You are on page 1of 1425

Telegram @unacademyplusdiscounts

Telegram @unacademyplusdiscounts

MASTER
RES URCE
Book for

JEE Main
Physics
Specially Prepared Questions for JEE Main with
Complete Theory 2 Levels Exercises Exams Questions

DB SINGH

ARIHANT PRAKASHAN (Series), MEERUT


Telegram @unacademyplusdiscounts

MASTER
RES URCE
Book for JEE Main

Arihant Prakashan (Series), Meerut


All Rights Reserved

© Author
No part of this publication may be re-produced, stored in a retrieval system or distributed
in any form or by any means, electronic, mechanical, photocopying, recording, scanning,
web or otherwise without the written permission of the publisher. Arihant has obtained
all the information in this book from the sources believed to be reliable and true. However,
Arihant or its editors or authors or illustrators don’t take any responsibility for the absolute
accuracy of any information published and the damages or loss suffered there upon.

All disputes subject to Meerut (UP) jurisdiction only.

Administrative & Production Offices


Regd. Office
‘Ramchhaya’ 4577/15, Agarwal Road, Darya Ganj, New Delhi -110002
Tele: 011- 47630600, 43518550; Fax: 011- 23280316
Head Office
Kalindi, TP Nagar, Meerut (UP) - 250002
Tele: 0121-2401479, 2512970, 4004199; Fax: 0121-2401648

Sales & Support Offices


Agra, Ahmedabad, Bengaluru, Bareilly, Chennai, Delhi, Guwahati,
Hyderabad, Jaipur, Jhansi, Kolkata, Lucknow, Meerut, Nagpur & Pune

ISBN : 978-93-13195-48-1

Published by Arihant Publications (India) Ltd.


For further information about the books published by Arihant
log on to www.arihantbooks.com or email to info@arihantbooks.com

/arihantpub /@arihantpub Arihant Publications /arihantpub


Telegram @unacademyplusdiscounts

MASTER
RES URCE
Book for JEE Main

PREFACE
In sync with the recent changes in the test pattern and format of JEE Main (Joint Engineering
Entrance), it is my pleasure to introduce Master Resource Book in Physics for JEE Main, for the
Students aspiring a seat in a reputed Engineering College. JEE Main is a gateway examination for
candidates expecting to seek admission in Bachelor in Engineering (BE), Bachelor of Technology
(B.Tech) and Bachelor of Architecture (B.Arch) at Indian Institutes of Information Technology (IIITs),
National Institutes of Technology (NITs), Delhi Technological University and other Centrally Funded
Technical Institutes (CFTIs).

JEE Main is also an examination which is like screening examination for JEE Advanced
(The gateway examination to India's most reputed Technical Institutes, Indian Institutes of
Technology IITs). Only the top 2.2 lacs students passed in JEE Main will be able to attempt
JEE Advanced.

Gradually, the number of students aspiring for the seat in the Engineering College has increased
rapidly in the last 5 Years or so. This year nearly 10 lacs students appeared for JEE Main and only a
few were able to reserve a seat in the college of their choice, so there is a cut throat competition
among the aspirants. Thus, it calls for a systematic mastery of all the subjects of the test with
paramount importance to problem-solving. Most of the books now in the market have become
repetitive with scant respect to the needs of true and effective learning. This book has been
designed to fulfill the perceived needs of the students as such.

— This book comprehensively covers all the topics of JEE Main Physics syllabus. The chapters have
been sequenced according to the syllabus of class 11th & 12th. Each chapter has essential
theoretical discussion of the related concepts with sufficient number of solved examples,
practice problems and other solved problems. In each chapter previous years' questions of
AIEEE and JEE Main have been included to help students know the difficulty levels and nature
of questions asked in competitive exams at this level.
— All types of questions have been included in this book: Single Correct Answer Types, Multiple
Correct Answer Types, Reasoning Types, Matches, Passage-based Questions etc.
— This is the only book which has its subject matter divided as per class 11th & 12th syllabus. It
covers almost all questions of NCERT Textbook & NCERT Exemplar problems.

It is hoped this new effort will immensely benefit the students in their goal to secure a seat in the
prestigious engineering college, and would be convenient to teachers in planning their teaching
programmes. Suggestions for further improvement are welcome from the students and teachers.

DB Singh
Telegram @unacademyplusdiscounts

MASTER
RES URCE
Book for JEE Main

CONTENTS
PART I 20. Magnetostatics 906-944
21. Electromagnetic Induction
Chapters from Class 11th Syllabus and Alternating Current 945-1001
1. Units and Measurements 3-40 22. Electromagnetic Waves 1002-1027
2. Kinematics 41-84 23. Ray Optics and Optical
3. Vector Analysis 85-115 Instruments 1028-1088
4. Projectile Motion 116-151 24. Wave Optics 1089-1130
5. Circular Motion 152-177 25. Dual Nature of Radiation
and Matter 1131-1155
6. Laws of Motion and Friction 178-229
26. Electronic Devices 1156-1190
7. Work, Energy and Power 230-267
27. Atoms, Molecules and Nuclei 1191-1247
8. Centre of Mass 268-306
28. Communication Systems 1248-1270
9. Rotational Motion 307-351
29. Experimental Physics 1271-1308
10. Gravitation 352-396
11. Properties of Solids 397-434
12. Properties of Liquids 435-488 JEE Main Solved Papers
13. Heat and Kinetic Theory of Gases 489-571 Solved Papers 2013 (Online & Offline) 1-34
14. Thermodynamics 572-614
Solved Papers 2014 35-42
15. Oscillations 615-662
16. Waves 663-716 Solved Papers 2015 43-52
Solved Papers 2016 53-60
PART II Solved Papers 2017 1-7
Chapters from Class 12th Syllabus Solved Papers 2018 1-8
17. Electrostatics 719-803
18. Current Electricity 804-862 Online JEE Main 2019 Solved Papers
(April & January Attempt) 1-32
19. Magnetic Effect of Current 863-905
Telegram @unacademyplusdiscounts

MASTER
RES URCE
Book for JEE Main

SYLLABUS
NOTE The syllabus contains two Sections - A & B. Section A pertains to the Theory Part, having 80%
weightage, while Section B contains Practical Component (Experimental Skills) having 20% weightage.

SECTION- A
UNIT 1 Physics and Measurement UNIT 5 Rotational Motion
Physics, technology and society, SI units, Fundamental Centre of mass of a two-particle system, Centre of mass of
and derived units. Least count, accuracy and precision of a rigid body; Basic concepts of rotational motion;
measuring instruments, Errors in measurement, moment of a force, torque, angular momentum,
Significant figures. Dimensions of Physical quantities, conservation of angular momentum and its
dimensional analysis and its applications. applications; moment of inertia, radius of gyration. Values
of moments of inertia for simple geometrical objects,
UNIT 2 Kinematics parallel and perpendicular axes theorems and their
Frame of reference. Motion in a straight line: Position-time applications.
graph, speed and velocity. Uniform and non-uniform Rigid body rotation, equations of rotational motion.
motion, average speed and instantaneous velocity.
Uniformly accelerated motion, velocity-time, position UNIT 6 Gravitation
time graphs, relations for uniformly accelerated motion. The universal law of gravitation.
Scalars and Vectors, Vector addition and Subtraction, Zero Acceleration due to gravity and its variation with altitude
Vector, Scalar and Vector products, Unit Vector, Resolution and depth.
of a Vector. Relative Velocity, Motion in a plane, Projectile Kepler's laws of planetary motion.
Motion, Uniform Circular Motion. Gravitational potential energy; gravitational potential.
Escape velocity. Orbital velocity of a satellite.
UNIT 3 Laws of Motion Geo-stationary satellites.
Force and Inertia, Newton's First Law of motion;
Momentum, Newton's Second Law of motion; Impulse; UNIT 7 Properties of Solids & Liquids
Newton's Third Law of motion. Law of conservation of Elastic behaviour, Stress-strain relationship, Hooke's. Law,
linear momentum and its applications, Equilibrium of Young's modulus, bulk modulus, modulus of rigidity.
concurrent forces.
Pressure due to a fluid column; Pascal's law and its
Static and Kinetic friction, laws of friction, rolling friction. applications.
Dynamics of uniform circular motion: Centripetal force Viscosity, Stokes' law, terminal velocity, streamline and
and its applications. turbulent flow, Reynolds number. Bernoulli's principle
and its applications.
UNIT 4 Work, Energy and Power
Surface energy and surface tension, angle of contact,
Work done by a constant force and a variable force;
application of surface tension - drops, bubbles and
kinetic and potential energies, work-energy theorem,
capillary rise.
power.
Heat, temperature, thermal expansion; specific heat
Potential energy of a spring, conservation of mechanical
capacity, calorimetry; change of state, latent heat.
energy, conservative and nonconservative forces; Elastic
and inelastic collisions in one and two dimensions. Heat transfer-conduction, convection and radiation,
Newton's law of cooling.
Telegram @unacademyplusdiscounts

MASTER
RES URCE
Book for JEE Main

UNIT 8 Thermodynamics Electric potential and its calculation for a point


charge, electric dipole and system of charges;
Thermal equilibrium, zeroth law of thermo-dynamics,
Equipotential surfaces, Electrical potential energy of a
concept of temperature. Heat, work and internal
system of two point charges in an electrostatic field.
energy. First law of thermodynamics.
Conductors and insulators, Dielectrics and electric
Second law of thermodynamics: reversible and
polarization, capacitor, combination of capacitors in
irreversible processes. Camot engine and its efficiency.
series and in parallel, capacitance of a parallel plate
UNIT 9 Kinetic Theory of Gases capacitor with and without dielectric medium
between the plates, Energy stored in a capacitor.
Equation of state of a perfect gas, work done on
compressing a gas. UNIT 12 Current Electricity
Kinetic theory of gases - assumptions, concept of Electric current, Drift velocity, Ohm's law, Electrical
pressure. Kinetic energy and temperature: rms speed of resistance, Resistances of different materials, V-I
gas molecules; Degrees of freedom, Law of characteristics of Ohmic and nonohmic conductors,
equipartition of energy, applications to specific heat Electrical energy and power, Electrical resistivity,
capacities of gases; Mean free path, Avogadro's Colour code for resistors; Series and parallel
number. combinations of resistors; Temperature dependence
of resistance.
UNIT 10 Oscillations and Waves
Electric Cell and its Internal resistance, potential
Periodic motion - period, frequency, displacement as difference and emf of a cell, combination of cells in
a function of time. Periodic functions. Simple harmonic series and in parallel.
motion (S.H.M.) and its equation; phase; oscillations of a
Kirchhoff's laws and their applications. Wheatstone
spring - restoring force and force constant; energy in
bridge, Metre bridge.
S.H.M. - kinetic and potential energies; Simple
pendulum - derivation of expression for its time period; Potentiometer - principle and its applications.
Free, forced and damped oscillations, resonance.
UNIT 13 Magnetic Effects of Current
Wave motion Longitudinal and transverse waves,
speed of a wave. Displacement relation for a and Magnetism
progressive wave. Principle of superposition of waves, Biot-Savart law and its application to current carrying
reflection of waves, Standing waves in strings and organ circular loop. Ampere's law and its applications to
pipes, fundamental mode and harmonics, Beats, infinitely long current carrying straight wire and
Doppler effect in sound. solenoid. Force on a moving charge in uniform
magnetic and electric fields Cyclotron.
UNIT 11 Electrostatics Force on a current-carrying conductor in a uniform
Electric charges Conservation of charge, Coulomb's magnetic field. Force between two parallel current-
law-forces between two point charges, forces between carrying conductors-definition of ampere. Torque
multiple charges; superposition principle and experienced by a current loop in uniform magnetic
continuous charge distribution. field, Moving coil galvanometer, its current sensitivity
Electric field Electric field due to a point charge, Electric and conversion to ammeter and voltmeter.
field lines, Electric dipole, Electric field due to a dipole, Current loop as a magnetic dipole and its magnetic
Torque on a dipole in a uniform electric field. dipole moment. Bar magnet as an equivalent
Electric flux, Gauss's law and its applications to find field solenoid, magnetic field lines; Earth's magnetic field
due to infinitely long, uniformly charged straight wire, and magnetic elements. Para, dia and ferro-magnetic
uniformly charged infinite plane sheet and uniformly substances
charged thin spherical shell. Magnetic susceptibility and permeability, Hysteresis,
Electromagnets and permanent magnets.
Telegram @unacademyplusdiscounts

MASTER
RES URCE
Book for JEE Main

UNIT 14 Electromagnetic Induction UNIT 17 Dual Nature of Matter


and Alternating Currents and Radiation
Electromagnetic induction; Faraday's law, induced Dual nature of radiation. Photoelectric effect, Hertz
emf and current; Lenz's Law, Eddy currents. Self and and Lenard's observations; Einstein's photoelectric
mutual inductance. equation; particle nature of light. Matter waves-wave
Alternating currents, peak and rms value of nature of particle, de Broglie relation. Davisson-
alternating current/ voltage; reactance and Germer experiment.
impedance; LCR series circuit, resonance; Quality
factor, power in AC circuits, wattless current. AC UNIT 18 Atoms and Nuclei
generator and transformer. Alpha-particle scattering experiment; Rutherford's
model of atom; Bohr model, energy levels, hydrogen
UNIT 15 Electromagnetic Waves spectrum.
Electromagnetic waves and their characteristics. Composition and size of nucleus, atomic masses,
Transverse nature of electromagnetic waves. isotopes, isobars; isotones. Radioactivity-alpha, beta
Electromagnetic spectrum (radio waves, and gamma particles/rays and their properties;
microwaves, infrared, visible, ultraviolet, X-rays, radioactive decay law. Mass-energy relation, mass
gamma rays). Applications of e.m. waves. defect; binding energy per nucleon and its variation
with mass number, nuclear fission and fusion.
UNIT 16 Optics
UNIT 19 Electronic Devices
Reflection and refraction of light at plane and
Semiconductors; semiconductor diode: I-V
spherical surfaces, mirror formula, Total internal
characteristics in forward and reverse bias; diode as a
reflection and its applications, Deviation and
rectifier; I-V characteristics of LED, photodiode, solar
Dispersion of light by a prism, Lens Formula,
cell, and Zener diode; Zener diode as a voltage
Magnification, Power of a Lens, Combination of thin
regulator. Junction transistor, transistor action,
lenses in contact, Microscope and Astronomical
characteristics of a transistor transistor as an amplifier
Telescope (reflecting and refracting) and their
(common emitter configuration) and oscillator. Logic
magnifying powers.
gates (OR, AND, NOT, NAND & NOR). Transistor as a
Wave optics wave front and Huygens' principle, switch.
Laws of reflection and refraction using Huygen's
principle. Interference, Young's double slit UNIT 20 Communication Systems
experiment and expression for fringe width, coherent
Propagation of electromagnetic waves in the
sources and sustained interference of light.
atmosphere; Sky and space wave propagation,
Diffraction due to a single slit, width of central
Need for modulation, Amplitude and Frequency
maximum. Resolving power of microscopes and
Modulation, Bandwidth of signals, Bandwidth of
astronomical telescopes, Polarisation, plane polarized
Transmission medium, Basic Elements of a
light; Brewster's law, uses of plane polarized light and
Communication System (Block Diagram only)
Polaroids.
Telegram @unacademyplusdiscounts

MASTER
RES URCE
Book for JEE Main

SECTION- B
UNIT 21 Experimental Skills (ii) Internal resistance of a cell.
Familiarity with the basic approach and observations of 14. Resistance and figure of merit of a
the experiments and activities galvanometer by half deflection method.
1. Vernier callipers - its use to measure internal and 15. Focal length of
external diameter and depth of a vessel. (i) Convex mirror
2. Screw gauge - its use to determine thickness/ (ii) Concave mirror
diameter of thin sheet/wire. (iii) Convex lens
3. Simple Pendulum - dissipation of energy by 16. Using parallax method. Plot of angle of
plotting a graph between square of amplitude deviation vs angle of incidence for a triangular
and time. prism.
4. Metre Scale - mass of a given object by principle 17. Refractive index of a glass slab using a travelling
of moments. microscope.
5. Young's modulus of elasticity of the material of a 18. Characteristic curves of a p-n junction diode in
metallic wire. forward and reverse bias.
6. Surface tension of water by capillary rise and 19. Characteristic curves of a Zener diode and
effect of detergents. finding reverse break down voltage.
7. Coefficient of Viscosity of a given viscous liquid 20. Characteristic curves of a transistor and finding
by measuring terminal velocity of a given current gain and voltage gain.
spherical body.
21. Identification of Diode, LED, Transistor, IC,
8. Plotting a cooling curve for the relationship Resistor, Capacitor from mixed collection of
between the temperature of a hot body and time. such items.
9. Speed of sound in air at room temperature 22. Using multimeter to
using a resonance tube.
(i) Identify base of a transistor.
10. Specific heat capacity of a given (i) solid and (ii) (ii) Distinguish between npn and pnp type
liquid by method of mixtures. transistor.
11. Resistivity of the material of a given wire using (iii) See the unidirectional flow of current in
metre bridge. case of a diode and an LED.
12. Resistance of a given wire using Ohm's law. (iv) Check the correctness or otherwise of a
13. Potentiometer given electronic component (diode,
transistor or IC).
(i) Comparison of emf of two primary cells.
Telegram @unacademyplusdiscounts

Part - I
th
Chapters from Class 11 Syllabus
Telegram @unacademyplusdiscounts
Telegram @unacademyplusdiscounts

1 Units and
Measurements
JEE Main MILESTONE
< Physics, Technology and Society < Errors in Measurement
< SI units < Significant Figures
< Fundamental and Derived Units < Dimensions of Physical Quantities
< Least Count < Dimensional Analysis and its Applications
< Accuracy and Precision of Measuring Instruments

1.1 Physics, Technology and Society


Science is a systematic attempt to understand natural phenomena in as much
detail and depth as possible and use the knowledge, so gained to predict, modify
and control phenomena. Science is exploring, experimenting and predicting from
what we see around us.
In physics, we attempt to explain diverse physical phenomena in terms of a few
concepts and laws. The effort is to see the physical world as manifestation of some
universal laws in different domains and conditions. The connection between
physics, technology and society can be seen in many examples. The discipline of Physics describes the laws of
thermodynamics arose from the need to understand and improve the working of nature. This description is
heat engines. Also wireless communication technology followed from the
quantitative and involves
discovery of the basic laws of electricity and magnetism. Let another important
measurement and comparison of
example of physics giving rise to technology is the silicon chip that triggered the
computer revolution in the last three decades of the twentiath century. Hence, we
physical quantities. To measure a
observe that how much strong influence physics is having on technology physical quantity, we need some
development and society. standard unit of that quantity. In

this chapter, we shall study about the units and measurement.


1.2 SI Units
The quantities by means of which we describe the laws of physics are called
physical quantities. To measure a physical quantity, some standard unit of that
quantity is required, e. g. , if length of some metal rod is measured to be 20 m, then
m is the unit of length and 20 is the numerical value. So,
Physical quantity = Numerical value ´ Unit

Note
● If the numerical value of any physical quantity in different units u1 and u 2 are n1 and n 2
respectively, then n1u1 = n 2u 2 .
● As the unit will change, numerical value will also change, e . g ., acceleration due to gravity,
–2 –2
g = 32 fts = 9.8 ms .
Telegram @unacademyplusdiscounts

4 JEE Main Physics

(ii) Steradians for solid angle It is the solid angle which has
1.3 Fundamental and the vertex at the centre of the sphere, and cut-off an
Derived Units area of the surface of sphere equal to that of square
with sides of length equal to radius of sphere.
The number of physical quantities is quite large, but we Note
need a limited number of units only for expressing all the ● Angle and solid angle are considered supplementary base units
physical quantities because they are interrelated with one because although these have units but they are both
another. Thus, we may define a set of fundamental dimensionless.
(or base) quantities and all other quantities may be ● 2 p radians = 360°
expressed in terms of these fundamental quantities. All
other quantities are called derived quantities. Metric Prefixes for Powers of 10
Units of fundamental and derived quantities are The physical quantities whose magnitude is either too
respectively, known as the fundamental units and derived large or too small can be expressed more compactly by
units. A complete set of these units, both fundamental and the use of certain prefixes as given in the table.
derived units, is known as the system of units.
Table 1.2 Metric Prefixes
System of Units Power of 10 Prefix Symbol
The common system of units are -1
deci d
10
(i) FPS System The units of length, mass and time are
10-2 centi c
respectively foot, pound and second.
(ii) CGS System The units of length, mass and time are 10-3 milli m
respectively centimetre, gram and second. 10 -6
micro m
(iii) MKS System The units of length, mass and time are 10 -9
nano n
respectively metre, kilogram and second.
-15
10 femto f
SI System (International System of Units) 10-18
atto a
-21
In 1971, CGPM held its meeting and decided a system of 10 zepto z
units which is known as the International System of Units.
10-24 yocto y
It is abbreviated as SI from the French name Le Systeme
1
International d¢ unites. This system is widely used through 10 deca da
out the world. 10 2
hecto h
The SI is based on the following seven fundamental units 10 3
kilo k
and two supplementary units. 6
10 mega m
Table 1.1 Units and Symbol of Quantities 10 9
giga g
S. No. Quantity Unit Symbol 1012 tera t
1. Mass kilogram kg 1015 peta p
2. Length metre m 18
10 exa e
3. Time second s 21
10 zetta z
4. Electric current ampere A
24
5. Temperature kelvin K 10 yotta y

6. Amount of substance mole mol


7. Luminous intensity candela cd
Practical Units
A large number of units are used in general life for
The two supplementary units in SI system are measurement of different quantities in comfortable
(i) Radians for angle It is the angle subtended at the centre manner. But they are neither fundamental units nor
by an arc of a circle having a length equal to radius of derived units.
the circle. Its symbol is rad.
Telegram @unacademyplusdiscounts

Units and Measurements 5

Some practical units are listed below

Table 1.3 Practical Units of Length, Mass and Time


S.No. Practical Units of Length Practical Units of Mass Practical Units of Time
1. 1 light year = 9.46 × 1015 m 1 quintal = 102 kg 1
1 year = 365 solar days
4
2. 1 astronomical unit 1 metric ton =103 kg 1 lunar month = 27.3 solar days
or 1 AU = 1.5 × 1011 m
3. 1 parsec = 3.26 light year 1 atomic mass unit (amu) =1.66 × 10–27kg 1 solar day = 86400 s
4. 1 seamile = 6020 ft 1 pound = 0.4537 kg Tropical year It is that year in which solar eclipse
occurs.
5. 1 micron = 1 mm = 10–6 m 1 chandrasekhar limit = 1.4 times the mass Leap year It is that year in which the month of
of sun =2.8 × 1030 kg February has 29 days.
6. 1 angstrom = 10–10 m 1 slug = 14.59 kg 1 shake = 10–8 s
7. 1 fermi = 10–15 m

Sample Problem 1 What is the SI unit of surface tension? 2. Screw gauge,


(a) Nm-1 (b) Nm-2 (c) Nm (d) N Value of 1 pitch scale reading
Least count =
Total number of head scale divisions
Force N
Interpret (a) Surface tension = = = Nm–1 1mm
Length m Least count =
100 divisions
= 0.01 mm

1.4 Least Count 3. Travelling microscope,


Value of 1 main scale division
Least count =
The least count of a measuring device is the least distance Total number of vernier scale divisions
(resolution/accuracy), that can be measured using the 0.5 mm
device. The general formula that can be used for least =
50 divisions
count (LC).
= 0.01mm
Value of 1 main scale division
LC =
Total number of vernier scale divisions 4. Spectrometer,
0.5 degree
Every measuring instrument has no error, when readings Least count =
30 divisions
are taken. The least count uncertainity or maximum 30°
= = 1°
possible error characterises such errors. Instruments error 30 divisions
can be compared by calculating the percentage of
uncertainity of their readings. The instrument with the Note 1 degree (angle) = 60' and1' = 60"
least uncertainity is taken to measure objects, as all
measurements consider accuracy. Least Count Error
The percentage uncertainity is calculated with the
Measured values are good only upto its least count . The
following formula
least count error is the error associated with the resolution
Maximum possible error of the instrument.
= ´ 100
Measurement of object in question
Least count error belongs to the category of random errors
The smaller the measurement, the larger the percentage but within a limited scale, it occurs with both systematic
uncertainity. The least count of an instrument is indirectly and random errors. If we use a metre scale for
proportional to the precision of the instrument. measurement of length, it may have graduations as 1 mm
division scale spacing or interval. Instruments of higher
Least Count of Certain Measuring precision, improving experimental techniques etc., can
reduce the least count error. Repeating the observations
Instruments and taking the arithmetic mean of the result, the mean
1. Vernier calliper, value would be very close to the true value of the
1 mm measured quantity.
Least count (LC) = = 0.1mm
10 divisions
Telegram @unacademyplusdiscounts

6 JEE Main Physics

Sample Problem 2 In an experimental set up, the density of Sample Problem 4 A vernier calliper has 1 mm mark on
a small sphere is to be determined. The diameter of the small the main scale. It has 20 equal divisions on the vernier scale
sphere is measured with the help of a screw gauge, whose pitch which match with 16 main scale divisions. For this vernier
is 0.5 mm and there are 50 divisions on the circular scale. The callipers, the least count is
reading on the main scale is 2.5 mm and that on the circular (a) 0.02 mm (b) 0.05 mm
scale is 20 divisions. If the measured mass of the sphere has a (c) 0.1 mm (d) 0.2 mm
relative error of 2%, the relative percentage error in the density is
Interpret (d) Least count of a vernier calliper,
(a) 0.03% (b) 3.11%
(c) 0.08% (d) 8.2% LC = 1MSD – 1 VSD
Value of 1 MSD
Interpret (b) Least count of screw gauge =
Total divisions on the circular scale
Pitch
= 20 divisions of Vernier scale = 16 divisions of main scale
Total divisions on circular scale 16
0.5 1 VSD = mm = 0.8 mm
\ Least count = = 0.01mm = Dr 20
50
\ LC = 1MSD –1 VSD
Diameter = Main scale + Circular scale ´ Least count
= 1mm– 0.8 mm
0.5
= 2.5+20 ´ = 2.70 mm = 0.2 mm
50
Dr 0.01
Q =
r 2.70
Dr 1
1.5 Accuracy and Precision of
´ 100 =
r 2.7 Measuring Instruments
m m
Density, D = = 3 Measurements is the foundation of all experimental
V 4 ærö
pç ÷ science and technology. The result of every measurement
3 è2ø
by any measuring instrument contains some uncertainity.
Here, r is diameter This uncertainity is called error.
DD ì Dm æ Dr ö ü
\ ´ 100 = í + 3 ç ÷ý ´ 100
D î m è r øþ Resolution, Accuracy and Precision
Dm Dr of an Instrument
= ´ 100 + 3 ´ ´ 100
m r
1 Resolution Stands for least count or the minimum reading
= 2% + 3 ´ = 3.11% which an instrument can read.
2.7
Accuracy An instrument is said to be accurate, if the
Sample Problem 3 A screw gauge gives the following physical quantity measured by it resembles very closely to
reading, when used to measure the diameter of a wire. its true value.
Main scale reading : 0 mm
Precision An instrument is said to have high degree of
Circular scale reading : 52 divisions
precision, if the measured value remains unchanged, how
sions of Given that 1 mm on main scale corresponds to
so ever, large number of times it may have been repeated.
the circular scale
The diameter of the wire from the above data is Sample Problem 5 A man wishes to Distant object
(a) 0.026 cm (b) 0.016 m estimate the distance of a nearby tower O
(c) 0.052 cm (d) 0.062 m from him. He stands at a point A is front of
C
Interpret (c) Diameter of wire = MSR + CSR ´ LC the tower C and spots A infront of the
θ
tower C and spots a very distant object O
Given main scale reading (MSR) = 0 mm
in line with AC. He then walks θ
Circular scale reading = 52
perpendicular to AC upto B, a distance of A
Value of 1 main scale division 100 m
B
Least count = 100 m and looks at O and C again. Since
Total divisions on circular scale O is very distant, the direction BO is
1 practically the same as AO, but he finds the line of sight of C
LC = 0 + 52 ´
100 shifted from the original line of sight by angle q = 40°
= 0.52 mm ( q is known as parallax), the distance of tower C from his
= 0.052 cm original position A is
Telegram @unacademyplusdiscounts

Units and Measurements 7

(a) 100 m (b) 10 m


(c) 19 m (d) 119 m
1.6 Errors in Measurement
Interpret (d) Given, parallax angle q = 40° There are many causes of errors in measurement. Errors
may be due to instrumental defects, ignoring certain facts,
From the given figure, AB = AC tan q
AB 100 m 100 carelessness of experimenter, random change in
Þ AC = = = = 119 m temperature, pressure, humidity, etc. When an
tan q tan 40° 0.8391
experimenter tries to reach accurate value of
Sample Problem 6 The moon is observed from two measurement by doing large number of experiments, the
diametrically opposite points A and B on the earth. The angle q mean of a large number of the results of repeated
subtended at the moon by the two directions of observation is experiments is close to the true value.
1° 54¢. The distance of the moon from the earth is
[Given diameter of earth = 1.276 ´ 107 m] Calculation of Magnitude of Errors
8 3
(a) 3.84 ´ 10 m (b) 1.5 ´ 10 m (i) True value If a1, a2, a3, ¼, an are the observed values of
4
(c) 2.81 ´ 10 m (d) 3.2 ´ 10 5 m a measurement, then true value of measurement is the
mean of these observed values.
Interpret (a) Given,
\ atrue = amean = a0
1° 54 ¢ = 114 ¢= (114 ´ 60) ¢¢ ´ ( 4.85 ´ 10 –6) rad
a + a2 + a3 + ¼+ an
= 3.32 ´ 10 –2 rad = 1
n
Since, 1" = 4.85 ´ 10 –6 rad 1 i=n
Also, AB = b = 1.276 ´ 10 7 m
= å ai
n i =1
b
D= (ii) Absolute error The absolute errors in various
q individual measured values are found by substracting
1.276 ´ 10 7 the observed value from true value. Thus,
D= = 3.84 ´ 10 8 m
3.32 ´ 10 –2
Da1 = a0 - a1, Da2 = a0 - a2,

Sample Problem 7 The sun’s angular diameter is Da3 = a0 - a3, ¼, Dan = a0 - an


measured to be 1920". The distance D of the sun from the earth The absolute error may be positive or negative.
is 1.496 ´ 1011 m, the diameter of the sun is [NCERT] (iii) Mean absolute error The arithmetic mean of the
3
(a) 9.31 ´ 10 m (b) 1.39 ´ 10 m 9 magnitudes of different values of absolute errors is
known as the mean absolute error.
(c) 3.26 ´ 10 6 m (d) 8.32 ´ 10 4 m
\ Mean absolute error,
Interpret (b) Sun’s angular diameter a = 1920" | Da1 | + | Da2 | + | Da3 | + ¼ + | Dan |
= 1920 ´ 4.85 ´ 10 –6 rad Damean =
n
= 9.31 ´ 10 –3 rad The final result of measurement can be written as
Sun’s diameter, d = aD a = am ± Da . This implies that value of a is likely to lie
= (9.31 ´ 10 –3) ´ (1.496 ´ 10 11) am + Da and am - Da .
= 1.39 ´ 10 9 m (iv) Relative or fractional error The ratio of the mean value
of absolute error and the true value is known as the
mean relative error.
Check Point 1
\ Mean relative error
1. Do AU and Å represent the same unit of length? Mean absolute error
=
2. In defining the standard of length, we have to specify the Mean value of measurement
temperature at which the measurement should be made. Are Damean Damean
= =
we justified in calling length a fundamental quantity, if amean a0
another physical quantity (temperature) has to be specified in
choosing a standard? Relative error is also known as fractional error. When
expressed in terms of percentage, relative error is
3. Can there be a physical quantity which has no unit and
called the “relative percentage error”. Hence,
dimensions? Can a physical quantity have unit without having
dimensions? Da
Percentage error = mean ´ 100%
amean
Telegram @unacademyplusdiscounts

8 JEE Main Physics

Sample Problem 8 The average speed of a train is Interpret (a) Given, V1 = (10.2 ± 0.02) cm3
measured by 5 students. The results of measurements are given and V2 = (6.4 ± 0.01) cm3
below
Number of Students Speed (m/s)
DV = ± ( DV1 + DV2)
= ± (0.02+0.01) cm3 = ± 0.03 cm3
1 10.2 ms–1
2 10.4 ms–1 V1 + V2 = (10.2+6.4) cm3 = 16.6 cm3
3 9.8 ms–1 and V1 - V2 = (10.2 – 6.4) cm3 = 3.8 cm3
4 10.6 ms–1 Hence, sum of volume = (16.6 ± 0.03) cm3
5 10.8 ms–1
and difference of volume = (3.8 ± 0.03) cm3
(a) 2.6% (b) 3.5%
(c) 4.5% (d) 5.5% In product If Z = AB, then maximum fractional error is
DZ æ DA DB ö
10.2+10.4+9.8+10.6+10.8 = ±ç + ÷
Interpret (a) Mean value, v m = Z è A B ø
5
51.8 Therefore, maximum fractional error in product of two (or
= = 10.0 ms–1
5 more) quantities is equal to sum of fractional errors in the
Dv1 = v m - v1 = 10.4 –10.2 = 0.2 individual quantities.
Dv 2 = v m - v 2 = 10.4 –10.4 = 0.0 In division If Z = A / B, then maximum fractional error is
Dv3 = v m - v3 = 10.4 – 9.8 = 0.6 DZ æ DA DB ö
Dv 4 = v m - v 4 = 10.4 –10.6 = –0.2 = ±ç + ÷
Z è A B ø
Dv5 = v m - v5 = 10.4 –10.8 = –0.4
Mean absolute error, Therefore, maximum fractional error in product of two (or
| Dv1| + | Dv 2| + | Dv3| + | Dv 4| + | Dv5| more) quantities is equal to sum of fractional errors in the
Dv = individual quantities.
5
0.2+0.0 +0.6+0.2+0.4 1.4
= = = 0.28 ms–1 Sample Problem 10 Object distance, u = (50.1 ± 0.5) cm
5 5
Dv 0.28 and image distance v = (20.1 ± 0.2) cm, then focal length is
Relative error = ± =± (a) (12.4 ± 0.4) cm (b) (12.4 ± 0.1) cm
vm 10.4
(c) (14.3 ± 0.4) cm (d) (14.3 ± 0.1) cm
Dv 0.28
Percentage error = ± ´ 100 = ± ´ 100 = ± 2.6% 1 1 1
vm 10.4 Interpret (c) Focal length is given by = +
f v u
uv (50.1) (20.1)
Combination of Errors or f= =
u + v (50.1) + (20.1)
= 14.3 cm

In Sum If Z = A + B, then DZ = ± (DA + DB), maximum Df æ Du Dv Du + Dv ö


DZ DA + DB Also, =±ç + + ÷
fractional error in this case = f è u v u+v ø
Z A+ B
æ 0.5 0.2 0.5+0.2 ö
i. e. , when two physical quantities are added, then the =±ç + + ÷
è 50.1 20.1 50.1+20.1ø
maximum absolute error in the result is the sum of the
= ± [0.00998+0.00995+0.00997]
absolute errors of the individual quantities.
= ± (0.0299)
In Difference If Z = A - B, the maximum absolute error is
\ Df = 0.0299 ´ 14.3
DZ = ± (DA + DB) and maximum fractional error in this case
= 0.428 = 0.4 cm
DZ DA + DB
= \ f = (14.3 ± 0.4) cm
Z A-B
DZ DA
In power If Z = An, then =n
Sample Problem 9 The volumes of two bodies are Z A
measured to be V1 = (10.2 ± 0.02) cm3 and V2 = (6.4 ± 0.01) cm3. AxBy ,
In more general form if Z =
The sum and difference in volumes with error limits is Cq
(a) (16.6 ± 0.03) cm3 and (3.8 ± 0.03) cm3 then the maximum fractional error in Z is
3
(b) (16.6 ± 0.01) cm and (3.8 ± 0.01) cm 3 DZ DA DB DC
=x +y +q
(c) (16.2 ± 0.03) cm3 and (3.6 ± 0.03) cm3 Z A B C
(d) (16.2 ± 0.01) cm3 and (3.6 ± 0.01) cm3 (Note that there is no negative sign)
Telegram @unacademyplusdiscounts

Units and Measurements 9

Sample Problem 11 Calculate percentage error in Sample Problem 13 Two resistors of resistances
determination a time period of a pendulum R1 = 100 ± 3 W and R2 = 200 ± 4 W are connected in parallel,
l ælö
1/ 2 then the equivalent resistance in parallel is (in ohm)
T = 2p or 2 p ç ÷ æ 1 1 1 DR ¢ DR1 DR2 ö
g ègø çUse = + and = + 2 ÷
è R ¢ R1 R2 DR ¢2 R12 R2 ø
where, l and g are measured with ± 2% and ± 3% errors.
(a) 66.7 ± 1.8 (b) 300 ± 7
(a) 2.9% (b) ± 2.5%
(c) 150.8 ± 2 (d) 92.3 ± 3
(c) 1.5% (d) ± 1.9%
DT æ 1 Dl 1 Dg ö Interpret (a) The equivalent resistance of parallel combination is
Interpret (b) ´100 = ± ç ´ ´ 100 + ´ ´ 100 ÷ R1R2 200
T è2 l 2 g ø R¢ = = = 66.7 W
R1 + R2 3
æ1 1 ö
= ± ç ´ 2 + ´ 3÷ = ± 2.5% 1 1 1
è2 2 ø From, = + , we get
R ¢ R1 R2
Sample Problem 12 The period of oscillation of a simple DR ¢ DR1 DR2
= 2 + 2
pendulum is measured, in successive measurement the R ¢2 R1 R2
readings turn out to be 2.63 s, 2.56 s, 2.42 s, 2.71 s and 2.80 s. 2 DR1 DR
DR ¢ = (R ¢ ) 2 + (R ¢2) 22
Then, the relative error or percentage error is R1 R2
(a) 2% (b) 4% 2 2
æ 66.7 ö æ 66.7 ö
(c) 6% (d) 8% =ç ÷ 3+ ç ÷ 4 = 1.8 W
è 100 ø è 200 ø
Interpret (b) The mean period of oscillation of the pendulum is Hence, R ¢ = (66.7 ± 1.8) W
(2.63+2.56+2.42+2.71+2.80) s
T=
5 Sample Problem 14 The temperature of two bodies
13.12 measured by a thermometer are t1 = 20° C + 0.5° C and
T= s = 2.624 s = 2.62 s
5 t 2 = 50° C ± 0.5° C. The temperature difference is
As the periods are measured to a resolution of 0.01 s, all times are to (a) ± 1°C (b) ± 2°C
the second decimal, it is proper to put this mean period also to the (c) ± 3°C (d) ± 4°C
second decimal. The errors in the measurement are Interpret (a) The temperature difference is given by
2.63 s – 2.62 s = 0.01 s t ¢ = t 2 - t1 = (50° C ± 0.5° C) - (20° C ± 0.5° C)
2.56 s – 2.62 s = –0.06 s t ¢ = 30° C ± 1° C
2.42 s – 2.62 s = – 0.20 s
V
2.71s – 2.62 s = 0.09 s Sample Problem 15 The resistance R = , where
I
2.80 s – 2.62 s = 0.18 s
V = (100 ± 5) V and I = (10 ± 0.2) A. The percentage error in R is
Note that the errors have the same units as the quantity to be (a) 5% (b) 2%
measured. The arithmetic mean of all the absolute errors (for (c) 3% (d) 7%
arithmetic mean, we take only the magnitudes) is
Interpret (d) The percentage error in V is 5% and in I it is 2%.
DTmean = [(0.01+0.06+0.20+0.09+0.18) s] / 5
Hence, the total error in R would be
= 0.54 s / 5 = 0.11s
% error = 5% + 2% = 7%
That means, the period of oscillation of the simple pendulum is
(2.62 ± 0.11) s i. e. , it lies between (2.62 + 0.11) s and (2.62 – 0.11) s Sample Problem 16 The period of oscillation of a simple
or between 2.73 s and 2.51 s. As the arithmetic mean of all the L
absolute errors in 0.11 s, there is already an error in the tenth of a
pendulum is T = 2p . Measured value of L is 20 cm known to
g
second. Hence, there is no point in giving the period to a
hundredth. A move correct way will to be write, 1 mm accuracy and time for 100 oscillations of the pendulum is
found to be 90 s using a wrist-watch of 1 s resolution. The
T = 2.6 ± 0.1 s
accuracy in the determination of g is
Note that the last numeral 6 is unreliable, since it may be anything (a) 1% (b) 2%
between 5 and 7. We indicate this by saying that the measurement (c) 3% (d) 4%
has two significant figures. In this case, the two significant figures
are 2, which is reliable and 6, which has an error associated with it. L
Interpret (c) Given, T = 2p
Hence, the relative error or percentage error is g
0.1 4 p 2L
Sa = ´ 100 = 4% or g =
2.6 T2
Telegram @unacademyplusdiscounts

10 JEE Main Physics

t Dt (iv) All zeros to the right of a derived point and to the left of
Hence, T= and DT =
n n a non-zero digit are not significant.
DT Dt
\ = Number Significant number
T t 0.08 1
The errors in both L and t are the least count errors. Therefore, 0.008 1
æ Dg ö æ DL ö æ DT ö 0.1 æ 1ö 0.0846 3
ç ÷ = ç ÷ + 2 çè ÷= + 2 ç ÷ = 0.027
è g ø è L ø T ø 20.0 è 90 ø (v) All zeros to the right of a decimal point and to the right
The percentage error in g is of a non-digit are significant.
æ Dg ö æ DL ö æ DT ö Number Significant number
ç ÷ ´ 100 = çè ÷ø ´ 100 + 2 ´ çè ÷ ´ 100 = 3%
è g ø L T ø 0.40 2
0.430 3
(vi) The powers of ten are not counted as significant digits
Few more examples of
e. g ., 1.4 ´ 10–7 has only two significant figures 1 and 4.
Combination of Errors
Simple pendulum, T µ l1/2 Þ
DT
=
1 Dl Rounding off the Digits
T 2 l
Certain rules are applied in order to round off the
4
For sphere, A = 4 pr 2, V = pr 3 measurements
3
DA Dr DV Dr (i) If the number lying to the right of digit to be rounded is
Þ = 2× and = 3×
A r V r less than 5, then the rounded digit is retained as such.
GM However, if it is more than 5, then the digit to be
For gravity, g=- 2
R rounded is increased by 1.
Dg DR For example, x = 6.24 is rounded off to 6.2 to two
= -2 (whereM is constant)
g R significant digits and x = 8.356 is rounded off to 8.36 to
three significant digits.
(ii) If the digit to be dropped is 5 followed by digits other
1.7 Significant Figures than zero, then the preceding digit is increased by 1.
Significant figures in the measured value of a physical For example, x = 14.252 is rounded off to x = 14.3 to
quantity tell the number of digits in which we have three significant digits.
confidence. Larger the number of significant figures
(iii) If the digit to be dropped is simply 5 or 5 followed by
obtained in a measurement, greater is the accuracy of the
zeros, then the preceding digit is left unchanged if it is
measurement. “The significant figures are those number
even.
of digits in a quantity that are known reliably plus one
digit that is uncertain.” For example, x = 6.250 or x = 6.25 becomes x = 6.2 after
rounding off to two significant digits.
Rules for Significant Figures (iv) If the digit to be dropped is 5 or 5 followed by zeros,
(i) All non-zero digits are significant figures. then the preceding digit is raised by one if it is odd.
Number Significant figures For example, x = 6.350 or x = 6.35 becomes x = 6.4 after
16 2 rounding off to two significant digits.
1683 4
16835 5 Rounding off to Three
Measured Values
Significant Digits
(ii) All zeros occuring between non-zero digits are
significant figures. 7.364 7.36
Number Significant figures 7.367 7.37
802 3
80004 5 8.3251 8.33
(iii) All zeros to the right of the last non-zero digits are not 9.445 9.45
significant.
9.4450 9.45
Number Significant number
40 1 15.75 15.8
410 2 15.7500 15.8
40240 4
Telegram @unacademyplusdiscounts

Units and Measurements 11

Algebraic Operations with


Significant Figures Check Point 2
In addition, subtraction, multiplication or division 1. What importance do we attach to the final zeros in a number
inaccuracy in the measurement of any one variable without any decimal point?
affects the accuracy of the final result. Hence, in general , 2. The length of a table as measured by two students is given as
the final result have significant figures according to the 2.5 m and 2.50 m. Which of the following measurement is
rules given below more accurate and why?
3. Two students A and B made the length measurement in the
Addition and Subtraction laboratory and wished to find their sum. The student A insisted
that they should round off and then add, while the student B
The number of decimal places in the final result of any of
argued that they should add the measurements directly and
these operations has to be equal to the smallest number of then round off their sum. Which student was correct?
decimal places in any of the terms involved in calculation
e. g. , sum of terms 2.29 and 62.7 is 64.99. After rounding off
to one place of decimal, it will become 65.0. Subtraction of
Sample Problem 19 Each side of a cube is measured to be
62.7 from 82.27 gives 19.57. After rounding off to one place 7.203 m. The volume of the cube to appropriate significant
of decimal, it will become 19.6. figures is [NCERT]
(a) 31.3 m3 (b) 313 m3
Multiplication and Division (c) 373.7 m3 (d) 37.3 m3
In these operations, the number of significant figures in
Interpret (c) The number of significant figures in the measured
the result is same as the smallest number of significant length is 4. The volume should therefore be rounded off to 4
figures in any of the factors. significant figures.
e. g. , 1.2 ´ 1.3 = 1.56. After rounding off to two significant As, V = a3
figures, it becomes 1.6. Given, a = 7.203 m
1100 \ V = (7.203)3 = 373.714754 m3 = 373.7 m3
Similarly, if gives 107.84. Thus, the result when
10.2
rounded off to two significant digits becomes 108. Sample Problem 20 5.74 g of a substance occupies 1.2 cm3.
Keeping the significant figure in view, its density is given by
Sample Problem 17 If L = 2.5 ´ 10 4 and B = 3.9 ´ 10 5, (a) 4.8 g cm-3 (b) 1.5 g cm-3
then L - B is (c) 2.1 g cm-3 (d) 9.2 g cm-3
(a) 1.4 ´ 10 4 (b) 1.4 ´ 10 5
(c) 3.6 ´ 10 4 (d) 3.6 ´ 10 5
Interpret (a) There are 3 significant figures in the measured
mass whereas there are only 2 significant figures in the measured
Interpret (d) Given, L = 2.5 ´ 10 4 = 25000, volume. Hence, the density should be expressed to only 2
B = 3.9 ´ 10 5 = 390000 significant figures.
Mass
\ L - B = 390000 - 25000 = 365000 Q Density =
Volume
= 3.65 ´ 10 5 = 3.6 ´ 10 5
5.74
(rounded to one place of decimal) \ Density = g cm–3 = 4.8 g cm–3
1.2
Sample Problem 18 The area enclosed by a circle of
diameter 1.06 m to correct number of significant figures is
(a) 0.88 m2 (b) 0.088 m2 1.8 Dimensions of Physical
(c) 0.882 m2 (d) 0.530 m2 Quantities
1.06
Interpret (c) Here, r = = 0.530 m Dimensions of a physical quantity are the powers to which
2
the fundamental quantities must be raised to represent
Area enclosed = pr 2 = 3.14 (0.53) 2
the given physical quantity. In mechanics, all physical
= 0.882026 m2 = 0.882 m2 quantities can be expressed in terms of mass [M], length [L]
(rounded to three significant figures) and time [T].
Telegram @unacademyplusdiscounts

12 JEE Main Physics

For example, Further, if we represent force by [F], then [F] = [M1L1T –2] is
Force = Mass ´ Acceleration called the dimensional equation of force.
Velocity m ´ s m ´ s
= Mass ´ = =
Time t ´t t2 Dimensionless Quantity
= [M] [L] [T –2]
In the equation [MaLb T c], if a = b = c = 0, then the quantity is
So, the dimensions of force are 1 in mass, 1 in length and called dimensionless.
-2 in time.
For example, strain, specific gravity, angle. They are ratio
of two similar quantities.
Dimensional Formula and
Follwing physical quantities are dimensionless angle, solid
Dimensional Equations angle, relative density, specific gravity, Poisson’s ratio,
The expression which shows how and which of the base Reynold’s number, all trigonometric ratios refractive index,
quantities represent a physical quantity is called the relative permittivity, dielectric constant, magnetic
dimensional formula of the given physical quantity. susceptibility. A dimensionless quantity has same numeric
value in all system of units.
For example, as above, [M1L1T –2] is the
deduced
dimensional formula of force. It reveals that unit of force
depends on [M], [L] and [T].

Table 1.4 Dimensional Formulae and SI Units of Some Physical Quantities


S. No. Physical quantity Relation with other quantities Dimensional formula SI unit
1. Area Length × breadth [L ´ L] =[L2 ] =[M0L2 T 0 ] m2
2. Volume Length × breadth × height [L ´ L ´ L] =[L3 ] =[M0L3 T0 ] m3
3. Density Mass é M ù = [ML–3 T0 ] kg m–3
Volume êë L3 úû

4. Specific gravity Density of body é M/L3 ù No unit


ê 3ú
= [M0L0 T0 ]
Density of water at 4° C M/L
ë û
5. Speed or velocity Distance or Displacement é L ù = [LT–1 ] = [M0LT–1 ] ms –1
Time êë T úû

6. Linear momentum Mass × velocity [MLT–1 ] =[MLT–1 ] kg ms -1


7. Acceleration Change in velocity é L / T ù = [LT–2 ] =[M0LT–2 ] ms -2
Time taken êë T úû

8. Acceleration due to gravity (g) Change in velocity é L / T ù = [LT–2 ] = [M0LT–2 ] ms -2


ê ú Time taken ë T û
9. Force (F) Mass × acceleration [M] ´ (LT–2 ] =[MLT–2 ] N (newton)
–2 –1
10. Impulse Force × time [MLT ] ´ [T] =[MLT ] Ns
11. Pressure Force/area é MLT ù
–2
–1 –2 Nm -2
ê 2 ú = [ML T ]
ë L û
12. Universal constant of Gm1m2 Fr 2 [MLT–2 ] [L2 ] Nm2 kg –2
F= or G = , where F is G= = [M–1L3 T–2 ]
gravitation (G) 2
r m1m2 [MM]
force between masses m1, m2 at a
distance r.
13. Work Force × distance [MLT–2 ] ´ [L ] = [ML2 T–2 ] J (joule)
14. Energy (including potential Work [ML2 T–2 ] J (joule)
energy, kinetic energy, heat
energy, light energy etc.)
Telegram @unacademyplusdiscounts

Units and Measurements 13

S. No. Physical quantity Relation with other quantities Dimensional formula SI unit
–2 2 –2
15. Moment of force Force × distance [MLT ] ´ [L] =[ML T ] Nm
16. Power Work é ML T ù
2 –2
2 –3 W (watt)
Time ê ú = [ML T ]
ë T û
17. Surface tension Force é MLT–2 ù 0 –2 Nm -1
Length ê ú = [ML T ]
ë L û
18. Surface energy Potential energy/Area [ML0 T–2 ] Jm -2
19. Force constant Force é MLT–2 ù 0 –2 Nm -1
Length ê ú = [ML T ]
ë L û
20. Thrust Force [MLT–2 ] N (newton)
–2
21. Tension Force [MLT ] N (newton)
22. Stress Force é MLT ù
–2
–1 –2 Nm -2
Area ê 2 ú = [ML T ]
ë L û
23. Strain Change in configuration é L ù = [M0L0 T0 ] No unit
Original configuration êë L úû

24. Coefficient of elasticity Stress é ML–1T–2 ù –1 –2 Nm–2


Strain ê ú = [ML T ]
ë 1 û

25. Radius of gyration (K) Distance [L ] = [M0LT0 ] m


26. Moment of inertia (I) Mass × (distance) 2 [ML2 ] =[ML2 T0 ] kgm2
27. Angle (q) Length(l) / Radius( r) é L ù = [M0L0 T0 ] radian
ëê L ûú
28. Angular velocity (w) Angle(q) é 1 ù = [T–1 ] = [M0L0 T–1 ] rads –1
Time (t ) êë T úû

29. Angular acceleration (a) Change in angular velocity é 1/ T ù = [T–2 ] =[M0L0 T–2 ] rads –2
Time taken êë T úû

30. Angular momentum Moment of inertia × angular velocity [ML2 ][T–1 ] =[ML2 T–1 ] kgm2 s –1

31. Torque Moment of inertia × angular [ML2 ][T–2 ] =[ML2 T–2 ] N-m
acceleration
32. Wavelength (l ) Length of one wave i.e., distance [L] =[M0LT0 ] m
–1 0 0 –1
ons/sec 33.1/Frequency n( )L T ]
T =[T ]=[M s –1 or Hz
(hertz)
34. Velocity of light in vacuum(c) Distance travelled/Time taken é L ù = [M0LT–1 ] ms –1
êë T úû

35. Velocity gradient Velocity/Distance é LT–1 ù –1 0 0 –1 s –1


ê ú = [T ] = [M L T ]
ë L û
36. Rate of flow Volume/Time é L3 ù 3 –1 0 3 –1 m3 s –1
ê ú = [L T ] [M L T ]
ëTû
37. Planck’s constant(h) Energy (E ) é ML2 T–2 ù J-s
ê –1 ú
= [ML2 T–1 ]
Frequency (n)
ë T û
38. Molar gas constant (R) Pressure ´ Volume [ML2 T–2 ] kgm2 s –2 K –1
Moles ´ Temperature [K]
39. Wien’s constant Wavelength × Temperature [M0LT0K ] mK
Telegram @unacademyplusdiscounts

14 JEE Main Physics

S. No. Physical quantity Relation with other quantities Dimensional formula SI unit
40. Avagadro’s number (N) Number of atoms/Molecules in one mol –1
gram atom/Mole 0 0 0
[M L T ]
41. Electric field strength (E) Force [MLT–2 ] NC –1
Charge [AT]
42. Capacitance (C) Charge
Potential difference [AT] F (farad)
= [ML2 T–3 A –1 ]
[ML2 T–3 A –1 ]
43. Coefficient of viscosity (h) Force [ML–1T–1 ] Pascal-econd
Velocity gradient ´ area
or poise
44. Resistance (R) Potential difference 2 –3
[ML T A ] –1
W (ohm)
= [ML2 T3 A –2 ]
Current [A]
45. Self inductance (L) dt [ML2 T–2 ] [T] H (henry)
L=e
dt [AT] [A ]

46. Magnetic induction Force [MLT–2 ] T (tesla)


= [ML0 T–2 A –1 ]
Charge ´ velocity [AT] [LT–1 ]

Hot Spot D i men s i o n al A n al y s i s


and Its Applications
Applications of dimensional analysis is the most important topic of this chapter. There are three applications of
dimensional analysis.
1. To check the correctness of a given physical 2. Derivation of formula
equation If we know the factors on which a given physical quantity may depend
As per principle of homogeneity, if the dimensions of each term on we can find a relation correlating the quantity with these factors.
both sides of a physical relation are same, then the relation is Assume the dimensions of the given physical quantity in terms of
dimensionally correct otherwise wrong. these factors, combine them to form an equation, write the
Sample Problem 21 Is the given expression of velocity of dimensions of various quantities in terms of mass ( M ), length ( L) and
æE ö time (T ) on either side of the equations.
sound given by v = ç ÷ is dimensionally correct? Using the principle of homogeneity of dimensions, equate the powers
èr ø
of M, L and T on the both sides. The three equations , so obtained are
Here, E = coefficient of elasticity, solved to obtaining the values of three unknown powers or
ρ = density of medium dimensions.
(a) Yes (b) No
E Sample Problem 22 The time period T of simple
(c) Cannot be predicted (d) The correct expression is pendulum depends upon length l of the pendulum and
r
gravitational acceleration. The formula for time period of
Interpret (a) [LHS] = [v ] = [LT –1] simple pendulum is given by
é æ E ö 1/ 2ù l g
[RHS] = ê ç ÷ ú (a) T = 2p (b) T = 2p
g l
êë è r ø úû
1 2 pl
é æ ML–1T –2 ö 1/ 2ù (c) T = lg (d) T =
2p g
= êç ÷ ú = [LT –1]
ê è ML–3 ø ú
ë û Interpret (a) Let T µ l a and T µ g b
[LHS] = [RHS] where a and b are dimensionless constants
Hence, equation is dimensionally correct. T = kl a g b
Telegram @unacademyplusdiscounts

Units and Measurements 15

where, k is dimensionless constant. As, n1 (u1) = n2(u2)


[LHS] = [ T ] = [M 0L0 T 1] or n1 [M1 L21 T1–2] = n2 [M 2 L22 T2–2]
and [RHS] = ( l a g b) = [L ]a [LT –2]b Here, M1 = kg, L1 = m, T1 = s
= [La + b T -2b ] = [M 0La + b T -2b ] M2 = g, L2 = cm, T2 = s
According to homogeneity principle, M1 = 1000 M2,
[LHS] = [RHS] L1 = 100 L2
or [M 0L0 T] = [M 0La + b T -2b ] T1 = T2 and n = 1
2 -2
For dimensional balance, dimensions on both sides should éM ù éL ù é T ù
\ n2 = n1 ê 1 ú ê 1 ú ê 1 ú
be same. ë M 2 û ë L 2 û ë T2 û
\ a+ b =0 é1000 M 2 ù é100 L 2 ù é T1 ù
2 2
7
and -2 b = 1 =1ê úê ú ê ú = 10
ë M 2 û ë L 2 û ë T2 û
1
\ b=- \ 1 joule = 10 7 erg
2
1
and a= Sample Problem 24 Consider a simple pendulum having
2
a bob attached to a string, that oscillates under the action of the
l
\ T = 2p force of gravity. Suppose that the period of oscillations of the
g simple pendulum depends on its length ( l ), mass of the bob (m)
[since, numerical value of k in case of simple pendulum is 2 p] and acceleration due to gravity ( g ). Using the method of
dimensions, expression for its time period is
3. To convert a physical quantity from one
g 2g
system to the other (a) T µ k (b) T µ k
l l
Dimensional formula is useful to convert the value of a physical l l
quantity from one system to the other. Physical quantity is expressed (c) T µ k (d) T µ k
g 2g
as a product of numerical value and unit. In any system of
measurement, this product remains constant. Interpret (c) The dependence of time period T on the
quantities l, g and m as a product may be written as
Let dimensional formula of a given physical quantity be [M aLb T c ] . If in
T = kl xg ymz
a system having base units [M1L1T1 ] the numerical value of given
quantity ( Q ) be n1 and numerical value n2 in another unit system where, k is dimensionless constant and x, y and z are the exponents.
having the base units M 2, L 2, T2 then Taking dimensions on both sides, we have
Q = n1u1 = n2u2 [L0M 0 T 1] = [L1]x [L1T –2]y [M1]z
n1 [M1aLb1 T1c ] = n2 [Ma2 Lb2 T2c ]
[M0 L0 T 1] = M zLx+ y T -2y
a b c
éM ù éL ù éT ù On equating the dimensions on both sides, we have
Þ n2 = n1 ê 1 ú ê 1 ú ê 1 ú
ë M2 û ë L 2 û ë T2 û x+ y = 0,
-2 y = 1
Sample Problem 23 In SI system, the magnitude work 1 1
Þ y = - and x =
done is joule. In another system, where the fundamental 2 2
physical quantities are in gram, centimetre and second, the and z =0
magnitude of work is
So that T = kl1/ 2g - 1/ 2
(a) 10 7 erg (b) 10 5 erg
l
(c) 10 8 erg (d) 10 10 erg or T =k
g
Interpret (a) Joule is SI unit of work. The dimensions of work
in SI. Note The value of constant k cannot be obtained by the method of
= [W1] = [M1 L21 T1–2] dimensions. Here, it does not matter if some number multiplies the right
side of this formula, because that does not affect its dimensions.
But erg is CGS unit of work. The dimensions of work in l
Actually, k = 2 p so thatT = 2p
CGS unit = [W2] = [M 2 L22 T2–2] g
Telegram @unacademyplusdiscounts

16 JEE Main Physics

Sample Problem 25 The SI unit of energy is J = kg m 2s-2, 1


Interpret (b) Given, mv 2 = mgh
-1 -2 2
that of speed v is ms and of acceleration a is ms which one
of the formula for kinetic energy given below is correct on the The dimensions of LHS are
basis of dimensional arguments. [M] [LT –1]2 = [M] [L2T –2] = [ML2T –2]
[Given m stands for the mass of body] The dimensions of RHS are
(a) K = m2v 2 (b) K = ma [M] [LT –2] [L] = [M] [L2T –2] = [ML2T –2]
1 1
(c) K = mv 2 + ma (d) K = mv 2
2 2 The dimensions of LHS and RHS are the same and hence the
equation is dimensionally correct.
Interpret (a) Every correct formula or equation must have the
same dimensions on both sides of the equation. Also, only Sample Problem 28 If force F, length L and time T be
quantities with the same physical dimensions can be added or
considered fundamental units of mass will be
subtracted. The dimensions of the quantity on the right side for (a) is
(a) [FLT -2] (b) [FL-2T -1]
K = m2v 2
(c) [FL-1T 2] (d) [F 2LT -2]
Putting, m = [M], v = [LT –1]
\dimensions are [M 2L2T –2] Interpret (c) Let [M] µ [FaLbT c ]
for (b), K = ma So, using dimensions, we have

putting m = [M], a = [LT –2] [M1 L0 T 0 ] = K [MLT –2]a [L ]b [ T ]c

\dimensions are [MLT –2] = K [M aLa + b T - 2a + b ]


We have, a = 1, a = b = 0 and k = 1
option (c) has no proper dimensions,
1 \ b = -1
option (d), K = mv 2, putting m = [M] and - 2a + c = 0
2
Þ c =2
and V = [LT –1], we have
So unit of mass is [FL–1 T 2]
K = [M] [LT –1]2 =[ML2T –2]
in units it is written as kg m2 s–2. Limitations of Theory of Dimensions
Although dimensional analysis is very useful but it is not
Sample Problem 26 The angle of 1" (second of arc) in
universal, it has some limitations as given below
radians is (Given 360° = 2 p rad, 1° = 60 ¢ and 1¢ = 60")
[NCERT] (i) This method gives no information about dimensional
–2
(a) 1.745 ´ 10 rad constants. Such as universal constant of gravitation (G)
or Planck’s constant (h) and where they have to be
(b) 2.91 ´ 10–4 rad
introduced.
(c) 4.85 ´ 10–6 rad
(d) 3.42 ´ 10–3 rad (ii) Numerical constant (k), having no dimensions such as
3/4, e, 2p etc., cannot be deduced by the method of
Interpret (c) Given, 360° = 2p rad dimensions.
æ p ö –2 (iii) This technique is useful only for deducing and
1° = ç rad = 10
÷1.745 ´ rad
è180 ø verifying power relations. Relationship involving
exponential, trignometric functions etc., cannot be
1° = 60 ¢ = 1.745 ´ 10 –2 rad
obtained or studied by this technique.
1¢ = 2.90 s ´ 10 –4 rad ~
- 2.91 ´ 10 –4 rad (iv) In this method, we compare the powers of
1¢ = 60" = 2.908 ´ 10 –4 rad fundamental quantities (like M,L,T etc.,) to obtain a
numbers of independent equations to find the
1" = 4.87 ´ 10 –4 rad ~
- 4.85 ´ 10 –6 rad
unknown powers. Since, the total number of such
equations cannot exceed the number of fundamental
1
Sample Problem 27 Given equation mv 2 = mgh, where quantities we cannot use this method to obtain the
2 required relation if the quantity of interest depends
m is the mass of the body, v is velocity, g is the acceleration due upon more parameters than the number of
to gravity and h is the height. Then the given equation is fundamental quantities used.
(a) dimensionally incorrect
(v) Even if a physical quantity depends on three physical
(b) dimensionally correct quantities, out of which two have same dimensions,
(c) wrong the formula cannot be derived by theory of
(d) None of above dimensions.
Telegram @unacademyplusdiscounts

WORKED OUT
Examples
Solution Energy = Force × distance
Example 1 The ratio of one micron to one nanometer is
\ Its unit will be kgm2s-2
(a) 10 3 (b) 10 -3
(c) 10 -6 (d) 10 -9 Energy does not have the units of kg ms -1.

One micron 10 -6 Example 7 What are the units of magnetic permeability?


Solution = -9 = 10 3
One nanometre 10 (a) WbA-1m-1 (b) Wb -1Am
(c) WbAm-1 (d) WbA-1m
Example 2 Magnetic intensity is measured in m 0 idl sin q
(a) Am-1 (b) tesla Solution From Biot-Savart's law, B =
4p r2
(c) gauss (d) weber
4pBr 2
idl sin q Am m0 =
Solution B= = 2 = Am-1 idl sin q
r2 m
tesla m2
Magnetic intensity is measured in Am-1 \Unit of magnetic permeability = = WbA -1m-1
Am

Example 3 Nuclear cross-section is measured in barn which Example 8 The dimensions of electromotive force in terms
is equal to of current A are
(a) 10 -20 m2 (b) 10 -28 m2 (a) [MT -2A-2] (b) [ML2T -2A2]
-30 2 -14 2
(c) 10 m (d) 10 m (c) [ML2T -2A-2] (d) [ML2T -3 A-1]

Solution 1 barn = 10 -28 m2 Solution Electromotive force = potential difference


W [ML2T -2]
30 V= = = [ML2T -3 A -1]
Example 4 The order of 2 is approximately q [AT]
(a) 10 9 (b) 10 5
(c) 10 15 (d) 10 20 Example 9 Which of the following is a dimensional
constant?
Solution (2)30 = 1073741824 = 10 9 (a) Refractive index
(b) Dielectric constant
Example 5 Units of magnetic flux is
(c) Relative density
(a) weber/metre
(d) Gravitational Constant
(b) newton metre/ampere
(c) joule × coulomb / metre Solution Gravitational constant G has a constant value and
(d) tesla dimensions. In SI system value of gravitational constant G is
6.6 ´ 10 -11 Nm2 kg -2. Dimensional formula of G is [M -1L3 T -2].
F newton ´ m2
Solution Magnetic flux f = B ´ A = ´ A=
il ampere ´ m Example 10 The dimensions of solar constant are
0 0 0
(a) [M L T ] (b) [MLT -2]
Example 6 Which of the following quantities has not been 2 -3
(c) [ML T ] (d) [ML-3 ]
expressed in proper units?
Stress
(a) = Nm-2 (b) Surface tension = Nm -1 Solution Solar constant = Energy / sec × area
Strain
[ML2T -2]
(c) Energy = kgms-1 (d) Pressure = Nm-2 = [MT -3 ]
[TL2]
Telegram @unacademyplusdiscounts

18 JEE Main Physics

Example 11 The physical quantity denoted by Solution Let X = M aLbT c


mass ´ pressure DX
is æ aDM bDL cDT ö
density ´ 100 = ç + + ÷ ´ 100
X è M L T ø
(a) force (b) work = ( aa + bb + cg)%
(c) momentum (d) angular momentum
mass ´ pressure [M] ´ [ML-1T -2] Example 16 The significant figures in 300.500 are
Solution Given, = (a) 6 (b) 5
density [ML-3 ]
(c) 4 (d) 2
= [ML2T -2]
Which represents work. Solution As per rules, all zeros in the given number are
significant. Therefore, number of significant figures is 6.
Example 12 The density of a cube is measured by
measuring its mass and length of its sides. If the maximum Example 17 The result after adding 3.8 ´ 10 -6 to
errors in the measurement of mass and length are 3% and 2% 4.2 ´ 10 -5
with due regard to significant figures is
respectively, then the maximum error in the measurement of (a) 4.58 ´ 10 -5
density is
(b) 0.458 ´ 10 -4
(a) 7% (b) 5% (c) 1% (d) 9%
(c) 4.6 ´ 10 -5
Solution We know density, (d) 45.8 ´ 10 -6
M [M]
r= = Solution 3.8 ´ 10 -6 + 4.2 ´ 10 -5
V [L3 ]
Dr DM DL = (3.8 ´ 10 -1 + 4.2 ) ´ 10 -5
\ ´ 100 = ´ 100 + 3 ´ 100
r M L = (0.38 + 4.2) ´ 10 -5
= 3% + 3 (2%) = 9% = ( 4.58) ´ 10 -5
Example 13 The mass of a box measured by a grocer's Rounding off to one place of decimal.
balance is 2.3 kg. Two gold pieces of masses 20.15 g and 20.17 g The sum = 4.6 ´ 10 -5
are added to the box. The total mass of the box is
(a) 2.30 kg (b) 2.340 kg (c) 2.34 kg (d) 2.3 kg Example 18 Which of the following numerical values have
Solution Total mass = (2.3 + 0.02015 + 0.02017) kg
three significant figures?
(a) 3.033 (b) 0.030
= 2.34032 kg, upto one decimal place
(c) 30.30 (d) 0.300
= 2.3 kg
Solution Options (a) and (c) have four significant figures; (b)
Example 14 When a current of (2.5 ± 0.5) A flows through a has two and (d) has three significant figures.
wire, it developes a potential difference of (20 ± 1) V . The
resistance of wire is Example 19 What is the number of significant figures in
(a) (8 ± 2) W (b) (8 ± 1.6) W (3.20 + 4.80) ´ 10 5?
(c) (8 ± 1.5) W (d) (8 ± 3) W (a) 5 (b) 4
V 20 (c) 3 (d) 2
Solution We know resistance R = = =8W
I 2.5
DR DV DI 1 0.5 1 Solution (3.20 + 4.80) ´ 10 5 = 8.00 ´ 105
= + = + =
R V I 20 2.5 4 Number of significant figures is 3.
1
DR = ´ 8 = 2 W = R (8 ± 2) W Example 20 Subtract 0.2 J from 7.26 J and express the result
4
with correct number of significant figures.
Example 15 A quantity is represented by x = M aLbT c . The (a) 7.1 J (b) 7.06 J
percentage error in measurement of M, I, and T are a%, b% and (c) 7.0 J (d) 7J
g% respectively. The percentage error in X would be
(a) ( aa + bb + gc)% (b) ( aa - bb + gc)% Solution Subtraction is correct upto one place of decimal,
corresponding to the least number of decimal places.
(c) ( aa - bb - gc) ´ 100% (d) None of these
7.26 - 0.2 = 7.06 = 7.1J.
Telegram @unacademyplusdiscounts

Start Practice for


JEE Main
Round I (Topically Divided Problems)

Unit & Dimensional Formulae 9. Which one of the following pairs of quantities and
1. The SI unit of electrochemical equivalent is their unit is properly matched?
-1 (a) Electric field-coulomb/m
(a) kg C (b) C kg
(b) Magnetic flux- Weber/m2
(c) kg C -1 (d) kg2 C-1
(c) Power-Farad
2. The sum of numbers 436.32, 227.2 and 0.301 in (d) Capacitance-Henry
appropriate significant figures in [NCERT]
10. The mean length of an object is 5 cm. Which is the
(a) 663.821 (b) 664
following measurements is most accurate?
(c) 663.8 (d) 663.82
[NCERT Exemplar]
3. A sextant is used to measure
(a) 4.9 cm (b) 4.805 cm (c) 5.25 cm (d) 5.4 cm
(a) area of hill
(b) height of an object 11. Energy per unit volume represents
(c) breadth of a tower (a) pressure (b) force (c) thrust (d) work
(d) volume of the building 12. Which of the following pairs of physical quantities
4. You measure two quantities as does not have same dimensional formula? [NCERT]
A = 10 . m. We should report
. m ± 0.2m, B = 2.0m ± 02 (a) Work and torque
correct value for AB as [NCERT Exemplar] (b) Angular momentum and Planck’s constant
(a) 1.4 m ± 0.4 m (b) 1.41 m ± 0.15 m (c) Tension and surface tension
(c) 1.4 m ± 0.3 m (d) 1.4 m ± 0.2 m (d) Impulse and linear momentum
5. A pressure of 106 dyne cm -2 is equivalent to 13. The surface tension of mercury is 32 dyne cm -1. Its
5 -2 4 -2
(a) 10 Nm (b) 10 Nm value in SI units is
-2
(a) 0(c) 106 Nm(b)
. 032 0 32. (c) 3200 (d) 32000
6. Universal time is based on 14. In the relation y = r sin ( wt - kx), the dimensional
(a) rotation of earth on its axis formula of w/ k are
(b) oscillations of quartz crystal (a) [M0L0 T 0 ] (b) [M0L1T -1 ] (c) [M0L0 T1 ] (d) [M0L1T 0 ]
(c) vibrations of cesium atom
(d) earth's orbital motion around the sun
15. One light year is defined as the distance travelled by
light in one year. The speed of light 3 ´ 108 ms -1. The
7. Which of the following cannot be regarded as an same in metre is
essential characteristic of a unit of measurement?
(a) 3 ´ 1012 m (b) 9.461 ´ 1015 m
(a) Inaccessibility (b) Indenstructibility
(c) 3 ´ 1015 m (d) None of these
(c) Invariability (d) Reproductibility
16. One slug is equivalent to 14.6 kg. A force of 10 pound
8. Young modulus of steel is 1.9 ´ 1011 N/m2 . When is applied on a body of 1 kg. The acceleration of the
2
expressed in CGS units of dyne/cm , it will be equal
body is
to (1 N = 105 dyne, 1 m 2 = 104 cm 2) [NCERT]
(a) 44.5 ms -2 (b) 4.448 ms -2
(a) 1.9 ´ 1010 (b) 1.9 ´ 1011
(c) 44.4 ms -2 (d) None of these
(c) 1.9 ´ 1012 (d) 1.9 ´ 1013
Telegram @unacademyplusdiscounts

20 JEE Main Physics

17. If the acceleration due to gravity is 10 ms -2 and the = a n sin -1 æç - 1ö÷. The value of n is
dx x
units of length and time are changed in kilometre
27. ò 2ax - x2 èa ø
and hour respectively, the numerical value of (a) 0 (b) –1
acceleration is (c) 1 (d) None of these
(a) 360000 (b) 72000 (c) 36000 (d) 129600 You may use dimensional analysis to solve the problem.
18. One amu is equivalent to 931 MeV energy. The rest 28. SI unit of intensity of wave is
mass of electron is 9.1 ´ 10-31 kg. The mass energy is (a) J m -2 s -1 (b) J m -1s -2
(1 amu = 1.67 ´ 10-17 kg) (c) W m -2 (d) J m -2
(a) 0.5073 MeV (b) 0.693 MeV
29. A suitable unit for gravitational constant is
(c) 4.0093 MeV (d) None of these
(a) kg-m s -1 (b) Nm -1 s
19. The value of universal gas constant is (c) Nm2 kg –2 (d) kg m s –1
R = 8.3 J/ k-mol. The value of R in atmosphere litre
per kelvin per mol 30. If L denotes the inductance of an inductor through
(a) 8.12 (b) 0.00812 (c) 81.2 (d) 0.0812 which a current I is flowing, then the dimensional
formula of LI2 is
20. Electron-volt is the unit of energy
(a) [MLT -2 ]
(1 eV = 1.6 ´ 10-19 J). In H-atom, the binding energy of
(b) [ML2 T -2 ]
electron in first orbit is 13.6 eV. The same in joule (J) is
(c) [M2 L2 T -2 ]
(a) 10 ´ 10 -19 J (b) 21.76 ´ 10 -19 J
(d) not expressible in terms of M, L, T
(c) 13.6 ´ 10 -19 J (d) None of these
31. One yard in SI unit is equal
21. The expression for centripetal force ( F ) depends upon (a) 1.9144 m (b) 0 .9144 m
mass of body ( m), speed (v) of the body and the radius (c) 0.09144 km (d)1.0936 km
( r) of circular path will be expression for centripetal
32. The equation of alternating current is I = I 0 e- t / CR
force
where t is time, C is capacitance and R is resistance of
mv2 mv2
(a) F = (b) F = coil, then the dimensions of C R is
2r3 r
(a) [MLT -1] (b) [M 0 LT]
mv2 m2 v2 (c) [M 0 L 0 T] (d) None of these
(c) F = 2 (d) F =
r 2r
33. Which of the following pairs has same dimensions?
22. The damping force of an oscillating particle is (a) Current density and charge density
observed to be proportional to velocity. The constant (b) Angular momentum and momentum
of proportionality can be measured in (c) Spring constant and surface energy
(a) kg s–1 (b) kg s (d) Force and torque
(c) kg ms–1 (d) kg m–1 s–1
34. How many wavelengths of Kr86 are there in one
23. The fundamental unit, which has the same power in metre?
the dimensional formulae of surface tension and (a) 1553164.13 (b) 1650763.73
viscosity is (c) 652189.63 (d) 2348123.73
(a) mass (b) length
(c) time (d) None of these
35. Taking frequency f, velocity v and density r to be the
fundamental quantities, then the dimensional
24. The mass and volume of a body are 4.237 g and formula for momentum will be
2.5 cm 3 respectively. The density of material of the (a) [rv 4f -3 ] (b) [rv3f -1 ]
body in correct significant figures is. [NCERT]
(c) [rvf 2 ] (d) [r2 v2f 2 ]
(a) 1.6048 g cm -3 (b) 1.69 g cm -3
(c) 1.7 g cm -3 (d) 1.695 g cm -3 36. If p represents radiation pressure, c represents speed
of light and q represents radiation energy striking a
25. What is the power of a 100 W bulb in CGS units? unit area per second, then non-zero integers a, b and
(a) 106 ergs–1 (b) 107 ergs–1 c are such that paqbc c is dimensionless, then
(c) 109 ergs–1 (d) 1011 ergs–1
(a) a = 1, b = 1, c = -1
26. If the units of M and L are increased three times, (b) a = 1, b = -1, c = 1
then the unit of energy will be increased by (c) a = -1, b = 1, c = 1
(a) 3 times (b) 6 times (c) 27 times (d) 81 times (d) a = 1, b = 1, c = 1
Telegram @unacademyplusdiscounts

Units and Measurements 21

37. Farad is not equivalent to 47. The dimensional formula of the ratio of angular to
(a)
q
(b) qV2
linear momentum is
V (a) [M 0 LT 0 ] (b) [MLT]
q2 J (c) [ML2 T -1] (d) [M -1L -1T -1]
(c) (d) 2
J V 48. The maximum static friction on a body is F = mN .
(q = coulomb, V = volt and J = joule) Here, N = normal reaction force on the body
38. In the equation y = a sin ( wt + kx), the dimensional m = coefficient of static friction. The dimensions ofm are
formula of w is (a) [MLT -2 ] (b) [M 0 L 0 T 0 q-1]
(a)[M0L0T–1] (b) [M0LT–1] (c) dimensionless (d) None of these
(c) [ML0T0] (d) [M0L–1T0] 49. One mole of an ideal gas at standard temperature
39. A new unit of length is chosen such that the speed of and pressure occupies 22.4 L (molar volume). The
light in vacuum is unity. Then the distance between ratio of molar volume to the atomic volume of a mole
the sun and the earth in terms of the new unit, if light of hydrogen? (Take the size of hydrogen molecule to
takes 8 min and 20 s to cover this distance? be about 1 Å) [NCERT]
[NCERT] (a) 9.1 ´ 10 4 (b) 6 ´ 10 4
(a) 300 new unit of length (b) 500 new unit of length (c) 7.1 ´ 10 4 (d) 8.1 ´ 105
(c) 600 new unit of length (d) None of these
50. If I is the moment of inertia and w the angular
40. The dimensiona l formula of magnetic permeability
velocity, what is the dimensional formula of
is
rotational kinetic energy
(a) [M0L-1T] (b) [M0L2 T -1 ]
(a) [ ML2 T -1] (b) [ M2 L -1T -2 ]
(c) [M0L2 T -1A2 ] (d) [MLT -2 A -2 ]
(c) [ML2 T -2 ] (d) [M2 L -1T -2 ]
41. [ML-2T -2 ] represents dimensional formula of which 51. A gas bubble from an explosion under water
of the following physical quantities? oscillates with a time period T, depends upon static
(a) Energy (b) Pressure pressure p, density of water r and the total energy of
(c) Torque (d) Pressure gradient explosion E. The expression for the time period T.
42. The period of a body under SHM is respected by (where, k is a dimensionless constant) is
T = pa DbS c , where p is pressure, D is density and S (a) T = kp -5 /6r1 /2 E 1 /3 (b) T = kp -4 /7r1 /2 E 1 /3
is surface tension. The value of a, b and c are (c) T = kp -5 /6r1 /2 E1 /2 (d) T = kp -4 /7r1 /3E 1 /2
3 1
(a) - , ,1 (b) -1,-2, 3 52. Solar constant is defined as energy received by earth
2 2
1 -3 -1 1 per cm2 per minute. The dimensions of solar constant
(c) , , (d) 1, 2,
2 2 2 3 are
(a) [ML2 T -3] (b) [M2 L 0 T -1] (c) [ML 0 T -3] (d) [MLT -2 ]
43. The length, breadth and thickness of a rectangular
sheet of metal are 4.234 m, 1.005 m and 2.01 cm 53. Electric displacement is given by D = eE,
respectively. The area and volume of the sheet to Here, e = electric permittivity
correct significant figures are [NCERT] E = electric field strength
(a) 8.72 m2 and 0.0855 m3 The dimensions of electric displacement are
(b) 8.7 m2 and 0.085 m3 (a) [ML–2TA] (b) [L–2T –1A]
(c) 0.87 m2 and 0.855 m3 (c) [L–2TA] (d) None of these
(d) 0.087 m2 and 0.0855 m3 54. The work done by a battery is W = e Dq, where Dq
44. The dimensions of emf in MKS is charge transferred by battery, e = emf of the battery.
(a) [ML–1T–2Q–2] (b) [ML2T –2Q–2] What are dimensions of emf of battery?
(c) [MLT –2Q–1] (d) [ML2T –2Q–1] (a) [M 0 L 0 T -2 A -2 ] (b) [ML2 T -3A -2 ]
45. The physical quantity which has the dimensional (c) [M2 L 0 T -3A 0 ] (d) [ML2 T -3A -1]
formula [M1T -3] is 55. In the formula, a = 3 bc2 , a and c have dimensions of
(a) surface tension (b) density electric capacitance and magnetic induction
(c) solar constant (d) compressibility respectively. What are dimensions of b in MKS
46. Force constant has same dimensions as system?
(a) coefficient of viscosity (b) surface tension (a) [M -3L -2 T 4 Q 4 ] (b) [M -3T 4 Q 4 ]
(c) frequency (d) impulse (c) [M -3T3Q] (d) [M -3L2 T 4 Q -4 ]
Telegram @unacademyplusdiscounts

22 JEE Main Physics

56. The dimensions of the power of lens are 63. If C is the restoring couple per unit radian twist and I
(a) [LT ]-2 0 -1 0
(b) [M L T ] is the moment of inertia, then the dimensional
0 0 0
(c) [M L T ] (d) None of these I
representation of 2p will be
2 -3 -2
C
57. [ML T A ] is the dimensional formula of (a) [M0L0 T -1 ] (b) [M0L0 T]
(a) Electric resistance -1
0
(c) [M LT ] (d) [ML2 T -2 ]
(b) Capacity
(c) Electric potential 64. The velocity v of water waves may depend on their
(d) Specific resistance wavelength (l), the density of water (r) and the
58. The concorde is the fastest airlines used for acceleration due to gravity ( g ). The method of dimensions
commercial service. It can cruise at 1450 mile per gives the relation between these quantities as
hour (about two times the speed of sound or in other (a) v2 µ l-1r -1 (b) v2 µ gl
words mach 2). What is it in m/s? (c) v2 µ glr (d) g -1 µ l3
(a) 644.4 m/s (b) 80 m/s 65. If E, m, J and G represent energy, mass, angular
(c) 40 m/s (d) None of these momentum and gravitational constant respectively,
59. Which of the following is the most precise device for then the dimensional formula of EJ2/m5G2 is
measuring length? [NCERT] (a) [MLT –2] (b) [M0L0 T ]
(a) A vernier callipers with 20 divisions on the sliding scale (c) [M0L2T0] (d) dimensionless
(b) A screw gauge of pitch 1 mm and 100 divisions on the
66. Crane is British unit of volume (one crane = 170.4742).
circular scale
Convert crane into SI units.
(c) An optical instrument that can measure length to within a
wavelength of light? (a) 0.170474 m3 (b) 17.0474 m3
(d) All are equally precise device for measuring length (c) 0.00170474 m3 (d) 1704.74 m3

60. A student measures the thickness of a human hair by 67. The wavelength associated with a moving particle
looking at it through a microscope of magnification depends upon power p of its mass m, qth power of its
100. He makes 20 observations and finds that the velocity v and rth power of Planck’s constant h. Then
average width of the hair in the field of view of the the correct set of values of p, q and r is
microscope is 3.5 mm. The thickness of hair is[NCERT] (a) p = 1, q = -1, r = 1 (b) p = 1, q = 1, r = 1
(a) 0.035 mm (b) 0.04 mm (c) p = -1, p = -1, r = -1 (d) p = -1, q = -1, r = 1
(c) 0.35 mm (d) 0.40 mm 68. The time taken by an electron to go from ground state
61. The photograph of a house occupies an area of to excited state is one shake (one shake = 10–8s). This
2
1 . 75 cm on a 35 mm slide. The slide is projected on to time in nanosecond will be
a screen and the area of the house on the screen is (a) 10 (b) 4 (c) 2 (d) 25
1.55 m 2 . The linear magnification of the
projector-screen arrangement, is [NCERT] Error and Measurement
(a) 84.1 (b) 96.1 69. If x = a - b, then the maximum percentage error in
(c) 94.1 (d) 86.1 the measurement of x will be
62. A highly rigid cubical block A of small mass M and æ Da + Db ö
(a) ç ÷ ´ 100%
side L is fixed rigidly on to another cubical block of è a-b ø
same dimensions and of low modulus of rigidity h æ Da Db ö
(b) ç - ÷ ´ 100%
such that the lower face of A completely covers the è a bø
upper face of B. The lower face of B is rigidly held on a æ Da Db ö
(c) ç + ÷ ´ 100%
horizontal surface. A small force F is applied è a - a a - bø
perpendicular to one of the side faces of A. After the æ Da Db ö
force is withdrawn, block A executes small (d) ç - ÷ ´ 100%
è a - a a - bø
oscillations, the time period of which is given by
Mh 70. If X = A ´ B and DX , DA and DB are maximum
(a) 2p MhL (b) 2p absolute errors in X, A and B respectively, then the
L
maximum relative error in X is given by
ML M
(c) 2p (d) 2p (a) DX = DA + DB (b) DX = DA - DB
h hL
DX DA DB DX DA DB
(c) = - (d) = +
X A B X A B
Telegram @unacademyplusdiscounts

Units and Measurements 23

71. The percentage errors in the measurement of mass 80. The internal and external diameters of a hollow
and speed are 2% and 3% respectively. How much cylinder are measured with the help of a
will be the maximum error in the estimate of kinetic vernier callipers. Their values are 4.23 ± 0.01 cm and
energy obtained by measuring mass and speed? 3.87 ± 0.01 cm respectively. The thickness of the wall of
(a) 11% (b) 8% (c) 5% (d) 1% the cylinder is
72. Error in the measurement of radius of sphere is 2%. (a) 0.36 ± 0.02 cm (b) 0.18 ± 0.02 cm
The error in the measurement of volume is (c) 0.36 ± 0.01 cm (d) 0.18 ± 0.01cm
(a) 1% (b) 5% (c) 3% (d) 6% 81. The density of the material of a cube is measured by
73. There are atomic clocks capable of measuring time measuring its mass and length of its side. If the
with an accuracy of 1 part in 1011.
If two such clocks maximum errors in the measurement of mass and
are operated with precision, then after running for the length are 3% and 2% respectively, the maximum
5000 yr, these will record error in the measurement of density is
(a) a difference of nearly 2s (a)1% (b) 5%
(b) a difference of 1 day (c) 7% (d) 9%
(c) a difference of 1011 s 82. When the planet Jupiter is at a distance of 824.7
(d) a difference of 1 yr million km from the earth, its angular diameter is
74. If there is a positive error of 50% in the measurement measured to be 35.72¢ ¢ of arc. The diameter of Jupiter
of speed of a body, then the error in the measurement can be calculated as [NCERT]

of kinetic energy is (a) 1329 ´ 107 km (b) 1429 ´ 105 km


(a) 25% (b) 50% (c) 100 % (d) 125% (c) 929 ´ 105 km (d) 1829 ´ 105 km
75. The radius of the sphere is (4.3 ± 0.1) cm. The 83. In an experiment, we measure quantities a, b and c.
percentage error in its volume is ab2
Then x is calculated from the formula, x = . The
0.1 0.1 ´ 100 c3
(a) ´ 100 (b) 3 ´
4.3 4.3 percentage errors in a, b, c are ±1%, ±3% and ±2%
1 0.1 ´ 100 0.1 ´ 100
(c) ´ (d) 3 + respectively. The percentage error in x can be
3 4.3 4.3 (a) ±1% (b) ±4%
76. A public park, in the form of a square, has an area of (c) 7% (d) ±13%
(100 ± 0.2) m 2 . The side of park is 84. The time dependence of a physical quantity P is given
(a) (10 ± 0.01)m (b) (10 ± 0.1)m 2
by P = P0e -at , where a is a constant and t is time.
(c) (10.0± 0.1)m (d) (10.0 ± 0.2)m Then constant a is
77. The specific resistance r of a circular wire of radius r, (a) dimensionless (b) dimension of t–2
pr2 R (c) dimensions of P (d) dimension of t2
resistance R and length l is given by r = .
l 85. The least count of a stop watch is 0.2 s. The time of
Given, r = 0: (24 ± 0.02) cm, R = (30 ± 1) W and 20 oscillations of a pendulum is measured to be 25 s.
l = (4.80 ± 0.01) cm. The percentage error in r is nearly The percentage error in the measurement of time
(a) 7% (b) 9% will be
(c) 13% (d) 20% (a) 8% (b) 1.8% (c) 0.8 % (d) 0.1%
78. The initial temperature of a liquid is (80.0 ± 0.1)° C. 86. The pressure on a square plate is measured by
After it has been cooled, its temperature is measuring the force on the plate and the length of the
(10.0 ± 0.1)° C. The fall in temperature in degree F
sides of the plate by using the formula p = 2 .If the
centigrade is l
(a) 70.0 (b) 70.0 ± 0.3 maximum errors in the measurement of force and
(c) 70.0 ± 0.2 (d) 70.0 ± 0.1 length are 4% and 2% respectively, then the
maximum error in the measurement of pressure is
79. A physical quantity is represented by X = M a L bT - c .
(a) 1% (b) 2% (c) 8% (d) 10%
If percentage errors in the measurements of M, L and
T are a%, b% and g% respectively, then total percentage 87. Given, potential difference V = (8 ± 05
. ) V and current
error is I = (2 ± 0.2)A. The value of resistance R is
(a) ( aa + bb - gc ) % (b) ( aa + bb + gc ) % (a) 4 ± 16.25% (b) 4 ± 6.25%
(c) ( aa - bb - gc ) % (d) 0% (c) 4 ± 10% (d) 4 ± 8%
Telegram @unacademyplusdiscounts

24 JEE Main Physics

88. The length, breadth and thickness of a block is Df Du Dv


(a) = +
measured to be 50 cm, 2.0 cm and 1.00 cm. The f u v
percentage error in the measurement of volume is Df 1 1
(b) = +
(a) 0.8% (b) 8% f Du / u Dv / v
(c) 10% (d) 12.5 % Df Du Dv D(u + v )
(c) = + +
f u v u+v
89. Given p = 3.14. The value of p2 with due regard for Df Du Dv Du Dv
significant figures is (d) = + + +
f u v u+v u+v
(a) 9.86 (b) 9.859
(c) 9.8596 (d) 9.85960 95. The measured mass and volume of a body are 23.42 g
and 4.9 cm3 respectively with possible error 0.01 g
90. One side of a cubical block is measured with the help
and 0.1 cm3. The maximum error in density is nearly
of a vernier callipers of vernier constant 0.01 cm.
(a) 0.2% (b) 2%
This side comes out to be 1.23 cm. What is the
(c) 5% (d) 10%
percentage error in the measurement of area?
1.23 0.01 T
(a) ´ 100 (b) ´ 100 96. The velocity of transverse wave in a string is v =
0.01 1.23 M
0.01 0.01 where T is the tension in the string and M is mass per
(c) 2 ´ ´ 100 (d) 3 ´ ´ 100
1.23 1.23 unit length. If T = 3.0 kgf, mass of string is 2.5 g and
91. A physical quantity P is related to four observables length of string is 1.00m, then the percentage error in
3 2
a, b, c and d are as follows P = a b / cd the measurement of velocity is
(a) 0.5 (b) 0.7
The percentage errors of measurement in a, b, c and d
(c) 2.3 (d) 3.6
are 1%, 3%, 4% and 2% respectively. What is the
percentage error in the quantity P, if the value of P 97. The unit of length convenient on the atomic scale is
calculated using the above relation turns out to be known as an angstrom and is denoted by Å. 1 Å =
3.763, to what value should you round-off the result ? 10-10 m. The size of the hydrogen atom is about 0.5 Å.
[NCERT]
The total atomic volume in m 3 of a mole of hydrogen
atoms would be [NCERT]
(a) 13% and 3.8 (b) 1.3% and 0.38
(c) 1.3% and 3.8 (d) 3.8% and 13 (a) 3.15 ´ 10 -7 m3 (b) 3.0 ´ 10 -8 m3
(c) 3.85 ´ 10 -7 m3 (d) 2.85 ´ 10 -7 m3
92. Length is measured in metre and time in second as
usual. But a new unit of mass is so chosen that G = 1. 98. The relative density of the material of a body is the
This new unit of mass is equal to ratio of its weight in air and the loss of its weight in
(a) 1.5 × 107 kg (b) 1.5 × 1010 kg water. By using a spring balance, the weight of the
(c) 6.67 ×10 –11 kg (d) 6.67 × 10–8 kg body in air is measured to be 500 . N. The weight
. ± 005
93. The length, breadth and thickness of a metal block is of the body in water is measured to be 4.00 ± 005 . N.
given by l = 90 cm, b = 8 cm, t = 2.45 cm. The volume of Then, the maximum possible percentage error in
the block is relative density is
(a) 2 × 102 cm3 (b) 1.8 × 102 cm3 (a) 11% (b) 10%
2
(c) 1.77 × 10 cm 3 (d) 1.764 × 102 cm2 (c) 9% (d) 7%
1 1 1 99. The length l, breadth b and thickness t of a block are
94. The focal length of a mirror is given by = +
f u v measured with the help of a metre scale. Given
where u and v represent object and image distances l =15.12 ± 0.01 cm, b = 10.15 ± 0.01 cm, t = 5.28 ± 0.01 cm.
respectively. The percentage error in volume is
The maximum relative error in f is (a) 0.64% (b) 0.28%
(c) 0.37% (d) 0.48%
Telegram @unacademyplusdiscounts

Units and Measurements 25

Round II (Mixed Bag)

Only One Correct Option unit volume for the values x1 and x2 of x respectively.
1. The circular divisions of shown screw guage are 50. It Then the dimensional formula of diffusion constant
moves 0.5 mm on main scale in one rotation. The D is
diameter of the ball is (a) [M0LT 0 ] (b) [M0L2 T -4 ] (c) [M0LT -3 ] (d) [M0L2 T -1 ]
1 e2
10 30 9. The dimensional formula of is
5
0
25
20
e 0 hc
(a) [M0L0 T 0 A 0 ] (b) [M-1L3T 2 A]
3 -4 -2
(c) [ML T A ] (d) [M-1L-3T 4 ]
10. The radius of the proton is about 10–15 m. The radius
(a) 2.25 mm (b) 1.20 mm of the observable universe is 1026 m. Identify the
(c) 2.20 mm (d) 1.25 mm distance which is half-way between, these two
2. Two quantities A and B are related by the relation extremes on a logarithmic scale.
A
= m, where m is linear mass density and A is force. (a) 1021 m (b) 106 m (c) 10–6 m (d) 100m
B 3 1/ 2
11. Given X = (Gh / c ) , where G, h and c are
The dimensions of B will be gravitational constant, Planck’s constant and the
(a) same as that of squared speed velocity of light respectively. Dimensions of X are the
(b) same as that of pressure same as those of
(c) same as that of work (a) mass (b) time
(d) same as that of momentum (c) length (d) acceleration
3. In the equation X = 3 YZ 2 , X and Z have dimensions 12. The dimensional formula of coefficient of
of capacitance and magnetic induction respectively. permittivity for free space ( e 0 ) is
In MKSQ system, the dimensional formula of Y is (a) [ML3A -2 T -4 ] (b) [M–1L–3T4A2]
-3 -2 -2 -4
(a) [M L T Q ] (b) [ML–2] -1 -3 -2 -4
(c) [M L A T ] (d) [ML3A2 T -4 ]
-3 -2 4 8 -3 -2 4 4
(c) [M L Q T ] (d) [M L Q T ]
13. The thrust developed by a rocket-motor is given by
2
4. Given that r = m sin pt, where t represents time. If F = mv + A( p1 - p2 ), where m is the mass of the gas
the unit of m is N, then the unit of r is ejected per unit time, v is velocity of the gas, A is area of
(a) N (b) N2 (c) N-s (d) N2s cross-section of the nozzle, p1, p2 are the pressures of the
exhaust gas and surrounding atmosphere. The formula is
5. When a wave transverses a medium the displacement
dimensionally
of a particle located at x at a time t is given by
(a) correct
y = a sin ( bt - cx) , where a, b and c are constants of
(b) wrong
the wave. Which of the following is dimensionless?
(c) sometimes wrong, sometimes correct
y b
(a) (b) bt (c) cx (d) (d) data is not adequate
a c
14. What is the unit of k in the relation where,
6. In a new system of units, unit of mass is 10 kg, unit of ky
U= where U represents the potential energy,
length is 1 km and unit of time is 1 min. The value of y2 + a 2
1 J in this new hypothartical system is
y represents the displacement and a represents
(a) 3.6 ×10–4 new units (b) 6 × 107new units amplitude?
(c) 1011 new units (d) 1.67 ×104 new units
(a) m s–1 (b) m s (c) J m (d) J s–1
7. [ML3T -1Q-1 ] is the dimensional formula of 15. A calorie is a unit of heat and equals 4.2 J. Suppose
(a) resistance (b) resistivity we employ a system of units in which the unit of mass
(c) capacitance (d) conductivity is a kg, the unit of length is b metre and the unit of time
n2 - n1 is g s. In this new system, 1 calorie =
8. The number of particles given by n = D are
x2 - x1 (a) a -1b -2 g 2 (b) 4.2 ab 2 g -2
crossing a unit area perpendicular to x-axis in unit (c) ab 2 g 2 (d) 4.2 a -1b -2 g 2
time, where n1 and n2 are the number of particles per
Telegram @unacademyplusdiscounts

26 JEE Main Physics

16. Let us choose a new unit of length such that the 25. The SI unit of length is metre. Suppose we adopt a
velocity of light in vacuum is unity. If light takes new unit of length which equal x metre. The area of
8 min and 20 s to cover the distance between sun and 1 m2 expressed in terms of the new unit has a
earth, this distance in terms of the new unit is magnitude
(a) 5 (b) 50 (c) 500 (d) 3 × 108 (a) x (b) x2
17. For the equation F µ A v d , where F is the force, A is
a b c (c) x -1 (d) x–2
the area, v is the velocity and d is the density, the 26. The following observations were take for
values of a, b and c are respectively determining surface tension of water by capillary
(a) 1, 2, 1 (b) 2, 1,1 (c) 1, 1, 2 (d) 0, 1,1 tube method. Diameter of capillary, D = 1.25 × 10–2 m
18. An important milestone in the evolution of the and rise of water in capillary, h = 1.46 × 10–2 m.
universe just after the Big Bang is the Planck time t p , Taking g = 9.80 ms –2 and using the relation
the value of which depends on three fundamental T = ( rgh / 2) ´ 103 Nm-1, what is the possible error in
constants speed c of light in vacuum, gravitational surface tension T ?
constant G and Planck’s constant h. Then, t p µ (a) 2.4% (b) 15 %
c5 (c) 1.6% (d) 0.15%
(a) Ghc5 (b)
Gh
1 /2
(c)
Gh æ Gh ö
(d) ç 5 ÷ More Than One Correct Option
c5 èc ø
27. Which of the following combinations have the
19. If 1 g cm s–1 = x newton-sec, then the number x is dimensions of time? L-C-R represents inductance,
equal to capacitance and resistance respectively?
(a) 1 × 10–3 (b) 3.6 × 10–3 (a) RC (b) LC (c) R / C (d) C / L
(c) 1 × 10–5 (d) 6 × 10–4 28. Photon is quantum of radiation with energy E = hn
20. The frequency of vibration f of a mass m suspended where n is frequency and h is Planck's constant. The
from a spring of spring constant k is given by relation dimensions of h are the same as that of
of the type f = cm x k y , where c is a dimensionless [NCERT Exemplar]
constant. The values of x and y are (a) Linear impulse (b) Angular impulse
(a) 1/2, 1/2 (b) –1/2, –1/2 (c) Linear momentum (d) Angular momentum
(c) 1/2, –1/2 (d) –1/2, 1/2 29. Which of the following is a unit of permeability
21. What will be the unit of time in that system in which (a) H/m (b) Wb/Am
the unit of length is metre, unit of mass is kg and unit (c) ohm × s/m (d) V × s/m2
of force is kg-wt? 30. If Planck's constant (h) and speed of light in vacuum
1 (c) are taken as two fundamental quantities, which
(a) (9.8)2 s (b) 9.8 s (c) 9.8 s (d) s
9.8 one of the following can, in addition, be taken to
22. The dimensions of a rectangular block measured express length, mass and time in terms of the three
chosen fundamental quantities? [NCERT Exemplar]
with callipers having least count of 0.01 cm are
(a)5 Mass
mm ´of 10
electron
mm (m´ 5) mm. The maximum percentage e
(b) Universal gravitational constant (G)
error in the measurement of the volume of the
(c) Charge of electron (e)
block is (d) Mass of proton (mp )
(a) 5% (b) 10 % (c) 15 % (d) 20%
31. The pitch of a screw guage 15 mm and there are 100
23. A resistor of 10 kW having tolerance 10% is connected divisions on the circular scale. While measuring
in series with another resistor of 20 kW having tolerance diameter of a thick wire. The pitch scale reads 1 mm
20%. The tolerance of the combination will be and 63 rad division on the circular scale coincides
approximately with the reference. The length of the wire is 5.6 cm.
(a) 10% (b) 13% (c) 17% (d) 20% (a) The least count of screw guage is 0.001 cm
24. A resistor of 4 kW with tolerance 10% is connected in (b) The volume of the wire is 0.117 cm3
parallel with a resistor of 6 kW with tolerance 100%. (c) The diameter of the wire is 1.63 m
The tolerance of the parallel combination is nearly (d) The cross-section area of the wire is 0.0209 cm3
(a) 10 % (b) 20 % (c) 30 % (d) 40 %
Telegram @unacademyplusdiscounts

Units and Measurements 27

Comprehension Based Questions Matching Type


Passage I 39. Column I gives three physical quantities. Select the
Planck, propounder of the quantum nature of appropriate units for the choice given in Column II.
radiation found dimentionally that dimensions of Some of physical quantities may have more than one
Gh choice correct.
are same as that or a base quantity used in
c3 Column I Column II
mechanics, where A. Capacitance (p) Ohm-second
G = gravitational constant = 6.67 ´ 10–11 N m2 kg–2, B. Inductance (q) Coulmb2 -joule -1
h = Planck’s constant = 6.63 ´ 1034 J-s and C. Magnetic induction (r) Coulomb (volt) -1
c = speed of light = 3.0 ´ 108 ms–1. (s) Newton (Ampere meter) -1
Gh (t) Volt second (Ampere) -1
32. The numerical value of is of the order of
c3 A B C
(a) 10–35 (b)10–31 (a) q p s
(c) 10–32 (d) 10–36 (b) q r p
(c) r s p
33. Dimensions of which base quantity corresponds to
(d) r t q
Gh
that of = ? 40. Match the physical quantities given in column I with
c3
dimension expressed in terms of mass ( m), length ( L),
(a) Time (b) Length
(c) Mass (d) Temperature time ( T) and change (Q) given in column II.
Column I Column II
Passage II
(A) Angular momentum (p) [ML2 T –2 ]
All quantities is mechanics are represented in terms
of base units of length, mass and time. Additional (B) Torque (q) [ML2 T –1 ]
base unit of temperature (kelvin) is used in heat and (C) Inductance (r) [M–1L–2 T 2Q2 ]
thermodynamics. An magnetism and electricity, the
additional base unit of electric current (ampere) is (D) Latent heat (s) [ML2Q–2 ]
used. (E) Capacitance (t) [ML3T –1Q–2 ]
34. The dimensions of distance travelled in nth second (F) Resistivity (u) [L2 T –2 ]
are A B C D E F
(a) [M0LT] (b) [M0L0T0] (a) q s p t r u
(c) [M0LT–1] (d) [M0LT0] (b) q p s u r t
35. The dimensions of universal gravitational constant (c) p s u r t q
are (d) s u r t q p
(a) [ML–3T2] (b) [ML2T–3]
(c) [M–1L3T–2] (d) [M2L2T–2] Assertion and Reason
36. Coefficient of thermal conductivity has the Directions Q. No. 41 to 46 are Assertion-Reason type. Each of
dimensions these contains two Statements: Statement I (Assertion),
(a) [ML–1T3K3] (b) [ML–1T–3K–1] Statement II (Reason). Each of these questions also has four
(c) [MLT–3K–1] (d) [MLT–3K] alternative choice, only one of which is correct. You have to
select the correct choices from the codes (a), (b), (c) and (d) given
37. The dimensions of electrical conductivity are below
(a) [ML3T -3A -2 ] (b) [M-1L-3T3A2 ] (a) If both Assertion and Reason are true and the Reason
(c) [M-1L-3T3A2 ] (d) [M-1L-3T -3A2 ] is correct explanation of the Assertion
38. The dimensions of pole strength are (b) If both Assertion and Reason are true but Reason is
not correct explanation of the Assertion
(a) [M0LT0A] (b) [M0LTA]
(c) If Assertion is true but Reason is false
(c) [M0L–1TA–1] (d) [M0L–1T0A–1]
(d) If Assertion is false but the Reason is true
Telegram @unacademyplusdiscounts

28 JEE Main Physics

41. Assertion Impulse has the dimensions of force. Distance


Reason Velocity = .
Reason Impulse = force ´ time. Time
1 T 45. Assertion Pressure has the dimensions of energy
42. Assertion In the relation, n = ,where symbols
2l m density.
have standard meaning, m represents total mass. energy [ML2 T -2 ]
Reason Energy density = =
volume [L3 ]
Reason Linear mass density = mass/volume.
[ML–1T –2 ] = pressure
43. Assertion The dimensions of rate of flow are [M0L3T -1 ].
Reason Rate of flow is velocity/s. 46. Assertion The unit used for measuring nuclear
cross-section is ‘barn’.
44. Assertion If error in measurement of distance and
Reason 1 barn = 10–14 m2
time are 3% and 2% respectively, error in calculation
of velocity is 5%.

Previous Years’ Questions


47. In an experiment, the angles are required to be line with the main scale as 35. The diameter of the
measured using an instrument. 29 divisions of the wire is [AIEEE 2008]
main scale coincide with 30 divisions of the vernier (a) 3.32 mm (b) 3.37 mm
scale. If the smallest division of the main scale is (c) 3.67 mm (d) 3.38 mm
half a degree (= 0.5°), then the least count of the
53. The dimensions of magnetic field in M, L, T and C
instrument is [AIEEE 2009]
(Coulomb) are given as [AIEEE 2008]
(a) half minute (b) one degree
(a) [MLT–1C–1] (b) [MT2C–2]
(c) half degree (d) one minute
(c) [MT–1C–1] (d) [MT–2C–1]
48. If 3.8 × 10–6 is added to 4.2 × 10–5 giving due regard
to significant figures, then the result will be 54. The dimensional formula of magnetic flux is
[UP SEE 2009] [BVP 2007]
(a) 4.08 × 10–5 (b) 4.6 × 10–5 (a) [ML0T –2A–1] (b) [ML2T –1A–1]

(c) 4.5 × 10–5 (d) None of these (c) [ML2T –1A–2] (d) [ML2T –2A–1]

49. If ‘muscle times speed equals power’, what is the 55. Dimensions of resistance in an electrical circuit, in
ratio of the SI units and the CGS unit of muscle? terms of dimension of mass M, of length L, of time T
[BVP 2008] and of current I, would be [UP SEE 2007]
(a) 105 (b) 103 (a) [ML2T –3I–1] (b) [ML2T –2]
(c) 107 (d) 10 -5 (c) [ML2T –1I–1] (d) [ML2T –3I–2]

50. Resistance of a given wire is obtained by measuring 56. Which of the following units denotes the dimensions
the current flowing in it and the voltage difference [ML2/Q2], where Q denotes the electric charge?
applied across it. If the percentage errors in the [AIEEE 2006]
measurement of the current and the voltage (a) Henry (b) Hm–2 (c) Weber (Wb) (d) Wbm–2
difference are 3% each, then error in the value of
resistance of the wire is [AIEEE 2012] 57. The dimensions of permittivity e0 are [BVP 2006]

(a) 6% (b) zero (c) 1% (d) 3% (a) [M -1L-3A 2T4] (b) [M -1L3A -2 T -4 ]
(c) [M -1L -1A2 T2 ] (d) [M -1L -3A2 T -4 ]
51. The respective number of significant figures for the
number 23.023, 0.0003 and 21 ´ 10–3 are [AIEEE 2010] 58. What is dimensional formula of thermal conductivity?
(a) 5, 1, 2 (b) 5, 1, 5 (c) 5, 5, 2 (d) 4, 4, 2 [UP SEE 2006]
(a) [MLT -1 q-1] (b) [MLT -3 q-1]
52. Two full turns of the circular scale of a screw gauge
cover a distance of 1 mm on its main scale. The total (c) [M2 LT -3 q-2 ] (d) [ML2 T -2 q ]
number of divisions on the circular scale is 50. R
59. The dimensions of are
Further, it is found that the screw gauge has a zero L [BVP 2005]
error of – 0.03 mm. While measuring the diameter of (a) [T2] (b) [T]
a thin wire, a student notes the main scale reading of (c) [T –1] (d) [T –2]
3 mm and the number of circular scale divisions in
Telegram @unacademyplusdiscounts

Units and Measurements 29

60. Dimensions of potential energy are [BVP 2005] 64. A cube has a side of length 1.2 ´ 10–2 m. Calculate its
(a) [MLT –1] (b) [ML2T –2] volume [IIT JEE 2003]
(c) [ML–1T –2] (d) [ML–1T –1] (a) 1.7 × 10–6 m3 (b) 1.73 × 10–6 m3
61. 1 Wbm–2 is equal to [UP SEE 2005] (c) 1.70 × 10–6 m3 (d) 1.732 × 10–6 m3
(a) 104Gauss (b) 4 × 10–3
Gauss 65. A spectrometer gives the following reading when
(c) 102 Gauss (d) 10–4 Gauss used to measure the angle of a prism.
62. A wire has a mass (0.3 ± 0.003) g, radius Main scale reading = 58.5°
(0.5 + 0.005) mm and length (6 ± 0.06) cm. The Vernier scale reading = 09 division
maximum percentage error in the measurement of its Given that 1 division on main scale corresponding to
density is [IIT JEE 2004] 0.5°. Total division on the vernier scale is 30 and
(a) 1 (b) 2 match with 29 divisions of the main scale. The angle
(c) 3 (d) 4 of the prism from the above data [AIEEE 2012]
63. In an experiment to measure the height of a bridge by (a) 58.59° (b) 58.77° (c) 58.65° (d) 59°
dropping stone into water underneath, if the error in 66. Let [ e0 ] denote the dimensional formula of the
measurement of time is 0.1s at the end of 2 s, then the perimitivity of vacuum. If M = mass, L = length,
error in estimation of height of bridge will be T = Time and A = electric current, then [JEE Main 2013]
[Kerala CEE 2004]
(a) [ e 0 ] = [M–1L–3T2 A ] (b) [ e 0 ] = [M–1L–3T 4 A2 ]
(a) 0.49 m (b) 0.98 m
(c) [ e 0 ] = [M2L2 T –1A –2 ] (d) [ e 0 ] = [M–1L2 T –1A2 ]
(c) 1.96 m (d) 2.12 m

Answers
Round I
1. (c) 2. (b) 3. (b) 4. (d) 5. (a) 6. (a) 7. (a) 8. (c) 9. (b) 10. (a)
11. (a) 12. (b) 13. (a) 14. (b) 15. (b) 16. (a) 17. (d) 18. (a) 19. (d) 20. (b)
21. (b) 22. (a) 23. (a) 24. (c) 25. (c) 26. (c) 27. (a) 28. (d) 29. (c) 30. (b)
31. (c) 32. (c) 33. (c) 34. (b) 35. (a) 36. (b) 37. (b) 38. (a) 39. (b) 40. (d)
41. (d) 42. (a) 43. (a) 44. (d) 45. (c) 46. (b) 47. (a) 48. (c) 49. (c) 50. (c)
51. (a) 52. (d) 53. (c) 54. (d) 55. (a) 56. (b) 57. (c) 58. (a) 59. (c) 60. (a)
61. (c) 62. (d) 63. (b) 64. (b) 65. (d) 66. (a) 67. (d) 68. (a) 69. (a) 70. (d)
71. (b) 72. (b) 73. (a) 74. (d) 75. (b) 76. (a) 77. (d) 78. (c) 79. (b) 80. (b)
81. (d) 82. (b) 83. (d) 84. (b) 85. (c) 86. (c) 87. (a) 88. (b) 89. (a) 90. (c)
(b) (d)
91. (a) (a)
92. (b) (a)
93. 94. 95. 96. 97. 98. 99. (c)

Round II
1. (b) 2. (a) 3. (d) 4. (b) 5. (d) 6. (a) 7. (b) 8. (d) 9. (a) 10. (b)
11. (c) 12. (b) 13. (a) 14. (c) 15. (d) 16. (c) 17. (a) 18. (d) 19. (c) 20. (a)
21. (d) 22. (a) 23. (c) 24. (c) 25. (d) 26. (c) 27. (a,b) 28. (b,d) 29. (a,c) 30. (a,b,d)
31. (a,b,c) 32. (a) 33. (b) 34. (c) 35. (c) 36. (c) 37. (b) 38. (a) 39. (a) 40. (b)
41. (c) 42. (c) 43. (b) 44. (a) 45. (a) 46. (b) 47. (d) 48. (b) 49. (a) 50. (a)
51. (a) 52. (d) 53. (c) 54. (d) 55. (a) 56. (b) 57. (a) 58. (b) 59. (c) 60. (b)
61. (a) 62. (d) 63. (c) 64. (a) 65. (c) 66. (b)
Telegram @unacademyplusdiscounts

the Guidance
Round I
1. According to Faraday’s first law of electrolysis, m = Zq or [Energy] [ML2T -2]
11. As, = = [ML-1T -2]
m [Volume] [L3 ]
Z= . So, SI unit of Z is kg C-1.
q [MLT -2]
and [Pressure] = = [ML-1T -2]
2. Sum of 436.32, 227.2 and 0.301 is 663.821. Because figure. [L2]
8 is more than 5, so 1 add in 663.
12. Because, dimension formula of tension same as force
Þ 663 + 1 = 664
= [MLT –2]
3. The height of tree, building, tower, hill etc.,can be force
determined with the help of sextant.
and surface tension = = [ML0 T –2]
length
4. Given A =10
. m ± 0.2 m Work and torque, both are product of force and length.
B = 2.0 m ± 0.2 m Impulse is equal to change in momentum.
x = AB = 10
. ´ 2.0 = 1414
. m 32 ´ 10 -5
13. As, T = = 32 ´ 10 -3Nm-1 = 0.032 Nm–1
Rounding-off to two significant digits, (10) -2

x = AB = 14
. m 14. Given, y = r sin( wt - kx)
Dx 1 é DA DB ù where, wt = angle
Now, = +
x 2 êë A B úû 1
\ w = = [ T -1] (Q angle is dimensionless)
1 é 0.2 0.2 ù 0.6 T
= ê + ú =
2 ë 10
. 0.2 û 2 ´ 2.0 Similarly kx = angle
0.6 ´ x 1
Dx = = 0.15 ´ 1414
. = 0.2121 \ k = = [L-1]
2 ´ 2.0 x
w [T -1]
Rounding-off to one significant digits, Dx = 0.2 m \ = = [LT -1]
k [L-1]
Thus, AB = 1.4m ± 0.2m
15. One light year = 3 ´10 8m/s ´ 1 yr
5. 1 Newton = 105 dyne and 1m = 100 cm
3 ´ 10 8 m
10 6 dyne cm–2 = 10 6 ´ 10 -5 N ´ (10 -2 m) 2 = 10 5 Nm–2 = ´ 365 ´ 24 ´ 60 ´ 60 s
s
6. Time defined in terms of rotation of the earth is called = 3 ´ 10 8 ´ 365 ´ 24 ´ 60 ´ 60 m
Universal Time (UT).
= 9.461 ´ 10 15 m
7. Indestructibility, invariability and reproductibility are
essential characteristics of a unit of measurment. 16. Force, F = ma
F 10 pound
8. Young modulus Y = 1.9 ´1011 N / m2 \ a= =
m 1 kg
Q 1N = 10 5 dyne,1 m2 = 10 4 cm2 pound 10 slug ft
= 10 =
1.9 ´ 10 11 ´ 10 5 kg kg s2
So in CGS Y= dyne / cm2
10 4 ft ft
Y = 1.9 ´ 10 12 dyne/cm2 = 10 ´ 14.6 kg 2
= 146 2
kg s s
9. Magnetic flux has the unit as weber/ m2 = 146 ´ 0.30 ms–2 = 44.5 ms–2
10. Here, x = 5 cm 1 -2 1 -2
æL ö æT ö m ö æ sec ö
Dx1 = 5 - 4.9 = 0.1 cm
17. As, n2 = n1 ç 1 ÷ ç 1 ÷ = 10 æç ÷ ç ÷
è L2 ø è T2 ø è kmø è hr ø
Dx2 = 5 - 4.805 = 0.195 cm
(Here, n1 and n2 are numerical values)
Dx3 = 5 - 5.95 = - 0.25 cm 1
Dx4 = 5 - 5.4 = 0.4 cm æ m ö æ sec ö -2
n2 = 10 ç 3 ÷ ç ÷ = 129600
Hence, option (a) is most accurate. è10 mø è 3600 sec ø
Telegram @unacademyplusdiscounts

Units and Measurements 31

. ´ 10 -31
91 Mass 4.237 g
18. The mass of electron = 24. Q Density = = = 1.694 g /cm3
. ´ 10 -27
167 Volume 2.5 cm3
. ´ 10 -31
91 Þ = 1.7 g / cm3
\ E= ´ 931 MeV
. ´ 10 -27
167 25. As, 100 W = 100 Js-1 = 10 9 erg s–1
= 0.5073 MeV
26. [Energy] =[ML2T -2]. Increasing M and L by a factor of 3, energy
19. As, R = 8.3 J/K-mol is increased 27 times.
Now, n1u1 = n2 u2
27. Trigonometric function has no dimension.
(u1 and u2 are units while n1 and n2 are the numerical values) x
n1 u1 So, = dimensionless
\ n2 = a
u2
Thus, a has the dimensions as x in equation.
8.3 J / K-mol x
= RHS has dimension of i.e., dimensionless
atmL / K-mol a
8.3 J / K-mol and hence, an is dimensionless
=
(1.013 ´ 10 N / m2) (10 -3 m3) / K -mol
5
Þ n =0
8.12 Energy J
= = 0.0812 28. Intensity (I) = =
10 2 Area ´ Time m2s
\ 8.3 J/K-mol = 0.0812 atm L/K-mol
m1m2
29. Q F =G
20. Given, 1 eV = 1.6 ´ 10 -19 J r2
\ 13.6 eV = 13.6 ´ 1.6 ´ 10 -19 J Fr 2 Nm2
G= 2 Þ
. ´ 10 -19 J
= 2176 m kg 2

21. According to question, 30. LI 2 represents energy, i. e. ,[ML2T –2]


F µ mav br c 31. 1 yard = 36 inch = 36 ´ 2.54 cm = 0.9144 cm
a b c
F = km v r 32. CR is known as time constant
k, being a dimensionless constant. CR = [ T ]
From homogeneity of dimensions, F
33. Spring constant = = [ML0 T -2]
LHS = RHS L
[MLT -2] = [M]a [LT -1]b[L ]c Energy
Surface energy = = [ML0 T -2]
or [MLT -2] = [M aLb + c T -b ] Area
Comparing the powers, we obtain 34. According to definition, metre is the distance containing
a =1 1650763.73 wavelength in vacuum of radiation corresponding
b + c =1 to orange red light emitted by an atom of Kr-86.
- b = -2 Þ b = 2
35. Momentum, p µ f av brc
abc
\1 1 1 3 2 +
----
c =1
Þ c = -1 [MLT ] = [ T ] [LT ] [ML ]
kmv 2 [MLT -1] = [M cLb -3c T -a - b ]
Therefore, F = kmv 2r -1 = .
r Þ c =1
The experimental value of k is found to be here b - 3c = 1
mv 2 Þ b=4
\ F=
r - a - b = -1
[F] [MLT -2] a + b = 1, a = -3
22. Force, F = kv , [k] = = = [MT -1].
[v] [LT -1] [P ] = [ f -3v 4r ]
So, unit is kg s–1. 36. Here, [M0L0T 0 ] = [ML-1T -2]a [ML-3 ]b[LT -1]c
23. [Surface tension] = [ML0T -2], [viscosity] = [ML-1T -1]. or [M 0L0 T 0 ] = [M a + bL- a + c T -2a -3b - c ]

Clearly, mass has the same exponent in these physical Comparing powers of M, L and T, we get
quantities. a + b = 0 ,- a + c = 0 ,-2a - 3b = 0
Solving, a = 1, b = -1, c = 1
Telegram @unacademyplusdiscounts

32 JEE Main Physics

Charge q As thickness has least number of significant figures 3,


37. Capacitance C = =
Potential V therefore rounding-off area up to three singificant figures, we
Work æ Wö get
Also potential = çQ V = ÷
Charge è qø Area of sheet (A) = 8.72 m2
q2 J Volume of sheet (V) = l ´ b ´ t
\ C= as well as C = 2 .
J V = 4.234 ´ 1005
. ´ 0.0201
Thus, (a), (c), (d) are equivalent to farad but (b) is not = 0.0855289
equivalent to Farad.
Rounding-off up to three significant figures, we get
38. Given, y = a sin ( wt + kx) Volume of the sheet = 0.0855 m3
Here,wt should be dimensionless Ldi éAù
é 1ù 44. e = Þ [ e] = [ML2T -2A -2] ê ú
\ [ w] = ê ú dt ëTû
ët û
[ e] = [ML2T -2Q-1]
[ w] = [M 0L0 T -1]
[ML2T -2]
-1 45. [MT -3 ] =
39. The speed of light in vacuum (c) =1(new unit of length s ) [L2][ T]
Time taken by light to reach the earth = energy / area ´ time = dimension of solar constant.
t = 8 min + 20 s 46. Both force constant and surface tension represent force per
= (8 ´ 60 + 20) s = 500s unit length.
\Distance between the sun and the earth Angular momentum [ML2T -1]
= Speed of light ´ Time 47. As, = = [M 0LT 0 ]
Linear momentum [MLT -1]
x= c ´t
48. Friction, F = mN
=1(new unit of length s-1 ) ´ 500 s
F
= 500 new unit of length \ m=
N
F m 0I1 I2 -2
40. As, = é F ù é MLT ù
l 2 pr \ [m ] = ê ú = ê -2 ú
= dimensionless
ë N û ë MLT û
[F ] [MLT -2]
or [m ] = =
[l1l2] [A 2] 49. Given, molar volume of one mole of hydrogen
-2 –2
= [MLT A ] = 22.4 L = 22.4 ´ 10 -3 m3

[MLT -2] Diameter of hydrogen molecule (d) = 1 Å = 10 -10 m


41. As, [ML-2T -2] =
[L ][L2] d 10 -10
\ Radius of hydrogen molecule (r) = =
Force 2 2
=
Distance ´ area = 0.5 ´ 10 -10 m
Pressure 4 3
= Volume of one molecule of hydrogen = pr
Distance 3
= Pressure gradient 4
= . ´ (0.5 ´ 10 -10)3
´ 314
42. By substituting the dimensions of each quantity, we get 3
T = [ML–1T –2]a [L3M]b [MT –2]c = 5.234 ´ 10 -31 m3
3 1 Number of molecules in one mole of hydrogen
By solving, we get a = - , b = and c = 1
2 2 = Avogadro number (N)
43. Given, length (l) = 4.234 m = 6.023 ´ 10 23

Breagth (b) =1005


. m \Atomic volume of one mole of hydrogen
Thickness (t) = 2.01 cm = 0.0201 m = Number of molecules in one mole of hydrogen
Area of sheet ( A) ´ Volume of one molecule of hydrgen
= 2 ( l ´ b + b ´ t + t ´ l) = 6.023 ´ 10 23 ´ 5.234 ´ 10 31
= 2 [( 4.234 ´ 1005
. ) + (1005
. ´ 0.0201) + (0.0201 ´ 4.234)] = 3152
. ´ 10 -7 m3
= 2 ´ 4.3604739 Molar volume 22.4 ´ 10 -3
2 Now, = . ´ 10 4
= 71
= 8.7209478 m Atomic volume 3154
. ´ 10 -7
Telegram @unacademyplusdiscounts

Units and Measurements 33

50. Do not think in terms of I and w .Remember; kinetic energy is 57. Electric potential, V = IR,
fundamentally,'work'.
é V ù é Work done ù
W = Force ´ distance [R ] = ê ú = ê ú
ë I û ë Charge ´ I û
= [MLT -2] ´ [L]
[ML2T –2]
= [ML2T -2] = = [ML2T –3 A –2]
[A 2T]
51. Time period, 58. As, n1u1 = n2u2
T µ p ar bE c
(n1 and n2 are numerical values and u1 and u2 are the unit in
or T = kp ar bE c proper system)
k, is a dimensionless constant. nu
\ n2 = 1 1
According to homogeneity of dimensions, u2
LHS = RHS 1450 mile / h 1450 s / mile
= =
\ -1 -2 a
[ T] = [ML T ] [ML ] [ML T ]3 b 2 -2 c m/ s mh

[T] = [M a+b+c ][L-a -3b + 2c ][T -2a - 2c ] 1450 s ´ 1.6 km


= = 644.4
10 –3 km ´ 60 ´ 60 s
Comparing the powers, we obtain
1450 mile/h = 644.4 m/s
a+ b+ c =0
-a - 3b + 2c = 0 59. The instrument whose least count is minimum, is called the
-2 a - 2 c = 1 most precise device.
On solving, we get (a) Number of division (MSD) = 20
5 1 1 Main Scale Division (MSD) = 1mm
a=- ,b= ,c=
6 2 3 As 20 divisions on vernier scale will be equal to the 19
energy divisions on main scale.
52. Solar constant =
cm2 min 19
\ Vernier Scale Division (VSD) = MSD
\The dimension of solar constant 20
[ML2T -2] Least count of varnier callipers = 1MSD - 1VSD
= = [ML0 T -3 ]
[L2T] = 1MSD -1VSD
19 1
53. Electric displacement, D = eE = 1MSD - MSD = MSD
20 20
C2 N
Unit of D = 1 1
Nm2 C = mm = cm
20 200
æ C ö [ AT]
\ [D ] = C ç 2 ÷ = 2 = [L-2TA] = 0.005 cm
è m ø [L ]
(b) Pitch of screw guage = 1mm
54. Work done, W = eDq Number of divisions on circular scale = 100
W [ML2T -2] Least count of screw guage
\ e= =
Dq [AT] Pitch
=
\ [e] = [ML2T -3 A -1] Number of divisions on circular scale
1 1
55. Given, a = 3bc2 mm = cm
a 100 1000
Þ b= = 0.001 cm
3 c2
Writing dimensional for a and c, we have (c) Wavelength of light ( l) » 10 -7m = 10 -5 cm = 0.00001 cm
[Q / V ] [Q ] / [ML2T -2Q-1] \ As the given optical instrument can measure length to
[ b] = =
[B2] [MT -1Q-1]2 within a wavelength of light, therefore least count of the
given optical instrument = Wavelength of light.
= [M -3L-2T 4 Q4 ]
= 0.00001 cm
1
56. Power of lens, P = The least count is minimum for the given optical
f
instrument. Therefore, the given optical instrument is the
1 1
\ [P ] = = = [M 0L-1T 0 ] most precies.
[ f ] [L ]
Telegram @unacademyplusdiscounts

34 JEE Main Physics

60. Magnification of microscope = 100 68. As, n1u1 = n2u2


Observed width of the hair = 3.5 mm 1 shake 10 -8 s
Þ n2 = = –9
Ovserved width 1 ns 10 s
Magnification =
Real width \ n2 = 10
Ovserved width 3.5
Real width = = 69. Maximum absolute error is Da + Db. Now work out the
Magnification width 100
relative error and finally the percentage error.
= 0.35 mm
70. When two quantities are multiplied, their maximum relative
61. Area of object = 1.75cm2 = 1.75 ´ 10 -4 m2 errors area added up.
Area of image = 1.55 m 2 1
71. We know that kinetic energy = mv 2
Area of image 2
\Areal magnification = Required percentage error is 2% + 2 ´ 3%, i. e. , 8%
Area of object
155
. 72. Volume µ r3
= » 8857
. ´ 10 -4
175 So, error is 3 ´ 2% = 6%
Linear magnification = Area magnification 5000 ´ 86400 ´ 365.25
73. Required time = s = 1.6 s ~
-2s
= 8857 = 941
. 10 11
1
62. Modulus of rigidity [h ] = [ML–1T –2] 74. Kinetic energy, E = mv 2
2
1/ 2
éMù DE Dv 2 - v 2
or [T ] = ê ú \ ´ 100 = ´ 100
ë Lh û E v2
M = [(1.5) 2 - 1] ´ 100 = 125%
Time period = 2p
Lh 0.1
75. Percentage error in radius is ´100
4.3
63. Use formula for time period in angular SHM.
Again, V µ R3
and dimension of time = [M 0L0 T ] 3 ´ 0.1
\ Error in volume = ´ 100
64. Velocity v = klarbg c 4.3
1 0.2
Þ [M 0LT –1] = [La ] [M bL-3b ] [Lc T -2c ] 76. Percentage error inside = éê ù
´ 100 ú = 0.1m
2 ë100 û
or [M 0LT –1] = [M bLa -3b + c T -2c ]
0.1
Equating powers of M, L and T, we get Absolute error inside = ´ 10 = 0.01 cm and side
100
- 2 c = -1
1 = 100 = 10 m
Again, a - 3 b + c = 1, b = 0 , c =
2 \ side = (10 ± 0.01) m
\ v = kl1/ 2r 0 g 1/ 2 2 ´ 0.02 1
77. Required percentage = ´ 100 + ´ 100
or 2
µv l
g 0.24 30
0.01
+ ´ 100
[E][J]2 [ML2T –2] [ML2T –1]2 4.80
65. = [M 0L0 T 0 ]
[M]5 [G ]2 [M]5 [M –1L3 T –2]2 = 16.7+3.3+0.2 = 20%

66. n1u1 = n2u2 78. When quantities are subtracted,their maximum absolute
errors are added up.
n u 170.474 L 170.474 ´ 10 –3 M3 \ Result = (80 - 10) ± (0.1+0.1)
n2 = 1 1 = =
u2 M3 M3 = 70 ± 0.2
= 0.170474
67. l = mpv qhr 79. Given, X = [M aLb T - c ]
0 0 p –1 q 2 -2 r
[M LT ] = [M ] [LT ] [ML T ] DX é DM DL DT ù
\ =± êë a M + b L + c T úû
[M 0LT 0 ] = [M p + rLq + 2r T -q - r ] X
\ p + r = 0 ,q + 2 r = 1, - q - r = 0 = ± [ aa + bb + gc]%
After solving, we get 80. Subtract 3.87 from 4.23 and then divide by 2.
p = -1,q = -1,r = 1
Telegram @unacademyplusdiscounts

Units and Measurements 35

81. Required error in density = 3% + 3 ´ 2% = 9% 89. As p = 3.14


\ p 2 = (3.14) 2 = 9.8596
82. Distance of Jupiter from earth (d) = 824.7 million km
on rounding off p 2 = 9.86
= 824.7 ´ 10 6 km
0.01
Angular diameter of Juptier (q) = 35.73¢¢ 90. Percentage error in measurement of a side = ´ 100
1.23
But 1° = 60 ¢ = (60 ´ 60) ¢¢ 0.01
\Percentage error in measurement of area = 2 ´ ´ 100
æ 1 ö 1 p 1.23
1¢¢ = ç ÷° = ´ rad
è 60 ´ 60 ø 60 ´ 60 180
91. Given, P = a3b2 / cd
314
.
= rad Maximum relative error in physical quantity P is given by
60 ´ 60 ´ 180
DP é æ Da ö æ Db ö 1 æ Dc ö æ Dd ö ù
1¢¢ = 4.85 ´ 10 -6 rad = ± ê3ç ÷ + 2ç ÷ + ç ÷ + ç ÷
P ë è a ø è b ø 2 è c ø è d ø úû
\Angular diameter of Jupiter ( q) = (35.72 ´ 4.85 ´ 10 -6) rad \Maximum percentage error in P is given by
If D is the diameter of the Jupiter then, angular diameter DP é æ Da ö æ Db ö
D ´ 100 = ± ê3ç ´ 100 ÷ + 2ç ´ 100 ÷
( q) = P è
ë a ø è b ø
d 1 æ Dc ö æ Dd öù
+ ç ´ 100 ÷ + ç ´ 100 ÷ ú
D = qd 2è c ø è d øû
= (35.72 ´ 4.85 ´ 10 -6) ´ 824.7 ´ 10 6 Da Db
But ´ 100 = 1%, ´ 100 = 3%
= 142873 a b
= 1429
. ´ 10 5 km Dc Dd
´ 100 = 4%, ´ 100 = 2%
c d
83. Percentage error in x = 1% + 2 ´ 3% + 3 ´ 2% = 13% DP é 1 ù
\ ´ 100 = ± ê3 ´ (1) + 2 ´ (3) + ´ ( 4) + (2) ú
The sign ± has been used because the words ‘maximum P ë 2 û
percentage error’ have not been used. ± [3 + 6 + 2 + 2]%
= ± 13%
Note Percentage error is ± DA ´ 100
A As the result (13%) has two significant figures, therefore the
DA value of P = 3.763 should have only two significant figures.
Maximum percentage error is ´100
A Rounding-off the value of P up to two significant figures, we
1 get P = 3.8.
84. Here at 2 is a dimensionless. Therefore, a = and has the
1
t2 92. New unit of mass is kg
dimension of [ T –2]. 6.67 ´ 10 –11
0.2 i. e. , 1.5 ´ 10 10 kg
85. % error = ´ 100 = 0.8
25
93. The result has to be in one significant number only.
86. Maximum percentage error in, P = 4% + 2 ´ 2% = 8% uv Df Du Dv D (u + v)
94. f = , = + +
u+v f u v u+v
87. As, V = (8 + 0.5)
0.01
and I = (2 + 0.2) 95. Percentage error in mass = ´ 100 = 0.04
23.42
8
\ R= =4 (R = resistance) 0.1
2 Percentage error in volume = ´ 100 = 2.04
4.9
DR æ DV DI ö æ 0.5 0.2 ö
Þ %= ç + ÷=ç + ÷ ´ 100 = 16.25% Adding up the percentage errors, we get nearly 2%.
R è V I ø è 8 2 ø
1/ 2 1/ 2
\ R = (4 ± 16.25%) T é m¢ g ù é m¢ lg ù
96. v = =ê ú =ê
1 m ëM / lû ë M úû
88. Percentage error in length = ´ 100 = 2
50 D v 1 é D m¢ D l D M ù
It follows from here, = ê + +
0.1 v 2ë m l M úû
Percentage error in breadth = ´ 100 = 5
2.0 1 é 0.1 0.01 0.1ù
= ê + +
Percentage error in thickness =
0.1
´ 100 = 1 2 ë 3.0 1.000 2.5 úû
1.00 1
= [0.03+0.001+0.04] = 0.036
Percentage error in volume = 2 + 5 + 1 = 8 2
Percentage error in the measurement = 3.6
Telegram @unacademyplusdiscounts

36 JEE Main Physics

97. Radius of hydrogen atom (r) = 0.5Å = 0.5 ´ 10 -10m w1 D (RD)


98. RD = , ´100
w1 - w2 RD
4 3
Volume of each hydrogen atom (V ) = pr Dw1 D (w1 - w2)
3 = ´ 100 + ´ 100
4 w1 w1 - w2
= . ´ (0.5 ´ 10 -10)3
´ 314
3 0.05 0.05+0.05
= ´ 100 + ´ 100
= 5.234 ´ 10 -31
m3 5.00 1.00
= 1 + 10 = 11%
Number of atoms in one mole of hydrogen
Dl 0.01
= Avogadro numver (N) 99. ´ 100 = ´ 100 = 0.07
23 l 15.12
= 6.023 ´ 10
Db 0.01
\Atomic volume of 1 mole of hydrogen atoms (V ¢) ´ 100 = ´ 100 = 0.1
b 15.12
= Volume of a hydrogen atom ×Number of atoms Dt 0.01
´ 100 = ´ 100 = 0.2
V¢ =V ´N t 5.25
= 5.236 ´ 10 -31 ´ 6.023 ´ 10 23m3 Required percentage = 0.07+0.1+0.2 = 0.37%
= 3152
. ´ 10 -7 m3

Round II
0.5 2p 2p
1. Zero error = 5 ´ = 0.05 mm we get, b=
,c =
50 T l
0.5 b 2p / T
Actual measurement = 2 ´ 0.5 mm+25 ´ - 0.05 mm Dimension of = = [LT –1] and other three quantity is
50 c 2p / l
= 1mm+0.25 mm– 0.05 mm = 1.20 mm dimensionless.
A 6. We know that the dimensional formula of energy is [ML2T –2]
2. =m
B 1 2 2
é 1 kg ù é 1 m ù é 1 s ù
Force mass n2 = 1 ê ú ê ú ê ú
Þ = ë10 kg û ë1 kmû ë1 min û
B length
Force ´ length 1 1 1
Þ B= = ´ ´
mass 10 10 6 (60) -2
MLT -2 ´ L 3600
= = L2T –2 = [LT –1]2 = = 3.6 ´ 10 –4
M 10 7
l RA
3. Capacitance X = [M–1L–2T 2Q2] 7. As R = r \ r =
–1 –1
A l
Magnetic induction Z = [MT Q ]
\ dimension = [ML2T -1Q2 × L] = [ML3 T –1Q–1]
2 2 –2 –2
[ Z ] = [M T Q ]
n ( x2 - x1)
Given, X = 3 YZ 2
8. From the given relation, D =
n2 - n1
X [X ] é 1 ù 1
or Y= or [Y ] = Here, [n] = ê = 2 = [L-2T –1]
3 Z2 [ Z ]2 ë area ´ time úû [L T ]
[M –1L–2T 2Q2] x2 - x1 = [L]
\ [Y ] = = [M –3L–2T 4 Q4 ]
[M 2T –2Q–2] é 1 ù é 1ù
and n2 - n1 = ê = 3 = [L–3 ]
4. T-ratios are dimensionless. So, the unit of r is N 2. ë volume úû êë L úû
r [L–2T –1L ]
As, has the dimension as angle. So, [D ] = = [L2T –1]
m2 [L–3 ]
5. Given equation, Y = a sin ( bt - cx) e2
9. The formula for fine structure constant =
Comparing the given equation with general wave equation. æ h ö
4 pe 0 ç ÷c
æ 2 pt 2 px ö è2 p ø
y = a sin ç - ÷
è T l ø It is dimensionless.
Telegram @unacademyplusdiscounts

Units and Measurements 37

10. r1 = 10 -15 m,r2 = 10 26 m 21. We know [F ] = [MLT –2]


1 ML 1kg ´ 1m 1kg ´ 1m
log r = [log 10 -15 + log 10 26 ] T2 = = =
2 F 1kg - wt 9.8 N
1 1
= [ -15 + 26] = 5.5 » 6 T= s
2 9.8
Þ r = 10 6 m
22. Required percentage error
1/ 2
é M –1L3 T –2 ´ ML2T –1 ù 0.01 0.01
11. [ X ] = ê ú = [L] =2´ ´0 + ´ 10 = 4 + 1 = 5
ë L3 T –3 û 15.12 10.15
10 20
[ A 2T 2] 23. As, DRs = DR1 + DR2 = éê ´ 10 +
ù
´ 20 ú kW = 5 kW
12. [ e 0 ] = = [M –1L–3 T 4 A 2] ë100 100 û
[ML3 T –2]
DRs 5 50
´ 100 = ´ 100 = = 17
13. Each of the three terms in the given equation has the Rs 30 3
dimensional formula of force.
R1R2
k 24. Equivalent, Rs =
14. The right hand side of the given relation is basically . R1 + R2
metre
DRs DR1 DR2 D (R1 + R2)
But, since the left hand side is joule, therefore k should be Jm. ´100 = ´ 100 + ´ 100 + ´ 100
Rs R1 R2 R1 + R2
15. [ Calorie] = [ML2T –2] 10
Now, DR1 = ´ 4 kW = 0.4 kW
Comparing with general dimensional formula [M aLb T c ], we 100
get 10
DR2 = ´ 6 kW = 0.6 kW
a = 1, b = 2, c = 2 100
DRs 0.4 0.6 0.4+0.6
1
é 1 kg ù é1 mù é 1 s ù
2 -2 Again, ´ 100 = ´ 100 + ´ 100 + ´ 100
n2 = 4.2 ê = 4.2 a –1 b -2g 2 Rs 4 6 10
ú ê ú ê ú
ë a kg û ë b m û ë g s û = 10 + 10 + 10 = 30
16. New unit of distance = Speed in new units ´ 500 s. 2 2 1
25. As, n ( x m) = 1 m or n = 2
–2 2a b -b c -3 c c 2a + c -3 c -b x
17. As, [MLT ] = [L ] [L T ] [M L ] = [M L T ]
0.01 0.01 0.01
Comparing powers of M, L and T, we get 26. Percentage error in T = ´ 100 + ´ 100 + ´ 100
1.26 9.80 1.45
c = 1, 2a + b - 3c = 1, - b = -2 or b = 2 = 0.8+0.1+0.7 = 1.6
2a + 2 - 3 (1) = 1
27. We know that
Þ 2a = 2 R = [M1 L2T –3 A –2]
or a =1
C = [M –1 L–2T 4 A 2]
5
18. Note carefully that every alternative has Gh and c . L = [ML2T –2A –2]
–1 3 –2 2 –1 0 5 –3
[Gh] = [M L T ] [ML T ] = [M L T ]
\ RC = [ T ] and LC = [ T ]
–1
[ c] = [LT ]
1/ 2
28. From E = hn
æ Gh ö
\ ç 5÷ = [T] E [ML2T -2]
èc ø h= = = [ML2T -1]
n [ T -1 ]
1 g cm s–1 1 g cms–1
19. As, x = 2
= Angular impulse = t ´ t = [ML2T -2 ´ T ] = [ML2T -1]
T 1kg ´ 1ms–1 ´ 1 s
Angular momentum = mvr = M [LT -1] L = [ML2T -1].
1 g cms–1
= 3 = 10 –5
10 g ´ 10 2 cms2 ´ 1 s 29. Units of permeability (m) are WbA –1m–1 = Hm–1
= ohm- s - m–1
20. [M0L0T –1] = [Mx ] [My T -2y ] = [Mx+ y T -2y ]
Equating powers of M and T. 30. Here, h = [ML2T -1]; c = [LT -1]
x + y = 0 , -2y = -1 e = AT; mP = M
1 1 hc [ML2T -1] [LT -1]
or y = ,x+ = 0 Now, = = M2
2 2 G [M -1L3 T -2]
1 hc
or x= M=
2 G
Telegram @unacademyplusdiscounts

38 JEE Main Physics

h [ML2T -1] 39. Capacitance — Coulomb/volt, Coulomb 2 /Joule


Again, = = [ML ]
c [LT -1] Inductance — Ohm second, volt second (ampere) -1
h h G Gh Magnetic inductance — Newton (ampere-metre) -1
L= = =
cM c hc c3 / 2 2 –1
40. Angular momentum — [ML T ]
L Gh Gh
From c = [LT -1], T = = 3 / 2 = 5 / 2 Inductance — [ML2Q–2]
c c .c c Latent heat — [M 0L2T –2]
Hence, out of (a), (b), (d), any one can be taken to express L, Capacitance — [M –1L–2T 2Q+2]
M, T in terms of three chosen fundamental quantites. Resistivity — [ML3 T –1Q–2]
31. Least count 1 =
1
mm = 0.01 = 0.001 cm Torque — [ML2T –2]
100
41. Impulse = Force ´ time
Diameter of wire D = 1mm + 63 ´ 0.01mm
\ Impulse has no dimension of force.
= 1.63 mm or 0.163 cm
1 T T
pD 2l 3.14 ´ (0.163) 2 ´ 5.6 42. From n= , n2 = 2
Volume of wire = = 2l m 4l m
4 4
= 0.117 cm3 T [MLT –2] [M] mass
m= 2 2
= 2 –2 = =
4l n [L T ] [T] length
Gh (6.67 ´ 10 –11) (6.63 ´ 10 –34)
32. = = linear mass density
c3 (3 ´ 10 8)3
volume [L3 ]
= 1.64 ´ 10 –69 = 4.04 ´ 10 –35 43. Rate of flow = = = [L3 T –1] = [M 0L3 T –1]
time [T ]
Þ order = 10 -35 [L ]
44. In fact, [v ] =
33. As, –1 3 –2
[G ] [h] = [M L T ] [ML T ] 2 –1 [T]
= [M 0L5 T –3 ] Dv æ DL DT ö
\ =±ç + ÷
v è L T ø
[ c] = [LT –1]
1/ 2 = ± (3% + 2%) = ±5%
é Gh ù [L5 T –3 ]1/ 2
Now, êë c3 úû = = [L2]1/ 2 = [L ] Force
[LT –2]3 / 2 45. Pressure =
Area
Hence, [L ] = length Force ´ distance energy
= = = energy density
L Area ´ distance volume
34. Distance travelled in nth second = = [LT –1] = [M 0LT –1]
T
46. The assertion is true, but the reason is false, because 1 barn
Gm1m2 = 10 -28 m2.
35. From F = ,
r2
Fr 2 [MLT –2] [L2] 47. As, 30 VSD = 29 MSD
G= = = [M –1L3 T –2] 29
m1m2 [M × M] 1 VSD = MSD
30
Dq dT Least count = 1MSD –1 VSD
36. From = KA
Dt dx æ 39 ö 1 1°
Aq dx = ç1 - ÷ MSD = ´ 0.5° = = 1 minute
K= ´ è 30 ø 30 60
Dt AdT
[ML2T –2] [L ]
48. We will use the general rule of addition by making the powers
= = [MLT –3K –1] same.
[ TL2K ]
i. e. , we will add 3.8 ´ 10 –6 and 42 ´ 10 –6 , we get
1
37. As, s = = [M –1L–3 T 3 A 2] = 45.8 ´ 10 –6 = 4.58 ´ 10 –5
r
As least number of significant figures in given values are 2. So,
M A -m2 we round off the result to 4.6 ´ 10 –5 .
38. Pole strength = = = A-m = AL = [M 0LT 0 A]
2l m
Telegram @unacademyplusdiscounts

Units and Measurements 39

49. Muscle ´ Speed = Power So, Dimensions of R


Power Work [Dimensions of work]
\ Muscle = = =
Speed Time ´ Speed [Dimensions of charge] [Dimensions of current]
[ML2T –2] [ML2T –2]
= = [MLT –2] = = [ML2T –3I–2]
[T] [LT –1] [IT] [I]

= Mass ´ acceleration = Force ML2 [ML ]2


56. As, = = [ML2T –2A –2]
v1 Q 2 [ AT ]2
Now, Henry (H) = SI unit of inductance
v2 e edt W dt
t = = = ´
di /dt di q di
–v1 v 2 – v1
[ML2T –2]
= = [ML2T –2A –2]
[AT] [A]
SI unit of force kg ´ m ´ s–2 q1q 2
Hence, = 57. As, e 0 =
CGS unit of force g ´ cm ´ s–2 4 pFr 2
= 10 3 ´ 10 2 = 105 The dimensions of permitivity
V dimensions of q1 ´ dimensions of q 2
50. R = =
i dimensions of force ´ dimensions of r 2
\ log R = log V - log i [ AT ] ´ [ AT ]
=
DR DV i Dv [MLT –2] [L2]
= +
R V i = [M –1L–3 A 2T 4 ]
= 3% + 3% = 6%
58. Substituting dimensions for corresponding quantities in the
51. All non-zero numbers are signifiicant figures. After point zero relation having coefficient of thermal conductivity.
is not a significant figure. Power of 10’s is not a significant
Heat DQ transferred through a rod of length L and area A in
figure.
time Dt is
1
52. As, pitch = mm = 0.5 mm æT -T ö
2 DQ = KA ç 1 2 ÷ Dt
è L ø
0.5
Least count = mm = 0.01mm where, K = coefficient of thermal conductivity,
50
T1 - T2 = temperature difference.
Zero error = -0.03 mm
DQ ´ L
Zero correction = + 0.03 mm \ K= …(i)
A (T1 - T2) Dt
Observed diameter of wire = 3.35 mm+0.03 mm
= 3.35 mm Substituting dimensions for corresponding quantities in
Eq. (i), we have
Corrected diameter of wire = 3.35 mm+0.03 mm
[ML2T –2] [L ]
= 3.38 mm [K ] = = [MLT –3 q–1]
[L2][ q] [ T]
53. From F = Bqv R 1
F [MLT –2] 59. Dimension of = Dimension of frequency = = [ T -1]
B= = = [M1L0 T –1C–1] L T
qv C[LT –1] (L/R is called time constant for an electrical circuit)
2 –2 –1
54. Dimensional formula of magnetic flux = [ML T A ] 60. Dimension of potential energy
Potential difference V W = Dimension of M ´ Dimension of g ´ Dimension of h
55. Resistance, R = = =
Current i qi = [M] ´ [LT –2] ´ [L ] = [ML2T –2]
(\Potential difference is equal to work done per unit charge).
Telegram @unacademyplusdiscounts

40 JEE Main Physics

61. In CGS system, the magnetic field is expressed in Gauss. If a 64. Here, L = 1.2 ´10 –2 m, V = ?
charge of 1 C moving with a velocity of 1 ms -1 perpendicular
V = L3 = (1.2 ´ 10 –2)3 = 1.728 ´ 10 –6 m3
to a uniform magnetic field experiences a force of 1 N, then
the magnitude of the field is 1 T. The SI unit of magnetic field As the result can have only two significant digits, therefore,
is Wbm–2. Thus, rounding off, we get
1 T = 1NA –1 m–1 = 1 Wbm–2 V = 1.7 ´ 10 –6 m3
In CGS system, 65. As, 30 VSD = 29 MSD
1 tesla = 10 4 gauss = 1Wbm -2 20
1 VSD = MSD
Dm 0.003 Dr 0.005 DL 0.06 30
62. Here, = , = × =
m 0.3 r 0.5 L 6 Least count of vernier scale = 1MSD - 1 VSD
m 29
As r= = 0.5° - ´ 0.5°
( pr 2) L 30
æ Dr ö æ Dm 2 Dr DL ö 0.5°
\ ç ÷ ´ 100 = ç + + ÷ ´ 100 =
è r ø è m r L ø 30
æ 0.003 2 ´ 0.005 0.06 ö Reading of vernier = MS reading + VS reading ´ 2.5
=ç + + ÷ ´ 100 0.5°
è 0.3 0.5 6 ø = 58.5° +9 ´ = 58.65°
30
= 1 + 2 + 1 = 4%
1 q1 q 2
1 2 66. From Coulomb’s law F =
63. From, s = ut + at 4pe 0 R 2
2
1 q1 q 2
h = 0 + ´ 9.8 (2) 2 = 19.6 m e0 =
2 4pFR 2
Dh æ Dt ö Substituting the units
=±ç ÷ (Q a = g = constant)
h è t ø C2
Hence, e0 =
æ 0.1ö 1 N-m2
= ±2 ç ÷ = ±
è 2 ø 10 [ AT ]2
=
h 19.6 [MLT –2][L2]
Dh = =
10 10
= [M –1 L–3 T 4 A 2]
= 1.96 m
Telegram @unacademyplusdiscounts

2 Kinematics
JEE Main MILESTONE
< Frame of Reference < Average Speed and Instantaneous Velocity
< Motion in a Straight Line < Acceleration
< Speed and Velocity < Graphs in One Dimensional Motion
< Uniform and Non-uniform Motion < Relative Velocity

The branch of physics which in space and time deals with the motion of particles or objects, is
called mechanics.
Statics (Study of stationary objects)
Mechanics
Dynamics (Study of moving objects)
An object can have uniform motion, even when a number of forces are acting on it. Such
forces are said to be in equilibrium. Thus, statics is the study of the motion of an object under
the effect of forces in equilibrium.
The motion of objects is studied under two separate headings
An object is said to be at rest, if it
Kinematics The study of the motion of an object without taking into consideration cause
does not change its position with
of its motion is called kinematics.
time and in a state of motion, if it
Dynamics The study of the motion of an object by taking into account the cause of its
motion (whether rest or uniform motion) is called dynamics. continuously changes its position
with time.

2.1 Frame of Reference


The frame of reference is a suitable coordinate system involving space and time
used as a reference to study the motion of different bodies. The most common
reference frame is the cartesian frame of reference involving (x, y, z and t).
Frame of reference is of two types :
(i) Inertial frame of reference
(ii) Non-inertial frame of reference.

(i) Inertial Frame of Reference


A frame of reference which is either at rest or moving with constant velocity is
known as inertial frame of reference. Inertial frame of reference is one in which
Newton’s first law of motion holds good.

(ii) Non-Inertial Frame of Reference


A frame of reference moving with some acceleration is known as non-inertial
frame of reference. Non-inertial frame of reference in one which Newton’s law of
motion does not hold good.
Telegram @unacademyplusdiscounts

42 JEE Main Physics

Position Vector Distance Vs Displacement


It describes the instantaneous position of a particle with
respect to the chosen frame of reference. It is a vector 1. Distance is a scalar quantity and displacement is a vector quantity.
joining the origin to the particle. If at any time (x, y, z) be 2. For motion between two points displacement is single valued while
coordinates of the particle, then its position vector is given distance depends on actual path and so can have many values.
by r = x$i + y$j + zk$ . 3. Path length or distance is a positive scalar quantity which does not
decrease with time and can never be zero for a moving body.
In one dimensional motion position vector may be given Displacement of a body can be zero.
by r = x$i, y = z = 0 (along x-axis). In two dimensional
4. Magnitude of displacement can never be greater than distance.
motion, r = x$i + y$j (in x-y plane z = 0).
5. When a body returns to its initial position, its displacement is zero
y but distance or path length is non-zero.
6. In general, magnitude of displacement is not equal to distance.
B However, it can be so if the motion is along a straight line without
any change in direction.
r2 r
A
One, Two and Three Dimensional
r1
Motions
O x
One Dimensional Motion
Distance and Displacement If only one of the three coordinates is required to specify
Distance is the total length of the path travelled by the the position of an object in space changing w.r.t. time, then
particle in a given interval of time. the motion of the object is called one dimensional motion.
Motion of a particle in a straight line can be described by
Displacement is a vector joining the initial position of the
only one component of its velocity and acceleration. For
particle to its final position in a given interval of time.
instance, motion of a block in a straight line, motion of a
Mathematically, it is equal to the change in position
train along a straight track, a man walking on a level and
vectors i. e. , Dr = r2 - r1
narrow road and object falling under gravity, etc.
y

B
Two Dimensional Motion
r If two of the three coordinates are required to specify the
r2
C A position of an object in space changing w.r.t. time, then the
motion of the object is called two dimensional motion.
r1 The motion of a particle through its vertical plane at some
angle with horizontal (¹ 90°) is an instance of two
x
O dimensional (2-D) motion. This is a projectile motion.
Suppose a body is at point A x y z Similarly, a circular motion is an instance of 2-D motion. A
2-D motion takes place in a plane and its velocity (or
It reaches at point B (x2, y2, z2 ) at t = t2 through path ACB
acceleration) can be described by two components in any
with respect to the frame shown in figure. The actual
two mutually perpendicular directions (vx and vy ).
length of curved path ACB is the distance travelled by the
body in time, Dt = t2 - t1. Three Dimensional Motion
If we connect point A (initial position) and point B (final
If all the three coordinates are required to specify the
position) by a straight line, then the length of the straight
position of an object in space changing w.r.t. time, the
line AB gives the magnitude of displacement of body in
motion of an object is called three dimensional motion.
time interval, Dt = t2 - t1.
Such a motion is not restricted to a straight line or plane
The direction of displacement is directed from A to B but takes place in space. In a 3-D motion, velocity and
through the straight line AB and the magnitude of acceleration of a particle can be resolved in three
displacement is components (vx, vy , vz , a x, ay , az ). A few instances of 3-D
| AB | = (x2 - x1 ) 2 + (y2 - y1 ) 2 + (z2 - z1 ) 2 motion are a flying bird, a flying kite, a flying aeroplane,
the random motion of gas molecules, etc.
Telegram @unacademyplusdiscounts

Kinematics 43

Velocity of an object in motion is defined as the ratio of


2.2 Motion in a Straight Line displacement and the corresponding time interval taken
Here we will consider the motion of a point object in a by the object, i. e. ,
straight line in one dimension, during such motion, the displacement
Velocity =
point object occupies is definite position on the path at time interval
each instant of time. Therefore, the motion of the point Velocity is a vector quantity as it has both the magnitude
object can be described by specifying the distance x of the (speed) and direction.
point object and the corresponding instant of time t.
Mathematically, the position of the object in one
dimensional motion can be expressed as follows 2.4 Uniform and
x = x (t )
or x = f (t )
Non-uniform Motion
Here, the distance x is the function of the time t. Uniform Motion
Sample Problem 1 A particle moves along a circle of An object is said to uniform motion, if its velocity is
radius R. It starts from A and moves in an anti-clockwise uniform, i. e. , it under goes equal displacements in equal
direction. What is the distance and displacement of the particle intervals of time, howsoever small these intervals may be,
from A to D? for a uniform motion along a straight line in a given
B direction, the magnitudes of displacement is equal to the
actual distance covered by the object.

Non-uniform Motion
O
C A An object is said to be non-uniform motion, if it undergoes
equal displacements in unequal intervals of time,
howsoever small there intervals may be. Clearly, in
non-uniform motion, the velocity of an object is different
D at different instants.
pR
(a) , 2R (b) pR , 2 R
2
3
(c) pR , 2 R (d) 2 pR, zero
2.5 Average Speed and
2
Instantaneous Velocity
Interpret (c) For the motion from A to D,
2 pR ´ 3 3 Average Velocity
Distance travelled = = pR
4 2
Average velocity v of an object moving through a
\ Displacement = | AD| = (OA) 2 + (OD) 2 displacement (Dx ) during a time interval (Dt ) is given by
= R2 + R2 = 2 R Dx
v=
Dt
The velocity vector v of an object that has positions x (t ) at
2.3 Speed and Velocity time t and x (t + Dt ) at time t + Dt, can be computed as the
derivative of position
Speed of an object in motion is defined as the ratio of total
x (t + Dt ) - x (t ) dx
path length (i. e. , actual distance covered) and the v = lim =
Dt ® 0 Dt dt
corresponding time taken by the object, i. e. ,
total path length Velocity is also defined as rate of change of displacement.
Speed =
time taken Average velocity in magnitude is always smaller than or
equal to average speed of a given particle.
actual distance covered
=
time taken Note The average velocity is not given by v = v1 + v 2
Speed is a scalar quantity. It gives one idea about the 2
Since the velocities are vectors in different directions and the
direction of motion of the object.
acceleration is not constant.
Telegram @unacademyplusdiscounts

44 JEE Main Physics


v1 v2
Average Speed x-axis
v + v2
Average speed is a measure of the distance travelled in a vaverage =v = 1
given period of time. It is sometimes referred to as the 2
distance per time ratio. Velocity is a vector quantity and For this special case, these expressions give the same result.
average velocity can also be stated as the ratio of
displacement and time. Instantaneous Velocity
For straight line motion is the x-direction, the average The average velocity of an object in motion tells us how
velocity takes the form fast an object has been moving over a given interval of
displacement time. But it does not tell, how fast the given object is
x-axis
(x1, t1) (x2, t2) moving at different instants of time during motion. For
x2 - x1 Dx this, the instantaneous velocity of an object at an instant of
\ vaverage = v = = time t, is defined as the limit of average velocity as time
t2 - t1 Dt
interval Dt, around time t becomes infinitesimally small.
Unit (MKS) metre/second or in general any distance unit Thus, instantaneous velocity at instant of time t is
over any time unit. Ds ds
vi = lim =
If the begining and ending velocities for this motion are Dt ® 0 D t dt
known and the acceleration is constant, the average where, ds / dt = the differential coefficient of s w.r.t. time.
velocity can also be expressed as

Important Points about Speed and Velocity


1. The velocity in the uniform circular motion does not depend upon The graph, describes the motion of a particle moving along x-axis
the time interval. (along a straight line).
2. Velocity can be negative, zero or positive, but speed is never negative. Suppose, we wish to calculate the average velocity betweent = t1 and
3. If motion takes place in the same direction, then the average speed t = t2. The slope of chord AB [shown in Fig. (b)] gives the average
and average velocity are the same. velocity.
4. If a particle travels equal distances at speeds v1, v2 , v3 ,¼ etc x2 - x1
Mathematically, v av = tan q =
respectively, then the average speed is harmonic mean of individual t2 - t1
speeds. 8. If a body moves with a constant velocity, the instantaneous velocity
5. If a particle moves a distance at speed v1 and comes back with speed is equal to average velocity. The instantaneous speed is equal to
2 v1v2 modulus of instantaneous velocity.
v2, then v av = but v av = 0
v1 + v2
9. x-component of displacement is Dx = ò vx dt
6. If a particle moves in two equal intervals of time at different speedsv1
v +v y-component of displacement is Dy = ò vy dt
and v2 respectively, then v av = 1 2
2
z-component of displacement is Dz = ò v z dt
7. The average velocity between two points in a time interval can be
obtained from a position versus time graph by calculating the slope Thus, displacement of particle is
of the straight line joining the coordinates of the two points.
Dr = Dx$i + Dy$j + Dzk$
x2 x2 B 10. The magnitude of instantaneous velocity is equal to the
x1 x1 A instantaneous speed at a given instant.
(x2 – x1)
(t2 – t1) 11. If during motion, velocity remains constant through out a given
interval of time the motion is said to be uniform. For uniform motion,
instantaneous velocity = average velocity = uniform velocity.
t1 t2 t1 t2
However, converse may or may not be true i .e ., if average velocity =
(a) (b) instantaneous velocity, the motion may or may not be uniform.
Telegram @unacademyplusdiscounts

Kinematics 45

Sample Problem 2 A car travels a distance A to B at a 2 u + v1 + v 2 2 u (v1 + v 2)


(a) (b)
speed of 40 kmh–1and returns to A at a speed of 30 kmh–1. 2 u (v1 + v 2) 2 u + v1 + v 2
What is the average speed for the whole journey? v1 + v 2 v -v
(c) (d) 1 2
(a) Zero (b) 34.3 kmh–1 v1 - v 2 v1 + v 2
(c) 68.6 kmh–1 (d) 120 kmh–1 Interpret (b) When time intervals are equal,
s then distance are equal
Interpret (b) Let AB = s, time taken to go from A to B, t1 = h
40 æv + v ö
2u ç 1 2÷
s 2 uv m è 2 ø 2 u (v1 + v 2)
and time taken to go from B to A, t 2 = h v av = = =
30 u + vm æ v + v ö 2 u + v1 + v 2
u + ç 1 2÷
s s è 2 ø
\ Total time taken = t1 + t 2 = +
40 30
(3 + 4) s 7 s Sample Problem 5 A particle travels according to the
= = h equation a = A - Bv where a is acceleration, A and B are
120 120
constants, v is velocity of the particle. It’s velocity as a function of
Total distance travelled = s + s = 2 s
time is
Total distance travelled B B
\ Average speed = (a) (1- e-At ) (1- e-Bt )
(b)
Total time taken A A
2s 120 ´ 2 A A
= = = 34.3 kmh –1 (c) (1- e-Bt ) (d) (1- e-At )
7 s / 120 7 B B
dv
Interpret (c) Given acceleration, a =
Sample Problem 3 A man walks on a straight road from his dt
home to a market 3 km away with a speed of 6 kmh–1 finding dv
\ = A - Bv
the market closed, he instantly turns and walks back with a dt
speed of 9 kmh–1. What is the magnitude of average velocity dv
Þ = dt
and average speed of the man, over the interval of time 0 to A - Bv
40 min ? V dv t

(a) 2.25 kmh–1, 6.75 kmh–1 (b) 6.75 kmh–1, 2.25 kmh–1 ò0 A - Bv = ò0 dt
v
(c) zero, 2.25 kmh–1 (d) zero, 6.75 kmh–1 1
- log e ( A - Bv) = t
Interpret (a) Time taken by man to go from his home to B 0

market, A - Bv
Þ log e = -Bt
distance 3 km 1 A
t1 = = –1
= h = 30 min Þ A - Bv = Ae-Bt
speed 6 kmh 2
\ A
Time taken by man to go from market to home, v = (1 - e-Bt )
B
3 km 1
t2 = = h = 20 min
9 kmh –1 3 Sample Problem 6 Between two stations a train starting
Distance moved in 30 min (from home to market) = 3.0 km from rest first accelerates uniformly, then moves with constant
Distance moved in 10 min (from market to home) with speed velocity and finally retards uniformly to come to rest. If the ratio
1 of the time taken be 1 : 8 : 1 and the maximum speed attained be
9 kmh –1 = 9 ´ = 1.5 km 60 km/h, then the average speed over the whole journey is
6
(a) 35 km/h
So, displacement = 3.0 - 1.5 = 1.5 km
(b) 54 km/h
Total path length travelled = 3.0 +1.5 = 4.5 km (c) 40 km/h
1.5 km
Average velocity = = 2.25 kmh –1 (d) 15 km/h
(40/60) h
Interpret (b) Given, u = 0. Let during three phases time taken
4.5 km
Average speed = = 6.75 kmh –1 are t, 8 t and t.
(40 /60) h v max = at = 60 km/h
1 2 1
at + v max 8 t + at 2
Sample Problem 4 A particle travels half the distance with v av = 2 2
velocity u. The remaining part of the distance is covered with t + 8t + t
velocity v1 for the first half time and v2 for the remaining half 60 + 8 ´ 60
v av = = 54 km/h
time. The average velocity of the particle during the complete 10
motion is
Telegram @unacademyplusdiscounts

46 JEE Main Physics

Sample Problem 7 A particle is moving according to graph Uniformly Accelerated Motion


is shown in figure. What is the average velocity in the interval of
In an accelerated motion, if the change in velocity of an
3 to 8 s?
object in each unit of time is constant, the object is said to
moving with constant acceleration and such a motion is
10 ms–1 called uniformly accelerated motion.
● If a particle is accelerated for a time t1 with acceleration a1
v and for time t2 with acceleration a2, then average
t 10 s acceleration is
a1t1 + a2t2
Interpret In uniform motion, average velocity equal to aav =
t1 + t2
instantaneous velocity.
Average velocity will be 10 ms-1
● If a body starts from rest and moves with uniform
acceleration then distance travelled by the body in t
second is proportional to t2 (i.e., s µ ft2)
Sample Problem 8 A particle moving according to the
So, we can say that the ratio of distance covered in 1s, 2s,
equation x = 5 t 2 - 20 t + 4 , what is the average velocity
3s, is 12 : 22 : 32 or 1 : 4 : 9.
between time t1 = 0 s to t 2 = 4 s ? (where x = displacement, t =
time)
● A particle moving with an uniform acceleration from A to
B along a straight line has velocities v1 and v2 at A and B
respectively. If C is the mid-point between A and B, then
v12 + v22
v velocity of the particle at C is equal to v = .
2
2 t 4
Sample Problem 9 A body moves along a straight line
–20 ms–1 with an acceleration 3 ms–2 for 2 s and then with an
acceleration 4 ms–2 for 3 s. What is his average acceleration?
Interpret Given that,
(a) 3.4 ms–2 (b) 3.5 ms–2
x = 5 t 2 - 20 t + 4
(c) 3.6 ms–2 (d) 3.7 ms–2
dx
v= = 10 t - 20 Interpret (c) Average acceleration
dt
a1t1 + a2 t 2 3 ´2 + 4 ´3
At t = 0 , vi = - 20 ms–1 aav = = = 3.6 ms–2
t1 + t 2 2+3
t = 2 s, v f = - 0 ms–1
t = 4 s, v = - 20 ms–1 Uniformly Accelerated Motion
Area of v-t graph gives displacement and distance For uniformly accelerated motion, acceleration is
Average velocity = 0 constant. If u be the initial velocity, v the final velocity and
40 distance covered by the body in time t is s, then the
Average speed = = 10 ms-1 equations of motion are as under :
4
(i) v = u + at
1
(ii) s = ut + at 2
2
2.6 Acceleration (iii) v2 - u2 = 2 as and
Acceleration of an object is defined as rate of change of a
(iv) snth = u + (2 n - 1)
velocity. It is a vector having unit m/s2 or ms-2. 2
Average acceleration for a given time For motion in a plane, we may consider motion of an object
v -v Dv along x-axis and y-axis independently and then combine
aav = 2 1 = the two motions so as to get the net motion of the particle.
t2 - t1 Dt
Thus, we have
Instantaneous acceleration at a particular instant is (i) v = u + a t,
defined as vx = u x + a x t ,
Dv dv æ dvx $ dvy $ dvz ö vy = uy + ayt
ains = lim = =ç i+ j+ k$ ÷
Dt ® 0 Dt dt è dt dt dt ø and | v | = v2x + vy2
= (a x$i + ay $j + az k$ ) = (v x$i + vy $j ) = (ux$i + uy $j ) + (a x$i + ay $j )t
Telegram @unacademyplusdiscounts

Kinematics 47

1 2
(ii) s = u t + at , Sample Problem 10 A ball is dropped from a high tower
2
1 such that distance covered by it in last second of its motion is
s x = uxt + a xt 2,
2 same as the distance covered by it during first three seconds.
1 Find the time taken by ball to reach ground and height of tower.
sy = uyt + ayt 2
2 (Take g = 10 ms-2)
and |s | = s 2x + sy2 Interpret Let ball takes t seconds to reach the ground and h be
(iii) v2x - uy2 = 2 a x × s x, vy2 - uy2 = 2 ay × sy the height of tower. Then,
1 2
h =0 + gt …(i)
Equations of motion in free space are similar to those for 2
motion in a plane. g 1
and hnth = 0 + (2 t - 1) = 0 + g (3) 2
Sign Conventions Normally, vertically upward motion is 2 2
taken as negative and vertically downward motion is or 2 t -1 = 9 …(ii)
taken as positive. Similarly, for horizontally rightward Þ t =5s
motion is taken positive and leftward motion is taken 1
and h = ´ 10 ´ (5) 2 = 125m
negative. 2
Sample Problem 11 A particle is projected vertically
Motion Under Gravity upwards with velocity 40 ms–1. Find the displacement and
distance travelled by the particle in 6 s. [ Take g =10 ms–2]
The most familiar example of motion with constant
acceleration on a straight line is motion in a vertical (a) 60 m, 100 m (b) 60 m, 120 m
direction, near the surface of earth. If air resistance is (c) 40 m, 100 m (d) 40 m, 80 m
neglected, the acceleration of such type of particle is Interpret (a) Here, u is positive (upwards) and a is negative).
gravitational acceleration which is nearly constant for a So, first we will find t 0 , the time when velocity becomes zero i.e.,
height negligible with respect to the radius of earth. The when the particle is the highest point.
magnitude of gravitational acceleration, which is nearly u 40
t0 = = = 4s
constant for a height negligible with respect to the radius a 10
of earth. The magnitude of gravitational acceleration near Here, t > t0
the surface of earth is g = 9.8 ms-2 = 32 fts-2. Hence, distance > displacement
1
s = 40 ´ 6 - ´ 10 ´ 36 = 60 m
Cases of Motion Under Gravity 2
u2 1
Case I If particle is moving upwards While, d= + | a (t - t 0) 2|
2a 2
In this case, applicable kinematics relations are g ( 40) 2 1
v = u - gt ... (i) = + ´ 10 ´ (6 - 4) 2 = 100 m
1 2 2 ´ 10 2
h = ut - gt ... (ii)
2 u Sample Problem 12 A ball is thrown upwards from the
v2 = u2 - 2 gh ... (iii) ground with an initial speed u. The ball is at a height of 80 m at
Here, h is the vertical height of the particle in two times, the time interval being 6s. Then u is
upward direction. (Take g = 10 ms–2)
Note For maximum height attained by a projectile (a) 20 ms–1 (b) 30 ms–1
h = h max' v = 0 (c) 40 ms–1 (d) 50 ms–1
i .e ., ( 0) 2 =u 2 - 2gh max Interpret (d) Here, a = g = -10 ms–2 and s = 80 m
u2 1 2
\ h max = Substituting the values egh, in s = ut + at , we have
2g 2
Case II If particle is moving vertically downwards. 80 = ut - 5 t 2 s = 80 m
In this is case, u or 2
5 t - ut + 80 = 0 +ve
n = u + gt ... (i) 2
1 u + u - 1600
h = ut + gt 2 ... (ii) g or t=
2 10 –ve
u
v2 = u2 + 2 gh ... (iii) u - u 2 - 1600
and t=
Here, h is the vertical height of particle in downward 10
direction.
Telegram @unacademyplusdiscounts

48 JEE Main Physics

Now, it is given that Now, y 0 = 25 m, y = 0 m, v 0 = 20 ms-1, a = -10 ms-2


u + u 2 - 1600 u - u 2 - 1600 æ 1ö
- =6 \ 0 = 25 + 20 t + ç ÷ ( -10) t 2
10 10 è2ø
5 t 2 - 20 t - 25 = 0
u 2 - 1600
or =6
5 Solving this quadratic equation for t, we get
or 2
u - 1600 = 30 t =5s

or u 2 - 1600 = 900 Note The second method is better since we do not have to worry about
\ u 2 = 2500 the path of the motion as the motion is under constant acceleration.
or u = ± 50 ms-1
Ignoring the negative sign, we have
Non-Uniformly Accelerated Motion
u = 50 ms-1 When motion of a particle is not uniform i. e. , acceleration
of particle is not constant or acceleration is a function of
Sample Problem 13 A ball is thrown vertically upwards time, then following relations hold for one dimensional
with a velocity of 20 ms–1 from the top of a multistorey building. motion
The height of the point from where the ball is thrown is 25 m ds dv dv
(i) v = (ii) a = =v
from the ground. How long will it be before the ball hits the dt dt ds
ground (Take g = 10 ms-2)? (iii) ds = v dt and (iv) dv = a dt or v dv = a ds
(a) 1 s (b) 3 s Such problems can be solved either by differentiation or
(c) 5 s (d) 7 s integration on applying some boundary conditions.
Interpret (c) There are two ways of solving the problem
Sample Problem 14 A particle is moving with a velocity of
Method I We split two parts the upward motion (A to B) and the
v = (3 + 6 t + 9 t 2) cms-1
downward motion (B to C) and calculate the corresponding time
taken t1 and t 2. Since, the velocity at B is zero, we have The displacement of the particle in the interval t = 5 to t = 8 is
B (a) 1352 cm (b) 1287 cm
y – y0 (c) 1182 cm (b) 11000 cm

A
Interpret (b) Given, v = (3 + 6 t + 9 t 2) cms–1
y
ds
g = –10 m/s2
or = (3 + 6 t + 9 t 2)
dt
or ds = (3 + 6 t + 9 t 2) dt
C
s s 2
v = v 0 + gt \ ò 0 ds = ò 0 (3 + 6 t + 9 t ) dt
0 = 20 - 10 t1 8
\ s = [3 t + 3 t 2 + 3 t 2]5
t1 = 2 s
or s = 1287 cm
This is the time in going from A to B. From B or the point of the
maximum height the ball falls freely under the acceleration due to
gravity. The ball is moving in negative y-direction. We use equation
Sample Problem 15 The motion of a particle along a
1 straight line is described by the function x = (2 t - 3) 2, where x is
y = y 0 + v 0t + at 2
2 in metres and t is in seconds. The acceleration at t = 2 s is
We have, y 0 = 45 m, y = 0 ,v 0 = 0 , a = - g = -1ms -2 (a) 5 ms–2 (b) 6 ms–2 (c) 7 ms–2 (d) 8 ms–2
1 Interpret (d) Position, x = (2 t - 3) 2
0 = 45 + ( -10) t 22
2
dx
Þ t2 = 3 s Velocity v= = 4 (2 t - 3) ms-1
dt
Total time taken by the ball before it hits the ground
dv
= t1 + t 2 = 2 s + 3 s = 5 s and acceleration, a = = 8 ms–2
dt
Method II The total time taken can also be calculated by noting At t = 2 s,
the coordinates of initial and final positions of the ball with respect x = (2 ´ 2 - 3) 2 = 1.0 m
to the origin chosen and using equation
1 \ v = 4 (2 ´ 2 - 3) 2 = 4 ms–1
y = y 0 + v 0t + at 2
2 and a = 8 ms–2
Telegram @unacademyplusdiscounts

Kinematics 49

2.7 Graphs in One Dimensional Motion


The tabular forms of s-t and v-t graphs are given for one dimensional motion with uniform velocity or with constant
acceleration.

Position-Time Graph
(i) Position-time graph gives instantaneous value of displacement at any instant.
(ii) The slope of tangent drawn to the graph at any instant of time gives the instantaneous velocity at that instant.
(iii) The s-t graph cannot make sharp turns.

Table 2.1 Different Cases of Position-Time Graph

S.No. Different Cases s-t Graph The main Features of Graph


1. At rest s Slope = v = 0

t
2. Uniform motion s Slope = constant, v = constant
a=0

s = νt

t
3. Uniformly accelerated motion with s u = 0, i .e.,
u = 0, s = 0 at t = 0 Slope of s-t graph
1 at t = 0, should be zero.
s = at 2
2

t
4. Uniformly accelerated motion with s Slope of s-t graph gradually goes on
u ¹ 0 but s = 0 at t = 0 increasing
1
s = ut + at 2
2

t
5. Uniformly retarded motion s q is decreasing
so, v is decreasing, a is negative

t
t0

Velocity-Time Graph
(i) Velocity-time graph gives the instantaneous value of velocity at any instant.
(ii) The slope of tangent drawn on graph gives instantaneous acceleration.
(iii) Area under v-t graph with time axis gives the value of displacement covered in given time.
(iv) The v-t curve cannot take sharp turns.
Telegram @unacademyplusdiscounts

50 JEE Main Physics

Table 2.2 Different Cases in Velocity-Time Graph


S.No. Different Cases v-t Graph The main Features of Graph
1. Uniform motion v (i) q = 0º
(ii) v = constant
v = constant (iii) Slope of v-t graph = a = 0

t
2. Uniformly accelerated motion with v So slope of v-t graph is constant
u = 0 and s = 0 at t = 0 u = 0 i .e., so, a = constant u = 0
v = at i.e., v = 0 at t = 0

t
3. Uniformly accelerated motion with v Positive constant acceleration because q
u ¹ 0 but s = 0 at t = 0 v = u + at is constant and <90º but the initial
u velocity of the particle is positive

t
4. Uniformly decelerated motion v Slope of v-t graphs = – a (retardation)
u
v = u – at

t
t0
5. Non-uniformly accelerated motion v Slope of v-t graph increases with time.
q is increasing, so, acceleration is
increasing
t
6. Non-uniformly decelerating motion v q is decreasing, so acceleration
decreasing

Note Sample Problem 16 Position-time graph of a particle


1. Slope of s-t or v-t graphs can never be infinite at any point, moving in a straight line is as shown in figure. State whether the
because infinite slope of s-t graphs means infinite velocity. motion is accelerated or not. Describe the motion in detail.
Similarly infinity slope of v-t graph means infinite acceleration. Given, s0 = 20 m and t 0 = 4 s.
Hence, the following graphs are not possible. s
s v

t
t0
–s0
t t

2. At one time, two values of velocity or displacement are not Interpret Slope of s-t graph is constant. Hence, velocity of
possible. Hence, the following graphs are not acceptable particle is constant. Further at time t = 0, displacement of the
s v particle from the mean position is -s0 or –20 m. Velocity of particle,
s 20
s1 v1 v = slope = 0 = = 5 ms–1
t0 4
s2 v2 v = 5 ms–1

+ve
s = – 20 ms–1 s= 0
t t
t0 t0 t=0
3. Different values of displacements in s-t graph corresponding to Motion of the particle is as shown in figure. At t = 0, particle is at
given v-t graphs can be calculated just by calculating areas under –20 m and has a constant velocity of 5 ms–1. At t 0 = 4 s, particle will
v-t graph. There is no need of using equations like v = u + at , etc. pass through its mean position.
Telegram @unacademyplusdiscounts

Kinematics 51

Hot Spot RelativeVelocity


The term relative is frequently used for comparision of displacement, velocity and acceleration of the two objects.
The time rate of change of relative position of one object with respect to another is called relative velocity.
Let two objects A and B are moving along the +ve direction of x-axis. A which will be represented by diagonal OC of rectangle OACB
time t, their displacement from the origin be xA and xB.
\ vrm = v2r + v2m + 2v r v m cos 90° = v2r + v2m
vA A vB
O B
xA –vm O vm
A A
xB
dxA dx vrm q
\ Their velocities arev A = and v B = B vr
dt dt
C B
The displacement of B relative to A.
sBA = xB - xA If q is the angle which vrm makes with the vertical direction then
Rate of change of relative displacement of w.r.t. time is BC v æv ö
tan q = = m or q = tan -1 ç m ÷
d ( xBA ) d OB vr è vr ø
= ( xB - xA )
dt dt Here, angle q is from vertical towards west and is written as q, west of
dxBA dx dx vertical.
Þ = B - A
dt dt dt
\ vBA = vB - vA Note In the above problem if the man wants to protect himself from the
rain, he should hold his umbrella in the direction of relative velocity of rain
w.r.t. man i.e., the umbrella should be held making an angle
Different Cases
q( = tan-1 v m / v r ) west of vertical.
Case I When the two objects move with equal velocities, i. e.,
v A = v B or v B - v A = 0. It means, the two objects stay at 2. Crossing the River
constant distance apart during the whole journey. To cross the river over shortest distance, i. e., to cross the river straight,
y the man should swim upstream making an angle q with OB such that,
tB
jec OB gives the direction of resultant velocity ( vmR ) of velocity of
ob
s(m) tA
jec swimmer and velocity of river water as shown in figure. Let us consider
ob A vR B

x
t (s) vm vmR
q
In this case, the position-time graph of two objects are
parallel straight lines. O
Case II If both objects A and B move along parallel straight lines in AB = v R (velocity of river water)
the opposite direction, then relative of B w.r.t. A is given as OA = vm (velocity of man in still river water)
vBA = vB - ( - vA ) = vB + vA OB = vmR (relative velocity of man w.r.t. river)
and the relative velocity of A w.r.t. B is given by v mR = v2m - v2R
vAB = vB - vA v
In DOAB, sin q = R
vm
Examples of Relative Motion where q is the angle made by man with shortest dstance OB,
1. Relative Velocity of Rain v
tan q = R =
vR
Consider a man walking west with velocity vm , represented by OA. Let vm v - v2
2
m R
the rain be falling vertically downwards with velocity vr , represented
(a) Time taken to cross the river If d be the width of the
by OB . To find the relative velocity of rain w.r.t. man ( i. e., vrm ) bring
river, then time taken cross to the river given by
the man at rest by imposing a velocity -vm on man and apply this
d d
velocity on rain also. Now the relative velocity of rain w.r.t man will be t1 = =
the resultant velocity of vr ( = OB) and - vm ( = OA), v mR v - v2
2
m R
Telegram @unacademyplusdiscounts

52 JEE Main Physics

(b) To cross the river in possible shortest time The man vB = vB cos 60° $i + vB sin 60° $j
should go along OA. Now the swimmer will be going along æ 1ö æ 3ö $
OB, which is the direction of resultant velocity of v m and v R = 6 ç ÷ $i + 6 ç ÷ j = 3 $i + 3 3 $j
è2ø è 2 ø
A vR B
To find the velocity,
x
vBA = vB - v A = (3$i + 3 3 $j) - 6 $i
d vm –vmR
θ = -3 $i + 3 3 $j

upstream
downstream | vBA| = ( -3) 2 + (3 3) 2 = 9 + 27
O
AB v = 36 = 6 ms-1
In DOAB tan q = = R
OA v m Here, $i is –ve and $j is +ve. So, second quadrant is possible.
and v mR = v2m + v2R Direction,
d OB x2 + d 2 coefficient of $j
Time of crossing the river t = = = tan a =
coefficient of $i
v m v mR v2m + v2R
3 3
The boat will be reading the point B instead of point A. If = =- 3
-3
BD = x,
if AB = x, then Þ a = 60°
v x Sv
tan q = R = Þ x= R Sample Problem 18 Two parallel rail tracks run
vm d vm
north-south. Train A moves north with a speed of 54 kmh–1 and
In this case, the man will reach the opposite bank at a distance
train B moves south with a speed of 90 kmh-1. The relative
AB downstream.
velocity of B w.r.t. A is
Sample Problem 17 A man A moves due to east with (a) - 40 ms-1
-1
velocity 6 ms and another man B moves in N-30°E with (b) -20 ms-1
-1
6 ms . Find the velocity of B w.r.t. A. (c) 50 ms-1
-1 -1
(a) 3 ms (b) 3 ms (d) -30 ms-1
-1 -1
(c) 6 ms (d) 6 ms
Interpret (a)
Interpret (d)
Let positive direction of motion be from south to north.
Given, v A = 6 $i,
Given, v A = + 54 kmh –1 = 15 ms–1;
N
vB = - 90 kmh –1 = 25 ms–1
30° v = 6 ms–1 \ The relative velocity of B w.r.t. A
B
vBA = vB - v A = - 25 - 15 = - 40 ms–1
W E
vA = 6 ms–1 i. e. , the train B appears to A to move with a speed of 40 ms–1 from
north to south.
S
Telegram @unacademyplusdiscounts

WORKED OUT
Examples
Solution Distance travelled by A in 5 th second,
Example 1 A particle moving in a straight line covers half
a 9
the distance with speed of 3 ms-1. The other half of the distance sA = 0 + 1 (2 ´ 5 - 1) = a1
2 2
is covered in two equal time intervals with speed of 4.5 ms-1
Distance travelled by B in 3 rd second,
and 7.5 ms-1 respectively. The average speed of the particle
a 5
during this motion is sB = 0 + 2 (2 ´ 3 - 1) = a2
(a) 4.0 ms-1 (b) 5.0 ms-1 2 2
(c) 5.5 ms-1 (d) 4.8 ms-1 As per question, sA = sB
9 5
So, a1 = a2
Solution If s the total distance travelled by the particle, then 2 2
( s /2) s a1 5
t1 = = ; s1 = 4.5 t 2 and s2 = 7.5 t 2 or =
s 6 a2 9
s
Also, s1 + s2 = = 4.5 t 2 + 7.5 t 2 = 12 t 2
2 Example 4 A train accelerating uniformly from rest attains a
So, t 2 = s /24 maximum speed of 40 ms-1 in 20 s. It travels at this speed for
s 2´s s 20 s and is brought to test with uniform retardation in further
\ t = t1 + 2t 2 = + =
6 24 4 40 s. What is the average velocity during this period?
s
Hence average speed, v av = = 4 ms-1 (a) 80 /3 ms-1 (b) 25 ms-1
t (c) 40 ms-1 (d) 30 ms-1

Example 2 The table shows the distance covered in Solution (i) v = u + at1
successive seconds by a body accelerated uniformly from rest
40 = 0 + a ´ 20
Time interval I II III IV \ a = 2 ms-2
Distance 2 6 10 16 v 2 - u2 = 2a s
What is the speed of the body at the end of 4 th second? 40 2 - 0 = 2 ´ 2 ´ s1
-1 -1
(a) 4 ms (b) 8 ms \ s1 = 400 m
(c) 14 ms-1 (d) 16 ms-1 (ii) s2 = v ´ t 2 = 40 ´ 20 = 800 m
a (iii) v = u + at
Solution From, sn = u + (2n - 1)
2 0 = 40 + a ´ 40
a
2 = 0 + (2 ´ 1 - 1) \ a = - 1 ms-2
2
v 2 - u 2 = 2as
\ a = 4 cms-1
0 2 - 40 2 = 2( -1)s3
Again from v = u + at = 0 + 4 ´ 4 = 16 cms-1
\ s3 = 800 m
Example 3 A body A starts from rest with an acceleration a1. Total distance travelled = s1 + s2 + s3
After two seconds, another body B starts from rest with an = 400 + 800 + 800 = 2000 m
acceleration a2. If they travel equal distances in the 5 th second Total time taken = 20 + 20 + 40 = 80 s
after the start of A, then the ratio of a1: a2 is equal to 2000
Average velocity = = 25 ms-1
(a) 5 : 9 (b) 1 : 3 80
(c) 3 : 1 (d) 9 : 1
Telegram @unacademyplusdiscounts

54 JEE Main Physics

4 ´ 10 2 ´ 10 2 ´ 10
Example 5 On displacement-time graphs, two straight lines = + 8 ´ 10 + - - 4 ´ 10
make angle 60° and 30° with time axis. The ratio of the 2 2 2
velocities represented by them is = 20 + 80 + 10 - 10 - 40 = 60 m
x (m)
Example 8 Two particles A and B get 4m closer each second
A while travelling in opposite direction. They get 0.4 m closer
every second while travelling in same direction. The speeds of
A and B are respectively.
60° 30° (a) 2.2 ms-1 and 0.4 ms-1 (b) 2.2 ms-1 and 1.8 ms-1
O t (s)
C B (c) 4 ms-1 and 0.4 ms-1 (d) None of these
(a) 1 : 2 (b) 1 : 3
(c) 2 : 1 (d) 3 : 1
Solution If u and v are speeds of A and B respectively, then
u + v = 4 and u - v = 0.4m
Solution As v µ tan q Adding 2u = 4.4, u = 2.2 ms-1
v1 tan q1 tan 60° 3 v = 1.8 ms-1
= = = =3
v 2 tan q2 tan 30° 1/ 3
Example 9 A car is moving on a road and rain is falling
Example 6 Form the figure find the ratio of speed in first two vertically. Select the correct answer
seconds to the speed in the next four second. (a) The rain will strike the hind screen only
s (b) The rain will strike the front screen only
A (c) The rain will strike both the screens
(d) The rain will not strike any of the screens
s
Solution The relative velocity of rain w.r.t. car is inclined to the
t vertical in the backward direction. Therefore, it will strike the front
0 1 2 3 4 5 6 screen.
(a) 1 : 2 (b) 2 : 1
(c) 2 :1 (d) 3 : 1 Example 10 Two cars are moving in the same direction with
the same validity of 30 kmh-1. They are separated by a distance
s
Solution Speed in two second, v = of 5 km. A truck moving in the opposite direction meets two
2
cars at an interval of 4 min. What is the velocity of the truck?
Speed in time interval 2 to 6 s is (a) 30 kmh -1 (b) 45 kmh -1
s s -1
v¢ = = (c) 60 kmh (d) 65 kmh -1
6 -2 4

\
v s /2
= =2 Solution Let v be the velocity of truck in kmh -1.
v ¢ s/ 4 Relative velocity of truck w.r.t. car = (v + 30)
Separation, d = 5 km
Example 7 The graph below represents motion of a car. The 4 5
Time of crossing, t = 4 min = h=
displacement of the car in 20 s is 60 v + 30
v (ms–1)
On solving, v = 45 kmh -1

10
Example 11 A ball is thrown upwards with a speed u from a
height h above the ground. The time taken by the ball to hit the
t (s)
4 8 12 16 20 ground is
(a) 2h / g (b) 8h / g
–10
2
u + 2gh u 2h
(c) (d) +
g g g
(a) 60 m (b) 20 m
(c) 90 m (d) 10 m Solution Time taken to reach the highest point from the height h
is obtained from v = u + at
Solution Displacement in 20 s = algebraic sum of the area under u
velocity-time graph 0 = u - gt , t =
g
Telegram @unacademyplusdiscounts

Kinematics 55

Height attained above h is obtained from Example 13 Acceleration due to gravity on moon is
v 2 - u2 = 2a s 1.6 ms -1. An inflated balloon is released on moon. It will
u2 (a) move down with acceleration1.6 ms-2
0 - u 2 = 2( - g ) s, s =
2g (b) move up with acceleration1.6 ms-2
u 2
(c) move up with acceleration 9.8 ms-2
Total height = +h
2g (d) move down with acceleration 9.8 ms-2
Time taken to hit the ground is obtained from
1 Solution The balloon will move down with an acceleration
s = ut + at 2 1.6 ms-2, as moon has no atmosphere and no upward thrust acts on
2
balloon due to atmosphere.
u2 1
+ h = 0 + gt 2
2g 2
Example 14 A ball is dropped from a bridge122.5 m above
(u 2 + 2g h) a river. After the ball has been falling for 2 s, a second ball is
\ Total time taken t =
g thrown straight down after it. What must the initial velocity of
the second ball be so that both hit the water at the same time?
Example 12 Two balls are dropped to ground from (a) 49 ms-1 (b) 55.5 ms-1
different heights. One ball is dropped 2 s after the other but the (c) 26.1ms-1 (d) 9.8 ms-1
first ball hits the ground in 5 s, then, the difference in their
heights is Solution Let 1st ball hits water in t sec.
(a) 125 m 1
For first ball, s = ut + at 2
(b) 45 m 2
(c) 80 m 1
122.5 = 0 + ´ 9.8 t 2 = 4.9 t 2
(d) 170 m 2
122.5
Solution Second ball hits the ground in (5 - 2) = 3 s t= = 25 = 5s
4.9
First ball hits the ground in 5s
For second ball,
1 2
From, s = ut + at 1
2 122.5 = u(5 - 2) + ´ 9.8 (5 - 2) 2
1 2
h1 = 0 + ´ 10 ´ 5 2 = 3u + 44.1
2
1 3u = 122.5 - 44.1 = 78.4
h2 = 0 + ´ 10 ´ 3 2
2 u = 26.1 ms-1
h1 - h2 = 5 (5 2 - 3 2) = 80 m
Telegram @unacademyplusdiscounts

Start Practice for


JEE Main
Round I (Topically Divided Problems)

Speed, Velocity and Acceleration 5. The displacement of a body along x-axis depends on
1. A wheel of radius 1 m rolls forward half a revolution time as x = t + 1. Then, the velocity of body
on a horizontal ground. The magnitude of the (a) increase with time (b) decrease with time
displacement of the point of the wheel initially in (c) independent of time (d) None of these
contact with the ground is 6. A car moving along a straight highway with speed of
(a) 2 p (b) 2 p 126 km/h is brought to a stop within a distance of
200 m. What is the retardation of the car (assumed
(c) p2 + 4 (d) p
uniform) and how long does it take for the car to stop?
2. A point particle starting from rest has a velocity that [NCERT]
increases linearly with time such that v = pt, where (a 3.06 m / s2 and11.4 s (b) 2.06 m / s2 and11.4 s
p = 4 ms–2. The distance covered in the first 2 s will be (c) 3.06 m / s2 and10.4 s (d) 3.06 m / s2 and 4.1s
(a) 6 m (b) 4 m
(c) 8 m (d) 10 m 7. A particle of mass m is initially
situated at the point P inside a
3. Among the four graphs, there is only one graph for hemispherical surface of radius r
which average velocity over the time intervel (0, T) as shown in figure. A horizontal
P
can vanish for a suitably chosen T. Which one is it? acceleration of magnitude a 0 is suddenly produced on
[NCERT Exemplar]
the particle in the horizontal direction. If
x x
gravitational acceleration is neglected, the time
taken by particle to touch the sphere again is
(a) t (b)
4 r sin a 4 r tan a
(a) (b)
a0 a0
t
4 r cos a
x x
(c) (d) None of these
a0
8. A body starts from rest and moves with a constant
(c) (d) acceleration. The ratio of distance covered in the nth
second to the distance covered in n second is
t t 2 1 1 1 2 1 2 1
(a) - (b) - (c) - (d) +
n n2 n2 n n2 n n n2
4. A particle moves with constant acceleration and v1, v2
and v3 denote the average velocities in the three 9. A particle moving with a uniform acceleration along a
successive intervals t1, t2 and t3 of time. Which of the straight line covers distance a and b in successive
following relations is correct? intervals of p and q second. The acceleration of the
v1 - v2 t1 - t2 v1 - v2 t1 - t2 particle is
(a) = (b) =
v2 - v3 t2 + t3 v2 - v3 t1 - t2 pq ( p + q) 2( aq - bp )
(a) (b)
v - v2 t1 - t2 v1 - v2 t1 + t2 2 ( bp - aq) pq ( p - q)
(c) 1 = (d) =
v2 - v3 t2 - t3 v2 - v3 t2 + t3 bp - aq 2( bp - aq)
(c) (d)
pq ( p - q) pq ( p - q)
Telegram @unacademyplusdiscounts

Kinematics 57

10. A bee flies a line from a point A to another point B in 17. The driver of a car moving with a speed of 10 ms -1
-1
4 s with a velocity of |t – 2| ms . The distance sees a red light ahead, applies brakes and stops after
between A and B in metre is covering 10 m distance. If the same car were moving
(a) 2 (b) 4 (c) 6 (d) 8 with a speed of 20 ms -1, the same driver would have
11. A 2 m wide truck is moving with a uniform speed stopped the car after covering 30 m distance. Within
v0 = 8 ms -1 along a straight horizontal road. A what distance the car can be stopped if travelling
pedestrian starts to cross the road with a uniform with a velocity of 15 ms -1? Assume the same reaction
speed v when the truck is 4 m away from him. The time and the same deceleration in each case.
minimum value of v, so that he can cross the road (a) 18.75 m (b) 20.75 m (c) 22. 75 m (d) 25 m
th
safely is 18. A lift is coming from 8 floor and is just about to
(a) 2.62 ms–1 (b) 4.6 ms–1 reach 4 th floor. Taking ground floor as origin and
(c) 3.57 ms–1 (d) 1.414 ms–1 positive direction upwards for all quantities, which
12. A bus moves over a straight level road with a one of the following is correct? [NCERT Exemplar]

constant acceleration a. A body in the bus drops a ball (a) x < 0, v < 0, a > 0 (b) x > 0, v < 0, a < 0
outside. The acceleration of the ball with respect to (c) x > 0, v < 0, a > 0 (d) x > 0, v > 0, a < 0
the bus and the earth are respectively 19. A target is made of two plates, one of wood and the
(a) a and g other of iron. The thickness of the wooden plate is
(b) a + g and g - a 4 cm and that of iron plate is 2 cm. A bullet fired goes
(c) a2 + g2 and g through the wood first and then penetrates 1 cm into
iron. A similar bullet fired with the same velocity
(d) a2 + g2 and a
from opposite direction goes through iron first and
13. Two cars move in the same direction along parallel then penetrates 2 cm into wood. If a1 and a2 be the
roads. One of them is a 100 m long travelling with a retardation offered to the bullet by wood and iron
velocity of 7.5 ms -1. How long will it take for the first plates respectively, then
car to overtake the second car? (a) t1 + t2 (b) a2 = 2 a1
(a) 24 s (c) 40 s (c) a1 = a2 (d) data insufficient
(c) 60 s (d) 80 s 20. If the velocity v of a particle moving along a straight
14. A car is moving along a straight road with uniform line decreases linearly with its displacement s from
acceleration. It passes through two points P and Q 20 ms -1 to a value approaching zero at s = 30 m, then
separated by a distance with velocities 30 kmh–1 and acceleration of the particle at s = 15 m is
40 kmh -2 respectively. The velocity of car midway
between P and Q is 20
(a) 33.3 km–1 (b) 1 km–1
(c) 25 2 km–1 (d) 35.35 km–1 v
(in ms–1)
15. A particle starts from the origin and moves along the
X-axis such that the velocity at any instant is given O
u (in m)
30
by 4 t 3 - 2 t, where t is in second and velocity is in
ms–1. What is the acceleration of the particle when it 2 –2 2 20 20
(a) ms (b) - ms–2 (c) ms–2 (d) - ms–2
is 2 m from the origin? 3 3 3 3
(a) 10 ms–2 (b) 12 ms–2 21. The velocity of a particle moving in a straight line
(c) 22 ms–2 (d) 28 ms–2 varies with time in such a manner that v versus t
16. The retardation experienced by a moving motor boat, graph is velocity is vm and the total time of motion is t0
dv
after its engine is cut-off, is given by = - kv 3, v
dt
where k is a constant. If v0 is the magnitude of the
velocity at cut-off, the magnitude of the velocity at vm
time t after the cut-off is
v0 t
(a) v 0 (b) t0
2 p
- kt v0 (i) Average velocity of the particle is vm
(c) v 0 e (d) 4
2v20 kt + 1 (ii) Such motion cannot be realized in practical terms
Telegram @unacademyplusdiscounts

58 JEE Main Physics

(a) Only (i) is correct 28. The engine of a train can impart a maximum
(b) Only (ii) is correct acceleration of 1 ms–2 and the brakes can give a
(c) Both (i) and (ii) are correct maximum retardation of 3 ms–2. The least time
(d) Both (i) and (ii) are wrong during which a train can go from one place to the
22. A police van moving on a highway with a speed of other place at a distance of 1.2 km is nearly
30 km/h fires a bullet at a thief’s car speeding away in (a) 108 s (b) 191 s
the same direction with a speed of 192 km/h . If the (c) 56.6 s (d) time is fixed
muzzle speed of the bullet is 150 m/s, with what speed 29. The acceleration of a particle increasing linearly with
does the bullet hit the thief’s car? (Note Obtain that time t is bt. The particle starts from the origin with
speed which is relevant for damaging the thief’s car. an initial velocity v0 . The distance travelled by the
[NCERT] particle in time t will be
(a) 105 m/s (b) 100 m/s 1 3 1 2
(a) v 0 t + bt (b) v 0 t + bt
(c) 95 m/s (d) 110 m/s 6 6
1 1
23. A particle starts from rest and travels a distance s (c) v 0 t + bt3 (d) v 0 t + bt2
3 3
with uniform acceleration, then it travels a distance
2s with uniform speed, finally it travels a distance 3s
with uniform retardation and comes to rest. If the
Graphical Representation
complete motion of the particle in a straight line 30. The displacement-time graph of a moving particle is
then the ratio of its average velocity to maximum shown below. The instantaneous velocity of the
velocity in particle is negative at the point
(a) 6/7 (b) 4/5 (c) 3/5 (d) 2/5 D
24. A particle moving in a straight line with uniform

Displacement
acceleration is observed to be a distance a from a E F
fixed point initially. It is at distances b, c, d from the C
same point after n, 2n, 3n second. The acceleration of
the particle is
c - 2b + a c + b+ a Time
(a) (b)
n2 9 n2 (a) C (b) D (c) E (d) F
c + 2b + a c -b+ a 31. In the given v-t graph, the distance travelled by the
(c) (d)
4 n2 n2 body in 5 s will be
25. A body is moving along a straight line path with 40
constant velocity. At an instant of time the distance
v 30
of time the distance travelled by it is s and its (in ms–1) 20
displacement is D, then
10
(a) D < s (b) D > s
(c) D = s 0 Tme (in s)
2 3 4 5 D £ s
(d)
26. Three particles start from the origin at the same –10
time, one with a velocity v1 along x-axis, the second –20
along the y-axis with a velocity v2 and the third along (a) 20 m (b) 40 m (c) 80 m (d) 100 m
x = y line. The velocity of the third so that the three
32. The displacement-time graphs of two moving
may always lie on the same line is
particles make angles of 30° and 45° with the x-axis.
vv 2 v1v2 3 v1v2
(a) 1 2 (b) (c) (d) zero The ratio of the two velocities is
v1 + v2 v1 + v2 v1 + v2

27. In one dimensional motion, instantaneous speed v


satisfies 0 £ v < v0 .
Displacement

[NCERT Exemplar]
(a) The displacement in time T must always take
non-negative values
(b) The displacement x in time T satisfies - v 0 T < x < v 0T 30°
45°
(c) The acceleration is always a non-negative number Time
(d) The motion has no turning points (a) 3 : 1 (b) 1 : 1 (c) 1 : 2 (d) 1 : 3
Telegram @unacademyplusdiscounts

Kinematics 59

33. A rocket is fired upwards. Its engine explodes fully is 37. The given graph shows the u
12 s. The height reached by the rocket as calculated variation of velocity with v0
from its velocity-time graph is displacement. Which one of the
1200 graph given below correctly
represents the variation of x0
x
acceleration with displacement?
v (in ms)–1

a a

(a) (b)
x x
12 t (in s) 132
(a) 1200 × 66 m (b) 1200 × 132 m
1200 a a
(c) m (d) 1200 × 122 m
12
34. v-t graph for a particle is v (m/s) (c) (d)
x
as shown. The distance
8
travelled in the first 4 s is
(a) 12 m 4 38. A ball A is thrown up vertically with a speed u and at
(b) 16 m the same instant another ball B is released from a
(c) 20 m height h. At time t, the speed of A relative to B is
(d) 24 m 0 2 4 6 t (s) (a) u (b) 2u
35. The velocity-time graph of a (c) u - gt (d) (u2 - gt )
body is shown in figure. The C 39. A body falls freely from rest. It covers as much
ratio of the ..... during the
distance in the last second of its motion as covered in
intervals OA and AB is ..... v
the first three seconds. The body has fallen for a time
(a) average velocities : 2 60° 30°
O of
OA 1 A t B
(b) : (a) 3 s (b) 5 s
AB 3
(c) 7 s (d) 9 s
(c) average accelerations, same as distances covered
1 40. Rain is falling vertically with a speed of 30 m/s. A
(d) distances covered :
2 woman rides a bicycle with a speed of 10 m/s in the
north to south direction. What is the direction in
36. Figure shows the acceleration-time graphs of a
which she should hold her umbrella? [NCERT]
particle. Which of the following represents the
corresponding velocity-time graphs? (a) 18° with vertical (b) 18° with horizontal
(c) 28° with vertical (d) 28° with horizontal
41. A ball P is dropped vertically and another ball Q is
a thrown horizontally with the same velocities from
the same height and at the same time. If air
resistance is neglected, then
(a) ball P reaches the ground first
t
(b) ball Q reaches the ground first
v v (c) both reach the ground at the same time
(a) (b) (d) the respective masses of the two balls will decide the time
42. A particle moves along x-axis as
t t x = 4 ( t - 2) + a ( t - 2)2
v v Which of the following is true?
(a) The initial velocity of particle is 4
(c) (d)
(b) The acceleration of particle is 2 a
(c) The particle is at origin at t = 0
t t (d) None of the above
Telegram @unacademyplusdiscounts

60 JEE Main Physics

43. A ball is thrown vertically upwards. It was observed 50. A body thrown vertically upward with an initial
at a height h twice with a time a interval Dt the initial velocity u reaches maximum height in 6 second. The
velocity of the ball is ratio of the distances travelled by the body in the first
æ gDt ö
2 second and seventh second is
(a) 8 gh + g2 ( Dt )2 (b) 8 gh + ç ÷ (a) 1 : 1 (b) 11 : 1 (c) 1 : 2 (d) 1 : 11
è 2 ø
(c) 1 / 2 8 gh + g2 ( Dt )2 (d) 8 gh + 4 g2 ( Dt )2
51. A juggler keeps on moving four balls in the air throws
the balls in regular interval of time. When one ball
44. A frictionless wire AB is fixed on a sphere of radius R. leaves his hand (speed = 20 ms -1), the position of
A very small spherical ball slips on this wire the time other ball will be (Take g = 10 ms -2 )
taken by this ball to slip from A to B is (a) 10 m, 20 m, 10 m (b) 15 m, 20 m, 15 m
(c) 5 m, 15 m, 20 m (d) 5 m, 10 m, 20 m
2 gR
(a) A
52. The velocity-time graph of a particle in
g cos q
cos q θ one-dimensional motion is shown in figure. Which of
(b) 2 gR the following formulae are correct for describing the
g B O motion of the particle over the time interval t1 to t2 .
R
(c) 2 v
g
gR C
(d)
g cos q

45. A jet airplane travelling at a speed of 500 km/h ejects


its products of combustion at the speed of 1500 km/h O t1 t2 t
relative to the jet plane. What is the speed of the
1
latter with respect to an observer on the ground? (i) x( t2 ) = x( t1) + v( t1)( t2 - t1) + a( t2 - t1)2
[NCERT]
2
(a) -1000 km / h (b) 1000 km / h (ii) v( t2 ) = v( t1) + a( t2 - t1)
(c) 100 km / h (d) -100 km / h é x( t ) - x( t1) ù
(iii) vav = ê 2 ú
46. A body is thrown vertically up with a velocity u. It ë ( t2 - t1) û
passes three points A, B and C in its upward journey [v( t2 ) - v( t1)]
u u u (iv) aav =
with velocities , and repectively. The ratio of ( t2 - t1)
2 3 4 1
the separations between points A and B and between aav ( t2 - t1)2
(v) x( t2 ) = x( t1) + vav ( t2 - t1) +
AB 2
B and C, i. e., is (vi) x( t2 ) - x( t1) = Area under v-t curve bounded by
BC
10 20 the t-axis and the dotted line shown
(a) 1 (b) 2 (c) (d) (a) (iii) and (vi) (b) (iii), (iv) and (vi)
7 7
(c) (ii), (iii) and (iv) (d) (iv) and (vi)
47. A boy released a ball from the top of a building. It will
clear a window 2 m high at a distance 10 m below the 53. Water drops fall from a tap on the floor 5 m below at
top in nearly regular intervals of time, the first drop striking the
(a) 1 s (b) 1.3 s (c) 0.6 s (d) 0.13 s floor when the fifth drop begins to fall. The height
48. A stone is allowed to fall from the top of a tower at which the third drop will be, from ground, at
100 m high and at the same time another stone is the instant when first drop strikes the ground, will be
projected vertically upwards from the ground with a (g = 10 ms–2)
velocity of 254 ms -1. The two stones will meet after (a) 1.25 m (b) 2.15 m
(a) 4 s (b) 0.4 s (c) 0.04 s (d) 40 s (c) 2.73 m (d) 3.75 m

49. From a balloon rising vertically upwards at 5 m/s a 54. From an elevated point A, a stone is projected
stone is thrown up at 10 m/s relative to the balloon. vertically upwards. When the stone reaches a
Its velocity with respect to ground after 2 s is (assume distance h below A, its velocity is double of what was
g = 10 m /s2 ) at a height above A? The greatest height attained by
(a) 0 (b) 20 m/s the stone is
h 2h h 5h
(c) 10 m/s (d) 5 m/s (a) (b) (c) (d)
3 2 2 3
Telegram @unacademyplusdiscounts

Kinematics 61

55. A ball is thrown vertically upwards from the top of a 63. A particle covers 4 m, 5 m, 6 m and 7 m in 3rd, 4th,
tower of height h with velocity v. The ball strikes the 5th and 6th second respectively. The particle starts
ground after (a) with an initial non-zero velcoity and moves with uniform
v é 2 gh ù v é 2 gh ù acceleration
(a) ê1 + 1 + 2 ú (b) ê1 + 1 - 2 ú (b) from rest and moves with uniform velocity
g ë v û g ë v û
1 /2 1 /2 (c) with an initial velocity and moves with uniform velcoity
v æ 2 gh ö v æ 2 gh ö (d) from rest and moves with uniform acceleration
(c) ç1 + 2 ÷ (d) ç1 - 2 ÷
g è v ø g è v ø 11
64. A balls is released from the top of a tower travels of
56. A body freely falling from rest has a velocity v after it 36
falls through distance h. The distance it has to fall the height of the tower in the last second of its
down further for its velocity to become double is journey. The height of the tower is (Take g = 10 ms -2 )
(a) h (b) 2 h (c) 3 h (d) 4 h (a) 11 m (b) 36 m
(c) 47 m (d) 180 m
57. Two balls A and B are thrown simultaneously from
the top of a tower. A is thrown vertically up with a
speed of 4 ms–1. B is thrown vertically down with a Relative Motion
speed of 4 ms–1. The ball A and B hit the ground with 65. At a metro station, a girl walks up a stationary
speed v A and vB respectively. Then, escalator in time t1. If she remains stationary on the
(a) v A < v B (b) v A > v B (c) v A ³ v B (d) v A = v B escalator, then the escalator take her up in time t2 .
58. A particle starting from rest falls from a certain The time taken by her to walk up on the moving
height. Assuming that the value of acceleration due escalator will be [NCERT Exemplar]
to gravity remains the same throughout motion, its (a) (t1 + t2 )/2 (b) t1t2 /(t2 - t1 )
displacements in three successive half second (c) t1t2 /(t2 + t1 ) (d) t1 - t2
intervals are S1, S2 , S3. Then, 66. A 120 m long train is moving in a direction with speed
(a) S1 : S2 : S3 = 1 : 5 : 9 (b) S1 : S2 : S3 = 1 : 2 : 3 20 m/s. A train B moving with 30 m/s in the opposite
(c) S1 : S2 : S3 = 1 : 1 : 1 (d) S1 : S2 : S3 = 1 : 3 : 5 direction and 130 m long crosses the first train in a
59. A ball thrown upward from the top of a tower with time.
speed v reaches the ground in t1 second. If this ball is (a) 6 s (b) 36 s
thrown downward from the top of the same tower (c) 38 s (d) None of these
with speed v it reaches the ground in t2 second. In 67. For a body moving with relative speed of the velocity
what time the ball shall reach the ground if it is is doubled, then
allowed to falls freely under gravity from the top of
(a) its linear momentum is doubled
the tower?
(b) its linear momentum will be less than doubled
t1 + t2 t1 - t2
(a) (b) (c) t1t2 (d) t1 + t2 (c) its linear momentum will be more than doubled
2 2
(d) its linear momentum remains unchanged
60. A ball is dropped on the floor from a height of 10 m. It 68. An express train is moving with a velocity v1 its driver
rebounds to a height of 2.5 m. If the ball is in contact
finds another train is moving on the same track in
with the floor for 0.01 s, the average acceleration
the same direction with velocity v2 . To avoid collision
during contact is nearly (Take g = 10 ms -2 )
driver applies a retardation a on the train. The
(a) 500 2 ms–2 upwards (b) 1800 ms–2 downwards minimum time of avoiding collision will be
(c) 1500 5 ms–2 upwards (d) 1500 2 ms–2 downwards v1 - v2 v21 - v22
(a) t = (b) t =
61. A stone thrown vertically upwards attains a a 2
maximum height of 45 m. In what time the velocity of (c) None (d) Both (a) and (b)
stone become equal to one-half the velocity of throw? 69. Rain drops fall vertically at a speed of 20 ms -1. At
(Given g = 10 ms -2 ) what angle do they fall on the wind screen of a car
(a) 2 s (b) 1.5 s (c) 1 s (d) 0.5 s moving with a velocity of 15 ms -1, if the wind screen
62. A body released from a great height falls freely velocity inclined at an angle of 23° to the vertical?
towards the earth. Another body is released from the é -1 æ 4 ö ù
same height exactly one second later. The separation êëcot çè 3 ÷ø » 36° úû
between the two bodies two second after the release (a) 60º (b) 30º
of the second body is (c) 45º (d) 90º
(a) 9.8 m (b) 4.9 m (c) 24.5 m (d) 19.6 m
Telegram @unacademyplusdiscounts

62 JEE Main Physics

70. A man wants to reach point B (a) east-north direction (b) west-north direction
B on the opposite bank of a (c) south-east direction (d) None of the above
river flowing at a speed as 78. A boat crosses a river from part A to part B which are
u
shown in figure. What just on opposite side. The speed of the water is vw and
minimum speed relative to 45°
that of boat is vb relative to still water. Assume
water should the man have vb = 2 vw . What is the time taken by the boat? If it has
A
so that he can reach point B? to cross the river directly on the AB line.
(a) u 2 (b) u / 2 (c) 2u (d) u /2 2D 3D D D 2
(a) (b) (c) (d)
71. A steam boat goes across a lake and comes back (i) on vb 3 2 vb vb 2 vb
a quiet day when the water is still and (ii) on a rough 79. Two cars A and B are moving with same speed of
day when there is a uniform current so as to help the 45 km/h along same direction. If a third car C coming
journey onwards and to impede the journey back. If from the opposite direction with a speed of 36 km/h
the speed of the launch on both days was same, the meets two cars in an interval of 5 minutes. The
time required for complete journey on the rough day, distance between cars A and B should be (in km)
as compared to the quiet day will be (a) 6.75 (b) 7.25 (c) 5.55 (d) 8.35
(a) more (b) less
(c) same (d) None of these 80. Two trains A and B of length 400 m each are moving
on two parallel tracks with a uniform speed of
72. Two trains travelling on the same track are 72 km/h in the same direction, with A ahead of B. The
approaching each other with equal speeds of 40 ms–1. driver of B decides to overtake A and accelerates by
The drivers of the trains begin to decelerate 1 m/s2 . If after 50 s, the guard of B just brushes past
simultaneously when they are just 2 km apart. If the the driver of A, what was the original distance
decelerations are both uniform and equal, then between them? [NCERT]
the value of deceleration to barely avoid collision
(a) 1250 m (b) 1350 m
should be
(c) 1450 m (d) None of these
(a) 0.8 ms–2 (b) 2.1 ms–2 (c) 11.0 ms–2 (d) 13.2 ms–2
81. On a two lane road, car A is travelling with a speed of
73. A 210 m long train is moving due North at a speed of 36 km/h. Two cars B and C approach car A in opposite
25 m/s. A small bird is flying due South, a little above
directions with a speed of 54 km/h each. At a certain
the train with speed 5 m/s. The time taken by the bird
instant, when the distance AB is equal to AC, both
to cross the train is
being 1 km, B decides to overtake A before C does. In
(a) 6 s (b) 7 s (c) 9 s (d) 10 s
this case, the acceleration of car B is required to avoid
74. A police jeep is chasing with velocity of 45 km/h a an accident [NCERT]
theif in another jeep moving with velocity 153 km/h. (a) 1m / s2 (b) 0.1m / s2 (c) 1.9 m / s2 (d) 0.2m / s2
Police fires a bullet with muzzle velocity of 180 m/s.
The velocity with which is will strike of the car of the 82. A passenger arriving in a new town wishes to go from
thief is the station to a hotel located 10 km away on a
(a) 150 m/s (b) 27 m/s (c) 450 m/s (d) 250 m/s straight road from the station. A dishonest cabman
takes him along a circuitoius path 23 km long and
75. A boat is sent across a river with a velocity of 8 km/h. reaches the hotel in 28 min. What are the average
If the resultant velocity of boat is 10 km/h, then speed of the taxi and the magnitude of average
velocity of river is velocity respectively (in km/h)? [NCERT]
(a) 10 km/h (b) 8 km/h (c) 6 km/h (d) 4 km/h
(a) 49.3 and 21.43 (b) 48.3 and 24.43
76. The distance between two particles moving towards (c) 21 and 20 (d) 21.3 and 49.3
each other is decreasing at the rate of 6 m/s. If these
83. A man can swim with a speed of 4 km/h in still water.
particles travel with same speed and in the same
How long does he take to cross a river 1 km wide, if
direction then the separatioon increase at the rate of
the river flows steadily 3 km/h and he makes his
4 m/s. The particles have speed as
strokes normal to the river current. How far down
(a) 5 m/s 1 m/s (b) 4 m/s; 1 m/s
the river does he go when he reaches the other bank?
(c) 4 m/s; 2 m/s (d) 5 m/s; 2 m/s
[NCERT]
77. A train is moving towards east and a car is along (a) 850 m (b) 750 m
north, both with same speed. The observed direction (c) 650 m (d) None of these
of a car to the passenger in the train is
Telegram @unacademyplusdiscounts

Kinematics 63

Round II (Mixed Bag)

Only One Correct Option 8. A particle is dropped vertically from rest from a
1. An automobile travelling with a speed of 60 kmh -1 height. The time taken by it to fall through successive
can brake to stop with a distance of 20 m. If the car is distances of 1 m each, will then be
going twice as fast i. e., 120 kms -1, the stopping (a) all being equal to 2 / 9 second
distance will be [AIEEE 2004] (b) in the ratio of the square roots of the integers 1, 2, 3, …
(a) 20 m (b) 40 m (c) 60 m (d) 80 m (c) in the ratio of the difference in the square roots of the
2. Two balls of same size but the density of one is greater integers is ( 2 - 1) ( 3 - 2 ) ( 4 - 3 )
than that of the other are dropped from the same (d) in the ratio of the reciprocal of the square roots of the
height, then which ball will reach the earth first æ 1 1 1 1 ö
integers is ç , , , ÷
(air resistance is negligible)? è 1 2 3 4ø
(a) Heavy ball
(b) Light ball 9. A man throws balls with the same speed vertically
(c) Both simultaneously upwards one after the other at an interval of 2 s.
(d) Will depend upon the density of the balls What should be the speed of the throw so that more
than two balls are in the sky at any time?
3. A boat takes two hours to travel 8 km and back in still
(Given g = 9.8 m/s2)
water. If the velocity of water 4 kmh -1, the time
(a) At least 0.8 m/s
taken for going ups tream 8 km and coming back is
(b) Any speed less than 19.6 m/s
(a) 2 h
(c) Only with speed 19.6 m/s
(b) 2 h 40 min
(d) More than 19.6 m/s
(c) 1 h 20 min
(d) cannot be estimated with the information given 10. A motion boat covers a given distance in 6 h moving
4. A particle moving in a straight line covers half the down stream of a river. It covers the same distance in
distance with speed of 3 m/s. The other half of the 10 h moving upstream. The time (in hour) it takes to
distance is covered in two equal time intervals and cover the same distance in still water is
with speeds of 4.5 m/s and 7.5 m/s respectively. The (a) 6 h (b) 7.5 h
average speed of the particle during this motion is (c) 10 h (d) 15 h
(a) 4.0 m/s (b) 5.0 m/s (c) 5.5 m/s (d) 4.8 m/s 11. A point initially at rest moves along x-axis. Its
5. In a race for 100 m dash, the first and the second acceleration varies with time as a = (6 t + 5) m/s2. If it
runners have a gap of one metre at the mid way starts from origin, the distance covered in 2 s is
stage. Assuming the first runner goes steady, by (a) 20 m (b) 18 m
what percentage should the second runner increases (c) 16 m (d) 25 m
his speed just to win the race.
(a) 2% (b) 4%
12. From the top of a tower of height 50 m, a ball is
(c) more than 4% (d) less than 4% thrown vertically upwards with a certain velocity. It
hits the ground 10 s after it is thrown up. How much
6. Two cars A and B are travelling in the same direction time does it take to cover a distance AB where A and
with velocities v A and vB (v A > vB ). When the car A is
B are two points 20 m and 40 m below the edge of the
at a distance s behind car B, the driver of the car A
tower? (g = 10 ms -2 )
applies the brakes producing a uniform retardation
(a) 2.0 s (b) 1.0 s
a, there will be no collision when
2 2 (c) 0.5 s (d) 0.4 s
(v A - v B ) (v A - v B )
(a) s < (b) s =
2a 2a 13. The acceleration of a particle is increasing linearly
(v A - v B ) 2
(v A - v B ) 2 with time t as bt. The particle starts from the origin
(c) s ³ (d) s £ with an initial velocity v0 . The distance travelled by
2a 2a
the particle in time t, is
7. A bird flies for 4 s with a velocity of |t–2|ms -1 in a 1 2 1 3
straight line, where t = time in second. It covers a (a) v 0t + bt (b) v 0t + bt
3 6
distance of 1 1
(c) v 0t + bt3 (d) v 0t + bt2
(a) 8 m (b) 6 m (c) 4 m (d) 2 m 3 2
Telegram @unacademyplusdiscounts

64 JEE Main Physics

14. A body of mass m is resting on a wedge of angle q as 19. A graph of x versus t is shown in figure. Choose
shown in figure. The wedge is given an acceleration correct alternatives from below. [NCERT Exemplar]
a. What is the value of a that the mass m just falls x
freely? B
A C
m
E
θ
(a) g (b) g sin q t
D
(c) g tan q (d) g cot q
15. From the top of a tower, a stone is thrown up and (a) The particle was released from rest at t = 0
reaches the ground in time t1 = 9 s. A second stone is (b) At B, the acceleration a > 0
thrown down with the same speed and reaches the (c) At C, the velocity and the acceleration vanish
ground in time t2 = 4 s. A third stone is released (d) The speed at D exceeds that at E
from rest and reaches the ground in time t3, which is 20. The motion of a body falling from rest in a resisting
equal to medium is described by the equation
(a) 6.5 s (b) 6.0 s dv
5 = A - Bv
(c) s (d) 65 s dt
36
where A and B are constants. Then
(a) initial acceleration of the body is A
More Than One Correct Option (b) the velocity at which acceleration becomes zero is A/B
16. The motion of a body is given by the equation A
(c) the velocity at any time t is (1 - e Bt )
dv ( t) B
= 6.0 - 3 v ( t), where v( t) is speed in ms–1 and t in
dt (d) All of the above are wrong
second. If body was at rest at t = 0 21. A spring with one end attached to a mass and the
(a) the terminal speed is 2.0 ms–1 other to a rigid support is stretched and released
(b) the speed varies with the times as v (t ) = 2 (1 - e -3 t ) ms -1 [NCERT Exemplar]
(c) the speed is 1.0 ms–1 when the acceleration is half the (a) Magnitude of acceleration, when just released is
initial value maximum
(d) the magnitude of the initial acceleration is 6.0 ms–2 (b) Magnitude of acceleration, when at equilibrium position
is maximum
17. An elevator ascends with an upward acceleration
(c) Speed is maximum when mass is at equilibrium position
of 2.0 ms -2 . At the instant its upward speed is
(d) Magnitude of displacement is always maximum whenever
2.5 ms -1, loose bolt is dropped from the ceiling of the
speed is minimum
elevator 3.0 m from the floor. If g = 10 ms–2, then
(a) the time of flight of the bolt from the ceiling to floor of 22. The displacement (x) of a particle depends on time (t)
the elevator is 0.11 s as
x = a t2 - b t 3
(b) the displacement of the bolt during the free fall relative to
(a) The particle will come to rest after time 2 a / 3 b
the elevator shaft is 0.75 m
(b) The particle will return to its starting point after time a / b
(c) the distance covered by the bolt during the free fall
relative to the elevator shaft is 1.38 m (c) No net force will act on the particle at t = a /3 b
(d) the distance covered by the bolt during the free fall (d) The initial velocity of the particle was zero but its initial
relative to the elevator shaft is 2.52 m acceleration was not zero

18. A particle of mass m moves on the x-axis as follows : it 23. Suppose a and v denotes the acceleration and
velocity respectively of a body in one dimensional
starts from rest at t = 0 from the point x = 0 and comes
motion, then
to rest at t = 1 at the point x = 1. No other information
is available about its motion at intermediate time (a) speed must increase when a > 0
(b) speed will increase when v and d are > 0
(0 < t < 1). If a denotes the instantaneous acceleration
of the particle, then (c) speed must decreases when a < 0
(d) speed will decrease when v < 0 and a > 0
(a) a cannot remain positive for all t in the interval 0 £ t £ 1
(b) |a | cannot exceed 2 at any points in its path 24. A particle is projected vertically upwards in vacuum
(c) |a | must be ³ 4 at some point or points in its path with a speed v.
(d) a must change sign during the motion but no other (a) The time taken to rise to half its maximum height is half
assertion can be made with the information given the time taken to reach its maximum height
Telegram @unacademyplusdiscounts

Kinematics 65

(b) The time taken to rise to three-fourth of its maximum (a) The displacement of the particle in time 2 T is zero
height is half the time taken to reach its maximum height (b) The initial and final speeds of the particle are the same
(c) When it rises to half its maximum height, its speed (c) The acceleration of the particle remains constant
becomes v / 2 throughout the motion
(d) When it rises to half its maximum height, its speed (d) The particle changes its direction of motion at same point
becomes v /2
25. A particle is moving with a uniform acceleration Comprehension Based Questions
along a straight line AB. Its speed at A and B are
2 ms–1 and 14 ms–1 respectively. Then
Passage
(a) its speed at mid-point of AB is 10 ms–1 When two bodies A and B are moving with velocity v A
(b) its speed at a point P such that AP : PB = 1:5 is 4 ms–1 and v B , then relative velocity of A w.r.t. B is
(c) the time to go from A to mid-point of AB is double of that v AB = v A - v B
to go from mid-point to B Relative velocity of B w.r.t. A is
(d) None of the above v BA = v B - v A = v B + ( - v A )
When body C is moving with velocity vC on a body A,
26. A ball is bouncing elastically with a speed 1 m/s
which is moving with velocity v A , then velocity of C
between walls of a railway compartment of size 10 m
w.r.t. ground is vC + v A .
in a direction perpendicular to walls. The train is
Suppose two parallel rail tracks run north-south.
moving at a constant velocity of 10 m/s parallel to the
Train A moves north with a speed of 54 kmh–1 and
direction of motion of the ball. As seen from the
train B moves south with a speed of 90 kmh–1.
ground. [NCERT Exemplar]
(a) the direction of motion of the ball changes every 10 s
29. Relative velocity of ground w.r.t. B is
(b) speed of ball changes very 10 s (a) 25 ms–1 due north (b) 25 ms–1 due south
(c) average speed of ball over any 20 s interval is fixed (c) 40 ms–1 due north (d) 40 ms–1 due south
(d) the acceleration of ball is the same as from the train 30. A monkey is moving with a velocity 18 kmh–1 on the
roof of train A against the motion of train A. The
27. The figure shows the velocity (v) of a particle moving
velocity of monkey as observed by a man standing on
on a straight line plotted against time (t).
the ground is
v (ms–1)
(a) 5 ms–1 towards south
10 (b) 10 ms–1 towards north
(c) 10 ms–1 towards south
5
(d) 20 ms–1 towards south
0 t (s)
5 10 15
–5 Assertion and Reason
–10 Directions Question No. 31 to 35 are Assertion-Reason type.
Each of these contains two Statements : Statement I (Assertion),
(a) The particle has zero displacement Statement II (Reason). Each of these questions also has four
(b) The particle has never turned around alternative choice, only one of which is correct. You have to
(c) The particle has constant acceleration select the correct choices from the codes (a), (b), (c) and (d) given
(d) The average speed in the interval 0 to 5 s is the same as below
the average speed in the interval 5 to 10 s (a) If both Assertion and Reason are true and the Reason
is correct explanation of the Assertion
28. The figure shows the velocity (v) of a particle plotted (b) If both Assertion and Reason are true but Reason is
against time (t). not correct explanation of the Assertion
v (ms–1)
(c) If Assertion is true but Reason is false
B
10 (d) If Assertion is false but the Reason is true
31. Assertion A body is dropped from a height of 40.0 m.
0
D
t (s)
After it falls by half the distance, the acceleration due
T 2T to gravity ceases to act. The velocity with which it
hits the ground is 20 ms–1 (Take g = 10 ms–2).
–10 A Reason v2 = u2 + 2 as
Telegram @unacademyplusdiscounts

66 JEE Main Physics

32. Assertion A car moving with a speed of 25 ms–1 takes 34. Assertion The slope of displacement-time graph of a
U turn in 5 s, without changing its speed. The body movng with high velocity is steeper than the
average acceleration during these 5 s is 5 ms–2. slope of displacement-time graph of a body with low
Change in velocity velocity.
Reason Acceleration =
Time taken Reason Slope of displacement-time graph = Velocity
33. Assertion The average velocity of the object over an of the body.
interval of time is either smaller than or equal to the 35. Assertion A body having non-zero acceleration can
average speed of the object over the same interval. have a constant velocity.
Reason Velocity is a vector quantity and speed is a Reason Acceleration is the rate of change of velocity.
scalar quantity.

Previous Years’ Questions


36. A ball is dropped from a bridge at a height of 176.4 m 42. A body of mass m is accelerated uniformly from rest
over a river. After 2s, a second ball is thrown straight to a speed v in a time T. The instantaneous power
downwards. What should be the initial velocity of the delivered to the body as a function of time is given by
second ball so that both hit the water [AIEEE 2008]
simultaneously? [UP SEE 2009] 1 mv 2 2
1 mv 2
(a) t (b) t
(a) 2.45 ms–1 (b) 49 ms–1 2 T2 2 T2
(c) 14.5 ms–1 (d) 24.5 ms–1 mv2 mv2
(c) 2 t2 (d) 2 t
37. A scooterist sees a bus 1 km ahead of him moving T T
with a velocity of 10 ms -1. With what speed the 43. A body moves with initial velocity 10 ms -1. If it covers
scooterist should move so as to overtake the bus in a distance of 20 m in 2 s then acceleration of the
100 s.? [Orissa JEE 2008] body is [Orissa JEE 2011]
(a) 10 ms–1 (b) 20 ms–1 (a) zero (b) 10 ms -2
(c) 50 ms–1 (d) 30 ms–1 (c) 5 ms -2 (d) 2 ms -2
38. A bullet emerges from a barrel of length 1.2 m with a 44. A particle moves in a straight line with retardation
speed of 640 ms–1. Assuming constant acceleration, proportional to its displacement. Its loss of kinetic
the approximate time that it spends in the barrel energy for any displacement x is proportional to
after the gun is fired is [WB JEE 2008] [UP SEE 2007]
(a) 4 ms (b) 40 ms (a) x 2
(b) e x
(c) 400 ms (d) 1 s (c) x (d) log e x
39. A body is fired vertically upwards. At half the
maximum height, the velocity of the body is 10 ms–1. 45. A ball which is at rest, is dropped from a height
The maximum height raised by the body is h metre. As it bounces off the floor its speed is 80% of
(g = 10 ms -2 ). [Orissa JEE 2008]
what it was just before touching the ground? The ball
will then rise to nearly a height [BVP Engg. 2007]
(a) zero (b) 10 m
(a) 0.94 h (b) 0.80 h
(c) 15 m (d) 20m
(c) 0.75 h (d) 0.64 h
40. The velocity of a particle is v = v0 + gt + ft2 . If its
position is x = 0 at t = 0, then its displacement after 46. A particle has an initial velocity of 3 $i + 4 $j and
unit time ( t = 1) is [AIEEE 2007] acceleration of 0.4 $i + 0.3 $j. Its speed after 10 s is
(a) 10 units (b) 7 2 units
(a) v 0 - g / 2 + f (b) v 0 + g / 2 + 3 f
(c) v 0 + g / 2 + f / 3 (d) v 0 + g + f (c) 7 units (d) 8.5 units
41. A proton in a cyclotron changes its velocity from 47. A particle is moving with velocity v = k ( 4 $i + x $j )
30 kms–1 due north to 40 kms–1 due east in 20 s. where k is a constant. The general equation for its
What is the magnitude of average acceleration path is [AIEEE 2010]
during this time? [BVP 2008] (a) y = x2 + constant (b) y2 = x + constant
(a) 2.5 kms–2 (b) 12.5 kms–2 (c) xy = constant (d) y2 = x2 + constant
(c) 22.5 kms–2 (d) 32.5 kms–2
Telegram @unacademyplusdiscounts

Kinematics 67

48. A particle located at x = 0 at time t = 0, starts moving decelerates at 2 ms -2 . He reaches the ground with a
along the positive x-direction with a velocity v that speed of 3 ms -1. At what height, did he fallen out?
varies as v = a x . The displacement of the particle [AIEEE 2005]
varies with time as [AIEEE 2006] (a) 111 m (b) 293 m
(a) t3 (b) t2 (c) 182 m (d) 91 m
(c) t (d) t1/2 52. A car starting from rest, accelerates at the rate f
49. An object, moving with a speed of 6.25 m/s, is through a distance S, then continues at constant
dv f
declerated at a rate given by = - 2.5 v , where v is speed for time t and then decelerates at the rate to
dt 2
the instantaneous speed. The time taken by the come to rest. If the total distance traversed in 15 s,
object, to come to rest would be [AIEEE 2011] then [AIEEE 2005]
(a) 2 s (b) 4 s 1 2 1 2
(a) s = ft (b) s = ft
(c) 8 s (d) 1 s 4 72
1
(c) s = ft2 (d) s = ft
50. A train accelerated uniformly from rest attains a 6
-1
maximum speed of 40 ms in 20 s. It travels at this
53. A body projected vertically upwards crosses a point
speed for 20 s and is brought to rest with uniform
twice in it journey at a height h first after t1 and t2
retardation in 40 s. The average velocity during this
second. Maximum height reached by the body is
period is [BVP Engg. 2006]
[EAMCET 2005]
(a) (80/3) ms–1 (b) 30 ms–1 g
(c) 25 ms–1 (d) 40 ms–1 (a) 2g (t1 + t2 ) (b) (t1 + t2 )2
4
2
51. A parachutist after alling out falls 50 m g æt + t ö
(c) (t1 t2 ) (d) 2 g ç 1 2 ÷
without friction. When parachute opens, it 4 è 4 ø

Answers
Round I
1. (c) 2. (c) 3. (b) 4. (d) 5. (a) 6. (a) 7. (c) 8. (a) 9. (b) 10. (b)
11. (c) 12. (c) 13. (a) 14. (d) 15. (c) 16. (d) 17. (a) 18. (a) 19. (b) 20. (d)
21. (c) 22. (a) 23. (c) 24. (a) 25. (c) 26. (b) 27. (b) 28. (c) 29. (a) 30. (c)
31. (c) 32. (d) 33. (a) 34. (b) 35. (b) 36. (b) 37. (a) 38. (a) 39. (b) 40. (a)
41. (c) 42. (b) 43. (a) 44. (d) 45. (a) 46. (b) 47. (d) 48. (a) 49. (a) 50. (b)
51. (c) 52. (b) 53. (d) 54. (d) 55. (a) 56. (c) 57. (d) 58. (d) 59. (c) 60. (d)
61. (b) 62. (c) 63. (a) 64. (d) 65. (c) 66. (d) 67. (c) 68. (a) 69. (a) 70. (b)
75. (c) 76. (a)
71. (a) 77. (b)
72. (a) 78. (a)
73. 79. (a) 80. (a)
81. (a) 82. (a) 83. (b)

Round II
1. (d) 2. (c) 3. (b) 4. (a) 5. (c) 6. (c) 7. (c) 8. (c) 9. (d) 10. (b)
11. (b) 12. (d) 13. (b) 14. (b) 15. (b) 16. (b,c,d) 17. (b,c) 18. (a,c,d) 19. (a,c,d) 20. (a,b,c)
21. (a,c) 22. (a,b,d) 23. (b,d) 24. (b,c) 25. (a,c) 26. (b,c,d) 27. (c,d) 28. (a,b,c,d) 29. (a) 30. (b)
31. (a) 32. (d) 33. (a) 34. (a) 35. (d) 36. (d) 37. (b) 38. (a) 39. (b) 40. (c)
41. (c) 42. (d) 43. (a) 44. (a) 45. (d) 46. (b) 47. (d) 48. (b) 49. (a) 50. (c)
51. (b) 52. (b) 53. (d)
Telegram @unacademyplusdiscounts

the Guidance
Round I
1. Horizontal distance covered by the wheel in half revolution 6. Initial velocity of the car (u) = 126 km/h
pR. 5 æ 5 ö
A′ Final = 126 ´ m/s çQ 1 km /h = m/s÷
18 è 18 ø
= 35 m/s
2h Final velocity of the car (v) = 0
Distance travelled ( s) = 200 m
A πR From equation of motion, v 2 = u 2 + 2as
So, the displacement of the point which was initially in v 2 - u 2 0 - (35) 2
or a= =
contact with ground 2s 2 ´ 200
= AA¢ = ( pR) 2 + (2 R) 2 -35 ´ 35 49
= =- m/s 2
400 16
= R n2 + 4 = p 2 + 4 [Q R = 1m ]
= - 3.06 m/s 2
2. Given, v = pt
\Retardation of the car = - 3.06 m/s 2
2
x 2 pt 4´4
Þ ò 0 dx = pò0 t dt = 2
=
2
=8m From equation of motion, v = u + at
v - u (0 - 35)
or t= =
3. In graph (b), for one value of displacement, there are two a ( -49 /16)
timings. As a result of it, for one time, the average velocity is 35 ´ 16
positive and for other time is equivalent negative. Due to it, =
49
the average velocity for the two timings (equal to time period)
5 ´ 16 80
can vanish. = = s
7 7
u + v1¢ u + u + at1 1
4. As, v1 = = = u + at1 = 114
. s
2 2 2
v ¢ + v ¢ (u + at1) + u + a (t1 + t 2) 1 \Car will stop after 11.4 s
v2 = 1 2 = = u + at1 + at 2
2 2 2 7. Let the particle touches the sphere at the point A.
v ¢ + v ¢ (u + at1 + at 2) + u + a (t1 + t 2 + t3) Let PA = l
v3 = 2 3 =
2 2 l
\ PB =
1 2
= u + at1 + at 2 + at3
2 PB
In D OPB, cos a =
1 r
Then, v1 - v 2 = - a (t1 + t 2)
2 O
1 α α
v 2 - v3 = - a (t 2 + t3)
2 r r
v1 - v 2 t1 + t 2
\ = α α
v 2 - v3 t 2 + t3 P A
B
5. x = t + 1 \ PB = r cos a
Squaring both sides, we get l
or = r cos a
x = (t + 1) 2 = t 2 + 1 + 2 t 2
Differentiating it w.r.t. t, we get l = 2.4 cos a
dx 1
= 2t + 2 But l = a0t 2
dt 2
dx æ 2l ö æ 2 ´ 2r cos a ö æ 4r cos a ö
Velocity = v = = 2t + 2
dt \ t= ç ÷ = ç ÷ = ç ÷
è a0 ø è a0 ø è a0 ø
so increase with time.
Telegram @unacademyplusdiscounts

Kinematics 69

1 2 dv
8. Here, Sn = an For minimum v, =0
n dq
S nth = distance travelled in n second - 8 (2 cos q - sin q)
or =0
– distance travelled in (n -1) second (2 sin q + cos q) 2
æ 2n - 1ö or 2 cos q - sin q = 0
=ç ÷a
è 2 ø or tan q = 2,
S nth 2n - 1 2 1 2
\ = 2 = - 2 So, sin q =
Sn n n n 5
1
9. According to problem, when cos q =
5
s = a, t = p
8
1 \ v min =
Q s = ut + ft 2 (here, f = acceleration) æ 2 ö 1
2 2ç ÷+
è 5ø 5
fp 2
\ a = up + …(i) 8
2 = = 3.57 ms–1
5
For s = b, t = q
fq 2 12. Let a rel = acceleration of ball with respect to ground-
b = uq + …(ii) acceleration of bus with respect to ground.
2
y
After solving Eqs. (i) and (ii), we get
2 ( aq - bp)
f= a
pq ( p - q)
Motion
10. Here, v =|t - 2| ms–1 of bus
g
v = t - 2, when t > 2 s
v = 2 - t , when t < 2 s
= - g$i - a$j
dv
\ a= = 1ms–2 when t > 2 s
dt | a rel| = g 2 + a2
a = -1 ms–2 when t < 2 s
13. From the figure, the relative displacement is
a = 1 ms–2 a = 1 ms–2
C
A B
t = 2s
7.5 ms–1
In the direction of motion from A to C, bee decelerates but
for C to B, bee accelerates. 20 ms–1
Let AC = s1,BC = s2
100 m
uA = 2 ms–1 ,t = 0
200 100 (m) 300 mrel = + = uC = 0 at s
æ u + uC ö v rel = v1 - v 2 = (20 - 7.5) ms–1
\ s1 = ç A ÷t
è 2 ø 1 = 12.5 ms–1
æ u + uB ö srel
s2 = ç C ÷t \ t=
è 2 ø 2 v rel
æ2 + 0 ö æ0 + 2ö 300
\ s = s1 + s2 = ç ÷2+ ç ÷2 = 4m = = 24 s
è 2 ø è 2 ø 12.5
11. Let the man starts crossing the road at an angle q with the s
14. 40 2 - 30 2 = 2 as, and v 2 - 30 2 = 2a
roadside. For safe crossing, the condition is that the man must 2
cross the road by the time truck describes the distance or 2 (v 2 - 30 2) = 2 as
( 4 + 2 cot q),
Comparing,
4 + 2 cos q 2l sin q
So, = 2 (v 2 - 900) = 1600 - 900 = 700
8 v
8 or v 2 = 900 + 350 = 1250
or v=
2 sin q + cos q or v = 35.35 kmh –1
Telegram @unacademyplusdiscounts

70 JEE Main Physics

dx Dividing Eq. (iii) by Eq. (i), we get


15. = 4 t3 - 2 t
dt 225 x - 15 t 0
=
or dx = 4t 3dt - 2 t dt 100 10 - 10 t 0
1
4t 4 2t 2 x - 15 ´
Integrating, x= - = t4 -t2 9 2
4 2 or =
4 10 - 10 ´ 1
When x = 2, 2
\ t4 -t2 -2 = 0 45 = 4x - 30
- ( -1) ± 1 + 8 or 4x = 75
t2 =
2 75
or x= m = 18.75 m
2 1± 3 4
or t = =2 (Ignoring –ve sign)
2 18. As lift is coming from 8th to 4th floor, the value of x becomes
d 2x less hence negative, i. e. , x < 0. Velocity is downwards
Again, = 12 t 2 - 2
dt 2 (i. e. , negative). So, v < 0. Before reaching 4th floor lift is
When t 2 = 2, acceleration = 12 ´ 2 - 2 = 22 ms–2 retarded, i. e. , acceleration is upwards. Hence, a > 0.

dv 19. Let a1 and a2 be the retardations offered to be bullet by wood


16. As, = - kv3
dt and iron respectively.
v dv t For A ® B ® C,
Þ òv 0 v3
= -kò dt
0 v12 - u 2 = 2 a1( 4)
1 v and 0 2 - v12 = 2a1(1)
Þ - ò v -3dv = t
k v0
Adding, we get - u 2 = 2 ( 4 a1 + a2) …(i)
-3 + 1 v
1 v For A¢ ® B¢ ® C ¢,
or - =t
k -3 + 1v v 22 - u 2 = 2 a2(2)
0

1 é1 1ù and 0 2 - v 22 = 2a1(2)
or ê - ú =t
2k ë v 2 v 02 û Adding, we get
1 1 -u 2 = 2 (2a + 2a2) …(ii)
or - = 2 kt
v 2 v 02 Equating Eqs. (i) and (ii) and solving, we get
1 1 4a1 + a2 = 2a1 + 2a2
or = + 2 kt
v 2 v 02 Þ a2 = 2a1
1 1 + 2v 02kt 2
or = 20. Slope of line = -
v2 v 02 3
v0 2
or v= Equation of line is (v - 20) = - ( s - 0)
2v 02kt + 1 3
2
Þ v = 20 - s …(i)
17. If t 0 is the reaction time, then the distance covered during 3
decelerated motion is10 - 10 t 0 . Velocity at s = 15 m, i. e. ,
Now, in the first case, ds 2
v= = 20 - (15) = 10 ms–1
10 2 = 2a (10 - 10 t 0) …(i) dt s =15 m 3
Similarly, in the second case,
Differentiate Eq. (i) with respect to time, acceleration
20 2 = 2a (30 - 20 t 0) …(ii) dv 2 ds
= =-
Again, in the third case, dt 3 dt
15 2 = 2a ( x - 5 t 0) …(iii) dv 2 ds
\ a= =-
Dividing Eq. (ii) by Eq. (i), dt s =15 m 3 dt s =15 m
20 2 30 - 20 t 0 20
= =- ms–2
10 2 10 - 10 t 0 3
or 40 - 40 t 0 = 30 - 20 t 0
21. The displacement of the particle is determined by the area
or 20 t 0 = 10 bounded by the curve. This area is
1 p
or t0 = s s = vm t 0
2 4
Telegram @unacademyplusdiscounts

Kinematics 71

The average velocity is 1


s p 24. As, b - a = un + An2
< v > = = vm 2
t0 4 \ 2b - 2a = 2 un + An 2 …(i)
Such motion cannot be relized in practical terms since at the t=0 t=n t = 2n
initial and final moments, the acceleration (which is slope of
a
v-t graph) is infinitely large. Hence, both (i) and (ii) are correct. b
22. Speed of police van (vP) = 30 km/h c

5 5 1
= 30 ´ m/ s (Q 1 km/s = m/s) Again, A (2 n) 2
c - a = u (2 n) + …(ii)
18 18 2
25 Subtracting, Eq. (i) in Eq. (ii), we get
= m/s
3 c - a - 2b + 2a = An 2
Speed of thief’s car (vT ) = 192 km/h c -2 b + a
A=
5 n2
= 192 ´ m/s
18 25. A body is moving on straight line with constant velocity.
160 Between A and B the straight line is the shortest distance. This
= m/s
3 is the distance travelled. The particle starts at A and reaches B
Muzzle speed of bullet (vB) = 150 m/s along the straight line. Therefore displacement is also AB . i.e.,
D = s.
The bullet is sharing the speed of the police van, therefore
effective speed of the bullet 26. Let time interval be chosen as 1 s
vB ¢ = vB + vP PA OA v x
= =
25 475 PB OB v y
= 150 + = m/s
3 3 So, P ( x, y) divides AB in the ratio of v x : v y .
Speed of the bullet with which it hits the thief‘s car
= Relative speed of the bullet w.r.t. thief‘s car (vBT ) (0, vy) B

vBT = vB ¢ - vT
æ 475 160 ö vy P (x, y)
=ç - ÷ m/s
è 3 3 ø
315
= = 105 m/s A
3 O (vy, 0)
Therefore, bullet will hit the thief’s car with a speed 105 m/s
Using section formula,
23. When a particle is moving with uniform acceleration, let v be vx ´ 0 + vy ´ vx v xv y
the velocity of particle at a distance s, x= =
vx + vy vx + vy
0 +v v
then average velocity = = v xv y + v y ´ 0 v xv y
2 2 y= =
s 2s vx + vy vx + vy
Time taken, t1 = =
(v / 2) v v xv y
v = x2 + y 2 = 2
When particle moves with uniform velocity, time taken, v xv y
2s Now, replace v x by v1 and v y by v 2.
t2 =
v 2 v1v 2
v=
When particle moves with uniform acceleration, time taken, v1 + v 2
3s 6s
t3 = = 27. The maximum distance covered in time T = v 0 T. Therefore,
(0 + v) / 2 v
for the object having one dimensional motion, the
Total time = t1 + t 2 + t3 displacement x in time T satisfies -v 0 T < x < v 0T.
2 s 2 s 6 s 10 s v v
= + + = 28. As, 1 = and 3 =
v v v v t1 t2
s + 2 s + 3s 6 v
\ v av = = 1
10 s / v 10 \ 1200 = (t1 + t 2) v ,
2
v av 6 3 1æ vö 1 4 v2 2 v2
or = = 1200 = çv + ÷ v = =
v 10 5 2è 3ø 2 3 3
Telegram @unacademyplusdiscounts

72 JEE Main Physics

or v 2 = 1800 37. Given line have positive intercept but negative slope so its
1 equation can be written as
\ 1200 = t ´ 1800
2 V = -mx + v 0 …(i)
2400 é v0 ù
t= s
1800 ê where, m = tan q = ú
ë x0 û
2400
= s = 56.6 s By differentiating w.r.t. time, we get
42.43
dv dx
dv = - m = - mv
29. Given, = bt or, dv = bt dt dt dt
dt
v t
Now substituting the value of v from Eq. (i), we get
Þ òv 0
dv = ò bt dt
0
dv
= -m ( -mx + v 0) = m2x - mv 0
dt
bt 2
or v - v0 = \ a = m2x - mv 0
2
bt 2 The graph between a and x should have positive slope but
or v = v0 + negative intercept on a-axis. So, graph (a) is correct.
2
bt 2 38. At time t B uA = 0
or dx = v 0dt + dt
2 Velocity of A, v A = u - gt upward
x t b t Velocity of B, vB = gt downward
ò 0 dx = ò 0 v 0dt + ò t 2 dt h
2 0 If we assume that height h is smaller than or
1 bt 3 bt 3 equal to maximum height reached by A u =u
or x = v 0t + = v 0t + then at every instant v A and vB are in
A A
2 3 6
opposite direction
30. Slope is negative at the point E. \ v AB = v A + vB
31. Area between v-t graph and time axis gives the distance = u - gt + gt
1 1 (Speeds in opposite directions get added)
\ D= ´ 2 ´ 20 + 15 ´ 3 + 2 ´ ´ 15 ´ 1
2 2 =u
= 80 m . 1 9
39. As, g (3) 2 = (2n - 1)
tan 30° 1 2 2
32. = = 1: 3
tan 45° 3 Þ n =5s
1
33. Height reached = ´132 ´1200 m = 66 ´1200 m 40. Velocity of rain falling vertically downward
2
v r = 30 m/s
34. Distance covered = Area enclosed by v-t graph
Rain
= Area of triangle
1
= ´ 4 ´ 8 = 16 m
2 α
B –vw vw
1 N 1 O S
´ OA ´ AC ´ AB ´ AC
α
35. (i) 2 +2 =1 vr
OA AB
vw
CA CA C A
(ii) tan 60° = and tan 30° =
OA AB
Velocity of woman riding a bicycle
OA tan 60° = AB tan 30°
vw = 10 m/s (north to south)
OA tan 30°
or = To protect herself from rain, the woman should hold her
AB tan 60°
umbrella in the direction of relative velocity of the rain with
1 1 1
= ´ = respect to the woman, i.e., v rw .
3 3 3
The relative velocity of rain with respect to the woman, i.e.,
36. Since acceleration is constant, therefore there is a uniform v rw = v r - vw
increase in velocity. So, the v-t graph is a straight line slopping
upward to the right. When acceleration becomes zero, \ | v rw| = (30) 2 + (10) 2
velocity is constant. So, v-t graph is a straight line parallel to = 900 + 100 = 1000 m/s
the time-axis.
= 10 10 m/s
Telegram @unacademyplusdiscounts

Kinematics 73

If v rw makes an angle a with the vertical, then Relative velocity of plane with respect to the observer
v 10 v j - v 0 = 500 - 0 = 500 km/h …(i)
tan a = w =
v r 30 Relative velocity of products of combustion with respect to
1 the jet plane
= = 0.3333
3 vg - v j = -1500 km/h (given) …(ii)
or a = 18°26¢ (Velocity of ejected gas vg and velocity of v j are in opposite
directions)
Hence, woman should hold her umbrella at an angle of 18°
26¢ with the vertical towards south. Adding Eqs. (i) and (ii), we get
(v j - v 0) + (vg - v j ) = 500 - 1500
41. Vertical component of velocities of both the balls are same
2h vg - v 0 = - 1000 km/h
and equal to zero. So, t = .
g Therefore, relative velocity of the ejected gases with respect
Same for both the balls. to the observer is 1000 km/h, -ve sign shows that this velocity
is in a direction opposite to the motion of the jet airplane.
42. Given, x = 4 (t - 2) + a (t - 2) 2 u2
dx 46. Here, A= - u 2 = - 2 gh1
v= = 4 + 2a (t - 2) 4
dt
u2
At t = 0 , v = 4 (1 - a) B= - u 2 = -2 gh2
9
d 2x
Acceleration a = 2 = 2a u2
dt and C= - u 2 = - 2 gh3
16
43. Let the ball be at height h at time t and (t + D t). Then, u2 ì 8 3 ü 42 5
1 \ AB = í - ý= ×
h = ut - gt 2 …(i) 2g î 9 4þ 2 g 36
2
u 2 ì15 8ü u 2 7
1 BC = í - ý= ×
and h = u (t + D t ) - g (t + D t ) 2 …(ii) 2 g î16 9þ 2 g 144
2
Equating Eqs. (i) and (ii), we get AB 5 144 20
\ = ´ =
2u - g Dt BC 36 7 7
t=
2g 2 ´ 12 2 ´ 10 æ 2H ö
47. Dt = - çQ t = ÷
Substituting Eq. (ii) in Eq. (i), we get, 10 10 è g ø
4 u 2 - g 2 ( Dt ) 2 = 1.549 s –1.414 s = 0.135 s
h=
8g 1 2 1
48. As, x = gt and 100 - x = 25 x - g t 2,
1 2 2
Þ u= 8 gh + g 2 ( D t ) 2
2 Adding 25 t = 100
44. Acceleration of body along AB is g cos q or t = 4s
1 49. Initial velocity of balloon with respect to ground
Distance travelled in time t sec = AB = ( g cos q ) t 2
2 v = 10 + 5 = 15 m/s upward
From D ABC, AB = 2R cos q After 2 s its velocity v = u - gt
1 = 15 - 10 ´ 2 = -5 m/s
2 R cos q = g cos q t 2
2 = 5 m/s (downward)
4R
t2 = u
g 50. Time of ascent = = 6 s
g
R
t =2 Þ u = 60 m/s
g
Distance in first second,
45. Let jet airplane be moving upwards right (+ve direction) with g
hfirst = 60 - (2 ´ 1 - 1) = 55 m
velocity v j and ejected gases be moving downwards (-ve 2
direction) with velocity vg while observer be at rest on the
Distance in seventh second will be equal to the distance in
ground i.e., v 0 = 0
first second of vertical downward motion.
\ v j = 500 km/h g
vg = -1500 km/h hseventh = (2 ´ 1 - 1) = 5 m
2
v0 = 0 Þ hfirst / hseventh = 11 : 1
Telegram @unacademyplusdiscounts

74 JEE Main Physics

51. v=0 1 2
55. As, h = - vt + gt
2
or f t 2 - 2 vt - h = 0
u
t=g
20 m - ( - 2v) ± 4 v 2 + 4 gh 2 v ± v 2 + gh
=2 s Þ t= =
15 m 2g 2g
Ground v [v 2 + 2 gh]1/ 2 v é 2 gh ù
= ± = ê1 ± 1 + 2 ú
g g g ë v û
Position of balls
1 2 1
56. Now, retain only the positive sign.
h1 =gt = ´ 10 ´ 12 = 5 m (2 v) 2 - v 2 = 2 gh ¢
2 2
1 2 1 or 4 v 2 - v 2 = 2 gh ¢
h2 = gt = ´ 10 ´ 2 2 = 20 m
2 2 or 3 v 2 = 2 gh ¢
From ground, 5 m, 20 m, 15 m (shown in figure)
or 3 ´ 2 gh = 2 gh ¢
52. The slope of the given graph over the time interval t1 to t 2 is or h¢ = 3 h
not constant and is not uniform. It means acceleration is not
constant and is not uniform, therefore relations (a), (b) and (e)
57. When A returns to the level of top of tower, its downward
velocity is 4 ms -1. This velocity is the same as that of B. So,
are not correct which is for uniform accelerated motion, but
both A and B hit the ground with the same velocity.
relations (c), (d) and (f) are correct, because these relations are
true for both uniform or non-uniform accelerated motion. 58. As, s¢ µ t 2
53. By the time 5th water drop starts falling, the first water drop 1 9
Now, s1¢ : s¢2 : s3¢ = : 1 : or 1 : 4 : 9
reaches the ground. 4 4
1 2 For successive intervals,
As u = 0 , h= gt
2 s1 : s2 : s3 = 1 : ( 4 - 1) : (9 - 4)
1 or s1 : s2 : s3 = 1 : 3 : 5
= ´ 10 ´ t 2
2 1 2
1 59. As, h = - vt1 + gt1
or 5 = ´ 10 ´ t 2 2
2 h 1
or = -v + gt1 …(i)
or t = 1s t1 2
1s 1 2
Hence, the interval of each water drop = = 0.25 s and h = vt 2 +
gt 2
4 2
When the 5th drop starts its journey towards ground, the third h 1
or - = -v + gt 2 …(ii)
drop travels in air for t2 2
t1 = 0.25+0.25 = 0.5 s h h 1
\ + = g (t1 + t 2)
\ Height (distance) covered by 3rd drop in air is t1 t 2 2
1 1 2 1 2
= =h´1gt´ 1 or h= gt1t 2
2 2 2
= 5 ´ 0.25 = 1.25 m For fall under gravity from the top of the tower
So, third water drop will be at a height of 1
h = gt 2
= 5 - 1.25 = 3.75 m 2
1 1
54. Let u be the velocity with which the stone is projected \ gt1t 2 = gt 2
vertically upwards. 2 g
Given that, v -h = 2 v h Þ t = t1 t 2
(v - h) 2 = 4 v h2 Dv
60. Average acceleration =
\ 2 2
u - 2 g ( -h) = 4 (u - 2 gh) Dt
10 gh 2 gh ¢ - ( - 2 gh) 2 gh ¢ + 2 gh
\ u2 = = =
3 Dt Dt
u2 5 h 2 ´ 10 ´ 2.5 + 2 ´ 10 ´ 10
Now, hmax = = = ms–2
2g 3 0.01
Telegram @unacademyplusdiscounts

Kinematics 75

50 + 200 5 2 + 10 2 L
= ms–2 = ms–2 65. Velocity of girl, vg =
0.01 0.01 t1
15 2 L
= ms–2 = 1500 2 ms–2 Velocity of escalator, v e =
0.01 t1
The upward velocity has been taken as positive. Since
Effective velocity of girl on escalator = vg + v e
average acceleration is positive therefore its direction is
vertically upward. L L
= +
t1 t2
61. Let us solve the problem in terms of relative initial velocity,
relative acceleration and relative displacement of the coin L L L
If t is the time taken, then = +
with respect to floor of the lift. t t1 t2
Given, u = 0 ms–1, a = 9.8 ms–2, s = 4.9 m,t = ? t1t 2
or t=
1 t1 + t 2
As, 4.9 = 0 ´ t + ´ 9.8 ´ t 2
2
or 4.9 t 2 = 4.9 66. Total distance = 130 + 120 = 250 m
or t = 1s Relative velocity = 30 - ( -20) = 50 m/s
15 = 30 - 10 t 250
Hence, t= =5s
or 10 t = 15 50
or t = 1.5 s m0v
67. Relativistic momentum =
1 2 1 v2
62. As, D x = gt - g (t - 1) 2 1- 2
2 2 c
1 1 If velocity is doubled, then the relativistics mass also increases.
= g [ -(t - 1) 2] = g (2 t - 1)
2 2 Thus, value of linear momentum will be more than doubled.
1
= ´ 9.8 ´ 5 m = 24.5 m 68. As the train are moving in the same direction so, the initial
2
relative speed (v1 - v 2) and by applying retardation final
a
63. 4 = u + (2 ´ 3 -1) relative speed becomes zero.
2
From v = u - at
5a
or 4 =u+ Þ 0 = (v1 - v 2) - at
2
a æv -v ö
5 = u + (2 ´ 4 - 1) Þ t = ç 1 2÷
2 è a ø
7a
or 5 =u+ 69. Let the required angle is q.
2
7 a 5 a 2a 20
Subtracting, 1= - = =a tan (90° - q) =
2 2 2 15
5 20 4
Again, 4 =u+ \ cos q = =
2 15 3
5 Þ q = 37°
or u = 4 - = 1.5 ms–1 \ q = 37° + 23° = 60°
2
So, the initial velocity is non-zero and acceleration is uniform. 70. Let v be the speed of boatman in still water.
11h 9.8 B
64. Clearly, = (2n - 1) y
36 2
11 1 9.8
or ´ ´ 9.8 n 2 = (2 n - 1) v vb
36 2 2 45°
11 2 x
or 2n - 1 = n A u
36
or 11n 2 = 72 n - 36 Resultant of v and u should be along AB. Components of v b
or 11n 2 - 72 n + 36 = 0 (absolute velocity of boatman) along x and y directions are,
or 11n 2 - 66 n - 6 n + 36 = 0 v x = u - v sin q
or 11n (n - 6) - 6 (n - 6) = 0 and v y = v cos q
vy
Þ n = 6 (Rejecting fractional value) Further, tan 45° =
1 vx
h = ´ 10 ´ 6 ´ 6 m = 180 m
2
Telegram @unacademyplusdiscounts

76 JEE Main Physics

v cos q 77. v ct = v c - v t
or 1=
u - v sin q vct vc
u u
v= =
sin q + cos q 2 sin ( q + 45° )
v is minimum at, 45°

q + 45° = 90° or q = 45° –vt vt


u
and v min = v ct = v c + ( - v t )
2
Velocity of car w.r.t. train (v ct ) is towards west-north.
x x 2x x x 2 xv
71. Here, t1 = + = ,t 2 = = + 78. From figure,
v v v v + w v - w v 2 - w2 B
2 xv 2x
or t2 = 2ö
=
æ w æ w2 ö
v 2 ç1 - 2 ÷ v ç1 - 2 ÷ vB cos θ
è v ø è v ø vB
t1 θ
or t2 = or t 2 > t1
w2
1- 2
v vB sin θ vw
72. Let us calculate relative deceleration by considering relative vB sin q = v w
velocity. v 1
Using, v 2 - u 2 = 2 as, sin q = w =
vb 2
0 2 - 80 2 = 2 ´ a ´ 2000
80 ´ 80 64 Þ q = 30°
or a=- =- ms–2 = -1.6 ms–2
4000 40 Time taken to cross the river
1.6 –2 D D 2D
Deceleration of each train is ms , i. e. ,0.8 ms–2 t= = =
2 v b cos q v b cos 30° v b 3
73. Relative velocity of bird w.r.t. train = 25 + 5 = 30 m/s 79. Distance between the cars A and B remains constant. Let the
Time taken by the bird to cross the train distance be x.
210 Velocity of C w.r.t. A and B,
t= =7s
30 v = 45 + 36 = 81 km/h
74. Effective speed of the bullet 5
\ Distance = 81 ´ = 6.75 km
= Speed of bullet + speed of police jeep 60
= 180 m/s + 45 km/h 80. Length of each train,
= (180 + 12.5) m/ s = 192.5 m/ s l A = lB = 400 m
Speed of thief’s jeep = 153 km/h = 42.5 m/s Initial velocities of both trains,
Velocity of bullet w.r.t. thief’s car uA = uB = 72 km/h
= 192.5 – 42.5 = 150 m/s 5 æ 5 ö
= 72 ´ m/s çQ1 km/h = m/s÷
75. Given AB = velocity of boat = 8 km/h 18 è 18 ø
AC = Resultant velocity of boat = 10 km/h = 20 m/s
B
C Distance travelled by train A in 50 s
sA = uA ´ t
(As for unaccelerated motion distance = Speed ´ Time)
θ sA = 20 ´ 50 = 1000 m
Distance travelled by train B in 50 s,
A 1
sB = uBt + aBt 2
\ BC = Velocity of river = AC 2 - AB2 2
= (10) 2 - (8) 2 = 6 km/h (As motion of train B is an accelerated motion)
1
76. When two particles moves towards each other, then sB = 20 ´ 50 + ´ 1 ´ (50) 2
2
v1 + v 2 = 6 …(i) = 1000 + 1250 = 2250 m
When these particles moves in the same direction, then Original distance between the two trains = sB - sA
v1 - v 2 = 4 …(ii)
= 2250 - 1000 = 1250 m
By solving Eqs. (i) and (ii), we get, v1 = 5 and v 2 = 1 m/s
Telegram @unacademyplusdiscounts

Kinematics 77

81. Speed of car A(uA) = 36 km/h Total distance travelled


The average speed of the taxi =
Total time taken
5 æ 5 ö
= 36 ´ m/s çQ1 km / h = m/s÷ 23
18 è 18 ø =
(7 / 15)
= 10 m/s
345
= km/h
1 km 1 km 7
A = 49.3 km/h
uA
The magnitude of average velocity
Magnitude of the total displacement
B
uB uC
C =
Total time taken
Speed of car B and car C 10
=
uB = uC = 54 km/h ( 7 / 15)
5 150
= 54 ´ m/s = 15 m/s = km/h
18 7
Relative velocity of car B w.r.t. car A = 21.43 km/h
uBA = uB - uA = 15 - 10 = 5 m/s No, the average speed of the taxi is not equal to the magnitude
Relative velocity of car C w.r.t. car A of the average velocity of the taxi.
uCA = uC - uA = 15 - ( -10) = 25 m/s 83. Given, speed of man (v m) = 4 km/h
Distance between car A and car B = 1km = 1000 m A D
Time taken by car C to travel distance AC = 1000 m
β
Distance
Time (t ) = β
Relative velocity of car C w.r. t. car A
1000 C O B
= s = 40 s
25
Speed of river (v r ) = 3 km/h
Let car B start to accelerate with an acceleration a.
Width of the river (d) = 1km
Using equation of motion,
1 B vr C
s = ut + at 2
2
1 1 km v
or s = uBAt + at 2
2 vm β
1
1000 = 5 ´ 40 + a ´ ( 40) 2 A
2
= 200 + 800 a Time taken by the man to cross the river
or 800 a = 800 Width of the river
t=
or a = 1m/s 2 Speed of the man
Therefore, car B should accelerate with an acceleration 1km 1
= = h
1 m/s 2. 4 km/ h 4

82. Given, shortest distance between the station and the hotel 1
= ´ 60 = 15 min
= 10 km 4
\ Displacement of the taxi = 10 km Distance travelled along the river = v r ´ t
Distance travelled by the taxi = 23 km 1 3
= 3 ´ = km
4 4
Time taken by the taxi = 28 min
28 7 3000
= = h = = 750 m
60 15 4
Telegram @unacademyplusdiscounts

78 JEE Main Physics

Round II
2
12 ö
1. x¢n2x = æç ÷ ´ 20 = 80 m
5. Let v1 and v 2 be the initial speeds of first and second runners
è 60 ø respectively. Let t be time by them when the first runner has
Let a be the retardation in both the cases. Using the relation, completed 50 m. During this time, the second runner has
v 2 = u 2 + 2 as, when automobile is stopped, v = 0. covered a distance = 50 - 1 = 49 m.
50 49
So, 0 = u 2 + 2 as So, t= = …(i)
v1 v 2
or s µ u2
Suppose, the second runner increases his speed to v3 so that
\ s2 = 4 s1 = 4 ´ 20 = 80 m
he covers the remaining distance ( = 51 m) in time t.
2. We know that gravity is a universal force with which all 51 49
So t= =
bodies are attracted towards the earth. Hence, g is same for v3 v 2
both the balls. Also, if t is the time taken by the balls to reach 51
the ground, then from equation of motion. or v3 = v2
49
1
s = ut + gt 2 æ 2 ö
2 or v3 = ç1 + ÷v
è 49 ø 2
2 ( s - ut ) v3 2
Þ t= or -1 =
g v2 49
Since s, u and g are same for both, hence time taken by both v3 - v 22
the balls is same.
or =
v2 49
3. Boat covers distance of 16 km in a still water in 2 hours 2
or % increase = ´ 100 = 4.1%
16 49
i. e. , vB = = 8 kmh –1
2 6. For no collision, the speed of car A should be reduced to vB
Now, velocity of water before the cars meet, i. e. , final relative velocity of car A with
vw = 4 kmh –1 respect to car B is zero, i. e. , vr = 0
Time taken for going upstream Here, initial relative velocity, ur = v A - vB
8 8 Relative acceleration, ar = - a - 0 = - a
t1 = = =2h Let relative displacement = sr
vB - vw 8 - 4
The equation,
As water current oppose the motion of boat, therefore time v r2 = ur2 + 2 ar sr
taken for going downstream
8 8 8 (0) 2 = (v A - vB) 2 - 2 asr
t2 = = = h
vB + vw 8 + 4 12 (v A - vB) 2
sr =
(water current helps the motion of boat) 2a
\ Total time = t1 + t 2 For no collision, sr £ s
8ö (v A - vB) 2
.i .e, £ s 2= +æç ÷
è 12 ø 2a

4. If t1 and 2 t 2 are the time taken by particle to cover first and 7. Since, v = (t - 2), so v µ t . On plotting a graph between v and t ,
second half distance respectively we get a straight line ABand BC as shown in figure (below).
x/2 x v (ms–1)
t1 = =
3 6
A C
Clearly, x1 = 4.5 t 2 2
and x2 = 7.5 t 2
x x 1
So, x1 + x2 = = 4.5 t 2 + 7.5 t 2 =
2 2
x D
t2 = t
24 0 1 2 3 4
x x x
Total time t = t1 + 2 t 2 = + = The distance covered in 4 s is equal to the area under the
6 12 4 velocity- time graph = Area of D AOB + Area of D BCD
So, average speed = 4 m/s 2 ´2 2 ´2
= + =2+2 = 4m
2 2
Telegram @unacademyplusdiscounts

Kinematics 79

1
8. As, h = ut + gt 2 v = 3t2 + 5t + c
2 where c is constant of integration
1 2
Þ 1 = 0 ´ t1 + gt1
2 When t = 0 ,v = 0 so c = 0
Þ t1 = 2 / g \ v = 3t2 + 5t

Velocity after travelling 1m distance æ ds ö


Þ ds = (3 t 2 + 5 t ) dt ç as v = ÷
è dt ø
v 2 = u 2 + 2 gh
Þ v 2 = (0) 2 + 2 g ´ 1 Integrating it within the condition of motion is as t changes
from 0 to 2 s, S changes from 0 to S, we have
Þ v= 2g S 2 2
For second 1 m distance
ò 0 ds = ò 0 (3 t + 5 t ) dt
2
1 2 é 5 ù
1 = 2g ´ t 2 + gt 2 \ s = êt 2 + t 2ú = 8 + 10 = 18 m
2 ë 2 û0
Þ gt 22 + 2 2 gt 2 - 2 = 0
12. Let the body be projected upwards with velocity u from top of
-2 2 g ± 8 g + 8 g tower. Taking vertical downward motion of boy from top of
t2 =
2g tower to ground, we have
- 2 ±2 u = - u, a = g = 10 ms–2, s = 50 m,t = 10 s
= 1
g As s = ut + at 2
2
Taking +ve sign, t 2 = (2 - 2 t ) / g
1
So, 50 = -u ´ 10 + ´ 10 ´ 10 2
t1 2/g 1 2
\ = = and so on.
t 2 (2 - 2) / g 2 -1 On solving, u = 45 ms–1

9. Interval of all ball throw = 2 s If t1 and t 2 are the time taken by the ball to reach points A and B
respectively, then
If we want that minimum three (more than two) ball remain in
1
air then time of flight of first ball must be greater than 4 s 20 = 45 t1 + ´ 10 ´ t12
2
t >4s
1
2u and 40 = - 45 t 2 + ´ 10 ´ t 22
>4s 2
g
On solving, we get, t1 = 9.4 s and t 2 = 9.8 s
Þ u > 19.6 m/s
Time taken to cover the distance AB
For u = 19.6 first ball will just about to strike the ground (in air) = (t 2 - t1) = 9.8 – 9.4 = 0.4 s
Second ball will be highest point (in air). dv
13. As, = bt
Third ball will beat point of projection or at ground (not in air) dt
10. Let vw be velocity of water and v b be the velocity of motor boat Þ dv = bt dt
in still water. If xis the distance covered, then as per question bt 2
1 Þ v= +K
x = (v 2+ v ) ´ 6 = (v - v ) ´ 10
b w b w
On solving, vw = v b / 4 At t = 0 ,v = v 0
Þ K1 = v 0
\ x = (v b + v b / 4) ´ 6 = 7.5 v b
1
Time taken by motor boat to cross the same distance in still we get, v = bt 2 + v 0
water is 2
x 7.5 dx 1 2
t= = = 7.5 h Again = bt + v 0
xb v b dt 2
1 bt 3
11. Given, acceleration a = (6 t + 5) m/s 2 Þ x= + v 0t + K2
2 3
dv At t = 0 , x=0
Þ a= = (6 t + 5),
dt Þ K2 = 0
dv = (6 t + 5) dt 1
v 4 \ x = bt 3 + v 0t
Integrating it, we have ò dv = ò (6 t + 5) dt 6
0 0
Telegram @unacademyplusdiscounts

80 JEE Main Physics

14. The horizontal acceleration a of the wedge should be such or v = 2 (1 - e-3t )


that in time the wedge moves the horizontal distance BC. The when t = 0 , v = 2 (1 - e-3t )
body must fall through a vertical distance AB under gravity.
Initially, v = 0, From Eq. (i) acceleration,
Hence, dv
1 1 a0 = = 6 - 3 ´ 0 = 6 ms–2
BC = at 2 and AB = gt 2 dt
2 2
AB g a 6
tan q = = When a = 0 = = 3 then from Eq. (i);
BC a 2 2
g 3 = 6 -3v
or a= = g cot q
tan q or 3v = 6 -3 = 3
or v = 1ms–1
15. Let u be the initial upward velocity of the ball from A and h be
the height of the tower. 17. Velocity of bolt relative to elevator = 2.5 – 2.5 = 0
Taking the downward motion of the first stone from A to the Acceleration of bolt relative to elevator,
ground, we have a = 10 - ( - 2) = 12 ms–2 (Q g = 10 m/ s2)
1
h = - ut1 + gt12 …(i) 1 2
2 Using the relation, s = ut + at
2
Taking the downward motion of the second stone from A to
1
the ground, we have we have, 3.0 = 0 ´ t + ´ 12 ´ t 2
1 2
h = ut 2 + gt12 …(ii)
2 1
or t= s = 0.707 s = 0.7 s
Multiplying Eq. (i) t 2 and Eq. (ii) by t1 and adding, we get 2
1 1
h (t1 + t 2) = gt1 t 2 (t1 + t 2) Displacement = - 2.5 ´ 0.71+ ´ 10 ´ (0.71) 2
2 2
1 = - 1.775+2.521 = 0.746 = 0.75 m
So, h = gt1 t 2 …(iii)
2 u2
Distance covered = 2 ´ + displacement
For falls under gravity from the top of the tower 2g
1
h = gt32 …(iv) (2.5) 2
2 =2´ + 0.75
2 ´ 10
From Eqs. (iii) and (iv),
= 0.63+0.75 = 1.38 m
t32 = t1t 2
or t3 = t1 t 2 = 9 ´ 4 = 6 s
18. The body is at rest initially and again comes to rest at t = 1s at
position x = 1.
dv Thus, firstly acceleration will be positive then negative. Thus
16. Given, = 6 -3v …(i)
dt a have to change the direction so that body may finally come
dv to rest in the interval 0 £ t £ 1. If we plot v-t graph.
or = dt
6 -3v The total displacement = 1m = area under v-t graph
Integrating it, we have v
é 1 ù B C D
êë - 3 log (6 - 3 v) úû = t + K …(ii)
vmax
At t = 0,v = 0
1
\ K = - log 6
3 A t (s)
1/2 1
Putting this value in Eq. (ii), we have
1 1 1
- log (6 - 3 v) = t - log 6 Now v max × t = s
3 3 2
æ6 -3v ö 2´s
or log ç ÷ = -3t Þ v max =
è 6 ø t
6 -3v 2 ´1
or = e- 3 t v max = = 2 m/s
6 1
v The maximum velocity = 2 m/s
or 1 - = e- 3 t
2
Telegram @unacademyplusdiscounts

Kinematics 81

Now just see the v-t graph At equilibrium position, the whole PE of the stretched spring
During AB a >4 m/s2 is converted into whole KE of the mass. Hence, speed of mass
For ABC, is maximum at equilibrium position.
During BE a <– 4 m/s2
In damped SHM, the speed of mass is minimum at the
During AC a = 4 m/s2
For ACE extreme position, where displacement is maximum.
During CE a = – 4 m/s2
But the magnitude of displacement is not always maximum
During AD a <4 m/s2 whenever speed is minimum.
For ADF
During DE a > – 4 m/s2 22. Given, x = at 2 - bt 3
Thus, a ³ 4 at some point or points in its path. dx
v= = 2 at - 3 bt 2 …(i)
dt
19. From graph, when t = 0 , the particle is released from rest at A, dv
hence, v = 0. \Acceleration, a= = 2 a - 6 bt …(ii)
dt
At B, the graph is parallel to time axis, hence velocity is The particle will come to rest, if v = 0. From Eq. (i),
constant there. Thus acceleration a is zero.
0 = 2 at - 3 bt 2
At C, the graph changes slope, where velocity and
2a
acceleration vanish. or t=
3b
Average velocity for motion between A and D is negative
because the value of x is decreasing with time t. The slope of The particle when returns to its starting point, then x = 0
graph (which represents speed) is more at D than at E. Now, 0 = at 2 - bt 3
Therefore, speed at D is more than that at E. a
or t=
d b
20. Given acceleration, = A - Bv
v Force on particle is zero when a = 0. From Eq. (ii)
(a) When t = 0 , v = 0, therefore initial acceleration, 0 = 2a t 2 - 6 b t 3
æ dv ö a
ç ÷ =A
è dt ø t =0 or t=
b
dv
(b) When acceleration is zero, then = 0. When t = 0, from Eq. (i) v = 0 and from Eq. (ii) a ¹ 0, has
dt
a = 2 a.
Hence, A - Bv = 0
or v = A/B 23. If the initial velocity is negative (i. e. , v < 0) and a is a positive,
dv i. e. ,( a > 0) then speed will decrease. The speed will increase
(c) = dt
A - Bv when v and d are both positive.
Integrating it within the limits of motion, i. e. , as time changes v2
24. Maximum height reached, sm =
from 0 to t, velocity changes 0 to v, we have 2g
v v
é log e ( A - Bv) ù t Time taken to reach the maximum height, Tm =
-ê úû = [t ]0
ë B 0
g
1 2
Þ log e ( A - Bv) - log e A = - Bt Height s reached in time t is s = ut - gt
A - Bv 2
or = e-Bt T v
A If t= =
A 2 2g
or v = (1 - e-Bt ) 2
B v ´v 1 æ v ö
then s= - g ç ÷
21. Maximum restoring force set up in the spring when stretched 2 g 2 è2 g ø
by distance r is F = - kr and potential energy of stretched v2 v2 3 v2 3
1 = - = = sm
spring = kx2. As F µ r and this force is directed towards 2g 8g 8g 4
2
equilibrium position, hence if mass is left free, it will execute Speed at height s is v 2 = u 2 - 2 gs
damped SHM due to gravity pull. sm v 2
When s= =
Magnitude of acceleration in the mass attached to one end 2 4g
of spring when just released is v2 v2
F k then v ¢2 = v 2 - 2 g ´ =
a = = r = (maximum) 4g 2
m m
v
k or v¢ =
At equilibrium position, r = 0 \a = ´ 0 = 0 2
m
Telegram @unacademyplusdiscounts

82 JEE Main Physics

25. Here, u = 2 ms–1, v = 14 ms–1 Since OA = BC, so initial and final speeds are the same.
A C B
Distance between A and B = s The slope of velocity-time graph represents acceleration.
Here, the velocity-time graphs AB is a straight line inclined to
v 2 - u 2 14 2 - 2 2 194 97
Then acceleration, a = = = = time axis hence has equal acceleration throughout. The
2s 2s 2s s particle changes its direction of motion after time T.
The speed at mid- point C, 29. vGB = vG + ( -vB) = 0 + ( -25 ms-1) = 25 ms-1 due north
s 97 s
v 2 = u2 + 2 a = 22 + 2 ´ ´ = 101
2 s 2 30. v mG = v m + vG = 5 + ( -15 ms-1)
v = 101~ - 10 ms–1 = -10 ms–1 towards south
1 1 = 10 ms–1 towards north
As per question, AP = [ AB] = s
6 6 40
s 32. Here, u = 0 , a = 10 ms–2, s = = 20 m
When, s= , 2
6 Using the relation v 2 = u 2 + 2 as = 0 + 2 ´ 10 ´ 20 = 400
97 s
then, v 2 = 22 + 2 ´ ´ or v = 20 ms–1
s 6
97 change in velocity 25 - ( -25)
or v =3+2´ = 36.3 ms–1 33. Acceleration = = = 10 ms–2
3 time taken 5
\ v = 36.3 » 6 ms–1 Hence, Assertion is wrong but Reason is correct.
Since velocity at mid- point C is 10 ms–1. 34. As displacement is either smaller or equal to distance but
\Taking motion from A to C, we have never be greater than distance.
10 = 2 + a ´ t1 35. Since slope of displacement-time graph measures velocity of
10 - 2 8 a moving object.
or t1 = =
a a 36. As per definition, acceleration is the rate of chagne of velocity
Taking motion from C to B, we have dv
is a =
14 = 10 + 1 ´ t 2 dt
14 - 10 4 dv
or t2 = = If velocity is constant =0 \ a =0
a a dt
t1 t Therefore, if a body has constant velocity it cannot have non
\ = 2 or t 2 = 1 zero acceleration but uniform.
t2 2
1 2
26. Since the ball is moving with a very small speed in the moving 37. For first ball, gt = 176.4
2
train, the direction of motion of the ball is the same as that of
176.4 ´ 2
the train. The direction of motion of ball does not change with Þ t=
respect to observer on ground. 10
Þ t = 5.9 s
The speed of the ball as observed by observer on ground
before collision with side of train is 10 + 1 = 11 m/s and after For second ball, t = 3.9s
elastic collision is10 - 1 = 9 m/s. 1
u (3.9)+ g (3.9) 2 = 176.4
2
Since the collision of the ball with side of train is perfectly
10
elastic; the total momentu and kinetic energy are conserved, Þ 3.9 u + (3.9) 2 = 176.4
so average speed of the ball over any 20 seconds interval is 2
constant as observed by observer on ground. Þ u = 25.7 ms–1
Since train is moving with constant velocity, it is an inertial This value is approximated to 24.5 ms–1.
frame,so acceleration of ball is same as from the train.
37. Let u be the velocity of scooterist in order to catch the but in
27. The displacement is the area which the velocity-time graphs 100 s. Then
encloses with time axis for a given interval of time. Since the 100 u = 2000
area of velocity-time graph for time 0 to 5 s is the same as area 2000
of the velocity-time graph for time 5 s to 10 s, hence average or u= = 20 ms–1
100
speed in these intervals is the same.
38. Here, u = 0 ,v = 640 ms–1, s = 1.2 m, a = ? and t = ?
28. Displacement = velocity ´ time.
As, v 2 = u 2 + 2 as
In time 0 to 2 the displacement
v 2 - u 2 (640) 2 (640) 2 –1
= - Area of D OAB + Area of D OAD + Area of D DBC or a= = = ms
2s 2 ´ 1.2 2.4
= 0.
Telegram @unacademyplusdiscounts

Kinematics 83

v - u 640 - 0 45. Clearly, v = 2 gh …(i)


Also, t= =
a (640) 2
After rebounce, v 2 = u 2 - 2gh
2.4
Þ u 2 = v 2 + 2gh
2.4
= = 3.75 ´ 10 –3 s~
- 4 ms and u 2 = 2gh ¢ …(ii)
640
v 2 2gh
39. Taking motion of the body from half the maximum height \ =
u 2 2gh ¢
upto the highest point, we have 2
u2 æ 80 ö
u = 10 ms–1, a = -10 ms–2, v = 0 , s = h / 2 Þ h' = h ´ =h´ç ÷ = 0.64 h
v2 è100 ø
As v 2 = u 2 + 2 as
h 46. Given, u = 3 $i + 4 $j;
\ 0 = 10 2 + 2 ( -10) ´
2 and a = 0.4 $i + 0.3 $j
or h = 10 m Q v = u + at = 3 $i + 4 $j + (0.4 $i + 0.3$j) 10
40. Given, velocity v = v 0 + gt + ft 2 = 3$i + 4$j + 4$i + 3$j
dx v = 7 $i + 7$j
As, v=
dt Thus, speed is 7 2 + 7 2 = 7 2 unit
x t t 2
So, ò 0 dx = ò 0 v dt = ò 0 (v 0 + gt + ft ) dt 47. Given, v = ky$i + kx$j
gt f 2 dx
or + ft 2 +
v x = v 0t + = ky
2 3 dt
g f dy
When t = 1, then x = v 0 + + and = kx
2 3 dt
dy dy dt kx
41. Magnitude of change in velocity = ( 40) 2 + (30) 2 = 50 km/s \ = ´ =
dx dt dx ky
50
\Average acceleration = = 2.5 kms–2 Þ y dy = xdy
20
Þ y 2 = x2 + C (after integrating)
v mv
42. As, acceleration, a = and F = ma = dx dx
T T 48. Velocity, = a x or = a dt
v dt x
\Velocity acquired, v = at = t
T Integrating it, we have
mv v mv 2 òx
-1/ 2
dx = ò a dt
Instantaneous power, P = F × v = ´ t= 2 t
T T T
or 2x1/ 2 = at or x µ t 2
–1
43. Here u = 10 ms , t = 2 s, s = 20 m dv
49. Given, = 2.5 v
1 dt
Using s = ut + at 2
2 dv
Þ = - 2.5 dt
1 v
\ 20 = 10 ´ 2 + a ´ i 2
2 0 t
Þ -1/ 2 = -2.5 v dv dt ò 6.26 ò0
0 =2a Þ a=0
0
44. From given information a = - kx, where a is acceleration, x is Þ - 2.5 [t ] t0 = [2 v1/ 2]6.25
displacement and k is a proportionality constant. t =2s
v dv
i. e. , = - kx 50. The acceleration of train in 20 s is given by
dx 40 - 0
Þ v dx = - kxdx a= = 2 ms–2
20
Let for any displacement from 0 to x, the velocity changes
[from the formula v = u + at (here, u = 0)].
from v 0 to v.
v x Now the distance travelled is given by equation of motion,
Þ òv 0
v dv = - ò kxdx
0 v 2 = u 2 + 2 as
v 2 - v 02 kx2 æ v 2 - u2 ö 40 ´ 40 - 0
Þ =- So, s1 = ç ÷ a= = 400 m
2 2 è 2 ø 2 ´2
æ v 2 - v 02 ö mkx2
Þ mç ÷=- Now distance travelled with constant speed of 40 ms -1 in
è 2 ø 2
t = 20 s is
Þ D K µ x2 [D K is loss in KE] s2 = 40 ´ 20 = 800 m
Telegram @unacademyplusdiscounts

84 JEE Main Physics

Again the distance covered in 3rd case is given by The velocity of car at A = velocity of car at B = (2 fS)1/ 2
40 ´ 40 - 0
s3 = = 800 m As magnitude of retardation of the car from Bto C is half of that
2 ´1 of acceleration from O to A when velocity changes by v, so
Therefore, average speed of the train is given by distance BC = 2 S
400 + 800 + 800 200 Distance, AB = 15 S - ( S + 2 S) = 12 S
v av = = = 25 ms–1
20 + 20 + 40 80 As distance AB is covered with constant velocity in time t
51. First 50 m fall is under the effect of gravity only. The velocity So, 12 S = vt = (2 fS)1/ 2 ´ t
–1
acquired, u = 2 gh = 2 ´ 9.8 ´ 50 ms . Taking onward or 144 S 2 = 2 fSt 2
–2
motion of parachutist with retardation 2 ms , we have, 1 2
or S= ft
u = 10 9.8 ms–1. 72
d = - 2 ms–2,v = 3 ms–1 53. Time taken by the body to reach the point A is t1 (During
2
v -u 2 2
(3) - (2 ´ 9.8 ´ 50) upward journey).
s= = = 243 m The body crosses this point again (during downward journey)
2a 2 ´ ( -2)
after t 2, i. e. , the body takes the time (t 2 - t1) to come again at
\Total height = 50 + 243 = 293 m point A.
52. Constant Retardation = f /2 So, the time taken by the body to reach at point B(at maximum
Acceleration = f
velocity height)
æt -t ö
t = t1 + ç 2 1 ÷
O A S B S C è 2 ø
Taking motion of car from 0 to A, (Q Time of ascending = Time of descending)
Here, u = 0 , a = f , s = S ,v = ? t +t
t= 1 2
As v 2 = u 2 + 2 as 2
So, maximum height
So v 2 = 0 + 2 ´ f ´ S or v = 2 fS 1 1 æt + t ö
2
æt + t ö
2
H = gt 2 = g ç 1 2 ÷ = 2g ç 1 2 ÷
2 2 è 2 ø è 4 ø
Telegram @unacademyplusdiscounts

3 Vector Analysis
JEE Main MILESTONE
< Scalars and Vectors < Resolution of Vectors
< Addition of Vectors < Scalar Product or Dot Product
< Subtraction of Vectors < Vector Product or Cross Product

3.1 Scalars and Vectors


On the basis of magnitude, direction and rules of addition, all physical quantities
are classified into two groups as scalars and vectors.

Scalars
A scalar quantity is one whose specification is completed with its magnitude only.
Two or more than two similar scalar quantities can be added according to the
ordinary rules of algebra. We found that the directional
For example, mass, distance, speed, energy, electric flux, current electricity, etc. aspect of the physical quantities
In the above example, current electricity has magnitude and direction both, but it can be taken care of by positive
is a scalar quantity because two different electrical currents can be added only (+ ve) and negative (- ve) signs, as
with simple algebra, as in one dimension only two
4A direction are possible. But, in order
to describe the motion of an
12 A object in two dimensions (a plane)
8A
or three dimensions (space), we
need to use vectors. Therefore, it
Vectors is first necessary to learn the
A vector quantity is one whose specification is completed with its magnitude and language of vectors.
direction both. Two similar vector quantities can be added according to the law of
parallelogram or triangle law.
For example, displacement, velocity, acceleration, force, electric field intensity,
current density etc.
A vector quantity can be represented by an arrow. The front end (arrow head)
represents the direction and length of the arrow gives its magnitudes as
O
P A
; P = OA
head
tail

OA = magnitude of the vector (not according to scalar). The magnitude of a vector P


can be written as | P | i. e. , modulus of P.
Telegram @unacademyplusdiscounts

86 JEE Main Physics

Vector = (Magnitude of the vector) ´ (direction of vector)


Types of Vectors
or A = | A | A$
Polar Vectors A$ is the unit vector drawn in the direction of A.
A vector whose, direction is along the direction of the A Vector
\ A$ = =
motion of a body or particle is known as a polar vector. | A| Magnitude of the vector
Vectors producing straight line or line effect are called
polar vectors. For example, force, momentum, velocity, Orthogonal Unit Vectors
displacement are polar vectors. The unit vectors along x-axis, y-axis and z-axis are
$
denoted by $i, $j and k.
Axial Vector y
These are orthogonal unit vectors.
A vector whose direction is along the axis of rotation of the
body or a particle is called an axial vector. An axial vector $i = x Þ x = x$i ^
j
always produces rotational effect on the body. For x
^
i
example, angular velocity (w), angular acceleration (a ), $j = y Þ y = y$j x
torque (t ) and angular momentum (L) are axial vectors. y ^
k
ω (Angular velocity) τ z
k$ = Þ z = zk$
z z
F
A
e.g., a vector of magnitude 3 along x-axis is
x = 3$i
A vector of magnitude 6 along –x axis is
Axis of x = 6 (- $i ) = - 6 $i
rotation

Equal Vectors Note With a unit vector, no units are to be attached (like newtonN,ms -1,
m etc.), i .e ., unit vector is dimensionless physical quantity.
Two vectors are said to be equal vectors, if they have equal
magnitude and same directions. The Zero Vector or Null Vector
A
A vector whose magnitude is zero and not having any
direction is called zero vector or null vector. It is
B represented by 0.
The vectors A and B are equal vectors i.e., A = B. The position vector of origin, the acceleration of a particle
moving with uniform velocity etc., represents a zero or
Co-initial Vectors null vector.
Vectors having common initial point are called co-initial Addition or subtraction of zero vector from a given vector
vectors. The vectors A, B, C and D are said to be co-initial does not affect the given vector.
vectors. i.e., A+0 = A
A
and A -0 = A
B
If a zero vector is multiplied by a scalar number it gives the
C zero vector i.e., n 0 = 0.
D
Sample Problem 1 A vector may change, if
(a) frame of reference is translated
(b) vector is rotated
Unit Vector (c) frame of reference is rotated
A vector of unit magnitude and whose direction is the (d) vector is translated parallel to itself
same as that of the given vector is called unit vector.
Basically, unit vector represents the direction of the given
Interpret (b) Vector will change, if rotated because its
direction changes.
vector. Consider a vector A. This vector can be written as
Telegram @unacademyplusdiscounts

Vector Analysis 87

1 $ $
Sample Problem 2 The expression ( i + j) is a (ii) Addition of Two Vectors Pointing in
2
(a) unit vector (b) null vector
Different Directions
(c) vector of magnitude 2 (d) scalar When two vectors are pointing in different directions, they
1/ 2 can be added using laws of vector addition as triangle law,
é æ 1 ö 2 æ 1 ö 2ù parallelogram law and polygon law.
Interpret (a) We have|R| = ê ç ÷ +ç ÷ ú =1
êë è 2 ø è 2ø ú
û Suppose A and B are two vectors
S
Multiplication of a Vector by a Real
Number +B
B =A B
The multiplication of a vector by a scalar quantity n gives a R
new vector whose magnitude is n times the magnitude of
the given vector. Its direction is same as that of the given O P
A A
vector, if n is a positive real number. Suppose a vector a is
then, OS = OP + PS or R = A + B
multiplied by a scalar quantity n.
\ A = na
For example, Laws of Vector Addition
If n = 4, then A = 4a
If n = - 4, then A = - 4 a, If n = 0, then A = 0 (null vector). 1. Triangle Law of Vector Addition
If two vectors are represented both in
Sample Problem 3 If A = 2 $i - 3$j + 4 k$ , is multiplied by a magnitude and direction by the two
number 5, then the vector along y-direction is sides of a triangle taken in the same R
B
(a) -15$j (b) 5$j order, then the resultant of these
vectors is represented both in α θ
(c) -5$j (d) 15$j
magnitude and direction by the third O A
side of the triangle taken in reverse
Interpret (d) As, n ´ A = nA, order as shown below
So, 5 ´ (2 $i - 3 $j + 4 k$) = 10 $i - 15 $j + 20k$
or R = A + B = B+ A
\ Vector along y-direction = -15 $j
The resultant, R can be calculated is given by
R= A2 + B2 + 2 AB cos q
3.2 Addition of Vectors If q is the angle between A and B, then
|A + B|= A2 + B2 + 2 AB cos q of R makes an angle a with A,
(i) Addition of Two Collinear Vectors B sin q
then, tan a = .
Suppose A and B are two collinear vectors. A + B cos q
A
O P
B 2. The Parallelogram Law
O′ O′
If two non-zero vectors A and B are represented by the two
Now, the resultant vector, R = A + B adjacent sides of a parallelogram then, the resultant is
i.e., given by the diagonal of the parallelogram passing
A B through the point of intersection of the two vectors.
O Q
P R
B
Suppose a body is displaced through 4 m due west and B
A+
then is further displaced through 6 m due west. Then, the R=
resultant displacement of the body = (4 m + 6 m) = 10 m β
α θ θ
due west.
A
Telegram @unacademyplusdiscounts

88 JEE Main Physics

The magnitude of R is Sample Problem 5 The resultant of two forces acting at an


R = |R | = A2 + B2 + 2 AB cos q …(i) angle of 150º is 10 N and is perpendicular to one of the forces.
where q is the angle between A and B.
Two other force is
B sin q (a) 20 / 3 N (b) 10 / 3 N
Here, tan a =
A + B cos q (c) 20 N (d) 20 / 3 N
A sin q
and tan b = …(ii) Interpret (c) We have, R 2 = A2 + B2 + 2 AB cos q
B + A cos q
(10) 2 = A2 + B2 + 2 AB cos150°
Special Cases = A2 + B2 + 2 AB ( - 3 / 2)
or 100 = A2 + B2 - 3 AB …(i)
If q = 0°, Rmax = A + B
B sin 150°
q = 180°, Rmin = A ~ B tan 90° =
A + B cos150°
and if q = 90° ,R = A2 + B2 B ´1/ 2 B
= =
In all other cases Eqs. (i) and (ii) can be used to calculate magnitude A + B ( - 3 /2) 2 A - 3 B
and direction of R. B
or ¥=
2A- 3B
3. Polygon Law of Vector Addition or 2A- 3B =0
If a number of vectors are represented both in magnitude 3
or A= B
and direction by the sides of a polygon taken in the same 2
order, then the resultant vector is represented both in Putting the value of A in Eq. (i), we get
magnitude and direction by the closing side of the 3 3
polygon taken in the opposite order. 100 = B2 + B2 - 3 ´ B´B
4 2
E U 1
V = B2
D 4
2
or B = 4 ´ 100
T
or B = 20 N
R C
Sample Problem 6 Three vectors each of magnitude A are
S
acting at a point such that angle between any two vectors is
B 60º. The magnitude of their resultant is
P
A Q (a) zero (b) 2 A
(c) 3 A (d) 6 A
Properties of Vector Addition
(i) Vector addition is commutative i.e., A + B = B + A Interpret (d) We have, R = | A + B + C|
(ii) Vector addition is associative i.e., (A + B) + C = A + (B + C ) = [ A2 + B2 + C 2 + 2 A × B + 2 B × C + 2 C × A ]1/ 2
(iii) Vector addition is distributive i.e., l (A + B) = lA + lB
Given, A=B=C
and q = 60°
Sample Problem 4 If A = B + C and the magnitude of A, B
R = [3 A2 + 2 A × B + 2 B × C + 2 C × A ]1/ 2
and C are 5, 4 and 3 minutes respectively, then angle between
A and C is = [3 A2 + (2 A × A cos 60° ) ´ 3]1/ 2
(a) cos–1 (4/5) (b) cos–1 (3/5) = 6A
(c) tan–1 (3/4) (d) sin–1(3/5)
Interpret (b) As, R 2 = A2 + B2 + 2 AB cos q and 52 = 43 + 32,
Þ AB cos q = 0 3.3 Subtraction of Vectors
So, angle between A and C is 90º. If q is the angle between A and B, Negative of a vector (–A) is a vector of the same magnitude
B sin q B
then, tan a = = as vector A but pointing in a direction opposite to that of A.
A + B cos q A
B 3 Therefore, A - B = A + (- B).
cos a = =
B2 + A2 5 Let the angle between vectors A and B be q, then the angle
or -1
a = cos (3 / 5) between A and - B will be 180° - q.
Telegram @unacademyplusdiscounts

Vector Analysis 89

R =A + B
Sample Problem 7 A car moving towards south changes
B its direction towards west moving with the same speed. Find
the change in the direction of velocity of the car.
θ
α (a) North-West
β (b) North-East
A
(c) South-East
180° – θ
–B (d) South-West
R = A + (–B) Interpret (a)
O N
Magnitude of S = A - B will be given by
|S|= |A - B|= A2 + B2 + 2 AB cos (180° - q) v
–v1 v1 W E
2 2
or S= A + B - 2 AB cos q
W S S
For direction of S, we will either calculate angle a or b, v2
B sin (180° - q) Here, |v1| = |v 2| = v (say)
tan a =
A + B cos (180° - q) \Change in velocity of car, Dv = v 2 - v1
B sin q Magnitude of the change in velocity,
=
A - B cos q
| Dv| = v12 + v 22 - 2 v1v 2 cos 90°
A sin (180° - q) A sin q
tan b = = = v2 + v2 - 0
B + A cos (180° - q) B - A cos q
= 2 v2 = 2 v
Note A - B or B - A can also be found by making triangles as shown in
figure. The direction of change in velocity,
|v | v
tan q = 1 = = 1
| v 2| v
or
A –B B –A or q = 45°
B

The change in velocity of the car is along north-west direction.


A A
(a) (b) Sample Problem 8 Find A - B from the diagram shown in
figure. Given A = 4 units and B = 3 units
Change in velocity of a particle moving along circular path with B
a constant speed
v1 = v

P θ = 60°
A
O
Q
(a) 18 units
(b) 17 units
(c) 14 units
v2 = v (d) 13 units

When a particle moves along a circular path with a constant Interpret (d) Addition R = A 2 + B2 - 2 AB cosq
speed, then its velocity changes due to change in direction. = 16 + 9 - 2 ´ 4 + 3 cos 60°
\ Change in velocity, Dv = v1 - v 2 = v - ( - v) = 2 v = 13 units
Telegram @unacademyplusdiscounts

90 JEE Main Physics

Hot Spot Resolution


of Vectors
Resolution of vectors is the process of resolving a vector into its components in such a way that, these components
produce the same effect on working together, as the given vector.
Rectangular Perpendicular components of a vector According to the vector addition rule, it may be written as
When a vector is resolved into its components and the components are A = A x+ A y + A z
right angles to each other, then such components are called If $i, $j and k$ are the unit vectors along x, y and z-axis respectively, then
rectangular perpendicular components. A x = Ax$i, A y = A y $j
To find the rectangular components of a vector lying in the and A = A k$
z z
plane, \ A = Ax$i + A y $j + Azk$
Suppose A x and A y are rectangular components of A.
y
\ | A |2 = A2x + Ay2 + Az2

\ |A | = A2x + A2y + A2z


N P
A

AY
Sample Problem 9 The magnitudes of vectors OA, OB
AY
and OC in figure. The direction of OA + OB - OC is
θ C Y
X A
O AX M

According to the triangle law of vector addition, 45° 30°


X
OP = OM + MP = OM + ON O 60°
\ A = Ax + A y
Here, A x and A y are two rectangular components of A. If $i and $j be the B
unit vectors along x-axis and y-axis respectively, then é (1 - 3 - 2) ù é (1 + 3 + 2) ù
(a) tan -1 ê ú (b) tan -1 ê ú
A = A $i + A $j
x y ë (1 + 3 + 2) û ë (1 - 3 - 2) û

If q is the angle subtended by vector A with x-axis, Ax = A cos q and é (1 - 3 - 2) ù é (1 + 3 + 2) ù


(c) cot -1 ê ú (d) cot -1 ê ú
A y = A sin q represented the rectangular components of A along two ë (1 + 3 + 2) û ë (1 - 3 - 2) û
perpendicular directions.
Interpret (a) Let, OA = OB = OC = F
\ A2 = A2x + A2y or A = A2x + A2y
3
x-component of OA = F cos 30° = F
For the directions of vectors 2
Ay F æ Ay ö
tan q = or q = tan -1 ç ÷ x-component of OB = F cos 60° =
Ax è Ax ø 2
F
To find the rectangular components of a vector lying in the x-component of OC = F cos135° = -
2
space
\x-component of OA + OB - OC
Suppose there is a vector A in space as shown in the figure. Let the æF 3 ö æF ö æ F ö
rectangular components of A along x-axis, y-axis and z-axis be A x, A y =ç ÷ + ç ÷ - ç- ÷
è 3 ø è2ø è 2ø
and A z respectively.
F
Y = ( 3 + 1 + 2)
P 2
F
y-component of OA = F cos 60° =
A 2
F 3
O X y-component of OB = - ×
2
F
Z y-component of OC = F cos 45° =
2
Telegram @unacademyplusdiscounts

Vector Analysis 91

\ y-component of OA + OB - OC The magnitude of R using the law of cosine,


æF ö æ F 3 ö æ F ö R = v b2 + v c2 + 2 v bv c cos120°
= ç ÷ + ç- ÷-ç ÷
è2ø è 2 ø è 2ø
Given, v b = 25 km/h
F
= (1 - 3 - 2) and v c = 10 km/h
2
æ 1ö
Angle of OA + OB - OC with x-axis \ R = 25 2 + 10 2 + 2 ´ 25 ´ 10 ç - ÷
è 2ø
éF ù
(1 - 3 - 2) ú
-1 ê 2 @ 22 km/h
= tan ê ú
F
ê (1 + 3 + 2) ú
ë2 û Sample Problem 13 A particle starts from origin at t = 0
é (1 - 3 - 2 ) ù with a velocity 5 $i m/s and moves is x-y plane under action of a
= tan -1 ê ú
ë (1 + 3 + 2) û force which produces a constant acceleration of (3 i$ + 2 $j) m/s 2.
At the instant its x-coordinate is 84 m, the speed of the particle
Sample Problem 10 A force of 8N makes an angle 30º at this time is
with x-axis. Find the x and y components of the force. (a) 16 ms -1 (b) 26 ms -1
(a) Fx = 4 3 N ,Fy = 4 N (b) Fx = 4 N ,Fy = 4 3 N (c) 20 ms -1 (d) 6 ms -1
(c) Fx = 2 N , Fy = 2 3 N (d) Fx = 2 3 N , Fy = 2 N
Interpret (b) The position of the particle is given by
Interpret (a) Here, F = 8 N, q = 30° 1 2
r (t ) = v 0t + at
3 2
x-component of force, Fx = F cos q = 8 ´ =4 3N 1
2 = 5 $i t + (3 $i + 2 $j) t 2
1 2
y-component of force, Fy = F sinq = 8 ´ = 4N
2 = (5 t + 1.5 t 2) $i + t 2 $j
\ x (t ) = 5 t + 1.5 t 2
Sample Problem 11 A force of 10.5 V acts on a particle
along a direction making an angle of 37° with the vertical. The y (t ) = 1.0 t 2
component of force in the vertical direction is Given, x (t ) = 84 m
(a) 4.5 N (b) 8.4 N then 5 t + 1.5 t 2 = 84
(c) 6.9 N (d) 3.2 N
Þ t =6s
Interpret (b) The component of the force in the vertical At t = 6 s, y = 1.0 (6) 2 = 36 m
direction will be
dr
4 Now the velocity, v= = (5 + 3 t ) $i + 2 t$j
Fv = F cos q = (10.5 N) cos 37° = 10.5 N ´ = 8.4 N dt
5
At t = 6 s, v = 23 $i + 12 $j
Sample Problem 12 A motorboat is racing towards north \ Speed = |v | = 23 2 + 12 2
at 25 km/h and the water current in that region is 10 km/h in the
~= 26 ms-1
direction of 60° east of south. The resultant velocity of the boat is
(a) 10 km/h (b) 20 km/h
(c) 12 km/h (d) 22 km/h
Check Point 1
Interpret (d) The vector v b representing the velocity of the
1. Can a vector be zero, if any of its components is non-zero?
motorboat and the vector v c representing the water current. Using
the parallelogram method of addition, the resultant R is obtained in 2. Under what condition, the magnitude of sum of two vectors is
the direction shown in the figure. equal to the magnitude of difference between them.
N
3. Can resultant be zero in case of
(a) two unequal vectors
R (b) three coplanar vectors
φ
vb (c) three non-coplanar vectors
W θ
E
vc 4. Can the flight of a bird be an example of composition of
60° vectors?

S
Telegram @unacademyplusdiscounts

92 JEE Main Physics

or A × B = A (B cos q) = (magnitude of A) (component of B in


3.4 Scalar Product or the direction of A).
Dot Product Dot product or scalar product of two vectors gives the
scalar quantity.
The scalar product of two vectors A and B is defined as the
product of magnitude of A and B multiplied by the cosine Note
of the smaller angle between them. (i) The dot product of force F and displacement s gives work (scalar
i.e., A × B = AB cos q quantity) i.e., F × s = W.
(ii) The dot product of force ( F ) and velocity ( v ) is equal to power
(scalar quantity) i.e., F × v = P.
B (iii) The dot product of magnetic induction ( B) and area vector ( A ) is
equal to the magnetic flux ( f) linked with the surface (scalar
quantity) B × A = fB .
θ (iv) The dot product of electric field intensity ( E ) and area ( A ) vector
O
A is the dot product of electric field equal to the electric flux ( fE )
B cos θ
linked with the surface (scalar quantity) E × A = fE .

Properties of Scalar Product or Dot Product


1. Dot product is commutative A × B = ABcos q and \ cos 90° = 0
B × A = BA cos q = AB cos q If two vectors A and B are perpendicular, then

\ A × B = B × A which is commutative law. A × B = AB cos 90° = 0


2. Dot product is distributive over the addition of vectors Now, $i × $j = 1 ´ 1 ´ cos 90° = 0,
i.e., A × (B + C ) = A × B + A × C $j × k$ = 1 ´ 1 ´ cos 90° = 0

3. Dot product of two parallel vectors k$ × $i = 1 ´ 1 ´ cos 90° = 0


A
Thus, $i × $j = $j × k$ = k$ × $i = 0
B
6. Dot product of two antiparallel vectors
Here, q = 0° A
\ A × B = AB cos0° = AB (Q cos 0° = 1)
4. Dot product of two equal vectors B
A

Here, q = 180°
A
\ cos180° = -1
The angle between two equal vectors is zero. Then, A × B = ABcos 180°
i.e., q = 0° = - AB
\ cos0° = 1
7. Dot product of two vectors in terms of their
\ A × A = AA cos0° = A2 components
Similarly, $i × $i = 1 ´ 1 ´ cos 0° = 1, $j × $j = 1 ´ 1 ´ cos 0° = 1 If A = x1$i + y1$j + z1k$
k$ × k$ = 1 ´ 1 ´ cos 0° = 1
and B = x2$i + y2$j + z2k$
\ $i × $i = $j × $j = k$ × k$ = 1
\ $ × ( x $i + y $j + z k)
A × B = ( x1$i + y1$j + z1k) $
2 2 2
5. Dot product of perpendicular vectors
or A × B = x1 x2 + y1y2 + z1 z2
where $i × $i = $j × $j = k$ × k$ = 1
B
and $i × $j = $j × k$ = k$ × $i = 0
θ = 90°
A
Telegram @unacademyplusdiscounts

Vector Analysis 93

Sample Problem 14 What is the angle f between The direction of the vector given by the cross product of
the two vectors is perpendicular to the plane containing
a = 3.0 $i - 4.0 $j and b = 2.0 i$ - 3.0 k$ ?
the two vectors.
(a) 60° (b) 53° ^
n
(c) 110º (d) 75°
C=A×B
Interpret (c) The angle between the directions of two vectors is
included in the definition of their scalar product
a × b = ab cos f …(i) θ
A B
In this equation, a is the magnitude of a, or
a = (3.0) 2 + ( - 4.0) 2 = 5 …(ii)
and b is the magnitude of b, or i.e., A ´ B = ( AB sin q) n$ = C

b = (2.0) 2 +(3.0) = 3.61 …(iii) where, n$ is the unit vector which gives the direction of
vector C.
We can separately evaluate the left side of Eq. (i) by writing the
vectors in unit-vector notation and using the distribution law The unit vector normal to the plane containing vectors A
a × b = (3.0 $i + 4.0 $j) ( -2.0 $i + 3.0 k$ ) and B, is given by
A´B
= (3.0 $i) × ( -2.0 $i) + (3.0 $i) × (3.0 k$ ) n$ =
AB sin q
+ ( - 4.0 $j) × ( -2.0 $i) + ( -4.0 $j) × (3.0 k$ ) A´B
=
or a × b = - (6.0) (1)+9.0 (0)+(8.0) (0) - 12 (0) | A ´ B|
= - 6.0
Substituting this and the result of Eqs. (ii) and (iii) into Eq. (i), we get
- 6.0 = (5.00) (3.61) cos f
Right Hand Rule for Direction of Vector
- 6.0 Product
So, f = cos-1
(5.00) (3.61) If a right handed screw is placed over the plane containing
= 109° @ 110° A and B as shown in the figure and is turned from A to B
(anti-clockwise) through a small angle then the direction
Sample Problem 15 The work done by a force F during a of advancement of the screw gives the direction of n$ or
displacement r is a given by F × r. A force of 12 N acts on a A ´ B i.e., upward perpendicular to the plane containing A
particle in vertically upward direction and the particle and B.
displaced through 2 m in vertically downward direction. The ^ n^
C =( AB sin θ) n
work done by the force during this displacement is
(a) - 24 J (b) – 12 J
n^
(c) – 38 J (d) 40 J
Interpret (a) The angle between the force F and the
displacement r is 180°. Therefore, the work done is θ θ
A B
W = F × r = Fr cos q
= 12 ´ 2 ´ cos 180° A B
= - 24 N-m = - 24 J Note
(i) The cross product of angular velocity ( w) and the radius vector r

3.5 Vector Product or is equal to the velocity (r) i.e., v = r ´ w


(ii) The cross product of position vector (r) and force ( F ) is equal to
Cross Product the torque ( t ) i.e.,
t = r´F
The vector product of two vectors is a vector quantity. The (iii) Angular momentum is the vector product of position vector ( r )
cross product of two vectors is a single vector whose and linear momentum (p)
magnitude is equal to the product of the magnitudes of i.e., L = r´p
two given vectors multiplied by the sine of the smaller
angle between the two given vectors.
Telegram @unacademyplusdiscounts

94 JEE Main Physics

Properties of Vector Product Scalar Triple Product


or Cross Product Scalar triple product of three vectors is given by
1. Cross product of two vectors does not obey the a1 a2 a3
commutative law. A × (B ´ C) = b1 b2 b3
i. e., A ´ B ¹ B´ A c1 c2 c3
Here, A ´ B = -B ´ A
2. Cross product of two vectors is distributive over vector It gives volume of parallelopiped formed with A, B and C as
addition adjacent sides.
i. e., A ´ (B + C ) = A ´ B + A ´ C Vector triple product is given by
3. Cross product of two parallel vectors or equal vectors A × (B ´ C) = B (A × C) - C (A × B)
is zero. In this case the angle between vectors will be
zero degree.
\ A ´ B = ( AB sin 0° ) n$ = 0 [Q sin 0° = 0] Check Point 2
The cross product of two equal vectors is given by
It is worth noting that
A ´ A = ( AA sin q) n$
(i) It represents the volume of parallelopiped represented by edges
or A´A = 0
$i ´ $i = (1 ´ 1 ´ sin 0° ) n$ = 0, A, B and C.
Similarly,
$j ´ $j = (1 ´ 1 ´ sin 0° ) n$ = 0 (ii) A × ( B ´ C ) = 0 implies that vectors are coplanar.
(iii) In scalar triple product, dot and cross can be interchanged
and k$ ´ k$ = (1 ´ 1 ´ sin 0° ) n$ = 0
provided that their cyclic order is maintained.
4. Cross product of two perpendicular vectors : In this (iv) Four points A, B, C and D are coplanar, if AB × ( BC ´ CD) = 0.
case, q = 90°
A ´ B = ( AB sin 90° ) n$ = ( AB) n$
5. Cross product between the pair of unlike unit vectors Sample Problem 16 The vector A has a magnitude of
$i ´ $j = 1 ´ 1 ´ sin 90° k$ = k$ 5 unit, B has a magnitude of 6 unit and the cross product A and
Similarly, $j ´ k$ = $i, k$ ´ $i = $j B has the magnitude of 15 unit. The angle between A and B is
$j ´ $i = - k$ , $i ´ k$ = - $j (a) 90° (b) 60°
Now,
(c) 30º (d) 120°
and k$ ´ $j = - $i
Interpret (c) If the angle between A and B is q, the cross
6. Cross product of two vectors in terms of their product will have a magnitude,
rectangular components
| A ´ B| = AB sin q
A = x1$i + y1$j + z1k$ and B = x2$i + y2$j + z2k$
or 15 = 5 ´ 6 sin q
\ $ ´ ( x $i + y $j + z k)
A ´ B = ( x $i + y $j + z k) $
1 1 1 2 2 2 1
or sin q =
A ´ B can be determined as follows 2
$i $j k$ \ q = 30°
A ´ B = x1 y1 z1
x2 y2 z2
Sample Problem 17 If A = i$ + 2 k$ and B = $j - k$ , then,
A ´ B is equal to
7. Magnitude of cross product of two vectors A and B (a) 2 $i - $j - k$
represents the area of the parallelogram. Suppose
(b) -2 $i + $j + k$
OPRQ be a parallelogram whose adjacent sides OP
and OQ are represented both in magnitude and (c) $i + $j - k$
direction by two vectors A and B. (d) 2 $i + $j - k$
Q R
Interpret (b) We have, A ´ B = ( $i + 2 k$ ) ´ ( $j - k$ )
B sin $i $j k$
B
= 1 0 2
0 1 -1
O
N A P = $i (0 - 2) + $j (0 + 1) + k$ (1 - 0)
\ | A ´ B | = AB sin q = A (B sin q) = OP ´ QN = Area of the
= -2 $i + $j + k$
parallelogram.
Telegram @unacademyplusdiscounts

WORKED OUT
Examples
Example 1 Two quantitites A and B have different or A + B cos q = 0
dimensions. Which mathematical operation given below is or B cos q = - A
physically meaningful? \ 8 3 = [ A2 + B2 + 2A ( - A)]1/ 2
(a) A/B (b) ( A + B) or 192 = B2 - A2 = (B - A) (B + A) = (B - A) ´ 16
(c) ( A - B) (d) None of these
or B - A = 192 / 16 = 12
Solution The quantities having different dimensions can only On solving A = 2 and B = 14
be multiplied or divided but cannot be added or subtracted.
Example 4 The vectors A1 and A 2 each of magnitude A are
Example 2 A particle has two equal accelerations in two
inclined to each other such that their resultant is equal to 3A.
given directions. If one of the accelerations is halved, then the
angle which the resultant makes with the other is also halved. Then the resultant of A1 and A 2 is
The angle between the accelerations is (a) 2A (b) 3A
(a) 120° (b) 90°
(c) 2A (d) A
(c) 60° (d) 45°
B tan q Solution Let q be the angle between A1 and A 2. Then,
Solution tan b =
A + B cos q A12 + A22 + 2A1A2 cos q = R 2
A sin q or A2 + A2 + 2AA cos q = 3 A2
= (\A = B)
A + A cos q
1
sin q or cos q = = cos 60°
= …(i) 2
(1 + cos q) or q = 60°
b ( A / 2) sin q The angle between A1 and -A 2 is (180° - 60° ) = 120°
tan =
2 A + ( A / 2) cos q
\Resultant of A1 and -A 2 is
sin q R ¢ = [ A12 + A22 + 2A1A2 cos(180°-60° )]1/ 2
= …(ii)
2 + cos q
= [ A2 + A2 + 2AA cos120° ]1/ 2 = A
The questions are satisfied if q = 120°

Example 3 The sum of magnitudes of two forces acting at a Example 5 If A and B are perpendicular vectors, where
point is 16 and magnitude of their resultant is 8 3. If the
$ $ $
A = 5 i + 7 j - 3k and B = 2 i$ + 2 $j - ak$ , then the value of a is
resultant is at 90° with the force of smaller magnitude, their (a) - 2 (b) 8
magnitudes are (c) - 7 (d) - 8
(a) 3, 13 (b) 2, 14
(c) 5, 11 (d) 4, 12 Solution For perpendicular vectors, A × B = 0
So, (5$i + 7$j - 3k$ ) × (2$i + 2$j - ak$ ) = 0
Solution Given A + B = 16
or 10 + 14 + 3a = 0 or 3a = - 24
R = ( A2 + B2 + 2AB cos q)1/ 2
or a = -8
8 3 = ( A2 + B2 + 2AB cos q)1/ 2
B sin q Example 6 The area of a parallelogram whose adjacent
and tan 90° =
A + B cos q sides are P = 2 i$ + 3$j and Q = $i + 4$j is
B sin q (a) 5 square units (b) 15 square units
or ¥=
A + B cos q (c) 20 square units (d) 25 square units
Telegram @unacademyplusdiscounts

96 JEE Main Physics

Solution P ´ Q = (2$i + 3$j ) ´ ( $i + 4$j) Example 8 The angle between the two vectors
$i $j k$ A = 3i$ + 4 $j + 5 k$ and B = 3$i + 4$j - 5 k$ will be
= 2 3 0 (a) zero
1 4 0 (b) 45°
(c) 90°
= $i (0 - 0) - $j (0 - 0) + k$ (8 - 3) = 5k$
(d) 180°
Area of parallelogram
= |P ´ Q| = 5 square units Solution A × B = (3$i + 4 $j + 5k$ ) × (3$i + 4 $j - 5k$ ) = 9+16 - 25 = 0
or AB cos q = 0
Example 7 If A = i$ + 2 $j + 3k$ and B = 3$i - 2 $j + k$ , then the
or cos q = 0
area of parallelogram formed from these vectors as the adjacent
or q = 90°
sides will be
(a) 2 3 square units (b) 4 3 square units
(c) 6 3 square units (d) 8 3 square units
Example 9 The vectors P = a i$ + a$j + 3k$ and Q = a i$ - 2 $j - k$
are perpendicular to each other. The positive value of a is
$i $j k$ (a) 3 (b) 2
Solution A ´B = 1 2 3 (c) 1 (d) 0
3 -2 1
Solution For perpendicular vectos P × Q = 0
= $i (2 + 6) + $j (9 - 1) + k$ ( - 2 - 6) So, ( a$i + a$j + 3k$ ) × ( a$i - 2$j - k$ ) = 0
= 8$i + 8 $j - 8 k$ or a2 - 2 a - 3 = 0
2 2 2
Area = | A ´ B| = 8 + 8 + ( - 8) On solving a = 3 or -1
= 8 3 square units
Telegram @unacademyplusdiscounts

Start Practice for


JEE Main
Round I (Topically Divided Problems)

Addition of Vectors 7. Given R = A + B and R = A = B. The angle between A


P and B is
1. Two forces, each equal to act at right angles. Their
2 (a) 60° (b) 90°
effect may be neutralised by a third force acting along (c) 120° (d) 180°
their bisector in the opposite direction with a 8. The magnitude of the X and Y components of A are 7
magnitude of and 6. Also the magnitudes of X and Y components of
P A + B are 11 and 9 respectively. What is the
(a) P (b)
2 magnitude of B ?
P
(c) (d) 2 P (a) 5 (b) 6
2 (c) 8 (d) 9
2. What is the numerical value of the vector 3 $i + 4$j + 4k$ ? 9. One of the rectangular components of a velocity of
(a) 3 2 (b) 5 2 60 kmh–1 is 30 kmh –1. The other rectangular
(c) 7 2 (d) 9 2 component is
3. A = 3 i$ - $j + 7 k$ and B = 5 $i - $j + 9 k$ . The direction (a) 30 kmh-1 (b) 30 3 kmh-1
-1
cosine, m of the vector A + B is (c) 30 2 kmh (d) zero
3 10. The angle between the z-axis and the vector
(a) zero (b)
31 $ is
$i + $j + 2 k
8
(c) (d) 5 (a) 30° (b) 45°
336
(c) 60° (d) 90°
4. Given A = $i + 2$j - 3 k$ . When a vector B is added to A,
11. The resultant of two forces, each P, acting at an angle
we get a unit vector along X-axis. Then, B is q is
(a) -2$j + 3 k$ (b) - $i - 2$j q q
(a) 2P sin (b) 2P cos
(c) - $i + 3 k$ (d) 2$j - 3 k$ 2 2
(c) 2P cos q (d) P 2
5. Two forces F1 and F2 are acting at right angles to each
other. Then their resultant is 12. The resultant of two vectors of magnitudes 2A and
2 A acting at an angle q is 10 A. The correct value
(a) F1 + F2 (b) F12 + F2 2
of q is
F +F
(c) F12 - F2 2 (d) 1 2 (a) 30° (b) 45°
2 (c) 60° (d) 90°
6. The x and y components of a force are 2 N and – 3N. 13. If, 0.5 $i + 0.8 $j + c k$ is a unit vector, then the value of
The force is
c is
(a) 2$i - 3$j (b) 2$i + 3$j
(a) 0.11 (b) 0.22
(c) -2$i - 3$j (d) 3 $i + 2$j
(c) 0.33 (d) 0.89
Telegram @unacademyplusdiscounts

98 JEE Main Physics

14. Two forces, each equal to F, act as shown in figure. 22. If the resultant of A and B makes angle a with A and
Their resultant is b with B, then
(a) a < b, always (b) a < b, if A < B
(c) a < b, if A > B (d) a < b, if A = B
F
23. A proton in a cyclotron changes its velocity from
60° 30 kms–1 north to 40 kms–1 east in 20 s. What is the
F average acceleration during this time
F (a) 2.5 km s–2 at 37º E of S
(a) (b) F (c) 3 F (d) 5 F (b) 2.5 km s–2 at 37º N of E
2
(c) 2.5 km s–2 at 37º N of S
15. If P = 4 i$ - 2 $j + 6 k$ and Q = i$ - 2 $j - 3 k$ , then the (d) 2.5 km s–2 at 37º E of N
angle which P + Q makes with x-axis is
24. What is the angle between P and Q. The resultant of
æ 3 ö æ 4 ö (P + Q) and (P - Q)?
(a) cos -1 ç ÷ (b) cos -1 ç ÷
è 50 ø è 50 ø
(a) zero (b) tan -1 ( P / Q )
æ 5 ö æ 12 ö
(c) cos -1 ç ÷ (d) cos -1 ç ÷ (c) tan -1 ( Q / P ) (d) tan -1 ( P - Q )/( P + Q )
è 50 ø è 50 ø
25. The resultant of two forces at right angle is 5N. When
16. If A + B = C and A = 3, B = 3 and C = 3, then the
the angle between them is 120°, the resultant is 13.
angle between A and B is
Then, the forces are
(a) 0° (b) 30° (c) 60° (d) 90°
(a) 12 N, 13 N (b) 20 N, 5 N
17. The angle between A = $i + $j and B = $i - $j is (c) 3 N, 4 N (d) 40 N, 15 N
[NCERT Exemplar]
(a) 45° (b) 90° (c) - 45° (d) 180° 26. If the resultant of the vectors ( $i + 2$j - k$ ), ( $i - $j + 2k)
$

18. If the magnitude of the sum of the two vectors is and C is a unit vector along the y-direction, then C is
equal to the difference of their magnitudes, then the (a) -2$i - k$ (b) -2$i + k$ (c) 2$i - k$ (d) -2$i + k$
angle between vectors is 27. Which one of the following statements is true?
(a) 0° (b) 45° [NCERT Exemplar]
(c) 90° (d) 180° (a) A scalar quantity is the one that is conserved in a process
19. The simple sum of two co-initial vectors is 16 units. (b) A scalar quantity is the one that can never take negative
Their vector sum is 8 units. The resultant of the values
vectors is perpendicular to the smaller vector. The (c) A scalar quantity is the one that does not vary from one
magnitudes of the two vectors are point to another in space
(a) 2 units and 14 units (b) 4 units and 12 units (d) A scalar quantity has the same value for observers with
different orientations of the axes
(c) 6 units and 10 units (d) 8 units and 8 units
20. If, the resultant of two forces ( A + B) and ( A - B) is 28. (P + Q) is a unit vector along X-axis. If, P = $i - $j + k$ ,
A2 + B2 , then the angle between these forces is then what value is Q?
2
é (A - B )ù 2 (a) $i + $j - k$ (b) $j - k$ (c) $i + $j + k$ (d) $j + k$
-1
(a) cos ê - 2 2 ú
ë A +B û 29. What vector must be added to the sum of two vectors
$ and 3 $i - 2 $j - 2 k
2 $i - $j + 3 k $ so that the resultant is a
é ( A2 + B2 ) ù
(b) cos -1 ê - 2 2 ú
ë (A - B )û unit vector along Z-axis?
(a) 5 $i + k$ (b) -5 $i + 3 $j (c) 3 $j + 5 k$ (d) -3 $j + 2 k$
é A2 + B2 ù
(c) cos -1 ê - 2 2 ú
ë 2( A - B ) û 30. The resultant of a system of forces shown in figure is
é 2 ( A2 + B2 ) ù a force of 10 N parallel to given forces through R,
(d) cos -1 ê - ú where PR equals
ë A2 - B2 û
P R Q
21. If, A = $i + $j - 2k$ and B = 2i$ - $j + k$ , then the
magnitude of 2 A - 3 B is 4N
(a) 90 (b) 50
6N
(c) 190 (d) 30 (a) (2/5) RQ (b) (3/5) RQ (c) (2/3) RQ (d) (1/2) RQ
Telegram @unacademyplusdiscounts

Vector Analysis 99

Product of Vectors 42. Given, r = 4 $j and p = 2 $i + 3 $j + k$ . The angular


$ momentum is
31. Given, vector, A = i$ - $j + 2 k and vector
$ $ $ (a) 4 $i - 8 k$ (b) 8$i - 4 k$
B = 3 i - 3 j + 6 k, then which one of the following
(c) 8 $j (d) 9 k$
statements is true?
(a) A is perpendicular to B
$ N acts on a body of mass
43. A force of (10 $i - 3 $j + 6k)
(b) A is parallel to B $ ) m to
100 g and displaces it from (6 $i + 5 $j - 3 k
(c) Magnitude of A is half of that of B $
(10 i$ - 2 $j + 7 k) m. The work done is
(d) Magnitude of B is equal to that of A
(a) 21 J (b) 121 J
32. Given, q is the angle between A and B. Then,|A ´ B| (c) 361 J (d) 1000 J
is equal to
(a) sin q (b) cos q
44. A force, F = 2 i$ + 2 $j N displaces a particle through
$ m in 16 s. The power developed by F is
S = 2 $i + 2 k
(c) tan q (d) cot q
(a) 0.25 J s–1 (b) 25 J s–1
33. Given, p = 3 $j + 4 k$ and Q = 2 i$ + 5 k$ . The magnitude
(c) 225 J s–1 (d) 450 J s–1
of the scalar product of these vectors, is
(a) 20 (b) 23 45. Projection of P on Q is
(c) 26 (d) 5 33 (a) PQ$ $
(b) PQ
(c) P ´ Q$ (d) P ´ Q
34. If P × Q = 0, then|P ´ Q|is
(a) |P ||Q| (b) zero 46. Two vectors a and b are such that |a + b|= |a - b|.
(c) 1 (d) PQ What is the angle between a and b?
35. Given, c = a ´ b. The angle which a makes with c is (a) 0° (b) 90° (c) 60° (d) 180°
(a) 0° (b) 45° 47. Given, A = 4 $i + 6 $j and B = 2 $i + 3 $j. Which of the
(c) 90° (d) 180° following is correct?
36. If P = 2 $i - 3 $j + k$ and Q = 3 i$ - 2 $j, then P × Q is (a) A ´ B = 0 (b) A × B = 24
|A | 1
(a) zero (b) 6 (c) = (d) A and B are anti-parallel
|B| 2
(c) 12 (d) 15
48. If A × B = 0 and A ´ B = 1, then A and B are
37. If AB = AB, then the angle between A and B is
(a) perpendicular unit vectors
(a) 0° (b) 45°
(b) parallel unit vectors
(c) 90° (d) 180°
(c) parallel
38. What is the unit vector along $i + $j ? (d) perpendicular
$i + $j
(a) (b) 2 ( $i + $j ) 49. The torque of a force F = - 3 $i + $j + 5 k$ acting at a
2
point is t. If the position vector of the point is
(c) i + $j
$ (d) k$ $ , then t is
7 $i + 3 $j + k
39. The adjacent sides of a parallelogram are (a) 7 $i - 8 $j + 9 k$ (b) 14 $i - $j + 3 k$
represented by co-initial vectors 2 i$ + 3 $j and i$ + 4 $j.
(c) 2 $i - 3 $j + 8 k$ (d) 14 $i - 38 $j + 16 k$
The area of the parallelogram is
(a) 5 units along z-axis (b) 5 units in x-y plane 50. The area of a parallelogram formed by the vectors
(c) 3 units in x-z plane (d) 3 units in y-z plane $ and B = 3 i$ - 2 $j + k
A = $i - 2 $j + 3 k $ as its adjacent

40. The magnitudes of the two vectors a and b are a and sides, is
b, respectively. The vector product of a and b cannot (a) 8 3 units (b) 64 units
be (c) 32 units (d) 4 6 units
(a) equal to zero (b) less than ab
51. Given that, A + B + C = 0. Out of three vectors, two
(c) equal to ab (d) greater than ab
are equal in magnitude and the magnitude of third
41. Given, P = A + B and P = A + B. The angle between vector is 2 times that of either of the two having
A and B is equal magnitude. Then, the angles between vectors
p p are given by
(a) 0° (b) (c) (d) p
4 2 (a) 45°, 45°, 90° (b) 90°, 135°, 135°
(c) 30°, 60°, 90° (d) 45°, 60°, 90°
Telegram @unacademyplusdiscounts

100 JEE Main Physics

52. The magnitude of the vectors product of two vectors 62. The momentum of a particle is p = 2 cos t × i$ + 2 sin t $j.
is 3 times their scalar product. The angle between What is the angle between the force F acting on the
the two vectors is particle and the momentum p.
(a) 90º (b) 60º (c) 45º (d) 30º (a) 65° (b) 90°
53. If, A = 2 i$ + 3 $j + 4 k$ and B = 4 i$ + 3 $j + 2 k$ , then angle (c) 150° (d) 180°

between A and B is 63. A vector F1 is along the positive Y-axis. If its vector
æ 25 ö æ 29 ö product with another vector F2 is zero, then F2 could
(a) sin -1 ç ÷ (b) sin -1 ç ÷ be
è 29 ø è 25 ø
æ 25 ö æ 29 ö (a) 4 $j (b) $j + k$
(c) cos -1 ç ÷ (d) cos -1 ç ÷
è 29 ø è 25 ø (c) $j - k$ (d) -4 $i

54. Consider the quantities, pressure, power, energy, 64. If the vectors A = 2 i$ + 4 $j and B = 5 $i - p $j are parallel
impulse, gravitational potential, electrical charge, to each other, the magnitude of B is
temperature, area. Out of these, the only vector (a) 5 5 (b) 10
quantities are [NCERT Exemplar] (c) 15 (d) 2 5
(a) Impulse, pressure and area
65. If the magnitudes of scalar and vector products of two
(b) Impulse and area
vectors are 6 and 6 3 respectively, then the angle
(c) Area and gravitational potential
between two vectors is
(d) Impulse and pressure
(a) 15º (b) 30º
55. Three vectors A, B and C satisfy the relation AB = 0 (c) 60º (d) 75º
and AC = 0. If B and C are not lying in the same
66. What is the angle between $i + $j + k$ and $i?
plane, then A is parallel to
(a) 0º (b) p/6
(a) B (b) C (c) B ´ C (d) BC
(c) p/3 (d) None of these
56. A force of (7 $i + 6 k$ ) N makes a body move on a rough
plane with a velocity of (3 $j + 4 k$ ) ms -1. Calculate the
67. An object moves along a straight line path from P to
$ ) N. If the
Q under the action of a force ( 4 $i - 3 $j + 3 k
power in watt
(a) 24 (b) 34 coordinates of P and Q in metres are (3, 3, –1) and
(c) 21 (d) 45 (2, –1, 4) respectively, then the work done by the force
$ and $i? is
57. What is the angle between ( $i + 2$j + 2k)
(a) + 23 J (b) –23 J
(a) 0º (b) p /6
(c) 1015 J (d) 35 ( 4 $i - 3$j + 2 k$ ) J
(c) p /3 (d) None of these
58. If A = B, then which of the following is not correct? 68. For motion in a plane with constant acceleration a,
initial velocity v 0 and final velocity v after time t, we
(a) A$ = B$ (b) A$ × B
$ = AB
(c) |A | = |B| (d) AB || BA$
$ have
(a) v × ( v - at ) = v0 × ( v0 + at )
59.(b)For× = what value of a, A = 2 i$ + a $j + k$ will be v v at2
0
$?
perpendicular to B = 4 i$ - 2 $j - k (c)v × v0 = a × v0t
(a) 4 (b) zero (d) v0 × v0 = a × v0t
(c) 3 (d) 1 69. Given that A and B are greater than 1. The
60. The sum of two vectors A and B is at right angles to magnitude of ( A ´ B) cannot be
their difference. Then (a) equal to AB (b) less than AB
(a) A = B (c) more than AB (d) equal to A/B
(b) A = 2B 70. A force, F = - K ( y i$ + x $j) (where, K is a positive
(c) B = 2 A constant) acts on a particle moving in the xy plane.
(d) A and B have the same direction Starting from the origin, the particle is taken along
61. A point of application of a force F = 5 $i - 4 $j + 2 k$ is the positive x-axis to the point (a, 0) and then parallel
$ to r = - 5 i$ + 2$j + 3 k
moved from r1 = 2 $i + 7 $j + 4 k $ to the y-axis to the point (a, a). The total work done by
2
the force, F on the particle is
the work done is
(a) - 2 Ka2 (b) 2 Ka2
(a) –17 units (b) –22 units
(c) 33 units (d) –33 units (c) - Ka2 (d) Ka2
Telegram @unacademyplusdiscounts

Vector Analysis 101

(a) P = W tan q
Components of a Vector (b) t + P + W = 0
71. The coordinates of a moving particle at time t are (c) T 2 = P 2 + W 2
given by x = ct2 and y = bt2 . The instantaneous speed (d) T = P + W
of the particle is
(a) 2t ( b + c ) (b) 2 t ( b + c )1 /2
76. The X and Y components of vector A have numerical
values 6 and 6 respectively and that of ( A + B) have
(c) 2 t ( c2 - b2 ) (d) 2 t ( c2 + b2 )1 /2 numerical values 10 and 9. What is the numerical
72. Following forces start acting on a particle at rest at value of B?
the origin of the coordinate system simultaneously (a) 2 (b) 3
$ , F = 2 i$ + 8 $j + 6k
F1 = 5$i - 5$j + 5 k $,
$ , F = - 6$i + 4 $j - 7 k (c) 4 (d) 5
2 3
$ . The particle will move
F = - $i - 3 $j - 2 k 77. A particle of mass = 5 is moving with a uniform speed
4
(a) in x-y plane (b) in y-z plane v = 3 2 in the XOY plane along the line y = x + 4. The
(c) in x-z plane (d) along x-axis magnitude of the angular momentum of the particle
about the origin is
73. A force is inclined at 60° to the horizontal. If its (a) 60 units
rectangular component in the horizontal direction is (b) 40 2 units
50 N, then magnitude of the force in the vertical
(c) 7.5 units
direction is
(d) zero
(a) 25 N (b) 75 N
(c) 87 N (d) 100 N 78. The component of a vector r along X-axis will have
maximum value if [NCERT Exemplar]
74. Figure shows the orientation of two vectors u and v
(a) r is along positive Y-axis
in the XY plane. [NCERT Exemplar]
(b) r is along positive X-axis
Y
(c) r makes an angle of 45° with the X-axis
(d) r is along negative Y-axis
v 79. There are two forces each of magnitude 10 units. One
u
inclined at an angle of 30° and the other at an angle
of 135° to the positive direction of x-axis. The x and y
O X components of the resultant are respectively,
If u = ai$ + b$j and v = pi$ + q$j, (a) 1.59 $i and 12.07 $j
which of the following is correct? (b) 10 $i and 10 $j
(a) a and p are positive while b and q are negative
(c) 1.59 $i
(b) a, p and b are positive while q is negative
(d) 15.9 $i and 12.07 $j
(c) a, q and b are positive while p is negative
(d) a, b, p and q are all positive 80. A body of mass 3 kg is suspended by a string from a
75. A small sphere is hung by a string rigid support. The body is pulled horizontally by a
fixed to a wall. The sphere is force F until the string makes an angle of 30° with the
T
pushed away from the wall by a θ vertical. The value of F and tension in the string are
stick. The force acting on the P (a) 19.6 N; 19.6 N
sphere are shown in figure. (b) 9.8 N, 9.8 N
Which of the following (c) 9.8 N, 19.6 N
statements is wrong? W (d) 19.6 N, 9.8 N
Telegram @unacademyplusdiscounts

102 JEE Main Physics

Round II (Mixed Bag)

Only One Correct Option 6. The sum of the magnitudes of two forces acting at a
1. The vector which can give unit vector along x-axis point is 16 N. The resultant of these forces is
with $,
A = 2 i$ - 4 $j + 7 k $
B = 7 i$ + 2 $j - 5 k and perpendicular to the smaller force has a magnitude of
$ $ $ 8 N. If the smaller force is magnitude x, then, the
C = - 4 i + 7 j + 3 k is
value of x is
(a) 4 $i + 5 $j + 5 k$ (b) -5 $i - 5 $j + 5 k$ (a) 2 N (b) 4 N (c) 6 N (d) 7 N
(c) -4 $i - 5 $j - 5 k$ (d) 4 $i - 5 $j - 5 k$ 7. Four concurrent coplanar forces 3N
2. A man 80kg is supported by two cables as shown in in newton are acting at a point F
the figure. Then the ratio of tensions T1 and T2 is and keep it in equilibrium
(figure). Then values of F and q θ
A B are 1N
60° 30°
(a) 1N, 60º (b) 2 N, 60º θ
T1
(c) 2 N, 90° (d) 2 N, 90º
T2
8. If A 1 and A 2 are two 2N
C non-collinear unit vectors and if
80 kg |A 1 + A 2|= 3 , then the value of
( A 1 - A 2 ) × (2 A 1 + A 2 ) is
(a) 1: 1 (b) 1 : 3 (a) 1 (b) 1/2 (c) 3/2 (d) 2
(c) 3 : 1 (d) 1 : 3
9. Two vectors a and b are at an angle of 60° with each
3. If A = 2 i$ + 3 $j + 6 k$ and B = 3 i$ - 6$j + 2 k$ , then vector other. Their resultant makes an angle of 45° with a.
perpendicular to both A and B has magnitude k If|b|= 2 units, then|a|is
$ . That k is equal to
times that of (6 $i + 2 $j - 2 k) (a) 3 (b) 3 - 1 (c) 3 + 1 (d) 3 / 2

(a) 1 (b) 4 (c) 7 (d) 9 10. The x-y plane is the boundary between two
4. A proton of velocity (3 i$ + 2 $j) ´ 105 ms -1 enters a transparent media. A medium I has a refractive
$ ) T . If the specific charge is
magnetic field (2 $i + 3 k index m1 = 2 and medium II has a refractive index
m2 = 3 . A ray of light in medium I, given by vector,
9.6 ´ 107 C kg –1, the acceleration of the proton in $ is incident on the plane of separation.
A = 3 $i - k
ms–2 is
The unit vector in the direction of the refracted ray in
(a) (6 $i - 9 $j + 4 k$ ) ´ 9.6 ´ 1012
medium II is
(b) (6 $i + 9 $j + 4 k$ ) ´ 9.6 ´ 1012 1 $ $ 1 $ $
(a) ( i + k) (b) ( i + j)
(c) (6 $i - 9 $j - 4 k$ ) ´ 9.6 ´ 1012 2 2
(d) (6 $i + 9 $j - 4 k$ ) ´ 9.6 ´ 1012 1 $ $ 1 $ $
(c) (k - i ) (d) ( i - k)
2 2
5. Three forces of magnitudes 6 N, 6 N and 72 N act at
a corner of a cube along three sides as shown in 11. If A, B and C are the unit vectors along the incident
figure. Resultant of these forces is ray, reflected ray and outward normal to the
reflecting surface, then
72 N (a) B = A - C (b) B = A + ( AC) C
G
D (c) B = 2 A - C (d) B = A - 2( AC) C
12. Two vectors A and B are inclined at an angle q. Now if
F E the vectors are interchanged then the resultant turns
6N through an angle b. Which of the following relation is
C true?
2
a æ A - Bö q a æ A - Bö q
6N A B (a) tan =ç ÷ tan (b) tan = ç ÷ tan
2 è A + Bø 2 2 è A + Bø 2
a æ A - Bö q a æ A - Bö q
(a) 12 N along OB (b) 18 N along OA (c) tan = ç ÷ cot (d) tan = ç ÷ cot
2 è A+ B ø 2 2 è A+ B ø 2
(c) 18 N along OC (d) 12 N along OE
Telegram @unacademyplusdiscounts

Vector Analysis 103

13. The vectors $,


2 i$ + 3 $j - 2 k $
5 i$ + a $j + k and 22. A body constrained to move in y-direction, is
$ are coplanar when ‘a’ is $)
subjected to a force given by F = ( - 2 $i + 15 $j + 6 k
- $i + 2 $j + 3 k
(a) –9 (b) 9 (c) –18 (d) 18 done by this force in moving the body through a
distance of 10m along y-axis?
14. A vector A when added to the vector B = 3 i$ + 4 $j (a) 190 J (b) 160 J
yields a resultant vector that is in the positive (c) 150 J (d) 20 J
y-direction and has a magnitude equal to that of B.
23. Consider a vector F = 4 i$ - 3 $j. Another vector that is
Find the magnitude of A.
perpendicular to F is
(a) 10 (b) 10 (c) 5 (d) 15
(a) 4 $i + 3 $j (b) 6 $j (c) 7 $j (d) 3 $i - 4 $j
15. In a two dimensional motion of a particle, the particle
moves from point A, position vector r1 to point B,
24. Work done when a force, F = ( i$ + 2 $j + 3 k$ ) N acting on
$ m to
a particle takes it from the point r1 = ( $i - $j + k)
position vector r2 . If the magnitudes of these vectors
are respectively, r1 = 3 and r2 = 4 and the angles they $
the point r = ( $i + $j + 2 k) m is
2
make with the x-axis are q 1 = 75° and 15°, (a) –3 J (b) –1 J (c) zero (d) 2 J
respectively, then find the magnitude of the
displacement vector 25. The radius vector and linear momentum are
A respectively given by vector 2 i$ + 2 $j + k$ and
$ . Their angular momentum is
2 $i - 2 $j + k
B
r1 (a) 2 $i - 4 $j (b) 4 $i - 8 k$
θ1 r2 (c) 2 $i - 4 $j + 2 k$ (d) 4 $i - 8 $j
θ2

(a) 15 (b) 13 (c) 17 (d) 15 More Than One Correct Option


16. The resultant of two vectors A and B is perpendicular 26. Three vectors A, B and C add up to zero. Find which is
to the vector A and its magnitude is equal to half of false [NCERT Exemplar]
the magnitude of vector B. Then, the angle between (a) (A ´ B) ´ C is not zero unless B, C are parallel
A and B is (b) (A ´ B) × C is not zero unless B, C are parallel
(a) 30º (b) 45º (c) 150º (d) 120º (c) If A, B, C defined a plane, ( A ´ B) ´ C is in that plane
(d) ( A ´ B) × C = |A ||B||C|® C2 = A2 + B2
17. The magnitude of resultant of three vectors of
magnitude 1, 2 and 3 whose directions are those of 27. If vectors A and B are given by A = 5 i$ + 6$j + 3 k$ and
the sides of an equilateral triangle taken in order is $ . Which is/are of the following
B = 6 $i - 2 $j - 6 k
(a) zero (b) 2 2 unit (c) 4 3 unit (d) 3 unit
correct?
18. The area of the parallelogram represented by the (a) A and B are mutually perpendicular
vectors, A = 4 $i + 3 $j and B = 2 $i + 4 $j is (b) Product of A ´ B is the same B ´ A
(a) 14 units (b) 7.5 units (c) The magnitude of A and B are equal
(c) 10 units (d) 5 units (d) The magnitude of A × B is zero
19. If A and B denote the sides of a parallelogram and its 28. It is found that |A + B|= |A|. This necessarily
1 implies,
area is AB (A and B are the magnitude of A and B [NCERT Exemplar]
2 (a) B = 0 (b) A, B are antiparallel
respectively), the angle between A and B is (c) A, B are perpendicular (d) A × B £ 0
(a) 30º (b) 60º (c) 45º (d) 120º
29. Which of the following statements is/are correct?
20. Two vectors A and B are inclined to each other at an (a) The magnitude of the vector 3 $i + 4 $j is 5
angle q. Which of the following is the unit vector
(b) A force (3 $i + 4 $j ) N acting on a particle causes a
perpendicular to both A and B?
displacement 6 $j. The work done by the force is 30 N
A´B A$ ´ B
$ A´B A´B
(a) (b) (c) (d) (c) If A and B represent two adjacent sides of a
AB sin q AB sin q ABcos q
parallelogram, then |A ´ B| give the area of that
21. Angle between A and B is q. What is the value of parallelogram
A × (B ´ A ) ? (d) A force has magnitude 20 N. Its component in a direction
(a) A2 B cos q (b) A2 B sin q cos q making an angle 60° with the force is 10 3 N
(c) A2 B sin q (d) zero
Telegram @unacademyplusdiscounts

104 JEE Main Physics

30. For two vectors A and B, |A + B |=|A - B| is always (b) If both Assertion and Reason are true but Reason is
not correct explanation of the Assertion
true when [NCERT Exemplar]
(c) If Assertion is true but Reason is false
(a) |A | = |B| ¹ 0
(d) If Assertion is false but the Reason is true
(b) A ^ B
(c) |A | = |B| ¹ 0 and A and B are parallel or anti parallel 33. Assertion Angle between i$ + $j and $i is 45°
(d) when either |A | or |B| is zero. Reason $i + $j is equally include to both $i and $jand the
angle between $i and $j is 90°.
Comprehension Based Questions 1 $ 1 $ 1 $
34. Assertion The vector i+ j+ k is a unit
3 3 3
Passage vector.
A motor cyclist is riding north in still air at 36 kmh -1. Reason Unit vector is one which has unit magnitude
The wind starts blowing west ward with a velocity and a given direction.
18 kmh -1.
35. Assertion A physical quantity cannot be called a
31. The direction of apparent velocity is vector if, its magnitude is zero.
(a) tan–1 (1/2) West of North
(b) tan–1 (1/2) North of West Reason A vector has both, magnitude and direction.
(c) tan–1 (1/2) East of North 36. Assertion A vector A points vertically upwards and
(d) tan–1 (1/2) North of East B points towards north. The vector product A ´ B is
32. If the wind velocity becomes 36 kmh–1 due west, then along east.
how much more distance the motor cyclist would Reason The direction of A ´ B is given by right hand
cover in 10 min? rule.
(a) 2.4 km (b) 1.8 km 37. Assertion The resultant of three vectors OA and OB
(c) 3.6 km (d) 8.5 km and OC as shown in figure is R (1 + 2). Here, R is the
radius of circle.
Assertion and Reason C
B
Directions Question No. 33 and 37 are Assertion-Reason type. 45°
Each of these contains two Statements: Statement I (Assertion), 45°
Statement II (Reason). Each of these Questions also has four O A
alternative choice, only one of which is correct. You have to
select the correct choices from the codes (a), (b), (c) and (d) given
below
(a) If both Assertion and Reason are true and the Reason Reason OA + OC is along OB and (OA + OC) + OB is
is correct explanation of the Assertion
along OB.

us Years’ Questions
38. Given, C = A ´ B and D = B ´ A. What is the angle 41. A sphere is rolling without slipping on a fixed
between C and D ? [WB JEE 2009] horizontal plane surface. In the figure, A is the point
(a) 30º (b) 60º (c) 90º (d) 180º of contact, B is the centre of the sphere and C is its
topmost point. Then, [IIT JEE 2009]
39. A and B are two vectors given by A = 2 i$ + 3 $j and
C
B = $i + $j. The magnitude of the components of A and
B is [WB JEE 2009]
5 3 7 1 B
(a) (b) (c) (d)
2 2 2 2
40. A particle has an initial velocity of 3 $i + 4 $j and A
acceleration of 0.4 $i + 0.3 $j. Its speed after 10 s is (a) vC - vA = 2( vA - vC )
[AIEEE 2009] (b) vC - vB = vB - vA
(a) 7 2 units (b) 7 units (c) |vC - vA | = 2 |vB - vC |
(c) 8.5 units (d) 10 units (d) |vC - vA | = 4 vB
Telegram @unacademyplusdiscounts

Vector Analysis 105

42. A mass of 10 kg is suspended from a spring balance. E


It is pulled by a horizontal string so that it makes an 150°
angle of 60° with the vertical. The new reading of the 150°
D
balance is [Karnataka CET 2008]
(a) 10 3 kg-wt (b) 20 3 kg-wt
(c) 20 kg-wt (d) 10 kg-wt
43. The component of vector A = ax $i + a y $j + a z k$ along 120°
the direction of ( $i - $j) is [EAMCET 2008] G
C
90°
F
(a) ( ax - a y + az ) (b) ( ax + a y )
(c) ( ax - a y )/ 2 (d) ( ax - a y + az ) B
120°
120°
44. The angle subtended by the vector, A = 4 $i + 3 $j + 12 k$ A
with the x-axis is [WB JEE 2008] (a) 10 N, 11 N
-1 æ3ö -1 æ4ö (b) 10 N, 6N
(a) sin ç ÷ (b) sin ç ÷
è13 ø è13 ø (c) 10 N, 10 N
æ4ö æ3ö (d) can’t calculate due to insufficient data
(c) cos -1 ç ÷ (d) cos -1 ç ÷
è13 ø è13 ø 48. A plumb line is suspended from a ceiling of a car
45. A pendulum of length 1 m is released from q = 60° . moving with horizontal acceleration of a. What will
The rate of change of speed of the bob at q = 30°, is be the angle of inclination with vertical?
( g = 10 ms -2 ) [Orissa JEE 2003]
[Kerala CET 2007] (a) tan -1 ( a / g ) (b) tan -1 ( g / a )
(a) 10 ms–2 (b) 7.5 ms–2 (c) 5 ms–2 (d) 5 3 ms–2 (c) cos -1 ( a / g ) (d) cos -1 ( g / a )
46. A particle is displaced from a position (2 i$ - $j - k$ ) to 49. A force, F = (5 i$ + 3 $j + 2 k$ ) N is applied over a particle
another position (3 $i + 2 $j - 2 k $ ) under the action of
which displaces it from its origin to the point
$ ). The work done by the force is an
the force (2 $i + $j - k r = (2 i$ - $j) m.The work done on the particle in joule is
[AIEEE 2004]
arbitrary unit is [Kerala CET 2006]
(a) –7 (b) + 7 (c) + 10 (d) +13
(a) 8 (b) 10 (c) 12 (d) 16
47. The adjacent figure is the part of horizontal
50. If A ´ B = B ´ A, then the angle between A and B is
[AIEEE 2004]
stretched net. Section AB is stretched with a force of
(a) p (b) p /3 (c) p /2 (d) p /4
10 N. The tensions in the sections BC and BF are
[Karnataka CET 2005]

Answers
Round I
1. (c) 2. (b) 3. (a) 4. (a) 5. (b) 6. (a) 7. (c) 8. (a) 9. (b) 10. (b)
11. (b) 12. (b) 13. (a) 14. (b) 15. (c) 16. (c) 17. (b) 18. (d) 19. (c) 20. (c)
21. (a) 22. (c) 23. (b) 24. (a) 25. (c) 26. (a) 27. (d) 28. (b) 29. (b) 30. (c)
31. (b) 32. (a) 33. (c) 34. (a) 35. (c) 36. (a) 37. (a) 38. (a) 39. (a) 40. (d)
41. (a) 42. (a) 43. (b) 44. (a) 45. (a) 46. (b) 47. (a) 48. (a) 49. (d) 50. (d)
51. (b) 52. (b) 53. (c) 54. (b) 55. (c) 56. (a) 57. (d) 58. (b) 59. (c) 60. (a)
61. (a) 62. (b) 63. (a) 64. (a) 65. (c) 66. (d) 67. (a) 68. (a) 69. (c) 70. (c)
71. (d) 72. (b) 73. (c) 74. (b) 75. (d) 76. (d) 77. (a) 78. (b) 79. (a) 80. (d)

Round II
1. (c) 2. (c) 3. (c) 4. (c) 5. (d) 6. (c) 7. (d) 8. (b) 9. (b) 10. (d)
11. (d) 12. (b) 13. (d) 14. (a) 15. (b) 16. (c) 17. (d) 18. (c) 19. (a) 20. (b)
21. (d) 22. (c) 23. (b) 24. (b) 25. (b) 26. (b,d) 27. (d) 28. (a,b) 29. (a,c) 30. (b,d)
31. (a) 32. (b) 33. (a) 34. (a) 35. (d) 36. (d) 37. (a) 38. (d) 39. (a) 40. (a)
41. (c) 42. (c) 43. (c) 44. (c) 45. (c) 46. (a) 47. (c) 48. (a) 49. (b) 50. (a)
Telegram @unacademyplusdiscounts

the Guidance
Round I
1. As, R = a2 + b 2 + 2 ab cos q 10. From, A × B = AB cos q
A ×B
æ pö æ pö
2
æ pö æ pö
2 Þ cos q =
Þ R = ç ÷ + ç ÷ + 2 ç ÷ ç ÷ cos 90° AB
è2ø è2ø è2ø è2ø
( $i + $j + 2 k$ )
or cos q =
p p
Þ R = 2× = 1 12 + 12 + ( 2) 2
2 2
2 1
or cos q = = Þ q = 45°
2. Required numerical value is 32 + 42 + 52 ,i. e. , 50 or 5 2. 2 2
3. Given, C = A + B = 3 $i - $j + 7 k$ + 5$i - $j + 9 k$ 11. As, R 2 = P 2 + P 2 + 2 P 2 cos q
C = A + B = 8 $i - 2 $j + 16 k$ or R 2 = 2 P 2 + 2 P 2 cos q
Direction cosine, i. e. , angle between the A + B vector is zero, or R 2 = 2 P 2 (1 + cos q)
as the two vectors are parallel to each other. æ qö
or R 2 = 2 P 2 ç cos2 ÷
è 2ø
4. As, B + ( $i + 2$j - 3 k$ ) = $i or B = -2 $j + 3 k$
q
or R 2 = 4 P 2 cos2
5. As, resultant of two vectors is given by, 2
q
R = A2 + B2 + 2AB cos q or R = 2 P cos According to question
2
2
\ F = F12 + F22 + 2 FF
1 2 cos 90 °
12. Resultant, R = A2 + B2 + 2AB cos q
Given, q = 90°
Þ 10 A2 = 4 A2 + 2 A2 + 2 ´ 2 A ´ 2 A ´ cos q
2
or F = F12 + F22 Þ F= F12 + F22 or 4 A2 = 4 2A2 cos q
6. Here, F = Fx$i + Fy $j or F = 2 $i - 3 $j 1
or cos q = Þ q = 45°
2
7. For the resultant,
13. Clearly, (0.5) 2 + (0.8) + c2 = 1
R 2 = R 2 + R 2 + 2 R 2 cos q
0.25+0.64+ c2 = 1
or R 2 = 2 R 2 + 2 R 2 cos q
or c2 = 1 - 0.25 – 0.64 = 0.11
1
= 1 + cos q or c = 0.11
2
1 14. Note that the angle between two forces is 120° and not 60°.
or cos q = - or q = 120°
2 Þ R 2 = F 2 + F 2 cos120°
8. Let, A + B = R. Given, Ax = 7 and Ay = 6 æ 1ö
or R 2 = 2 F 2 + 2 F 2 ç - ÷ = F 2 or R = F
è 2ø
Also, Rx = 11 and Ry = 9
Therefore, Bx = Rx - Ax = 11 - 7 = 4 15. As, P + Q = 5 $i - 4$j + 3 k$
and By = Ry - Ay = 9 - 6 = 3 5 5
Þ cos a = =
Hence, magnitude of B = Bx2 + By2 2
= 4 +3 =52 2
5 + 4 +3 2 2 50
æ 5 ö
9. v or a = cos-1 ç ÷
è 50 ø
80 km/h
60 km/h 16. As, A + B = C (Given)
So, it is given that C is the resultant of A and B
\ C 2 = A2 + B2 + 2 AB cos q
30 km/h
60 2 = 30 2 + v 2 or 3 2 = 3 + 3 + 2 ´ 3 ´ cos q
or 3 = 6 cos q
or v 2 = 90 ´ 30
1
or v = 30 3 kmh -1 or cos q = Þ q = 60°
2
Telegram @unacademyplusdiscounts

Vector Analysis 107

A ×B ( $i + $j) × ( $i - $j) 1-1 22. We can make the digram as below


17. cos q = = = 0 = cos 90°
AB 1 + 1 ´ 1 + ( -1)
2 2 2 2 2

\ q = 90° R
B
2 2
18. According to question, P + Q + 2 PQ cos q = (P - Q)
β
Þ P 2 + Q 2 + 2 PQ cos q = P 2 + Q 2 - 2 PQ α
Þ 2 PQ (1 + cos q) = 0 A
B sin q
But, 2 PQ ¹ 0 Clearly, tan a =
A + B cos q
\ 1 + cos q = 0
A sin q
or cos q = -1 and tan b =
B + A cos q
or q = 180°
From the above equation, it is clear that when a < b, then
19. According to question, P + Q = 16 …(i) B < A.
Þ P 2 + Q 2 + 2 PQ cos q = 64 23. Here, v1 = 30 kms-1 due north = OA
Q sin q v 2 = 40 kms-1 due east = OB
\ tan 90° = …(ii)
P + Q cos q
Change in velocity in 20 s
Q sin q
or ¥= Dv = v 2 - v1 = v 2 + ( - v1)
P + Q cos q
= OB + OC = OD
Þ P + Q cos q = 0
| Dv| = v 22 + v12 = 40 2 + 30 2 = 50 kms–1
or Q cos q = -P ...(iii)
| Dv| 50
From Eqs. (ii) and (i), we get Acceleration, a= = = 2.5 kms–2
Dt 20
P 2 + Q 2 + 2 P ( -P) = 64
v 30 3
Þ tan b = 1 = = = 0.75 = tan 37°
or Q 2 - P 2 = 64 v 2 40 4
or (Q - P) (Q + P) = 64 ...(iv) \ b = 37° north of east
Now from Eqs. (i) and (iv), we get N
64
or Q -P = =4 …(v) A
16
v1
Adding, Eqs. (i) and (v), we get
2 Q = 20 v2 B
W E
or Q = 10 units O β
From Eq. (i), P + 10 = 16 v
or P = 6 units v
C D
20. Here, P = ( A + B),
S
Q = ( A - B)
and R = A2 + B2
24. Resultant, R = (P + Q) + (P - Q) = 2 P. Thus, angle between R
R2 - P 2 - Q 2
Þ cos q = and P is 0°.
2 PQ
25. Let, A and B be the two forces.
( A2 + B2) - ( A + B) 2 - ( A - B) 2 é A2 + B2 ù
= =-ê 2 2 ú As per question A2 + B2 = 5
2 ( A + B) ( A - B) ë 2 (A - B ) û
or A2 + B2 = 25 …(i)
é A2 + B2 ù
\ q = cos-1 ê - 2 2 ú
and 2 2
A + B + 2 AB cos120° = 13
ë 2 (A - B ) û
or 25 + 2 AB ´ ( -1 / 2) = 13
21. Clearly, 2 A - 3 B = 2 ( $i + $j - 2 k$ ) - 3 (2 $i - $j + k$ ) or AB = 25 - 13 = 12
= - 4 $i + 5 $j - 7 k$ or 2 AB = 24 …(ii)
2 2 2 Solving Eqs. (i) and (ii), we get
\Magnitude of 2A - 3B = ( -4) + (5) + ( -7)
A = 3N
= 16 + 25 + 49 = 90
and B = 4N
Telegram @unacademyplusdiscounts

108 JEE Main Physics

26. Given, ( $i + 2$j - k$ ) + ( $i - $j + 2 k$ ) + C = $j A


38. We have A = AA$ or A$ =
A
\ C = $j - ( $i - 2$j - k$ ) - ( $i - $j + 2 k$ ) $i + $j $i + $j
= - 2 $i - k$ \Required unit vector is =
| $i + $j| 2
27. A scalar quantity has the same value for observers with
different orientations of the axes.
39. The required area, A ´ B = (2$i + 3$j) ´ ( $i + 4$j)
= 8 ( $i ´ $j) + 3( $j ´ $i) = 8 k$ - 3 k$ = 5 k$
28. Given, P = $i - $j + k$ , then we have
P + Q = $i 40. As,| a ´ b| = ab sin q
Q = $i - $i + $j - k$ = $j - k$ Since, sin q cannot be greater than 1.
\| a ´ b| cannot be greater than ab.
29. As, A = 2$i - $j + 3 k$ and B = 3$i - 2$j - 2 k$ ; C = ?
R = k$ = A + B + C
41. Given,| p| = A + B
Þ | p|2 = ( A + B) 2
k$ = (2$i - $j + 3 k$ ) + (3$i - 2$j - 2 k$ ) + C
or | A + B|2 = ( A + B) 2
= 5$i - 3$j + k$ + C
or A + B + 2 AB cos q = A2 + B2 + 2 AB
2 2

\ C = -5 $i + 3 $j or cos q = 1Þ q = 0°
30. Equating the moments about R, $i $j k$
F1 ´ l1 = F2 ´ l2 42. As, angular momentum, L = r ´ p = 0 4 0
or 6 ´ PR = 4 ´ RQ 2 3 1
2
or PR = RQ = $i [ 4 - 0 ] + $j [0 - 0 ] + k$ [0 - 8] = 4 $i - 8 k$
3
31. A vector A is parallel to that of vector B, if it can be written as 43. Here, displacement,
A = mB S = (10 $i - 2$j + 7 k$ ) - (6$i + 5$j - 3 k$ ) = 4$i - 7$j + 10 k$
1 W = F × S = Force ´ displacement
Here, A = ( $i - $j + 2 k$ ) = (3$i - 3$j + 6 k)
$ Now,
3
= (10 $i - 3$j + 6 k$ ) × ( 4 $i - 7 $j + 10 k$ )
1
A= B = ( 40 + 21 + 60) J = 121 J
3
1 44. As, power,
This implies A is parallel to B and magnitude of A is times the
3
magnitude of B. F × S (2 $i + 2$j) × (2 $i + 2 k$ ) -1 4 -1
P= = Js = Js = 0.25 Js–1
t 16 16
32. The vector product, A ´ B = AB sin q
| A ´ B| = (1) (1) sin q = sin q 45. Projection of P on Q is P cos q
33. Using, $i × $j = $j × k$ = k$ × $i = 1 P
|P × Q| = (3$j + 4 k$ ) × (2$i + 5 k$ ) = 6 + 20 = 26 θ
Q
34. Since, P × Q = PQ cos q, then
PQ cos q P × Q $
P× Q = 0 Here, P cos q = = = P× Q
Q Q
Þ P ^ Q or q = 90°
|P ´ Q| = PQ sin 90° = PQ or|P|| Q| 46. From the condition given in question
35. The direction of the vector given by the cross product of the a2 + b 2 + 2 ab cos q = - a2 + b 2 - 2 ab cos q
two vectors is perpendicular to the plane containing the two or 4 ab cos q = 0
vectors, i. e. , A ´ B = ( AB sin q) A$ = C. Therefore, the angle But 4 ab ¹ 0
which A makes with C is 90°. \ cos q = 0
36. Here, P× Q = (2$i - 3$j + k$ ) × (3$i + 2$j) = 6 ( $i × $i) - 6 ( $j × $j) = 0 or q = 90°
Again, ( a + b) and (a - b) are the diagonals of a parallelogram
37. We know that, [ $i × $j = $j × k$ = k$ × $i = 0, $i × $i = $j × $j = k$ × k$ = 1] whose adjacent sides are a and b.
As, AB cos q = AB Since, | a + b| = | a - b|, therefore, the two diagonals of a
or cos q = 1 parallelogram are equal. So, think of square.
or q = 0° This leads to q = 90°
Telegram @unacademyplusdiscounts

Vector Analysis 109

47. We have A ´ B = ( 4 $i + 6$j) ´ (2 $i + 3 $j) 54. Out of the given quantities impulse and area are vector
quantities other are scalar quantities.
= 12 ( $i ´ $j) ´ 12( $j ´ $i) = 12 ( $i ´ $j) - 12 ( $i ´ $j) = 0
55. As, A × B = 0 so, A is perpendicular to B. Also A × C = 0 means A
48. Given, A × B = 0 is perpendicular to C. Since B ´ C is perpendicular to B and C.
Þ A^B (Since, A × B = AB cos q) Then clearly A parallel to B ´ C.
Now, A ´B = 1
56. Using, k$ × k$ = 1, $i × $j = 0
or AB sin q = 1
AB sin 90° = 1 or AB = 1 Power, P = F × v = (7$i + 6 k$ ) × (3$j + 4 k$ ) = 24 watt
Þ A = 1and B = 1 57. Using, A × B = AB cos q
So, A and B are perpendicular unit vectors. A ×B
Þ cos q =
$i $j k$ AB
49. Torque, t = r ´ F = 7 3 1 ( i + 2$j + 2k$ ) × $i
$ 1 1
or cos q = = = = 0.4472
-3 1 5 (12 + 2 2 + 2 2)1 / 2 9 3
Þ q = 63°12 ¢
= $i [15 - 1] + $j [ -3 - 35] + k$ [7 + 9] = 14 $i - 38$j + 16 k$
58. Here A$ × B$ = (1) (1) cos 0° = 1 ¹ AB.
50. Required area of parallelogram,
A ´ B = ( $i - 2$j + 3 k$ ) ´ (3$i - 2$j + k$ ) 59. A ^ B, if A × B = AB cos 90° = 0
= -2k$ - $j - 6( - k$ ) - 2 $i + 9 $j - 6 ( - $i) = 4 $i + 8$j + 4 k$ (2$i + a$j + k$ ) × ( 4$i - 2$j - 2 k$ ) = 0
or 8 - 2a - 2 = 0 or a = 3
Modulus is 4 2 + 8 2 + 4 2 = 32 + 64
= 96 = 4 6 units 60. Using, A × B = AB cos q, given q = 90° Þ cos 90° = 0
Then, ( A + B) × ( A - B) = 0
51. If | A| =|B| = x, then| C| = 2 x
A2 - B2 = 0 or A = B
Now, A+B= -C
or ( A + B) × ( A + B) = ( - C) × ( - C) 61. Displacement, r = ( r2 - r1) and workdone = F × r = w
or cos q = 0 or q = 90° Given F = 5$i - 4$j + 2 k$
or A × A + C× C + 2 A × C = B 2
and r = r2 - r1 = ( -5 $i + 2$j + 3 k$ ) - (2 $i + 7$j + 4 k$ )
or x + 2x2 + 2x2 2 cos q = x2
2
= -7 $i - 5$j - k$
1 \ W = F × r = (5$i - 4$j + 2 k$ ) × ( -7 $i - 5$j - k$ )
or cos q = -
2
= -35 + 20 - 2 = -17 units
Þ q = 135° dp
Again, B + C = -A 62. As, force F = = ( -2 sin t ) $i + (2 cos t ) $j
dt
or (B + C) × (B + C) = ( - A) × ( - A) F×P
Now cos q = =0
or x2 + 2x2 + 2x2 2 cos q = x2 Fp
- 2x2 1 \ q = 90°
or cos q = 2
=-
2x 2 cos q 2 (As, F = - 2 sin t$i + 2 cos t$j and P = 2 cos t$j + 2 sin t$j)
Þ q = 135°
63. As, F1 = F1 $j ; F1 ´ F2 is equal to zero only if angle between F1 and
52. Given, | A ´ B| = 3A × B
F2 is either 0° or 180°. So, F2 will be 4 $j. (In direction of y-axis)
Þ AB sin q = 3AB cos q
Using, $i ´ $i = $j ´ $j = 0
or tan q = 3
Þ q = 60° 64. As, A = 2 $i + 4$j and B = 5 $i - p $j
53. As, A × B = AB cos q \ A = 2 2 + 4 2 = 20
A × B (2$i + 3$j + 4 k$ ) × ( 4 $i + 3$j + 2 k$ ) and B = 52 + p2
Þ cos q = =
AB 4 + 9 + 16 × 16 + 9 + 4 Now, A × B = 10 - 4 p
8 + 9 + 8 25
= = If | A||B| then,
29 29
A × B = AB cos 0° = AB
-1 æ 25 ö
Þ q = cos ç ÷ 10 - 4 p = 20 25 + p 2
è 29 ø
Telegram @unacademyplusdiscounts

110 JEE Main Physics

Squaring, 100 + 16 p 2 - 80 p 73. Given, Ax = 50 and q = 60°


= 20 (25 + p 2) = 500 + 20 p 2 Then tan q = Ay / Ax
or 2 2
20 p - 16 p + 80 p + 400 = 0 or Ay = Ax tan q
or Ay = 50 tan 60° = 50 ´ 3 = 87 N
or p 2 + 20 p + 100 = 0
or ( p + 10) 2 = 0 74. As per figure, in u = a$i + b$j , both a and b are positive. In
\ p = -10 v = p$i + q$j, p is positive and q is negative.
\ B = 5 $i + 10 $j Thus a, b and p are positive and q is negative.
B = 5 2 + (10) 2 = 125 = 5 5 75. Here, W = T cos q ...(i)
65. Given, A × B = AB cos q = 6 T T cos θ
and P = T sin q ...(ii) θ
and | A ´ B| = AB sin q = 6 3 In equilibrium the horizontal and P
vertical component of forces are T sin θ
AB sin q 6 3
\ = = 3 equated.
AB cos q 6
So, P + W = T (cos q + sin q) < T W
or tan q = 3
where as (a), (b) and (c) are correct
and q = 60° and (d) is wrong.
66. Using A × B = AB cos q 76. As, Bx = 10 - 6 = 4 and By = 9 - 6 = 3
Q A = $i + $j + k$ ,
So, B = (Bx2 + By2)1/ 2 = 4 2 + 3 2
\ A = 12 + 12 + 12 = 3
A × $i 1 3 = 16 + 9 = 25 = 5
Þ cos q = = =
3 3 3 77. Motion is along the time; y = x + 4
1.732
= = 0.5773 = cos 54° 44 ¢ Differentiating it w.r.t. time, we have
3
dy dx
or q = 54° 44 ¢~- 55° = , i. e. ,v y = v x
dt dt
67. Here, PQ = r = (2 - 3) $i + ( -1- 3) $j + ( 4 + 1) k$ = -$i - 4$j + 5 k$ As, v = (v x2 + v y2)1/ 2 = 3 2
\Work done = F × r = ( 4 $i - 3$j + 3 k$ ) × ( -1 $i - 4$j + 5 k$ )
and vx = 3 = vy
= - 4 + 12 + 15 = 23 J When x = 0 , from the given equation,
v + v0 ö
68. Since, v 2 - v 02 = 2a × s = 2a × æç ÷t y =0 + 4 = 4
è 2 ø Magnitude of angular momentum of particle
or v × v - v 0 × v 0 = ( v + v 0) × a t = mvr = mvy (Q y = r)
or v × ( v - at ) = v 0 × ( v 0 + at ) = 5 ´ 3 ´ 4 = 60 units
69. We have,| A ´ B| = AB sin q. As sin q £ 1, therefore AB sin qcan 78. If r makes an angle q with x-axis, then component of r along
not be more than AB. x-axis = r cos q.
It will
70.beDisplacement,
maximum if cos =q(=a1or
rmax $=
i $+ aqj=
)0-°.( ai.e.,
i) =raisj along
positive x-axis.
F = -K(y i + x j) = -K( a$i + a$j)
$ $
Work done, W = F × r = -K ( a$i + a$j)& × a $j = -Ka 2 79. Here, A - OP = 10 units along OP
dx dy B - ( OQ) = 10 units along OQ
71. As, v x = = 2 ct and v y = = 2 bt \ ÐXOP = 30°
dt dt
\ v = v x2 + v y2 = 2t ( c2 + b2)1 / 2 and ÐXOQ = 135°
\ ÐQOX ¢ = 180° - 135° = 45°
72. The resultant of all the forces,
F1 + F2 + F3 + F4 = (5$i - 5$j + 5 k$ ) + (2 $i + 8$j + 6 k)
$
B sin 45°

Q A sin 30°
+ ( - 6 $i + 4 $j - 7 k$ ) + ( - $i - 3$j - 2k$ ) P
$
= 4 i+2k $
B
135° A
This force is in y-z plane. Therefore, particle will move in y-z 45° 30°
plane. X X
B cos 45° A cos 30°
Telegram @unacademyplusdiscounts

Vector Analysis 111

Resolving A and B into two rectangular components we have 80. From the diagram, for vertical equilibrium, T cos 30° = mg
A cos 30° along OX and A sin 30° along OY. While B cos 45°
mg
along OX ¢ and B sin 45° along OY ¢. or T= O
cos 30°
Resultant of components of forces along x-axis.
3 ´ 9.8
= ( A cos 30° - B cos 45° ) $i =
3 /2 30°
= (10 ´ 3 / 2 - 10 ´ 1 / 2) $i = 1.59 $i
= 19.6 N 30°
Resultant of components forces along y-axis
F = T sin 30°
= ( A sin 30° + B sin 45° ) $j T
1 B
æ 1 1 ö$ = 19.6 ´ F
= ç10 ´ + 10 ÷ j = 12.07 $j 2
è 2 2ø A mg
= 9.8 N

Round II
1. The vector is $i - ( A + B + C) 6. Given, x + y = 16, Also y 2 = 82 + x2
= $i - [(2$i - 4$j + 7 k$ ) + (7 $i + 2 $j - 5 k$ ) + ( -4 $i + 7$j + 3 k$ )]
= - 4 $i - 5$j - 5 k$ y

2. From figure in equilibrium position


T1 sin 30° = T2 sin 60°
1 3
or T1 ´ = T2 ´ x
2 2
or y 2 = 64 + (16 - y) 2 (Q x = 16 - y)
T1
or = 3 or 2
y = 64 + 256 + y - 32 y 2
T2
60° 30° or 32 y = 320
or y = 10 N
T1 30° 60° T2
\ x + 10 = 16 or x = 6 N
7. In equilibrium position
C
Along y-direction,
mg
2 sin 60° = 3 + F cos q
3. Let, C be a vector perpendicular to A and B. Then as per 3
question, kC = A ´ B or 2´ = 3 + F cos q
2
( A ´ B) (2$i + 3$j + 6 k$ ) ´ (3 $i - 6$j + 2 k$ ) or F cos q = 0
or k= =
C (6 $i + 2$j - 3 k$ ) As F ¹0
( 42 $i + 14 $j - 21 k$ ) \ cos q = 0 Þ q = 90°
= =7
(6$i + 2$j - 3 k$ ) Along x-direction,
mv 2 1
4. As, q ( v ´ B) = F sin 90° = 1 + 2 cos 60° = 1 + 2 ´
r 2
v2 q or F =2N
and acceleration = = ( v ´ B)
r m 8. Here, A1 = A2 = 1
= 9.6 ´ 10 7 ´ [(3 $i + 2$j) 10 5 ´ (2 $i + 3 k$ )] and A12 + A22 + 2A1A2 cos q = ( 3) 2 = 3
= 9.6 ´ 10 12 ´ (6 $i - 9 $j - 4 k)
$ ms–2
or 1 + 1 + 2 ´ 1 ´ 1 ´ cos q = 3
5. The resultant of 5 N along OC and 5 N along OA is 1
or cos q =
2
R = 62 + 62
Now, ( A1 - A 2) × (2A1 + A 2) = 2A12 - A22 - A1 A2 cos q
= 72 N along OB
1 1
The resultant of 72 N along OB and 72 N along OG is = 2 ´ 12 - 12 - 1 ´ 1 ´ =
2 2
R ¢ = 72 + 72 = 12 N along OE.
Telegram @unacademyplusdiscounts

112 JEE Main Physics

2 sin 60° 3 12. As, A = AA$ = BB$ . Let, q be the angle between A and B. As per
9. Here, tan 45° = =
a + 2 cos 60° a + 1 question.
3 ( AA$ + BB$ ) × ( AB$ + BA$ )
1= cos a =
a +1 | AA$ + BB$ || AB$ + BA$|
or a +1= 3 2AB + ( A2 + B2) cos q
or cos a =
a = 3 -1 ( A2 + B2 + 2AB cos q) 2
10. From figure, here, A = 3 $i - k$ or 2AB + ( A2 + B2) cos q = ( A2 + B2) cos a + 2AB cos q cos a
z
C
or 2AB(1 - cos a cos q) = ( A2 + B2) (cos a - cos q)
A Medium I 2 AB cos a - cos q
i1 or 2 2
=
µ2 = 2 A + B 1 - cos a cos q
x 2 AB cos a - cos q
µ2 = 3 or =
r ( A2 + B2) 1 - cos a cos q
^
n Medium II 2 AB + ( A2 + B2) (cos a - cos q) + (1 - cos a cos q)
or =
( A2 + B2) - 2 AB (1 - cos a cos q) + (cos a - cos q)
3
So, tan i1 = = 3 = tan 60° ( A + B) 2 (1 + cos a) (1 - cos q)
1 or =
( A - B) 2 (1 + cos q) (1 - cos a)
i1 = 60°
Using Snell’s law, m1 sin i1 = m 2 sin r tan 2 q/2
=
m1 2 1 tan 2 a /2
or sin r = sin i1 = sin 60° = = sin 45°
m2 3 2 a æ A -Bö q
or tan =ç ÷ tan
2 è A+B ø 2
or r = 45°
The unit vector in the direction of the refracted ray will be 13. If the three vectors are coplanar then their scalar triple product
n$ = 1 sin 45° $i - 1 cos 45° k$ is zero. So, ( A ´ C) × B = 0
1 $ $ or [(2$i + 3$j - 2k$ ) ´ ( - $i + 2 $j + 3 k$ )] × [5 $i + a$j + k$ ] = 0
= ( i - k)
2 or [(13 $i - 4 $j + 7 k$ ] × [5$i + a$j + k$ ] = 0
11. Let A, B and C be as shown in figure. Let, q be the angle of or 65 - 4 a + 7 = 0
incidence, which is also equal to the angle of reflection. or a = 18
Resolving these vectors in rectangular components, we have
14. Given, C =|B| $j Þ C = 5 $j
Y
Let, C = A + B = A + 3 $i + 4 $j
C
B 5 $j = A + 3 $i + 4$j
A θ θ Þ A = -3 $i + $j
X 2 2
10= + =| | ( ) ( )
3 1A
θ
15. Displacement = AB
angle between r1 and r2
q = 75° - 15° = 60°
A = sin q $i - cos q $j From figure,
and B = sin q $i + cos q $j AB2 = r12 + r22 - 2 r1 r2 cos q
\ B - A = 2 cos q $j = 3 2 + 4 2 - 2 ´ 3 ´ 4 cos 60° = 13
or B = A + 2 cos q $j Þ AB = 13
B sin q
Now A × C = 2 cos q $j 16. Here, tan 90° =
A + B cos q
or B = A cos q $j
or A + B cos q = 0
\ B = A - 2 ( A × C ) $j or cos q = -A /B …(i)
or B = A - 2 ( A × C) C (As, $j = C ) B
Now, R = = [ A2 + B2 + 2 AB cos q]1 / 2
2
Telegram @unacademyplusdiscounts

Vector Analysis 113

B2 22. Since displacement is along the y-direction, hence


or = A2 + B2 + 2 AB ( - A /B) = B2 - A2
4 displacement s = 10 $j.
2
A 3
or = \ Work done = F × s = ( -2 $i + 15$j + 6 k$ ) ×10 $j = 150 J
B2 4
A 3 23. Since F = 4 $i - 3 $j is lying in xy-plane, hence the vector
or =
B 2 perpendicular to F must be lying perpendicular to xy-plane,
3 i. e. , along z-axis.
From Eq. (i), cos q = - = cos150°
2 24. Displacement, r = r2 - r1
Þ q = 150° = ( $i - $j + 2 k$ ) - ( $i - $j + k$ ) = -2 $i + k$
17. The three vectors A, B and C are represented as shown in \Work done, W = F × r = ( $i + 2$j + 3 k$ ) × ( -2 $j + k$ ) = -1 J
figure (a) where A = 1, B = 2 and C = 3. Here the sides of the
equilateral triangle represent only the directions and not the 25. Angular momentum,
magnitudes of the vectors. L = r´p
Y
= (2 $i + 2 $j + k$ ) ´ (2 $i - 2$j + k$ ) = 4 $i - 8 k$
B 26. Given, A + B+ C = 0, then A, B and C are in one plane and
C
60°
60° O
are represented by the three sides of a triangle taken in one
X' X order.
60° A
B (i) \B ´ ( A + B + C ) = B ´ 0 = 0
60° 60°
C or B´ A + B´B+ B´ C=0
A
or B ´ A + 0 + B ´ C = 0 or A ´ B = B ´ C …(i)
(a) (b)
\ ( A ´ B ) ´ C = (B ´ C) ´ C; It cannot be zero.
In figure (b), these vector are drawn from a common point, O If B|| C, then B ´ C = 0, then (B ´ C) ´ C = 0
and they are lying in XY-plane. Resolving these vectors into Thus, option (a) is correct.
two rectangular components along XY-axis and Y-axis, we (ii) ( A ´ B) × C = (B ´ C) × C = 0
have, the X-component of resultant vector as
If B|| C, then B ´ C = 0, then (B ´ C) ´ C = 0
RX = | A| + |B| cos (180° - 60° ) + | C| cos (180° + 60° )
Thus, option (b) is false.
= 1 - 2 cos 60° - 3 cos 60°
(iii) ( A ´ B) = D = AB sin q D. The direction of D is
1 1 3
= 1- 2 ´ - 3 ´ = - perpendicular to the plane containing A and B.
2 2 2
(A ´ B) ´ C = D ´ C. Its direction is in the plane of A, B
Y-component of resultant vector is and C. Thus, option (c) is correct.
RY = 0 + |B| sin (180° - 60° ) + | C| sin (180° + 60° )
(iv) If C 2 = A2 + B2. then then angle between A and Bis 90°.
= 0 + 2 sin 60° - 3 sin 60° = - sin 60° = - 3 / 2
\ ( A ´ B) × C = ( AB sin 90° D) × C = AB (D × C)
Magnitude of resultant vector, = ABC cos 90° = 0.
2 2
æ 3ö æ 3ö Thus, option (d) is false.
R = RX2 + RY2 = ç - ÷ + ç - ÷
è 2ø è 2 ø 27. Here, A × B = (5 $i + 6$j + 3 k$ ) × (6 $i - 2$j - 6 k$ ) = 0
= 3 units So, A is perpendicular to B and A is not equal to A ´ B, as cross
product of two vectors in anticommunicative. The magnitude
18. Area = | A ´ B| = |( 4$i + 3$j) ´ (2$i + 4$j) = |10 k$ | = 10 units
of A is 70 and of B is 76.
19. Area of parallelogram =| A ´ B| 28. If | A + B| =| A|, then either |B| = 0 or A and B will be
1 antiparallel, where,|B| = 2 | A|
AB sin q = AB
2
1 29. (i) If, A = 3 $i + 4 $j, then| A| = 32 + 42 = 5
\ sin q = Þ q = 30°
2 (ii) W = (3$i + 4$j) × 6$j = 24 J
20. The required unit vector should be (iii) | A ´ B| = Area of parallelogram whose two adjacent sides
A ´B AA$ ´ BB$ A$ ´ B$ are represented by two A and B.
n$ = = =
AB sin q AB sin q sin q (iv) Component of force F in the direction making an angle
1
21. A × (B ´ A) = 0 According to rules for scalar triple product q = F cos q = 20 cos 60° = 20 = 10 N
2
Telegram @unacademyplusdiscounts

114 JEE Main Physics

30. When A is perpendicular to B, then \ v x = ux + axt = 3 + (0.4) ´ 10 = 7 units


v y = uy + ayt = 4 + 0.3 ´ 10 = 7 units
| A + B| = A2 + B2 + 2AB cos 90° = A2 + B2
v = v x2 + v y2
| A - B| = A2 + B2 - 2AB cos 90° = A2 + B2
When | A| = 0 or|B| = 0 then| A + B| = | A - B| \ = 7 2 + 7 2 = 7 2 units
AC 5 1 41. Let, w be the angular velocity of rotation of sphere v A , vB , vC
31. From figure, tan b = = =
OA 10 2 be the velocities at A, B and C respectively. Then, A is at rest,
N i. e. , v A = 0.
C
C A vC
β
R 10 ms–1
B B
W E vB
5 ms–1 O

A
S vC = 2wR about A
æ 1ö and v A = wR about A
So, b = tan -1 ç ÷ west of north
è2ø \ vC - vB = 2wR - wR = wR
Since, ( vC - vB) = ( vB - v A )
32. New apparent velocity, R ¢ = 10 2 + 10 2 = 10 2 ms-1
Also | vC - vB| = 2 wR - 0 = 2wR
Distance covered, | vB - v A| = wR - 2wR = - wR
s¢ = (10 2) ´ (10 ´ 60) m \ | vC - v A| = 2| vB - vC|
= 6 2 km = 8.5 km
42. As shown at position A, the reading of balance will be the
Extra distance covered = 8.5 – 6.7 = 1.8 km tension, T in the spring. As, the spring balance is in
( $i + $j) × $i 1 equilibrium position at A
33. As, cos q = = cos 45°
$ $
| i + j|| i| $ 2
So, q = 45° 60°

1 $ 1 $ 1 $ T
34. Here, A = i+ j+ k F
3 3 3
1/ 2 F
é æ 1 ö 2 æ 1 ö 2 æ 1 ö 2ù
A = êç ÷ +ç ÷ +ç ÷ ú =1 kg-wt
êë è 3 ø è 3ø è 3ø ú
û
Hence, T cos 60° = 10 kg-wt
36. In assertion, the direction of ( A ´ B) according to right hand 10
rule is towards West. T=
cos 60°
37. As, OA = OC 10
or = = 20 kg-wt
OA + OC is along OB (bisector) and its magnitude is (1/2)
2R cos 45° = R 2 43. Here, A = ( ax$i + ay $j + azk$ )
( OA + OC) + OB is along OB and its magnitudes is
Let, B = ( $i - $j)
R 2 + R = R (1 + 2)
B $i - $j $i - $j
Then, B$ = = =
38. Since, ( A ´ B) = -(B ´ A), so C = D B (1) 2 + ( -1) 2 2
i. e. , C and D are antiparallel to each other.
Component of A along the direction of B is
39. Magnitude of component of A and B ( $i - $j) ( ax - ay )
A × B = ( ax$i + ay $j + azk$ ) × =
A × B (2$i + 3$j) 5 2 2
= = =
|B| 2
1 +1 2 2
44. Let, q be the angle which A make with x-axis. Then
40. Here, ux = 3 units and uy = 4 units; Ax 4 4
cos q = = =
A 4 2 + 3 2 + 12 2 13
a = ax$i + ay $j = 0.4 $i + 0.3 $j
æ4ö
So, ax = 0.4 units and ay = 0.33 units \ q = cos-1 ç ÷
è13 ø
Telegram @unacademyplusdiscounts

Vector Analysis 115

45. When q = 30°, the restoring force on the bob is Also, T1 cos 60° + T2 cos 60° = T = 10 N
O or 2 T1 cos 60° = 10
10
or T1 =
2 cos 60°
30°
10
= = 10 N
T 1

F 2
B So, T1 = T2 = 10 N
θ 48. Here, horizontal force on plumb line = ma and vertical
sin
30°

g downward force on plumb line = mg.


m mg mg cos θ
If q is the angle with vertical through which plumb line is
F = mg sin 30° inclined, then
\ Rate of change of speed at this instant
ma a
F 1 tan q = =
a = = g sin 30° = 10 ´ = 5 ms-2 mg g
m 2
æ aö
46. Work done, W = F × ( r2 - r1) or q = tan -1 ç ÷
ègø
= (2$i + $j - k$ ) × [(3 $i + 2 $j - 2 k$ ) - (2$i - $j + k$ )]
49. The work done, W = F × r = (5$i + 3$j + 2 k$ ) × (2$i - $j) = 7
= (2$i + $j - k$ ) × ( $i + 3$j - 3 k)
$
[Using $i × $i = $j × $j = k$ × k$ = 1]
= 2 + 3 + 3 = 8 units of work
47. From the diagram, T1 sin 60° = T2 sin 60° or T1 = T2 50. As, A ´ B = B ´ A or ( A ´ B) - (B ´ A) = 0
or ( A ´ B) + ( A ´ B) = 0
C T2cos 60° F or 2 ( A ´ B) = 0
T1cos 60°
T1 T2 or 2 AB sin qn$ = 0
60° 60° As, A¹0
nor B¹0
T1sin 60° B T2sin 60° So, sin q = 0
Hence, q = 0° or p
T
A
Telegram @unacademyplusdiscounts

4 Projectile Motion
JEE Main MILESTONE
< Equation of Trajectory < Some Applications of General Equations
< Important Terms in Projectile Motion of Projectile Motion
< Projectile Motion on an Inclined Plane

4.1 Equation of Trajectory


Let us consider a projectile launched so that its initial velocity u makes an angle q
with the horizontal. Horizontal direction is taken as x-axis and vertical direction as
the y-axis.
u = u $i + u $j
x y

Þ u = u cos q $i + u sin q $j A particle when given a velocity at


y an arbitrary angle made with the
horizontal surface is known as a
projectile, the path followed by
u the particle (called trajectory) is a
H parabola and the motion of the
θ particle is called projectile motion.
x
ux = u cos θ Projectile motion is a
R
two-dimensional motion.
The x-axis is parallel to the horizontal, y-axis is parallel to the vertical and u lies in
the plane x-y. The constant acceleration a is given as
a = a $i + a $j
where, ax = 0
(Since, there is not acceleration along x-axis)
ay = - g
(Since, the acceleration is downwards and equal to g)
Now, velocity after time t is given as
vt x = u x + a xt = u cos q (as a x = 0)
vt y = uy + ayt = u sin q - gt
\ v = v $i + v $j
t tx ty

Þ vt = v cos q$i + (u sin q - gt ) $j


Telegram @unacademyplusdiscounts

Projectile Motion 117

The direction of v with the x-axis is given by


4.2 Important Terms in
-1 æ vy
ö
q = tan ç ÷
è vx ø Projectile Motion
Coordinates of the projectile after time t is given by Let us now make ourselves familiar with certain
Þ x = u cos q t …(i) important terms used in projectile motion.
y
1
Þ y = u sin q t - gt 2 …(ii) g
2
u A
From Eqs. (i) and (ii), eliminating t, we get
x 1 x2 H
y = u sin q - g 2 θ
u cos q 2 u cos2 q x
O C B
2
gx R
Þ y = x tan q - …(iii)
2u cos2 q
2
Let a particle projected from the point O with an initial
The equation between x and y represents the path of the velocity u at an angle q with the horizontal and it hits the
projectile known as trajectory. The Eq. (iii) shows that it is ground at same level at point B. The point O is known as
the equation of parabola of the form the point of projection, the angle q is called the angle of
y = bx + cx 2 projection.
where, b = tan q = constant and The distance OB is called the horizontal range (R) or simply
g range and the vertical height AC is known as the maximum
c= = constant.
2u2 cos2 q height (H). The total time taken by the particle in describing
the path OAB is called the time of flight (T).
Note Projectile motion is a two-dimensional motion with constant
acceleration ( g ). So, we can use
Time of Flight (T )
v = u + at
1 As the motion from the point O to A and then from
s = ut + at 2 etc. the point A to B are symmetrical, the time of ascent
2
In projectile motion as well. Here,
(for journey from the point O to A) and the time of descent
(for the return journey from the point A to B) will be equal
u = u cos q$i + u sin q $j
to T/2. Further on reaching the highest point A, the vertical
and a = -g $j component of the velocity of the projectile must become
Now, suppose we want to find velocity at time t. zero i. e. , vy = 0
v = u + at T
So, uy = u sin q ; ay = - g ;t = and vy = 0,
= (u cos q$i + u sin q $j ) - gt $j 2
From equation of motion,
or v = u cos q$i + (u sin q - gt ) $j
v = u + gt
y
Substituting the valves, we get
u T
0 = u sin q + (- g )
2
2 u sin q
T =
θ
g
O x
g
Sample Problem 1 A very broad elevator is going up
vertically with a constant acceleration of 2 ms-2. At the instant,
Similarly, displacement at time t will be,
1 when its velocity is 4 ms-1 a ball is projected from the floor of
s = ut + at 2
2 the lift with a speed of 4 ms-1 relative to the floor at an elevation
= (u cos q$i + u sin q $j ) t -
1 2$
gt j of 30º. If g = 10 ms-1, then what is the time taken by the ball to
2 return to the floor?
= ut cos q$i + æçut sin q - gt 2 ö÷ $j 1 1 1
1
è ø (a) s (b) s (c) s (d) 1s
2 2 3 4
Telegram @unacademyplusdiscounts

118 JEE Main Physics

Interpret (b) Here, u = 4 ms–1, q = 30° u2 sin 2 q u2 sin2 q


Then, =n
g 2g
Acceleration of the ball relative to the lift = 10 + 2 = 12 ms–2 acting
4 æ 4ö
in the negative y-direction or vertically downwards. It means, here i. e. , tan q = or q = tan-1 ç ÷
g ¢ = 12 ms–2. n è nø

2 u sin q 2 ´ 4 ´ sin 30° 1


Time of flight, T = = = s Note If air resists the projectile motion, then
g¢ 12 3
(i) Time taken by projectile during upward motion < Time taken
during downward motion.
Horizontal Range (R) (ii) The values of height attained and of range of a projectile decrease.
y
Distance OB is known as horizontal range or simply range
I
of the projectile. u
II
The horizontal range R is the horizontal distance covered
by the projectile with uniform velocity u cos q in a time
equal to the time of flight. Therefore, θ1 θ θ0
x
R = u cos q ´ T O θ > θ1
2 u sin q u2 sin 2 q (iii) The projectile returns to the ground with less speed. At its
= u cos q ´ Þ R=
g g trajectory, its horizontal velocity also decreases.
(iv) Time of flight also decreases.
Range will be maximum, if
(v) The angle which the projectile makes with the ground, increases.
sin2 q = 1
sin 2 q = sin 90° Sample Problem 2 An object is thrown along a direction
2 q = 90° or q = 45° inclined at an angle of 45° with the horizontal direction. The
u2 horizontal range of the particle is
Rmax = (at q = 45°)
g (a) four times the vertical height
(b) thrice the vertical height
A projectile has the same range at angles of projection q
(c) twice the vertical height
and (90° - q), though time of flight, maximum height and
(d) equal to vertical height
trajectories are different. This is also true for a range of
projectile for q1 = (45° - a ) and q2 = (45° + a ) and is equal to Interpret (a) We know that,
u2 cos 2 a u 2 sin 2 q
. Range, R=
g g
Here, q = 45°
Maximum Height (H) \ R=
u2
× Hmax =
u 2 sin 2 45°
=
u2
At the maximum height at point A, the vertical component g 2g 4g
of velocity is zero. So, clearly, R = 4 Hmax
y
vy = 0
Sample Problem 3 Two stones having different masses m1
A
u ux and m2 are projected at angles q and (90° – q) with same
velocity from the same point. The ratio of their maximum
H heights is
θ
x (a) 1 : 1 (b) 1: tan q (c) tan q : 1 (d) tan 2 q : 1
O
u 2 sin 2 q
Q vy2 = uy2 - 2 gh Interpret (d) Maximum height, H1 = ...(i)
2g
(0) 2 = (u sin q) 2 - 2 gH u 2 sin 2(90° - q)
and H2 = ...(ii)
u2 sin2 q 2g
H=
2g From Eqs. (i) and (ii), we get
H1 sin 2 q
In case of projectile motion, range R is n times the, = 2
H2 sin (90° - q)
maximum height H.
i. e. , R = nH H1 tan 2 q
= \ H1 : H2 = tan 2 q : 1
H2 1
Telegram @unacademyplusdiscounts

Projectile Motion 119

Important Points Regarding Projectile Motion


1. We know that in principle a x = 0 i.e., motion of 6. There are two unique times at which the projectile is
projectile in horizontal direction is uniform. Hence, at the same height h (< H ) and the sum of these two
horizontal component of velocity u cos q does not times equals the time of flight T. Since,
change during its motion. 1 2
h = (u sin q) t - gt is quadratic in time, so it has
y 2
2. Motion invertical
two unique roots t1 and t2 (say) such that sum of
direction is first
retarded, then u A 2 u sin q 2h
y roots (t1 + t2 ) is and product (t1t2 ) is . The
accelerated in u g g
opposite direction. θ time lapse (t1 - t2 ) between these two events is
Because uy is upwards O ux B x
(t1 - t2 ) 2 = (t1 + t2 ) 2 - 4 t1t2
and ay is downwards.
Hence, vertical component of its velocity first 4u2 sin 2 q 8h
t1 - t2 = -
decreases from O to A and then increases from A to B. g2 g
3. The coordinates and velocity components of the
7. If K ¢ is the kinetic energy at the point of launch then
projectile at time t are
kinetic energy at the highest point is
x = s x = uxt = (u cos q)t 1 1
1 1 K ¢ = mv2x = mu2 cos2 q
y = sy = uyt + ayt 2 = (u sin q)t - gt 2 2 2
2 2
Þ K ¢ = K cos2 q
v x = ux = u cosq
8. For complementary angles f and 90° - f, if Tf and
and vy = uy + ayt = u sin q - gt
T90° -f are the times of flight and R is the range, then
Therefore, speed of projectile at time t is v = v2x + vy2
2 Rf 2 R90°- f 2R
and the angle made by its velocity vector with positive Tf T90°- f = = =
x-axis is g g g
æv ö 2 R89°
q = tan -1 ç y ÷ e. g ., T1° T89°
2R
= 1° =
èvx ø g g
4. Equation of trajectory of projectile 9. The velocity of the projectile is minimum at the
x highest point (= u cos q) and is maximum at the point
x = (u cos q)t \ t =
u cosq of projection or at the point of striking the ground.
1 2
Substituting this value of t in, y = (u sin q)t - gt , we 10. At the maximum point of projectile motion, the
2
get velocity is not zero, but is ucos q acting in the
gx2 horizontal direction.
y = x tan q -
2u2 cos2 q
11. The angle between velocity and acceleration varies
gx2 2 from 0° < q < 180°.
= x tan q - sec q
2u2 12. Path of a projectile w.r.t. other projectile is a straight
gx2 2 line.
= x tan q - 2
(1 + tan q)
2u 13. In oblique projection of a projectile, the following
These are the standard equations of trajectory of a physical quantities remains constant during motion.
projectile. (i) horizontal component of velocity (u cos q)
The equation is quadratic in x. This is why the path of a (ii) acceleration due to gravity ( g)
projectile is a parabola. The above equation can also be (iii) total energy of the projectile.
written in terms of range (R) of projectile as
The following physical quantities which change during
x
y = x æç1 - ö÷ tan q the motion are
è Rø
(i) speed and velocity
5. Range R is given by,
2 (ii) direction of motion
R= (u cos q) (u sin q)
g (iii) linear momentum
2 æ Horizontal component ö æ Vertical component ö (iv) kinetic energy and potential energy
= ç ÷ç ÷
g è of initial velocity ø è of initial velocity ø (v) angle between direction of motion and acceleration
due to gravity. (It changes from 90° + q to 90° - q).
Telegram @unacademyplusdiscounts

120 JEE Main Physics

Sample Problem 4 A projectile is fixed at an angle q with Sample Problem 6 A biker stands on the edge of a cliff
the horizontal, (as shown in the figure), condition under which 490 m above the ground and throws a stone horizontally with
it lands perpendicular on an inclined plane of inclination a is an initial speed of 15 ms -1. Neglecting air resistance, the speed
with which the stone hits the ground is
(a) 10 ms-1 (b) 99 ms-1
θ
(c) 15 ms-1 (d) 30 ms-1
α
Interpret (b) We choose the origin of the x-axis and y-axis at the
(a) sin a = cos ( q - a) edge of the cliff and t = 0 s at the instant the stone is thrown. Choose
(b) cos a = sin ( q - a) the positive direction of x-axis to be along the initial velocity and
(c) tan a = cot ( q - a) the positive direction of y-axis to be the vertically upward
(d) cot ( q - a) = sin a direction. The x and y- components of the motion can be treated
independently. The equations of motion are
Interpret (c) From the equation of projectile, we have
1 x = x0 + uxt
y = 0 = u sin ( q - a) t - gt cos at 2 1 2
2 y = y 0 + uyt + ayt
2 u sin ( q - a) 2
or t= Here, x0 = y 0 = 0 ,uy = 0 , ay = - g = - 9.8 ms–2,
g cos a
The projectile lands perpendicular if its horizontal velocity will ux = 15 ms-1
vanish, that is, v x = 0. The stone hits the ground when,
[2u sin ( q - a)]
0 = u cos ( q - a) - g sin a y = -490 m
g cos a
1
Þ tan a = cot ( q - a) - 490 = - (9.8) t 2
2
Sample Problem 5 A particle moves according to the Þ t = 10 s
equation The velocity components are un = u0x and uy = u0y - gt
v = a$i + b$j so that, when the stone hits the ground
At the initial point, x = 0 = y, the radius of the trajectory is ux = 15 ms-1
bx2 2a
(a) (b) 2 uy = 0 - 9.8 ´ 10 = -98 ms–1
2a bx
x2 a \The speed of the stone is
(c) (d)
a x2 ux2 + uy2 = 15 2 + 98 2 = 99 ms-1
Interpret (a) Distance = speed ´ time
Given, v = a$i + b$j Sample Problem 7 A cricket ball is thrown at a speed of
At the initial point, 28 ms -1 in a direction 30° above the horizontal. The distance
dy from the thrower to the point where the ball returns to the same
x = at , = bx
dx level is [NCERT]
x (a) 10 m (b) 20 m (c) 69 m (d) 82 m
Þ t=
a
dx Interpret (c) The distance from the thrower to the point where
or dt = the ball returns to the same level is given by range
a
dx u02 sin 2 q
Þ dy = bxdt = bx R=
a g
xn + 1 Given, u0 = 28 ms–1, 2 q = 60°, g = 9.8 ms–2
On integrating the above expression, using ò xndx =
n +1
28 ´ 28 ´ sin 60°
bx2 \ R=
Thus, y= . 9.8
2a
= 69 m
Telegram @unacademyplusdiscounts

Projectile Motion 121

(c) Let at time t, the coordinates of the position of


projectile are ( x, y), then
Check Point 1 1
x = ut and y = 0 - gt 2
1. A person sitting in a moving train throws a ball vertically 2
upwards. How does the ball appear to move to an observer Therefore, at time t, position vector
outside the train? 1
r = x$i + y$j = ut$i - gt 2$j
2. While firing one has to aim a little above the target and not 2
exactly on the target. Why? 2
1
3. In long jumping, does it matter how high you jump? What |r | = x 2 + y2 = (ut )2 + æç - gt 2 ö÷
è 2 ø
factors determine the span of jump?
and tan q = y / x
4. A man can jump on moon six times as high as on earth. Why?
(d) Let at time t, the horizontal and vertical velocities of
5. An object is dropped through the window of a fast running
projectile be v x and vy .
bus. Then,
(a) the object moves straight horizontally Hence, vx = u
(b) the object falls down vertically and vy = 0 + (- gt )t = - gt
(c) the object follows an elliptical path r = v $i + v $j = u$i + (- gt )$j
x y

(d) to a man standing near the track, path of object will and v= v2x + vy2 = u2 + (- gt )2
appear to be part of parabola. vy
and tan q =
vx
Case II
4.3 Some Applications of General Projectile Projected at an Angle q above Horizontal
u x = u cos q, a x = 0
Equations of Projectile Motion
uy = u sin q, ay = - g
Projection from a Height From equation of horizontal motion,
x = u cos qt …(i)
Case I Equation of vertical motion
Projectile Projected in Horizontal Direction Let a projectile 1 2
- h = u sin qt - gt ...(ii)
be projected with a velocity u. Observation is being taken 2
from point O at a height h from ground.
vb = 0 vx = ux = u cos θ
u
+ve A
uy = u sin θ

O u +ve O θ B u cos θ ay = –g
θ u cos θ θ
r θ y (x, y)
ay =– g u sin θ
x
vx h
h θ

vy v
P D C
P B
From Eqs. (i) and (ii),
Here, u x = u,
gt 2 - 2u sin qt - 2 h = 0 …(iii)
uy = 0, a x = 0
u sin q u2 sin2 q 2 h
ay = - g or t= ± ´
g g2 g
(a) Horizontal motion, x = ut …(i)
1 Horizontal distance covered in time T
Vertical motion, - h = 0 (t ) - gt 2 …(ii)
2 PC = (u cos q) T
From Eqs. (i) and (ii), we get and horizontal distance covered during this time
2h u2 sin 2 q
t= OB =
g g
(b) Horizontal range, (R) = u ´ t = u 2h / g In such case for range PC to become maximum, q should
be 45°.
Telegram @unacademyplusdiscounts

122 JEE Main Physics

Case III From equation of motion,


Projection at an Angle q below Horizontal 1 2
s = ut + at
u x = u cos q, a x = 0 2
uy = - u sin q, ay = - g 1
- h = (-u sin q) t + (- g ) t 2
y 2
or gt 2 + (2u sin q) t - 2 h = 0
uy = u sin θ

O ux cos θ
θ On solving this equation, value of t can be obtained
ay = –g
u - 2u sin q 4 u2 sin2 q + 8 gh
t= ±
h 2g 2g
Neglect –ve root of t.
x In this time the horizontal distance covered on the earth
P A
PA = (u cos q) t

Hot Spot Projectile Motion


on an Inclined Plane
In case the projection is from an inclined plane, we consider two axes x and y along and perpendicular to the inclined
plane. Different cases of projectile motion on an inclined plane are shown below.
Case I Motion up the Plane In xy-plane, i. e., sin (2q - b ) = 1
y x u2 [1 - sin b ]
Þ Rmax =
g [1 - sin2 b ]
u2
β Þ Rmax = up the plane
sin g [1 + sin b ]
g
R
Vertical Height at which Projectile
x'

g sβ Strikes From equation,


θ co 1 2
β g
y = u sin qt - gt
2
2
ux = u cos ( q - b ) 2 u sin ( q - b ) 1 æ 2 u sin ( q - b ) ö
= (u sin q) - gç ÷
u y = u sin ( q - b ) g cos b 2 è g cos b ø
ax = - g sin,b 2 u2 cos q sin b sin ( q - b )
=
a y = - g cos b g cos2 b
1
Since, y = u sin ( q - b ) t - g cos bt2 Case II Motion down the Plane Let the particle be thrown with a
2 velocity u at an angle q with the horizontal as shown in the figure.
At t = T , y = 0 where, T = time of flight
y
2 u sin ( q - b )
Þ T =
g cos b
Again x = (u cos q) T θ
β
2 u sin ( q - b )
x = u cos q
g cos b β
sin
So, range along inclined plane ( R) = x /cos b g R
β β
2 u2 cos q sin ( q - b ) s
\ R= co
g cos2 b β g

[use formula 2cos A sin B = sin ( A + B) - sin ( A - B)] 1


u sin ( q + b ) T - g cos bT2 = 0 [for y = 0]
2
u [sin (2q - b ) - sin b ] 2
R= 2 u sin ( q + b )
g cos2 b Þ T =
g cos b
Now, R will be maximum, when sin (2q - b ) is maximum,
Telegram @unacademyplusdiscounts

Projectile Motion 123

u2 é sin (2q + b ) + sin b ù Interpret (a) Take X, Y-axes as shown in figure below. Suppose
R=
g êë 1 - sin2 b
ú
û
that the particle strikes the plane at a point P with coordinates ( x, y).
Consider the motion between A and P.
Since, q is variable and the maximum value of sine function u
A X
is 1.
For R to be maximum,
y
sin (2q + b ) = 1
P
u2 æ 1 + sin b ö u2 x
and Rmax = ç ÷ = down the plane. θ
g è1 - sin2 b ø g (1 - sin b )
Y

Sample Problem 8 A cannon fired from under a shelter Motion in x- direction


inclined at an angle a to the horizontal. The cannon is at point Initial velocity = u
A distant L from the base (B) of the shelter. The initial velocity of Acceleration = 0
the cannon is v0 and its trajectory lies in the plane. The x = ut …(i)
maximum range Rmax of the shell is Motion in y- direction
v 02
(a) sin 2a Initial velocity = 0
g
Acceleration = g
g
(b) 2 sin 2 ( f - a) 1
v0 y = gt 2 …(ii)
2
v 02 æ gR sin 2a ö Eliminating t from Eqs. (i) and (ii), we get
(c) sin 2 ç a + sin -1 ÷
g è v0 ø
1 x2
y= g
v2 æ R sin 2a ö 2 u2
(d) 0 sin 2 ç a + sin -1 ÷
2g è g ø Also, y = x tan q
Interpret (c) For h¢ to be maximum, g x2 2u 2 tan q
Thus, 2
= x tan q giving x = 0 or
v 02 sin 2 ( f - a) 2u g
h ¢ = R sin a =
2 g cos a 2 u 2 tan q
Clearly, the point P corresponds to x = , then
gR sin 2 a = v 02 sin2 ( f - a) g

gR sin 2a 2u 2 tan 2 q
Þ f = a + sin -1 y = x tan q -
g g
The distance, AP = l = x2 + y 2
2u 2
= tan q 1 + tan 2 q
h′ g
α 2u 2
h′ φ– = tan q sec q
α α g
B
R A
v 2 sin 2f Sample Problem 10 Two bodies are projected from the
Range, Rmax = 0 same point with equal speeds in such directions that they both
g
strike the same point on a plane whose inclination is a. If q be
v20 æ gR sin 2a ö the angle of projection of the first body with the horizontal,
= sin 2 ç a + sin -1 ÷
g è v0 ø then the ratio of their time of flight is
cos ( q - a)
(a)
Sample Problem 9 A particle is projected horizontally cos b
with a speed u from the top of a plane inclined at an angle q sin ( q - a)
(b)
with the horizontal. How far from the point of projection will cos q
the particle strike the plane? sin ( q - a)
2 u2 2u (c)
(a) tan q sec q (b) tan 2 q sec q sin b
g g cos ( q - a)
2 u2 2u (d)
(c) tan q cos q (d) tan q cos2 q sin q
g g
Telegram @unacademyplusdiscounts

124 JEE Main Physics

u2 T sin ( q - a)
Interpret (b) Range R = [sin (2q - a) - sin a ] =
g cos2 q T ¢ sin ( q¢ - a)
sin ( q - a)
=
ì p ü
u sin í - ( q - a) - aý
î2 þ
u sin ( q - a) sin ( q - a)
= =
æp ö cos q
sin ç - q÷
è2 ø
θ
α Note If two particles are projected at angles q1 and q2 respectively as
shown in figure, then the relative motion of 1 with respect to 2 is a straight
Range of both the bodies is same.
line at an angle.
Therefore, y y
sin (2q - a) = sin (2q¢ - a)
u1 u2
2 q¢ - a = p - (2 q - a)
p
q¢ = - ( q - a)
2
2u sin ( q - a) θ1 θ2
Now, T= ...(i) x x
g cos a
æu ö
2u sin ( q¢ - a) a = tan-1 ç 12 y ÷ with positive x-axis.
and T¢ = ...(ii) è u12x ø
g cos a
where, u12x = u1x - u 2x = u1 cos q1 - u 2 cos q2
Dividing Eq. (i) by Eq. (ii), we get
u12 y = u1y - u 2 y = u1 sin q1 = u 2 sin q2
Telegram @unacademyplusdiscounts

WORKED OUT
Examples
Example 1 An aeroplane in a level flight at 144 kmh -1 is at (a) 25 m (b) 50 m
an altitude of 1000 m. How far from a given target should a (c) 100 m (d) 77.6 m
body be released to hit the target? u2
(a) 571.43 m (b) 671.43 m
Solution Range R1 = sin 2 ´ 15° = 50
g
(c) 371.43 m (d) 471.43 m
u2 50 50
144 ´ 1000 or = = = 100
Solution Horizontal velocity, u = = 40 ms-1 g sin 30° 1/ 2
60 ´ 60
u2 u2
Time of flight, T = 2h/g = 2 ´ 1000 / 9.8 Horizontal range R2 = sin 2 ´ 45° = = 100 m
g g
Horizontal range = ut 40 2 ´ 1000 / 9.8 = 571.43 m
Example 5 A projectile is projected at an angle of 15° to the
Example 2 A ball thrown by one player reaches the other in horizontal with some speed v. If another projectile is projected
2s. The maximum height attained by the ball above the point of with the same speed, then it must be projected at an angle with
projection will be ( g = 10 ms-2) the horizontal so as to have the same range.
(a) 12.5° (b) 75°
(a) 10 m (b) 7.5 m
(c) 65° (d) It is never possible
(c) 5 m (d) 25 m
2u sin q u sin q Solution We know for same horizontal range q1 + q2 = 90°
Solution Given, T = = 2 or =1
g g q2 = 90° - q1 = 90° - 15° = 75°
2 2 2 2
u sin q g æ u sin q ö 10
Maximum height H = = ç ÷= ´ 12 = 5 m
2g 2 è g2 ø 2 Example 6 A body is projected at an angle of 30° to the
horizontal with kinetic energy E. The kinetic energy at the top
most point is
Example 3 Two stones are projected with the same velocity
(a) 3E/4 (b) E/4
but making different angles with the horizontal. Their ranges
(c) 0 (d) E/2
are equal. If angle of projection of one is 30° and its maximum
height is y, then the maximum height of other will be 1
Solution Initial KE = E =mv 2
(a) 3y (b) 2y 2
(c) y/2 (d) y/3 1 1 3 3
KE at the highest point = m (u cos 30° ) 2 = mu 2 ´ = E
2 2 4 4
Solution As horizontal range of the two stones is same. So the
sum of angles of projection of two stones must be 90° Example 7 A man can throw a stone to a maximum distance
30° + q = 90° or q = 60° of 80 m. The maximum height to which it will rise in metre is
u 2 sin 2 30° u 2(1 / 2) 2 (a) 30 m (b) 20 m
According to question, y = =
2g 2g (c) 10 m (d) 40 m
u 2 sin 2 36° u 2(3 3/ 2) 2
And y¢ = = Solution Maximum horizontal range = 80 m
2g 2g
y¢ When q = 45°
\ = 3 or y ¢ = 3y u2
y i. e. , = 80 m
g
Example 4 The range of projectile fired at an angle of 15° is u 2 sin 2 q 80
Maximum height = h = = (sin 45° ) 2 = 20 m
50 m. If it is fired with the same speed at an angle of 45°, its 2g 2
range will be
Telegram @unacademyplusdiscounts

126 JEE Main Physics

Example 8 When the angle of projection is 75°, a ball falls Þ u sin q = gt


10 m shorter of the target. When the angle of projection is 45°, 1 2
and h = u sin q t - gt ...(ii)
it falls 10 m ahead of the target. Both are projected from the 2
same point with the same speed in the same direction, the Using Eqs. (i) and (ii) in Eq. (iii),
distance of the target from the point of projection is 1
(u cos q ) (t ) = (u sin q ) (t ) - (u sin q )t
(a) 15 m (b) 30 m (c) 45 m (d) 10 m 2
2
u sin 2 ´ 75° tan q = 2
Solution If range is R then, = R - 10 ...(i)
g
Example 10 A marble starts falling from rest on a smooth
u 2 sin 2 ´ 45°
and = R + 10 inclined plane of inclination a. After covering a distance h, the
g ball rebounds off the plane. The distance from the impact point
u2 where the ball rebounds for the second time is
or = R + 10
g
From Eq. (i), (R + 10) sin 150° = R - 10 h
1
or (R + 10) = R - 10 or R = 30 m
2 α

Example 9 A heavy particle is projected from a point at the (a) 8h cos a (b) 8h sin a
foot of a flying plane, inclined at an angle 45° to the horizontal, (c) 2h tan a (d) 4h sin a
in the vertical plane containing the line of greatest slope
through the point. If q ( > 45° ) is the inclination with the Solution Velocity before strike u = 2gh
horizontal of the initial direction of projection, for what value
Component of acceleration along the inclined plane = g sin a and
of tan f will the particle strike the plane?
the perpendicular component = g cos a
(a) tan q = 1 (b) tan q = 2
1
1 1 Using s = ut + at 2,
(c) sin q = (d) sin q = 2
2 2
For vertical direction, we get
Solution Let the particle be projected from O with velocity u 1
0 = v cos at - g cos at 2
and strikes the plane at a point P after time t. 2
Let ON = PN = h, then OP = h 2 and for horizontal direction,
If the particle strikes the plane horizontally, then its vertical 1
x = u sin at + g sin at 2
component of velocity at P is zero. 2
h = (u cos q) (t ) ...(i) 2
2u 1 æ 2u ö æ 2u ö
Along vertical direction 0 = u sin q - gt = u sin a + g sin a ç ÷ çQ t = ÷
g 2 èg ø è g ø
P
u cos α 2u 2 sin a 2u 2 sin a
= +
g g
h 4 u 2 sin a
u =
g
θ 2gh ´ sin a
45° =4´ = 8h sin a
O N g
Telegram @unacademyplusdiscounts

Start Practice for


JEE Main
Round I (Topically Divided Problems)

Horizontal Projection 6. Water is flowing from a horizontal pipe fixed at a


height of 2 m from the ground. If it falls at a
1. The horizontal range of a projectile fired at an angle of
horizontal distance of 3 m, as shown in figure, the
15° is 50 m. If it is fired with the same speed at an
speed of water when it leaves the pipe is
angle of 45°, its range will be [NCERT Exemplar]
( Take, g = 9.8 ms –2 )
(a) 60 m (b) 71 m (c) 100 m (d) 141 m
2. A tennis ball rolls off the top of a sister case way with
a horizontal velocity u ms -1. If the steps are b metre 2m
wide and h meter high, the ball will hit the edge of the
nth step, if 3m
2hu 2hu2 (a) 2.4 ms -1 (b) 4.7 ms -1 (c) 7.4 ms -1 (d) 6.2 ms -1
(a) n = 2 (b) n =
gb gb2 7. A stone is just released from the window of a moving
2hu2 hu2 train along a horizontal straight track. The stone will
(c) n = (d) n = 2 hit the ground following
gb gb
(a) straight path (b) circular path
3. A bomber plane moves horizontally with a speed of (c) parabolic path (d) hyperbolic path
500 ms -1 and a bomb releases from it, strikes the 8. A man standing on a hill top projects a stone
ground in 10 s. Angle at which it strikes the ground horizontally with speed v0 as shown in figure. Taking
will be ( g = 10 ms -2 ) the coordinate system as given in the figure. The
æ 1ö æ 1ö
(a) tan -1 ç ÷ (b) tan ç ÷ coordinates of the point where the stone will hit the
è5ø è5ø
hill surface
(c) tan -1(1) (d) tan -1(5 ) æ 2v2 tan q - 2v20 tan2 q ö
(a) ç 0 , ÷ y
4. An aeroplane is flying in a O è g g ø v0
x
(0, 0)
horizontal direction with a æ 2v20 2v20 2
tan q ö
(b) ç , ÷
velocity 600 kmh -1 at a è g g ø
height of 1960 m. When it is h æ 2v2 tan q 2v20 ö
vertically above the point A (c) ç 0 , ÷ θ
è g g ø
on the ground, a body is
dropped from it. The body A B æ 2v2 tan2 q 2v20 tan q ö
(d) ç 0 , ÷
strikes the ground at point B. è g g ø
Calculate the distance AB.
(a) 3.33 km (b) 333 km (c) 33.3 km (d) 3330 km 9. A particle moves in the xy-plane with velocity
vx = 8 t - 2 and v y = 2. If it passes through the point
5. The height y and distance x along the horizontal for a x = 14 and y = 4 at t = 2 s, find the equation
body projected in the xy-plane are given by (x-y relation) of the path.
y = 8 t - 5 t2 and x = 6 t. The initial speed of (a) x = y2 - y + 2
projection is (b) x = 2 y2 + 2 y - 3
(a) 8 m/s (b) 9 m/s
(c) x = 3 y2 + 5
(c) 10 m/s (d) (10/3) m/s
(d) Cannot be found from above data
Telegram @unacademyplusdiscounts

128 JEE Main Physics

10. The ceiling of a long hall is 25 m high. Then, the 17. An arrow is shot into air. Its range is 200 m and its
maximum horizontal distance that a ball thrown time of flight is 5 s. If = 10 m / s2 , then horizontal
with a speed of 40 m/s can go without hitting the component of velocity and the maximum height will
ceiling of the hall, is [NCERT Exemplar] be respectively
(a) 95.5 m (b) 105.5 m (a) 20 m/s, 62.50 m (b) 40 m/s, 31.25 m
(c) 100 m (d) 150.5 m (c) 80 m/s, 62.5 m (d) None of these

11. A particle leaves the origin with an initial velocity 18. A body of mass m thrown horizontally with velocity v,
^ from the top of tower of height h touches the level
v = (3.00 i) ms -1 and a constant acceleration
^ ^ ground at distance of 250 m from the foot of the
. i - 0.50 j) ms -2 . When the particle reaches
a = ( - 100 tower. A body of mass 2 m thrown horizontally with
its maximum x -coordinate, what is its y-component a v
velocity , from the top of tower of height 4h will
velocity? 2
(a) –2.0 ms -1 (b) –1.0 ms -1 touch the level ground at a distance x from the foot of
(c) –1.5 ms -1 (d) 1.0 ms -1 tower. The value of x is
(a) 250 m (b) 500 m
12. A projectile can have same range from two angles of
(c) 125 m (d) 250 2 m
projection with same initial speed. If h1 and h2 be the
maximum heights, then 19. A ball is thrown up with a certain velocity at an angle
(a) R = h1h2 (b) R = 2 h1h2 q to the horizontal. The kinetic energy (KE) of the ball
(c) R = 2 h1h2 (d) R = 4 h1h2 varies in the horizontal displacement x as

13. A stone is thrown at an angle q to be the horizontal


reaches a maximum height H. Then, the time of
KE KE
flight of stone will be
(a) (b)
2H 2H
(a) (b) 2
g g
O O x
2 2 H sin q 2 H sin q x
(c) (d)
g g

14. A bomb is dropped on an enemy post by an aeroplane KE KE


flying horizontally with a velocity of 60 kmh -1 and at (c) (d)
a height of 490 m. At the time of dropping the bomb,
how far the aeroplane should be from the enemy post
O O
so that the bomb may directly hit the target? x x
400 500
(a) m (b) m 20. Two paper screen A and B are separated by a
3 3
1700 distance of 100 m. A bullet pierces A and B. The hole
(c) m (d) 498 m
3 in B is 10 cm below the hole in A. If the bullet is
travelling horizontally at the time of hitting A. Then,
15. A body projected with velocity u at projection angle q
the velocity of the bullet at A is
has horizontal range R. For the same velocity and
(a) 100 m/s (b) 200 m/s
projection angle, its range on the moon surface will
(c) 600 m/s (d) 700 m/s
be g moon = g earth / 6)
R 21. It was calculated that a shell when fired from a
(a) 36 R (b)
36 gun with a certain velocity and at an angle of
R 5p
(c) (d) 6 R elevation rad should strike a given target. In
16 36
16. A boy throws a ball with a velocity u at an angle q actual practice, it was found that a hill just prevented
with the horizontal. At the same instant he starts the trajectory. At what angle of elevation should the
running with uniform velocity to catch the ball before gun be to hit the target?
5p 11 p
if hits the ground. To achieve this he should run with (a) rad (b) rad
36 36
a velocity of
7p 13 p
(a) u cos q (b) u sin q (c) rad (d) rad
(c) u tan q (d) u sec q 36 36
Telegram @unacademyplusdiscounts

Projectile Motion 129

22. Two projectiles thrown from the same point at angles 29. A projectile shot into air at some angle with the
60° and 30° with the horizontal attain the same horizontal has a range of 200 m. If the time of flight is
height. The ratio of their initial velocities is 5 s, then the horizontal component of the velocity of
(a) 1 (b) 2 the projectile at the highest point of trajectory is
1 (a) 40 ms -1
(c) 3 (d)
3 (b) 0 ms -1
(c) 9.8 ms -1
23. A projectile is thrown at angle b with vertical. It
(d) equal to the velocity of projection of the projectile
reaches a maximum height H. The time taken to
reach the highest point of its path is 30. The kinetic energy of a project at the height point is
H 2H half of the initial kinetic energy. What is the angle of
(a) (b)
g g projection with the horizontal?
H 2H (a) 30° (b) 45° (c) 60° (d) 90°
(c) (d)
2g g cos b 31. A ball is projected from a certain point on the surface
of a planet at a certain angle with the horizontal
24. A cricket ball is hit at 30° with the horizontal with surface. The horizontal and vertical displacement x
kinetic energy E k. What is the kinetic energy at the and y vary with time t in second as x = 10 3 t and
highest point? y = 10 t - t2 . The maximum height attained by the
Ek 3Ek Ek
(a) (b) (c) (d) Zero ball is
2 4 4
(a) 100 m (b) 75 m
25. A particle is projected with a velocity of 30 m/s, at an (c) 50 m (d) 25 m
3
angle q 0 = tan -1 æç ö÷. After 1 s, the particle is moving 32. A projectile A is thrown at an angle of 30° to the
è4ø
horizontal from point P. At the same time, another
at an angle q to the horizontal, where tan q will be
projectile B is thrown with velocity v2 upwards from
equal to (g = 10 m/s2 )
1 1 the point Q vertically below the highest point. For B
(a) 1 (b) 2 (c) (d) v
2 3 to collide with A, 2 should be
v1
26. When a projectile is projected at a certain angle with Highest
point
the horizontal, its horizontal range is R and time of
flight is T1. When the same projectile is throwing with
the same speed at some other angle with the v1 B v2
horizontal, its horizontal range is R and time of flight
A
is T2 . The product of T1 and T2 is 30º
R 2R P Q
(a) (b) 1
g g (a) 1 (b) 2 (c) (d) 4
3R 4R 2
(c) (d)
g g 33. For a projectile thrown into space with a speed v, the
3v2 v2
27. A projectile of mass m is thrown with a velocity v horizontal range is × The vertical range is .
2g 8g
making an angle of 45° with the horizontal. The
change in momentum from departure to arrival along The angle which the projectile makes with the
vertical direction, is horizontal initially is
(a) 2mv (b) 2 mv (a) 15° (b) 30° (c) 45° (d) 60°

(c) mv (d)
mv 34. The velocity of projection of an oblique projectile is
^ ^
2 (6 i + 8 j) ms -1. The horizontal range of the projectile
28. Two stones thrown at different angles have same is
initial velocity and same range. If H is the maximum (a) 4.9 m (b) 9.6 m (c) 19.6 m (d) 14 m
height attained by one stone thrown at an angle of
35. A body is projected at an angle q to the horizontal
30°, then the maximum height attained by the other
with kinetic energy E k. The potential energy at the
stone is
H highest point of the trajectory is
(a) (b) H (a) Ek (b) Ek cos2 q
2
2
(c) 2 H (d) 3H (c) Ek sin q (d) Ek tan2 q
Telegram @unacademyplusdiscounts

130 JEE Main Physics

36. Two projectiles A and B are thrown with velocities v 41. Two particles are simultaneously projected in
v opposite directions horizontally from a given point in
and respectively. They have the same range. If B is
2 space whose gravity g is uniform. If u1 and u2 be their
thrown at an angle of 15° to the horizontal, A must initial speeds, then the time t after which their
have been thrown at an angle velocites are mutually perpendicular is given by
æ1ö æ 1ö
(a) sin -1 ç ÷ (b) sin -1 ç ÷ u1u2 u21 + u22
è16 ø è 4ø (a) (b)
g g
æ 1ö 1 -1 æ 1 ö
(c) 2 sin -1 ç ÷ (d) sin ç ÷ u1 (u1 + u2 ) u2 (u1 + u2 )
è 4ø 2 è 8ø (c) (d)
g g
37. A particle slides down a frictionless parabolic ( y = x2 )
track ( A - B - C) starting from rest at point A. Point 42. A plane surface is inclined making an angle q with
B is at the vertex of parabola and point C is at a the horizontal. From the bottom of this inclined
height less than that of point A. After C, the particle plane, a bullet is fired with velocity v. The maximum
moves freely in air as a projectile. If the particle possible range of the bullet on the inclined plane is
v2 v2
reaches highest point at P, then [NCERT Exemplar] (a) (b)
g g(1 + sin q)
A y
v2 v2
(c) (d)
g(1 - sin q) g(1 + sin q)2
P
43. A projectile is fired with a velocity v at an angle q with
the horizontal. The speed of the projectile when its
direction of motion makes an angle b with the
horizontal is
v0
θ (a) v cos q (b) v cos q cos b
C (c) v cos q sec b (d) v cos q tan b
–x2 –x1 B –x0 x
44. A ball is projected up an incline of 30° with a velocity
(x = 0)
of 30 ms -1 at an angle of 30° with reference to the
(a) KE at P = KE at B inclined plane from the bottom of the inclined plane.
(b) height at P = height at A If g = 10 ms -2 , then the range on the inclined plane is
(c) total energy at P = total energy at A (a) 12 m (b) 60 m (c) 120 m (d) 600 m
(d) time of travel from A to B = time of travel from B to P.
45. A cricketer can throw a ball to a maximum horizontal
38. The horizontal range of an oblique projectile is equal distance of 100 m. How much high above the ground
to the distance through which a projectile has to fall can the cricketer throw the same ball? [NCERT Exemplar]
freely from rest to acquire a velocity equal to the
(a) 40 m (b) 45 m
velocity of projection in magnitude. The angle of
(c) 500 m (d) 50 m
projection is
(a) 15° (b) 60° (c) 45° (d) 30° 46. A piece of marble is projected from earth’s surface
39. A projectile is thrown with velocity v making an angle with velocity of 50 ms -1. 2 s later it just clears a wall
q with the horizontal. It just crosses the tops of two 5 m high. What is the angle of projection?
(a) 45° (b) 30°
poles, each of height h, after 1 s and 3 s respectively.
(c) 60° (d) None of these
The time of flight of the projectile is
(a) 1 s (b) 3 s 47. A body is projected with speed v ms -1 at angle q. The
(c) 4 s (d) 7.8 s kinetic energy at the highest point is half of the
40. Two stones are projected so as to reach the same initial kinetic energy. The value of q is
(a) 30° (b) 45° (c) 60° (d) 90°
distance from the point of projection on a horizontal
surface. The maximum height reached by one 48. A ball is projected with velocity u at an angle a with
exceeds the other by an amount equal to half the sum horizontal plane. Its speed when it makes an angle b
of the height attained by them. Then, angle of with the horizontal is
u
projection of the stone which attains smaller height is (a) u cos a (b)
(a) 45° (b) 60° cos b
(c) 30° (d) tan -1 (3 / 4 ) u cos a
(c) u cos a cos b (d)
cos b
Telegram @unacademyplusdiscounts

Projectile Motion 131

49. The angle of projection of a projectile for which the


horizontal range and maximum height are equal to v0
(a) tan -1(2) (b) tan -1( 4 ) H
(c) cot -1 (2) (d) 60°
θ
50. A particle is projected from horizontal making an
-1
angle 60° with initial velocity 40 ms . The time √3 H
taken by the particle to make angle 45° from
æ 1 ö
horizontal, is (a) tan -1 ç ÷ (b) tan -1 3
(a) 15 s (b) 2.0 s (c) 20 s (d) 1.5 s è 3ø
æ 2 ö æ 3ö
51. Two bodies are projected from the same point with (c) tan -1 ç ÷ (d) tan -1 ç ÷
è 3ø è 2 ø
equal speeds in such directions that they both strike
the same point on a plane whose inclination is b. If a
57. A particle is projected with speed v at an angle
be the angle of projection of the first body with the p
horizontal the ratio of their times of flight is q æç0 < q < ö÷ above the horizontal from a height H
è 2ø
cos a sin( a + b )
(a) (b) above the ground. If v = speed with which particle
sin ( a + b ) cos a
hits the ground and t = time taken by particle to
cos a sin( a - b )
(c) (d) reach ground, then
sin( a - b ) cos a (a) as q increases, v decreases and t increases
52. A particle is projected with velocity 2 gh so that it (b) as q increases, v increases and t increases
just clears two walls of equal height h, which are at a (c) as q increases, v remains same and t increases
distance of 2h from each other. What is the time (d) as q increases, v remains same and t decreases
interval of passing between the two walls? 58. Two inclined planes are located as shown in figure. A
2h gh h h particle is projected from the foot of one frictionless
(a) (b) (c) (d) 2
g g g g plane along its line with a velocity sufficient to carry
it to top after which the particle slides down the other
53. A projectile is thrown with a velocity of 10 m/s at an
frictionless inclined plane. The total time it will take
angle 60° with horizontal. The interval between the
to reach the point C is
moment when speed is 5 g m/s, is ( g = 10 m/s2 ).
(a) 1 s (b) 3 s
(c) 2 s (d) 4 s
9.8 m
54. A particle is projected from the ground with an initial
speed of v at an angle q with horizontal. The average
velocity of the particle between its point of projection 45° 45°
and highest point of trajectory is (a) 2 s (b) 3 s (c) 2 2 s (d) 4 s
v v
(a) 1 + 2 cos2 q (b) 1 + cos2 q 59. The equation of motion of a projectile are given by
2 2
v x = 36 t. If and 2 y = 96 t - 9.8 t2 m. The angle of
(c) 1 + 3 cos2 q (d) v cos q projectile is
2
æ 4ö æ 3ö
(a) sin -1 ç ÷ (b) sin -1 ç ÷
55. A body of mass m is thrown upward at an angle q with è5ø è5ø
the horizontal with velocity v. While rising up the æ 4ö æ 3ö
velocity of the mass after t second will be (c) sin -1 ç ÷ (d) sin -1 ç ÷
è 3ø è 4ø
(a) ( v cos q)2 + ( v sin q)2
60. Trajectories of two projectiles are shown in figure.
(b) ( v cos q - v sin q)2 - gt Let T1 and T2 be the time periods and u1 and u2 their
(c) 2 22
v + g t - (2v sin q) gt speeds of projection. Then,
y
(d) v2 + g2t2 - (2v cos q) gt

56. A projectile is thrown at an angle q such that it is just


able to cross a vertical wall as its highest point as 1
2
x
shown in the figure. The angle q at which the
projectile is thrown is given by (a) T2 > T1 (b) T1 = T2 (c) u1 > u2 (d) u1 < u2
Telegram @unacademyplusdiscounts

132 JEE Main Physics

61. A projectile A is thrown at an angle 30° to the point C if both are projected simultaneously?
horizontal from point P. At the same time another ( g = 10 ms-2 )
projectile B is thrown with velocity v2 upwards from B
the point Q vertically below the highest point A would 5 ms–1
v
reach. For B to collide with A the ratio 2 should be
v1
h 10 ms–1

v1
v2 60°
C
A
30°
P Q (a) 10 m (b) 30 m
3 1 2 (c) 15 m (d) 25 m
(a) (b) 2 (c) (d)
2 2 3
65. A very broad elevator is going up vertically with a
62. A fighter plane enters inside the enemy territory, at constant acceleration of 2ms -2 . At the instant when
time t = 0 with velocity v0 = 250 ms -1 and moves its velocity is 4 ms-1 a ball is projected from the floor
horizontally with constant acceleration a = 20 ms -2 of the list with a speed of 4 ms-1 relative to the floor
(see figure). An enemy tank at the border, spot the at an elevation of 30°. The time taken by the ball to
plane and fire shots at an angle q = 60° with the return the floor is ( g = 10 ms-2 )
horizontal and with velocity u = 600 ms -1. At what (a) 1/2 s (b) 1/3 s
altitude H of the plane it can be hit by the shot? (c) 1/4 s (d) 1 s
66. A projectile is fired at an angle of 30° to the
horizontal such that the vertical component of its
600 ms–1 initial velocity is 80 ms-1. Its time of flight is T. Its
T
velocity at t = has a magnitude of nearly
H 4
(a) 200 ms -1 (b) 300 ms -1
θ = 60° (c) 140 ms -1 (d) 100 ms -1
(a) 1500 3 m (b) 125 m 67. A car is moving rectilinearly on a horizontal path
(c) 1400 m (d) 2473 m with acceleration a0 . A person sitting inside the car
observes that an insect S is crawling up the screen
63. An aircraft, diving at an angle of 53.0° with the with an acceleration a. If q is the inclination of the
vertical releases a projectile at an altitude of 730 m. screen with the horizontal the acceleration of the
The projectile hits the ground 5.00 s after being insect
released. What is the speed of the aircraft? (a) parallel to screen is a0 cos q
(a) 282 ms -1 (b) along the horizontal is a0 - a cos q
(b) 202 ms -1 (c) perpendicular to screen is a0 sin q
(c) 182 ms -1 (d) perpendicular to screen is a0 tan q
(d) 102 ms -1 68. A particle is projected from the ground at an angle of
60° with horizontal with speed u = 20 ms -1. The
64. A particle A is projected from the ground with an radius of curvature of the path of the particle, when
initial velocity of 10 ms-1 at an angle of 60° with its velocity makes an angle of 30° with horizontal is
horizontal. From what height h should an another ( g = 10 ms -2 )
particle B be projected horizontal with velocity 5 ms-1 (a) 10.6 m (b) 12.8 m
so that both the particles collide with velocity 5 ms-1 (c) 15.4 m (d) 24.2 m
so that both the particles collide on the ground at
Telegram @unacademyplusdiscounts

Projectile Motion 133

Round II (Mixed Bag)

Only One Correct Option 7. A car is travelling at a velocity of 10 kmh -1 on a


straight road. The driver of the car throws a parcel
1. A stone is projected with a velocity 20 2 ms-1 at an
with a velocity of 10 2 kmh -1 when the car is
angle of 45° to the horizontal. The average velocity of
passing by a man standing on the side of the road. If
stone during its motion from starting point to its
the parcel is to reach the man, the direction of throw
maximum height is ( g = 10 ms-2 )
makes the following angle with direction of the car
(a) 5 5 ms -1 (b) 10 5 ms -1
(a) 135° (b) 45°
(c) 20 ms -1 (d) 20 5 ms -1 -1 æ 1 ö
(c) tan ( 2) 60° (d) tan ç ÷
è 2ø
2. A ball is dropped from a height of 49 m. The wind is
blowing horizontally. Due to wind a constant 8. A particle is dropped from a height h. Another
horizontal acceleration is provided to the ball. Choose particle was what initially at a horizontal distance d
the correct statement (s). ( Take g = 9.8 m /s2 ) from the first, is simultaneously projected with a
(a) Path of the ball is a straight line horizontal velocity u and two particles just collide on
(b) Path of the ball is a curved one the ground. The three quantities h, d and u are
(c) The time taken by the ball to reach the ground is 316 s related to
(d) Actual distance travelled by the ball is more than 49 m u2 h 2 u2 h
(a) d2 = (b) d2 =
3. A shell is fired from a cannon with a velocity v at 2g g
angle q with horizontal. At the highest point, it (c) d = h (d) gd2 = u2 h
explodes into two pieces of equal mass. One of the
pieces retraces its path to the cannon. The speed of 9. A body of mass 1 kg is projected with velocity 50 m/s
the other piece just after explosion is at an angle of 30° with the horizontal. At the highest
(a) 3v cos q (b) 2v cos q point of its path a force 10 N starts acting on body for
3 3 5 s vertically upward besides gravitational force,
(c) v cos q (d) v cos q what is horizontal range of the body? ( g = 10 m / s2 )
2 2
(a) 125 3 m (b) 200 3 m
4. The speed of projection of a projectile is increased by
(c) 500 m (d) 250 3 m
10%, without changing the angle of projection. The
percentage increase in the range will be 10. If a stone is to hit at a point which is at a distance d
(a) 10% (b) 20% (c) 15% (d) 5% away and at a height h above the point from where
5. A projectile is launched with a speed of 10 m/s at an the stone starts, then what is the value of initial
angle 60° with the horizontal from a sloping surface speed u, if the stone is launched at an angle q?
of inclination 30°. The range R is. (Take, g = 10 m / s2 ) u

10 m/s
60° h

R θ

30° d

g d
(a)
(a) 4.9 m (b) 13.3 m (c) 9.1 m (d) 12.6 m cos q 2 ( d tan q - h)

6. Two stones are projected with the same velocity in (b)


d d
magnitude but making different angles with the cos q 2 ( d tan q - h)
horizontal. Their ranges are equal. If the angle of
p gd2
projection of one is and its maximum height is y1, (c)
3 h cos2 q
the maximum height of the other will be gd2
y y (d)
(a) 3 y1 (b) 2 y1 (c) 1 (d) 1 ( d - h)
2 3
Telegram @unacademyplusdiscounts

134 JEE Main Physics

11. Figure shows four paths for a kicked football ignoring Y


the effects of air on the flight rank the paths u
according to the initial horizontal velocity component
highest first
θ
X
P Q
2 2
(a) mu sin q (b) mu cos q
1 1
(c) mu2 sin 2 q (d) mu2 cos 2 q
2 2
1 2 3 4
16. A projectile is fired with a velocity v at right angle to
(a) 1, 2, 3, 4 (b) 2, 3, 4, 1 the slope which is inclined at an angle q with the
(c) 3, 4, 1, 2 (d) 4, 3, 2, 1 horizontal. What is the time of flight?
12. After one second the velocity of a projectile makes an 2v2 v2
(a) tan q (b) tan q
angle of 45° with the horizontal. After another one g g
second it is travelling horizontally. The magnitude of
2v2 2v2
its initial velocity and angle of projection are (c) sec q (d) tan q sec q
( g = 10 ms -2 ) g g
(a) 14.62 ms –1, tan -1 (2) (b) 22.36 ms –1, tan -1(2)
17. A body is projected up smooth inclined plane with a
–1
(c) 14.62 ms , 60° (d) 22.36 ms –1, 60° velocity v0 from the point A as shown figure. The angle
13. A particle of mass m is released V(x) of inclination is 45° and top B of the plane is connected
from rest and follows a parabolic to a well of diameter 40 m. If the body just manages to
m
path as shown. Assuming that cross the well, what is the value of v0 ? Length of the
the displacement of the mass O
x inclined plane is 20 2 m, and g = 10 ms -2 .
from, the origin is small, which B C
graph correctly depicts the
45°
position of the particle as a
function of time?
A
x(t) x(t)

(a) 0 t (b) 0 t 40 m

(a) 20 ms -1 (b) 20 2 ms -1
-1
(c) 40 ms (d) 40 2 ms -1
x(t) x(t)
18. Two projectiles A and B thrown with speeds in the
ratio 1 : 2 acquired the same heights. If A is thrown
(c) 0 t (d) 0 t
at an angle of 45° with the horizontal, the angle of
projection of B will be
(a) 0° (b) 60°
(c) 30° (d) 45°
14. A particle of mass m is projected with a velocity v at (e) 15°
an angle of 60° with horizontal. When the particle is
at its maximum height. The magnitude of its angular 19. A particle is projected with a velocity 200 ms -1 at an
momentum about the point of projection is angle of 60°. At the highest point, it explodes into
3 mv 2 three particles of equal masses. One goes vertically
(a) zero (b) upwards with a velocity 100 ms -1, the second particle
16 g
goes vertically downwards. What is the velocity of
3 mv2 3 mv2
(c) (d) third particle?
16 g 3g (a) 120 ms -1 making 60° angle with horizontal
15. Average torque on a projectile of mass m, initial (b) 200 ms -1 making 30° angle with horizontal
speed u and angle of projection q, between initial and (c) 300 ms -1
final position P and Q as shown in figure about the (d) 200 ms -1
point of projection is
Telegram @unacademyplusdiscounts

Projectile Motion 135

20. The trajectory of a projectile in vertical plane is Comprehension Based Questions


y = ax - bx2 , where a and b are constants and x and y
are respectively horizontal and vertical distances of Passage I
the projectile from the point of projection. The A projectile is thrown from the ground with a speed of
maximum height attained by the particle and the
2 gh at an angle of 60° to the horizontal from a point
angle of projection from the horizontal are
b2 a2 on the horizontal ground.
(a) , tan -1( b) (b) , tan -1(2b)
4b b 26. The horizontal range of projectile is
a2 2a2 (a) 3h (b) 2h 3
(c) , tan -1( a ) (d) , tan -1( a )
4b b (c) 3h/ 2 (d) 3h/ 2

27. The time spent by projectile above a height h is


More Than One Correct Options h h 2h 3h
(a) 4 (b) (c) (d)
21. A particle is projected from a point A with a velocity v g g g g
at an angle of elevation q. At a certain point B, the
particle moves at right angle to its initial direction. 28. The maximum height attained by projectile is
Then (a) 2h/3 (b) 3h
(a) velocity of particle at B is v sin q (c) 3h/ 4 (d) 3h/ 2
(b) velocity of particle at B is v cot q
Passage II
(c) velocity of particle at B is v tan q
v Two second after projection, a projectile is travelling
(d) velocity of flight from A to B is
g sin q in a direction inclined at 30° to the horizontal. After
1 more second, it is travelling horizontally
22. Two projectiles A and B are projected with same (use g = 10 ms -2 )
speed at angles 15° and 75° respectively to the
maximum and have same horizontal range. If h be 29. The initial velocity of its projection is
(a) 10 ms -1 (b) 10 3 ms -1
the maximum height and T total time of flight of a
projectile, then (c) 20 ms -1 (d) 20 3 ms -1
(a) hA > hB (b) hA < hB
30. The angle of projection of the projectile is
(c) TA < TB (d) TA > TB
(a) 30° (b) 45°
23. Two particles are projected in air with speed v0 at (c) 60° (d) None of these
angles q 1 and q 2 (both acute) to the horizontal,
respectively. If the height reached by the first Assertion and Reason
particle is greater than that of the second, then tick
the right choices [NCERT Exemplar] Directions Question No. 31 to 35 are Assertion-Reason type.
(a) angle of projection : q 1 > q2 Each of these contains two Statements : Statement I (Assertion),
(b) time of flight : T1 > T2 Statement II (Reason). Each of these questions also has four
(c) horizontal range : R1 2> R alternative choices, only one of which is correct. You have to
select the correct choices from the codes (a), (b), (c) and (d) given
(d) total energy : U1 > U2
ahead
24. A projectile has the same range R for two angles of (a) If both Assertion and Reason are true and Reason is
projections. If T1 and T2 be the times of flight in the correct explanation of the Assertion
two cases, then (using q as the angle of projection (b) If both Assertion and Reason are true but Reason is
corresponding to T1) not correct explanation of the Assertion
(a) T1 T2 µ R (b) T1 T2 µ R2 (c) If Assertion is true but Reason is false
(d) If Assertion is false but the Reason is true
(c) T1 /T2 = tan q (d) T1 /T2 = 1
31. Assertion If a particle is projected vertices upwards
25. A particle is hurled into air from a point on the
with velocity u, the maximum height attained by the
horizontal ground at an angle with the vertical. If the
particle is h1. The same particle is projected at angle
air exerts a constant resistive force
30° from horizontal with the same speed u. Now the
(a) the path of projectile will be parabolic path
maximum height is h2 . Thus, h1 = 4 h2
(b) the time of ascent will be equal to time of descent
(c) the total energy of the projectile is not conserved Reason In first case v = 0 at highest point and in
(d) at the highest point, the velocity of projectile is second case v ¹ 0 at highest point.
horizontal
Telegram @unacademyplusdiscounts

136 JEE Main Physics

32. Assertion At highest point of a projectile dot product 34. Assertion If in a projectile motion, we take air
of velocity and acceleration is zero. friction into consideration, then tascent < tdescent .
Reason At highest point velocity and acceleration Reason During ascent magnitude of retardation is
are mutually perpendicular. greater than magnitude of acceleration during
33. Assertion A particle is projected with speed u at an descent.
angle q with the horizontal. At any time during 35. Assertion In projectile motion if time of flight is 4 s,
motion, speed of particle is v at angle a with the then maximum height will be 20 m. ( g = 10 m / s2 ).
vertical, then v sin a is always constant throughout Reason Maximum height =
gT
.
the motion. 2
Reason In case of projectile motion, magnitude of
radial acceleration at top most point is maximum.

Previous Years’ Questions


36. A projectile is given an initial velocity of ( $i + 2$j ) m/s, 41. A ball rolls of the top of stair-way with a horizontal
where $i is along the ground and $jis along the vertical. velocity of magnitude 1.8 ms -1. The steps are 0.20 m
If g = 10 m / s2 , the equation of its trajectory is high and 0.20 m wide. Which step will the ball hit
[JEE Main 2013] first? [Orissa JEE 2011]
(a) y = x - 5 x2 (b) y = 2x - 5 x2 (a) First (b) Second
(c) 4 y = 2x - 5 x2 (d) 4 y = 2x - 25 x2 (c) Third (d) Fourth
37. Two cars of masses m1 and m2 are moving in circles of 42. A projectile is projected with velocity kve vertically
radii r1 and r2 respectively. Their speeds are such that upward direction from the ground into the space
they make complete circles in the same time t. The (ve is the escape velocity and k < 1). If air resistance is
ratio of their centripetal acceleration is [AIEEE 2012] considered to be negligible then the maximum
(a) m1r1 : m2 r2 (b) m1 : m2 height from the centre of earth to which it can go will
(c) r1 : r2 (d) 1 : 1 be (R = radius of earth) [Orissa JEE 2010]
R R
38. A boy can throw a stone up to a maximum height of (a) 2
(b) 2
10 m. The maximum horizontal distance that the boy k +1 k -1
R R
can throw the same stone up to will be [AIEEE 2012] (c) (d)
1 - k2 k +1
(a) 20 2 m (b) 10 m
(c) 10 2 m (d) 20 m 43. A small particle of mass m in projected at an angle q
with the x-axis with an initial velocity v0 in the X
39. A particle of mass m is projected with a velocity v v sin q
Y - plane as shown in figure . At a time t < 0
making an angle of 30° with the horizontal. The g
magnitude of angular momentum of the projectile the angular momentum of the particle is
about the point of projection when the particle is at [AIEEE 2010]
its maximum height h is [AIEEE 2011] Y
3 mv2
(a) (b) zero v0
2 g
mv3 3 mv3
(c) (d)
2g 16 g
θ
40. A large number of bullets are fired in the all X
directions with same speed v. What is the maximum 1
area on the ground on which these bullets will (a) mgv0 t2 cos q i$
2
spread? [AIEEE 2011]
(b) - mgv0 t2 cos q $j
v2 v4
(a) p (b) p (c) mgv0 t cos qk$
g g2
1
v4 v2 (d) - mgv0 t2 cos qk$
(c) n2 (d) p2 2
g2 g2
Telegram @unacademyplusdiscounts

Projectile Motion 137

44. A point P moves in counter-clockwise direction on a 49. Two particles A and B are projected with same speed
circular path as shown in figure. The movement of P so that the ratio of their maximum heights reached is
is such that it sweep out a length s = t 3 + 5, where s is 3 : 1. If the speed of A is doubled without altering
in metres and t is in seconds. The radius of the path is other parameters, the ratio of the horizontal ranges
20 m. The acceleration of P when t = 2 s is nearly obtained by A and B is [Kerala CET 2008]
[AIEEE 2010] (a) 1 : 1 (b) 2 : 1
y
(c) 4 : 1 (d) 3 : 2
50. A body is projected at such angle that the horizontal
P(x, y) range is three times the greatest height. The angle of
projeciton is [BCECE 2008]
20 m (a) 42°8¢ (b) 53°7¢
O x (c) 33°7¢ (d) 25°8¢
(a) 14 m/s2 (b) 13 m/s2 51. A particle is projected with certain velocity at two
(c) 12 m/s2 (d) 7.2 m/s2 different angles of projections with respect to
45. The maximum range of projectile fired with some horizontal plane so as to have same range R on a
initial velocity is found to be 1000 m, in the absence of horizontal plane. If t1 and t2 are the time taken for the
wind and air resistance. The maximum height two paths, the which one of the following relations is
reached by the projectile is [Orissa JEE 2009]
correct? [UP SEE 2008]
2R R
(a) 250 m (b) 500 m (c) 1000 m (d) 2000 m (a) t1 t2 = (b) t1 t2 =
g g
46. A particle is projected with velocity v0 along x-axis. g 4R
(c) t1 t2 = (d) t1 t2 =
The deceleration on the particle is proportional to the 2g g
square of the distance from the origin, i. e., a = ax2 , the
distance at which the particle stop is [MP PET 2009] 52. A particle is projected at 60° to the horizontal with an
1/ 3 energy E. The kinetic energy and potential energy at
3 v0 æ3 v ö
(a) (b) ç 0 ÷ the highest point are [KCET, AIEEE 2007]
2a è 2a ø
æ E Eö æ 3E E ö
1/ 3 (a) ç , ÷ (b) ç , ÷
2 v20 æ 3 v2 ö è 2 2ø è 4 4ø
(c) (d) ç 0 ÷
3a è 2a ø æ E 3E ö
(c) ( E, 0 ) (d) ç , ÷
è4 4 ø
47. If a body is projected with an angle to the horizontal,
then [EAMCET 2008] 53. The maximum height attained by a projectile when
(a) its velocity is always perpendicular to its acceleration thrown at an angle q with the horizontal is found to
(b) its velocity becomes zero at maximum height be half the horizontal range. Then, q is equal to
(c) its velocity makes zero angle with the horizontal at its [KCET 2007]
maximum height -1 p
(d) the body just before hitting the ground, the direction of (a) tan (2) (b)
6
velocity coincides with the acceleration p æ 1ö
(c) (d) tan -1 ç ÷
48. A body is thrown upwards from the earth surface 4 è 2ø
with velocity 5 ms -1 and from a planet surface with
54. A particle is thrown in the upward direction making
velocity 3 ms -1. Both follow the same path. What is
an angle of 60° with the horizontal direction with a
the projectile acceleration due to gravity on the
velocity of 147 ms -1. Then, the time after which its
planet? Acceleration due to gravity on earth is
inclination with the horizontal is 45°, is
10 ms -1. [Orissa JEE 2008]
[UP SEE 2006]
(a) 2 ms -2 (b) 3.6 ms -2
(a) 15 s (b) 10.98 s
(c) 4 ms -2 (d) 5 ms -2
(c) 5.49 s (d) 2.745 s
Telegram @unacademyplusdiscounts

Answers
Round I
1. (c) 2. (b) 3. (a) 4. (a) 5. (c) 6. (b) 7. (c) 8. (a) 9. (a) 10. (d)
11. (c) 12. (d) 13. (b) 14. (b) 15. (d) 16. (c) 17. (b) 18. (a) 19. (c) 20. (d)
21. (d) 22. (d) 23. (b) 24. (b) 25. (d) 26. (b) 27. (b) 28. (d) 29. (a) 30. (b)
31. (d) 32. (c) 33. (b) 34. (b) 35. (c) 36. (d) 37. (c) 38. (a) 39. (c) 40. (b)
41. (a) 42. (b) 43. (c) 44. (b) 45. (a) 46. (b) 47. (b) 48. (d) 49. (b) 50. (d)
51. (d) 52. (d) 53. (a) 54. (c) 55. (c) 56. (c) 57. (c) 58. (d) 59. (a) 60. (c)
61. (c) 62. (d) 63. (b) 64. (c) 65. (b) 66. (c) 67. (c) 68. (c)

Round II
1. (b) 2. (c) 3. (a) 4. (b) 5. (b) 6. (d) 7. (b) 8. (b) 9. (d) 10. (b)
11. (d) 12. (b) 13. (d) 14. (b) 15. (c) 16. (a) 17. (b) 18. (c) 19. (c) 20. (c)
21. (d) 22. (c) 23. (a,b) 24. (a,c) 25. (a.c,d) 26. (b) 27. (d) 28. (d) 29. (d) 30. (c)
31. (b) 32. (a) 33. (b) 34. (a) 35. (c) 36. (b) 37. (c) 38. (d) 39. (d) 40. (b)
41. (d) 42. (c) 43. (d) 44. (a) 45. (a) 46. (d) 47. (c) 48. (b) 49. (c) 50. (b)
51. (a) 52. (d) 53. (a) 54. (c)

the Guidance
Round I
u 2 sin 2 ´ 15° u2 50 50 \ Angle with which it strikes the ground
1. Here, 50 = or = = = 100
g g sin 30° 1/ 2 æ vy ö æ 100 ö
q = tan -1ç ÷ = tan -1ç ÷
u 2 sin 2 ´ 45° u 2 è vx ø è 500 ø
R= = = 100 m
g g æ 1ö
q = tan -1ç ÷
1 è5ø
2. nh = gt 2
2 1
4. From h = gt 2,
æ 2nh ö 2
Þ t= ç ÷ …(i)
è g ø 2hOA 2 ´ 1960
We have tOB = = = 20 s
Horizontal distance travelled by ball g 9.8
æ 2nh ö Horizontal distance AB = vtOB
nb = ut , nb = u ç ÷ …(ii) æ 5ö
è g ø = ç600 ´ ÷ (20)
è 18 ø
Squaring Eq. (ii), we get
u 22nh 2u 2h = 3333.33 m = 3.33 km
n 2b 2 = \ n= dy dx
g gb 2 5. v y = = 8 -10 t ,v x = = 6
dt dt
3. Horizontal component of velocity v x = 500 ms-1 and vertical
At t = 0 , v y = 8 m/s and v x = 6 m/s
components of velocity while striking the ground
\ v = v x2 + v y2 = 10 m/s
v y = 0 + 10 ´ 10 = 100 ms-1
2h 4
u = 500 ms–1 6. t = = = 0.64 s
g (9.8)
s 3
Now, v= = = 4.7 m/s
500 ms–1 t 0.64
θ
7. Stone will must follow the parabolic path.
100 ms–1
Telegram @unacademyplusdiscounts

Projectile Motion 139

8. Range of the projectile on an inclined plane (down the plane) = sin 33.6°
is, or q = 33.6°
u2 u 2 sin 2q
R= [sin(2a + b) + sin b ] \ Horizontal range (R) =
g cos2 b g
Here, u = v 0 , a = 0 and b = q ( 40) 2 sin 2 ´ 33.6°
=
2v 02 sin q 9.8
\ R= 1600 ´ sin 67.2°
g cos2 q =
9.8
v2
1600 ´ 0.9219
θ = = 150.5 m
9.8
R 11. The velocity of the particle at any time t
v = v0 + a t
The x-component is
θ v x = v ax + axt
The y-component is
2v 02 tan q
Now x = R cos q = v y = v oy + ax = ( -0.5 t ) ms-1
g
2v 02 tan 2 q When the particle reaches its maximum x-coordinate,
and y = - R sin q = -
g v x = 0. i.e.,
9. v x = 8t - 2 3 -t =0 Þ t =3 s
dx The y-component of the velocity of this time is
or = 8t - 2 v y = - 0.5 ´ 3 = - 1.5 ms-1
dt
x t
or ò14 dx = ò2 (8t - 2)dt u 2 sin q
12. R = at angle q and 90° - q
g
or x - 14 = [ 4t 2 - 2t ]t2 = 4t 2 - 2t - 12
u 2 sin 2 q
or x = 4t 2 - 2t + 2 …(i) Now, h1 =
2g
Further, v y = 2
u 2 sin 2 (90° - q) u 2 cos2 q
dy and h2 = =
or =2 2g 2g
dt
y t æ u 2 sin 2q ö 1 R 2
\ ò4 dy = ò2 2 dt h1 h2 = ç
è g
÷× =
ø 16 16
or y - 4 = [2t ]t2 = 2t - 4 or y = 2t
\ R = R h1 h2
y
or t= …(ii)
2 u 2 sin 2 q
13. H =
Substituting the value of t from Eq. (ii) in Eq. (i), we have 2g
x = y2 - y + 2 2u sin q
and T=
g
10. Given, initial velocity (u) = 40 m/s
4u 2 sin 2 q T2 8
Þ T2 = \ =
Height of the hall (H) = 25 m g2 H g
Let the angle of projection of the ball be q, when maximum 8H 2H
height attained by it be 25 m. Þ T= =2
g g
Maximum height attained by the ball
2h 2 ´ 490
u 2 sin 2 q 14. t = = = 100 = 10 s
H= g 9.8
2g
æ 5ö 500
( 40) 2 sin 2 q x = vt = ç60 ´ ÷ ms-1 ´ 10 s = m
25 = è 18 ø 3
2 ´ 9.8
1
25 ´ 2 ´ 9.8 15. R µ \ Rmoon = 6 Rearth
or sin 2 q = g
1600
= 0.3063 16. Velocity of a body should be equal to the horizontal
or sin q = 0.5534 component of velocity of ball.
Telegram @unacademyplusdiscounts

140 JEE Main Physics

2 uy 4
17. T = 25. Given, ux = u cos q0 = 20 ´ = 24 m/s
g 5
gT 3
\ uy = = 25 m/s and uy sin q0 = 30 ´ = 18 m/s
2 5
uy2 (25) 2 After 1 s, ux will remain as it is uy will decreases by 10 m/s or it
Now, H= = = 31.25 m will remain 8 m/s
2g 20
vy 8 1
Further, R = uxT \ tan q = = =
v x 24 3
R
\ ux = = 40 m/s
T 26. The two angles of projection are clearly q and (90° - q).
2h 2v sin q
18. t = T1 =
g g
Distance from the foot of the tower 2v sin(90° - q)
and T2 =
2h g
d = vt = v = 250 m
g 2(v) 2(2 sin q cos q) 2R
\ T1T2 = =
v g ´g g
When velocity =
2 27. Change in momentum is the product of force and time.
and height of tower = 4h 2 sin q æ Dp ö
Dp = mg ´ çQ F = ÷
v 2( 4h) g è Dt ø
Then, distance x=
2 g = 2mv sin q = 2mv sin 45°
2h 2mv
x=v = 250 m = = 2 mv
g 2
19. At the highest point, KE will be minimum but not zero. 28. Since, range is given to be same therefore the other angle is
1 2 (90° - 30° ), i. e. , 60°.
20. h= gt (in vertical direction)
2 v 2 sin 2 30° 1 é v 2 ù
H= = ê ú
2h 2 ´ 0.1 2g 4 ë 2g û
\ t= = = 0.141s
g 10
v 2 sin 2 60° 3 é v 2 ù
H¢ = = ê ú
Now, in horizontal direction 2g 4 ë 2g û
S 100
vx = x = » 700 m/s H¢ 3 4
t 0.141 = ´ = 3 or H ¢ = 3H
H 4 1
p 5 p 18 p - 5 p
21. Required angle = - = v 2 sin 2q 2v sin q
2 36 36 29. R = = 200 , T = =5
g g
13 p
= rad
36 v 2 ´ 2 sin q cos q g 200
Dividing, ´ = = 40
v 2 sin 2 q g 2v sin q 5
22. As, hmax =
2g or v cos q = 40 ms-1
In the given problem, hmax is same in both the cases. It may be noted here that the horizontal component of the
\ v12 sin 2 60° = v 22 sin 2 30° velocity of projection remains the same during the flight of
the projectile.
v1 sin 30° 1 2 1
or = = ´ = 1
v 2 sin 60° 2 3 3 30. (KE)H = (KE)i
2
v 2 cos2 b 1 1 æ1 ö 1
23. As, H = or v cos b = 2gH mv 2 cos2 q = ç mv 2÷ = mv 2
2g 2 2 è2 ø 4
v cos b 2gH 1
t= = or cos2 q =
g g 2
2H 1
or t= or cos q =
g 2
3E k or q = 45°
24. As, E k ¢ = E k cos2 30° =
4
Telegram @unacademyplusdiscounts

Projectile Motion 141

d d d v 2 sin 2q
31. v y (y) = (10t ) - (t 2) = 10 - 2t 36. As, R =
dt dt dt g
At maximum height, v y = 0 In the given problem v 2 sin 2q = constant
\ 10 - 2t = 0 2
æv ö v2
or 2t = 10 v 2 sin 2q = ç ÷ sin 30° =
è2ø 8
or t =5 s
1
\ y = (10 ´ 5 - 5 ´ 5) m = 25 m or sin 2q =
8
32. Equating velocities along the vertical, é 1ù
or 2q = sin -1ê ú
v 2 = v1 sin 30° ë8û
v2 1 1 é 1ù
or = or q = sin -1ê ú
v1 2 2 ë8û
v 2 sin 2q 3v 2 37. Since y = xz , the motion is in two dimensions. Velocity at B is
33. As, =
g 2g
greater than at P. In the given motion of a particle, the law of
3 conservation of energy is obeyed. Therefore, total energy at
or sin 2q =
2 P = total energy at A. As vertical distance AB > BP , time of
or 2q = 60° travel from A to B is greater than that from B to P.
or q = 30° 38. Using, v 2 - u2 = 2as, we get
Let us cross check with the help of data for vertical range. v2
2 2 2 s=
v sin q v 2g
=
2g 8g v 2 sin 2q v 2
1 Now, =
or sin 2 q = g 2g
4 1
1 or sin 2q =
or sin q = 2
2
or sin 2q = sin 30°
or q = 30°
or q = 15°
^ ^
34. Here, v = 6 i + 8 j ms -1 The other possible angle of projection is (90° - 15° ), i. e. ,75°.
^ ^ 1
Comparing with v = v x i + v y j , we get 39. h = v sin qt - gt 2
2
ux = 6 ms-2 1 2
or gt - v sin qt + h = 0
and uy = 8 ms-2 10 8 2
2 -v sin q
Also, u = v x2 + v y2 t1 + t 2 = -
1
θ g
36 + 64 = 100 2
-1 6
or v = 10 ms 2v sin q
or + =1 2= t t T
8 6 g
sin q = and cos q =
10 10 T = (1 + 3) s = 4 s
v 2 sin 2q 2v 2 sin q cos q H + H2
R= = 40. As, H1 - H2 = 1
g g 2
8 6 or H1 = 3 H2
R = 2 ´ 10 ´ 10 ´ ´ ´ 10 m = 9.6 m
10 10 2 2 ì u 2 sin2 (90° - q) ü
u sin q
\ = 2í ý
35. Let v be the velocity of projection and q the angle of 2g î 2g þ
projection.
Kinetic energy at highest point tan 2 q = 3
1
= mv 2 cos2 q or E k cos2 q \ tan q = 3
2
or q = 60°
Potential energy at highest point
= E k - E k cos2 q = E k(1 - cos2 q) = E k sin 2 q Therefore, the other angle is (90° - q) or 30°.
Telegram @unacademyplusdiscounts

142 JEE Main Physics

41. Since, v1 ^ v 2 43. As, v ¢ cos b = v cos q


\ v1 × v1 = 0 (Q horizontal component of velocities are always equal)
or (u1 i - gtj ) × ( -u2 i - gt$j ) = 0
$ $ $ or BV ¢ = v cos q sec b
\ g 2 t 2 = u1 u2 2 ´ 30 ´ 30 sin 30° cos 60°
44. R =
10 cos2 30°
u1 u2 1 1 2 ´2
or t= = 180 ´ ´ ´ m = 60 m
g 2 2 3
42. As, v x = v cos( a - q); v y = v sin( a - q) 45. Horizontal range of a projectile is given by
ax = - g sin q; ay = - g cos q u 2 sin 2q
R=
If T is the time of flight, then g
If q = 45°, then R is maximum and is equal to
A Y
u2
Rmax =
v inθ g
g cos θ

Y gs
α–θ g Given, Rmax = 100 m
α u2
θ \ 100 = …(i)
O B g
1 When cricketer throws the ball vertically upward, then ball
0 = v sin( a - q) × T - g cos q × T 2
2 goes upto height H.
2v sin( a - q) Using equation of motion,
or T=
g cos q v 2 = u 2 + 2as
OB = v cos a ´ T (0) 2 = u 2 + 2( - g )H
OB
Now, cos q = u2 1 æ u2 ö
OA or H= = ç ÷
2g 2èg ø
OB
or OA = 1
cos q = ´ 100 [using Eq. (i)]
2
v sin a × T
or OA = = 50 m
cos q
2v sin( a - q) 1 46. Horizontal component = u cos q
or OA = v cos a ´ ´
g cos q cos q Vertical component = u sin q
v 2
g = - 10 ms-2, u = 50 ms-1,h = 5 m, t = 2 s
or OA = [2 sin( a - q) cos a ]
g cos2 q 1 2
h = uy t + gt
v 2 2
or OA = [sin (2a - q) + sin ( - q)]
g cos2 q
v2 θ
or OA = [sin(2a - q) - sin q]
g cos2 q 1
\ 5 = 50 sin q - ´ 10 ´ 4
Clearly, the range R ( = OA) will be maximum when sin(2a - q) 2
is maximum, i. e. , 1. This would mean or 5 = 50 sin q - 2q
p 25 1
2a - q = or sin q = =
2 50 2
q p \ q = 30°
or a= +
2 4 1
47. Given, (KE) highest = (KE)
Maximum range up the inclined plane, 2
v2 v 2(1 - sin q) 1 1 1
Rmax = (1 - sin q) = mv 2 cos2 q = × mv 2
2
g cos q g (1 - sin 2 q) 2 2 2
1
v 2(1 - sin q) v2 cos2 q =
= = 2
g (1 - sin q) (1 + sin q) g (1 + sin q) 1
Þ cos q = Þ q = 45°
2
Telegram @unacademyplusdiscounts

Projectile Motion 143

48. As, v cos b = u cos a 1 2


Further, h = uyt - gt
2
(horizontal component of velocities are always equal) or 2
gt - 2uyt + 2h = 0
u cos a
\ v= 2uy + 4uy2 - 8gh
cos b
\ t1 =
2g
49. Given, R = H
2uy - 4uy2 - 8gh
u 2 sin 2a u 2 sin 2 a and t2 =
= 2g
g 2g
sin a 2 4uy2 - 8gh
or 2 sin a cos a = Dt = t1 - t 2 = d
2 g
sin a g 2( Dt ) 2
or = 4 or tan a = 4 or uy2 = + 2gh
cos a 4
\ a = tan -1( 4) Given, ux2 + uy2 = (2 gh) 2
50. At 45°, v x = v y 4h 2 g 2( Dt ) 2
\ 2
+ + 2gh = 4gh
or ux = uy - gt ( Dt ) 4
uy - ux g2
\ t= ( Dt ) 4 - 2gh( Dt ) 2 + 4h 2 = 0
g 4
40(sin 60° - sin 30° )
= = 1.5 s 2gh ± 4g 2h 2 - 4g 2h2 4h
9.8 ( Dt ) 2 = 2
=
g /2 g
51. Let a¢ be the angle of projection of the second body.
u h
or Dt = 2
g
u
53. v 2 = v y2 + v x2
or 5 g = (uy - gt ) 2 + ux2
α
β or 50 = (5 3 - 10 t ) 2 + (5) 2
u2 \ (5 3 - 10 t ) = ± 5
R= [sin(2a - b)]
g cos b
5 3 -5
Range of both the bodies is same. Therefore, t1 =
10
sin(2a - b) = sin(2a ¢ - b) 5 3 -5
or 2a ¢ - b = p - (2a - b) and t2 =
\ t1 - t 2 = 1 s
10
p Displacement
a ¢ = - ( a - b) 54. Average velocity =
2 Time
2u sin( a - b) 2u sin( a ¢ - b)
Now, T = and T ¢ = R2
g cos b g cos b H2 +
4 v av = …(i)
T sin( a - b) sin( a - b)
\ = = T /2
T ¢ sin( a ¢ - b) ìp ü
sin í - ( a - b) - bý y
î 2 þ
sin( a - b) sin( a - b)
= =
æa ö cos a H
sinç - a ÷
è2 ø x
R/2
52. Let Dt be the time interval. Then, v sin 2 q
2
Here, H = maximum height =
y 2g
v v 2 sin 2q
R = range =
uy g
θ 2v sin q
x and T = time of light =
ux 2h g
2h = (ux) ( Dt ) Substituting in Eq. (i), we get
2h v
or ux = …(i) v av = 1 + 3 cos2 q
Dt 2
Telegram @unacademyplusdiscounts

144 JEE Main Physics

55. Instantaneous velocity of rising mass after t s will be 1 2


\ s = ut + at
2
v t = v x2 + v y2
1
s = 0 ´ t + ( g sin 45° )t 02
where, v x = v cos q = Horizontal component of velocity 2
v y = v sin q - gt = Vertical component of velocity 9.8 2
or 9.8 2 = t0
v t = (v cos q) 2 + (v sin q - gt ) 2 2 2
\ t 02 = 4
v = v 2 + g 2 t 2 - (2v sin q) gt
\ t0 = 2 s
R /2 3H \ T = 2t 0 = 4 s
56. = = 3
H H
(v 02 sin q cos q)/ g
59. x = 36 t
or = 3 dx
(v 02 sin 2 q)/2 g \ vx = = 36 m/s
2 cot q = 3 dt
2 y = 48 t - 4.9 t 2
tan q =
3 \ v y = 48 - 4.8 t
-1 æ 2 ö at t = 0, v x = 36 m/s
or q = tan ç ÷
è 3ø
and v y = 18 m/s
57. From figure, æ vy ö æ 4ö
So, angle of projection q = tan -1 ç ÷ = tan -1 ç ÷
v0 sin è vx ø è3ø
v0
æ 4ö
θ or q = sin -1 ç ÷
g v0 cos θ è5ø

60. Maximum height and time of flight depend on the vertical


H component of initial velocity.
H1 = H2 Þ uy1 = uy 2
Ground Here T1 = T2
1 2 u 2 sin 2q
H = ( -v 0 sin q)t + gt Range R=
2 g
v x = v 0 cos q 2(u sin q) (u cos q) 2uxuy
= =
v y2 = (v 0 sin q) 2 + 2gHd g g
R2 > R1
v = v x2 + v y2 at ground
\ ux2 > ux1 or u2 > u1
v = v 02 + 2gH 61. Vertical component of velocity of A should be equal to
It means speed is independent of angle of projection. vertical velcotiy of B.
1 2 or v1 sin 30° = v 2
Also, gt = +H t v v1 v 1
2 or = v2 \ 2 =
From this where q increase, t increases. 2 v1 2

58. The time of ascent = time of descent = t 0 62. If it is being hit, then
1 2
T = total time of flight = 2t 0 d = v 0t + at = (u cos q)t
2
(Q acceleration in horizontal direction is zero)
9.8 m

g sin 45° Q
600 ms–1
45° H
A
9.8 9.8
sin 45° = = θ = 60°
BC s
d
\ s = 9.8 2
Telegram @unacademyplusdiscounts

Projectile Motion 145

u cos q - v 0 and acceleration of ball relative to lift is 12 ms -2 in negative


or t=
a /2 y- direction or vertically downwards. Hence, time of flight
1 2uy uy 2 1
600 ´ - 250 T= = = = s
2 12 6 6 3
\ t= =5 s
10 u
1
66. x = cot 30° = 3
H = (u sin q)t - ´ gt 2 uy
2
\ ux = 80 3 ms-1
3 1
= 600 ´ ´ 5 - ´ 10 ´ 25 2uy 2 ´ 80
2 2 T= = = 16 s
g 10
H = 2473 m
T
63. Since, the projectile is released its initial velocity is the same At t = = 4 s, v x = 80 3 ms-1
4
as the velocity of the plane at the time of release.
Take the origin at the point of release. v y = 80 - 10 ´ 4 = 40 ms-1
Let x and y( = - 730 m ) be the coordinates of the point on the
ground where the projectile hits and let t be the time when it \ v = (80 3) 2 + ( 40) 2 » 140 ms-1
hits. Then,
67. Acceleration of insect with respect to car a sc is a in the
1
y = - v 0 t cos q - gt 2 direction shown in figure. Absolute acceleration of insect is
2
a
where, q = 53.0°
θ s
This equation gives ac
1
y + gt 2
v0 = - 2
t cos q
1 a s = a sc + a c
-730 + (9.8) (5) 2 Component of a s along horizontal is a0 - a cos q and
= 2 = 202 ms-1
5 cos 53° perpendicular to screen is a0 sin q.

64. Horizontal component of velocity of A is10 cos 60° or 5 ms -1 68. Let v be the velocity of particle when it makes 30° with
which is equal to the velocity of B in horizontal direction. horizontal. Then,
They will collide at C if time of flight of the particles are equal y v
or t A = tB
2u sin q 2h æ 1 2ö 30°
= çQ h = gtB ÷
g g è 2 ø v cos 30°
30

2 x
°

æ 3ö g g cos 30°
2(10) 2ç ÷
2u 2 sin 2 q è 2 ø v cos 30° = u cos 60°
or h= = = 15 m
g 10
æ 1ö
(20) ç ÷
65. Components of velocity of ball relative to lift are u cos 60° è 2 ø 20
or v= = = ms-1
y cos 30° æ 3ö 3
ç ÷
è 2 ø
u = 4 ms–1 v2
Now, g cos 30° =
R
2
æ 20 ö
30° x 2 ç ÷
v è 2ø
or R= =
g cos 30° 3
ux = 4 cos 30° = 2 3 ms -1 (10)
2
and udy = 4 sin 30° = 2 ms-1 = 15.4 m
Telegram @unacademyplusdiscounts

146 JEE Main Physics

Round II
1. Shown figure are when projectile is at A, then dH 2du 1
\ = =2 ´
H u 10
A dH
u \ % increase in H = ´ 100
H
2
= ´ 100 = 20%
θ 10
O
R/2 B
5. At B, S y = 0
R 1 u2 1 (20 2) 2
OC = = sin 2q = ´ sin 2 ´ 45° y
2 2 g 2 10
= 40 m 10 m/s
u 2 sin 2 q x
AC = H = A
2g
(20 2) 2
= sin 2 45° = 20 m B
2 ´ 10
30°
\Displacement, OA = OC 2 + CA2 1
\ ay t 2 = 0
uyt +
= 40 + 202 2 2
2 uy - 2 (10) 4
Time of projectile from O to A or t =- = = s
ay -10 ´ 3 /2 3
1 æ 2u sin q ö u sin q
= ç ÷= 1 2
2è g ø 2g Now, AB = R = axt
2
(20 2) sin 45° 1æ 1 ö æ16 ö
= =2 s = ç10 ´ ÷ ç ÷ = 13.33 m
10 2è 2ø è 3 ø
Displacement
\ Average velocity = 6. Given, q1 = p / 3 = 30°
Time
Horizontal range is same if q1 + q2 = 90°
40 2 + 20 2
= \ q2 = 90° - 30° = 60°
2
u 2 sin 2 30°
= 10 5 ms-1 y1 =
2g
2. As initial velocity is zero. Particle will move in a straight line u 2 sin 2 60°
along anet . and y2 =
2g
a
2
y 2 sin 2 30° æ 1 / 4 ö 1
\ = =ç ÷ =
y1 sin 2 60° è 3 / 4 ø 2
1 y
or y2 =
3
g anet
7. Let v1 be the velocity of the car and v 2 be the velocity of the
2h 2 ´ 49 parcel. The parcel is thrown at an angle q from O, it reaches
Further, t= = = 10 = 3.16 s the mass at M.
g 9.8 M
3. According to law of conservation of linear momentum at the v2
highest point.
m m
mv cos q = ( -v cos q) + v1
2 2 O A
v1
or v1 = 3 v cos q v1 10
\ cos q = =
u 2 sin 2 q v 2 10 2
4. H =
2g 1
= = cos 45°
2u sin 2 q 2
dH = du
2g So, q = 45°
Telegram @unacademyplusdiscounts

Projectile Motion 147

1 or u cos q = 2g - g (\ t = 1 s)
8. As, h = gt 2
2 or u cos q = g …(ii)
2h Squaring and adding Eqs. (i) and (ii), we have
Distance d = ut = u
g u 2 = 5 g 2 = 5(10) 2 = 500

Þ d 2 = u2 ×
2h or u = 500 = 22.36 ms-1
g Dividing Eq. (i) by Eq. (ii), we have
d tan q = 2 or q = tan -1(2).

13. Potential v ( x) versus x is parabolic. SHM starts for extreme


h position and x versus t should be cosine curve.
v 2 sin 2 60° v 2 3 3 v 2
14. Maximum height, H = = ´ =
2g 2g 4 8g
Momentum of particle at highest point
9. For 5 s weight of the body is balanced by the given force.
Hence, it will move in a straight line as shown. mv
p = mv cos 60° =
2
mv 2 v 2 3 mv 2
Angular momentum = pH = ´ =
2 8g 16 g
2u sin q
15. Time of flight, t =
g
5s
u 2 sin 2q
2
u sin 2 q Horizontal range, R =
R= + (u cos q) (5) g
g
Change in angular momentum,
(50) 2 × sin 60° |dL| = (L f - Li ) about point of projection
= + (50 ´ cos 30° ) (5)
10
u 2 sin 2q
= 250 3 m = (mu sin q) ´
g
1 mu3 sin q sin 2q
10. h = (u sin q)t - gt 2 =
2 g
d
d = (u cos q)t or t = Change in angular momentum
u cos q Torque | t| =
Time of flight
2
d 1 d dL mu3 sin q 2 sin q × cos q g
h = u sin q × - g× 2 = = ´
u cos q 2 u cos2 q T g 2u sin q
d g 1
u= or 2
t = mu sin 2 q
cos q 2(d tan q - h) 2
16. We know that the range of projectile projected with velocity
u 2 sin 2q 2 ux uy u, making an angle q with the horizontal direction up the
11. R = = inclined plane, whose inclination with the horizontal
g g
direction is q0 , is
\Range µ horizontal initial velocity component (v x)
u2
In path 4 range is maximum of football has maximum R= [sin(2q - q0) - sin q0 ]
g cos2 q0
horizontal velocity component in this path.
Here, u = v , q = (90° + q), q0 = q
12. Time of flight of this particle, T = 4 s. If u is its initial speed and
v2
q is the angle of projection, then \ R= {sin[2(90° + q)] - q} - sin q}
g cos2 q0
2u sin q
T=4= v2
g = [sin(180° + q) - sin q]
g cos2 q0
or u sin q = 2g …(i)
After 1 s, the velocity vector particle makes an angle of 45° v2 2u 2
=- 2 sin q = - tan q sec q
with horizontal, so g cos2 q0 g
vx = vy 2v 2
= tan q (in magnitude)
i. e. , u cos q = (u sin q) - gt g
Telegram @unacademyplusdiscounts

148 JEE Main Physics

17. Let v be the velocity acquired by the body at B which will be æ dy ö


and ç ÷ = a = tan q
moving making an angle 45° with the horizontal direction. As è dx ø x = 0
v2 where, q = angle of projection
the body just crosses the well so = 40
g \ q = tan -1( a).
or v 2 = 40 g = 40 ´ 10 = 400
21. At initial point, v x = v cos q and v y = v sin q. At second point,
or v = 20 ms-1
where particle moves at right angle to its direction, let its
Taking motion of the body from A to B along the inclined velocity be v¢. Then, v ¢x = v sin q = v x = v cos q
plane, we have cos q
10 Þ v¢ = cot q
u = v 0 , a = - g sin 45° = - ms-2, sin q
2 Since, v ¢y = gt
s = 20 m, v = 20 ms-1 v y - v ¢y
or t=
As v 2 = u 2 + 2as g
æ 10 ö v sin q - v ¢ cos q v sin q - v cos q × cos q
\ 400 = v 02 + 2 ç - ÷ ´ 20 2 \ t= =
è 2ø g g
or v 02 = 400 + 400 = 800 v sin q v
= (1 - cot q) =
g g sin q
or v = 20 2 ms-1
22. For qA = 15° and qB = 75° , RA = RB
18. Given, condition h1 = h2
UA = UB ,
Þ u12 sin 2 45° = u22 sin 2 q 2
2 2
hA uA sin qA æ sin 15° ö
u2 1 1 1 But = =ç ÷ <1
Þ 2
sin q = 12 sin 2 45° = × = hB uA2 sin 2 qB è sin 75° ø
u2 2 2 4
1 or hA < hB
Þ sin q = TA uA sin qA
2 Again, =
TB uB sin qB
Þ q = 30°
sin 15°
19. If a particle is projected with velocity u at an angle q with the = <1
sin 75°
horizontal, the velocity of the particle at the highest point is
or TA < TB
v = u cos q = 200 cos 60° = 100 ms-1
v 02 2
sin q
If m is the mass of the particle, then its initial momentum at 23. Height, h = i. e. ,h µ sin 2 q
2g ,
highest point in the horizontal direction = mv = m ´ 100. It
means at the highest point, initially the particle has no h1 sin 2 q1
\ = >1
momentum vertically upwards or downwards. Therefore, h2 sin 2 q2
after explosion, the final momentum of the particles going
upwards and downwards must be zero. Hence, the final So, sin 2 q1 > sin 2 q2
momentum after explosion is the momentum of the third or q1 > q2
particle, in the2horizontal
v sin q direction. If the third particle moves 0
mv¢ Time of flight, T =
with velocity v¢ , then its momentum . According to law of g
3
mv ¢ or T µ sin q
conservation of linear momentum, we have = m ´ 100 or
3 T1 sin q1
\ = >1
v¢ = 300 ms-1. T2 sin q2

20. y = ax - bx2 or T1 > T2


u 2 sin 2q
For height of y to be maximum Horizontal range, R=
g
dy
=0 or R µ sin 2q
dx
or a - 2bx = 0 R1 sin 2q1
\ = £1
a R2 sin 2q2
or x=
2b or R1 £ R2
2
æ aö æ aö a2 Total energy of each particle will be equal to KE of each
\ y max = a ç ÷ - bç ÷ =
è 2b ø è 2b ø 4b particle at the time of its projection.
Telegram @unacademyplusdiscounts

Projectile Motion 149

24. Horizontal range is same when angle of projection is q and According to question,
(90° - q) v y 10
tan 30° = =
u 2 sin 2 q 2 u 2 sin q cos q ux ux
\ R= =
g g 10
or ux = = 10 3 ms-1
When angle of projection is q, then tan 30°
2 u sin q
T1 = \ u = ux2 + uy2 = (10 3) 2 + (30) 2 = 20 3 ms-1
g
uy 30
When angle of projection is (90° - q), then 30. tan q = = = 3 = tan 60°
2 u sin (90° - q) 2 u cos q ux 10 3
T2 = =
g g \ q = 60°
4 u 2 sin q cos q æ 2 u 2 sin q cos q ö æ 2 ö 2 R u 2
u sin 2 30° u 2
2
\ T1 T2 = =ç ÷ç ÷= 31. h1 = , h2 = =
g2 è g ø ègø g 2g 2g 8g
\ T1 T2 µ R 32. Velocity is horizontal and acceleration is vertical.
T1 2 u sin q / g
and = = tan q 33. Assertion v sin a = horizontal component of velocity
T2 2 u cos q / g
= constant
25. Since the projectile has two component velocities, i. e. , a
Reason ar = g 2 - at2
horizontal velocity which is constant without air friction and a
vertical velocity which changes with air friction and gravity At highest point at = 0.
pull, hence the path of projectile is a parabolic path. Thus, ar is maximum.
Due to air friction, the energy spent by projectile against air
34. v = velocity, w = weight and A = air resistance.
friction is not conserved, so the total energy of projectile is not
u u
conserved.
At the highest point, the projectile has horizontal component
velocity only.
26. Here, u = 2 gh and q = 60°
w 1A w1 A
u 2 sin 2 q ascent descent
Maximum range =
g
u 2 sin 2 q
4 gh ´ 3
35. As, H =
= = 2h 3 2g
2g 2 u sin q gT
and T= \ u sin q =
27. It is equal to half of the time of flight g 2
T 2 u sin q æ g 2 T 2 / 4 ö gT 2
\ t= = \ H=ç ÷=
2 2g è 2g ø g
2 ´ 2 gh ´ 3 3h
Þ t= = 36. Initial velocity v = ( $i + 2$j) m/s
2 ´2 g g
Magnitude of velocity v = (1) 2 + (2 2) = 5 m/s
28. From v 22 = v12 - 2 as,
Equation of trajectory of projectile
0 2 = ( 4 sin q) 2 - 2 gs
gx2 æ y 2 ö
2 y = x tan q = (1 + tan 2 q) ç tan q = = = 2÷
æ 3ö 2u 2 è x 1 ø
Þ ç2 gh × ÷ =2g s
è 2 ø 10 ( x) 2
3h \ y = x´2 - [1 + (2) 2]
Þ 3 gh = 2 g s \ s = 2 ( 5) 2
2
10 ( x2)
2uy = 2x - (1 + 4) = 2x - 5x2
29. Time of flight, T = 2 ´5
g
gT 10 ´ 6 37. As their period of revolution is same, so their angular speed.
uy = = = 30 ms-1 Centripetal acceleration is circular path, a = w2r
2 2
Vertical velocity after 2s,v y = uy - gT a1 w2r1 r1
Then, = =
= 30 - 10 ´ 2 a2 w2r2 r2
= 30 - 230 = 10 ms-1
Telegram @unacademyplusdiscounts

150 JEE Main Physics

38. Maximum speed with which the boy can throw stone is 44. s = t 3 + t
u = 2 gh = 2 ´ 10 ´ 10 = 10 2 ms -1 ds
\ Speed, v = = 3t2
dt
\Range is maximum when projectile is thrown at an angle of
dv
45°. and rate of change of speed, at = = 6t
dt
u 2 (10 2) 2
Thus, Rmax = = = 20 m \ Tangential acceleration at t = 2 s,
g 10
at = 6 ´ 2 - 12 ms–2
39. Angular momentum of the projectile
and at t = 2 s,v = 3 (2) 2 = 12 ms–1
L = mv hr^ = m (v cos q) h
(where h is the maximum height) v 2 144
\Centripetal acceleration, ac = = ms–2
æ v 2 sin 2 q ö R 20
Þ = m (v cos q) ç ÷
è 2g ø \ Net acceleration = at2 + ai2 » 14 ms–2
mv3 sin 2 q cos q 3 mv3 u2
L= = 45. Rmax = = 1000 m. In this case, q = 45°
2g 16 g g
40. Maximum range of water coming out of the fountain u 2 sin 2 45°
2 \ Maximum height =
v 2g
Rm =
g 1
2
æ 1ö
\Total area around fountains = ´ 1000 ´ ç ÷ = 250 m
2 è2ø
2 v4
A = pRm =p 46. Given, initial velocity = v 0
g2
Final velocity = 0
41. Given, x = 0.20 m,y = 0.20 m,u = 1.8 ms–1
Deceleration a = - a x2 ...(i)
Let the ball strike the nth step of stairs,
1 2 Let the distance travelled by the particle be s.
Vertical distance travelled = ny = n ´ 0.20 = gt
2 Now, we know that
Horizontal distance travelled, nx = ut dv dv dt v dv
nx a= = ´ =
or t= dt dt dx dx
u dv
1 n 2x2 or a=v ...(ii)
\ ny = g ´ 2 dx
2 u
From Eqs. (i) and (ii),
2 u 2 y 2 ´ (1.8) 2 ´ 0.20 dv
or n= = = 3.3 » 4 v = - ax2
g x2 9.8 ´ (0.20) 2 dx
42. Kinetic energy = potential energy or v dv = - ax2dx
1 mgh On integrating with limit v 0 ® 0 and 0 ® s
m (kv e) 2 =
2 h 0 s
1+ v dv = ò - ax2dx
R òx 0 0

1 2 mgh Rk2 æv2ö


0
æ x3 ö
s
Þ mk 2gR = Þ h=
2 h 1 - k2 or ç ÷ = -aç ÷
1+ è 2 øn è 3 ø0
R
Height of projectile from the earth’s surface = h -v 02 a ( s)3
=-
Rk2 2 3
Height from the centre r = R + h = R +
1 - k2 v 02 as3
R =
By solving, r = 2 3
1 - k2
3 v 02
43. L = m( r ´ v) = s3
2a
1/ 3
é 1 ù æ3 v2 ö
L = m êv 0 cos qt $i + (v 0 sin qt - gt 2 $j ú s = ç 0÷
ë 2 û è 2a ø
´ [v 0 cos q$i + (v 0 sin q - gt ) $j ] 47. In angular projection, the body at the highest point has
æ 1 ö 1 velocity = u cos q in the horizontal direction which makes
= mv 0 cos qt ç - gt ÷ k$ = - mgv 0 t 2 cos q k$
è 2 ø 2 zero angle with the horizontal direction.
Telegram @unacademyplusdiscounts

Projectile Motion 151

48. Let g ¢ be the acceleration due to gravity on the planet. As per 2 u cos q
t2 =
question, horizontal range is same, so g
u12 sin 2q u22 sin 2q 2 u sin q 2 u cos q
= t1 t 2 = ×
g g¢ g g

u12 u22 2 u 2 sin 2 q 2R


or = t1 t 2 = or t1 t 2 =
g g¢ g g
1
u22 32 52. Initial KE, E = mu2
or g¢ = 2
g = 2 ´ 10 = 3.6 ms-2 2
u1 5
u
At the highest point, velocity v = u cos 60° =
49. Let q1 and q2 be the angle of projection of two particles A and B 2
2
and u be their velocity of projection. As per question 1 1 æuö 1 æ1 ö E
KE at highest point = mv 2 = m ç ÷ = ç mu 2÷ =
H1 u 2 sin 2 q1 / 2g sin 2 q1 3 2 2 è2ø 4 è2 ø 4
= = =
H2 u 2 sin 2 q2 / 2g sin 2 q1 1 u 2 sin 2 60°
PE at highest point = mg ´
sin q1 2g
or = 3 = tan 60° 2
sin q2 1 æ 3ö 3
= mu 2 ç ÷ = E
sin 60° sin 60° 2 è 2 ø 4
= =
cos 60° sin 30°
u 2 sin 2 q
\ q1 = 60° and q2 = 30° 53. Maximum height, H =
2g
When speed of A is made then
u 2 sin 2q
R1 (2u) 2 sin 2q1 4 sin 2 ´ 60° Range, R=
= = g
R2 u 2 sin 2q2 /g sin 2 ´ 30°
R
4 sin 120° 4 ´ ( 3/2) Given, H=
= = =4 2
sin 60° ( 3/2) u 2 sin 2 q u 2 sin q cos q
\ =
\ R1 : R2 = 4 : 1 2g 2g
50. Let a body be projected at a velocity u at an angle q with or sin q = 2 cos q
horizontal. Then, horizontal range covered is given by or tan q = 2
a2 sin 2 q or q = tan -1(2)
R=
g u
54. At the two points of the trajectory during projection, the
C
and height H is horizontal component of the velocity remains same.
u 2 sin 2 q θ
H
ìhorizontal component of ü ìhorizontal component of ü
H= A
í ý =í ý
2g B
î velocity at angle 60° þ î velocity at angle 45° þ
R
Given, R = 3H i. e. , u cos 60° = u cos 45°
u 2 sin 2 q u 2 sin2 q 1 1
\ =3´ 147 ´ = v ´
g 2g 2 2
Also, sin 2 q = 2 sin q cos q 147 -1
or v= ms
u 22 sin q cos q u2 sin q 2
\ =3´
g 2g Vertical component of u = v sin 60°
or 2 cos q = 1.5 sin q 147 3
2 = m
or tan q = = 1.33 or q = 53°7 ¢ 2
1.5
Vertical compound of u = v sin 45°
Hence, angle of projection is 53°7 ¢. 147 1 147
= ´ = m
51. If the horizontal range is the same, then the angle of project of 2 2 2
an object is q or (90° - q) with the horizontal direction. So, the But v y = uy + at
angle of projection of first particle is q and the other particle is
(90° - q) 147 147 3
\ = - 9.8t
2 u sin q 2 2
t1 = 147
g or 9.8t = ( 3 - 1)
2 u sin (90° - q) 2
t2 =
g \ t = 5.49 s
Telegram @unacademyplusdiscounts

5 Circular Motion
JEE Main MILESTONE
< Kinematics of Circular Motion < Forces in Circular Motion
< Uniform Circular Motion < Conical Pendulum
< Non-uniform Circular Motion < Motion in a Verticle Circle
< Relation between Linear and Angular Variables

5.1 Kinematics of Circular Motion


For a particle in circular motion, following variables are needed to describe its
motion.

Angular Position
The position of the particle P at a given instant may be y
described by the angle q between OP and OX (reference When a particle moves in a
P' circular path, then its motion is
line). This angle q is called angular position.
θ P said to be circular motion. Circular
As the particle moves on the circle, its angular position q
θ x motion is a two dimensional
changes. O r
motion. When the speed of the
Here, P and P ¢ is given as P (r, q) and P ¢ (r, q + Dq)
particle performing circular
respectively.
motion is constant, then its
motion is said to be uniform
Angular Displacement
circular motion, if the speed of the
Referring to the above figure, angle between initial and final positions of particle, particle performing circular
which it tends on axis of rotation, is called angular displacement. In figure OP is the motion is changing, then its
initial and OP ¢ is the final position of particle. Then, angular displacement
non-uniform
ÐP
motion said=to
is¢OP Dqbe circular motion.
It is dimensionless (as q = l / r ). SI unit is radian while practical unit is degree or
revolution
2 p rad = 360° = 1 rev
If a body makes n revolutions, its angular displacement q = 2 pn radian.

Note Angular displacement is a vector quantity, provided Dq is small. Commutative law of vector
addition is not valid for large Dq .

Angular Velocity
If the angular position of a particle changes with time it is said to have angular
velocity. If q1 and q2 are the angular position of a particle at time t1 and t2 respectively,
the average angular velocity wav for this time interval is defined as
Telegram @unacademyplusdiscounts

Circular Motion 153

q2 - q1 Dq Thus, if a particle moves in a circle of radius r with an


wav = =
t2 - t1 Dt v2
uniform speed v, then its acceleration is . This
And instantaneous angular velocity is defined as the r
limiting value of this ratio acceleration is termed as centripetal acceleration.
Dq dq
i.e., w = lim = Sample Problem 1 The linear acceleration of a particle
Dt ® 0 D t dt
which completes a revolution in a circle of radius 10 cm with
In case of uniform circular or rotational motion, constant speed, in 4 s is
w = wav (a) p 2cms-2 (b) 1.5 p 2cms-2
(c) 2 p 2cms-2 (d) 2.5 p 2cms-2
It is also a vector quantity. w is also known as angular
frequency. Its unit is rads–1, rpm, rps etc. Interpret (d) Distance travelled in one revolution
Note = 2 pr = 2 p ´ 10 cm
2 pr 2 p ´ 10
(i) About different points angular velocity of a particle is different. Linear speed, v = = = 5 p cms-1
(ii) If a particle makes n rotations in t second. t 4
2 pn ù Linear acceleration
wav = é rads -1
êë t úû v 2 (5 p ) 2
a= = = 2.5 p 2 cms-2
Hence, if T is the time period and f is the frequency of uniform r 10
circular motion.
2 p ´1 éQf = 1 ù
w= = 2 pf
êë T ûú
T
(iii) Angular velocity depends on the point about which rotation is
5.3 Non-uniform Circular Motion
considered. If speed of a particle moving in a circle is not constant, then
the particle has both radial and tangential components of
Angular Acceleration (a ) acceleration.
dv/dt
If the angular speed of a particle is variable, the body has
an angular acceleration. Let w1 and w2 be the
instantaneous angular speeds at times t1 and t2 a
respectively, then the average angular acceleration a av is α
defined as O v2
w - w1 Dw r
a av = 2 =
t2 - t1 Dt

The instantaneous angular acceleration is the limit of this


ratio as Dt approaches zero, i.e., ar = Radial component
Dw dw d 2q v2
a inst = lim = = 2 = w2r =
Dt ® 0 D t dt dt r
The SI unit of angular acceleration is rad s–2. and at = Tangential component =
dv
If a = 0, circular motion is said to uniform. dt
It is having same characteristics as that of angular velocity. Magnitude of net acceleration,
a = ar2 + at2

5.2 Uniform Circular Motion or


æ v2 ö
a= ç ÷ +ç
2
æ dv ö
÷
2

è r ø è dt ø
If a particle in circular motion moves with an uniform
speed, then motion of the particle is called uniform circular This resultant acceleration makes an angle a with the
motion. In such a case, radius, where
dv é v2 ù
=0 ê ú
dt a r
tan a = t = ë û
and a = w2r ar é dv ù
v2 v2 ê dt ú
or a= r= ë û
2
r r
Telegram @unacademyplusdiscounts

154 JEE Main Physics

Note (ii) The period of revolution (T) for the motion of each
(i) In accelerated circular motion dv /dt is positive, tangential 2p
point and for the rigid body itself is given by T = .
acceleration of the particle is parallel to velocity v. w
(ii) In decelerated circular motion, dv /dt is negative and hence,
(iii) When a particle is moving along a curved path, then its
tangential acceleration is anti-parallel to velocity v.
tangential and angular velocities are related by
(iii) Regarding circular motion, following possibilities exist
v = w ´ r, where r is the vector joining the location of
(a) If ar = 0 and at = 0, then a = 0 and motion is uniform
particle and the point about which w has been
translatory.
computed. In other way, we can write, v = rw, where v is
(b) If ar = 0 and at ¹ 0, then a = at and motion is accelerated
translatory. the component of velocity perpendicular to r or we
(c) If ar ¹ 0 but at = 0, then a = ar and motion is uniform can say vt is the tangential velocity.
circular. (iv) If we differentiate above equation, we get,
(d) If ar ¹ 0 and at ¹ 0, then a = at2 + ar2 and motion is dv dw
= ´r
non-uniform circular. dt dt
i. e., at = a ´ r
Sample Problem 2 An insect trapped in a circular groove of
radius 12 cm moves along the groove steadily and completes where a t is the tangential component of acceleration
7 revolutions in100 s. The magnitude of acceleration is given by which is responsible for changing the magnitude of
velocity.
(a) 1.3 cms-2 (b) 2.3 cms-2
(c) 4.3 cms-2 (d) 6.3 cms-2
Sample Problem 4 A particle moves in a circular path of
Interpret (b) The acceleration is directed towards the centre of radius 0.5 m with a linear speed of 2 ms–1, find its angular
the circle. Since, this direction changes continuously acceleration speed.
here is not a constant vector. However, the magnitude of
(a) 12 rad s–1 (b) 3 rad s–1
acceleration is constant.
(c) 4 rad s–1 (d) None of these
a = w2R
2 p 2 p ´7 Interpret (c) The angular speed is
w= = = 0.44 rad/s
T 100 v 2
w= = = 4 rad s–1
Therefore, a = (0.44) 2 (12 cm) = 2.3 cms–2 r 0.5

Sample Problem 3 The speed of a particle moving in a Sample Problem 5 A particle moves in a circle of radius
circle of radius r = 2 m varies with time t as v = t 2 where, t is in 0.5 m with a speed that uniformly increases. Find the angular
second and v in ms–1. The net acceleration at t = 2 s is acceleration of the particle, if its speed changes from 2.0 ms–1
(a) 40 ms-2 (b) 60 ms-2 (c) 80 ms-2 (d) 10 ms-2 to 4.0 ms–1 in 4.0 s.
(a) 1 rad s–2 (b) 2 rad s–2
Interpret (c) Linear speed of particle at t = 2 s is (c) 3 rad s–2 (d) 4 rad s–2
v = (2) 2 = 4 ms-1
Interpret (a) Tangential acceleration of the particle is
v 2 ( 4) 2
\ Radial acceleration, ar = = = 8 ms–2 at =
dv 4.0 - 2.0
= = 0.5
r 2 dt 4.0
The tangential acceleration is a
dv The angular acceleration is a = t
at = = 2t r
dt 0.5
= = 1 rad s–2
At t = 2 s at = (2) (2) = 4 ms-2 0.5
Net acceleration of the particle at t = 2 s is
a = ( ar ) 2 + ( at ) 2 = (8) 2 + ( 4) 2 or a = 80 ms-2
5.5 Forces in Circular Motion
In circular motion of an object two kinds of forces occur
5.4 Relation between Linear and which are described below
Angular Variables
(i) If a reference line on a rigid body rotates by an angle q,
Centripetal Force
a point within the body at a position r from the rotation When a body moves along a circular path with an uniform
axis moves a distance s along a circular arc, where s is speed, its direction changes continuously i. e. , velocity
given by keeps on changing on account of a change in direction.
s = qr
According to Newton’s first law of motion, a change in the
The angle q must be measured in radian.
Telegram @unacademyplusdiscounts

Circular Motion 155

direction of motion of the body can take place only if some Centripetal Force in Different Situations
external force acts on the body.
Situation The centripetal force
v
A particle tied to a string and Tension in the string
whirled in a horizontal circle
Fc Vehicle taking a turn on a level Frictional force exerted by the
O m road road on the tyres
A vehicle on a speed breaker Weight of the body or a
component of weight
Revolution of earth around the Gravitational force exerted by
sun the sun
Thus, a particle performing circular motion is acted upon
Electron revolving around the Coulomb attraction exerted
by a force directed along the radius towards the centre of nucleus in an atom by the protons on electrons
the circle. This force is called the centripetal force.
A charged particle describing a Magnetic force exerted by the
If m is the mass of the particle, the magnitude of circular path in a magnetic field magnetic field
centripetal force is given by
Centripetal force = mass ´ centripetal acceleration Centrifugal Force
æ v2 ö
\ F =mç ÷ (in magnitude) “Centrifugal force can be defined as the radially directed
è r ø outward force acting on a body in circular motion, as
or F = mrw2 observed by a person moving with the body.”
v
In magnitude and direction, centripetal force is given by
mv2
F=- $r m
r O
Centrifugal
T
or = -mw2$r force on body

mg
Important Points Centrifugal force = Mass ´ centrifugal acceleration
1. In non-uniform circular motion, the particle simultaneously mv2
or F = = mrw2
possesses two forces r
mv2 or F = mvw
Centripetal force, Fc = mac = = mrw2
r This can be written in vector form as
Tangential force, Ft = mat mv2
F= r$
\ Net force, Fnet = ma = m a2c + a2t r

2. If a moving particle comes to stand still, i .e ., the particle will move where $r is the unit vector acting along r.
along the radius towards the centre and if radial acceleration ar is (i) In an inertial frame, the centrifugal force does not act
zero, the body will fly off along the tangent. So, a tangential velocity on the object.
and a radial acceleration (hence force) is a must for uniform circular (ii) In non-inertial rotating frames, pseudo force arises as
motion. centrifugal force and need to be considered.
mv2
3. Since, F = ¹ 0, so the particle is not in equilibrium and linear
r Circular Turning of Roads
momentum of the particle moving on the circle is not conserved.
It is most popular application of circular motion. When
But as the force is central, i.e.,t = 0 so, the angular momentum is
vehicles go through turns, they travel along a nearly
conserved i.e., p ¹ constant but L = constant.
circular arc. There must be some force which will produce
4. The work done by the centripetal force is always zero as it is the required centripetal acceleration. If the vehicles travel
perpendicular to velocity and displacement. Further, by on a horizontal circular path, this resultant force is also
work-energy theorem horizontal. The necessary centripetal force is being
Work done = change in kinetic energy provided to the vehicles by following three ways
\ DK = 0 (Q DW = 0 ) 1. By friction only,
i.e., K (Kinetic energy) remains constant. 2. By banking of roads only,
3. By friction and banking of roads both.
Telegram @unacademyplusdiscounts

156 JEE Main Physics

In practical, the necessary centripetal force is provided by Applying Newton’s second law along the radius and
friction and banking of roads both. Now, let us write the first law in the vertical direction.
equations of motion in each of the three cases separately N
and see what are the constraints in each case. θ
mv2 G
1. By friction only Suppose, a car of mass m is moving r
with a speed v in a horizontal circular arc of radius r. In
this case, the necessary centripetal force to the car will
be provided by force of friction f acting towards centre.
θ
v
mg
F mv2
N sin q =
r r
and N cosq = mg
From these two equations, we get
v2
tan q = ...(i)
2 rg
mv
Thus, f =
r or v = rg tan q …(ii)
Further, limiting value of f is mN 3. By friction and banking of roads both If on a banked
or f L = mN = m mg (\ N = mg ) circular turning, there is a frictional force between car
Therefore, for a safe turn without sliding and road, then the vector sum of normal reaction force
and frictional force provides the necessary centripetal
mv2
£ fL force
r
Centre of Bank r
mv2
or £ m mg
r
v2 N
N cos θ
or m³
rg N θ
or v £ m rg θ N sin
f cos θ
Here, two situations may arise. If m and r are known to
us, the speed of the vehicle should not exceed mrg h
f f
and if v and r are known to us, the coefficient of friction
v2
θ mg f sin θ
should be greater than . θ
rg b mg
(a) (b)
Note
(i) You might have seen that if the speed of the car is too high, car mv2
N sin q + f cos q = …(i)
starts skidding outwards. With this radius of the circle increases r
or the necessary centripetal force is reduced
N cos q = mg + f sin q …(ii)
æ centripetal force µ 1ö .
ç ÷
è rø (Q vertical force is balanced)
(ii) If the real width is b and the outer end is raised by h relative to the Taking limiting condition, we can write
h f = ms N …(iii)
inner one, then tanq = .
b
To obtain the value of N, Solve above three equations
2. By banking of roads only Friction is not always properly.
reliable at circular turns, if high speed and sharp mg
N=
turns are involved. To avoid dependence on friction, cos q - m s sin q
the roads are banked at the turn so that the outer After putting the value of N in Eq. (i), we get
part of the road is somewhat raised compared to the 1/ 2 1/ 2
inner part. é rg (sin q + m s cos q) ù é rg (m s + tan q) ù
vmax = ê ú =ê ú
ë cos q - m s sin q û ë 1 - m s tan q û
Telegram @unacademyplusdiscounts

Circular Motion 157

Sample Problem 5 A cyclist speeding at 18 km/h on a mv 2


we have, N sin q + f cos q = …(i)
level road takes a sharp circular turn of radius 3 m without r
reducing the speed. The coefficient of static friction between N cos q = mg + f sin q …(ii)
the tyres and the road is 0.1. Then, which one of the following Taking limiting condition
statement is true regarding the motion
f = m sN …(iii)
(a) While taking the circular turn cyclist will slip
To obtain the value of N solving with Eqs. (i), (ii) and (iii), we get
(b) While taking the circular turns cyclist will not slip
mg
(c) Frictional force is sufficient to provide necessary N= …(iv)
centripetal force cos q - m s sin q
(d) None of the above Putting the value of N from equation (iv) in equation (i) for
maximum permissible speed, we get
Interpret (a) On an unbanked road, frictional force alone can
provide the centripetal force needed to keep the cyclist moving on a v2 = Square of maximum permissible speed
1 /2
circular turn without slipping. In order to solve this problem, é rg (m s + tan q) ù
following steps should be remembered. =ê ú
ë (1 - m s tan q) û
To make a circular turn the required centripetal force is provided by
Now, putting the values in given in the question, i. e., r = 300 m,
the friction as,
mv 2 q = 15°, g = 9.8 ( » 10) ms–2 and m s = 0.2
f= ...(i)
r We obtain, v max = 38.1 ms–1
where, m = mass of the cyclist.
For limiting value of f Sample Problem 7 The driver of a car running with a
f = m sN = m mg …(ii) velocity v suddenly notices a blockade infront of him at a
Thus, for the safe turn without skiding distance s. What will be better option for him?
mv 2 (a) Apply brakes
£f (b) Make a circular turn without applying brakes in order to
r
2 just avoid crashing into the blockade
mv
Þ £ m mg (c) He can choose any one of them because both options are
r same
v2 (d) None of the above
Þ ms ³
rg
Interpret (a) First we see what happens if the driver stops the
or v 2 £ m srg …(iii)
car by applying brakes at a distance s,
Clearly, maximum value of v 2 is given by 1
FBrake ´ s = mv 2
v 2 = m srg …(iv) 2
Putting the given values, i. e. ,r = 3 m, g = 9.8 ms –2
and m s = 0.1in mv 2
i. e. , FBrake = …(i)
Eq. (iv), we get 2s
v 2 = 0.1 ´ 3 ´ 9.8 m2s–2 = 2.94 m2s–2 And, now we see if the driver takes a turn to avoid the collision, the
2
The obtained value of v is smaller than actual squared speed of force required will be
2
cyclist, i. e. ,15 m2s–2. mv
FTurn = …(ii)
s
So, cyclist will slip while taking the circular turn.

Sample Problem 6 A circular race track of radius 300 m is From Eqs. (i) and (ii). It is clear that
banked at angle of 15°. If the coefficient of friction between the 1
FBrake = FTurn
wheels of a race-car and the road is 0.2, then the maximum 2
permissible speed to avoid slipping is So, it is better to apply brakes than to take turn to avoiding crash
(a) 28.1 ms–1 (b) 50 ms–1 with the blockade.
(c) 38.1 ms–1 (d) 42 ms–1
Sample Problem 8 Find the maximum speed at which a
Interpret (c) On a banked road, the horizontal component of
car can turn round a curve of 30 m radius on a level road, if the
the normal force and the frictional force contribute to provide
centripetal force to keep the car moving on a circular turn without coefficient of friction between the tyres and the road is 0.4.
slipping. At the optimum speed, the normal reactions component is (acceleration due to gravity = 10 ms–2)
enough to provide the needed centripetal force and the frictional (a) 12 ms–1 (b) 10 ms–1
force is not needed. (c) 11 ms–1 (d) 15 ms–1
Telegram @unacademyplusdiscounts

158 JEE Main Physics

Interpret (c) We know that for turning, centripetal force is Dividing Eq. (ii) by Eq. (i), we get
provided by friction, so rw2
2 tan q =
mv g
£ fL
r g tan q
As, fL = mN = m mg (where, N = mg ) i. e. , w= …(iii)
r
mv 2 2p
Thus, £ m mg But, r = l sin q and w =
r t
i. e. ,v £ m gr , so that v max = m gr t being period of completing one revolution,
Here, m = 0.4,r = 30 m and g = 10 ms–2 2p g tan q
\ =
So, v max = 0.4 ´ 30 ´ 10 = 11 ms –1 t l sin q
l sin q
This gives, t =2p
Sample Problem 9 A turn of radius 600 m is banked for a g (sin q / cos q)
vehicle of mass 200 kg going with a speed of 180 kmh–1. Find
l cos q æ hö
the banking angle of its path. or Period, t = 2 p = 2p ç ÷
(a) q = 22.6° (b) q = 30°
g è gø
(c) q = 20.6° (d) q = 30.6° where, h = ON = l cos q
Interpret (a) The turn is banked for speed
5 –1
Sample Problem 10 A hemispherical bowl of radius R is
v = 180 kmh–1 = 180 ´ ms = 50 ms–1 rotating about its axis of symmetry which is kept vertical. A
18
small ball kept in the bowl rotates with the bowl without
v 2 50 ´ 50 slipping on its surface. If the surface of the bowl is smooth and
\ tan q = =
rg 600 ´ 10 the angle made by the radius through the ball with the vertical
25 is a, find the angular speed at which bowl is rotating.
= = 0.417 = tan 22.6°
60 R g
(a) w = (b) w =
g cos a cos a
g
5.6 Conical Pendulum (c)
R cos a
(d) None of these

It consists of a string OA, whose upper end O is fixed and Interpret (c) Let w be the angular speed of rotation of the bowl.
bob is tied at the other free end. The bob is given a Two forces are acting on the ball.
horizontal push through a small angular displacement q (i) Normal reaction, N (ii) Weight, mg
and arranged such that the bob describes a horizontal
The ball is rotating in a circle of radius, r = (R sin a) with centre at A
circle moving with an uniform angular velocity w in such a with an angular speed, w
way that the string always makes an angle q with the
Hence, N sin a = mrw2
vertical. As the string traces the surface of the cone, the
arrangement is called a conical pendulum. N sin a = mRw2 sin a …(i)
and N cos a = mg …(ii)
O
ω
θ
T
h l T cos θ
θ
T sin θ
N A
r R N

mg r A

mg
Let T be the tension in the string of length l and r the radius
of circular path. The vertical component of tension T Dividing Eq. (i) by Eq. (ii), we get
balances the weight of the bob and horizontal component 1 w2R
=
provides the necessary centripetal force. cos a g
Thus, T cos q = mg …(i) g
2 \ w=
and T sin q = mrw …(ii) R cos a
Telegram @unacademyplusdiscounts

Circular Motion 159

(iii) Tension at the lowest point A


Check Point 1 At, A h=0 (Q q = 0° )
m 2
\ TA = (u + gR) …(iv)
1. A stone tied to a string is revolved in a horizontal circle, with R
uniform speed v. If string breaks suddenly, then how will the
(iv) Tension at the highest point C
stone move?
At C, h = 2R (Q q = p )
2. A train moves on an unbanked circular bend of rails. Which rail
m 2
will wear out faster (inner rail or outer rail)? \ TC = (u - 5 gR) …(iv)
R
3. Why does a child in merry-go-round press the side of his seat
radially outwards?
4. Which provides the centripetal force to the satellite revolving
Condition for Completing the Loop
round the earth? Which furnishes the centripetal force for The particle will complete the circle, if the string does not
electrons to go round the nucleus? slack even at the highest point (q = p ). Thus, tension at the
highest point should be greater than or equal to zero (T ³ 0)
at q = p .
m 2
5.7 Motion in Vertical Circle At the highest point, TC =
R
(u - 5 gR)

This is an example of non-uniform circular motion. but for completing the loop, TC ³ 0
Consider a particle of mass m attached to a string of length Þ u2 - 5 gR ³ 0
R to be whirled in a vertical circle about a fixed point O. or u ³ 5 gR
g C
Therefore, minimum velocity at the bottom required to
complete the circle is given by
umin = 5 gR
O
v(t) At, u = 5 gR, velocity at the highest point is calculated
R– h θ T
from Eq. (ii),
h B v2 = u2 - 2 gh
A u mg mg cos θ or v2 = 5 gR - 2 g (2 R) (Q h = 2 R)
mg sin θ
or v= gR
(i) Velocity at any point on vertical loop
At the lowest point A, it is imparted a velocity u in the So, corresponding to minimum velocity, umin (= 5 gR ) at
horizontal direction. Let v (t ) be its velocity at point B at lowest point, velocity at highest point is gR.
height h, is given by
C TC = 0
v = u2 - 2 gh = u2 - 2 gR (1 - cos q) ...(i) vmin = √gR

[as, h = R - R cos q = R (1 - cos q)]


(ii) Tension at any point on the vertical loop
O
At any point B, the necessary centripetal force is
provided by the resultant of tension T and mg cos q. R
umin = √5 gR
mv2
TB - mg cosq = u
R A T = 6 mg
mv2
TB = mg cosq +
R For umin = 5 gR,
m 2
or TB = [u - gR (2 - 3 cos q)] …(ii)
R T A = 6 mg, TC = 0
[From Eq. (i)] Finally, we observe that, T A - TC = 6 mg
m 2
or TB = (u + gR - 3 gh ) …(iii)
R
Telegram @unacademyplusdiscounts

160 JEE Main Physics

Important Points Interpret (c) Velocity at the lowest point, v = 2 gl


At the lowest point, the tension in the string
1. The condition for a body to perform oscillations is that the velocity
should be zero earlier than the tension i.e., mv 2 m
T = mg + = mg + (2 gl) = 3 mg
l l
u2 u2 + gR
<
2g 3g Sample Problem 12 A block is released from rest at the top of
This gives, u < 2 gR an inclined plane which later curves into a circular track of radius r as
Thus, if u = 2 gR , the body continues to oscillate about the lowest shown in figure. Find the minimum height h from where it should be
point A. In particular, if m < 2gR the arc of the oscillation is a released so that it is able to complete the circle.
(a) 2 × r (b) 3 × r
semicircle.
(c) 2.5 × r (d) 3.5× r
2. The condition for the body to leave the circular path is that when the
tension in the string becomes zero earlier than the velocity, then Interpret (c) To ‘loop the loop’ body should not loose contact
body leaves the circular path i.e., with the track anywhere so,
u2 + gR u2 v ³ gr
<
3g 2g
A
This gives, u > 2 gR
B
Thus, if the speed u of the body at lowest point lies between 2gR
h
and 5gR the body leaves the circular path some where between A
2r
and C.
3. Oscillation of a pendulum is part of a circular motion. At points A and
C, since velocity is zero, net centripetal force will be zero. Only
tangential force is present. From A to B or C to B speed of the Applying law of conservation of mechanical energy between points
bob increases. Therefore, tangential force is parallel to velocity. From A and B, i. e. , magnitude of change in kinetic energy equals the
B to A or B to C speed of the bob decreases. Hence, tangential force is magnitude of change in potential energy.
antiparallel to velocity. 1
Þ mv 2 - 0 = mg (h - 2 r)
2
1
m ( gr ) 2 = mg (h - 2 r)
2
5
Þ h= r
2
Hence, h must be at least equal to 2.5 r.

C A Check Point 2
B
1. A string just supports a hanging ball without breaking. If the
ball is made to swing, will the string will break. Why?
Sample Problem 11 The string of a pendulum of length l 2. Explain, why the pilot of the aeroplane does not face down,
is displaced through 90° from the vertical and released. Then while completing the loop?
the minimum strength of the string in order to withstand the 3. In a vertical circular motion, if tension T £ 0, then how will the
tension as the pendulum passes through the mean position is body move?
(a) mg (b) 6 mg
(c) 3 mg (d) 5 mg
Telegram @unacademyplusdiscounts

WORKED OUT
Examples
Example 1 The blades of an aeroplane propeller are Example 4 A ferris wheel with radius 14 m is turning about
rotating at the rate of 600 revolutions per minute. Calculate its a horizontal axis through its centre. The linear speed of the
angular velocity. passenger on the rim is 7 ms-1. Find the acceleration of a
(a) 5 p rad s-1 (b) 10 p rad s-1 passenger at the highest point.
(c) 20 p rad s-1 (d) 25 p rad s-1 (a) 6.3 ms-2 downwards (b) 3.5 ms-2 upwards
(c) 13.3 ms-2 upwards (d) None of these
Solution Here,n = 600 revolutions/minute
2
v 72
600 Solution Acceleration, a = =
= revolutions/second r 17
60
= 3.5 ms-2 upwards
We know w = 2 pn = 2 p ´ 600 / 60
= 20 p rad s-1 Example 5 A circular disc rotates at 60 rpm. A coin of 18 g
is placed at a distance of 8 cm, from the centre. The centrifugal
Example 2 The radius of the earth’s orbit around the sun is force on the coin becomes
1.5 ´ 1011m. The linear velocity of the earth is (a) 5.7 ´ 10 -1 N (b) 5.7 ´ 10 -2 N
(a) 1.99 ´ 10 -7 ms-1 (b) 2.99 ´ 10 -7 ms-1 (c) 3.8 ´ 10 -1 N (d) 3.8 ´ 10 -2 N
(c) 1.99 ´ 10 4 ms-1 (d) 2.99 ´ 10 4 ms-1 Solution Here, n = 60 rpm =
60
= 1rps
60
Solution Here, r = 1.5 ´ 10 11 m; time period of revolution of m = 18g = 18 ´ 10 -3 kg
earth around the sun is 1 yr i.e.,
r = 8 cm = 8 ´ 10 -2 m
T = 1 yr = 365 ´ 24 ´ 60 ´ 60 s
Centrifugal force, F = mrw2 = mr (2pn) 2
2p
\Angular velocity, w = = 4p 2 mr n 2
T
2 ´ (22 / 7) 22 22
= =4´ ´ ´ (18 ´ 10 -3) ´ (8 ´ 10 -2) ´ 12
365 ´ 24 ´ 60 ´ 60 7 7
F = 5.689 ´ 10 -2 N
= 1.99 ´ 10 -7 ´ rad s-1
Linear velocity, v = w r Example 6 A car of mass 1500 kg is moving with a speed of
-7 11 12.5 ms-1 on a circular path of radius 20 m on a level road.
= 1.99 ´ 10 ´ 1.5 ´ 10 What should be the value of coefficient of friction to attain this
= 2.99 ´ 10 4 ms-1 force?
(a) 0.2 (b) 0.4
(c) 0.6 (d) 0.8
Example 3 A volunteer is rotated in a horizontal circle of
radius 7m. Find the period of rotation for which the acceleration Solution Here, m = 1500 kg, v = 12.5 ms-1,
is equal to 3g.
r = 20 m
(a) 2.61 s (b) 2.87 s
Frictional force = centripetal force required
(c) 3.07 s (d) 3.31 s
mv 2 1500(12.5) 2
æ 2p ö æ 4p 2 ö F= = = 1.172 ´ 10 4 N
Solution Acceleration, ar = r ç ÷ = 3g or 7 ç 2 ÷ r 20
èT ø è T ø As F = m R = m mg
= 3g or T = 3.07s F 1.172 ´ 10 4
m= = = 0.8
mg 1500 ´ 9.8
Telegram @unacademyplusdiscounts

162 JEE Main Physics

Example 7 A pendulum was kept horizontal and released. (b) 2.0625 ´ 10 5N downward
Find the acceleration of the pendulum when it makes an angle q (c) 2.0625 ´ 10 5N upward
with the vertical. (d) 2.0625 ´ 10 4N downward
θ mv 2 5 ´ 10 3 ´ (250) 2
Solution Force, F = + mg = + 5 ´ 10 4
v2 r 2 ´ 10 3
l cos θ l
= 2.0625 ´ 10 4N downward

Example 10 Find the tension in the pendulum at the


extreme position if amplitude is q 0 .
g sin θ
mv 2 mv 2
(a) (b) + mg cos q0
(a) g 1 + 3 cos2 q (b) g 1 + 3 sin 2 q r r
(c) g sin q (d) 2g cos q mv 2
(c) mg cos q0 (d) - mg cos q0
2 r
mn v2
Solution At a point, = mgl cos q or = 2g cos q
2 l Solution The necessary centripetal force is provided by the
Acceleration = at2 + ar2 = ( g sin q) 2 + ( g cos q) 2 resultant of tension T and mg cos q
mv 2
= g 1 + 3 cos2 q T - mg cos q =
r
At the extreme position v = 0; therefore T = mg cos q0
Example 8 A circular track of radius 100 m is designed for
an average speed 54 kmh-1. Find the angle of banking.
Example 11 One end of a string of length 1.5 m is tied to
æ 3 ö æ 9 ö
(a) tan -1ç ÷ (b) tan -1ç ÷ the stone of mass 0.4 kg and the other end to a small pivot on a
è 20 ø è 40 ø
smooth vertical board. What is the minimum speed of the stone
æ3ö required at its lowermost point so that the stone required at its
(c) tan -1ç ÷ (d) None of these
è10 ø lowermost point so that the string does not slack at any point in
its motion along the vertical circle?
Solution If the track is banked at an angle q, then
(a) 3.2 ms-1 (b) 4.2 ms-1
v2 15 ´ 15 9
tan q = = = [Q 54 kmh -1 = 15 ms-1] (c) 6.8 ms-1 (d) 8.6 ms-1
rg 100 ´ 10 40
æ 9 ö
q = tan -1ç ÷
Solution Here, r = 1.5 m,m = 0.4 kg;
è 40 ø g = 9.8 ms-2

Example 9 A fighter plane is pulling out for a dive at 900 The minimum speed at the lowest point of the vertical circle is
-1
kmh in a vertical circle of radius 2 km. Its mass is 5000 kg.
Find the force exerted by the air on it at the lowest point. vL = 5 gr = 5 ´ 9.8 ´ 1.5
4
(a) 2.0625 ´ 10
L N upward v = 8.6 ms-1
Telegram @unacademyplusdiscounts

Start Practice for


JEE Main
Round I (Topically Divided Problems)

Motion along Horizontal Circle C


O
1. A car-wheel is rotated to uniform angular A B
l l l
acceleration about its axis., Initially its angular
velocity is zero. It rotates through an angle q 1 in the (a) 3 : 5 : 7 (b) 3 : 4 : 5
first 2 s, in the next 2 s, it rotates through an (c) 7 : 11 : 6 (d) 3 : 5 : 6
q
additional angle q 2 , the ratio of 2 is 6. Two particles of equal mass are connected to a rope
q1
AB of negligible mass such that one is at end A and
(a) 1 (b) 2 other dividing the length of rope in the ratio 1 : 2 from
(c) 3 (d) 5 B. The rope is rotated about end B in a horizontal
2. A sphere of mass 0.2 kg is attached to an inextensible plane. Ratio of tensions in the smaller part to the
string of length 0.5 m whose upper end is fixed to the other is (ignore the effect of gravity)
ceilling. The sphere is made to describe a horizontal (a) 4 : 3 (b) 1 : 4
circle of radius 0.3 m. The speed of the sphere will be (c) 1 : 2 (d) 1 : 3
(a) 1.5 ms -1 (b) 2.5 ms -1 7. A coin is placed on a gramophone record rotating at a
(c) 3.2 ms -1 (d) 4.7 ms -1 speed of 45 rpm. It flies away when the rotational
3. Two wires AC and BC are tied at speed is 50 rpm. If two such coins are placed over the
A
C of small sphere of mass 5 kg, 30º other on the same record, both of them will fly away
B when rotational speed is
which revolves at a constant
speed v in the horizontal circle of 45º (a) 100 rpm (b) 25 rpm
radius 1.6 m. The minimum value (c) 12.5 rpm (d) 50 rpm
of v is C
8. The maximum and minimum tension in the string
(a) 3.01 ms -1 whirling in a circle of radius 2.5 m with constant
(b) 4.01 ms -1 1.6 velocity are in the ratio 5 : 3, then its velocity is
(c) 8.2 ms -1 (a) 98 ms -1 (b) 7 ms -1
(d) 3.96 ms -1 (c) 490 ms -1 (d) 4.9 ms -1
4. A particle describes a horizontal circle in a conical
9. A long horizontal rod has a bead, which can slide
funnel whose inner surface is smooth with speed of
along its length and initially placed at a distance L
0.5 m/s. What is height of the plane of circle from
from one end A of the rod. The rod is set in angular
vertex of the funnel?
acceleration a. If the coefficient of friction, between
(a) 0.25 m (b) 2 cm
the rod and the bead is m and gravity is neglected,
(c) 4 cm (d) 2.5 cm
then the time after which the bead starts slipping is
5. Three identical particles are joined together by a (a) m / a
thread as shown in figure. All the three particles are
(b) m / a
moving in a horizontal plane. If the velocity of the
outermost particle is v0 then the ratio of tensions in (c) 1/ ma
the three sections of the string is (d) infinitesimal
Telegram @unacademyplusdiscounts

164 JEE Main Physics

10. A car moving on a circular path and takes a turn. If 18. A weightless thread can bear tension upto 3.7 kg-wt.
R1 and R2 be the reactions on the inner and outer A stone of mass 500 g is tied to it and revolved in a
wheels respectively, then circular path of radius 4 m in a vertical plane. If
(a) R1 = R2 (b) R1 < R2 g = 10 ms -2 , then the maximum angular velocity of
(c) R1 > R2 (d) R1 ³ R2 the stone will be
11. The length of second’s hand in a watch is 1 cm. The (a) 4 rad/s (b) 16 rad/s
change in velocity of its tip in 15 s is (c) 21 rad/s (d) 2 rad/s
p 19. A 2 kg stone at the end of a string 1 m long is whirled
(a) zero (b) cm/s
30 2 in a vertical circle at a constant speed. The speed of
p p 2 the stone is 4 m/s. The tension in the string will be
(c) cm/s (d) cm/s
30 30 52 N, when the stone is
(a) at the top of the circle
12. A wheel rotates with a constant angular velocity of
(b) at the bottom of the circle
300 rpm. The angle through which the wheel rotates
(c) halfway down
in one second is
(d) None of the above
(a) p rad (b) 5 p rad
(c) 10 p rad (d) 20 p rad 20. An object is tied to a string and rotated in a vertical
circle of radius r. Constant speed is maintained along
13. If a particle covers half the circle of radius R with
the trajectory. If Tmax / Tmin = 2, then v2 / rg is
constant speed, then
(a) 1 (b) 2
(a) change in momentum is mvr
1 (c) 3 (d) 4
(b) change in KE is mv2
2 21. A body crosses the topmost point of a vertical circle
(c) change in KE is mv2 with critical speed. What will be its acceleration
(d) change in KE is zero when the string is horizontal?
14. The string of a pendulum of length l is displaced (a) g (b) 2 g
through 90° from the vertical and released. Then, the (c) 3 g (d) 6 g
minimum strength of the string in order to withstand 22. A frictionless track ABCDE ends in a circular loop of
the tension as the pendulum passes through the radius R, figure. A body slides down the track from
mean position is point A which is at a height h = 5 cm. Maximum value
(a) mg (b) 6 mg of R for the body to successfully complete the loop is
(c) 3 mg (d) 5 mg A
15. An object is being weighed on a spring balance
D
moving around a curve of radius 100 m at a speed
7 ms -1. The object has a weight of 60 kg-wt. The h
reading registered on the spring balance would be 2R C
E
(a) 60.075 kg-wt (b) 60.125 kg-wt
(c) 60.175 kg-wt (d) 60.225 kg-wt B

(a) 5 cm (b) 15/4 cm


Motion in Vertical Circle (c) 10/3 cm (d) 2 cm
16. A stone of mass m is tied to a string and is moved in a 23. A simple pendulum oscillates in a vertical plane.
vertical circle of radius r making n revolution per When it passes through the mean position the
minute. The total tension in the string when the tension in the string is 3 times the weight of
stone is at its lowest point is pendulum bob. What is the maximum displacement
(a) mg (b) m ( g + pnr2 ) of the pendulum with respect to the vertical?
(a) 30° (b) 45°
(c) m ( g + pnr ) (d) m {g + ( p2 n2 r ) / 900}
(c) 60° (d) 90°
17. A stone of mass 1 kg is tied to a string 4 m long and is 24. In figure, a particle is placed at the highest point A of
rotated at constant speed of 40 ms -1 in a vertical
a smooth sphere of radius r. It is given slight push,
circle. The ratio of the tension at the top and the
and it leaves the sphere at B, at a depth h vertically
bottom is
below A such that h is equal to
(a) 11 : 12 (b) 39 : 41
(c) 41 : 39 (d) 12 : 11
Telegram @unacademyplusdiscounts

Circular Motion 165

A
Bending of Raods and Railway Tracks
h
C B 29. A fan is making 600 revolution per minute. If after
r some time it makes 1200 revolution per minute, then
O increase in its angular velocity is
(a) 10 p rad/s (b) 20 p rad/s
(c) 40 p rad/s (d) 60 p rad/s

r 1
30. A body moves along a circular path of radius 5 m. The
(a) (b) r coefficient of friction between the surface of path and
6 4
1 1 the body is 0.5. The angular velocity, in rad/s, with
(c) r (d) r which the body should move so that it does not leave
3 2
the path is (g = 10 ms–2)
25. A stone tied to a string of length L is whirled in a (a) 4 (b) 3 (c) 2 (d) 1
vertical circle, with the other end of the string at the
31. A car is moving on a circular level road of radius of
centre. At a certain instant of time, the stone is at its
curvature 300 m. If the coefficient of friction is 0.3
lowest position, and has a speed u. The magnitude of
and acceleration due to gravity 10 ms–2, the
change in its velocity as it reaches a position, where
maximum speed of the car can have is (in kmh–1)
the string is horizontal is (a) 30 (b) 81 (c) 108 (d) 162
(a) u2 - 2 gL (b) 2 gL
32. A railway carriage has its centre of gravity at a
(c) u2 - gL (d) 2 (u2 - gL) height of 1 m above the rails, which are 1.5 m apart.
The maximum safe speed at which it could travel
26. Read each of the following statements carefully and round an unbanked curve of radius 100 m is
state with reasons, chose the correct statement (s)
(a) 12 ms–1 (b) 18 ms–1
(i) The net acceleration of a particle in the circular (c) 22 ms–1 (d) 27 ms–1
motion is always along the radius of the circle
33. A car of mass 2000 kg is moving with a speed of
towards the centre.
10 ms–1 on a circular path of radius 20 m on a level
(ii) The velocity vector of a particle at a point is road. What must be the frictional force between the
always along the tangent to the path of the car and the road so that the car does not slip?
particle at that point. (a) 104 N (b) 103 N (c) 105 N (d) 102 N
(iii) The acceleration vector of a particle in uniform 34. A stone tied to the end of a string 80 cm long is
circular motion averaged over one cycle is a null whirled in a horizontal circle with a constant speed.
vector. If the stone makes 14 revolutions in 25 s, what is the
(a) (i) and (iii) magnitude and direction of acceleration of the stone?
(b) (ii) and (iii) (a) 9.9 m/ s2 along the tangent
(c) (iii) Only (b) 7.9 m/ s2 along the radius
(d) All the three (c) 9.9 m/ s2 along the radius
27. A particle is moving in a vertical circle. The tensions (d) None of the above
in the string when passing through two positions at
35. An aircraft executes a horizontal loop of radius 1 km
angles 30° and 60° from vertical (lowest position) are
with a speed of 900 km/h. Compare its centripetal
T1 and T2 respectively.
acceleration with the acceleration due to gravity.
(a) T1 = T2 (a) 6 (b) 7 (c) 8 (d) 5
(b) T2 > T1
(c) T1 > T2 36. A coin placed on a rotation turn table slops when it is
(d) Tension in the string always remains the same placed at a distance of 9 cm from the centre. If the
angular velocity of the turn table is trippled. It will
28. A body of mass 1 kg is rotating in a vertical circle of just slip. If its distance from the centre is
radius 1 m. What will be the difference in its kinetic (a) 27 cm (b) 9 cm (c) 3 cm (d) 1 cm
energy at the top and bottom of the circle?
37. What should be the coefficient of friction between the
(Take g = 10 ms -2 )
tyres and the road, when a car travelling at 60 kmh–1
(a) 10 J (b) 20 J
makes a level turn of radius 40 m?
(c) 30 J (d) 50 J (a) 0.5 (b) 0.66 (c) 0.71 (d) 0.80
Telegram @unacademyplusdiscounts

166 JEE Main Physics

38. The maximum speed with which a car is driven 41. A car rounds an unbanked curve of radius 92 m
round a curve of radius 18 m without skidding without skidding at a speed of 26 ms–1. The smallest
(where, g = 10 ms -2 and the coefficient of friction possible coefficient of static friction between the tyres
between rubber tyres and the roadway is 0.2) is and the road is
(a) 36.0 kmh–1 (b) 18.0 kmh–1 (a) 0.75 (b) 0.60
(c) 21.6 kmh–1 (d) 14.4 kmh–1 (c) 0.45 (d) 0.30
39. What is the smallest radius of a circle at which a 42. A particle moves in circular path of radius R. If
cyclist can travel if its speed is 36 kmh–1, angle of centripetal force F is kept constant but the angular
inclination is 45° and g = 10 ms–2? velocity is double, the new radius of the path will be
(a) 20 m (b) 10 m (a) 2 R (b) R /2
(c) 30 m (d) 40 m (c) R / 4 (d) 4 R
40. The angle which the bicycle and its rider must make 43. A curved road of 50 m radius is banked at correct
with the vertical when going round a curve of 7 m angle for a given speed. If the speed is to be doubled
radius at 5 ms–1 is keeping the same banking angle, the radius of
(a) 20° (b) 15° curvature of the road should be changed to
(c) 10° (d) 5° (a) 25 m (b) 100 m
(c) 150 m (d) 200 m

Round II (Mixed Bag)

Only One Correct Option 5. The kinetic energy K of a particle moving along a
1. The bob of a pendulum of mass m and length L is circle of radius R depends on the distance covered s as
displaced, 90° from the vertical and gently released. K = as2 , where a is a constant. The force acting on
In order that the string may not break upon passing the particle is
1/2
through the lowest point, its minimum strength must s2 æ s2 ö
(a) 2 a (b) 2 as ç1 + 2 ÷
be R è R ø
(a) mg (b) 2 mg R2
(c) 3 mg (d) 4 mg (c) 2 as (d) 2 a
s
2. A body is acted upon by a constant force directed 6. A particle of mass m is moving in circular path of
towards a fixed point. The magnitude of the force
constant radius r such that its centripetal
varies inversely as the square of the distance from
acceleration ac is varying with time t as ac = k2 rt2 .
the fixed point. What is the nature of the path?
The power delivered to the particle by the forces
(a) Straight line (b) Parabola
acting on it is
(c) Circle (d) Hyperbola
(a) 2 pmk2 r2t (b) mk2 r2t
3. A string of length L is fixed at one s
2 mk 4 r2t5
end and the string makes rev/s (c) (d) zero
θ L 3
p
around the vertical axis through the 7. When the road is dry and coefficient of friction is m,
fixed and as shown in the figure, the maximum speed of a car in a circular path is
then tension in the string is R 10 ms–1. If the road becomes wet and m ¢ = m/2, what is
(a) ML (b) 2 ML the maximum speed permitted?
(c) 4 ML (d) 16 ML (a) 5 ms–1 (b) 10 ms–1
4. When a ceiling fan is switched on, it makes 10 (c) 10 2 ms–1 (d) 5 2 ms–1
rotations in the first 4 s. How many rotations will it 8. A body of mass 1 kg is moving in a vertical circular
make in the next 4 s? (Assuming uniform angular path of radius 1 m. The difference between the
acceleration). kinetic energies at its highest and lowest point is
(a) 10 (b) 20 (a) 20 J (b) 10 J
(c) 40 (d) 30 (c) 4 5 J (d) 10 5 J
Telegram @unacademyplusdiscounts

Circular Motion 167

20 ö
9. A particle moves along a circle of radius æç ÷ m with 15. For a particle in uniform circular motion the
èpø acceleration a at a point P ( R, q ) on the circle of radius
constant tangential acceleration. If the velocity of the R is (here q is measured from the x-axis)
particle is 80 ms–1, at the end of seconds revolution v2 v2 v2 v2
after motion has begun, the tangential acceleration is (a) - cos q $i + sin q $j (b) - sin q $i + sin q $j
R R R R
(a) 40 ms–2 (b) 640 p ms–2
v2 v2 v2 $ v2 $
(c) 1609 p ms–2 (d) 40 p ms–2 (c) - cos q i$ - sin q $j (d) i+ j
R R R R
10. The distance r from the origin of a particle moving in
16. A car is moving in a circular horizontal track of
xy-plane varies with time as r = 2 t and the angle
radius 10 m with a constant speed of 10 ms–1. The
made by the radius vector with positive x-axis is
angle made by the rod with track is
q = 4 t. Here, t is in second, r in metre and q in radian.
(a) zero (b) 30° (c) 45° (d) 60°
The speed of the particle at t = 1 s is
(a) 10 ms–1 (b) 16 ms–1 17. An object of mass 10 kg is whirled round a horizontal
(c) 20 ms–1 (d) 12 ms–1 circle of radius 4 m by a revolving string inclined 30°
to the vertical. If the uniform speed of the object is
11. A stone of mass 1 kg tied to a light in extensible 5 ms–1, the tension in the string (approximately) is
string of length L = 10 m is whirling in a circular (a) 720 N (b) 960 N (c) 114 N (d) 125 N
3
path of radius L in a vertical plane. The ratio of the
maximum tension in the string to the minimum More Than One Correct Option
tension in the string is 4 and if g is taken to be 18. A particle moves along a circle with a constant speed.
10 m /s2 . The speed of stone at the highest point of the If a is acceleration and E is kinetic energy of the
circular is particle, then
(a) 20 m/s (b) 10 3 m/s (a) a is constant (b) E is constant
(c) 5 2 m/s (d) 10 m/s (c) a is variable (d) E is variable
12. A body of mass m is moving with a uniform speed v 19. A body of mass m is moving in a circle of radius r with
along a circle of radius r, what is the average a constant speed v. The force on the body is mv2 / r and
acceleration in going from A to B? is directed towards the centre. What is the work done
B by this force in moving the body over half the
circumference of the circle?
v mv2 mv2 pr
(a) ´ pr (b) 2 (c) zero (d)
r r mv2
A
20. For a particle performing uniform circular motion,
choose the correct statement (s) from the following
[NCERT Exemplar]
(a) Magnitude of particle velocity (speed) remains coonstant
(a) 2 v2 / pr (b) 2 2 v2 / pr
(b) Particle velocity remains directed perpendicular to radius
(c) v2 / pr (d) None of these vector
13. A string is would round the rim of a mounted (c) Direction of acceleration keeps changing as particle
fly wheel of mass 20 kg and radius 20 cm. A steady moves
ball of 25 N is applied on the cord. Neglecting friction (d) Angular momentum is constant in magnitude but
direction keeps changing
and mass of the string, the angular acceleration of
the wheel is 21. The speed of revolution of a particle moving round a
(a) 50 rad s -2 (b) 25 rad s -2 circle is doubled and its angular speed is halved.
(c) 6.25 rad s -2
(d) 12.5 rad s -2 What happens to the centripetal acceleration?
(a) Unchanged (b) Halved
14. An aeroplane flying at a velocity of 900 kmh–1 loops (c) Doubled (d) 4 times
the loop. If the maximum force pressing the pilot
22. If ar and at represent radial and tangential
against the seat is five times its weight, the loop
acceleration respectively, the motion of a particle will
radius should be
be circular if
(a) 1594 m (b) 1402 m
(a) ar = 0 and at = 0 (b) ar = 0 but at ¹ 0
(c) 1315 m (d) 1167 m
(c) ar ¹ 0 and at = 0 (d) ar ¹ 0 and at ¹ 0
Telegram @unacademyplusdiscounts

168 JEE Main Physics

Comprehension Based Questions 28. If the speed of car is more than this safe speed, the
car would topple
Passage I (a) inwards (b) outwards
When a stone tied to one end of a string is rotated in a (c) sometimes inwards (d) Cannot say
vertical circle, its velocity goes on changing on 29. The safe speed on banked road is
account of gravity. For looping the loop, velocity at (a) 33.6 ms–1 (b) 46. 4 ms–1
lowest point is vL = 5 gr; tension in the string, (c) 9.8 ms–1 (d) 19.6 ms–1
TL ³ 6 mg. The string does not slacken at the highest
point when n H ³ gr and TH > 0. The stone leaves the
30. At this safe speed, frictional force is
vertical circle when tension in the string vanishes (a) inwards
before its velocity becomes zero. For this, (b) outwards
2 gr < vL < 5 gr. Read the above passage and (c) zero
answer the following questions when mass of stone is (d) Cannot say
100 g, length of string is 1 m and g = 9.8 ms–2.
23. If vL = 7 ms–1, velocity at highest point will be Assertion and Reason
(a) 1 ms–1 (b) zero Directions Question No. 31 to 35 are Assertion-Reason type.
(c) 3.13 ms–1 (d) cannot say Each of these contains two Statements: Statement I (Assertion),
24. When vL = 7 ms–1, tension in the string at highest Statement II (Reason). Each of these questions also has four
point will be alternative choice, only one of which is correct. You have to
(a) 9.8 N (b) 9.8 kg (c) 0.98 J (d) zero select the correct choices from the codes (a), (b), (c) and (d) given
below
25. The velocity of projection at the lowest point is (a) If both Assertion and Reason are true and the Reason
6 ms–1. Will the stone loops the loop? is correct explanation of the Assertion:
(a) Yes (b) No (b) If both Assertion and Reason are true but Reason is
(c) May or may not (d) Cannot be predicted not correct explanation of the Assertion
(c) If Assertion is true but Reason is false
Passage II
(d) If Assertion is false but the Reason is true
When a vehicle rounds a curve, it requires some
centripetal force = mv2 / r. If the road is unbanked, the 31. Assertion When an automobile is going too fast around
curve overturns, its inner wheels leave the ground
necessary force is provided by the force of friction
first.
between the tyres and road. To avoid skidding, the
Reason For a safe turn the velocity of automobile
speed of vehicle must be £ m rg and to avoid
grx should be less than the value of safe limiting velocity.
overturning, the speed must be £ , where 2x is
h 32. Assertion During a turn the value of centripetal
the wheel base and h is height of centre of gravity, force should be less than the limiting frictional force.
above the road. The dependence on friction can be Reason The centripetal force is provided by the
avoided if the road is suitably banked. The safe speed frictional force between the tyres and the road.
then rises to r g tan q . In no case, the speed limits 33. Assertion As the frictional force increases the safe
depend upon mass of the vehicle. velocity limit for taking a turn on an unbanked road
Read the above passage and answer the following also increases.
questions, when mass of car is 800 kg, wheel base is Reason Banking of roads will increase the value of
1.1 m, height of centre of gravity is 50 cm, banking limiting velocity.
angle is 30° and radius of curve is 200 m. 34. Assertion A safe turn by a cyclist should neither be
(take g = 9.8 ms -2 , and m = 0.2) fast nor sharp.
26. The safe speed to avoid skidding on the unbanked Reason The bending angle from the vertical would
curve is decrease with increase in velocity.
(a) 9.8 ms–1 (b) 19.8 ms–1 35. Assertion Improper banking of roads causes wear
(c) 10 ms–1 (d) 1.98 ms–1 and tear of tyres.
27. The safe speed to avoid toppling on unbanked curve is Reason The necessary centripetal force in that event
(a) 19.8 ms–1 (b) 198 ms–1 is provided by the force of friction between the tyres
(c) 46.4 ms–1 (d) 1.98 ms–1 and the road.
Telegram @unacademyplusdiscounts

Circular Motion 169

Previous Years’ Questions


36. A body of mass 5 kg is moving in a circle of radius 1 m (a) 30 rad/s (b) 20 rad/s
with angular velocity of 2 rad/s. The centripetal force (c) 10 rad/s (d) 25 rad/s
is [Orissa JEE 2011] 41. Centripetal acceleration is [Kerala CEE 2010]
(a) 10 N (b) 20 N (c) 30 N (d) 40 N
(a) a constant vector
37. If a car is to travel with a speed v along the (b) a constant scalar
frictionless banked circular track of radius r, the (c) a magnitude changing vector
required angle of banking so that the car does skid is (d) not a constant vector
[J&K 2010]
æ v2 ö
42. A particle moves in a circle of radius 5 cm with
ævö
(a) q = tan -1 ç ÷ (b) q = tan -1 ç ÷ constant speed and time period 0.2 p s. The
è rg ø è rg ø
acceleration of the particle is
æ r2 ö æ v2 ö (a) 5 m / s2 (b) 15 m / s2 (c) 25 m / s2 (d) 36 m / s2
(c) q = tan -1 ç ÷ (d) q < tan -1 ç ÷
è vg ø è rg ø 43. A car of mass 1000 kg negotitates a banked curve of
38. A car is moving along a circular path of radius 500 m radius 90 m on a frictionless road. If the banking
with a speed of 30 ms–1.
If at some instant, its speed angle is 45°, the speed of car is [BVP Engg. 2006]
increases at the rate of 2 ms–1, then at that instant (a) 20 ms -1 (b) 30 ms -1 (c) 5 ms -1 (d) 10 ms -1
the magnitude of resultant acceleration will be 44. A motorcycle moving with a velocity of 72 kmh–1 on a
[UP SEE 2009] flat road takes a turn on the road at a point where the
(a) 4.7 ms–2 (b) 3.8 ms–2 radius of curvature of the road is 20 m. The
(c) 3 ms–2 (d) 2.7 ms–2 acceleration due to gravity is 10 ms–2. In order to
39. A heavy small-sized sphere is suspended by a string avoid skidding, he must not bent with respect to the
of length l. The sphere rotates uniformly in a vertical plane by an angle greater than [BVP Engg. 2006]
horizontal circle with the string making an angle q (a) q = tan -1 (2) (b) q = tan -1 (6 )
with the vertical. Then, the time-period of this (c) q = tan -1 ( 4 ) (d) q = tan -1 (25.92)
conical pendulum is [BVP Engg. 2008]
l l sin q 45. A particle is moving in a circle of radius R in such a
(a) t = 2p (b) t = 2p way that at any instant the normal and tangential
g g
components of its acceleration are equal. If its speed
l cos q l
(c) t = 2p (d) t = 2p at t = 0 is v0 , the time taken to complete the first
g g cos q revolution is [Kerala CET 2005]
40. A stone of mass 2 kg is tied to a string of length (a)
R
(b)
R
(1 - e -2 p )
0.5 cm. If the breaking tension of the string is 900 N, v0 v0
then the maximum angular velocity, that stone can R -2 p 2 pR
(c) e (d)
have in uniform circular motion is [Kerala CET 2010] v0 v0

Answers
Round I
1. (c) 2. (a) 3. (d) 4. (d) 5. (d) 6. (a) 7. (d) 8. (a) 9. (a) 10. (b)
11. (d) 12. (c) 13. (d) 14. (c) 15. (a) 16. (d) 17. (b) 18. (a) 19. (b) 20. (c)
21. (c) 22. (d) 23. (d) 24. (c) 25. (d) 26. (b) 27. (c) 28. (b) 29. (b) 30. (d)
31. (c) 32. (d) 33. (a) 34. (c) 35. (a) 36. (d) 37. (c) 38. (c) 39. (b) 40. (a)
41. (a) 42. (c) 43. (d)

Round II
1. (c) 2. (c) 3. (d) 4. (d) 5. (b) 6. (b) 7. (d) 8. (a) 9. (a) 10. (b)
11. (d) 12. (b) 13. (d) 14. (a) 15. (b) 16. (c) 17. (d) 18. (b) 19. (c) 20. (a,b,c)
21. (a) 22. (c,d) 23. (c) 24. (d) 25. (b) 26. (b) 27. (c) 28. (b) 29. (a) 30. (c)
31. (a) 32. (a) 33. (b) 34. (c) 35. (a) 36. (b) 37. (a) 38. (d) 39. (c) 40. (a)
41. (d) 42. (a) 43. (b) 44. (a) 45. (b)
Telegram @unacademyplusdiscounts

the Guidance
Round I
w q
1. a = and w = T1 cos 30º
t t
+ T2 cos 45º
q
\ a=
t2
but a = constant T1 sin 30º
q1 (2) 2 + T2 sin 45º
So, =
q1 + q2 (2 + 2) 2 mg
q1 1
or =
q1 + q2 4 But T1 ³ 0
q1 + q2 4 mv 2
or = mg -
q1 1 \ r ³0
q 4 3 -1
or 1+ 2 = 2
q1 1
q2 mv 2
\ =3 or mg ³
q1 r
or v £ rg
mv 2
2. In figure, T sin q = ; T cos q = mg ; \ v max = rg = 1.6 ´ 9.8 = 3.96 ms–1
r
O 4. The particle is moving in circular path
R R sin θ
θ θ
l
R cos θ
r
T cos θ
T (R = reaction)
mg
T sin θ A mv 2
B r r h
mg
θ
2
v r
So, tan q = =
rg l - r2
2

1/ 2 1/ 2
From the figure mg = R sin q …(i)
é r 2g ù é 0.09 ´ 10 ù
v = ê 2 2 1/ 2 ú =ê mv 2
1/2 ú = R cos q …(ii)
ë (l - r ) û ë (0.25 – 0.09) û r
= 1.5 ms–1 From Eqs. (i) and (ii), we get
rg r
3. From force diagram shown in figure tan q = 2 and tan q =
v h
T1 cos 30° + T2 cos 45° = mg …(i)
v 2 (0.5) 2
mv 2 h= = = 0.25 m = 2.5 cm
T1 sin 30° + T2 cos 45° = …(ii) g 10
r
After solving Eqs. (i) and (ii), we get 5. Let w is the angular speed of revolution
mv 2 O
T1 A T2 B T3
C
mg -
T1 = r l l l
æ 3 - 1ö
ç ÷ T3 = mw33l
è 2 ø
T2 - T3 = mw22l Þ T2 = mw25l
Telegram @unacademyplusdiscounts

Circular Motion 171

T1 - T2 = mw2l Þ T1 = mw26l where, r = radius of circular path,


Þ T3 : T2 : T1 = 3 : 5 : 6 2a = distance between two wheels and
h = height of centre of gravity of car.
6. Tension in the respective parts are shown in figure.
11. In 15 seconds hand rotate through 90°
v
v
m T2 T2 m T1
A B
2r r 90°

Let w be angular velocity, then


T1 - T2 = mw2 ´ r …(i)
and T2 = mw2(r + 2 r) \Change in velocity ,
æ qö
T2 = 3 mw r 2
…(ii) | Dv| = 2 v sin ç ÷
è2ø
From Eqs. (i) and (ii), we get
æ 90° ö
T1 = 4 mw2r = 2 (rw) sin ç ÷
è 2 ø
T1 4 2p 1
Þ = = 2 ´1´ ´
T2 3 T 2
7. A coin flies off when centrifugal force just exceeds the force of 4p p 2 cm
2
= = [ As T = 60 s]
friction i. e. ,mrw ³ mmg 60 2 30 s
mg 300
or w³ 12. Frequency of wheel, v = = 5 rps. Angle rotated by wheel
r 60
Thus, w does not depend upon mass and will remain the in one rotation= 2 p rad. Therefore, angle rotated by wheel in
same. (i. e. , 50 rpm) 1 s = 2 p ´ 5 rad = 10 p rad.

8. In this problem it is assumed that particle although moving in 13. As momentum is vector quantity
mv
a vertical loop but it speed remain constant.
mv 2
Tension at lowest point Tmax = + mg 180°
r
mv 2
Tension at highest point Tmin = - mg
r
mv 2
+ mg mv
Tmax 5
= r 2 = \Change in momentum
Tmin mv 3
- mg æ qö
r Dp = 2 mv sin ç ÷
è2ø
\ v = 4 gr = 4 ´ 9.8 ´ 2.5 = 98 ms–1
= 2 mv sin (90° ) = 2 mv
9. Tangential acceleration, a = La But kinetic energy remains always constant so change in
\Normal reaction, N = Ma = MLa kinetic energy is zero.
\Frictional force, F = mN = m MLa
14. Velocity at the lowest point
For no sliding along the length, frictional force ³ centripetal
force. v = 2 gl
i. e. , m MLa ³ MLw2 At the lowest point, the tension in the string
As w = w0 + at = at mv 2
T = mg +
\ m MLa ³ ML ( at ) 2 l
m m
Þ t= = mg + (2 gl ) = 3 mg
a l

1 æ v 2h ö 15. Here, r = 100 m, v = 7 ms–1, m = 60 kg.


10. Reaction on inner wheel R1 = M ç g - ÷
2 è ra ø Reading registered = resultant force = ?
Two forces are acting as weight (mg) and centripctal force,
1 æ v 2h ö
Reaction on outer wheel R2 = M çg + ÷ æ mv 2 ö
2 è ra ø ç ÷ are at 90° to each other
è r ø
Telegram @unacademyplusdiscounts

172 JEE Main Physics

2 1/ 2
æ mv 2 ö é æ v 2 ö 2ù 21. The body crosses the top most position of a vertical circle with
2
\ Resultant force = (mg ) + ç ÷ = mg ê1 + ç ÷ ú critical velocity, so the velocity at the lowest point of vertical
è r ø ê è rg ø ú
ë û circle, u = 5 gr .
1/ 2
é æ 7 ´ 7 ö 2ù Velocity of the body when string is horizontal, is
= 60 ´ 9.8 ê1+ ç ÷ ú
êë è100 ´ 9.8 ø úû v 2 = u 2 - 2 gr = 5 gr - 2 gr = 3 gr

= 60.075 ´ 9.8 N = 60.075 kg-wt v 2 3 gr


\Centripetal acceleration ac = = =3g
r r
16. Here, T = mg + mw2r = m ( g + 4 p 2n2r)
22. For successfully completing the loop,
ïì æ n ö öïü
æ 2
ïì æ p 2n 2r öïü
= m í g + çç 4 p 2 ç ÷ r ÷÷ý = m í g + ç ÷ý v bottom should be 5 gR
è 60 ø øï è 900 øþï
ïî è þ îï
From energy conservation
mv 2 1
17. As, Ttop = - mg …(i) mgh = m ( 5 gR ) 2
r 2
mv 2 5 2h 2 ´5
and Tbottom = + mg …(ii) Þ h = R ÞR = = = 2 cm
r 4 5 5
v2 40 ´ 40 mv 2
Ttop -g - 10 23. Tension at mean position, mg + = 3 mg
Þ = 2r = 4 r
Tbottom v 40 ´ 40 v = 2 gl …(i)
+g + 10
r 4
and if the body displaced by angle q with the vertical, then
400 - 10 390 39
= = = v = 2 gl (1 - cos q) …(ii)
400 + 10 410 41
Comparing Eqs. (i) and (ii), cos q = 0 Þ q = 90°
18. Maximum tension that string can bear = 3.7 kg -wt = 37 N
Tension at lowest point of vertical loop = mg + mw2r
24. If v is velocity acquired at B, then A mv 2
2 r
v = 2 gh h
= 0.5 ´ 10 ´ 0.5 ´ w ´ 4 C B
2 The particle will leave the θ θ
=5+2w
sphere at B, when
O
\ 37 = 5 + 2 w2 mv 2
mg
³ mg cos q
or w = 4 rad/s r
mv 2 2 ´ ( 4) 2 2 gh g (r - h)
19. Here, mg = 2 ´10 = 20 N and = = 32 N =
r 1 r r
r
It is clear that 52 N tension will be at the bottom of the circle which gives, h=
because we know that 3
mv 2 25. The velocity at B is v, where v 2 = u2 - 2 gL, in vertically
Tbottom = mg +
r upward direction. As is clear from figure change in velocity.
mv 2 v
20. At the lowest point, = TL - mg …(i) L
r O B B'
2
mv
At the highest point, = TH + mg …(ii) v
r L
Tmax TL
As = =2 O A'
Tmin TH A u v

\ TL = 2 TH OB = OA = A ¢B¢
From Eqs. (i) and (ii), we have = u 2 + v 2 = u 2 + (u 2 - 2 gL) = 2 (u 2 - gL)
2 TH - mg = TH + mg
TH = 2 mg 26. (a) False, because in uniform circular motion, the
centripetal acceleration is along the radius of the circle
mv 2 towards the centre, but in non-uniform circular motion, the
From Eq. (ii), = 3 mg
r direction of the resultant acceleration is not along the radius
v2 of the circle.
or =3
rg
Telegram @unacademyplusdiscounts

Circular Motion 173

aT
33. To avoid slipping friction force
mv 2
a F=
r
2000 ´ 100
O ac F= = 10 4 N
20
34. Radius of the horizontal circle = Length of the strain
= 80 cm = 0.80 m
(b) True, the velocity vector of a particle is always along the 14 -1
Frequency of revolution (n) = s
tangent to the path of the particle either it is in rectilinear, 25
circular or curvilinear motion. Angular speed of the revolution of the stone
(c) True, the direction of acceleration vector in uniform w = 2 pn
circular motion is directed towards the centre of the
22 14
circular path, which is continuously changing with time. =2 ´ ´
Therefore, the resultant of all these vectors over one cycle 7 25
will be a null vector. 88
= rad/s
2 25
mv
27. Tension, T = + mg cos q Centripetal acceleration of the stone ( a) = rw2
r
2
mv 2 æ 88 ö
For q = 30°, T1 = + mg cos 30° = 0.80 ´ ç ÷
r è 25 ø
mv 2 88 88
For q = 60°, T2 = + mg cos 60° = 0.80 ´ ´
r 25 25
\ T1 > T2 = 9.91 m/s 2
1 The direction of the acceleration is towards the centre of the
28. Difference in KE = m [( 5 gr) 2 - ( gr) 2]
2 horizontal circle, along its radius.
= 2 mgr = 2 ´ 1 ´ 10 ´ 1 = 20 J 35. Radius of horizontal loop (r) = 1km = 1000 m
29. Increment in angular velocity w = 2 p (n2 - n1) Speed of aircraft (v) = 900 km/h
rad 5
w = 2 p (1200 - 600) = 900 ´ m/s
min 18
2 p ´ 600 rad æ 5 ö
= çQ1 km/ h = m/ s÷
60 s è 18 ø
rad = 250 m/s
= 20 p
s Centrepetal acceleration of the aircraft
30. Here, r = 5 m,m = 0.5, w = ?, g = 10 ms–2 v2
a=
As, mrw2 = F = mR = m mg r
(250) 2 62500
mg 0.5 ´ 10 = =
w= = = 1 rad s–1 1000 1000
r 5
= 62.5 m/s 2
31. Here, r = 300 m,m = 0.3, g = 10 ms–2
Acceleration due to gravity ( g ) = 9.8 m/s 2
As, v max = m rg = 0.3 ´ 300 ´ 10 = 30 ms–1
Centrepetal acceleration ( a) 62.5
18 \ = = 6.38
= 30 ´ kmh –1 = 108 kmh –1 Acceleration due to gravity ( g ) 9.8
5
36. In the given condition friction provides the required
32. Here, h = 1m, r = 100 m, x = 1.5 m centripetal force and that is constant i. e. ,mw2r = constant.
For no skidding 1
Þ rµ 2
mv 2 w
´ h = mgx 2
r æw ö æ 1ö
2
\ r2 = r1 ç 1 ÷ = 9 ´ ç ÷
grx 9.8 ´ 100 ´ 0.75 è w2 ø è3ø
v= =
h 1
= 1cm
v = 27.1 ms–1
Telegram @unacademyplusdiscounts

174 JEE Main Physics

v 2 (60 ´ 5 / 18) 2 41. Here, r = 92 m, v = 26 ms–1, m = ?


37. As, m = = = 0.71
rg 40 ´ 9.8
mv 2
–2
As = F = mR = m mg
38. Here, v max = ?, r = 18 m, g = 10 ms r
and m = 0.2 v2
Þ m=
2
mv max rg
Now = F = mR = m mg 26 ´ 26
r = = 0.75
v max = m rg = 0.2 ´ 18 ´ 10 = 6 ms–1 92 ´ 9.8

18 42. As, F = mw2R


=6´ kmh –1 = 21.6 kmh –1 1
5 \ Rµ ( m and F are constants)
v2 w2
39. From, tan q = 1 R
rg If w is doubled then radius will become times is i. e. ,
4 4
v2 10 ´ 10
Þ r= = = 10 m v2
g tan q 10 ´ tan 45° 43. As, tan q = , therefore, when speed v is doubled; r must be
rg
40. Here, r = 7 m, v = 5 ms–1, q = ? made 4 times, if q remains the same.
v2 5 ´5 \ New radius of curvature,
\ tan q = = = 0.364
rg 7 ´ 9.8 r ¢ = 4 r = 4 ´ 50 m = 200 m
q = tan -1 (0.364) = 20°

Round II
1. If v is velocity of the bob on reaching the lowest point, then æ 40 p ö
Þ N = 40 ç as a = ÷
1 è 16 ø
mv 2 = mgL
2 \Required number of rotation = 40 - 10 = 30
To avoid breaking, strength of the string 1
5. According to given problem, Mv 2 = as2 (Here, M = mass)
mv 2 2 mgL 2
TL = + mg = + mg = 3 mg
L L 2a
Þ v=s …(i)
2. When the force is acting on a body is directed towards a fixed M
point, then it changes only the direction of motion of the body v 2 2 as2
without changing its speed. So, the particle will describe a So, aR =
=
R MR
circular motion.
dv dv ds dv
Further more as at = = × =v (By chain rule) which
3. From figure in question dt ds dt ds
T sin q = Mw2R …(i) is light of Eq. (i)
and T sin q = Mw2L sin q …(ii) i. e. , v=s
2a
yields
Þ 2
T = Mw L = M × 4 p n L2 2 M
2 é 2a ù é 2a ù 2 as æ v 2a ö
æ2ö at = ê s úê ú= çQ = ÷
= M × 4 p 2 ç ÷ L = 16 ML
èpø ë Mû ë Mû M è s mø

1 So that, a = aR2 + at2


4. As, q = 2 pn = w0t + at 2
2 2
æ 2 as2 ö 2 æ 2ö
1 æ 2 as ö 2 as ç 1 + æç s ö÷ ÷
Þ 2 p ´ 10 = a ( 4) 2 = ç ÷ +ç ÷ =
2 è MR ø è M ø M ç èRø ÷
è ø
40 p
or a= \ F = Ma = 2 as 1 + ( s / R) 2
16
Let is makes N rotations in first 8s, then 6. Here the tangential acceleration also exists which requires
1 power.
2 pN = a 8 2
2 Given that, ac = k2 × rt 2
Telegram @unacademyplusdiscounts

Circular Motion 175

v2 vB2 + gL 4
and ac = or =
r vT2 - gL 1
v2 or vB2 + gL = 4 vT2 - 4 gL
\ = k2 r t 2
r but vB2 = vT2 + 4 gL
or v 2 = k2r 2t 2 or v = krt
\ vT2 + 4 gL + gL = 4 vT2 - 4 gL
dv
Tangential acceleration a = = kr Þ 3 vT2 = 9 gL
dt
10
Now force F = m × a = mkr \ vT2 = 3 ´ g ´ L = 3 ´ 10 ´
2 2 3
So power P = F ´ v = mkr ´ krt = mk r t
or vT = 10 m/s
7. The maximum speed without skidding is 2 pr pr
12. Here, T = =
v = m rg 4v 2v
v2 m2 m /2 1 Change in velocity is going from A to B = v 2
\ = = = (for rg = constant)
v1 m1 m 2 v 2 2 2 v2
Average acceleration = =
v1 pr /2 v pr
v2 = = 5 2 ms–1 (Q v1 = 10 ms-1)
2
13. The mass of flywheel = 20 kg
1
8. Here, (KE)L - (KE)H = m (vL2 - vH2 ) 20 1
2 Radius = 20 cm = m= m
100 5
1
= m (5 gr - gr) = 2 mgr 1 1 1
2 The moment of inertia = mR 2 = ´ 20 ´
2 2 5
= 2 ´ 1 ´ 10 ´ 1 = 20 J
Þ I = 0.4 kg-m2
9. Initial angular velocity w0 = 0. \ t = I a ÞF ´ r = I a
Final angular velocity 1
Þ 25 ´ = 0.4 ´ a
v 80 5
w= = = 4 p rads–1
r (20 / p )
\ a = 12.5 rad / s2
angle described, q = 4 p rad 900 ´ 1000
w2 - w20 14. Here, v = 900 kmh–1 = ms–1 = 250 ms–1
\Angular acceleration, a = 60 ´ 60
2q Maximum force is at bottom of the vertical circle
( 4 p) 2 - 0 mv 2
= = 2 p rads–2 Fmax = + mg = 5 mg
2´4p r
Linear acceleration, a = ar = 2 p ´
20
= 40 ms–2 \ v 2 = 4 gr
p
v 2 250 ´ 250
or r= = = 1594 m
10. Here, r = 2 t , q = 4 t 4g 4 ´ 980
As, l = rq = (2 t ) ( 4 t ) = 8 t 2 15. For a particle in uniform circular motion.
dl d y
\ v= = (8 t 2) = 16 t
dt dt
P(R, θ)
Þ v t = 16 ms–1 (at t = 1s) ac

11. Since the maximum tension TB in the string moving in the


x
vertical circle is at the bottom and minimum tension TT is at ac
the top.
mvB2
\ TB = + mg
L
mvT2 v2
and TT = - mg a= towards centre of circle
L R
mvB2 v2
+ mg a= ( - cos q $i - sin q $j)
TB 4 R
\ = L2 =
TT mvT 1 v2 v2
- mg or a=- cos q $i - sin q $j
L R R
Telegram @unacademyplusdiscounts

176 JEE Main Physics

16. As we know that 27. Here, 2 x = 1.1m, x = 1.1/ 2 m


v2 h = 50 cm = 0.5 m,r = 200 m
tan q =
rg grx 9.8 ´ 200 1.1
v= = ´
where, v = speed and r = radians h 0.5 2
(10) 2 = 46.4 ms–1
\ tan q = =1
10 ´ g
28. When speed of car exceeds 46.4 ms -1, the reaction on inner
Þ q = 45° wheel reduces to zero. Therefore, the car topples outwards.
AB
17. In figure, sin 30° = 29. When the curve is banked, safe speed
OA
v = rg tan q
O
200 ´ 9.8
= 200 ´ 9.8 tan 30° = = 33.6 ms–1
θ 3

30. In calculating the value of this safe speed, we have neglected


T friction, i. e. , frictional force is taken as zero.
31. Let m be the coefficient of static friction between the tyres and
B
A the road, the magnitude of friction force F will not exceed
4
m mg, so that F £ m mg
mg
mv 2
Hence, for a safe turn £ m mg
AB 4 r
or OA = = =8m
sin 30° 1/2 v2
or m³ or v £ m rg
T F mg rg
= =
AO AB OB Hence, when speed of car (automobile) exceeds the value of
AO AO mv 2 8 52 m rg then it overturns, as the inner wheels are moving in a
F= ´F = = ´ 10 ´ » 125 N
AB AB r 4 4 cricle of smaller radius, the maximum possible velocity is less
18. When a particle moves along a circle with constant speed, it for it. Therefore, the wheels leave the ground first and car will
v2 overturn on the outside.
has centripetal acceleration a = . Its direction is always
r 32. The body is able to move in a circular path due to centripetal
towards the centre of the circle, which goes on varying. force. The centripetal force in case of vehicle is provided by
1 frictional force. Thus if the value of frictional force m mg is less
Therefore, a is variable. But kinetic energy of particle = mv 2,
2 than centripetal force, then it is not possible for a vehicle to
which remains constant. take a turn and the body would overturn.
19. Centripetal force always acts perpendicular to the mv 2
Thus condition for safe turning of vehicle is m mg ³
instantaneous displacement of the body on circular path so, r
q = 90° and W = Fs cos 90° = 0.
33. On an banking road friction provides the necessary
2 v mv
21. Centripetal acceleration ac = = vw centripetal force = m mg
r r
When v is doubled, wis halved, (vw) shall remain unchanged. \ v = m rg
22. In uniform circular motion, at = 0, but in non-uniform circular Thus, with increase in friction, safe velocity limit also
motion, at ¹ 0. In both cases, ar ¹ 0. increases.
23. When vL = 5 gr ,vH = gr = 9.8 ´1 = 3.13 ms-1 When the road is banked with angle of q then its limiting
rg (tan q + m)
velocity is given by, v =
24. When vL = 5 gr ,TH = 0 1 - m tan q

25. vL = 5 gr = 5 ´ 9.8 ´1 = 49 = 7 ms–1 Thus, limiting velocity increase with banking of road.
v2
As velocity of projection at the lowest point is 6 ms -1, i. e. ,less 34. For safe turn, tan q ³
than 5 gr, the stone cannot loops the loop. rg
It is clear that for safe turn v should be small and r should be
26. To avoid skidding on unbanked curve, large. Also bending angle from the vehicle would increase in
v = m rg = 0.2 ´ 200 ´ 9.8 = 19.8 ms–1 velocity.
Telegram @unacademyplusdiscounts

Circular Motion 177

35. When roads are not properly banked force of friction between v2
41. Centripetal acceleration, ac =
tyres and road provides partially the necessary centripetal R
force. This causes wear and tear. where v is the speed of an object and R is the radius of the
36. Centripetal force = Mrw2 = 5 ´1´ (22) = 20 N circle.

v2 It is always directed towards the centre of the circle. Since, v


37. Q tan q < and R are constants for a given uniform circular motion,
rg
therefore, the magnitude of centripetal acceleration is also
æ v2 ö constant. However the direction of centripetal acceleration
So, q < tan -1 ç ÷
è rg ø changes continuously. Therefore a centripetal acceleration is
not a constant vector.
38. Centripetal acceleration, 2
æ 2p ö -2
v 2 (30) 2 42. As, ac = w2R = ç ÷ (5 ´ 10 ) = 5 m/s
2
ac = = = 1.8 ms–2 è 0.2 p ø
r 500
Tangential acceleration, at = 2 ms–1 v2
43. For banking, tan q =
rg
\Resultant acceleration, a = at2 + ac2
v2
2 2 –2 tan 45° = =1
= (1.8) + (2) = 2.7 ms 90 ´ 10
39. Radius of circular path in the horizontal plane v 2 = 900
r = l sin q v = 900 m/s

Resolving T along the vertical and horizontal directions, we = 30


get, 44. Using the formula for motor cycle not to skid
T cos q = Mg …(i) æ v2 ö
2 2 q = tan -1 ç ÷
T sin q = Mrw = M ( l sin q) w è rg ø
or T = Mlw2 …(ii) where, r = 20 m
Dividing Eq. (ii) by Eq. (i), we get v = 72 kmh –1
1 lw2 g 5
= or w2 = = 72 ´ = 20 ms–1
cos q g l cos q 18
æ 20 ´ 20 ö
\ q = tan -1 ç ÷
è 20 ´ 10 ø
θ θ
l or q = tan -1 (2)
T
dv v 2
45. As, at = =
T cos θ dt R
v
t dt v dv é 1ù
A T sin θ r B
Þ ò0 R

v0 v2
,t = -R ê ú
ë v ûv 0
v 0R
Mg v=
(R - v 0t )
\ Time period,
dr v 0R
2p l cos q Now, =
t= = 2p dt (R - v 0t )
w g
2pR T dt
40. Here, mass of stone, m = 2 kg ò0 dr = v 0R ò
0 (R - v 0t )
…(i)

Length of string r = 0.5 m R


Þ T= (1 - e-2p)
Breaking tension T = 900 N v0
At T = mrw2 Put R - vt = z
T 900 and 0 - vdt = dz
or w2 = = = 900
mr 2 ´ 0.5 in Eq. (i) to get the result.
Þ w = 900 = 30 rad/s
Telegram @unacademyplusdiscounts

Laws of Motion
6 and Friction
JEE Main MILESTONE
< Force < Impulse
< Free Body Diagram (FBD) < Equilibrium of Concurrent Forces
< Inertia < Connected Motion
< Newton’s Laws of Motion < Friction
< Linear Momentum

6.1 Force
Force is a push or pull which
(i) generates or tends to generate motion in a body at rest.
(ii) stops or tends to stop a body in motion.
(iii) increases or decreases the magnitude of velocity of the moving body.
(iv) changes or tries to change the direction of a moving body.
(v) tends to change the shape of the body.
Newton’s laws of motion are of
central importance in classical
Classification of Forces mechanics (physics). A large
number of laws and results may
Based on the nature of interaction between two bodies, forces may be broadly
be derived by Newton’s laws. The
classified as under
first two laws related to the type
(a) Field forces are the forces that act between two bodies separated by a distance of motion of the system that
without any actual contact (Non-contact forces). Gravitational force between two
results from a given set of forces.
bodies and electrostatic force between two charges are examples of field forces.
Weight (w = mg ) of the body comes in this category.
(b) Contact forces are the forces that act between two bodies in contact e. g ., tension,
normal reaction, friction, etc.

Some Particular Types of Forces


Tension (T )
When a string, thread or wire is held taut, the pull on O T A
whatever bodies are attached to them in the direction
of the string is known as tension. T

If the string is massless, then


B
(i) the tension T has the same magnitude at all points
throughout the string.
(ii) the magnitude of acceleration of any number of
masses connected through string is always same.
Telegram @unacademyplusdiscounts

Laws of Motion and Friction 179

(iii) if there is friction between string and pulley, tension is These forces are known as normal reaction forces.
different on two sides of the pulley but if there is no Normal reaction forces in different situations are shown
friction between pulley and string, tension will be below
same on both sides of the pulley. N
Note
¾ If string slacks, tension in string becomes zero. ⇒
(a) N
¾ The direction of tension on a body or pulley is always away from
the body or pulley. Direction of normal Direction of normal
reaction on reaction on
the block the surface
The direction of tension in some cases are shown below N
(b)
(a) T T m m m
m2 m1 F ⇒ m2 1

θ θ
T T
m1 ⇒ m1
Inclined plane Direction of normal
reaction on the block
N
(b) T
String is massless
T
and pulley is
light and smooth m2
m2 θ N

Direction of normal Normal reaction on


reaction on the horizontal surface
inclined plane
(c) The number of normal reaction pairs is equal to
(c) number of contact surfaces.
Þ T2 T1
T3 m2 B NB
NA
T4 A m1 ⇒ B
m1 m1

m2 m2 The normal reaction on upper block is in the upward


direction and normal reaction on lower block is in the
String is not massless and there is
friction between pulley and string downward direction.
N2
wall

The Normal Reaction Force N1


(d)
When a body is pressed against a surface, the surface Sphere N1
(even a seemingly rigid surface) deforms and pushes on N2
the body with a normal reaction force N that is
perpendicular to the surface. For a simple introduction to
the normal reaction force, consider a situation in which
you put a book on your head and continue your stationary N2 N1
position. In this case, the pain you feel is due to the force
⇒ C
that the book exerts on your head. (e)
C
N1 N2
In the language of physics, the book exerts a force on your
θ θ θ
head normal to the surface of contact in downward
direction. According to Newton’s third law of motion, the
N2 N2
head exerts a force of same magnitude on the book
normal to the surface of contact and in upward direction. (f) ⇒ N1 N1
Normal reaction is the adjustable force that acts at the
contact points of two bodies in direction normal to the
surface.
Telegram @unacademyplusdiscounts

180 JEE Main Physics

Sample Problem 1 Find the normal reaction on a block of Interpret (b)


N
mass M lying over a horizontal surface.

M
N′
M N

(a) zero (b) Mg Mg


(c) 2 Mg (d) 3 Mg
N - N¢ = Mg
Interpret (b) Here the number of contact points between block Þ N = (M + m) g
and surface are infinite so the normal reaction means the net normal Þ N = ( 4 + 2) ´ 10 = 60 N
reaction. Let it be N
N Note In the above two problems we see that between the same
surfaces, the normal reaction takes different value means it is adjustable.

Spring Force
w (Mg) Consider a light spring tied to a vertical wall, which is
Free body diagram of block being pulled to right and the final elongation of the spring
is x and at that moment, the force applied is said F.
As body is under static equilibrium the net force on the body in
vertical direction is zero.
N - Mg = 0 Þ N = Mg
F
x
Sample Problem 2 A round table with four symmetrical
pointed legs is lying over a horizontal surface. If mass of the
table is M, then normal reaction of the surface on each leg is Ideal spring follows Hooke’s law which says that force
equal to applied by spring on bodies connected to it is
(a) 2 Mg (b) Mg/4 proportional to extension or compression (change over
1 natural length) and is always in an opposite direction to
(c) 3 Mg (d) Mg
2 extension or compression produced. So,
Interpret (b) Since all four legs are identical therefore 4N F µ -x
normal reactions at all contact points are same and are in Þ F = - kx
vertical direction. Let it be N.
where k is a constant that is a characteristic of the spring
As table is at rest, the net force acting is zero known as spring constant or force constant.
4 N - Mg = 0 (considering vertical direction) Mg
Mg L L+x L–x
Þ N=
4 m m m

Spring in natural Spring in the condition Spring in the condition


Sample Problem 3 A block of mass 4 kg lies over a length of elongation of compression
horizontal surface (g = 10 ms–2). The normal reaction between (a) (b) ©
the block and the surface is
(a) 10 N (b) 30 N L L–x L+x
(c) 40 N (d) 1 N F=0 F = kx F = kx
m m m
Interpret (c) N - Mg = 0
No Spring force Spring force on the Spring force on the
Þ N = Mg = 4 ´ 10 = 40 N on body body is leftward body is leftward
(d) (e) (f)
Sample Problem 4 In the above problem if another block
of mass 2 kg is placed over the first block, the normal reaction
between the first block and the ground surface in new case is Weight (w )
(a) 40 N It is a field force, the force with which a body is pulled
(b) 60 N towards the centre of the earth due to its gravity. It has the
(c) 20 N magnitude mg, where m is the mass of the body and g is
(d) 10 N the acceleration due to gravity.
Telegram @unacademyplusdiscounts

Laws of Motion and Friction 181

Weight of a Body in a Lift


Earth attracts every body towards its centre. The force of So, apparent weight
attraction exerted by the earth on the body is called w¢ = mg + R = mg + ma = m ( g + a ).
gravity force. If m be the mass of the body then the Hence, apparent weight w¢ > actual weight w.
gravity force on it is mg. Normally, the weight of a body
is equal to the gravity force w = mg. Case III The lift is accelerated in downward direction
As shown in Fig. (iii). The weight w = mg is acting
But when the body is on an accelerating platform, the
in downward direction while the reaction
weight of the body appears to be changed. The changed
R = ma acts upward we assume a < g
weight is termed as apparent weight. Here, we consider
the apparent weight of a man standing in a moving lift. \ Apparent weight,
w¢ = w - R
Consider the following cases
= mg - ma = m ( g - a )
a=0 Hence, apparent weight w¢ < actual weight w.
a a<g
Special case when, g = a
In this case apparent weight w¢ = 0.
Thus in a freely falling lift, the man will
experience a state of weightlessness.
w = mg w = mg + ma w = mg – ma
Case IV The lift is accelerated in downward direction
(I) (ii) (iii)
such that a > g.
Case I The lift is moving with a uniform velocity (i.e., a = 0). In this case the reaction force, R = ma and is
In this case normal reaction, R = ma = 0 greater than the weight mg.
Hence, apparent weight w = actual weight, w = mg Apparent weight,
w¢ = m ( g - a ) = (–ve)
Case II The lift is accelerated in upward direction
Hence, the man will be accelerated in upward
In this case, two forces acting on the man i. e. ,
direction and will stick to the ceiling of the lift.
weight mg and reaction, R = ma both are acting in
the downward direction.

Gravitational Force 6.2 Free Body Diagram (FBD)


The force of attraction between two bodies by virtue of A free body diagram (FBD) consists of a diagramatize
their masses is called gravitational force. representation of a single body or sub-system of bodies
Let two objects of masses m1 and m2 be separated by a isolated from its surroundings showing all forces acting
distance d. on it.
The force on object A due to object B is FAB acting towards A free body diagram is helpful in solving problems like
m2 along the line joining m1 and m2. Similarly, the force on several bodies connected by strings, springs, chains etc.,
object B due to object A is FBA acting towards m1 along the using Newton’s second law.
line joining m2 and m1. While sketching a free body diagram the following points
A B should be kept in mind.
d 1. Normal reaction (N) always
FAB FBA acts normal to the surface on A B
which the body is kept.
Gm1m2
So, FAB = FBA = 2. When two objects A and B are connected by a string,
d2
the tension for object A is towards B and for object B, it
Here, G = gravitational constant = 6.67 ´ 10–11 Nm2 kg–1. is towards A.
Telegram @unacademyplusdiscounts

182 JEE Main Physics

Note Always remember that while sketching a FBD, tension in each


section must form an action-reaction pair. Check Point 1
3. If the pulley is light and frictionless and a string passes
1. If a body is not at rest position, then the net external force
over it without any links, then tension on either side of
acting on it cannot be zero. Is this statement true?
the string is the same.
2. The distance travelled by a moving body is directly
4. While sketching FBD of an object, always take into proportional to time. Is any external force acting on it?
account the forces acting on the body and not those
which the body exerts on others.
3. Can a body remain at rest position when external forces are
acting on it?
5. The friction is a tangential force acting tangentially to
4. If force is acting on a moving body perpendicular to the
the surface in contact.
direction of motion, then what will be its effect on the speed
For Example and direction of the body?
5. The two ends of a spring-balance are pulled each by a force of
A rectangular object of mass M lying on F
10 kg-wt. What will be the reading of the balance?
a smooth horizontal surface is being θ 6. A lift is being accelerated upwards. Will the apparent weight of
pulled by a force F at an angle q with the
a person inside the lift increase, decrease or remain the same
horizontal. M
relative to its real weight? If the lift is going with an uniform
Forces acting on the object are speed, then?
(i) Applied force represented as F 7. A 5 kg body is suspended from a spring-balance and an
identical body is balanced on a pan of a physical balance. If
(ii) Weight = Mg in vertical downward direction.
both the balances are kept in an elevator then what would
Reaction of surface on object N in vertical upward happen in each case when the elevator is moving with an
direction. upward acceleration?
Free body diagram Break up of forces
is shown along axes
N F sin θ
y
6.3 Inertia
θ F cos θ
The inability of a body to change by itself its state of rest or
x
state of uniform motion along a straight line is called
inertia of the body.
(Mg) w
w As inertia of a body is measured by the mass of the body.
Heavier the body, greater the force required to change its
Three blocks A, B and C are placed one over the other as state and hence greater is its inertia. inertia obviously
shown in figure. three types (i) inertia of rest (ii) inertia of motion (iii) inertia
of direction.
A

C
6.4 Newton’s Laws of Motion
Free body diagrams of A, B and C are as shown below
First Law of Motion (Law of Inertia)
Here, N1 = normal reaction between A and B, It states that a body continues to be in a state of rest or of
uniform motion along a straight line, unless it is acted
N 2 = normal reaction between B and C and
upon by some external force to change of state. This is also
N3 = normal reaction between C and ground called law of inertia.
N1 N2 If F = 0, Þ v = constant Þ a = 0
(i) This law defines force.
(ii) The body opposes any external change in its state of
wA wB wC
rest or of uniform motion.
N1 N2 N3 (iii) It is also known as the law of inertia given by Galileo.
FBD of A FBD of B FBD of C
Telegram @unacademyplusdiscounts

Laws of Motion and Friction 183

Examples Third Law of Motion


(i) When a stationary vehicle suddenly moves, then the
To every action, there is an equal and opposite reaction.
passengers inside the vehicle fall backward.
F12 = - F21
(ii) If a moving vehicle suddenly stops, then the
passengers inside the vehicle bend forward. (i) Action and reaction are mutually opposite and act on
two different bodies.
(iii) When a carpet or a blanket is beaten with a stick, then
the dust separates out from it. (ii) The force acting on a body is known as action.
(iii) When a force acts on a body, then the reaction acts
normally to the surface of the body.
6.5 Linear Momentum Examples
(i) Jumping of a man from a boat onto the bank of a river.
It is defined as the total amount of motion of a body and is
measured as the product of the mass of the body and its (ii) Jerk is produced in a gun when bullet is fired from it.
velocity. (iii) Pulling of cart by a horse.

The momentum of a body of mass m moving with a


Sample Problem 5 A block of metal weighing 2 kg is
velocity v is given by p = mv.
resting on a frictionless plane. It is struck by a jet releasing
Its unit is kg-ms–1 and dimensional formula is [ML T –1] water at the rate of 1 kgs–1 and with a speed of 5 ms–1. The
initial acceleration of the block is
Second Law of Motion (a) 2 ms–2 (b) 2.5 ms–2
(c) 3 ms–2 (d) 3.5 ms–2
The rate of change of momentum of a body is directly
proportional to the applied force and takes place in the Interpret (b) The water jet striking the block at the rate of
direction in which the force acts. According to second law, 1 kgs–1 with a speed of 5 ms–1 will exert a force on the block
dm
dp dp F=v = 5 ´1 = 5 N
F µ or F = k dt
dt dt
where, k is constant. Under the action of this force of 5 N, the block of mass 2 kg will
move with an acceleration given by
dp d
as, = (mv) = ma F 5
dt dt a = = = 2.5 ms–2
m 2
m dv
or = ma
dt Sample Problem 6 Momentum of a block as a function of
i. e. , second law can be written as time is given by p = (10 t 2$i + 5 t$j) kgms-1. The force acting on
dp
F = = ma the block at t = 2 s is
dt
(a) F = (20 $i + 6$j) N (b) F = ( 40 $i + 5$j) N
The SI unit of force is newton (N) and in CGS system is
(c) F = (10 $i + 5$j) N (d) F = (5 $i + 10 $j) N
dyne.
1 N = 105 dyne Interpret (b) As, F = dp = d (10t 2 $i + 5 t$j)
dt dt
d d
Þ F = (10 t 2 $i) + (5 t$j)
6.6 Impulse dt dt
Þ F = 20 t $i + 5$j
Impulse received during an impact is defined as the
product of the average force and the time for which the Þ F = ( 40 $i + 5$j) N at, t = 2 s
force acts.
Impulse, I = Fav t Sample Problem 7 An astronaut accidentally gets
separated out of his small spaceship accelerating in inter stellar
Impulse is also equal to the total change in momentum of
space at a constant rate of 100 ms -2. The acceleration of the
the body during the impact.
astronaut the instant after he is outside the spaceship is (assume
Impulse, I = p2 - p1 that there are no nearby stars to exert gravitational force on
Impulse = Change in momentum him).
(a) 9.8 m/s 2 (b) zero
(c) infinity (d) 18.3 m/s 2
Telegram @unacademyplusdiscounts

184 JEE Main Physics

Interpret (b) Since, there are no rearby stars to exert Interpret (b) Change in momentum = m Dv
gravittional force on him and the small spaceship exerts negligible
Given, m = 0.15 kg,
gravitational attraction on him, the net force acting on the astronaut,
once he is out of the spaceship is zero. By the first law of motion, the \ Dp = 0.15 ´ 12 - ( -0.15 ´ 12)
acceleration of the astronaut is zero. Dp = 3.6 N s
In the direction from the batsman to the bowler.
Sample Problem 8 A bullet of mass 0.04 kg moving with a
speed of 90 ms -1 enters a having wooden block and is stopped
after a distance of 60 cm. The average resistive force exerted by
Principles of Conservation of Linear
the block on the bullet is Momentum
(a) 100 N (b) 270 N (c) 50 N (d) 298 N It states that if no external force is acting on a system, the
Interpret (b) The retardation a of the bullet is given by momentum of the system remains constant.
u 2 -90 ´ 90 According to second law of motion,
a=- = ms–2 = - 6750 ms–2
2s 2 ´ 0.6 dp
F=
The retarding force, by the second law of motion is dt
F = ma = 0.04 ´ 6750 = 270 N If no force is acting, then F = 0
dp
Note The actual resistive force, and therefore retardation of the bullet \ =0
dt
may not be uniform. The answer therefore only indicates the average
resistive force. Þ p = constant
or m1v1 = m2v2 = constant
Sample Problem 9 The motion of a particle of mass m is
decribed by Applications of the principle of
1 2
y = ut + gt
2 conservation of linear momentum
The force acting on the particle is [NCERT] (i) When a man jumps out of a boat to the shore, the
mg 3 boat is pushed slightly away from the shore. The
(a) mg (b) (c) 2 mg (d) mg
2 2 momentum of the boat is equal and opposite to that
of the man in accordance with the law of
Interpret (a) Given, y = ut + 1 gt 2 conservation of linear momentum.
2
dy (ii) Recoiling of gun When a bullet is fired from a gun, the
Now, v= = u + gt
dt gun recoils, i. e., moves in a direction or opposite to
dv the direction of motion of the bullet. The recoil
Acceleration, a= =g
dt velocity of the gun can be calculated from the
Force = Mass ´ Acceleration conservation of linear momentum.
F = ma = mg
1
From equation, v2 µ
Note The actual resistive force, and therefore retardation of the bullet m2
may not be uniform. The answer therefore, only indicates the average
It means that a heavier gun will recoil with a smaller
resistive force.
velocity and vice-versa.
Sample Problem 10 A batsman hits back a ball straight in (iii) Explosion of bomb When bomb falls vertically
the direction of the bowler without changing its initial speed of downwards, its horizontal velocity is zero and hence
12 ms -1. If the mass of the ball is 0.15 kg, the impulse imparted its horizontal momentum is zero. When bomb
to the ball is [NCERT Exemplar] explodes, its pieces are scattered horizontally in
(a) 1.5 Ns (b) 3.6 Ns different direction so that the vector sum of momenta
(c) 7.2 Ns (d) 10.2 Ns of these pieces becomes zero in accordance with the
law of conservation of momentum.
Telegram @unacademyplusdiscounts

Laws of Motion and Friction 185

Important Points of Newton’s Laws of Motion


1. A frame of reference which is at rest or which is moving with a 5. If a pulley is massless, net force T1
uniform velocity along a straight line, is called an inertial frame of on it is zero even, if it is
reference. accelerated. For example, in the
2. Newton’s laws are not valid in the non-inertial frame of reference. following figure.
T1
They are to be modified by introducing the concept of pseudo force. T2
Note If rope falls freely, T = 0 m1
3. The pseudo force is always directed opposite to the direction of the everywhere. P
acceleration of the non-inertial frame. T2 T2
FBD of pulley P
4. While drawing free body diagrams (FBD) in which pseudo force is
T2
involved, we must first see the acceleration of the non-inertial frame T2
and then in the FBD, plot the pseudo force with a value ma in a
direction opposite to the acceleration of the non-inertial frame.

6.7 Equilibrium of Concurrent In case, a number of forces act at a point, then they will be
in equilibrium, if they can be represented completely by
Forces the sides of a closed polygon taken in order.
If a number of forces act at the same point, they are called
concurrent forces. Lami Theorem
Consider that a body is under the action of a number of For three concurrent forces in equilibrium posision.
forces. Suppose that the body remains in equilibrium If three forces acting at a point be in
under the action of these forces i. e. , the body remains in its equilibrium, then each force is P R
β
state of rest or of uniform motion along a straight line, proportional to the sine of the angle
when acted upon these forces. γ α
between the other. Thus, if the forces
The condition that body may be in equilibrium or the are P, Q and R; a, b, g be the angles
number of forces acting on the body may be in between Q and R, R and P, P and Q
equilibrium is that these forces should produce a zero respectively, also the forces are in Q
resultant force. equilibrium, we have
Therefore, the resultant of three concurrent forces will be P Q R
= =
zero and hence they will be in equilibrium , if they can be sin a sin b sin g
represented completely by the three sides of any triangle.
Telegram @unacademyplusdiscounts

186 JEE Main Physics

Hot Spot Connected Motion


When two objects of mass m 1 and m 2 tied at the ends of an inextensible string which passes over the light and frictionless
pulley. Suppose m 1 > m 2 , the heavier object m 1 moves downward and lighter object m 2 moves upward. If a be the
common acceleration of two objects. Since pulley is frictionless and light, then the tension in the string will be same on
both sides of pulley.
æ 2 m1m2 ö
or T =ç ÷g
è m1 + m2 ø
Therefore, Tension (T ) in the string can be calculated.

a T Sample Problem 11 Two masses M and M/2 are joined


together by means of light inextensible string passes over a
T
mg frictionless pulley as shown in figure. When bigger mass is
released, the small are will ascend with an acceleration of
m2g a

m1g

For the heavier object, the various forces are


(i) Its weight m1 g acting downwards.
(ii) The tension T acting upwards in the string. M/2
When this object moves downwards with acceleration a, then the net
M
downward force on it = m1a.
Hence, m1 g - T = m1a …(i) (a) g (b) g /2
For the lighter object, the various forces are (c) g /3 (d) 2 g
(i) Its weight m2 g acting downwards.
Interpret (c) Given, the mass of the heavier object, m1 = M and
(ii) The tension T acting upwards in the string. When this body
the mass of the light object, m2 = M /2.
moves upwards with acceleration a, then the net upward force
on it = m2 a Now, we have acceleration,
Hence, T - m2 g = m2 a …(ii) æ m - m2 ö
a=ç 1 ÷g …(i)
Adding Eq. (i) and (ii), we get è m1 + m2 ø
m 1g - m 2 g = ( m1 + m2 ) a Putting these values in Eq. (i), we get
( m - m2 ) g = ( m1 + m2 ) a æ Mö
çM - ÷
æ m - m2 ö è 2ø
or a=ç 1 a= g
÷g M
è m1 + m2 ø M+
2
Clearly, a < g æ 2M - M ö
\ Dividing Eq. (i) by (ii), we get ç ÷
=ç 2 g
m1g - T ma m 2M + M ÷
= 1 = 1 ç ÷
T - m2 g m2 a m2 è 2 ø
or m1m2 g - Tm2 = m1T - m1m2 g g
=
2 m1m2 g = T ( m1 + m2 ) 3
Telegram @unacademyplusdiscounts

Laws of Motion and Friction 187

Important Cases
1. Motion of blocks in contact 5. Pulley-mass system
B (i) Body accelerated on a horizontal surface by a falling body,
A N
F m2 s
m1
T
m2
F
Acceleration, a = Smooth
m1 + m2 T
m2 g
2. Motion of blocks connected by massless string m1
a
B
A m1g
m2 F
m1
æ m1 ö
Acceleration, a = ç ÷g
F è m1 + m2 ø
Acceleration, a=
m1 + m2 æ mm ö
Tension, T = ç 1 2 ÷ g
m1F è m1 + m2 ø
and tension, F =
m1 + m2 (ii) Motion on a smooth inclined plane

3. Motion of massive string N T


a
T
m F m2
M a m1
m2 g sin θ a
m2 g cos θ
θ
F m1 g
a= m2 g
M+ m
MF
T = æ m - m sin q ö m m (1 + sin q)g
( M + m) \ a=ç 1 2 ÷ g and T = 1 2
è m1 + m2 ø (m1 + m2 )
4. Tension in spring
(iii) Body accelerated on a wedge due to a another falling body(m2 > m1)
A
a a
P'
T T
m1 m2
B
L
x
α Smooth β
C m1m2
F T = (sin a + sinb ) g
m1 + m2
Tension, T = F + Mg æ m sinb - m1 sin a ö
and a=ç 2 ÷g
M è m2 + m1 ø
T¢=F + xg
L
Telegram @unacademyplusdiscounts

188 JEE Main Physics

Sample Problem 12 As shown in figure A, B and C are T1 cos q = T2 = 60 N


1 kg, 3 kg and 2 kg respectively. The acceleration of the system is T1 sin q = 50 N
(a) 5 ms–2 (b) 4.11 ms–2 Which gives that
(c) 4 ms–2 (d) 5.11 ms–2 5
tan q =
6
Interpret (b) Net pulling force
æ5ö
or q = tan -1 ç ÷ = 40°
è6ø

B C
Sample Problem 14 A wooden block of mass 2 kg rests on
a soft horizontal floor. When an iron cylinder of mass 25 kg is
A
placed on top of block, the floor yields steadily and the block and
60° the cylinder together go down with acceleration of 0.1 ms -2. The
action of the block on the floor after the floor yields is [NCERT]
= mA g sin 60° + mg g sin 60° - mC g sin 30°
3 3 1 (a) 500 N (b) 250.4 N
= 1 ´ 10 + 3 ´ 10 - 2 ´ 10 ´
2 2 2 (c) 267.3 N (d) 651.3 N
= 24.64 N
Interpret (c) The system (block + cylinder) accelerated
Total mass being pulled = 1 + 3 + 2 = 6 kg
downwards with 0.1ms–2. The free body diagram of the system
\Acceleration of the system, shows two forces on the system, the force of gravity due to the earth
24.66 (270 N), and the normal force R ¢ by the floor.
a= = 4.11ms–2
6 R′

Sample Problem 13 A mass of 6 kg is suspended by a rope


of length 2 m from the ceiling. A force of 50 N in the horizontal
direction is applied at the mid-point P of the rope as shown. The
angle that rope makes with the vertical in equilibrium is
270 N
T1
1m θ 0.1 ms–2 Free-body diagram
of the block + cylinder
P 50 N system
1m T2 Applying the second law to the system,
270 - R ¢ = 27 ´ 0.1N
w
i. e. , R ¢ = 267.3 N
60 N
By the thrid law, the action of the system on the floor is equal to
(a) 10° (b) 20°
267.3 N vertically downwards.
(c) 30° (d) 40°
Interpret (d) Making the free body diagram of P and T2
w, we have consider the point of equilibrium of the 6.8 Friction
weight w
w
\ T2 = 6 ´ 10 = 60 N Whenever an object actually slides or rolls over the
60 N surface of another body or tends to do so, a force opposing
Consider the equilibrium of the point P under the action
of three forces, the tensions T1 and T2 and the horizontal the relative motion acts between these two surfaces in
force 50 N. The horizontal and vertical components of contact. It is known as friction or force due to friction.
the resultant force must vanish separately. Force of friction acts tangential to the surfaces in contact.
Limiting friction is the limiting (maximum) value of static
T1 friction, when a body is just on the verge of starting motion
θ over the surface of another body.

P 50 N f1 = mN

T2
Telegram @unacademyplusdiscounts

Laws of Motion and Friction 189

Kinetic friction is the opposing force that comes into play, Angle of Repose (a)
when one body is actually sliding over the surface of
This angle is relevant to an inclined plane. If a body is
another body.
placed on an inclined plane and it is just on the point of
Static friction is opposing force that comes into play, when sliding down, then the angle of inclination of the plane
one body is at rest and tends to move over the surface of with the horizontal is called the angle of repose (a).
another body f s = F R F
The static friction between two contact surfaces is given by
f s £ m s N , where N is the normal force between the contact
α
surfaces and m s is a constant coefficient of static friction,
which depends on the nature of the surfaces and is called mg sin α
coefficient of static friction. mg mg cos α
α
Rolling friction is the opposing force that comes into play,
when one body of symmetric shape (wheel or cylinder or In limiting condition, F = mg sin a and
disc, etc.) rolls over the surface of another body. Force of R = mg cos a
rolling friction f r is directly proportional to the normal F F
reaction N and inversely proportional to the radius (r) of So, = tan a \ = m s = tan a
R R
wheel. Thus,
N Thus, the coefficient of limiting friction is equal to tangent
fr µ
r of the angle of repose.
N
or fr = m r As well as, a=q
r
i. e. , Angle of repose = Angle of friction
The constant m r is known as the coefficient of rolling
friction, m r has the unit and dimensions of length. Angle of Friction (q)
Magnitude wise As shown in figure, a body A is in contact with surface B.
m r << m k or m l
The forces acting on A are as shown. Surface B applies two
contact forces on body A.
Laws of Friction P N
(i) If the body does not move, then the static frictional
force f s and the component of F that is parallel to the
surface are equal in magnitude and f s is directed
opposite to that component. If the parallel component θ A
F
of force increases, f s also increases. If the applied fL
parallel component exceeds a certain (maximum) θ Angle of friction O B
value, the body slides on the surface.
(ii) Static friction takes its peak value ( f s (max) = m s N ) when
the surface is about to slide on the other. Static friction
in this case is called limiting friction.Otherwise, w = mg
f s £ m s N. Under these conditions, the value of
(i) Normal reaction force N, whose magnitude is equal to
frictional force is determined from the analysis of the
weight w ( = mg ) of body and is directed upwards.
physical situation. We suggest, you have to apply
S F = ma (or with the given constraints) for computing (ii) Maximum static frictional force f L (= mN ), which is
the values of f s . tangential to the surface of A and directed opposite to
the direction of applied force F.
(iii) If the body begins to slide on the surface, the
magnitude of the frictional force rapidly decreases to a The resultant of these two forces is represented by the
constant value fk (kinetic friction) vector OP. The angle between vector OP and normal
Given, fk = m k N reaction force N is the angle of friction (q).
where, m k is the coefficient of kinetic friction. f mN
\ tan q = L = =m
N N
or q = tan-1 (m )
Telegram @unacademyplusdiscounts

190 JEE Main Physics

Sample Problem 15 What is the acceleration of the block Sample Problem 17 A mass of 4 kg rests on a horizontal
and trolley system shown in the figure, if the coefficient of plane. The plane is gradually inclined until at an angle q = 15°
kinetic friction between the trolley and the surface is 0.04. with the horizontal, the mass just begins to slide. The
(g = 10 ms-2, neglect the mass of the string) [NCERT] coefficient of static friction between the block and the surface is
20 kg [NCERT]
N fs B
w T
fk
θ
sin mg cos θ
mg
T mg
O A
3 kg w
(a) 1.5 (b) 0.27
30 N (c) 0.83 (d) 5.5

(a) 0.42 ms -2
(b) 0.96 ms -2 Interpret (b) The forces acting on a block of mass m at rest on
an inclined plane are
(c) 20 ms -2 (d) 40 ms -2
(i) the weight mg acting vertically downwards
Interpret (b) As the string is inextensible, and the T
(ii) the normal force N of the plane on the block
pulley is smooth, the 3 kg block and the 20 kg trolley both
(iii) the static frictional force fs opposing the impending motion.
have same magnitude of acceleration. w
Applying second law to motion of the block In equilibrium, the resultant of these forces must be zero. Resolving
the weight mg along the two directions, shown, we have
30 - T = 3 a …(i) 30 N mg sin q = fs
Applying second law to motion of the trolley,
mg cos q = N
fk w
T
As q increases, the self adjusting frictional force fs increases until at
T - fk = 20 a q = qmax , fs achieves its maximum value
fk = m k N ( fs ) max = m sN
Here, m k = 0.04 \ tan qmax = m s
N = 20 ´ 10 = 200 N or qmax = tan -1 m s
Thus, the equation for the motion of the trolley is When q becomes just a little more than qmax , there is a small net
T - 0.04 ´ 200 = 20 a force on the block and it begins to slide. Note that qmax depends
or T - 8 = 20 a …(ii) only on m s and is independent of the mass of the block.
From Eqs. (i) and (ii), we get For qmax = 15°
a = 0.96 ms–2 m s = tan15°
= 0.27
Note The answer does not depend on the length of the rope
(assumed massless) nor on the point at which the horizontal force is Sample Problem 18 A particle of mass 1 kg rests on rough
applied. contact with a plane inclined at 30° to the horizontal and is just
about to slip. Then the coefficient of friction between the plane
Sample Problem 16 The maximum acceleration of the and the particle will be
train in which a box lying on its floor will remain stationary, is 1
(a)
[Given coefficient of static friction between the box and the 2
train’s floor is 0.15] [NCERT] 1
(b)
(a) 0.12 ms -2 (b) 1.5 ms -2 3
(c) 3.4 ms -2 (d) 5.2 ms -2 1
(c)
3
Interpret (b) Since the acceleration of the box is due to static
3
friction, (d)
2
ma = fs £ m sN = m s mg
i. e. , a £ m sg Interpret (b) The given angle 30° is the angle of repose a.
–2 1
i. e. , amax = m s g = 0.5 ´ 10 ms So, m = tan a = tan30° =
3
= 1.5 ms–2
Telegram @unacademyplusdiscounts

Laws of Motion and Friction 191

Some Special Cases


Case I Acceleration of a block on a horizontal and net retarding force = mg sin l + f
surface As shown in figure, we have = mg sin l + m mg cos l
N \ External force needed (up the inclined plane) to
maintain sliding motion
F = mg (sin l + m cos l )

F
In the absence of external force, the motion of given block
f = µN
will be retarded and the value of retardation will be
a = g (sin l + m cos l )

mg
Case IV Motion of two bodies, one resting on the
other
N = mg (i) Let a body A of mass M is placed on a smooth surface
and f = mN = m mg and a block B of mass m be placed on A. Let coefficient
of friction between surfaces of A and B be m . If a force F
\Net acceleration produced
is applied on the lower body A as shown in figure, then
F-f
a=
m f ′ = ma B
m f = µmg
F - m mg F
= = - mg A
F
m m M

where, m = coefficient of (kinetic) friction between the two Smooth surface


surfaces in contact. Common acceleration of the two bodies
F
Case II Acceleration of a block sliding down a rough a=
( M + m)
inclined plane Let there be an inclined plane having
angle of inclination l, which is more than the angle of pseudo force acting on block B due to the accelerated
motion f ¢ = ma. The pseudo force tends to produce a
repose a . Then as shown in figure
relative motion between bodies A and B and
N consequently a frictional force, f = mN = m mg is
µN
f=

developed. For equilibrium,


ma £ m mg or a £ mg
a If under the influence of force F the acceleration
m λ
produced exceeds mg, two bodies will not move
λ

sin mg mg cos λ together and the block B will slide backwards and after
g
m
λ(λ > α) some time fall from body A.
(ii) Let friction is also present between the ground surface
N = mg cos l and body A. Let m1 = coefficient of friction between the
and f = mN = m mg cos l given surface and body A and m 2 = coefficient of
friction between the surfaces of bodies A and B. If a
\ Net accelerating force down the inclined plane,
force F is applied on the lower body A as shown in
mg sin l - f = mg sin l - m mg cos l = ma figure, then
Acceleration, a = g (sin l - m cos l )
f ′ = ma B
m fB = µ2 mg
Case III Retardation of a N F A
block sliding up a rough fA = µ1(M + m) g M F

inclined plane In this case,


m
a Net accelerating force = F - f A
various forces have been shown
λ

sin λ = F - m1 ( M + m) g
in figure g
m \ Net acceleration,
N = mg cos l µN mg cos λ
f= λ mg F - m1 ( M + m) g F
a= = - m1 g
Force of friction, ( M + m) ( M + m)
f = mN = m mg cos l
Telegram @unacademyplusdiscounts

192 JEE Main Physics

Pseudo force acting on the block B, From, v2 = u2 - 2 as


f ¢ = ma Þ 0 = u2 - 2 mgs
The pseudo force tends to produce a relative motion
u2
between the bodies A and B and consequently a \ s=
frictional force fB = m 2mg is developed. For 2 mg
equilibrium, au é At v = u - at ù
and time taken to come to rest t = ê
ma £ m 2mg or a £ m 2 g mg ë 0 = u - mgt úû
If acceleration produced under the effect of force F is
more than m 2 g, then two bodies will not move together. Case VII Motion of an insect in the rough bowl The
insect crawls up the bowl upto a certain height h only till
Case V Maximum length of hung chain A uniform the component of its weight along the bowl is balanced
chain of length l is placed on the table in such a manner by limiting frictional force
that its l ¢ part is hanging over the edge of table without
O
sliding. Since, the chain has uniform linear density r
therefore, the ratio of mass and ratio of length for any R θ
y
part of the chain will be equal

L
F
We know, Insect
m mass hanging from the table h
m= 2 =
m1 mass lying on the table mg cos θ
mg mg sin θ
\ For this case, we can rewrite the above expression in
Let m = mass of the insect,
the following manner.
length hanging from the table r = radius of the bowl and
m=
length lying on the table m = coefficient of friction

(As chain has uniform linear density) For limiting condition at point A,
l¢ R = mg cos q …(i)
\ m=
l -l¢ and FL = mg sin q …(ii)
ml Dividing Eq. (ii) by (i), we get
By solving, l¢ =
(m + l ) F
tan q = L = m
R
Case VI Stopping of block due to friction A block of
mass m is moving initially with a velocity u on a rough r 2 - y2
\ =m
surface and due to friction it comes to rest after covering y
a distance s. r
Retarding force, f = ma = mR or y=
1+ m 2
Þ ma = m mg \ a = mg

Sample Problem 19 A 1200 mg automobile of annels a


level of curve of radius 200 m, on an unbanked road with a But, flimiting friction ³ ffriction
velocity of 72 kmh-1. What is the minimum coefficient of or m mg ³ ffriction
friction between tyres and road in order that the automobile
mv 2
may not skid? (Take g = 10 ms-2). or m mg ³
r
(a) 0.2 (b) 0.4
v2
(c) 0.34 (d) 0.41 \ m min =
gr
Interpret (a) In an unbanked road, the centripetal force is
20 ´ 20
provided by the frictional force. =
10 ´ 200
mv 2
\ ffriction = = 0.2
r
Telegram @unacademyplusdiscounts

Laws of Motion and Friction 193

Sample Problem 20 Figure below shows a man standing Therefore, net force on the man,
stationary with respect to a horizontal conveyor belt that is ma = 65 ´ 1 = 65 N
accelerating with 1 ms–2. If the coefficient of static friction The limiting friction between the shoes of the man and the belt is
between the man’s shoes and the belt is 0.2, upto what given by
acceleration of the belt can the man continue to be stationary F = m Mg = 0.2 ´ 65 ´ 9.8 N
relative to the belt? (Mass of the man = 65 kg).
If, the man can remain stationary upto an acceleration say a¢, then
Ma¢ = F
F 0.2 ´ 65 ´ 9.8
or a¢ = =
M 65
= 1.96 ms–2

Check Point 2
(a) 1.25 ms–2
(b) 1.96 ms–2 1. A thief jumps from the roof of a house with a box of weight w
on his head, what will be the weight of the box as experienced
(c) 2.5 ms–2 by the thief during jump?
(d) 3.6 ms–2
2. Action and reaction forces do not balance each other always,
Interpret (b) As the man is standing stationary w.r.t. the why?
horizontal conveyor belt, he is also accelerating at 1 ms–2, the 3. Automobile tyres have generally irregular projections over
acceleration of the belt. Thus, their surfaces. Why ?
Acceleration of the man, a = 1ms–2 4. Why is it difficult to climb up a greasy pole?
Mass of the man, 5. Why frictional force increase when two surfaces in contact are
polished beyond a certain limit?
M = 65 kg
Telegram @unacademyplusdiscounts

WORKED OUT
Examples
Example 1 A 0.2 kg object at rest is subjected to a force Þ R = m ( g + a) …(ii)
(0.3 $i - 0.4 $j) N. What is the velocity after 6 s ? where a is the acceleration of the nail. Since the nail penetrates a
distance x.
(a) (9$i - 12$j) (b) (8$i - 16$j)
v 02 - 0 2 = 2ax …(iii)
(c) (12$i - 9$j) (d) (16$i - 8$j)
Substituting v 0 From Eq. (i) and a from Eq. (ii) in
Solution Here, m = 0.2 kg, u = 0 Eq. (iii), we get
æ R - mg ö
F = (0.3 $i - 0.4 $j) 2gh = 2 ç ÷x
è m ø
v = ?, t = 6 s mg (h + x)
F (0.3 $i - 0.4 $j) æ 3 $ Þ R=
ö x
a= = = ç i - 2$j ÷
m 0.2 è2 ø
æh ö
Þ R = mg ç + 1÷
From v = u + at èx ø
æ3 ö
v = 0 + ç $i - 2$j ÷ ´ 6 = 9$i - 12$j
è2 ø Example 3 Two blocks of masses m1 = 2 kg and m2 = 1 kg
are in contact on a smooth horizontal surface as shown in the
Example 2 An iron nail is dropped from a height h onto a figure. A horizontal force F = 3 N is applied on the block m1.
sand bed. If it penetrates through a distance x in the sand before Find the contact force between blocks.
coming to rest, the average force exerted by the sand on the nail
m1 m2
is F
æx ö æx ö
(a) mg ç - 1÷ (b) mg ç + 1÷
èh ø èh ø (a) 1 N (b) 2 N
æh ö æh ö (c) 3 N (d) 4 N
(c) mg ç + 1÷ (d) mg ç - 1÷
èx ø èx ø
Solution Let the contact force between the blocks be F1. Since F1
Solution The nail hits the sand with a speed v 0 after falling is responsible for the acceleration of m2 hence it will be in the
direction of acceleration on m2 and on m1, it will be opposite to the
through a height h
direction of acceleration. Equations of motion are
Þ v 02 = 2 gh F - F1 = m1a …(i)
Þ v 0 = 2gh ...(i) F1 = m2a …(ii)
Let the nail stops after time t and after having penetrated through N1 N2
a distance x, into the sand. Since its velocity decreases gradually,
the sand exerts a retarding upward force R. The net force acting F1 F
F
on the nail is given by
Y m1g m2g
FBD of m1 FBD of m2
R
where a = acceleration of the blocks.
From Eqs. (i) and (ii), we get
X
F
a=
m1 + m2
m2F
Þ F1 = = 1N
å Fy = R - mg = ma m1 + m2
Telegram @unacademyplusdiscounts

Laws of Motion and Friction 195

Example 4 A mass M is hung with a light inextensible string Example 6 The pulleys and strings shown in figure are
as shown in the figure. Find the tension of the horizontal string. smooth and of negligible mass. For the system to the under
(a) 2 Mg equilibrium, the angle q should be
(b) 3 Mg
(c) 2 Mg
(d) 3 Mg θ

Solution As there is a load at P , so tension in AP and PB will be


different. Let these be T1 and T2 respectively. For vertical equilibrium m 2
of P
m m
B
30° (a) 0° (b) 60°
T1
(c) 45° (d) 30°
A P T2
T1 Solution In equilibrium,
For mass 2m, T cos q + T cos q = 2 mg
M
And for mass m, T = mg
Mg Þ 2T cos q = 2mg cos q = 2 mg
p
T2 cos 60° = Mg i. e. ,T2 = 2 Mg …(i) Þ q= = 45°
4
And for horizontal equilibrium of P
T1 = T2 sin 60° = T2 ( 3 / 2) …(ii) Example 7 A body of mass 5 kg is supported by a light cord.
Substituting the value of T2 from Eqs. (i) and (ii) Find the tension in the cord.
T1 = (2Mg ) ´ ( 3 / 2) = 3 mg (a) 20 N (b) 30 N
(c) 50 N (d) 40 N
Example 5 Two blocks of masses 6 kg and 4 kg connected Solution
by a rope of mass 2 kg are resting on a frictionless floor as
shown in the figure. If a constant force of 60 N is applied to the
6 kg block, find the tension in the rope at points A, B and C. m = 5kg
(a) 30 N, 25 N, 20 N respectively
(b) 20 N, 20 N, 30 N respectively (i) The body is isolated
(c) 30 N, 20 N, 25 N respectively (ii) The two forces acting on it are the tension (T) in the cord (­)
and the weight ( mg) ( ¯)
(d) None of the above
(iii) Equation of equilibrium is
Solution As the mass of the system is (6 + 4 + 2) = 12 kg and T - mg = 0
applied force is 60 N, the acceleration of the system is or T = (5) (10) = 50 N
F 60
a= = = 5 ms-2 (iv) Tension in the cord equals the weight of the body.
m 12
2 kg Example 8 If the stretch in a spring of force constant k is
6 kg
4 kg C B A F = 60 N doubled, find the ratio of final to initial force in the spring
P
Q T stretched from y to 2y.
(a) 1 N (b) 2 N
Now, at point A as tension is pulling the rope and the block of mass (c) 3 N (d) 4 N
4 kg.
Solution As we know that for a spring F = ky
TA = (2 + 4) ´ 5 = 30 N
F2 ky 2
Similarly for B and C =
F1 ky1
TB = (1 + 4) ´ 5 = 25 N
2y
and TC = 20 N = =2
y
Telegram @unacademyplusdiscounts

196 JEE Main Physics

Example 9 A box of mass 8 kg is placed on a rough inclined (a) 0.346 (b) 0.436
plane of inclination 30°. Its downward motion can be (c) 0.463 (d) 0.364
prevented by applying a horizontal force F then value of F for
Solution When the body is projected up the plane, there will be
which friction between the block and the incline surface is retardation aA such that
minimum, is
80 mg sin q + fL = maA
(a) (b) 40 3
3
40
or aA = g (sin q + m cos q) [as fL = mR = m mg cos q] …(i)
(c) (d) 80 3 Now from equation of motion
3
v = u + at
Solution For friction to be minimum and v 2 = u 2 + 2as
θ
os For the motion up the plane
Fc
0 = u - aAt A
θ F
and 0 = u 2 - 2aA s (Qv = 0)
sin
mg θ Eliminating u between these, we get
N
mg t A = 2s / AA ...(ii)

F cos q = mg sin q Now when the body slides down the plane there will be
1 acceleration aD such that
F = mg tan q = 80
3 R
tio
n R
Mo FL
Example 10 A block is placed over a plank. The coefficient
θ θ
of friction between the block and the plank is m = 0.2. Initially sin sin
mg θ mg θ
both are at rest, suddenly the plank starts moving with an mg cos θ mg cos θ
acceleration a0 = 4 ms-2. The displacement of the block in 1 s is 30° 30°
(g = 10 ms -2)
mg sin q - fbL = maD
(a) 1 m relative to ground (b) 2 m relative to plank
aD = g (sin q - m cos q) (Q fL = mF = mg cos q) …(iii)
(c) zero relative to plank (d) 2 m relative to ground
And from equation of motion
Solution Since mg = 0.2 ´ 10 = 2 ms-2 < a0 1
s = ut + at 2
2
The block stops over the plank
Acceleration of block relative to ground tD = 2s / aD (as u = 0) …(iv)
-2 According to the given problem
a = m g = 2 ms (in the forward direction)
1
And acceleration of the block relative to plank t A = 1 tD
2
ar = a - a0 = (2 - 4) = - 2 ms-2
From Eqs. (ii) and (iv)
i. e. , ar is 2 ms-2 in the backward direction.
tA aD 1
= =
Since, magnitude of a and ar are equal in magnitude. The tD aA 2
displacement of block relative to ground and plank are equal.
1 Now, substituting the values of aA and aD from Eqs. (i) and
From s = at 2 (iii) in the above, we get
2
sin q - m cos q 1
1 =
s = ´ 2 ´ 12 = 1 m sin q + m cos q 4
2
which on simplification gives
Example 11 A body of mass 5 ´ 10 -3 kg is launched upon a 3
m = tan q
rough inclined plane making an angle of 30° with the 5
horizontal. Find the coefficient of friction between the body 3 1 3
= ´ = = 0.346
and the plane if the time of ascent is half of the time of descent. 5 3 5
Telegram @unacademyplusdiscounts

Start Practice for


JEE Main
Round I (Topically Divided Problems)

Force, Momentum and Friction 5. A body of mass 0.05 kg is observed to fall with an
1. Two blocks are in contact on a frictionless table. One acceleration of 9.5 ms –2 . The opposite force of air on
has mass m and other 2m. A force f is applied on 2m the body is ( g = 9.8 ms –2 )
as shown in figure. Next the same force F is applied (a) 0.015 N (b) 0.15 N
from the right on m. In the two cases respectively, the (c) 0.030 N (d) zero
force of contact between the two blocks will be 6. A rocket with a lift-off mass 20000 kg is blasted
upwards with an initial acceleration of 5.0 m /s2 .
F F
2m m Calculate the initial thrust (force) of the blast.
[NCERT]
(a) 2 : 1 (b) 1 : 3 (a) 3 ´ 10 N5
(b) 2 ´ 10 N 5
(c) 1 : 2 (d) 3 : 1
(c) 4 ´ 105 N (d) 5 ´ 105 N
2. A ball of mass 0.2 kg is thrown vertically upwards by
applying a force by hand. If the hand moves 0.2 m 7. A lift is moving upwards with a uniform velocity v in
while applying a force and the ball goes upto 2 m which a block of mass m is lying. The frictional force
height further find, the magnitude of the force. offered by the block, when coefficient of the frictional
Consider g = 10 m/s2 force is m, will be
(a) 16 N (b) 20 N (a) zero (b) mg
(c) 22 N (d) 44 N (c) m mg (d) 2m mg

3. A wooden wedge of mass M and inclination angle a 8. Two blocks of masses m1 = 4 kg and m2 = 2 kg are
rests on a smooth floor. A block of mass m is kept on connected to the ends of a string which passes over a
wedge. A force P is applied on the wedge as shown in massless, frictionless pulley. The total downward
thrust onfigure,
the pulley
suchisthat
nearly
a block remains stationary with
respect to wedge. The magnitude of force P is (a) 27 N (b) 54 N
(c) 0.8 N (d) zero
9. A man wants to slide down a rope. The breaking load
m 2
P for the rope rd of the weight of the man. With what
M 3
α minimum acceleration should fireman slide down?
g g
(a) (b)
(a) ( M + m ) g tan a (b) g tan a 4 3
(c) mg cos a (d) ( M + m ) gcosec a 2g g
(c) (d)
4. Conservation of momentum in a collision between 3 6
particles can be understood from [NCERT Exemplar] 10. A body weighs 8 g when placed in one pan and 18 g
(a) conservation of energy when placed on the other pan of a false balance. If the
(b) Newton’s first law only beam is horizontal when both the pans are empty,
(c) Newton’s second law only the true weight of the body is
(d) both Newton’s second and third law (a) 13 g (b) 12 g (c) 15.5 g (d) 15 g
Telegram @unacademyplusdiscounts

198 JEE Main Physics

11. A body of mass 5 kg is acted upon by two 17. A constant force acting on a body of mass 3.0 kg
perpendicular forces 8 N and 6 N. Give the changes its speed from 2.0 m/s to 3.5 s. The direction
magnitude and direction of the acceleration of the of motion of the body remains unchanged. What is
body the magnitude and direction of the force ?
(a) 2 m/s2 at an angle 37° to force (a) 0.18 N, along the direction of motion
(b) 2 m/s2 at an angle 57° to force (b) 0.18 N, opposite to the direction of motion
(c) 0.28 N, along the direction of motion
(c) 4 m/s2 at angle 37° to force
(d) 0.28 N, opposite to the direction of motion
(d) 4 m/s2 at an angle 57° to force
18. If a body of mass m is moving on a rough horizontal
12. A sphere is accelerated upward by a cord whose surface of coefficient of kinetic friction m, the net
breaking strength is four times its weight. The electromagnetic force exerted by surface on the body
maximum acceleration with which the sphere can is
move up without breaking the cord is (a) mg 1 + m2 (b) mmg
(a) g (b) 3 g (c) 2 g (d) 4 g
(c) mg (d) mg 1 - m2
13. A body of mass 2 kg travels according to law
x ( t) = pt + qt2 + rt 3, where p = 3 ms –1, q = 4 ms –2 and 19. An open carriage in a goods train is moving with a
r = 5 ms –3. The force acting on the body at t = 2 s is uniform velocity of 10 ms–1. If the rain adds water
[NCERT Exemplar] with zero velocity at the rate of 5 kgs–1, then the
(a) 136 N (b) 134 N additional force applied by the engine to maintain
(c) 158 N (d) 68 N the same velocity of the train is
(a) 0.5 N (b) 2.0 N
14. The adjacent figure is the part of a horizontally (c) 50 N (d) 25 N
stretched net section AB is stretched with a force of
10 N. The tension in the section BC and BF are 20. A batsman deflects a ball by an angle of 45° without
E changing its initial speed which equal to 54 km/h.
What is the impulse imparted to the ball? (Mass of
150° 150°
D the ball is 0.15 kg).
(a) 4 kg-m/s
(b) kg-m/s
120°
(c) 2 kg-m/s
G C F H
(d) 5 kg-m/s
B 21. A body floats in a liquid contained in a beaker. If the
120° 120° whole system as shown in figure of all freely under
A gravity, then the upthrust on the body due to liquid is
(a) 10 N, 11 N
(b) 10 N, 6 N
(c) 10 N, 10 N
(d) Can’t calculate due to insufficient data
15. A bag of sand of mass m is suspended by a rope. A (a) zero
m
bullet of mass is fired at it with a velocity v and (b) equal to the weight of liquid displaced
20
(c) equal to the weight of the body in air
gets embedded into it. The velocity of the bag finally
(d) None of the above
is
v 20 v 22. An object at rest in space suddenly explodes into
(a) ´ 21 (b)
20 21 three parts of same mass. The momentum of the two
v v parts are 2 p$i and p$j . The momentum of the third
(c) (d)
20 21 part
16. The engine of a car produces an acceleration of 6 ms –2 (a) will have a magnitude p 3
in the car. If this car pulls another car of the same (b) will have a magnitude p 5
mass, then the acceleration would be (c) will have a magnitude p
(a) 6 ms–2 (b) 12 ms–2 (d) will have a magnitude 2p
(c) 3 ms–2 (d) 1.5 ms–2
Telegram @unacademyplusdiscounts

Laws of Motion and Friction 199

23. In figure, the blocks A, B and C each of mass m have 28. Two weights w1 and w2 are suspended from the ends
acceleration a1, a2 and a3 respectively. F1 and F2 of a light string over a smooth fixed pulley. If the
are external forces of magnitude 2 mg and mg pulley is pulled up with acceleration g, the tension in
respectively. Then the string will be
4 w1w2 2 w1w2 w1 - w2 w1w2
(a) (b) (c) (d)
A B C w1 + w2 w1 + w2 w1 + w2 2( w1 + w2 )

29. A ball of mass 1 kg hangs in equilibrium from two


strings OA and OB as shown in figure. What are the
tensions in strings OA and OB? (Take g = 10 ms–2)
A B
m A m B m C 30° 60°

T1 90° T2
2m m 120° O 150°
F1 = 2 mg F2 = mg

(a) a1 = a2 = a3 (b) a1 > a3 > a2


(c) a1 = a2 , a2 = a3 (d) a1 = a2 , a1 = a3
w = 10N
24. If the surface is smooth, the acceleration of the block (a) 5 N, zero (b) Zero, N
m2 will be
(c) 5 N, 5 3 N (d) 5 3 N, 5 N
m
30. A disc of mass 10 g is kept floating horizontally in air
by firing bullets, each of mass 5 g, with the same
velocity at the same rate of 10 bullets per second. The
bullets rebound with the same speed in positive
direction. The velocity of each bullet at the time of
impact is
(a) 196 cms–1 (b) 98 cms–1
m (c) 49 cms–1 (d) 392 cms–1
m2 g 2 m2 g 31. A ball is travelling with uniform translatory motion.
(a) (b)
4 m1 + m2 4 m1 + m2 This means that [NCERT Exemplar]
2 m1g 2 m1g (a) it is at rest
(c) (d)
m1 + 4 m2 m1 + m2 (b) the path can be a straight line or circular and the ball
25. If a force of 250 N act on body the momentum travels with uniform speed
acquired is 125 kg-m/s. What is the period for which (c) all parts of the ball have the same velocity (magnitude
and direction) and the velocity is constant
force acts on the body?
(d) the centre of the ball moves with constant velocity and
(a) 0.5 s (b) 0.2 s (c) 0.4 s (d) 0.25 s
the ball spins about its centre uniformly
26. A point mass m is moving along inclined plane with 32. A satellite in force free space sweeps stationary
acceleration a with respect to smooth triangular
interplanetary dust at a rate dM / dt = av, where M is
block. The triangular block is moving horizontally
the mass, v is the velocity of the satellite and a is a
with acceleration a0 . The value of a is
constant. What is the deacceleration of the satellite?
(a) g sin q + a0 cos q (b) g sin q - a0 cos q
(a) -2 av2 / M (b) -av2 / M
(c) g cos q - a0 sin q (d) None of these
(c) + av2 / M (d) - av2
27. When a force F acts on a body F2
of mass m, the acceleration 33. A metre scale is moving with uniform velocity. This
produced in the body is a. implies [NCERT Exemplar]
If three equal forces (a) the force acting on the scale is zero, but a torque about
135° 90°
F1 = F2 = F3 = F act on the m
F1 the centre of mass can act on the scale
same body as shown in figure, F (b) the force acting on the scale is zero and the torque acting
3
the acceleration produced is about centre of mass of the scale is also zero
(c) the total force acting on it need not be zero but the
(a) ( 2 - 1) a (b) ( 2 + 1) a torque on it is zero
(c) 2 a (d) a (d) neither the force nor the torque need to be zero
Telegram @unacademyplusdiscounts

200 JEE Main Physics

34. A bird is sitting in a large closed cage which is placed 41. In a rocket of mass 1000 kg fuel is consumed at a rate
on a spring balance. It records a weight of 25 N. The of 40 kg/s. The velocity of the gases ejected from the
bird (mass m = 0.5 kg) flies upward in the cage with rocket is 5 ´ 104 m/s. The thrust on the rocket is
an acceleration of 2 m/s2 . The spring balance will now (a) 2 ´ 103 N (b) 5 ´ 10 4 N
record a weight of (c) 2 ´ 106 N (d) 2 ´ 10 9 N
(a) 24 N (b) 25 N (c) 26 N (d) 27 N
42. A block is kept on a frictionless
35. A cricket ball of mass 150 g has an initially inclined surface with angle of
velocity u = (3 $i + 4 $j) ms -1 and a final velocity inclination (a ). The incline is a
v = ( -3 $i + 4 $j) ms -1. After behind hit. The change in given an acceleration a to keep α
momentum (final momentum – initial momentum) is the block stationary. Then a is
(in kg ms -1) [NCERT] equal to
(a) zero (b) - ( 0.45 $i + 0.6 $j ) (a) g (b) g tan a
(c) - ( 0.9 $i + 1.2 $j ) (d) -5 ( $i + $j ) (c) g / tan a (d) g cosec a

36. In the above question the magnitude of the 43. A horizontal force F is applied
on a block of mass m placed on
momentum transferred during the hit is
[NCERT]
a rough inclined plane of F
(a) zero (b) 0.75 kg ms -1
inclination q. The normal
reaction N is
(c) 1.5 kg ms -1 (d) 14 kg ms -1
θ
37. A frictionless inclined plane of length l having (a) mg cos q
inclination q is placed inside a lift which is (b) mg sin q
accelerating downward with an acceleration a ( < g). (c) mg cos q - F cos q
If a block is allowed to move, down the inclined plane, (d) mg cos q + F sin q
from rest, then the time taken by the block to slide 44. For the system shown in figure, the pulleys are light
from top of the inclined plane to the bottom of the and frictionless. The tension in the string will be
inclined plane is
2l 2l
(a) (b)
g g-a
2l 2l
(c) (d) m
g+a ( g - a ) sin q
m
38. A rocket with a lift-off mass 105 kg is blasted upward
with an initial acceleration of 5 ms–2. If g = 10 ms–2, 2 3
then the initial thrust of the blast is (a) mg sin q (b) mg sin q
3 2
(a) 1.5 × 102 N (b) 1.5 × 103 N 1
(c) 1.5 × 105 N (d) 1.5 × 106 N (c) mg sin q (d) 2 mg sin q .
2
39. A block of mass 3 kg rests 10 N
y
45. An object is kept on a smooth inclined plane of 1 in l.
on a horizontal frictionless The horizontal acceleration to be imparted to the
xy-plane. What would be O x
inclined plane so that the object is stationary relative
the acceleration of the 60°
5N
to the inclined is
block if it is subjected to g g
two forces as shown in (a) g l 2 - 1 (b) g ( l 2 - 1) (c) (d)
2
l -1 l2 - 1
figure?
46. The monkey B shown in figure is holding
(a) 2.5 ms–2 (b) 5 ms–2 along y-axis
on to the tail of the monkey A which is
(c) 10 ms–2 along x-axis (d) 15 ms–2 along y-axis
climbing up a rope. The masses of the
40. Two masses of 3 kg and 5 kg are suspended from the monkeys A and B are 5 kg and 2 kg
A
ends of an unstreatchable massless cord passing over respectively. If A can tolerate a tension of
a frictionless pulley. When the masses are released, 30 N in its tail, what force should it apply
the pressure on the pulley is on the rope in order to carry the monkey B B
(a) 2 kgf (b) 7.5 kgf with it? (Take g = 10 ms–2)
(c) 8 kgf (d) 15 kgf
(a) 105 N (b) 108 N (c) 10.5 N (d) 100 N
Telegram @unacademyplusdiscounts

Laws of Motion and Friction 201

47. The acceleration of the 500 g block in figure is 53. A block is gently placed on a conveyor belt moving
horizontally with constant speed. After 4 s the velocity
100 g of the block becomes equal to the velocity of belt. If the
coefficient of friction between the block and the belt is
0.2, then velocity of the conveyor belt is
500 g (a) 2 ms -1 (b) 4 ms -1
30° (c) 6 ms -1 (d) 8 ms -1
54. A cricket ball of mass 150 g collides straight with a
50 g
bat with a velocity of 10 ms–1. Batsman hits it
6g 7g straight back with a velocity of 20 ms–1. If ball
(a) downwards (b) downwards remains in contact with bat for 0.1s, then average
13 13
8g 9g force exerted by the bat on the ball is
(c) downwards (d) upwards
13 13 (a) 15 N (b) 45 N
(c) 150 N (d) 4.5N
48. An elevator and its load have a total mass of 800 kg.
The elevator is originally moving downwards at 55. A block of mass 1 kg is at rest on a horizontal table.
10 ms–1, it slows down to stop with constant The coefficient of static friction between the
acceleration in a distance of 25 m. Find the tension T block and the table is 0.5. If g = 10 ms -2 , then the
in the supporting cable while the elevator is being magnitude of the force acting upwards at an angle of
brought to rest. (Take g = 10 ms–2) 60° from the horizontal that will just start the block
(a) 8000 N (b) 1600 N (c) 9600 N (d) 6400 N moving is
49. A body with mass 5 kg is acted upon by a force (a) 5 N (b) 5.36 N
F = ( - 3 $i + 4 $j) N. If its initial velocity at t = 0 is (c) 74.6 N (d) 10 N
v = (6i$ - 12$j) ms -1, the time at which it will just have 56. 100 g of an iron ball having velocity 10 ms–1 collides
a velocity along the y-axis is [NCERT Exemplar] with wall at an angle 30° and rebounds with the same
(a) never (b) 10 s (c) 2 s (d) 15 s angle. If the period of contact between the ball and
50. A 1000 kg lift is supported by a cable that can support wall is 0.1s, then the average force experienced by
2000 kg. The shortest distance in which the lift can the wall is
be stopped when it is descending with a speed of (a) 10 N (b) 100 N
2.5 ms–1 is [Take g = 10 ms–2] (c) 1.0 N (d) 0.1 N
5 5 57. A heavy uniform chain lies on a horizontal table top.
(a) 1 m (b) 2 m (c) m (d) m
32 16 If the coefficient of friction between the chain and the
51. A block of weight 5 N is pushed against a vertical table surface is 0.25 then the maximum fraction of
wall by a force 12 N. The coefficient of friction length of the chain, that can hang over one edge of
between the wall and block is 0.6. The magnitude of the table is
the force exerted by the wall on the block is (a) 20% (b) 25%
(c) 35% (d) 15%
58. A block A with mass 100 kg is resting on another
12 N block B of mass 200 kg. As shown in figure, a
horizontal rope tied to a wall holds it. The coefficient
of friction between A and B is 0.2 while coefficient of
(a) 12 N (b) 5 N (c) 7.2 N (d) 13 N friction between B and the ground is 0.3. The
minimum required force F to start moving B will be
52. A blumb bob is hung from the ceiling of a train
compartment. The train moves on an inclined track A
of inclination 30° with horizontal. Acceleration of
train up the plane is a = 9/2. The angle which the
string supporting the bob makes with normal to the B F
ceiling in equilibrium is
æ 2 ö æ 3ö
(a) 30° (b) tan -1 ç ÷ (c) tan -1 ç ÷ (d) tan -1 (2) (a) 900 N (b) 100 N
è 3ø è 2 ø
(c) 1100 N (d) 1200 N
Telegram @unacademyplusdiscounts

202 JEE Main Physics

59. A maximum speed that can be achieved without 66. A smooth inclined plane of length L having
skidding by a car on a circular unbanked road of inclination q with the horizontal is inside a lift which
radius R and coefficient of static friction m is is moving down with retardation a. The time taken
(a) m Rg (b) Rg m by a body to slide down the inclined plane, from rest,
will be
(c) m Rg (d) m Rg
2L 2L
(a) (b)
60. A chain lies on a rough horizontal table. It starts ( g + a ) sin q ( g - a ) sin q
sliding when one-fourth of its length hangs over the 2L 2L
edge of the table. The coefficient of static friction (c) (d)
g sin q a sin q
between the chain and the surface of the table is
1 1 1 1 67. A wooden box of mass 8 kg slides down an inclined
(a) (b) (c) (d)
2 3 4 5 plane of inclination 30° to the horizontal with a
constant acceleration of 0.4 ms–2. What is the force of
61. A fireman of mass 60 kg slides down a pole He is
friction between the box and inclined plane?
pressing the pole with a force of 600 N. The
(g = 10 ms–2)
coefficient of friction between the hands and the pole
(a) 36.8 N (b) 76.8 N (c) 65.6 N (d) 97.8 N
is 0.5 with what acceleration with the fireman slide
down? ( g = 10 m/s2 ) 68. The coefficient of kinetic friction between a 20 kg box
(a) 1 m/s 2
(b) 2.5 m/s2 and the floor is 0.40. How much work does a pulling
(c) 10 m/s2 (d) 5 m/s2
force do on the box in pulling it 8.0 m across the floor
at constant speed? The pulling force is directed 37°
62. The minimum velocity (in ms–1) with which a car above the horizontal
driver must traverse a flat curve of radius 150 m and (a) 343 J (b) 482 J
coefficient of friction 0.6 to avoid skidding is (c) 14.4 J (d) None of these
(a) 60 ms–1 (b) 30 ms–1 (c) 15 ms–1 (d) 25 ms–1
69. A car starts from rest to cover a distance s. The
63. A block of mass 3 kg resting on a horizontal surface. coefficient of friction between the road and the tyres
A force F is applied on the block as shown in figure. If is m. The minimum time in which the car can cover
1
coefficient of friction between the block be what the distance is proportional to
2 3 (a) m (b) m
can be the maximum value of force F so that block
(c) 1/m (d) 1/ m
does not start moving? (Take g = 10 ms–2)
70. A block of mass m lying on a rough horizontal plane is
F acted upon by a horizontal force P and another force
Q inclined at an angle q to the vertical. The block will
60° remain in equilibrium if the coefficient of friction
√3 kg
between it and the surface is

(a) 20 N (b) 10 N (c) 12 N (d) 15 N Q


θ
64. A heavy uniform chain lies on horizontal table top. If
the coefficient of friction between the chain and the
P
table surface is 0.25, then the maximum fraction of
the length of the chain that can hang over one edge of P + Q sin q P cos q + Q
(a) (b)
the table is mg + Q cos q mg - Q sin q
(a) 20% (b) 25% (c) 35% (d) 15% P + Q cos q P sin q - Q
(c) (d)
65. A block moves down a smooth inclined plane of mg + Q sin q mg - Q cos q
inclination q. Its velocity on reaching the bottom is v.
71. A partly hanging uniform chain of length L is resting
If it slides down a rough inclined plane of same
on a rough horizontal table. l is the maximum
inclination, its velocity on reaching the bottom is v/n,
possible length that can hang in equilibrium. The
where n is a number greater than 1. The coefficient
coefficient of friction between the chain and table is
of friction is given by
l L
æ 1ö æ 1ö (a) (b)
(a) m = tan q ç1 - 2 ÷ (b) m = cot q ç1 - 2 ÷ L-l l
è n ø è n ø l lL
1 /2 1 /2 (c) (d)
æ 1ö æ 1ö L L+ l
(c) m = tan q ç1 - 2 ÷ (d) m = cot q ç1 - 2 ÷
è n ø è n ø
Telegram @unacademyplusdiscounts

Laws of Motion and Friction 203

72. A box of mass m kg is placed on the rear side of an T


A B
open truck accelerating at 4 ms -2 . The coefficient of
friction between the box and the surface below it is
0.4. The net acceleration of the box with respect to
the truck is zero. The value of m is
[Given g = 10 ms -2 ] C

(a) 4 kg (b) 8 kg L
(c) 9.78 kg (d) It could be any value
mm mm
(a) g (b) g
73. A 40 kg slab rests on a frictionless floor. A 10 kg block M (M + m)
rests on top of the slab. The static coefficient of 2mm 2mm
friction between the block and the slab is 0.60 while (c) g (d) g
M (M + m)
the kinetic coefficient of friction is 0.40. The 10 kg
block is acted upon by a horizontal force of 100 N. 78. A man weighing 60 kg is standing on a trolley
If g = 9.8 ms–2, the resulting acceleration of the slab weighing 240 kg. The trolley is resting on frictionless
will be horizontal rails. If the man starts walking on the
10 kg
trolley with a velocity of 1 ms -1, then after 4 s, his
100 N
No friction displacement relative to the ground is
40 kg (a) 6 m (b) 4.8 m (c) 3.2 m (d) 2.4 m

(a) 1.47 ms–2 (b) 1.69 ms–2 79. A block of mass 5 kg, resting on a horizontal surface,
(c) 9.8 ms–2 (d) 0.98 ms–2 is connected by a cord, passing over a light
frictionless pulley to a hanging block of mass 5 kg.
74. A body of mass 40 kg resting on a rough horizontal The coefficient of kinetic friction between the block
surface is subjected to a force P which is just enough
and the surface is 0.5. Tension in the cord is (Take g =
to start the motion of the body. If m s = 0.5, m k = 0.4,
9.8 ms–2)
g = 10 ms -2 and the force P is continuously applied on
(a) 49 N (b) 36 N
the body, then the acceleration of the body is
(c) 36.75 N (d) 2.45 N
(a) zero (b) 1 ms–2
(c) 2 ms–2 (d) 2.4 ms–2 80. Three blocks are placed at rest on a smooth inclined
plane with force acting on m1 parallel to the inclined
plane. Find the contact force between m2 and m3.
75. The coefficient of friction between a body and the
surface of an inclined plane at 45° is 0.5 if m3
m2
g = 9.8 m /s2. The acceleration of the body downwards m1
in m/s2 is
4.9 F θ
(a) (b) 4.9 2 (c) 19.2 2 (d) 4.9
2
( m1 + m2 + m3 ) F m3F
76. A rope of length L and mass M is hanging from a right (a) (b)
3 123 m m +m +m
from support. The tension in the rope at a distance
(c) F - ( m1 + m2 ) g (d) None of these
the rigid support is
æ L - xö
(a) Mg (b) ç ÷ Mg
è L ø Connected Body Motion
æ L ö
(c) ç (d)
x 81. Three equal weight A, B and C of mass
÷ Mg Mg
è L - xø L 2 kg each are hanging on a string
passing over a fixed frictionless pulley
77. A plate of mass M is placed on a horizontal
as shown in the figure. The tension in
frictionless surface (see figure) and a body of mass m
the string connecting weight B and C is
is placed on this plate. The coefficient of dynamics
(a) zero
friction between this body and the plate is m. If a force A
(b) 13 N
2 mmg is applied to the body of mass m along the B
(c) 3.3 N
horizontal, the acceleration of the plate will be
(d) 19.6 N
C
Telegram @unacademyplusdiscounts

204 JEE Main Physics

82. Two blocks are connected by a string as F 87. In the figure, the ball A is released
shown in the diagram. The upper block is T from rest when the spring is at its
hung by another string. A force applied on natural length. For the block B of
the upper string produces an acceleration of 2 kg mass M to leave contact with the
2 m/s2 in the upward direction in both the ground at same stage, the minimum
T
blocks. If T and T ¢ be the tensions in the two mass of A must be
parts of the string, then ( g = 9.8 m /s2 ) (a) 2 M m
(a) T = 70.8 N and T ¢ = 47.2 N 4 kg (b) M
(b) T = 58.8 N and T ¢ = 47.2 N (c)
M
(c) T = 70.8 N and T ¢ = 58.8 N 2 M
(d) T = 70.8 N and T ¢ = 0 (d) a function of M and the force
constant of the spring
83. A block is dragged on a v
F
smooth horizontal plane with 88. A shell is fired from a cannon with velocity v ms–1 at
the help of a light rope which an angle q with the horizontal direction. At the
moves with a velocity v highest point in its path it explodes into two pieces of
as shown in figure. The equal mass. One of the pieces retraces its path to the
horizontal velocity of the block θ
cannon and the speed in m/s of the piece immediately
is after the explosion is
(a) 3 v cos q
(a) v (b) v sin q m (b) 2 v cos q
v v
(c) (d) 3v
sin q cos q (c) cos q
2
84. A mass of 3 kg descending vertically downward 3 v cos q
(d)
supports a mass of 2 kg by means the end of 5 s, the 2
string breaks. How much higher the 2 kg mass will go
89. A body of weight 2 kg is suspended as shown in
further?
figure. The tension T1 in the horizontal string (in
kg-wt) is
30°

m
m T
2m

(a) 4.9 m 2 kg-wt


(b) 9.8 m
(a) 2 / 3 (b) 3/ 2
(c) 19.6 m
(d) 2.45 m (c) 2 3 (d) 2

85. Two bodies of masses m1 and m2 are connected by a 90. Two blocks of masses m and 2 m are connected by a
light, inextensible string which passes over a light string passing over a frictionless pulley. As
frictionless pulley. If the pulley is moving upward shown in the figure, the mass m is placed on a smooth
with uniform acceleration g, then the tension in the inclined plane of inclination 30° and 2 m hangs
string is vertically. If the system is released, the blocks move
4 m1m2 m1m2 with an acceleration equal to
(a) g (b) g
m1 + m2 4 m1m2
m1m2 m - m2 2
(c) g (d) 1 g m
m1 + m2 m1 + m2

86. In the given arrangement, n number of equal masses 2m


are connected by strings of negligible masses. The 30°
tension in the string connected to nth mass is
g g
mMg mMg (a) (b)
(a) (b) 4 3
nm + M nmM
g
(c) mg (d) mng (c) (d) g
2
Telegram @unacademyplusdiscounts

Laws of Motion and Friction 205

91. Refer to the system shown in figure. The ratio of


tensions T1 and T2 is
T1 T1 T2
m1 m2
T2
m1 m2 3 kg
(a) (b) B T2
m1 + m2 m1 + m2
T1
m m
(c) 1 (d) 2 T1
m2 m1 A 5 kg
1 kg C
92. In the figure a smooth pulley of negligible g g g g
weight is suspended by a spring balance. (a) (b) (c) (d)
3 6 9 12
Weights of 1 kg and 5 kg are attached to
the opposite ends of a string passing over 95. Two block of masses 7 kg and 5 kg are placed in
the pulley and move with acceleration contact with each other on a smooth surface. If a force
because of gravity. During their motion, of 6 N is applied on a heavier mass the force on the
the spring balance reads a weight of lighter mass is
7 kg
(a) 6 kg 1 kg
5 kg
(b) less than 6 kg 6N
5 kg
(c) more than 6 kg
(d) may be more or less than 6 kg
(a) 3.5 N (b) 2.5 N (c) 7 N (d) 5 N
93. Three blocks of masses m1, m2 and m3 are connected
by massless strings as shown on a frictionless table in
96. A trolley T (mass 5 kg) on a horizontal smooth
surface is pulled by a load L (2 kg) through a uniform
figure. They are pulled with a force T3 = 40 N. If
rope ABC of length 2 m and mass 1kg. As the
m1 = 10 kg, m2 = 6 kg and m3 = 4 kg, the tension T2
load falls from BC = 0 to BC = 2 m, its acceleration
will be
T1 T2 T3 (in ms–2) changes from (Take g = 10 ms–2)
m1 m2 m3
A B
T

(a) 20 N
(b) 40 N
(c) 10 N C
(d) 32 N L
94. Refer to the system shown in figure. The acceleration
(a) 20/6 to 20/5 (b) 20/8 to 30/8
of the masses is
(c) 20/5 to 30/6 (d) None of these

Round II (Mixed Bag)

Only One Correct Option 2. Two masses 8 kg and 12 kg are connected at the two
1. A vessel containing water is given a constant ends of a light inextensible string that goes over a
acceleration a towards the right along a straight frictionless pulley. The acceleration of the masses
horizontal path. Which of the following represent the and the tension in the string when the masses are
surface of the liquid? released, are respectively [NCERT]
(a) 2 m/s2 and 90 N
a a a (b) 4 m/s2 and 90 N
(c) 2 m/s2 and 60 N
(A) (B) (C) (D)
(d) 4 m/s2 and 99 N
(a) A (b) B (c) C (d) D
Telegram @unacademyplusdiscounts

206 JEE Main Physics

3. Two elastic blocks P and Q of equal masses m and 8. A circular disc with a groove along its diameter is
connected by a massless spring rest on a smooth placed horizontally. A block of mass 1 kg is placed as
horizontal surface, as in figure. A third block R of the shown. The coefficient of friction between the block
same mass M strikes the block P. After the collision, 2
and all surfaces of groove in contact is m = , the disc
P and Q will 5
has an acceleration of 25 m/s2 . Find the acceleration
R P Q of block with respect to disc

(a) always move in same direction


(b) sometimes move in same direction and sometime move a = 25 m/s2
in opposite directions
θ 4 3
(c) always move in opposite directions cos θ = — , sin θ = —
5 5
(d) be at rest with respect to each other
(a) 10 m/s2 (b) 5 m/s2
4. Two persons are holding a rope of negligible weight
(c) 20 m/s2 (d) 1 m/s2
tightly at its ends so that it is horizontal. A 15 kg
weight is attached to rope at the mid-point which 9. A cylinder roll up an inclined plane, reaches some
now no more remains horizontal. The minimum height and then rolls down (without slipping
tension required to completely straighten the rope is throughout these motions). The directions of
(a) 15 kg (b) 15/2 kg frictional force acting on the cylinder are
(c) 5 kg (d) infinitely large (a) up the inclined while ascending and down the incline
while descending
5. Two bodies of mass 4 kg and 6 kg
(b) up the incline while ascending as well as descending
are attached to the ends of a string
(c) down the incline while ascending and up the incline
passing over a pulley. The 4 kg while descending
mass is attached to the table by (d) down the incline while ascending as well as descending
another string. The tension in this T
string T1 is 10. When forces F1, F2 , F3 are acting on a particle of mass
(a) 19.6 N m T m such that F2 and F3 are mutually perpendicular,
(b) 25 N 4 kg then the particle remains stationary. If the force F1 is
(c) 10.6 N T1 6 kg
now removed, then the acceleration of the particle is
(d) 10 N (a) F1/ m (b) FF
2 3/ mF1 (c) ( F2 - F3 ) / m (d) F2 / m
11. A particle moves in a circular path with decreasing
6. A block of mass 2 kg is placed on the floor. The speed. Choose the correct statement.
coefficient of static friction is 0.4. If a force of 2.8 N is (a) Angular momentum remains constant
applied on the block parallel to floor, the force of (b) Acceleration a is towards the centre
friction between the block and floor (c) Particle moves in a spiral path with decreasing radius
(Taking g = 10 ms–2) is (d) The direction of angular momentum remains constant
(a) 2.8 N (b) 8 N (c) 2 N (d) zero
12. A spring balance, A reads 2
7. The pulley and strings shown in figure are smooth kg with a block m
and of negligible mass. For the system to remain in suspended from it. A
A
equilibrium, the angle q should be balance B reads 5 kg when
a beaker filled with liquid
is put on the pan of the
balance. The two balances
θ
are now so arranged that B
the hanging mass is inside
√2 m the liquid as shown in figure. In this situation
(a) the balance A will read more than 2 kg
m m
(b) the balance B will read more than 5 kg
(c) the balance A will read less than 2 kg and B will read more
(a) 0º (b) 30º
than 5 kg
(c) 45º (d) 60º
(d) the balance A and B will read 2 kg and 5 kg
Telegram @unacademyplusdiscounts

Laws of Motion and Friction 207

13. While waiting in a car at a stoplight and 80 kg man 19. A body of mass M is kept on a rough horizontal
and his car are suddenly accelerated to a speed of surface (friction coefficient m). A person is trying to
5 ms–1 as a result or rear end collision. If the time of pull the body by applying a horizontal force but the
impact is 0.4 s, find the average force on the man body is not moving. The force by the surface on the
(a) 100 N (b) 200 N (c) 500 N (d) 1000 N body is F, where
14. A body of mass m is suspended by two strings making (a) F = Mg (b) F = mMgF
angle a and b with the horizontal as shown in figure. (c) Mg £ f £ Mg 1 + m2 (d) Mg ³ f ³ Mg 1 + m2
Tensions in the two strings are 20. A 5 kg stationary bomb is exploded in three parts
having mass 1 : 1 : 3 respectively. Parts having same
T2 T1 mass move in perpendicular directions with velocity
39 ms–1, then the velocity of bigger part will be
10
β α (a) 10 2 ms -1 (b) ms -1
2
15
(c) 13 2 ms -1 (d) ms -1
2
mg
21. The upper half of an inclined plane with inclination f
mg cos b is perfectly smooth, while the lower half is rough. A
(a) T1 = = T2
sin ( a + b ) body starting from rest at the top will again come to
mg sin b rest at the bottom if coefficient of friction for the
(b) T1 = = T2
sin ( a + b ) lower half is given by
mg cos b mg cos a (a) 2 sin f (b) 2cos f
(c) T1 = ; T2 =
sin ( a + b ) sin ( a + b ) (c) 2 tan f (d) tan f
(d) None of the above 22. An insect crawls up a hemispherical surface very
15. A block of mass 15 kg is resting on a T slowly, figure. The coefficient of friction between the
rough inclined plane as shown in insect and the surface is 1/3. If the line joining the
M centre of the hemispherical surface to the insect
figure. The block is tied by a
horizontal string which has a makes an angle a with the vertical, the maximum
tension of 50 N. The coefficient of possible value of a is given by
friction between the surfaces of 45°
contact is (g = 10 ms–2) α

(a) 1/2 (b) 3/4 (c) 2/3 (d) 1/4


16. A stone of mass 0.25 kg tied to the end of a string is
whirled round in a circle of radius 1.5 m with speed (a) cot a = 3 (b) sec a = 3
40 rev/min in a horizontal plane. What is the tension (c) cosec a = 3 (d) None of these
in the string? What is the maximum speed with 23. A mass of 6 kg is suspended by a rope of length 2 m
which the stone can be whirled around if the string from a ceiling. A force of 50 N is applied in the
can withstand a maximum tension of 200 N? [NCERT]
horizontal direction at the mid-point of the rope. The
(a) 200 N (b) 300 N (c) 2000 N (d) 250 N angle made by the rope, with the vertical, in
17. The mass of a body measured by a physical balance in equilibrium position will be (take g = 10 ms -2 , neglect
a lift at rest is found to be m. If the lift is going up the mass of the rope)
with an acceleration a, its mass will be measured as (a) 90º (b) 60º
æ aö æ aö (c) 50º (d) 40º
(a) m ç1 - ÷ (b) m ç1 + ÷
è gø è gø 24. A rope of mass 0.1 kg is connected at the same height
(c) m (d) zero of two opposite walls. It is allowed to hang under its
18. A given object takes n times more time to slide down own weight. At the constant point between the rope
a 45° rough inclined plane as it takes to slide down a and the wall, the rope makes an angle q = 10° with
perfectly smooth 45° incline. The coefficient of kinetic respect to horizontal. The tension in the rope at its
friction between the object and the incline is mid-point between the wall is
(a) 2.78 N (b) 2.56 N
1 1 1 1 (c) 2.82 N (d) 2.71 N
(a) (b) 1 - (c) 1 - (d)
1 - n2 n2 n2 1 - n2
Telegram @unacademyplusdiscounts

208 JEE Main Physics

25. A 24 kg block resting on a floor has a rope tied to its 32. The motion of a particle of mass m is given by x = 0 for
top. The maximum tension, the rope can withstand t < 0 s, x( t) = A sin 4 pt for 0 < t < (1/ 4) s ( A > 0), and
without breaking is 310 N. The minimum time in x = 0 for t > (1 / 4) s. Which of the following
which the block can be lifted a vertical distance of statements is true ? [NCERT Exemplar)
4.6 m by pulling on the rope is (a) The force at t = (1 / 8) s on the particle is - 16 p2 Am
(a) 1.2 s (b) 1.3 s (b) The particle is acted upon by on impulse of magnitude
(c) 1.7 s (d) 2.3 s 4 p2 A m at t = 0 s and t = (1 / 4 ) s
26. Two small balls of same size and masses m1 and m2 (c) The particle is not acted upon by any force
(m1 > m2 ) are tied by a thin weightless thread and (d) There is no impulse acting on the particle
dropped from a certain height. Traing upward
33. Mass m1 moves on a slope making an angle q with the
bouyancy force F into account, the tension T of the
horizontal and is attached to mass m2 by a string
thread during the flight after the motion of the ball
passing over a frictionless pulley as shown in figure.
becomes uniform will be
The coefficient of friction between m1 and the sloping
(a) ( m1 - m2 ) g (b) ( m1 - m2 ) g / 2
surface is m.
(c) ( m1 + m2 ) g (d) ( m1 + m2 ) g / 2
27. A solid disc of mass M is just held in air horizontal by
throwing 40 stones per sec vertically upwards to
m1
strike the disc each with a velocity 6 ms–1. If the mass
m2
of each stone is 0.05 kg. What is the mass of the disc?
B
(g = 10 ms–2) θ
(a) 1.2 kg (b) 0.5 kg (c) 20 kg (d) 3 kg
28. If coefficient of friction between an insect and bowl is Which of the following statements are true ?
m and radius of the bowl is r, the maximum height to [NCERT Exemplar]
which the insect can crawl in the bowl is (a) If m2 > m1 sin q, the body will move up the plane
é 1 ù r (b) If m2 > m1 (sin q + m cos q), the body will move up the
(a) r ê1 - ú (b) plane
êë 1 + m2 úû 1 + m2
(c) If m2 < m1 (sin q + m cos q) , the body will move up the
(c) r 1 + m2 (d) r [ 1 + m2 - 1] plane
(d) If m2 < m1 (sin q - m cos q), the body will move down
29. A block of mass m is pushed with a velocity u towards the plane
a movable wedge of mass nm and height h, figure. All
the surfaces are smooth. The minimum value of u for 34. A man of mass M is standing on a board of mass m. The
which the block will reach the top of wedge is friction coefficient between the board and the floor is m,
1ö 1ö shown in figure. The maximum force that the man can
æ æ
(a) 2 gh ç1 - ÷ (b) 2 gh ç1 + ÷ exert on the rope so that the board does not move is
è nø è nø
(c) 3 gh (d) 2 gh
T T
F
More Than One Correct Option
30. 80 railway wagons all of same mass 5 × 103 kg are T
pulled by an engine with a force of 4 × 105 N. The R
tension in the coupling between 30th and 31st wagon
T
from the engine is
f
(a) 25 × 104 N (b) 40 × 104 N
(c) 20 × 104 N (d) 32 × 104 N (m + M)g

31. A gardner waters the plants by a pipe of diameter (a) m ( m + M ) g


1 mm. The water comes out at the rate of 10 cm3 s–1. m (m + M ) g
(b)
The reactionary force exerted on the hand of the m +1
gardner is m (m + M ) g
(c)
(a) zero (b) 1.27 ´ 10 –2 N m -1
–4 (d) None of the above
(c) 1.27 ´ 10 N (d) 0.127 N
Telegram @unacademyplusdiscounts

Laws of Motion and Friction 209

Comprehension Based Questions Assertion and Reason


Passage I Directions Question No. 43 to 51 are Assertion-Reason type.
Each of these contains two Statements: Statement I (Assertion),
A body of mass 10 kg is lying on a rough horizontal
Statement II (Reason). Each of these questions also has four
surface. The coefficient of friction between the body
alternative choice, only one of which is correct. You have to
and horizontal surface is 0.577. When the horizontal select the correct choices from the codes (a), (b), (c) and (d) given
surface is inclined gradually, the body just begins to below
slide at a certain angle a. This is called angle of (a) If both Assertion and Reason are true and the Reason
repose. When angle of inclination is increased is correct explanation of the Assertion
further, the body slides down with some acceleration. (b) If both Assertion and Reason are true but Reason is
35. The minimum force required just to slide the block on not correct explanation of the Assertion
the horizontal surface is (c) If Assertion is true but Reason is false
(d) If Assertion is false but the Reason is true
(a) 57.7 N (b) 100 N (c) 100 kg (d) 57.7 kg
43. Assertion An electric fan continues to rotate for some
36. The minimum force required just to move the body up
time after the current is switched off.
the incline is
Reason It is because of inertia of rest.
(a) 100 N (b) 57.7 N (c) 111.5 N (d) 157.7 N
44. Assertion Force is required to move a body uniformly
37. Value of angle of repose in this case is
along a circle.
(a) 60º (b) 57.7º (c) 5.77º (d) 30º
Reason When the motion is uniform, acceleration is
Passage II zero.
A force that acts on a body for a very short time is 45. Assertion Angle of repose is equal to angle of limiting
called impulsive force. Impulse measures the effect of friction.
the force. It is the product of force and time for which Reason When a body is just at the point of motion, the
the force acts. Impulse is measured by the change in force of friction in this stage is called as limiting
momentum of the body. For a given change in friction.
momentum, Fav ´ t = constant. By increasing the tie 46. Assertion A string can never remain horizontal, when
(t) of impact, we can reduce the average force fav . loaded at the middle, howsoever large the tension
Read the above passage carefully and answer the may be.
following questions (g = 10 ms -2).
Reason For horizontal spring, angle with vertical,
38. If the impact lasts for 0.1s, force exerted by the w w
q = 90° Þ T = = =¥
impinging ball on the ground is 2 cos q 2 cos 90°
(a) 45.2 N (b) 45.2 kg-wt 47. Assertion Use of ball bearing between two moving
(c) 42.5 N (d) 42.5 kg-wt parts of machine is a common practice.
39. The force exerted by the ground on the ball during Reason Ball bearings reduce vibration and provide
impact is good stability.
(a) 90.4 N (b) 42.5 N (c) 84.5 N (d) 45.2 N 48. Assertion The maximum speed with which a vehicle
40. The loss of energy during impact is can go round a level curve of diameter 20 m without
Two small balls of same size and and masses and skidding is 10 ms–1, given m = 0.1.
(a) 75 J (b) 5 J (c) 57 J (d) 6.38 J Reason It follows from v £ m rg .
41. A cricket player lowers his hands while catching a 49. Assertion A man is closed cabin falling freely does
ball, because not experience gravity.
(a) ball is heavy Reason Inertial and gravitational mass have
(b) ball is coming with high speed equivalence.
(c) it increases the time of impact and reduces the impact of 50. Assertion A cyclist always bends inwards while
force on his hands negotiating a curve.
(d) None of the above Reason By bending cyclist lower his centre of
42. A ball of mass 250 g falls from a height of 5 m above gravity.
the ground, and rebounds to a height of 2.45 m. The 51. Assertion Aeroplane always fly at low altitudes.
impulse on collision is Reason According to Newton’s third law of
(a) 4.25 kgms–1 (b) 45.2 kgms–1 motion for every action there is an equal and opposite
(c) 52.4 kgms–1 (d) 54.2 kgms–1 reaction.
Telegram @unacademyplusdiscounts

210 JEE Main Physics

Previous Years’ Questions


52. A force of (5 + 3 x) N acting on a body of mass 20 kg 58. A block of mass m is connected to another block of
along the x-axis displaces it from x = 2 m to x = 6 m. mass M by a massless spring of constant k. The
The work done by the force is [UP SEE 2009] blocks are kept on a smooth horizontal plane.
(a) 20 J (b) 48 J (c) 68 J (d) 86 J Initially, the blocks are at rest and the spring is
unstretched. Then a constant force F starts acting on
53. Block A of mass m and block B of mass 2m are placed the block of mass M to pull it. Find the force on the
on a fixed triangular wedge by means of a massless,
block of mass m. [AIEEE 2007]
inextensible string and a frictionless at 45° to the
MF mF
horizontal on both the side. If the coefficient of (a) (b)
m+M M
friction between the block A and the wedge is 2/3 and (M + m) F mF
that between the block B and the wedge is 1/3 and (c) (d)
m m+M
both blocks A and B are released from rest, the
acceleration of A will be [UP SEE 2008]
59. A player caught a cricket ball of mass 150 g moving at
a rate of 20 ms–1. If the catching process is completed
in 0.1s, the force of blow exerted by the ball on the
hands of the player is equal to [AIEEE 2006]
(a) 3 N (b) –30 N
m 2m (c) 300 N (d) 150 N
60. A ball of mass 0.2 kg is thrown vertically upwards by
45° 45° applying a force by hand. If the hand moves 0.2 m
while applying the force and the ball goes upto 2 m
(a) –1 m/s2 (b) 1.2 m/s2 (c) 0.2 m/s2 (d) 0 m/s2 height further, find the magnitude of the force
[take g = 10 ms–2] [AIEEE 2006]
54. A block of mass 5 kg is moving horizontally at a speed
of 1.5 ms–1. A vertically upward force 5 N acts on it (a) 16 N (b) 20 N
for 4 s. What will be the distance of the block from the (c) 22 N (d) 4 N
point where the force starts acting? [BVP Engg. 2008] 61. A bomb of mass 16 kg at rest explodes into two pieces
(a) 2 m (b) 6 m (c) 8 m (d) 10 m of masses 4 kg and 12 kg. The velocity of the 12 kg
mass is 4 ms–1. The kinetic energy of the other mass
55. A block B is pushed momentarily B v is [AIEEE 2006]
along a horizontal surface with an
initial velocity v. If m is the coefficient (a) 288 J (b) 192 J
of sliding friction between B and the surface, block B (c) 96 J (d) 144 J
will come to rest after a time [UP SEE 2007] 62. Two masses M and M/2 are joined together by means
v gm g v of light inextensible string passed over a frictionless
(a) (b) (c) (d)
gm v v g pulley as shown in figure. When the bigger mass is
released, the small one will ascend with an
56. An ice cart of mass 60 kg rests on a horizontal snow acceleration of [Kerala CET 2005]
patch with coefficient of static friction 1/3 Assuming
that there is no vertical acceleration, find the
magnitude of the maximum horizontal force required
to move the ice cart. (g = 9.8 ms–2) [BVP Engg. 2007]
(a) 100 N (b) 110 N
(c) 209 N (d) 196 N
57. A uniform metal chain is placed on a rough table such
that one end of it hangs down over the edge of the M/2
table. When one-third of its length hangs over the
edge, the chain starts sliding. Then, the coefficient of
M
static friction is [BVP Engg. 2007]
3 1 g 3g
(a) (b) (a) (b)
4 4 3 2
2 1 g
(c) (d) (c) g (d)
3 2 2
Telegram @unacademyplusdiscounts

Laws of Motion and Friction 211

63. A block of mass m is at rest under the action of force F 67. A man drags a block through 10 m on rough surface
against a wall as shown in figure. Which of the (m = 0.5). A force of 3 kN acting at 30° to the
following statements is incorrect? [IIT JEE 2005] horizontal. The work done by applied force is
[Orissa JEE 2011]
a
(a) zero (b) 15 kJ (c) 5 kJ (d) 10 kJ
a 68. A block at rest slides down a smooth inclined plane
F which makes an angle 60° with the vertical and it
reaches the ground in t1 seconds. Another block is
dropped vertically from the some point and reaches
the ground in t2 seconds, then the ratio of t1 : t2 is
[Kerala CET 2011]
(a) f = mg [where f is the friction force]
(a) 1 : 2 (b) 2 : 1 (c) 1 : 3 (d) 1 : 2
(b) F = N [where N is the normal force]
(c) F will not produce torque 69. A block is moving on an inclined plane making an
(d) N will not produce torque angle 45° with the horizontal and the coefficient of
friction is m. The force required to just push it up the
64. A block is kept on a frictionless inclined surface with
inclined plane is 3 times the force required to just
angle of inclination a as shown in figure. The incline
prevent it from sliding down. If we define N = 10 m,
is given an acceleration a to keep to block stationary.
then N is [IIT JEE 2011]
Then, n is equal to [AIEEE 2005]
mg/√2

F1 m

a µ mg/√2
α
45°
(a) g tan a (b) g (c) g cosec a (d) g / tan a
(a) 2 (b) 4 (c) 5 (d) 6
65. A particle of mass m is at rest at the origin at time
g = 0. It is subjected to a force F ( t) = f0 e- bt in the x 70. The acceleration of system of two bodies over the
direction. Its speed v( t) is depicted by which of the wedge as shown in figure. [Orissa JEE 2011]
following curves? [AIEEE 2012]

F0 F0
—– —– M
mb mb M
(a) (b)
37° 53°
v(t) v(t)
-2
t t
(a) 1 ms (b) 2 ms -2
(c) 0.5 ms -2 (d) 10 ms -2
F0 F0
—–
mb
—–
mb 71. A force vector applied on a mass is represented as
$ and acceleration with 1 m/s2 . What
F = 6 i$ - 8 $j + 10 k
(c) (d)
will be mass of the body?
v(t) v(t)
(a) 10 2 kg (b) 2 10 kg
t t
(c) 10 kg (d) 20 kg
66. A mass m hangs with the help of a string wrapped 72. The figure shows the position-time (x-t) graph of
around a pulley on a frictionless bearing. The pulley one-dimensional motion of a body of mass 0.4 kg. The
has mass m and radius R. Assuming pulley to be the magnitude of each impulse is [AIEEE 2010]
perfect uniform circular disc, the acceleration of the
mass m. If the string does not slip on the pulley, is
2
[AIEEE 2011]
2
(a) g (b) g x(m)
3 t
g 3 0 2 4 6 8 10 12 14 16
(c) (d) g
3 2 (a) 0.2 Ns (b) 0.4 Ns (c) 0.8 Ns (d) 1.6 Ns
Telegram @unacademyplusdiscounts

212 JEE Main Physics

73. Two fixed frictionless inclined planes making an 75. A light string passes over a frictionless pulley. To one
angle 30° and 60° with the vertical are shown in the of its ends a mass of 6 kg is attached. To its other end
figure. Two blocks A and B are placed on the two a mass of 10 kg is attached. The tension in the thread
planes. What is the relative vertical acceleration of A will be [AMU Engg. 2010]
with respect to B ? [AIEEE 2010]

6 kg
60°
30°
10 kg
(a) 4.9 m/s2 in vertical direction
(b) 4.9 m/s2 in horizontal direction (a) 24.5 N (b) 2.45 N (c) 79 N (d) 73.5 N
(c) 9.8 m/s2 in vertical direction
76. A body of mass m = 3.613 kg is moving along. The
(d) zero
x-axis with a speed of 5.00 ms -1. The magnitude of its
74. A block of mass 200 kg is being momentum is retarded as [AIEEE 2008]
pulled up by men on an inclined (a) 17.565 kg ms -1 (b) 17.56 kg ms -1
plane at angle of 45° as shown in 45° (c) 17.57 kg ms -1 (d) 17.6 kg ms -1
figure. The coefficient of static
friction is 0.5. Each man can only apply a maximum 77. Rocket pollution is associated with [J & K CET 2010]

force of 500 N. Calculate the number of men required (a) the conservation of angular momentum
for the block to just start moving up the plane. (b) the conservation of mass
[AMU Engg. 2010] (c) the conservation of mechanical energy
(a) 10 (b) 15 (c) 5 (d) 3 (d) Newton’s III law of motion

Answers
Round I
1. (c) 2. (c) 3. (a) 4. (d) 5. (a) 6. (b) 7. (a) 8. (b) 9. (b) 10. (b)
11. (a) 12. (b) 13. (a) 14. (c) 15. (d) 16. (c) 17. (a) 18. (a) 19. (c) 20. (a)
21. (a) 22. (b) 23. (b) 24. (a) 25. (a) 26. (b) 27. (a) 28. (a) 29. (c) 30. (b)
31. (c) 32. (c) 33. (b) 34. (c) 35. (c) 36. (c) 37. (d) 38. (d) 39. (b) 40. (b)
41. (c) 42. (b) 43. (d) 44. (c) 45. (c) 46. (a) 47. (c) 48. (c) 49. (b) 50. (d)
55. (b) 56. (a)
51. (d) 57. (a)
52. (b) 58. (c)
53. 59. (d) 60. (b)
61. (d) 62. (b) 63. (a) 64. (a) 65. (a) 66. (a) 67. (a) 68. (b) 69. (d) 70. (a)
71. (a) 72. (d) 73. (d) 74. (b) 75. (a) 76. (b) 77. (a) 78. (c) 79. (c) 80. (b)
81. (b) 82. (a) 83. (a) 84. (a) 85. (a) 86. (a) 87. (c) 88. (a) 89. (c) 90. (c)
91. (a) 92. (b) 93. (d) 94. (c) 95. (b) 96. (b)

Round II
1. (c) 2. (a) 3. (b) 4. (d) 5. (a) 6. (a) 7. (c) 8. (a) 9. (b) 10. (a)
11. (d) 12. (b) 13. (d) 14. (c) 15. (a) 16. (a) 17. (c) 18. (b) 19. (c) 20. (c)
21. (c) 22. (a) 23. (d) 24. (c) 25. (c) 26. (b) 27. (a) 28. (a) 29. (b) 30. (a)
31. (d) 32. (a,b,d) 33. (b,d) 34. (b) 35. (a) 36. (c) 37. (d) 38. (c) 39. (b) 40. (d)
41. (c) 42. (a) 43. (c) 44. (c) 45. (a) 46. (b) 47. (a) 48. (c) 49. (a) 50. (b)
51. (a) 52. (c) 53. (a) 54. (d) 55. (a) 56. (d) 57. (d) 58. (d) 59. (d) 60. (b)
61. (a) 62. (a) 63. (d) 64. (a) 65. (a) 66. (b) 67. (b) 68. (b) 69. (c) 70. (a)
71. (a) 72. (c) 73. (b) 74. (c) 75. (d) 76. (d) 77. (d)
Telegram @unacademyplusdiscounts

the Guidance
Round I
1. When force F is applied on 2m from left, contact force, 6. Initial mass of the rocket m = 20000 kg T
m F Initial acceleration a = 5.0 m/s2 in upwards a
F1 = F=
m + 2m 3 direction Rocket
When force F is applied on m from right, contact force Let initial thrust of the blast be T.
2m 2F \ T - mg = ma
F2 = F= \ F1 : F2 = 1 : 2
m + 2m 3 or T = mg + ma
2. Let the ball starts moving with velocity u and = m( g + a) w = mg
it reaches up to maximum height Hmax then = 20000 ´ (9.8 + 5.0)
from Hmax = 2 m
= 2 ´ 10 4 ´ 14.8 N
u2
Hmax = = 29.6 ´ 10 4 N = 2.96 ´ 10 5 N
2g
u = 2g (Hmax ) 7. The mass m is not moving with respect to the lift and also has
0.2 m no tendency to move. Hence, friction force acting on it is
= 2 ´ 10 ´ 2 equal to zero.
= 2 10 m/s 2 m1m2 2 ´ 4 ´ 2 ´ 10 160
8. T = g = = = 26.6 » 27 N
This velocity is supplied to the ball by the hand and initially m1 + m2 4+2 6
the hand was at rest it requires this velocity is distance of Total downward thrust on the pulley = 2 T = 2 ´ 27 = 54 N
0.2 m, it requires this velocity is distance of 0.2 m
2
u2 40 9. Tension in rope, T < Breaking load, mg
\ a= = = 100 m/s2 3
2 s 2 ´ 0.2 2
\ m ( g - a) < mg
So upward force on the ball F = m ( g + a) = 0.2 (10 +100) 3
= 0.2 ´ 110 = 22 N g
or a>
3
3. Since, P = (M + m) a
10. From figure, 8 x = wy
Now as in free body diagram of block,
ma x w
co = …(i)
sα N y 8
x y
α
ma mg
sin
mg cos α α α
+
ma sin α mg α 8g w

ma cos a = mg sin a wx = 18 y
sin a x 18
\ a=g = g tan a = …(ii)
cos a y w
or P = (M + m) g tan a x y

4. Conservation of momentum in a collision between particles


can be understood from both, Newton’s 2nd law and 3rd law.
5. Here, mass of the body w 8g
fair
M = 0.05 kg Dividing Eq. (i) by Eq. (ii), we have
–2
Acceleration g = 9.8 ms , a = 9.5 ms –2 x w
y
\ mg - fair = ma = 8
a x 18
Þ fair = m ( g - a) y w
= 0.05 (9.8 – 9.5) = 0.015 N Þ w = 18 ´ 8 = 12 g
mg
Telegram @unacademyplusdiscounts

214 JEE Main Physics

11. Mass of the body m = 5 kg 15. Applying law of conservation of momentum,


Force acting on body F1 = 8 N m æ mö
F v = çm + ÷V
Force perpendicular to force F1 on F2 = 6 N 20 è 20 ø
the body v 20 v
α or V= ´ =
F2 = 6 N 20 21 21
F1 = 8 N
Angle between two forces q = 90° 16. Force applied by engine = 6 m
Resultant force acting on the body When two cars are pulled,
F = F12 + F22 + 2FF
1 2 cos q (m + m) a = 6 m
2
= (8) + (6) 2 + 2 ´ 8 ´ 6 ´ cos 90° or 2 ma = 6 m or a = 3 ms–2

= 64 + 36 (Q cos 90° = 0) 17. Mass of the body m = 3.0 kg


= 10 N Initial speed, u = 2.0 m/s
If resultant force F makes an angle a with force F1, then Final speed, v = 3.5 m/s
F 6 Time, t = 25 s
tan a = 2 = = 0.75 = tan 36° 33¢ Force, F =?
F1 8
Using the first equation of motion, v = u + at
a = 36°53¢
\ 3.5 = 2.0 + a ´ 25
Using relation F = ma 3.5 - 2.0
F 10 or a= m/s2
Acceleration a = = = 2 m/s2 25
m 5 1.5
Acceleration a = m/s-2
\ An acceleration of 2 m/s2 is acting on body at an angle of 25
36°33¢ from the direction of force F1 = 8 N \Force acting on the body
1.5 4.5
12. Here the tension in the cord is given by F = ma = 3.0 ´ = N = 0.18 N
25 25
T = mg + ma T
Here upward acceleration = a As direction of motion of the body remains unchanged,
Mass of sphere = M therefore the direction of force acting on the body is along the
a direction of motion.
T = 4 mg
Þ 4 mg = mg + ma 18. The net electromagnetic force = N 2 + f 2
3 mg = ma mg But N = mg , f = m mg
Þ a=3g Force = mg 1 + m 2
13. Given, x (t) = pt + qt 2 + rt 3 and p = 3 ms–1, q = 4 ms–2, dm
19. F = v = 10 ´ 5 N = 50 N
–3
r = 5 ms , then x (t ) = 3 t + 4 t 2 + 5 t 3 dt
d 2x (t ) 20. Mass of the ball m = 0.15 kg
a= = 8 + 30 t
dt 2 Velocity of the ball v = 54 km/h
\ t =2s 5 æ 5 ö
= 54 ´ m/s çQ1 km/h = m/s÷
\ a = 8 + 30 ´ 2 = 68 18 è 18 ø
Now F = m ´ a = 2 ´ 68 = 136 N = 15 m/s
Let the ball be incident along path PO. Batsman deflects the
14. By drawing the free body diagram of point B. Let the tension in ball by an angle of 45° along both OQ.
the section BC and BF are T1 and T2 respectively. N
From Lami’s theorem P Q
u cos θ
C F
120° u u
T1 T2
°
22.5°

120°
22.5

120°
10 N = T
T3 45°
u sin θ
O u sin θ
A θ
T1 T2 T
= = u cos θ u
sin 120° sin 120° sin120°
Þ T = T1 = T2 = 10 N
Telegram @unacademyplusdiscounts

Laws of Motion and Friction 215

45° 24.
ÐPON = ÐNOQ = = 22.5° a1 x1
2 T1
The horizontal component of velocity u sin q remains m1
unchanged while vertical component of velocity is just
reversed. T1 T1
\ Impulse imparted to the ball
= change in linear momentum of the ball
= mu cos q - ( - mu cos q)
T2
= 2 mu cos q
= 2 ´ 0.15 ´ 15 ´ cos 22.5° m2 x2
= 4.5 ´ 0.9239 kg-m/s = 4.16 kg-m/s a2
m2g
21. Upthrust on the body = Vs g for freely falling body effective g
becomes zero so upthrust becomes zero. From force diagram,
T1 = m1a1 …(i)
22. Combined momentum = 2p$i + p$j
T2 = 2 T1 …(ii)
Magnitude of combined momentum
m2g - T2 = m2a1
= (2p) 2 + p 2 = 5 p 2 = 5p
m2g - 2T2 = m2a1 …(iii)
This must be equal to the momentum of the third part. Total work done by tensions should be zero.
23. For A, T = f = 2 mg 2mg \ T1x1 - T2x2 = 0
2 mg - mg = ma1 or T1x1 = T2x2
\ a1 = g A or T1x1 = 2 T1x2
For B, or x1 = 2 x2
From force diagram shown in figure, mg d 2x1 2 d 2x2
or =
dt 2 dt 2
\ a1 = 2 a2 …(iv)
After solving Eqs. (i), (iii) and (iv),
m2g
a2 =
4 m1 + m2
mg m 2m 2mg
B m 25. Change in momentum = Impulse
mg 3m 2mg Þ Dp = F ´ Dt
mg
2m Dp
Þ Dt =
2mg
F
125
2 mg - mg = 3 ma2 = = 0.5 s
250
g
For \ 2 = mass
discussion of motion of theapoint , we assume that 26. m
observer is situated at the triangular block.
For C,
The force diagram of point mass m is shown in figure.
m
a
0
co

N
s
θ

θ m
m
g

mg 2m 2mg ma0
si

m
n

B m
θ

mg 3m 2mg
mg m θ
mg cos θ + a0
F2 = mg ma0 sin θ
mg
\ 2 mg - mg = 2 ma3
From force diagram,
g
\ a3 = mg sin q - ma0 cos q = ma
2
So, a1 > a3 > a2 \ a = g sin q - a0 cos q
Telegram @unacademyplusdiscounts

216 JEE Main Physics

27. Resultant acceleration ( a)R = 2a - a 31. In uniform translatory motion, all parts of the ball have the
a same velocity in magnitude and direction, and this velocity is
constant.
√2 a dp æ dm ö 2
32. F = =v ç ÷ = av
dt è dt ø
a F
\ a=
M
a
av 2
=
( a)R = ( 2 - 1) a M
28. For solving the problem, we assume that observer is situated 33. When a metre scale is moving with uniform velocity, the force
in the frame of pulley (non-inertial reference frame). acting on the scale is zero and the torque acting about centre
m1g = w1 of mass of the scale is also zero.
m2g = w2 34. Reading = Weight of cage + Reaction by bird
From force diagram, = 20 + 0.5 (10 +2) = 26 N
T T 35. Dp = p2 - p1 = mv - mu
a
a = 0.15 ´ - (3$i - 4$j) - 0.15 (3$i+ 4$j)
= - (0.9 $i - 1.2 $j) kg ms -1

w2 m2a0 m1a0 w1 36. | Dp| = ( -0.9) 2 + ( -1.2) 2 = 1.5 kg ms–1


T - m2a0 - w2 = m2a 37. Effective value of acceleration due to gravity in the lift = g - a
or T - m2g - w2 = m2a (Q a0 = g ) Acceleration down the inclined plane
or T - 2 w2 = m2a …(i) 1
Using, s = ut + at 2, we get
From force diagram 2
1
m1a0 + w1 - T = m1a l = ( g - a) sin qt 2, we get
2
or m1a0 + w1 - T = m1a
2l
or 2 w1 - T = m1a …(ii) (Q a0 = g ) t=
( g - a) sin q
From Eqs. (i) and (ii),
4 w1w2 38. Initial thrust mg + ma = m ( g + a)
T=
w1 + w2 = 10 5 (10 + 5) N
29. Various forces acting on the ball are as shown in figure. The = 15 ´ 10 5 N
three concurrent forces are in equilibrium. Using Lami’s
= 1.5 ´ 10 6 N
theorem.
T1 T2 10 1
= = 39. Horizontal component of 10 N is10 cos 60° i. e. ,10 ´ N
sin 150° sin 120° sin 90° 2
T1 T2 10 or 5 N. It is balanced y 5 N force along x-axis.
= =
sin 30° sin 60° 1 Y-component of 10 N force is 10 sin 60 N.
\ T1 = 10 sin 30° 3
i. e. , 10 ´ N or 5 3 N
= 10 ´ 0.5 = 5 N 2
T2 = 10 sin 60° 5 3N
ay =
3 3 kg
and = 10 ´ =5 3 N
2 = 5 ms–2
30. 2 mnv = Mg 2 ´3 ´5 15
40. T = kgf = kgf
Mg 3+5 4
\ v= 15
2 mn Pressure on the pulley = 2 T = 2 ´ kgf = 7.5 kgf
10 ´ 980 4
or u= cms–1
dm ö
2 ´ 5 ´ 10 41. Thrust F = u æç 4 6
÷ = 5 ´ 10 ´ 40 = 2 ´ 10 N
9800 è dt ø
= cms–1 = 98 cms–1
100
Telegram @unacademyplusdiscounts

Laws of Motion and Friction 217

42. Let the mass of a block is m. It will remains stationary if forces Suppose A is climbing with acceleration a such that T = 30 N
acting on it are in equilibrium T -2 a = 2 a
i. e. , ma cos a = mg sin a 30 - 2 ´ 10 = 2 a T
Þ a = g tan a or a = 5 ms –2 A 5g
ma T
co Again, T¢ -T -5 g = 5 a 5g

or T¢ = T + 5 g + 5 a T
ma α or T ¢ = (30 + 50 + 25) N B 2g
mg
sin
α = 105 N 2g
mg α
47. 500 g - T = 500 a …(i)
Here, ma = pseudo force on block, mg = weight T - 100 g sin 30° - T ¢ = 100 a
43. As is clear from figure.
F sin θ T T
a
F T
θ a 0º a
sin 3
θ 100g
mg cos θ
mg
θ 500 g 50 g

R = mg cos q + F sin q or T - T ¢ - 50 g = 100 a …(ii)


44. Let force in downward to the incline mg sin q - T = ma Again, T ¢ - 50 g = 50 a …(iii)
T = ma From Eqs. (i), (ii) and (iii), 400 g = 650 a
400 g 8 g
T or a= =
a 650 13
m a This acceleration is downwards.
T 48. As the elevator is going down with T
n

m
si

decreasing speed, so acceleration is


g
m

upward direction. Let it is a


\ mg sin q - T = T T - 800 g = 800 a v = 10 ms–1
or 2 T = mg sin q T = 800 ( g + a)
1 v=0
or T = mg sin q From v 2 = u 2 - 2as,
2
\ a = 2 ms–2 800 g
1
45. Here, sin q = \ T = 800 (10 +2),
l
R θ \ T = 9600 N
ma

θ
ma 49. Here, m = 5 kg, F = ( - 3$i + 4$j) N
a
sin
mg θ Initial velocity at t = 0 , u = (6$i - 12$j)
mg cos θ F
θ
Retardation, a=
m
Let required acceleration of inclined plane be for the object to
remain stationary relative to inclined. æ 3$i 4$j ö
= ç- + ÷ m/s 2
We have è 5 5ø
ma cos q = mg sin q As final velocity is along Y-axis, its x-component must be zero.
1 From v = u + at , for X-component only,
a = g tan q = g
2
l -1 3$i
0 = 6$i - t
46. If A is climbing with constant velocity, then 5
T ¢ = 5 g + T and T = 2 g 5 ´6
t= = 10 s
T¢ = 5 g + 2 g = 7 g 3
Telegram @unacademyplusdiscounts

218 JEE Main Physics

50. 2000 g = 1000 g + 1000 a F æ 3Fö


or = 0.5 ç1 ´ 10 - ÷
2 è 2 ø
or a=g
Direction is upward 3F
or F+ = 10
Now, 0 2 - 2.5 2 = -2 ´ 10 ´ s 2
2.5 ´ 2.5 625 25 5 20
or s= = = m= m or F=
20 100 ´ 20 80 16 2+ 3
51. Wall applies 2 forces of the block (i) normal reaction, R = 12 N, 20
or F= N = 5.36 N
and (ii) frictional force, f2 = mg = 5 N tangentially upward. 3.732
\Total force exerted by wall on block 56. Change in momentum
5N Dp = 2 mu sin 30°
13 N 1 30°
= 2 ´ 0.1 ´ 10 ´
f 2
N
12 N 12 N
= 1kg-ms–1
Dp 1
\ Fav = = = 10 N
5N Dt 0.1
æ m ö 0.25 l l
F = N 2 + fs2 = (12) 2 + (5) 2 = 13 N
57. l ¢ = ç ÷l= = = 20% of l
è m + 1ø (0.25+1) 5
52. 58. As, F = FAB + FBG
g /2
a=

° A
30 fAB
in
m gs B
30°
fBG
Ground
cos
mg
From diagram, T sin q - mg sin q = ma = m AB ma g + mBG (mA + mB) g
mg = 0.2 ´ 100 ´ 10 +0.3 ´ (300) ´ 10
T sin q = mg sin q + …(i)
2
= 200 + 900
T cos q = mg cos q …(ii)
= 1100 N
Dividing Eq. (i) by Eq. (ii), we get
2 59. In the given condition the required centripetal force is
tan q =
3 provided by frictional force between the road and tyre.
mv 2
53. For block to continue motion on belt, acceleration = mmg
a = + mg = 0.2 ´ 10 = 2 ms–2 R
\ v = mRg
\ Velocity of belt = Velocity of block after 4 s
= 2 ´ 4 = 8 ms –1 1 3
60. Weight of chain on table = mg - mg = mg
4 4
m (v - u) 0.15 [20 – (–10)] 0.15 ´ 30
54. F = = = = 45 N For maximum possible friction,
t 0.1 0.1
3 1 1
mmg = mg \ m=
55. R + F sin 60° = mg 4 4 3
3F 61. Net downward acceleration
or R = mg - F
2 Weight - Frictional force
=
F sin 60º Mass
R F
mg - mR R 600 N
=
60º
F cos 60º m
1 60 ´ 10 - 0.5 ´ 600
=
60
300 W
= = 5 m/s2
mg 60
F cos 60° = f = mR
Telegram @unacademyplusdiscounts

Laws of Motion and Friction 219

62. Using the relation 66. Downward retardation means upward acceleration.
mv 2 g¢ = g + a
= mR ,R = mg
r 2L
Now, t=
mv 2 g ¢ sin q
= m mg
r
2L
or v 2 = mrg or t=
( g + a) sin q
or v 2 = 0.6 ´ 150 ´ 10
or v = 30 ms–1 67. ma = m sin q - f
g
8k f
63. From acting on block are shown in adjoining figure.
N a
mg
F cos 60° 30°
60°
60° or f = mg sin q - ma
f =µN F æ 1 ö
= 8 ç10 ´ - 0.4÷ N
è 2 ø
Mg + F sin 60°
= 8 ´ 4.6 N = 36.8 N
As the block does not move, hence
F cos 60° = f = mN = m (Mg + F sin 60° ) 68. The work done by the force is F cos 37°,
where F cos 37° = f = mN
1 1 æ 3ö
\ F = ç 30 ´ 10 + F ÷ F sin 37°
2 2 3 è 2 ø
On simplification, we get F = 20 N F

64. Let length of chain be l and mass m. Let a part x of chain can 37°
F cos 37°
hang over one edge of table having coefficient of friction.
f 20 kg
l x

x mg N

In this case, N = mg - F sin 37°,


m mg
mx So that, F=
\Pulling force, F= g (cos 37° + m sin 37° )
l
m Here, m = 0.40 and m = 20 kg
and friction force, f = mN = m ( l - x) g
l \ F = 75.4 N
For equilibrium, F = f , hence Hence, W = (75.4 cos 37° ) (8.0) = 482 J
mx m m
× g = m ( l - x) g = 0.25 ( l - x) g 69. Force on the car
l l l
1 F = mR
Þ x= or ma = m mg (Q R = mg )
5
x 1 or a = mg
or = = 20%
l 5 Now from 2nd equation of motion
1
65. For a smooth plane, v = 2g sin q × s and for a rough plane, s = ut + at 2
2
v
= 2g (sin q - m cos q) s 1
n or s = 0 + at 2 (Q u = 0)
2
sin q
\ n= 2s
sin q - m cos q or t=
mg
sin q
or n2 = 2s
sin q - m cos q \ t=
Þ 2 2
(n - 1) sin q = n m cos q mg
1
æ n 2 - 1ö æ 1ö or tµ
or m = ç 2 ÷ tan q = tan q ç1 - 2 ÷ m
è n ø è n ø
Telegram @unacademyplusdiscounts

220 JEE Main Physics

70. For equilibrium of the block M


76. Tx = (mass of rope of length L - x) = (L - x) g
N = mng + Q cos q K
Mg (L - x)
Q sin q + P = mN Þ Tx =
L
Q sin q + P = m (mg + Q cos q)
N
77. Reaction on m is mg. Maximum friction force by m on M is
mmg. So, the force on M is mmg forward.
N
m mg
Acceleration =
P M
µN P + Q sin θ
78. The trolley shall move backwards to conserve momentum.
θ The backward momentum would be shared by both the
Q
mg
trolley and man.
mg + Q cos θ Applying conservation of momentum
60 ´ 1 (240 + 60) v
æ Q sin q + P ö
\ m=ç ÷ or 60 = 300 v
è mg + Q cos q ø
60
or v=
71. If m is the mass/length, then 300
1
Weight of hanging length = m lg = ms–1 = 0.2 ms–1
5
Weight of chain on table = m (L - l) g
Speed of man w.r.t. ground = (1 - 0.2) ms–1 = 0.8 ms–1
R = m (L - l) g
f = m sR = m sm (L - l) g Displacement of man = 0.8 ´ 4 m = 3.2 m

Equating, m sm (L - l) g = mlg 79. Refer to the free body diagram of block B


l 5 g -T = 5 a
or ms =
L-l or T = 5 g -5 a
72. Pseudo force on the block = m ´ 4 N (backward) Refer to the free body diagram of block A
Force of friction = 0.4 ´ m ´ 10 N (forward)
T a
Equating, m ´ 4 = 0.4 ´ m ´ 10 = 4 m
Clearly, the equation holds good for all values of m. B a A T

73. fms = 0.6 ´10 ´ 9.8 N = 58.8 N


Since the applied force is greater than fms therefore the block 5g
will be in motion. So, we should consider fk. T - f = 5a
fk = 0.4 ´ 10 ´ 9.8 N or 5g - 5a - 0.5 ´ 5 ´ g = 5 a
or fk = 4 ´ 9.8 N or 10 a = 2.5g = 2.5 ´ 9.8
This would cause acceleration of 40 kg block or a = 0.25 ´ 9.8 ms–2 = 2.45 ms–2
4 ´ 9.8 N
Acceleration = Again, T = (5 ´ 9.8 – 5 ´ 2.45) N
40 kg
= (49 - 12.5) N
74. Force, P = fms = m smg (when body is at rest) = 36.75 N
When the body starts moving with acceleration a, then Net pushing force
80. Acceleration of system a =
P - fk = ma Total mass
m smg - m kmg = ma F - (m1 + m2 + m3) g sin q
or a=
or a = (m 0 - m k) g (m1 + m2 + m3)
or a = (0.5 - 0.4)10 Equation of motion for m3
= 0.1 ´ 10 ms –2
= 1ms –2 N - m3 g sin q = m3 a
ì F - (m1 + m2 + m3) g sin qü
75. Net acceleration, or N = m3 g sin q + m3 í ý
î (m1 + m2 + m3) þ
a = g (sin q - m cos q) = 9.8 ( sin 45° - 0.5 cos45° )
m3F
4.9 =
= m/s2 m1 + m2 + m3
2
Telegram @unacademyplusdiscounts

Laws of Motion and Friction 221

81. Tension between m2 and m3 is given by After breaking of string, mass m2 moves under gravity and go
further higher through a height h, where final velocity is zero.
Hence,
(0) 2 - (9.8) 2 = 2 ´ ( -9.8) ´ h
or h = 4.9 m
85. When the system accelerates upwards, the effective value of
acceleration due to gravity is given by
m1 A
B m2 f¢ = g + a = g + g = 2 g
2 (m1) (m2)
Now, T= (2 g )
m1 + m2
C m3 4 m1m2
or T= g
m1 + m2
2m1m2 2 ´2 ´2
T= ´g = ´ 9.8 = 13 N
m1 + m2 + m3 2+2+2 86. The system may be represented as follow
a
82. FBD of mass 2 kg FBD of mass 4 kg T
nm
T T′
4N 8N

T
2 kg 4 kg
m2 a
19.6 N
T′
Mg
39.2 N
T - T ¢ = 19.6 = 4 …(i)
From the force diagram,
T ¢ - 39.2 = 8 …(ii)
Mg - T = Ma …(i)
From Eq. (ii)
and T = nma …(ii)
T ¢ = 47.2 N
From Eqs. (i) and (ii), we get
and substituting T ¢ in Eq. (i), we get
Mg
T = 4 + 19.6+ 47.2 = 70.8 N a=
nm + M
83. From geometry l = x2 + y 2 but y is constant, hence on
2
The force diagram of nth block is shown in figure.
dl dx
differentiating, we have, 2 l = 2x N
dt dt
dl
But = v. Hence, horizontal velocity of block,
dt m Tn
dx
vx = n
dt
mg

l θ From the figure,


y mMg
θ Tn = ma =
nm + M
87. For minimum mass of m, mass M breaks off contact when
x elongation in spring is maximum.
Þ lv = x × v x At the time of break off, block A is at lowest position and its
l ×v v speed is zero. At an instant t1
or vx = =
x sin q mg - kx = ma T = kx
84. Acceleration of combined system, dv mg - kx
v = mg – kx = mg
m1 - m2 3 -2 dx m
a= ×g = ´ 9.8 = 1.96 ms–2
m1 + m2 3+2 0 t æ k ö
ò0 v dv = ò0 çè g - m x÷ø dx mg
Vertically upward velocity of 2 kg mass at the time of breaking
of string, v = at = 5 ´ 1.96 = 9.8 ms–2. where x0 is maximum elongation in spring
Telegram @unacademyplusdiscounts

222 JEE Main Physics

kx02 91. Net force, T2 - T1 = m2a …(i)


0 = gx0 -
2m
and T1 = m1a …(ii)
2 mg
x= Dividing Eq. (i) by Eq. (ii) a
k
T2 - T1 m2
At the time of break off of block B = m2
T1 m1 T1 T2
Mg = kx0
T2 m2 m + m1 a
Mg = 2 mg or = +1= 2
M T1 m1 m1 m1
m= T1 m1
T2
2 or =
T2 m1 + m2
88. In case of projectile motion at the highest point
(v) vertical = 0 92. Spring balance reading in terms of kgf
and (v) horizontal = v cos q 4 m1m2 4 ´ 5 ´ 1 10
= =
The initial linear momentum of the system will be mv cos q. m1 + m2 6 3
Now as force of blasting is internal and force of gravity is This is less than 6 kgf.
vertical.
40
So, linear momentum of the system along horizontal is 93. Common acceleration, a = ms–2 = 2 ms–2
conserved. 10 + 6 + 4
T2 40 N
p1 + p2 = mv cos q 4 kg
m1v1 + m2v 2 = mv cos q
m Now, 40 - T2 = 4 ´ 2
But it is given that m1 = m2 = and as one part retraces its
2 or T2 = ( 40 - 8) N = 32 N
path,
v1 = -v cos q 94. From diagram, 5 g - T2 = 5 a …(i)
1 1 T2 - T1 = 3 a …(ii)
\ m ( -v cos q) + mv 2 = mv cos q
2 2 T1 - g = a …(iii)
or v 2 = 3 v cos q T2 T2 T1
89. T sin 30° = 2 kg-wt
30° T sin 30° B A a B
a a
T
30°
T cos 30°
T1 3g T1 5g g

Solving Eqs. (i), (ii) and (iii), we get


2 kg-wt g =9a
g
or a=
Þ T = 4 kg-wt 9
Newton’s second law T1 = °T 95.
= 4 cos 30° = 2 3 F = ma
Þ 6 = (7 + 5) a
90. 2 mg - T = 2 ma …(i)
1
a = m/s2
T - mg sin 30° = ma …(ii) 2
R f ¢ = 5 kg
T
1
T Now f ¢ = 5 ´ = 2.5 N
2
° 2M 96. Initially, the weight of load L is the force on the system of mass
30°

30
in
gs mg cos 30°
8 kg.
m 30° 2 ´ 10 20
Acceleration = = unit
8 8
Adding Eq. (i) and (ii), we get
mg Towards the end, force = (2 + 1) ´ 10 N = 30 N
2 mg - = 3 ma 30
2 So, acceleration now is units
g 8
Þ a=
2
Telegram @unacademyplusdiscounts

Laws of Motion and Friction 223

Round II
1. Due to acceleration in forward direction vessel is in an 7. If T is tension in each part of the string holding mass 2 m,
accelerated frome therefore a pseudo force will be exerted in then in equilibrium,
backward directon. Therefore water will be displaced in T cos q + T cos q = 2 mg
backward direction.
2T cos q = 2 mg
2. Masses connected at the two ends of a light
But T = mg
inextensible string are
\ 2mg cos q = 2 mg
m1 = 8 kg, m2 = 12 kg
1
Let T be the tension in the string and T T a cos q =
2
masses moves with an acceleration a when
masses are released. q = 45°
m1
a
For mass m1 8. Making FBD of block with respect to disc.
T - m1g = m1a …(i)) m 1g Let a be the acceleration of block with respect to disc
For mass m2 m2 N1 ma sin q
m2g - T = m2a …(ii) ma
Adding Eqs. (i) and (ii), we get m2g ma cos q
m2g - m1g = (m1 + m2) a µ
(m - m1)
\ a= 2 g …(iii)
(m1 + m2)
12 - 8 N2 mg
= ´ 10 = 2 m/s 2
12 + 8 N1 = mg
Substituting value of a in Eq. (i), we get N2 = ma sin q
T = m1g + m1a ma cos q - mN2 - mN1
A= = 10 m/s2
= m1( g + a) = 8(10 + 2) = 90 N m

3. When block R collides with block P, it transfers its momentum 9. As shown in figure, component of weight( mg sin q ) is always
to block P, due to which it moves towards blocks Q. The down the inclined plane, whether the cylinder is following up
spring connecting blocks P and Q gets compressed, which or it is rolling down. Therefore, for no slipping, sense of
will push the block Q outwards. Due to outward motion of angular acceleration must be the same in both the cases.
the block Q, the spring gets stretched, the block Q is pulled Therefore, force of friction (f) acts up the inclined plane in
back. The spring gets compressed, it pushes the block P both the cases.
towards left and so on.
f
4. Let T be the tension in the string. Since mg sin θ
the system is in equilibrium, therefore
from figure. θ
θ T
T
2T cos q = mg
or T = mg / 2 cos q 10. As F2 and F3 are mutually perpendicular, their
The string will be straight if q = 90° resultant = F22 + F32
mg
\ T = mg /2 cos 90° = mg /2(0) = ¥ As particle is stationary under F1, F2, F3 therefore, F22 + F32 must
5. For body of mass 6 kg be equal and opposite to F1.
T = 6g = 6 ´ 9.8 = 58.8 N 11. Angular momentum is an axial vector, so its direction is along
For body of mass 4 kg the axis, perpendicular to the plane of motion which is not
T - T1 = 4g = 4 ´ 9.8 = 39.2 N changing because of change of speed. Therefore, the
direction of angular momentum remains, the same and its
T1 = T - 39.2
magnitude may vary.
= 58.8 - 39.2 = 19.6 N
12. The reading of balance A will decrease due to the upward
6. Minimum force required to move the block thrust caused by buoyancy. The upthrust will be equal to the
= mR = mmg = 0.4 ´ 2 ´ 10 = 8 N weight of water displaced. The net downward force due to
Since the force applied is only 2.8 N, the block fails to move mass immersed in water will add to effective weight of the
and static fraction = applied force = 2.8 N system. So, the reading of balance B will increase.
Telegram @unacademyplusdiscounts

224 JEE Main Physics

13. Impulse = Change in momentum 17. Mass measured by physical balance remains unaffected due
F ´ t = m (v - u) to variation in acceleration due to gravity.
F ´ 0.4 = 80(5 - 0) 1 1
18. From s = ut + at 2 = 0 + at 2
80 ´ 5 2 2
Þ F= = 1000 N
0.4 2s
t=
14. Applying Lami’s theorem a
T1 T2 mg For smooth plane, a = g sin q
= =
sin(90°+ b) sin(90°+ a) sin[180°- ( a + b)] For rough plane, a, = g (sin q - m cos q)
T1 T mg
or = 1 = 2s
cos b cos a sin( a + b) \ t¢ =
g (sin q - m cos q)
mg cos b mg cos a
\ T1 = ; T2 = 2s
sin( a + b) sin( a + b) = nt = n
g sin q
15. Figure shows free body diagram of the block.
\ n 2g (sin q - cos q) = g sin q
T cos 45°
when q = 45° ,
T
R sin q = cos q = 1 / 2
µR

1
f=

Solving we get, m = 1 -
n2
°
45

mg cos 45°
n

19. Maximum force by surface when friction works


si

mg
g
m

45° F = f 2 + R 2 = mR 2 + R 2

For equilibrium, along the plane = R m2 +1


mR + T cos 45° = mg sin 45° Maximum force = R when there is no friction
T mg Hence, ranging from R to R m 2 + 1
mR + = …(i)
2 2
we get Mg £ f £ mg m 2 + 1
For equilibrium, in direction perpendicular to inclined plane,
R = T sin 45° = mg cos 45° 20. As m1 : m2 : m3 = 1: 1: 3
T mg and momentum is conserved,
= =
2 2 \ P12 + P22 + P32 = 3v3
m 1
Put in Eq. (i), (T + mg ) = - (mg - T) 1 ´ 39 2 + 1 ´ 39 2 = 3v3
2 2
m(50 + 15 ´ 10) = (15 ´ 10 - 50) 39 2 = 3v3
100 1 39 2
m= = v3 = = 13 2 ms-1
200 2 3
For Mass
21.16. of a stone
the smooth m = 0.25
portion BC, kg
Radius of the string r = 1.5 m u = 0 , s = l, g sin f, u = ?
40 2 From v 2 - u 2 = 2as
Frequency n = 40 rev/min = rev/s = rev/s
60 3 v 2 - 0 = 2g sin f ´ l
Centripetal force required for circular motion is obtained For the rough portion CO
from the tension in the string.
u = v = 2g sin f. l
\Tension in the string = centripetal force
T = mrw2 v = 0, a = g (sin f = m cos f)
s=l
= mr(2pn) 2 [Q w = 2p n ]
From v 2 - u 2 = 2as
2 2
= mr 4p v
0 - 2gl sin f = 2g (sin f - m cos f) l
2 2
æ 22 ö æ2ö - sin f = sin f - m cos f
= 0.25 ´ 1.5 ´ 4 ´ ç ÷ ´ ç ÷ = 6. 6 N
è7ø è3ø m cos f = 2 sinf
Maximum tension which can be withstand by the string m = 2 tan f
Tmax = 200 N
Telegram @unacademyplusdiscounts

Laws of Motion and Friction 225

22. As is from figure. 26. As both the balls are of same size, force of buoyancy on each
is same. Therefore, in equilibrium,
R α
F F m2g

α
mg sin α
mg cos α mg T
F = mg sin a m1g
R = mg cos a
F F
= tan a
R
1 F + F = m1g + m2g
i.e., m = tan a =
3 g
or F = (m1 + m2)
\ cot a = 3 2
Considering the equilibrium of lower ball,
23. The three forces acting on the mass B
at location A have been shown in T + F = m1g
figure. Since the mass is in θ T = m1g - F
T
equilibrium, therefore, the three g
forces acting on the mass must be T = m1g - (m1 + m2)
2
represented by the three sides of a g
triangle in one order, Hence S A T = (m1 - m2)
50 N 2
50 6 ´ 10 27. Weight of the disc will be balanced by the force applied by
In D SBA, =
SA SB the bullet on the disc in vertically upward direction.
SA 50 5
or = = 6 × 10 N F = nmv = 40 ´ 0.05 ´ 6 = Mg
SB 60 6 40 ´ 0.05 ´ 6
SA 5 M= = 1.2kg
or tan q = = 16
SB 6
= 0.8333 = tan 40° 28. In figure O is the centre of the bowl of radius r. The insect will
\ q = 40° crawl (from B to A ) till component of its weight (mg ) along the
bowl is balanced by the force of limiting friction (f)
24. Mass of rope M = 0.1kg, q = 10°
r O
From figure
f θ y
T sin θ R
θ θ A C
T T sin θ B
h
θ
θ

T
s
co

m
g
g
m

si

T cos θ T cos θ mg

i. e. , mg sin q = f = mR = mmg cos q


mg AC
or m = tan q =
2T sin q = mg OC
mg 0.1 ´ 9.8 OA2 - OC 2 r2 - y2
T= = = 2.82 N or = =
2 sin q 2sin10° OC y
25. Effective upward force = 310 - mg r2 - y2
or m2 =
= 310 - 24 ´ 9.8 = 74.8 N y2
Upward acceleration
m 2y 2 + y 2 = r 2
\ a = 74.8/24 = 3.12ms-2
r
1 y=
As s = ut + at 2 m2 +1
2
1 h = BC = OB - OC = r - y
4.6 = 0 + ´ 3.12 ´ t 2
2 r æ 1 ö÷
2 4.6 =r- = r çç1 - ÷
or t = = 2.95 m2 +1 è m 2 + 1ø
1.56
or t = 2.95 » 1.7s
Telegram @unacademyplusdiscounts

226 JEE Main Physics

1 1
29. If v is common velocity of the block and movable wedge, then At t = s, a(t ) = - 16 p 2 A sin 4p ´ = - 16 p 2 A ´ 1
applying the principle of conservation of linear momentum 8 8
we get, F = ma (t ) = - 6p 2Am
mu + 0 = (m + nm)v Impluse = chagne in linear momentum
mu u 1
v= = = F ´ t = - 16 p 2 Am ´ = - 4 p 2 Am
m (1 + n) 1 + n 4
This infact, can be taken as velocity of centre of mass of the The impulse (change in linear momentum) at t = 0 is same
block and wedge 1
as at t = sec.
u 4
i. e. , v CM = v =
1+ n Further, as F depends on A which is not constant, therefore,
Applying the principle of conservation of energy the particle is not acted upon by a constant force.
1 1 u2 33. In figure, f is the force of friction. When the body moves up
mu 2 = mgh + m (1 + n)
2 2 (1 + n) 2 the plane, f acts down the plane.
u2
or u 2 = 2gh +
1+ n R
æ 1ö
u = 2ghç1 + ÷ θ
è nø in
gs f θ m1g cosθ m2
m1
30. Total mass of 80 wagons = 80 ´ 5 ´10 -3 5
= 4 ´ 10 kg θ m2g
m1g
5
F 4 ´ 10
Acceleration, a= = = ms-2 f = mR = mm1 g cos q
M 4 ´ 10 5
In that event, m2g > m1 g sin q + f
Tension in the coupling between 30th and 31st wagon will be
due to mass of remaining 50 wagons. Now, mass of remaining m2g > m1g sin q + m m1 g cos q
50 wagons. m2 > m1 (sin q + m cos q)
3
m = 50 ´ 5 ´ 10 kg = 25 ´ 10 kg 4 Choice (b) is correct.
\Required tension, T = mg = 125 ´ 10 4 ´ 1 When the body moves down the plane, f acts up the plane.
In that event
= 25 ´ 10 4 N
(m2g + f ) < m1g sin q
v
31. Rate of flow water = 10 cm3s-1 m2g < m1g sin q - f
t
m2g < m1g sin q - m m1 g cos q
Density of water r = 10 3 kg /m3
or m2 < m1 (sin q - m cos q)
Cross-sectional area of pipe OA = p (0.5 ´ 10 -3) 2
Choice (d) is correct.
dv mv Vrv rv v
Force = m = = = ´ 34. As is clear from figure,
dt t t t At
2 R + T = (m + M) g
æv ö l æ v ö
() = + - R m=Mçè g T
÷ ç\ V = ÷
tø A At
ø The system will not move till
(10 ´ 10 -6) 2 ´ 10 3
F= T £ F or (T £ mR)
p ´ (0.5 ´ 10 -3) 2
T £ m[(m + M) g = T ]
= 0.127 N m(m + M) g m (m + M) g
T£ \ Fmax =
32. Here, x = 0 for t < 0 s m +1 m +1
1
x(t ) = A sin 4 pt for 0 < t < s 35. Minimum force required just to slide the block = force of static
4 friction
1
x=0 for t > s f = mR = mmg = 0.577 ´ 10 ´ 10 = 57.7 N
4
1 36. When we tend to move the body up the incline, force of
For 0 < t < s
4 friction acts down the plane.
dx F = mg (sin q + m cos q)
v(t ) = = 4pA cos 4pt
dt = 10 ´ 10(sin 40° + 0.577cos 45° )
dv 100
a(t ) = - 16p 2 A sin 4 pt F= ´ 1.577 = 111.5 N
dt 2
Telegram @unacademyplusdiscounts

Laws of Motion and Friction 227

37. From tan a = m = 0.577 For the string to become horizontal


q = 90°, cos q = cos 90° = 0
a = tan -1(0.577) = 30°
w
T= =¥
38. As impulse = F ´ t 2 cos 90°
4.25 = F ´ 0.1 Both the assertion and reason are true and latter is correct
4.25 explanation of the former.
F= = 42.5 N (downwards)
0.1
47. Bearings are used to reduce friction.
39. As action and reaction are equal and opposite force exerted 48. v = mrg = 0.1´10 ´10 = 10 ms-1
by the ground on the ball
F ¢ = F = 42.5 N (upward) 49. Mgrav g - N = minertial a for freely falling a = g
40. Loss of energy = E 2 = E1 = mg (h2 - h1) since mgrav = minert Þ N = 0
1 50. The purpose of bending is to acquire centripetal force for
= ´ 10(2.45 - 5) = -6.38 J
4 circular motion.By doing so component of normal reaction
Negative sign is for loss. will counter balance the centrifugal force.

41. As F ´ t = change in momentum of ball = constant, 51. The wings of the aeroplane. Pushes the external air backward
and aeroplane move forward by reaction of pushed air. At low
\ when t increases, F decreases.
altitudes density of air is high and so the aeroplane gets
i.e., by lowering his hands while catching the ball, he sufficient force to move forward.
increases the time of impact and in force exerted by the ball x 6
on his hands decreases. 52. Work done W = ò 2 F × dx = ò (5 + 3x ) dx
x1 2
250 1 2ö6
42. Here m = 250 g = kg = kg æ 3x
1000 4 = ç5x + ÷ = 68 J
è 2 ø2
h1 = 5 m, h2 = 2.45 m
Velocity of striking the ground, 53. The situation is as shown in the figure.
u = 2gh1 = 2 ´ 10 ´ 5 = 10 ms-1 a a

Velocity of rebounding from the ground T T


2 R2 A R1
µ2 = B
v = - 2gh2 = - 2 ´ 10 ´ 2.45 3 µ1R1 1
m 2m µ1 = 3
-1 mg cos 45°
= -7 ms 2m
°
45

2mg cos 45° g


n

Impulse = change in momentum of ball R2 mg co


si

µ 2 45° 2mg s4
g
m

1 45° 5°
= m(u - v) = (10 + 7)
4
The equation of motion for body B.
= 4.25 kgms-1 2mg sin 45° - m1R1 - T2 = 2ma
43. Assertion is true, but the Reason is false. The fan continue to 1
2mg sin 45°- 2mg cos 45°-T = 2ma
rotate due to inertia of motion. 3
1 1 1
44. This is because the direction of motion is changing Þ 2mg ´ - 2mg ´ - T = 2ma
2 3 2
continuously. Hence, the velocity is changing and
acceleration is being produced. In this problem as (mB = mA ) g sin q = (mg / 2) is lesser than
(mB mB + m AmA ) g cos q = ( 4mg / 3 2), the masses will not move
45. Angle of repose is equal to angle sliding friction and
maximum value of static friction is called the limiting friction. and hence .
46. As is clear from figure. Acceleration of B = Acceleration of A = 0
5
54. a = = 1 ms-2
q 5
T T
Upward distance covered in 4 s
1 1
y = at 2 = ´ 1 ´ ( 4) 2 = 8 m
2 2
w
Horizontal distance covered in 4s
2T cos q = w x = vt = 1.5 ´ 4 = 6 m
w s = x2 + y 2 = 6 2 + 8 2
T=
2 cos q
= 36 + 64 = 10 m
Telegram @unacademyplusdiscounts

228 JEE Main Physics

55. Block B will come to rest, if force applied to its will vanish due 63. The various forces acting on the block are shown in figure. As
to frictional force acting between block B and surface. the block remains stationary under the effect of these forces.
force applied = frictional force So, mg = f (force of friction)
mmg = ma f
Y
æv ö
or mmg = mç ÷ a
èt ø
v N a
or t=
mg
F X
C F
56. Force applied against friction N
1
fk = m kR = m kmg = ´ 60 ´ 9.8 = 196 N
3
mg
F mg /3 1
57. m = = = F = N (Normal reaction)
R 2 mg /3 2
Torque due to F or mg is zero as the line of action of F or mg
F passes through centre C. Since the body is in equilibrium so
58. Acceleration of system, a =
m+M the sum of torque due fo force of friction ( t f ) and torque due
to normal reaction ( t N ) must be zero, i. e. , t f + t N = 0. As
m M f
line of action of f and N may not pass through centre C of
block, so tf may not be zero and t N may also not be zero.
mF Therefore, f does not produce angle torque is wrong and N
So, force acting on mass = ma =
m+M does not produce a torque is also wrong. Hence, the incorrect
statement is (d) only.
59. Here, m = 150 g,u = 20 ms-1,v = 0,
t = 0.1 s, F = ? 64. As is clear from figure, the block will remain stationary, when
ma cos a = mg sin a
F ´ t = m(v - u) g sin a
m(v - u) 150(0 - 20) a= = g tan a
F= = = -30 N cos a
t 1000 ´ 0.1 dv
65. m = F0 e- bt
dt
60. Let v be the velocity given by hand and h be the height to dv f0 - bt v f0 t - bt
which the ball goes. Þ = e
dt m
Þ ò0dv = m ò0e dt
1
\ mv 2 = mgh …(i) t
2 f0 æ e-bt ö f0
Þ v= ç ÷ Þ v= (1 - ebt )
If F is the force applied by the hand as it moves through 0.2 m, m è -b ø 0 mb
then
1
66. For the motion of the block
mv 2 + F ´ 0.2 = mg (h + 2) mg - T = ma
2
For the rotation of the pulley
Using Eq. (i), mgh + F ´ 0.2 = mgh + mg ´ 2 t = TR = Ia
mg ´ 2 1 R
F= = 10 mg T = mRa m
2
= 10 ´ 0.2 ´ 10 = 20 N As string does not slip on the pulley T
a = Ra T
61. Here, m1 = 4 kg , m2 = 12 kg 2g
On solving Eqs. (i),(ii) and (iii) a =
u1 = ?, u2 = 4 ms-1 3 m

As momentum is conserved 67. The component of applied force F in the direction of motion is
\ m1u1 = m2 u2 F cos 30°. The work done by the applied force is
m2 u2 12 ´ 4 F sin 30°
u1 = = = 12 ms-1
m1 4 F = √3 kN
1 1
Kinetic energy = m1u12 = ´ 4 ´ (12) 2 = 288 J 30°
2 2 F cos 30°

62. Here, m1 = M, m2 = M / 2
(m1 - m2) g (M - M / 2) g g 3
a= = = W = (F cos 30° ) s = 3 ´ 10 3 ´ ´ 10 J
m1 + m2 M + M /2 3 2
= 15 ´ 10 3 J = 15 kJ
Telegram @unacademyplusdiscounts

Laws of Motion and Friction 229

68. Let L be the length and H be the Adding Eq. (i) and Eq. (ii)
height of the inclined plane mg (sin 53°- sin 37° )
60° a=
respectively. Acceleration of the H L 2m
block slide down the smooth = g cos 45° sin 8°
inclined plane is
é æ A + Bö æ A - Bö ù
a = g cos 60° êQ sin A - sin B = 2 cos çè 2 ÷ø sin çè 2 ÷ø ú
ë û
1
\ L = g cos 60° t12 (\ u = 0) 1
2 = 10 ´ ´ 0.139
2
Acceleration to another block dropped vertically down from
= 0.98 ms-1 @ 1 ms-2
the same inclined plane is
a=g f 6 2 + 8 2 + 10 2
1 1
71. m = = = 200 10 2 kg
\ H = at 22 = gt 22 (\ u = 0) a 1
2 2
72. Impulse D p = m(v f - vi ) = 0.4 [ -1-1] = 0.8 N-s
H
From figure, cos 60° =
L 73. Relative vertical acceleration of A will respect to B
Þ H = L cos 60° = g(sin 2 60°- sin 2 30° )
1 æ 3 1ö
\ L cos 60° = gt 22 …(i) = 9.8ç - ÷ = 4.9 m/ s2
2 è 4 4ø
Divide Eq. (i) by Eq. (ii), we get
74. Here mass of the block, m = 200 kg, coefficient of static
t12 cos 60° 1 1
= friction m s = 0.5 =
t 22 cos 60° 2
t12 1 4 NF
Þ = = R
t 22 cos2 60° 1
t1 2 200 kg
Þ =
t2 1 45° 200 g cos 45°
mg mmg mg mmg 200 g sin 45° 200 g
69. f1 = + , f2 = - F2 45°
2 2 2 2 µ mg/√2
F1 = 3 2 m Angle to inclined plane q = 45°
Maximum force that each man can apply F = 500 N
1+ m = 3 - 3m
Þ 4m = 2 mg/√2 Let N number of men are required for the block to just start
moving up the plane NF = mg sin q + f = mg sin q + m sR
1
Þ m= 45° = mg sin q + m smg cos q = mg [sin q + m s cos q]
2
æ 1 ö
N = 10m NF = 200 ´ 10 ç sin 45°+ cos 45° ÷
è 2 ø
Þ N =5
200 ´ 10 ´ 3
70. Let T be the tension in the string. Let a be the acceleration of =
2 2
the system. The equation of motion are
200 ´ 10 ´ 3
Ma = Mg sin 53°-T …(i) N= =5
2 2 ´ 500
and Ma = T - mg sin 37° …(ii)
2m1m2 2 ´ 10 ´ 6
75. T = g = ´ 9.8 = 73.5 N
T m1 + m2 10 + 6
T
N 76. In terms of three significant figure
a
° a Momentum p = mv = 3.5 /13 ´ 5.00 = 17.6 kg ms-1
37

sin
s5
s3

37° 53°
77. Roket pollution is associated with Newton's third law of
g

g
co
co

sin

m mg motion.
mg
mg

53

37° mg
53°
°
Telegram @unacademyplusdiscounts

7 Work, Energy
and Power
JEE Main MILESTONE
< Work < Potential Energy of a Spring
< Energy < Conservation of Mechanical Energy
< Work-Energy Theorem < Conservative and Non-Conservative Forces
< Power

7.1 Work
The scientific meaning of work is transfer of energy by mechanical means. Work is
said to be done by a force, when a body a displaced actually through some
distance in the direction of applied force. The SI unit work is joule (J) and in CGS is
erg.
1 joule (J) = 107 erg
The terms work, energy and power
Work Done by a Constant Force are frequently used in everyday
Let us first consider the simple case of a constant force F acting on a body. Further, language. In physics, however the
let us also assume that the body moves in a straight line; in the direction of force. In word ‘work’ converse a definite
this case, we define the work done by the force on the body as the product of the and precise meaning. ‘Energy’ is
magnitude of the force F and the distance S through which the body moves. our capacity to do work. In
F physics, the term energy is related
to work. The word ‘power’ is used
S
in everyday life with different
That is, the work W is given by shades of meaning. In boxing, we
W = F ×S talk of powerful punches. These
On the other hand, in a situation when the constant force does not act along the are delivered at a great speed. This
same direction as the displacement of the body, the component of force F along the shade of meaning is close to the
displacement S is effective in doing work. meaning of the word power used
F in physics.
θ

Thus, in this case, work done by a constant force F is given by


W = (component of force along the displacement) ´ (displacement)
or W = (F cos q) (S )
or W = F × S (from the definition of dot product)
So, work done is a scalar or dot product of F and S.
Telegram @unacademyplusdiscounts

Work, Energy and Power 231

Regarding work it is worth noting that Sample Problem 1 A body of mass 2 kg initially at rest
Work can be positive, negative or even zero also, depending moves under the action of an applied horizontal force of 7 N on
on the angle (q) between the force vector F and a table with coefficient of kinetic friction = 0.1. The work done
displacement vector S. Work done by a force is zero when by applied force in 10 s will be equal to
q = 90°, it is positive when q < 90° and negative when (a) 800 J (b) 825 J
q > 90°. For example, when a person lifts a body, the work (c) 882 J (c) 856 J
done by the lifting force is positive (as q = 0° ) but work Interpret (c) Here, m = 2 kg, u = 0 ,F = 7 N, m = 0.1, t = 10 s
done by the force of gravity is negative (as q = 160° ).
Acceleration produced by applied force,
Similarly, work done by centripetal force is always zero (as
F 7
q = 90° ). a1 = = = 3.5 ms-2
m 2
Note Positive work means that force or its component is parallel to Force of friction, f = mR = mmg
displacement while negative work means that force or its component is = 0.1 ´ 2 ´ 9.8 = 1.96 N
opposite to displacement. Retardation produced by friction
-f 1.96
a2 = =- = -0.98 ms-2
Work Done by a Variable Force m 2
Let us assume that a particle is moving along path A-B Net acceleration with which body moves,
(as shown in figure) w.r.t. a particular frame of reference a = a1 + a2 = 3.5 - 0.98 = 2.52 ms-2
under the action of force F.
Distance moved by the body in 10 s.
Y 1 1
(x1, y1, z1) Apply, s = ut + at 2 = 0 + ´ 2.52 ´ 10 2 = 126 m
ds 2 2
A
\Work done by the applied force
θ
s
F B = F ´ s = 7 ´ 126 = 882 J
(x2, y2, z2)
O X Sample Problem 2 A woman pushes a trunk on a railway
platform which has a rough surface. She applies a force of
Work done by the force on the particle is given by 100 N over a distance of 10 m. Thereafter, she gets
B
WA® B = ò F × ds progressively tired and her applied force reduces linealry with
A
distance to 50 N. The total distance through which the trunk
Here, F × ds is to be integrated along the path, the particle has been moved is 20 m. Frictional force is 50 N versus
follows displacement. The work done by the two forces over 20 m is
B (a) –25 J (b) –50 J (c) –100 J (d) –1000 J
The vector integral ò F × ds is equivalent to
A
Interpret (d) The plot of the applied force is as shown.
x2 y2 z2
WA® B = ò Fxdx + ò Fy dy + ò Fz dz At x = 20 m, F = (50 N) ( ¹ 0)
x1 y1 z1
Frictional force, f = 50 N. It opposes motion and acts in a direction
If a number of forces act on a body or particle, then
opposite to F.
W = W1 + W2 + W3 +¼

or W = ò F1 × ds + ò F2 × ds+¼ 100
B F C

Force E
W = ò (F1 + F2 +¼ ) × ds D
(N) A
x
10 m 20 m
or W = ò FR × ds (as FR = SF )
f = –50
H
G
Work done in displacing a particle under the action of a
number of forces is equal to the work done by the The work done by the woman is WF ® area of the rectangle ABCD +
resultant force. Further, if the body is in equilibrium (static area of trapezium CEID.
or dynamic) W = SW = 0 as for equilibrium FR = 0. Here, 1
WF = 100 ´ 10 + (100 + 50) ´ 10 = 1000 + 750 = 1750 J
work done by individual forces may or may not be zero, 2
but work done by some forces is positive while by others it Work done by the frictional force is
is negative, and positive work is equal to negative work, so WF = ( -50) ´ 20 = -1000 J
the net work becomes zero. The area on the negative side of the force axis has a negative sign.
Telegram @unacademyplusdiscounts

232 JEE Main Physics

Sample Problem 3 A cyclist comes to a skidding stop in Sample Problem 6 A force F = (2 + x) acts on a particle in
10 m. During this process, the force on the cycle due to the road x-direction where F is in newton and x in metre. The work done
is 200 N and is directly opposed to the motion. Work done by by this force during a displacement from x = 1.0 m to x = 2.0 m is
the road on the cycle is (a) 2 J (b) 3.5 J
(a) –2000 J (b) 4000 J (c) 4.5 J (d) None of these
(c) – 8000 J (d) 10000 J
Interpret (b) The work done in small displacement from x to
Interpret (a) Work done on the cycle by the road is the work x + dx is
done by the stopping (frictional) force on the cycle due to the road. dW = Fdx = (2 + x) dx
2 2 2 2
The stopping force and the displacement make an angle of 180° Hence, W= ò1 dW = ò1 (2 + x) dx = ò1 2 dx + ò1 xdx
(p rad) with each other. Thus, work done by the road is 2
é x2 ù
W = Fd cos q = ê2x + ú = 3.5 J
ë 2 û1
= 200 ´ 10 ´ cos p
= - 2000 J
Work Depends on the Frame of
Note It is the work that brings the cycle to a halt in accordance with Reference
work-energy theorem.
Considering an example, if a man is pushing a box
Sample Problem 4 A drop of mass 1 g falling from a inside a moving train, the work done in the frame of
height 1.00 km. It hits the ground with a speed of 50 ms -1. Work train will be F × s. While in the frame of earth will be
done by the gravitational force is F × (s + s0 ) where s0 is the displacement of the train
(a) 50 J (b) 10 J relative to the ground.
(c) 15 J (d) 20 J Work done by friction may be zero, positive or negative
depending upon the situation. When force applied on a
Interpret (b) The change is kinetic energy of the drop is
body is insufficient to overcome the friction, work done
1
mv 2 - 0
DK = by the friction force is zero. When this force is large
2
enough to overcome friction then work done by the
1
= ´ 10 -3 ´ 50 ´ 50 = 1.25 J friction force is negative.
2
Let us consider the situation in which there is a
where, we have assumed that the drop is initially at rest.
horizontal rough conveyer belt on which a block and a
Assuming that g is a constant with a value 10 m/ s2, the work done man is standing, is accelerating along the horizontal
by the gravitational force is direction. Block is not slipping on the belt.
Wg = mgh = 10 -3 ´ 10 ´ 10 3 = 10 J The following conclusions can be drawn from above.
(a) In this case work done by friction (between belt and
Sample Problem 5 The angle between force the block) is zero as observed by the man on conveyor
F = (3$i + 4$j - 5 k$ ) unit and displacement d = (5 $i + 4$j - 3 k$ ) belt.
(b) Work done by friction (between trolley and the block)
unit is
is positive as observed by an observer on the ground.
(a) – cos-1 (0.32) (b) sin -1 (0.31)
(c) Work done by friction is negative as observed by
(c) tan -1 (0.32) (d) cosec-1 (0.32)
observer who is moving along the direction of motion
Interpret (a) As, F × d = Fxd x + Fyd y + Fzd z of conveyor belt with higher speed.

= 3 (5) + 4 ( 4) + ( -5) (3)


Sample Problem 7 Over a horizontal plank a small block
= 16 unit of mass m is lying at rest. Now plank is moved with constant
Hence, F × d = Fd cos q = 16 unit acceleration as such that there is no relative motion between
Now, F × F = F 2 = Fx2 + Fy2 + Fz2 block and plank. Find the work done
by friction of plank on block in first t m
= 9 + 16 + 25 = 50 unit
seconds. a
and d × d = d 2 = d x2 + d y2 + d z2 (i) in ground frame
= 25 + 16 + 9 = 50 unit (ii) in plank frame
16 16 1 1
\ cos q = = = 0.32 (a) ma2t 2,zero (b) zero, ma2t 2
50 50 50 2 2
1 221 22
q = cos-1 (0.32) (c) zero, zero (d) ma t , ma t
2 2
Telegram @unacademyplusdiscounts

Work, Energy and Power 233

Interpret (a) by Newton’s second law of motion,


(i) In ground frame, F = ma
f
Friction force acting on the block f = ma v2 = 2 as (by third equation of motion)
Displacement in first t second or s = v2 / 2 a
1
s = at 2 Work done by the constant force = Fs
2
æ v2 ö 1
1
Work done = F × s = f æç at 2 ö÷ or W = ma ç ÷ = mv2
è2 ø è 2a ø 2
1 1
= ma × at 2 = ma 2t 2 But the kinetic energy of the body is equivalent to the work
2 2
done in giving the body this velocity.
F(= ma) 1
Hence, KE = mv2
f 2
(ii) In plank frame, Some practical units of energy and their relation with SI
f = ma
But displacement = 0 unit of energy (joule) are
(because there is no relative motion between plank and (i) 1 cal = 4.2 J
block) . ´ 106 J
(ii) 1 kilowatt hour (kWh) = 36
Hence, work done on the block in the plank frame is zero.
Sample Problem 8 A 120 g mass has a velocity at
v = (2 $i + 5 $j) ms-1 a certain instant, KE of the body at that
7.2 Energy instant is
Capacity of a body to do work is called its energy. Like (a) 3.0 J (b) 1.74 J (c) 4.48 J (d) 5.84 J
work, energy is a scalar quantity. The units of Interpret (b) Here, m = 120 g = 0.12 kg,
measurement of energy are same as the units of work. In
v = (2$i + 5$j) ms-1
SI, the unit of energy of joule (J) and is CGS, the unit of
energy is erg. There are so many types of energy e. g. , v =|v|= 2 2 + 52 = 29 ms-1
kinetic, potential, electrostatic, magnetic, geothermal, 1 1
\ KE = mv 2 = ´ 0.12 ´ 29 = 1.74 J
elastic, solar etc. In this chapter, we will discuss only 2 2
mechanical energy. Mechanical, energy consists of kinetic
energy and potential energy. Sample Problem 9 A bob of mass m is suspended by a
light string of length L. It is imparted a horizontal velocity v0 at
ME = KE + PE
the lowest point A, such that it completes a semicircular
Note KE is always positive but PE may be positive or negative. Infact, trajectory in the vertical plane with the string becoming slack
when forces involved are repulsive, PE is positive and when forces only on reaching the topmost point C. The ratio of the kinetic
involved are attractive, PE is negative. æK ö
energies ç B ÷ at B and C is
è KC ø
Kinetic Energy (a) 3 : 1 (b) 1 : 3 (c) 5 : 2 (d) 2 : 5
Kinetic energy (KE) is the capacity of a body to do work by Interpret (a) There are two external C
virtue of its motion. The faster the object moves, the greater forces on the bob, gravity and tension (T) mg
is the kinetic energy. When the object is stationary, its in the string. The latter does not work,
since the displacement of the bob is Tc
kinetic energy is zero.
always normal to the string. The potential B
An object of mass m moving in a reference frame with energy of the bob is thus associate with
velocity v (well below the velocity of light) is said to have the gravitational force only. The total L TA
kinetic energy given by mechanical energy of the system is
1 é1 1 2ù
conserved. We take the potential energy A v0
KE = mv2 2
êë 2 mv = 2 m (| v | ) úû of the system to be zero at the lowest
2 mg
point A. Thus at A
Let us consider a constant force F which when acting on a
1
mass at rest, produces velocity v. If on reaching this E= mv 02
2
velocity, the particle has attained an acceleration a and
mv 02
displacement s, then TA - mg = (Newton’s second law)
L
Telegram @unacademyplusdiscounts

234 JEE Main Physics

where, TA is the tension in the string at A. At the highest point C, Note 18 kmh -1 has been converted to 5 ms -1. At maximum
the string slackers, as the tension in the string (Tc ) becomes zero. compression x m , the potential energy V of the spring is equal to the
Thus, at C kinetic energy K of the moving car from the principle of conservation of
1 mechanical energy.
E = mv c2 + 2 mgL
2
mv c2 Sample Problem 11 A block of mass m = 1kg moving on a
mg = (Newton’s second law) horizontal surface with speed v = 2 ms-1 enters a rough patch
L
where, v c is the speed at C. ranging from x = 0.10 m to x = 2.01m. The retarding force Fr on
5 the block in this range inversely proportional to x over this
E=
mgL range
2
k
Equating this to the energy at A Fr = - for 0.1 < x < 2.01 m
x
5 m
mgL = v 02 =0 for x < 0.1 m and x > 2.01 m
2 2
where, k = 0.5 J. The final kinetic energy of the block as it crosses
v 0 = 5 gL
this patch is
mv c2 (a) 5 J (b) 50 J
Also from equation, mg = , we have
L (c) 0.5 J (d) 500 J
v c = gL
Interpret (c) If Ki and Kf are initial and final kinetic energies
At B, the energy is corresponding to xi and xf , then
1
E=mvB2 + mgL Kf - Ki = ò
xf
F dx
2 xi
Equating this to the energy at A and employing the equation 2.01 ( -k)
v 02 = 5 gL, we have Kf = Ki + ò dx
0.1 x
1 1 5
mvB2 + mgL = mv 02 = mgL 1
2.01
2 2 2 Kf = mvi2 - k ln ( x)
2 0.1
\ vB = 3 gL
The ratio of the kinetic energies at B and C is = 2 - 0.5 ln (20.1)
1 = 2 - 1.5 = 0.5 J
mvB2
KB 2 3
= =
KC 1 mv 2 1 Sample Problem 12 In a ballistic demonstration, a police
2
C
officer fires a bullet of mass 50 g with a speed 200 ms -1 on soft
plywood of thickness 2.00 cm. The bullet emerges with only
Note At point C, the string becomes slack and the velocity of the bob is 10% of its initial kinetic energy. The emergent speed of the
horizontal and to the left. If the connecting string is cut at this instant, the bullet is
bob will execute a projectile motion with horizontal projection in to a rock
(a) 40 ms -1 (b) 63.2 ms -1
kicked horizontally from the edge of a cliff. Otherwise the bob will
continue on its circular path and complete the revolution. (c) 52.3 ms -1 (d) 20 ms -1
Interpret (b) The initial kinetic energy of the bullet is
Sample Problem 10 A car of mass 1000 kg moving with a mv 2
speed 18 km/h on a smooth road and colliding with = 1000 J
2
a horizontally mounted spring of spring constant
6.25 ´ 103 Nm -1. The maximum compression of the spring is It has a final kinetic energy of 0.1 ´ 1000 = 100 J
(a) 1 m (b) 2 m If v f is the emergent speed of the bullet
(c) 3 m (d) 4 m 1
mv f2 = 100 J
2
Interpret (b) At maximum compression, the kinetic energy of
the car is converted entirely into the potential energy of the spring. 2 ´ 100
Þ vf =
0.5
The kinetic energy of the moving car is
1 1 = 63.2 ms–1
K = mv 2 = ´ 10 3 ´ 5 ´ 5
2 2
Note The speed is reduced by approximately 68%.
K = 1.25 ´ 10 4 J
Telegram @unacademyplusdiscounts

Work, Energy and Power 235

Potential Energy The change in gravitational potential energy of particle-


earth system when the particle is at a certain height y is
Potential energy (PE) of a body is the energy stored in the
U - U i = mg (y - y1 )
system of particles in the body by virtue of their position of
configuration in a field. Here, we take U i to be the gravitational potential energy of
the system when it is in a reference configuration in which
The concept of PE exists only for conservative forces, like
the particle is at a reference point yi . Usually, we take
gravitational forces, electrical forces, magnetic forces etc.
U i = 0 and y1 = 0. Doing these changes, we obtain
We cannot define potential energy corresponding to
non-conservative forces like frictional forces. U (y ) = mgy

The change in potential energy (dU ) of a system 3. Electric potential energy


corresponding to a conservative force is
The electric potential energy of two point charges q1 and q2
æ dU ö
dU = - F × ds = - dW çQ F = - ÷ separated by a distance r in vacuum is given by
è ds ø
1 q1q2
Uf s2 U= ×
4pe 0
òU
i
dU = - ò
s1
F × ds r
1 N-m2
or U f - U i = -ò
s2
F × ds Here, = 9.1 ´ 109 = constant
s1 4pe 0 C2

We generally choose the reference point at infinity and Note Work done by a non-conservative force is the sum of change in
assume potential energy to be zero at that point potential energy and change in kinetic energy or work done by a
i. e. , s1 = ¥ and U i = 0 non-conservative force equals the change in value of total mechanical
s energy. Thus,
then U f = - ò F × ds = - W
¥
æ ö æ Total initial ö
Wnet = ç Total final ÷-ç ÷
Thus, potential energy of a body is negative of work done èmechanical energy ø èmechanical energy ø
by the conservative forces in bringing it from infinity to the
Change in potential energy is equal to the negative of work done by the
present position. conservative force ( DU = - DW ). If work done by the conservative force is
Different types of potential energy are given below negative, change in potential energy will be positive and vice-versa. This
can be understood by a simple example. Suppose a ball is taken from the
ground to some height, work done by gravity is negative, i.e., change in
1. Elastic potential energy potential energy should increase or potential energy of the ball will
This type of energy is associated with the state of increase. Which happens so.
compression or extension of an elastic (spring like) object.
If you compress or extend a spring, you do work to change
the relative locations of the coils within the spring. This Ball
work is an increase in the elastic potential energy of the Ground
spring. DWgravity = - ve
The elastic potential energy of the spring-block system is \ DU = + ve (Q DU = - DW )
1 or U f - U i = + ve
U = kx 2 dU
2 F =- , i .e ., conservative forces always act in a
dr
where, k is the spring constant and x is the compression direction where potential energy of the system F
or expansion in spring. If a spring is stretched from initial decreases. This can also be shown in figure. If a ball
position x1 to final position x2, then increase in elastic is dropped from a certain height, the force on it (its
potential energy weight) acts in a direction in which its potential
1 energy decreases.
= k(x22 - x12 )
2
Sample Problem 13 When a body is projected vertically
up, its PE is twice its KE , when it is at a height h above the
2. Gravitational potential energy
ground. At what height will its KE be twice the KE?
This type of energy is associated with the state of h
(a) 2h (b)
separation between objects, which attract one another via 3
the gravitational force. h h
(c) (d)
2 4
Telegram @unacademyplusdiscounts

236 JEE Main Physics

Interpret (c) Total energy, Note This theorem can be applied to non-inertial frames also. In a
E1 = PE+KE non-inertial frame, it can be written as
1 3 3 Work done by all the forces (including the pseudo forces) = change in
= PE+ PE = PE = mgh kinetic energy in non-inertial frame.
2 2 2
E 2 = PE+KE = PE+2PE+3PE = 3 mgh'
Sample Problem 15 A particle of mass 0.5 kg travels in a
As, E 2 = E1
3 h straight line with velocity v = ax3/ 2 , where a = 5m -1/ 2s-1 . The
3mgh'= mgh Þ h' = work done by the net force during its displacement from x = 0 to
2 2
x = 2 m is
Sample Problem 14 The potential energy of diatomic (a) 50 J (b) 45 J
molecule is given by (c) 25 J (c) None of these
A B
U= - Interpret (a) Here, m = 0.5 kg,
r12 r 6
where, r is the distance between the atoms that make up the v = ax3 / 2, a = 5 m–1/2 s-1, W = ?
molecule and A and B are positive constants. Find the Initial velocity, at x = 0, v1 = a ´ 0 = 0
equilibrium separation between the atoms. Final velocity, at x = 2, v 2 = 5 ´ 23 / 2
1/ 6 1/ 6
é Aù é 2A ù Work done = increase in KE
(a) s = ê ú (b) s = ê ú
ë Bû ë Bû =
1
m(v 22 - v12)
1/5 1/5 2
é Bù é 2B ù
(c) s = ê ú (d) s = ê ú 1
ë Aû ë Aû W= ´ 0.5 [(5 ´ 23 / 2) 2 - 0 ] = 50 J
2
Interpret (b) The interaction force between the atoms is
given by Sample Problem 16 A ball of mass m is thrown in air with
dU -12 A 6B
F=- = - éê 13 + 7 ùú speed v1 from a height h and it is caught at a height h2 > h1 when
ds ë s s û
its speed becomes v2. Find the work done on the ball by air
1/ 6
é 2A ù resistance.
At equilibrium F = 0, therefore s = ê ú
ë Bû
Interpret Work done on the ball by gravity is
Wg = - mg(h2 - h1)
Work done on the ball by air resistance is Wair = ?
7.3 Work-Energy Theorem Q Wg + Wair = DKE
This theorem is a very important tool that relates the work 1
Þ -mg (h2 - h1) + Wair = m (v 22 - v12)
to kinetic energy. 2
Accordingly, work done by all the forces (conservative or 1
Þ Wair = mg (h2 - h1) + m (v 22 - v12)
non-conservative, external or internal) acting on a particle 2
or an object is equal to the change in its kinetic energy of
the particle. Thus, we can write
D ==- WKKK f
7.4 Power
We can also write, The time rate of doing work is called power. If an external
K f = Ki + W
force is applied to an object (which we assume as a
Which says that particle), and if the work done by this force is DW in the
æ Kinetic energy after ö time interval Dt, then the average power during this
ç ÷
è the net work is doneø interval is defined as
æ Kinetic energy beforeö æ The net ö DW
P=
=ç ÷+ç ÷ Dt
è the net work done ø è work doneø
The work done on the object contributes to increasing the
These statements are known traditionally as the energy of the object. The general definition of power is the
work-kinetic energy theorem for particles. They hold for time rate of energy transfer. The instantaneous power is
both positive and negative works. If the net work done on the limiting value of the average power as Dt approaches
a particle is positive, then the particle’s kinetic energy zero.
increases by the amount of the work done. If the net work DW dW
done is negative, then the particle’s kinetic energy i.e., P = lim =
Dt ® 0 D t dt
decreases by the amount of the work.
Telegram @unacademyplusdiscounts

Work, Energy and Power 237

where we have represented the infinitesimal value of the Interpret (b) Force required to keep the belt moving = rate of
work done by dW (even though it is not a change and increase of horizontal momentum of sand = mass per second
therefore not differential) dm
´ velocity change = 2 ´ 0.1 = 0.2 N
dW ds é ds ù dt
P=
dt
=F
dt
= F×v ê as, dt = vú Power = Force ´ velocity
ë û
(a) Power is equal to the scalar product of force and = 0.2 ´ 0.1 = 0.02 W
velocity.
(b) Power is a scalar with dimensions [ML2T -3]. The SI unit
of power is Js–1 and is called watt (W) (after James Watt)
7.5 Potential Energy of a Spring
Practical unit of power is horse power (HP)
Consider the situation shown in figure. One end of a
1 HP = 746 W spring is attached to a fixed vertical support and the other
Work end to a block which can move on a horizontal table. Let
(c) Since, Power = , any unit of power multiplied by a
Time x = 0denote the position of the block, when the spring is in
unit of time gives unit of work (or energy), i. e., its natural length. We shall calculate the work done on the
kilowatt-hour or watt-day are units of work or energy block by the spring force as the block moves from x = 0 to
but not of power. x = x1
1 kWh = 103 ´ Js–1 ´ (60 ´ 60 s) = 3.6 ´ 106 J
(d) The slope of work-time curve gives the instantaneous x=0 x = x1
dW
power as P = = tan q [from Fig. (a)] while the area F
dt
dW A A
under P-t curve gives the work done. Since, P = .
dt We have to find the work done during a small interval in
which means W = ò Pdt = area under P-t curve [as shown which the block moves from x to x + dx. The force in this
interval is kx and the displacement is dx. The restoring
in Fig. (b)].
force and displacement are opposite in direction.
Work Power
So, dW = F × ds = | F | | ds| cos 180°
= - | F | | ds| = - kxdx
during this interval. The total work done as the block is
displaced from x = 0 to x = x1 is
θ x
Time dt Time x1 é 1 ù 1
(a) (b) W =ò - kxdx = ê - kx 2 ú
0 ë 2 û0
1 2
Sample Problem 17 An advertisement claims that a =- kx1
2
certain 1200 kg car can accelerate from rest to a speed of
25 ms–1 in a time of 8 s. What average power must the motor If the block moves from x = x1 to x = x2, the limits of
produce to cause this acceleration? (ignore friction) integration are x1 and x2 and the work done is
(a) 45 kW (b) 45.9 kW æ1 1 ö
W = ç kx12 - kx22 ÷ = potential energy
(c) 46.9 kW (d) None of these è2 2 ø
Interpret (c) The work done in accelerating the car is given by
1 1 Note If the block is displaced from x1 to x 2 and brought back to x = x1
W = DK = m(v f2 - v12) = (1200) [(25) 2 - 02 ]
2 2 the work done by the spring force is zero. The work done during the return
journey is negative of the work during the onward journey. The net work
or W = 375 kJ
done by the spring force in a round trip is zero.
W 375
Power = = = 46.9 kW
t 8 Sample Problem 19 A block of mass m has a velocity v0
when it just touches a spring. The block moves through a
Sample Problem 18 Sand drops vertically at the rate of distance l before it stops. The spring constant of spring is k, what
2 kgs–1 on to a conveyor belt moving horizontally with a is the work done on it by the spring force?
velocity of 0.1 ms–1. The extra power needed to keep the belt kl 2 3kl 2
moving is (a) (b)
2 2
(a) 0.05 W (b) 0.02 W
kl 2 3kl 2
(c) 0.06 W (c) 0.03 W (c) - (d) -
2 2
Telegram @unacademyplusdiscounts

238 JEE Main Physics

Interpret (c) Here, the speed of both the blocks are same. Let the speed is v ms–1.
v0 k Since block of 2 kg is coming down hence the gravitational
m potential energy is decreasing while the gravitational potential
energy of 1 kg block is increasing.
The net force acting on the block by the spring is equal to So, kinetic energy of both the blocks will increase.
|Fspring| = kx 1 1
Hence, mBgh = mA gh + mAv 2 + mBv 2
2 2
where, x is the compression in the spring.
1 1
Work done by the spring ò Fspring. ds or 2 ´ 10 ´ 1 = 1 ´ 10 ´ 1 + ´ 1´ v 2 + ´ 2v 2
2 2
= ò Fspring | ds| cos180° or 20 = 10 + 0.5 v 2 + v 2

l -kl 2 or 1.5v 2 = 10
= -ò kxdx = 10
0 2 \ v2 = = 6.67
15
.
or v = 2.58 ms–1
7.6 Conservation of
Sample Problem 21 In the above problem work done by
Mechanical Energy friction in 10 s is equal to
(a) –236.8 J (b) –245.2 J
The mechanical energy E of a system is the sum of its
(c) –246.9 J (d) 246 J
kinetic energy K and its potential energy U.
E = K +U Interpret (c) Work done by the force of friction
When the forces acting on the system are conservative in Wf = f ´ s = - 1.96 ´ 126
nature, the mechanical energy of the system remains = –246.9 J
constant,
K + U = constant
Examples of Conservation of Mechanical
Þ DK + DU = 0
Energy
There are physical situations, where one or more non-
conservative force act on the system but net work done by 1. Object thrown vertically upwards
them is zero, then too the mechanical energy of the system
remains constant. Energy at the lowest point (at A) is only kinetic energy h = 0,
in the middle, energy is both kinetic and potential energy
If S Wnet = 0
(as h = h1 ) and at the highest point, energy is only potential.
Mechanical energy, E = constant. ( as v = 0)
Note If only conservative forces are acting on a system of particles and C v=0
work done by any other external force is zero, then mechanical energy of
the system will remain conserved. In this case, some fraction of
mechanical energy will be decreasing while the other fraction will
increase.
h B v1

Sample Problem 20 In the arrangement


h1
shown in figure, string is light and inextensible
and friction is absent everywhere.
A v
The speed of both the blocks after the block A
has ascend a height of 1 m will be
(a) 2 ms–1 (b) 2.58 ms–1 \ E = K A = K B + UB = UC
(c) 3 ms–1 (c) 3.58 ms–1 1
A B or E = mv2
Given that, mA = 1 kg and 2 kg. 2
1
Interpret (b) Since, there is no friction anywhere, so = mv12 + mgh = mgh
mechanical energy will be conserved. 2
Telegram @unacademyplusdiscounts

Work, Energy and Power 239

2. Freely falling object (a) KC = U A


At the maximum height, total energy is 1
C UC =0 or mvC2 = mgh = mg (1 - cos q1 )
in the form of potential energy. In the 2
middle, total energy is in the form of \ vC = vmax = 2 g (1 - cos q1 )
both kinetic and potential energy. At the (b) U B + KB = U A
lowest point, total energy is in the form B
h v1 or KB = U A - U B = mg (h1 - h2 )
of kinetic energy. 1
or mvB2 = mgl (cos q1 - cos q2 )
\ E = UC = K B + UB = K A h1 2
or \ vB = 2 gl (cos q1 - cos q2 )
1 1 v
E = mgh = mv12 + mgh1 = mv2 A (c) If pendulum of length l is released from horizontal
2 2
position as shown in adjacent figure, then
U A = KB
3. Projectile motion
O
At the highest point, potential energy is maximum and A
2 2
u sin q 1
U H = mgh = mg = mu2 sin2 q
2g 2 l

At the highest point, the kinetic energy will be minimum


but not zero because at the highest point only vertical
component of velocity is zero. B
u
v
1
or mgl = mvB2
H 2
θ \ vB = vmean = 2 gl

1 1
KH = mu2x = mu2 cos2 q
2 2
1
Check Point
Hence, EH = U H + K H = mu2 = Einitial
2 1. Does kinetic energy depend on the direction of motion
Hence, in projectile motion, mechanical energy is involved? Can it be negative? Does it depend on frame of
reference.
conserved.
2. Can kinetic energy of a system be increased or decreased
4. Oscillator without applying any external force on the system?
3. Out of joule, kilowatt, calorie and electron volt, which one is
O
A not the unit of energy?
θ2 E=U 4. The protons are brought towards each other. Will the potential
energy of the systemθdecrease
1
or increase? If a proton and an
l Extreme position electron are brought closer, then?
h1
5. A pump motor is used to deliver water at a certain rate from a
E=U+K given pipe. To obtain n times water from the same pipe in the
h2 B
C
same time by what amount (a) the force and (b) power of
E=K
motor should be increased?

Equilibrium or mean position


Telegram @unacademyplusdiscounts

240 JEE Main Physics

Fig. (a) Let W1, W2, W3 denote the amounts of work done
7.7 Conservative and moving a body from A to B along three different paths, 1, 2,
Non-Conservative Forces 3 respectively. If the force is non-conservative
W1 ¹ W2 ¹ W3.
Conservative Forces 1

A force is said to be conservative, if work done by or


A B
against the force in moving a body depends only on the
initial and final positions of the body and not the nature of
2
path followed between the final and initial positions. This Fig. (b)
means, work done by or against a conservative force is
moving a body over any path between fixed initial and Fig. (b) Shown that a particle moving a closed path.
final positions will be the same. A ® 1, B ® 2 ® A. If W1 is work done in moving the particle
For example, gravitational force, electrostatic force etc., from A ® 1 ® B and W2 is work done in moving the
are conservative forces. particle from B ® 2 ® A, then for a non-conservative force
| W1 | ¹ | W2 |.
In case of gravitational force, if we take work done in
moving the body from A to B, against gravity as negative, \ Net work done along the closed path, A ® B ® A is not
the work done in moving the body from B to A, by gravity zero.
has to be taken as positive, i. e. , ò F × ds ¹ 0
i. e. , WAB = - WBA \ WAB + WBA = 0
In fact, work done is taking the body from A to B is speed in Sample Problem 22 A particle is taken from point A to
the body in the form of PE. This energy which is spent in point B via the path ACB and then come back to point A via the
moving body from B to A. Thus, over the round trip path BDA. What is the work done by gravity on the body over
( A ® B ® A), net work done is zero. this closed path, if the motion of the particle is in the vertical
plane?
Non-Conservative Forces (a) mgh (b) –mgh
1
A force is said to be non-conservative, if work done by or (c) mgh (d) zero
2
against the force in moving a body from one position to
another, depends on the path followed between these two Interpret (d) Here, displacement of the particle is AB, gravity is
acting vertically downwards. The vertical component of AB is h
positions.
upwards. Hence,
For example, force of friction and viscous force are W( ACB) = -mgh
non-conservative forces.
For the path BDA, component of the displacement acting along
2 vertical direction is h (downward)
1 In this case, W(BDA ) = mgh
A B
ACB BDA Total work done W +W =0
Fig. (a)
Telegram @unacademyplusdiscounts

WORKED OUT
Examples
Example 1 A uniform rope of linear density d and length l is Example 5 The work done in time t on a body of mass m
hanging from the edge of a table. The work done in pulling the which is accelerated from rest to a speed v in time t1 as a
rope on the table is function of time t is given by
dgl dgl 2 1 v 2 v 2
(a) (b) (c) dgl 2 (d) d 2gl (a) m t (b) m t
2 2 2 t1 t1
2
Solution Here, mass of rope m = l ´ d 1 æ mv ö 2 1 v2 2
(c) ç ÷ t (d) m t
2 è t1 ø 2 t12
For pulling the rope on the table,
distance of centre of gravity moved = l /2
Solution We know, v = u + at ,v = 0 + at1
l l dgl 2
\Work done = F ´ = ldg ´ = v
2 2 2 or a=
t1
Example 2 A force of (10 $i - 3$j + 6 k$ ) N acts on a body of From second law of motion
v
5 kg and displaces it from A(6 i$ - 5 $j + 3k$ ) m to F = ma = m
t1
B(10 i - 2 $j + 7k$ ) m. The work done is
$ 1 2
Distance travelled from relation s = ut + at
(a) zero (b) 55 J (c) 100 J (d) 221 J 2
1 vt 2 vt 2
Solution Given force F = (10 $i - 3$j + 6k$ ) N, s =0 + =
2 t1 2t1
m = 5 kg mv vt 2 1 mv 2t 2
AB = (10 $i - 2$j + 7k$ ) - (6$i - 5$j + 3k$ ) W =F ´ s ´ ´ =
t1 2t1 2 t12
= ( 4$i - 3$j + 4k$ )
W = F. s = F . AB Example 6 A uniform force of 4 N acts on a body of mass
= (10 $i - 3$j + 6k$ ) × ( 4$i - 3$j + 4 k$ ) 40 kg for a distance of 2.0 m. The kinetic energy acquired by
the body is
= 40 - 9 + 24 = 55 J
(a) 4 ´ 2 J (b) 4 ´ 4 ´ 2 J
Example 3 A ball of mass 5 kg experiences a force (c) 4 ´ 4 ´ 2 ´ 10 8 erg (d) 4 ´ 2 ´ 2 erg
2 2= + .FWork
x x done in displacing the ball by
Solution KE acquired = Work done
(a) 22/3 J (b) 44/3 J (c) 32/3 J (d) 16/3 J
F ´ s = 4 ´2J
x x
Solution Work done, W = ò F dx = ò (2x2 + x) dx
0 0 Example 7 Calculate the KE and PE of the ball half way up ,
é 2x 2 2ù2 when a ball of mass 0.1 kg is thrown vertically upwards with an
x æ16 4 ö 22
=ê + ú =ç + ÷= J initial speed of 20 ms-1.
ë 3 2 û0 è 3 2ø 3
(a) 10 J , 20 J (b) 10 J ,10 J
Example 4 A box is dragged across a floor by a rope which (c) 15 J , 8 J (d) 8 J ,16 J
makes an angle 45° with the horizontal. The tension in the rope Solution Total energy at the time of projection
is 100 N while the box is dragged by 10 m. The work done is 1 1
= mv 2 = ´ 0.1(20) 2 = 20 J
(a) 607.1 J (b) 707.1 J (c) 1414.2 J (d) 900 J 2 2
Solution Work done W = Fs cos q = 100 ´ 10 cos 45° Half way up, PE becomes half the PE at the top
20
100 i.e., PE = = 10 J
= = 707.1J 2
2
\ KE = 20 - 10 = 10 J
Telegram @unacademyplusdiscounts

242 JEE Main Physics

Example 8 A spring is kept compressed by a small cart of \ Decrease in PE of block


mass 150 g. On releasing the cart, it moves with a speed of = increase in KE of block + increase in elastic potential
0.2 ms-1. The potential energy of the spring is energy of spring
1 1
(a) 1 ´ 10 -4 J (b) 6 ´ 10 -3 J (c) 4 ´ 10 -4 J (d) 3 ´ 10 -3 J So, 45 ´ 9.8 ´ 0.012 = ´ 45 ´ v 2 + ´ 1050
2 2
1 [(0.075 + 0.024) 2 - (0.075) 2]
Solution PE of spring = KE of mass = mv 2
2 v = 0.37 ms-1
1 æ 150 ö 2 -3
= ç ÷ (0.2) = 3 ´ 10 J
2 è1000 ø Example 12 A single conservative force F(x) acts on a
1.0 kg particle that moves along the x-axis. The potential energy
Example 9 The potential energy of a certain spring when U(x) is given by U(x) = 20 + ( x - 2) 2 where x is in metre. At x =
stretched through a distance s is 10 J. The amount of work (in J )
5.0 m the particle has kinetic energy of 20 J. What is the
That must be done on this spring to stretch it through an
mechanical energy of the system?
additional distance s will be
(a) 44 J (b) 45 J (c) 48 J (d) 49 J
(a) 30 (b) 40 (c) 10 (d) 20
1 2 Solution Potential energy at x = 5 m is U = 20 + (5 - 2) 2 = 29 J
Solution Potential energy E1 = 10 = ks
2 \ Mechanical energy = KE + PE = 20 + 29 = 49 J
1
E 2 = k ( s + s) 2 = 4 ´ 10 = 40 J
2 Example 13 In the above example, the maximum kinetic
Amount of work required = E 2 - E1 = 40 - 10 = 30 J energy of the particle is
(a) 39 J (b) 29 J
Example 10 An obdect of mass 5 kg falls from rest through (c) 30 J (d) None of these
a vertical distance of 20 m and attains a velocity of 10 ms-1.
How much work is done by the resistance of the air on the Solution Maximum kinetic energy at x = 2 m
object? ( g = 10 ms-2) where PE is minimum and the maximum kinetic energy is
(a) 750 J (b) - 750 J (c) 850 J (d) - 650 J Kmax = E - Umin = 49 - 20 = 29 J

Solution Applying work-energy theorem Example 14 When a belt moves horizontally at a constant
Work done ball the forces = Change in kinetic energy speed of 1.5 ms-1, gravel is falling on it at 5 kgs -1. Then, the
1 extra power needed to drive the belt is
or Wmg + Wair = mv 2
2 (a) 11.25 W (b) 37.5 W
1 1 (c) 7.5 W (d) 0.75 W
\ Wair = mv 2 - Wmg = mv 2 - mgh
2 2
Solution Here, v = 1.5 ms-1
1 2
= ´ 5 ´ (10) - 5 ´ 10 ´ 20 = - 750 J dm dm
2 = 5 kg ms-1,F = ´ v = 5 ´ 1.5 = 7.5 N
dt dt
P = F ´ v = 7.5 ´ 1.5 = 11.25 W
Example 11 The system is released from rest with the
spring initially stretched 75 mm. Calculate the velocity of the
block after it has dropped 12 mm. The spring has a stiffness of Example 15 A machine gun fires 360 bullets per minute,
1050 Nm -1. Neglect the mass of the small pulley. with a velocity of 600 ms-1. If the power of the gun is 5.4 kW.
(a) 0.371 ms-1 (b) 0.45 ms-1 (c) 5 ms-1 (d) 2.2 ms-1
mass of each bullet is
(a) 5 kg (b) 0.5 kg (c) 5 g (d) 0.5 g
Solution When the block descends 12 mm, spring further
Solution Here, n =
360
= 6 bullets s -1
stretches by 24 mm 60
v = 600 ms-1, m = ?
Power of gun = Power of bullets
1
5.4 ´ 10 3 = (nm) v 2
2
2 ´ 5400 = 6 ´ m(600) 2
2 ´ 5400
or m= kg
6 ´ 600 ´ 600
45 kg 1 1000
= kg = g =5g
200 200
Telegram @unacademyplusdiscounts

Start Practice for


JEE Main
Round I (Topically Divided Problems)

Work 6. A 5 kg brick of 20 cm × 10 cm × 8 cm dimensionless


1. Under the action of a force, a 2 kg body moves such lying on the largest base. It is now made to stand
that its position x as a function of time t is given by with length vertical. If g =10 ms -2 , then the amount
x = t 3 / 3, where x is in metre and t in second. The of work done is
work done by the force in the first two seconds is (a) 3 J (b) 5 J (c) 7 J (d) 9 J
(a) 1.6 J (b) 16 J 7. A block of mass 10 kg slides down a rough slope
(c) 160 J (d) 1600 J which is inclined at 45° to the horizontal. The
2. The work done in pulling up a block of wood coefficient of sliding friction is 0.30. When the block has
weighing 2 kN for a length of 10 m on a smooth plane slide 5 m, the work done on the block by the force of
inclined at an angle of 15° with the horizontal is friction is nearly
[sin 15° = 0.2588] (a) 115 J (b) 75 2 J
(a) 4.36 kJ (b) 5.13 kJ (c) 321.4 J (d) –321.4 J
(c) 8.91 kJ (d) 9.82 kJ 8. In a children’s park, there is a slide which has a total
3. A mass M is lowered with the help of a string by a length of 10 m and a height of 8.0 m. A vertical ladder
distance h at a constant acceleration g/2. The work is provided to reach the top. A boy weighing 200 N
done by the string will be climbs up the ladder to the top of the slide and slides
Mgh -Mgh 3Mgh -3Mgh down to the ground. The average friction offered by
(a) (b) (c) (d) the slide is three-tenth of his weight. The work done
2 2 2 2
by the slide on the boy as he comes down is
4. An electron and a proton are moving under the
influence of mutual forces. In calculating the change
in the kinetic energy of the system during motion,
one ignores the magnetic force of one on another.
This is because, [NCERT Exemplar]
(a) the two magnetic forces are equal and opposite, so they
produce no net effect
(b) the magnetic forces do no work on each particle
(a) zero (b) + 600 J (c) –600 J (d) +1600 J
(c) the magnetic forces do equal and opposite (but
non-zero) work on each particle 9. A proton is kept at rest. A positively charged particle
(d) the magnetic forces are necessarily negligible is released from rest at a distance d in its field.
5. A ball of mass 0.2 kg is thrown vertically upwards by Consider two experiments; one in which the charged
applying a force by hand. If the hand moves 0.2 m particle is also a proton and in another, a positron. In
while applying the force and the ball goes upto 2 m the same time t, the work done on the two moving
height further. Find the magnitude of force charged particles is [NCERT Exemplar]
(Consider g = 10 m / s2 ) (a) same as the same force law is involved in the experiments
(a) 22 N (b) 4 N (b) less for the case of a positron, as the positron moves away
(c) 16 N (d) 20 N more rapidly and the force on it weakness.
Telegram @unacademyplusdiscounts

244 JEE Main Physics

(c) more for the case of a positron, as the positron moves 17. A 5 kg stone of relative density 3 is resting at the bed
away a larger distance of a lake. It is lifted through a height of 5 m in the
(d) same as the work done by charged particle on the lake. If g = 10 ms–2, then the work done is
stationary proton 500 350
(a) J (b) J
10. A man squatting on the ground gets straight up and 3 3
stand. The force of reaction of ground on the man 750
(c) J (d) zero
during the process is [NCERT Exemplar] 3
(a) constant and equal to mg in magnitude 18. A force acts on a 30 g particle in such a way that the
(b) constant and greater than mg in magnitude position of the particle as function of time is given by
(c) variable but always greater than mg x = 3 t - 4 t2 + t 3, where x is in metre and t is in
(d) at first greater than mg, and later becomes equal to mg second. The work done during the first 4 seconds is
11. A ball is released from the top of a tower. The ratio of (a) 5.28 J (b) 450 mJ
work done by force of gravity in Ist second, 2nd (c) 490 mJ (d) 530 mJ
second and 3rd second of the motion of ball is 19. A car weighing 1400 kg is moving at a speed of
(a) 1 : 2 : 3 (b) 1 : 4 : 16 54 kmh–1 up a hill when the motor stops. If it is just
(c) 1 : 3 : 5 (d) 1 : 9 : 25 able to reach the destination which is at a height of
12. A plate of mass m, length b and breadth a is initially 10 m above the point, then the work done against
lying on a horizontal floor with length parallel to the friction (negative of the work done by the friction) is
floor and breadth perpendicular to the floor. The (Take g = 10 ms–2)
work done to erect it on its breadth is (a) 10 kJ (b) 15 kJ
é bù é bù (c) 17.5 kJ (d) 25 kJ
(a) mg (b) mg a +
êë 2 úû êë 2 úû 20. A cord is used to lower vertically a block of mass M by
é b - aù éb+ aù a distance d with constant downward acceleration
(c) mg (d) mg
êë 2 úû êë 2 úû g / 4 work done by the cord on the block is
d
13. The displacement x in metre of a particle of mass m kg (a) Mg
4
moving in one dimension under the action of a force is d
related to the time t in second by the equation (b) 3 Mg
4
t = x + 3 , the work done by the force (in joule) in first d
six seconds is (c) -3 Mg
4
(a) 18 m (b) zero (d) Mgd
(c) 9 m/2 (d) 36 m
21. Water is drawn from a well in a 5 kg drum of capacity
14. A position-dependent force F = 3x2 – 2x + 7 acts on a 55 L by two ropes connected to the top of the drum.
body of mass 7 kg and displaces it from x = 0 m to x = 5m. The linear mass density of each rope is 0.5 kgm–1.
The work done on the body is x′ joule. If both F and x The work done in lifting water to the ground from the
are measured in SI units, the value of x′ is surface of water in the well 20 m below is (g = 10 ms–2)
(a) 135 (b) 235 (c) 335 (d) 935 (a) 1.4 ´ 10 4 J (b) 1.5 ´ 10 4 J
15. A body of mass 0.5 kg travels in a straight line with (c) 9.8 ´ 10 ´ 6 J (d) 18 J
velocity, v = ax 3/ 2 , where a = 5 m -1/ 2 /s. What is the
22. A wire of length L suspended vertically from a rigid
work done by the net force during its displacement
support is made to suffer extension l in its length by
from x = 0 to x = 2 m? [NCERT]
applying a force F. The work is
(a) 30 J (b) 40 J (c) 20 J (d) 50 J Fl
(a) (b) Fl
16. A uniform chain of length L and mass M overhangs a 2
horizontal table with its two-third part on the table. (c) 2 Fl (d) Fl
The friction coefficient between the table and the 23. A bicyclist comes to a skidding stop in 10 m. During
chain is m. The work done by the friction during the this process, the force on the bicycle due to the road is
period the chain slips off the table is 200 N and is directly opposed to the motion. The work
1 2
(a) - mMgL (b) - mMgL done by the cycle on the road is [NCERT Exemplar]
4 9
4 6 (a) + 2000 J (b) - 200 J
(c) - m MgL (d) - mMgL (c) zero (d) - 20,000 J
9 7
Telegram @unacademyplusdiscounts

Work, Energy and Power 245

24. A uniform chain of length L and mass M is lying on a 30. The relationship between force and position is shown
smooth table and one third of its length is hanging in figure given (in one dimensional case). The work
vertically down over the edge of the table. If g is done by the force is displaying a body from x = 1 cm to
acceleration due to gravity, the work required to pull x = 5 cm is
the hanging part on the the table is
(a) MgL (b) MgL/3 20
(c) MgL/9 (d) MgL/18

Force (dyne)
10
25. During inelastic collision between two bodies, which of 0
the following quantities always remain conserved? 1 2 3 4 5 6
–10 x (cm)
[NCERT Exemplar]
(a) Total kinetic energy (b) Total mechanical energy –20
(c) Total linear momentum (d) Speed of each body
26. A spring of spring constant 5 ×103 Nm–1 is stretched (a) 20 erg (b) 60 erg
initially by 5 cm from the unstretched position. Then (c) 70 erg (d) 700 erg
the work required to stretch it further by another 31. A 10 kg brick moves along an x-axis. Its acceleration
5 cm is as a function of its position is shown in figure. What
(a) 12.50 N-m (b) 18.75 N-m is the net work performed on the brick by the force
(c) 25.00 N-m (d) 6.25 N-m causing the acceleration as the brick moves from x = 0
to x = 8.0 m ?
27. A rod AB of mass 10 kg and length 4 m rests on a
horizontal floor with end A fixed so as to rotate it in
vertical. Work done on the rod is 100 J. The height to 20
which the end B be raised vertically above the floor is (ms–2 ) 15
(a) 1.5 m (b) 2.0 m 10
(c) 1.0 m (d) 2.5 m
5
28. Two inclined frictionless tracks, one gradual and the 0
other steep meet at A from where two stones are 1 2 3 4 5 6 7 8
x (m)
allowed to slide down from rest, one on each track as
shown in figure.
A (a) 4 J (b) 8 J
(c) 2 J (d) 1 J
I
II 32. A 2.0 kg block is dropped from a height of 40 cm onto
h a spring of spring constant k = 1960 Nm–1. Find the
maximum distance the spring is compressed.
θ1 θ2 (a) 0.080 m
B
C (b) 0.20 m
Which of the following statement is correct? (c) 0.40 m
[NCERT Exemplar]
(d) 0.10 m the stones reach the bottom at the same time but
(a) Both 33. The graph between the resistive force F acting on a
not with the same speed body and the distance covered by the body is shown in
(b) Both the stones reach the bottom with the same speed the figure. The mass of the body is 2.5 kg and initial
and stone I reaches the bottom earlier than stone II velocity is 2 m/s. When the distance covered by the
(c) Both the stones reach the bottom with the same speed body is 4 m, its kinetic energy would be
and stone II reaches the bottom earlier than stone I
(d) Both the stones reach the bottom at different times and
F (newton)

with different speeds 20

29. A force F = Ay2 + By + C acts on a body in the 10


y-direction. The work done by this force during a
displacement from y = – a to y = a is x (cm)
0 1 2 3 4
2 Aa5 2 Aa5
(a) (b) + 2 Ca
3 3
(a) 50 J (b) 40 J
2 Aa5 Ba2
(c) + + Ca (d) None of these (c) 20 J (d) 10 J
3 2
Telegram @unacademyplusdiscounts

246 JEE Main Physics

34. A stone tied to a string of length L is whirled in a 39. A body of mass 0.5 kg travels in a straight line with
vertical circle with the other end of the string at the velocity v = a x 3/ 2 where a = 5m -1/2s -1 . The work
centre. At a certain instant of time, the stone is at its done by the net force during its displacement from
lowest position and has a speed u. The magnitude of x = 0 to x = 2 m is [NCERT Exemplar]
the change in its velocity as it reaches a position where (a) 1.5 J (b) 50 J (c) 10 J (d) 100 J
the string is horizontal is
40. When a man increases his speed by 2 ms–1, he finds
(a) u2 - 2gl (b) 2gl that his kinetic energy is doubled, the original speed
(c) u2 - gl (d) 2 (u2 - gL) of the man is
(a) 2 ( 2 - 1) ms -1 (b) 2 ( 2 + 1) ms -1
35. The potential energy function for a particle executing
1 2 (c) 4.5 ms–1 (d) None of these
linear SHM is given by V ( x) = kx where k is the
2 41. A 0.5 kg ball is thrown up with an initial speed 14 m/s
force constant of the oscillator. For k = 0.5 N/m , the and reaches a maximum height of 8.0 cm. How much
graph of V ( x) versus x is shown in the figure. A energy is dissipated by air drag acting on the ball
particle of total energy E turns back when it reaches during the time of ascent?
x = ± xm . If V and K indicate the PE and KE (a) 19.6 J (b) 4.9 J
respectively of the particle at x = ± xm then which of (c) 10 J (d) 9.8 J
the following is correct? [NCERT Exemplar]
42. A body is moving unidirectionally under the
V(x) influence of a source of constant power supplying
energy. Which of the diagrams shown in figure
correctly shows the displacement-time curve for its
motion? [NCERT Exemplar]

x d d
–xm xm

(a) V = O, K = E (b) V = E, K = O (a) (b)


(c) V < E, K = O (d) V = O, K < O
36. An elastic string of unstretched length L and force
t t
constant k is stretched by a small length x. It is
further stretched by another small length y. The d d
work done in the second stretching is
1 2 1 2 (d)
(a) ky (b) k( x + y2 ) (c)
2 2
1 1
(c) k( x + y )2 (d) ky (2x + y )
2 2 t t

43. A stone of mass 2 kg is projected upward with kinetic


Energy energy of 98 J. The height at which the kinetic energy
37. A ball is projected vertically upwards with a certain of the body becomes half its original value, is given by
initial speed. Another ball of the same mass is (Take g = 10 ms–2)
projected at an angle of 60° with the vertical with the (a) 5 m (b) 2.5 m
same initial speed. At highest point of their journey, (c) 1.5 m (d) 0.5 m
the ratio of their potential energies will be 44. A ball whose kinetic energy is E, is projected at an
(a) 1 : 1 (b) 2 : 1 angle 45° to the horizontal. The kinetic energy of the
(c) 3 : 2 (d) 4 : 1 ball at the highest point of its flight will be
38. The kinetic energy K of a particle moving in straight (a) E (b)
E
(c)
E
(d) zero
line depends upon the distance s as 2 2
K = as2 45. A body is falling freely under the action of gravity
The force acting on the particle is alone in vacuum. Which of the following quantities
(a) 2 as (b) 2 mas
remain constant during the fall? [NCERT Exemplar]
(a) Kinetic energy (b) Potential energy
(c) 2a (d) as2
(c) Total mechanical energy (d) Total linear energy
Telegram @unacademyplusdiscounts

Work, Energy and Power 247

46. The potential energy as a function of the force 52. Which of the diagrams shown in figure represents
between two atoms in a diatomic molecules is given variation of total mechanical energy of a pendulum
a b oscillating in air as function of time? [NCERT Exemplar]
by U( x) = 12 - 6 , where a and b are positive
x x
constants and x is the distance between the atoms. E E
The position of stable equilibrium for the system of
the two atoms is given (a) (b)
t
a a
(a) x = (b) x = t
b b
3a æ2 aö E
(c) x = (d) x = 6 ç ÷
b è bø E

47. The potential energy of a particle of mass 5 kg (c) (d)


moving in the xy-plane is given by U = ( -7x + 24 y) J, x t t
and y being in metre. If the particle starts from rest
from origin, then speed of particle at t = 2 s is
(a) 5 ms–1 (b) 01 ms–1
53. A 50 g bullet moving with a velocity of 10 ms–1 gets
embeded into a 950 g stationary body. The loss in kinetic
(c) 17.5 ms–1 (d) 10 ms–1
energy of the system will be
48. A running man has half the kinetic energy of that of a (a) 95% (b) 100% (c) 5% (d) 50%
boy of half of his mass. The man speeds up by 1 m/s,
so as to have same kinetic energy as that of the boy. 54. A car is moving with a speed of 100 kmh–1. If the
The original speed of the man will be mass of the car is 950 kg, then its kinetic energy is
(a) 0.367 M J (b) 3.67 J
(a) 2 m/s (b) 2 - 1 m/s
1 1 (c) 3.67 M J (d) 367 J
(c) m/s (d) m/s
2 -1 2 55. A simple pendulum is released A
from A as shown. If M and l
49. If a body looses half of its velocity on penetrating 3 cm
represent the mass of the bob and 30°
in a wooden block, then how much will it penetrate
length of the pendulum
more before coming to rest?
respectively, the gain in kinetic
(a) 1 cm (b) 2 cm B
energy at B is
(c) 3 cm (d) 4 cm
50. In the given curved road, if particle is released from Mgl 3
(a) (b) Mgl
A, then 2 2
Mgl 2
M A (c) (d) mgl
2 3
56. Two masses of 1 g and 4 g are moving with equal
h kinetic energies. The ratio of the magnitudes of their
linear momenta is
(a) 4 : 1 (b) 2 : 1
B
(c) 1 : 2 (d) 1 : 16
(a) kinetic energy at B must be mgh
(b) kinetic energy at B may be zero 57. A mass of 5 kg is moving along a circular path of
(c) kinetic energy at B must be less than mgh radius 1 m. If the mass moves with 300 revolutions
(d) kinetic energy at B must not be equal to zero per minute, its kinetic energy would be
[NCERT Exemplar]
51. Two springs have force constants k1 and k2 . These are (a) 250 p2 (b) 100 p2 (c) 5 p2 (d) 0
extended through the same distance x. If their elastic
E 58. A body of mass 2 kg is thrown up vertically with
energies are E1 and E2 , then 1 is equal to
E2 kinetic energy of 490 J. The height at which the
(a) k1 : k2 (b) k2 : k1 kinetic energy of the body becomes half of its original
value is
(c) k1 : k2 (d) k12 : k22
(a) 50 m (b) 12.25 m (c) 25 m (d) 10 m
Telegram @unacademyplusdiscounts

248 JEE Main Physics

59. In a shotput event an athlete throws the shotput of 66. A bomb of mass 9 kg explodes into 2 pieces of mass
mass 10 kg with an inital speed of 1 m s -1 at 45° from 3 kg and 6 kg. The velocity of mass 3 kg is 1.6 m/s, the
a height 1.5 m above ground. Assuming air kinetic energy of mass 6 kg is
resistance to be negligible and acceleration due to (a) 3.84 J (b) 9.6 J
gravity to be 10 ms -2 , the kinetic energy of the (c) 1.92 J (d) 2.92 J
shotput when it just reaches the ground will be 67. An engine pumps water continuously through a hole.
[NCERT Exemplar] Speed with which water passes through the hole
(a) 2.5 J (b) 5.0 J nozzle is v and k is the mass per unit length of the
(c) 52.5 J (d) 155.0 J water jet as it leaves the nozzle. Find the rate at
60. A machine which is 75% efficient uses 12 J of energy which kinetic energy is being imparted to the water.
in lifting up a 1 kg mass through a certain distance. 1 2 1 3 v2 v3
(a) kv (b) kv (c) (d)
The mass is then allowed to fall through that 2 2 2k 2k
distance. The velocity of the ball at the end of its 68. In the stable equilibrium position, a body has
fall is (a) maximum potential energy
(a) 24 ms -1 (b) 32 ms -1 (b) minimum potential energy
(c) 18 ms -1 (d) 3 ms–1 (c) minimum kinetic energy
(d) maximum kinetic energy
61. A body of mass 4 kg is moving with momentum of
8 kg-ms–1. A force of 0.2 N acts on it in the direction of 69. A stone is dropped from the top of a tall tower. The
motion of the body for 10 s. The increase in kinetic ratio of the kinetic energy of the stone at the end of
energy in joule is three seconds to the increase in the kinetic energy of
the stone during the next three seconds is
(a) 10 (b) 8.5 (c) 4.5 (d) 4
(a) 1 : 1 (b) 1 : 2 (c) 1 : 3 (d) 1 : 9
62. A body of mass M is dropped from a height h on a 70. A rectangular plank of mass m1 and height a is kept
sand floor. If the body penetrates x cm into the sand, on a horizontal surface. Another rectangular plank of
the average resistance offered by the sand to the body mass m2 and height b is placed over the first plank.
is The gravitational potential energy of the system is
æ hö æ hö é æ m + m2 bö ù
(a) Mg ç ÷ (b) Mg ç1 + ÷ (a) [ m1 + m2 ( a + b)] g (b) ê ç 1 a + m2 ÷ ú g
è xø è xø
ëè 2 2øû
æ hö
(c) Mgh + Mgx (d) Mg ç1 - ÷ éæ m ö bù éæ m ö bù
è xø (c) ê ç 1 + m2 ÷ a + m2 ú g (d) ê ç 1 + m2 ÷ a + m1 ú g
è
ë 2 ø 2û è
ë 2 ø 2û
63. A mass of 50 kg is raised through a certain height by a
machine whose efficiency is 90%, the energy is 5000 J.
If the mass is now released, its kinetic energy on hitting
Power
the ground shall be 71. A body is initially at rest. It undergoes
(a) 5000 J (b) 4500 J (c) 4000 J (d) 5500 J one-dimensional motion with constant acceleration.
The power delivered to it at time t is proportional to
64. Given that the position of the body in metre is a
[NCERT]
function of time as follows /
12
(a) t (b) t
x = 2 t4 + 5 t + 4 (c) t32/ (d) t2
The mass of the body is 2 kg. What is the increase in 72. A 10 m long iron chain of linear mass density
its kinetic energy, one second after the start of 0.8 kg m –1 is hanging freely from a rigid support. If
motion? g = 10 ms–2, then the power required to lift the chain
(a) 168 J (b) 169 J upto the point of support in 10 s is
(c) 32 J (d) 144 J (a) 10 W (b) 20 W
65. A bomb of mass 3.0 kg explodes in air into two pieces (c) 30 W (d) 40 W
of mass 2.0 kg and 1.0 kg. The smaller mass goes at a 73. A 10 HP motor pump out water from a well of depth
speed of 80 m/s. The total energy imparted to the two 20 m and falls a water tank of volume 22380 litre at a
fragment is height of 10 m from the ground the running time of
(a) 1.07 kJ (b) 2.14 kJ the motor to fill the empty water tank is ( g = 10 ms -2 )
(c) 2.4 kJ (d) 4.8 kJ (a) 5 min (b) 10 min
(c) 15 min (d) 20 min
Telegram @unacademyplusdiscounts

Work, Energy and Power 249

74. An engine of power 7500 W makes a train move on a 80. A 500 kg car, moving with a velocity of 36 kmh–1 on a
horizontal surface with constant velocity of 20 ms–1 . straight road unidirectionally, doubles its velocity in
The force involved in the problem is one minute. The power delivered by the engine for
(a) 375 N (b) 400 N doubling the velocity is
(c) 500 N (d) 600 N (a) 750 W (b) 1050 W
75. A one kilowatt motor is used to pump water from a (c) 1150 W (d) 1250 W
well 10 m deep. The quantity of water pumped out 81. The power of a water jet flowing through an orifice of
per second is nearly radius r with velocity v is
(a) 1 kg (b) 10 kg (a) zero (b) 500 pr2 v2
(c) 100 kg (d) 1000 kg (c) 500 pr2 v3 (d) pr 4 v
76. A car manufacturer claims that his car can be 82. A motor of power P0 is used to deliver water at a
accelerated from rest to a velocity of 10 ms–1 in 5 s. If certain rate through a given horizontal pipe. To
the total mass of the car and its occupants is 1000 kg, increase the rate of flow of water through the
then the average horse power developed by the same pipe n times. The power of the motor is
engine is increassed to p1. The ratio of p1 to p0 is
103 10 4 (a) n : 1 (b) n2 : 1
(a) (b)
746 746 (c) n3 : 1 (d) n 4 : 1
105
(c) (d) 8 83. A quarter horse power motor runs at a speed of
746
600 rpm. Assuming 40% efficiency, the work done by
77. Which of the diagrams in figure correctly shows the the motor in one rotation will be
change in kinetic energy of an iron sphere falling (a) 7.46 J (b) 7400 J
freely in a lake having sufficient depth to impart it a (c) 7.46 erg (d) 74.6 J
terminal velocity? [NCERT Exemplar]
84. Ten litre of water per second is lifted from well
through 20 m and delivered with a velocity of
10 ms–1, then the power of the motor is
KE KE (a) 1.5 kW (b) 2.5 kW (c) 3.5 kW (d) 4.5 kW
(a) (b)
85. A body is moved along a straight line by machine
delivering a constant power. The distance moved by
Depth Depth
the body in time t is proportional to
(a) t3/ 4 (b) t3/2 (c) t1/4 (d) t1 /2

(d) KE
86. One man takes 1 minute to raise a box to a height of
(c) KE 1
1 m and another man takes minute to do so. The
2
energy of the two is
Depth Depth
(a) different
78. A dam is situated at a height of 550 m above sea level (b) same
and supplies water to a power house which is at a (c) energy of the first is more
height of 50 m above sea level. 2000 kg of water (d) energy of the second is more
passes through the turbines per second. What would 87. The power supplied by a force acting on a particle
be the maximum electrical power output of the power moving in a straight line is constant. The velocity of
house if the whole system were 80% efficient? the particle varies with the displacement x as
(a) 8 MW (b) 10 MW (a) x1 /2 (b) x (c) x2 (d) x1/3
(c) 12.5 MW (d) 16 MW
88. A particle of mass m is moving in a circular path of
79. An automobile weighing 1200 kg climbs up a hill that constant radius r such that its centripetal
rises 1 m in 20 s. Neglecting frictional effects, the acceleration a c is varying with time t as ac = k2 rt2 .
minimum power developed by the engine is 9000 W. The power is
If g = 10 ms -2 , then the velocity of the automobile is (a) 2 pmk2 r2t (b) mk2 r2t
(a) 36 kmh–1 (b) 54 kmh–1
mk 4 r2t5
(c) 72 kmh–1 (d) 90 kmh–1 (c) (d) zero
3
Telegram @unacademyplusdiscounts

250 JEE Main Physics

Round II (Mixed Bag)

Only One Correct Option 8. A particle is released from a height s. At certain


1. The bob of a pendulum is released from a horizontal height its kinetic energy is three times its potential
position A as shown in the figure. If the length of the energy. The height and speed of the particle at that
pendulum is 1.5 m, then the speed with which the instant are respectively
bob arrives at the lower most point B. Given that it s 3 gs s 3 gs
(a) , (b) ,
dissipated 5% of its initial energy against air 4 2 4 2
resistance? [NCERT] s 3 gs s 3 gs
(c) , (d) ,
(a) 6.0 m/s (b) 6.5 m/s (c) 4.5 m/s (d) 5.3 m/s 2 2 4 2
2. If a man speeds up by 1 ms–1,
his kinetic energy 9. A body of mass 3 kg acted upon by a constant force is
1 2
increases by 44%. His original speed in ms–1 is displaced by s metre, given by relation s = t , where
(a) 1 (b) 2 (c) 5 (d) 4 3
t is in second. Work done by the force in 2 s
3. A particle moves in a straight line with retardation 8 19
proportional to its displacement. Its loss of kinetic (a) J (b) J
3 5
energy for any displacement x is proportional to 5 3
(c) J (d) J
(a) x (b) x2 (c) x0 (d) e x 19 8
4. A bullet fired from a gun with a velocity of 10. A body of mass 3 kg is under a force which causes a
104 ms–1goes through a bag full of straw. If the bullet 3
displacement is given by s = t (in m). Find the work
loses half of its kinetic energy in the bag, its velocity 3
when it comes out of the bag will be done by the force in first 2 seconds.
(a) 7071.06 ms–1 (b) 707 ms–1 (a) 2 J (b) 3.8 J (c) 5.2 J (d) 24 J
(c) 70.71 ms–1 (d) 707.06 ms–1
11. A gun of mass 20 kg has bullet of mass 0.1 kg in it.
5. A motor drives a body along a straight line with a The gun is free to recoil 804 J of recoil energy
constant force. The power P developed by the motor are released on firing the gun. The speed of bullet
must vary with time t as shown in figure (ms–1) is
2010
P P (a) 804 ´ 2010 (b)
804
(a) (b)
804
(c) (d) 804 ´ 4 ´ 103
2010
t t
12. Power supplied to a particle of mass 2 kg varies with
3 t2
P P time as P = watt. Here t is in second. If the
(c) (d) 2
velocity of particle at t = 0 is v = 0, the velocity of
particle at time t = 2 s will be
t t
(a) 1 ms–1 (b) 4 ms–1
6. A bob of mass m accelerates uniformly from rest to v1 (c) 2 ms–1 (d) 2 2 ms -1
in time t1. As a function of t, the instantaneous power
13. A body of mass 3 kg is under a force which causes a
delivered to the body is
displacement in it, given by s = t2 / 3 (in m). Find the
mv1t mv1t mv1t2 mv2t
(a) (b) (c) (d) 2 1 work done by the force in 2 s
t2 t1 t1 t1
(a) 2 J (b) 3.8 J (c) 5.2 J (d) 2.6 J
7. Two blocks of mass m each are connected to a spring 14. Power supplied to a particle of mass 2 kg varies with
of spring constant k as shown in figure. The time as P = t2 /2 watt, where t is in second. If velocity of
maximum displacement in the block is particle at t = 0 is v = 0, the velocity of particle at t = 2 s
k
m m will be
v v
(a) 1 ms–1 (b) 4 ms–1
2
2 mv mv2
mv2
k 2
(a) (b) (c) 2 (d) 2 (c) 2 ms–1 (d) 2 2 ms -1
k k k mv2 3
Telegram @unacademyplusdiscounts

Work, Energy and Power 251

15. Power applied to a particle varies with time as 21. The potential energy of a certain spring when
P = (3 t2 - 2 t + 1) watt, where t is in second. Find the stretched through a distance s is 10 J. The amount of
change in its kinetic energy between t = 2 s and t = 4 s. work (in joule) that must be done on this spring to
(a) 32 J (b) 46 J (c) 61 J (d) 100 J stretch it through additional distance s will be
16. A car of mass 1000 kg moves at a constant speed of (a) 30 (b) 40 (c) 10 (d) 20
20 ms–1 up an incline. Assume that the frictional 22. A bullet when fired at a target with velocity of
force is 200 N and that sin q = 1/20, where, q is the 100 ms–1 penetrates 1 m into it. If the bullet is fired
angle of the incline to the horizontal. The g = 10 ms–2. at a similar target with a thickness 0.5 m, then it will
Find the power developed by the engine? emerge from it with a velocity of
(a) 14 kW (b) 4 kW 50
(c) 10 kW (d) 28 kW (a) 50 2 m/s (b) m/s
2
17. The human heart discharges 75 cc of blood through (c) 50 m/s (d) 10 m/s
the arteries at each beat against an average pressure
of 10 cm of mercury. Assuming that the pulse
23. Velocity-time graph of a particle of mass 2 kg moving
in a straight line is as shown in figure. Work done by
frequency is 72 per minute the rate of working of
all forces on the particle is
heart in watt, is (Density of mercury = 13.6 g/cc and
g = 9.8 ms–2) 20
(a) 11.9 (b) 1.19
(c) 0.119 (d) 119 v (m/s)
18. A particle of mass 2 kg starts moving in a straight
line with an initial velocity of 2 ms–1 at a constant
acceleration of 2 ms–2. Then rate of change of kinetic t(s) 2
energy
(a) is four times the velocity at any moment (a) 400 J (b) – 400 J
(b) is two times the displacement at any moment (c) – 200 J (d) 200 J
(c) is four times the rate of change of velocity at any moment 24. A particle moves on a rough horizontal ground with
(d) is constant through out some initial velocity v0 . If 3 th of its kinetic energy is
19. The potential energy of a system represent in the 4
first figure. The force acting on the system will be lost due to friction in time t0 , the coefficient of friction
represent by between the particle and the ground is
v0 v0 3 v0 v0
f (x) (a) (b) (c) (d)
2 gt 0 4 gt 0 4 gt 0 gt 0

25. A box of mass 50 kg is pulled up on an incline 12 m


x long and 2 m high by a constant force of 100 N from
O a
rest. It acquires a velocity of 2 ms–1 on reaching the
f (x) f (x) top. Work done against friction (g = 10 ms–2) is
(a) 50 J (b) 100 J (c) 150 J (d) 200 J
a a
(a) x (b) x 26. A man running has half the kinetic energy of a boy of
half his mass. The man speeds up by 1 ms–1 and then
has kinetic energy as that of the boy. What were the
f (x) f (x)
original speeds of man and the boy?
(a) 2 ms -1; 2 2 - 1 ms -1

(c)
a
x x (b) ( 2 - 1) ms -1, 2( 2 - 1) ms -1
(d) a
(c) ( 2 + 1) ms -1; 2( 2 + 1) ms -1
(d) None of the above
27. An engine pumps up 100 kg of water through a
20. A ball is dropped from a height of 20 cm. Ball height of 10 m in 5 s. Given that the efficiency of the
rebounds to a height of 10 cm. What is the loss of engine is 60%. If g = 10 m/s2 , the power of the engine
energy? is
(a) 25% (b) 75% (c) 50% (d) 100% (a) 3.3 kW (b) 0.33 kW (c) 0.033 kW (d) 33 kW
Telegram @unacademyplusdiscounts

252 JEE Main Physics

28. An ideal spring with spring constant k is hung from 34. Two blocks M1 and M2 having equal mass are to
the ceiling and a block of mass M is attached to its move on a horizontal frictionless surface. M2 is
lower end. The mass is released with the spring attached to a massless spring as shown in figure.
initially unstretched. Then the maximum extension Initially M2 is at rest and M1 is moving toward M2
in the spring is with speed v and collides head-on with M2 .
4 Mg 2 Mg [NCERT Exemplar]
(a) (b)
k k M1 = m M2 = m
Mg Mg
(c) (d) v
k 2k

29. A 0.5 kg ball is thrown up with an initial speed


14 ms -1 and reaches a maximum height of 8 m. How (a) While spring is fully compressed all the KE of M1 is stored
much energy is dissipate by air drag acting on the as PE of spring
ball during the ascent? (b) While spring is fully compressed the system momentum is
(a) 19.6 J (b) 4.9 J not conserved though final momentum is equal to initial
(c) 10 J (d) 9.8 J momentum
(c) If spring is massless, the final state of the M1 is state of
30. The kinetic energy k of a particle moving along a rest
circle of radius R depends upon the distance s as (d) If the surface on which blocks are moving has friction,
k = as2 . The force acting on the particle is then collision cannot be elastic
1/2
s2 é s2 ù
(a) 2 a (b) 2 as ê1 + 2 ú
R ë R û Comprehension Based Questions
(c) 2 as (d) 2 a
Passage I
The stopping distance for vehicle is obtained by
More Than One Correct Option dividing their kinetic energy by the stopping force
31. A ball moves over a fixed track as shown in figure. applied. If a car of mass mc and a bus of mass mb,
From A to B the ball rolls without slipping. Surfaces having kinetic energies K c and K b are stopped under
BC is frictionless. K A , K B and K C are kinetic the action of the same retarding force in distance xc
energies of the ball at A, B and C respectively. Then and xb in time tc and tb respectively, then
1/ 2
C t æm ö
(i) For and K c = K b,xc = xb and c = ç c ÷
A tb è mb ø
hC
hA i.e., bus would take longer time to stop, but they
would cover the same distance before stopping.
(a) hA > hC : KB > KC (b) hA > hC : KC > KA xc mb t
(ii) If pc = pb then = and c = 1
(c) hA = hC : KB = KC (d) hA < hC : KA > KC xb mc tb

32. A man of mass m, standing at the bottom of the i.e., stopping distance for car is more than
the bus, though
staircaseof they
height take the
L climbs itsame time. Three
and stands at its top.
[NCERT Exemplar] cars A, B, C having masses 1000 kg, 2000 kg and
(a) Work done by all forces on man is equal to the rise in 2500 kg are moving with velocities 10 2 ms–1, 10
potential energy mgL ms–1 and 8 ms–1 respectively. Exactly same force
(b) Work done by all forces on man is zero is applied to stop the cars, A and B. Time taken to
(c) Work done by the gravitational force on man is mgL stop and the distances travelled before stopping
(d) The reaction force from a step does not do work because are measured. Similar measurements are made by
the point of application of the force does not move while applying same opposite force on the cars B and C.
the force exists 35. If car A takes 5 s to stop, the time taken by car A to
33. If the kinetic energy of a body is directly proportional stop will be
to time t, the magnitude of the force acting on the (a) 5 s (b) 5 2 s
body is (c) 5 / 2 s (d) 2.5 s
(a) directly proportional to t
36. Out of cars B and C, which one stops quickly?
(b) inversely proportional to t
(a) B (b) C
(c) directly proportional to the speed of the body
(c) Both take same time (d) Cannot be said
(d) inversely proportional to the speed of the body
Telegram @unacademyplusdiscounts

Work, Energy and Power 253

37. Out of cars A and B, which one travels longer Assertion and Reason
distance before stopping.?
(a) A Directions Question No. 44 to 49 are Assertion-Reason type. Each
(b) B of these contains two Statements : Statement I (Assertion),
(c) Both travel the same distance Statement II (Reason). Each of these questions also has four
(d) Cannot say alternative choice, only one of which is correct. You have to
select the correct choices from the codes (a), (b), (c) and (d) given
38. Out of cars A and B, which one stops first? below :
(a) A (a) If both Assertion and Reason are true and the Reason
(b) B is correct explanation of the Assertion
(c) Both stop at the same time (b) If both Assertion and Reason are true but Reason is
(d) Cannot say not correct explanation of the Assertion
39. Out of cars B and C, which one would travel larger (c) If Assertion is true but Reason is false
distance before stopping? (d) If Assertion is false but the Reason is true
(a) B 44. Assertion The change in kinetic energy of a particle
(b) C is equal to the work done on it by the net force.
(c) Both travel equal distance Reason Change in kinetic energy of particle is equal
(d) Cannot say to the work done only in case of a system of one
particle.
Passage II
In a conservative force field, we can find the radial
45. Assertion Power developed in circular motion is
component of force F from the potential energy always zero.
function (U) using the relation F = dU . Positive values Reason Work done in case of circular motion is zero.
dr Kinetic energy
of F mean repulsive forces and vice-versa. We can find 46. Assertion Stopping distance =
Stopping force
the equilibrium position, where force is zero. We can
also calculate ionisation energy, which is the work Reason Work done in stopping a body is equal to
done to move the particle from a certain position to change in kinetic energy of the body.
infinity. 47. Assertion Two springs of force constants k1 and k2
Let us consider a particle bound to a certain point are stretched by the same force. If k1 > k2 , then work
at a distance r from the centre of the force. The done in stretching the first (W1) is less than work done
potential energy function of the particle is given by in stretching the second (W2 ).
A B
U ( r) = 2 - where A and B are positive constants. Reason F = k1x1 = k2 x2
r r
x1 k2
40. The nature of equilibrium is =
x2 k1
(a) neutral (b) stable 1
(c) unstable (d) Cannot be predicted k x2 2
æ k2 ö
W1 2 1 1 k k2
= = 1 ç ÷ =
-3 B2 W2 1 k x 2 k2 è k1 ø k1
41. If E = represents total energy of particle and the 2 2
16 A 2
motion is radial only, then velocity will be zero at As k1 > k2, W1 < W2
2r 2r r
(a) r0 (b) 0 (c) 0 (d) 0 48. Assertion Mass and energy are not conserved
3 5 3
separately, but are conserved as a single entity called
42. The work required to move the particle from ‘mass-energy’.
equilibrium position to infinity is Reason This is because one can be obtained at the
B2 4 B2 4B 4A cost of the other as per Einstein equation.
(a) (b) (c) (d)
4A A A B E = mc 2
43. The nature of the force is 49. Assertion Energy released when a mass of one
(a) attractive always microgram disappears in a process is 9 ´ 107 J.
(b) repulsive always 1
Reason It follows from E = mv2
(c) may be attractive or repulsive 2
(d) Cannot predict
Telegram @unacademyplusdiscounts

254 JEE Main Physics

Previous Years’ Questions


50. This question has statement I and statement II. Of 56. A block of mass 2 kg is free F(t)
the four choices given after the statements, choose to move along the x-axis its N
the one that best describes the two statements. is at rest and form t = 0
Statement I A point particle of mass m moving with onwards it is subjected to a 4.5 s
t
speed v collides with stationary point particle of mass time dependent force F ( t) in O 3s
M. If the maximum energy loss possible is given as the X-direction. The force
æ1 ö æ m ö. F ( t) varies with t as shown
f ç mv2 ÷, then f = ç ÷
è2 ø èM + mø in the figure. The kinetic energy of the block after 4.5
second is [IIT JEE 2010]
Statement II Maximum energy loss occurs when the
particles get stuck together as a result of the (a) 4.50 J (b) 7.50 J (c) 5.06 J (d) 14.06 J
collision. [JEE Main 2013] 57. A variable force given by the two-dimensional vector
(a) Statement I is true, Statement II is true, and Statement II F = (3 x2 i$ + 4 $j) acts on a particle. The force is in
is the correct explanation of Statement I newton and X is in metre. What is the change in the
(b) Statement I is true, Statement II is true, but Statement II kinetic energy of the particle as it moves from the
is not the correct explanation of Statement I point with coordinates (2, 3) to (3, 0) (The coordinates
(c) Statement I is true, Statement II is false are in metres)? [AMU (Med) 2010]
(d) Statement I is false, Statement II is true
51. A block of mass 5 kg is resting on a smooth surface at (a) –7 J (b) zero (c) +7 J (d) +19 J
what angle a force of 20 N be acted on the body so 58. Water falls from a height of 60 m at the rate of 15 kg/s
that it will acquired a kinetic energy of 40 J after to operate a turbine. The losses due to frictional force
moving 4 m [Orissa JEE 2011] are 10% of energy. How much power is generated by
(a) 30° (b) 45° the turbine ( g = 10 m/s2 )? [CBSE PMT 2008]
(c) 60° (d) 120° (a) 12.3 kW (b) 7.0 kW (c) 8.1 kW (d) 10.2 kW
52. A force ( 4 $i + $j - 2k$ ) N acting on a body maintains its 59. A body of mass m is accelerated uniformly for rest to a
$ ) ms -1. The power exerted is
velocity at (2 $i + 2 $j + 3 k speed v in a time T. The instantaneous power
[Kerala CET 2010]
delivered to the body as a function of time is given by
[AIEEE 2005]
(a) 4 W (b) 5 W (c) 2 W (d) 8 W
1 mv2 2 1 mv2
53. A body of mass M is moving with a F1 F2 (a) t (b) t
M 2 T2 2 T2
uniform speed of 10 m/s on
mv2 mv2
frictionless surface under the influence of two forces (c) 2 t2 (d) 2 t
T T
F1 and F2 . The net power of the system is
[MP PET 2010] 60. A particle is placed at the origin and a force F = kx is
(a) 10 FF12 M (b) 10 ( F1 + F2 ) M acting on it (where k is a positive constant). If U(0) = 0,
(c) ( F1 + F2 ) M (d) zero the graph of U(x) versus x will be, figure (where U is
the potential energy function) [UP SEE 2004]
54. A 2 kg block slides on a horizontal floor with a speed
of 4 ms–1. It strikes an uncompressed spring and (a) U ( x) (b) U(x)
compresses it till the block is motionless. The
kinetic friction force is 15 N and spring constant is
x x
10000 ms–1. The spring compresses by [AIEEE 2007]
(a) 8.5 cm (b) 5.5 cm
(c) 2.5 cm (d) 11.0 cm
55. An engine pumps water through a hose pipe. Water (c) U(x) (d) U(x)
passes through the pipe and leaves to with a velocity
of 2 m/s. The mass per unit length of water in the pipe
x x
is 100 kg/m. What is the power of the engine?
[CBSE PMT 2010]
(a) 800 W (b) 400 W (c) 200 W (d) 100 W
Telegram @unacademyplusdiscounts

Work, Energy and Power 255

Answers
Round I
1. (b) 2. (b) 3. (b) 4. (b) 5. (d) 6. (a) 7. (b) 8. (c) 9. (c) 10. (d)
11. (c) 12. (c) 13. (b) 14. (a) 15. (d) 16. (b) 17. (a) 18. (a) 19. (c) 20. (c)
21. (a) 22. (a) 23. (c) 24. (d) 25. (c) 26. (b) 27. (b) 28. (c) 29. (b) 30. (a)
31. (b) 32. (d) 33. (d) 34. (d) 35. (b) 36. (d) 37. (d) 38. (a) 39. (b) 40. (b)
41. (d) 42. (b) 43. (b) 44. (c) 45. (c) 46. (d) 47. (d) 48. (c) 49. (a) 50. (b)
51. (a) 52. (c) 53. (a) 54. (a) 55. (c) 56. (c) 57. (a) 58. (b) 59. (d) 60. (c)
61. (c) 62. (b) 63. (b) 64. (d) 65. (d) 66. (c) 67. (b) 68. (b) 69. (c) 70. (c)
71. (a) 72. (d) 73. (a) 74. (a) 75. (b) 76. (b) 77. (b) 78. (a) 79. (b) 80. (d)
81. (c) 82. (a) 83. (a) 84. (b) 85. (b) 86. (b) 87. (d) 88. (b)

Round II
1. (d) 2. (c) 3. (b) 4. (a) 5. (a) 6. (d) 7. (a) 8. (b) 9. (a) 10. (d)
11. (d) 12. (c) 13. (d) 14. (c) 15. (b) 16. (a) 17. (b) 18. (a) 19. (c) 20. (c)
21. (a) 22. (a) 23. (b) 24. (a) 25. (b) 26. (c) 27. (a) 28. (b) 29. (d) 30. (b)
31. (a,b,d) 32. (b,d) 33. (b,d) 34. (c,d) 35. (b) 36. (c) 37. (c) 38. (a) 39. (a) 40. (b)
41. (a) 42. (b) 43. (a) 44. (a) 45. (a) 46. (a) 47. (a) 48. (a) 49. (c) 50. (d)
51. (c) 52. (a) 53. (d) 54. (b) 55. (a) 56. (c) 57. (c) 58. (c) 59. (d) 60. (a)

the Guidance
Round I
dx d æ t 3 ö 2 4. As the magnetic forces due to motion of electron and proton
1. v = = ç ÷ =t
dt dt è 3 ø act in a direction perpendicular to the direction of motion, no
work is done by these forces. That is why one ignores the
When t = 0, then v = 0, when t = 2, then magnetic force of one particle on another.
v = 4 m/s
1 1 5. Work done by hand = maximum PE of the ball
W = m[( 4) 2 - (0) 2] = ´ 2 ´ 16 = 16J F ´ s = mgh
2 2
mgh
F=
2. W = mg sin q ´ S s
= 2 ´ 10 3 ´ sin 15° ´ 10 = 5.13 kJ 0.2 ´ 10 ´ 2
F= = 20 N
0.2
s
6. Initial height of CG = 4 cm
Final height of CG = 10 cm
θ Increase in height = 6 cm = 0.06 m
sin
g Work done = 5 ´ 10 ´ 0.06 = 3 J
m
θ
7. As, F = mmg cos q
3. Tension in the string or F = 0.30 ´ 10 ´ 10 cos 45°
æ g ö Mg 30
T = M( g - a) = Mç g - ÷ = or F= N
è 2ø 2 2
W = Force ´ displacement 30 150 2
W =F´s= ´5 = ´ = 75 2 J
Mgh 2 2 2
=-
2 This is negative work because F and s are oppositely directed.
Telegram @unacademyplusdiscounts

256 JEE Main Physics

3 15. Mass of the body (m) = 0.5 kg


8. Force, F = mg
10
Velocity of the body (v) = ax3 / 2
As, W = - Fs
3 where, a = 5 m-1/ 2/s
or W =- mgs
10 Velocity of the body at x = 0,
3 v1 = 5 ´ 0 = 0
or W =- ´ 200 ´ 10 J = - 600 J
10 Velocity of the body at x = 2 m,
v 2 = 5 ´ (2)3 / 2 m/s
9. Force between two protons = force between a proton and a
positron. As positron is much lighter than proton, it moves According to work-energy theorem,
away through much larger distance compared to proton . As Work done = Change in kinetic energy
work done = force ´ distance, therefore in the same time t, 1 1
work done in case of positron is more than that in case of = mv 22 - mv12
2 2
proton. 1
= m(v 2 - v12)
2
10. When the man gets straight up and stand, reaction of ground 2
on the man = mg. However, when he is squatting on the 1
= ´ 0.5 [5 (23 / 2) - (0) 2]
ground, reaction of ground is more than mg , as the man is to 2
exert some extra force on the ground to stand up. 1
= ´ 0.5 ´ 2.5 ´ 23
11. Initial velocity of ball is zero i. e. ,u = 0 2
1
\ Displacement of ball in t th second = ´ 12.5 ´ 8 = 50 J
2
1 æ 1ö M
s = gt - g = g çt - ÷
2 è 2ø 16. dW = - m éê ùú gl dl
ëLû
æ 1ö 2L
mMg
s µ çt - ÷
è 2ø W =ò3 - l dl
0 L
æ 1ö æ 1ö æ 1ö 2L
or s1: s2: s3 = ç1 - ÷ : ç2 - ÷ : ç3 - ÷ = 1 : 3 : 5
è 2ø è 2ø è 2ø mMg é l 2 ù 3
or W =-
Now, W = mgs L êë 2 úû 0
W µs mMg 4 L2
or W =- -0
\ W1: W2: W3 = 1: 3 : 5 2L 9
a 2
12. Initial height of CG = or W = - mMgL
2 9
b Weight in air
Final height of CG = 17. Relative density =
2 Loss of weight in water
é b aù æ b - aö 5 ´ 10
Work done = mg - = mg ç ÷ \Loss of weight in water = N
êë 2 2 úû è 2 ø 3
13. Here t = x + 3 æ 50 ö 100
Weight in water = ç50 - ÷N= N
3 3
or x = (t -è3) 2 = t 2 -ø 6 t + 9
100 500
dx Work done = N ´ 5m = J
v= = 2t - 6 3 3
dt
dx
at t = 0 s,v = 2 ´ 0 - 6 = - 6 18. As, v = = 3 - 8 t + 3 t 2
dt
at t = 6 s, v = 2 ´ 6 - 6 = + 6
\ v 0 = 3 m/s and v 4 = 19 m/s
Initial and final KE are same hence no work is done 1
1 W = m (v 42 - v 02)
W = m (v12 - v 22) = 0 2
2 (According to work energy theorem)
14. This is the case of work done by a variable force 1
= ´ 0.03 ´ (19 2 - 33) = 5.28 J
5 2
W = ò (3x2 - 2x + 7) dx
0 1
19. Here, 1400 ´10 ´10 + W = ´15 ´15
W = | x3 + x2 + 7x|50 2
or W = (5 ´ 5 ´ 5 - 5 ´ 5 + 7 ´ 5) or W = 700 ´ 15 ´ 15 - 1400 ´ 10 ´ 10
= 700(225 - 200)J
or W = (125 - 25 + 35) = 135 J
= 700 ´ 25J = 17.5 kJ
Telegram @unacademyplusdiscounts

Work, Energy and Power 257

h B
20. When the block moves vertically downward with 27. Work done = mg æç ö÷
g è ø 2
acceleration , then tension in the cord
4 h
10 ´ 10 ´ h h/2
æ gö 3 100 =
T = M ç g - ÷ = mg 2
è 4ø 4 A
h = 2.0 m 10 × g
Work done by the cord F × s = Fs cos q = Td cos180°
æ 3 ö d 28. As both surfaces I and II are frictionless and two stones slide
= ç - Mg ÷ ´ d = -3 Mg from the same height, therefore, both the stones reach the
è 4 ø 4
æ1 ö
bottom with same speed ç mv 2 = mgh÷ . As acceleration
21. Work done in lifting water and drum = 60 ´10 ´ 20 J è2 ø
= 12000 J down plane II is larger ( a2 = g sin q2 greater than a1 = g sin q1 ),
Total mass of ropes = 4 ´ 0.5 kg = 20 kg therefore, stone II reaches the bottom earlier than stone I.
+a
Work done in the case of ropes 29. W = ò Fdy = ò ( Ay 2 + By + C)dy
-a
= 20 ´ 10 ´ 10 = 2000 J +a
Total work done = 14000 J é Ay3 By 2 ù é Aa3 Ba2 ù é Aa3 Ba2 ù
ê 3 + 2 + Cy ú = ê 3 + 2 + Caú - ê - 3 + 2 - Caú
22. As, dW = Fdl ë û -a ë û ë û
l Fl 2Aa3
W = ò Fdl Y = = + 2Ca
0 dl 3
Yal
l 30. Work done = Area between the graph and position axis
or W =ò dl
L
0
W = 10 ´ 1 + 20 ´ 1 - 20 ´ 1 + 10 ´ 1 = 20 erg
Yal
or F= 31. According to the graph the acceleration a varies linearly with
L
the coordinate x.We may write a = ax, where a is the slope of
Ya l the graph
L ò0
or W= l dl
20
a= mg 0 = 2.5 ms-2
Ya æ l 2 ö 8
or W= ç ÷
L è2ø The force on the brick is in the positive x- direction and
1 Yal 1 according to Newton's second law, its magnitude is given by
or W= l = Fl a a
2 L 2 F= = x
m m
23. As the road does not move at all, therefore, work done by the
if xf is the final coordinate, the work done by the force is
cycle on the road must be zero.
xf a xf
L 1 W = ò Fdx = ò xdx
24. The weight of hanging part æç ö÷ of chain is æç Mg ö÷. This 0 m 0
è3ø è3 ø
a 2 2.5
weight acts at centre of gravity of the hanging part which is at = xf = ´ (8) 2 = 8 J
2m 2 ´ 10
a depth of L /6 from the table.
As work done = force ´ distance 32. Let m be the mass of the block,h the height from which it is
dropped, and x the compression Mgof the
L spring.
MgL
\ W= ´ =
3 6 18 Since, energy is conserved, so
Final gravitational potential energy
25. In inelastic collision between two bodies, total linear
momentum remains conserved. = final spring potential energy
1
1 1´ 5 or mg (h + x) = kx2
26. As, W1 = kx12 = ´ 10 3 ´ (5 ´ 10 -2) 2 = 6.25 J 2
2 2 1
1 or mg (h + x) + kx2 = 0
W2 = k ( x1 + x2) 2 2
2
or kx2 - 2mg (h + x) = 0
1
´ 5 ´ 10 3(5 ´ 10 -2 + 5 ´ 10 -2) 2 = 25 J kx2 - 2mgx - 2mgh = 0
2
This is a quadratic equation for x. Its solution is
Net work done = W2 - W1
mg ± (mg ) 2 + 2mghk
= 25 - 6.25 x=
k
= 18.75 J
Now, mg = 2 ´ 9.8 = 19.6 N
= 18.75 N - m
Telegram @unacademyplusdiscounts

258 JEE Main Physics

and hk = 0.40 ´ 1960 = 784 N 1 2 1


or W= kx - k ( x + y) 2
2
19.6 ± (19.6) + 2(19.6)(784) 2 2
\ x= 1 2 1 2
1960 or W = kx - k ( x + y 2 + 2xy)
2 2
= 0.010 m -0.080 m
or
1 2 1 2 1 2 1 1
Since, x must be positive (a compression), we accept the = kx - kx - ky - k (2xy) = - kxy - ky 2
2 2 2 2 2
positive solution and reject the negative solution. Hence 1
x = 0.10 m. = ky( -2x - y)
2
1 1
33. Initial KE of the body = mv 2 = ´ 25 ´ 4 = 50 J The work done against elastic force is
2 2
ky
Work done against resistive force Wext = -W = (2x + y)
2
= Area between F-x graph 1
1
= ´ 4 ´ 20 = 40 J
37. For first ball mgh1 = mu2
2
2
u2
Final KE = Initial KE – Work done against resistive force i. e. , h1 = h1
2g u
= 50 - 40 = 10 J
For second ball
m
34. In this case motion of stone is in vertical circle of radius L and u 2 cos2 q
centre at O. mgh2 = mg
2g
The change in velocity is
1 1
v
= mu 2 cos2 q = mu 2 cos2 60°
O
Y
L 2 2
2
1 æ 1ö 1 æ 1ö
= mv 2 ç ÷ = mv 2 ç ÷
2 è 2 ø 2 è 4ø
L u2
Þ h2 =
8g
u
h1 u 2 8 g
Dv = v - u = v$j - u$i = ´
h2 2 g u 2
| Dv| = (v) 2 + ( -u) 2 = v 2 + u 2
u cos 60°
According to work-energy theorem,
W = DK u
1 1 60° h2
or WT + Wg = mv 2 - mu 2 …(i)
2 2
u sin 60°
WT = work done by the force of tension = 0 h1 4
Wg = work done by the force of gravity Þ =
h2 1
= mgL (path independent)
1
1 1 38. Given, K = as2 or mv 2= as2
From Eq. (i), 0 - mgL = mv 2 - mu 2 2
2 2
Differentiating w.r.t. time t ,
Q v 2 = u 2 - 2gL æ dv ö æ ds ö
m (2v) ç ÷ = (2a)(2s) ç ÷
\ | Dv| = v 2 + u 2 = 2 (u 2 - gl) è dt ø è dt ø
ds
35. At x = + xm , the particle turns back. Therefore, its velocity at \ =v
dt
this point is zero.Therefore, kinetic energy K = 0. The total dv
energy E is in the form of potential energy i.e., V = E. \ 2m = 4as
dt
36. Elastic force in string is conservative in nature. dv
Þ m = 2as
W = -DV1 dt
or Ft = 2as
where, W = work done by elastic force of string
W = - (Vf - Vi ) = Vi - Vf F = Ft2 + Fr2 (\Fr = 0)
F = Ft = 2as
Telegram @unacademyplusdiscounts

Work, Energy and Power 259

39. Here, m = 0.5 kg, v = ax3/ 2, where a = 5 m-1/ 2 s-1,W = ? K2 h2


\ =
K1 h1
dv d 3 dx
Acceleration, A = = ( ax3 / 2) = a ´ x1/ 2 K1 K1 h2
dt dt 2 dt Given K2 = = =
3a 1/ 2 3 / 2 2 2K1 5
= x ( ax )
2 \ h2 = 2.5 m
3 a2 2 44. At the highest point of its flight, vertical component of velocity
A= x
2 is zero and only horizontal component is left which is
3 a2 2 3 a2 2 ux = u cos q
F = mA = 0.5 ´ x = x
2 4 Given q = 45°
2
2 2 3 a2 2 3a2 é x3 ù \ ux = u cos 45° =
u
W = ò F. dx = ò x dx =
0 0 4 4 êë 3 úû 0 2
3 Hence, at the highest point kinetic energy
= (5) 2 [23 - 0 ] 1
4 ´3 E' = mux2
2
W = 50 J 2
1 1 æ u ö 1 æ u2 ö E æ 1 2 ö
40. KE = mv 2 = mç ÷ = mç ÷ = ç\ mu = E ÷
2 è 2ø 2 è2ø 2 è 2 ø
2
Given, v 2 = (v1 + 2)
2
45. Due to conservation of total energy.
K1 æ v1 ö a b dU
\ =ç ÷ 46. U( x) = - 6 at the stable equilibrium =0
K2 è v 2 ø x12
x dx
1 v12 12a 6b æ 2a ö
1/ 6
Þ = (\ K2 = 2 K1) \ - + 7 =0 Þ x= ç ÷
2 (v1 + 2) 2 x13
x èbø
v12 + 4v1 + 4 = 2v12 ¶U $ ¶U $
47. F = i- j = 7 $i - 24 $j
v12 - 4v1 - 4 =0 ¶x ¶y
4 ± 16 + 16 F 7
v1 = \ ax = X = = 14 . ms-2 along positive x- axis
2 m 5
4 + 32 F 24
v1 = = 2( 2 + 1) ms-1 ay = Y = -
2 m 5
= 4.8 ms–2 along negative y-axis
41. If there is no air drag then maximum height
u 2 14 ´ 14 \ v x = axt = 1.4 ´ 2 = 2.8 ms–2
H= = = 10 m
2 g 2 ´ 9.8 and v y = 4.8 ´ 2 = 9.6 ms-1
But due to air drag ball reaches up to height 8 m only so, loss \ v = v x2 + v y2 = 10 ms-1
of energy
= Mg (10 - 8) = 0.5 ´ 9.2 ´ 2 = 9.8 J 48. Let m = mass of boy, M = mass of man
2 -3
42. Here, P = [ML T ] = constant v = velocity of boy;
V = velocity of man
As, mass M of body is fixed,
1 1 é1 ù
L2 T -3 = constant mV 2 = mv 2 …(i)
2 2 êë 2 úû
2
L 1
= constant or L µ T 3 / 2 or displacement (d) µ t 3 / 2. æ1 ö
T3 M (V + 1) 2 = 1 ç mv 2÷ …(ii)
2 è2 ø
1
43. As, K = mv 2 M
Putting m = and solving, we get, V =
1
2 2 2 -1
98 ´ 2
v2 = = 98
2 49. For first condition,
v 2
98 initial velocity = u, final velocity = u / 2, s = 3 cm
h= = =5
2g 2 ´ 9.8 From v 2 = u 2 - 2 as
2
1 1 æuö 2
K1 = mv 2 = m ´ 2gh Þ ç ÷ = u - 2 as
2 2 è2ø
Telegram @unacademyplusdiscounts

260 JEE Main Physics

3 u2 55. Vertical height = h = l cos q = l cos 30°


Þ a=
8s 3
Loss of potential energy = mgh = mgl cos 30° = mgl
For second condition, 2
Initial velocity = u / 2, final velocity = 0, s = x A
From v 2 = u 2 - 2 as
l
u2 h 30°
Þ 0= - 2 ax
4
B
u2 u2 ´ 8 s 8
x= = = = 1cm
4 ´ 2 a 4 ´ 2 ´ 3 u2 3 3
\ Kinetic energy gained = mgl
2
50. (a) If the surface is smooth, then the kinetic energy at B never
be zero. 56. p = 2mEK
(b) If the surface is rough, the kinetic energy at B be zero.
or pµ m [Q E k is given to be constant]
Because, work done by force of friction is negative. If
work done by friction is equal to mgh then, net work done p1 m1 1 1
\ = = =
on body will be zero. Hence, net change in kinetic energy p2 m2 4 2
is zero. Hence, (b) is correct.
(c) If the surface is rough, the kinetic energy at Bmust be lesser 57. Here, m = 5 kg, r =1m
than mgh. If surface is smooth, the kinetic energy at B is 300
w= rps = 5 rps = 5 ´ 2 p rad s-1
equal to mgh. 60
(d) The reason is same as in (a) and (b) 1 1 1
KE = mv 2 = m (rw) 2 = ´ 5 (1 ´ 10 p ) 2 = 250 p 2 J
1 2 2 2
51. E = kx2 490
2 58. Potential energy at the required height = = 245J
F µk 2
E1 k1 Again, 245 = 2 ´ 10 ´ h
= 245
E 2 k2 or h= m = 12.25 m
20
52. When a pendulum oscillates in air, it loses energy
continuously in overcoming resistance due to air. Therefore,
59. As the shotput reaches the ground, its KE
total mechanical energy of the pendulum decreases = PE of shotput when it is thrown + KE given
continuously with time. The variation of total mechanical 1 1
= mgh + mv 2 = 10 ´ 10 ´ 1.5 + ´ 10 (1) 2
energy E with time t is shown correctly by curve (c). 2 2
= 150 + 5 = 155 J
53. Applying principal of conservation of linear momentum
75
velocity of the system (v) is 60. Useful work = ´ 12J = 9 J
100
m1v1 = (m1 + m2)v 1
m1v1 50 ´ 10 1 Now, ´1´ v 2 = 9
Þ v= = = ms-1 2
m1 + m2 (50 + 950) 2 or v = 18 ms-1
1
Initial KE, E2 = m1v12 61. Initially, 4 u = 8 Þ u = 2 m/ s
2
1 æ 50 ö 2
Now, mv - mu = Ft
= ´ç ÷ ´ 10 = 2.5 J mv - 8 = 0.2 ´ 10
2 è1000 ø
1 or v = 5 / 2 ms-1
Final KE, E 2 = (m12 + m22) v 2 1
2 Increase in KE = m(v 2 - u 2)
1 (50 + 950) 1 2
= ´ = 0.125J 2
2 1000 2 1 é æ5ö ù
= ´ ê 4 ç ÷ - (2) 2ú = 4.5 J
Percentage loss in KE 2 êë è 2 ø úû
E1 - E 2 2.5 - 0.125
´ 100 = = 95% 62. If the body strikes the sand floor with a velocity v, then
E1 2.5
1
2 Mgh = mv 2
1 5ö 2
54. Kinetic energy = ´ 950 ´ æç100 ´ ÷ J
2 è 18 ø With this velocity v, when body passes through the sand floor
= 0.3665 ´ 10 6 J = 0.376 MJ it comes to rest after travelling a distance x. Let F be the
resisting force acting on the body. Net force in downwards
direction = Mg - F
Telegram @unacademyplusdiscounts

Work, Energy and Power 261

Work done by all the forces is equal to change in KE 68. Net force on the body is zero and its potential energy should
1 be minimum for stable equilibrium.
(Mg - F) x = 0 - Mv 2
2 1
(Mg - F) x = -Mgh
69. As, E = mg 2t 2
2
or Fx = Mgh + Mgx 1
mg 2 ´ 3 2
æ hö E1 2 9 1
or F = Mg ç1 + ÷ = = =
è xø 1
E2 mg 2(6 2 - 3 2) 9 ´ 3 3
2
63. Because the efficiency of machine is 90%, hence, potential
a
energy gained by the mass 70. Height of CG of mass m1 =
90 2
= ´ energy spent b
100 Height of CG of mass m2 = a +
90 2
= ´ 5000 = 4500 J \ Gravitational potential energy of system
100
a æ bö ém ù b
When the mass is released now , gain in KE on hitting the = m1g + m2g ç a + ÷ = 1 + m2 ga + m2g
ground = Loss of potential energy = 4500 J 2 è 2 ø êë 2 úû 2
dx éæm ö bù
64. x = 2t4 + 5t + 4 = v = = 8t + 5 = ê ç 1 + m2÷ a + m2 ú g
dt ëè 2 ø 2û
At t = 0 ,v = 5 m/s 71. Let a body of mass m which is initially at rest undergoes
At t = 1s,v = 8 ´ 1 + 5 = 13 m/s one-dimensional motion under a constant force F with a
1 constant acceleration a.
Increase in KE = m [(13) 2 - (5) 2] = 144 J
2 F
Acceleration ( a) = …(i)
m
65. Both fragments will possess the equal linear momentum
m1v1 = m2v 2 Using equation of motion, v = u + at
F
Þ 1 ´ 80 = 2 ´ v 2 Þ v = 0 + ×t (Q u = 0)
m
Þ v 2 = 40 m/s
F
1 1 Þ v= t …(ii)
\ Total energy of system = m1v12 + m2v 22 m
2 2
1 1 Power delivered (P) = Fv
= ´ 1 ´ (80) 2 + ´ 2 ´ ( 40) 2 Substituting the value from Eq. (ii), we get
2 2
F
= 4800 J = 4.8 kJ Þ P =F ´ ´t
m
m1 m2
66. 9 kg At rest v1 = 1.6 m/s F2
3 kg 6 kg Þ P= t
Before explosion m
After explosion
Dividing and multiplying by m in RHS,
At the bomb initially was at rest therefore initial momentum of F2
bomb = 0 P= ´ mt = a2mt [Using Eq. (i)]
m2
Final momentum of system = m1v1 + m2v 2
As mass m and acceleration a are constants.
As there is no external force
\ P µt
\ m1v1 + m2v 2 = 0
3 ´ 1.6+6 ´ v 2 = 0 72. As, m = 10 ´ 0.8 kg, h = 5 m
Velocity of 6 kg mass v 2 = 0.8 m/s (numerically) mgh 8 ´ 10 ´ 5
\ P= =
1 1 t 10
its kinetic energy = m2v 22 = ´ 6 ´ (0.8) 2 = 1.92 J
2 2 = 40 W
mass dm 73. Volume of water of raise = 22380 L = 22380 ´10 -3 m3
67. K = =
length dt mgh Vrgh
1 P= =
KE = mv 2 t t
2 Vrgh
Þ t=
d æ dm ö 2 1 æ dm dx ö 2 1 1 r
Þ ç ÷v = ç ´ ÷ v = kvv 2 = kv3
dt è dt ø 2 è dx dt ø 2 2
22380 ´ 10 -3 ´ 10 3 ´ 10 ´ 10
t= = 5 min
10 ´ 746
Telegram @unacademyplusdiscounts

262 JEE Main Physics

74. Power = 7500 W = 7500 Js-1, Velocity v = 20 ms-1 82. Power of motor initially = P0
-1
P 7500 Js Let rate of flow of motor = ( x)
P = Fv or F = = = 375 N
v 20 ms-1 work mgy æy ö
Since, power P0 = = = mg ç ÷
time t èt ø
mgh m P
75. As, P = or = y
t t gh = x = rate of flow of water = mgx …(i)
m 1000 t
or = kg = 10 kg If rate of flow of water is increased by n times, i. e. ,(nx)
t 10 ´ 10
Increased power,
10 - 0
76. As, a = ms-2 = 2 ms-2 mgy ¢ æ y¢ö
5 P1 = = mg ç ÷ = nmgx …(ii)
t èt ø
Q F = ma
\ F = 1000 ´ 2 N = 2000 N The ratio of powers
0 + 10 P1 nmgx
Average velocity = ms-1 = 5 ms-1 =
2 P0 mgx
Average power = 2000 ´ 5W = 10 4 W Þ P1 : P0 Þ n : 1
10 4
Required horse power is 83. Motor makes 600 revolution per minute
746 revolution rev
\ n = 600 = 10
77. When an iron sphere falls freely in a lake, its motion is minute sec
accelerated due to gravity and retarded due to viscous force. 1
The overall effect is increase in velocity and hence increase in \Time required for one revolution = sec
10
KE till the sphere acquires terminal velocity, which is
Energy required for one revolution = Power ´ time
constant. Hence KE of sphere beyond this depth of lake
1 1 746
becomes constant. Choice (b) is most appropriate. = ´ 746 ´ = J
4 10 40
dm
78. Given, h = 500 m , = 2000 kgs-1 But work done = 40% of input
dt
80 dm 746
\ Power output = ´ gh = 40% ´
100 dt 40
4 40 746
= ´ 2000 ´ 10 ´ 500 W = ´ = 7.46 J
5 100 40
= 8 ´ 10 6 W = 8 MW 1
mgh + mv 2
total energy 2
79. Minimum force mg sin q. So, minimum power is given by 84. As power, P = =
t t
P = mg sin q v (Q F = mg sin q ) 1
10 ´ 10 ´ 20 + ´ 10 ´ 10 ´ 10
P 2
or v= =
mg sin q 1
9000 ´ 2 = 2000 + 500 = 2500 W
or v= = 15 ms-1
1200 ´ 10 ´ 1 = 2.5 kW
18
= 15 ´ = 54 kmh -1 85. P = constant
5
Þ Fv = P [Q P = force ´ velocity]
80. Given, u = 10 ms-1, v = 20 ms-1 Þ Ma ´ v = P [Q F = Ma]
\ Work done = Increase in kinetic energy P
1 Þ va =
= ´ 500 [20 2 - 10 2] M
2 vdv P vdv ù
é
500 ´ 30 ´ 10 Þ v´ = Qa=
= ds M êë ds úû
2
v 2 s P
500 ´ 30 ´ 10
Power = = 1250 W Þ ò0 v dv =ò0 M ds
2 ´ 60
[Assuming at t = 0 it starts from rest, i. e., from s = 0 ]
81. Volume = av = pr 2v
v3 P
Mass = pr 2v ´ 1000 SI units Þ = s
3 M
1
mv 2 æ 3P ö
1/3
2 1 Þ v=ç ÷ ´ s1/3
Power of water jet = = ´ pr 2v ´ 1000 ´ v 2 = 500 pr 2v3 èMø
t 2
Telegram @unacademyplusdiscounts

Work, Energy and Power 263

1/3 2
ds é æ 3P ö ù æ dx ö
Þ = ks1/3 êk = ç ÷ ú ç ÷ = k1t
dt êë èMø ú è dt ø
û
S ds t dx
= k1t
Þ ò0 s1/3 = ò0kdt dt
dx
s2/3 = k2(t )1/ 2 (\ k11/ 2 = k2)
Þ = kt dt
2 /3 2 ö
æ
3/ 2 x = k3t 3 / 2 çQ k3 = k2÷
æ2 ö 3/ 2 è 3 ø
\ s = ç k÷ ´t
è3 ø dx 1/ 2
Hence µ t µ x1/3
Þ s µ t3/ 2 dt
v2
86. Energy required = mgh 88. Here ac = = k2rt (Q v = krt )
r
In both, cases, h is the same. Hence, energy given by both is \ v = krt
same. [ It is worth noting here that powers of two men will be dv
different as power is the energy expense per unit time and The integral acceleration is at =
dt
times are different]
d(krt )
æ d 2x ö = = kr
æ dx ö dt
87. P = Fv = (ma) v = m ç ÷ ç ÷
è dt 2 ø è dt ø
The work done by centripetal force will be zero.
Since, power is constant. So power is delivered to the particle by only tangential force
æ d 2x ö æ dx ö which acts in the same direction of instantaneous velocity.
ç 2÷ ç ÷ = k
è dt ø è dt ø \ Power = Fv
t = matkrt
2 = m (kr)(krt )
d æ dx ö
or ç ÷ =k = mk2r 2t
dt è dt ø

Round II
1. Length of the pendulum = 15
. m 1
2. As, E1 = mv 2
2
1.5 m 1
O A m (v + 1) 2
E2 =
2
1
m [(v + 1) 2 - v 2]
1.5 m (E 2 - E1) 2 44
= =
E1 1 2 100
mv
2
B On solving, we get v = 5 ms-1.
B
3. Loss of KE = force ´ distance = (ma) x = work done
Potential energy of the bob at position A = mgh As a µ x \ Loss of KE µ x2
As bob moves from position A towards position B its 1 1 1
potential energy converted into kinetic energy. 5% of 4. KE left, mv 2 = æç mu2ö÷
2 è 2 2 ø
its potential energy is dissipated against air
resistance. u 10 4
\ Velocity left, v = = = 7071.06 ms–1
2 2
\ KE at position B = 95% of its PE at position A
1 95 L
mv 2 = ´ mgh 5. We know that, P = F × v = F ×
2 100 T
2 ´ 95 ´ gh As F = [MLT –2] = constant
or v=
100 \ L µT2

=
19
´ 9.8 ´ 15
. L T2
\ P =F× =F× = F ×T
10 T T
= 5.28 m/s or P µT
Telegram @unacademyplusdiscounts

264 JEE Main Physics

æ v1 ö 11. Here, m1 = 20 kg
6. From v = u + at ,v1 = 0 + at1 çQ a = ÷
è t1 ø
m2 = 0.1 kg
mv1
F = ma = v1 = velocity of recoil of gun.
t1
v 2 = velocity of bullet
v1
Velocity acquired in t sec = at = t As m1v1 = m2v 2 (Q momentum is conserved)
t1
m2 0.1 v
mv1 v1t mv12t v1 = v2 = v2 = 2
Power = F × v = ´ = 2 m1 20 200
t1 t1 t1 2
1 1 æ v ö
1 2 1 1 Recoil energy of gun = m1v12 = ´ 20 ç 2 ÷
7. kx = mv 2 + mv 2 = mv 2 2 2 è 200 ø
2 2 2
10 v 22 v 22
2mv 2 804 = =
x= 4 ´ 10 4
4 ´ 10 3
k
v 2 = 804 ´ 4 ´ 10 3 ms-1
8. Velocity at B when dropped from A
where, AC = s 12. From work energy theorem,
v 2 = u 2 + 2g ( s - x) …(i) DKE = Wnet
v 2 = 2g ( s - x) …(ii) or Kf - Ki = ò Pdt
Potential energy at B = mgx 1 2 æ3 ö
\ kinetic energy = 3 ´ potential energy or mv 2 = ò ç t 2÷ dt
2 0 è2 ø
1
m ´ 2g ( s - x) = 3 ´ mgx A 2
2 2 é t3 ù
v =ê ú
or ( s - x) = 3x ë 2 û0
or s = 4x
s v = 2 ms-1
or x= B – KE = 3 PE
4 2 2 2 d 2s 2 d 2s ds
13. As, W = ò Fds = ò Mads = ò M ds = ò0 M dt 2 × dt dt
From Eq. (i) 0 0 0 dt 2
v 2 = 2g ( s - x) 2
2 æ2ö æ2 ö 4 ét2ù
æ s ö 2g ´ 3s 3gs = ò 3 ç ÷ ç t ÷ dt = ê ú
= 2g ç s - ÷ = = 0 è3ø è3 ø 3 ë 2 û0
è 4ø 4 2
4 4 8
s 3gs W = ´ = = 2.6 J
\ x= and v = 3 2 3
4 2
1 14. From work-energy theorem, DKE = Wnet
9. Given, s = t 2
3 Kf - Ki = ò P dt
ds 2 d 2s 2
\ v= = t,a = 2 = 1 2 æt2ö
dt 3 dt 3 mv 2 - 0 = ò ç ÷ dt
2 2 0 è2ø
3=F =ma
´= 2
3 1 1 é t3 ù
1 2 or (2)v 2 = ê ú
W =2´ t 2 2 ë 3 û0
3
1 8 2
At t = 2 s, W = 2 ´ 2 ´2 = J v =2 ms-1
3 3 3
t3 dE
10. Given, s= 15. Given, P = 3t 2 - 2t + 1 =
3 dt
\ ds = t 2 dt \ dE = (3t 2 - 2t + 1)dt
2 2 3 t =4 s
d s d ét ù 2 Þ E=ò (3t 2 - 2t + 1) dt
Þ a= = ê ú = 2 t m/s t =2s
dt 2 dt 2 ë 3 û
t =4 s
Now work done by the force é 3t 3 2t 2 ù
2 2
=ê = + 1ú
W = ò F × ds = ò m × ads ë 3 2 û t =2s
0 0
2 2 3 = [( 43 - 23) - ( 4 2-2 2) + ( 4 + 2)]
2 2
ò0 3 ´2t ´t dt = ò 6 t dt = [t 4 ]0 = 24 J
3
0 2 or E = 56 - 12 + 2 = 46 J
Telegram @unacademyplusdiscounts

Work, Energy and Power 265

16. As, P = (mg sin q + F)v 3 1


24. KE lost is th, therefore, KE left is th. Hence, velocity of
4 4
æ 1 ö v0
= ç1000 ´ 10 ´ + 200 ÷ ´ 20 particle reduces from v 0 to = v 0 - mg t 0
è 20 ø
2
= 14000 W = 14kW v
or m= 0
dW dv 2gt 0
17. As, P= =P
dt dt
25. If W1 = work done by applied force
P = h d g = 10 ´ 13.6 ´ 980
W2 = work done against friction then applying work-energy
= 1.3328 ´ 10 6dyne/cm2 theorem
dv W1 - W2 = PE + KE (at the top)
and = pulse frequency ´ blood discharged per pulse
dt 1
F ´ s - W2 = mgh + ´ mv 2
dv 72 2
\ = ´ 75 = 90 cc/s
dt 60 1
100 ´ 12 - W2 = 50 ´ 10 ´ 2 + ´ 50 ´ 2 2
\ Power of heart = 1.3328 ´ 10 5 ´ 90 erg /s = 1.19 W 2
1 1200 - W2 = 1100
18. As, K = mv 2 W2 = 100 J
2
dK dv æ dv ö 26. Let mass of boy be m, therefore, mass of man = 2 m,
= mv × = çm ÷v = mav = 4v
dt dt è dt ø
1
As, KE of man = KE of boy
Q m = 2 kg and a = ms-2 2
1 1 1
19. As slope of problem graph is positive and constant upto \ (2 m) u 2 = ´ mu ¢2
certain distance and then it becomes zero. 2 2 2
- dU 2 u'2 u'
So, from F = , upto distance u = ,u =
dx 4 2
F = constant (negative) and become zero suddenly. When man speeds up to 1 m/s
KE of man = KE of boy
20. As, K1 = mgh1 and K2 = mgh2 1 1 1
(2 m) (u + 1) 2 = mu'2 = m (2 u) 2
K1 - K2 h -h 2 2 2
\ %Loss = ´ 100 = 1 2 ´ 100 = 50%
K1 h1 (u + 1) 2 = 2 u 2
1 u +1= 2u
21. As, U = ks2 = 10 J
2 1 2 +1
1 æ1 ö u= =
and U' = k ( s + s) 2 = 4 ç ks2÷ = 40 J 2 - 1 ( 2 - 1) ( 2 + 1)
2 è2 ø
u = ( 2 + 1) ms-1
\ W = U' - U = 40 - 10 = 30 J
u' = 2 u = 2 ( 2 + 1) ms-1
22. Let v be the velocity with which the bullet will emerge.
Now, change in kinetic energy = work done 27. Work output of engine = mgh = 100 ´10 ´10 = 10 4 J
45
1 output 10 10
For first case, m (100) 2 - m ´ 0 = F Efficiency (h) = = ´ 100 = J
2 2 input 60 6
1 1 input energy 10 5 / 6
For second case, m(100) 2 - mv 2 = F ´ 0.5 \ Power = =
2 2 time 5
Dividing Eq. (ii) by Eq.(i), we get 10 5
(100) 2 - (v) 2 0.5 1 100 = = 3.3 kW
= = or v= = 50 2 ms-1 30
(100) 2 1 2 2
28. Let x be the maximum extension of the spring
23. Initial velocity of particle, vi = 20 ms-1 as shown in figure. From conservation of
Final velocity of the particle, v f = 0 mechanical energy; decreases in gravitational k
According to work-energy theorem potential energy = increase in elastic potential
energy
Wnet = DKE = Kf - Ki
1 1 M v =0
= m(v f2 - vi2) \ Mgx = kx2
2 2
x
1 2 Mg
= ´ 2(0 2 - 20 2) = -400 J \ x=
2 k M u =0
Telegram @unacademyplusdiscounts

266 JEE Main Physics

1 1 tC
29. Energy supplied = mv 2 = (0.5) 142 = 49 J 36. When PC = PB; =1
2 2 tB
Energy stored = mgh = 0.5 ´ 9.8 ´ 8 = 39.2J we find that both cars B and C will stop simultaneously.
\ Energy dissipated = 49 - 39.2 = 9.8 J 37. From car A,
1 1 1
30. Here K = mv 2 = as2 KA =mAv A2 = ´ 1000(10 2) 2 = 105 J
2 2 2
\ mv 2 = 2as2 1 1
For car B, KB = mBvB2 = ´ 2000(10) 2 = 10 5 J
Differentiating w.r.t. time t 2 2
dv ds dv Thus, cars A and B have same KE. When same stopping force
2mv = 4as = 4 asv Þ m = 2as
dt dt dt is applied, both would travel the same distance before
This is tangential force, Ft = 2as stopping.
1/ 2
mv 2 2as2 t A æ mA ö
Centripetal force Fc =
= 38. As =ç ÷ and mA < mB \ t A < tB
R R tB è mB ø
\ Force acting on the particle i. e. , car A would stop in a shorter time.
2
æ 2as ö 39. Linear momentum of car B = mBvB
F = Ft2 + Fc2 = (2as) 2 + ç 2
÷ = 2as 1 + s / R
2
è R ø
= 2000 ´ 10 = 2 ´ 10 4 kg ms-1
31. As, E A = mghA + KA Linear momentum of C = mC vC = 2500 ´ 18 = 2 ´ 10 4 kg ms-1
EB = KB; EC = mghC + KC Thus PB = PC
xC mB 2000
Using conservation of energy \ = = = 0.8
xB mC 2500
E A = EB = EC ,
KB > KC \ xB > xC
i. e. , car B will travel large distance before stopping than car C.
if hA > HC ; KC > KA
dU B 2A
and if hA < hC ; KC < KA 40. From = -
dr r 2 r3
32. When a man of mass m climbs up the staircase of height L, d 2U 2B 6A
work done by the gravitational force on man is (-) mgL, and =- 3 + 4
dr 2 r r
work done by muscular force is mgL. If we ignore air
resistance and friction, then the work done by all forces on 2A d 2U 2B4 6AB4 B
As r ® r0 = , 2 =- 3+ = ,
man is equal to - mgL + mgL = zero. Further, force from a B dr 8A 16A4 8A3
step does not do work because the point of application of which is positive.
force does not move while the force exists. \ U is minimum.
1 Hence, the equilibrium is stable.
33. mv 2 = a(t )
2 41. The velocity of particle is zero, where total energy is
or v µ t 1/ 2 completely potential.
dv 3B2 A B
a= µ t -1/ 2 \ E=- = U(r) = 2 -
dt 16A r r
1 1 2r0 2 æ 2A ö 4 B
or Fµ and F µ at r= = ç ÷= -
t v 3 3è B ø 3 r
34. While spring is fully compressed, the entire KE of M1 is not 2r 2 æ 2A ö 4A
Þ r= 0 = ç ÷=
3 3 è B ø 3B,
stored as PE of spring as M2 may move. If spring is massless,
than as M1 = M2, velocities of M1 and M2 are interchanged on A B A9B2 B.3B 9B2 3B2 3B2
U(r) = - = - = - = -
collision.M1 comes to rest, instead of M2. Choice (c) is correct. r 2 r 16A2 4A 16A 4A 16A
If surface on which blocks are moving has friction loss of which is the total energy.
energy is involved . Collision cannot be elastic. Choice (d) is 2A A B AB2 B2 B2
correct. 42. At r = r0 = , U1 = 2 - = 2
=- =-
B r r 4A 2A 4A
1/ 2
t A æ mA ö At r = ¥ , U2 = 0
35. From =ç ÷
tB è mB ø \Work done to move the particle from equilibrium position
1/ 2
t A æ 1000 ö 1 to infinity,
=ç ÷ = æ B2 ö B2
5 è 2000 ø 2 W = U2 - U1 = 0 - ç - ÷=
è 4A ø 4A
tA = 5 / 2 s
Telegram @unacademyplusdiscounts

Work, Energy and Power 267

m v2
43. Always attractive. 55. Power = F × v = V æç ö÷ v =
è ø t (r AV )
44. Change in kinetic energy = Work done by net force. This
relationship is valid for particle as well as system of particles. = r AV = 100 (2)3 = 800 W
3

45. Work done and power developed is zero in uniform circular 56. ò F dt = D p
motion only.
1 1
46. These tells about work energy theorem. Þ ´ 4 ´ 3 - ´ 1.5 ´ 2 = pf - 0
2 2
47. As force = kx 9
Þ pf = 6 - 1.5 =
Greater the k greater will be force for constant x. 2
48. The mass may be converted into energy p2 81
KE = =
as E = mc2 2m 4 ´2 ´2

49. The exact reason is governed by E = mc2 KE = 5.06 J


1 57. Given, F = 3 x i + 4$j , r = x$i + y k$
2$
and not by E = mv 2
2
Let, dr = dx$i + dy $j
p2 p2 æ p2 ö
50. Maximum energy less = - çQKE = ÷ Work done W = ò F dr = ò
(3 , 0 )
(3 x2$i + 4$j) (dx $i + dy$j)
2m 2 (m + M) è 2m ø ( 23 )
Before collision the mass m and after collision the mass is (3 , 0 ) (3 , 0 )
=ò (3x2 dx + 4 dy) = ( x3 + 4 y)( 2, 3)
m+M ( 2, 3 )
p2 é M ù 1 ì M üæ M ö = 33 + 4 ´ 0 - (23 + 4 ´ 3)
= ê ú = mv 2 í ý çf = ÷
2m ë (m + m) û 2 î m + Mþ è m + Mø
= 27 + 0 - 20 = + 7 J
51. According to work-energy theorem, mgh
58. Power given to turbine =
W = change in kinetic energy t
1 1 æ mö
FS cos q = mv 2 - mv 2 Pin = ç ÷ gh = Pin = 15 ´ 10 ´ 60
2 2 èt ø
Substituting the given values, we get
Pin = 9000 W
20 ´ 4 ´ cos q = 40 - 0
40 1 Þ Pin = 9 kW
cos q = =
80 2 As efficiency of turbine is 90% therefore power generat
æ 1ö = 90% of 9 kW
q = cos-1 ç ÷ = 60°
è2ø 90
Þ Pout =9´
52. Here, force F = ( 4 $i + $j - 2 k)
$ N 100
Þ Pout = 8.1 kW
Velocity, v = (2 $i + 2$j+ 3 k)
$ ms-1

\ Power P = F × v = ( 4 $i + $j - 2 k)
$ (2 $i + 2$j+ 3 k)
$
v mv
59. Acceleration, a, and F = ma =
= (8 + 2 - 6) = 4 W T T
v
53. Q Speed is constant. Velocity acquired, V = at = t
T
\ Work done by force = 0
mv v
work Instantaneous power, P = F ´ v = ´ t
\ Power = =0 T T
time
mv 2
54. Here, m = 2 kg, v = 4 s-1 = t
T2
F = 15 N, k = 1000 Nms-1, x = ? dU
60. From F = -
KE spent = Work done against friction + PE of spring dx
1 1 dU = - Fdx
mv 2 = F ´ x + kx2 U( x ) x x
2 2 U=ò dU = - ò Fdx = - ò kxdx
1 1 0 0 0
´ 2 ´ 4 2 = 15x + ´ 10000 x2
2 2 kx2
U=
\ 5000 x2 + 15x - 16 = 0 2
On solving it, we get x = 0.055 m = 5.5cm As U(0) = 0 , µ x2 and U is negetive.
Telegram @unacademyplusdiscounts

8 Centre of Mass
JEE Main MILESTONE
< Centre of Mass < Centre of Mass (CM) of Continuous Bodies
< Position of Centre of Mass < Collision

8.1 Centre of Mass


Centre of mass of a system (body) is a point that moves when external forces are
applied on the body as though all the mass were concentrated at that point and all
external forces were applied there. So far, we have studies the motion
If particles of masses m1, m2, m3, ¼, mn are placed at x1, x2, x3, ¼, xn distances from of the particle treated as the point
the origin, then position of the centre of mass of this particle system will be given by of mass having practically no size.
n Any real body, which we come
å mixi across in our daily life has a finite
m1x1 + m2x2 + m3x3 + ¼ + mnxn i =1
X CM = = n size. Such as extended body
m1 + m2 + m3 + ¼ + mn
å mi consists of number of particles.
i =1

The position vector rCM of the centre of mass can be expressed in terms of position
vectors r1, r2, … of the particles as
n
å miri
m1r1 + m2r2 + ¼ i =1
rCM = = n
m1 + m2 + ¼
å mi
i =1

1 kg 1 kg
Sample Problem 1 Particles of mass 2 kg, 2 kg, 1 kg and 1 kg are D C
placed at the corners A, B, C and D of a square of side L as shown in
figure. Find the centre of mass of the system.
L L L
(a) (b)
3 2
(c) L (d) None of these A B
2 kg L 2 kg
Interpret (d) If A is taken as origin
m1x1 + m2x2 + m3 x3 + m4 x4
Then, X CM =
m1 + m2 + m3 + m4
2 ´ 0 + 2 ´ L + 1´ L + 1´ 0 1
= = L
6 2
Telegram @unacademyplusdiscounts

Centre of Mass 269


Y
Note that, if we put x1 = 0, then x2 becomes d and Eq. (ii)
D(0, L)
C(L, L) reduces to Eq. (i), as it must. Note also that inspite of the
1 kg shift of the coordinate system, the centre of mass is still at
the same distance from each particle.
We can rewrite Eq. (ii) as
m1x1 + m2x2
X xCM = ...(iii)
A(0, 0) B(L, 0) M
m1y1 + m2y 2 + m3y3 + m4y 4 In which M is the total mass of the system ( M = m1 + m2 )
YCM =
m1 + m2 + m3 + m4
2. System of Large Number of Particles
2 ´ 0 + 2 ´ 0 + 1´ L + 1´ L 1
= = L If we have a system consisting of n particles, of masses m1,
6 3
m2, ¼ mn with r1, r2, ¼, rn as their position vectors, at a given
Note The centre of mass of a body may lie within or outside the body. It instant of time, the position vector of CM, i.e., rCM of the
is not at all necessary that some mass has to be present at the centre of system at that instant is given by
mass. n n
å miri å miri
m1r1 + m2r2 + ¼ mnrn i =1 i =1
rCM = = =
8.2 Position of Centre of Mass m1 + m2 + ¼+ mn n
å mi
M
i =1
We start with a system of few particles. We can classify a
system of particles in two groups
n ü
å mixi ï
1. Two particles system i =1 ï
xCM =
2. System of large number of particles and M ï
n ï
Now let us take them one by one. å i i ïï
m y
i =1 ï
Further, yCM = ý …(iv)
1. Two Particles System M ï
n ï
Fig. (a) shows two particles of masses m1 and m2 separated å mizi ï
by a distance d. We have arbitrarily chosen the origin of zCM =
i =1 ï
the x-axis to coincide with the particle of mass m1. We M ï
ï
define the position of the centre of mass (CM) of this two ïþ
particle system to be
m2
xCM = d ...(i) Sample Problem 2 The position vectors of three particles of
m1 + m2
masses m1 = 1kg, m2 = 2 kg and m3 = 3 kg are r1 = ( $i + 4$j + k$ ) m,
y r2 = ( i$ + $j + k$ ) m and r3 = (2 $i + $j + 2 k$ ) m respectively. Find
x CM x CM
m1 m2 m1 m2 position vector of their centre of mass.
x 1 x 1
CM
x1
CM (a) (3$i + 2$j - 4 k$ ) (b) (3$i + $j - k$ )
d 3 2
d x2 1 1
(a) (b) (c) (3$i + $j - k$ ) (d) (3$i + 2$j - 4 k$ )
3 2
From the above discussion, we see that Interpret (b) The position vector of CM of the three parts will
d be given by
xCM = , if m1 = m, i. e. , CM lies mid-way between the two m1r1 + m2r2 + m3 r3
2 rCM =
m1 + m2 + m3
particles of equal masses.
Substituting the values, we get
Fig. (b) shows a more generalized situation, in which the
1 ( $i + 4$j + k$ ) + 2 ( $i + $j + k$ ) + 3 (2$i - $j - 2 k$ )
coordinate system has been shifted leftward. The position rCM =
1+ 2 + 3
of the centre of mass is now defined as
9$i + 3$j - 3 k$
m x + m2x2 =
xCM = 1 1 ...(ii) 6
m1 + m2 1 $ $ $
\ rCM = (3 i + j - k) m
2
Telegram @unacademyplusdiscounts

270 JEE Main Physics

Hot Spot Centre of Mass


(CM) of Continuous Bodies
An ordinary object, such as a baseball bat, contains so many particles æL ö
Therefore, the coordinates of centre of mass of the rod are ç , 0, 0 ÷
(atoms) that we can treat it as a continuous distribution of matter. The è2 ø
particles then become differential mass elements dm, and the lies at the centre of the rod.
coordinates of the centre of mass are defined as
Proceeding in the similar manner, we can find centre of mass of
1

xCM = x dm certain rigid bodies. Centre of mass of source well known rigid bodies
are given below.
1

y CM = y dm Centre of Mass of a Rigid Bodies
1 1. Centre of mass of a uniform rectangular, square or circular

zCM = z dm plate lies at its centre.

For uniform objects which have uniform density or mass per unit
volume, we have
dm M
r= = CM CM
dV V CM
where dV is the volume occupied by a mass element dm and V is the
total volume of the object. Thus, we find that 2. Centre of mass of a uniform semicircular ring lies at a
1 2R
xCM = ò x dV distance of h = from its centre, on the axis of symmetry,
V p
1 where R is the radius of the ring.
y CM = ò y dV
V
1 R CM 2R
zCM = ò z dV
V π
O
Centre of Mass of a Uniform Rod
3. Centre of mass of a uniform semicircular disc of radius R lies
Consider a rod of mass M and length L is lying along x-axis with its one
4R
end at x = 0 and the other end at x = L at a distance of h = from the centre on the axis of
3p
P Q
x symmetry as shown in figure.
x=0 x=L
dx
M CM
Mass per unit length of the rod = R 4R
L 3π
The mass of the element PQ of length dx is O
4. Centre of mass of a hemispherical shell of radius R lies at a
dm = dx
L distance of h = R /2 from its centre on the axis of symmetry as
shown in figure.
The coordinates of the element PQ are ( x, 0, 0 ).
\ x-coordinates of centre of mass of the rod will be
R CM R
L æM ö
L x dm ò0 ( x) çè L dx÷ø 2
xCM = ò = O
0 M M
1 L L 5. Centre of mass of a solid hemisphere of radius R lies at a
= ò x dx =
L 0 2 3R
distance of h = from its centre on the axis of symmetry.
The y-coordinate of centre of mass is 8

y CM =
ò y dm =0 (as y = 0)
M R CM 3R
8
Similarly, zCM = 0
O
Telegram @unacademyplusdiscounts

Centre of Mass 271

Sample Problem 3 Find the centre of mass of a uniform æ a ö


(a) ç - , 0 ÷ (b) (0, 0)
semicircular ring of radius R and mass M. è 6 ø
2R 3R æa ö æa aö
(a) 0 , 2 pR (b) 0 , (c) 0 , (d) 0 , pR (c) ç , 0 ÷ (d) ç , - ÷
p p è6 ø è6 6ø
Interpret (a) Consider the centre of the ring as origin. Consider Interpret (a) Here, A1 = area of complete circle = pa2 and
a differential element of length dl of the ring whose radius vector
makes an angle q with the X-axis. If the angle subtended by the A2 = area of small circle
2
length dl is dq at the centre, then, dl = R dq. æ aö p a2
=pç ÷ =
Let l be the mass per unit length. è2ø 4
Then, mass of this element is dm = l R dq
1 p ( x1, y1) = coordiantes of centre of mass of large circle
X CM = ò (R cos q) l R dq = 0 = (0 , 0)
m 0
1 p and ( x2, y 2) = coordinates of centre of mass of small circle
Þ YCM = ò (R sin q) × lR dq
m 0 æa ö
= ç ,0÷
lR 2 p lR 2 è2 ø
or = ò sin q dq = [ - cos q]p0
m 0 l pR A x - A2x2
Using, xCM = 1 1
2R A1 - A2
Þ YCM =
p - p a2 æ a ö
p ç ÷
As m = ò l R dq = 2 p R 4 è2ø
0 We get, xCM =
p a2
p a2 -
4
Sample Problem 4 Find the position of centre of mass of
the uniform lamina shown in figure is æ 1ö
-ç ÷
y è8ø a
= a =-
æ3ö 6
ç ÷
è 4ø
a
O x and y CM = 0 as y1 and y 2 both are zero. Therefore, the coordinates of
æ a ö
centre of mass of the limina shown in figure are ç - , 0 ÷ .
è 6 ø

The Position of Centre of Mass after Removal of a Part of a Body


If some mass or area is removed from a rigid body, then m1y1 - m2y2
\ yCM =
the position of centre of mass of the remaining portion is m1 - m2
obtained from the following formulae A1y1 - A2y2
or yCM =
m1r1 - m2r2 A1 - A2
(a) rCM =
m1 - m2 m1 z1 - m2 z2
and zCM =
m1 - m2
A r - A2r2
or rCM = 11 A1 z1 - A2 z2
A1 - A2 or zCM =
A1 - A2
m1 x1 - m2 x2
(b) xCM =
m1 - m2 Here, m1, A1, r1, x1, y1 and z1 are the values for the mass of
the whole body before the mass has been removed
A1 x1 - A2 x2
or xCM = while m2, A2, r2x2, y2 and z2 are the values for the mass
A1 - A2 which has been removed.
Telegram @unacademyplusdiscounts

272 JEE Main Physics

Sample Problem 5 A uniform metal disc of radius R is


taken out of it a disc of diameter is cut-off from the end. The Shift in the Position of Centre of Mass
centre of mass (CM) of the remaining part will be In the absence of an external force, the position of the
(a) R/3 from the centre (b) R /4 from the centre centre of mass of a stationary system does not change.
(c) R/6 from the centre (d) R /5 from the centre However, if under the influence of external forces, the
Interpret (c) The centre of mass of complete disc should lie at position of different particles of masses m1, m2, m3, ¼
point O. changes by Dr1, Dr2, Dr3, ¼, then shift in the position
vector of the centre of mass is given by
n
C1 O C2 å miDri
m1Dr1 + m2Dr2 + ¼ i =1
x x2 DrCM = = n
m1 + m2 + ¼
å mi
i =1

C1 is the position of centre of mass of remaining portion and C 2 the Sample Problem 7 A dog of mass 10 kg is standing on a
position of centre of mass of the removed disc. flat boat so that it is 20 m from the shore. It walks 8 m on the
x (Area of remaining portion) boat towards the shore and then stops. The mass of the boat is
R 40 kg and friction between the boat and the water surface is
= (Area of removed disc)
2 negligible. How far is the dog from the shore now?
æ 2 pR 2 ö R æ pR 2 ö
\ x ç pR - ÷= ç ÷
è 4 ø 2è 4 ø
R
\ x=
6 20 m

Sample Problem 6 A thin uniform circular disc of mass M (a) 13 m


has a radius R. From it a circular hole of radius R/2 is cut-off as (b) 13.6 m
shown in figure. Then centre of mass of the remaining part will (c) 14 m
be (d) 14.6 m
R Y
(a) Interpret (b) Take the boat and dog together as a system.
2 Initially centre of mass of the system is at rest. Since no external
R force is acting on the system, hence centre of mass of the system
(b) M
4 R will remain stationary.
R r=R
(c) 2 Let initially distance of the centre of mass of the boat from the shore
6 X
O O' is x m. Hence,
R
(d) 40 ´ x + 10 ´ 20
8 x1CM = ...(i)
40 + 10

Interpret (c) Let centre O of disc be the origin. Due to where, x1CM = distance of the centre of mass of the system from the
symmetry the centre of mass of the remaining part will be on the shore. Since dog moves towards the shore and for the centre of
X-axis. Here, mass of the cut-off portion is mass of the system to be at rest, the boat has to move away from the
2 shore. Let distance moved by the boat be x ¢, then x1CM
æRö
pç ÷ 40( x + x¢) + 10(20 - 8 + x¢)
è2ø M x2CM =
m= 2
M= 40 + 10
pR 4
R 40 x + 200 40 ( x + x¢) + 10 (12 + x¢)
and position of its centre of mass x = OO ¢ = Þ =
2 50 50
Hence, for remaining part
Þ 50 x ¢ = 80
æMö æRö
M ´0 - ç ÷ ç ÷
è 4ø è2ø R Þ x ¢ = 1.6 m
xCM = =-
M 6 Thus, distance of the dog from the shore = x + x¢
M-
4
= 12 + 1.6
Thus, centre of mass of the remaining part lies to the left of the
= 13.6 m
origin at a distance R/6.
Telegram @unacademyplusdiscounts

Centre of Mass 273

Velocity of the Centre of Mass of a y


200 g B (0.25, 0.25√3)
System of Particles
We know that position vector of the centre of mass of a
system of particles is given by
C ( 5 1
— , —–
18 3√3 )
m r + m2r2 + ¼+ mnrn 100 g 150 g
rCM = 1 1
m1 + m2 + ¼+ mn O (0, 0) A (0.5, 0)
Differentiating on both sides w.r.t. time, we obtain m1x1 + m2x2 + m3 x3
X=
dr dr dr m1 + m2 + m3
m1 1 + m2 2 + ¼+ mn n
drCM dt dt dt [100 (0) + 150 (0.5)+200 (0.25)] g
= X=
dt m1 + m2 + ¼+ mn (100 + 150 + 200) g
m1v1 + m2v2 + ¼+ mnvn 75 + 50 125 5
Þ vCM = X= = = m
m1 + m2 + ¼ + mn 450 450 18
[(100 (0) + 150 (0) + 200 (0.25 3)]
Y=
Acceleration of the Centre of Mass 450
of a System of Particles =
50 3
m=
3
m=
1
m
450 9 3 3
Velocity of the centre of mass of a system of particles
m v + m2v2 + ¼ mnvn
vCM = 1 1 Note Centre of mass is not the geometric centre.
m1 + m2 + ¼ mn

Differentiating the above equation w.r.t. t, we get Sample Problem 9 The centre of mass of a uniform
dv dv dv
L-shaped lamina (a thin flat plate) with dimensions as shown is
m1 1 + m2 2 + ¼+ mn n (g mass of lamina is 3 kg)
dvCM dt dt dt
= y
dt m1 + m2 + ¼+ mn
2m
F + F2 + ¼+ Fn S Fext F (0, 2) E (1, 2)
= 1 =
M M C3 D(1, 1)
and if total net force acting on a system of particles is zero, B(2, 1)
then aCM = 0. Hence, in the absence of any net external C1 1m
C2
force acting on a system, the centre of mass of the system x
O(0, 0) A(2, 0)
is either at rest or in uniform motion along a given straight
line.
æ6 5ö æ5 5ö æ5 6ö æ7 6ö
(a) ç , ÷ (b) ç , ÷ (c) ç , ÷ (d) ç , ÷
Sample Problem 8 Three particles of masses 100 g, 150 g è5 6ø è6 6ø è6 5ø è5 5ø
and 200 g are placed at the vertices of an equilateral triangle.
Each side of the triangle is 0.5 m long. The centre of mass of the Interpret (b) Taking the L-shape to consist of 3 squares each of
three particles is [NCERT Exemplar] length 1m. The mass of each square is 1kg, since the lamina is
æ18 ö uniform. The centre of mass C1, C 2 and C3 of the squares are, by
(a) ç , 3 3 ÷
è5 ø æ 1 1ö
symmetry their geometric centres and have coordinates ç , ÷ ,
è2 2ø
æ 1 5ö
(b) ç , ÷ æ 3 1ö æ 1 3 ö
è 3 3 13 ø ç , ÷ , ç , ÷ respectively. We take the masses of the squares to
è2 2ø è2 2ø
æ5 1 ö
(c) ç , ÷ be concentrated at these points. The centre of mass of the whole L
è18 3 3 ø
shape ( x, y) is the centre of mass of these mass points.
æ18 1 ö
(d) ç , ÷ é æ 1ö æ3ö æ 1ö ù
è5 13 ø ê1 çè 2 ÷ø + 1 çè 2 ÷ø + 1 çè 2 ÷ø ú kg -m 5
Hence, X = ë û = m
Interpret (c) With the x and y-axes chosen the coordinates of (1 + 1 + 1) kg 6
points O , A and B forming the equilateral triangle are respectively é æ 1ö æ 1ö æ3öù
(0, 0), (0.5, 0), (0.25, 0.25 3). Let the masses 100 g, 150 g and 200 g ê1 çè 2 ÷ø + 1 çè 2 ÷ø + 1 çè 2 ÷ø ú kg -m 5
be located at O , A and B. Then, and Y=ë û = m
(1 + 1 + 1) kg 6
Telegram @unacademyplusdiscounts

274 JEE Main Physics

Fig. (c) The spring will compress till velocity of both the
8.3 Collision blocks become equal. So, at maximum compression (say
Contrary to the meaning of the term ‘collision’ in our xm) velocities of both the blocks are equal (say v).
everyday life, in physics it does not necessarily mean one Fig. (d) Spring force is still in the directions shown
particle ‘striking’ against other. Indeed two particles may in figure, i.e., velocity of block m1 is further
not even touch each other and may still be said to collide. decreased and that of m2 is increased. The spring now
All that is implied is that as the particles approach each starts relaxing.
other,
Fig. (e) The two blocks are separated from one another.
(i) an impulse (a large force for a relatively short time) acts
Velocity of block m2 becomes more than the velocity of
on each colliding particles.
block m1, i.e., v2¢ > v1¢ .
(ii) the total momentum of the particles remain conserved.
Assuming spring to be perfectly elastic following two
The collision is in fact a redistribution of total momentum of the equations can be applied in the above situation.
particles. Thus, law of conservation of linear momentum is (i) In the absence of any external force on the system the
indispensible in dealing with the phenomenon of linear momentum of the system will remain
collision between particles. Consider a situation shown in conserved before, during and after collision, i.e.,
figure.
m1v1 + m2v2 = (m1 + m2 )v
Two blocks of masses m1 and m2 are moving with
= m1v1 ¢ + m2v2 ¢ …(i)
velocities v1 and v2 (< v1 ) along the same straight line in a
smooth horizontal surface. A spring is attached to the (ii) In the absence of any dissipative forces, the
mechanical energy of the system will also remain
block of mass m2.
conserved, i.e.,
Now, let us see what happens during the collision 1 1 1 1 2
between two particles. m1v12 + m2v22 = (m1 + m2 ) v2 + kxm
2 2 2 2
v1 v2 F F
1 1
= m1v1¢ 2 + m2v2¢ 2 …(ii)
m1 m2 m1 m2 2 2

v1 > v2
Note In the above situation, we have assumed spring to be perfectly elastic,
(a) (b) i.e., it regains its original shape and size after the two blocks are separated.
In actual practice, there is no such spring between the two blocks. During
v v F F collision both the blocks (or bodies) are a little bit deformed. This situation
is similar to the compression of the spring. Due to deformation two equal
m1 m2 m1 m2 and opposite forces act on both the blocks. These two forces redistribute
their linear momentum in such a manner that both the blocks are
Maximum compression = xm separated from one another.
(c) (d) The collision is said to be elastic, if both the blocks regain their original
shape and size completely after they are separated.
v1' v2' On the other hand, if the blocks do not return to their original form the
collision is said to be inelastic. If the deformation is permanent and the
m1 blocks move together with same velocity after the collision, the collision is
said to be perfectly inelastic.
v1' < v2'
(e) Sample Problem 10 Two blocks A and B of equal mass
m =1.0 kg are lying on a smooth horizontal surface as shown in
Fig. (a) Block of mass m1 is behind m2. Since, v1 > v2, the figure. A spring of force constant k = 200 N/m is fixed at one end
blocks will collide after some time. of block A. Block B collides with block A with velocity v0 = 2.0
m/s. Find the maximum compression of the spring.
Fig. (b) The spring is compressed. The spring force F (= kx )
2.0 m/s
acts on the two blocks in the directions shown in figure.
This force decreases the velocity of m1 and increases the B A
velocity of m2.
Telegram @unacademyplusdiscounts

Centre of Mass 275

Interpret At maximum compression ( xm ), velocity of both the Further, a collision is said to be head on (or direct), if the
blocks is same, say it is v. Applying conservation of linear directions of the velocity of colliding objects are along the
momentum, we have, line of action of the impulses, acting at the instant of
(mA + mB)v = mBv 0 collision. If just before collision, at least one of the
or (1.0 + 1.0) v = (1.0) v 0 colliding objects was moving in a direction different from
v 2.0
or v= 0= the line of action of the impulses, the collision is called
2 2
oblique or indirect.
=1.0 m/s
Using conservation of mechanical energy, we have
1 1 1 2 (i) Head on Elastic Collision
mBv 02 = (mA + mB ) v 2 + kxm
2 2 2 Let the two balls of mass m1and m2, collide each other
Substituting the values, we get elastically with velocities v1 and v2 in the directions shown
1 1 1 in Fig. (a). Their velocities become v1 ¢ and v2 ¢ after the
2
´ (1) ´ (2.0) 2 = ´ (1.0 +1.0) ´ (1.0) 2+ ´ ( 200 ) ´ xm
2 2 2 collision along the same line. Applying conservation of
2
or 2 = 1.0 + 100 xm linear momentum, we get
or xm = 0.1 m = 10.0 cm m2 m1
v2 v1

Types of Collision (a)


Collision between two bodies may be classified in two Before collision
ways m2
v'2
m1
v'1
1. Elastic collision and inelastic collision.
2. Head on collision or oblique collision. (b)
After collision
The collision between two bodies is said to be elastic if
both the bodies come to their original shape and size after m1v1 + m2v2 = m1v1 ¢ + m2v2 ¢ …(i)
the collision, i.e., no fraction of mechanical energy
In an elastic collision, kinetic energy before and after
remains stored as deformation potential energy in the
collision is also conserved. Hence,
bodies. Thus, in addition to the linear momentum, kinetic
1 1 1 1
energy also remains conserved before and after collision. m1v12 + m2v22 = m1v1 ¢2 + m2v2 ¢2 …(ii)
2 2 2 2
On the other hand, in an inelastic collision, the colliding Solving Eqs. (i) and (ii) for v1 ¢ and v2 ¢, we get
bodies do not return to their original shape and size
æ m - m2 ö æ 2 m2 ö
completely after collision and some part of the v1 ¢ = ç 1 ÷ v1 + ç ÷ v2 …(iii)
è m1 + m2 ø è m1 + m2 ø
mechanical energy of the system goes to the deformation
potential energy. Thus, only linear momentum remains æ m - m1 ö æ 2 m1 ö
and v2 ¢ = ç 2 ÷ v2 + ç ÷ v1 …(iv)
conserved in case of an inelastic collision. è m1 + m2 ø è m1 + m2 ø
Telegram @unacademyplusdiscounts

276 JEE Main Physics

Special Cases
Case I If m1 = m2, then from Eqs. (iii) and (iv), we can see that æ m ö
With these two substitutions çv1 = 0 and 2 = 0÷
v1 ¢ = v2 and v2 ¢ = v1 è m1 ø

i.e., when two particles of equal mass collide We get the following two results,
elastically and the collision is head on, they exchange v1 ¢ » 0
their velocities., e.g. and v2 ¢ » - v2
4 m/s 3 m/s i.e., the particle of mass m1 remains at rest while the
m m
particle of mass m2 bounces back with same speed
Before collision v2 .

3 m/s 4 m/s Case III If m2 > > m1 and v1 = 0


m m v1 = 0
After collision v2 m1
m2
2 m/s v=0
m m Before collision

v1′ ≈ 0
Before collision v2′ ≈ – v2
v2
v=0 m2 m1
2 m/s
m m
After collision
After collision
m1
With the substitution » 0 and v1 = 0, we get the results
Case II If m1 > > m2 and v1 = 0. m2
v1 = 0
v1 ¢ » 2v2
v2 m1
m2 and v2 ¢ » v2

Before collision i.e., the mass m1 moves with velocity 2v2, while the
velocity of mass m2 remains unchanged.
v1′ ≈ 0
v2′ ≈ – v2
v2 Note It is important to note that Eqs. (iii) and (iv) and their three special
m2 m1
cases can be used only in case of a head-on elastic collision between
After collision
two particles. Many students apply these two equations even if the
collision is inelastic and do not apply these relations where clearly a head
m2
Then, »0 on elastic collision is given in the problem.
m1

2m 2 2
Sample Problem 11 Two particles of mass m and or v 2¢ = v- v =0
moving in opposite directions collide elastically with velocities 3 3
v and 2v. Find their velocities after collision. 3v
2m m
Interpret Here, v1 = - v ,v 2 = 2v ,m1 = m and m2 = 2m
2v v
2m m +ve i.e., the second particle (of mass 2m) comes to a rest while the first
(of mass m) moves with velocity 3v in the direction shown in figure.

Substituting these values in Eqs. (iii) and (iv), we get Sample Problem 12 Two pendulum bobs of mass m and
æ m - 2m ö æ 4m ö 2m collide elastically at the lowest point in their motion. If both
v1¢ = ç ÷ ( - v) + ç ÷ (2v)
è m + 2m ø è m + 2m ø the balls are released from a height H above the lowest point, to
v 8v what heights do they rise for the first time after collision?
or v1¢ = + = 3v
3 3 Interpret Given, m1 = m,m2 = 2m,
æ 2m - m ö æ 2m ö v1 = - 2 gH and v 2 = 2 gH
and v 2¢ = ç ÷ (2v) + ç ÷ ( - v)
è m + 2m ø è m + 2m ø
Telegram @unacademyplusdiscounts

Centre of Mass 277

Note Since the collision is elastic, mechanical energy of both the balls
will remain conserved, or
Ei = Ef
Þ ( m + 2m) gH = mgh1 + 2mgh 2
+ve
3mgH = ( mg ) æç
25 ö
H ÷ + ( 2mg ) æç ö÷
2 1 H
Þ
è 9 ø è9ø
v2 v1 Þ 3mgH = 3mgH
Since, the collision is elastic. Using Eqs. (iii) and (iv) discussed in the
theory the velocities after collision are (ii) Head on Inelastic Collision
æ m - 2mö æ 4m ö In an inelastic collision, the particles do not regain their
v1¢ = ç ÷ ( - 2 gH ) + ç ÷ 2 gH
è m + 2 mø è m + 2 mø shape and size completely after collision. Some fraction of
2 gH 4 2 gH 5 mechanical energy is retained by the colliding particles in
= + = 2 gH the form of deformation potential energy. Thus, the kinetic
3 3 3
energy of the particles no longer remains conserved.
æ 2m - mö æ 2m ö
and v 2¢ = ç ÷ ( 2 gH ) + ç ÷ ( - 2 gH ) However, in the absence of external forces, law of
è m + 2 mø è m + 2 mø
conservation of linear momentum still holds good.
2 gH 2 2 gH 2 gH v2 v1
= - =- m2 m1
3 3 3
i.e., the velocities of the balls after the collision are as shown in Before collision
figure.
v2′ v1′
m2 m1
+ve
After collision

Suppose the velocities of two particles of mass m1 and m2


before collision be v1 and v2 in the directions shown in
figure. Let v1¢ and v2¢ be their velocities after collision. The
v2' =
√2gH v1' = 5 √2gH law of conservation of linear momentum gives
3
3
v′
Therefore, the heights to which the balls rise after the collision are
(v1¢ ) 2
h1 = (using, v 2 = u 2 - 2 gh)
2g
2 m1v1 + m2 v 2 = m1v¢1 + m2 v 2¢ …(v)
æ5 ö
ç 2 gH ÷
è3 ø Collision is said to be perfectly inelastic, if both the
or h1 =
2g particles stick together after collision and move with same
velocity, say v¢ as shown in figure. In this case, Eq. (v) can be
25 (v ¢ ) 2
written or
as h1 = H and h = 2
9 2g
2
m1v1 + m2v2 = (m1 + m2 )v¢
æ 2gH ö m v + m2v 2
ç ÷ or v¢ = 1 1 …(vi)
è 3 ø H m1 + m2
or h2 = or h2 =
2g 9
Telegram @unacademyplusdiscounts

278 JEE Main Physics

Newton’s Law of Restitution Special Cases


When two objects are in direct (head on) impact, the
Case I
speed with which they separate after impact is usually
less than or equal to their speed of approach before If collision is elastic, i.e., e = 1, then
impact. æ m - m2 ö æ 2m2 ö
v1¢ = ç 1 ÷ v1 + ç ÷ v2
Experimental evidence suggests that the ratio of these m
è 1 + m 2ø è m1 + m2 ø
relative speeds is constant for two given set of objects. This æ m - m1 ö æ 2m1 ö
and v2¢ = ç 2 ÷ v2 + ç ÷ v1
property formulated by Newton, is known as the law of è m1 + m2 ø è m1 + m2 ø
restitution and can be written in the form
which are same as Eqs. (iii) and (iv).
separation speed
=e …(vii) Case II If collision is perfectly inelastic, i.e., e = 0, then
approach speed
m1v1 + m2v2
v1¢ = v2¢ = = v¢ (say )
The ratio e is called the coefficient of restitution and is m1 + m2
constant for two particular objects.
which is same as Eq. (vi).
In general, 0£ e£1
Case III If m1 = m2 and v1 = 0, then
e = 0, for completely inelastic collision, as both the objects
stick together. So, their separation speed is zero or e = 0 m2 m1
v
from Eq. (vii). v1 = 0

e = 1, for an elastic collision, as we can show from Before collision


Eqs. (iii) and (iv), that v2' v1'

v1 ¢ - v2 ¢ = v2 - v1
After collision
or separation speed = approach speed
or e=1 æ 1+ e ö
v1¢ = ç ÷v
Let us now find the velocities of two particles after è 2 ø 2
collision, if they collide directly and the coefficient of æ 1- e ö
and v2¢ = ç ÷v
restitution between them is given as e. è 2 ø 2
m2 m1
v2
v1 Note
(i) If mass of one body is very-very greater than that of the other,
Before collision then after collision velocity of heavy body does not change
v2′ v1′ appreciably. (Whether the collision is elastic or inelastic).
u=0
After collision
h
Applying conservation of linear momentum
m1v1 + m2v2 = m1v1 ¢ + m2v2 ¢ …(viii)
Further, separation speed = e (approach speed) v0 = √2gh
or v1¢ - v2¢ = e (v2 - v1 ) …(ix) (ii) In the situation shown in figure, if e is the coefficient of restitution
Solving Eqs. (viii) and (ix), we get between the ball and the ground, than after nth collision with the
æ m - em2 ö æ m + em2 ö floor the speed of ball will remain e nv 0 and it will go upto a height
v1¢ = ç 1 ÷ v1 + ç 2 ÷ v2 …(x)
è m1 + m2 ø è m1 + m2 ø e 2nh or,
v n = e nv 0 = e n 2gh
æ m - em1 ö æ m + em1 ö
and v2¢ = ç 2 ÷ v2 + ç 1 ÷ v1 …(xi) and h n = e 2nh
è m1 + m2 ø è m1 + m2 ø
Telegram @unacademyplusdiscounts

Centre of Mass 279

Elastic Collision in Two Dimensions or cos q =


2 v12 sin2 q
The collisions are two dimensional, where the initial and 2 v1v2 cos f
final velocities may lie in a plane. Thus, when two bodies æ v ö sin2 q
travelling initially along the same straight line collide = ç 1÷ …(viii)
è v2 ø cos f
without loss of KE and move along different directions in a
plane after collision, then the collision is said to be elastic Now, cos (q + f) = cos q cos f - sin q sin f
collision in two dimensions. v1 sin2 q v
v1 \ cos (q + f ) = cos f - 1 sin2 q = 0
y v2 cos f v2
m1 \ q + f = 90° …(viii)
m2 θ v2
A X A B
u1 B φ
B at rest
v2
y′ \ q + f = 90° …(ix)
Let m1 and m2 are two masses of body and their velocities Before collision
are u1 and u2 respectively. If u1 > u2, then two bodies are v1
collide. A
Let velocity of body A = v1 at angle q along x-axis and
velocity of body where, B = v2 at angle f along x-axis. 90°

where, q is angle of scattering and f is angle of recoil


B
As the collision inelastic, KE is conserve.
v2
\ Total KE after collision = Total KE before collision After collision
1 1 1 1
m1v12 + m2v22 = m1u12 + m2u22
2 2 2 2
or m1v12 + m2v22 = m1u12 + m2u22 …(i)
Inelastic Collision in Two Dimension
When two bodies travelling initially along the same
Total linear moment after collision,
straight line collide involving some loss of KE, and move
m1v1 cos q + m2v2 cos f = m1u1 + m2u2 …(ii) after collision, along the different directions in a plane, the
The linear momentum along y-axis is zero. collisions is said to be inelastic collision in
\ m1v1 sin q - m2v2 sin f = 0 …(iii) two-dimension. The total linear momentum of system
A special case two objects of same mass under go remains constant, i. e. ,
perfectly elastic collision in two-dimensions. p f = pi
i. e. , m1 = m2 \ m1v1 cos q + m2v2 cos f = m1v1 + m2v2 …(x)
For this, let u1 = u and v2 = 0 As initial momentum of two bodies along y-axis is zero.
\ From Eq. (i), v12 + v22 = u2 …(iv) Then, law of conservation of linear momentum along
y-axis.
From Eq. (ii), v1 cos q + v2 cos f = u …(v)
m1v1 sin q - m2v2 sin f = 0 …(xi)
and from Eq. (iii),
v1 sin q - v2 sin f = 0 …(vi) As the system is closed and isolated, then total linear
momentum of system remains constant.
Using Eq. (v), we get from Eq. (iv)
\ m1v1 cos q + m2v2 = (m1 + m2 ) v cos f …(xii)
v12 + v22 = (v1 cos q + v2 cos f ) 2 = v12 cos2 q + v22 cos2 f
m1 y
+ 2 v1v2 cos q cos f v1
or v12 2
(1 - cos q) + v22 2
(1 - cos f) = 2 v1v2 cos q cos f θ m2 u2
x
O φ
or v12 sin2 q + v2 sin2 f = 2v1v2 cos q cos f …(vii)
v
y'
From Eq. (vi),
v2 sin f = v1 sin q
\ Laws of conservation of linear momentum along y-axis,
Putting in Eq. (vii), we get we get
2 v12 sin2 q = 2 v1v2 cos q cos f m1v1 sin q = (m1 + m2 ) v sin f
Telegram @unacademyplusdiscounts

280 JEE Main Physics

Sample Problem 13 Consider the collision as shown in Speed of C just after collision is
the figure, to be between two billiand balls with equal masses (a) 2 m/s (b) 2 2 m/s
m1 = m2. (c) 5 m/s (d) ( 2 - 1) m/s
y
v1f Interpret (a) As, the balls A and B are constrained to move
m1 horizontally (immediately after collision), if I be the impulse
v1i θ1 imparted by ball C to each of A and B, the impulse received by ball C
m1 m2 θ2 from them would be 2 I cos q. Now, each of ball B and C received
impulse I but moves horizontally as its vertical component gets
balanced by impulse imparted to ball B and C by respective strings,
m2 hence
v2f

The first ball is called the cue while the second ball is called the I I
target. The billiard player wants to sink the target ball in a θ
corner pocket, which is at angle q 2 = 37°. Assume that the C
collision is elastic and that friction and rotational motion are
not important, then value of q 1 is
(a) 37° (b) 43° (c) 53° (d) 22° A
B
θ
Interpret (c) From the principle of conservation of momentum,
we have
I I
m1v1i = m2v1f + m2v 2f
Given, m1 = m2, I cos q = MAv A = MBvB
We have v1i = v1f + v 2f M v
Þ I= A A (I = magnitude of impulse)
or v12i = ( v1f + v 2f ) × ( v1f + v 2f ) cos q

= v12f + v 22f + 2 v1f × v 2f Now, for ball C, if its final velocity v c¢ downwards, we have
mcv c¢ = mcv c - 2 I cos q
= v12f + v 22f + 2 v1f v f cos ( q1 + 37° ) …(i)
2 Ma
Þ v c¢ = v c - vA
Since, the collision is elastic and m1 = m2 , it follows from Mc
conservation of kinetic energy that
Given, v c = 10 m/s, ma = 2 kg, mc = 1kg, v A = 3 m/s
v12f = v12f + v 22f …(ii) 2
Þ v c¢ = 10 - 2 ´ ´ 3
Comparing Eqs. (i) and (ii), we have 1
cos ( q1 + 37° ) = 0 Þ v c¢ = -2 m/s
q1 + 37° = 90° (–ve sign indicates that it is directed upwards)
q1 = 53°
Sample Problem 15 A ball of mass m moving with a
Note This proves that when two equal masses undergo a glancing speed v makes a head-on collision with an identical ball at rest.
elastic collision with one of them at rest, after the collision, they will move
The kinetic energy after collision of the balls is three-fourths the
at right angles to each other.
original kinetic energy. The coefficient of restitution (e) is
Sample Problem 14 Two identical balls A and B each of 1 1 1 1
(a) (b) (c) (d)
mass 2 kg and radius R, are suspended vertically from 2 3 2 3
inextensible strings as shown. Third ball C of mass 1 kg and Interpret (c) From law of conservation of momentum is
radius r = ( 2 - 1) R falls and hits A and B symmetrically with
mv = mv1 + mv 2
10 m/s. Speed of both A and B just after the collision is 3 m/s. or v = v1 + v 2 …(i)
v -v
1 kg which gives, e= 2 1
v -0
g
m m m
v

Before collision
R R v2′ v1′

2 kg 2 kg
After collision
A B
Telegram @unacademyplusdiscounts

Centre of Mass 281

or v 2 - v1 = ev …(ii) Sample Problem 17 A ball is moving with velocity 2 m/s


Adding Eqs. (i) and (ii), we get towards a heavy wall moving towards the ball with speed 1 m/s
v + ev (1 - e) v as shown in figure. Assuming collision to be elastic, find the
v2 = and v1 =
2 2 velocity of ball immediately after the collision.
3 mv 2 m é v 2 (1 + e) 2 (1 - e) 2 v 2 ù
= ê + ú
4 2 2 ë 4 4 û 2 m/s 1 m/s
Þ 3 = (1 + e2) + (1 - e) 2 = 2 (1 + e2)
1 1
Þ e2 = or e =
2 2
Interpret The speed of wall will not change after the collision.
Sample Problem 16 A ball falls on an inclined plane of So, let v be the velocity of the ball after collision in the direction
inclination q from a height h above the point of impact and shown in figure. Since, collision is elastic ( e = 1),
makes a perfectly elastic collision. Where will it hit the inclined
plane again
8h 4h 2 m/s 1 m/s v 1 m/s
(a) (b) 8hsinq (c) 4hsinq (d)
sinq sinq

Interpret (b) We have,


v = 2 gh …(i) Before collision After collision

Applying conservation of momentum the ball will recoil with separation speed = approach speed
velocity v making an angle q with the perpendicular to the plane. or v -1= 2 + 1
Along horizontal direction or v = 4 m/s
g
AP = v sin qt + sin q t 2 …(ii) Sample Problem 18 After perfectly inelastic collision
2
between two identical particles moving with same speed in
different directions, the speed of the particles becomes half the
θ θ initial speed. Find the angle between the two before collision.
(a) 120° (c) 140°
A
(c) 90° (d) 100°
Interpret (a) Let q be the desired angle. Linear momentum of
θ the system will remain conserved. Hence,
P
P 2 = P12 + P22 + 2P1P2 cos q
Along vertical direction displacement is zero
v
1 2m
0 = v cos qt - g cos q t 2 v
2 m
θ 2
2v m
Þ t= …(iii)
g
v
Substituting the value of t from Eq. (iii) in Eq. (ii), we get 2
2 ì æ v öü 2 2
æ2v ö g æ2v ö or í2m çè ÷øý = (mv) + (mv) + 2(mv)(mv) cos q
AP = v sin q ç ÷ + sin q ç ÷ î 2 þ
è g ø 2 è g ø
1
or 1 = 1 + 1 + 2 cos q or cos q = -
4 v2 2
= sin q = 8 h sin q along the incline
g \ q = 120°
Telegram @unacademyplusdiscounts

WORKED OUT
Examples
Example 1 A block of mass m slides with velocity v along a Solution X CM =
å mi xi = m ´ 0 + m ´ 0 + m ´ 2 = 2
frictionless level surface towards a block of mass 4m initially at å mi m+m+m 3
rest. The velocity of centre of mass is
(a) v / 5 (b) v / 4 YCM =
å myi i = m ´ 2 + m ´ 0 + m ´ 0 = 2
(c) 5v / 2 (d) ( 4 / 5)v å mi m+m+m 3
2 $ $
Solution Here, m1 = m, v1 = v \ Position vector of centre of mass is ( i + j).
3
m2 = 4m, v 2 = 0 , v CM = ?
m v + m2v 2 Example 4 Two blocks of mass 5 kg and 2 kg are placed on
v CM = 1 1 a frictionless surface and connected by a spring. An external
m1 + m2
kick gives a velocity of 14 ms-1 to the heavier block in the
m ´ v + 4m ´ 0 v
= = direction of lighter one. Calculate the velocity gained by the
m + 4m 5 centre of mass.
(a) 14 ms-1 (b) 7 ms-1 (c) 10 ms-1 (d) zero
Example 2 The centre of mass of two particles with masses
4 kg and 2 kg located at (1,0,1) and (2,2,0) respectively has Solution We know that,
coordinates. m v + m2v 2 5 ´ 14 + 2 ´ 0
v CM = 1 1 = = 10 ms-1
(a) (1/3, 2/3, 2/3) (b) (4/3, 1/3, 1/3) m1 + m2 5+2
(c) (2/3, 1/3, 1/3) (d) (4/3, 2/3, 2/3)
Example 5 A circular plate of diameter 56 cm. A circular
Solution Here, m1 = 4 kg, m2 = 2 kg
portion of diameter 43 cm is removed from one edge of the plate
x1 = 1, y1 = 0, z1 = 1, x2 = 2, y 2 = 2, z 2 = 0 as shown in the figure. The centre of mass of the remaining
The coordinates of centre are portion is at what distance from the centre of the given plate?
m x + m2x2 4 ´ 1 + 2 ´ 2 4
x= 11 = =
m1 + m2 4+2 3
m1y1 + m2y 2 4 ´ 0 + 2 ´ 2 2
y= = = O1
m1 + m2 4+2 3 O2 O
A
X
m x + m2z 2 4 ´ 1 + 2 ´ 0 2
z = 11 = =
m1 + m2 4+2 3

Example 3 Three identical spheres, each of mass m are


placed at the corners of a right angled triangle with mutually (a) 10 cm (b) 9 cm (c) 8 cm (d) 7 cm
perpendicular sides equal to 2m. Taking their point of
intersections as the origin, the position vector of centre of Solution Let O be the centre of circular plate and O1, the centre
mass is of circular portion removed from the plate. Let O 2 be the centre of
mass of the remaining part.
1 $ $ A 2
(a) ( i - j) æ 56 ö
3 Area of original plate = pR 2 = p ç ÷ = (28) 2 p cm2
2 è2ø
(b) ( $i - $j)
3 Area removed from circular part = pr 2
2 $ $ 2m æ 42 ö
(c) ( i + j) = p ç ÷ = (21) 2 p cm2
3 è 2 ø
1 Let s be the mass per cm 2. Then mass of original plate
(d) ( $i + $j) O 2m B
3
m = (28) 2 s p
Telegram @unacademyplusdiscounts

Centre of Mass 283

Mass of the removed part, m1 = (21) 2 sp Solution Since on striking the wooden block, the bullet comes
Mass of the remaining part, m2 = (28) 2sp - (21) 2sp = 343sp to rest w.r.t. the block of wood, the collision is inelastic in nature.
Therefore, the principle of conservation of energy will not hold for
Now the masses m1 and m2 may be supposed to be concentrated
this collision i.e., only the principle of conservation of momentum
at O1 and O 2 respectively.
will hold.
Their combined centre of mass is at O. Taking O as the origin we
Here, mass of the bullet, M1 = 0.012 kg
have from definition of centre of mass
m x + m2x2 Mass of the block, M2 = 0.4 kg
xCM = 1 1
m1 + m2 Initial velocity of the bullet, u1 = 70 ms -1
x1 = OO1 = OA - O1 A = 28 - 21 = 7 cm Initial velocity of the bullet, u2 = 0
x2 = OO 2 = ?, xCM = 0 Let v be the velocity acquired by the block and bullet together.
Þ m1v1 = m2v 2 = p Then, according to the principle of conservation of momentum,
Energy of explosion = DKEsystem M1 u1 + M2 u2 = (M1 + M2) v
p2 p2 p 2 é m1 ù M1 u1 + M2 u2
= + = 1+ or v=
2m1 2m2 2m1 êë m2 úû M1 + M2
Putting p = m1v1 0.012 ´ 70 + 0.4 ´0
=
where, m1 = 3 kg,m2 = 6 kg,v = 16 ms–1 0.012 ´ 0.4
(3 ´ 16) 2 é 3ù = 2.04 ms -1
We obtain, E= êë1 + 6 úû
2 ´3
If the block and the bullet rise to a height h, then
E = 576 J 1
(M1 + M2)v 2 = (M1 + M2) gh
2
Example 6 A bullet of mass 0.012 kg and horizontal speed
70 ms-1 strikes a block of wood of mass 0.4 kg and instantly v 2 (2.04) 2
or h= =
comes to rest inside the block. The block is suspended from the 2g 2 ´ 9.8
ceiling by means of thin wires. Calculate the height to which = 0.212 m
the block rises.
= 21.2 cm
(a) 21.2 cm (b) 25.3 cm (d) 28.4 cm (d) 29.5 cm
Telegram @unacademyplusdiscounts

Start Practice for


~
JEE Main
Round I (Topically Divided Problems)

Centre of Mass 5. Which of the following points is the likely position of


1. A cricket bat is cut at the location of its centre of mass the centre of mass of the system shown in figure.
[NCERT Exemplar]
as shown. Then,
Hollow sphere
Air

(a) the two pieces will have the same mass R/2
A
(b) the bottom piece will have larger mass B
(c) the handle piece will have larger mass
C
(d) mass of handle piece is double the mass of bottom piece R/2

2. Three rods of the same mass are placed as shown in


figure. What will be the coordinates of centre of mass of Sand
the system? (a) A (b) B
(0, a) (c) C (d) D
6. Find the velocity of centre of the system shown in the
figure.
1 kg 2 ms–1
(a, 0) y
O

é a aù é a a ù
(a) , (b) ê , x′ x 2 kg
êë 2 2 úû ë 2 2 úû 30°
é a aù
(c) [ 2 a, 2 a ] (d) ,
ëê 3 3 ûú y′
2 ms–1
3. Which of the following does the centre of mass lie
outside the body? [NCERT Exemplar] æ2 + 2 3ö $ 2 $
(a) ç ÷i - j (b) 4 $i
(a) A pencil è 3 ø 3
(b) A shotput æ2 - 2 3ö $ 2 $
(c) A dice (c) ç ÷i - j (d) None of these
è 3 ø 3
(d) A bangle
7. Four particles of mass 1 kg, 2 kg, 3 kg and 4 kg are
4. A man of mass M stands at one end of a plank of
placed at the corners A, B, C and D respectively of
length L which is at rest on a frictionless horizontal
a square ABCD of edge X-axis and edge AD is
surface. The man walks to the other end of the plank.
taken along Y-axis, the coordinates of centre of mass
If mass of the plank is M/3, the distance that the man
in SI unit is
moves relative to ground is
(a) (1, 1) m (b) (5, 7) m
(a) L (b) L/4
(c) (0.5, 0.7) m (d) None of these
(c) 3L/4 (d) L/3
Telegram @unacademyplusdiscounts

Centre of Mass 285

8. Three bricks each of length L Wall 15. In the HCL molecule, the separation between the
and mass M are arranged as nuclei of the two atoms is about 1.27 Å (1 Å = 10-10
shown from the wall. The m). Find the approximate location of the centre of
distance of the centre of mass L/4 mass of the molecule, given that a chlorine atom is
of the system from the wall is L/2
about 35.5 times as massive as a hydrogen atom and
(a)
L
(b)
L L nearly all the mass of an atom is concentrated in its
4 2 nucleus. [NCERT]
3 11 (a) rCM = 1.24 Å (b) rCM = 2.24 Å
(c) L (d) L
2 12 (c) rCM = 0.24 Å (d) rCM = 3.24 Å
9. Two blocks A and B are connected by a massless 16. When a meteorite burns in the atmosphere, then
string (shown in figure). A force of 30 N is applied on (a) the momentum conservation principle is applicable to the
block B. The distance travelled by centre of mass in meteorite system
2 s starting from rest is (b) the energy of meteorite remains constant
B (c) the conservation principle of momentum is applicable to
A a system consisting of meteorites, earth and air molecules
20 kg (d) the momentum of meteorite remains constant
10 kg F = 30 N

Smooth
17. A particle of mass m f

(a) 1 m (b) 2 m
moving with a velocity v
f0
(c) 3 m (d) None of these
makes an elastic one
dimensional collision with
10. A circular hole of radius 1 cm is cut-off from a disc of a stationary particle of
radius 6 cm. The centre of hole is 3 m from the centre
mass establishing a O
of the disc. The position of centre of mass of the T/4 3T/4 T
contact with it for
remaining disc from the centre of disc is
3 1
extremely small time T. Their force of contact
(a) - cm (b) cm increase from zero to f0 linearly in time T / 4 remains
35 35
T
3 constant for the further time and decrease
(c) cm (d) None of these 2
10 T
linearly from f0 to zero in further time as shown.
11. A non-uniform thin rod of length L is placed along 4
X-axis as such its one of ends is at the origin. The The magnitude possessed by f0 is
linear mass density of rod is l = l0 x. The distance of mv 2 mv 4 mv 3 mv
centre of mass of rod from the origin is (a) (b) (c) (d)
T T 3T T
L 2L L L
(a) (b) (c) (d)
2 3 4 5 18. The density of a non-uniform rod of length 1m is
given by r ( x) = a (1 + bx2 ) where a and b are constants
12. Two boys of masses 10 kg and 8 kg are moving along a
vertical rope. The former climbing up with and 0 £ x £ 1. The centre of mass of the rod will be at
acceleration of 2 ms–1 while later coming down with [NCERT Exemplar]
uniform velocity of 2 ms–1. Then tension in rope at 3 (2 + b) 4 (2 + b) 3 (2 + b) 4 (3 + b)
(a) (b) (c) (d)
fixed support will be (Take g = 10 ms–2) 4(3 + b) 3 (3 + b) 4 (2 + b) 3 (2 + b)
(a) 200 N (b) 120 N (c) 180 N (d) 160 N 19. If momentum of a body remains constant, then
13. A body of mass M moving with velocity v ms–1 mass-speed graph of body is
suddenly breaks into two pieces. One part having (a) circle (b) straight line
mass M/4 remains stationary. The velocity of the (c) rectangular hyperbola (d) parabola
other part will be 20. A machine gun fires a steady stream of bullets at the
3v 4v rate of n per minute into a stationary target in which
(a) v (b) 2 v (c) (d)
4 3 the bullets get beaded. If each bullet has a mass ma
14. A particle of mass M is moving in a horizontal circle and arrive at the target with a velocity v, the average
of radius R with uniform speed v. When it moves from force on the target is
one point to a diametrically opposite point, its 60 v
(a) 60 mnv (b)
(a) momentum does not change mn
(b) momentum changes by 2Mv mnv mv
(c) (d)
(c) kinetic energy changes by Mv 2 60 60 n
(d) None of the above
Telegram @unacademyplusdiscounts

286 JEE Main Physics

21. A body in equilibrium may not have 27. When two blocks A and B coupled by a spring on a
(a) momentum (b) velocity frictionless table are stretched and then released,
(c) acceleration (d) kinetic energy then
22. A bomb is kept stationary at a point. It suddenly (a) kinetic energy of body at any instant after releasing is
inversely proportional to their masses
explodes into two fragments of masses 1g and 3g. The
(b) kinetic energy of body at any instant may or may not be
total kinetic energy of the fragments is 6.4 ´ 10 4 J.
inversely proportional to their masses
What is the kinetic energy of the smaller fragment? KE of B mass of B
(a) 2.5 × 104 J (b) 3.5 × 104 J (c) = , when spring is massless
KE of A mass of A
(c) 4.8 × 104 J (d) 5.2 × 104 J
(d) both (b) and (c) are correct
23. In the given figure, two bodies of mass m1 and m2 are 28. An isolated particle of mass m is moving in a
connected by massless spring of force constant k and
horizontal plane (x-y), along the x-axis, at a certain
are placed on a smooth surface (shown in figure),
height above the ground. It suddenly explodes into
then
k two fragments of masses m/4 and 3m/4. An instant
F m1 m2 F later, the smaller fragment is at y = +15 cm. The
larger fragment at this instant is at
(a) the acceleration of centre of mass must be zero at every
instant (a) y = – 5 cm (b) y = + 20 m
(b) the acceleration of centre of mass may be zero at every (c) y = + 5 cm (d) y = –20 cm
instant 29. A bomb at rest explodes in air into two equal
(c) the system always remains in rest fragments. If one of the fragments is moving
(d) None of the above vertically upwards with velocity v0 , then the other
24. A particle moves in the xy-plane under the action of a fragment will move
force F such that the value of its linear momentum p (a) vertically up with velocity v 0
at any time t is px = 2 cos t, py = 2 sin t. The angle q (b) vertically down with velocity v 0
between F and p at a given time t will be (c) in arbitrary direction with velocity v 0
(a) 90º (b) 0º (c) 180º (d) 30º (d) horizontally with velocity v 0

25. A set of n identical cubical blocks lies at rest parallel


to each other along a line on a smooth horizontal Collision
surface. The separation between the near surfaces of 30. Which of the following is not a perfectly inelastic
any two adjacent blocks is L. The block at one end is collision?
given a speed v towards the next one at time t = 0. All (a) Striking of two glass ball
collisions are completely elastic. Then (b) A bullet striking a bag of sand
k (c) An electron captured by a proton
m2 m1 F
(d) A man jumping onto a moving car
A B
1 31. A ball of mass m moving with velocity v collides with
(a) the last block starts moving at time t = ( a - 1)
v another ball of mass 2m and sticks to it. The velocity
( n - 1) L of the final system is
(b) the last block starts moving at time t =
2v (a) v/3 (b) v/2
(c) the centre of mass of the system will have a final speed v (c) 2v (d) 3v
(d) the centre of mass of the system will have a final 32. An example of inelastic collision is
speed v/n (a) scattering of a-particle from a nucleus
26. Two blocks of masses m1 and m2 are connected by a (b) collision of ideal gas molecules
massless spring and placed at smooth surface. The (c) collision of two steel balls lying on a frictionless table
spring initially stretched and released. Then (d) collision of a bullet with a wooden block
(a) the momentum of each particle remains constant separately 33. In an elastic collision
(b) the magnitude of momentum of both bodies are same to (a) only KE of system is conserved
each other
(b) only momentum is conserved
(c) the mechanical energy of system remains constant
(c) both KE and momentum are conserved
(d) both (b) and (c) are correct
(d) neither KE nor momentum is conserved
Telegram @unacademyplusdiscounts

Centre of Mass 287

34. A body of mass 3 kg is moving with a velocity of 41. Two particles of masses m1 and m2 in projectile
4 ms -1 towards right, collides head on with a body of motion have velocities v1 and v2 respectively at time
mass 4 kg moving in opposite direction with a t = 0. They collide at time t0 . Their velocities become
velocity of 3 ms -1. After collision the two bodies stick v1 and v2 at time 2 t0 while still moving in air. The
together and move with a common velocity, which is value of [( m1v1 + m2v 2 ) - ( m1v1 - m2v 2 )] is
(a) zero (b) 12 ms–1 towards left (a) zero (b) ( m1 + m2 ) gt 0
12 –1 1
(c) 12 ms–1 towards right (d) ms towards left (c) 2 ( m1 + m2 ) gt 0 (d) ( m1 + m2 ) gt 0
7 2
35. A sphere of mass m moving with a constant velocity v 42. In an elastic head on collision between two particles
hits another stationary sphere of same mass. If e is (a) velocity of separation is equal to the velocity of approach
the coefficient of restitution, then the ratio of velocity (b) velocity of the target is always more than the velocity of
of two spheres after collision will be the projectile
1- e 1+ e (c) the maximum velocity of the target is double to that of
(a) (b)
1+ e 1- e the projectile
e +1 e -1 2 (d) maximum transfer of kinetic energy occurs when masses
(c) (d) t
e -1 e +1 of both projectile and target are equal
36. Two perfectly elastic objects A and B of identical 43. In a one dimensional collision between two identical
mass are moving with velocities 15 ms -1 and 10 ms -1 particles A and B, where B is stationary and A has
respectively, collide along the direction of line joining momentum p before impact. During impact B gives
them. Their velocities after collision are respectively an impulse J to A. Then coefficient of restitution
(a) 10 ms–1, 15 ms–1 (b) 20 ms–1, 5 ms–1 between the two is
(c) 0 ms–1, 25 ms–1 (d) 5 ms–1, 20 ms–1 2J 2J
(a) -1 (b) +1
p p
37. Two spherical bodies of the same mass M are moving
J J
with velocities v1 and v2 . These collide perfectly (c) + 1 (d) - 1
p p
inelastically,
1 1 44. A body of mass 2 kg moving with a velocity of 3 m/s
(a) M ( v1 - v2 ) 2 (b) M ( v21 - v22 )
2 2 collides head-on with a body of mass 1 kg moving in
1 opposite direction with a velocity of 4 m/s. After
(c) M ( v1 - v2 ) 2 (d) 2 M ( v21 - v22 )
4 collision two bodies stick together and moves with a
38. Two equal masses m1 and m2 moving along the same common velocity which in m/s is equal to [NCERT]
straight line with velocities + 3 m/s and - 5 m/s 1 1
(a) (b)
respectively collide elastically. Their velocities after 4 3
the collision will be respectively 2 3
(c) (d)
(a) + 4 m/s for both (b) - 3 m/s and + 5 m/s 3 4
(c) - 4 m/s and + 4 m/s (d) - 5 m/s and + 3 m/s 45. A body is dropped and observed to bounce a height
39. A particle of mass m collides with another stationary greater than the dropping height. Then
particle of mass M. If the particle m stops just after (a) the collision is elastic
collision, the coefficient of restitution for collision is (b) there is additional source of energy during collision
equal to (c) it is not possible
m (d) this type of phenomenon does not occur in nature
(a) 1 (b)
M 46. A particle of mass m moving with a velocity
M-m m (3 $i + 2 $j ) ms -1 collides with a stationary body of mass M
(c) (d)
M+m M+m and finally moves with a velocity ( -2 $i + $j ) ms -1. If
m 1
40. A bullet of mass m hits a target of mass M hanging by = , then
a string and gets embedded in it. If the block rises to a M 13
height h as a result of this collision, the velocity of the (a) the impulse received by M is m (5 i$ + $j)
bullet before collision is 1
(b) the velocity of the M is (5 $i + $j)
æ mö 13
(a) v = 2 gh (b) v = 2 gh ç1 + ÷ 11
è Mø (c) the coefficient of restitutions
17
æ Mö æ mö
(c) v = ç1 + ÷ 2 gh (d) v = 2 gh ç1 - ÷ (d) All of the above are correct
è mø è Mø
Telegram @unacademyplusdiscounts

288 JEE Main Physics

47. A small block of mass M moves with velocity 5 ms -1 51. A body at rest breaks up into 3 parts. If 2 parts
towards an another block of same mass M placed at a having equal masses fly off perpendicularly each
distance of 2 m on a rough horizontal surface. after with a velocity of 12 m/s, then the velocity of the
Coefficient of friction between the blocks and ground third part which has 3 times mass of each part is
is 0.25. Collision between the two blocks is elastic, (a) 4 2 m/s at an angle of 45° from each body
the separation between the blocks, when both of (b) 24 2 m/s at an angle of 135° from each body
them come to rest, is (g = 10 ms -2 ) (c) 6 2 m/s at 135° from each body
(a) 3 m (b) 4 m (d) 4 2 m/s at 135° from each body
(c) 2 m (d) 1.5 m
52. Two bodies A and B of definite shape (dimensions of
48. A thick uniform bar lies on a frictionless horizontal bodies are not ignored). A is moving with speed of
surface and is free to move in any way on the surface. 10 ms–1 and B is in rest, collides elastically. The
Its, mass is 0.16 kg and length is 1.7 m. Two particles
(a) body A comes to rest and B moves with speed of 10 ms -1
each of mass 0.08 kg are moving on the same surface
(b) they may move perpendicular to each other
and towards the bar in the direction perpendicular to
(c) A and B may come to rest
the bar, one with a velocity of 10 ms -1 and other with
(d) they must move perpendicular to each other
velocity 6 ms -1. If collision between particles and bar
is completely inelastic, both particles strike with the
bar simultaneously. The velocity of centre of mass Coefficient of Restitution
after collision is 53. If a ball is dropped from rest, its bounces from the
(a) 2 ms -1 floor. The coefficient of restitution is 0.5 and the
(b) 4 ms -1 speed just before the first bounce is 5 ms–1. The total
(c) 10 ms -1 time taken by the ball to come to rest is
(d) 167 ms -1 (a) 2 s (b) 1 s
(c) 0.5 s (d) 0.25 s
49. In the given figure four identical spheres of equal
mass m are suspended by wires of equal length l0 , so 54. Three identical blocks A, B and C are placed on
that all spheres are almost touching to one other. If horizontal frictionless surface. The blocks A and C
the sphere 1 is released from the horizontal position are at rest. But A is approaching towards B with a
and all collisions are elastic, the velocity of sphere 4 speed 10 ms–1. The coefficient of restitution for all
just after collision is collisions is 0.5. The speed of the block C just after
collision is
A B C

(a) 5.6 ms–1 (b) 6 ms–1


(c) 8 ms–1 (d) 10 ms–1
1 2 3 4
55. A smooth steel ball strikes a fixed smooth steel plate
(a) 2 gl0 at an angle q with the vertical. If the coefficient of
(b) 3 gl0 restitution is e, the angle at which the rebounce will
take place is
(c) gl0
æ tan q ö
g l0 (a) q (b) tan -1 ç ÷
(d) è e ø
2 æ e ö
(c) e tan q (d) tan -1 ç ÷
50. A ball moving with a certain velocity hits another è tan q ø
identical ball at rest. If the plane is frictionless and 56. A tennis ball bounces down flight of stairs striking
collision is elastic, the angle between the directions each step in turn and rebounding to the height of the
in which the balls move after collision, will be step above. The coefficient of restitution has a value
(a) 30º (b) 60º
(a) 1/2 (b) 1
(c) 90º (d) 120º
(c) 1 / 2 (d) 1 / 2 2
Telegram @unacademyplusdiscounts

Centre of Mass 289

Round II (Mixed Bag)

Only One Correct Option 6. Two balls each of mass m are palced on the vertices A
1. ( n - 1) equal point masses each of mass m are placed and placed at vertex C. The centreB of an equilateral
at the vertices of a regular n-polygon. The vacant triangle ABC of side 1 m. A ball of mass 2 m is of mass
vertex has a position vector a with respect to the of this system from vertex A (located at origin) is
centre of the polygon. Find the positive vector of C
m
centre of mass [NCERT Exemplar]
1 1
(a) a (b) - a 1m 1m
n -1 ( n - 1)
æ n - 1ö
(c) ( n - 1) a (d) - ç ÷
è a ø m m
A 1m B
2. A gas molecule of mass m strikes the wall of the
æ1 1 ö æ1 ö
container with a speed v at an angle q with the normal (a) ç m, m÷ (b) ç m, 2 m÷
è2 2 ø è2 ø
to the wall at the point of collision. The impulse of the
gas molecule has a magnitude æ1 3 ö æ 3 3 ö
(c) ç m, m÷ (d) ç m, m÷
(a) 3mv (b) 2 mv cos q è 2 4 ø è 4 4 ø
(c) mv (d) zero 7. A neutron travelling with velocity u and kinetic
3. A body of mass M at rest explodes into three pieces, energy K collides head on elastically with the nucleus
two of which of mass M/4 each are thrown off in of an atom of mass number A at rest. The fraction of
mutually perpendicular directions with speeds of its kinetic energy retained by the neutron even after
3 ms–1 and 4 ms–1 respectively. Then the third piece the collision is
2 2 2 2
will be thrown off with a speed of æ1 - A ö æ A + 1ö æ A - 1ö æ A + 1ö
(a) ç ÷ (b) ç ÷ (c) ç ÷ (d) ç ÷
(a) 1.5 ms–1 (b) 2 ms–1 è A + 1ø è A - 1ø è A ø è A ø
(c) 2.5 ms–1 (d) 3.0 ms–1 8. Three identical balls A, B and C are lying on a
4. A particle is projected with 200 ms -1, at an angle of horizontal frictionless table as shown in figure. If ball
60º. At the highest point it explodes into three A is imparted a velocity v towards B and C and the
particles of equal masses. One goes vertically upward collisions are perfectly elastic, then finally
with velocity 100 ms -1, the second particle goes v
vertically downward with the same velocity as the A B C
first. Then, what is the velocity of the third particle?
(a) 120 ms–1 with 60º angle (a) ball A comes to rest and balls B and C roll out with speed
(b) 200 ms–1 with 30º angle v/2 each
(c) 50 ms–1 vertically upwards (b) balls A and B are at rest and ball C rolls out with speed v
(d) 300 ms–1 horizontally (c) all the three balls roll out with speed v/3 each
5. From a unifrom disc of radius R, a circular section of (d) all the three balls come to rest
R R 9. In a two blocks system an initial velocity v (with respect
radius is cut out. The centre of the hole is at from
2 2 to the ground) is given to block A. Choose the correct
the centre of the original disc. Locate the centre of statement.
gravity of the resulting flat body. [NCERT]
A v Rough
R
(a) to the right of centre O B
6
R
(b) to the right of centre O Smooth
3
R (a) The momentum of block A is not conserved
(c) to the left of centre O (b) The momentum of system of blocks A and B is conserved
3
R (c) The increase in momentum of B is equal to the decrease
(d) to the left of centre O in momentum of block A
6
(d) All of the above
Telegram @unacademyplusdiscounts

290 JEE Main Physics

10. Two blocks of masses 14 ms –1 15. Two small particles of each v A


10 kg and 4 kg are equal masses start moving in 2v
connected by a spring of 10 kg 4 kg opposite direction from a
negligible mass and point A in a horizontal
placed on a frictionless circular orbit. Their
horizontal surface. An impulsive force gives a tangential velocities are v and
velocity of 14 ms–1 to the heavier block in the 2v respectively as shown in the figure between
direction of the lighter block. The velocity of centre of collisions, the particles move with constant speeds.
mass of the system at that moment is After making how many elastic collisions other than
(a) 30 ms–1 (b) 20 ms–1 (c) 10 ms–1 (d) 5 ms–1 that at A. These two particles will again reach the
11. A bullet of mass 0.01 kg and travelling at a speed of point A
500 ms–1 strikes a block of mass 2 kg, which is (a) 4 (b) 3 (c) 2 (d) 1
suspended by a string of length 5 m. The centre of 16. A bullet of mass m is fired with a velocity of 50 ms–1
gravity of the block is found to rise a vertical distance at an angle q with the horizontal. At the highest point
of 0.1 m. What is the speed of the bullet after it of its trajectory, it collides head-on with a bob
emerges from the block? connected with a massless string of length l = 10 / 3 m
(a) 580 ms–1 (b) 220 ms–1 and gets embedded with the bob. After the collision,
(c) 1.4 ms–1 (d) 7.8 ms–1 the string moves to an angle of 120°. What is the
12. The two bodies of mass m1 and m2 ( m1 > m2 ) angle q ?
respectively are tied to the ends of a
massless string, which passes over a
light and frictionless pulley. The 10 m 120°
masses are initially at rest and then l=
3
released. Then acceleration of the 50 ms–1
centre of mass of the system is
2 a2 T
æ m - m2 ö
(a) ç 1 ÷ g
è m1 + m2 ø æ 4ö æ5ö
m2 (a) cos -1 ç ÷ (b) cos -1 ç ÷
2 è5ø è 4ø
æ m - m2 ö T
(b) ç 1 ÷ æ 4ö æ5ö
è m1 + m2 ø (c) sin -1 ç ÷ (d) sin -1 ç ÷
m1
è5ø è 4ø
(c) g
(d) zero 17. A bullet of mass M hits a block of mass M ¢. The
energy transfer is maximum, when
13. A spherical hollow is made in a
(a) M ¢ = M (b) M ¢ = 2 M
lead sphere of radius R such that
(c) M ¢ << M (d) M ¢ >> M
its surface touches the outside
surface of lead sphere and passes O O1
18. A nucleus ruptures into two nuclear parts which
through the centre. What is the have their velocity ratio equal to 2 : 1. What will be
shift in the centre of mass of lead the ratio of their nuclear size?
sphere as a result of this (a) 21/3 : 1 (b) 1 : 21/3 (c) 31/2 : 1 (d) 1 : 31/2
hollowing? 19. Four identical spheres each of mass M and radius
R R R 10 cm, each are placed on a horizontal surface
(a) (b) (c) (d) R
7 14 2 touching one another so that their centres are located
14. Two identical blocks A and B, each of mass m resting at the corners of a square of side 20 cm. What is the
on smooth floor are connected by a light spring of distance of their centre of mass from centre of any of
natural length L and spring constant k with the the spheres ?
spring at its natural length. A third identical block C (a) 5 cm (b) 10 cm
(mass m) moving with a speed v along the line joining (c) 20 cm (d) 10 2 cm
A and B collides with A. The maximum compression 20. A bullet of mass 50 g is fired from a gun of mass 2 kg.
in the spring is If the total kinetic energy produced is 2050 J, the
m v mv mv kinetic energy of the bullet and the gun respectively
(a) v (b) m (c) (d)
2k 2k K 2k are
Telegram @unacademyplusdiscounts

Centre of Mass 291

(a) 200 J, 5 J (b) 2000 J, 50 J 27. The masses of five balls at rest and lying at equal
(c) 5 J, 200 J (d) 50 J, 2000 J distances in a straight line are in geometrical
21. Two spherical bodies of masses M and 5M in free progression with ratio 2 and their coefficients of
space with initial separation between their centres restitution are each 2/3. If the first ball be started
equal to 12 R. If they attract each other due to towards the second with velocity u, then the velocity
gravitational force only, then the distance covered by communicated to 5th ball is
2 3 4
the smaller body just before collision is 5 æ5ö æ5ö æ5ö
(a) u (b) ç ÷ u (c) ç ÷ u (d) ç ÷ u
(a) 2.5 R (b) 4.5 R (c) 7.5 R (d) 1.5 R 9 è9ø è9ø è9ø

22. A bullet of mass m leaves a gun of mass M kept on a 28. Two bodies having masses m1 and m2 and velocities
smooth horizontal surface. If the speed of the bullet u1 and u2 collide and form a composite system of
relative to the gun is v, the recoil-speed of the gun m1v1 + m2 v2 = 0 ( m1 ¹ m2 ). The velocity of the
will be composite system is
m m M M u1 + u2
(a) v (b) v (c) v (d) v (a) zero (b) u1 + u2 (c) u1 - u2 (d)
M M+m M+m m 2
23. A system of three particles having masses m1 = 1 kg, 29. Two carts on horizontal straight rails are pushed
m2 = 2 kg and m3 = 4 kg respectively is connected by apart by an explosion of a powder charge Q placed
two light springs. The acceleration of the three between the carts. Suppose the coefficient of friction
particles at any instant are 1 ms–2, 2 ms–2 and between carts and rails are identical. If the 200 kg
0.5 ms–2 respectively directed as shown in the figure. cart travels a distance of 36 m and stops, the distance
The net external force acting on the system is covered by the cart weighing 300 kg is
(a) 1 N (b) 7 N (c) 3 N (d) 6 N (a) 32 m (b) 24 m (c) 16 m (d) 12 m
24. A loaded spring gun of mass M fires a shot of mass m 30. Three identical sphere lie at rest along a line on a
with a velocity v at an angle of elevation q . The gun smooth horizontal surface. The separation between
was initially at rest on a horizontal frictionless surface. any two adjacent spheres is L. The first sphere is
After firing, the centre of mass of gun-shot system moved with a velocity u towards the second sphere at
mv time t = 0. The coefficient of restitution for collision
(a) moves with a velocity
M between any two blocks is 1/3. Then choose the
mv correct statement.
(b) moves with a velocity in the horizontal direction
M cos q 5L
(a) The third sphere will start moving at t =
(c) remains at rest 2u
(M - m) v 4L
(d) moves with velocity in the horizontal direction (b) The third sphere will start moving at t =
(M + m) u
25. In the shown figure the magnitude of acceleration of (c) The centre of mass of the system will have a final
centre of mass of the system is ( g = 10 ms -2 ) speed u/3.
(d) The centre of mass of the system will have a final speed u
µ = 0.2 5 kg
31. Three stationary particles A, B, C of masses m A , mB
and mC are under the action of same constant force
for the same time. If m A > mB > mC , the variation of
momentum of particles with time for each will be
5 kg
correctly shown as
A C
(a) 4 ms -2 (b) 10 ms -2 (c) 2 2 ms -2 (d) 5 ms -2 p p
(a) B (b) B
26. A bomb travelling in a parabolic path under gravity, C A
explodes in mid air. The centre of mass of fragments
will move O
t
O
t
(a) vertically upwards and then downwards
(b) vertically downwards A, B, C p
p
(c) in an irregular path (c) (d)
A, B, C
(d) in the parabolic path as the unexploded bomb would
have travelled
O O
t t
Telegram @unacademyplusdiscounts

292 JEE Main Physics

32. A ball strikes a horizontal floor at an angle q = 45°. 38. A set on n identical cubical blocks lies at rest parallel
The coefficient of restitution between the ball and the to each other along a line on a smooth horizontal
floor is e = 1/2. The fraction of its kinetic energy lost in surface. The separation between the near surface of
collision is any two adjacent block is L. The block at one end is
(a) 5/8 (b) 3/8 (c) 3/4 (d) 1/4
given a speed v towards the next one at time t = 0. All
33. A ball falls freely from a height of 45 m. When the collisions are completely inelastic, then
ball is at a height of 25m, it explodes into two equal ( n - 1)
(a) the last block starts moving at t = L
pieces. One of them moves horizontally with a speed v
of 10 ms–1. The distance between the two pieces when n ( n - 1) L
(b) the last block starts moving at t =
both strike the ground is 2v
(a) 10 m (b) 20 m (c) the centre of mass of the system will have a final speed v
(c) 15 m (d) 30 m v
(d) the centre of mass of the system will have a final speed
n
More Than One Correct Option
34. A body of mass 2 kg moving with a speed of 3 ms–1 Comprehension Based Questions
collides with a body of mass 1 kg moving with a speed Passage I
of 4 ms–1. If the collision is one dimensional and We sometimes encounter examples where a large
completely inelastic, the speed of composite mass force acts for very short duration of time producing an
after the collision may be appreciable and finite change in linear momentum of
3 –1 2 –1 the body. Such forces are know as impulsive forces. As
(a) ms (b) ms
2 3 an example, consider two identical cricket balls of
10 mass m and initial speed u, approaching a rigid wall.
(c) 4 ms–1 (d) ms–1
5 One ball strikes the wall normally and rebounds with
35. A man of mass m is standing at one end of a boat of same speed. Another ball strikes the wall making and
mass  M and length L. The body walks to the other angle of 30° from normal and is elastically reflected
end, the displacement of the back as shown in figure. Now answer the following
(a) centre of mass of the system is zero questions.
m u
(b) boat is L
M+m
m u 30°
(c) man is L u
M+m 30°
m
(d) boat is L u
M
Ball I Ball II
36. The velocity of the centre of mass of a two particle
system is v and total mass of particles is M. The
kinetic energy of the system 39. What is the magnitude of force on the wall due to first
1 2 ball ?
(a) may be equal to Mv
2 (a) mu (b) 2 mu
(b) must be equal to or less than Mv2 (c)
mu
(d) Data insufficient
1 2
(c) may be equal to or greater than Mv2
2 40. The ratio of magnitudes of the impulse imparted to
(d) cannot be exactly calculated as the information given is the two balls is
insufficient
2 3 1
(a) (b) (c) (d) 2
37. A ball hits the floor and rebounds after an inelastic 3 2 2
collision. In this case choose the correct statement(s).
41. The force exerted on the wall by ball I and ball II is
(a) The momentum of the ball just after the collision is the (a) parallel to the wall for both the balls
same as that just before the collision
(b) normal to the wall for both the balls
(b) The mechanical energy of the ball remains the same in
(c) normal to the wall for ball number 1 and inclined at 30º
the collision
to the horizontal for 2nd ball
(c) The total energy of the ball and earth is conserved
(d) normal to the wall for 1st ball and inclined at 30º to the
(d) The total momentum of the ball and earth is conserved vertical for 2nd ball
Telegram @unacademyplusdiscounts

Centre of Mass 293

Passage II 44. Assertion The centre of mass of a two particle system


lies on the line joining the two particles, being closer
A tennis ball is dropped from a height h0 on a
to the heavier particle.
horizontal marble flooring. The ball rebounds to a
height h1, then again falls on the floor, again rebounds Reason This is because product of mass of one
and so on. particle and its distance from centre of mass is
numerically equal to product of mass of other particle
42. The maximum height of rebound hn after n rebounds
and its distance from centre of mass.
will be
(a) ne × h0 (b) n e × h0 (c) e n × h0 (d) e2 n × h0 45. Assertion A quick collision between two bodies is
more violent than slow collision even when initial
43. If h0 = 10 m and e = 1/2, then compute the total and final velocities are identical.
distance travelled by the ball before it stops
Reason The rate of change of momentum
bouncing.
50 25
determines that force is small or large.
(a) ¥ (b) 50 m (c) m (d) m
3 3 46. Assertion The centre of mass of an electron and
proton, when released moves faster towards proton.
Assertion and Reason Reason This is because proton is heavier.
47. Assertion Torque is time rate of change of a
Directions (Q. Nos. 44 to 48) Assertion-Reason type. Each of
parameter, called angular momentum.
these contains two Statements : Statement I (Assertion), Statement II
(Reason). Each of these questions also has four alternative choice, only Reason This is because in linear motion, force
one of which is correct. You have to select the correct choices from the represents time rate of change of linear momentum.
codes (a), (b), (c) and (d) given below 48. Assertion Two particles moving in the same
(a) If both Assertion and Reason are true and the Reason direction do not lose all their energy in a completely
is correct explanation of the Assertion inelastic collision.
(b) If both Assertion and Reason are true but Reason is
not correct explanation of the Assertion
Reason Principle of conservation of momentum
(c) If Assertion is true but Reason is false
holds true for all kinds of collisions.
(d) If Assertion is false but the Reason is true

Previous Years’ Questions


49. Which of the following statement(s) is wrong? 52. A block C of mass m is moving with velocity v0 and
[UP SEE 2009] collides elastically with block A of mass m and
(a) KE of a body is independent of the direction of motion connected to another block B of mass 2m through
(b) In an elastic collision of two bodies, the momentum and spring constant k. What is k? If x0 is compression of
energy of each body is conserved spring when velocity of A and B is same? [UP SEE 2006]
(c) If two protons are brought towards each other, the
C 0 A the systemBdecreases v
potential energy of
(d) A body cannot have energy without momentum
mv 20 mv 20 3 mv 20 2 mv 20
50. The acceleration of the centre of mass of a uniform (a) (b) (c) (d)
solid disc rolling down an inclined plane of angle a is x02 2x20 2 x02 3 x02
[UP SEE 2008] 53. A bullet of mass 20 g and moving with 600 ms -1
(a) g sin a collides with a block of mass 4 kg hanging with the
(b) 2/3 g sin a string. What is velocity of bullet when it comes out of
(c) 1/2 g sin a block, if block rises to height 0.2 m after collision?
(d) 1/3 sin a [UP SEE 2006]
51. A 10 kg object collides with stationary 5 kg object and (a) 200 ms–1 (b) 150 ms–1
after collision they stick together and move forward (c) 400 ms–1 (d) 300 ms–1
with velocity 4 ms -1. What is the velocity with which 54. Consider a rubber ball freely falling from a height
the 10 kg object hit the second one? [BVP 2007] h = 4.9 m onto a horizontal elastic plate. Assume
(a) 4 ms–1 (b) 6 ms–1 that the duration of collision is negligible and the
(c) 10 ms–1 (d) 12 ms–1 collision with the plate is totally elastic. Then the
Telegram @unacademyplusdiscounts

294 JEE Main Physics

velocity as a function of time the height as function of 56. A body A of mass M while falling vertically
time will be [AIEEE 2009] downwards under gravity breaks into two parts, a
v v body B of mass 1/3 M and a body C of mass 2/3 M. The
+v1 centre of mass of bodies B and C taken together shifts
v1
compared to that the body A towards [AIEEE 2005]
(a) (b)
O t (a) depends on height of breaking
O t
–v1 (b) does not shift
(c) body C
v
v
+v1
(d) body B
+v1
(c) O 3t1 (d) O 3t1 57. A mass m moves with a velocity v and collides
t t
t1 2t1 4t1 t1 2t1 4t1 inelastically with another identical mass. After
v
–v1 –v1 collision the 1st mass moves with velocity in a
3
direction perpendicular to the initial direction of
55. Consider a two particle system with particles having motion. Find the speed of the 2nd mass after collision.
masses m1 and m2 . If the first particle is pushed [AIEEE 2005]
towards the centre of mass through a distance d, by (a) v
what distance should the second particle be moved,
(b) 3 v
so as to keep the centre of mass at the same position?
[AIEEE 2006] 2
(c) v
m2 m1 3
(a) d (b) d
m1 m1 + m2 v
(d)
m 3
(c) 1 d (d) d
m2

Answers
Round I
1. (b) 2. (d) 3. (d) 4. (c) 5. (c) 6. (a) 7. (c) 8. (d) 9. (b) 10. (a)
11. (b) 12. (a) 13. (d) 14. (b) 15. (a) 16. (c) 17. (c) 18. (a) 19. (c) 20. (c)
21. (c) 22. (c) 23. (a) 24. (a) 25. (d) 26. (d) 27. (c) 28. (a) 29. (b) 30. (a)
31. (b) 32. (d) 33. (c) 34. (a) 35. (a) 36. (a) 37. (c) 38. (d) 39. (b) 40. (c)
41. (c) 42. (a) 43. (a) 44. (c) 45. (b) 46. (d) 47. (a) 48. (b) 49. (a) 50. (c)
55. (b) 56. (c)
51. (d) 52. (b) 53.

Round II
1. (b) 2. (b) 3. (c) 4. (d) 5. (d) 6. (c) 7. (a) 8. (b) 9. (d) 10. (c)
11. (b) 12. (a) 13. (b) 14. (a) 15. (c) 16. (a) 17. (a) 18. (b) 19. (d) 20. (b)
21. (c) 22. (b) 23. (c) 24. (c) 25. (c) 26. (d) 27. (d) 28. (a) 29. (c) 30. (a)
31. (d) 32. (b) 33. (b) 34. (b,d) 35. (a,b) 36. (a, c) 37. (c, d) 38. (b, d) 39. (d) 40. (a)
41. (b) 42. (d) 43. (c) 44. (a) 45. (a) 46. (d) 47. (d) 48. (a) 49. (d) 50. (b)
51. (b) 52. (d) 53. (a) 54. (c) 55. (c) 56. (b) 57. (c)
Telegram @unacademyplusdiscounts

the Guidance
Round I
1. Centre of mass is closer to massive part of the body, therefore mA xA + mBxB + mC xC + mDxD
7. xCM =
the bottom piece of bat has larger mass. mA + mB + mC + mD
2. As shown in figure, centre of mass of respective rods are at y
their respective mid-points.
D C(1, 1)
Hence centre of mass of the system has coordinates ( X CM , YCM), (0, 1)
then
(0, a)

A x
a a (0, 0) B(1, 0)
,
a 2 2
0,
2 1´ 0 + 2 ´1+ 3 ´1+ 4 ´ 0
=
1+ 2 + 3 + 4
2+3 1
O a (a, 0) = = = 0.5 m
2
,0 10 2
mAYA + mBYB + mC YC + mDYD
Similarly, YCM =
a a mA + mB + mC + mD
m´ + m ´ + m ´0
2 2 a
X CM = = 1´ 0 + 2 ´ 0 + 3 ´1+ 4 ´1
3m 3 =
1+ 2 + 3 + 4
a a
m ´0 + m ´ +m´ 7
YCM = 2 2 =a = = 0.7 m
10
3m 3
8. From figure,
3. Centre of mass of a bangle lies at the centre of the bangle,
Wall
which is outside the body. L L L
x1 = , x2 = + = L
4. If speed of man relative to plank be v, then it can be shown 2 2 2
L/4
easily that speed of man relative to ground L L L 5L
x3 = + + = L/2
M 3 2 4 2 4
v mg = v = v
æ Mö 4
çM + ÷ M1x1 + M2x2 + M3 x3 L
è 3ø \ xCM = x1
M1 + M2 + M3 x2
3L x3
\ Distance covered by man relative to ground must be
4 L 5L
M ´ + M ´L + M ´
= 2 4
5. The position of centre of mass of the system shown in figure is M+M+M
likely to be at C. This is because lower part of the sphere
containing sand is heavier then upper part of the sphere 11
ML
11L
containing air. = 4 =
3M 12
6. Here, m1 = 1 kg, v1 = 2 $i
9. The acceleration of centre of mass is
m2 = 2 kg, v 2 = 2 cos $i - 2 sin 30 $j
F 30
m1v1 + m2v 2 aCM = = = 1ms–2
v CM = mA + mB 10 + 20
m1 + m2
1
\ s= aCM t 2
1 ´ 2 $i + 2 (2 cos 30° $i - 2 sin 30° $j) 2
=
1+ 2 1
= ´1´ 22 = 2 m
2 i + 2 3 i - 2 $j æ 2 + 2 3 ö $ 2 $
$ $ 2
= =ç ÷ i- j
3 è 3 ø 3
Telegram @unacademyplusdiscounts

296 JEE Main Physics

10. For the calculation of the position of Since, body of mass m1 moves with T
centre of mass, cut-off mass is taken as acceleration a = 2 ms–2 in upward direction.
negative. The mass of disc is O' \ f1 = m1g + m1a
m1 = pr12s O = 10 ´ 10 + 10 ´ 2
= p (6) 2s = 36 ps = 120 N
where s is surface mass density. \Tension in string = f1 + f2
The mass of cutting portion is = 120 + 80 = 200 N f1 f2
2 M 3
m2 = p (1) s = ps 13. Mv = v1 + Mv 2
m1x1 - m2x2 4 4
xCM = 3
m1 - m2 Mv = Mv 2 (\ v1 = 0)
4
Taking origin at the centre of disc,
4v
x1 = 0 , x2 = 3 cm v2 =
3
36 ps ´ 0 - ps ´ 3
xCM =
36 ps - ps 14. Change in momentum
- 3ps 3 Dp = p2 - p1 = MV - ( -MV ) = 2 MV
= =- cm
35 ps 35 15. Given, separation between the nuclei of H and Cl = 1.27Å
11. The mass of considered element is = 1.27 ´ 10 -10 m
Let mass of hydrogen atom = m
\Mass of chlorine atom = 35.5 m
dm
O CM
x Cl
dx
m 35.5 m
1.27 Å
dm = l dx = l 0 xdx
1 1 Let hydrogen atom be at origin i.e., position vector of it,
ò 0 xdm ò 0 x ( l0xdx) r1 = 0
\ xCM = = 1
\Position vector of chlorine atom r2 = 1.27 ´10 - 10 m
ò dm ò 0 l0xdx
Position vector to centre of mass is given by
L
é x3 ù 3 m r + m2 r2
lê ú L rCM = 1 1
l0 m1 + m2
ë 3 û0 3 = 2L
= L
=
é x2 ù L2 3 m ´ 0 + 35.5 m ´ 1.27 ´ 10 -10
l0 ê ú l0 =
2 2 m+35.5 m
ë û0
35.5 ´ 1.27 ´ 10 -10
12. Since, m2 moves with constant velocity =
36.5
a = 2 ms–1 m1 = 10 kg, m2 =8 kg = 1.235 ´ 10 -10 m = 1.24 Å
From hydrogen atom on the line joining H and Cl atoms.
m1 16. The law of conservation of momentum is applicable on the
process.

m2
17. Change in momentum = Impulse
= Area under force-time graph
v = 2ms–1
f2
\ mv = Area of trapezium
f1
1æ Tö
Þ mv = çT + ÷ f0
m1 m2 2è 2ø
3T
m1g m2g mv = f0
4
\ f2 = m2g 4 mv
Þ f0 =
3T
f2 = 8 ´ 10 = 80 N
Telegram @unacademyplusdiscounts

Centre of Mass 297

18. Here, r ( x) = a (1+ bx2) L


25. Time taken by first block to reach second block = . Since
v
When b ® 0 ,( x) = a = constant collision is 100% elastic, now first block comes to rest and
i.e., density of rod of length 1 m is constant. In that event, 2nd block starts moving towards the 3rd block with a velocity
centre of mass of rod would lie at 0.5 m , (i.e., at the centre of L
v and takes time = to reach 3rd block and so on.
rod.) v
When we try b ®0 in all the four given options, we find L
\ Total time = t + t + ¼(n - 1) time = (n - 1)
3 (2 + b) 6 v
choice (a) alone given x = = = 0.5
4 (3 + b) 12 Finally only the last nth block is in motion velocity v, hence
final velocity of centre of mass.
19. As p = mv
p mv v
\ m= VCM = =
v nm n
Hence, m-v graph will be rectangular hyperbola. 26. Since, no external force is present on the system, so
Dp Dm nm conservation principle of momentum is applicable.
20. F = =v =v
Dt Dt t \ p f = p f = p1 + p 2
n mvn \ p1 = - p 2 (Q pi = 0)
F = vm =
60 60
\ | p1| = |- p 2|
21. When a body is equilibrium, net force is zero. Hence, \ p1 = p 2
acceleration is zero.
From this point of view, it is clear that momenta of both
22. v1 v2
1g 3g particles are equal in magnitude but opposite in direction.
E1 E2 Also, fraction is absent. So total mechanical energy of system
As the momentum of both fragments are equal, therefore remains conserved.
E1 m2 3 27. When spring is massless then according to momentum
= =
E 2 m1 1 conservation principle,
pi = p f
According to problem,
or m1v1 = m2v 2
E1 + E 2 = 6.4 ´ 10 4 J
\ m1v1 = - m2v 2
By solving Eqs. (i) and (ii), we get m1v1 = m2v 2 or p1 = p 2
E1 = 4.8 ´ 10 4 J p2
Q K1 = 1
2 m1
and E 2 = 1.6 ´ 10 4 J
p22 K m
23. The resultant force on the system is zero. So, the centre of K2 = \ 1 = 2 (Q p1 = p2)
2 m2 K2 m1
mass of system has no acceleration.
24. p = px2 + py2 = (2 cos t) 2 + (2 sin t) 2 = 2 28. Since there is no external force acting on the particle, hence
m1y1 + m2y 2
y CM = = 0,
If m be the mass of the body, then kinetic energy m1 + m2
p 2 (2) 2 2 æ mö æ 3 mö
= = = Hence, ç ÷ ´ (15) + ç ÷ (y ) = 0
2m 2m m è 4ø è 4 ø 2

Since kinetic energy does not change with time, both work Þ y 2 = - 5 cm
done and power are zero. 29.Velocity or momentum is such that the linear momentum
Now, Power = Fv cos q = 0 would be conserved.
as F ¹ 0,v ¹ 0 30. Because in perfectly inelastic collision from colliding bodies
\ cos q = 0 stick together and move with common velocities.
or q = 90° 31. From conservation of linear momentum
As direction of p is same that v (Q p = mv), hence angle mv = 3 mv ¢
between F and p is equal to 90°. v
\ v¢ =
3
Telegram @unacademyplusdiscounts

298 JEE Main Physics

34. u1 = 4m/s u 2 = 3m/s 40. If initial velocity of bullet be v, then after collision combined
velocity of bullet and target is
mv
m1 = 3 kg m2 = 4 kg v¢ =
(M + m)
m1u1 + m2u2 = (m1 + m2) v
v ¢2
3 ´ 4 + 4 ´ ( -3) = (3 + 4) v , v = 0 and h= or v ¢ = 2 gh
2g
35. Given, m1 = m2 = m, u1 = 4 and u2 = 0 mv
\ = 2 gh
v (M + m)
v1 = (1 - e)
2 æ M + mö æ Mö
Þ v=ç ÷ × 2 gh = ç1 + ÷ 2 gh
v è m ø è mø
v 2 = (1 + e)
2
41. From t1 = 0 to 2 t 0 the external force acting on the combined
v1 æ 1 - e ö
\ =ç ÷ system is m1g + m2g .
v 2 è1+ e ø
\ Total change in momentum of the system
(From conservation of linear momentum)
= F ´ t = (m1 + m2) g ´ 2t 0
36. 15 m + 10 m = mv1 + mv 2
42. For elastic collision e = 1and velocity of separation is equal to
25 = v1 + v 2 …(i) velocity of approach. The velocity of the target may be more,
v 2 - v1 equal or less than that of projectile depending on their
and =1 masses.
u1 - u2
v 2 - v1 The maximum velocity of target is double to that of projectile,
Þ =1 when porjectile is extremely massive as compared to the
15 - 10
target.
Þ v 2 - v1 = 5 …(ii)
Maximum kinetic energy is transferred from projectile to
Solving Eqs. (i) and (ii), we have target when their masses are exactly equal.
v 2 = 15 ms–1,v1 = 10 ms–1 43. Let p1 and p2 be the momenta of A and B after collision.
1 m1m2 p2 p1
37. Loss of kinetic energy = (v1 - v 2) 2 A p B A
J J
B A B
2 m1 + m2
1 M ´M Before collision After collision
= (v1 - v 2) 2
2 (M + M)
Then applying impulse = change in linear momentum for the
M ×M M
= (v1 - v 2) 2 = (v1 - v 2) 2 two particles
2 (2 M) 4
For B J = p1 …(i)
38. v1 = + 3 m/s v2 = – 5 m/s
For A J = p - p2 …(ii)
m1 m2
or p2 = p - J …(iii)
As m1 = m2, therefore after collision velocities of masses get p1 - p2
interchanged. Coefficient of restitution, e =
p
i. e. , velocity of mass m1 = - 5 m/s p1 - p + J
=
and velocity of mass m2 = + 3 m/s p
39. As net horizontal force acting on the system is zero, hence J-p+ J 2J
= = -1
momentum must remain conserved. p p
Hence, mu + 0 = 0 + mv 2
44. m1v1 - m2v 2 = (m1 + m2) v
mu
Þ v2 = Þ 2 ´ 3 - 1 ´ 4 = (2 + 1) v
M
6 - 4 = 3v
As per definition,
2 = 3v
mu
(v 2 - v1) v 2 - 0 v 2 m 2
e= = = = M = v = m/s
(u2 - u1) 0 - u u u M 3
Telegram @unacademyplusdiscounts

Centre of Mass 299

46. (a) Impulse received by m y

J = m ( v f - vi )
12 m/s M
= m( -2 $i + $j - 3 $i - 2 $j)
12 m/s
= m( -5 $i - $j) M x
and impulse received by M 135°
M
= - J = m(5$i + $j)
v
(b) mv = m (5 $i + $j)
Let v is the velocity of third part.
m $ $ 1 $ $
or v= (5 i + j) = (5 i + j) By the conservation of linear momentum,
M 13
(c) e = (relative velocity of separation/relative velocity of 3 m ´ v = m ´ 12 2
approach) in the direction of - $j = 11 / 17 Þ v = 4 2 m/s
47. Retardation due to friction 52. (a) This is only possible when collision is head-on elastic.
a = mg = (0.25) (10) = 2.5 ms–2 (b) When collision is oblique elastic, then in this case, both
Collision is elastic, i. e. , after collision first block comes to rest bodies move perpendicular to each other after collision.
and the second block acquires the velocity of first block. Or (c) Since, in elastic collision, kinetic energy of system
we can understand it is this manner that second block is remains constant so, this is no possible.
permanently at rest while only the first block moves. Distance (d) The same reason as (b).
travelled by it will be v -u
53. Acceleration a=
v2 (5) 2 t
s= = =5m
2 a (2) (2.5) v - v0
or a=
t
\ Final separation will be ( s - 2) = 3 m
v - v0
48. Here, m = 0.08 kg, m0 = 0.16 kg or g = \ v =0
t
According to conservation principle of m v1 Speed before first bounce
momentum,
v 0 = - 5 ms–1
mv1 + mv 2 = (2 m + m0) v CM
m0 vB - v A 0 - ( -5) 5
mv1 + mv 2 \ t= = = = 0.5 s
\ v CM = g 10 10
2 m + m0 vCM

0.08 ´ 16 1.28 54. For collision between block A and B,


= = vB - v A vB - v A vB - v A
0.16 +0.16 1.32 e= = =
uA - uB 10 - 0 10
128 v2
= = 4 ms–1 m
32 \ vB - v A = 10 e = 10 ´ 0.5 = 5 …(i)
49. When the sphere 1 is released from horizontal position, From principle of momentum conservation,
then from energy conservation, potential energy at height mAuA + mBuB = mAv A + mBvB
l0 = kinetic energy at bottom
1 or m ´ 10 + 0 = mv A + mvB
or mgl0 = mv 2 or v = 2 gl0
2 \ v A + vB = 10 …(ii)
Since, all collisions are elastic, so velocity of sphere 1 is Adding Eqs. (i) and (ii), we get
transferred to sphere 2, then from 2 to 3 and finally from 3 to vB = 7.5 ms–1 …(iii)
4. Hence, just after collision, the sphere 4 attains a velocity to
2 gl0 . Similarly for collision between B and C,
vC - vB = 7.5 e = 7.5 ´ 0.5 = 3.75
50. This is an example of elastic oblique collision. When a \ vC - vB = 3.75 ms–1 …(iv)
moving body collides obliquely with another identical body
in rest, then during elastic collision, the angle of divergence Adding Eqs. (iii) and (iv), we get
will be 90°. 2 vC = 11.25
11.25
51. The momentum of third part will be equal and opposite of the \ vC = = 5.6 ms–1
2
resultant of momentum of rest two equal parts.
Telegram @unacademyplusdiscounts

300 JEE Main Physics

55. Since, no force is present along the surface, so momentum 56. As shown in adjoining figure ball is falling from height 2 h and
conservation principle for ball is applicable along the surface rebounding to a height h only. It means that velocity of ball
of plate. just before collision.
v1

θ2 θ1
n
θ1

v Plate
mv sin q1 = mv1 sin q2
2 (2 h) 4h
or v sin q1 = v1 sin q2 u= =
g g
v cos q2 v1 cos q2
e= 1 =
v cos q1 v cos q and velocity just after collision.
\ v1 = cos q2 = ev cos q 2h
v=-
v1 sin q2 v sin q tan q g
\ = =
v1 cos q2 ev cos q e 2h
tan q -v g 1
\ tan q = \ e= = =
e u 4h 2
æ tan q ö g
\ q2 = tan -1 ç ÷
è e ø

Round II
1. Let the centre of mass be b 4. At the highest point momentum of particle before explosion
(n - 1) mb + ma p = mv cos 60°
Then, =0
mn 1
= m ´ 200 ´ = 100 m horizontally.
1 a 2
b=- ×a = -
(n - 1) (n - 1) Now as there is no external force during explosion, hence
2. From adjoining figure the component of momentum along p = p1 + p 2 + p3 = constant
x-axis (parallel to the wall of container) remains unchanged However, since velocities of two fragments, of masses m / 3
even after the collision.
each, are 100 ms -1 downward and 100 ms -1 upward.
Hence, p1 = - p 2
mvy θ θ m
v mvx or p1 + p 2 = 0
m
= × = =3100
3 horizontally
pvp m v m
mvx 3
mvy
v3 = 300 ms-1 horizontally
\ Impulse = change in momentum of gas molecule along
y-axis, i. e. , in a direction normal to the wall = 2 mv cos q 5. Let mass per unit area of the disc be m.
3. As there is no external force, hence \Mass of the disc (M) = Total area of disc ´ Mass per unit
area = pR 2 m.
p = p1 + p 2 + p3 = constant
Mass of the portion removed from the disc (M¢ )
Þ | p3| = | p1 + p 2| 2 2
æRö pR
m 5m = pç ÷ m = m
= (3) 2 + ( 4) 2 = è2ø 4
4 4
[Since v1 and v 2 are mutually perpendicular] M
=
m 5m 4
\ p3 = v3 =
2 4 The centre of mass of the original disc is O and the centre of
5 mass of the removed part of O1 and let centre of mass of the
Þ v3 = = 2.5 ms–1 remaining part be O 2.
2
Telegram @unacademyplusdiscounts

Centre of Mass 301

According to the question, figure can be drawn as 7. Here, m1 = u,m2 = Au,u1 = u and u2 = 0
(m1 - m2) u1 2 m2 u2 æ 1- A ö
\ v1 = + =ç ÷u
(m1 + m2) (m1 + m2) è1 + A ø
O R v1 æ 1 - A ö
O1 R Þ =ç ÷
O2 x 2 u è1+ A ø
2 2
Kfinal æ v1 ö æ 1- A ö
\ =ç ÷ =ç ÷
Kinitial è u ø è1+ A ø
R
Here, OO1 = 8. When two identical balls collide head-on elastically, they
2 exchange their velocities. Hence when A collides with B, A
The remaining portion of the disc can be considered as a transfers its whole velocity to B. When B collides with C , B
M
system of two masses M at O and- M¢ = - at O1. transfers its whole velocity to C. Hence finally A and B will be
4 at rest and only C will be moving forward with a speed v.
If the distance of the centre of mass of the remaining part
from the centre O is at a distance x, then 9. Due to presence of contact (frictional) force momenta of
R blocks A and B separately change but total sum of momenta of
M ´ 0 - M¢ ´ A and B taken together is constant because no net external
x= 2
force is acting on the system.
M - M¢
M R 10. At the time of applying the impulsive force on block of 10 kg
- ´ pushes the spring forward but 4 kg mass is at rest. Hence,
= 4 2 m v + m2v 2 10 ´ 14 + 4 ´ 0
M v CM = 1 1 =
M-
4 m1 + m2 10 + 4
MR 4 R 140
=- ´ =- = = 10 ms–1
8 3M 6 14
R 11. The speed acquired by block, on account of collision of bullet
Therefore, centre of mass of the remainig part is at to the
6 with it, be v 0 ms–1. Since the block rises by 0.1 m, hence
left of centre O.
v 02
6. The centre of mass is given by 0.1 =
2g
m1x1 + m2x2 + m3 x3
x= Þ v 02 = 2 ´ g ´ 0.1
m1 + m2 + m3
or v 0 = 2 ms–1
1 , √3
C —
2m — Now as per law of conservation of momentum for collision
2 2
between bullet and block,
1m 1m mu = mu + Mv 0
M 2 kg
Þ v = u - v 0 = 500 - ´ 2 ms–1
m 0.01 kg
m
m
A(0, 0) 1m = (500 - 200 2) ms–1
B(1, 0)
= 220 ms–1
æ 1ö æ m1 - m2 ö
m ´ 0 ´ m ´ 1 ´ 2m ´ ç ÷
è2ø 12. In the pulley arrangement,| a1| =| a 2| = a = ç
÷ g but a1
x= è m1 + m2 ø
m + m + 2m is in downward direction and in the upward direction,
2m 1 i. e. , a 2 = - a1.
x= = m
4m 2 \ Acceleration of centre of mass
m y + m2y 2 + m3y3 m a + m2a 2
y = 11 a CM = 1 1
m1 + m2 + m3 m1 + m2
m ´ 0 + m ´ 0 + 2m ´ 3 /2 é m - m2 ù é m1 - m2 ù
y = m1 ê 1 ú g - m2 ê m + m ú g
m + m + 2m m
ë 1 + m2û ë 1 2û
=
3 (m1 + m2)
= m
4 2
é m - m2 ù
æ1 3 ö =ê 1 ú g
\Centre of mass is ç m, m÷ ë m1 + m2 û
è 2 4 ø
Telegram @unacademyplusdiscounts

302 JEE Main Physics

13. Let centre of mass of lead sphere after hollowing be at point 15. Let the initially particle x is moving in anti-clockwise direction
O 2, where OO 2 = x . and y in clockwise direction.
Mass of spherical hollow vx 1
As the ratio of velocities of x and y particles are = ,
2 vy 2
4 æRö
pç ÷ M therefore ratio of their distance covered will be in the ratio of
3 è2ø M
m= = + 2 : 1. It means they collide at point B.
æ 4 3ö 8
ç p R ÷ A
è3 ø A y 2v
x
R v
and x = OO1 =
2 2v
x
120° 120°

120° B y C
x B C

O2 O O1
v
After first collision at B velocities of particles get interchanged,
R
i. e. ,xwill move with 2 v and particle y with v. Second collision
2
with take place at point C. Again at this point velocities get
R interchanged and third collision take place at point A. So after
two collision these two particles will again reach the point A.
æ M ö R MR
M ´0 - ç ÷ ´ 16. Velocity of bullet at highest point of its trajectory = 50 cos q in
è8ø 2 R
\ x= = 16 = - horizontal direction.
M 7M 14
M-
8 8 As bullet of mass m collides with pendulum bob of mass 3 m
R and two stick together, their common velocity
\ Shift = m 50 cos q 25
14 v¢ = = cos q ms–1
m + 3m 2
14. C
C A B Now under this velocity v ¢ pendulum bob goes up to an angle
m m m 120°, hence
v ¢2 10 é æ 1 ö ù
= h = l (1 - cos120° ) = 1- ç f - ÷ = 5
2g 3 êë è 2 ø úû
Þ v ¢2 = 2 ´ 10 ´ 5 = 100 or v ¢ = 10
Initial momentum of the system (block C ) = mv
æ 4ö
or q = cos-1 ç ÷
After striking with A the block C comes to rest and now both è5ø
block A and B moves with velocity v. When compression in Comparing two values of v ¢, we get
spring is maximum. 25
cos q = 10
By law of conservation of linear momentum, 2
4
Þ M +qm
mv = (cos =) V
5
v
Þ V= (Q M = m) æ 4ö
2 q = cos-1 ç ÷
è5ø
By law of conservation of energy,
17. If M = M¢, then bullet will transfer whole of its velocity
KE of block C = KE of system + PE of system (and consequently 100% of its KE) to block and will itself
1 1 1 come to rest as per theory of collision.
mv 2 = (2M) V 2 + kx2
2 2 2
2
18. From conservation law of momentum,
1 1 æv ö 1 4 3
Þ mv 2 = (2M) ç ÷ + kx2 pr r 3
2 2 è2ø 2 v1 2 m2 3 2 ær ö
= = = = ç 2÷
1 v 2 1 m1 4 pr3r è r1 ø
Þ kx2 = mv 2 3
1
2
r2
m Þ = (2)1/3 : 1
Þ x=v r1
2k
or r1 : r2 = 1 : (2)1/3
Telegram @unacademyplusdiscounts

Centre of Mass 303

19. As shown in figure, 26. As there is no net external force, hence motion of centre of
M ´ 0 + M ´ 20 + M ´ 20 + M ´ 0 mass of fragments should have been as before.
xCM = = 10 cm
4M 27. We know that velocity of 2nd ball after collision is given by
Similarly, y CM = 10 cm u1 (1 + e) m1 (m - m1e)
v2 = + u2 2
Hence, distance of centre of mass from centre of any one (m1 + m2) (m1 + m2)
sphere, say
In present problem u2 = 0 ,m2 = 2 m1 and e = 2 / 3, hence
r = (10 - 0) 2 + (10 - 0) 2 = 10 2 cm æ 2ö
u ç1 + ÷ m1
è 3ø 5
20. Since, there is no external force acting on gun-bullet system, v2 = = u
Kb mg 2 kg 4 (m1 + 2 m1) 9
hence pb = pg and = = =
Kg mb 50 g 1 As four exactly similar type of collisions are taking place
Kb successively, hence velocity communicated to fifth ball
or Kg = 4
40 æ5ö
v5 = ç ÷ u
Now total energy = Kb + Kg è9ø
Kb 41 28. Since net momentum of the composite system is zero, hence
or = , Kb = 2050
40 40 resultant velocity of the composite system should also be
2050 ´ 40 zero.
Þ Kb = = 2000 J
41 29. Consider the two cart system as a single system. Due to
and Kg = 2050 - 2000 = 50 J explosion of power total momentum of system remains
unchanged, i. e. , p1 + p 2 = 0 or m1v1 = m2v 2, hence
21. Distance between the centres of spheres = 12 R
v1 m2
=
\ Distance between their surfaces = 12 R - (2 R + R) = 9 R. v 2 m1
Since there is no external force, hence centre of mass must
remain unchanged and hence As coefficient of friction between carts and rails are identical,
hence a1 = a2 and at the time of stopping final velocity of cart
Þ r = m2 r2
m11
is zero. Using equation v 2 - u 2 = 2 as, we have
Þ Mx = 5 M (9 R - x ),
s1 v12 m22
where, x = distance covered by smaller body. = =
s2 v 22 m12
Þ x = 7.5 R
s1m12 36 ´ (200) 2
22. Speed of the bullet relative to ground v b = v+ v r , where v r is Þ s2 = = = 16 m
m22 (300) 2
recoil velocity of gun. Now for gun-bullet system applying the
conservation law of momentum, we get 30. First sphere will take a time t1, to start motion in second sphere
mv b + Mv r = 0 L
on colliding with it, where t1 = .
u
or m ( v+ v r ) + Mv r = 0
mv mv Now speed of second sphere will be
Þ u v r 2= - or v r = 1ö
m+M m æ+ M v2 = (1 + e) = u çQ e = ÷
2 3 è 3ø
Feq m a + m2a 2 + m3 a3
23. Q a CM = = 1 1
(m1 + m2 + m3) (m1 + m2 + m3) Hence, time taken by second sphere to start motion in third
L 3L
\ Feq = m1a1 + m2a 2 + m3 a3 sphere t 2 = = .
2 /3 u 2 u
= 1 ´ 1 + 2 ´ 2 + 4 ´ ( -0.5) = 1+ 4 – 2 = 3 N L 3L 5L
\ Total time t = t1 + t 2 = + =
24. Since gun-shot system is an isolated closed system, its centre u 2u 2u
of mass must remain at rest.
5 g - 5 mg
31. Change in momentum = Ft and does not depend on mass of
25. asystem = = 4 m/s2 the bodies.
5+5
1
m a + m2a 2 5 ( 4 $i) + 5 ( 4 $j) 32. Let ball strikes at a speed u the K1 = mu2.
aCM = 1 1 = 2
m1 + m2 10
Due to collision tangential component of velocity remains
b 42 + 42 unchanged at u sin 45°, but the normal component of velocity
= = 2 2 m/s2
10 1
change to u sin 45° = u cos 45°
2
Telegram @unacademyplusdiscounts

304 JEE Main Physics

\Final velocity of ball after collision 36. As total mass is M and velocity of centre of mass is v, hence
2 kinetic energy of the system may have any value equal to or
æ1 ö 1
v = (u sin 45° ) + ç u cos 45° ÷ greater than Mv 2. However exact value of kinetic energy
è2 ø
2
2 2 can be calculated only when values of m1, m2, v1 and v 2 are
æ u ö æ u ö 5
= ç ÷ +ç ÷ = u known to us.
è 2ø è2 2 ø 8
37. In an inelastic collision neither momentum of ball nor
Hence, final kinetic energy mechanical energy of ball will remain same. However, total
1 5 energy and total momentum of earth-ball system will remain
K2 = mv 2 = mu 2
2 16 constant.
\ Fractional loss in KE 38. L L L
1 5
mu 2 - mu 2 v
K1 - K2 2 16 3
= = =
K1 1 8
mu 2
2
Since collision is perfectly inelastic, so all the block will stick
33. Let at the time of explosion velocity of one piece of mass m/2 together one by one and move in a form of combined mass.
is (10 $i). If velocity of other be v 2, then from conservation law Time required to cover distance (d) by first block = .
L
of momentum (since there is no force in horizontal direction), v
horizontal component of v 2, must be -10 $i. Now first and second block will stick together and move with
v / 2 velocity (by applying conservation of momentum) and
\Relative velocity of two parts in horizontal direction L 2L
= 20 ms-1. combined system will take = to reach upto block third.
v /2 v
Time taken by ball to fall through 45 m, Now these three blocks will move with velocity v/ 3 and
L 3L
2h 2 ´ 45 combined system will take time = to reach upto the
= 20 = = =3s v/ 3 v
g 10
fourth block.
and time taken by ball to fall through first 20 m, L 2L 3L (n - 1) L n (n - 1 ) L
So, total time + + +¼ =
v v v v 2v
2 h¢ 2 ´ 20
t¢ = = =2s
g 10 39. We cannot calculate the value of force exerted because time
is not known to us.
Hence time taken by ball pieces to fall from 25 m height to
ground = t - t ¢ = 3 - 2 = 1s. 40. Impulse of ball I, J1 = change in momentum of ball I = 2 mu
\ Horizontal distance between the two pieces at the time of and impulse of ball II J2 = change in momentum of
striking on ground = 20 ´ 1 = 20 m. ballII = 2 mu cos 30°.
J1 1 2
Þ = =
34. If v1 and v 2 are in same direction, then J2 cos 30° 3
m1v1 + m2v 2
v comp = 41. Force will be normal to the wall in both situations.
m1 + m2
h1 h h hn
42. Q e = = 2 = 3 ¼=
2 ´ 3 + 1´ 4 h0 h1 h2 hn -1
=
2 +1 \ h1 = e2h0 ,h2 = e2h1 = e4h0 and so on
Þ hm = e2nh0
10
= ms-1
3 43. The total distance travelled by the balls is
However, if v1 and v 2 are in mutually opposite directions, then H = h0 + 2 h1 + 2 h3 + ¼= h0 + 2 e2h0 + 2 e4h0 + 2 e4h0 + ¼
2 ´3 +1 ´ - 4 = h0 [1 + 2 e2 (1 + e2 + e4 + ¼)]
v comp =
2 +1
é æ 1 öù
2 –1 = h0 ê1 + 2 e2 ç ÷
= ms ë è1 - e2 ø úû
3
é 1 + e2 ù
35. As there is no external force on the system hence = h0 ê 2ú
displacement of the centre of mass of the system is zero. ë 1- e û
Telegram @unacademyplusdiscounts

Centre of Mass 305

As h0 = 10 m 51. As, m1u1 + m2u2 = (m1 + m2) v


1
and e= Þ 10 ´ u1 + 5 ´ 0 = (10 + 5) ´ 4
2
15 ´ 4
é 1ù Þ u1 =
1+ 10
ê 4 ú = 50 m
Hence, H = 10 ê
1ú 3 = 6 ms-1
ê 1- ú
ë 4û
52. In an elastic collision, the conservation of linear momentum
45. In a quick collision, time t is small as F ´ t = constant, and conservation of energy hold. Using conservation of linear
therefore, force involved is large in collision is more violent in momentum, we have
comparison to slow collision.
mv 0 = mv + 2 mv
46. The position of centre of mass of electron and proton remains v
at rest, at their motion is due to (internal) forces of electrostatic Þ v= 0
3
attraction, which are conservative. No external force, what so
ever is acting on the two particles. Using conservation of energy, we have
47. Angular momentum is rotational analogue of linear 1 1 1
mv 02 = x02 + (3 m) v 2
momentum, and torque is rotational analogue of force. 2 2 2
48. If it is a completely inelastic collision, then where, x0 = compression in the spring.
m1v1 + m2v 2 = m1v + m2v v 02
\ mv 02 = kx02 + (3 m)
m v + m1v 2 9
v= 11
m1 + m2 mv 02
Þ kx02 = mv 02 -
p12 p2 3
KE = + 2
2 m1 2 m2 2 mv 02
Þ kx02 =
As p1 and p 2 both simultaneously cannot be zero, therefore 3
total kinetic energy cannot be lost. 2 mv 02
Þ k=
49. If momentum is zero, i. e. , p = 0, then kinetic energy 3 x02
K = p 2/ 2 m = 0 . In an inelastic collision between two bodies, only
But potential energy cannot be zero, thus, a body can have conservation of linear momentum holds.
energy without momentum.
53. Conservation of linear momentum holds here. According to
50. The acceleration of the body which is rolling down an conservation of linear momentum,
inclined plane of angle a is
g sin a
g¢ =
k2
1+ 2 α
R
where, k = radius of gyration, m1v1 = m1v + m2v 2
R = radius of body. where v is the velocity of bullet after the collision and v 2 is the
velocity of block.
Now, here the body is a uniform solid disc.
\ 0.02 ´ 600 = 0.02 v + 4 v 2
k2 1
So, = Here, v 2 = 2 gh
R2 2
g sin a = 2 ´ 10 ´ 0.2
\ a=
1 = 2 ms–1
1+
2 \ 0.02 ´ 600 = 0.2 v + 4 v 2
g sin a Þ 0.02 v = 12 - 8
or a= (as g ¢ = a)
3 /2 4
\ v=
2 g sin a 0.02
or a=
3 = 200 ms–1
Telegram @unacademyplusdiscounts

306 JEE Main Physics

1 –ve sign shows that both the particles have to move in


54. h = gt 2 (parabolic)
2 opposite directions.
v = - gt and after the collision v = gt (straight line) md
So, 1 is the distance moved by 2nd particle to keep centre
Collision is perfectly elastic, then ball reaches to same height m2
again and again with same velocity. of mass at the same position.
v v 56. Since, the acceleration of centre of mass in both the cases is
+v1 same equal to g, so the centre of mass of the bodies B and C
h taken together does not shift compared to that of body A.
O t t
1 2t1 3t1 4t1 57. In x-direction
–v1 t
mv + 0 = 0 + mv x
Þ mv = mv x
55. To keep the centre of mass at the same position, velocity of Þ vx = v
centre of mass is zero, so
In y- direction
m1v1 + m2v 2
=0 æ v ö
m1 + m2 0 +0 =mç ÷ - mv y
è 3ø
[where v1 and v 2 are velocities of particles 1 and 2 v
respectively.] vy =
3
dr1 dr é dr dr ù
Þ m1 + m2 2 = 0 Q v1 = 1 and v 2 = 2 \Velocity of second mass after collision
dt dt ëê dt dt ûú
2
Þ m1dr1 + m2 dr2 = 0 æ v ö 2
v¢ = ç ÷ +v
è 3ø
[dr1 and dr2 represent the change in displacement of particles]
Let 2nd particle has been displaced by distance x. 4 2
= v
3
Þ m1 ( x) + m2 ( x) = 0
m1d 2
Þ x=- v¢ = v8
m2 3
Telegram @unacademyplusdiscounts

9 Rotational Motion
JEE Main MILESTONE
< Basic Concepts of Rotational Motion < Law of Conservation of Angular Momentum
< Moment of Inertia < Pure Rotational Motion (Spinning)
< Theorems on Moment of Inertia < Combined Rotational and Translational Motion or
< Values of Moment of Inertia for Simple Rolling
Geometrical Objects < Rigid Body Rotation
< Torque < Equation of Rotational Motion
< Angular Momentum

9.1 Basic Concepts of Rotational Motion


In this chapter, we will analyse the rotation of a rigid body. Let us consider the
example of a disc of radius R and mass M rotating about a fixed axis passing
through its centre as shown in figure.
Consider a pulley fixed at a typical
This rotating disc can be imagined as a group of infinite masses revolving in circles. well on which a rope is covered
The points which are at different distances from axis, are moving with different with one end attached to a
speeds in circles of different radii.
bucket. When the bucket is
The time period (T ) and the angular velocity (w) of revolution ω
released, the pulley starts rotating.
(w = 2 p /T ) are same for each point as all of them complete one As the bucket goes down, the
revolution in same time interval. For example, a point at a pulley rotates more rapidly till the
distance r from the axis is rotating with a speed rw while a point
bucket goes into water. The pulley
on the circumference of the disc is rotating with speed Rw. Points
is said to be executing rotational
on the axis are at rest. Points on the circumference have
maximum tangential velocities.
motion. Rotational motion is not
uniform. In this chapter, shall
If the disc is rotating with a constant angular velocity w, its
angular displacement in time interval t is simply given by
study the rotational motion.
Dq = wDt.
If the disc is rotating with a constant angular velocity. The rate ω Rω
of change of angular velocity is known as angular acceleration
Dw
(a ). If Dw is the change in angular velocity in Dt, then a =
Dt r
(unit of a = rads–2 ).
If the angular velocity of rotation varies with the magnitude of rω
tangential velocity as v = rw. The rate of change of tangential
velocity is known a tangential acceleration (at ) given as
Dv Dw
at = = r×
Dt Dt
Þ at = ra

Note w and a are same for all particles in the body while v and at are different for different points. For a
point at a distance r from the axis v = rw and at = r a.
Telegram @unacademyplusdiscounts

308 JEE Main Physics

Rotation with Constant Angular Interpret (c) Angular displacement q is given by


w2f - wi2 0 - ( 5p/ 3) 2
Acceleration q=
2a
=
-2 ( 5p/ 24)
If the angular velocity increases or decreases at a constant 20 p
rate, the angular acceleration a is a constant. Þ q= rad
3
Let wi = angular velocity at some critical instant q 10
\ Number of revolutions of = revolutions anti-clockwise.
w f = final angular velocity t second later 2p 3
w f - wi æ 1ö
Angular acceleration, a = Hence, it completes ç3 + ÷ revolution before stopping.
è 3ø
t
Þ w f = wi + at
Sample Problem 3 A flywheel of radius 30 cm starts from
If q is the angular displacement during these t second rest and accelerates with constant acceleration of 0.5 rad s–2.
1 The tangential acceleration of a point on its circumference is
q = wit + at 2
2 (a) 1 ms–2 (b) 0.15 ms–2
w2f = w2i + 2 aq (c) 2 ms–2 (d) 0.25 ms–2
Interpret (b) Given,
Note a = 0.5 rads–2,

These relations can be compared with those in uniformly accelerated
R = 0.3 m,
linear motion i.e.,
1 2 2 and w = wi = 0 rads–1
v = u + at , s = ut + at , v = u 2 + 2 as
2 Tangential acceleration, at = Ra

q, w, a can be clockwise or anti-clockwise. We can take = (0.3) (0.5) = 0.15 ms–2
anti-clockwise direction as positive and clockwise direction as
negative or vice-versa.

w and q are always in same direction. Check Point 1

a and w are opposite if the rotating body slows down (compare with 1. A disc of metal is melted and recast in the form of a solid
the case of retardation in linear motion) sphere. What will happen to the moment of inertia about a
vertical axis passing through the centre?
Sample Problem 1 A wheel is rotating at the rate of 2. What are the factors on which moment of inertia of a body
50 rev min–1 in an anti-clockwise direction. What should be the depends?
magnitude and direction of the angular acceleration of the
3. Is radius of gyration of a body a constant quantity?
wheel so that it stops it in 8s?
5p 7p 4. Two solid spheres of the same mass are made of metals of
(a) rads-2, Clockwise (b) rads–2, Clockwise different densities. Which of them has a larger moment of
24 24
inertia about the diameter?
5p 7p
(c) rads-2, Anti-clockwise (d) rads-2, Anti-clockwise 5. Why there are two propellers in a helicopter?
24 24
6. Two satellites of equal masses, which can be considered as
Interpret (a) Let initial angular velocity = wi particles are orbiting the earth at different heights. Will their
æ 50 ö 5 p moment of inertia be same or different?
wi = 2 p ç ÷ = rads–2
è 60 ø 3
Here wf (final angular velocity) = 0
w - wi 0 - 5p /3 - 5p
a= f = = rads-2 9.2 Moment of Inertia
t 8 24
Hence, angular acceleration of 5p/24 rad s–2, must be imparted to The inability of a body to change its state of rest or of
the block in clockwise direction. uniform linear motion by itself is known as inertia.
Similarly, a body rotating about an axis is unable to
Sample Problem 2 In the above example, how many produce a change in its rotational motion by itself. This
revolutions will it cover before stopping? inertness in this case is known as rotational inertia or
æ 1ö æ 1ö
(a) ç3 + ÷ revolution (b) ç1 + ÷ revolution moment of inertia.
è 2ø è 2ø
1ö The moment of inertia of a rigid body about any axis of
æ
(c) ç3 + ÷ revolution (d) None of these rotation is the sum of the product of masses of the
è 3ø
Telegram @unacademyplusdiscounts

Rotational Motion 309

particles and the square of their respective distances from


axis of rotation.
9.3 Theorems on Moment of
I = MR2 Inertia
Let us consider a body of n particles of masses There are two important theorems on moment of inertia
m1, m2, ¼, mn with r1, r2, ¼, rn as their respective distances which, in some cases, enable the moment of inertia of a
from axis of rotation as shown in figure. Then, moment of body to be determined about an axis, if its moment of
inertia of the body is given by inertia about some other axis is known.
z
1. Theorem of Parallel Axes
r1
r2
m1 It states that the moment of inertia of a rigid body about
m2
r3 m3 any axis is equal to its moment of inertia about a parallel
rn mn axis through its centre of mass plus the product of the
mass of the body and the square of the perpendicular
y distance between the two axes.
O

CM
I = m1r12 + m2r22 + ¼+ mnrn2
n
or I = å miri2 r
i =1

The unit of moment of inertia in SI system is kg-m . 2 Two such axes are shown in figure for a body of mass M. If
r is the distance between the axes and I CM and I are the
Radius of Gyration respective moments of inertia about these axes, then
I = I CM + Mr 2
It is defined as the distance from the axis of rotation at
which, if whole mass of the body were supposed to be
concentrated, the moment of inertia would be same as 2. Theorem of Perpendicular Axes
with the actual distribution of the mass of body into small It states that the moment of inertia of a plane lamina
particles. about an axis perpendicular to its plane is equal to the
sum of the moments of inertia of the lamina about any
two mutually perpendicular axes in its plane and
intersecting each other at the point, where the
K M
M ⇒ perpendicular axis passes through it.
y z

xi
If a body has mass M and radius of gyration is K, then ri P
xi
x
Moment of inertia, O
I = MK 2
I
K =
M
Let x and y axes be chosen in the plane of the body and
Here, K is radius of gyration.
z-axis perpendicular to this plane, three axes being
Radius of gyration is also defined as the root mean square mutually perpendicular, then according to the theorem
distance of all the particles about the axis of rotation.
Iz = I x + Iy
r12 + r22 + r32 + ¼+ rn2 where, I x, I y and I z are the moments of inertia about the x,
i.e., K =
n y, z axes respectively.
Telegram @unacademyplusdiscounts

310 JEE Main Physics

Important Points 9.4 Values of Moment of Inertia


1. Theorem of parallel axes is applicable for any type of rigid body for Simple Geometrical
whether it is a two dimensional or three dimensional, while the Objects
theorem of perpendicular axes is applicable for laminar type or two
dimensional bodies only. 1. Circular ring
(a) About the axis passing through the centre and
2. In theorem of perpendicular axes, the point of intersection of the
perpendicular to the plane of ring
three axes (x, y and z) may be any point on the plane of body (it may
even lie outside the body). This point may or may not be the centre of I = MR2
mass of the body.
R
3. Moment of inertia of a part of a rigid body (symmetrically cut from (b) About the axis passing through
the whole mass) is the same as that of the whole body e .g ., in figure the centre about its diameter
moment of inertia of the section shown (a part of a circular disc)
1
about an axis perpendicular to its plane and passing through point O I = MR2
1 1 2
is MR 2 as the moment of inertia of the complete disc is also MR 2.
2 2 ω
2. Hollow cylinder
R (a) About its geometrical axis
O M
R

(a)
(b) Hollow

1
Suppose the given section is th part of the disc, then mass of the
h
disc will be nM. Axis
1
I disc = (nM )R 2 I = MR2
2
1 1 3. Solid cylinder and a disc
I section = I disc = MR 2
n 2 (a) About its geometrical axis
4. Calculation for moment of y
R
inertia by digits b
x' x
Moment of inertia about an axis of a
symmetry is
Solid
y'

Mass ´ the sum of squares of perpendicular semi -axis


3 or (4 or 5) Axis
where denominator to be 3 or 4 or 5 according as the body is (i) (ii)
rectangular, elliptical (including circular) or ellipsoidal (including 1
I = MR2
spherical) e.g., for ellipse, 2
M (b) About the perpendicular axis
I z = (a 2 + b 2 )
4
5. Expression for moment of inertia of a lamina about an axis passing
through origin making an angle q with x-axis is h
2 2
I = Ix cos q + Iy sin q - 2 F sin q cos q
where, F = Smxy =product of inertia
l
6. If a rigid body consists of a great many adjacent particles (it is
continuous, like a frishbee), then we define the rotational inertia of æ l2 R2 ö
the body as I=M ç + ÷
è 12 4 ø
2
I= òI dm
Telegram @unacademyplusdiscounts

Rotational Motion 311

(c) Moment of inertia of a disc about ω 6. Cuboid (l ´ b ´ h ) Axis through


its diameter passing through centre and parallel to the height h
centre of gravity (h)
1 M 2
I= MR2 I= (l + b2) b
4 12

4. (a) Solid sphere About the axis passing through the 7. Solid cone About the axis joining the vertex and centre
centre of the base

R
h

O R
2
I = MR2
5
(b) Hollow sphere About the axis passing through the 3
I0 = MR2
centre 10
ω
8. Rectangular plate
(a) About the axis lying in the plane of the plate and
passing through centre of mass
R
x
l

2
I = MR2
5 b
y
5. Thin rod
(a) About the axis passing through mid-point and
perpendicular to the length.
Mb2
Ix = ,
12
Ml 2
I = , y
12
and Iz = I x + Iy
l
M (l2 + b2)
Iz =
MR2 12
I=
12 (b) A rectangular plate about one edge
(b) About the axis passing through an end and ω
perpendicular to the rod.

l
l

MR2 Ml2
I= I=
3 3
Telegram @unacademyplusdiscounts

312 JEE Main Physics

9. Annular disc Sample Problem 5 Three mass points m1, m2, m3 are
(a) About the axis passing through the centre and located at the vertices of an equilateral triangle of length a.
perpendicular to the plane What is the moment of inertia of the system about an axis along
the altitude of the triangle passing through m1?
a2 a2
(a) (m2 + m3) (b) (m1 + m3)
R2 2 2
2 2
a a
R1 (c) (m2 + m3) (d) (m1 + m2)
4 4
M
Interpret (c) ABC is an equilateral triangle. Let AO ^ BC, so
AO is also a bisector of BC i.e., AO is median of DABC.
A
m1
M
I = [ R 21+ R 22]
2 a a
(b) Moment of inertia of an annular disc about an
axis passing through the centre about its
diameter 90º
B C
m2 a/2 O a/2 m3
M
I = [ R 21+ R 22]
4 We have to calculate moment of inertia of the system about AO.
10. Sphere with cavity i=n

Mass = M, inner radius = R1,


I= å mi ri 2
i =1
Outer radius = R2 2 2
æ aö æ aö
2M æ R 52 - R 51 ö I = m1 ´ 0 + m2 ´ ç ÷ + m3 ´ ç ÷
è2ø è2ø
I0 = ç ÷
5 è R 32- R 31 ø
a2
I= (m2 + m3)
4
Sample Problem 4 Calculate the moment of inertia of
(a) a ring of mass M and radius R about an axis coinciding with Sample Problem 6 Four spheres, each of diameter 2a and
a diameter of the ring. mass M are placed with their centres on the four corners of a
(b) a thin disc about an axis coinciding with a diameter. square of side b. Then moment of inertia of the system about
Interpret (a) Let x and y axes be along two perpendicular one side of the square taken as the axis is
2 2
diameters of the ring. By symmetry (a) M( 4a2 + 5b 2) (b) M( 4a2 + 5b 2)
z 5 3
1 1
y (c) M(2a2 + 5b 2) (d) M(2a2 + 5b 2)
3 4
Interpret (a) ABCD is a square of side b. Four spheres, each of
x mass M and radius a are placed at the four corners of the square.
a a
b
A D

Ix = Iy and Iz = Ix + Iy
But we know that, b b
Iz = MR 2
MR 2 = Ix + Iy a a
2 B C
MR = 2 Ix b
2
MR Moment of inertia of the system about any side, say CD is
Ix = Iy =
2 = MI of sphere at A about CD
Similarly for a thin disc (i. e. , a circular plate). + MI of sphere at B about CD
(b) Moment of inertia about the diameter is + MI of sphere at C about CD
1 æ1 ö 1
I = ç MR 2÷ = MR 2 + MI of sphere at D about CD
2 è2 ø 4
Telegram @unacademyplusdiscounts

Rotational Motion 313

æ2 ö æ2 ö 2 2 Torque, t = force ´ force arm


= ç Ma2 + Mb 2÷ + ç Ma2 + Mb2÷ + Ma2 + Ma2
è5 ø è5 ø 5 5 t = r ´ F = rF sin q
8 (where q is the angle between r and F)
= Ma2 + 2 Mb 2
5
Unit of torque is Nm.
2
= M( 4a2 + 5b 2)
5 Torque is an axial vector i.e., its direction is always
perpendicular to the plane containing vectors r and F in
Sample Problem 7 The moment of inertia of a rod of mass accordance with right hand screw rule.
M, length l about an axis perpendicular to it through one end is Moreover, rotation analogue of Newton’s law equation
Ml 2 2 F = Ma is given by,
(a) (b) Ml 2
12 3
Torque, t = Ia
3 2 Ml 2
(c) Ml (d)
2 3 Couple
Interpret (d) For the rod of mass M and length l, A pair of two equal and opposite forces acting along
parallel lines but having different lines of action
constitutes a couple. Moment of couple or torque = F1 =
M
(Force) ´ (perpendicular distance between force)
l F

Ml 2
Moment of inertia of rod I =
12
Using the parallel axes theorem,
l
I ¢ = I + Ma2 F
l
with a = , we get
2
2
Ml 2 ælö Ml 2 Work done by Torque
I¢ = +M ç ÷ =
12 è2ø 3
Consider a rigid body acted upon by a force F at a
Note We can check this independently since I is half the moment of perpendicular distance r from the axis of rotation. Let
inertia of a rod of mass 2 M and length 2 l about its mid-point. under the action of this force, the body rotates through an
angle Dq.
4 l 2 1 Ml 2 Work done = torque ´ displacement = t d q
I ¢ = 2M × ´ =
12 2 3 dW dq
Power = =t = tw
dt dt

9.5 Torque Sample Problem 8 The force 7 $i + 3$j - 5 k$ acts on a


Torque is a quantity which measures the capability of a particle whose position vector is i$ - $j + k$ , then the torque of the
force to rotate a body. Torque due to a force is also known force is
as the moment of a force. It is defined as the product of the (a) 12 $i + 2 $j + 7 k$ (b) 2 $i + 12 $j + 10 k$
force and the perpendicular distance between the line of (c) 13 $i + 3 $j + 4 k$ (d) 10 $i + $j + 5 k$
action of the force and the axis of rotation. This
perpendicular distance is known as the force arm. Interpret (b) Given, r = $i - $j + k$ , F = 7 $i + 3 $j - 5 k$
F \ t = r ´F
$i $j $k

r t = 1 -1 1 = (5 - 3) $i - ( -5 - 7) $j + [3 - ( -7)] k$
O A
r
θ 7 3 -5

\ t = 2 $i + 12 $j + 10 k$
Telegram @unacademyplusdiscounts

314 JEE Main Physics

Sample Problem 9 A grind stone in the form of a solid or L=r´p (vector product)
cylinder, has a radius of 0.5 m and a mass 50 kg. What torque The angular momentum is the same at every point on an
will bring it from rest to an angular velocity of 300 rev min–1 orbit. When it is closer, it increases speed.
in10 s. mv = p
(a) 18.6 Nm (b) 15.6 Nm
r
(c) 19.6 Nm (d) 20.6 Nm
r
Interpret (c) Let wi = 0 , wf = 2p (300 /600) = 10 p rads–2
mv
We know,
wf - wi 10 p - 0 The direction is given by the right hand rule which would
a= = = 10 p rads–1
t 10 give L the direction out of the diagram.
æ1 ö For an orbital angular momentum is conserved, and this
Torque required, t = Ia = ç MR 2÷ a
è2 ø
leads to one of Kepler’s laws. For a circular orbit, L
1 becomes
Þ t = 50 ´ (0.5) 2p = 19.6 Nm
2 L = mvr
Angular momentum of a rigid body is defined as the
9.6 Angular Momentum product of the moment of inertia and the angular velocity.
L = Iw
The angular momentum of a particle of mass m with
respect to a chosen origin is given by It is a vector quantity.
L = mvr sin q

Hot Spot Law of Conservation


of Angular Momentum
The law of conservation of angular momentum states that when no external torque acts on an object or a closed system of
objects, no change of angular momentum can occur. Hence, the angular momentum before an event involving only
internal torques or no torques is equal to the angular momentum after the event.
We know that, t ext =
dL always towards some fixed point, the centre, then there is no torque
dt on the body with respect to the centre, and so the angular momentum
dL of the body about the centre is constant.
If t ext = 0, then = 0 or L must be a constant. Therefore, in the
dt Constant angular momentum is extremely useful when dealing with
absence of any external torque, the total angular momentum of a the orbits of planets and satellites and also when analyzing the Bohr’s
system must remain conserved. model of the atom.
As L = Iw, the law of conservation of momentum leads us to the
conclusion. Applications of Angular Momentum
For an isolated system, Iw = constant 1. The conservation of angular momentum explains the angular
or I1w1 = I2 w2 acceleration of an ice skater as she brings her arms and legs
close to the vertical axis of rotation. By bringing part of her
This principle is often made use by gymnast swimmers, circus acrobats
body closer to the axis she decreases her body’s moment of
and ballet dancers etc.
inertia. Because angular momentum is constant in the absence
It is assumed that internal interaction forces obey Newton’s third law of external torques, the angular velocity (rotational speed) of
of motion in its strong form, i. e., the forces between particles are the skater has to increase.
equal and opposite and act along the line between the particles.
2. Conservation of angular momentum in earth-moon system
In orbits, the angular momentum is distributed between the spin of results in the transfer of angular momentum from earth to
the planet itself and the angular momentum of the orbit. moon. This in turn results in slowing down of the rotation
Ltotal = Lspin + Lorbit rate of earth and in gradual increase of the radius of moon’s
Conservation of angular momentum is used extensively in analyzing orbit.
central force motion. If the net force on some body is directed
Telegram @unacademyplusdiscounts

Rotational Motion 315

Special Cases of Angular Momentum Sample Problem 10 A man of mass 100 kg stands at the
rim of a turn table of radius 2 m and moment of inertia
Case I For a particle moving in a circle of mv
4000 kg m 2 mounted on a vertical frictionless shaft at its centre.
radius r with a speed v, its linear
momentum mv is along the tangent The whole system is initially at rest. The man now walks along
and q = 90°, then its angular the outer edge of the turn table (anti-clockwise) with a velocity
momentum ( L) is given as r of 1 m/s relative to the earth. With what angular velocity and in
L = mvr what direction does the turn table rotate?
Case II For a rigid body (about a fixed axis) (a) The table rotates anti-clockwise (in the direction of man
L = Sum of angular momenta of all particles motion) with angular velocity 0.05 rad/s
= m1v1r1 + m2 v2 r2 + m3v33r + ¼ (b) The table rotates clockwise (opposite to man) with angular
= m1r12 w + m2 r22 w + m33r2 w + ¼ (Q v = rw) velocity 0.1 rad/s
= ( m1r12 + m2 r22 + m33r2 + ¼ ) w (c) The table rotates clockwise (opposite to man) with angular
L = Iw velocity 0.05 rad/s
Case III For a particle in linear motion (d) The table rotates anti-clockwise (in the direction of the
Angular momentum about origin ( O ) is given by man motion) with angular velocity 0.1 rad/s
mv
Interpret (c) By conservation of linear momentum on the
A
man-table system.
Li = Lf
θ
r 0 + 0 = Imwm + It wt
I w
r Þ wt = - m m
It
O
v 1
L = r ´ ( mv) where wm = = rad/s
r 2
where, r = position vector of the particle,
v = velocity æ 1ö
100 (2) 2 ´ ç ÷
è2ø
Þ L = mvr sin q = mv ( OA) sin q = mvr^ =-
where, r^ = perpendicular distance of velocity vector from O. 4000
v 1
Note A body, which translates, can also have an angular momentum wm = = rad/s
r 2
besides its linear momentum. It is not at all necessary for a body to rotate
for it to have angular momentum. The value of angular momentum 1
Þ wt = - rad/s
depends on the point about which it is being measured. 20

Sample Problem 11 Point masses m1 and m2 are placed at From work-energy theorem
1
the opposite ends of a rigid rod of length l and negligible mass. W = D KE = [m1x2 + m2 ( l - x) 2] w20
The rod is to be set rotating2about an axis perpendicular to it.
The position on the rod through which the axis should pass in For minimum work,
dW
order that the work required to set the rod rotating with angular =0
dx
velocity w0. be minimum is
or 2 m1x + 2 m2 ( l - x) = 0
M2 L M + M2 M2 L
(a) (b) 1 Þ x=
M1 + M2 M2 L M1 + M2
M1L M - M2
(c) (d) 1 Sample Problem 12 A particle of mass m is projected
M1 - M2 M2 L
with velocity v at an angle q with the horizontal. Find its
Interpret (a) In order angular momentum about the point of projection when it is at
m1 m2
the highest point of its trajectory.
mv 2 sin q sin 2 q mv3 sin q sin 2 q
(a) (b)
x l–x 4g 2g
mv3 sin q cos q
(c) (d) None of these
4g
Telegram @unacademyplusdiscounts

316 JEE Main Physics

Interpret (a) At the highest point of its trajectory velocity of Interpret (a) The angular momentum of the sphere about the
projectile is v cos q in the horizontal direction and value of point of contact P will be
maximum height is

O v0
A
u v cos

hmax L p = ICM w + r ´ PCM = ICM w + r ´ mv CM


θ As sphere is rolling without slipping, thus
O B
C
v
2
v sin q 2 w= 0
hmax = r
2g æ 2 2ö æ v 0 ö
L p = ç mr ÷ ç ÷ + r ´ m ´ v 0
\ Angular momentum about the point of projection is è3 øè r ø
v 2 sin 2 q 2 7
L = phmax = (mv cos q) = mv 0r + mv 0r = mv 0r
2g 5 5
mv 2 cos q × sin 2 q
Þ L=
2g Relation between Torque and
3
mv sin q × sin 2 q Angular Momentum
Þ L=
4g As angular momentum, L = r ´ p
dL dr dp
\ = ´p+ r´
Angular Momentum of a Rigid Body dt dt dt
Performing Rotational and Translational = p ´ mv + r ´ F = 0 + r ´ F = t ext
Motion dL
Þ = t ext
dt
When a rigid body describes pure rotational motion, all its
constituent particles describe circular motion about the Thus, rate of change of angular momentum is equal to
axis of rotation. In such case, angular momentum of the the torque due to an external force.
rigid body Relation for work done, power, angular impulse in
L = Smvr = Smr 2w = (S mr 2 ) × w = Iw rotational motion.
(i) In rotational motion, total work done by a torque is
Direction of L is same as w. Hence, w angular momentum is
given by = ò t × dq
an axial vector.
When a rigid body is rotating about its centre of mass axis (ii) Instantaneous power in rotational motion P = t × w
with angular velocity wCM , simultaneously moving (iii) Angular impulse = ò t dt = L f - Li
translationally with a linear velocity v, the angular
momentum of that body about a point P in the laboratory Hence, angular impulse of torque is equal to total
frame is given by change in angular momentum of the body in given
time.
L p = I CM w + rX m vcm

Sample Problem 13 A solid sphere rolls without slipping Sample Problem 14 A star of mass equal to two solar
on a rough surface with centre of mass having a constant speed masses and radius10 -6 km rotates about its axis with an angular
v0. If mass of the sphere be m and r be its radius, then what is the speed of 10–6 rad s–1. What is the angular speed of the star, when
value of angular momentum of the sphere about the point of it collapses (due to inward gravitational forces) to a radius of
contact with rough surface? 104 km?
7 (a) 10 -1 rad s-1 (b) 10 -2rad s-1
(a) mv 0 r
5 (c) 10 -3 rad s-1 (d) 10 -4 rad s-1
5
(b) mv 0 r
7 Interpret (a) Here, R1 = 10 6 km,
3 w1 = 10 -6 rad s–1
(c) mv 0 r
5
5 w2 = ?,R2 = 10 4 km
(d) mv 0 r
3 As, I1w1 = I2 w2
Telegram @unacademyplusdiscounts

Rotational Motion 317

2
MR12w1 K2 K2
I1w1 5 Here, 2
is a constant for different bodies. Value of 2 = 1
\ w2 = = R R
I2 2 K2 1
MR22 for ring and cylindrical shell and 2 = for disc and solid
5 R 2
2
æR ö K2 2
= ç 1 ÷ w1 cylinder and 2 = for a solid sphere.
è R2 ø R 5

æ10 6 ö
2 Example of this type of motion are rotation of door about
w2 = ç 4 ÷ ´ 10 -6 hinge, rotation of a fan about its axis of rotation, rotation of
è10 ø
pulley, etc.
= 10 -1 rad s–1

Sample Problem 15 What will be the duration of the day, 9.8 Combined Rotational
if the earth suddenly, shrinks to 1/64th of its original volume,
mass remaining unchanged? and Translational Motion
(a) 0.5 h (b) 1 h or Rolling
(c) 1.5 h (d) 2 h
1 In this type of rotational motion, axis of rotation is in
Interpret (c) Here, final volume V2 = , initial volume V1 motion. Plane motion can be considered as combination
64
4 3 1 4 3 of translational motion of the centre of mass and
i. e. , pR2 = ´ pR1 rotational motion of the body about an axis passing
3 64 3
3 through the centre of mass.
æ1 ö ω, α ω, α
R32 = ç R1÷
è4 ø
+
v, a v, a
1
or R2 = R1
4 Plane motion Translational Rotational motion of
motion of CM body about an AOR
As no external torque is acting,
passing through CM
æ 2p ö
L = Iw = ç ÷ = constant
èT ø Body possess both translational and rotational kinetic
i. e. , I2 /T2 = I1 /T1 energy.
æ2 2ö 1 æ2 ö 1 Net kinetic energy = (Translatory + Rotatory) kinetic energy
ç MR2 ÷ = ç MR12÷ ´
è5 ø T2 è 5 ø T1 1 1 1 1 K2
KN = KT + KR = mv2 + Iw2 = mv2 + mv2 2
R22 2 2 2 2 R
T2 = ´ T1
R12 1 æ K2ö
\ KN = mv2 ç1 + 2 ÷
As T1 = time taken by earth to complete one revolution about its 2 è R ø
axis.
i.e., normal length of the day = 24 h Sample Problem 16 A disc of mass 5 kg and radius 50 cm
–1
2 rolls on the ground at the rate of 10 ms . The kinetic energy of
æ 1ö 1
\ T2 = ç ÷ ´ 24 = 1.5 h the disc is (Given, I = MR2)
è 4ø
2
(a) 300 J (b) 325 J
(c) 350 J (d) 375 J
9.7 Pure Rotational Motion Interpret (d) Here, mass of the disc, M = 5 kg
(Spinning) Radius of the disc, R = 50 cm =
1
m
2
In this type of rotational motion, axis of rotation is fixed.
Linear velocity of the disc, v = 10 ms-1
Questions based on pure rotational motion can be solved
by using, t = Ia (about axis of rotation) and basic kinematic As, v = Rw
equations. In spinning, body possess only rotatory kinetic 1
\ 10 = w
energy, 2
1 1 v2 1 æK2ö or w = 10 ´ 2 = 20 rad s–1
K R = Iw2 = mK 2 2 = mv2 ç 2 ÷
2 2 R 2 èR ø Also, moment of inertia of disc about an axis through its centre
Telegram @unacademyplusdiscounts

318 JEE Main Physics

2
1 1 æ 1ö 5 or R1 - R2 = 1.2 gN = 11.76 N …(iv)
MR 2 = ´ 5 ´ ç ÷ = kgm2
I=
2 2 è 2 ø 8 From Eqs. (iii) and (iv), we get
1 2 R1 = 54.88 N,R2 = 43.12 N
As, KE of translation = Mv
2 Thus, the reactions of the support are about 55 N at K1 and 43 N
1 at K2.
= ´ 5 ´ (10) 2 = 250 J
2
1 2 1 5 Classification of Rolling Motion
and KE of rotation = Iw = ´ ´ (20) 2 = 125 J
2 2 8
Depending on the fact that relative velocity of point of
Total KE = 250 + 125 = 375 J
contact of the body undergoing plane motion, with the
platform (on which the body is performing plane motion)
Sample Problem 17 A metal bar 70 cm long and 4 kg is
is zero or non-zero, rolling motion is classified into two
mass supported on two knife-edges placed 10 cm from each
end. A 6 kg load is suspended at 30 cm from one end. The 1. Pure rolling or rolling without slipping/sliding or
reactions at the knife-edges are perfect rolling motion.
(a) 54.88 N, 43.12 N (b) 98 N, 43.2 N 2. Impure rolling or rolling with slipping/sliding or
(c) 30 N, 15.32 N (d) 15.2 N, 20.1 N imperfect rolling motion.

Interpret (a) The following figure shows the positions of the 1. Pure Rolling Motion
knife edges K1 and K2, the centre of gravity of the rod at G and the
suspended load at P. If the relative velocity of the point of contact (between
R1
P G
R2 body and platform) is zero, then the rolling motion is said
A B to be pure rolling motion.
For pure rolling motion, v AB = 0
K1 K2
i. e., (v - Rw) - v0 = 0 Þ r - rw = v0
w
v
Note the weight of the rod W acts at its centre of gravity G. The rod is ω
uniform in cross-section and homogeneous, hence G is at the centre A
v
of the rod; AB = 70 cm, AG = 35 cm, AP = 30 cm, PG = 5 cm, v0 Rω v0
AK1 = BK2 = 10 cm and K1G = K2G = 25 cm.
B
Also, W = weight of the rod = 4 kg
W1 = suspended load = 6 kg If the platform is stationary i.e., v0 = 0, then for pure rolling
R1 and R2 are the normal reactions of the support at the knife edges. motion v = Rw.
For translational equilibrium of the rod In general, friction is responsible for pure rolling motion.
R1 + R2 - W1 - W = 0 …(i) In pure rolling motion, friction is static in nature. In pure
rolling motion, friction is non-dissipative in nature i.e.,
Note W1 and W act vertically down and R1 and R2 act vertically up. For work done by friction force is zero, this is because of the
considering rotational equilibrium, we take moments of the forces. A fact that point of contact is relatively at rest.
convenient point to take moments about is G. The moments of R 2 and W1
In case of pure rolling motion, velocity of different
are anti-clockwise (+ve), whereas the moment of R1 is clockwise (–ve).
particles of the body is as shown below
For rotational equilibrium, 2v
-R1 (K1G) + W1 (PG) + R2 (K2G) = 0 …(ii)
2v
It is given that, v
⇒ v
W = 4 gN and W1 = 6 gN ω 2v
where, g = acceleration due to gravity.
We take, g = 9.8 m/s2 with numerical values inserted, Body performing pure Diagram showing velocity
rolling motion on ground. of certain points on body
R1 + R2 - 4 g - 6 g = 0
or R1 + R2 = 10 gN = 98 N …(iii) For above body, total kinetic energy is given by
From Eq. (ii), Iw2 mv2
K = KR + KT = +
- 0.25 R1 + 0.05 W1 + 0.25 R2 = 0 2 2
Telegram @unacademyplusdiscounts

Rotational Motion 319

Iw2 m (Rw) 2 (I + mR2 ) w2 Note


K = + =
2 2 2 • In the case of rolling, slipping and falling from the same height, the
I 2 w2 speeds of sliding and falling are equal but that of rolling is lesser,
K =
2 acceleration is maximum in falling and minimum in rolling.

where, I p = I + mR2 i. e. , the moment of inertia of the body • The falling body reaches the bottom first and the rolling body reaches
the bottom last.
about an axis passing through point of contact. K2
• As factor, b = 1 + 2 depends on shape of body and is independent
R
Rolling Motion on an Inclined Plane of mass, so if a solid and hollow body of same shape are allowed to
roll down an inclined plane then as b S < b H , solid body will reach the
We consider an inclined plane of q inclination on which a
bottom first with greater velocity.
body performs pure rolling motion. At any instant t, the
body is at a height from the horizontal surface.
• If a cylinder, ring, disc and sphere roll on inclined plane then as
bR = max while b S = min, the sphere will reach the bottom first with
We suppose that body is released from height h. greater velocity while ring will reach the bottom with least velocity.
According to conservation principle of energy, Table 9.1 Comparison of Various Motions
1 1
mgh = Iw2 + mvCM 2
R of a Body on an Inclined Plane.
2 2
T Motion \
But vCM = rw
h Physical Velocity Acceleration Time of Descend
1 1 s
\ mgh = Iw2 + mr 2w2 Quantity
2 2
θ Rolling æ 2 gh ö ar =
g sinq 1 æ 2 hb ö
1 æ I ö vr = ç ÷ Tr = ç ÷
mgh = mv2 ç1 + ÷ motion è b ø b sinq è g ø
2 è mr 2 ø g sinq
æ 2 gh ö =
1 æ K2ö = çç ÷ æ K2 ö
or = mv2 ç1 + 2 ÷ 2 2÷
è1 + K / R ø
çç1 + 2 ÷÷
2 è R ø è R ø
1 Sliding v s = 2 gh a s = g sinq 1 æ2 h ö
or mgh = mv2b = ç ÷
2 motion sinq è g ø
K2
where, b = 1+ 2
R Falling v f = 2 gh af = g æ2 h ö
motion tf = ç ÷
b is a constant for a given body. èg ø

The value of b does not depend upon mass and radius of


the body. It only depends upon shape of the body. The 2. Impure Rolling Motion
value of b is always greater or equal to one.
In impure rolling motion, the point of contact of the body
2 gh with the platform is not relatively at rest w.r.t. platform on
From the equation, v =
b which it is performing rolling motion, as a result sliding
2 2 gh occurs at the point of contact.
where , v =
b
For impure rolling motion,
h = s sin q
vAB ¹ 0 i. e. , v - Rw ¹ v0
dv 2 g sin q ds
or 2v =
dt b dt If platform is stationary i.e., v0 = 0, then condition for
ds impure rolling motion is v ¹ Rw.
but v=
dt
dv g sin q v
\ Acceleration in motion a = =
dt b ω
1 2
\ s = at A
v = Rω
2 v0
v0
1 2 hb
\Time of descent, t = B
sin q g
Here as, v ¹ Rw,
So, two bodies of the same shape but of different masses
and radii reach the bottom at the same time. So, a ¹ Ra
Telegram @unacademyplusdiscounts

320 JEE Main Physics

As in impure rolling motion, velocity of point of contact is whole body, q is measured from a fixed direction in the
not zero relative to the platform, kinetic friction comes into plane of motion, of P, which we take to be the x ¢-axis,
the existence and kinetic friction is given by m kN. chosen parallel to x-axis.
In this case, friction is opposing the relative motion i. e. , Angular velocity is the time rate of change of angular
rolling motion and is dissipative in nature i.e., work done displacement
by friction force is non-zero. dq
w=
dt

Check Point 2 dw
Angular acceleration, a =
dt
1. A ballet-dancer stretches her hands out for slowing down. This
is based on principle of conservation of.........
2. A cannon ball and a marble ball roll from rest down an incline
which goes to the bottom first?
9.10 Equation of
3. In a flywheel, most of the mass is concentrated at the rim. Rotational Motion
Why?
4. There is a stick half of which is wooden and half is of steel. It is The kinematical quantities in rotational motion, angular
pivoted at the wooden end and a force is applied at the steel displacement (q), angular velocity (w) and angular
end at right angles to its length. Next, it is pivoted at the steel acceleration (a ) respectively correspond to kinematic
end and the same force is applied at the wooden end. In which quantities in linear motion, displacement (x ), velocity (v)
case is the angular acceleration is more and why?
and acceleration (a ). The kinematical equations of linear
5. Why is it more difficult to revolve a stone by trying it to a motion with uniform (i. e. , constant) acceleration
longer string than by trying it to a shorter string?
v = v0 + at , …(i)
6. Explain, why the speed of whirlwind in a tornado is alarmingly
high? 1
x = x0 + v0 t + at 2 …(ii)
7. If the ice on the polar caps of the earth melts how will it effect 2
the duration of the day? and v2 = v02 + 2 ax …(iii)

where, x0 = initial displacement and v0 = initial velocity.


The word initial refers to values of the quantities at t = 0.
9.9 Rigid Body Rotation The corresponding kinematic equations for rotational
motion with uniform angular acceleration are
Taking rotation around a fixed axis, this case involves only
one degree of freedom i. e. , needs only one independent w = w0 t + at,
vaiable to describe the motion, this is translation, 1 2
q = q0 + w0 t + at
corresponds to linear motion. 2
z
and w2 = w20 + 2 ´ (q - q0 )

where, q0 = initial angular displacement of the rotating


body and w0 = initial angular velocity of the body.

Equivalence between Rotational and Linear Motion


y
C θ0
P
Linear Momentum Rotational Motion
p0
θ
Displacement x Angular displacement q
dx dq
x Linear velocity v = Angular velocity w =
dt dt
y Linear momentum p = mv Angular momentum L = Iw
O
dv dw
x Acceleration a = Angular acceleration a =
dt dt
[Axis of rotation is the z-axis and plane of motion is the x-y Force F = ma Torque t = Ia
plane] Impulse I = FDt = Dp Rotational impulse J = ò t dt = DL
The angular displacement of the rotating body, we take
any particle like P of the body. Its angular displacement q Work W = ò F × dS Work = ò t × dq
in the plane it moves is the angular displacement of the Power P = F × v Rotational power P = t × w
Telegram @unacademyplusdiscounts

Rotational Motion 321

Sample Problem 18 A cord of negligible mass is wound Interpret (c) We assume conservation of energy of the rolling
round the run of a flywheel of mass 20 kg and radius 20 cm. A body, i. e. ,there is no loss of energy due to friction etc. The potential
steady pull of 25 N is applied on the cord as shown. The energy lost by the body in rolling down the inclined plane ( = mgh)
flywheel is mounted on a horizontal axle with frictionless must, therefore be equal to kinetic energy gained.
bearings. Assume that the wheel starts from rest, the kinetic Since the bodies start from rest the kinetic energy gained is equal to
energy of the wheel is the final kinetic energy of the bodies.
1 2 æ k2 ö
K = mv CM ç1 + 2 ÷
2 è R ø
R where v CM is the final velocity of (the centre of mass) of the body.
Equating k and mgh, we have
1 æ k2 ö
M = 20 kg mgh = mv 2 ç1 + 2 ÷
F = 25N
2 è R ø
R = 20 cm

(a) 10 J (b) 20 J æ 2 gh ö
or v2 = ç ÷
(c) 30 J (d) 50 J è1 + k2/ R 2 ø

Interpret (a) We use, Ia = t Note It is independent of the mass of the rolling body.
The torque, t = FR
For a ring, k2 = R 2
= 25 ´ 0.20 Nm (as R = 0.20 m)
2 gh
= 5 Nm v ring = = gh
1+ 1
MR 2
I = moment of inertia of flywheel about its axis = R2
2 For a solid cylinder, k2 =
2
20 ´ (0.2) 2
= = 0.4 kg m2 2 gh 4
2 v disc = = gh
1 + 1/2 3
5 Nm
a = angular acceleration = = 12.5 rad s-2 2 R2
0.4 kgm2 For a solid sphere, k2 =
5
Let w be the final angular velocity. The kinetic energy gained
1 2 gh 10
= Iw2 v sphere = = gh
2 7
2 1+
5
Since the wheel starts from rest. Now,
From the results obtained it is clear that among the three bodies,
w2 = w20 + 2 aq, w0 = 0 the sphere has the greatest and the ring has the least velocity of
The angular displacement q = length of unwound string/radius of the centre of mass at the bottom of the inclined plane.
wheel
2m Sample Problem 20 The angular speed of a motor wheel
= = 10 rad is increased from 1200 rpm to 3120 rpm is 16 s. Number of
0.2m
revolutions the engine makes during this time is
w2 = 2 ´ 12.5 ´ 10 = 250 (rad/s) 2
(a) 50 (b) 326
1 (c) 252 (d) 576
\ KE gained = ´ 0.4 ´ 250 = 50 J
2
Interpret (d) The angular displacement is time t is
Sample Problem 19 Three bodies, a ring, a solid cylinder 1 2
and a solid sphere roll down the same inclined plane without q = w0t + at
2
slipping. They start from rest. The radii of the bodies are æ 1 ö
identical. Which of the body reaches the ground with maximum = ç 40 p ´ 16 + ´ 4 p ´ 16 2÷ rad
è 2 ø
velocity?
(a) Ring = (640 p + 512 p ) rad
(b) Cylinder = 1152 p rad
(c) Sphere 1152 p
Number of revolutions = = 576
(d) All have the same velocity 2p
Telegram @unacademyplusdiscounts

WORKED OUT
Examples
Example 1 A disc starts rotating with constant angular Example 4 The moment of inertia of uniform semicircular
acceleration p rad s-2 about a fixed axis perpendicular to its disc of mass M and radius R about a line perpendicular to the
plane and through its centre. The angular velocity of the disc plane of disc and passing through the centre is
after 4 s is MR 2 2
(a) (b) MR 2
-1 -1 4 5
(a) 2 p rad s (b) 3p rad s
(c) 4p rad s-1 (d) 5p rad s-1 MR 2
(c) MR 2 (d)
2
Solution Here, a = p rad s-2
Solution The given section is 1 of the disc, then mass of the disc
w0 = 0 , t = 4 s 2
w4s = 0 + p ´ 4 = 4p rad s -2 is 2 M
1
Idisc = (2M)R 2
2
Example 2 In the above example what is the angular
1
displacement the disc after 4s ? Isection = Idisc
2
(a) 2 p rad (b) 3p rad
1
(c) 6p rad (d) 8p rad = MR 2
2
Solution Angular displacement
1 2 1 Example 5 If I1 is the moment of inertia of a thin rod about
q = w0t + at = 0 + ´ 16 = 8 p rad
2 2 an axis perpendicular to its length and passing through its
centre of mass and I2 is the moment of inertia of ring about an
Example 3 The motor of an engine is rotating about its axis axis perpendicular to plane of ring and passing through its
with an angular velocity of 100 rev min -1. It come to rest in centre formed by bending the rod, then
15 s, after being switched off. Assuming constant angular I1 3
(a) =
deceleration. What are the number of revolutions made by it I2 p 2
before coming to I 2
(b) 1 = 2
(a) 15.6 (b) 32.6 I2 p
(c) 12.5 (d) 40
I1 p 2
(c) =
Solution As wf = wi + at I2 2
Here, 0 = w0 - a t I1 p 2
(d) =
w0 (100 ´ 2p ) / 60 I2 3
\a= = = 0.7 rad s -2
t 15
Solution Here, as ring is made by bending the rod l = 2 pR,
Angle rotated before coming to rest.
l
w20 R=
q= 2p
2a
ml 2
æ100 ´ 2 p ö
2 I1 = ,
ç ÷ 12
è 60 ø
or q= = 78.33 rad ml 2
2 ´ 0.7 I2 = mR 2 =
4p 2
q 78.33
Hence, number of rotations n = = = 12.5 I1 ml 2 ml 2 p 2
2p 2p = / =
I2 12 4p 2 3
Telegram @unacademyplusdiscounts

Rotational Motion 323

Example 6 The ratio of radii of gyration of a hollow sphere Example 9 A constant torque acting on a uniform circular
and a solid sphere of same radii about a tangential axis is wheel changes its angular momentum from A0 to 4A0 in 4s. the
7 5 21 25 magnitude of this torque is
(a) (b) (c) (d) 3A0
3 21 5 9 (a) (b) 4A0
4
2
Solution Moment of Inertia, I = mK (c) A0 (d) 12A0
2
mR 2 + mR 2 Solution Angular impulse = Change in angular momentum
I 3
\ Radius of gyration Khollow = = \ t ´ t = Lf - Li
m m
4t = 4A 0 - A 0
5
= R 3A 0
3 \ Torque t=
4
[I = ICM + mR 2 parallel axis theorem]
2 Example 10 A solid sphere of mass M rolls without slipping
mR 2 + mR 2
I 5 7 on an inclined plane of inclination q. What should be the
\Similarly, Ksolid = = = R
m m 5 minimum coefficient of friction, so that the sphere rolls down
Khollow 25 5 without slipping ?
= = 2 2
Ksolid 21 21 (a) tan q (b) tan q
5 7
5
Example 7 A particle of mass 5 g is moving with uniform (c) tan q (d) tan q
7
speed of 3 2 cms-1 is x-plane along the line y = 2 5 cm. The
magnitude of its angular momentum about the origin in Solution This is a case of rolling on rough inclined plane Force
g cm 2 s-1is of friction in this case will be backward.
Mg sin q - f
(a) 30 (b) Zero a= …(i)
M
(c) 30 2 (d) 30 10
fR
a=
Solution Angular momentum L = mvr^ I
M, R, I
= 5.3 2 × 2 5 = 30 10 gcm2 s-1 a

α
Example 8 A particle of mass m = 5 unit is moving with a θ
sin
g
uniform speed v = 3 2 unit is XY-plane along the line y = x + 4. M
the magnitude of the angular momentum about origin is θ
(a) zero (b) 60 unit
(c) 7.5 units (d) 40 2 unit For pure rolling to take place
a = Ra …(iii)
Solution Angular momentum L = m [ r ´ v ] = mvr sin q Solving Eqs. (i), (ii) and (iii), we get
Mg sin q
f= …(iv)
(0, 4) MR 2
1+
I
Further, the force of friction calculated in Eq. (iv) for pure rolling
to take place should be less than or equal to the maximum
(–4, 0) friction m mg cos q
Mg sin q
or £ m mg cos q
MR 2
Given y =x+ 4 1+
I
Comparing with general equation y = x + 4,
tan q
we get m =1 m min £
MR 2
tan q = 1 1+
I
Þ q = 45° tan q 2
1 = = tan q
Now, L = mvr sin q = 5 ´ 3 2 ´ 4 ´ = 60 unit 5 7
1+
2 2
Telegram @unacademyplusdiscounts

324 JEE Main Physics

Example 11 A solid sphere and a solid cylinder of same Example 12 An energy of 484 J is spent in increasing the
mass are rolled down on two inclined planes of heights h1 and h2 speed of flywheel from 60 rpm to 360 rpm. The moment of
respectively. If at the bottom of the plane of the two objects inertia of wheel is
have some linear velocities, then the ratio of h1 : h2 is (a) 0.2 kg m2 (b) 0.7 kg m2
(a) 2 : 3 (b) 7 : 5 (c) 2 kg m2 (d) 3 kg m2
(c) 14 : 15 (d) 15 : 14
Solution Here, energy spent, W = 484 J;
60
Initial speed, w1 = 60 rev min -1 = ´ 2p rad s -1
s 60
θ = 2 p rad s -1
Final speed, w2 = 360 rev min -1
Solution Here, v1 = v 2 360
= = 2p rad s -1
60
2as1 = 2a2s2
= 12 p rad s -1

h1 h I =?
2a1 = 2a2 2
sin q sin q As work done = increase in KE of rotations
h1 a2 \ W = KE2 - KE1
\ =
h2 a1 1 1
= Iw22 - w12
g sin q g sin q g sin q 2 2 2
Here, a2 = = = = g sin q 1
I2 1 1 3 484 = I[(12p ) 2 - (2p ) 2]
1+ 1 + 1 +
m2 R 2 2 2 2
1
g sin q g sin q 5 = I ´ 140 p 2
a1 = = = g sin q 2
I1 2 7
1+ 1 + 22 22
m1 R 2 5 484 = 70I ´ ´
7 7
h1 14
= 484 ´ 49
h2 15 I= = 0.7 kg m2
70 ´ 484
Telegram @unacademyplusdiscounts

Start Practice for


JEE Main
Round I (Topically Divided Problems)

Moment of Inertia 6. Three particles each of mass m x C

1. Moment of inertia of a uniform circular disc about a gram, are situated at the
diameter is I. Its moment of inertia about an axis vertices of an equilateral l l

perpendicular to its plane and passing through a triangle ABC of side l cm (as
point on its rim will be shown in figure). The moment B
y
A
(a) 5 I (b) 3 I
of inertia of the system about a l

(c) 6 I (d) 4 I
line AX perpendicular to AB
and in the plane of ABC in g-cm2 unit will be
2. In a rectangle ABCD ( BC = 2 AB). B
F
C 3 2
The moment of inertia along (a) ml (b) 2 ml2
4
which axes will be minimum. E G 5 3 2
(c) ml2 (d) ml
(a) BC (b) BD 4 2
(c) HF (d) EG A D
H 7. The moment of inertia of a sphere of mass M and
radius R about an axis passing through its centre is
3. Two wheels A and B are mounted on the same axle. 2
Moment of inertia of A is 6 kgm2 and it is rotating at MR2 . The radius of gyration of the sphere about a
5
600 rpm when B is at rest. What is moment of inertia parallel axis to the above and tangent to the sphere is
of B, if their combined speed is 400 rpm? æ 7ö æ 3ö
7 3
(a) 8 kg m2 (b) 4 kg m2 (a) R (b) R (c) ç ÷R (d) ç ÷R
5 5 è 5ø è 5ø
(c) 3 kg m2 (d) 5 kg m2
4. Moment of inertia of a thin rod of mass M and length 8. A particle of mass m is moving in z
2 yz-plane with a uniform velocity v
ML
L about an axis passing through its centre is . Its v
with its trajectory running a 12
moment of inertia about a parallel axis at a distance parallel to + ve y-axis and
L intersecting z-axis at z = a. The
of from this axis is given by
4 change in its angular momentum y
ML2 ML3 ML2 7 ML2 about the origin as it bounces
(a) (b) (c) (d)
48 48 12 48 elastically from a wall at y = constant is
[NCERT Exemplar]
5. Two thin uniform circular rings each of radius 10 cm
and mass 0.1 kg are arranged such that they have a (a) mva e x (b) 2 mva e x
common centre and their planes are perpendicular to (c) ymv e x (d) 2ymv e x
each other. The moment of inertia of this system 9. Three identical thin rods each of length l and mass M
about an axis passing through their common centre are joined together to form a ladder H. What is the
and perpendicular to the plane of one of the rings in moment of inertia of the system about one of the sides
kgm2 is of H ?
(a) 15 ´ 10 -3 (b) 5 ´ 10 -3 Ml 2 Ml 2 Ml 2 Ml 2
(a) (b) (c) 2 (d) 4
(c) 1.5 ´ 10 –3 (d) 18 ´ 10 -4 4 3 3 3
Telegram @unacademyplusdiscounts

326 JEE Main Physics

10. Two discs have same mass and thickness. Their 17. Four spheres of diameter 2 a and mass M are placed
materials have densities d1 and d2 . The ratio of their with their centres on the four corners of a square of
moments of inertia about central axis will be side b. Then the moment of inertia of the system
(a) d1 : d2 (b) d1d2 : 1 about an axis along one of the sides of the square is
(c) 1 : d1d2 (d) d2 : d1 4 8
(a) Ma2 + 2 Mb2 (b) Ma2 + 2 Mb2
5 5
11. When a disc rotates with uniform angular velocity,
8 4
which of the following is not true? [NCERT Exemplar] (c) Ma2 (d) Ma2 + 4 Mb2
5 5
(a) The sense of rotation remains same
(b) The orientation of the axis of rotation remains same 18. A uniform square plate has a small piece Q of an
(c) The speed of rotation is non-zero and remains same irregular shape removed and glued to the centre of
(d) The angular acceleration is non-zero and remains same the plate leaving a hole behind. The moment of
inertia about the z-axis is then [NCERT Exemplar]
12. Of the two eggs those have identical sizes, shapes
y y
and weights, one is raw, and other is half boiled. The
(a) increased hole
ratio between the moment of inertia of the raw to the Q
(b) decreased
half boiled egg about central axis is x x
(c) the same
(a) one (b) greater than one
(d) changed in unpredicted manner
(c) less than one (d) not comparable
19. The radius of gyration of a uniform rod of length L
13. Moment of inertia of a solid cylinder of length L and
about an axis passing through its centre of mass and
diameter D about an axis passing through its centre
perpendicular to its length is
of gravity and perpendicular to its geometric axis is
(a) L / 12 (b) L2 /12 (c) L / 3 (d) L / 2
æ D 2 L2 ö æ L2 D2 ö
(a) M ç + ÷ (b) M ç + ÷
è 4 12 ø è16 8 ø
æ D 2 L2 ö æ L2 D 2 ö
Torque, Angular Momentum
(c) M ç + ÷ (d) M ç + ÷
è 4 6ø è 12 16 ø and Kinetic Energy of Rotation;
14. A thin rod of length L and mass M O
Conservation of Angular Momentum
is bent at the middle point O at an 20. If the earth suddenly changes its radius x times the
60° present value, the new period of rotation would be
angle of 60°. The moment of L/2 L/2
inertia of the rod about an axis (a) 6 x2 h (b) 12 x2 h (c) 24 x2 h (d) 48 x2 h
passing through O and
21. What torque will increase angular velocity of a solid
perpendicular to the plane of the
disc of mass 16 kg and diameter 1 m from zero to
rod will be
120 rpm in 8 s?
ML2 ML2 p p p
(a) (b) (a) N-m (b) N-m (c) N-m (d) p N-m
6 12 4 2 3
ML2 ML2
(c) (d) 22. A force of 100 N is applied perpendicularly to the left
24 3
edge of the rectangle as shown in figure. The torque
15. A bullet of mass 10 g and speed 500 m/s is fired into a (magnitude and direction) produced by this force
door and gets embedded exactly at the centre of the with respect to an axis perpendicular to the plane of
door. the door is 1.0 m wide and weight 12 kg. It is the rectangle at corner A and with respect to a
hinged at one end and rotates about a vertical axis similar axis at corner B are respectively.
practically without friction. The angular speed of the A
door just after the bullet embeds into it is 0.75 m
(a) 0.390 rad/s (b) 0.625 rad/s
100 N
(c) 0.062 rad/s (d) 3.9 rad/s
1.25
16. The moment of inertia of a dumb-bell, consisting of
B
point masses m1 = 2.0 kg and m2 = 1.0 kg, fixed to the
ends of a rigid massless rod of length L = 0.6 m, about (a) 75 N-m counter clockwise, 125 N-m clockwise
an axis passing through the centre of mass and (b) 125 N-m counter clockwise, 75 N-m clockwise
perpendicular to its length, is (c) 125 N-m clockwise, 75 N-m counter clockwise
(a) 0.72 kg m2 (b) 0.36 kg m2 (d) 125 N-m clockwise, 75 N-m counter clockwise
(c) 0.27 kg m2 (d) 0.24 kg m2
Telegram @unacademyplusdiscounts

Rotational Motion 327

23. What constant force tangential to the equator should 30. A ring of diameter 0.4 m and of mass 10 kg is rotating
be applied to the earth to stop its rotation is one day? about its axis at the rate of 1200 rpm. The angular
22
(a) 1.3 ´ 10 N (b) 8.26 ´ 10 N 28 momentum of the ring is
(c) 1.3 ´ 1023 N (d) None of these (a) 60.28 kg-m2 s–1 (b) 55.26 kg-m2s–1
(c) 40.28 kg-m2s–1 (d) 50.28 kg-m2s–1
24. A cord is wound round the circumference of a wheel of
radius r. The axis of the wheel of horizontal and
31. The oxygen molecule has a mass of 5.30 ´ 10-26 kg
and a moment of inertia of 1.94 ´ 10-46 kg-m -2 about
moment of inertia about it is I. A weight mg is
an axis through its centre perpendicular to the lines
attached to the end of the cord and falls from rest.
joining the two atoms. Suppose the mean speed of
After falling through a distance h, the angular
such a molecule in a gas is 500 m/s and that is KE of
velocity of the wheel will be 2
1/2 1/2 rotation is of its KE translation. Find the average
æ 2 gh ö æ 2 mgr ö 3
(a) ç ÷ (b) ç ÷
è1 + mr ø è1 + mr2 ø angular velocity of the molecule.
1/2
æ 2 mgh ö (a) 3.75 ´ 1012 rad/s (b) 5.75 ´ 1012 rad/s
(c) ç ÷ (d) (2 gh)1/2
è1 + 2m ø (c) 9.75 ´ 1012 rad/s (d) 6.75 ´ 1012 rad/s
25. A 3 kg particle moves with constant speed of 2 ms–1 32. A ring and a disc of different masses are rotating
in the xy-plane in the y-direction along the line with the same kinetic energy. If we apply a retarding
x = 4 m. The angular momentum (in kg-m2s -1) torque t on the ring stops after making n revolutions,
relative to the origin and the torque about the origin then in how many revolutions will the disc stop under
needed to maintain this motion are respectively the same retarding torque?
(a) 12, 0 (b) 24, 0 (a) n (b) 2n
(c) 0, 24 (d) 0, 12 (c) 4 n (d) n/2
26. A horizontal force F is applied a
33. A flywheel of moment of inertia 0.4 kg-m2 and radius
such that the block remains 0.2 m is free to rotate about a central axis. If a string
stationary, then which of the a
is wrapped around it and it is pulled with a force of
following statement is false? F
10N, then its angular velocity after 4 s will be
(a) F = mg (where f is the frictional (a) 10 rad s–1 (b) 5 rad s–1
force) (c) 20 rad s–1 (d) None of these
(b) F = N (where N is the normal
force) 34. A hoop of radius 2 m weight 100 kg. It rolls along a
(c) F will not produce torque horizontal floor so that its centre of mass has a speed
(d) N will not produce torque of 20/cm. How much work has to be done to stop it?
(a) zero (b) mv2 / 4 2g
27. What is the torque of the force F = (2 i - 3 $i + 4 k$ ) N 2
$ ) m about the (c) mv / 2g (d) m (2 gh2 )
acting at the point r = (3 $i + 2 $j + 3 k
origin?
35. A stone of mass m tied to a string of length l is
rotating along a circular path with constant speed v.
(a) -17i$ + 6 $j + 13 k$ (b) -6i$ + 6$j - 12 k$
The torque on the stone is [NCERT]
(c) 17$i - 6 $j - 13 k$ (d) 6 $i - 6 $j + 12 k$
(a) 6.0 J (b) 5.0 J
28. The moment of inertia of a body about a given axis is (c) 9.0 J (d) 4.0 J
1.2 kg-m2. Initially, the body is at rest. In order to 1
36. Four 2 kg masses are connected by m spokes to an
produce a rotational kinetic energy of 1500 J, and 4
angular acceleration of 25 rads–2 must be applied axle. A force of 24 N acts on a lever 1/2 m long to
about that axis for a duration of produce angular acceleration a. The magnitude of a
(a) 4 s (b) 2 s in rad s–2 is
(c) 8 s (d) 10 s (a) 24 (b) 12 (c) 6 (d) 3
2 2
29. A flywheel of moment of inertia 3 ´ 10 kg-m is 37. A gramophone turn table rotating at 75 rpm slow
rotating with uniform angular speed of 4.6 rad/s. If a down uniformly and stops in 5 s after the motor is
torque of 6.9 ´ 102 N-m retards the wheel, then the turned-off. Its angular acceleration (rad s–2)
time in which the wheel comes to rest is (a) –0.42 (b) –0.89
(a) 1.5 s (b) 2 s (c) –1.57 (d) –1.96
(c) 0.5 s (d) 1 s
Telegram @unacademyplusdiscounts

328 JEE Main Physics

38. When a ceiling fan is switched off, its angular 44. A ballet dancer spins with 2.8 rev s–1 with her arms
velocity fall to half while it makes 36 rotations. How out stretched. When the moment of inertia about the
many more rotations will it make before coming to same axis becomes 0.7 I, the new rate of spin is
rest? (Assume uniform angular retardation) (a) 3.2 rev s–1 (b) 4.0 rev s–1
(a) 36 (b) 24 (c) 4.8 rev s–1 (d) 5.6 rev s–1
(c) 18 (d) 12 45. A Merry-go-round, made of a ring-like platform of
39. A wheel starts from rest and acquires a rotational radius R and mass M, is revolving with angular
speed of 240 rps in 2 min. Its acceleration is speed w. A person of mass M is standing on it. At one
(a) 5 rps2 (b) 2 rps2 instant, the person jumps off the round, radially
(c) 8 rps2 (d) 11 rps2 away from the centre of the round (as seen from the
40. A particle of mass M moves along the line PC with round). The speed of the round afterward is
[NCERT Exemplar]
velocity v as shown. What is the angular momentum
w
of the particle about O ? (a) 2 w (b) w (c) (d) 0
2

C
46. A particle performs uniform circular motion with an
L
v
angular momentum L . If the frequency of a particle’s
M
motion is doubled and its kinetic energy is halved,
P
90° r the angular momentum becomes
l (a) 2 L (b) 4 L (c) L/2 (d) L/4
O 47. A circular platform is mounted on a vertical friction
(a) mvL (b) mvl less axle. Its radius is r = 2 m and its moment of
(c) mvr (d) Zero inertia I = 200 kg-m 2 . It is initially at rest. A 70 kg
man stands on the edge of the platform and begins to
41. A particle starts from rest with an acceleration of
walk along the edge at speed v0 = 1.0 ms –1 relative to
2 rad s–2 in a circle of radius 2 m. Its linear speed
the ground. The angular velocity of the platform is
after 6 s is
0.7 rad s–1. When the man has walked once around
(a) 12 ms–1 (b) 24 ms–1
the platform, so that he is at his original position on
(c) 4 ms–1 (d) None of these
it, his angular displacement relative to ground is
42. A thin and circular disc of mass and radius R is 6 5 4 5
(a) p (b) p (c) p (d) p
rotating in a horizontal plane about axis passing 5 6 5 4
through its centre and perpendicular of its plane
48. If earth where to shrink to half its present diameter
with an angular velocity w. If another disc of same
M without any change in its mass, the duration of the
dimensions but of mass is placed gently on the day will be
4
(a) 48 h (b) 6 h (c) 12 h (d) 24 h
first disc coaxially, then the new angular velocity of
the system is 49. Two discs of moment of inertia I1 and I2 about their
5 2 respective axes and rotating with angular speed w1
(a) w (b) w
4 3 and w2 are brought into contact face to face with their
4 3 axes of rotation coincident. Then the loss of in kinetic
(c) w (d) w
5 2 energy of the system in the process is
I1I2 I1 I2
43. A thin uniform rod AB of mass m and length L is (a) ( w1 - w2 ) 2 (b) - ( w1 - w2 ) 2
2( I1 + I2 ) 2( I1 + I2 )
hinged at one end A to the level floor. Initially, it
I I
stands vertically and is allowed to fall freely to the (c) 1 2 ( w1 - w2 ) 2 (d) zero
( I1 + I2 )
floor in the vertical plane. The angular velocity of the
rod, when its end B strikes the floor is 50. A man of 80 kg mass is standing on the rim of a
(g is acceleration due to gravity) circular platform of mass 200 kg rotating about its
1/2
æ mg ö æ mg ö axis. The mass of the platform with the man on it
(a) ç ÷ (b) ç ÷
è L ø è 3L ø rotates at 12.0 rpm. If the man now moves to centre of
1/2 the platform, the rotational speed would become
æ gö æ3 gö
(c) ç ÷ (d) ç ÷ (a) 16.5 rpm (b) 25.7 rpm
è Lø è L ø
(c) 32.3 rpm (d) 31.2 rpm
Telegram @unacademyplusdiscounts

Rotational Motion 329

Motion of Rolling Bodies on 58. A body of mass M slides down an inclined and
reaches the bottom with a velocity v if the same mass
Horizontal and Inclined Planes were in the form of a ring which rolls down the
51. A sphere and a hollow cylinder roll without slipping incline, the velocity of the ring at bottom
down two separate inclined planes and travel the [NCERT Exemplar]
same distance in the same time. If the angle of the (a) v (b) 2 v
plane down which the sphere rolls is 30°, the angle of 1 2
the other plane is (c) v (d) v
2 5
(a) 60º (b) 53º
(c) 37º (d) 45º 59. A circular disc rolls down an inclined plane. The ratio
of the rotational kinetic energy to total kinetic energy
52. A rupee coin starting from rest rolls down a distance
is
of 1 m on an inclined plane at angle of 30° with the 1 1
horizontal. Assuming that g = 9.81 ms–2, time taken (a) (b)
2 3
is 2 3
(a) 0.68 s (b) 0.8 s (c) (d)
3 4
(c) 0.5 s (d) 0.7 s
60. An inclined plane makes an angle of 30° with
53. A solid cylinder (SC), a hollow cylinder (HC) and a horizontal. A solid sphere rolling down the inclined
solid sphere (S) of the same mass and radius are plane from rest without slipping has a linear
released simultaneously from the same height of acceleration equal to
incline. The order in which these bodies reach the (a) 5g/14 (b) 5g/4 (c) 2g/3 (d) g/3
bottom of the incline is
(a) SC, HC, S (b) SC, S, HC 61. A sphere of mass m and radius r rolls on a horizontal
(c) S, SC, HC (d) HC, SC, S plane without slipping, with the speed u. Now if it
rolls up vertically the maximum height that would be
54. A solid sphere rolls down without slipping on an attained, is
inclined plane at angle 60° over a distance of 10 m.
3u 2 5u 2
The acceleration (in ms–2) is (a) (b)
4g 2g
(a) 4 (b) 5
7u 2 u2
(c) 6.06 (d) 7 (c) (d)
10 g 2g
55. A hemispherical bowl or radius R is kept on a
horizontal table. A small sphere of radius r ( r << R) is 62. A solid cylinder on moving with constant speed v0
placed at the highest point at the inside of the bowl reaches the bottom of an incline of 30°. A hollow
and let go. The sphere rolls without slipping. Its cylinder of same mass and radius moving with the
velocity at the lowest point is same constant speed v0 reaches the bottom of a
(a) 5 gR /7 (b) 3 gR / 2 different incline of inclination q. There is no slipping
and both of them go through the same distance in the
(c) 4 gR / 3 (d) 10 gR /7
same time; q is then equal to
56. A marble and a cube have the same mass starting (a) 37º (b) 30º (c) 42º (d) 45º
from rest, the marble rolls and the cube slides down a 63. A ring starts to roll down the inclined plane of height
frictionless ramp. When they arrive at the bottom, h without slipping. The velocity with which it reaches
the ratio of speed of the cube to the centre of mass and the ground is
speed of the marble is 10 gh 4 gh 4 gh
(a) 7 : 5 (b) 7 : 5 (a) (b) (c) (d) 2 gh
7 7 3
(c) 2 : 1 (d) 5 : 2 (e) gh
57. A thin metal disc of radius 0.25 m and mass 2 kg 64. A wheel of mass 8 kg and radius 40 cm is rolling on a
starts from rest and rolls down an inclined plane. If horizontal road with angular velocity of 15 rad s–1.
its rotational kinetic energy is 4 J at the foot of the The moment of inertia of the wheel about its axis is
inclined plane, then its linear velocity at the same 0.64 kg m–2. Total kinetic energy of wheel is
point is (a) 288 J (b) 216 J
(a) 1.2 ms–1 (b) 2 2 ms–1 (c) 72 J (d) 144 J
(c) 20 ms–1 (d) 2 ms–1
Telegram @unacademyplusdiscounts

330 JEE Main Physics

Round II (Mixed Bag)

Only One Correct Option 8. A cubical block of side a is a

1. The moment of inertia of a uniform horizontal moving with velocity v on a v


M
cylinder of mass M about an axis passing through its horizontal smooth plane as
edge and perpendicular to the axis of the cylinder shown. It hits a ridge at point
when its length is 6 times its radius R is O. The angular speed of the
39 39 block after is hits O is O
(a) MR2 (b) MR 3v 3v 3v
4 4 (a) (b) (c) (d) zero
49 49 4a 2a 2a
(c) MR (d) MR2
4 4
9. A disc of mass M and radius R y
2. The moment of inertia of two spheres of equal masses is rolling with angular speed w
about their diameters are equal. If one of them is on a horizontal plane as shown. ω
solid and other is hollow, the ratio of their radii is The magnitude of angular M
(a) 3 : 5 (b) 3 : 5 momentum of the disc about O x
(c) 5 : 3 (d) 5 : 3 the origin O is
1 3
3. A thin wire of length l and mass m is bent (a) MR2 w (b) MR2 w
2 2
in the form of semicircle. Its moment of (c) MR2 w (d) 2 MR2 w
m
inertia about an axis joining its free ends
will be 10. A particle performs uniform circular motion with an
O
(a) ml2
(b) zero
r angular momentum L, if the frequency of particles
(c) ml2 / 2p2 (d) None of these motion is doubled and its kinetic energy is halved,
P the angular momentum becomes
(a) 4 L (b) 0.5 L
4. A flywheel rotates with a uniform angular (c) 2 L (d) 0.25 L
acceleration. Its angular velocity increases from
20 p rads–1 to 40 p rads–1 in 10 s. How many rotations 11. A uniform rod of length 2 L is placed with one end in
did it make in this period? contact with the horizontal and is then inclined at an
(a) 80 (b) 100 (c) 120 (d) 150 angle a to the horizontal and allowed to fall without
stopping at contact point when it becomes horizontal.
5. Two bodies of moment of inertia I1 and I2 ( I1 > I2 ) Its angular velocity will be
have equal angular momenta. If E1, E2 are their
3 g sin a 2L
kinetic energies of rotation, then (a) w = (b) w =
2L 3g sin a
(a) E1 > E2 (b) E1 = E2
(c) E1 < E2 (d) Cannot be said 6 g sin a L
(c) w = (d) w =
L g sin a
6. A solid cylinder is rolling down on an inclined plane
of angle q. The coefficient of static friction between 12. Two thin discs each of mass M and radius R are
the plane and cylinder is m s . The condition for the placed at either end of a rod of mass m, length l and
cylinder not to slip is radius r. Moment of inertia of the system about an
(a) tan q ³ 3 m s (b) tan q > 3 m s axis passing through the centre of rod and
(c) tan q £ 3 m s (d) tan q < 3 m s perpendicular to its length is
7. A thin circular ring of mass M and radius r is rotating mL2 1 1
(a) + MR 2 + ML2 M Axis of rotation M
about its axis with a constant angular velocity w. Two 12 4 4
objects each of mass M are attached gently to the ML2 1 1
(b) + mR 2 + mL2
opposite ends of a diameter of the ring. The ring will 12 2 2
2 R m 2r R
now rotate with an angular velocity 1 mR ML2
(c) mL2 + +
w( M - 2m ) wM 2 2 12
(a) (b)
M + 2m M + 2m mL2 2 1 2
wM w( M + 2M ) (d) + MR + ML
(c) (d) 12 2 L
M+m M
Telegram @unacademyplusdiscounts

Rotational Motion 331

13. Four particles each of mass m are lying 19. Three rods each of length L and mass M are placed
symmetrically on the rim of a disc of mass M and along X, Y and Z axes in such a way that one end of
radius R. Moment of inertia of this system about an each rod is at the origin. The moment of inertia of the
axis passing through one of the particles and system about Z-axis is
perpendicular to plane of disc is ML2 2 ML2 3 ML2 2 ML2
(a) (b) (c) (d)
R2 3 3 2 12
(a) 16 mR2 (b) 3 ( M + 16 m )
2 20. A solid sphere of mass M and radius R spins about an
R2 axis passing through its centre making 600 rpm. Its
(c) (3M + 12m ) (d) zero
2 kinetic energy of rotation is
2 2 2
14. Two uniform thin rods each of mass M and length l (a) p MR (b) pM2 R2 (c) 80 p2 MR2 (d) 80 pR
5 5
are placed along X and Y axis with one end of each at
the origin. Moment of inertia of the system about 21. A ring of radius R is first rotated with an angular
Z-axis is velocity w0 and then carefully placed on a rough
3 2 2 2 horizontal surface. The coefficient of friction between
(a) ML (b) ML the surface and the ring is m. Time after which its
2 3
angular speed is reduced to half is
(c) 2 ML2 (d) None of these
w0mR 2 w0 R w0 R w0 g
(a) (b) (c) (d)
15. If the radius r of earth suddenly changes to x times 2g mg 2m g 2 mR
the present values, the new period of rotation would 22. What is the moment of inertia of solid sphere of
be density r and radius R about its diameter?
(a) dT /dt = (T /r ) ( dr/dt ) (b) dT /dt = (2T /r ) ( dr/dt ) 105 5 105 2 176 5 176 2
æ 1 ö æ dr ö (a) R r (b) R r (c) R r (d) R r
(c) dT /dt = ( r/T ) ( dr/dt ) (d) dT /dt = ç T /r ÷ ç ÷ 176 176 105 105
è 2 ø è dt ø
23. If the moment of inertia of a disc about an axis
16. The curve between log e L and log e p is (L is angular tangential and parallel to its surface be I, then what
momentum and p is linear momentum) will be the moment of inertia about the axis
(a) (b) tangential but perpendicular to the surface?
6 3 3 5
(a) I (b) I (c) I (d) I
5 4 2 4
log L

log L

24. A thin uniform rod pivoted at O z


ω
is rotating in the horizontal
log p log p plane will constant angular O v
speed was shown in the figure. At
(c) (d) time t = 0 a small insect starts
from O and moves with constant
log L

log L

speed v with respect to the rod towards the other end


it reaches the end of the rod at t = T and stops. The
angular speed of system remains w throughout the
log p log p magnitude of torque ( t) on the system about O as a
function of time is best represented by which plot
17. A rod of length l is hinged at one end and kept z z
horizontal. It is allowed to fall. The velocity of the
other end of the rod is
(a) |r| (b) |r|
(a) 3 gl (b) 2 gl
(c) 2 Ml 2 (d) None of these
t t
18. Two particles, each of mass m and speed v, travel in T T
opposite directions along parallel lines separated by z z
a distance d. The vector angular momentum of the
two particle system is whatever be the point an out (d) |r|
(c) |r|
which the angular momentum is taken.
(a) same (b) different
(c) may be same or not (d) can not be said t t
T T
Telegram @unacademyplusdiscounts

332 JEE Main Physics

25. A rectangular block has a square base measuring ω(t) ω(t)


a ´ a, and its height is h. It moves on a horizontal
surface in a direction perpendicular to one of its ω0 ω0
(a) (b)
edges. The coefficient of friction is m. It will topple if
(a) m > h / a (b) m > a / h
2a a
(c) m > (d) m > t t
h 2h
ω(t) ω(t)
26. Two spheres each of mass M and radius R / 2 are
connected with a massless rod of length 2 R as shown ω0 ω0
(c) (d)
in the figure. What will be the moment of inertia of
the system about an axis passing through the centre
of one of the sphere and perpendicular to the rod?
t t
M M
R/2 R/2 30. The mass of the earth is increasing at the rate of
1 part in 5 ´ 1019 per day by the attraction of meteors
2R falling normally on the earth’s surface. Assuming
21 2 that the density of earth is uniform, the rate of
(a) MR2 (b) MR2 change of the period of rotation of the earth is
5 5
5 5 (a) 2.0 × 10–20 (b) 2.66 × 10–19
(c) MR2 (d) MR2
2 21 (c) 4.33 × 10–18 (d) 5.66 × 10–17

27. A uniform rod of mass m 31. A cylinder of mass M, length L and radius R. If its
and length l is suspended moment of inertia about an axis passing through its
by means of two light centre and perpendicular to its axis is minimum, the
inextensible strings as ratio L / R must be equal to
shown in figure. Tension in (a) 3/2 (b) 2/3
one string immediately A B (c) 2 / 3 (d) 3 / 2
after the other string is
32. Four holes of radius R are cut from a thin square
cut is
plate of side 4 R and mass M. The moment of inertia
mg
(a) (b) mg of the remaining portion about z-axis is
2 p
mg (a) MR2 y
(c) 2 mg (d) 12
4
æ4 pö
(b) ç - ÷ MR2
28. Two particles of masses m1 and m2 are connected by a è3 4ø
rigid massless rod of length r to constitute a æ4 pö
x
dumb-bell which is free to move in the plane. The (c) ç - ÷ MR2
è3 6ø
moment of inertia of the dumb-bell about an axis 10 p ö
æ8 2
(d) ç - perpendicular
÷ MR to the plane passing through the è3 16 ø
centre of mass is
m1m2 r2
(a) (b) ( m1 + m2 ) r2 More Than One Correct Option
m1 + m2
2
m1mr r 33. Consider a bicycle wheel C
(c) (d) ( m1 - m2 ) r2
m1 - m2 rolling without slipping on D ω
a rough level road at a
29. A circular platform is free to rotate in a horizontal linear speed as shown in B vc
O
plane about a vertical axis passing through its figure. Then
centre. A tortoise is sitting at the edge of the (a) the speed of the particle A
platform. Now the platform is given an angular is zero
velocity w0 . When the tortoise moves along a chord of (b) the speed of B, C and D are all equal to v 0
the platform with a constant velocity (w.r.t. the (c) the speed of C is 2 v 0
platform), the angular velocity of the platform will (d) the speed of B is greater than the speed of O
vary with the time t as
Telegram @unacademyplusdiscounts

Rotational Motion 333

34. Choose the correct alternatives [NCERT Exemplar] (a) Torque t caused by F about z-axis is along - k
(a) For a general rotation motion, angular momentum L and (b) Torque t¢ caused by F abut z¢ axis is along - k$
angular velocity w need not be parallel (c) Torque t caused by F about z axis is greater in magnitude
(b) For a rotational motion about a fixed axis, angular than that about z axis
momentum L and angular velocity w are always parallel (d) Total torque is given be t = t + t ¢
(c) For a general translational motion, momentum p and 38. With referene to figure of a cube of edge a and mass
velocity v are always parallel.
m, state whether the following are true or false. (O is
(d) For a general translational motion, acceleration a and
the centre of the cube). [NCERT Exemplar]
velocity v are always parallel
z ¢¢
35. The net external torque on a system of particles H
about an axis is zero. Which of the following are G
compatible with it? [NCERT Exemplar] z'
(a) The forces may be acting radially from a point on the axis D C
O
(b) The forces may be acting on the axis of rotation
(c) The forces may be acting parallel to the axis of rotation E
F
(d) The torque caused by some forces may be equal and Y
opposite to that caused by other forces
36. The figure shows a system consisting of (i) ring of A a B
x
outer surface 3 R rolling clockwise without slipping
on a horizontal surface with angular speed w and (ii) (a) The moment of inertia of cube about z-axis is Iz = Ix + I y
an inner disc OP radius 2 R rotating anticlockwise ma2
(b) The moment of inertia of cube about z ¢ is Iz¢ = Iz +
with angular speed w/ 2. The ring and the disc 2
separated by frictionless ball bearing the system is in mz 2
the xz-plane. The point P on the inner disc is at a (c) The moment of inertia of cube about z¢¢ is = Iz +
2
distance R from the origin where OP makes an angle (d) Ix = I y
of 30° with the horizontal. Then with respect to the
horizontal surface.
z Comprehension Based Questions
ωD Passage I
The three equations of rotational motion are
ω
w = w0 + at; q = w0 t + 1 at2 and w2 - w20 = 2 aq , where
3p Rρ
30° x 2
2R the symbols have their usual meanings. Also,
2p
v = r w; w = = 2 pn are the known standard
T
relations. Use them to answer the following questions

(a) The point O has a linear velocity 3 Rw $i 39. The angular velocity of minutes hand of a watch is
p p
11 $ 3 (a) rad s–1 (b) rad s–1
(b) The point P has a linear velocity Rw i + Rwk$ 30 60
4 4 p p
13 3 (c) rad s–1 (d) rad s–1
(c) The point P has a linear velocity Rw i$ - Rw k$ 1800 3600
4 4
(d) The point P has a linear velocity 40. The linear velocity of tip of hours hand of a clock,
æ 3ö 1 which is 5 cm long is
ç3 - ÷ Rw i$ + Rw0 k$ p p
è 4 ø 4 (a) ms -1 (b) ms -1
120 ´ 60 120 ´ 60 ´ 60
37. Figure shows a lamina in xy- plane. Two axes z and z¢ (c)
p
ms -1 (d)
p
ms -1
pass perpendicular to its plane. z z 120 60 ´ 60 ´ 60
A force F acts in the plane of F
41. The spin driver of a washing machine revolving at
lamina at point P as shown.
15 rps slow down to 5 rps, while making
Which of the following are P
50 revolutions. Angular acceleration of the driver is
true? (The point P is closer to
(a) - 4 p rads -2 (b) - 4 p rads -2
z¢-axis than the z-axis.)
[NCERT Exemplar] (c) 8p rads -2 (d) - 8p rads -2
Telegram @unacademyplusdiscounts

334 JEE Main Physics

42. A solid cylinder of mass 20 kg rotates about its axis (b) If both Assertion and Reason are true but Reason is
with angular speed 100 rad/s. The radius of the not correct explanation of the Assertion
cylinder is 0.25 m. What is the kinetic energy (c) If Assertion is true but Reason is false
associated with the rotation of the cylinder? What is (d) If Assertion is false but the Reason is true
the magnitude of angular momentum of the cylinder 44. Assertion The centre of mass of a body will change
about its axis ? with the change in shape and size of the body.
(a) 3125 J, 62.5 J-s i=n
(b) 72.5 J s and 62.5 J-s å mi i i
i =1
(c) 3125 J, 82.5 J-s Reason r = i=n
(d) None of the above å mi
i =1
43. Torque of equal magnitude are applied to a hollow
cylinder and a solid sphere, both having the same 45. Assertion The velocity of a body at the bottom of an
mass and radius. The cylinder is free to rotate about inclined plane of given height is more when it slides
its standard axis of symmetry and the sphere is free down the plane compared to when it rolls down the
to rotate about an axis passing through its centre. same plane.
Which of the two will aquire a greater angular speed Reason In rolling down, a body acquires both, kinetic
after a given time ? energy of translation and kinetic energy of rotation.
(a) solid sphere 46. Assertion A ladder is more opt to slip when you are
(b) hollow sphere high on it than when you just begin to climb.
(c) both have some angular speed Reason At the high up on ladder the torque is
(d) cannot be said large and on climbing up the torque is small.
47. Assertion When ice on polar caps of earth melts,
Assertion and Reason duration of the day increases.
2p
Directions Question No. 44 to 48 are Assertion-Reason type. Reason L = Iw = I × = constant.
T
Each of these contains two Statements: Statement I (Assertion),
Statement II (Reason). Each of these questions also has four 48. Assertion Moment of inertia of circular ring about a
alternative choice, only one of which is correct. You have to given axis is more than moment of inertia of the
select the correct choices from the codes (a), (b), (c) and (d) given circular disc of same mass and same size, about the
below same axis.
(a) If both Assertion and Reason are true and the Reason
Reason The circular ring hollow so its moment of
is correct explanation of the Assertion
inertia is more than circular disc which is solid.

Previous Years’ Questions


49. A pulley of radius 2m is rotated about its axis by a end. During the journey of the insect, the angular
force F = (20 t - 5 t2 ) newton (where t is measured in speed of the disc [AIEEE 2011]
sec) applied tangentially. If the moment of inertia of (a) continuously of the disc
the pulley about its axis of rotation is 10 kg-m2 , the (b) continuously decreases
number of rotation made by the pulley before its (c) first increases and then decreases
direction of motion of reversed is [AIEEE 2011] (d) remains unchanged
(a) less than 3 51. A thin uniform rod of length l and mass m is swinging
(b) more than 3 but less than 6 freely about a horizontal axis passing through its
(c) more than 6 but less than 9 end. Its maximum angular speed is w. Its centre of
(d) more than 9 mass rises to a maximum height of [AIEEE 2009]
50. A thin horizontal circular disc is rotating about a lw l 2 w2
vertical axis passing through its centre. An insect is (a) (b)
6g 2g
at rest it a point near the rim of disc. The insect now
l 2 w2 l 2 w2
moves along a diameter of the disc to reach its other (c) (d)
6g 3g
Telegram @unacademyplusdiscounts

Rotational Motion 335

52. Consider a uniform square plate of side a and mass 59. What is moment of inertia in terms of angular
m. The moment of inertia of this plate about an axis momentum (L) and kinetic energy (K)? [UP SEE 2006]
perpendicular to its plane and passing through one of L 2
L 2

its corners is [AIEEE 2008]


(a) (b)
K 2K
5 ma 2 7 2 L L
(a) ma 2 (b) (c) ma 2 (d) ma 2 (c) (d)
6 12 12 3 2 K2 2K
53. A motor is rotating at a constant angular velocity of 60. A disc of mass 2 kg and radius 0.2 m is rotating with
500 rpm. The angular displacement per second is angular velocity 30 rads–1. What is angular velocity,
[BVP Engg. 2007]
if a mass of 0.25 kg is put on periphery of the disc?
3 3p
(a) rad (b) rad [UP SEE 2006]
50 p 50
(a) 24 rads–1 (b) 36 rads–1
25 p 50 p
(c) rad (d) rad (c) 15 rads–1 (d) 26 rads–1
3 3
61. The moment of inertia of a rod about an axis through
54. A wheel has angular acceleration of 3.0 rads–2 and an 1
initial angular speed of 2.00 rads–2.
In a time of 2 s it its centre and perpendicular to it is ML2 (where,
12
has rotated through an angle (in radian) of M is the mass and L the length of the rod). The rod is
[UP SEE 2007] bent in the middle to that the two halves make an
(a) 6 (b) 10 (c) 12 (d) 4 angle of 60°. The moment of inertia of the bent rod
about the same axis would be [UP SEE 2006]
55. For the given uniform square lamina ABCD, whose
centre is O as shown in figure. 1 1 2 1 ML2
[AIEEE 2007] (a) ML2 (b) ML (c) ML2 (d)
48 12 24 8 3
D F C 62. If the earth is treated as a sphere of radius R and
mass M, its angular momentum about the axis of
rotation with time period T is [BVP Engg. 2006]
O pMR3 MR2T 2 pMR2 4 pMR2
(a) (b) (c) (d)
T 2p T 5T
A B 63. A force of –F k acts on O, the origin of the coordinate
E
system as shown in figure. The torque about the
(a) IAC = 2 IEF (b) 2 IAC = IEF point (1, –1) is [AIEEE 2006]
Z
(c) IAD = 3 IEF (d) IAC = IEF (a) - F( $i + $j)
56. Angular momentum of the particle rotating with a (b) F( i$ + $j)
central force is constant due to [AIEEE 2007] (c) - F( $i - $j) O X
(a) constant torque (d) F( $i - $j) (1, –1)
Y
(b) constant force
(c) constant linear momentum 64. A hoop of radius r and mass m rotating with an
(d) zero torque angular velocity w0 is placed on a rough horizontal
surface. The initial velocity of the centre of the hoop
57. A circular disc of radius R is removed from a bigger is zero. What will be the velocity of the centre of the
circular disc of radius 2R such that the circumference hoop when it ceases to slip? [JEE Main 2013]
of new disc is a R from the centre of the bigger disc. rw0 rw0 rw0
The value of a is [AIEEE 2007] (a) (b) (c) (d) rw0
4 3 2
(a) 1/4 (b) 1/3
65. A circular disc of radius R rolls without slipping
(c) 1/2 (d) 1/6
along the horizontal surface with constant velocity
58. Four point masses, each of value m, are placed at the v0 . We consider a point A on the surface of the disc.
corners of a square ABCD of side l. The moment of Then the acceleration of the point A is [UP SEE 2005]
inertia of the system about an axis passing through A (a) constant in magnitude as well as in direction
and parallel to BD is [AIEEE 2006] (b) constant in direction
(a) 3 ml 2 (b) 3 ml 2 (c) constant in magnitude
(c) ml 2 (d) 2 ml 2 (d) constant
Telegram @unacademyplusdiscounts

336 JEE Main Physics

66. A solid cylinder of mass 20 kg has length 1 m and 40


(a) 4 MR2 (b) MR2
radius 0.2 m. Then its moment of inertia in kg m 2 9
about its geometrical axis is 37
[Kerala CET 2005] (c) 10 MR2 (d) MR2
(a) 0.8 (b) 0.4 (c) 0.2 (d) 20.4 9

67. A ring of radius 0.5 m and mass 10 kg is rotating 70. A T shape object with dimensions l
about its diameter with angular velocity of 20 rads–1. shown in figure is lying on a smooth A O
B

Its kinetic energy is [BVP Engg. 2005] floor. A force F is applied at the point P
parallel to AB, such that the object has
(a) 10 J (b) 100 J (c) 500 J (d) 250 J
only the translational motion without F P
D 2l
68. Two discs of the same material and thickness have rotation. Find the location of P with
radii 0.2 m and 0.6 m. Their moments of inertia about respect to C. [AIEEE 2005]
the axes will be in the ratio of [BVP Engg. 2005]
(a) 1 : 81 (b) 1 : 37 (c) 1 : 9 (d) 1 : 3 4
(a) l (b) l C
69. From a circular disc of radius R and mass 9 M, a 3
3 2
small disc or radius R/3 is removed. The moment of (c) l (d) l
inertia of the remaining disc about an axis 2 3
perpendicular to the plane of the disc and passing 71. An angular ring with inner and outer radii R1 and R2
through O is [IIT JEE 2005] is rolling without slipping with a uniform angular
speed. The ratio of the forces experienced by the two
particles situated on the inner and outer parts of the
R/3 ring. F1/ F2 is [AIEEE 2005]
R1
2R/3 (a) (b) 1
R2
2
R
æR ö R2
(c) çç 1 ÷÷ (d)
è R2 ø R1

Answers
Round I
1. (c) 2. (d) 3. (c) 4. (d) 5. (c) 6. (c) 7. (c) 8. (b) 9. (d) 10. (d)
11. (d) 12. (d) 13. (d) 14. (b) 15. (b) 16. (b) 17. (b) 18. (b) 19. (a) 20. (c)
21. (d) 22. (a) 23. (a) 24. (c) 25. (d) 26. (d) 27. (c) 28. (b) 29. (b) 30. (d)
35. (d) 36. (b)
31. (d) 37. (c)
32. (b) 38. (d)
33. 39. (b) 40. (b)
41. (b) 42. (c) 43. (d) 44. (b) 45. (a) 46. (d) 47. (b) 48. (b) 49. (b) 50. (d)
51. (d) 52. (b) 53. (c) 54. (c) 55. (d) 56. (b) 57. (b) 58. (c) 59. (b) 60. (a)
61. (c) 62. (c) 63. (e) 64. (b)

Round II
1. (d) 2. (c) 3. (d) 4. (d) 5. (c) 6. (c) 7. (b) 8. (a) 9. (c) 10. (d)
11. (a) 12. (a) 13. (b) 14. (b) 15. (b) 16. (b) 17. (a) 18. (a) 19. (b) 20. (c)
21. (c) 22. (c) 23. (a) 24. (b) 25. (a) 26. (a) 27. (a) 28. (a) 29. (c) 30. (a)
31. (d) 32. (d) 33. (c) 34. (a,c) 35. (a,b,c,d) 36. (a,b) 37. (b,c) 38. (b,d) 39. (c) 40. (b)
41. (b) 42. (a) 43. (d) 44. (a) 45. (d) 46. (a) 47. (a) 48. (b) 49. (b) 50. (c)
51. (c) 52. (d) 53. (d) 54. (b) 55. (d) 56. (c) 57. (b) 58. (b) 59. (b) 60. (a)
61. (b) 62. (d) 63. (b) 64. (c) 65. (a) 66. (b) 67. (d) 68. (a) 69. (a) 70. (b)
71. (a)
Telegram @unacademyplusdiscounts

the Guidance
Round I
1. Moment of inertia of uniform circular disc about diameter = I 2
7. Given, I = MR 2
According to theorem of perpendicular axes, 5
Using the theorem of parallel axes, moment of inertia of the
Moment of inertia of disc about its axis = 2 I sphere about a parallel axis tangential to the sphere is
æ 1 2ö 2 7
ç where, I = mr ÷ I ¢ = I + MR 2 = MR 2 + MR 2 = MR 2
è 2 ø 5 5
Applying theorem of parallel axes 7 æ 7 ö
\ I' = MK 2 = MR 2, K = ç ÷R
Moment of inertia of disc about the given axis 5 è 5ø
= 2 I + mr 2 = 2 I + 4 I = 6 I (Here, K is radius of gyrations)
2. About EG, the minimum distance from the axis is the least i. e. , 8. Linear momentum of particle before colliding = mv = mv ey
distribution of mass is minimum.
Linear momentum of particle after it bounces
3. Applying the principle of conservation of angular = - mv = - mvey
momentum, Change in linear momentum,
(I1 + I2) w = I1w1 + I1w2 Dp = - mv - (mv) = - 2 mv = - 2mv ey
400 600
(6 + I2) ´ 2p = 6 ´ ´ 2p + I2 ´ 0 Change in angular momentum = DL = r ´ Dp, where
60 60 r = (yey + aez)
which gives, I2 = 3 kg-m 2 = (yey + aez) ´ ( - 2mvey ) = 2mv aex
4. Apply parallel axis theorem, 9. Moment of inertia of the system about rod xshown the figure
I = ICM + Mh 2, we get x z
2
ML2 æLö 7 ML2
Þ = +Mç ÷ =
12 è 4ø 48
y
5. Here, m1 = m2 = 0.1kg
r1 = r2 = 10 cm = 0.1m
1 3 æ Ml 2 Ml 2 ö
and r 2 + m2r22 = m11
I = I1 + I2 = m11 r2 (Q m1 = m2) 4
2 2 I = Ix + Iy + Iz = 0 + ç + ÷ + Ml 2 = Ml 2
è 12 4 ø 3
3
= ´ 0.1(0.1) 2 = 1.5 ´ 10 –3 kg-m2
2 10. As, m1 = m2
6. Moment of inertia of the system about axis AX Þ pR12xd1 = pR22xd 2
X R12 d 2
rC C =
R22 d1
1
l l mR12
I1 2 R2 d
Now, = = 12 = 2
I2 1
60° mR22 R2 d1
A Y 2
l
rB
11. When a disc rotates with uniform angular velocity, angular
= IA + IB + IC acceleration of the disc is zero. Choice (d) is not true.
= (MA (rA ) 2 + MB(rB) 2 + MC (rC ) 2 12. A raw egg behaves like a spherical shell and a half boiled egg
= M (0) 2 + m ( l) 2 + m ( l cos 60° ) 2 behaves like a solid sphere
ml 2
5 ml 2
1ö Ir 2 / 3 mr 2 5
æ \ = = >1
= ml 2 + = çQ cos 60° = ÷ Ib 2 / 5 mr 2 3
4 4 è 2ø
Telegram @unacademyplusdiscounts

338 JEE Main Physics

æ L2 r2ö æ L2 D 2 ö Total moment of inertia,


13. Required moment of inertia, I = M ç + ÷ =Mç + ÷
è12 4ø è12 16 ø æ2 ö æ2 ö
I = ç Ma2÷ ´ 2 + ç Ma2 + Mb2÷ ´ 2
è5 ø è5 ø
ml 2
14. Moment of inertia of a uniform rod about one end = 8
3 = Ma2 + 2 Mb 2
5
\ Moment of inertia of the system, which rod is bent,
æ M ö (L / 2)
2
ML2
18. According to the theorem of perpendicular axes, Iz = Ix + Iy .
=2´ç ÷ =
è2ø 3 12 With the hole, Ix and Iy both decrease gluing the removed
piece at the centre of square plate does not affect Iz. Hence, Iz
15. Given, mass of bullet (m) = 10 g = 0.01kg decreases, overall.
Speed of bullet (v) = 500 m/s ML2 L
Width of the door ( l) = 1. 0 m 19. As, I = MK 2 = \ K=
12 12
Mass of the door (M) = 12 kg
20. As no torque is applied, angular momentum
As bullet gets embedded exactly at the centre of the door,
æ2 ö æ 2p ö
therefore its distance from the hinged end of the door L = Iw = constant = ç MR 2÷ ç ÷ = constant i. e. ,
è5 øèT ø
l 1
(r) = = m
2 2 R2
Þ = constant
Angular momentum transferred by the bullet to the door, T
(L) = mv ´ r R12 R22
Þ =
1 T1 T2
= 0.01 ´ 500 ´ 2
2 R22 æ xR ö
\ T2 = T1 = ç 1 ÷ ´ 24 h = 24x2h
= 2.5 J-s T2 è R1 ø
Moment of inertia of the door about the vertical axis at one 2
of its end, 1 2 1 æ 1ö
21. As, I= mr = ´ 16 ç ÷ = 2 kg-m2
2 2 è2ø
Ml 2 12 ´ (1) 2
(I) = = = 4 kg-m 2 2p (n2 - n1) 2 p (2 - 0) p
3 3 a= = = rads–2
t 8 2
But angular momentum, (L) = Iw
p
\ 2.5 = 4 ´ w Now, t = Ia = 2 × = p N-m
2
2.5
or w=
4 22. As toruqe = force ´ perpendicular distance
w = 0.625 rad/s \ t A = 100 ´ 0.75 = 75 Nm counter clockwise,
tB = 100 ´ 1.25 = 125 Nm clockwise
16. Required moment of inertia,
r 2 + m2r22 = 2 (0.3) 2 + 1(0.3) 2 = 0.27 kg-m2
I = m11 23. w1 = 2p rad/day, w2 = 0 and t = 1day
w2 - w1 0 - 2 p
17. We calculate moment of inertia of the system about AD, \ a= =
t 1
a b rad 2p rad
2p A= = B
day 2 (86400) 2 s2
Torque required to stop the earth, t = I a = FR
b 2
MR 2 ´ a
Ia 5 2
F= = = MR ´ a
Da aC R R 5
2 2p
= ´ 6 ´ 10 ´ 6400 ´ 103 ´
24
5 (86400) 2
Moment of inertia of each of the spheres A and D about
2 = 1.3 ´ 10 22 N
AD = Ma2
5 24. By conservation of energy
Moment of inertia of each of the sphere B and C about AD 1 2 1 1 1 w2
mgh = Iw + mv 2 = Iw2 + mr 2w2 = [1 + mr 2]
æ2 ö 2 2 2 2 2
= ç Ma2 + Mb 2÷
è5 ø é 2 mgh ù
1/ 2
w=ê
ë I + mr úû
2
Using theorem of parallel axis.
Telegram @unacademyplusdiscounts

Rotational Motion 339

25. Here, m = 3 kg, v = 2 ms–1 4.6


6.9 ´ 10 2 = 3 ´ 10 2 ´
y t
3 ´ 10 2 ´ 4.6
t= = 2s
6.9 ´ 10 2
30. Here, r = 0.2 m,M = 10 kg,
O x
n = 1200 rpm = 20 rps
4m
L = Iw = (Mr 2) (2 pn)
r = 4 m,L = ?,T = ?
22
L = mvr sin 90° = 3 ´ 2 ´ 4 ´ 1 = 24 kg m2s–1 = 10 ´ (0.2) 2 ´ 2 ´ ´ 20 = 50.28 kg-m2s–1
7
dL
and t= =0 31. Mass of oxygen molecule (M) = 5.30 ´ 10 -26 kg
dt
Moment of inertia I = 1.94 ´ 10 -46 kg-m 2
26. As the block remains stationary therefore for translatory
equilibrium Mean speed of the molecule (v) = 500 m/s
S fx = 0 \ f = N 2
Given, KE of rotation = ´ KE of translation
and S fy = 0 \ f = mg 3
For rotational equilibrium S t = 0 1 2 2 1
Iw = ´ Mv 2
2 3 2
By taking the torque of different force about point O
t f + t f + t N + t mg = 0 2Mv 2
or w=
As f and mg passing through point O I
\ tf + tN = 0 2 ´ 5.30 ´ 10 -26 ´ (500) 2
=
As t f ¹ 0 1.94 ´ 10 -46
\t N ¹ 0 and torque by friction and normal reaction will be = 1.35 ´ 10 10 ´ 500
in opposite direction.
= 6.75 ´ 10 12 rad/s
27. As, t = r ´ F = (3$i + 2$j + 3 k)
$ ´ (2$i - 3$j + 4 k)
$
32. Work done in stopping = change in KE = final KE - initial KE
= - 9 k$ - 12 $j - 4 k$ + 8$i + 6$j + 9$i i. e. ,t q = K = constant. As t is same in the two cases, q must be
= 17 $i - 6 $j - 13 k$ same i. e. , number of revolution must be same.
(w - w )
28. As, I = 1.2 kg-m2, E r = 1500 J 33. As, t = F ´ r = Ia = I 2 1
t
a = 25 rad s–2 w1 = 0 , t = ? F ´ r ´ t 10 ´ 0.2 ´ 4
\ w2 - w1 = = = 20 rad s–1
1 2 I 0.4
Now, Er = Iw
2 v 2 sin 2 q
2 Er
34. Maximum height attained (h) =
Þ w= 2g
I
v 2 sin 2 45° v2
2 ´ 1500 = =
= = 50 rad s–1 2g 4g
1.2
At highest point, momentum = mv cos 45°
From, w2 = w1 + a t mv
50 = 0 + 25 t =
2
Þ t =2s mv v 2 mv3
2 2 \Angular momentum = ´ =
29. Here, moment of inertia, I = 3 ´10 kgm 2 4g 4 2 g
Torque, t = 6.9 ´ 10 2 Nm
35. Given, R = 2 m
Initial angular speed, w0 = 4.6 rad s–1
M = 100 kg
Final angular speed, w0 = 0 rad s–1
Speed of centre of mass (v) = 20 cm/s = 0.20 m/s
As w0 = w0 + at
w - w0 0 - 4.6 4.6 Work done to stop the hoop
\ a= = =- rad s–2
t t t = Total kinetic energy of the loop
Now, negative sign is for deceleration 1 1
W = Mv 2 + Iw2
torque, t =Ia 2 2
Telegram @unacademyplusdiscounts

340 JEE Main Physics

v
But moment of inertia I = Mr 2 and angular velocity w = 43. As, the rod is highed at one end, its moment of inertia about
R ML2
2ö this end is I =
1 1 æv 3
\ W= Mv 2 + (Mr 2) ´ ç 2 ÷
2 2 èR ø Total energy in upright position
1 1 = total energy on striking the floor
\ = Mv 2 + Mv 2 = Mv 2
2 2 MgL 1 2 1 ML2 2
0+ = Iw + 0 = w
= 100 ´ (0.20) 2 2 2 2 3
= (100 ´ 0.04) J = 4.0 J Lw2 3g
Þ g = or w =
36. As, Force ´ distance = t = Ia 3 L
é 2
1 æ 1ö ù 44. Here, n1 = 2.8 rps,n2 = ?
Þ F sin 30° ´ = 4 ê2 ´ ç ÷ ú a
2 êë è 4ø ú l2 = 0.7 I1
û
1 1 a w2 I1 1
or 24 ´ ´ = As, = =
2 2 2 w1 I2 0.7
\ a = 12 rad s–2 n2 10
\ = (Q w = 2pn)
n1 7
75 5
37. Here, n1 = rps = rps 10 10
60 4 n2 = n1 = ´ 2.8 = 4.0 rps
7 7
For n2 = 0 ,t = 5 s, a = ?
w - w1 2 p (n2 - n1) 2p (0 - 5 / 4) 45. When the person jumps off the round, radially away from the
a= 2 = =
t t 5 centre, no torque is exerted i. e. , t = 0. According to the
p –2
principle of conservation of angular momentum,
= - = -1.57 rad s I ´ w = constant. As mass reduces to half (from 2M to M),
2
moment of inertia I becomen half. Therefore, w must become
38. Using, w22 - w12 = 2 aq, we get twices ( = 2 w).
( w / 2) 2 - w2 = 2 a (36 ´ 2p ) …(i) 1 1
46. Rotational kinetic energy, E = Iw2 = L ´ 2 pn
2 2 2 2
Similarly, 0 - ( w / 2) = 2a (n ´ 2p ) …(ii)
L2 E 2 n1
Dividing Eq. (i) by Eq. (ii), we get \ E µL ´n Þ = ´
L1 E1 n2
3
- w2 L2 é E1 /2 ù é n1 ù
4 36 = ´
=
w 2
n L1 êë E1 úû êë 2 n1 úû
-
4 I
I2 = 1
\ n = 12 4
L
39. In rotation per second (rps) or L2 = (as I1 = I)
4
w2 - w1 240 - 0
a= = = 2 rps2 1
t 2 ´ 60 Þ L2 =
4
40. Angular momentum
47. Angular velocity of man relative to platform is
= linear momentum ´ perpendicular distance of line of v0
action of linear momentum from the axis of rotation wr = w +
r
= mv ´ l 1
= 0.7+ = 1.2 rad s–1
41. As, w = w0 + at = 0 + 2 ´ 6 = 12 rads–1 2
Also v = rw Time taken by the man to complete one revolution,
2p 2p
\ v = 2 ´ 12 = 24 ms–1 T= = s
wr 1.2
42. According to conservation of angular momentum Angular displacement of the man w.r.t. ground,
I1w1 = I2w2 vT
q = w0T = 0
1 æ1 1 ì Mü ö r
Þ MR 2w = ç MR 2 + í ý R 2÷ w2
2 è2 2 î 4þ ø 1 æ2 p ö 5
= ç ÷ = p rad
4 2 è 1.2 ø 6
\ w2 = w
5
Telegram @unacademyplusdiscounts

Rotational Motion 341

48. As L = Iw = constant 2
1+
Þ 5 = 1+ 1
2 æ2 p ö
\ MR 2 ´ ç ÷ = constant sin 30° sin q2
5 è 7 ø
5
R2 or sin q2 = = 0.7143
i. e. , = constant 7
T
or q = 45°
where R is halved, R 2 becomes 1/4th. Therefore, T becomes
1/4th i. e. , 6 h. 52. Here, l = 1m, q = 30°, g = 9.81ms–2,t = ?
1 é (I w + I w ) 2 ù 2l (l + K 2/ R 2)
49. Here, Kf = (I1 + I2) ê 1 1 2 22 ú Q t=
2 ë (I1 + I2) û g sin q
1 (I1w1 + I2 w2) 1 2
= For a rupee coin, K 2 = R
2 I1 + I2 2
1 2 ´ 1(1 + 1 / 2) 6
and Ki = (I1w12 + I2 w22) Þ t= = = 0.78 s
2 9.81 sin 30° 9.81
Þ D K = Kf - Ki 53. Time taken in reaching bottom of incline is
I I
= - 1 2 ( w1 - w2) 2 2l (1 + K 2/ R 2)
2 (I1 + I2) t=
g sin q
50. Here, mass of man, m = 80 kg K2
Greater the value of , greater will be the time
Mass of platform, M = 200 kg R2
Let R be the radius of platform. For solid cylinder (SC), K 2 = R 2/ 2
When man is standing on the rim, For hollow cylinder (HC), K 2 = R 2
I1 = M (R / 2) 2 + mR 2 2
2
For solid sphere (S), K 2 = R2
æRö 5
= ç ÷ (M + 4 m)
è2ø 54. Here, q = 60°, l = 10 m, a = ?
When man reaches the centre of platform, 2 2
For solid sphere, K 2 = R
I2 = M (R / 2) 2 + m ´ 0 = m (R / 2) 2 5
As angular momentum is conserved, æ ö
ç ÷
I1 w2 2 pn2 n2 9.8 sin 60° çQ a = g sin q ÷
= = = \ a=
2 ç 2
K ÷
I2 w1 2 pn1 n1 1+ ç 1+ 2 ÷
5 è R ø
I1
n2 = ´ n2 5 3 -2
I2 or a = ´ 9.8 ´ = 6.06 ms
7 2
(M + 4 m) (R / 2) 2
= ´ 12 55. As, it is clear from figure,
M (R / 2) 2
(200 + 4 ´ 80)
= ´ 12 R
200
520 ´ 12
= On reaching the bottom of the bowl, loss in PE = mgR,
200
and gain in
n1 = 31.2 rpm 1 1
KE = mv 2 + Iw2
2 2
2l (1 + K 2/ R 2) Þ
1 1 æ2 ö
| DK| = mv 2 + ´ ç mr 2÷ w2
51. For rolling, t = = same (given in question)
g sin q 2 2 è5 ø

2l (1 + K12/ R 2) 2l (1 + K22/ R 2) 1 1 7
\ = = mv 2 + mv 2 = mv 2
g sin q1 g sin q2 2 5 10
2 2 As, again in KE = loss in PE
For sphere, K12 = R , q1 = 30° 7
5 \ mv 2 = mgR
10
For hollow cylinder, K22 = R 2, q2 = ?
10 gR
v=
7
Telegram @unacademyplusdiscounts

342 JEE Main Physics

56. If h is height of the ramp, then in rolling of marble, speed 61. The rolling sphere has rotational as well as translational
2 gh kinetic energy.
v= 1 1
1 + K 2/ R 2 \Kinetic energy = mu 2 + Iw2
2 2
The speed of the cube to the centre of mass
1 2 1 æ 2 2ö 2
v ¢ = 2 gh = mu + ç mr ÷ w
2 2 è5 ø
v¢ K2 1 mv 2 7
\ = 1+ 2 mu 2 +
= = mu 2
v R 2 5 10
2 2 2 Loss in potential energy = Gain in kinetic energy
For marble sphere, K = R
5 7
\ mgh = mu 2
v¢ 2 7 10
\ = 1+ = = 7: 5
v 5 5 7 u2
Þ h=
57. Here, r = 0.5,m = 2 kg 10 g

1 2 1 æ 1 2ö 2 1
Rotational KE = Iw = ´ ç mr ÷ w 62. For solid cylinder, q = 30° ,K 2 = R 2
2 2 è2 ø 2
1 1 For hollow cylinder, q = ?,K 2 = R 2
Þ 4 = mv 2 = ´ 2 v 2
4 4 Hence,
\ v = 8 = 2 2 ms –1 æ 1ö
ç1 + ÷
è 2 ø 1+ 1
58. When a body of mass m slides down an inclined plane, then =
sin 30° sin q
v = 2 gh
2
\ sin q = = 0.6667
When it is in the form of ring, then 3
2 gh 2 gh 2 gh v q = 42°
v ring = = = =
æ K2 ö 1+ 1 2 2 2 gh
ç1 + 2 ÷ 63. As, v =
è R ø K2
1+ 2
1 R
59. Rotational kinetic energy KR = Iw2
2 where K is the radius of gyration.
2
1 MR 1 K2
KR = ´ ´ w2 = Mv 2 (Q v = Rw ) For ring =1
2 2 4 R2
1
Translational kinetic energy KT = Mv 2 \ v=
2 gh
= gh
2 1+ 1
1 1
Total kinetic energy = KT + KR = Mv 2 + Mv 2 2
2 4 64. Here, m = 8 kg, r = 40 cm = m,
5
3
= Mv 2 w = 12 rad s–1,I = 0.64 kg m2
4
1 1 2 1
Mv 2 Total KE = Iw + mv 2
Rotational kinetic energy 4 1 2 2
\ = =
Total kinetic energy 3 2 3
Mv 1 2 1 2 2
4 = Iw + mr w
2 2
g sin q g sin 30° 5 1 5g
60. As, a = = = g´ = 1 1 æ2ö
2
K 2 2 7 2 14 = ´ 0.64 ´ 15 2 + ´ 8 ´ ç ÷ ´ 15 2
1+ 2 1+ 2 2 è5ø
R 5
(R = Radius of sphere) = 216 J
Telegram @unacademyplusdiscounts

Rotational Motion 343

Round II
1. Moment of inertia of cylinder about an axis through the centre g sin q
6. Linear acceleration for rolling, a =
and perpendicular to its axis is K2
1+
æ R 2 L2 ö R2
Ic = M ç + ÷
è 4 12 ø f

Using theorem of parallel axes, moment of inertia of the


cylinder about an axis through its edge would be
2 θ
æLö æ R 2 L2 L2 ö æ R 2 L2 ö
I = Ic + M ç ÷ = M ç + + ÷ =M ç + ÷ K2 1
è2ø è 4 12 4 ø è 4 3ø For cylinder, 2 =
R 2
49 2
When L = 6 R , Ih = MR 2 \ acylinder = g sin q
4 3
2 2 For rotation, the torque
2. As, Is = MRs2,Ih = MRh2
5 3 fR = Ia × (MR 2a) / 2
As Is = Ih (where, f = force of friction)
2 2 M
\ MRs2 = MRh2 But Ra = a \ f = a
5 3 2
M 2 M
Rs 5 \ f = × g sin q = g sin q
\ = 2 3 3
Rh 3
m s = f / N, where N is normal reaction,
3. Here, pr = l \ r = l / p M
g sin q
tan q
1 2 \ ms = 3 =
Moment of inertia of a ring about its diameter = Mr Mg cos q 3
2
2 \ For rolling without slipping of a roller down the inclined
1 é æ l ö ù ml 2 plane, tan q £ 3 m s .
\ Moment of inertia of semicircle = êm ç ÷ ú =
2 êë è p ø úû 2 p 2
7. Initial angular momentum of ring L = Iw = MR 2w
4. As, w2 = w1 + at Final angular momentum of system (Ring + Two particles)
\ 40 p = 20 p + a ´ 10 = (MR 2 + 2 mR 2) w¢
or a = 2 p rad s–2 As there is no external torque on the system therefore
From, w22 - w12 = 2 aq MR 2w = (MR 2 + 2 mR 2) w¢
( 40 p ) 2 - (20 p ) 2 = 2 ´ 2 pq Mw
Þ w¢ =
2 (M + 2 m)
1200 p
Þ q= = 300 p
4p 8. Angular momentum of block w.r.t. a
q 300 a v
Number of rotations completed = = = 150 O before collision with O = Mv M
2p 2p 2
On collision, the block will rotate
5. From conservation of angular momentum, about the side passing through O.
I1w1 = I2w2 Now its angular momentum = Iw O
w1 I2 By law of conservation of angular
\ = momentum
w2 I1
a
1 2 Mv = Iw
Iw 2
E1 2 1 1
Now, = a æ Ma2 Ma2 ö 3v
E 2 1 I w2 Þ Mv = ç + ÷w Þ w=
2
2 2 2 è 6 2 ø 4a
2
I1 æ I2 ö I where I is moment of inertia of the block about the axis
= ´ç ÷ = 2 perpendicular to the plane passing through O.
I2 è I1 ø I1
As I1 > I2 9. Angular momentum about origin = Itranslation + Irotation
\ E1 < E 2 1 3
= MvR + Ic w = M (Rw) R + MR 2w = MR 2w
2 2
Telegram @unacademyplusdiscounts

344 JEE Main Physics

1 14. According to theorem of parallel axes, moment of inertia of a


10. From E = Iw2, we find that when frequency (n) is doubled,
2 rod about one of its ends
w = 2 pn is doubled, w2 becomes 4 times. As E reduces to half, Ml 2 Ml 2 Ml 2
1 = + =
I must have been reduced to th. From L = Iw, L becomes 12 4 3
8
1 1 \Moment of inertia of two rods about Z-axis
´ 2 = times i.e., 0.25 L.
8 4 = Moment of inertia of 2 rods placed along X and Y-axis
11. By the conservation of energy, 2 Ml 2
=
3
Loss in PE of rod = gain of rotational KE
1 1 15. As no torque is being applied, angular momentum
mg sin a = Iw2
2 2 L = Iw = constant
1 1 ml 2 2
æ 2 2ö 2 p
mg sin a = w l/2 Þ ç MR ÷ = constant
2 2 3 α è5 ø T
3 g sin a r2
Þ w= or = constant
l T
But in the problem length of the rod 2 L is given Differentiating w.r.t. time (t), we get
dr dT
3g sin a T ×2 r - r2
\ w= dt dt = 0
2L
T2
mL2 or 2 Tr
dr
= r2
dT
12. Moment of inertia of rod about the given axis = .
12 dt dt
Moment of inertia of each disc about disc about its diameter dT 2 T dr
or =
dt r dt
MR 2
=
4 16. As, L = rP
Using theorem of parallel axes, moment of inertia of each disc Þ log e L = log e P + log e r
MR 2
2 If graph is drawn between log e L and log e P then, it will be
æLö
about the given axis = +Mç ÷ straight line which will not pass through the origin because of
4 è2ø
presence of constant in the equation.
MR 2 ML2
= + 17. As the mass is concentrated at the centre of the rod, therefore,
4 4
l 1 2 1 æ ml 2 ö 2
\ For theorem of parallel axes, moment of inertia about the mg ´ = Iw = ç ÷w
2 2 2 è 3 ø
given axis is
mL2 æ MR 2 ML2 ö 3g
I= +ç + ÷ Þ w=
12 è 4 4 ø l

mL2 mR 2 ML2
I= + + l/2
12 4 4
13. According to the theorem of parallel axes, moment of inertia
of disc about an axis passing through K and perpendicular to
plane of disc,
Velocity of other end of the rod
R v = wl = 3 gl
K
O R
18. Let L1, L2 and r1, r2 are the angular momenta and position
vectors of the particles at that instant about any arbitrary point
1 3 O.
= MR 2 + MR 2 = MR 2
2 2 Angular momentum of the particles,
Total moment of inertia of the system L1 = r1 ´ mv and L 2 = r2 ´ mv
3 It resultant angular momentum of the system is L, then
= MR 2 + m (2R) 2 + m ( 2R) 2 + m ( 2R) 2
2 L = L1 + L 2 = r1 ´ mv + ( - r2 + mv)
2
R Negative sign shown that both particles are moving in
= 3 (M + 16 m) opposite directions.
2
Telegram @unacademyplusdiscounts

Rotational Motion 345

5
23. MI of disc about tangent in a plane = MR 2 = I
4
M θ1
2 4
θ1 P1 v \ MR = I
r1 5
r1sinθ1 d 3
MI of disce about tangent I to plane I ¢ = MR 2
θ2 2
N nθ
2 θ2
si
P2 –v 3 æ4 ö 6
r2 r2 \ I¢ = ç I÷ = I
2 è5 ø 5

O 24. Angular momentum about an axis z


ω
2
Lt = [I + m (vt ) ] w
\ | L | = |L1 | - |L 2 | v
d Lt O
= mvr1 sin q1 - mvr2 sin q2 = 2 Mv 2tw v
dt
= mv(r1 sin q1 - r2 sin q2) …(i)
Þ torque, t = (2 mv 2w) t x
x = vt
where q1 and q2 are the angles between r1, v and r2, v
respectively. When particles changes their position with time, 25. As shown in figure normal reaction, R = mg . Frictional force,
their direction of motion ( v) remains unchanged and therefore F = mR = m mg . To topple, clockwise moment must be more
distances OM = r1 sin q1 and ON = r2 sin q2 remains same. than the anticlockwise moment
But OM - ON = MN = d (Given) h a
i. e. , m mg ´ > mg ´
\ r1 sin q1 - r2 sin q2 = d …(ii) 2 2
From Eqs. (i) and (ii), we get or m > a/h
|L | = mvd R = mg
It is constant with time.
The direction of L is perpendicular to the plane of r and v and
is inward to the plane of paper, which also remains G
unchanged with time. Motion
a/2
2 h/2
ML
19. Moment of inertia of a rod about one end =
3 F = µmg
As I = I1 + I2 + I3
mg
ML2 ML2 2 ML2
\ I =0 + + = 26. y
3 3 3
R/2
1 2 1 2 R/2
20. KE of rotation = Iw = ´ MR (2 pn) 2
2
2 2 5 2R y’
1
= ´ 4 p n MR 2
2 2
Moment of inertia of the system about yy¢
5
2 Iyy¢ = Moment of inertia of sphere P about yy¢
æ 600 ö
= 0.8 p 2 ç 2 2
÷ MR = 80 p MR
2
è 60 + Moment of inertia of sphere Q about yy¢
t m mgR mg
21. Angular retardation, a = = =
I mR 2 R Moment of inertia of sphere P about yy¢
As w = w0 - at 2
3 æRö
w - w w0 - w0 / 2 = M ç ÷ + M ( x) 2
\ t= 0 = 5 è2ø
a mg / R 2
2 æRö
wR = M ç ÷ + M (2R) 2
= 0 5 è2ø
2 mg
MR 2
2 2 æ4 ö = + 4 MR 2
22. As, I = MR 2 = ç mR3r ÷ R 2 10
5 5 è3 ø 2
2 æRö
8 22 5 Moment of inertia of sphere Q about yy¢ is Mç ÷
= ´ Rr 5 è2ø
15 7
176 5 2
Þ I= Rr MR 2 2 æRö 21
Now, Iyy¢ = + 4 MR 2 + M ç ÷ = MR 2
105 10 5 è2ø 5
Telegram @unacademyplusdiscounts

346 JEE Main Physics

27. When one string is cut off, the rod will rotate about the other MR 2
Initial angular momentum, I1 = mR 2 +
point A. Let a be the linear acceleration of centre of mass of 2
the rod and a be the angular acceleration of the rod about A. At any time t, let the tortoise reach D moving with velocity v.
As it clear from figure.
\ AD = vt
AC = R 2 - a2
T
As DC = AC - AD = ( R 2 - a2 - vt )
A \ OD = r = a2 + [ R 2 - a2 - vt ]2
a
mg Angular momentum at time t
MR 2
I2 = mr 2 +
mg - T = ma …(i) 2
t mg ( l / 2) 3 g As angular momentum is conserved
a= = = …(ii)
I ml 2/ 3 4
\ I1w0 = I2 w (t )
l l 3g 3g This shows that variation of w(t ) with time is non-linear.
a = ra = a = =
2 2 2l 4
30. As angular momentum is conserved in the absence of a
3 mg mg
From Eq. (i), T = mg - ma = mg - = torque, therefore
4 4
I0 w0 = Iw
28. x1 + x2 = r …(i) é2 2 ù
æ2 2ö æ 2 p ö 2 2 MR 2p
ç MR ÷ ç ÷ = ê MR + ú
and m1x1 = m2x2 …(ii) è3 ø è T0 ø ë 5 5 5 ´ 10 19 û T
T 1
= 1+
m1 C m2 T0 5 ´ 10 19
x1 x2 T 1
-1 = = 2 ´ 10 -20
T0 5 ´ 10 19
B
From Eqs. (i) and (ii),
31. Moment of inertia of the cylinder about an axis perpendicular
to the axis of the cylinder and passing through the centre is
m2 r
x1 = æ R 2 L2 ö
m1 + m2 I =M ç + ÷ …(i)
è 4 12 ø
m1r
and x2 = If r is volume density of the cylinder, then
m1 + m2
M = ( pR 2L) r = constant …(ii)
m1 m2 r 2
\ IAB = m1x12 + m2x22 = M
m1 + m2 \ L=
pR 2r
29. As there is no external torque, angular momentum will Put in Eq. (i)
remain constant. When the tortoise moves from A to C, figure, æ R2 M2 ö
moment of inertia of the platform and tortoise decreases. I =M ç + 2 4 4
÷
12Rp r 4
Therefore, angular velocity of the system increases. When the
tortoise moves fromè C to B, momentø of inertia increases. dI
For I to be minimum, =0
Therefore, angular velocity decreases. dR
dI æR M2 ö
=M ç - 2 5
÷ =0
dR è2 3 p R ø
R M2 2 M2
O = or R 6 =
R 2 3 p 2r 2R5 3 p 2r 2
r a
A B 2 p 2R 4L2r 2
D C Using Eq. (ii), R6 =
3 p 2r 2

If, M = mass of platform 2 2 L2 3


or R2 = L or =
R = radius of platform 3 R2 2
L
m = mass of tortoise moving along the chord AB or = 3 /2
R
a = perpendicular distance of O from AB.
Telegram @unacademyplusdiscounts

Rotational Motion 347

32. If M mass of the square plate before cutting the holes, then 38. Choice (a) is false, as theorem of perpendicular axes applies
mass of portion of each hole. only to a plane lamina.
M p Now, Z axis parallel to Z¢ axis and distance between them
m= 2
´ pR 2 = M
16 R 16 a 2 a
= = . Therefore, according to the theorem of parallel
\Moment of inertia of remaining portion 2 2
I = Isquare - 4 Ihole axes,
2
M é mR 2 ù æ a ö ma2
= (16 R 2 + 16 R 2) - 4 ê + m ( 2R) 2ú Iz ¢ = Iz + mç ÷ = Iz +
12 è 2ø 2
ë 2 û
M Choice (b) is true.
= ´ 32 R 2 - 10 mR 2
12 Again, choice (c) is false as Z¢ ¢ axis is not parallel to Z-axis.
8 10 p æ 8 10 p ö Choice (d) is true as from symmetry, we find that Ix = Iy .
= MR 2 - MR 2 = ç - ÷ MR
2
3 16 è 3 16 ø 39. For minutes hand, T = 1h = 60 ´ 60 s
33. From theory of rolling motion without slipping speed of 2p 2p
w= = rad s–1
particle at point of contact A is zero and at the top point C speed T 60 ´ 60
is 2 v 0 . Moreover, speed of point O is v 0 but that of B is v 0 2. p
= rad s–1
34. From the study of theory, we know that for general rotational 1800
motion, angular momentum L and angular velocity wneed not 40. For hour’s hand, T = 12 h = 12 ´ 60 ´ 60 s
be parallel.
æ2 p ö
Again, for a general translational motion, linear momentum p v = rw = r ç ÷
è T ø
and linear velocity v are always parallel. This is because p is
5 2p
directed along v only. = ´ ms–1
100 12 ´ 60 ´ 60
35. When net external torque on a system of particles about an
axis is zero, i. e. , t = r ´ F = r F sin q t = Zero, where q is angle p
= ms–1
between r and F, t is unit vector along t, then all the four 120 ´ 60 ´ 60
statements (a), (b), (c), (d) are compatible.
41. Here, n0 = 15 rps; n = 5 rps
36. As, v 0 = 3 wR$i q = 50 revolution = 50 ´ 2 rad
From, w2 - w20 = 2 aq
60° P w2 - w20 4 p 2 (n 2 - n02)
3 ωR a= =
30° 2q 2q
4 p 2 (5 2 - 15 2)
a= = - 4 p rad s–2
2 ´5 ´2 p
3 ωR v = 3 ωR
For pure rolling, 42. Given, M = 20 kg
v 0 = 3wR$i w = 100 rad/s
æR = 0.25wmR ö wR
v p = ç 3 wR - cos 60° ÷ $i + sin 60 $j Moment of inertia of the solid cylinder about its axis of
è 2 ø 2
symmetry.
11Rw $ 3 wR $ 1
= i+ j I = MR 2
4 4 2
37. According to right handed screw rule, the direction of torque 1
$ So choice (a) is false. = ´ 20 ´ (0.25) 2
t caused by F about Z-axis is along k. 2
However choice (b) is true as direction of torque ( t¢ ) caused = 10 ´ 0.0625
$ As t = r ´ F and P is closer to Z¢
by F about Z¢ axis is along - k. = 0.625 kg - m2
axis, therefore t caused by F about Z-axis is greater in Kinetic energy associated with the rotation of the cylinder is
magnitude than that about Z¢ axis. Choice (c) is true. Choice given by
(d) is false as it is meaningless to add torques about differemt 1
axes K = Iw2
2
Telegram @unacademyplusdiscounts

348 JEE Main Physics

1
= ´ 0.625 ´ (100) 2 47. Both, the assertion and reason are true and latter is a correct
2 explanation of the former. Infact, as ice on polar caps of earth
= 0.3125 ´ 10000 melts, mass near the polar axis spreads out, I increases.
= 3125 J Therefore, T increases i.e., duration of day increases.
Angular momentum, L = Iw 48. In the case of circular ring the mass is concentrated on the rim
= 0.625 ´ 100 (at maximum distance from the axis) therefore moment of
= 62.5 J-s inertia increase as compared to that in circular disc.

43. Let M and R be the mass of radius of the solid sphere and 49. To reverse the direction
hollow cylinder. ò t dq = 0 (work done is zero)
Moment of inertia of the hollow cylinder about its axis of As, t = (20 t - 5 t 2) 2 = 40 t - 10 t 2
symmetry,
I1 = MR 2 t 40 t - 10 t 2 dw
a= = = 4t -t2 =
I 10 dt
Moment of linear of the solid sphere about its diameter
t t3
2 w = ò a dt = 2 t 2 -
I2 = MR 2 0 3
5
w is zero at
Let torque t of triangle magnitude be applied on hollow
cylinder and solid sphere. the angular acceleration produced t3
2t2 - =0
in it are a1 and a 2 respectively. 3
\ t = I1a1 Þ t3 = 6 t 2
and t = I2a 2 Þ t =6 s
Therefore, I1a1 = I2a 2 dq
As =w
2 dt
MR 2
a1 I2 5 2 6 6 æ t3 ö
or = = = Þ q = ò w dt = ò ç2 t 2 - ÷ dt
a 2 I1 MR 2 5 0 0 è 3ø
5
or a2 = a1 é 2 t3 t 4 ù
6
2 é 2 1ù
=ê - ú = 216 ê - ú = 36 rad
= 2.5 a1 …(i) ë 3 12 û 0 ë3 2û
Let after time t , w1 and w2 be the angular speeds of the 36
Number of revolution is less than 6.
hollow cylinder and solid sphere respectively. 2p
\ w1 = w0 + a1t …(ii) 1
and w2 = w0 + a 2t 50. MI = MR 2 + mx2
2
= w0 + 2.5 a1t …(iii) where, m = mass of insect
From Eqs. (ii) and (iii), we get and x = distance of insect from centre
w2 > w1 Clearly as the insect moves along the diameter of the disc
Therefore, solid sphere will acquire a greater angular speed moment of inertia first decreases then increases.
after a given time.
By conservation of angular momentum, angular speed first
44. Position vector of centre of mass depends on masses of increases then decreases.
particles and their location. Therefore, change in shape/size
of body do change the centre of mass.
51. If centre of mass rises to a maximum height h,
then from loss in KE = gain in PE, we get
45. In sliding down, the entire potential energy is converted only
1 2
into linear kinetic energy. In rolling down, a part of same Iw = mgh
potential energy is converted into kinetic energy of rotation. 2
Therefore, velocity acquired is less. 1 æ ml 2 ö 2 l 2w2
or ç ÷ w = mgh Þ h =
46. When a person is high up on the ladder. Then a large torque is 2è 3 ø 6g
produced due to his weight about the point of contact
between the ladder and the floor whereas when he starts
52. Moment of inertia of the square plate about an axis passing
through the centre and perpendicular to its plane is
climbing up the torque is small, Due to this reason the ladder
is more opt to slip when one is high up on it. m ( a2 + a2) ma2
I= =
12 6
Telegram @unacademyplusdiscounts

Rotational Motion 349

53. When the axis passes through one of its corners, we use 57. In figure is centre of a circular disc of radius 2 R and mass M.
theorem of parallel axes.
2
æa 2ö ma2 ma2 2
\ I' = I + mç ÷ = + = ma2
è 2 ø 6 2 3 x
500 ´ 2p 50 p
\ q = wt = = rad C2 O C1
60 3
54. Angular accelerations is time derivative of angular speed and
angular speed is time derivative of angular displacement.
dw
By definition a = M = p (2R) 2r, where r is mass/area of disc.
dt
C1 is centre of disc of radius R, which is removed.
i. e. , dw = adt
Mass of removed disc,
So, if in time t the angular speed of a body changes from w0 to w
M
w t M1 = p (R) 2r =
òw 0
dw = ò adt
0
4
Mass of remaining disc,
If a is constant
M 3M
w - w0 = at ...(i) M2 = M - M1 = M - =
4 4
Now, as by definition
Let its centre of mass be at C 2, where OC 2 = x
dq
w= \ M1 ´ OC1 = M2 ´ OC 2
dt
M 3M
and ´R = x
4 4
dw
q= R 1
dt or x = = aR Þ a =
3 3
Eq. (i) becomes
dq 58. As is clear from figure
= w0 + at
dt AC = BD = l 2 + l 2 = l 2
i. e. , dq = ( w0 + at )dt Moment of inertia of four point masses about BD
2 2
So, if in time t angular displacenent is q æl 2ö æl 2ö
q t IBD = m ç ÷ + m ´0 + m ç ÷ + m ´0
ò0 dq = ò0 ( w0 + at) dt è 2 ø è 2 ø
1 2 ml 2 ml 2
q = w0t + at = + = ml 2
2 2 2
Y
Given, a = 3.0 rads -2, , w0 = 2.0 rads -1 ,t = 2s
1
Hence, q = 2 ´2 + ´ 3 ´ (2) 2 A B
2
or q = 4 + 6 = 10 rad
l
Eqs. (i) and (ii) are similar to first and second equations of X
O
linear motion.
55. From symmetry considerations, D l C
1
IAC = moment of inertia of lamina about an axis through O Applying the theorem of parallel axes,
2
and ^ ABCD. IXY = IBD + M ( AO) 2
2
1 æ l ö
and IEF = moment of inertia of lamina about an axis through = ml 2 + 4 m ç ÷ = 3 ml
2
2 è 2ø
O and ^ ABCD
59. Angular momentum of a rigid body about a fixed axis is given
\ IAC = IEF
by
56. Torque due to central force is zero L = Iw
d
As t = (L) = 0 where I is moment of inertia and w is angular velocity about
dt that axis.
\ L = constant
Telegram @unacademyplusdiscounts

350 JEE Main Physics

Kinetic energy of body is given by 63. Here, F = -Fk$


1
K = Iw2 r = ( $i - $j), t = ?
2
As t = r ´ F = ( $i - $J) ´ ( -F k$ )
1 L2
\ K= (Iw) 2 =
2I 2I = -F ( $i ´ k$ - $j ´ k)
$

L2 = - F ( - $j - $i)
Þ I=
2K = F( $i + $j)
60. Angular momentum in absence of any external torque R
64.
remains constant.
v
If no external torque acts on a system of particles, then angular
momentum of the system remains constant, i. e. , t = 0
dL
\ =0 From conservation of angular momentum
dt
v
Þ I1w1 = I2w2 …(i) mr 2w0 = mvr + mr 2 ´
r
Here, M = 2 kg, m = 0.25 kg, r = 0.2 m w0 r
Þ v=
w1 = 30 rad s–1 2
Hence, we get after putting the given values in Eq. (i) 65. The circular disc of radius R rolls without slipping. Its centre of
1 1 mass is C. P is point where body is in contact with the surface
´ 2 ´ (0.2) 2 ´ 30 = ´ (2 + 2 ´ 0.25) (0.2) 2 ´ w2
2 2 at any instant. At this instant, each particle of body is moving
Þ 1.2 = 0.05 w2 at right angles to the line which joins the particle with point P
\ w2 = 24 rad s–1 with velocity proportional to distance. In other words, the
combined translational and rotational motion gives pure
61. Since, rod is bent at the middle, so each part of it will have rolling and body moves with constant velocity in magnitudes
æLö æMö as well as direction.
same length ç ÷ and mass ç ÷ as shown
è2ø è2ø

R
M/2 C
L/2

60°
M/2 66. Here, m = 20 kg, l = 1m, r = 0.2 m
O Moment of inertia about its geometrical axis is
L/2
1
I = mr 2
Moment of inertia of each part about an axis passing through 2
its one end 1
1 æMö æ L ö
2 = ´ 20 (0.2) 2 = 0.4 kg m2
= ç ÷ç ÷ 2
3 è 2 ø è2ø
67. Moment of inertia of a ring about its diameter
Hence, net moment of inertia about an axis passing through
1 2
its middle point O is I= mr
2 2 2
1 æMö æ L ö 1 æMö æ L ö 1 é ML2 ML2 ù ML2
I= ç ÷ç ÷ + ç ÷ç ÷ = ê + ú= and kinetic energy is given by
3 è 2 ø è2ø 3 è 2 ø è2ø 3ë 8 8 û 12
1
E k = Iw2
62. Radius of earth = R 2
1 1
Mass of earth = M = ´ mr 2 ´ w2
2 2
Angular momentum about the axis of rotation is
1
2 2 p 4 pMR 2 = ´ 10 ´ 0.5 ´ 0.5 ´ 20 ´ 20
J= MR 2 ´ = 4
5 T 5T
= 250 J
Telegram @unacademyplusdiscounts

Rotational Motion 351

68. The moment of inertia of a disc about the axis is l


1 A B
I = mR 2 O
2
1
M R2
I1 2 1 1
Hence, = F P
I2 1 M R 2
2 2 D 2l
2
pR12 td ´ R12
= [Q M = pR12 td and M1 = M2]
pR22 td ´ R22
4
I1 R14 (0.2) 4 æ 0.1ö 1
\ = 4 = =ç ÷ = C
I2 R2 (0.6) 4 è 0.3 ø 81
Let DP = x
(Q R1 = 0.2 cm,R2 = 0.6 cm)
As P is the centre of mas, therefore
9M
69. Mass per unit area of disc = ml ( l - x) = 2 mlx
pR 2
2 Þ l : x = l /3
9M æRö l 4l
Mass of removed portion of disc = ´p ç ÷ =M \ CP = CD + DP = l + =
pR 2 è3ø
3 3
Moment of inertia of removed portion about an axis passing
71. Since w is constant, v would also be constant, so, no net force
through centre of disc and perpendicular to the plane of disc,
or toruqe is acting on ring. The force experienced by any
using theorem of parallel axis is
particle is only along radial direction or we can say it the
2 2
M æRö æ2Rö 1 2 centripetal force.
I1 = ç ÷ +M ç ÷ = MR
2 è3ø è 3 ø 2
When portion of disc would not have been removed, then the F2
moment of inertia of complete disc about the given axis is R2 F1
R1
1 v
I2 = MR 2 ω
2
So moment of inertia of the disc with removed portion, about
the given axis is
9 1
I = I2 - I1 = MR 2 - MR 2 = 4 MR 2
2 2 The force experienced by inner part, F1 = mw2R1 and the force
70. The object will have translation motion without rotation, experienced by outer part, F2 = mw2R1
when force F is applied at the centre of mass of system. If m is F1 mR1w2 R1
mass per unit length, the mass of AB, m1 = ml at O and mass of \ = =
F2 mR2w2 R2
OC,m2 = m (2 l). Here, CD = l.
Telegram @unacademyplusdiscounts

10 Gravitation
JEE Main MILESTONE
< The Universal Law of Gravitation < Escape Velocity
< Acceleration due to Gravity < Motion of a Satellite of Mass ( m ) around the Earth ( M )
< Gravitational Field < Geostationary Satellite or Parking Satellite
< Gravitational Potential < Kepler’s Laws of Planetary Motion
< Gravitational Potential Energy

10.1 The Universal Law of Gravitation


In this universe, each body attracts other body with a force that is directly
proportional to the product of their masses and inversely proportional to the
square of the distance between them.
Let m1 and m2 be the masses of two bodies and r be the separation between them.
mm
F µ 12 2 Gravitation is one of the four
r
Gm1m2 classes of interactions found in
Þ F = nature. These are
r2
(i) the gravitational force
Here, G is the constant of proportionality which is called ‘Universal gravitational
(ii) the electromagnetic force
constant’. The value of G is 6.67 ´ 10–11 N-m2 kg–2.
(iii) the strong nuclear force and
The direction of the force F is along the line joining the two particles. Regarding (iv) the weak nuclear force
gravitational force, following points should be noted.
1. The gravitational force between two particles is independent of the presence of It is gravity that holds the
other bodies or the properties of the intervening medium. universe together. In this chapter,
2. Gravitational force is a conservative force therefore work done in displacing a we shall learn the basic laws that
body from one place to another is independent of the path followed: It depends govern gravitational interactions.
only on the initial and final positions.
3. The gravitational force obeys Newton’s third law
i. e., F12 = - F21

10.2 Acceleration due to Gravity m

If M is the mass of earth and R is the radius, the earth attracts a R


mass m on its surface with a force F given by
GMm Centre
F=
2
R
This force imparts an acceleration to the mass m which is
known as acceleration due to gravity (g).
Telegram @unacademyplusdiscounts

Gravitation 353

By Newton’s law, we have Sample Problem 3 Three equal masses of m kg each are
Force fixed at the vertices of an equilateral triangle ABC (as shown).
Acceleration = ,
Mass The force acting on a mass 2m placed at the centroid G of the
GMm triangle is
F 2 GM y
Acceleration ( g ) = = R = 2
m m R A m
GM
On the surface of earth, g = 2
R
G 2m
Substituting the values of G, M, R, we get g = 9.81 ms-2.
B C
Mass of the earth m = 6 ´ 1024 kg and radius of the earth
m m
R = 6.4 ´ 106 m.
(a) 2 Gm2 $j (b) Gm2 $j
Sample Problem 1 A mass of 5 kg is first weighed on a m2 $
balance at the top of a tower of 20 m height. The mass is then (c) G j (d) zero
2
suspended from a fine wire 20 m long and reweighed. What is
the difference in weights? Mass of earth = 6 ´ 10 24 kg and Interpret (d) The angle between GC and the positive x-axis is
G = 6.67 ´ 10 –11 Nm 2 kg -2. 30° and so is the angle between GB and the negative x-axis. The
individual forces in vector notation are [NCERT]
(a) 0.04 g
Gm (2m) $
(b) 0.004 g FGA = j
1
(c) 0.03 g
Gm (2m) $
(d) 0.003 g FGB = ( - i cos 30° - $j sin 30° )
1
GMm
Interpret (d) As, Force = Gm (2m) $
R2 FGC = ( + i cos 30° - $j sin 30° )
1
6.67 ´ 10 –11 ´ 6 ´ 10 24 ´ 5
= From the principle of superposition and the law of vector addition,
(6420 ´ 10 8) 2
the resultant gravitational force FR on (2 m) is
= 48.85522N FR = FGA + FGB + FGC
6.67 ´ 10–11 ´ 6 ´ 6 ´ 1024 ´ 5
Force at surface = FR = 2 Gm2 $j + 2 Gm2 ( - $i cos 30° - $j sin 30° )
(6400 ´ 103 ) 2
= 48.8525 N +2 Gm2 ( $i cos 30° - $j sin 30° ) = 0

Therefore, change in weight = 48.85522 – 48.8525 N = 0.003 N. Note On the basis of symmetry from the figure, the resultant force
If g = 10 N kg–1, this is equivalent to the weight of a 0.03 g mass on comes out zero.
the earth’s surface.
Sample Problem 4 Two uniform solid spheres of equal
Sample Problem 2 Two point masses each equal to 1 kg radii R, but mass M and 4 M have a centre to centre separation
nce 6 R as shown. The
attract two
one spheres
another areaheld
with fixed.
force of A projectile
–9 of
between the two point masses is approximately mass m is projected from the surface of the sphere of mass M
(G = 6.6 ´ 10 –11 MKS unit). directly towards the centre of the second sphere.
(a) 8.2 cm m v
R N R
(b) 0.8 cm
O C
(c) 80 cm M 4M
(d) 0.08 cm r
6R
Interpret (a) Here,
Gm1m2
F= The minimum speed (v) of the projectile so that it reaches the
r2 surface of the second sphere is
Gm1m2 æ 5R ö
1/ 2
æ 3 GM ö
1/ 2
\ r=
F (a) ç ÷ (b) ç ÷
è 3 GM ø è 5R ø
6.67 ´ 10 –11 ´ 1 ´ 1 æ2 R ö æ 3 GM ö
= (c) ç ÷ (d) ç ÷
9.8 ´ 10 –9 è 3 GM ø è2 R ø
= 0.082 m = 8.2 cm
Telegram @unacademyplusdiscounts

354 JEE Main Physics

Interpret (b) The projectile is acted upon by two mutually At the neutral point N, the speed approaches zero. The mechanical
opposing gravitational forces of the two spheres. The neutral point energy at N is purely potential.
N is defined as the position where the two forces cancel each other GMm 4 GMm
exactly. If ON = r, we have EN = - -
2R 4R
GMm 4 GMm
=
r2 (6 R - r) 2 From the principle of conservation of mechanical energy.
2 2 1 2 GM 4 GM GM GM
or (6 R - r) = 4 r v - - =- -
2 R 5R 2R R
Þ 6R -r = ± 2r
2 GM æ 4 1 ö
Þ r = 2 R or - 6 R or v2 = ç - ÷
R è 5 2ø
ON = r = 2 R. It is sufficient to project the particle with a speed 1/ 2
which would enable it to reach N. Thereafter, the greater æ 3 GM ö
Þ v=ç ÷
gravitational pull of 4 M would suffice. The mechanical energy at è 5R ø
the surface of M is
1 GMm 4 GMm Note The speed of the projectile is zero at N, but is non-zero when it
Ei = mv 2 - -
2 R 5R strikes the heavier sphere 4M.

Hot Spot V ar i at i o n of g With


Altitude and Depth
A look at previous year papers shows a regular trend of questions being asked almost every year on variation of (g)with
height and depth.
The acceleration due to gravity We see that the value of g ¢ decreases as one goes up. Thus,
F Gm g æ GM ö
g= …(i) g¢ = 2
= …(iv) çQ g = 2 ÷
2
m æ hö æ hö è R ø
R2 ç1 + ÷ ç1 + ÷
where, F is the exerted force by the earth on an object of mass m. This è Rø è Rø
force is affected by a number of factors, thus g depends on these
If h << R,
factors. -2
æ hö æ 2 hö
then, g ¢ = g ç1 + ÷ = g ç1 - ÷ …(v)
Variation in the values of g above the surface è Rø è Rø

of the earth
Variation in value of ‘g’ below the surface of
When an object is placed at a distance h above the surface of the
earth, the force of gravitation is
the earth
GMm Acceleration due to gravity on the surface of the earth is
F= …(ii)
( R + h)2 GM 4
g = 2 = prGR ...(vi)
R 3
m
where, r is the density of the earth.
h
Acceleration due to gravity at depth d from the surface of the
earth,
g
R
dP
g′
R
rO

Acceleration due to gravity ( g ) at trip point is given by


F GM 4
g¢ = = …(iii) g¢ = pr G( R - d ) ...(vii)
m ( R + h)2 3
Telegram @unacademyplusdiscounts

Gravitation 355

-2
From Eqs. (vi) and (vii), we get g¢ æ hö æ 2hö
Þ = ç1 + ÷ = ç1 - ÷
g è R ø è R ø
é dù
g ¢ = g ê1 - ú g
ë Rû Here, g¢ =
g' 2
g /2 æ 2 h ö 2h 1 R
Below \ = ç1 - ÷ Þ = or h =
GM surface g è R ø R 2 4
—– Above
R2 surface
1 Sample Problem 6 A what depth below the surface of the
g∝—
r
earth, the value of g is the same as that at a height of 5 km?
O R (a) 5 km (b) 2.5 km
(c) 10 km (d) 6 km
where, g = acceleration due to gravity on the surface.
Interpret (c) Acceleration due to gravity at depth d below the
Note It should be noted that the value of g decreases, if we move above surface of earth,
the surface or below the surface of the earth. æ dö
g d = g ç1 - ÷ …(i)
è Rø
Variation on in the value of (g) due to Rotation of the earth
Due to rotation of the earth, the value of g decreases as the speed of Acceleration due to gravity at height h from the surface of the earth,
æ 2hö
rotation of the earth increases. The value of acceleration due to gravity g h = g ç1 - ÷ …(ii)
è R ø
at a latitude is
g l¢ = g - Rw2 cos2 l Here, gh = gd
æ 2hö æ dö
Following conclusions can be drawn from the above discussion \ g ç1 - ÷ = g ç1 - ÷
è R ø è Rø
(a) The effect of centrifugal force due to rotation of the earth is to
reduce the effective value of g. 2h d
Þ =
(b) The effective value of g is not truely in vertical direction. R R
(c) At the equators, l = 0° \ d = 2h
Therefore, g ¢ = g - Rw2 (minimum value) Thus, d = 2 ´ 5 = 10 km
(d) At the poles, l = 90°
Therefore, g¢ = g (maximum value) Sample Problem 7 The effect of rotation of the earth on
the value acceleration due to gravity is
Note (a) g is maximum at the equator and minimum at the pole
● At the equator, the rotation of the earth is maximum and value of g is (b) g is minimum at the equator and maximum at the pole
minimum. (c) g is equal at the equator and pole both
● At the poles, effect of rotation of the earth is zero and value of g is (d) g is maximum at the both poles
maximum.
Interpret (a) The value of acceleration due to gravity,
Sample Problem 5 The acceleration due to gravity g l¢ = g - Rw2 cos2 l …(i)
becomes g / 2 (g = acceleration due to gravity on the surface of
For the pole l = 90°,
the earth) at a height equal to
R R R R Putting the value of l = 90° in Eq. (i), we get
(a) (b) (c) (d)
4 2 3 5 g pole = g - w2R cos2 90°
\ g pole = g
Interpret (a) The acceleration due to gravity,
GM Therefore, there is no effect of rotational motion of the earth on the
g = …(i)
R2 value of g at the poles, i. e. , minimum.
At a height h above the surface of the earth, the acceleration due to For the equator, l = 0°, putting the value of l = 0°, in Eq. (i), we get
gravity is g equator = g - w2R cos20°
GM
g¢ = …(ii)
(R + h) 2 \ g equator = g - w2R
From Eqs. (i) and (ii), we get
2 2
Therefore, the effect of rotation of the earth on the value of g at the
g æR + hö æ hö equator is maximum.
=ç ÷ = ç1 + ÷
g¢ è R ø è Rø
Telegram @unacademyplusdiscounts

356 JEE Main Physics

(iv) Due to a solid sphere having a uniform mass


10.3 Gravitational Field distribution.
The space surrounding a material body in which its E
gravitational force of attraction can be experienced is GM
called its gravitational field. R2

Gravitational Field Intensity (E)


Gravitational field intensity at any point is defined as the O r=R r
gravitational force experienced by any test mass divided
For inside point (r < R),
by the magnitude of test mass when placed at the desired
point. Gmr
E=
F R3
E= r
m0
For outside point (r ³ R),
It means that the mass is so small that it doesn’t affect the Gm
original field when brought there. E=-
r2
E is a vector quantity and its direction is same as that of Fr .
It is expressed in Nkg–1.

Gravitational Field Intensity due to Various 10.4 Gravitational Potential (V )


Mass Distributions Gravitational potential at a point is defined as –ve of the
(i) Due to point mass, the gravitational field intensity at P work done by a gravitational force per unit mass in
distant r from a point mass m, bringing a unit test mass from some reference point
Gm (Generally considered at infinite) to the desired point.
is given by E = 2 , i.e., it is directed towards the point ¥
r
mass. V =
ò dWg = ò r F¥ dr
r m m
M r P It is a scalar quantity and its SI unit is J kg -1.
(ii) Due to a ring having a uniform mass distribution.
Gravitational potential comes out to be always negative.
m, R
M Relation between gravitational field intensity and
gravitational potential is expressed as
R
dV
E=-
O O r P dr
é ¶V $ ¶V $ ¶V $ ù
or E = -ê i+ j+ k
ë ¶x ¶y ¶z úû
At the centre, E = 0
where, V is a function of x, y and z
Gmr dV
on the axis, E= PO if V is depends only on x . E = - ,
(R2 + r2 )3 /2 dx
(iii) Due to hollow sphere having a m, R
uniform mass distribution. For Gravitational Potential due to Various Mass
inside point, (r < R), E = 0. For Distributions
outside point, (r ³ R),
(i) Due to point mass
GM
E= Gm
r2 V=-
E r
where, r is the distance of
(ii) Due to ring having a uniform mass distribution.
point from the centre.
r Gm
So, from the above expression, R At the centre, V=-
we can say, gravitational field intensity due to hollow R
sphere having uniform mass distribution at any Gm
On the axis, V=-
outside point is same as if the entire mass is R2 + r2
concentrated at its centre.
Telegram @unacademyplusdiscounts

Gravitation 357

(iii) Due to a hollow sphere of radius R having a uniform The gravitational intensity at O due to mass m at A is
mass distribution. For internal points (r £ R). Gm Gm
EA = = along OA
Gm (OA) 2 ( a / 3) 2
V =-
R Similarly, the gravitational intensity at O due to mass m at B is
V
Gm Gm
EB = 2
= along OB
(OB) ( a / 3) 2
r=R
r
and gravitational intensity at O due to mass m at C is
Gm Gm
–GM EC = = along OC
R (OC) 2 ( a / 3) 2
Gm As E A , EB and EC are equal in magnitude and equally inclined to
For external points (r > R), V = -
r each other, the resultant gravitational intensity at O is zero.
(iv) Due to a solid sphere of radius R having a uniform
Sample Problem 9 At a point above the surface of the
mass distribution.
earth, the gravitational potential is -5.12 ´ 107 J kg –1and the
V
acceleration due to gravity is 6.4 ms–2. Assuming the mean
radius of the earth to be 6400 km, the height of this point above
O r the earth’s surface is
(a) 1400 km (b) 1500 km
(c) 1600 km (d) 1700 km
– 3GM
2R
R Interpret (c) Let r be the distance of the given point from the
centre of the earth. Then
For inside points (r < R), GM
Gravitational potential = - = - 5.12 ´ 107 …(i)
- Gm (3 R2 - r 2 ) r
V =
2 R3 and acceleration due to gravity,
GM
For outside points (r ³ R), g = 2 = 6.4 ...(ii)
Gm r
V =-
r Dividing Eq. (i) by Eq. (ii), we get
5.12 ´ 10 7
r=
Sample Problem 8 Three particles, each of mass m are 6.4
placed at the vertices of an equilateral triangle of side a. What is = 8 ´ 10 6 m
the gravitational field at the centroid of the triangle?
= 6000 km
Gm
(a) Zero (b) -3 3 \Height of the point from earth’s surface
a2
3 Gm 2 Gm = r - R = 8000 - 6400
(c) (d) = 1600 km
a2 a2
Interpret (a) In the figure, O is the centroid of triangle ABC,
Am

10.5 Gravitational Potential
Energy
IA
The potential energy of a system corresponding to a
E
F conservative force was defined as
O
f
U f - U i = ò F × dr …(i)
i
IB IC
m• • •m We can calculate the change in gravitational potential
B D C
energy of the earth particle system, when the particle is
2 2 raised through a small height over the surface of the earth.
where, OA = AD = ( AB sin 60° )
3 3 \The change in potential energy
2 3 a
= ´a´ = U f - U i = mgh …(ii)
3 2 3
a Now, we can derive the general expression for the change
Thus, OA = OB = OC = in gravitational energy of a two-particle system.
3
Telegram @unacademyplusdiscounts

358 JEE Main Physics

r For an N particle system, there are N (N - 1) /2 pairs and


dr
r1 the potential energy is calculated for each pair and added
A B D E C to get the total potential energy of the system. For example
r2 N = 3, 4 , 5, ¼

Consider a particle of mass m1 be kept fixed at a point A


Sample Problem 10 A thin uniform annular disc of mass
and another particle of mass m2 is taken from a point B as M has outer radius 4R and inner radius 3R. The work required to
shown in figure. The distance between the particles is take a unit mass from point P on its axis to infinity is
AB = r1 and AC = r2.
Now, we can calculate the change in potential energy of
the system of two particles as distance change from r1 to r2. 4R
Consider a small displacement, when particles changes 2R
3R
from r to r + dr. From figure, we see that the second
particle going from D to E.
\Force on second particle , 2 GM 2 GM
(a) ( 4 2 - 5) (b) - ( 4 2 - 5)
Gm1m2 7R 7R
F = along DA …(iii)
r2 GM 2 GM
(c) (d) ( 2 - 1)
\Work done by the gravitational force in the displacement 4R 5R
is Interpret (a) W = DU = U f - Ui = U ¥ - U p
Gm1m2
dW = - dr …(iv) = - Up = -mVp
r2
= -Vp (as m = 1)
\Increase in potential energy of the two particles system is
4R √16R + r
2 2
Gm1m2
dU = - dW = dr …(v)
r2
The increase in potential energy from r1 to r2 is
r
U (r2 ) - U (r1 ) = ò dU dr

r2 Gm1m2 r2 1
=ò 2
= Gm1m2 ò 2 dr
r1 r 1 r
r
r Potential at point P will be obtained by integration. Let dM be
æ 1ö 2 mass of small ring as shown
= Gm1m2 ç - ÷
è r ør M
1
dM = (2 pr) dr
æ 1 1ö p ( 4 R) 2 - p (3 R) 2
= Gm1m2 ç - ÷ …(vi)
è r1 r2 ø 2 Mrdr
=
7 R2
We choose the potential energy of the two particle system
G × dM
to be zero, when the distance between them is ¥, i. e. , dVp = -
U (¥ ) = 0. 16 R 2 + r 2

From Eq. (vi), the potential energy U (r ), when the 2 GM 4R r


2 ò 3R
=- dr
separation between the particles is r, is 7R 16 R 2 + r 2
U (r ) = U (r ) - U (¥ ) 2 GM
=- ( 4 2 - 5)
æ 1 1ö Gm1m2 7R
= Gm1m2 ç - ÷ = - …(vii)
è¥ rø r 2 GM
W= ( 4 2 - 5)
The gravitational potential energy of a two particle system is 7R
Gm 1m 2
U (r ) = - …(viii) Sample Problem 11 Infinite number of masses each 1 kg,
r
are placed along the x-axis at x = ±1 m, ± 2 m, ± 4m, ± 8 m,
It should be noted that the potential energy depends ±16 m, …. .The magnitude of the resultant gravitational potential
only on the separation and not on the location of the in terms of gravitational constant G at the origin ( x = 0) is
particles. (a) G / 2 (b) G
(c) 2 G (d) 4 G
Telegram @unacademyplusdiscounts

Gravitation 359

Interpret (c) Gravitational potential 4 Gm2 Gm2


\ W (r) = - -2
æ1 1 1 ö æ1 1 1 1 1 ö l 2l
V = GM ç + + + ¼÷ = G ´ 1 ç + + + + + ¼÷
è r1 r2 r3 ø è1 2 4 8 16 ø 2 Gm2 æ 1 ö
=- ç2 + ÷
a l è 2ø
Sum of GP =
1- r Gm2
æ 1 ö = - 5.41
V =G ç ÷ = 2G l
è1- 1/ 2 ø Gm2
Hence, potential energy is proportional to .
l
Sample Problem 12 If g is acceleration due to gravity on
earth’s surface, the gain of the potential energy of an object of
mass in raised from the surface of the earth to a height equal to
the radius R of the earth is
10.6 Escape Velocity
(a) 2 mgR (b) mgR It is the minimum velocity with which a body must be
1 1
(c) mgR (d) mgR projected from the surface of the earth so that it escapes
2 4 the gravitational field of the earth. We can also say that a
Interpet (c) The potential energy of an object at the surface of body, projected with escape velocity, will be able to go to a
the earth is point which is at infinite distance from the earth.
GMm
U1 = - Earth
R
The potential energy of the object at a height h = R from the surface
of the earth is R
ve
m
GMm GMm Radius = R
U2 = - =-
R+h R+R Mass = M
Gain in potential energy of the object is
DU = U2 - U1 Let us imagine what happens to a body of mass m if it is
GMm GMm thrown from the earth with a velocity ve (escape velocity).
DU = - +
R+R R As the body moves away from the earth, it slows down
1 GMm (due to gravitational pull of the earth) and hence, its
DU =
2 R kinetic energy is converted into gravitational potential
Also gR 2 = GM energy of the mass-earth system. Let us imagine that it is
1 R 2m just able to reach upto infinity (where gravitational
Hence, DU = g potential energy is zero)
2 R
1 KE lost by mass m = gain in gravitational potential energy
Þ DU = mgR
2 of mass-earth system
1
Sample Problem 13 The potential energy of a system of mve2 = U f - U i
2
four particles placed at the vertices of a square of side l is
1 æ GMm ö
proportional to [NCERT] mve2 = 0 - ç - ÷
2 2 2 è R ø
ml lm
(a) (b)
G G (Q final potential energy is zero)
Gl Gm2 2 GM
(c) 2 (d) Þ ve =
m l R
Interpret (d) Consider four masses each of mass m at the or v e = 2gR
corners of a square of side l. We have four mass pairs at distance l
and two diagonal pairs at distance 2 l. Substituting the values of g = 9.81 ms– 2 and R = 6400 km,
l we get,
m m
r v e = 11.2 kms –1

2l Hence, any object thrown with a velocity of 11.2 kms–1 or


more will escape the gravitational field of the earth and
m
will never come back to the earth.
m
Telegram @unacademyplusdiscounts

360 JEE Main Physics

Application of Concepts of Escape 10.7 Motion of a Satellite of Mass


Velocity (m) around the Earth (M)
Maximum height attained by a particle
Consider a satellite of mass m h v
Suppose a particle of mass m is projected vertically revolving in a circle around the earth.
m
upwards with a speed v and we want to find the If the satellite is at a height h above Earth
maximum height h attained by the particle. Then, we can the earth’s surface, the radius of its r
use conservation of mechanical energy, i.e., orbit is r = R + h, where R is the radius
Decrease in kinetic energy of the earth. The gravitational force bit
= increase in gravitational potential energy of particle Or
between m and M provides the
1 1 mgh centripetal force necessary for a circular motion.
\ mv2 = DU or mv2 =
2 2 h
1+
R
Orbital Velocity
v2
Solving this, we get h = The velocity of a satellite in its orbit is called orbital
v2
2g- velocity. Let v be the orbital velocity of satellite, then
R
From this, we can see that G Mm mvo2
=
r2 r
(a) if v = ve or v2 = ve2 = 2 gR, h = ¥ and if
GM GM
v2 Þ vo = or vo =
(b) v is small, h = r R+ h
2g
Hence, the orbital velocity of a satellite is decided by the
Both the results are quite obvious. radius of its orbit or its height above the earth’s surface.

Sample Problem 14 Escape velocity on the earth is For a satellite very close to the earth’s surface,
11.2 kms–1. What would be the escape velocity on a planet GM GM
vo = = = gR
whose mass is 1000 times and radius is 10 times that of earth? r R
(a) 112 kms–1 (b) 11.2 kms–1
(c) 1.12 kms–1 (d) 3.7 kms–1 Time Period
Interpret (a) Escape velocity, v e = 2 gR = 2 GM ×R
The time taken to complete one revolution is called the
2
R time period. It is given by
or ve µ M / R 2 pr r
T = = 2 pr
Mass is 1000 times and radius is 10 times. Therefore, escape vo GM
velocity will become 10 times.
2 pr3/ 2
Þ T =
GM
Check Point 1 4 p2 3
Þ T2 = r (T = 24 h)
1. Why is Newton’s law of gravitation called a universal law? GM
2. On earth value of G = 6.67 × 10–11 Nm2 kg–2. What is its value The satellite which seems stationary, if observed from the
on moon, where g is nearly one-sixth than that of earth? surface of the earth is known as a Geostationary satellite.
3. Mass of a body can change without changing weight but
weight can changed without changing the mass. Explain. Total Energy of the Satellite
4. Where will the true weight of any body be zero? Total energy, TE = K + U
5. If the force of gravity acting on all bodies is proportional to (K = kinetic energy and U = potential energy)
their masses, why does not a heavy body fall correspondingly 1 æ GMm ö
faster than a light body? TE = mv2 + ç - ÷
2 è r ø
6. Where is the gravitational field zero and where is the
gravitational potential zero, incase of the earth? æ GMm ö æ G Mm ö
TE = ç ÷ -ç ÷
7. Why is gravitational potential energy negative? è 2r ø è r ø
8. What is the maximum value of gravitational potential energy GMm
TE = -
and where? 2r
Telegram @unacademyplusdiscounts

Gravitation 361

-GMm 2. If satellite and earth are rotating in the opposite


Binding energy = – (Total energy) =
2r direction,
T T
The energy that must be given to the orbiting satellite to TSE = S E
T E + TS
make it escape to infinity is known as binding energy.

Docking of Two Satellites/Spacecrafts


Trajectory of a Satellite for
Let us consider a spacecraft and a satellite to be in same
Different Speeds
circular orbit around the earth but at some separation,
Let v be the velocity given to a satellite. Let vo represent then the process to link the two is termed as docking.
the velocity for a circular orbit and ve be the escape B
A
velocity.
GM
vo =
r
2 GM
and ve =
r
where r is the distance of the satellite from centre of the
earth.
The docking process is not so simple as it seems to be, in
1. v > vo. The satellite follows an elliptical path with this one of the space vehicle is accelerated and docked
centre of the earth as the farther focus. In this case, if
with other. Because as soon as we increase or decrease the
satellite is projected from near the surface of the
speed of one (by using rocket firing system), the total
earth, it will hit the earth’s surface without
energy of this will increase or decrease and hence, the
completing its orbit.
orbit radius will change. If orbit itself changes, then
2. v = vo. The satellite follows a circular orbit with the
docking seems to be impossible.
centre of the earth as the centre of orbit.
3. vo < v < ve . The satellite follows an elliptical orbit with Let us say, we increase the speed of A by rocket-firing
the centre of the earth as the focus nearer to the point system, then it means kinetic energy of A increases and
of projection. hence, its total energy becomes less negative i. e. , value of r
4. v = ve . The satellite escapes from the field of the earth increases. Hence, we can say that A follows an elliptical
along a parabolic trajectory. path as shown, dotted line in the diagram. As semi-major
5. v > ve . The satellite escapes the field of the earth along axis increases, time period of A increases while of B
a hyperbolic trajectory. remains same as the initial one. As time periods of A and B
are different, they will rotate by different amount in same
time and hence, after a particular interval of time, they will
Time Period of a Satellite as Observed from come to the same point (from where the rockets has been
Earth fired) at same instant.
Consider a satellite in a circular orbit with the time period
TS . The earth also rotates with the time period, T E = 24 h. If
an observer on the earth sees this satellite, the angular 10.8 Geostationary Satellite or
velocity of the satellite relative to the earth will be Parking Satellite
wSE = wS - w E
If an artificial satellite revolves around the earth in an
Hence, the time for one revolution will
TS equatorial plane with a time period of 24 h in the same
appear to be different for T s , if TE
m sense as that of the earth, then it will appear stationary to
observed from the earth. This time Earth
the observer on the earth. Such a satellite is known as a
period can be calculated as TSE. r
geostationary satellite or parking satellite.
1. If satellite and earth are rotating t
rb
i At a given place (g = constant), the period of revolution of
in the same direction O earth’s satellite depends solely on its height above the
2p 2p 2p
= - surface of the earth. Let us calculate the height of the
TSE TS T E
satellite above the surface of the earth, so that the satellite
TS T E will have a period of revolution of 24 h and will appear as
Þ TSE =
| T E - TS | a stationary satellite.
Telegram @unacademyplusdiscounts

362 JEE Main Physics

Calculation of Height of
Check Point 2
Geostationary Satellite
1. The astronauts in a satellite orbiting the earth feel
r3 weightlessness. Does the weightlessness depend upon the
Time period, of satellite T =2p
gR2 distance of the satellite from the earth? If so how?
1/ 2 2. Is it possible to put an artificial satellite on an orbit in such a
2 p é (R + h)3 ù way that it always remains visible directly over Delhi?
= ê ú
R ë g û 3. An artificial satellite is revolving around the earth at a height
where, h = height of satellite above the surface of the 200 km from the earth’s surface. If a packet is released from
the satellite, what will happen to it? Will it reach the earth?
earth
1/ 3 4. Does the speed of a satellite remain constant in a particular
æ T 2R2 g ö orbit?
or h=ç ÷ -R
è 4p2 ø 5. According to Kepler’s second law, the radius vector to a planet
Putting T = 24 h = 24 ´ 3600 s, from the sun sweeps out equal area in equal interval of time.
The law is a consequence of which conservation law?
R = 6400 km = 6.4 ´ 106 m,
g = 9.8 ms–2.
We find 10.9 Kepler’s Laws of Planetary
1/ 3
2
é (24 ´ 3600) ´ (6.4 ´ 10 ) ´ 9.8 ù
h=ê ú
6 2
- 6.4 ´ 106 Motion
ë 4p2 û Based on trial and error, observation and using already
= 35930 km » 36000 km compiled data by earlier physicists, Kepler discovered
Thus, a satellite will appear stationary, if it revolves three empirical laws which accurately describe the
around the earth from west to east in an orbit coplanar motion of the planets. These laws are
with the equatorial plane at a height of about 36000 km
above the surface of the earth. Such an orbit is known as 1. Kepler’s First Law or Law of Orbits
synchronous or geostationary orbit or parking orbit and the It states ‘‘all the planets move around the sun in an
satellite revolving in this orbit is known as synchronous elliptical orbit with sun at one of the focus of ellipse’’. The
satellite. point when the planet is nearest to the sun is termed as
gR2 perihelion and the farthest one is known as aphelion.
Using relation, vo = , the orbital velocity of
R+ h
geostationary satellite comes out to be about 3.08 kms–1.
Planet

Sample Problem 15 A 400 kg satellite is in a circular S


orbit of radius 2 RE about the earth. Amount of energy required Perihelion Aphelion
or Perigee or Apogee
to transfer it to a circular orbit of radius 4RE is a
(a) 3.13´ 10 9 J
(c) 3.13 ´ 10 3 J (d) 5.29 ´ 10 9 J 2. Kepler’s Second Law or Law of Areas
GMEm It states “the line joining the sun
Interpret (a) Initially, Ei = - B
4 RE to the planet sweeps out equal A
GMEm areas in equal intervals of time, C I
While finally, Ef = -
8 RE i. e. , areal velocity of the planet II
The change in the total energy is w.r.t. sun is constant.’’ This is
D
called the law of area, which
GMEm æ GME ö mRE
DE = E f - Ei = =ç 2 ÷ indicates that a planet moves
8 RE è RE ø 8 faster near the sun and slowly
gmRE when away from the sun.
DE =
8
According to second law, area of region I = area of region II
9.81 ´ 400 ´ 6.37 ´ 10 6 where, the planet takes same time to move from A to B and
=
8 from C to D.
DE = 3.13 ´ 10 9 J
Telegram @unacademyplusdiscounts

Gravitation 363

3. Kepler’s Third Law or Law of Periods Then which one of the following statement regarding the
velocities vA and vB is true? [NCERT]
It states “the square of the planet’s time period of (a) v p < v A
revolution is directly proportional to the cube of (b) v p > v A
semi-major axis of its orbit.” (c) v p = v A
T 2 µ a3 where a is the semi-major axis. (d) v p = 0.075 v A
Kepler’s didn’t have any explanation for his laws,
neither he know why planets move in such way. But later Interpret (b) The magnitude of the angular momentum at P is
Newton found that all three Kepler’s laws can be Lp = mprpv p , since inspection tells us that rp and v p are mutually
perpendicular.
derived using Newton’s laws of motion and the law of
gravitation. Similarly, LA = mprAv A
From angular momentum conservation
Sample Problem 16 The planet Neptune travels around
mprpv p = mprAv A
the sun with a period of 165 yr. What is the radius of orbit
approximately, if the orbit is considered as circular? vP rA
=
(a) 20 R1 (b) 30 R1 (c) 25 R1 (d) 35 R1 v A rP

Interpret (b) T1 = Tearth = 1 yr Since, rA > rP ,vP > v A .

T2 = Tneptune = 165 yr Note The area SBAC bounded by the ellipse and the radius vectors SB
Let R1 and R2 be the radii of the circular orbits of the earth and and SC is larges than SBPC. From Kepler’s second law, equal areas are
Neptune respectively swept in equal times. Hence, the planet will take a longer time to traverse
BAC than CPB.
T12 R13 R3T 2
2
= 3 \ R32 = 1 22
T2 R2 T1 Sample Problem 19 The time period of moon’s
R3 ´ (165) 2 revolution is 27.3 days and radius of the earth is 6.37 ´ 106 m,
or R32 = 1 2
1 distance to the moon is 3.84 ´ 10 8 m, then the mass of the earth
\ R32 = 165 2 R13 is (approximately).
(a) 10 24 kg (b) 10 16 kg
or R2 = 30 R1
(c) 10 16 kg (d) 10 5 kg
Sample Problem 17 The maximum and minimum Interpret (a) Since moon is satellite of the earth, from Kepler’s
distances of a comet from the sun are 8 ´ 1012 m and third law, we have
1.6 ´ 1012 m. If its velocity when nearest to the sun is 60 ms-1.
4 p 2R3
-1
What will be its velocity in ms when it is farthest? T2 =
GME
(a) 12 (b) 60
(c) 112 (d) 6 4 p 2R3
ME =
GT 2
Interpret (a) By conservation of angular momentum
Putting the values, we have
mvr = constant
4 ´ 3.14 ´ 3.14 ´ (3.84)3 ´ 10 24
v min ´ rmax = v max ´ rmin ME =
6.67 ´ 10 –11 ´ (27.3 ´ 24 ´ 60 ´ 60) 2
60 ´ 1.6 ´ 10 22 60
\ v min = = = 12 ms–1 ME = 6.02 ´ 10 24 kg
8 ´ 10 22 5
Þ ME µ10 24 kg
Sample Problem 18 Let the speed of the planet at the
perihelion r be vp and the sun-planet distance SP be rP , and at 2

aphelion be vA and rA respectively. Note Alternate method of doing the question is M E = gRE putting the
G
B numerical values, we have
(9.8) 2 ´ (6.37 ´ 10 6 ) 2
ME =
6.67 ´ 10 –11
2b
P S′ A M E = 5.97 ´ 10 24 kg
Both methods yield almost the same answer, difference between them
C being less than 1%.
2a
Telegram @unacademyplusdiscounts

WORKED OUT
Examples
Example 1 Spheres of the same material and same radius r mw2r µ r5 / 2
are touching each other. Show that the gravitational force 1
Þ µ r -7 / 2
between them is directly proportional to T2
(a) r1 (b) r 2 Þ T 2 µ r 7/ 2
(c) r3 (d) r 4

Solution m1 = m2 = volume ´ density Example 4 At what distance (in metre) from the centre of the
moon, the intensity of gravitational field will be zero? Take
æ4 ö mass of earth and moon as 5.98 ´ 10 24 kg and 7.35 ´ 10 22 kg
= ç p r3 ÷ r
è3 ø
respectively and the distance between moon and earth is
3.85 ´ 10 8 m.
(a) zero (b) 3.85 ´ 10 7
8
(c) 8 ´ 10 (d) 3.46 ´ 10 8

r Solution Let x be the distance of the point from the centre of


earth where gravitational intensity is zero. Therefore,
æ4 ö æ4 ö
G ç pr3 ÷ ç pr3 ÷ r3 GMe GMm
Gm1m2 è3 ø è3 ø 2
=
\ f= = x (3.85 ´ 10 8 - x) 2
r2 r2
or F µ r4 x Me
or =
3.85 ´ 10 8 - x Mm
Example 2 Assuming earth to be a sphere of uniform mass 5.98 ´ 10 24
= @9
density, how much would a body weigh half way down the 7.35 ´ 10 22
centre of the earth, if it weighed 100 N on the surface?
x
(a) 20 N (b) 75 N or + x = 3.85 ´ 10 8
9
(c) 50 N (d) 30 N
or x = 9 ´ 3.85 ´ 10 8 / 10 = 3.46 ´ 10 8 m
Solution
Distance Given,
from
103.85
10 moon
3.46 mg= ´= -
100N
´ 8 8

Acceleration due to gravity at a depth d from the surface of earth = 3.9 ´ 10 7m


æ 1ö g
g ¢ = g ç1 - ÷ =
è 2ø 2 Example 5 If g is the acceleration due to gravity on the
mg 100 earth’s surface, the gain in potential energy of the body at a
Weight mg ¢ = = = 50N
2 2 height equal to three times the radius R of the earth will be
1
(a) mgR (b) mgR
Example 3 Imagine a light planet revolving around a very 2
massive star in a circular orbit of radius r with a period of 1 3
(c) MgR (d) mgR
revolution T. If the gravitational force of attraction between the 3 4
planet and star is proportional to r 5/ 2, then T 2 is proportional to
(a) r3 (b) r 7 / 2
Solution The graviational potential energy at any point at a
distance x from centre fo earth is E = - GMm/x on the surface of
(c) r5 / 2 (d) r3 / 2 earth x = R ,
Solution For revolution of the planet, centripetal force is -GMm
So, E1 = = - mgR
provided by gravitational force of attraction R
Telegram @unacademyplusdiscounts

Gravitation 365

At a height 3R, from the surface of earth, x = 4R Solution We know,


GMm
So, E2 = - = - mgR/ 4 2 pr 2 pr r3 / 2
4R T= = = 2p
v0 Gmr Gm
mgR 3
Increase in potential energy = - + mgR = mgR
4 4 or T µ r3 / 2
As h increases; r also increases, so T increases.
Example 6 R is the radius of earth and w is its angular
velocity and g p is the value of g at the poles. The effective value Example 9 The KE required to make a body move to infinity
of g at a latitude l = 60°. from the earth’s surface is
1 1 (a) infinite (b) 2 mgR
(a) g p - Rw2 (b) g p + Rw2
4 4 1
1 1 (c) mgR (d) mgR
(c) g p - Rw2 (d) g p + Rw2 2
2 2
1
Solution KE required = mv e2
Solution The acceleration due to gravity at a latitude l is 2
g ¢ = g - Rw2 cos2 l 1
= m ( 2gR) 2 = mgR
2
At the poles, l = 90°
\ g¢ = g = gp Example 10 The escape velocity of a body from the surface
\ g ¢ = g p - Rw2 cos2 l of earth is 11.2 kms -1. A body is projected with a velocity of
2 22.4 kms -1. Velocity of body at infinite distance from centre of
æ 1ö
or g ¢ = g p - Rw2 cos2 60° = g p - Rw2 ç ÷ earth would be
è2ø
(a) 11.2 kms -1 (b) zero
1
= g p - Rw2 (c) 11.2 3 kms -1 (d) None of these
4
Solution If v is the velocity of the body at infinite distance from
Example 7 A particle of mass 1 kg is kept on the surface of a the earth and u is the velocity of projection of body, then
uniform sphere of mass 20 kg and radius 1.0 m. The work to be 1 1 1
mv 2 = mv e2 + mv 2
done against the gravitational force between them to take the 2 2 2
particle away from the sphere, will be v 2 = u 2 - v e2
(a) 1.334 ´ 10 -9 J (b) 1.334 ´ 10 -10 J
or v = u 2 - v e2 = (22.4) 2 - (11.2) 2
(c) 2.33 ´ 10 -9 J (d) None of these
= 11.2 3 kms-1
Solution Potential on the surface of sphere
-GM - 6.67 ´ 10 -11 ´ 20 Example 11 A geostationary satellite is orbiting the earth at
V= = Jkg -1
R 1 a height 6R above the earth surface, where R is radius of earth.
= - 1.334 ´ 10 -9 Jkg -1 The time period of another satellite at a height 2.5R from earth’s
surface would be
i. e. ,1.33 ´ 10 -9 J work is obtained to bring a mass of 1 kg
6
from infinity to the surface of sphere. Hence, the same amount of (a) 24 h (b) h
2.5
work will have to be done to take the particle away from the
2.5
surface of sphere. (c) h (d) 2 2 h
6
Hence, W = 1.334 ´ 10 -9 J
Solution Here, T1 = 24h, R1 = 6R + R = 7R
Example 8 A satellite is orbiting round the earth at a height h T2 = ?, R2 = 2.5 R + R = 3.5R
above the surface of the earth. If this distance h is increased, the 3/ 2
time period of satellite will æR ö
Now, T2 = T1 ç 2 ÷
(a) decrease è R1 ø
(b) increase 3/ 2
æ 3.5 ö 24
(c) remain unaffected = 24 ç ÷ =
è 7 ø 2 2 =6 2h
(d) becomes zero
Telegram @unacademyplusdiscounts

Start Practice for


JEE Main
Round I (Topically Divided Problems)

Law of Gravitation 5. If a planet of given density were made larger its force
1. Two spheres of radius r and 2r are touching each of attraction for an object on its surface would increase
other. The force of attraction between them is because of planet’s greater mass but would decrease
proportional to because of the greater distance from the object to the
(a) r6 (b) r 4 centre of the planet. Which effect predominate?
(c) r2 (d) r -2 (a) Increase in mass
(b) Increase in radius
2. A solid sphere of
(c) Both affect attraction equally
uniform density and
(d) None of the above
radius R applies a
gravitational force of O R R 6. Both earth and moon are subject to the gravitational
attraction equal to F1
P force of the sun. As observed from the sun, the orbit of
on a particle placed at the moon [NCERT Exemplar]
P, distance 2 R from (a) will be elliptical
the centre O of the (b) will not be strictly elliptical because the total gravitational
sphere. A spherical cavity of radius R/2 is now made force on it is not central
in the sphere as shown in figure. The sphere with (c) is not elliptical but will necessarily be a closed curve
cavity now applies an gravitational force F2 on same (d) deviates considerably from being elliptical due to
particle placed at P. The ratio F2 / F1 will be influence of planets other than earth
(a) 1/2 (b) 7/9 (c) 3 (d) 7 7. Different points in earth are at slightly different
distances from the sun and hence experience
3. A uniform ring of mass M and radius r is placed
different forces due to gravitation. For a rigid body,
directly above a uniform sphere of mass 8 M and of
same radius R. The centre of the ring is at a distance we know that if various forces act at various points in
of d = 3 R from the centre of the sphere. The it, the resultant motion is as if a net force acts on the
gravitational attraction between the sphere and the CM (centre of mass) causing translation and a net
ring is torque at the CM causing rotation around an axis
through the CM For the earth-sun system
GM2 3 GM 2
(a) (b) (approximating the earth as a uniform density
R2 2 R2
sphere) [NCERT Exemplar]
2 GM2 3 GM2
(c) (d) (a) the torque is zero
2 R2 R2
(b) the torque causes the earth to spin
4. Imagine a light planet revolving around a very (c) the rigid body result is not applicable since the earth is
massive star in a circular orbit of radius r with a not even approximately a rigid body
period of revolution T. If the gravitational force of (d) the torque causes the earth to move around the sun
attraction between the planet and the star is 8. Two astronauts have deserted their space ships in a
proportional to R -3/ 2 , then T2 is proportional to region of space far from the gravitational attraction
(a) R3 (b) R5 /2 of any other body. Each has a mass of 100 kg and they
(c) R 3/2 (d) R7 /2 are 100 m apart. They are initially at rest relative to
Telegram @unacademyplusdiscounts

Gravitation 367

one another. How long will it be before the 14. The mass of the moon is 1/8 of the earth but the
gravitational attraction brings them 1 cm closer gravitational pull is 1/6 of the earth. It is due to the
together? fact that
(a) 2.52 days (b) 1.41 days (a) moon is the satellite of the earth
(c) 0.70 days (d) 0.41 days (b) the radius of the earth is 8.6 the moon
9. If three particles each of mass M are placed at the (c) the radius of the earth is 8 / 6 of the moon
three corners of an equilateral triangle of side a, the (d) the radius of the moon is 6/8 of the earth
forces exerted by this system on another particle of
15. If different planets have the same density but
mass M placed (i) at the mid point of a side and (ii) at
different radii, then the acceleration due to gravity
the centre of the triangle are respectively
on the surface of the planet is related to the radius
4 GM2
(a) 0, 0 (b) ,0 (R) of the planet as
3 a2
(a) g µ R2 (b) g µ R
2 2 2
4 GM 3 GM GM 1 1
(c) 0, (d) , 2 (c) g µ 2 (d) g µ
3 a2 a2 a R R
10. The gravitational attraction between the two bodies 16. A thief stole a box full of valuable articles of weight w
increases when their masses are and while carrying it on his head jumped down from
(a) reduced and distance is reduced a wall of height h from the ground. Before he reaches
(b) increased and distance is reduced the ground, he experienced a load
(c) reduced and distance is increased (a) zero (b) w /2
(d) increased and distance is increased (c) w (d) 2 w
11. A spherical hollow is made in a lead sphere of radius 17. Assuming the earth to be a sphere of uniform mass
R such that its surface touches the outside surface of density, how much would body weigh half way down
the lead sphere and passes through the centre. The to the centre of earth if it weighed 250 N on the
mass of the lead sphere before hollowing was M. The surface? [NCERT]
force of attraction that this sphere would exert on a (a) 225 N (b) 325 N
particle of mass m which lies at a distance d ( > R) (c) 100 N (d) 125 N
from the centre of the lead sphere on the straight line
joining the centres of the sphere and the hollow is
18. The maximum vertical distance through which a full
GM m GMm dressed astronaut can jump on the earth is 0.5 m.
(a) (b) Estimate the maximum vertical distance through
d2 8 d2
which he can jump on the moon, which has a mean
é ù é ù
ê ú ê ú density 2/3rd that of earth and radius one quarter
(c)
GMm ê
1 +
1 ú (d) GMm ê1 - 1 ú that of the earth
ê 2ú
d2 ê æ R öú d2 ê æ Rö ú (a) 1.5 m (b) 3 m
ê 8 ç1 + ÷ú
êë è 2 d ø úû ê 8 çè1 - 2 d ÷ø ú (c) 6 m (d) 7.5 m
ë û
19. In the above problem, the ratio of the time duration of
his jump on the moon to that of his jump on the earth is
Gravity and Acceleration due to Gravity (a) 1 : 6 (b) 6 : 1
12. If suppose moon is suddenly stopped and then (c) 6 : 1 (d) 1 : 6
released (given radius of moon is one-fourth the
radius of earth) and the acceleration of moon with 20. Particles of masses 2M, m and M are respectively at
1
respect to earth is 0.0027 ms–2), then the acceleration points A, B and C with AB = ( BC). m is much-much
2
of the moon just before striking the earth’s surface is
smaller than M and at time t = 0, they are all at rest
(Take g = 10 ms–2)
(a) 0.0027 ms–2 (b) 5.0 ms–2 2M m M
(c) 6.4 ms–2 (d) 10 ms–2
[NCERT Exemplar]
13. The acceleration due to gravity on a planet is (a) m will remain at rest
1.96 ms–2. If it is safe to jump from a height of 3 m on (b) m will move towards M
the earth, the corresponding height on the planet will (c) m will move towards 2M
be (d) m will have oscillatory motion
(a) 3 m (b) 6 m
(c) 9 m (d) 15 m
Telegram @unacademyplusdiscounts

368 JEE Main Physics

(a) S will run faster than P


Variation in the Acceleration due to (b) P will run faster than S
Gravity (c) both will run at the same rate as on the earth
21. The earth is an approximate sphere. If the interior (d) both will run at the same rate which will be different from
contained matter which is not of the same density that on the earth
everywhere, then on the surface of the earth, the 27. If the radius of the earth were to shrink by 1% its
acceleration due to gravity [NCERT Exemplar] mass remaining same, the acceleration due to gravity
(a) will be directed towards the centre but not the same on the earth’s surface would
everywhere (a) decrease by 2% (b) remain unchanged
(b) will have the same value everywhere but not directed (c) increase by 2% (d) become zero
towards the centre
(c) will be same everywhere in magnitude directed towards
28. Two spherical planets A and B have same mass but
the centre densities in the ratio 8 : 1. For these planets, the ratio
(d) cannot be zero at any point of acceleration due to gravity at the surface of A to its
value at the surface of B is
22. The masses and radii of the earth and moon are (a) 1 : 4 (b) 1 : 2
M1, R1 and M2 , R2 respectively. Then centres are (c) 4 : 1 (d) 8 : 1
distance d apart. The minimum velocity with which a
29. The height at which the acceleration due to gravity
particle of mass M should be projected from a point
decreases by 36% of its value on the surface of the
midway between their centres so that it escapes to
earth. (The radius of the earth is R).
infinity is R R R 2
G 2G (a) (b) (c) (d) R
(a) 2 ( M1 + M2 ) (b) 2 ( M1 + M2 ) 6 4 2 3
d d
30. If the value of g acceleration due to gravity at earth
GM GM ( M1 + M2 ) surface is 10 ms–2, its value in ms–2 at the centre of
(c) 2 ( M1 + M2 ) (d) 2
d d ( R1 + R2 ) the earth, which is assumed to be a sphere of radius R
metre and uniform mass density is
23. At a given place where, acceleration due to gravity is
(a) 5 (b) 10/R
g ms–2, a sphere of lead of density d kgm–3 is gently
(c) 10/2R (d) zero
released in a column of liquid of density r kgm–3. If
d > r, the sphere will 31. When of the following graphs correctly represents
(a) fall vertically with an acceleration of g ms–2 the variation of g on earth?
(b) fall vertically with no acceleration g g
æ d - rö
(c) fall vertically with an acceleration g ç ÷ (a) (b)
è d ø
(d) fall vertically with an acceleration r/ d R r R r
24. What is the height, the weight of body will be the g g
same as at the same depth from the surface of the
(c)earth? Radius of earth is R? (d)
R
(a) (b) 5 R - R r r
2 R R
5R-R 3R-R
(c) (d) 32. If the force inside the earth surface varies as x n ,
2 2
where r is the distance of body from the centre of
25. There is a mine of depth about 2.0 km. In this mine earth, then the value of n will be
the conditions as compared to those at the surface are (a) –1 (b) –2 (c) 1 (d) 2
(a) lower air pressure, higher acceleration due to gravity
33. 320 km above the surface of earth, the value of
(b) higher air pressure, lower acceleration due to gravity
acceleration due to gravity is nearly 90% of its value
(c) higher air pressure, higher acceleration due to gravity
on the surface of the earth. Its value will be 95% of
(d) lower air pressure, lower acceleration due to gravity
the value on the earth’s surface
26. A clock S is based on oscillation of a spring and a (a) nearly 160 km below the earth’s surface
clock P is based on pendulum motion. Both clock run (b) nearly 80 km below the earth’s surface
at the same rate on earth. On a planet having the (c) nearly 640 km below the earth’s surface
same density as earth but twice the radius, (d) nearly 320 km below the earth’s surface
Telegram @unacademyplusdiscounts

Gravitation 369

34. The acceleration due to gravity at a height 1/20th of 43. Two equal masses m and m are hung from a balance
the radius of the earth above the earth surface is whose scale pan differs in vertical height by h/2.
9 ms–2. Its value at a point at an equal distance below The error in weighing in terms of density of the earth
the surface of the earth in ms–2 is about r is
(a) 8.5 (b) 9.5 (c) 9.8 (d) 11.5 1
(a) pGrmh (b) pG pmh
3
35. At a distance 320 km above the surface of earth, the
4 8
value of acceleration due to gravity will be lower than (c) pG rmh (d) Grmh
3 3
its value on the surface of the earth by nearly
(radius of earth = 6400 km) 44. The radius of the earth is 6400 km and g = 10 m /s2 in
(a) 2% (b) 6% order that a body of 5 kg weight zero at the equator
(c) 10% (d) 14% the angular speed of the earth is
(a) 1/80 rad/s (b) 1/400 rad/s
36. The depth from the surface of the earth of radius R at
(c) 1/800 rad/s (d) 1/1600 rad/s
which the acceleration due to gravity will be 75% of
the value on the surface of the earth is 45. What should be the angular speed of earth in rads–1
(a) R / 4 (b) R /2 so that a body of 5 kg, weighs zero at the equator?
(c) 3 R / 4 (d) R /8 (Take g = 10 ms–2 and radius of earth = 6400 km).
37. Two equal masses m and m are hung from a balance (a) 1/1600 (b) 1/800
whose scale pans differ in height by h. If r is the mean (c) 1/400 (d) 1/80
density of earth, then the error in weighing machine is 46. The bodies situated on the surface of earth at its
(a) zero (b) 4 p Grmh / 3 equator, becomes weightless, when the earth has KE
(c) 8 p Grmh / 3 (d) 2 pGrmh / 3 about it axis
38. One goes from the centre of the earth to a distance (a) mgR (b) 2 mgR/5
two-third the radius of the earth, where will the (c) MgR/5 (d) 5 MgR/2
acceleration due to gravity be the greatest? 47. At what height above the earth’s surface, does the
(a) At the centre of the earth force of gravity decrease by 10%? The radius of the
(b) At a height half the radius of the earth earth is 6400 km?
(c) At a height one-third the radius of the earth (a) 345.60 km (b) 687.20 km
(d) At a height two-third the radius of the earth (c) 1031.8 km (d) 12836.80 km
39. Mass of moon is 7.34 ´ 1022 kg. If the acceleration due 48. The value of g on the earth’s surface is 980 cms–2. Its
to gravity on the moon is 1.4 ms –2 , the radius of the value at a height of 64 km from the earth’s surface is
moon is (G = 6.667 ´ 10–11 Nm 2 kg –2 ) (Radius of the earth R = 6400 km)
(a) 0.56 ´ 104 m (b) 1.87 ´ 106 m (a) 960.40 cms–2
(c) 1.92 ´ 106 m (d) 1.01 ´ 108 m
(b) 984.90 cms–2
40. The ratio of acceleration due to gravity at a height h (c) 982.45 cms–2
above the surface of the earth and at a depth h below (d) 977.55 cms–2
the surface of the earth for h < radius of earth
(a) is constant 49. The speed of earth’s rotation about its axis is w. Its
(b) increases linearly with h speed is increased to x times to make the effective
(c) decreases linearly with h acceleration due to gravity equal to zero at the
(d) decreases parabolically with h equator, then value of x is around
(g = 10 ms–2; R = 6400 km)
41. At what height in km over the earth’s pole the free
(a) 1 (b) 8.5
fall acceleration decreases by one percent?
(c) 17 (d) 34
(Assume the radius of the earth to be 6400 km)
(a) 32 (b) 64 50. For a body lying on the equator to appear weightless,
(c) 80 (d) 1.253 what should be the angular speed of the earth?
42. If a man weighs 90 kg on the surface of earth, the (Take g = 10 ms–2; radius of earth = 6400 km)
height above the surface of the earth of radius R, (a) 0.125 rads–1
where the weight is 30 kg, is (b) 1.25 rads–1
(a) 0.73 R (b) R / 3 (c) 1.25 ´ 10–3 rads–1
(c) R /3 (d) 3 R (d) 1.25 ´ 10–2 rads–1
Telegram @unacademyplusdiscounts

370 JEE Main Physics

Gravitational Field 55. A particle of mass m is placed inside a spherical shell,


away from its centre. The mass of the shell is M.
51. P is a point at a distance r from the centre of a solid (a) The particle will move towards the centre if m < M, and
sphere of radius r. The variation of gravitational away from the centre if m > M
potential at P (i.e., V) and distance r from the centre (b) The particle will move towards the centre
of sphere is represented by the curve. (c) The particle will oscillate about the centre of shell
O
r O r (d) The particle will remain stationary
r=R r=R
56. The distance between the earth and the moon is
(a) V (b) V 3.85 ´ 108 m . At what distance from the earth’s
centre, the intensity of gravitational field will be
r r zero? The masses of earth and moon are
O
r=R
O
r=R 5.98 ´ 1024 kg and 7.35 ´ 1022 kg respectively.
(a) 3.47 ´ 108 m (b) 0.39 ´ 108 m
(c) V (d) V
(c) 1.82 ´ 108m (d) None of these
57. Two bodies of masses 100 kg and 1000 kg are
separated by a distance of 1 m. What is the intensity
52. The gravitational field due to a mass distribution is of gravitational field at the mid point of the line
I = k / x 3 in the x-direction (k is a constant). Taking joining them?
the gravitational potential to be zero at infinity, its (a) 6.6 ´ 10–11 N m2 kg–2 (b) 2.4 ´ 10–8 Nkg–1
value at a distance x/ 2 is (c) 2.4 ´ 10–7 Nkg–1 (d) 2.4 ´ 10–6 Nkg–1
(a) k / x (b) k /2x (c) k / x2 (d) k /2x2 58. There are two bodies of masses 100000 kg and
1000 kg separated by a distance of 1 m. At what
53. Select the proper graph between the gravitational
distance (in metre) from the smaller body, the
potential ( V g) due to hollow sphere and distance (r)
intensity of gravitational field will be zero?
from its centre.
(a) 1/9 (b) 1/10
Vg Vg
r (c) 1/11 (d) 10/11
(a) (b) 59. In a certain region of space, the gravitational field is
given by –k/r, where r is the distance and k is a
r –Vg constant. If the gravitational potential at r = r0 be V0 ,
Vg Vg then what is the expression for the gravitational
R r
R
r
potential V ?
ærö ær ö
(c) (d) (a) k log ç ÷ (b) k log ç 0 ÷
è r0 ø èrø
–Vg ærö ær ö
(c) V0 + k log ç ÷ (d) V0 + k log ç 0 ÷
è r0 ø èrø
54. Which of the following graphs represents correctly
the variation of the intensity of gravitational field (I) 60. The depth d at which the value of acceleration due to
1
with the distance (r) from the centre of a spherical gravity becomes times, the value at the surface is
shell of mass M and radius a? n
I (R = radius of the earth)
R æ n - 1ö
GM GM (a) (b) R ç ÷
n è n ø
a2 a2
(a) (b) R æ n ö
(c) 2 (d) R ç ÷
n è n + 1ø
r O r
O t=a t=a 61. A solid sphere is of density r and radius R. The
I gravitational field at a distance r from the centre of
the sphere, where r < R, is
GM GM
a2 a2 rpGR 3 4 pGrr 2
(c) (d) (a) (b)
r 3
4 pGrR 3 4 pGrr
(c) (d)
O t=a
r O t=a
r 3 r2 3
Telegram @unacademyplusdiscounts

Gravitation 371

62. Two bodies of masses 2 kg and 8 kg are separated by 69. A mass m is placed at a point B in the gravitational
a distance of 9 m. The point where the resultant field of mass M. When the mass m is brought from B
gravitational field intensity is zero is at a distance of to near point A, its gravitational potential energy will
(a) 4.5 m from each mass (a) remain unchanged
(b) 6 m from 2 kg (b) increase
(c) 6 m from 8 kg (c) decrease
(d) 2.5 m from 2 kg (d) become zero
63. Gravitational potential on the surface of earth is 70. The gravitational field in a region is given by
(m = mass of the earth, R = radius of earth) I = ( 4 i$ + $j) Nkg -1. Work done by this field is zero
(a) - GM /2R (b) - gR when a particle is moved along the line
(c) gR (d) GM / R (a) x + y = 6 (b) x + 4 y = 6
64. A particle of mass m is placed at the centre of a (c) y + 4 x = 6 (d) x - y = 6
uniform spherical shell of mass 3 m and radius R. 71. A satellite orbits the earth at a height of 400 km
The gravitational potential on the surface of the shell above the surface. How much energy must be
is expended to rocket the satellite out of the earth’s
Gm 3 Gm gravitational influence? Mass of the satellite
(a) - (b) -
R R = 200 kg, mass of the earth = 6.0 ´ 1024 kg, radius of
(c) -
4 Gm
(d) -
2 Gm . ´ 10-11 N-m2 /kg2 .
the earth = 6.4 ´ 106 m, G = 667
R R [NCERT]
65. The gravitational field due to a mass distribution is (a) 5.2 ´1010 J (b) 3 ´ 106 J
1 = C2 in x direction. Here C is constant. Taking the (c) 4 ´ 106 J (d) 6 ´ 109 J
x 72. A body of mass m rises to a height h = R/5 from the
gravitational potential to be zero at infinity, surface of earth, where R is the radius of earth. If g is
potential at x is the acceleration due to gravity at the surface of earth,
2C C
(a) (b) the increase in potential energy is
x x
(a) (4/5) mgh (b) (5/6) mgh
2C C
(c) 2 (d) 2 (c) (6/7) mgh (d) mgh
x 2x
73. The gravitational potential difference between
the surface of a planet and a point 20 m above it is
Gravitational Potential Energy 14 J kg–1. The work done in moving a 2.0 kg mass by
66. A space ship moves from earth to moon and back. The 8.0 m on a slope of 60° from the horizontal, is equal to
greatest energy required for the space ship is to (a) 7 J (b) 9.6 J (c) 16 J (d) 32 J
overcome the difficulty in 74. If W1, W2 and W3 represent the A 3
(a) entering the earth’s gravitational field work done in moving a particle
(b) take off from earth’s field from A to B along three different
(c) take off from lunar surface paths 1, 2 and 3 respectively (as 2
(d) entering the moon’s lunar surface shown) in a gravitional field of 1
67. The mass of the earth is 6.00 ´ 1022 kg. The constant point mass m, then
B
of gravitation G = 6.67 ´ 10–11 Nm 2 kg –2 . The (a) W1 = W2 = W3
potential energy of the system is –7.79 ´ 1028 J. The (b) W1 > W2 > W3
mean distance between earth and moon is (c) W1 > W2 < W3
(a) 3.80 ´ 108 m (b) 3.37 ´ 106 m (d) W1 < W2 < W3
(c) 7.60 ´ 104 m (d) 1.90 ´ 102 m
68. The change in potential energy when a body of mass Satellite
m is raised to a height nR from the centre of earth 75. Out of the following, the only correct statement about
(R = radius of earth) satellites is
( n - 1) (a) A satellite cannot move in a stable orbit in a plane
(a) mgR (b) nmgR
n passing through the earth’s centre
n2 n (b) Geostationary satellites are launched in the equatorial
(c) mgR 2 (d) mgR
n +1 n +1 plane
Telegram @unacademyplusdiscounts

372 JEE Main Physics

(c) We can use just one geostationary satellite for global 82. The escape velocity from the earth is 11 kms -1. The
communication around the globe escape velocity from a planet having twice the radius
(d) The speed of satellite increases with an increase in the and the same mean density as the earth would be
radius of its orbit (a) 5.5 kms–1 (b) 11 kms–1
76. A satellite S is moving in an elliptical orbit around (c) 15.5 kms–1 (d) 22 kms–1
earth. The mass of the satellite is very small 83. The escape velocity for a body projected vertically
compared to the mass of the earth? upwards from the surface of the earth is 11.2 kms -1.
(a) The acceleration of S is always directed towards the If the body is projected in a direction making an angle
centre of the earth of 45° with the vertical, the escape velocity will be
(b) The angular momentum of S about the centre of the
(a) 11.2 kms–1 (b) 11.2 ´ 2 kms–1
earth changes in direction but its magnitude remains
constant (c) 11.2 ´ 2 kms–1 (d) 11.2/ 2 kms–1
(c) The total mechanical energy of S varies periodically with 84. The ratio of the radii of the planets P1 and P2 is a. The
time ratio of their acceleration due to gravity is b. The
(d) The linear momentum of S remains constant in
ratio of the escape velocities from them will be
magnitude
(a) ab (b) ab
77. A satellite is placed in a circular orbit around earth at (c) a / b (d) b / a
such a height that it always remains stationary with
respect to earth surface. In such case, its height from 85. The mass of the moon is 1/81 of earth’s mass and its
the earth surface is radius 1/4th that of the earth. If the escape velocity
(a) 32000 km (b) 36000 km from the earth’s surface is 11.2 kms–1, its value for
(c) 6400 km (d) 4800 km the moon will be
(a) 0.15 kms–1 (b) 5 kms–1
78. Satellites orbiting the earth have finite life and (c) 2.5 kms–1 (d) 0.5 kms–1
sometimes debris of satellites fall to the earth. This is
because, [NCERT Exemplar]
(a) the solar cells and batteries in satellites run out
Kepler’s Laws
(b) the laws of gravitation predict a trajectory spiralling 86. If the radius of earth’s orbit is made 1/4th, then
inwards duration of an year will become
(c) of viscous forces causing the speed of satellite and hence (a) 8 times (b) 4 times
height to gradually decrease (c) 1/8 times (d) 1/4 times
(d) of collisions with other satellites
87. The period of revolution of planet A around the sun is
79. The orbital velocity of an artificial satellite in a 8 times that B. The distance of a from the sun is how
circular orbit just above the earth’s surface is v. For a many times greater than that of B from the sun?
satellite orbiting at an altitude of half of the earth’s (a) 2 (b) 3 (c) 4 (d) 5
radius, the orbital velocity is 88. The largest and the shortest distance of the earth
3 3 from the sun are r1 and r2 , its distance from the sun
(a) v (b) v
2 2 when it is perpendicular to the major axis of the orbit
2 2 drawn from the sun, is
(c) v (d) v
3 3 r1 + r2 r12r
(a) (b)
4 r1 + r2
2 r12r r +r
Escape Velocity (c) (d) 1 2
r1 + r2 3
80. For a body to escape from earth, angle at which it
should be fired is? 89. In our solar system, the inter-planetary region has
(a) 45º (b) > 45º chunks of matter (much smaller in size compared to
(c) < 45º (d) any angle planets) called asteroids. They [NCERT Exemplar]
(a) will not move around the sun since they have very small
81. If the moon is to escape from the gravitational field of
masses compared to sun
the earth forever, it will require a velocity
(b) will move in an irregular way because of their small masses
(a) 11.2 kms–1
and will drift away into outer space
(b) less than 11.2 kms–1
(c) will move around the sun in closed orbits but not obey
(c) slightly more than 11.2 kms–1 Kepler’s laws
(d) 22.4 kms–1 (d) will move in orbits like planets and obey Kepler’s laws
Telegram @unacademyplusdiscounts

Gravitation 373

1 /2 1 /2
90. A comet of mass m moves in a highly elliptical orbit é GMr1 ù é GMmr1 ù
around the sun of mass M. The maximum and (a) ê ú (b) ê ú
ë ( r1 + r2 ) û ë ( r1 + r2 ) û
minimum distances of the comet from the centre of the
1 /2 1 /2
sun are r1 and r2 respectively. The magnitude of æ 2 Gm2 r12r ö æ 2 GMm2 r12r ö
(c) ç ÷ (d) ç ÷
angular momentum of the comet with respect to the è r1 + r2 ø è ( r1 + r2 ) ø
centre of sun is

Round II (Mixed Bag)

Only One Correct Option 5. The correct graph representing the variation of
1. If the diameter of mars is 6760 km and mass total energy ( E), kinetic energy ( K ) and potential
one-tenth that of earth. The diameter of earth is energy (U) of a satellite with its distance from the
12742 km. If acceleration due to gravity on earth is centre of earth is
9.8 ms–2, the acceleration due to gravity on mars is

Energy

Energy
E E
(a) 34.8 ms–2 (b) 2.84 ms–2 U K
(c) 3.48 ms–2 (d) 28.4 ms–2 (a) O r (b) O r

2. Halley’s comet has a period of 76, had a distance of K U


closest approach to the sun equal to 8.9 ´ 1010 m. The

Energy
Energy
comet’s farthest distance from the sun if the mass of
K
sun is 2 ´ 1030 kg and g = 6.67 ´ 1011 in MKS units is K
(c) O r (d) O r
(a) 2 ×1012 m (b) 2.7 × 1013 m E
(c) 5.3 × 1012 m (d) 5.3 × 1013 m U
U
3. Two identical trains P and Q move with equal speeds E
on parallel tracks along the equator. P moves from 6. The work that must be done in lifting a body
east to west and Q from west to east. of weight P from the surface of the earth to a height h
(a) Data is sufficient to arrive at a conclusion is
(b) Both exert equal force on track PRh
(a)
(c) Train Q exerts force on track R-h
(d) Train P exerts greater force on track R+ h
(b)
PRh
4. A spherical symmetric gravitational system of PRh
ìr for r £ R (c)
particles has a mass density r = í 0 R+ h
î 0 for r > R R-h
(d)
where r 0 is a constant. A test mass can undergo PRh
circular motion under the influence of the 7. A spaceship is launched into a circular orbit close to
gravitational field of particles. Its speed v as a earth’s surface. The additional velocity that should
function of distance r (0 < r < ¥) from the centre of the be imparted to the spaceship in the orbit to overcome
system is represent by the gravitational pull is (Radius of earth = 6400 km
v v and g = 9.8 ms–2)
(a) 11.2 kms–1 (b) 8 kms–1
(a) (b) (c) 3.2 kms–1 (d) 1.5 kms–1

r r 8. A particle is fired vertically upwards from the surface


R R of earth and reaches a height 6400 km. The initial
v v velocity of the particle is (R = 6400 km, g = 10 ms–2)
(a) 11.2 ms–1
(c) (d) (b) 8 kms–1
(c) 3.2 kms–1
r r
R R (d) None of the above
Telegram @unacademyplusdiscounts

374 JEE Main Physics

9. A shell of mass M and radius R has a point mass m 16 M M


placed at a distance r from its centre. O1 O2
O R r O R r
2a a

(a) (b)
10 a

U (r) U (r) 2 GM 3 5 GM
(a) (b)
O R r O r 3 a 2 a
2 5 GM 3 GM
(c) (d)
(c) GMm (d) 3 a 2 a
R
14. Three particles each of mass m rotate in a circle of
U(r) U (r) radius r with uniform angular speed w under their
10. An artificial satellite of the earth moves at an mutual gravitational attraction. If at any instant the
altitude to h = 670 km along a circular orbit. The points are on the vertex of an equilateral of side L,
velocity of the satellite is then angular velocity w is
(a) 7.5 kms–1 (b) 8.5 kms–1 2 Gm 3 Gm
(a) (b)
(c) 11.2 kms–1 (d) 4.5 kms–1 L3 L3
11. A star 2.5 times the mass of the sun and collasped to a 5 Gm Gm
(c) (d)
size of 12 km rotates with a speed of 1.2 rev/s. L3 L3
(Extremely compact stars of this kind are known as 15. How will you weight the sun i. e., estimate its mass?
neutron stars. Certain observed stellar objects called You will need to know the period of one of its planets
pulsars are belived to belong to this category). Will and the radius of the planetary orbit. The mean
an object placed on its equator remain stuck to its
orbital radius of the earth around the sun is
surface due to gravity? (Mass of the sun
1.5 ´ 108 km. Then the mass of the sun is
= 2 ´ 1030 kg). [NCERT]
[NCERT Exemplar]
(a) Yes (a) 4 ´ 1030 kg (b) 5 ´ 1030 kg
(b) No
(c) 2 ´ 1030 kg (d) 3 ´ 1030 kg
(c) Sometimes yes and sometimes no
(d) Can not be said 16. A rocket of mass M is launched vertically from the
12. Two identical thin rings each of radius R are surface of the earth with an initial speed v. Assuming
coaxially placed at a distance R. If the rings have a the radius of the earth to be R and negligible air
uniform mass distribution and each has mass m1 and resistance. The maximum height attained by the
m2 respectively, then the work done in moving a rocket above the surface of the earth.
mass m from centre of one ring to that of the other is æ gR ö æ gR ö
(a) R ç 2 - 1÷ (b) R ç 2 - 1÷
è 2 v ø è 2 v ø
R R æ 2 gR ö æ 2 gR ö
m1 (c) R ç 2 - 1÷ (d) R ç 2 - 1÷
Y Y
m2 è v ø è v ø
17. Two satellites S1 and S2 revolve around a planet in
R coplanar circular orbits in the same sense. Their
periods of revolution are 1 h and 8 h respectively. The
Gmm1 ( 2 + 1) Gm ( m1 - m2 ) ( 2 - 1) radius of orbit of S1 is 104 km. When S2 is closest to S1,
(a) (b) the speed of S2 relative to S1 is
m2 R 2R
(a) p ´ 10 4 kmh–1 (b) 2 p ´ 10 4 kmh–1
Gm 2 ( m1 + m2 )
(c) (d) zero (c) 3 p ´ 10 4 kmh–1 (d) 4 p ´ 10 4 kmh–1
R
13. Distance between the centres of two stars is 10 a. The 18. In the previous question, the angular speed of S2 as
masses of these stars are M and 16 M and their radii actually observed by an astronaut in S1
a and 2a respectively. A body of mass m is fired p
(a) radh-1 (b) p radh-1
straight from the surface of the larger star towards 2
the smaller star. The minimum initial speed for the 2p p
(c) radh-1 (d) radh-1
body to reach the surface of smaller star is 3 3
Telegram @unacademyplusdiscounts

Gravitation 375

19. A straight rod of length L extends from x = a to 26. The potential energy of gravitational interaction of a
x = L + a. Find the gravitational force it, exerts on a point mass m and a thin uniform rod of mass M and
point mass m at x = 0 if the linear density of rod length l, if they are located along a straight line at
m = A + Bx2 distance a from each other is
éA ù é æ1 1 ö ù GMm æa + lö
(a) Gm ê + BLú (b) Gm ê A ç - ÷ + BLú (a) U = log e ç ÷
ëa û ë è a a+ L ø û a è a ø
é A ù é Aù æ1 1 ö
(c) Gm ê BL + (d) Gm ê BL - ú (b) U = GMm ç - ÷
ë a + L úû ë aû èa a + lø
GMm æa + lö
20. If a planet was suddenly stopped in its orbit, k (c) U = log e ç
è a
÷
ø
l
suppose to be circular, find how much time will it
GMm
take in falling onto the sun? (d) U = -
a
(a) 2 / 8 times the period of the planet’s revolution
(b) 4 2 times the period of the planet’s revolution 27. A thin uniform annular disc (see
P
figure) of mass M has outer radius
(c) 3 2 times the period of the planet’s revolution 4R
4 R and inner radius 3 R. The
(d) 9 times the period of the planet’s revolution
work required to take a unit mass 3R
21. An object weighs 10 N at the north-pole of the earth. from point P on its axis to infinity 4R
In a geostationary satellite distant 7R from the is
centre of earth (of radius R), what will be its true
weight? 2 GM 2 GM
(a) (4 2 - 5) (b) - (4 2 - 5)
(a) 3 N (b) 5 N 7R 7R
(c) 2 N (d) 0.2 N GM 2 GM
(c) (d) ( 2 - 1)
4R 5R
22. In the above question, find apparent weight of the
object? 28. How much energy will be necessary for making a
(a) 3 N (b) Zero (c) 2 N (d) 0.2 N body of 500 kg escape from the earth?
23. Two equal mases m and m (g = 9.8 ms–2, radius of earth = 6.4 ´ 106 m)
are hung from balance (a) About 9.8 × 106 J (b) About 6.4 × 108 J
whose scale pans differ in m (c) About 3.1 × 1010 J (d) About 27.4 × 1012 J
vertical height by h. h 29. Two identical satellites are at R and 7R away from
Calculate the error in m earth surface, the wrong statement is
weighing, if any, in terms of (R = Radius of earth)
density of earth r. (a) ratio of total energy will be y
2 8 (b) ratio of kinetic energies will be y
(a) prR3Gm (b) prGmh
3 3 (c) ratio of potential energies will be y
8 4 (d) ratio of total energy will be 4 but ratio of potential and
(c) prR3Gm (d) prGm2 h
3 3 kinetic energy will be 1
24. If satellite is revolving around a planet of mass M in 30. A particle is projected vertically upwards from the
an elliptical orbit of semi-major axis a, find the surface of earth (radius Re ) with a kinetic energy
orbital speed of the satellite when it is at a distance r equal to half of the minimum value needed for it to
from the focus. escape. The height to which it rises above the surface
é2 1ù é 2 1ù of earth is
(a) v2 = GM ê - ú (b) v2 = GM ê 2 - ú
ë r aû ër aû (a) Re (b) 2 Re (c) 3 Re (d) 4 Re
é 2 1ù é 2 1 ù
(c) v2 = GM ê 2 - 2 ú (d) v2 = G ê - ú 31. A point P ( R 3 , 0, 0) lies on the axis of a ring of mass
ër a û ë r aû M and radius R. The ring is located in y-z plane with
25. The gravitational force between a point like mass M its centre at origin O. A small particle of mass m
and an infinitely long, thin rod of linear mass density starts from P and reaches O under gravitational
perpendicular to distance L from M is attraction only. Its speed at O will be
MGl 1 MGl GM Gm
(a) (b) (a) (b)
L 2 L R R
2 MGl GM Gm
(c) (d) infinite (c) (d)
L2 2R 2R
Telegram @unacademyplusdiscounts

376 JEE Main Physics

32. A spaceship is stationed on mars. How much energy 37. The magnitudes of the gravitational field at distance
must be expended on the spaceship to rocket it out of r1 and r2 from the centre of a uniform sphere of radius
the solar system? Mass of the spaceship = 1000 kg; R and mass M are r1 and r2 respectively. Then,
mass of sun = 2 ´ 1030 kg; mass of mars F1 r1
(a) = for r1 < R and r1 = R
= 6.4 ´ 1023 kg; radius of mars = 3395 km; radius of F2 r2
orbit of mars = 228 . ´ 108 km, G = 6.67 ´ 10-11 F1 r22
2 2 (b) = 2 for r1 > R and r1 > R
N-m /kg . [NCERT]
F2 r1
(a) 3 ´ 1011 J (b) 4 ´ 1011 J
F r
(c) 33 ´ 1011 J (d) None of these (c) 1 = 1 for r1 > R and r2 > R
F2 r2
F1 r12
More Than One Correct Option (d)
F2
= 2 for r1 < R and r2 < R
r2
33. A planet of mass m is revolving round the sun
(of mass ms ) in an elliptical orbit. If v is the velocity 38. A body is imparted a velocity v from surface of the
of the planet when its position vector from sun r then earth. If v0 is orbital velocity and ve be the escape
if the planet rotates in counter clockwise direction velocity, then for
then areal velocity has direction (a) v = v 0 , the body follows a circular track around the earth
(a) given by “Right Hand Thumb Rule” (b) v > v 0 , but < v e, the body follows elliptical path and
returns to surface of earth
(b) given by “Left Hand Thumb Rule”
(c) v < v 0 , the body follows elliptical path and returns to
(c) normal to the plane of orbit upwrads surface of earth
(d) normal to the plane of orbit downwards (d) v > v e, the body follows hyperbolic path and escapes the
34. If the law of gravitation, instead of being gravitational pull of the earth
inverse-square law, becomes an inverse-cube law 39. If the mass of sun were ten times smaller and
[NCERT Exemplar] gravitational constant G were ten times larger in
(a) planets will not have elliptic orbits magnitudes [NCERT Exemplar]
(b) circular orbits of planets is not possible (a) walking on ground would became more difficult
(c) projectile motion of a stone thrown by hand on the (b) the acceleration due to gravity on earth will not change
surface of the earth will be approximately parabolic (c) raindrops will fall much faster
(d) there will be no gravitational force inside a spherical shell (d) airplanes will have to travel much faster
of uniform density
40. If the sun and the planets carried huge amounts of
35. There have been suggestions that the value of the
opposite charges [NCERT Exemplar]
gravitational constant G becomes smaller when
(a) all three of Kepler’s laws would still be valid
considered over very large time period (in billions of
(b) only the third law will be valid
years) in the future. If that happens, for our earth, (c) the second law will not change
[NCERT Exemplar] (d) the first law will still be valid
(a) nothing will change
(b) we will become hotter after billions of years
(c) we will be going around but not strictly in closed orbits Comprehension Based Questions
(d) after sufficiently long time we will leave the solar system Passage I
36. Supposing Newton’s law of gravitation for A rocket is fired vertically upwards with a speed of
gravitation forces F1 and F2 between two masses v ( = 5 kms –1) from the surface of earth. It goes up to a
m1 and m2 at positions r1 and r2 read height h before returning to earth. At height h a body
n
r12 2 æ m1m2 ö is thrown from the rocket with speed v0 in such a way
F1 = - F2 = - 3 GM 0 ç ÷
r12 è M 20 ø so that the body becomes a satellite of earth. Let the
mass of the earth, M = 6 ´ 1024 kg, mean radius of the
where M 0 is a constant of dimension of mass,
earth, R = 6.4 ´ 106 m
r12 = r1 - r2 and n is a number. In such a case
(G = 6.6 ´ 10–11 Nm 2 kg –2 , g = 9.8 ms –2 )
[NCERT Exemplar]
(a) the acceleration due to gravity on earth will be different 41. The value of h is
for different objects (a) 1.5 × 105 m (b) 3.2 × 105 m
(b) none of the three laws of Kepler will be valid (c) 3.2 × 106 m (d) 1.6 × 106 m
(c) only the third law will become invalid 42. The value of v0 (i.e., orbital velocity of satellite is)
(d) for n negative, an object lighter than water will sink in (a) 6.7 kms–1 (b) 7.1 kms–1
water (c) 7.8 kms–1 (d) 8.2 kms–1
Telegram @unacademyplusdiscounts

Gravitation 377

43. The energy to be spent in taking the satellite out of Assertion and Reason
the gravitational field of the earth is (mass of the
satellite is 200 kg) Directions Question No. 48 to 56 are Assertion-Reason type.
(a) 5.0 × 109
J (b) 10.0 × J 109 Each of these contains two Statements : Statement I (Assertion),
(c) 2.5 × 1010 J (c) 5.0 × 1010 J Statement II (Reason). Each of these questions also has four
alternative choices, only one of which is correct. You have to
44. Time period of revolution of satellite around the select the correct choices from the codes (a), (b), (c) and (d) given
earth is below
(a) 3550 s (b) 7100 s (a) If both Assertion and Reason are true and the
(c) 5330 s (d) None of these Reason is correct explanation of the Assertion
(b) If both Assertion and Reason are true but Reason is
Passage II not correct explanation of the Assertion
A spaceship is in a circular orbit of radius r0 around a (c) If Assertion is true but Reason is false
star of mass M. The spaceship’s rocket engine can (d) If Assertion is false but the Reason is true
alter its velocity (instantaneously) by an Dv. Amounts 48. Assertion Gravitational force between two particles
direction of firing is measured by angle q between the is negligibly small compared to the electrical force.
ship’s velocity v and the vector from the tail to the Reason The electrical force is experienced by
nose of the ship. To conserve fuel in a sequence of N charged particles only.
N
fringes, it is desirable to minimise Dv = å |Dv i|, D v 49. Assertion There is no effect of rotation of earth on
acceleration due to gravity at poles.
i =1
is known as the specific impulse. We want to use the Reason Rotation of earth is about polar axis.
ship’s engine to cause it to crash into the star (assume 50. Assertion Generally the path of a projectile from the
the radius of the star to be negligible). earth is parabolic but it is elliptical for projectiles
v going to a very heigh.
Reason Upto ordinary height the projectile moves
θ
v under a uniform gravitional force, but for great
heights, projectile moves under a variable force.
51. Assertion When distance between two bodies is
doubled and mass of each body is also doubled.
45. What is the minimum specific impulse required to
Gravitational force between them remains the same.
escape from the star if the engine is fired in a single
Reason According to Newton’s law of gravitation,
rapid burst?
force is directly proportional to mass of bodies and
GM GM2 inversely proportional to distance between them.
(a) (b) ( 2 - 1)
r0 r0
52. Assertion A body becomes weightless at the centre of
GM GM earth.
(c) 2 (d) ( 2 - 1)
r0 r0 Reason As the distance from centre of earth
decreases, acceleration due to gravity increases.
46. In order to visit a planet in a circular orbit of radius
r1 > r0 , what is the minimum specific impulse 53. Assertion The binding energy of a satellite does not
required to reach the planet’s orbit if the engine is depend upon the mass of the satellite.
again fired in a single rapid burst ? Reason Binding energy is the negative value of total
energy of satellite.
GM é 2 r0 ù GM é 2 r0 ù
(a) ê3 - 2 ú (b) ê ú 54. Assertion If earth suddenly stops rotating about its
r1 êë r1 úû r1 ë r1 û
axis then the value of acceleration due to gravity will
GM
(c) Zero (d) become same at all the places.
r1
Reason The value of acceleration due to gravity is
47. If we want to use the ship’s engine to cause it to crash independent of rotation of earth.
into the star than find the minimum specific impulse 55. Assertion The speed of revolution of an artificial
for a firing strategy i.e., a single rapid burst at q = 180° satellite revolving very near the earth is 8 kms–1.
2
GM GM GM Reason Orbital velocity of a satellite, become
(a) (b) GM (c) (d)
r0 r20 r0 independant of height of satellite.
Telegram @unacademyplusdiscounts

378 JEE Main Physics

56. Assertion Two satellites are following one another in Reason The energy of earth satellites system in
the same circular orbit. If one satallite tries to catch GMm
circular orbit is given by E = - , where r is the
another (leading one) satellite, then it can be done by 2r
increasing its speed without changing the orbit. radius of the circular orbit.

Previous Years’ Questions


57. The height at which the acceleration due to gravity 65. If g 0 , g h and g d be the acceleration due to gravity at
becomes g/9 (where g is acceleration due to gravity on earth’s surface, at height h and at a depth d
the surface of the earth) in terms of R, (the radius of respectively, then [Orissa JEE 2008]
the earth) is [AIEEE 2009] (a) g 0 < g h and g 0 > g d (b) g 0 < g h and g 0 < g d
(a) R / 2 (b) R / 2 (c) 2 R (d) 2 R (c) g 0 > g h and g 0 > g d (d) g 0 < g h and g 0 > g d

58. The effect of rotation of the earth on the value of 66. The time period of a geostationary satellite at a
acceleration due to gravity is [UP SEE 2009] height 36000 km is 24 h. A spy satellite orbits very
(a) g is maximum at the equator and minimum at the poles close to earth surface (R = 6400 km). What will be its
(b) g is minimum at the equator and maximum at the poles time period? [Orissa JEE 2008]

(c) g is maximum at both places (a) 4 h (b) 1 h (c) 2 h (d) 1.5 h


(d) g is minimum at both places 67. Infinite number of masses, each of 1 kg are
59. The escape velocity from the earth is 11 kms–1.The placed along the x-axis at x = +1 m, ± 2 m1, ±4 m,
± 8 m, ± 16 m… The magnitude of the resultant
escape velocity from a planet having twice the radius
gravitational potential in terms of gravitational
and same mean density as that of earth is [UP SEE 2009] constant G at the origin ( x = 0) is [Kerala CET 2008]
(a) 5.5 kms–1 (b) 11 kms–1 (a) G/2 (b) G (c) 2 G (d) 4 G
(c) 22 kms–1 (d) None of these
68. A planet in a distant solar system is 10 times more
60. If g is the acceleration due to gravity on earth’s massive than the earth and its radius is 10 times
surface, the gain of the potential energy of an object smaller. Given that the escape velocity from the
of mass m raised from the surface of the earth to a earth is 11 kms–1, the escape velocity from the
height equal to the radius R of the earth is [UP SEE 2008]
surface of the planet would be [AIEEE 2008]
1 1
(a) 2mgR (b) mgR (c) mgR (d) mgR (a) 0.11 kms–1 (b) 1.1 kms–1
2 4 (c) 11 kms–1 (d) 110 kms–1
61. If the distance between the sun and the earth is 69. The escape velocity of a body on the surface of earth is
increased by three times, then attraction between 11.2 kms–1. If the mass of the earth is doubled and its
two will [BVP Engg. 2008] radius halved, the escape velocity becomes
(a) remain constant (b) decrease by 63% [Kerala CET 2007]
(c) increase by 63% (d) decrease by 89% (a) 5.6 kms–1 (b) 11.2 kms–1
62. The orbit of geostationary satellite is circular, the (c) 22.4 kms–1 (d) 44.8 kms–1
time period of satellite depends on (i) mass of the 70. The change in potential energy when a body of mass
satellite, (ii) mass of the earth, (iii) radius of the orbit, m is raised to a height nR from earth’s surface is
(iv) height of the satellite form the surface of the (R = radius of the earth) [Kerala CET 2007]
earth. [EAMCET 2008] n
(a) mgR (b) mgR
(a) (i) only (b) (i) and (ii) ( n - 1)
(c) (i), (ii) and (iii) (d) (ii), (iii) and (iv) n n2
(c) mgR (d) mgR 2
63. The weight of a body on surface of earth is 121.6 N. ( n + 1) ( n + 1)
When it is raised to a height half the radius of earth, 71. If g E and g M are the acceleration due to gravity on
its weight will be [WB JEE 2008] the surfaces of the earth and the moon respectively
(a) 2.8 N (b) 5.6 N (c) 12.6 N (d) 25.2 N and if Millikan’s oil drop experiment could be
64. If the earth shrinks such that its mass does not performed on two surfaces, one will find the ratio
change but radius decreases to one-quarter of its electronic charge on the moon
to be
original value, then one complete day will take electronic charge on the earth [AIEEE 2007]
[WB JEE 2008] (a) g m / g E (b) 1 (c) 0 (d) g E / g M
(a) 96 h (b) 48 h (c) 6 h (d) 1.5 h
Telegram @unacademyplusdiscounts

Gravitation 379

72. The escape velocity on the surface of earth is 77. The escape velocity for the earth is ves (s). The escape
11.2 kms–1. If mass and radius of a planet is 4 and 2 velocity for a planet whose radius is four times and
times respectively, than that of earth, the escape density is nine times that of the earth is [BVP Engg. 2005]
velocity on the planet [BVP Engg. 2006] (a) 36 v es( e) (b) 12 v es( e) (c) 6 v es( e) (d) 20 v es( e)
(a) 11.2 kms–1 (b) 1.12 kms–1 78. Two bodies of masses m and 4 m are placed at a
(c) 22.4 kms–1 (d) 15.8 kms–1 distance r. The gravitational potential at a point on
73. A simple pendulum has a time period T1 when on the the line joining, then where the gravitational field is
earth’s surface and T2 when taken to a height 2 R zero, is [AIEEE 2011]
above the earth’s surface when R is 2 R above earth’s - 4GM - 6 GM -9 GM
(a) (b) (c) (d) zero
surface where R is the radius of the earth. The value r r r
of ( T1/ T2 ) is [Kerala CET 2006] 79. The mass of spaceship is 1000 kg. It is to be launched
(a) 1/9 (b) 1/3 (c) 3 (d) 9 from the earth’s surface out into free space. The value
74. The mass of a planet is six times that of the earth. of g and R. (radius of earth) are 10 m/s2 and 6400 km
The radius of the planet is twice that of the earth. If respectively. The required energy for this work will
the escape velocity from the earth is v, then the be [AIEEE 2012]
escape velocity from the planet is [Kerala CET 2006] (a) 6.4 ´ 1011 J (b) 6.4 ´ 10 8 J
(a) 3 v e (b) 2 v e (c) v e (d) 5 v e (c) 6.4 ´ 10 9 J (d) 6.4 ´ 1010 J

75. A satellite in a circular orbit of radius R has a period 80. If ve and vo represent the escape velocity and orbital
of 4h. Another satellite with orbital radius 3R around velocity of satellite corresponding to a circular orbit
the same planet will have a period (in hours) of radius R, then [Orissa JEE 2011]

[Karnataka CET 2006)] (a) v e = v o (b) 2 v o = v e


(a) 16 (b) 4 (c) 4 27 (d) 4 8 v
(c) v e = o (d) v e and v o are not related
2
76. If earth suddenly shrinks by one-third of its present
radius, the acceleration due to gravity will be 81. What is the minimum energy required to launch a
[Manipal 2005] satellite of mass m from the surface of a planet of
2 3 mass M and radius R in a circular orbit at an altitude
(a) g (b) g of 2 R ? [JEE Main 2013]
3 2
4 9 5 GmM 2 GmM GmM GmM
(c) g (d) g (a) (b) (c) (d)
9 4 6R 3R 2R 3R

Answers
Round I
1. (d) 2. (b) 3. (d) 4. (c) 5. (a) 6. (b) 7. (a) 8. (b) 9. (b) 10. (b)
11. (d) 12. (c) 13. (d) 14. (c) 15. (b) 16. (a) 17. (d) 18. (b) 19. (b) 20. (c)
21. (d) 22. (a) 23. (c) 24. (c) 25. (b) 26. (b) 27. (c) 28. (c) 29. (b) 30. (d)
35. (c) 36. (a)
31. (a) 37. (c)
32. (c) 38. (d)
33. 39. (b) 40. (c)
41. (a) 42. (a) 43. (c) 44. (c) 45. (b) 46. (c) 47. (a) 48. (a) 49. (c) 50. (c)
51. (c) 52. (c) 53. (c) 54. (b) 55. (d) 56. (a) 57. (c) 58. (c) 59. (c) 60. (b)
61. (d) 62. (c) 63. (b) 64. (c) 65. (b) 66. (b) 67. (a) 68. (a) 69. (c) 70. (c)
71. (d) 72. (b) 73. (b) 74. (a) 75. (b) 76. (a) 77. (b) 78. (c) 79. (c) 80. (d)
81. (a) 82. (d) 83. (a) 84. (b) 85. (c) 86. (c) 87. (b) 88. (c) 89. (d) 90. (d)

Round II
1. (c) 2. (c) 3. (d) 4. (c) 5. (c) 6. (c) 7. (b) 8. (b) 9. (b) 10. (a)
11. (a) 12. (b) 13. (b) 14. (b) 15. (c) 16. (c) 17. (a) 18. (d) 19. (b) 20. (a)
21. (d) 22. (b) 23. (b) 24. (a) 25. (c) 26. (c) 27. (a) 28. (c) 29. (d) 30. (a)
31. (a) 32. (a) 33. (a,c) 34. (a,b,c) 35. (c,d) 36. (a,c,d) 37. (a,b) 38. (a,b,c,d) 39. (a,c,d) 40. (a,c,d)
41. (d) 42. (b) 43. (a) 44. (b) 45. (d) 46. (a) 47. (a) 48. (b) 49. (a) 50. (c)
51. (a) 52. (c) 53. (d) 54. (c) 55. (a) 56. (d) 57. (d) 58. (a) 59. (c) 60. (c)
61. (d) 62. (d) 63. (b) 64. (d) 65. (c) 66. (d) 67. (d) 68. (d) 69. (c) 70. (c)
71. (b) 72. (d) 73. (b) 74. (a) 75. (c) 76. (d) 77. (b) 78. (c) 79. (d) 80. (b)
81. (a)
Telegram @unacademyplusdiscounts

the Guidance
Round I
G m1m2 G m1m2 GMm G 8M ´ m
1. F = = i. e. ,F µ r -2 New force, F¢ = = = 2F
(r + 2 r) 2 9 r2 R2 (2 R) 2
4
Note that F µ r 4 by taking m = pr 4r and then i. e. , force of attraction increases due to the increase in mass of
3 the planet.
r3r3 4
F µ 2 ,i. e. ,F µ r 6. Moon is revolving around earth in almost circular orbit. Sun
r
exerts gravitational pull on both, earth and moon. When
is not correct because the gravitational law obeys inverse
observed from sun, the orbit of the moon will not be strictly
square law and is not related with densities.
elliptical because the total gravitational force (i. e. , force due
GMm to earth on moon and force due to sun on moon) is not
2. Gravitational force due to solid sphere, F1 = , where M
(2R) 2 central.
and m are masses of the solid sphere and particle respectively
and R is the radius of the sphere. The gravitational force on
7. The earth is revolving on circular orbit around sun due to
gravitational force (F) which acts along the radius of circular
particle due to sphere with cavity = force due to solid sphere
path, towards the sun i. e. , angle between r and F is zero. As
creating cavity, assumed to be present above at that position.
GMm G (M / 8) m 7 GMm Torque = | t | = | r ´ F| = r F sin 0° = 0
i. e. , F2 = - =
4 R2 (3R / 2) 2 36 R 2 Therefore, torque is zero.

F2 7 GMm æ GMm ö 7 8. Here, m1 = m2 = 100 kg,r = 100 m


So, = ç ÷=
F1 36 R 2 è 4 R2 ø 9 Acceleration of first astronaut,
Gm1m2 1 Gm1
3. From the figure, the gravitational a1 = ´ = 2
r2 m1 r
intensity due to the ring at a distance M R
d = 3 R on its axis is Acceleration of second astronaut,
Gm1m2 1 Gm2
GM a2 = ´ = 2
I= 2 d = v3 R r2 m2 r
(d + R 2)3 / 2
Net acceleration of approach
GM ´ 3 R 3 GM
= = Gm Gm 2 Gm1
(3R 2 + R 2)3 / 2 8 R2 a = a1 + a2 = 2 2 + 2 1 =
8M
r r r2
Force on sphere –11
2 ´ (6.67 ´ 10 ) ´ 100
3M 3 GM 2 =
= (8 M) I = (8 M) ´ = (100) 2
2
8R R2 = 2 ´ 6.67 ´ 10 –13 ms–2
æ GMm ö 1
4. Gravitational force = ç 3 / 2 ÷ provides the necessary As s = at 2
è R ø
2
2
centripetal force (i. e. , mRw ) 1/ 2 1/ 2
æ2 sö é 2 ´ (1 / 100) ù
GMm
2
æ2 p ö 4 p 2mR \ t =ç ÷ =ê ú second
So, = mRw2 = mR ç ÷ = è aø ë 2 ´ 6.67 ´ 10 –13 û
R3/ 2 è T ø T2
On solving we get t = 1.41days.
4 p 2R3 / 4
or T2 = ,
GM 9. (i) Gravitational force on the particle placed at mid-point D of
i. e. , T 2 µ R3 / 2 side BC of length a is
F = F1 + F2 + F3
5. Let R be the original radius of a planet. Then attraction on a Here, F2 = -F3
A
M
body of mass m placed on its surface will be
Þ F2 + F3 = 0
GMm
F= \ F = F1 + 0 = F1
R2
GMM
If size of the planet is made double, i. e. , R ¢ = 2 R, then mass of or F = F1 =
the planet becomes ( AD) 2
F1
4 3 GM 2 4 GM 2
M ¢ = p (2 R)3r = 8 ´ pR 2r = 8 M = 2
=
3 4 (3a / 4) 3 a2 B
F2 D F3
C
M
Telegram @unacademyplusdiscounts

Gravitation 381

(ii) Gravitational force on the particle placed at the point O, 13. A person is safe, if his velocity while reaching the surface of
i. e. , the intersection of three medians is moon from a height h ¢ is equal to its velocity while falling from
F = F1 + F2 + F3 = 0 height h on earth. So
A 2 g ¢h ¢ = 2 gh
M
gh 3
or h¢ = = 9.8 ´ = 15 m
g¢ 1.96
F1 g m G (M / 8) R2
14. As, = 2
= 32 …(i)
g e GM / Re 8 Rm
O
mg m 1
F2 F3 Given, =
B
D
C mg e 6
M M
gm 1
or F =0 \ = …(ii)
ge 6
Since, the resultant of F2 and F3 is equal and opposite to F1.
From Eqs. (i) and (ii), we get,
Gmm
10. F = 21 2, thus on increasing masses and reducing distance Re2 1
r =
2
r, force of gravitational attraction F will increases. 8 Rm 6

11. Gravitational force between sphere of mass M and the particle 8


or Re = Rm
of mass m at B is 6
G Mm GM G 4 3 4
F1 = 15. g = = pR r = pGrR, i. e. , g µ R
d2 R2 R2 3 3
If M1 is the mass of the removed part of sphere, then
æ ö 16. When the thief with box on his head jumped down from a
4 æRö
3
1æ4 M ç M ÷ wall, he along with box is falling down with acceleration due
2 ö
M1 = p ç ÷ r = ç pR r ÷ = çQ r = ÷ to gravity, so the apparent weight of box becomes zero,
3 è2ø 8 è3 ø 8 ç 4 3÷
pR
è 3 ø (because, R = mg - mg = 0), so he experiences no load till he
reaches the ground.
d
17. Weight of the body at earth’s surface
w = mg = 250 N …(i)
Acceleration due to gravity at depth h from earth’s surface
O A æ hö
g ¢ = g ç1 - ÷
è Re ø
Here, h = Re / 2
Gravitational force between the removed part and the particle æ R / 2ö
\ g ¢ = g ç1 - e ÷
of mass m at B is è Re ø
GM1m G (M / 8) m GMm
F2 = = = æ 1ö
(d - R / 2) 2 (d - R / 2) 2 8 (d - R / 2) 2 = g ç1 - ÷
è 2ø
\ Required force, g
GMm GMm g¢ =
F = F1 - F2 = - 2
d2 8 [d - (R / 2)]2
\ Weight of the body at depth h
é mg
GMm ê 1 ù w ¢ = mg =
= 1- ú 2
2 ê 2
d æ R ö ú Using Eq. (i), we get
ê 8 ç1 - ÷ ú
ê è 2d ø ú 250
ë û w¢ = = 125 N
2
12. Just before striking, the distance between the centre of earth
\ Weight of the body will be 125 N
and moon is,
R 5 Re 18. Here to point 7 of Problem Solving skills
r = Re + e =
4 4 h1 g 2
=
So, acceleration of moon at this moment is h2 g 1
GMe 16 hg 0.5 ´ g
a= 2
= ´ 10 = 6.4 ms–2 or h2 = 1 1 = = 3.0
(5Re / 4) 25 g2 g /6
Telegram @unacademyplusdiscounts

382 JEE Main Physics

Energy spent = mg ehe = mg mhm gR 2 æ hö


24. As, = g ç1 - ÷
or hm = g ehe / g m …(i) (R + h) 2 è Rø
éæ 4 3 2ö ù éæ 4 3 2ö ù æ h ö æ h2 2 h ö
ê çèG 3 pRe r / Re ÷ø he ú ê çèG pRe r / Re ÷ø he ú or ç1 - ÷ ç1 + 2 + ÷ =1
3 è Rø è R R ø
=ê ú=ê ú
ê æ G 4 pR 2 r / R 2 ö ú ê G 4 pR 2 r / R 2 ú
ç
êë è 3 m m m ú ÷ m m m ú h3 h 2 h
ø û êë 3 û or + - =0
R3 R 2 R
Re r e 3 4
= ´ ´ he = ´ ´ 0.5 = 3 m h æ h2 h ö
Rm r m 2 1 or ç + - 1÷ = 0
R è R2 R ø
ge Re r e 2 4
19. As, = = ´ =6 h -1 ± 1 + 4 5 -1 5 R -R
g m Rm r m 3 1 or = = or h =
R 2 2 2
ge
or gm = 25. Below the sea level the pressure is increasing with depth in
6
mine due to presence of atmospheric air there. The
For motion on earth, using the relation, acceleration due to gravity below the surface of the earth
1 decreases with the distance from the surface of the earth, as
s = ut + at 2
2 æ dö
1 1 1 g ¢ = g ç1 - ÷ .
We have, = 0 + ´ 9.8 t 2 or t = s è Rø
2 2 9.8 4
1 æ 9.8 ö 2 6 G ´ pR3r
GM 3 4
For motion on moon, 3 = 0 + ç ÷ t1 or t1 = s 26. As, g = 2 = = pGrR, i. e. , g µ R
2è 6 ø 9.8 R R 2
3
t1 For pendulum clock, g will increase on the planet, so time
\ = 6 or t1 = 6 t
t period will decrease. But for spring clock, it will not change.
Hence, P will run faster than S.
20. Resultant force on mass m due to masses at A and B is GM GM GM (100) 2
G2M ´ m GMm 27. As g = 2 Þ g ¢ = 2 =
F= - towards BA. Therefore, m will move R R¢ (99) 2R 2
( AB) 2 (BC) 2
( g ¢ - g ) ´ 100
towards 2M. % Increase in g =
g
21. If the earth is an approximate sphere of non-uniform density, æg¢ ö é æ100 ö 2 ù
then the centre of gravity of earth will not be situated at the = ç - 1÷ ´ 100 = ê ç ÷ - 1ú ´ 100
centre of earth. The distance of different points on earth will
èg ø êë è 99 ø úû
be at different distances from the centre of gravity of earth. As, éæ 1ö
2 ù
GM 1 ê ç1 + ÷ - 1ú ´ 100 » 2%
g = 2 or g µ 2, so g is different on different points on the êë è 99 ø úû
r r
surface of earth but never zero. 28. Mass of two planets is same, so
22. Gravitational potential at mid-point 4 3 4
pR1r1 = pR32 r 2
- GM1 - GM2 3 3
V= + 1/3
d /2 d /2 R1 æ r 2 ö æ 1ö
1/3
1
2 GM or =ç ÷ =ç ÷ =
Now, PE = M ´ V = - (M1 + M2) R2 è r1 ø è8ø 2
d 2
(M = mass of particle) g 1 GM / R12 æ R2 ö
= = ç ÷ = (2) 2 = 4
So, the projecting particle from mid-point to infinity g 2 GM / R22 è R1 ø
KE =|PE|
1 2 GM 29. The value of acceleration due to gravity at a height h reduces
Þ mv 2 = (M1 + M2) 64
2 d to = 100 - 36 = 64% = g
100
G (M1 + M2)
Þ v =2 64 gR 2
d \ g =
100 (R + h) 2
23. When a sphere of mass m is released in a liquid, it falls 8 R R
mg - FB or = or h =
vertically down with acceleration = 10 R + h 4
m
4 3 4 30. As, value of g at internal point which is at a distance xfrom the
pr dg - pr 3rg centre is given by
3 3 (d - r) g
= = GM
4 2 d g¢ = x,
pr d R3
3
when x = 0 then g ¢ = 0
Telegram @unacademyplusdiscounts

Gravitation 383

31. Below the surface of the earth and above the surface of earth 38. The acceleration due to gravity at a depth d inside the earth is
1 æ dö æR -d ö r
g µ . Therefore, the graph (a) is correct. g ¢ = g ç1 - ÷ = g ç ÷=g
r2 è Rø è R ø R
32. Force acting on a body of mass M at a point at depth d inside where, R - d = r = distance of a place from the centre of earth.
the earth is Therefore, g ¢ µ r
æ dö GM
F = mg ¢ = mg ç1 - ÷
è Rø 39. As, g = 2
R
mGM æ R - d ö GMm
= ç ÷= r (Q R - d = r) GM 7.34 ´ 10 22
R2 è R ø R3 or R = = 6.67 ´ 10 –11 ´ = 1.87 ´ 10 6 m
g 1.4
So, F µ r; Given F µ r n
Clearly, n =1 40. Acceleration due to gravity at height h,
g¢ 2 h 90 æ 2hö
33. At height h ¢, = 1 - = g 1 = g ç1 - ÷
è R ø
g R 100
2h 90 10 1 Acceleration due to gravity at depth h,
or = 1- = = æ hö
R 100 100 10 g 1 = g ç1 - ÷
è Rø
or R = 20 h = 20 ´ 320 = 6400 km
-1
g¢ d 95 g 1 1 - 2h / R æ 2 h ö æ hö æ hö
At depth d , = 1- = \ = = ç1 - ÷ ç1 - ÷ = ç1 - ÷
g R 100 g 2 1- h / R è R ø è Rø è Rø
d 95 5 1 g1
or = 1- = = \ decreases linearly with h.
R 100 100 20 g2
R 6400
or d= = = 320 km 41. At a height h, (Taking h << R) from the surface of earth
20 20
æ 2hö
gR 2 20 ´ 20 g h = g ç1 - ÷
34. Given, gh = 9 = = g è R ø
(R + R / 20) 2 21 ´ 21 gh 2 h 90
or = 1- =
9 ´ 21 ´ 21 g R 100
or g =
20 ´ 20 2h 99 1
æ dö or = 1- =
Now, g d = g ç1 - ÷ R 100 100
è Rø
R 6400
9 ´ 21 ´ 21 æ R / 20 ö or h= = = 32 km
= ç1 - ÷ = 9.5 ms
–2 100 200
20 ´ 20 è R ø mg ¢ 30 g¢ 1
42. Given, = or =
g 2h 2 ´ 320 1 9
35. As, ¢ = 1- = 1- = 1- = mg 90 g 3
g R 6400 10 10
R2
æ g - g ¢ö 1 Now, g¢ = g
\ % decrease in g = ç ÷ ´100 = ´ 100 = 10% (R + h) 2
è g ø 100
g¢ R2 1
=
or At depth d from the surface
36. = earth
of the g (R + h) 2 3
æ dö
g ¢ = g ç1 - ÷ R 1
è Rø or =
75 3 R+h 3
Given, g¢ = g = g
100 4 or (R + h) = 3 R
3g æ dö
Then, = g ç1 - ÷ or h = ( 3 - 1) R = 0.73 R
4 è Rø
R 43. Error in weight = difference in weight at two different heights
On solving, we get d =
4 æ 2h ö æ 2h ö
= mg ç1 - 1 ÷ - mg ç1 - 2 ÷
æ 2h ö è R ø è R ø
37. Error in weighing = mg - mg ¢ = mg - mg ç1 - ÷
è Rø
2 mg 2m GM h
mg 2h m2 hg = (h2 - h1) = ´ 2 ´
= = R R R 2
R R
4 [where, h2 - h1 = h /2]
G pR 2r
m 2h 8 p Grmh 2m 4 h 4
= ´ 3 2 = = 3 ´ G ´ pR 2r ´ = pGmrh
R R 3 R 3 2 3
Telegram @unacademyplusdiscounts

384 JEE Main Physics

44. For the condition of weightlessness at equator 53. Gravitational potential at a point outside the sphere
g - GM
w= Vg = . But Vg is same at a point inside the hollow sphere
R r
as on the surface of sphere.
10 1
\ w= = rad/s 54. Intensity of gravitational field at a point inside the spherical
6400 ´ 10 3 800
shell is zero and outside the shell is I µ1 / r 2.
2
45. Hence, g ¢ = g - Rw = 0
55. The gravitational intensity at a point inside the spherical shell
10 1 is zero.
w = g /R = 3
= rad/s
(6400 ´ 10 ) 800 GM GMm
56. As, 2 e =
46. When there is a weightlessness in the body at the equator, x (r - x ) 2
then g ¢ = r - Rw2 = 0
r-x Mm 7.35 ´ 10 22
or w = g / R and or = =
x Me 5.98 ´ 10 24
linear velocity or r = 0.11 x + x = 1.11 x
= wR = ( g / R) R = gR …(i) x = r / 1.11 = 3.85 ´ 10 8 / 1.11
1 2 1 2 = 3.47 ´ 10 8 m
\KE of rotation of earth = Iw = ´ MR 2 ´ w2
2 2 5
2 1 57. Resultant gravitational intensity at a mid-point on the line
M ( wR) 2 = MgR [from Eq. (i)] joining the two bodies, is
5 5
Gm2 Gm1 4G
10 g 90 I= - = 2 (m2 - m1)
47. As, g ¢ = g - = g (r / 2) 2 (r / 2) 2 r
100 100
R2 4 ´ 6.6 ´ 10 –11
\ g¢ = g = (1000 - 100)
(R + h) 2 12

9 R2 = 2.4 ´ 10 –7 Nkg –1
or =
10 (R + h) 2 58. Let x be the distance of point from the smaller body where
3 R gravitational intensity is zero.
or =
10 R + h Gm1 Gm
\ = 22
or h = ( 10 - 3) R / 3 (1 - x ) 2 x

( 10 - 3) ´ 6400 x m2 1000 1
= 345.60 km or = = =
3 1- x m1 100000 10

gR 2 æ 6400 ö
2 or 10 x = - x
–2
48. As, g ¢ = = 980 ´ ç ÷ = 960 cms or x = (1 / 11) m
(R + h) 2 è 6400 + 64 ø
dV
49. At equator, g ¢ = g - Rw2. 59. Here, I=- = -k / r
dr
When angular velocity be w¢ ( - xw), then 0 = g - Rw¢2 dr
or dV = k
or w¢ = g / R = xw r
Integrating it, we get
or x = ( g / R) / w V r dr
10 / (6.4 ´ 10 ) 6 òV0
dV = ò
r0
k
r
or x= ´ 24 ´ 60 ´ 60 = 17 or V = V0 + k log r / r0
2p
æ dö
50. At equator, g ¢ = g - Rw2 = 0 or w = g / R 60. As, g ¢ = g ç1 - ÷
è Rø
g æ dö
or w = 10 / (6.4 ´ 10 6) = 1.25 ´ 10 –3 rads–1 Þ = g ç1 - ÷
n è Rø
GM GM
51. Vp = - (3R 2 - r 2) inside the sphere and Vp = - outside æ n - 1ö
2R3 r or d=ç ÷R
è n ø
the sphere.
52. As, dV = -E dx 61. When r < R, Gravitational field intensity,
GM Gr æ 4 3 ö 4 pGrr
x/ 2 x/ 2 I= r= 3 ç pR r ÷ =
or V = -ò E dx = - ò kx-3dx = k / x2 R3
R è3 ø 3
¥ ¥
Telegram @unacademyplusdiscounts

Gravitation 385

62. If x is the distance of point on the line joining the two masses 1
tan q1 =
from mass m2¢ where gravitational field intensity is zero, then 4
Gm1 Gm æ 1ö
= 22 or q1 = tan -1 ç ÷ = 14°6'
(r - x) 2
x è 4ø
2 8 If q2 is the angle which the line y + 4 x = 6 makes with positive
or = 2
(9 - x) 2
x x-axis, then q2 = tan -1 ( - 4) = 75° 56' so, q1 + q2 = 90°
1 2 i. e. , the line y + 4 x = 6 is perpendicular to I.
or =
9-x x
71. Given, height of the satellite above the earth’s surface
On solving, x = 6 m.
(h) = 400 km = 0.4 ´ 10 6 m
63. Gravitational potential at a point on the surface of earth, Mass of the satellite (m) = 200 kg
- GM - gR 2 Radius of earth (Re) = 6.4 ´ 10 6 m
V= = = - gR
R R
Mass of earth (Me) = 6.0 ´ 10 24 kg
64. Gravitational potential on the surface of the shell is
Gravitational constant (G) = 6.67 ´ 10 -11 N-m 2/kg 2
V = Gravitational potential due to particle (V1)
+ Gravitational potential due to shell particle (V2) Energy required to send a satellite out of earth’s gravitational
influence is called its binding energy.
Gm æ G3m ö 4 Gm
=- + ç- ÷=- GMem
R è R ø R Binding energy of a satellite =
2(Re + h)
x x C C
65. As, V = -ò I dx = -ò 2
dx = 6.67 ´ 10 -11 ´ 6.0 ´ 10 24 ´ 200
¥ ¥ x x =
2(6.4 ´ 10 6 + 0.4 ´ 10 6)
66. When the spaceship is to take off, gravitational pull of earth
requires more energy to be spent to overcome it. 6.67 ´ 12 ´ 10 15
=
- GMm 2 ´ 6.8 ´ 10 6
67. As, U=
r = 5.885 ´ 10 9 J
- GMm
or r= = 5.9 ´ 10 9 J
U
- 6.67 ´ 10 –11 ´ 6 ´ 10 24 ´ 7.4 ´ 10 22 72. Gravitational force on a body at a distance x from the centre of
r= GMm
- 7.79 ´ 10 28 earth, F =
x2
= 3.8 ´ 10 8 m
\ Work done,
68. The change in potential energy in gravitational field is given R+ h R+ h GMm
by W =ò F dx = ò dx
R R x2
æ 1 1ö R+ h
D E = GMm ç - ÷ é 1ù æ1 1 ö
è r1 r2 ø = GMm ê - ú = mgR 2 ç - ÷
ë x ûR èR R + hø
In this problem; r1 = R and r2 = nR This work done appears as increase in potential energy
æ1 1 ö
D E = GMm ç - ÷ é1 1 ù
è R nR ø \ D U = mgR 2 ê - ú
ëR R + hû
GMm æ n - 1ö
= ç ÷ é 1 1ù 5
R è n ø = mg (5 h) 2 ê - ú = mgh
ë 5 h 6 hû 6
æ n - 1ö æ Gm ö
= mgR ç ÷ çQ g = 2 ÷
è n ø è R ø 73. Gravitational intensity,
dV 14
69. Gravitational potential energy of a body in the gravitational I= = = 0.7 Nkg –1
dx 20
- GMm
field, E = . When r decreases negative value of E Acceleration due to gravity,
r
increases i.e., E decreases. g = I = 0.7 Nkg –1

70. Work done by the gravitational field is zero, when Work done under this field in displacing a body on a slope of
displacement is perpendicular to gravitational field. Here, 60º through a distance s
gravitational field, I = 4$i + $j. If q1 is the angle which field = m ( g sin 60° ) s
makes with positive x-axis, then = 2 ´ (0.7 ´ 3 / 2) ´ 8 = 9.6 J
Telegram @unacademyplusdiscounts

386 JEE Main Physics

74. Since the gravitational field is conservative field, hence, the Now, ve µ R d
work done in taking a particle from one point to another in a v e Re de
gravitational field is path independent. \ =
v p Rp dp
GM
75. Orbital speed, v o = ; so speed of satellite decreases with Re d e
r =
2 Re d e
the increase in the radius of its orbit. We need more than one
satellite for global communication. For stable orbit it must vp = 2 ve
pass through the centre of earth. So, only (b) is correct. = 2 ´ 11 = 22 kms–1
76. When a satellite is moving in on elliptical orbit, it’s angular 83. Escape velocity of a body from the surface of earth is
momentum ( = r ´ p) about the centre of earth does not change
11.2 kms–1 which is independent of the angle of projection.
its direction. The linear momentum ( = mv) does not remain
constant as velocity of satellite is not constant. The total (v e)P1 2g 1 R1 g 1 R1
84. As, = = ´ = ab
mechanical energy of S is constant at all locations. (v e)P2 2 g 2 R2 g 2 R2
The acceleration of S (= centripetal acceleration) is always 2
directed towards the centre of earth. g m Mm æ Re ö 1 16
85. Here, = ´ç ÷ = ´ ( 4) 2 =
1/3 g e Me è Rm ø 81 81
æ T 2R 2 ö
77. As, h = ç ÷ -R 16
è 4 p2ø Þ gm = ge
81
1/3
é (24 ´ 60 ´ 60) 2 ´ (6.4 ´ 10 6) ´ 9.8 ù
=ê ú - 6.4 ´ 10 6 \ v e = 2 g eRe = 2 ´ 9.8 ´ (6400 ´ 1000)
ë 4 ´ (22 / 7) 2 û
» 11.2 kms–1
= 3.6 ´ 10 7 m = 36000 km
16 1
v m = 2 g m Rm = 2 ´ g e ´ Re
78. Orbital velocity of satellite at distance r from the centre of 81 4
GM 2 2
earth is, v = = 2 g e Re = ´ 11 » 2.5 kms–1
r 9 9
GMm 1 ær ö
3/ 2 3/ 2
Total energy of satellite = PE+ KE = - + mv 2 æ 1ö 1
r 2 86. As, T2 = T1 ç 2 ÷ = T1 ç ÷ = times
è r1 ø è 4ø 8
GMm 1 GM GMm
=- + m =-
r 2 r 2r 87. As T 2 µ r3
The viscous force acting on satellite decreases the energy of TA2 rA3
So, =
satellite. As a result of it, the value of r gradually decreases, TB2 rB3
consequently the height of satellite gradually decreases. rA æ TA 2/3 ö
or =ç ÷ = (8) 2/3 = 4 or rA = 4 rB
79. As, v =
GM rB çè TB ÷ø
R So, rA - rB = 4 rB - rB = 3 rB
GM 2 GM 2
v¢ = = = v 88. The earth moves around the sun is elliptical path. So by using
(R + R / 2) 3R 3
the properties of ellipse
80. The body can be fired at any angle because the energy is r1 = (1 + e) a and r2 = (1 - e) a
sufficient to take the body out of the gravitational field of r +r
earth. Þ a= 1 2
2
81. The escape velocity at the surface of earth is 11.2 kms–1. and 2 2
1 2 = (1 - e ) a
rr
where, a = semi-major axis
2 GM b = semi-minor axis
82. Escape velocity v e =
R e = eccentricity
b2
4 3 Now, required distance = semilatusrectum =
2G pR ´ d a
Þ ve = 3
a2 (1 - e2) r1 r2 2 rr
R = = = 12
a (r1 + r2) / 2 r1 + r2
4 8
= 2G pR 2 ´ d = R p Gd
3 3 89. Asteroids move in circular orbits like planets under the action
(Here, d = mean density of earth) of central forces.
Telegram @unacademyplusdiscounts

Gravitation 387

90. The gravitational force of sun on comet is radial, hence 1 GMm 1 GMm
mv12 - = mv 22 -
angular momentum is constant over the entire orbit. Using 2 r1 2 r2
law of conservation of angular momentum, at locations A and
æ 1 1ö
B, we get, or v 22 - v12 = 2 GM ç - ÷ …(ii)
è r2 r1 ø

Putting the values from Eq. (i) in Eq. (ii) and solving, we get
r2 r1 1/ 2
1 é 2 GMr2 ù
m M v1 = ê ú
B A ë r1 (r1 + r2) û
L = mv1 r1 = mv 2 r2 m
Sun 1/ 2
or 2 v = v11r
…(i) é 2 GMrr 12
ù
2 \ L = mv11
r =mê ú
r2
ë (r1 + r2) û
Using the principle of conservation of total energy at A and B,

Round II
GM GMm
1. As, g = If r > R, then F=
R2 r2
2 2 2
g M æ MM ö æ RE ö 1 æ12742 ö mv GMm
So, =ç ÷ ´ç ÷ = ´ç ÷ \ =
g E è ME ø è RM ø 10 è 6760 ø r r2
gM 1
\ = 0.35 Þ vµ
gE r
GMm
Þ g M = 9.8 ´ 0.35 = 3.48 ms–2 6. Force on the body = 2
x
2. It is self-evident that the orbit of the comet is elliptic with sun To move it by a small distance dx,
begin at one of the focus. Now, as for elliptic orbits, according GMm
to Kepler’s third law, Work done = F dx = dx
x2
4 p 2a3 R+ h
T2 = R+ h dx é - GMm ù
GM Total work done = ò F dx = GMm ò =
R x2 êë x úû R
æ T 2 GM1/3 ö
Þ a=ç ÷ é1 1 ù
ç 4 p2 ÷ = GMm ê -
è ø ë R R + h úû
1/3
é (76 ´ 3.14 ´ 10 7) ´ 6.67 ´ 10 –11 ´ 2 ´ 10 10 ù é (R + h) - R ù GMmh
or a=ê ú =ê ú=
ë 4 p2 û ë R (R + h) û R(R + h)
But in case of ellipse, GM mhR gmhR PRh
´ = =
2 a = rmin + rmax R2 R + h R + h R + h
\ rmax = 2 a - rmin = 2 ´ 2.7 ´ 10 12 - 8.9 ´ 1010
7. For orbiting space ship close to earth’s surface
@ 5.3 ´ 10 12 m mv o2 GMm
= ,
3. Since, earth from west to east, so train Q has effectively more R R2
angular velocity in comparison to train P and hence, GM
experiences a greater centrifugal force directed radially i. e. , vo = = gR
R
outwards. So, train Q will exert a lesser force on track Q in
comparison to train P. Hence, P exerts greater force on the track. \ v o = (9.8 ´ 6.4 ´ 10 6) @ 8 kms–1
- GMm GMm - GMm For escaping from closed to the surface of earth,
4. As, U = ,k = and E =
r 2r 2r GMm 1
= mv e2
For satellite U , K and E varies with r and also U and E remains R 2
negative whereas K remains always positive. 2 GM
ve = = 2 gR
R
GMm mv 2 GMm
5. If r < R, then F = 3
r= = r Þ v e = 2 ´ v o = 1.41 ´ 8 = 11.2 km /s
R r R3
Þ v µr \ The additional velocity to be imparted to the orbiting
satellite for escaping is11.2 – 8 = 3.2 kms–1.
Telegram @unacademyplusdiscounts

388 JEE Main Physics

8. If a body is projected from the surface of earth with a velocity 12. VA = (Potential at A due to m1 ) + (Potential at A due to m2 )
v and reaches a height h. Applying conservation of energy Gm1 Gm2
(relative to surface of earth) Þ VA = - -
R 2R
1 mgh
mv 2 = Similarly,
2 [1 + (h / R)]
h = R = 6400 km, g = 10 ms–2 VB = (Potential at B due to m1) + (Potential at B due to m2 )
Gm2 Gm1
So, v 2 = gh, Þ VB = - -
R 2R
i. e. , v = 10 ´ 6400 ´ 10 3 = 8 kms–1
Since, WA ®B = m (VB - VA ) Þ WA ®B
ì GMm Gm (m1 - m2) ( 2 - 1)
ï- r , r ³ R =
9. U(r) = í GMm 2R
ï- , r <R
î R 13. First we have to find a point where the resultant field due to
both is zero. Let the point P be at a distance x from centre of
10. For the satellite to move along closed orbit (a circle with a
bigger star.
radius R+h) it should be acted upon by a force directed
G (16 M) GM
towards the centre. In this case, this is the force of earth’s Þ 2
=
attraction. According to Newton’s second law x (10 a - x) 2
mv 2 GMm Þ x=8a (from O1)
=
R + h (R + h) 2
16 M M
GMm
At the earth’s surface, = mg O1 O2
R2 P
2a a
gR 2
Therefore, v= = 7.5 kms–1
R+h
10 a
11. If the gravitational force acting on an object is equal or greater
than the centripetal force required for the orbital motion, then
it will remain stuck to the surface of the star due to gravity i.e., once the body reaches P, the gravitational pull of
because in this condition the centrifugal force (acting away attraction due to M takes the lead to make m move
from the centre of rotation) is less than the gravitational force, towards it automatically as the gravitational pull of attraction
so the object will not be able to fly off. i.e., due to 16 M vanishes i.e., a minimum KE or velocity has to be
imparted to m from surface of 16 M such that it is just able to
mv 2 v2
mg ³ or g ³ overcome the gravitational pull of 16 M. By law of
r r conservation of energy.
Given, mass of the star M = 2.5 ´ 2 ´ 10 30 kg (Total mechanical energy at A)
= 5.0 ´ 10 30 kg = (Total mechanical energy at P)
3 é G (16 M) m G (16 M) m ù
Radius R = 12 km = 12 ´ 10 m 1 2
Þ mv min +ê - ú
GM 2 ë 2a 8a û
Acceleration due to gravity g = 2
é ù GMm G (16 M) m
6.67 ´ 10 -11 ´ 5.0 ´ 10 30 =0 + ê - ú
= ë 2a 8a û
(12 ´ 10 3) 2
1 2 GMm
= 0.2316 ´ 10 13 m/s 2 Þ mv min = ( 45)
2 8a
= 2.3 ´ 10 12 m/s 2
3 5 GM
Þ v min =
v 2 (rw) 2 2 a
Now, centripetal acceleration = (Q v = rw)
r r
Gm2 mw2L
= rw2 = r(2pn) 2 (Q w = 2pn) 14. From figure, 2
cos 30° = mw2r =
L 3
= 12 ´ 10 3 ´ (2 ´ 3.14 ´ 12) 2 L
\ r=
= (12 ´ 10 3 ´ 4 ´ 9.87 ´ 144) 3
= 1065.95 ´ 10 3 = 11
. ´ 10 6 m/s 2 3 Gm
\ w=
L3
v2
As g > , therefore the object will remain stuck to the star.
r
Telegram @unacademyplusdiscounts

Gravitation 389

15. Mean orbital radius of the earth around the sun T1 = 1 h,T2 = 8 h = 10 4 km
r = 1.5 ´ 10 8 km = 1.5 ´ 10 11 m æ8ö
3/ 2
R2 = ç ÷ ´ 10 4 km = 4 ´ 10 4 km
Time period of earth around the sun = 1yr è 1ø
= 365 days = 365 ´ 24 ´ 60 ´ 60 s 2 pR1 2 p ´ 10 4
v1 = = = 2 p ´ 10 4 kmh –1
As required centripetal force is obtained from the T1 1
gravitational force, therefore
2 pR2 2 p ´ 4 ´ 10 4
Centripetal force = Gravitational force v2 = = = p ´ 10 4 kmh –1
T2 8
mv 2 GMsm
= Relative velocity of S 2 with respect to S1 is
r r2
v = v 2 - v1 = ( p ´ 10 4 - 2 p ´ 10 4) kmh –1
GMs
v2 =
r |v| = p ´ 10 4 kmh –1
GMs
2
(rw) = (Q v = rw) |v| p ´ 10 4 p
r 18. As, w = = = rad h –1
R2 - R1 4 ´ 10 4 - 1 ´ 10 4 3
2 GMs
or w = 3 Gm (m dx)
r 19. Force on elementary part dF =
2p x2
But w=
T x
dx
2
æ 2p ö GMs
\ ç ÷ = 3 x
èT ø r
a +L dx
4p 2r3 Þ F = Gmò ( A + Bx)
or Ms = a x2
GT 2
é æ1 1 ö ù
4 ´ (3.14) 2 ´ (1.5 ´ 10 11)3 \ F = Gm ê A ç - ÷ + BL ú
= ë è a a+L ø û
6.67 ´ 10 -11(365 ´ 24 ´ 60 ´ 60) 2
» 2 ´ 10 30 kg
20. If the mass of sun is M and radius of the planet’s orbit is r, then
16. As, D KE = D U as v o = GM / r
1 æ1 1 ö 2 pr r
Þ mv 2 = GMe m ç - ÷ …(i) T= = 2 pr ´
2 èR R + hø
v0 GM
GMe
Also, g = …(ii) 4 p 2r 2
R2 i. e. , T2 = …(i)
GM
On solving Eqs. (i) and (ii), we have
Now, if the planet (When stopped in the orbit) has velocity v
R when it is at a distance x from the sun, by conservation of
h=
æ 2 gR ö mechanical energy, we get
ç 2 - 1÷
è v ø 1 æ GMm ö GMm
mv 2 + ç - ÷ =0 -
2 mv 2 GMm x rGM
17. For satellite, = Þ v2 =
R è R2 ø R æ dx ö 2 GM é r - x ù
or ç- ÷ =
v=
2 pR è dt ø r êë x úû
T
dx 2 GM (r - x)
4 p 2R 2 GM i. e. , - =
Þ 2
v = = dt r x
T2 R
t r 0 é x ù
\ T2 =
4 p 2R3 or ò0 dt = - 2 GM òt ê (r - x) ú dx
ë û
GM
Substituting x = r sin 2 q and solving the RHS, we get
If T1 and T2 are the time periods for satellites S1 and S 2
respectively. r æ pr ö
2 3 t= ´ç ÷ (After integrating)
æ T1 ö æR ö 2 GM è 2 ø
ç ÷ = ç 1÷
è T2 ø è R2 ø Now, Eq. (i) reduces to
æT 2/3 ö
1 æ 2ö
Þ R2 = çç 2 ÷÷ R1 t= T ,i. e. ,t = ç ÷T
è T1 ø 4 2 è 8 ø
Telegram @unacademyplusdiscounts

390 JEE Main Physics

21. The true weight of a body is given by mg and with height g So, from Eqs. (i) and (ii), we have
decrease 1 GMm GMm
mv 2 - =- ,
WS mg ¢ 1 é g ù 2 r 2a
So, = = ê As g ¢ = 2ú
WE mg [1 + (h / R)]2 ë [1 + (h / R)] û é 2 1ù
i. e. , v 2 = GM ê - ú
But here, h = 7 R - R = 6 R ,i. e. ,h / R = 6 ë r aû
WE 10 25.
So, WS = = = 0.2 N
(1 + 6) 2 49
dx y
22. If g is the acceleration due to gravity of earth at the position of L2 +x 2
satellite, the apparent weight of a body in the satellite will be dF
θ M θ dF sin θ
Wapp = m ( g ¢ - a) x x
L P θ dF sin θ
θ
But as satellite is a freely falling body, i. e. , g ¢ = a dF
L2 +x 2
So, Wapp = 0
23. As with height g varies as
g é 2hù Let the mass M be placed symmetrically.
g¢ = = g ê1 -
[1 + h / R ]2 ë R úû ¥
Þ Fnet = ò dF sin q

¥ GM ( l dx) L

-¥ X 2 + L2 X + L2
2
m
c h (dF cos q are cancelled out)
m ¥ dx
h
Þ Fnet = GMlL ò
h -¥ ( X 2 + L2)3 / 2
GMlL
Þ Fnet = (2)
and in according with figure h1 > h2, W1 will be lesser than W2 L2
and
2 GMl
éh h ù Þ Fnet =
i. e. , W2 - W1 = mg 2 - mg 1 = 2 mg ê 1 - 2 ú L2
ëR Rû
GM h
26. M l dx m
or W2 - W1 = 2 m
R2 R a
x
é GM ù
ê As g = R 2 and (h1 - h2) = hú
ë û æM ö
Gm ç dx÷
2 mhG æ 4 3 ö è l ø
or W2 - W1 = ç pR r ÷ Þ dU =
R3 è ø x
8 é 4 3 ù Integrating,
= pr Gmh ê as M = 3 pR r ú
3 ë û GmM a + l dx
Þ ò dU = l òa x
24. As in case of elliptic orbit of a satellite, mechanical energy
æ GMm ö GmM æ a+ lö
E = -ç ÷ remains constant, at any position of satellite in Þ U=- log e ç ÷
è 2a ø l è a ø
the orbit. 27. As, Wext = U¥ - Ur
GMm
i. e. , KE+PE = - …(i) P
2a
x
Now, if at position r , v is the orbital speed of satellite
1 3R
KE = mv 2 4R
r dr
2
GMm
and PE = - …(ii)
r
Telegram @unacademyplusdiscounts

Gravitation 391

æ GrM ö GMm 1 GMm


Wext = 0 - ò - ç - × dx÷ Þ - = mv 2 -
è x ø 2R 2 R
M 2 prdx 2 GM r dr GM
Wext = G ò 2
× = 2 ò Þ v=
p ´7R 2
16 R + r 2 7R 16 R 2 + r 2 R
2 GM 2 dz 2 GM 32. Mass of spaceship m = 1000 kg
Wext =
7 R2 ò 2
=
7 R2
[z ]
Mass of sun Ms = 2 ´ 10 30 kg
2 GM 4R
= [ 16 R 2 + r 2 ]3R Mass of mars Mm = 6.4 ´ 10 23 kg
7 R2
2 GM Radius of mars Rm = 3395 km = 3.395 ´ 10 6 m
= [ 4 2R - 5 R ]
7 R2 Radius of orbit of mars (r) = 2.28 ´ 10 8 km

=
2 GM
( 4 2 - 5) = 2.28 ´ 10 11 m
7R G = 6.67 ´ 10 -11 N-m 2/kg 2
28. Potential energy of a body at the surface of the earth Spaceship is present in gravitational field of the sun as well
GMm 9 R 2M as in the gravitational field of the mars.
PE = - =- = - mgR
R R
= 500 ´ 9.8 ´ 6.4 ´ 10 6
= - 3.1 ´ 10 10 J
Spaceship
So, if we give this amount of energy in the form of kinetic r
Sun Mars
energy then body escape from the earth.
29. Orbital radius of satellites r1 = R + R = 2 R
and r2 = R + 7 R = 8 R
GMm GMr \Potential energy of the spaceship due to gravitational field
U1 = - and U2 = -
r1 r2 of the sun
GMsm
GMm GMm =-
K1 = and K2 = r
2 r1 2 r2
Potential energy of the spaceship due to the gravitational
GMm GMm
E1 = and E 2 = field of the mars
2 r1 2 r2 GMmm
U1 K1 E1 =-
\ = = =4 Rm
U2 K2 E 2
\Total potential energy of the spaceship
2
1 æ 2 GM ö GMm æ GMsm ö æ GMmm ö
30. (KE) escape = m ç ÷ = = ç- ÷ + ç- ÷
2 è Re ø Re è r ø è Rm ø
1 GMm éM M ù
(KE) body initial = = - Gm ê s + m ú
2 Re r Rm û
ë
By law of conservation of energy, Potential energy of the spaceship out side the solar
Total initial mechanical energy system= 0
= Total final mechanical energy \Energy imparted to the spaceship required just rockets out
(KE+PE) surface = (KE+PE) at height h é æM M öù
of the solar system = 0 - ê - Gmç s + m ÷ ú
1 GMm GMm GMm êë è r Rm ø úû
Þ - =0 -
2 Re Re Re + h
éM M ù
(\velocity at maximum height is zero) = + Gm ê s + m ú
ë r Rm û
Þ h = Re
é 2 ´ 10 30 6.4 ´ 10 23 ù
31. As, (Total initial mechanical energy) p = 6.67 ´ 10 -11 ´ 1000 ê 11
+ ú
ë 2.28 ´ 10 3.395 ´ 10 6 û
= (Total final mechanical energy) 0
é 2 ´ 10 19 6.4 ´ 1013 ù
1 2 GMm 1 GMm = 6.67 ´ 10 -8 ê + ú
Þ m (0) - = mv 2 - ë 2.28 3.395 û
2 2 2
( 3 R) R 2 R
» 3.1 ´ 10 11 J
Telegram @unacademyplusdiscounts

392 JEE Main Physics

33. According to Right Hand Thumb Rule “curl the fingers of right (iv) v = v e parabolic path and it escape from the earth.
hand in the direction of rotation then thumb gives the (v) v > v e hyperbolic path and escape from earth.
direction of the areal velocity/angular momentum.” (10G) M
39. When G = 10 G , then g ¢ = = 10 g .
34. If the law of grvitation becomes an inverse cube law, then R2
2
GMm mv GM \Weight of person = mg ¢ = m ´ 10 g = 10mg i. e. , gravity pull
F= 3
= or v =
r r r on person will increase. Due to it, walking on ground would
2 pr 2 pr 2 become more difficult. As critical velocity of rain drop,
\ Time period of revolution of a planet, T = = or
v GM 2r 2 (r - s ) g
vc =
T 2 µ r 4 . It means a planet will not have an elliptical orbit. The 9h
circular orbit of a planet may not be possible as the where letters have the usual meanings, so v c µ g . Since g
gravitational attractive force obeys inverse square law and not increases, hence v c increases.
inverse cube law. A stone thrown by hand on the surface of To overcome the increased gravitational pull of earth, the
the earth will follow nearly parabolic path, under the areoplanes will have to travel much faster.
gravitational force. There will be some gravitational force
inside a spherical shell of uniform density. 40. Due to huge amounts of opposite charges on sun and earth
there will be a large force of electrostatic attraction as well as
35. When G decreases with time, the gravitational force gravitational attraction. Both the forces obey inverse square
GM m law and are central forces. Due to it, the distance between sun
F= will become weaker with time. Then r increases
r2 and earth will decrease. In this situation, all the three kepler’s
with time. Due to it, earth will be going around the sun not laws will be valid.
strictly in closed orbit and after long time it will leave the solar
system. 41. According to law of conservation of total mechanical energy,
n total energy of rocket at the surface of earth
GM02 æ m1m2 ö GM02 (1 - n)
36. F = 2
ç 2 ÷ = 2
(m1m2) n = total energy of rocket at the highest point
r12 è M0 ø r12 1 æ - GMm ö æ - GMm ö
or mv 2 + ç ÷ =0 + ç ÷
F GM02 (1 - n) (m1m2) n 2 è R ø è R+h ø
Acceleration due to gravity, g = = 2
mass r12 mass v 2 GM GM gR 2 gR 2
or = - = -
Thus accelaration due to gravity on earth is different for 2 R (R + h) R (R + h)
different objects. In this situation Kepler’s third law will not be æ R ö æ h ö
= gR ç1 - ÷ = gR ç ÷
valid. è R + hø èR + hø
When n is negative, then or v 2 (R + h) = 2 gRh
GM02 (1 + n) (m1m2) -n
or Rv 2 = 2 gRh - v 2h = (2 gR - v 2) h
g = 2
r12 mass Rv 2 6.4 ´ 10 6 ´ (5 ´ 10 3) 2
or h= =
which is positive as M0 > m1 or m2. (2 gR - v ) (2 ´ 9.8 ´ 6.4 ´ 10 6) - (5 ´ 10 3) 2
2

Therefore, object lighter than water will sink in water. = 1.6 ´ 10 6 m


4
37. As, g = mrGr (\ g µ r if r < R) 42. The orbital velocity of satellite is
3 1/ 2
GM 1 GM é (6.67 ´ 10 –11) ´ (6 ´ 10 24) ù
also, g = 2 (\g µ 2 if r > R) vo = =ê ú
r r R + h ë 6.4 ´ 10 6 + 1.6 ´ 10 6 û

if r1 < R and r2 < R, = 7.1 ´ 10 3 ms–1


F1 g 1 r1
then = = 43. Total energy of satellite = PE+KE
F2 g 2 r2
GMm 1
if r1 > R and r2 > R =- + mv 02
(R + h) 2
2
F1 g 1 æ r2 ö GMm 1 GM GMm
then = =ç ÷ =- + m =-
F2 g 2 è r1 ø (R + h) 2 (R + h) 2 (R + h)
Energy spent to take the body out of the earth’s gravitational
38. Velocity of nature of path of satellite field is
(i) v = v 0 circular path around the earth. = - (total energy of satellite)
(ii) v < v 0 elliptical path and body returns to earth. - GMm
=
(iii) v > v 0 but v e elliptical path around the earth and will not 2 (R + h)
escape.
Telegram @unacademyplusdiscounts

Gravitation 393

- (6.67 ´ 10 –11) ´ (6 ´ 10 24) ´ 200 48. If r is the distance between two electrons then according to
=
2 (6.4 ´ 10 6 + 1.6 ´ 10 6) Newton’s law, the gravitational force between them
= -5.0 ´ 10 J 9 m2 (9.1 ´ 10 –31) 5 ´ 10 -71
FG = G 2
= 6.67 ´ 10 –11 ´ @
r r2 r2
44. Time period of revolution of satellite,
and according to Coulomb’s law, the electrical force between
2 p (R + h) 2 ´ (22 / 7) ´ (6.4 ´ 10 6 + 1.6 ´ 10 6) electron is
T= =
vo 7.1 ´ 10 3 1 q ´q (1.6 ´ 10 –19) 2 2 ´ 10 -28
Fe = = 9 ´ 10 -9 ´ @
= 7082 » 7100 s 4 pe 0 r 2
r2 r2
45. Let v 0 be the speed of the spaceship in the circular orbit of FG 10 -71
\ @ = 10 - 43
radius r0 , and v e be the escape velocity for the orbit. Then, Fe 10 -28
mv 02 GMm mv e2 GMm i. e. , FG = 10 - 43Fe
= 2
and =
r0 r0 2 r0
i.e., gravitational force between two particles is negligible
GM 2 GM compared to the electrical force.
or v0 = , ve =
r0 r0
49. As a rotation of earth takes place about polar axis therefore,
As v e = v 0 + | Dv|cos q = v 0 + Dv cos q body places at poles will not feel any centrifugal force and its
The specific impulse required for escape is the least for q = 0 weight or acceleration due to gravity remains unchanged.
i.e., the initial velocity of the spaceship and the impulse are in 50. Upto ordinary heights the change in the distance of a projectile
the same direction, and is given by from the centre of the earth is negligible compared to the radius
GM of the earth. Hence, projectile moves under a nearly uniform
Dv = v e - v o = ( 2 - 1)
ro gravitational, force and its path is parabolic. But for projectile
going to great height, the gravitational force decreases in
46. After the first burst, the ship escape from the circular orbit inverse proportion to the square of the distance of the projectile
around the star and moves along a parabolic orbit. When the from the centre of the earth. Under such a variable force the
ship reaches the circular orbit r = r1 of the planet, the engine is path of projectile is elliptical.
again fired in a single rapid burst. For the ship to move along Gm1m2
GM 51. According to Newton’s law of gravitation, F =
the circular orbit of radius r1, its speed must be v1 = r2
r1 When m1, m2 and r all are doubled,
Letv1 e be the speed of the ship as it arrives atr = r1 and before the Gm1 m2
burst. Conservation of angular momentum requires F¢ =
r2
v e r0 = v1 e r1 cos f i. e. , force remains the same.
rv
or v1 e cos f = 0 e 52. Variation of g with depth from surface of earth is given by
r1
æ dö
1 GMm g ¢ = g R ç1 - ÷
Conservation of energy gives, mv12e = è Rø
2 r1
At the centre of earth, d = R
2 GM
or v1 e = æ Rö
r1 \ g ¢ = g ç1 - ÷ = 0
è Rø
Then the minimum specific impulse required is given by
2 GM GM Apparent weight of body = mg ¢ = 0.
( Dv) 2 = v12e + v12 - 2 v1 ev1 cos q = + v ev
r1 r1 53. Binding energy is the minimum energy required to free a
satellite from the gravitational attraction. It is the negative
3 GM 2 r0 2 GM GM æ 2 r0 ö
= - = ç3 - 2 ÷ value of total energy of satellite. Let a satellite of mass m be
r1 r1 r0 r1 è r1 ø revolving around earth of mass M e and radius Re
GMem 1
GM æ 2 r0 ö Total energy of satellite = PE+KE = - + mv 2 .
Hence, Dv = ç3 - 2 ÷ Re 2
r1 è r1 ø GMem GMem
= +
47. For a single rapid burst at q = 180°, the minimum specific Re 2Re
impulse is that which makes the speed of the spaceship v, GMm
=
v ¢ = v 0 - Dv = 0 , so that it will fall onto the star. Hence, the 2Re
GM \ Binding energy of satellite = – (total energy of satellite)
minimum impulse required is Dv = v 0 =
r0 which depend on mass of the satellite.
Telegram @unacademyplusdiscounts

394 JEE Main Physics

54. The value of g at any place is given by the relation, Since, the planet is having double radius in comparision to
2 2
g ¢ = g - w Re cos l. When λ is angle of latitude and w is the earth, therefore escape velocity becomes twice i.e., 22 kms–1.
angular velocity of earth. 60. The potential energy of an object at the surface of the earth
If there is no rotation w = 0 , GMm
U1 = - ...(i)
\ g¢ = g R
g The potential energy of the object at a height h = R from the
55. As, v 0 = Re surface of the earth
Re + h
GMm GMm
For a satellite revolving very near the earth surface, Re + h µ Re U2 = - =- ...(ii)
R+h R+R
and v 0 = Reg Hence, the gain in potential energy of the object
5
= 64 ´ 10 ´ 10 DU = U2 - U1
3
= 8 ´ 10 ms -1
= 8 kms -1 GMm GMm
or DU = - +
2R R
which is independent of height of satellite. GMm GMm
or DU = - +
gR 2 2R R
57. As, g ¢ =
(R + h) 2 1 GMm
or DU =
g gR 2 2 R
Þ =
9 (R + h) 2 But, is known that GM = gR2
1 R 1 gR 2m
or =± Hence, DU =
3 (R + h) 2 R
Taking + ve sign, 1 1
or DU = gRm or DU = gmR
R+g=2R 2 2
or h=2R Gm1m2
61. As, F =
Taking – ve sign, r2
Gm1m2 F
R + h = –3 and F¢ = =
(3r) 2 9
or h=–4R
h = – 4 R is not possible, Thus, h = 2R æ F - F¢ö 8
\% decrease in F = ç ÷ ´100 = ´ 100 = 89%
è F ø 9
58. As g ¢ = g - w2R cos2 l
Thus, attraction force between sun and earth is decreased by
The latitude at a point on the surface of the earth is defined as
89%.
the angle, which the line joining that point to the centre of earth
makes with equitorial plane. It is denoted by l. For the poles GMe
62. Orbital velocity v o =
l = 90° and for equator l = 0º. R+h
(i) Substituting l = 90° in the above expression, we get 2p(R + h) 2p(R + h)3 / 2
Time period, T= =
g pole = g - w2R cos2 90° vo (GMe)1/ 2

g pole = g Thus, time period of satellite is independent of mass of


i.e., there is no effect of rotational motion of the earth on the satellite but depends on mass of the earth, radius of the orbit
value of g at the poles. (R + h), height of the satellite from the surface of earth.
(ii) Substituting l = 0º in the above expression, we get 63. Here, mg = 12.6 N
g equator = g - w2R cos2 0° gR 2
At height h, g¢ =
(R + h) 2
\ g equator = g - w2R
2
é R ù 4
i.e., the effect of rotation of earth on the value of g at the When h = R / 2g ' = g ê ú =g 9
equator is maximum. ë R + (R / 2) û
4 4
59. Escape velocity, Weight at height, h = mg = 12.6 ´ = 5.6 N
9 9
2GM 8
ve = =R pGr
R 3 64. According to law of conservation of angular momentum,
\ v e µ R if r = constant Iw = Iw' (In moment of inertia)
Telegram @unacademyplusdiscounts

Gravitation 395

2
2 2p 2 æ R ö 2p 70. Change in PE, DU = U2 - U1
or MR 2 ´ = Mç ÷ ´
5 T 5 è 4ø T GMm GMm
=- +
T 24 (R + nR) R
or T' = = = 1.5h
16 16 GMm GMm
GM =- +
65. As, g 0 = = g (at earth’s surface) R(n + 1) R
R2
(R 2g )m n
æ 2d ö = ´
At height h, g h = g ç1 - ÷ R (n + 1)
è Rø
æ n ö
\ g 0 > g h and g 0 > g d = mgR ç ÷
è n + 1ø
1/ 2
2 pr 2 pr é r3 ù
66. As, T = = 1/ 2
= 2p ê ú 71. Electronic charge is a universal constant. It does not depend
v [GM / r ] ë GM û on the value of acceleration due to gravity.
As per question, 72. Escape velocity from the surface of earth is given by
1/ 2
é (6400 + 36000)3 ù
24 = 2p ê ú 2Gme
GM ve = ...(i)
ë û Re
1/ 2
é (6400)3 ù
and T' = 2p ê ú Here , v e = 11.2 kms-1,Mp = 4 me¢
ë GM û
1/ 2 2GMp 2G ´ 4Me
T¢ é (6400)3 ù ve = = ...(ii)
\ =ê 3ú
= (0.4)3 Rp 2Re
24 ë (6400 + 36000) û
or T ¢ = (0.4)3 ´ 24 = 1.53 h Dividing Eq. (ii) by Eq. (i), we get
é Gm æ Gm ö æ Gm ö ù So, v e¢ = 2 ´ v e = 1.414 ´ 11.2 = 15.8 ms–1
67. V = ê - + ç- ÷+ç ÷ + ...ú ´ 2
ë 1 è 2 ø è 4 ø û l
73. As, T = 2p
é 1 1 ù g
= -2Gmê1 + + + ...ú
ë 2 4 û i.e., T µ1/ g (for fixed value of l)
é ù 1/ 2
ê 1 ú T1 g ¢ é gR 2/ (R + 2R) 2 ù 1
= -2Gmê ú \ = =ê ú =
T2 g g 3
ê1- 1 ú ë û
ë 2û
74. Escape velocity from the surface of earth is
= - 4Gm = -4G ´ 1 = - 4G
2GM
In magnitude, V = 4 G ve =
R
2GM
68. For earth, v e = = 11kms-1 Escape velocity from the surface of earth is
R
2M(6M)
For planet, . e v= = 3v
(2R)
2G ´ 10M
v e¢ = 3/ 2 32
R / 10 æ R2 ö æ 3R ö
75. As, T2 = T1ç ÷ = 4ç ÷ = 4 27 h
è R1 ø èRø
2GM
= 10 = 10 ´ 11
R GM 1
76. As, g = i. e. , g µ 2
= 110 kms-1 R2 R

2GM Now, radius of earth


69. Escape velocity, v e = ;
R R' = R – R/3 = 2 R/3
M g' R2 9 9
So, ve µ So, = = or g ' = g
R g (2R / 3) 2
4 4
v e¢ 2M / (R / 2)
\ = =2 77. The escape velocity for the surface of earth is
ve M /R
2GMe
or v e¢ = 2v e = 2 ´ 11.2 = 22.4 kms-1 v es( e) =
Re
Telegram @unacademyplusdiscounts

396 JEE Main Physics

8 é 4 3ù 79. Potential energy on earth surface is - mgR while in free space


or v es( e) = Re Gpr êQ Me = 3 pR ú
3 ë û it is zero. So to free the spaceship minimum required energy is
8 U = mgR = 10 3 ´ 10 ´ 6400 ´ 10 3 = 6.4 ´ 10 10 J
= Re Gpr
3 80. As, v e = 2 gR
8 and v o = gR
v es( e) Re Gpr
3 1
Hence, = = \ 2 vo = ve
v es(P) 8 12
4Re Gp 9r
3 81. From conservation of energy.
So, v es( p) = 12 v es( e)
R
78. Let gravitational field is zero at P as shown in figure. A
P 2R
A B
x r–x
r
Total energy at the planet
GM G ( 4 m)
= = Total energy at the altitude
x2 (r - x) 2
Þ 2
4 x = (r - x) 2 - GMm - GMm 1
+ (KE) surface = + mv A2 …(i)
R 3R 2
Þ 2x = r - x
r In its orbit, the necessary centripetal force is provided by the
Þ x=
3 gravitational force
GM G ( 4 m) mv A2 GMm
\ Vp = - - \ =
x r-x (R + 2 R) (R + 2 R) 2
3 Gm 6 GM 9 GM
=- - =- Gm
r r r Þ v A2 = …(ii)
3R
From Eqs. (i) and (ii), we have
5 GMm
(KE) surface =
6 R
Telegram @unacademyplusdiscounts

Properties
11 Solids of
JEE Main MILESTONE
< Elastic Behaviour < Hooke’s Law
< Stress < Poisson’s Ratio ( s )
< Strain < Work Done or Potential Energy Stored in a
< Stress-Strain Relationship Stretched Wire
< Elastic Limit < Thermal Stresses and Strains

11.1 Elastic Behaviour


The elastic behaviour can be understood by taking the microscopic nature of
solids. A solid body is composed of atoms or molecules, which is surrounded by
other atoms or molecules. Those are bounded together by interatomic or
intermolecular forces and stay in stable equilibrium in their lattice position. This
can be visualised by taking a mode of spring ball system as shown in figure.
From figure, we see that the balls represent the atoms or molecules and the springs
represent the interatomic or intermolecular forces. In this system, if any ball is
displaced a little from its equilibrium position, the springs attached to that ball will A body is said to be elastic, if on
either be stretched or compressed. Therefore, the restoring forces are developed in, releasing the deforming force it
the springs and they will bring the ball back to its natural position. This is known regains its original shape. The
as, the elastic behaviour of the solid body. property of matter by virtue of
Interatomic/intermolecular force which a body tends to regain its
original shape and size after the
removal of deforming forces is
Atom/molecule elasticity.
called

11.2 Stress
When an external force is applied to a body then at each cross-section of the body
an internal restoring force is setup which tends to restore the body to its original
state. The restoring force setup inside the body per unit area is known as stress.
Restoring force
Stress =
Area
In SI system, unit of stress is Nm–2 or pascal (denoted by Pa) and in CGS system is
dyne/cm 2.
Telegram @unacademyplusdiscounts

398 JEE Main Physics

Different types of stresses are given below Here, Dx is the change (may be in length, in volume etc.)
and x the original value of quantity in which change has
1. Normal or Longitudinal Stress occurred. It has no dimension as it is a pure number.
If area of cross-section of a rod is A and a deforming force Since, a body may have three types of deformation, i. e. , in
F is applied along the length of the rod and perpendicular length, in volume or in shape, likewise there are following
to its cross-section, then in this case, stress produced in three types of strains.
the rod is known as normal or longitudinal stress.
F 1. Longitudinal Strain
Longitudinal stress = n
A
It is defined as the change in length per unit original
Longitudinal stress is of two types length of the body under deformation by the external
(i) Tensile stress When length of the rod is increased on force. Thus,
application of deforming force over it, then stress l
produced in rod is called tensile stress. F F
(ii) Compressive stress When length of the rod is
decreased on application of deforming force, then the Change in length
Longitudinal strain =
stress produced is called compressive stress. Original length
Dl
2. Volumetric Stress or el =
l
When a force is applied on a body such that it produces a
It is of two types
change in volume and density, shape remaining same
1. at any point, the force is perpendicular to its surface. (i) Tensile strain If on applying a deforming force, there
is an increase of Dl in length of a rod, then strain
2. at any small area, the magnitude of force is directly
produced in the rod is called tensile strain.
proportional to its area.
Then, force per unit area is called volumetric stress. F F
F l + ∆l
\ Volumetric stress = v
A
(ii) Compressive strain If on applying a deforming force,
3. Shearing or Tangential Stress there is decrease of Dl in length of a rod, then strain
When the force is applied tangentially to a surface, then it produced in the rod is called compressive strain.
is called tangential or shearing stress.
F F
F
Tangential stress = t l– l
A
It produces a change in shape, volume remaining same.
2. Volumetric Strain
It is defined as the change in volume per unit original
F
volume of the body under deformation by the external force.

F
F
∆V
Fixed face F F

V
11.3 Strain F
F
When the size or shape of a body is changed under an
external force, the body is said to be strained. The change Change in volume
Volumetric strain =
occurred in the unit size of the body is called strain. Original volume
Usually, it is denoted by e. DV
or eV =
Change in dimension Dx V
Strain = =
Original dimension x
Telegram @unacademyplusdiscounts

Properties of Solids 399

3. Shearing Strain using a material the working stress is always kept


small, as a fraction of breaking stress so that safety
This type of strain is produced x
æ breaking stressö
when a shearing stress is F factor ç = ÷ has a large value.
φ è working stress ø
present.
φ The materials of the wire, which break as soon as stress is
It is defined as the angle in L φ
increased beyond the elastic limit are called brittle.
radians through which a plane
Fixed face Graphically, for such materials the portion of graph
perpendicular to the fixed
between B and E is almost zero. While the materials of the
surface of the cubical body is
wire, which have a good plastic range (portion between B
turned under the effect of tangential force,
and E) are called ductile.
x
Shearing strain f =
L
11.5 Elastic Limit
11.4 Stress-Strain Relationship Elastic limit is the upper limit of deforming force after
which, on removing the deforming force, the body regains
For a solid, the graph between stress (either tensile or its original form completely and beyond, which if the
compressive) and normal strain is shown in figure. deforming force is increased the body looses its property
Plastic region of elasticity and gets permanently deformed.
Breaking D
strength C E (Fracture point) Note Elastic limit is the property of a body whereas elasticity is the
B
Elastic limit property of material of a body.
A
Proportional limit

Stress
OO' Strain
11.6 Hooke’s Law
Hooke’s law states that for metals within elastic limit,
Characteristics stress applied to a body is proportional to the resulting
(i) Part OA OA is a straight line showing that the material strain, i. e. ,
Stress
follows the Hooke’s law. If the applied load decreases, Stress µ Strain or = E = constant
Strain
the strain retraces the same curve backward and
regains the original state when the load is fully where, E is constant and is known as modulus of elasticity
removed. This point A is called the proportional limit. or coefficient of elasticity of the material of the body.
(ii) Part AB The Hooke’s law is no longer valid as the
graph is not found to be a straight line as indicated by Check Point 1
part AB of the graph. If the deforming force is removed
the curve does not trace the original path BAO and 1. Which is more elastic : water or air. Why ?
instead follows, the dotted curve. Then the point B is 2. Under what condition, the restoring forces are equal and
called the elastic limit and the portion of the graph opposite to the external deforming force?
between O and B is called elastic region. 3. Which of the two forces-deforming or restoring is responsible
(iii) Beyond B There is increase in strain even without any for elastic behaviour of the substance?
further increase in stress. The behaviour is irregular as 4. A hard wire is broken by bending it repeatedly in opposite
shown by the wavy line. The point at which the wire directions. Why?
yields to the applied stress and goes on increasing in 5. Why electric poles have a hollow structure?
length even if the load is kept constant is called the 6. Why do spring balances show wrong readings after they have
yield point. been used for a long time?
The increase in length of wire for virtually no increase
in stress is called plastic behaviour of the wire.
(iv) From C to E The strain is totally plastic. Wire starts Types of Modulus of Elasticity
flowing like a viscous liquid. At C some neck and waist Depending on the type of stress applied and resulting
are produced. Then the portion of the graph between strain, we have following three modulii of elasticity
points C and D is called plastic region. Finally, at a
1. Young’s modulus of Elasticity
point D, the wire may break. The maximum stress
corresponding to point D in the region CE is called the 2. Bulk modulus of Elasticity
breaking strength (stress) or tensile strength. While 3. Modulus of Rigidity.
Telegram @unacademyplusdiscounts

400 JEE Main Physics

Hot Spot Young’s Modulus


of Elasticity (Y)
Young’s modulus of elasticity (Y ) is defined as the ratio of normal stress (either tensile or compressive stress) to the
longitudinal strain within a elastic limit.
normal stress (3.18 ´ 10 8) (1m)
Thus, Y = DL = = 1.59 ´ 10 –3 m
longitudinal strain 2 ´ 10 11
Consider a metal wire PQ of length l, radius r and the DL = 1.59 mm
P
uniform area of cross-section A. Let it be suspended DL 1.59 ´ 10 –3 m
Strain = =
from a rigid support at P, a stretching force F be applied L 1m
normally at the free end Q and let its length increase by l = 1.59 ´ 10 –3 = 0.16%
Dl ( = QQ ¢).
Dl Sample Problem 2 A copper wire of length 2.2 m and a
Then, longitudinal strain =
l steel wire of length 1.6 m, both of diameter 3.0 mm are
Q connected end to end. When stretched by a load, the net
F F
Normal stress = = (Q A = pr2 ) l
elongation is found to be 0.70 mm. The load applied
A pr2
Normal stress
Q' (in Newton) is
Young's modulus (Y ) = F [Given, Yc = 1.1 ´ 1011 Nm -2, Ys = 2 ´ 1011 Nm -2]
Longitudinal strain [NCERT]

F / pr2 Fl (a) 1.8 ´ 10 2 N


= = (b) 2.5 ´ 10 6 N
Dl / l pr2 Dl
Since strain is dimensionless quantity, therefore, the unit of Young’s (c) 3.8 ´ 10 8 N
modulus is same as that of stress. Young’s modulus has unit of (d) 6.1 ´ 10 –2 N
pressure.
Interpret (a) The copper and steel wires are under a tensile
Note It is observe that for metals Young’s moduii are large. Therefore, stress because they have the same tension (equal to the load W)
these materials required a large force to produce a small change in and the same area of cross-section A.
length. Stress = strain ´ Young’s modulus
w æ DL ö æ DL ö
Sample Problem 1 A structural steel rod has a radius of \ = Yc ´ ç c ÷ = Ys ´ ç S ÷
A è Lc ø è LS ø
10 mm and a length of 1m. A 100 kN force stretches it along its
length Young’s modulus of structural steel is 2 ´ 1011 Nm -2. The where the subscripts C and S refer to copper and stainless steel
respectively.
strain on the rod is
DLC æ YS ö æ LC ö
(a) 1% (b) 0.04% = ç ÷ ´ç ÷ …(i)
DLS è YC ø è LS ø
(c) 0.08% (d) 0.16%
Given, LC = 2.2 m,LS = 1.6 m
Interpret (d) We assume that the rod is held by a clamp at one
DLC 2 ´ 10 11 2.2
end, and the force F is applied at the other end, parallel to the length = ´ = 2.5
of the rod. Then DLS 1.1 ´ 10 11 1.6
F F The total elongation is given to be
Stress = = 2
A pr DLC + DLS = 7 ´ 10 -4 m …(ii)
3 -2
Given, F = 100 kN = 100 ´ 10 N,r = 10 m Solving Eqs. (i) and (ii), we get
F 100 ´ 10 3 N DLC = 5 ´ 10 -4 m,
= =
A 3.14 ´ (10 –2) 2 DLS = 2 ´ 10 -4 m
8 –2
= 3.18 ´ 10 Nm
( A ´ Yc ´ DLc ) p (1.5 ´ 10 –3) 2 ´ (5 ´ 10 -4 ´ 1.1 ´ 10 11)
W= =
The elongation, Lc 2.2
(F/A) L
DL = = 1.8 ´ 10 2 N
y
Telegram @unacademyplusdiscounts

Properties of Solids 401

Sample Problem 3 A steel wire of length 4 m and diameter Interpret (c) Total change in length of the wire is
5 mm is stretched by 5 kg-wt. The increase in its length, if the a
Young’s modulus of steel wire is 2.4 ´ 1012dyne cm–2 is
(a) 0.003 cm (b) 0.0041 cm x
θ
(c) 0.00041 cm (d) 0.005 cm
dx
Interpret (b) Here, l = 4 m = 400 cm,2 r = 5 mm L

or r = 2.5 mm = 0.25 cm
F = 5 kg -wt = 5000 g -wt = 5000 ´ 980 dyne
Dl = ?,Y = 2.4 ´ 10 12 dyne cm -2 F
b

F l
As, Y= ´ DL F L dx
pr 2 Dl ò0 dy =
pY ò0 ( a + x tan q) 2
Fl (5000 ´ 980) ´ 400
or Dl = 2 = L
pr Y (22 / 7) ´ (0.25) 2 ´ 2.4 ´ 1012 F æ 1 ö
Dl = ç ÷
pY tan q è a + x tan q ø 0
= 0.0041 cm
FL
=
Sample Problem 4 A thin uniform metallic rod of length pa ( a + L tan q) Y
0.5 m and radius 0.1 m with an angular velocity 400 rad s -1 in a FL
horizontal plane about a vertical axis passing through one of its = (\a + L tan q = b)
p abY
ends. Elongation in the rod is (in m) [Given density of material of
3.14 ´ 9.8 ´ 10
the rod is 10 4 kgm -3 and Y = 2 ´ 1011 Nm -2] [NCERT] \ Dl =
3.14 ´ (9.8 ´ 10 –4) ´ 5 ´ 10 -4 ´ 2 ´ 10 11
10 -3 3 2 10 2
(a) (b) (c) (d) Dl = 10 -3 m
3 10 -3 10 2 2
Interpret (a) Consider an element of length dx at a distance x Sample Problem 6 A tension of 20 N is applied to a wire
from the axis of rotation, of cross-sectional area 0.01 cm 2. The decrease in
dF = dm xw2 cross-sectional area is [Young’s modulus of
= rAdx × x × w2 Cu = 1.1 ´ 1011 Nm -2, Poisson’s ratio = 0.32]
dx
L (a) 1.81 ´ 10 –4 cm2 (b) 1.16 ´ 10 –6 cm2
F = rAw2ò xdx x
x (c) 2.81 ´ 10 –8 cm2 (d) 5.23 ´ 10 –3 cm2
1 L
= rAw2 (L2 - x2) Dl F 20
2 Interpret (b) As, = = = 1.81 ´ 10 –4
l AY 10 -6 ´ 1.1 ´ 10 11
If dy is the elongation in the element of length dx, then Dr Dl
dy F =s ´ = 0.32 ´ 1.81 ´ 10 –4
= r l
dx AY
DA 2 Dr
F dx rw2 L = = 2 ´ 0.32 ´ 1.81 ´ 10 –4
A r 2 2
2Y ò 0
dy =
A × Y Dl
= 1.16 ´ 10 –4
2 2
rw × L and DA = A (1.16 ´ 10 –4)
Dl =
3Y
= 1.16 ´ 10 –6 cm2
3
æ 1ö
10 4 ´ ( 400) 2 ´ ç ÷
è2ø 10 -3 Sample Problem 7 A square lead slab of side 50 cm and
Dl = 11
= m
2 ´ 10 3 thickness 10 cm is subject to a sheaping force (on its narrow
face) of 9 ´ 10 4 N. The lower edge is riveted to the floor. The
Sample Problem 5 A body of mass 3.14 kg is suspended upper edge is displaced by [NCERT]
from one end of a wire of length 10 m. The radius of the wire is (a) 0.30 mm (b) 0.16 mm
changed uniformly from 9.8 ´ 10 –4 m at one end to 5 ´ 10 -4 m (c) 0.28 mm (d) 0.92 mm
at the other end. The change in length of the wire is
[Given Y = 2 ´ 1011 Nm -2] Interpret (b) The lead slab is fixed and the force is applied
-2 -1 parallel to the narrow face. The area of the face parallel to which
(a) 10 m (b) 10 m this force is applied is
-3 -5
(c) 10 m (d) 10 m
Telegram @unacademyplusdiscounts

402 JEE Main Physics

F /A
F K =
50 cm - DV /V
- FV pV
or K = =-
AD V DV
The negative sign indicates that on increasing stress the
volume of the sphere decreases. The units of bulk
A = 50 cm ´ 10 cm
modulus are Pa or Nm–2 in SI system.
= 0.5 m ´ 0.1m = 0.05 m2
For gases, bulk modulus is of two types
\ Stress applied is = (9.4 ´ 10 4 N / 0.05 m2)
(i) Isothermal bulk modulus of elasticity ( Ki ) , which is
= 1.80 ´ 10 6 Nm–2 equal to the pressure of gas (p).
Dx stress (ii) Adiabatic bulk modulus of elasticity Ka , Ka = g p
We know that shearing strain = =
L G where, g = C p/CV
Stress ´ L
\The displacement Dx =
G whereas, isothermal elasticity K i is given by
6
(1.8 ´ 10 ´ 0.5) Ki = p
Dx =
5.6 ´ 10 9 Ka Cp
\ =g= >1
= 1.6 ´ 10 –4 m = 0.16 mm Ki CV

As C p > CV
Sample Problem 8 The ratio of radii of two wires of same
material is 2 : 1. If these wires are stretched by equal force, what So, Ka > Ki
is the ratio of stress produced in them? K is maximum for solids, less for liquids and least for
(a) 1 : 2 (b) 1 : 3 gases.
(c) 1 : 4 (d) 1 : 5
Compressibility (C) The reciprocal of the bulk modulus of
Interpret (c) Here, r1 : r2 = 2 : 1, F1 = F2 = F the material of the body is called the compressibility of the
Force F material. Thus,
Stress ( S) = =
Area pr 2 1 -1 æ DV ö
Compressibility (C ) = = ç ÷
1 K V è Dp ø
\ Sµ 2,
r
2
Its unit is N–1m2 or Pa–1 in SI system.
S1 r22 æ 1 ö 1
\ = 2=ç ÷ =
S 2 r1 è 2 ø 4 Note Young’s modulus and Bulk modulus for a perfectly rigid body is
infinity.

Bulk Modulus of Elasticity (K) Modulus of Rigidity


It is defined as the ratio of the normal stress to the
It is defined as the ratio of tangential stress to shearing
volumetric strain. It is denoted by K. Thus,
strain. It is also called shear modulus. It is denoted by
Normal stress Greek letter h (eta). Thus,
K =
Volumetric strain Tangential stress
h=
Suppose a force F acts uniformly over the whole Shearing strain
surface of the sphere (shown), decreasing its volume by
Here, a body (shown) is acted upon by an external force
DV . Then, tangential to the surface of the body, the opposite face
F F being kept fixed, its volume remaining unchanged. Then,
V ∆V F
(V – ∆V) D D' C C'

F F

φ φ
F
F Fixed face
A B
Telegram @unacademyplusdiscounts

Properties of Solids 403

F /A p2 - p1 = 0.8 - 0.72 = 0.08 mm of Hg column


h=
DD ¢/AD = 0.08 ´ (13.6 ´ 10 3) ´ 9.8 Nm–2
DD ¢ = 10.66 ´ 10 3 = 1.07 ´ 10 4 Nm–2
Here, tan f » f =
AD
F /A Sample Problem 11 A 5 cm cube has its upper face
h=
f displaced by 0.2 cm by a tangential force of 8 N. The modulus
F of rigidity of the material of cube is
or h=
Af (a) 5 ´ 10 4 Nm-2 (b) 6 ´ 10 4 Nm-2
-2
4
(c) 7 ´ 10 Nm (d) 8 ´ 10 4 Nm-2
The units of modulus of rigidity are Pa or Nm–2 in SI
system. Interpret (d) Here, l = 5 cm = 5 ´ 10 -2 m,
Note Modulus of rigidity (or shear modulus) is involved with solids only. Dl = 0.2 cm = 0.2 ´ 10 –2 m, F = 8 N
Modulus of rigidity for a solid is generally less than its Young’s modulus. Shearing stress
Modulus of rigidity, h =
Shearing strain
Sample Problem 9 The increase in pressure required to F F 8
decrease the 200 L volume of a liquid by 0.004 % in kPa is Hence, shearing stress = = 2 = = 3200 Nm–2
A l ( 5 ´ 10-2) 2
(bulk modulus of the liquid = 2100 kPa) Dl 0.2
Shearing strain = = = 0.04
(a) 8.4 (b) 84 l 5
(c) 92.4 (d) 168 3200
\ h= = 80000 Nm–2 = 8 ´ 10 4 Nm–2
0.4
Interpret (b) Bulk modulus, K = -FV
A DV
F K DV 0.004
\ p= =- = (2100 ´ 103 ) ´ = 84 kPa 11.7 Poisson’s Ratio (s)
A V 100
It is defined as the ratio of lateral strain to the longitudinal
Sample Problem 10 One litre of ideal gas is compressed
strain is constant for a given material. This constant is
isothermally at 0.72 mm of Hg column so that its volume
called as Poisson’s ratio,
becomes 0.9 L. Its stress, if the density of mercury is
13.6 ´ 103 kgm -3 is Lateral strain æ - DR ö Dl
s= =ç ÷
Longitudinal strain è R ø l
(a) 1.07 ´ 10 4 Nm-2 (b) 1.18 ´ 10 4 Nm-2
(c) 2 ´ 10 4 Nm-2 (d) 5 ´ 10 4 Nm-2 Here, negative sign shows that if the length increases, then
the radius of wire decreases. Poisson’s ratio (s) has no units
Interpret (a) Here, V1 = 1L, p1 = 0.72 mm of Hg column, and dimensions.
V2 = 0.9 L,p2 = ? 1
Theoretically, -1 < s =
p1V1 = p2V2 2
p1V1 0.72 ´ 1 1
or p2 = = = 0.8 mm of Hg column. Practically, 0 < s <while practically no substance has
V2 0.9 2
Stress = increase in pressure been found for which s is negative.
Telegram @unacademyplusdiscounts

404 JEE Main Physics

Important Points for Modulus of Elasticity (Y, K and h )


1. The value of modulus of elasticity (Y, K and h) is independent of the 7. If a liquid of density r, volume V and bulk modulus K is compressed,
magnitude of the stress and strain. It depends only on the nature of then its density increases
the material of the body. M
As density r = ,
2. There are three modulii of elasticity i .e ., Y, K and h while elastic V
constants are four i .e ., Y, K, hand s. Poisson’s ratio s is not modulus Dr DV
so, =- ...(i)
of elasticity as it is the ratio of two strains and not of stress to strain. r V

Elastic constants are found to depend on each other through the But by definition of bulk modulus
relations -V Dp
K =-
Y = 3K (1 - 2 s) and Y = 2 h(1 + s) DV
DV Dp
Eliminating s or Y between these, we get Þ = ...(ii)
V K
9 Kh 3K - 2 h
Y = and s = From Eqs. (i) and (ii), we get
3K + h 6K + 2 h
Dr r ¢ - r Dp
= =
3. The modulii of elasticity has same dimensional formula and units as r r K
that of stress since strain is dimensionless i .e ., the dimensional
Dp ö
formula for Y, K or h is [ML–1T –2 ], while unit is dyne cm–2 or newton or r ¢ = r æç1 + ÷ = r (1 + C Dp )
è K ø
m–2.
4. Greater the value of modulii of elasticity, more elastic is the material. 8. In designing, a beam for its use to support a load (in construction of
æ 1ö roofs and bridges) it is advantageous to increase its depth rather than
AsY µ æç ö÷ , K µ
1 1
and h µ ç ÷ .
è Dl ø DV è fø the breadth of the beam because the depression in rectangular beam
Wl 3 WL 3
5. The modulii of elasticity Y and h exist only for solids as liquids and is d = 3
and the circular depression d =
4Ybd 12 pr4Y
gases cannot be deformed along one dimension only and also cannot
b
sustain shear strain. However, K exists for all states of matter i .e ., d
solid, liquid and gas. l
6. Gases being most compressible are least elastic while solids are most
δ
i .e ., the bulk modulus of gases is very low while that for liquids and
solids is very high i .e .,
Esolid > Eliquid > Egas

11.8 Work Done or Potential dW = F dx = (YA/l ) x dx

Energy Stored in a So, total work done in increasing the length by Dl


Dl YA
Stretched Wire W =ò
0 l
x dx

1 YA
When a wire is stretched work is done against the = (Dl ) 2
interatomic forces. This work is stored in the wire in the 2 l
form of elastic potential energy. \Work done per unit volume
2
Let us consider a wire of length l and A is the cross-section W 1 æ Dl ö 1
= Y ç ÷ = Y (strain) 2
area. If a force F acts along the length of the wire and V 2 è l ø 2
stretches it by x, then
æ Dl ö
Stress F /A Fl çQ V = Al and strain = ÷
Y = = = è l ø
Strain x/l Ax
W 1
YA or = Y ´ strain ´ strain
F = x V 2
l
æ Stress ö
So, work done for an additional small increase dx in çQ Y = or Y ´ strain = stress÷
è Strain ø
length
Telegram @unacademyplusdiscounts

Properties of Solids 405

W 1
or = Stress ´ strain
V 2 11.9 Thermal Stresses and Strains
W 1 F Dl When a body is allowed to expand or contract with
or = ´
Al 2 A l increasing temperature or decreasing temperature, no
1 stresses are induced in the body. But if the deformation of
or W= F Dl
2 the body is prevented, some stresses are induced in the
1 body. Such stresses are called thermal stresses or
= Load ´ elongation temperature stresses. The corresponding strains are called
2
thermal strains or temperature strains.
Thus, stored elastic energy is
1 l, α, Y, A
U= Stress ´ strain ´ volume A B
2
1
U= Y (Strain) 2 ´ volume
2
1 A
U= Load ´ elongation ∆l
2
A body having linear dimensions is shown in above figure.
Sample Problem 12 A 45 kg boy whose leg bones are Let the temperature of the rod is increased by an amount t.
5 cm2 in area and 50 cm long falls through a height of 2 m The length of the rod would increase by an amount Dl, if it
without breaking his leg bones. If the bones can stand a stress of were not fixed at two supports. Here,
0.9 ´ 10 8 Nm -2, then the Young’s modulus for the material of
Dl = l at
the bone is (Use, g = 10 ms-2)
But since the rod is fixed at the supports a compressive
(a) 2.25 ´ 10 7 Nm-2
strain will be produced in the rod. Because at the
(b) 2.25 ´ 10 9 Nm-2 increased temperature, the natural length of the rod is
(c) 8.5 ´ 10 7 Nm-2 l + Dl, while being fixed at two supports its actual length is
(d) 8.5 ´ 10 9 Nm-2 l. Hence, thermal strain
Dl l at
e= = = at
Interpret (b)Here, m = 45 kg; h = 2 m ; l l
L = 0.50 m; A = 5 ´ 10 -4 m2 or e = at
Therefore, thermal stress
Loss in gravitational energy
S = Ye (Q Stress = Y ´ strain)
= gain in elastic energy in both leg bones or S = Yat
æ1 ö
So, mgh = 2 ´ ç ´ stress ´ strain ´ volume÷
è2 ø
Check Point 2
Here, volume = AL = 5 ´ 10 -4 ´ 0.50
1. The ratio stress/strain remains constant for small deformation.
= 2.5 ´ 10 –4 m3 What will be the effect on this ratio, when the deformation
é1 ù made is very large?
\ 45 ´ 10 ´ 2 = 2 ´ ê ´ 0.9 ´ 10 8 ´ strain ´ 2.5 ´ 10 –4 ú
ë2 û 2. What is the value of Young’s modulus for a perfectly rigid
body?
45 ´ 10 ´ 2
or Strain = = 0.04 3. How does Young’s modulus change with rise in temperature?
0.9 ´ 2.5 ´ 10 4
4. What is the value of bulk modulus for an incompressible
Stress 0.9 ´ 10 8 liquid?
\ Y= =
Strain 0.04 5. Why do we prefer steel to copper in the manufacture of
= 2.25 ´ 10 9 Nm–2 spring?
Telegram @unacademyplusdiscounts

WORKED OUT
Examples
Example 1 A steel wire of length 4.7 m and cross-section A = p (r22 - r12 ) = p [(0.4) 2 - (0.3) 2] = 0.07pm2
. ´ 10 -5 m 2 stretches by the same amount as a copper wire of
30 The mass supported on the four columns, M = 50 ,000 kg.
length 3.5 m and cross-section 40 . ´ 10 -5 m 2 under a given Therefore, compressional froce on one column
load. What is the ratio of the Young's modulus of steel to that of Mg 50 ,000 ´ 9.8
F= = N
copper? 4 4
(a) 1.5 : 2 (b) 1.8 : 2 F/A
Now. Y=
(c) 1.5 : 1 (d) 1.8 : 1 Dl / l
Therefore, compressional strain,
Solution Given, for steel wire,
Dl F 50,000 ´ 9.8
A1 = 3.0 ´ 10 -5 m2 ; l1 = 4.7 m ; = = = 2.785 ´ 10 - 6
l AY 4 ´ 0.07p ´ 2.0 ´ 10 11
Dl1 = Dl ; F1 = F
For copper wire, Example 3 A solid ball 3 cm in diameter, is submerged in a
A2 = 4.0 ´ 10 -5m2 ; lake to a depth where the pressure is 103 kgfm -2. What is the
l2 = 3.5 m , Dl2 = Dl; F2 = F change in volume of the ball, if bulk modulus of the material of
the ball is 107 dyne cm -2?
Let Y1, Y2 be the Young's modulus of steel wire and copper wire
respectively. (a) 0.15 cm -3 (b) 1.5 cm3
F l (c) 1.386 cm3 (d) 0.1386 cm3
\ Y1 = 1 ´ 1
A1 Dl1 Solution Here, 2r = 3 ´ 10 -2 m
F 4.7
= -5
´ …(i) or r = (3 / 2) ´ 10 -2 m
3.0 ´ 10 Dl
F2 ´ l2 F ´ 3.5 Dp = 10 kgfm-2 = 10 3 ´ 9.8 Nm - 2
and Y2 = = …(ii)
A2 ´ Dl2 4 ´ 10 -5 ´ Dl K = 10 7 dyne cm2 = 10 6 Nm-2
3
Y1 4.7 ´ 4 ´ 10 -5 4 3 4 22 æ 3 ö
= = 1.8 Volume of the ball, V = pr = ´ ´ ç ´ 10 -2÷ m3
Y2 3.5 ´ 3.0 ´ 10 -5 3 3 7 è2 ø
Dp ´ V VDp
Now, Hence,K =:Y1 2 Y= 1.8 or
: 1VD =
DV K
3
Example 2 Four identical hollow chlindrical columns of 4 22 æ 3 ö
´ ´ ç ´ 10 -2÷ ´ 10 3 ´ 9.8
steel support a big structure of mass 50,000 kg. The inner and 3 7 è3 ø
=
outer radii of each column are 30 cm and 40 cm respectively. 10 6

Assuming the load distribution to be uniform, what will be the = 0.1386 ´ 10 -6 m3 = 0.1386 cm3
compressional strain of each column? The Young's modulus of
steel is 2.0 ´ 1011 Pa.
Example 4 A rubber cube of each side 7 cm has one side
(a) 2.7 ´ 10 -6 (b) 2.7 ´ 10 -4 fixed, while a tangential force equal to the weight of 300 kgf is
(c) 8.3 ´ 10 -6 (d) 8.3 ´ 10 -4 applied to the opposite face. What is the shearing strain
produced and the distance through which the strained side
Solution Here, Y = 2.0 ´ 10 11 Pa
moves? The modulus of rigidity for rubber is 2 ´ 107 dyne cm -2,
Inner radius of each column, r1 = 30 cm = 0.3 m
g = 10 ms -2.
Outer radius of each column, r2 = 40 cm = 0.4 m
(a) 0.3 rad, 2.1 cm (b) 0.2 rad, 2.1 cm
Therefore, area of cross-section of each column.
(c) 0.4 rad, 2.1 cm (d) 0.3 rad, 2.5 cm
Telegram @unacademyplusdiscounts

Properties of Solids 407

2
Solution Given l = 4 cm = 7 ´ 10 -2 m UA é lA ù æ rB ö
\ =ê ú ç ÷
F = 300 kgf = 300 ´ 10 N UB ë lB û è rA ø
2
h = 2 ´ 10 7 dyne cm-2 æ 1ö 3
= (3) ´ ç ÷ =
è2ø 4
= 2 ´ 10 6 Nm-2
F/A
As, h= Example 6 The strain-stress curves of three wires of
q
F F
different materials are shown in the figure. P, Q and R are the
or q= = elastic limits of the wires, The figure shows that
Ah l 2h
300 ´ 10 P
= = 0.3 rad Q

Strain
(7 ´ 10 -2 ) 2 ´ 2 ´ 10 6 R
Dl
q=
l
Stress
or Dl = l q = 7 ´ 0.3 = 2.1 cm
(a) elasticity of wire P is maximum
Example 5 Wires A and B are made from the same material. (b) elasticity of wire Q is maximum
A has twice the diameter and three times the langth of B. If the (c) tensile strength of R is maximum
elastic limits are not reached, when each is stretched by the
(d) None of the above
same tension, the ratio of energy stored in A to that of B is
(a) 2 : 3 (b) 3 : 4 Solution As stress is shown on X-axis and strain on Y-axis. So,
(d) 3 : 2 (d) 6 : 1 1 1
we can say that y = cot q = =
1 F 2l tan q slope
Solution Elastic energy per unit volume, U = FDl =
2 2 AY
1 So, elasticity of wire P is minimum and of wire R is miximum
U µ 2 (F and Y are constants)
r
Telegram @unacademyplusdiscounts

Start Practice for


JEE Main
Round I (Topically Divided Problems)

Young’s Modulus of Elasticity 2L


1. When a certain weight is suspended from a long
x
uniform wire, its length increases by 1 cm. If the
same weight is suspended from another wire of the m
same material and length but having a diameter half
of the first one, the increase in length will be x2 x x2 x2
(a) 0.5 cm (b) 2 cm (a) (b) (c) (d)
2 L2 L L 2L
(c) 4 cm (d) 8 cm
6. A copper wire of negligible mass, 1 m length and
2. The ratio of diameters of two wires of same materials
cross-sectional area 10–6 is kept on a smooth
is n : 1. The length of each wire is 4 m. On applying
horizontal table with one end fixed. A ball of mass
the same load, the increase in length of thin wire will
1 kg is attached to the other end. The wire and the
be (n > 1)
ball are rotated with an angular velocity 20 rad s–1. If
(a) n2 times (b) n times
the elongation in the wire is 10–3 m, then the Young’s
(c) 2n times (d) (2n + 1) times
modulus is
3. A rigid bar of mass M is supported symmetrically by (a) 4 × 1011 Nm–2 (b) 6 × 1011 Nm–2
three wires each of length l. Those at each end are of (c) 8 × 1011 Nm–2 (d) 10 × 1011 Nm–2
copper and the middle one is of iron. The ratio of their 7. A uniform wire, fixed at its upper end, hangs
diameter, if each is to have the same tension, is equal vertically and supports a weight at its lower end. If
to [NCERT Exemplar] its radius is r, its length L and the Young’s modulus
Yiron for the material of the wire is E, the extension is
(a) Ycopper /Yiron (b) 1. directly proportional to E
Ycopper
2. inversely proportional to r
Y2iron Yiron 3. directly proportional to L
(c) (d) (a) if only 3 is correct (b) if 1, 2 are correct
Y2copper Ycopper
(c) if 2, 3 are correct (d) if only 1 is correct
4. A thick rope of rubber of density 1.5 ´ 103 kgm–3 and 8. The Young’s modulus of a rubber string 8 cm long
Young’s modulus 5 ´ 106 Nm–2, 8 m in length is hung and density 1.5 kg / m 3 is 5 ´ 108 N / m 2 , is suspended
from the ceiling of a room, the increase in its length on the ceilling in a room. The increase in length due
due to its own weight is to its own weight will be
(a) 9.6 × 10–2 m (b) 19.2 × 10–2 m (a) 9.5 ´ 10 –5 m (b) 9.6 ´ 10 –11 m
(c) 9.6 × 10–3 m (d) 9.6 m (c) 9.6 ´ 10 –3 m (d) 9.6 m
5. A mild steel wire of length 2L and cross-sectional 9. A steel cable with a radius of 1.5 cm supports a chair
area A is stretched, well within elastic limit, lift at a ski area. If the maximum stress is not to
horizontally between two pillars as shown in figure. exceed 108 N/m2 , what is the maximum load the
A mass m is suspended from the mid point of the cable can support? [NCERT]

wire. Strain in the wire is [NCERT Exemplar]


(a) 7 ´ 105 N (b) 7 ´ 106 N
4
(c) 7 ´ 10 N (d) 9 ´ 105 N
Telegram @unacademyplusdiscounts

Properties of Solids 409

10. A rectangular frame is to be suspended 17. On increasing the length by 0.5 mm in a steel wire of
symmetrically by two strings of equal length on two length 2 m and area of cross-section 2 mm2, the force
supports (Fig.) It can be done in one of the following required is [Y for steel = 2.2 ´ 1011 Nm–2 ]
three ways; [NCERT Exemplar] (a) 1.1 × 105 N (b) 1.1 × 104 N
(c) 1.1 × 103 N (d) 1.1 × 102 N
18. Two wires of the same material and length are
stretched by the same force. Their masses are in the
ratio 3:2. Their elongations are in the ratio
(a) 3 : 2 (b) 9 : 4
(i) (ii) (iii) (c) 2 : 3 (d) 4 : 9

The tension in the strings will be 19. A 100 N force stretches the length of a hanging wire
(a) the same in all cases (b) least in (i) by 0.5 mm. The force required to stretch a wire, of the
(c) least in (ii) (d) least in (iii) same material and length but having four times the
diameter, by 0.5 mm is
11. A 1 m long wire is stretched without tension at 30°C (a) 100 N (b) 400 N
between two rigid supports. What strain will be (c) 1200 N (d) 1600 N
produced in the wire if the temperature falls to 0°C ?
(Given, a = 12 ´ 10-6 K -1 20. Two wires of the same length and same material but
(a) 36 ×10–5 (b) 64 ×10–5 radii in the ratio of 1 : 2 are stretched by unequal
(c) 0.78 (d) 0.32 forces to produce equal elongation. The ratio of the
two forces is
12. If x is longitudinal strain produced in a wire of (a) 1 : 1 (b) 1 : 2
Young’s modulus Y , then energy stored in the (c) 2 : 3 (d) 1 : 4
material of the wire per unit volume is
1 1 21. Two wires of the same material have lengths in the
(a) Yx2 (b) 2 Yx2 (c) Y 2 x (d) Yx2
2 2 ratio 1 : 2 and their radii are in the ratio 1 : 2. If they
are stretched by applying equal forces, the increase
13. Two identical wires are suspended from the same
in their lengths will be in the ratio of
rigid support but one is of copper and the other is of
(a) 2 : 2 (b) 2 : 2
iron. Young’s modulus of iron is thrice that of copper.
The weights to be added on copper and iron wires so (c) 1:1 (d) 1:2
that the ends are on the same level must be in the 22. A wire of length L and radius r is clamped rigidly at
ratio of one end. When the other end of the wire is pulled by a
(a) 1 : 3 (b) 2 : 1 (c) 3 : 1 (d) 4 : 1 force F its length increases by l. Another wire of the
14. The temperature of a wire of length 1 m and area of same material of length 4 L, radius 4r is pulled by a
cross-section 1 cm2 is increased from 0°C to 100°C. If force 4F. The increase in length will be
l
the rod is not allowed to increased in length, the force (a) (b) l
2
required will be (a = 10–5/°C and Y = 1011 N/m2)
(a) 103 N (b) 104 N (c) 10 (c) 2l (c) 4l

15. A substance breaks down by a stress of 106 Nm–2. 23. When a weight of 5 kg is suspended from a copper
If the density of the material of the wire is wire of length 30 m and diameter 0.5 mm, the length
3 ´ 103 kgm -3, then the length of the wire of that of the wire increases by 2.4 cm. If the diameter is
substance which will break under its own weight doubled, the extension produced is
(a) 1.2 cm (b) 0.6 cm
when suspended vertically is nearly
(a) 3.4 m (b) 34 m (c) 0.3 cm (d) 0.15 cm
(c) 340 m (d) 3400 m 24. The length of a wire is increased by 1 mm on
16. The dimensions of four wires of the same material the application of a given load. In a wire of the same
are given below. In which wire the increase in length material, but of length and radius twice that of
will be maximum? the first, on the application of the same load,
(a) Length 100 cm, Diameter 1 mm extension is
(b) Length 200 cm, Diameter 2 mm (a) 0.25 mm
(c) Length 300 cm, Diameter 3 mm (b) 0.5 mm
(d) Length 50 cm, Diameter 0.5 mm (c) 2 mm
(d) 4 mm
Telegram @unacademyplusdiscounts

410 JEE Main Physics

25. An aluminium rod, Young’s modulus 7.0 ´ 109 Nm –2 , 33. A steel wire has length 2 m, radius 1 mm and
has a breaking strain of 0.2%. The minimum Y = 2 ´ 1011 Nm -2 . A 1 kg sphere is attached to one
cross-sectional area of the rod in m 2 in order to end of the wire and whirled in a vertical circle with
support a load of 104 N is an angular velocity of 2 revolutions per second. When
(a) 1 ×10–2 (b) 1.4 × 10–3 the sphere is at the lowest point of the vertical circle,
(c) 1.0 ×10–3 (d) 7.1 × 10– 4 the elongation of the wire is nearly
26. A substance breaks down by a stress of 106 Nm–2. If (Take, g = 10 ms–2)
(a) 1 mm (b) 2 mm (c) 0.1 mm (d) 0.01 mm
the density of the material of the wire is
3 ´ 103 kg m -3, then the length of the wire of the 34. Two wires of equal cross-section but one made of steel
substance which will break under its own weight and the other of copper are joined end to end. When
when suspended vertically is the combination is kept under tension, the
(a) 66.6 m (b) 60.0 m elongations in the two wires are found to be equal.
(c) 33.3 m (d) 30.0 m What is the ratio of the lengths of the two wires?
27. A steel ring of radius r and cross-sectional area A is (Given, Y for steel = 2 ´ 1011Nm–2)
(a) 2 : 11 (b) 11 : 2 (c) 20 : 11 (d) 11 : 20
fitted on a wooden disc of radius R ( R > r). If Young’s
modulus be E, then the force with which the steel 35. The Young’s modulus of brass and steel are
ring is expanded is 10 ´ 1010 Nm -2 and 2 ×1011 Nm–2 respectively. A brass
R (R - r ) wire and a steel wire of the same length are extended
(a) AE (b) AE
r r by 1 mm under the same force. The radii of the brass
E æR - rö Er and steel wires are RB and RS respectively. Then
(c) ç ÷ (d)
Aè A ø AR RB
(a) RS = 2RB (b) RS =
2
28. Find the extension produced in a copper of length 2 m RB
and diameter 3 mm, when a force of 30 N is applied. (c) RS = 4 RB (d) RS =
4
Young’s modulus for copper = 1.1 ´ 1011 Nm–2
(a) 0.2 mm (b) 0.04 mm 36. When the tension in a metal wire is T1, its length is l1.
(c) 0.08 mm (d) 0.68 mm When the tension is T2 , its length is l2 . The natural
length of wire is
29. A wire extends by 1 mm when a force is applied. T2
Double the force is applied to another wire of same (a) ( l1 + l2 ) (b) T1l1 + T2 l2
T1
material and length but half the radius of
lT - lT l1T2 + l2T1
cross-section. The elongation of the wire in mm will (c) 1 2 2 1 (d)
T2 - T1 T2 + T1
be
(a) 8 (b) 4 (c) 2 (d) 1 37. A rubber rope of length 8 m is hung from the ceiling of
30. A 1 m long steel wire of cross-sectional area 1 mm2 is a room. What is the increase in length of the rope due
extended by 1 mm. If Y = 2 ´ 10 Nm 11 -2
, then the to its own weight? (Given Young’s modulus of
work done is elasticity of rubber = 5 ´ 106 Nm–2 and density of
(a) 0.1 J (b) 0.2 J (c) 0.3 J (d) 0.4 J rubber = 1.5 ´ 106 kgm–3. Take g = 10 ms–2 )
(a) 1.5 mm (b) 6 mm (c) 24 mm (d) 96 mm
31. A wire is stretched 1 mm by a force of 1 kN. How far
would a wire of the same material and length but of 38. Two identical wires of rubber and iron are stretched
four times that diameter be stretched by the same by the same weight, then the number of atoms in unit
force ? volume of iron wire will be
1 1 (a) equal to that of rubber
(a) mm (b) mm
2 4 (b) less than that of the rubber
1 1 (c) more than that of the rubber
(c) mm (d) mm
8 16 (d) None of the above

32. Two bars A and B of circular cross-section and of 39. Two wires, one made of copper and other of steel are
same volume and made of the same material are joined end to end (as shown in figure). The area of
subjected to tension. If the diameter of A is half that cross-section of copper wire is twice that of steel wire.
of B and if the force applied to both the rods is the
same and it is in the elastic limit, the ratio of F
Copper Steel
F
extension of A to that of B will be
(a) 16 : 1 (b) 8 : 1 (c) 4 : 1 (d) 2 : 1
Telegram @unacademyplusdiscounts

Properties of Solids 411

They are placed under compressive force of 44. The length of an elastic string is a metre when the
magnitudes F. The ratio for their lengths such that tension is 44 N, and b metre when the tension is 5 N.
change in lengths of both wires are same is (YS = 2 ´ The length in metre when the tension is 9 N, is
1011 Nm–2 and YC = 1.1 ´ 1011 Nm–2) (a) 4 a - 5 b (b) 5 b - 4 a
(a) 2.1 (b) 1.1 (c) 1.2 (d) 2 (c) 9 b - 9 a (d) a + b
40. A uniform slender rod of length L, cross-sectional area 45. The ratio of two specific heats of gas C p / CV for argon
A and Young’s modulus Y is acted upon by the forces is 1.6 and for hydrogen is 1.4. Adiabatic elasticity of
shown in the figure. The elongation of the rod is argon at pressure p is E. Adiabatic elasticity of
L hydrogen will also be equal to E at the pressure
8
(a) p (b) p
7
3F 2F 7
(c) p (d) 1.4 p
3FL 2FL 3FL 8FL 8
(a) (b) (c) (d)
5 AY 5 FY 8 AY 3 AY 46. Consider two cylindrical rods of identical
dimensions, one of rubber and the other of steel. Both
Bulk Modulus, Stress, Strain the rods are fixed rigidly at one end to the roof. A
mass M is attached to each of the free ends at the
41. Two wires of the same material centre of the rods. [NCERT Exemplar]
(Young’s modulus Y) and same length (a) Both the rods will elongate but there shall be no
L but radii R and 2R respectively are L, 2R perceptible change in shape
joined end to end and a weight w is (b) The steel rod will elongate and change shape but the
suspended from the combination as rubber rod will only elongate
shown in the figure. The elastic L, R
(c) The steel rod will elongate without any perceptible
potential energy in the system is change in shape, but the rubber rod will elongate and the
w
3w 2 L 3w 2 L shape of the bottom edge will change to an ellipse.
(a) (b)
4 pR2Y 8pR2Y (d) The steel rod will elongate, without any perceptible
5w2L w2L change in shape, but the rubber rod will elongate with
(c) (d) the shape of the bottom edge tapered to a tip at the
8pR2Y pR2Y
centre
42. A load suspended by a massless spring produces an
extension of x cm, in equilibrium. When it is cut into
47. If the compressibility of water is s per unit
atmospheric pressure, then the decrease in volume
two unequal parts, the same load produces an
(V) due to atmospheric pressure p will be
extension of 7.5 cm when suspended by the larger
(a) s p /V (b) spV
part of length 60 cm. When it is suspended by the
(c) s /pV (d) sV / p
smaller part, the extension is 5.0 cm. Then
(a) x = 12.5 48. A cube is compressed at 0°C equally from all sides by
(b) x = 3.0 an external pressure p. By what amount should be
(c) the length of the original spring is 90 cm temperature be raise to bring it back to the size it had
(d) the length of the original spring is 80 cm before the external pressure was applied? (Given K is
bulk modulus of elasticity of the material of the cube
43. In the figure three identical springs are shown. From
and a is the coefficient of linear expansion.)
spring A, a mass of 4 kg is hung and spring shows p p
elongation of 1 cm. But when a weight of 6 kg is hung (a) (b)
Ka 3Ka
on B, the hook descends through 3pa K
(c) (d)
p 3p

A A 49. When a 4 kg mass is hung vertically on a light spring


that obeys Hooke’s law, the spring stretches by 2 cm.
B The work required to be done by an external agent in
stretching this spring by 5 cm will be
(a) 4.9 J
(b) 2.45 J
(a) 1 cm (b) 2 cm (c) 0.495 J
(c) 3 cm (d) 4 cm (d) 0.245 J
Telegram @unacademyplusdiscounts

412 JEE Main Physics

50. A spring is extended by 30 mm when a force of 1.5 N 58. A spherical ball contracts in volume by 0.01% when
is applied to it. Calculate the energy stored in the subjected to a normal uniform pressure of 100
spring when hanging vertically supporting a mass of atmosphere. What is the bulk modulus of elasticity
0.20 kg if the spring was instructed before applying of the material of the ball ?
the mass? (Take, 1 atmosphere = 106 dyne cm–2)
(a) 0.01 J (b) 0.02 J (a) 109 dyne cm–2 (b) 1010 dyne cm–2
(c) 0.04 J (d) 0.08 J (c) 1012 dyne cm–2 (d) 1014 dyne cm–2
51. How much should the pressure on a litre of water be 59. When a rubber cord is stretched, the change in
changed to compress it by 0.10%. Bulk modulus of volume with respect to change in its linear
elasticity of water = 2.2 ´ 109 N/m2 . dimensions is negligible. The Poisson’s ratio for
(a) 2.2 ´ 10-6 N-m-2 (b) 4.2 ´ 10-6 N-m-2 rubber is
(c) 2.2 ´ 106 N-m2 (d) 4.2 ´ 104 N-m2 (a) 1 (b) 0.25
(c) 0.5 (d) 0.75
52. Compute the fractional change in volume of a glass
slab, when subjected to a hydraulic pressure of 60. A cube is subjected to a uniform volume compression.
10 atm [NCERT] If the side of the cube decreases by 1% the bulk strain
(a) 2.74 ´ 10 -5
(b) 3.74 ´ 10 -5 is
(a) 0.01 (b) 0.02
(c) 1.74 ´ 10 -5 (d) None of these
(c) 0.03 (d) 0.06
53. A height spring extends 40 mm when stretched by a 61. For most materials is Young’s modulus is n times, the
force of 10 N, and for tensions upto this value the rigidity modulus, where n is
extension is proportional to the stretching force. Two (a) 2 (b) 3 (c) 4 (d) 6
such springs are joined end-to-end and the
double-length spring is stretched 40 mm beyond its 62. A wire of Young’s modulus 1.5 × 1012 Nm–2 is
natural length. The total strain energy in (joule), stretched by a force so as to produce a strain of
stored in the double spring is 2 ´ 104. The energy stored per unit volume is
(a) 0.05 (b) 0.10 (c) 0.80 (d) 0.40 (a) 3 × 108 Jm–3 (b) 3 × 103 Jm–3
(c) 6 × 103 Jm–3 (d) 3 × 104 Jm–3
54. A cube is shifted to a depth of 100 m in a lake. The
change in volume is 0.1%. The bulk modulus of the 63. In the three states of matter, the elastic coefficient
material is nearly can be
(a) 10 Pa (b) 104 Pa (a) Young’s modulus
(c) 107 Pa (d) 106 Pa (b) coefficient of volume elasticity
(c) modulus of rigidity
55. A copper bar of length L and area of cross-section A is (d) Poisson’s ratio
placed in a chamber at atmospheric pressure. If
the chamber is evacuated, the percentage change 64. The force constant of a wire is k and that of another
in its volume will be (compressibility of copper is wire of the same material is 2k. When both the wires
8 × 1012 m2 N–1 and 1 atm = 105 N m–2) are stretched, then work done is
=
(a)1.5
W(a)W8 ×10–7 (b)=W2 W –5
2 1 2 1
(c) 1.25 ×10– 4 (d) 1.25 ×10–5 (c) W2 = W1 (d) W2 = 0.5 W1
56. An elastic material of Young’s modulus Y is subjected 65. Modulus of rigidity of ideal liquids is [NCERT Exemplar]
to a stress S. The elastic energy stored per unit (a) infinity
volume of the material is (b) zero
SY S2
(a) (b) (c) unity
2 2Y (d) some finite small non-zero constant value
S 2S
(c) (d)
2Y Y 66. One end of steel wire is fixed to ceiling of an elevator
moving up with an acceleration 2 ms–2 and a load of
57. A ball falling in a lake of depth 200 m shows a
decrease of 0.1% in its volume at the bottom. The 10 kg hangs from other end. Area of cross-section of
bulk modulus of elasticity of the material of the ball the wire is 2 cm2. The longitudinal strain in the wire
is (Take g =10 ms–2) is (Take g = 10 ms–2 and Y = 2 × 1011 Nm–2)
(a) 109 Nm–2 (b) 2 × 109 Nm–2 (a) 4 × 1011 (b) 3 × 10–6
(c) 3 × 109 Nm–2 (d) 4 × 109 Nm–2 (c) 8 × 10–6 (d) 2 × 10–6
Telegram @unacademyplusdiscounts

Properties of Solids 413

67. A wire suspended vertically from one of its ends is w1 é wù


(a) (b) ê w1 + ú s
stretched by attaching a weight of 200 N to the lower s ë 4û
end. The weight stretches the wire by 1 mm. Then, é 3w ù w +w
(c) ê w1 + s (d) 1
the elastic energy stored in the wire is ë 4 úû s
(a) 0.2 J (b) 10 J (c) 20 J (d) 0.1 J
75. Two rods A and B of the same material and length
68. A body of mass m = 10 kg is attached to a wire of have their radii r1 and r2 respectively. When they are
length 0.3 m. The maximum angular velocity with rigidly fixed at one end and twisted by the same
which it can be rotated in a horizontal circle is couple applied at the other end, the ratio of the angle
(Breaking stress of wire = 4.8 × 107 Nm–2 and area of of twist at the end of A and the angle of twist at the
cross-section of a wire = 10–2 m2) end of B is
(a) 4 rads–1 (b) 8 rads–1 r24 r14 r22 r12
(c) 1 rads–1 (d) 2 rads–1 (a) (b) (c) (d)
r14 r24 r12 r22
69. If the shear modulus of a wire material is
76. What among of work is done in increasing the length
5.9 ´ 1011 dyne cm –2 , then the potential energy of a
of a wire though unity?
wire of 4 ´ 103 cm in diameter and 5 cm long twisted
YL YL2 YA YL
through an angle of 10¢, is (a) (b) (c) (d)
2A 2A 2L A
(a) 1.253 × 10–12
J (b) 2.00 × 10–12J
(c) 1.00 × 10–12 J (d) 0.8 × 10–12 J 77. If the work done in stretching a wire by 1 mm is 2 J,
the work necessary for stretching another wire of
70. Two cylinders of same material and of A same material but with double radius of cross-section
same length are joined to end as
and half the length by 1 mm is
shown in figure. The upper end of A is 1
rigidly fixed. Their radii are in ratio of (a) J (b) 4 J
B 4
1 : 2. If the lower end of B is twisted by
(c) 8 J (d) 16 J
an angle q, the angle of twist of
cylinder A is 78. A wire (Y = 2 ´ 1011
Nm–2) has length 1 m and
15 16 16 17 cross-sectional area 1 mm2. The work required to
(a) q (b) q (c) q (d) q increase the length by 2 mm is
16 15 17 16
(a) 0.4 J (b) 4 J (c) 40 J (d) 400 J
71. In a wire stretched by hanging a weight from its end,
79. In above question, the work done in the two wires is
the elastic potential energy per unit volume in terms
(a) 0.5 J, 0.03 J (b) 0.25 J, 0 J
of longitudinal strain s and modulus of elasticity Y is
(c) 0.03 J, 0.25 J (d) 0 J, 0 J
Ys2 Ys
(a) (b)
2 2 80. A copper wire 2 m long is stretched by 1 mm. If the
2Ys2 Y2s energy stored in the stretched wire is converted to
(c) (d) heat, calculate the rise in temperature of the wire.
2 2
(Given, Y = 12 ´ 1011 dyne cm–2, density of copper =
72. Two wires of the same material and length but 9 gcm–3 and specific heat of copper = 0.1 cal g -1° C-1)
diameters in the ratio 1 : 2 are stretched by the same (a) 252°C (b) (1/252)°C
force. The potential energy per unit volume for the (c) 1000°C (d) 2000°C
two wires when stretched will be in the ratio
(a) 16 : 1 (b) 4 : 1 81. The stress versus strain graphs for wires of two
(c) 2 : 1 (d) 1 : 1 materials A and B are as shown in the figure. If y A
and yB are the Young’s modulus of the materials, then
73. A metal rod of Young’s modulus 2 ´ 1010 Nm–2
undergoes an elastic strain of 0.06%. The energy per A
unit volume stored in Jm–3 is
(a) 3600 (b) 7200 Stress B
(c) 10800 (d) 14400
74. One end of uniform wire of length L and of weight w 60°
is attached rigidly to a point in the roof and a weight 30°
X
Strain
w1 is suspended from its lower end. If s is the area of
cross-section of the wire, the stress in the wire at a (a) y B = 2 y A (b) y A = y B
height (3L/4) from its lower end is (c) y B = 3 y A (d) y A = 3 y B
Telegram @unacademyplusdiscounts

414 JEE Main Physics

82. The upper end of a wire of radius 4 mm and length 88. Equal torsional torques act on two rods x and y
100 cm is clamped and its other end is twisted having equal length. The diameter of rod y is twice
through an angle of 30°. Then angle of shear is the diameter of rod x. If q x and q y are the angles of
(a) 12° (b) 0.12° (c) 1.2° (d) 0.012° q
twist, then x is equal to
qy
83. If Eq and Ef denote the isothermal and adiabatic
(a) 1 (b) 2 (c) 4 (d) 16
elasticities respectively of a gas, then Eq / Ef
(a) < 1 (b) > 1 (c) = 1 (d) = 3.2 89. The increase in length on stretching a wire is 0.05%.
84. For a given material, the Young’s modulus is 2.4 If its Poisson’s ratio is 0.4, the diameter is reduced by
(a) 0.01% (b) 0.02% (c) 0.03% (d) 0.04%
times that of modulus of rigidity. Its Poisson’s ratio is
(a) 0.1 (b) 0.2 (c) 0.3 (d) 0.4 90. A force F is required to break a wire of length l and
85. Forces of 100 N each are applied in opposite radius r. What force is required to break a wire, of the
directions on the upper and lower faces of a cube of same material, having twice the length and six times
side 20 cm. The upper face is shifted parallel to itself the radius?
(a) F (b) 3 F
by 0.25 cm. If the side of the cube were 10 cm, then
(c) 9 F (d) 36 F
the displacement would be
(a) 0.25 cm (b) 0.5 cm 91. The following data were obtained when a wire was
(c) 0.75 cm (d) 1 cm stretched within the elastic region
86. The compressibility of water is 6 ´ 10-10 N –1m 2 . If one Force applied to wire 100 N
litre is subjected to a pressure of 4 ´ 107 Nm –2 . The Area of cross-section of wire 10–6 m2
decrease in its volume is Extension of wire 2 ´ 10–9m
(a) 2.4 cc (b) 10 cc (c) 24 cc (d) 15 cc Which of the following deductions can be correctly
made from this data?
87. The Young’s modulus of the material of a wire is
1. The value of Young’s modulus is 1011 Nm–2
6 ´ 1012 Nm–2 and there is no transverse strain in it,
then its modulus of rigidity will be 2. The strain is 10–3
(a) 3 × 1012 Nm–2 3. The energy stored in the wire when the load is
(b) 2 ×1012 Nm–2 applied is 10 J
(c) 1012 Nm–2 (a) 1, 2, 3 are correct (b) 1, 2 are correct
(d) None of the above (c) 1 only (d) 3 only

Round II (Mixed Bag)


Only One Correct Option 3. One end of a uniform rod of mass m1, uniform area of
cross-section A is suspended from the roof and mass
1. The ratio of lengths, radii and Young’s
2 Brass m is suspended from the other end. What is the
modulus of steel and brass wires shown
stress at the mid point of the rod?
in the figure are a, b and c respectively.
2 kg (a) ( m1 + m2 ) g / A
The ratio between the increase in
(b) ( m1 - m2 ) g / A
length of brass and steel wires would be Steel
é ( m / 2) + m2 ù
b2 a bc (c) ê 1 úû g
(a) (b) 2 ë A
2c 2a 4 kg
é m + ( m2 / 2) ù
ba2 c (d) ê 1 úû g
(c) (d) 2 ë A
2c 2b a
4. One litre of a gas is maintained at pressure 72 cm of
2. The figure shows the stress A mercury. It is compressed isothermally so that its
-strain graph of a certain volume becomes 900 cm3. The value of stress and
Stress

substance. Over which region B strain will be respectively


of the graph is Hooke’s Law (a) 0.106 Nm–2 and 0.1
C
obeyed ? (b) 1.106 Nm–2 and 0.1
D
(a) BC (b) CD (c) 106.62 Nm–2 and 0.1
(c) AB (d) OD O (d) 10662.4 Nm–2 and 0.1
Strain
Telegram @unacademyplusdiscounts

Properties of Solids 415

5. When a weight w is hung from one end of the wire 12. A load of 4.0 kg is suspended from a ceiling through a
other end being fixed, the elongation produced in it steel wire of length 2.0 m and radius 2.0 mm. It is
be l. If this wire goes over a pulley and two weights w found that the length of the wire increases
each are hung at the two ends, the elongation of the by 0.031 mm as equilibrium is achieved.
wire will be Taking, g = 3.1 p ms–2, the Young’s modulus of steel
(a) 4 l (b) 2 l (c) l (d) l / 2 is
6. When a force is applied on a wire of uniform cross- (a) 2.0 × 108 Nm–2 (b) 2.0 × 109 Nm–2
(c) 2.0 × 1011 Nm–2 (d) 2.0 × 1013 Nm–2
sectional area 3 ´ 10–6 m2 and length 4 m, the
increase in length is 1 mm. Energy stored in it will be 13. A steel wire of length 4.7 m and cross-sectional area
(Y = 2 ´ 1011 Nm–2) 3.0 ´ 10-5 m2 stretches by the same amount as a
(a) 6250 J (b) 0.177 J copper wire of length 3.5 m and cross-sectional area
(c) 0.075 J (d) 0.150 J 4.0 ´ 10-5 m 2 under a given load. The ratio of the
7. A wire of cross-sectional area A is stretched Young’s modulus of steel to that of copper is [NCERT]
horizontally between two clamps loaded at a distance (a) 1.2 (b) 1.8 (c) 1.5 (d) 1.19
2 l metres from each other. A weight w kg is suspended 14. A uniform cube is subjected to volume compression of
from the mid point of the wire. The strain produced in each side is decreased by 1%, then the bulk strain is
the wire, (if the vertical distance through which the (a) 0.01 (b) 0.06
(c) 0.02 (d) 0.03
mid point of the wire moves down x < l) will be
(a) x2 / l2 (b) 2x2 l 2 15. A solid block of silver with density 10.5 ´ 103 kg m–3
(c) x2 / 2 l 2 (d) x/2 l is subjected to an external pressure of 107 Nm–2. If
the bulk modulus of silver is 17 ´ 1010 Nm–2, the
8. The graph shown was obtained T2 change in density of silver (in kg m–3) is
from the experimental (a) 0.61 (b) 1.7 (c) 6.1 (d) 17 ´ 103
measurements of the period of
oscillation T for different masses 16. A rigid bar of mass 15 kg is supported symmetrically
M placed in the scale pan on the by three wires each 2 m long. These at each end are of
lower end of the spring balance. copper and middle one is of iron. Determine the ratio
The most likely reason for the line
M of their diameters if each is to have the same tension.
not passing through the origin is that Young’s modulus of elasticity for copper and steel are
(a) spring did not obey Hooke’s law 110 ´ 109 N/m2 and 190 ´ 109 N/m2 respectively.
(b) amplitude of oscillation was too large [NCERT]
(c) clock used needed regulation (a) 1 : 1.3 (b) 1.3 : 1 (c) 2.3 : 1.3 (d) 2.3 : 1
(d) mass of the pan was not neglected
17. A stress of 1 kg mm2 is applied on a wire. If the
9. What is the increase in elastic potential energy when modulus of elasticity of the wire is 1010 dyne cm–2,
the stretching force is increased by 200 kN? then the percentage increase in the length of the wire
(a) 238.5 J (b) 636.0 J (c) 115.5 J (d) 79.5 J will be
10. The work done in increasing the length of one metre (a) 0.0098% (b) 0.98% (c) 9.8% (d) 98%
long wire of cross-sectional area 1mm2 through 1 mm 18. A rectangular bar 2 cm in breadth and 1 cm in depth
will be (Y = 2 ´ 1011 Nm–2) and 100 cm in length is supported at its ends and a
(a) 0.1 J (b) 5 J load of 2 kg is applied at its middle. If Young’s
(c) 10 J (d) 250 J modulus of the material of the bar is 20 ´ 1011 dyne
11. A wire of length 2 L and radius 2L cm–2, the depression in the bar is
r is stretched between A and B x (a) 0.2450 cm (b) 0.3675 cm
(c) 0.1225 cm (d) 0.9800 cm
without the application of any
m
tension. If Y is the Young 19. Determine the volume contraction of a solid copper
modulus of the wire and it is cube, 10 cm on an edge, when subjected to a hydraulic
stretched like ACB, then the tension in the wire will pressure of 7 ´ 106 Pa. K for copper = 140 ´ 109 Pa.
be [NCERT]
pr 2Yd 3 pr 2Yd 2 p2 r 2Y × 2 L2 pr 2Y × 2 L (a) 5 ´ 10 -7 3
m (b) 4 ´ 10 -8 3
m
(a) (b) (c) (d)
2 L2 2 L2 d2 d (c) 5 ´ 10 -8 m3 (d) 6 ´ 10 -8 m3
Telegram @unacademyplusdiscounts

416 JEE Main Physics

20. A wire (Y = 2 ´ 1011 Nm–2) has length 1 m and area of 27. Figure shows a 80 cm square brass plate of thickness
cross-section 1 mm2. The work required to increase 0.5 cm. It is fixed at its bottom edge. What tangential
its length by 2 mm is force F must be exerted on the upper edge, so that the
(a) 400 J (b) 40 J displacement (x) of this edge in the direction of force
(c) 4 J (d) 0.4 J is 0.16 mm? The shear modulus of brass is
21. A steel wire of length 20 cm and uniform 3.5 ´ 1010 Pa.
cross-section 1 mm2 is tied rigidly at both the ends. F
The temperature of the wire is altered from 40°C to
20°C. Coefficient of linear expansion of steel is
a = 1.1 ´ 10–5 °C–1 and Y for steel is 2.0 ´ 1011 Nm2;
the tension in the wire is
(a) 2.2 ´ 106 N (b) 16 N
(c) 8 N (d) 44 N
(a) 2.8 ´ 10 4 N (b) 3.8 ´ 10 -4 N
22. If work done in stretching a wire by 1 mm is 2 J, the
work necessary for stretching another wire of same (c) 5 ´ 10 N 5
(d) 4 ´ 10 -5 N
material, but double the radius and half length by 28. The twisting couple per unit twist for a solid cylinder
1 mm joule is of radius 3 cm is 0.1 N-m. The twisting couple per
(a) 1/4 (b) 4
unit twist, for a hollow cylinder of same material
(c) 8 (d) 16
with outer and inner radius 5 cm and 4 cm
23. The Poisson’s ratio of a material is 0.1. If the respectively, will be
longitudinal strain of a rod of this material is 10–3, (a) 0.1 N-m (b) 0.455 N-m
then the percentage change in the volume of the rod (c) 0.91 N-m (d) 1.82 N-m
will be
(a) 0.008% (b) 0.08%
29. A solid sphere of radius r made of a material of bulk
modulus K is surrounded by a liquid in a cylindrical
(c) 0.8% (d) 8%
container. A massless piston of area a floats on the
24. The Poisson’s ratio of a material is 0.4. If a force is surface of the liquid. When a mass m is placed on the
applied to a wire of this material, there is a decrease piston to compress the liquid, the fractional change
of cross-sectional area by 2%. The percentage in the radius of the sphere (Dr/r) is
increase in its length is (a) Ka / mg (b) Ka / 3mg
(a) 3% (b) 2.5% (c) mg / 3Ka (d) mg / Ka
(c) 1% (d) 0.5%
30. The bulk modulus of a metal is 8 ´ 109 Nm–2 and its
25. Find the ratio of Young's modulus of wire A to wire B density is 11 gcm–2. The density of this metal under a
Stress pressure of 20,000 N cm–2 will be (in gcm–3)
B 440 431 451 40
(a) (b) (c) (d)
39 39 39 39
A 31. A uniform rectangular bar of area of cross-section A
30° is fixed at one end and on other end forces F is applied
30° as shown in figure. Find the shear stress at a plane
Strain
through the bar making an angle q with the vertical
(a) 1 : 1 (b) 1 : 1 (c) 1 : 3 (d) 1 : 4 as shown in figure.
26. The stress-strain graph for a metallic wire is shown
at two different temperatures, T1 and T2 which
temperature is high T1 or T2 ?
FF
Strain θ
T1

T2
Rigid wall
F F
Stress (a) (cos 2 q) (b)
2A 2A
(a) T1 > T2 (b) T2 > T1 F F
(c) (sin 2 q) (d) cos q
(c) T1 = T2 (d) None of these 2A 2A
Telegram @unacademyplusdiscounts

Properties of Solids 417

32. A uniform rod of length L and area of cross-section A (a) Material (ii) is more elastic than material (i) and hence
is subjected to tensile load F. If s be Poisson’s material (ii) is more brittle
ratio and Y be the Young’s modulus of the material of (b) Material (i) and (ii) have the same elasticity and the same
brittleness
the rod, then find the volumetric strain produced in
(c) Maerial (ii) is elastic over a larger region of strain as
rod.
F F compared to (i).
(a) (1 + 2 s ) (b) (1 - 2 s ) (d) Material (ii) is more brittle than material (i)
AY AY
(c) Zero (d) None of these 37. A rod of length l and negligible
mass is suspended at its two A B
More Than One Correct Option ends by two wires of steel (wire Steel Al
A) and aluminium (wire B) of
33. A wire is suspended from the ceiling and stretched equal lengths (figure) The
under the action of a weight F suspended from its m
cross-sectional areas of wires A
other end. The force exerted by the ceiling on it is and B are 1.0 mm2 and 2.0 mm2 , respectively.
equal and opposite to the weight. [NCERT Exemplar]
(YAl = 70 ´ 109 Nm -2 and Ysteel = 200 ´ 109 Nm -2 )
(a) Tensile stress at any cross section A of the wire is F/A
(b) TensiIe stress at any cross section is zero [NCERT Exemplar]
(c) Tensile stress at any cross section A of the wire is 2F/A (a) Mass m should be suspended close to wire A to have
(d) Tension at any cross section A of the wire is F equal stresses in both the wires
(b) Mass m should be suspended close to B to have equal
34. The wires A and B shown in figure, are stresses in both the wires
made of the same material and have radii A
(c) Mass m should be suspended at the middle of the wires to
rA and rB respectively. A block of mass m is m have equal stresses in both the wires
connected between them. When a force F is (d) Mass m should be suspended close to wire A to have
mg/3, one of the wires breaks. B equal strain in both wires
(a) A will break before B if rA < rB 38. A metal wire of length L is suspended vertically from
(b) A will break before B if rA = rB a rigid support. When a body of mass M is attached to
F
(c) Either A or B will break if rA = 2rB the lower end of wire, the elongation of the wire is l.
(d) The length of A and B must be known to decide which (a) The loss in gravitational potential energy of mass M is
wire will break Mgl
(b) The elastic potential energy stored in the wire is Mgl
35. A metal wire of length L, area of cross-section A and 1
Young’s modulus Y is stretched by a variable force F (c) The elastic potential energy stored in the wire is Mgl
2
such that F is always slightly greater than the elastic 1
forces of resistance in the wire. When the elongation (d) Heat produced is Mgl
2
of the wire is l
YAl 2
(a) the work done by F is
L
Comprehension Based Questions
YAl 2 Passage I
(b) the work done by F is
2L
A boy’s catapult is made of rubber cord 42 cm long and
YAl 2
(c) the elastic potential energy stored in the wire is 6 mm in diameter. The boy keeps a stone weighing
2L 0.02 kg on it and stretches the cord by 20 cm. When
(d) heat is produced during the elongation released, the stone flies off with a velocity of 20 ms -1.
36. The stress-strain graphs for two materials are shown Neglect the change in the cross-section of the cord in
in figure. (assume same scale). [NCERT Exemplar] stretched position.

Ultimate Tension
39. The stress in the rubber cord is
Ultimate Tension
Strength Strength (a) 1.8 × 106 Nm–2
Fracture Point Linear (b) 1.4 × 106 Nm–2
Linear Fracture Point
Stress

Stress

limit (c) 2.4 × 105 Nm–2


limit
(d) 1.8 × 105 Nm–2
40. The strain in the rubber cord is
Strain E Strain E (a) 2.1 (b) 1.8
Material (i) Material (ii)
(c) 0.96 (d) 0.48
Telegram @unacademyplusdiscounts

418 JEE Main Physics

41. The Young’s modulus of rubber is 46. Assertion Two identical springs of steel and copper
(a) 2.12 × 106 Nm–2 (b) 2.94 × 106 Nm–2 are equally stretched. More work will be done on
(c) 3.92 × 106 Nm–2 (d) 1.94 × 106 Nm–2 steel than copper.
Reason Steel is more elastic than copper.
Passage II
47. Assertion Young’s modulus for a perfectly plastic
A structural steel rod has a radius of 10 mm and
body is zero.
length of 1.0 m. A 100 kN force stretches it along
Reason For a perfectly plastic body, restoring force
its length. Young’s modulus of structural steel is
2 ´ 1011 Nm–2. is zero.

42. The elongation in the wire is 48. Assertion The bridges are declared unsafe after a
(a) 1.59 mm (b) 3.18 mm long use.
(c) 2.38 mm (d) 0.79 mm Reason The bridges lose their elastic strength with
time.
43. The percentage strain is about
(a) 0.16% (b) 0.32% 49. Assertion A solid shaft is found to be stronger, than a
(c) 0.24% (d) 0.08% hollow shaft of same material.
Reason The torque required to produce a given twist
44. Elastic energy density of stretched wire is in solid cylinder is smaller than that required to twist
(a) 1.26 × 105 Jm–3 (b) 2.53 × 105 Jm–3
a hollow cylinder of the same size and material.
(c) 3.79 × 105 Jm–3 (d) 5.06 × 105 Jm–3
YA
45. Stress produced in the steel rod is 50. Assertion Force constant, k = , where Y is
l
(a) 1.59 × 108 Nm–2 (b) 3.18 × 108 Nm–2 Young’s modulus, A is area and l is original length of
(c) 4.77 × 108 Nm–2 (d) 6.36 × 108 Nm–2 the given spring.
Reason Force constant in case of a given spring is
Assertion and Reason called spring constant.

Direction Question No. 46 to 52 are Assertion-Reason type. 51. Assertion The restoring force, F on a stretched string
Each of these contains two Statements : Statement I (Assertion), for extension x is related to potential energy, U as,
Statement II (Reason). Each of these questions also has four dU
F=-
alternative choices, only one of which is correct. You have to dx
select the correct choices from the codes (a), (b), (c) and (d) given 1
Reason F = - kx and U = kx 2 where, k is a spring
below 2
(a) If both Assertion and Reason are true and the Reason constant for the given stretched string.
is correct explanation of the Assertion
52. Assertion Identical springs of steel and copper are
(b) If both Assertion and Reason are true but Reason is
equally stretched. More work will be done on the
not correct explanation of the Assertion
steel spring.
(c) If Assertion is true but Reason is false
Reason Steel is more elastic than copper.
(d) If Assertion is false but the Reason is true

Previous Years’ Questions


53. A wooden wheel of radius R is made force that one part of the wheel applies on the other
of two semicircular parts (see part is [AIEEE 2012]
figure). The two parts are held
(a) 2 pSy µ DT (b) SY µ DT
together by a ring made on a metal R
strip of cross- sectional area S and (c) pSY µ DT (d) 2 SY µ DT
length L. L is slightly less than 2 pR. 54. The Poisson’s ratio of the material is 0.5. If a force is
To fit the ring on the wheel of is applied to a wire of this material, there is a decrease
heated so that it temperature rises by DT and it just in the cross-sectional area by 4%. The percentage
steps over the wheel. As it cools down to surrounding increase in its length is [WB JEE 2009]
temperature it presses the semicircular parts
(a) 1% (b) 2%
together if the coefficient of linear expansion of
(c) 2.5% (d) 4%
the metal is a and its young’s modulus is Y . The
Telegram @unacademyplusdiscounts

Properties of Solids 419

55. Two wires are made of the same material and have 62. If the volume of a block of aluminium is decreased by
the same volume. However, wire 1 has cross- 1%, the pressure (stress) on its surface is increased
sectional area 3A. If length of wire 1 increased by Dx by (Bulk modulus of aluminium = 7.5 ´ 1010 Nm–2)
on applying force F, how much force is needed to [Kerala CEE 2008]
stretch wire 2 by the same amount ? [AIEEE 2009] (a) 7.5 × 1010 Nm–2
(a) 4 F (b) 6 F (c) 9 F (d) F (b) 7.5 × 108 Nm–2
(c) 7.5 × 106 Nm–2
56. When a rod is heated but prevented from expanding,
the stress developed is independent of [BVP Engg. 2008] (d) 7.5 × 104 Nm–2
(a) material of the rod (b) rise in temperature 63. A student performs an experiment to determine the
(c) length of rod (d) None of these Young’s modulus of a wire, exactly 2 m long, by
57. The Bulk Modulus for an incompressible liquid is Searle’s method. In a particular reading, the
[UP SEE 2008] students measures the extension in the length of the
(a) zero (b) unity wire to be 0.8 mm with an uncertainty of ± 0.05 mm
(c) infinity (d) between 0 and 1 at a load of exactly 1.0 kg. The student also measures
the diameter of the wire to be 0.4 mm with an
58. A metal wire of length L1 and area of cross-section A
uncertainty of ± 0.01 mm. The Young’s modulus
is attached to a rigid support. Another metal wire of
obtained from the reading is (Take g = 9.8 ms–2).
length L2 and of the same area of the first wire. A
[IIT JEE 2007]
body of mass M is then suspended from the free end
of the second wire. If Y1 and Y2 are the Young’s (a) (2.0 ± 0.3) × 1011 Nm–2
modulii of the wires respectively, the effective force (b) (2.0 ± 0.2) × 1011 Nm–2
constant of the system of two wires is [NSEP 2008] (c) (20 ± 0.1) × 1011 Nm–2
Y1Y2 A Y1Y2 A (d) (2.0 ± 0.05) × 1011 Nm–2
(a) (b)
(Y1L2 + Y2 L1 ) ( L1L2 )1 /2 64. A wire 3 m in length and 1 mm in diameter at 30°C is
Y1Y2 A (Y Y )1 /2 A kept in a low temperature at –170°C and is stretched
(c) (d) 1 2 1 /2
(Y1L2 + Y2 L1 ) ( L1L2 ) by hanging a weight of 10 kg at one end. The change
in length of the wire is (Y = 2 ´ 1011 Nm–2, g = 10 ms–2
59. There is some change in length when a 33000 N
and a = 1.2 ´ 10–5°C–1) [UP SEE 2006]
tensile force is applied on a steel rod of area of
(a) 5.2 mm (b) 2.5 mm
cross-section 10–3 m2. The change in temperature
(c) 52 mm (d) 25 mm
required to produce the same elongation if the
steel rod is heated is (the modulus of elasticity is 65. Two rods of different materials having coefficients of
3 ´ 1011 Nm–2 and coefficient of linear expansion of linear expansion a 1 and a 2 and Young’s modulus, Y1
steel is 1.1 ´ 1011° C-1). [EAMCET 2008] and Y2 respectively are fixed between two rigid
(a) 20°C (b) 15°C (c) 10°C (d) 0°C massive walls. The rods are heated such that they
60. A load of 1kg weight is attached to one end of a steel undergo the same increase in temperature. There is
wire of area of cross-section 3 mm2 and Young’s no bending of rods. If a 1: a 2 = 2 : 3, the thermal stress
modulus 1011 Nm–2. The other end is suspended developed in the two rods are equal provided Y1: Y2
vertically from a hook on a wall, then the load is equal to [BVP Engg. 2006]
pulled horizontally and released. When the load (a) 2 : 3 (b) 4 : 9
passes through its lowest position, the fractional (c) 1 : 2 (d) 3 : 2
change in length is (g = 10 ms–2) [EAMCET 2008]
66. The pressure of a medium is changed from
(a) 10–4 (b) 10–3 (c) 103 (d) 104
1.010 ´ 105 to 1.165 ´ 105 Pa and change in volume is
61. A wire is stretched under a force. If the wire suddenly 10% keeping temperature constant. The Bulk
snaps, the temperature of the wire, [WB JEE 2008] modulus of the medium is [IIT Screening 2005]
(a) remains the same (a) 204.8 × 105 Pa
(b) decreases (b) 102.4 × 105 Pa
(c) increases (c) 51.2 × 105 Pa
(d) first decreases then increases (d) 1.55 × 105 Pa
Telegram @unacademyplusdiscounts

Answers
Round I
1. (c) 2. (a) 3. (b) 4. (a) 5. (a) 6. (a) 7. (a) 8. (b) 9. (c) 10. (c)
11. (a) 12. (d) 13. (a) 14. (b) 15. (b) 16. (d) 17. (d) 18. (c) 19. (d) 20. (d)
21. (c) 22. (b) 23. (b) 24. (b) 25. (d) 26. (c) 27. (b) 28. (c) 29. (a) 30. (a)
31. (d) 32. (a) 33. (a) 34. (c) 35. (b) 36. (c) 37. (d) 38. (c) 39. (b) 40. (d)
41. (c) 42. (a) 43. (c) 44. (b) 45. (b) 46. (d) 47. (b) 48. (b) 49. (b) 50. (c)
51. (c) 52. (a) 53. (b) 54. (d) 55. (b) 56. (b) 57. (b) 58. (c) 59. (c) 60. (c)
61. (b) 62. (d) 63. (b) 64. (b) 65. (b) 66. (b) 67. (d) 68. (a) 69. (a) 70. (c)
71. (a) 72. (a) 73. (a) 74. (c) 75. (a) 76. (c) 77. (d) 78. (a) 79. (a) 80. (b)
81. (d) 82. (b) 83. (a) 84. (b) 85. (b) 86. (c) 87. (a) 88. (d) 89. (b) 90. (d)
91. (b)

Round II
1. (d) 2. (d) 3. (c) 4. (d) 5. (c) 6. (c) 7. (c) 8. (d) 9. (b) 10. (a)
11. (b) 12. (c) 13. (b) 14. (d) 15. (a) 16. (b) 17. (b) 18. (c) 19. (c) 20. (d)
21. (d) 22. (d) 23. (b) 24. (b) 25. (c) 26. (a) 27. (a) 28. (b) 29. (c) 30. (a)
31. (c) 32. (b) 33. (a,d) 34. (a,c) 35. (b,c,d) 36. (c, d) 37. (b,d) 38. (a,c,d) 39. (b) 40. (d)
41. (b) 42. (a) 43. (a) 44. (b) 45. (b) 46. (a) 47. (a) 48. (a) 49. (d) 50. (b)
51. (a) 52. (a) 53. (d) 54. (d) 55. (c) 56. (c) 57. (c) 58. (a) 59. (c) 60. (a)
61. (c) 62. (b) 63. (b) 64. (a) 65. (d) 66. (d)

the Guidance
Round I
Fl 1
1. As, Y = Þ Yµ
aDL d2
1 1 1
or DL µ ; DL µ 2 Þ dµ
a D Y
DL2 D12 1
Þ = =4 then, d copper µ
DL1 D22 Ycopper
1
or DL2 = 4DL1 = 4 cm and d iron µ
iron Y
F
Fl d copper Yiron
2. As, Y= A = So, =
Dl A Dl d iron Ycopper
l
Fl ´ 4
4. If ( A) is the area of cross-section and l is the length of rope, the
or Y= Al
pD 2 ´ Dl mass of rope, m = × As the weight of the rope acts at the
r
1 mid-point of the rope.
or Dl µ
D2 mg ( l / 2)
So, Y= ´
DL2 D12 n 2 A Dl
or = =
DL1 D22 1 mgl Al rgl g rl 2
DL = = =
mgL 2 AY 2 AY 2Y
3. As, Y= (Y = Young’s modulus)
pr 2l 9.8 ´ 1.5 ´ 10 3 ´ 8 2
or DL =
4mgL 4 mgL 2 ´ 5 ´ 10 6
Þ Y= =
p (2r) 2l p (d) 2l1 = 9.6 ´ 10 -2 m
Telegram @unacademyplusdiscounts

Properties of Solids 421

5. According to the figure, 1


L L 12. Energy stored per unit volume = ´ stress ´ strain
B D C 2
increases in length = BO + OC - BC
1
DL = 2 BO - 2L = 2 (L2 + x 2 )1/ 2 - 2 L x = ´ Young’s modulus ´ (strain) 2
2
é x2 ù x2 O 1
or DL = 2 L ê1 + ú - 2L = = ´ Y ´ x2
m 2
ë L2 û L
D L x2 / L x2 13. As, Y µ F
\ Strain = = = 2 FCu YCu 1
2L 2L 2L Þ = =
FFe YEe 3
Fl (ml w2) l
6. As, Y = =
ADl A Dl 14. F = force developed
2 2
ml w = YA a ( Dq)
or Y=
A Dl = 10 11 ´ 10 -4 ´ 10 -5 ´ 100 = 10 4 N
1 ´ 1 ´ 1 ´ 20 ´ 20 LAdg
or Y= 15. From the question10 6 =
10 -6 ´ 10 -3 A
= 4 ´ 10 11 Nm-2 10 6
\ L= m
FL 3 ´ 10 3 ´ 9.8
7. As, E = 1000
pr 2DL = = 34.01m
3 ´ 9.8
FL
or DL = F L
pr 2E 16. As, Y=
Clearly, DL µ L A Dl
L L
L2dg (8 ´ 10 -2) 2 ´ 1.5 ´ 9.8 Þ Dl µ µ
8. As, l = = = 9.6 ´ 10 -11 m A pd 2
2Y 2 ´ 5 ´ 10 8
L
\ Dl µ 2
9. Given, radius of steel cable (r) = 1.5 cm = 1.5 ´ 10 -2 cm d
L
Maximum stress = 10 8 N/m 2 The ratio of is maximum for case (d).
d2
Area of cross-section of steel cable ( A) = pr 2
Fl
= 3.14 ´ (1.5 ´ 10 -2) 2 m 2 17. As, Y =
ADl
= 3.14 ´ 2.25 ´ 10 -4 m 2
YA Dl
Maximum force or F=
Maximum stress = l
Area of cross-section
2.2 ´ 10 11 ´ 2 ´ 10 -6 ´ 0.5 ´ 10 -3
Maximum force = Maximum stress ´ Area of cross-section or F=
2
= 10 8 ´ (3.14 ´ 2.25 ´ 10 -4) N = 1.1 ´ 10 2 N
4
= 7.065 ´ 10 N Fl 1
= 7.1 ´ 10 4 N
18. As, Y = Þ Dl µ
A Dl A
10. Let m be the mass of rectangular frame and q be the angle Again, m = Alr, m µ A
which the tension T in the string make with the horizontal 1
from figure. Then, \ Dl µ
m
2T sin q = mg T sin θ T sin θ Dl1 m2 2
mg T T
\ = =
or T= Dl2 m1 3
θ θ
2 sin q
T cos θ T cos θ F ´ 4 ´1
1 19. As, Y =
or Tµ m pD 2Dl
sin q
In the given problem, F µ D 2. Since, D is increased by a factor
T is least if sin q has maximum value
i.e., sin q = 1 = 90° or q = 90° of 4 therefore, F is increased by a factor of 16. So, F will be
100 ´ 16 = 1600 N
11. Strain = fractional change in length
Fl
Dl l at 20. As, Y =
= = = at = 12 ´ 10 -6 ´ 30 = 36 ´ 10 -5 ADl
l l
In the given problem, Y , l and Dl are constants.
\ F µA
Telegram @unacademyplusdiscounts

422 JEE Main Physics

or F = pr 2 (Q area = pr 2) 4Fl
28. As, Dl =
pD 2Y
or F µ r2
4 ´ 30 ´ 2 ´ 7
F1 r12 1 =
or = = 22 ´ (3 ´ 10 -3) 2 ´ 1.1 ´ 10 11
F2 r22 4
= 7.7 ´ 10 -5 m = 0.077 mm
Fl F
21. As, Y = 2 or Dl = 2 » 0.08 mm
pr Dl pr Y
1 2l Fl
Þ Dl µ 2 and Dl ¢ µ 29. As, Y =
ADl
r ( 2r ) 2
F
1 Þ Dl µ
\ Dl¢ µ 2 r2
r
Dl Dl2 F2 r12
Again, =1 Þ = ´
Dl¢ Dl1 F1 r22
FL Dl2
22. As, Y = 2 (Here, l = change in length) or =2 ´2 ´2 =8
pr l Dl1
FL FL or Dl2 = 8Dl1 = 8 ´ 1 mm = 8 mm
or l = 2 or l µ 2
pr Y r Fl
30. As, Y =
l1 F ´ L ( 4r) 2 ADl
= 2 ´ YADl
l2 r 4F ´ 4L or F=
l
or l1 = l2 = l
1
So, l remain unchanged. Work done = FDl
2
Mg ´ 4 ´ l 1 1 FA( Dl) 2 YA( Dl) 2
23. As, Y = Þ Dl µ 2 = =
pD 2 ´ Dl D 2 l 2l
When D is doubled, Dl becomes one-fourth, i. e. , 2 ´ 10 11 ´ 10 -6 ´ 10 -6
= = 0.1 J
1 2 ´1
´ 2.4 cm, i. e. , 0.6 cm
4 Fl
Fl Fl Fl 31. As, Y =
24. As, Y = or Dl = = 2 ADl
ADl AY pr Y
where, Y , l and F are constants.
1
In the given problem, Dl = 2 ; when both l and r are double, 1
r Þ Dl µ 2
D
Dl is halved.
Dl2 D12 1
F/A Þ = =
25. As, Y = Dl1 D22 16
Breaking strain
1
F 10 4 ´ 100 \ Dl2 = mm (Q Dl1 = 1 mm)
or A= = 16
Y ´ Breaking strain 7 ´ 10 ´ 0.2
F l
32. We have, Y= ´
= 0.71 ´ 10 -3 = 7.1 ´ 10 -4 A Dl
r 10 6 100 and V = Al
26. As, L = = = = 33.3 m
eg 3 ´ 10 3 ´ 10 3 V
or l=
A
27. Initial length (circumference) of the ring = 2pr
FV
Final length (circumference) of the ring = 2pR \ Y=
A2Dl
Change in length = 2pR - 2pr 1
Þ Dl µ
change in length 2p (R - r) R - r A2
Strain = = =
original length 2 pr r 1
F/A F/A or Dl µ
Now Young’s modulus, E = = D4
l / L (R - r) / r
DlA DB4 14
æR - r ö Þ = 4 = = 16
\ F = AE ç ÷ DlB DA æ 1 ö 4
è r ø ç ÷
è2ø
Telegram @unacademyplusdiscounts

Properties of Solids 423

(mg + mlw2) l Mg L / 2
33. As, Y = 37. As, Y = ´
pr 2Dl A DL
æ L ö
m ( g + mlw2) l çLength is taken as because weight acts at ÷
or Dl = 2
pr 2Y ç ÷
è centre of gravity (CG) ø
1(10 + 2 ´ 4p 2 ´ 4) 2
or Dl = Now, M = ALr
p (1 ´ 10 -3) 2 ´ 2 ´ 10 11
(For the purpose of calculation of mass, the whole of
(20 + 64 ´ 9.88)7
or Dl = geometrical length L is to be considered.)
2 ´ 22 ´ 10 5 AlrgL
\ Y=
4566.24 2 A DL
= ´ 10 3 mm = 1 mm
44 ´ 10 5 rgL2 1.5 ´ 10 3 ´ 10 ´ 8 ´ 8
or DL = =
stress 2Y 2 ´ 5 ´ 10 6
34. As, Y =
strain
= 9.6 ´ 10 -2 m = 9.6 ´ 10 -2 ´ 10 3 mm
stress
or strain = = 96 mm
Y
DL stress 38. As rubber is being more stretched as compare to the iron
or =
L Y under the action of same weight.
Since, cross-section are equal and same tension exists in FLS
39. As, YS =
both wires, therefore, the stresses developed are equal. Also, ASDLS
DL is given to be the same for both the wires. FL C
and YC =
\ L µY A C DL C
Ls Y 2 ´ 10 11 20 YC AC DLC
Þ = s = =
LCu YCu 1.1 ´ 10 11 11 LC F æ Y ö æ AC ö æ DLC ö
\ = =ç C÷ ç ÷ç ÷
F l LS Y A
S SD L S è YS ø è AS ø è DLS ø
35. As, Y = ´
pR 2 Dl F
F, l and Dl are constants. AC DLC Y 1.1
Here, = 2, = 1, C =
1 AS DLS YS 2
2
Þ R µ
Y LC 1.1
\ = ´ 2 ´ 1 = 1.1
RS2 YB 10 11
1 LS 2
Þ = = =
RB2 YS 2 ´ 10 11 2 40. Net elongation of the rod is
RS 1
\ = 3F 3F 2F 2F
RB 2
(2L /3) (L /3)
RB
or RS =
2 æ 2L ö æLö
3F ç ÷ 2F ç ÷
Fl è3ø è3ø
36. We have, Y = l= +
ADl AY AY
Y , l and A are constants. 8FL
l=
F 3AY
\ = constant Þ Dl µ F
Dl Yp (2R) 2 Yp (R) 2
41. As, k1 = and k2 =
Now, l1 - l µ T1 and l2 - l µ T2 L L
l1 - l T1 Since, k1x1 = k2x2 = w
Dividing, =
l2 - l T2 Elastic potential energy of the system
1 1
or l1T2 - lT2 = l2T1 - lT1 U = k1x12 + k2x22
2 2
or l(T1 - T2) = l2T1 - l1T2 2 2
1 æwö 1 æwö
l T - lT = k1ç ÷ + k2ç ÷
or l= 21 12 2 è k1 ø 2 è k2 ø
T1 - T2
1 2ì 1 1ü
l1T2 - l2T1 = wí + ý
or l= 2 î k1 k2þ
T2 - T1
Telegram @unacademyplusdiscounts

424 JEE Main Physics

1 1 L L Hence, final length = l + l ¢


Now, + = +
k1 k2 4YpR 2 YpR 2 = 5a - 4b + 9b - 9a
1 æ 5L ö 5w 2L l0 = 5 b - 4 a
\ U = w 2ç ÷ =
2 è 4YpR 2 ø 8pYR 2 45. Adiabatic elasticity E = gp
42. Assume original length of spring = l For argon E Ar = 1.6p
As, mg = kx For hydrogen E H2 = 1.4p¢
Þ k1(60) = k2( l - 60) = kl As elasticity of hydrogen and argon are equal
\ mg = k1 = (7.5) \ 1.6p = 1.4p¢
According to question, 8
Þ p¢ = p
and mg = k2 = (5.0) 7
kl kl p
\ k1 = , k2 = 47. As, K = (Here, K = bulk modulus of elasticity)
60 ( l - 60) DV
V
k1 5.0 ( l - 60)
= = 1 DV / V
k2 7.5 60 or =
K p
2 ( l - 60)
Þ = DV
3 60 or s=
pV
\ l = 100 cm
or DV = spV
and kx = k1 ´ 7.5,
pV pV p p
æ 5k ö 48. As, K = = = ÞT =
kx = ç ÷ ´ 7.5 DV gDT 3aT 3Ka
è3ø
\ x = 12.5 cm 49. As, mg = Kx
F 4 ´ 9.8
43. As, x = K= or K = 19.6 ´ 10 2 Nm-1
k 2 ´ 10 -2
k 1
If spring constant is k for the first case, it is for second case. \Work done = ´ 19.6 ´ 10 2 ´ (5 ´ 10 -2) 2 J = 2.45 J
2 2
4 1.5 N
For first case, 1= …(i) 50. K = = 50 Nm-1 (as mg = kx)
k 30 ´ 10 -3 m
6 12
For second case, x¢ = = …(ii) 0.2 ´ 10
k/2 k \ l= m = 0.04 m
50
Dividing Eq. (ii) by Eq. (i), we get 1
12 / k Now, energy stored = ´ 0.20 ´ 10 ´ 0.04 J = 0.04 J
x¢ = = 3 cm 2
4 /k 0.10
51. Change in volume, DV = V ´
44. As, T1 = K( l - l1) 100
DV 0.10
and T2 = K( l - l2) or = = 1 ´ 10 -3 …(i)
T1 l - l1 V 100
So, =
T2 ( l - l2) Bulk modulus of water (K) = 2.2 ´ 10 9 N/m 2
\ T1l - T1l2 = T2l - T2l1 Pressure on water ( Dp) = ?
(T1 - T2) l = T1l2 - T2l1 Dp
Bulk modulus of water (K) =
DV / V
T1l2 - T2l1
l=
(T1 - T2) DV
or Dp = K ´
V
l = (5a - 4b) …(i)
1 = 2.2 ´ 10 9 ´ 1 ´ 10 -3
k= …(ii)
b-a = 2.2 ´ 10 6 N-m 2
So, length of wire when tension is 9 N, is given by 52. Pressure ( p) = 10 atm = 10 ´ 1.013 ´ 105 Pa
9 = kl ¢ (l¢ = change in length)
(Q 1 atm = 1.013 ´ 10 5 Pa)
1
9= ´ dl ¢ = 9 b - 9 a = 1.013 ´ 10 6 Pa
( b - a)
Telegram @unacademyplusdiscounts

Properties of Solids 425

Bulk modulus for glass (K) = 37 ´ 10 9 N/m 2 59. As volume, V = pr 2l


æ DV ö DV D( pr 2l)
Fractional change in volume ç ÷ =? Þ =
è V ø
V pr 2l
p pV
Bulk modulus (K) = = DV r 2Dl + 2rlDr
DV / V DV or =
V r 2l
DV p 1.013 ´ 10 6 DV Dl 2 Dr
\ = = or = +
V K 37 ´ 10 9 V l r
101.3 Now, Poisson’s ratio
= ´ 10 -5
37 Dr / r Dr / r
= 2.74 ´ 10 -5 s =- =- = 0.5
Dl Dr
-2
æ DV ö -5 l r
\Fractional change in volume ç ÷ = 2.74 ´10
è V ø
60. Let, L be the length of each side of cube, then initial volume
53. As, mg = Kx = L3 . When each side decrease by 1%.Then
10 N 100 1 99L
k= -3
= Nm-1 = 250 Nm-1 New length, L ¢ = L - =
40 ´ 10 m 4 100 100
3
Spring constant of combination æ 99L ö
New volume = L ¢3 = ç ÷ ,
250 è100 ø
= Nm-1 = 125 Nm-1
2 \ Change in volume,
3
1 æ 99L ö
Energy = ´ 125 ´ ( 40 ´ 10 -3) 2 J = 0.1 J DV = L3 - ç ÷
2 è100 ø
3
54. 10 m column of water exerts nearly 1 atmosphere pressure. é æ 3 öù é 3 ù 3L
= L2 ê1 - ç1 - + K÷ ú = L3 ê ú =
So, 100 m column of water exerts nearly 10 atmospheric ë è 100 øû ë100 û 100
pressure, i. e. ,10 ´ 10 5 Pa or 10 6 Pa.
DV 3L3 / 100
\Bulk strain = = = 0.03
1 DV / V DV é 1ù V L3
55. As, = or = Dp ê ú
K Dp V ëKû 61. For most materials the modulus of rigidity, G is one third of the
DV Young’s modulus Y
or ´ 100 = 10 5 ´ 8 ´ 10 -12 ´ 100 = 8 ´ 10 -5
V 1
1 G= Y
56. As, energy stored per unit volume = ´ stress ´ strain 3
2
or Y = 3G
1 strain stress S 2 \ n =3
= ´ stress ´ Y= =
2 Y strain 2Y
62. Energy stored per unit volume
57. As, Dp = hrg = 200 ´ 10 3 ´ 10 Nm-2 1 1
= Y ( strain) 2 = ´ 1.5 ´ 10 12 ´ (2 ´ 10 -4) 2
6
= 2 ´ 10 Nm -2 2 2
= 3 ´ 10 4 Jm-3
Dp 2 ´ 10 6 2 ´ 10 8
\ K= = = Nm-2
DV 0.1 0.1 63. Every material has its certain volume. So, it may have
V 100 coefficient of volume elasticity an elastic coefficient.

= 2 ´ 10 9 Nm-2 64. Work done in stretching the wire


1
58. As, Dp = 100 atm = 100 ´ 10 6 dyne cm -2 W= ´ force constant ´ x2
2
= 10 8 dyne cm -2 1 1
For first wire, W1 = ´ kx2 = kx2
DV 0.01 2 2
\ = = 10 -4
V 100 1
For second wire, W2 = ´ 2k ´ x = kx2
2

æ ö 2
10 8 ç Dp ÷ Hence, W2 = 2W1
\ K = -4 dyne cm -2 = 10 12 dyne cm-2 çQ K = ÷
10 ç DV ÷
è
65. Because a liquid at rest begins to move under the effect to
V ø
tangential force.
Telegram @unacademyplusdiscounts

426 JEE Main Physics

66. Here, T = m( g + a0) = 10(10 + 2) = 120 N 1


72. As, energy density = stress ´ strain
2
T
\ Stress =
A 1 stress ( stress) 2 1
= stress ´ = µ 4
120 2 Y 2Y D
= = 60 ´ 10 4 Nm-2
2 ´ 10 -4 uA DB4
m1 Now, = = (2) 4 = 16
stress uB DA4
and Y=
strain (where uA and uB are energy densities)
stress 1
\ strain = m (g + a 0) 73. As, energy/volume = ´ stress ´ strain
Y 2
4
60 ´ 10 1 1
= = 30 ´ 10 -7 = 3 ´ 10 -6 = Y ´ strain ´ strain = Y ´ strain 2
2 ´ 10 11 2 2
1
67. Elastic energy stored in the wire is, = ´ 2 ´ 10 10 ´ 0.06 ´ 10 -2 ´ 0.06 ´ 10 -2
1 2
U= ´ stress ´ strain ´ volume = 3600 Jm-3
2
1 F Dl 1 3L
= ´ ´ ´ Al = FDl 74. Here, force = weight suspended + weight of of wire
2 A l 2 4
1 3w
or U = ´ 200 ´ 1 ´ 10 -3 = 0.1 J As, w1 =
2 4
68. Breaking strength = tension in the wire = mrw2 (centrefugal w1 +
3
w
force) force 4
\ stress = =
area S
Þ 4.8 ´ 10 7 ´ 10 -6 = 10 ´ 0.3 ´ w2
48 phr 4
w2 = = 16 75. As, torque t = q
0.3 ´ 10 2l
In the given problem, r 4 q = constant
w = 4 rads-1
1
69. To twist the wire through the angle dq, it is necessary to do the Þ qµ
r4
work
qA r24
dW = tdq Þ =
qB r14
10 p p
and q = 10 ¢ = ´ = rad 1
60 180 1080 76. As, work done = F ´ extension
q q hpr 4 q dq hpr 4 q 2
\ W = ò dW = ò t dq = ò = 1 YA YA
0 0 2l 4l = ´ ´ 1= (Q l = 1)
2 L 2L
5.9 ´ 10 11 ´ 10 -5 ´ p (2 ´ 10 -5) 4 p 2
or W= 1
10 -4 ´ 4 ´ 5 ´ 10 -2 ´ (1080) 2 77. As, W = FDl (where, Dl = entension)
2
or W = 1.253 ´ 10 -12 J
1 Ypr 2Dl
phr 4 Þ W= ´ Dl
70. For cylinder A, t= q¢ 2 l
2l
Ypr 2Dl
ph (2r) 4( q - q¢ ) or W=
For cylinder B, t= 2l
2l Fl
phr 4 q¢ nh (2r) 4( q - q¢ ) where, Y= 2
= (for equal torque) pr Dl
2l 2l
Ypr 2Dl
16 and F=
Þ q¢ = q l
17
1 r2 (2r 2) 22
71. Energy density = ´ stress ´ strain Þ Wµ and W µ
2 l l
stress W¢
Y= or stress = Ys Þ =8
s W
1 1 or W ¢ = 8 ´ 2 J = 16 J
\Energy density = Ys ´ s = Ys 2
2 2
Telegram @unacademyplusdiscounts

Properties of Solids 427

1 1 YADl 2 Y
78. As, work done = FDl = 84. As, h =
2 2 l 2(1 + s )
2 ´ 10 11 ´ 10 -6 (2 ´ 10 -3F) 2 2.4 h
= = 0.4 J or h=
2 ´1 2 (1 + s )
Fl or 1 + s = 1.2
where Y=
ADl or s = 0.2
YADl Fl Fl F 1
\ F= 85. As, h = = = or Dl µ (for, F = constant)
l ADl l 2Dl l Dl l
F 2l If l is halved, then Dl is doubled.
79. As, work done, W =
æ pD 2 ö P
2ç ÷Y 86. Bulk modulus, K = -
è 4 ø DV
V
Y , l and F are constants.
1 negative sign shown that an increase in pressure a decrease in
Þ Wµ 2 volume occurs.
D 1 - dV
W1 D22 Compressibility C = =
Þ = = 16 …(i) K PV
W2 D12 Decrease in volume, DV = PVC
1 = 4 ´ 10 7 ´ 1 ´ 6 ´ 10 -10 = 24 ´ 10 -3 Litre
Now, W1 = ´ 10 3 ´ 1 ´ 10 -3 = 0.5 J
2 = 24 ´ 10 -3 ´ 10 3 cm3 = 24 cc
-3
1 10 1 Y
\ W2 = ´ 10 3 ´ = = 0.03125 87. As, h =
2 16 32 2 (1 + s )
1 YA / Dl 2 Now, s =0
80. As, E =
2 l Y 6 ´ 10 12
then, h= = = 3 ´ 10 12 Nm-2
But m = Ald 2 2
m
or A= phr 4 ph(2r) 4
ld 88. As, t x = qx and t y = qy
2l 2l
YmDl 2
\ E= Since, tx = ty ,
2l 2d q
\ qx = 16qy or x = 16
YmDl 2 qy
E in calorie =
2l 2dJ
Lateral strain
YmDl 2
89. As, s =
Now, mSq = Longitudinal strain
2l 2dJ or Lateral strain = s ´ longitudinal strain
YDl 2 0.05 0.02
or q= 2 = 0.4 ´ =
2l d JS 100 100
So, percentage reduction in diameter is 0.02.
12 ´ 10 11 ´ 10 -1 ´ 10 -3 ´ 10 -3
or q=
2 ´ 2 ´ 2 ´ 9 ´ 10 3 ´ 4.2 ´ 0.1 ´ 10 3 90. Breaking force does not not depend upon length. Breaking
force = breaking stress ´ area of cross-section. For a given
12 ´ 10 5 1 material, breaking stress is constant.
= = °C
72 ´ 42 ´ 10 5 252 Q Breaking force µ Area of cross-section
YA tan qA tan 60° 3 F2 A2 p (6r) 2
81. As, = = = = 3 Þ YA = 3 YB Þ = = = 36
YB tan qB tan 30 1 / 3 F1 A1 pr 2
0.4 ´ 30° or F2 = 36F1 = 36F
82. As, pq = lf; so, f = = 0.12°
10 100 N 2 ´ 10 -3
91. As, stress = -6 2 = 10 8 Nm-2 and strain = = 10 -3 ,
83. Isothermal elasticity q = p, Adiabatic elasticity ( f) = gp 10 m 2
Eq 1 \Young modulus
\ =
Ef g 10 8
Y= Nm-2 = 10 11 Nm-2
10 -3
where g >1
1
Eq Thus, energy stored = ´ 100 ´ 2 ´ 10 -3 J = 10 -1 J = 0.1 J
\ <1 2
Ef
Telegram @unacademyplusdiscounts

428 JEE Main Physics

Round II
l1 r y V1 - V2
1. Given, = a, 1 = b, 1 = c Volumetric strain =
l2 r2 y2 V1
T 1000 - 900
T = = 0.1
Brass Steel 1000
w L wL
5. As, Y = ´ or l =
A l YA
2g When wire goes over a pulley and weight w is attached each
free end of wire, then the tension in the wire is doubled, but
2 kg
the original length of wire is reduced to half, so extension in
Let Young modulus of steel be Y1 and that of brass be Y2 the wire is
Fl 2w ´ (K / L) wL
Y1 = 1 1 …(i) l¢ = = =l
A1 Dl1 YA YA
F2l2 1 4Al 2
and Y2 = …(ii) 6. As, energy U = ´
A2 Dl2 2 L
Dividing Eq. (i) by Eq. (ii), we get 1 2 ´ 10 11 ´ 3 ´ 10 -6 ´ (1 ´ 10 -3) 2
= ´
Y1 F1A2 × l1Dl2 2 4
= …(iii)
Y2 F2A1 l2Dl1 = 0.075 J

Force on steel wire from free body diagram. 7. From figure the increase in length
F1 = T = 2g N Dl = (PR + RQ) - PQ
Force on brass wire from free body diagram = 2PR - PQ
1/ 2
F2 = T1¢ = T + 2g = 4g N æ x2 ö
= 2( l 2 + x2)1/ 2 - 2l = 2l ç1 + 2 ÷ -2l
Now putting the value of F1 × F2 in Eq. (iii), we get è l ø
Y2 æ 2g ö æ pr22 ö æ l1 ö æ Dl2 ö é 1 x2 ù
=ç ÷ç ÷ç ÷ç ÷ …(iv) = 2l ê1 +
Y1 è 4g ø è pr12 ø è l2 ø è Dl1 ø ú -2l
ë 2 l2 û
l1 r
Now given that, = a, 1 = b = x2 / l (By Binomial theorem)
l2 r2
2 2
\ Strain = Dl / 2l = x / 2l
Y1
and =c
Y2 l
S
then from Eq. (iv) we get, PQ
x
Dl1 c
= 2
Dl2 2b a T θ T

2. For Hooke’s law, stress µ strain i. e. , the graph between stress R


and strain is a straight line, which is so for portion O to D.
3. As, stress = (weight due to mass m2 + half of the weight of w
rod)/area 8. When no weight is placed in pan, and T 2 shows some value, it
= (m2g + m1g / 2) / A = [(m1 / 2) + m2]g / A means, the pan is weightless and hence, the mass of the pan
cannot be neglected.
4. According to Boyle’s law, p2V2 = p1V1
æV ö 9. Initial elastic potential energy,
or p2 = p1ç 1 ÷ 1 1
è V2 ø U1 = FDl = ´ (100 ´ 1000) ´ (1.59 ´ 10 -3) = 79.5 J
2 2
or p1 = 72 ´ 1000 / 900 = 80 cm of Hg. Let, Dl1, be the elongation in the rod when stretching force is
Stress = increase in pressure f l
increased by, 200 N. Since Dl = 2 ´ ; so, Dl µ F
pr Y
= p2 - p1 = 80 - 72 = 8
Dl1
= (8 ´ 10 -2) ´ 13.6 ´ 10 3 ´ 9.8 \ = 3Dl = 3 ´ 1.59 ´ 10 -3 m
Dl
= 10662.4 Nm-2 = 4.77 ´ 10 -3 m
Telegram @unacademyplusdiscounts

Properties of Solids 429

Final elastic potential energy is, 14. If side of the cube is L, then volume = V 3
1 1
U1 = F1Dl1 = ´ (300 ´ 10 3) ´ ( 4.77 ´ 10 -3) = 715.5 J dV 3 dL
2 2 Þ =
V L
Increase in elastic potential energy
\ % change in volume = 3 ´ % change in length
= 715.5 - 79.5 = 636.0 J
= 3 ´ 1% = 3%
1 1 DV
10. Work done, W = F ´ l = ´ Y stress ´ strain ´ volume \ Bulk strain, = 0.03
2 2 V
1 Dp ´ V
or W = Y ´ (stress) 2 ´ volume 15. Decrease in volume, DV =
2 K
2
1 æ Dl ö 1 Dl 2A Final volume, V ¢ = V - DV
= Y ç ÷ ´ Al = Y
2 è l ø 2 l VDp
2 ´ 10 11 ´ 10 -6 ´ 10 -6 =V - = V (11 - Dp / K)
= = 0.1 J K
2 ´1 m mæ Dp ö
or = ç1 - ÷
YAl r¢ r è K ø
11. As, T = (where, T = force)
L r
or r¢ =
increase in length of one segment of wire æ Dp ö
ç1 - ÷
æ 1 d2ö 1 d2 è K ø
l = çL + ÷ -L =
è 2 L ø 2 L 10.5 ´ 10 3
or r= = 0.61 kg m-3
Yp r 2 × d 2 (1 - 10 7 / 17 ´ 1010)
So, T=
2L2 16. Young’s modulus of copper (Y1) = 110 ´ 10 9 N/m 2
MgL 4 ´ (3.1 p ) ´ (2.0)
12. As, Y = 2 = Young’s modulus of steel (Y2) = 190 ´ 10 9 N/m 2
pr ´ l p ´ (2 ´ 10 -3) 2 ´ 0.031 ´ 10 -3)
= 2 ´ 10 11 Nm-2 Let d1 and d 2 be the diameters of steel and copper wires.
Since tension in each wire is same, therefore each wire has
13. Given, for steel wire same extension. As each wire in of same length, hence each
Length (l1) = 4.7 m wire has same strain.
Area of cross-section (A1) = 3.0 ´ 10 -5 m 2 Strees F/A
Young’s modulus (Y) = =
Strain Strain
For copper wire
F 4F
Length ( l2) = 3.5 m or Y= = 2
æ ld 2 ö pd ´ Strain
Area of cross-section (A2) = 4.0 ´ 10 -5 m 2 ç ÷ Strain
è 4 ø
Let F be the given load under which steel and copper wires
1 1
be stretched by the same amount Dl. \ Yµ or d 2 µ
d2 Y
F/A F´l
Young’s modulus (Y) = = d12 Y2
Dl / l A ´ Dl \ =
d 22 Y1
F ´ l1
For steel, Ys = …(i)
A1 ´ Dl d1 Y 190 ´ 10 9
or = 2 =
F ´ l2 d2 Y1 110 ´ 10 9
For copper, Yc = …(ii)
A2 ´ Dl 19
= = 173
.
Dividing Eq. (i) by Eq. (ii), we get 11
Ys F ´ l1 A ´ Dl = 1.31
= ´ 2
Yc A1 ´ Dl F ´ l2 \ d1 : d 2 = 1.31 : 1
l1 A2
= ´
l2 A1 Dl stress (1000 ´ 980) / (10 -1) 2
17. As, = = = 0.0098
4.7 4.0 ´ 10 -5 l Y 10 10
= ´
3.5 3.0 ´ 10 -5 % increase in length of wire
18.8 Dl
= = 1.79 = 1.8 = ´ 100 = 0.0098 ´ 100 = 0.98%
10.5 l
Telegram @unacademyplusdiscounts

430 JEE Main Physics

18. Here, W = 2 ´ 1000 ´ 980 dyne; l = 100 cm, b = 2 cm, r2


or = 1 - 10 -4 = 0.9999
r1
d = 1 cm, Y = 20 ´ 10 11 dyne cm-2,
æ V - V1 ö
Wl3 (2 ´ 1000 ´ 980) ´ (100)3 % increase in volume = ç 2 ÷ ´ 100
Now, d= = è V1 ø
3
4 Ybd 4 ´ (20 ´ 10 11) ´ 2 ´ (1)3 æ pr 2l - pr 2l ö æ r 2l ö
= 0.1225 cm = ç 2 2 2 1 1 ÷ ´ 100 = ç 22 2 - 1÷ ´ 100
è pr1 l1 ø è r1 l1 ø
19. Given, each side of cube ( l) = 10 cm = 0.1m = [(0.9999) 2 ´ 1.001 - 1] ´ 100 = 0.08%
6
Hydraulic pressure ( p) = 7 ´ 10 Pa Dd /d
24. Poisson’s ratio, s = 0.4 =
Bulk modulus for copper (K) = 140 ´ 10 9 Pa Dl / l
Volume contraction ( DV ) = ? pd 2
Area, A = pr 2 =
Volume of the cube (V ) = l 3 = (0.1)3 = 1 ´ 10 -3 m3 4
4A
Bulk modulus for copper (K) =
p pV or d2 =
= p
DV / V DV
Differentiating, we get
pV 4
or DV = 2d Dd = × DA
K p
7 ´ 10 6 ´ 1 ´ 10 -3 pd 2
DV = As A=
140 ´ 10 9 4
1 2 p dD d
= ´ 10 -6 m3 So, DA =
20 4
= 0.05 ´ 10 -6 m3 d
p Dd
DA Dd
= 5 ´ 10 -8 m3 = 22 =2
A pd / 4 d
F l YAx DA Dd
20. As, Y = ´ or F = Given ´ 100 = 2% = 2 = 2
A x l A d
1 1 Yax¢ Dd
Work done W = F ´ x = or = 1%
2 2 l d
1 ´ 2 ´ 10 11 ´ (10 -6) ´ (2 ´ 10 -3) 2 Dd / d
= = 0.4 J Given s= = 0.4
2 ´1 Dl / l
Dd Dl Dl 1 Dl
or = 0.4 = =
21. Increase in length due to rise in temperature DL = aL DT d l l 0.4 l
FL = 2.5 ´ 1% = 2.5%
As, Y=
ADL 25. As, Y = tan q
YADL YA ´ aLDT YA tan 30° 1 / 3 1
So, F= = = YAaDT \ = = =
L L YB tan 60° 3 3
\ F = 2 ´ 10 11 ´ 10 -6 ´ 1.1 ´ 10 -5 ´ 20 = 44 N
1
26. The slope of stress-strain curve with strain axis gives the value
of Young's work done =Y´ ´ (strain)
22. As,modulus.
2
2
In the above graph strain is taken along y-axis. Therefore the
1 æ DL ö YADL2 slope of graph at temperature T1 is less than the slope of graph
2 = ´Y ´ ç ÷ =
2 è L ø 2L at temperature T2.
Y( 4A) DL2 é YA DL2 ù Now as we know with increase in temperature the value of
New work done, W ¢ = =8ê ú8 ´ 2 = 16 J modulus of elasticity decreases. Therefore temperature T1 is
2 (L /2) ë 2L û
greater than temperature T2 .
23. Longitudinal strain x 1.6 ´ 10 -4 m
l -l 27. Shearing strain = = = 2.0 ´ 10 -4
Þ a = 2 1 = 10 -3 h 0.8 m
l1
Stress = (shearing strain) (shear modulus)
l2
= 1.001 = (2.0 ´ 10 -4) ´ (3.5 ´ 10 10 Pa)
l1
lateral strain b = 7.0 ´ 10 6 Pa
Poisson’s ratio, s= =
longitudinal strain a \ F = Stress ´ area
r -r = 7.0 ´ 10 6 Pa ´ 0.8 m ´ 0.005 m
or b = sa = 0.1 ´ 10 -3 = 10 -4 = 1 2
r1 = 2.8 ´ 10 4 N
Telegram @unacademyplusdiscounts

Properties of Solids 431

28. Twisting couple per unit twist for solid cylinder, 31. Consider A¢ is the area of cross-section of plane shown in
4 figure
phr
C1 = F cos θ
2l
\Hollow cylinder
r24 - r14 θ
C 2 = C1 F
r4
0.1 ´ (5 4 - 4 4) 36.9 θ
= =
34 81 F sin θ

= 0.455 Nm A
\ A¢ =
29. The volume of sphere in liquid, cos q
Now the tangential force on plane is F sin q
m F sin q
Shear stress =
A
cos q
r r F F
= sin q cos q = 2 sin q cos q
A 2A
F
(a) (b) = (sin 2q)
2A
4 3 F/A
V= pr …(i) 32. Y =
3 DL / A
When mass m is placed on the position, the increased DL F
Þ Y= = …(i)
mg L AY
pressure p = × Since, this increased pressure is equally
a Now, by definition of Poisson's ratio
applicable to all directions on the sphere, so there will be Dr / r
s=
decrease in volume of sphere, due to decrease in its radius. DL / L
From Eq. (i), change in volume is Dr sL sF
Þ =- =- [by using Eq. (i)]
4 r L AY
DV = p ´ 3r 2Dr = 4pr 2Dr Dr sF
3 Þ = - …(ii)
r AY
DV 4 pr 2Dr 3 Dr
= = Now V = pr 2L
V ( 4 /3) pr3 r
DV 2 Dr DL
p mg r Þ = +
\ Bulk modulusK = = ´ V r L
dV / V a 3Dr By using Eqs. (i) and (ii)
Dr mg DV æ - sF ö F
\ = = 2ç ÷+
r 3Ka V è AY ø AY
30. Here, p = 20 ,000 Ncm-2 = 2 ´ 10 8 Nm-2 DV F
Þ = [1 - 2s ]
V AY
pV
As, k= force F
DV 33. Tensila stress = =
area A
pV
or DV = Tension = applied force = F
k
2 ´ 10 8 ´ V V 34. Here, tension in B, TB = F = mg / 3
= =
8 ´ 10 9 40 mg 4mg
Tension in A, TA = TB + mg = + mg =
New volume of the metal, 3 3
V 39V \ TA = 4TB
V ¢ = V - DV = V - =
40 40 A wire will break when the stress is breaking stress
tension T
New mass of the metal Stress, ( S) = =
39V area of cross - section pr 2
= V¢ ´ r = r ¢ = V ´ 11 For rA = 2rB , S A = 4SB
40
40 ´ 11 440 So, A will break before B
or r¢ = = gcm-3
39 39
Telegram @unacademyplusdiscounts

432 JEE Main Physics

TB 38. Loss in gravitational PE of mass M = Mgl


For rA = 2rB , SB =
prB2
Elastic potential energy stored in wire = work done
TB 4TB T
and SA = = = B = SB = average force ´ extension
prA2 p (2rB) 2 prB2
æ 0 + Mg ö 1
=ç ÷ l = Mgl
As the stresses are equal, higher wire may break. è 2 ø 2
1 1
35. As, work done, W = Y ´ (strain) 2 ´ volume This work done appears as heat so, heat produced = Mgl
2 2
2
1 æ lö YAl 2
= Y ç ÷ ´ Al = 39. Here n = 0.02 kg; v = 20 ms-1;
2 èLø 2L
l = 42 cm = 0.42 m
This work done appears as elastic potential energy stored in
the wire. Here, heat is produced during the elongation of wire Dl = 120 cm = 0.20 cm
as the work is done against the restoring forces. r = 3 mm = 3 ´ 10 -3 m
36. From graphs it is clear that ultimate strength of meterial (ii) is
Due to extension, energy is stored in the cord. This is
greater than that of material (i). Therefore, the elastic
converted into kinetic energy when the stone flies off.
behaviour of material (ii) is over region of strain as compared
to material (i). If the fracture point of a material is closer to 1 1
\ Work done = mv 2 = FDl
ultimate strength point, then the material is a brittle material. 2 2
Therefore, the material (ii) is more brittle than material (i). mv 2 0.02 ´ (20) 2
or F= = = 40 N
37. Let the mass m be suspended at distance x from left end of the Dl 0.20
rod for equal stress in the wires. Let F1 and F2 be the tension in F 40
Stress = 2 =
the wires. Then, pr (22 / 7) (3 ´ 10 -3) 2

A2 = 2.0 mm2 = 1.4 ´ 10 6 Nm-2


A A1 = 1.0 mm2
L L B Dl 0.20
Steel 40. As, strain = = = 0.48
Al l 0.42
F1 F2
stress 1.4 ´ 10 6
41. Young’s modulus, Y = =
–x– strain 0.48
(l – x)
m = 2.94 ´ 10 6 Nm-2
F l-x
F1 x = F2 ( l - x ) or 1 = …(i) F l
F2 x 42. Elongation, Dl = ´
2
F F F F2 pr Y
S1 = 1 = 1-6 and S 2 = 2 = 1
A1 10 A2 2 ´ 10 -6 = (3.18 ´ 10 8) ´
2 ´ 10 11
For equal stress, S1 = S 2
F1 F2 F 1 = 1.59 ´ 10 -3 m = 1.59 mm
or = or 1 = …(ii)
10 -4 2 ´ 10 -6 F2 2 Dl (1.59 ´ 10 -3)
43. % strain = ´ 100 = ´ 100 » 0.16%
l (i) and (ii), we1get l - x 1
From Eqs.
x 2
44. Elastic energy density,
On solving, x = 2l / 3
1
It means mass m is suspended close to wire B. u= stress ´ strain
2
Stress
As, Strain = 1 ( stress) 2 1 (3.18 ´ 10 8) 2
x = = ´
2 Y 2 2 ´ 10 11
F1 / 10 6 F2 / (200 ´ 10 -6)
For equal strain, =
YS TAl = 2.53 ´ 10 5 Jm-3
F1 Y (200 ´ 10 9) 10 F 100 ´ 1000
= S = = …(iii) 45. As, stress = = = 3.18 ´ 10 8 Nm-2
F2 2 YAl 2 ´ 70 ´ 10 9 7 pr 2
3.14 ´ (10 -2) 2
l - x 10 46. Work done in stretching a spring of spring constant k is
From Eqs. (i) and (iii), =
x 7 1 2
W= kx or W µ k where x is constant.
7 2
On solving, x= l
17 Since, k for steel is more than for copper, hence more work
It means mass m is suspended close to wire A. will be done on steel than copper.
Telegram @unacademyplusdiscounts

Properties of Solids 433

strain Fl
47. Young’s modulus of a material, Y = 55. As, Y = ;
strain A Dl
Restaring force YA2 Dl YA2 Dl
and Stress = So, F= =
Area Al V
As, restoring force is zero \ Y = 0 where, Al = V = volume of wire
48. A bridge during its use undergoes alternating strains for a Hence, F µ A2
large number of times each day, depending upon the F ¢ (3A) 2
\ = =9
movement of vehicles on it. When a bridge is used for long F A2
time, it losses its elastic strength due to which the amount of or F ¢ = 9F
strain in the bridge for a given stress will become large and
ultimately, the bridge may collapse. This may not happens if 56. As, Lt = L0(1 + a D q )
the bridges are declared unsafe after long use. Þ DL = Lt - L0 = L0 a Dq …(i)
Here, Dq = change in temperature
49. A hollow shaft is found to be stronger than solid shaft of the
given size and material. Hence, Assertion is false. Torque If the same rod of length L0 is subjected to stress along
required to produce a given twist in hollow cylinder is greater its length, then extension in length can be calculated by
than that required to twist a solid cylinder. Hence, Reason is Hooke’s law
stress stress L0 ´ stress
true. Y= = =
strain DL DL
50. Here, both Assertion and Reason are true but Reason is not L0
the correct explanation of Assertion. L ´ stress
F l \ DL = 0 …(ii)
As, Y= Y
A Dl
If the rod is prevented from expanding, we have
F YA
or = =k (as F = K × Dl) L ´ stress
Dl l L0 a Dq = 0
Y
1 1
52. As, work done = ´ stress ´ strain = ´ Y ´ ( strain) 2 \ Stress = Y a Dq (independent of L0)
2 2
Since elasticity of steel is more than that of copper hence 57. The bulk modulus is,
more work has to be done in order to stretch the steel. pV
K=-
53. If temperature increases by DT DV
F
then increase in length L , DL = La DT If liquid is incompressible, so DV = 0
DL pV
\ a DT Hence, K=- =¥
L 0
Let, tension developed in the right is T Þ K = ¥ (infinity)
T DL 58. Let k1, k2 be the force constant of two wires and k be the
\ =Y = Ya D T T T
S Ld effective force constant of the system of two wires. Then,
\ T = SYa DT
F = 2T (from figure)
where, F is the force that one part of the wheel applies on the L, Y, A
other part.
\ F = 2SYa DT
L, Y, A
Þ 2 SY µ DT
54. Given, Poisson’s ratio = 0.5. It shows that the density of M
material is constant. Therefore, the change in volume of the
wire is zero. Thus,
Equivalent spring constant,
V = A ´ l = constant kk
Þ log V = log A + log l k= 12 …(i)
k1 + k2
DV DA Dl
\ =0 = + Mg Y1A
V A l where, k1 = =
Dl DA Dl1 L1
or =-
l A Mg Y2 A
k2 = =
Dl Dl2 L2
or % increase in length = ´ 100 = - ( - 4) = 4%
l
Telegram @unacademyplusdiscounts

434 JEE Main Physics

Putting values in Eq. (i), we get Fractional error in Y is


y1A y 2A DY æ 2DD D ( Dl) ö
´ =±ç + ÷
L1 L2 Y è D Dl ø
k=
æ Y1A ö æ Y2A ö æ 2 ´ 0.01 0.05 ö
ç ÷+ç ÷ or DY = ± ç + ÷ ´ 2 ´ 10 11
è L1 ø è L2 ø è 0. 4 0.8 ø
Y1 Y2A
= = ± 0.2 ´ 10 11 Nm–2
(Y1 L2 + Y2 L1)
\ Y = (2 ´ 10 11 ± 0.2 ´ 10 11) Nm-2
æFö
ç ÷l 64. The contraction in the length of the wire due to change in
è Aø
59. As, Dl = = a l Dq temperature
Y
\ F = YAa Dq = aLDT = 1.2 ´ 10 -5 ´ 3 ´ ( - 170 - 30)
F = - 7. 2 ´ 10 -3 m
or Dq =
Y Aa
The expansion in the length of wire due to stretching force
33000
= = 10° C Fl (10 ´ 10) ´ 3
(3 ´ 10 11) ´ (10 -3) ´ (1.1 ´ 10 -5) = =
AY (0.75 ´ 10 -6) (2 ´ 10 11)
60. When body is at A and released, its velocity while reaching B = 2 ´ 10 -3 m
is v = 2g L
Net change in length
Tension in the wire at B is
= - 7.2 ´ 10 -3 + 2 ´ 10 -3
O A
90° = - 5 . 2 ´ 10 -3 m = - 5.2 mm
Negative sign shows contraction.
L
65. From the formula for thermal stress,
B æFö
ç ÷ = a1Y1 DT …(i)
è A ø1
mg
æFö
ç ÷ = a 2Y2DT …(ii)
mv 2 m è Aø 2
F = mg + = mg + (2gL) = 3mg
L L
(Since, temperature DT is same)
F L
As, Y= ´ æFö
A Dl ç ÷
è A ø 1 a1Y1
Dl F 3 mg \ =
\ = = æFö a1Y2
L YA YA ç ÷
è A ø1
3 ´ 1 ´ 10
= 11 = 10 -4
10 (3 ´ 10 -6) For thermal stress to be equal
= Y Ya a
61. The work done on the wire to produce a strain in it will be 1 1 2 2

stored as energy which is converted into heat when wire Y1 a 2 3 æ a1 ö


or = = ç as = 2 : 3÷
snaps suddenly. Due to it, the temperature increases. Y2 a1 2 è a2 ø
Dp
62. As, K = 66. Here, Dp = (1.165 ´ 10 5 - 1.01 ´ 10 5)
DV / V
DV 1 = 0.155 ´ 10 5 Pa
or Dp = K= ´ 7.5 ´ 10 10
V 100
and DV / V = 10 / 100 = 1 / 10
= 7.5 ´ 10 8 Nm-2
Dp
It is the change in pressure. \Bulk modulus, K=
DV / V
4Fl 4 ´ (1 ´ 9.8) ´ 2
63. As, Y = = 0.155 ´ 10 5
pD 2 Dl æ 22 ö -3 2 -3 =
ç ÷ (0.4 ´ 10 ) ´ (0.8 ´ 10 ) 1 / 10
è7ø
= 1.55 ´ 10 5 Pa
= 2 ´ 10 11 Nm-2
Telegram @unacademyplusdiscounts

Properties of
12 Liquids
JEE Main MILESTONE
< Density and Relative Density < Torricelli’s Theorem
< Pressure due to a Fluid Column < Surface Tension
< Pascals’ Law and its Applications < Surface Energy
< Archimedes’ Principle and Buoyancy < Applications of Capillary Action or Capillarity
< Flow of Liquids < Viscosity
< Energy of a Flowing Liquid < Poiseuille’s Formula
< Bernoulli’s Principle < Stokes’ Law

12.1 Density and Relative Density


Fluid is characterised by density r at every point which is defined as the ratio of the
mass of the fluid contained in an infinitesimal volume element around the point to
the volume of element.
Dm dm The substances which flow are
r = lim =
DV ® 0 DV dV called fluids. Fluids include both
Another term, specific gravity is also used to represent density, it is a relative liquids and gases. The science of
measure of density of substance (fluid) w.r.t. density of water at 4°C. Specific gravity fluids at rest is called fluid statics
of liquid in this part of fluid mechanics
r liquid Density of liquid fluid mass is stationary w.r.t.
= =
r water Density of water container, containing the fluid.
Fluid statics includes hydrostatic
Relative Density pressure, floatation, Pascal’s law
It is defined on the ratio of density of substance to the density of water at 4°C.
and Archimedes’ principle, while
hydrodynamics includes continuity
Density of substance
i. e. , Relative density = equation and Bernoulli’s principle
Density of water at 4° C
and Torricelli’s theorem.
Density is a positive scalar quantity. In case of a homogeneous isotropic substance,
density has no directional properties, so it is a scalar. It has dimensions [ML–3 T 0]. SI
unit is kgm–3 while, CGS unit is gcc–1 with 1 gcc–1 = 103 kgm–3
Mass of substance
Density of substance =
Volume occupied by the substance
Mass of a body
and Density of a body =
Volume of the body
Telegram @unacademyplusdiscounts

436 JEE Main Physics

Hence, for a solid body, Vr1 + Vr 2


r=
Density of body = Density of substance. V +V
r1 + r 2
While, for a hollow body, density of body is lesser than i. e. , r= =4 …(i)
that of substance 2
[QVbody = Vsubstance ] and when equal masses are mixed, m1 = m2 = m and
m
V1 =
r1
Important Points m
and V2 =
1. When immiscible liquids of different densities are poured in a r2
container, the liquid of highest density will be at the bottom while, m +m
\ r= 1
that of lowest density at the top and interfaces will be plane. m m
+
r1 r 2
2. If a liquid of mass m1 and density r1 and other liquid of mass m2 and
2 r1 + r 2
density r2 are mixed. \ =3 …(ii)
r1 + r 2
m m é mù
m1 + m2 = M and V = 1 + 2 êQV = r ú
r1 r2 ë û
From Eqs. (i) and (ii) r1 and r 2, we find that quantities of metals
are 2 and 6.
Density of mixed liquid,
M m1 + m2 Sm1
r= = =
V m1 m2
+
r1 r2
ém ù
Dê i ú 12.2 Pressure due to a
ë ri û
2 r1 r2
Fluid Column
If m1 = m2 or r =
r1 + r2 Consider a small surface of area A such as a small card
submerged in the fluid. The fluid on one side of the card
3. Relative density has no units and no dimensions. It is positive vector
quantity. exerts a force F on the card which is balanced by an equal
but opposite force is exerted by the fluid on the other side
4. If a liquid of volume V1 and density r1 and other liquid of volume V2 of the card. If the card is very small, we can neglect the
and density r2 are mixed. Then, we have difference in depth of the card. The pressure p is defined
m = r1V1 + r2V2 and V = V1 + V2 éQr = m ù as the ratio of the magnitude of the force to the area of the
êë V úû card i. e. ,
m r1V1 + r2V2 D riVi F
r= = = p= …(i)
V V1 + V2 SVi A
5. With increase in pressure due to a decrease in volume, density will For a point at a depth h below the surface of a liquid
increase. density r, pressure p is given by
r m/V V éQr = m ù p = p0 + hrg …(ii)
\ = = 0
r 0 m/V 0 V êë V úû
where, p0 is the atmospheric pressure.
p0
Sample Problem 1 When equal volumes of two metals
are mixed together, the specific gravity of alloys is 4. When
h ρ
equal masses of the same two metals are mixed together, the
specific gravity of alloy is 3. What is the specific gravity of each p
metal?
(a) 3 and 6 (b) 5 and 4
(c) 2 and 6 (d) 4 and 4 The excess pressure above atmospheric pressure is
Interpret (c) In case of two metals, usually called gauge pressure and total pressure is called
(m1 + m2) absolute pressure. Thus,
r=
V1 + V2 Gauge pressure = absolute pressure - atmospheric pressure
If equal volumes are mixed V1 = V2 = V and i. e. , hrg = p - p0 …(iii)
m1 = Vr1 and m2 = Vr 2
Telegram @unacademyplusdiscounts

Properties of Liquids 437

Important Points
1. At same point on a fluid pressure is same in all directions.

1
p4
a
h
p1 p3
2
p2
If container is accelerating down, then p2 - p1 = r(g - a )h and if
p1 = p2 = p3 = p4 container is accelerating down with acceleration > g, then the liquid
2. Forces acting on a fluid in equilibrium have to be perpendicular to its occupies upper part of the vessel.
surface. Because it cannot sustain the shear stress. 6. Variation of pressure, when fluid container is under both horizontal
3. In the same liquid pressure will be same at all points at the same and vertical acceleration. Consider a container containing liquid is
level. moving up with constant acceleration on an inclined plane as shown
in the figure.
For example, in the given figure

p0 p0 θ
a
p1 p2 h
2
p3 h1 p4
p5 p6
Y

φ X

p1 ¹ p2 In this case, pressure difference at two points in same horizontal level


p3 = p 4 and p5 = p6 separated by a distance s is given by
Further, p3 = p4 p1 - p2 = rs ´ a cos f
p0 + r1hg1 = p0 + r2gh2 Invertical plane,
or r1h1 = r2h2 p2 - p1 = r (g + a sin f)h
1 a cos f
or hµ tan q = ,
r g + a sin f

4. Barometer is used to measure atmospheric pressure while, where q is the angle made by free surface of liquid with horizontal.
manometer measures pressure difference, i .e ., gauge pressure. 7. Variation of pressure within an accelerating closed
container.
Vacuum
(p = 0) 2
p0 h
p0 s p
h a
h 1

Hg Here, all the points lying on a particular line making an angle of


(a) (b) tan-1(a/g ) with the horizontal have the same pressure. In present
Barometer p0 = hρg Manometer p – p0 = hρg situation, point 2 is the least pressure point, if the vessel is
completely closed, we can take its pressure to be zero.
5. Pressure at two points within a liquid at vertical separation of h when p1 = p2 + rgh + rsa
the liquid container is accelerating up are related by expression as p2 = 0
p2 - p1 = r(g + a )h So, p1 = rgh + rsa
Telegram @unacademyplusdiscounts

438 JEE Main Physics

Sample Problem 2 The two thigh bones (femurs), each of (i) Pascal’s law is used in the working of the hydraulic lift
2
cross-sectional area 10 cm support the upper part of a human which is used to support or lift heavy objects. In
body of mass 40 kg. Average pressure sustained by the femurs is hydraulic lift,
(a) 10 3 N/m2 (b) 2 ´ 10 5 Nm-2 A2
F2 = F1
-5 2 -3 -2 A1
(c) 6 ´ 10 N /m (d) 10 Nm
where A1, A2 = area of cross-section of smaller and
Interpret (b) Total corss-sectional area of the femurs is larger piston of hydraulic lift. F1 = force applied on
A = 2 ´ 10 cm2 = 2 ´ 10 -4 m2. The force acting on them is
smaller piston.
F = 40 kgwt = 400 N (taking g = 10 ms–2). This force is acting
(ii) Hydraulic lift is a force multiplying device which is
vertically down and hence, normally on the femurs. Thus, the used in dentist’s chair, car lifts and jacks, many
average pressure is
elevators and hydraulic brakes.
F 400
pav = = = 2 ´ 10 5 Nm–2
A 2 ´ 10 -4
Atmospheric Pressure
Sample Problem 3 The pressure on a swimmer 10 m The pressure exerted by atmosphere is called atmospheric
below the surface of a lake is pressure. At STP, the value of atmospheric pressure is
(a) 10 atm (b) 5 atm (c) 15 atm (d) 2 atm 1.01 ´ 105 Nm–2 or 1.01 ´ 106 dyne cm–2.
Interpret (a) Total pressure, p = pa + rgh, where pa is (i) The sudden fall in atmospheric pressure produces the
atmospheric pressure, r is density and g is acceleration due to possibility of a storm.
gravity. (ii) Various units of atmospherical pressure are mm or cm
\ p = 1.01 ´ 10 5 pa + 1000 kgm–3 ´ 10 ´ 10 of Hg column, torr ( = 1 mm of Hg column). Pressure is
also measured in units of atmospheric pressure. The
= 2.01 ´ 105 pa » 2 atm
pressure at a depth of 10 m in water is about
2 atmosphere. The unit of pressure used for
Sample Problem 4 The average depth of Indian Ocean is metrerological purposes is called the bar; one bar is
about 3000 m. Bulk modulus of water is 2.2 ´ 10 4 Nm –2, about 105 Nm-2 .
æ DV ö
g = 10 ms–2, then fractional compression ç ÷ of water at the
è V ø Sample Problem 5 If a room has dimensions
bottom of the Indian Ocean is 3 m ´ 4 m ´ 5 m. What is the mass of air in the room, if density of
(a) 1.36% (b) 20.6% (c) 13.9% (d) 0.52% air at NTP is 1.3 kgm -3?
Interpret (a) The pressure exerted by a 3000 m column of (a) 78 kg (b) 75 kg
water on the bottom layer (c) 76 kg (d) 78.5 kg
p = hrg = 3000 ´ 1000 ´ 10 Density of water is 103 kgm -3 and g = 10 ms-2.
7 –1 –2 7 –2
= 3 ´ 10 kg m s = 3 ´ 10 Nm
Interpret (a) As we know that density, r = m
æ DV ö V
Fractional compression ç ÷
è V ø or m = rV = 1.3 ´ 3 ´ 4 ´ 5 = 78 kg
7
Stress ´ (3 10 Nm ) –2 i. e. , mass of air in the room is 78 kg which is not possible. Since, air
= = = 1.36 ´ 10
B (2.2 ´ 10 9) is considered as to be weightless. But it is true.
DV
= 1.36% Sample Problem 6 In the above example, what force does
V
water exert on the base of a house tank of base area 1.5 m 2,
when it is filled with water upto a height of 1 m?
12.3 Pascals’ Law and (a) 1.5 ´ 10 4 N
(b) 2.5 ´ 10 4 N
its Applications (c) 3 ´ 10 4 N
It states that if gravity effect is neglected, the pressure at (d) 3.5 ´ 10 4 N
every point of liquid in equilibrium or rest is same.
Pascal’s law also states that the increase in pressure at one Interpret (a) We know,
point of the enclosed liquid in equilibrium of rest is p - p0 = hrg = 1 ´ 10 3 ´ 10 = 10 4 Nm–2
transmitted equally to all other points or liquid provided
F = Dp ´ S = 10 4 ´ 1.5 = 1.5 ´ 10 4 N
the gravity effect is neglected.
Telegram @unacademyplusdiscounts

Properties of Liquids 439

Sample Problem 7 At a depth of 1000 m is an ocean the (iii) The buoyant force acts at the centre of buoyancy
force acting on the window of area 20 cm ´ 20 cm of a which is the centre of gravity of the liquid displaced by
submarine at this depth, the interior of which is maintained at the body when immersed in the liquid.
sea-level atmospheric pressure is [Given, density of seawater is (iv) The line joining the centre of gravity and centre of
1.03 ´ 103 kgm -3, g = 10 ms-2] buoyancy is called central line.
(a) 3.2 ´ 10 8 N (b) 4.12 ´ 10 5 N (v) Metacentre, is a point where the vertical line passing
2
(c) 8.3 ´ 10 N (d) 3.1 ´ 10 –5
N through the centre of buoyancy intersects the central
line.
Interpret (b) Given, h = 1000 m, r = 1.03 ´ 103 kgm–3, gauge
pressure, pg = rgh Laws of Floatation
pg = 1.03 ´ 10 3 ´ 10 ´ 1000 When a body of density r B and volume V is immersed in a
5 liquid of density s, the forces acting on the body are
pg = 103 ´ 10 Pa » 103 atm
(i) The weight of body W = mg = Vr B g acting vertically
The pressure outside the submarine is p = pa + rgh and pressure downwards through the centre of gravity of the
inside it is pa . Hence, the net pressure acting on the window is body.
gauge pressure pg . (ii) The upthrust F = Vsg acting vertically upwards
Since, area of window is A = 0.4 m2, the force acting on it is through the centre of gravity of the displaced liquid
F = rg A = 103 ´ 10 5 Pa ´ 0.04 m2 = 4.12 ´ 10 5 N i.e., centre of buoyancy.

So, the following three cases are possible.


Case I The density of body is greater
12.4 Archimedes’ Principle and than that of liquid
Buoyancy (i. e. , r B > s ). In this case, as weight will
be more than upthrust, the body will
Whenever a body is immersed in a fluid, the fluid exerts sink. As shown in Fig. (a)
an upward force on the body, which is called the buoyant (a) ρB > σ
force. In fact, any body wholly or partially immersed in a
Case II The density of body is equal to
fluid is buoyed up by a force equal to the weight of the
the density of liquid (i. e. , r B = s ). In this
displaced fluid. This result is known as Archimedes’
case, W = F . so, the body will float fully
principle. Thus, buoyant force = Vi r l g, where Vi is the
submerged in neutral equilibrium
volume of immersed part of body and r l is the density of
anywhere in the liquid as shown in
fluid.
Fig. (b). (b) ρB = σ
If a body of volume V and density r s is completely
immersed in a liquid of density, r l , then its observed Case III The density of body is lesser
weight than that of liquid (i. e. , r B < s ). In this
case, W < F ,, so the body will move
Wob = Wactual - upthrust upwards and in equilibrium will float
= Vr s × g - Vr l × g partially immersed in the liquid such
that
= V (r s - r l ) g W = Vinsg (c) ρB < σ
Buoyant force or buoyancy
(i) It is an upward force acting on the body immersed in a [Vin is the volume of body in the liquid]
liquid.
or Vr B g = Vinrg [as W = mg = r RV g ]
(ii) It is equal to the weight of liquid displaced by the
immersed part of the body.
or Vr B = Vins …(i)
Telegram @unacademyplusdiscounts

440 JEE Main Physics

Some Particular Cases Interpret (a) Mass of the man = mass of water displaced
(i) In liquid, the apparent weight of the body decreases, = volume ´ density
and this decrease in its weight is equal to the upthrust 1
= 3 ´2 ´ ´ 10 3 kg = 60 kg
acting on the body. 100
Hence, apparent weight
wapp = V ( r S - r L ) g Check Point 1
(ii) If object is immersed in water, then
1. One small and one big piece of cork are pushed below the
weight of body in air
RD = surface of water. Which will have greater tendency to rise
Loss in weight in water
swiftly?
weight of body in air 2. The bags and suitcases are provided with broad handles. Why?
RD =
wt. in air – weight in water
3. Why is it easier to swim in sea water than in river water?
So, by weighing a body in air and in water, we can 4. A boat carrying a number of large stones is floating in a water
determine the relative density of the body. tank. What will happen to the water level, if the stones are
(iii) The upthrust on a body immersed in a liquid of density unloaded into water?
r L in a lift moving downward with acceleration a is 5. A piece of ice is floating in a vessel containing water. What will
be the effect on the level of water in jar, when ice melts and
F = Vr L |g - a |
the temperature of water falls from 4ºC to 1ºC ?
(iv) The upthrust on a body immersed in a liquid of density
r L in a lift moving upwards with acceleration a is
F = Vr L |g + a |
12.5 Flow of Liquids
(v) If lift is falling freely, then
F=0 The flow of liquids (fluids) is of three types

(vi) The torque of hydrostatic forces per unit width of the


1. Streamline Flow
wall of a dam is
rgH3 The streamline flow of a liquid is the flow in which each
t=
6 element of the liquid passing through a point travels along
Here, H = length of wall, the same path and with the same velocity as the
r = density of water preceeding element passing through the same point.
and g = acceleration due to gravity. Hence, it is a regular flow. The path followed by each
element is called streamline. The tangent drawn at any
Sample Problem 8 An ice-berg is floating partly immersed point of streamline gives the direction of the flow of liquid
in seawater of density 1.03 gcm–3. The density of ice is at that point. From figure, velocity at different points may
0.92 gcm–3. The fraction of the total volume of the ice-berg be different. Hence, in the figure
above the level of seawater is v1 = constant, v3
(a) 8.1 % (b) 11 % (c) 34 % (d) 0.8 % v2 = constant, v1
Interpret (b) Let v be the volume of the ice-berg outside the v3 = constant v2
sea water and V be the total volume of ice-berg. According to But v1 ¹ v2 ¹ v3
question,
0.92 V = 1.03 (V - v)
2. Turbulent Flow
v 1 - 0.92 11 A liquid can possess streamlined motion only when its
or = =
V 1.03 103 velocity is less than a limiting velocity, called the critical
v 11 velocity. When the velocity of the liquid becomes greater
´ 100 = ´ 100 » 11% than the critical velocity for the liquid, the different
V 103
elements of the liquid move along a zig-zag path. As a
Sample Problem 9 A boat having a length of 3 m and result of unsteady motion of the elements of the liquid
breadth 2 m is floating on a lake. The boat sinks by 1 cm, when along zig-zag paths, the liquid gets churned up. Such a
a man gets on it. The mass of the man is motion of the liquid is called turbulent flow.
(a) 60 kg (b) 62 kg (c) 72 kg (d) 128 kg
Telegram @unacademyplusdiscounts

Properties of Liquids 441

(a) 0.37 cm (b) 0.67 cm


3. Laminar Flow (c) 37 cm (d) 67 cm
If a liquid is flowing over a horizontal surface with a
steady flow and moves in the from of layers of different
Interpret (b) Water is considered to be perfectly
incompressible. Volume covered by the movement of smaller
velocities which do not mix with each other, then the flow piston inwards is equal to the volume moved outwards due to
of liquid is called laminar flow. Thus a flow, in which the longer piston.
liquid moves in layers is called a laminar flow. L1A1 = L2A2
2
æ1 ö
Reynold’s Number p ç ´ 10 -2÷
A1 è2 ø
Þ L2 = L1 = 2
´ 6 ´ 10 -2
It is a pure number which determines the nature of flow of A2 æ3 ö
liquid through a horizontal tube. p ç ´ 10 -2÷
è2 ø
Value of critical velocity for flow of liquid of density r and = 0.67 ´ 10 –2 m » 0.67 cm
coefficient of viscosity h flowing through a horizontal tube
of radius r is given by Note Atmospheric pressure is common to both pistons and has been
h
vc µ ignored.
rr
Reynold’s number (N R) is a unitless and dimensionless Sample Problem 11 In a car, lift compressed air exent a
number given by force F1 on a small piston having a radius of 5 cm. This pressure
rvr is transmitted to a second piston of radius 15 cm. If the mass of
NR =
h the car be lifted is 1350 kg, the pressure necessary to
If the value of Reynold’s number accomplish this task is
(i) lies between 0 to 2000, the flow of liquid is streamline (a) 1.9 ´ 10 5 Pa (b) 3 ´ 10 6 Pa
or laminar. (c) 6.5 ´ 10 3 Pa (d) 0.23 ´ 10 3 Pa
(ii) lies between 2000 to 3000, the flow of liquid is unstable
and changing from streamline to turbulent flow. Interpret (a) Since, pressure is transmitted undiminished
throughout the fluid
(iii) above 3000, the flow of liquid is definitely turbulent.
A1 p (5 ´ 10 -2 m) 2
F1 = F2 = (1350 N ´ 9.8 ms–2)
Equation of Continuity A2 p (15 ´ 10 -2 m) 2

Let us consider Q = 1470 N » 1.5 ´ 10 3 N


P
stream line flow of an v2 The air pressure that will produce this force is
ideal, non-viscous v1
F1 1.5 ´ 10 3 N
fluid through a tube A2 p= = = 1.9 ´ 10 5 Pa
A1 p (5 ´ 10 –2) 2
of varying cross-
A1
section. Let at two
sections, the cross-section areas be A1 and A2 respectively
Note This is almost double the atmospheric pressure.
and fluid flow velocities are v1 and v2, then according to
equation of continuity
A1v1r1 = A2v2 r 2
12.6 Energy of a Flowing Liquid
where, r1 and r 2 are the respective densities of fluid. There are three types of energies in a flowing liquid.
Equation of continuity is based on the conservation of
mass. Pressure Energy
If fluid flowing is incompressible, then If p is the pressure on the area A of a fluid, and the liquid
r1 = r 2
moves through a distance l due to this pressure, then
and equation of continuity is simplified as
Pressure energy of liquid = work done
A1v1 = A2v2
= force ´ displacement = pAl
Sample Problem 10 Two syringes of different The volume of the liquid is Al.
cross-sections (without needles) filled with water are connected
Hence, pressure energy per unit volume of liquid
with a tightly fitted rubber tube filled with water. Diameters of
the smaller piston and longer pistion are 1 cm and 3 cm pAl
= =p
respectively. If the smaller piston is pushed in through 6 cm, Al
how much does the longer piston move out
Telegram @unacademyplusdiscounts

442 JEE Main Physics

Kinetic Energy In the same interval Dt, the fluid initially at D moves to E, a
distance equal to v2Dt. Pressures p1 and p2 act as shown on
If a liquid of mass m and volume V is flowing with velocity the plane faces of areas A1 and A2 binding the two regions.
1
v, then the kinetic energy is mv2. The work done on the fluid at left end (BC ) is
2
W1 = p1 A1 (v1Dt ) = p1DV
\ Kinetic energy per unit volume of liquid
Since the same volume DV passes through both the
1 æ mö 1
= ç ÷ v2 = rv2 regions (from the equation of continuity) the work done by
2 èV ø 2
the fluid at the other end (DE ) is
Here, r is the density of liquid. W2 = p2 A2 (v2Dt ) = p2DV
or work done on the fluid is - p2DV .
Potential Energy
\ The total work done on the fluid is
If a liquid of mass m is at a height h from the reference line
W1 - W2 = ( p1 - p2 ) DV
(h = 0), then its potential energy is mgh.
\Potential energy per unit volume of the liquid Part of this work goes into changing the kinetic energy of
the fluid, and part goes into changing the gravitational
æ mö
= ç ÷ gh = rgh potential energy. If the density of the fluid is r and
èV ø
Dm = rA1v1Dt = r DV is the mass passing through the pipe
in time Dt, then change in gravitational potential energy is
DU = rg DV (h2 - h1 )
12.7 Bernoulli’s Principle
The change in its kinetic energy is
Some useful properties for steady or streamline flows can
æ 1ö
be obtained using the principle of conservation of energy. DK = ç ÷ r DV (v22 - v12 )
è2ø
Consider a fluid moving in a pipe of varying
cross-sectional area. Let the pipe be at varying heights as Using the principle of work-energy theorem, we have
shown. æ 1ö
( p1 - p2 ) DV = ç ÷ r DV (v22 - v12 ) + rg DV (h2 - h1 )
D E è2ø
A2
C 2 Now, we divide each term by DV , we get
B 1
1
p2 ( p1 - p2 ) = r (v22 - v12 ) + rg (h2 - h1 )
A1 2

p1 v2Dt Re-arranging the above terms, we have


æ 1ö æ 1ö
v1Dt h2 p1 + ç ÷ rv12 + rgh1 = p2 + ç ÷ rv22 + rgh2
è2ø è2ø
h1
This is Bernoulli’s equation. Since 1 and 2 refer to any two
locations along the pipeline, Therefore, the expression in
An incompressible fluid is flowing through the pipe in a general can be written as
steady flow. Its velocity must change as a consequence of æ 1ö
p + ç ÷ rv2 + rgh = constant
equation of continuity. A force is required to produce this è2ø
acceleration, which is caused by the fluid surrounding it,
the pressure must be different in different regions. Bernoulli’s theorem may be stated as we move along a
Consider the flow at two regions 1 (i. e. , BC ) and 2 (i. e. , DE ). streamline, the sum of the pressure energy ( p), the kinetic
Consider the fluid initially lying between B and D. In a æ rv2 ö
energy (KE) per unit volume ç ÷ and the potential energy
small interval of time (Dt ), this fluid would have moved. Let è 2 ø
v1 is the speed of B and v2 at D, then fluid initially at B has
(pE) per unit volume ( rgh) remains constant.
moved a distance v1Dt to C (v, Dt is small enough to assume
constant cross-section along BC).
Telegram @unacademyplusdiscounts

Properties of Liquids 443

Limitations of Bernoulli’s Theorem


Important Points of Bernolli’s Theorem
(i) While deriving the Bernoulli’s equation, it is assumed
1. When a fluid is at rest, i .e ., its velocity is zero everywhere, Bernoulli’s that velocity of every particle of liquid across any
equation becomes cross-section of tube is uniform. Practically, it is not
p1 + rgh1 = p2 + rgh2 correct. Infact, the particles of the liquid in the inner
(p1 - p2 ) = rg (h2 - h1) most (i. e., central layer) have maximum velocity and
2. Bernoulli’s equation ideally applies to fluids with zero viscosity or those on layer in contact with the tube have least
non-viscous fluids. velocity. Therefore, we should take the mean velocity
of the liquid.
3. Bernoulli’s equation applies to fluids which must be incompressible,
(ii) The viscous drag of the liquid which comes into play
as the elastic energy of the fluid is also not taken into consideration.
when the liquid is in motion has not been taken into
4. Bernoulli’s equation does not hold for steady or turbulent flows, account.
because in that situation velocity and pressure are constantly
(iii) While deriving the above equation, it is assumed that
flctuating in time.
there is no loss of energy when liquid is in motion.
5. Bernoulli’s equation for flowing liquid is Infact some KE is converted into heat and is lost.
1
p + rv 2 + rgh = constant (iv) If the liquid is flowing along a curved path, the energy
2
due to centrifugal force should also be considered.
Dividing this equation by rg, we have
p v2 Sample Problem 12 At what speed, the velocity head of
+ + h = constant
rg 2 g water is equal to pressure head of 40 cm of Hg?
v2 p (a) 10.3 ms-1 (b) 2.8 ms-1
In this expression, is velocity head and is pressure head.
2g rg (c) 5.5 ms-1 (d) 8.4 ms-1

6. Bernoulli’s theorem for unit mass of liquid Interpret (a) From Bernoulli’s equation,
p 1 2 p v2
+ v = constant + + h = constant
r 2 rg 2 g
v2 p
Here, is velocity head and is pressure head.
Applications Based on Bernoulli’s Principle 2g rg

(i) The action of carburetor, paintgun, scent sprayer, Given that, velocity head = pressure head
atomizer and insect sprayer is based on Bernoulli’s v2 p
=
principle. 2 g rg
(ii) The action of Bunsen’s burner, gas burner, oil stove and 2p
Þ v2 =
exhaust pump is also based on Bernoulli’s principle. r
(iii) Motion of a spinning ball (Magnus effect) is based on Given, p = 40 cm of Hg = 40 ´ 10 -2 ´ 9.8 ´ 13.6 ´ 10 3
Bernoulli’s theorem.
2 ´ 13.6 ´ 10 3 ´ 40 ´ 10 –2 ´ 9.8
(iv) Blowing of roofs by wind storms, attraction between v2 =
two closely parallel moving boats, fluttering of a flag, 10 3
–1
etc., are also based on Bernoulli’s theorem. Þ v = 10.32 ms
(v) Aerofoil or lift on aircraft wing : Aerofoil is shaped to
provide an upward dynamic lift. When the aerofoil Sample Problem 13 A manometer connected to a closed
moves against the wind, the flow speed on top is tap reads 3.5 ´ 10 5 Nm -2 . When the value is opened, the
higher than that below it. There is an upward force reading of manometer falls to 3 ´ 10 5 Nm -2, then the velocity of
resulting in dynamic lift of the wings. It is found that
flow of water is
1
Dynamic lift F = A ( p1 - p2 ) = Ar (v22 - v12 ) (a) 100 ms-1 (b) 10 ms-1
2
(c) 1 ms-1 (d) 10 10 ms-1
Here, A = surface area of aerofoil
v1 = velocity below Interpret (b) Bernoulli’s theorem for unit mass of liquid is
p 1 2
v2 = velocity above + v = constant
r 2
and r = density of air
As the liquid starts flowing, it pressure energy decreases
Telegram @unacademyplusdiscounts

444 JEE Main Physics

1 2 p1 - p2
2
v =
r 12.8 Torricelli’s Theorem
1 2 3.5 ´ 10 5 - 3 ´ 10 5 It states that the velocity of efflux i. e. , the velocity with
Þ v =
2 10 3 which the liquid flows out of an orifice (i. e. , a narrow hole)
2 ´ 0.5 ´ 10 5 in a vessel containing liquid is equal to that which a freely
Þ v2 = falling body would acquire in falling through a vertical
10 3
distance equal to the depth of orifice below the free surface
Þ v 2 = 100
of liquid in vessel. Quantitatively, velocity of efflux,
Þ v = 10 m/s v = 2 gh, where h is the depth of orifice below the free
surface of liquid in vessel.
Sample Problem 14 A cylinder of height 20 m is
completely filled with water. The velocity of efflux of water (in
ms-1) through a hole on the side wall of the cylinder near its h
bottom is (ms-1) v
(a) 10 (b) 20 H
(c) 30 (d) 40
Interpret (b) Let p0 is the atmospheric pressure, r the density of
liquid and v the velocity at which water is coming out. Applying the R
Bernoulli’s theorem just inside and outside the hole,
(i) From figure, volume of the liquid coming out per
pv 2 second through an orifice of area of cross-section a at a
pinside + rgh + 0 = poutside +
2 depth h below the free surface of liquid, in the vessel is
rv 2 V = av = a 2 gh.
p0 + rgh = p0 +
2 (ii) The time after which the liquid strikes the horizontal
Þ v = 2 gh surface at the base level of liquid is
2 (H - h)
Þ v = 2 ´ 10 ´ 20 t=
g
Þ v = 20 ms–1
(iii) Horizontal range,

Sample Problem 15 Water is flowing with a speed of 2 (H - h)


. R = vt = 2 gh ´
2 ms-1 in a horizontal pipe with cross-sectional area decreasing g
from 2 ´ 10 -2m 2 to 0.01 m2 at pressure 4 ´ 10 4 Pa. What will be
= 2 h ( H - h)
the pressure at smaller cross-section?
(a) 2 ´ 10 4 Pa (b) 3.4 ´10 4 Pa Maximum range, Rmax = H , when h = H /2.
4
(c) 2.4 ´ 10 Pa (d) 4 ´ 10 Pa4 Regarding this theorem following points are worth noting
(a) From above relation,
Interpret (b) Here, v1 = 2 ms–1; A1 = 2 ´ 10-2 m2; v µ h . This implies that v1
p1 = 4 ´ 10 4 Pa; A2 = 0.01m2; p2 = ? greater is the distance of v2
As A1v1 = A2v 2 hole from the free surface
v3
Av of liquid greater will be
or v2 = 1 1 the velocity of efflux.
A2
2 ´ 10 -2 ´ 2 (b) The range is same for
= = 4 ms–1 h
0.01 liquid coming out of y
1 1 holes at same distance h H
Now, p1 + rv12 = p2 + rv 22
2 2 below the top and above h
1 the bottom, though their
or p2 = p1 + r (v12 - v 22) x=x'
2 velocities of efflux (v1 and A B
1 v2) and time taken (t1 and
or p2 = 4 ´ 10 4 + ´ 10 3 (2 2 - 4 2)
2 t2) in falling to the ground are different.
= 4 ´ 10 4 - 6 ´ 10 3
But v1t1 = v2t2 = R = 2h ( H - h )
= 3.4 ´ 10 4 Pa
Telegram @unacademyplusdiscounts

Properties of Liquids 445

(c) If a is the area of orifice at A


a depth h below the free h Check Point 2
surface and A is the area H a
of container, the volume 1. In a stream-lined flow, what is the velocity of the liquid in
of liquid coming out of contact with the containing vessel?
orifice per second will be 2. Why does the speed of a liquid increase and its pressure
decrease, when the liquid passes through a constriction in a
= av = a 2 gh (as v = 2 gh )
pipe? Explain.
If the hole is at the bottom of the tank, time taken to 3. Why two ships moving in parallel directions close to each other
empty the tank is get attracted?
A 2H 4. If a small ping pong ball is placed in a vertical jet of air or
t=
a g water, it will rise to a certain height above the nozzle and stay
at that level. Explain.
Venturimeter
It is a device used to measure the rate of flow of fluids
through pipes. In the arrangement shown, the rate of flow
of fluid V is given by
12.9 Surface Tension
The property of a liquid at rest by virtue of which its free
h surface behaves like a stretched membrane under tension
and tries to occupy as small area as possible is called
surface tension.
A C
B
A1 A2 E B
v1 v2

2 gh
V = A1 A2
( A12 - A22 ) F
A
2 ( p1 - p2 )
V = A1 A2
r ( A12 - A22 ) If we draw an imaginary line AB in any direction in a
liquid surface, the surface on either side of this line exerts
Sample Problem 16 The diameter of a pipe at two points, a pulling force on the surface of other side. This force is at
where a venturimeter is connected is 8 cm and 5 cm and the right angles to the line AB. The magnitude of such a force
difference of levels in it is 4 cm. The volume of water flowing per unit length of the line drawn on the surface of the
through the pipe per second is liquid gives the measure of the surface tension. Thus,
(a) 1889 ccs–1 (b) 1520 ccs–1 Force F
Surface tension, S = =
(b) 1321 ccs–1 (d) 1125 ccs–1 Length L
Interpret (a) Here, r1 = 8/2 = 4.0 cm; SI unit of surface tension is N/m or J/m2. It is a scalar and
r2 = 5 /2 = 2.5 cm; h = 4 cm its dimensional formula is [MT –2].
Now, A1 = pr12 = p ( 4) 2 = 16 pcm2 Surface tension is a molecular phenomenon which is due
and A2 = pr22 = p (2.5) 2 = 6.25 p cm2 to cohesive force and the root cause of the force is
electrical in nature.
Here, r = rm
So, the rate of flow of water in venturimeter is given by Surface tension of a liquid depends only on the nature of
liquid and is independent of the surface area of film or
2 gh
V = A1 A2 length of the line considered.Small liquid drops are
( A12 - A22)
spherical due to the property of surface tension.
2 ´ 980 ´ 4
= 6.25 p ´ 16 p Surface tension of a liquid decreases with an increase in
(16 p ) 2 - (6.25 p ) 2
temperature. A highly soluble substance like sodium
100 p 2 ´ 28 10 chloride in water, increases the surface tension of water.
=
(16 p - 6.25 p ) (16 p +6.25 p ) But the sparingly soluble substance like phenol when
dissolved in water, decreases the surface tension of water.
= 1889 ccs–1
Telegram @unacademyplusdiscounts

446 JEE Main Physics

Force due to Surface Tension If we establish a relation between surface energy and
surface tension, then it is found that the surface tension of
If a body of weight w is placed on the liquid surface whose liquid is numerically equal to its surface energy
surface tension is S. If F is the minimum force required to W
pull it away from the water then value of F for different Hence, S= or W = S DA
DA
bodies can be calculated from the following table
i. e. , surface tension may be defined as the amount of work
Body Force done in increasing the area of the liquid surface by unity
Needle (length) F = 2 lS
against the force of surface tension at constant
temperature.
Hollow disc (inner radius = r1, outer F = 2 p (r1 + r2 ),S = 4 pS
radius = r2 ) When the surface area of a liquid is increased, work is
Circular plate or disc F = 2 prS done against the cohesive force of molecules and this
(Radius = r) work is stored in the form of additional surface energy.
Square plate F = 4lS Increase in surface potential energy,
Square frame (side = l ) F = 8lS DU = Work done (DW ) = S × DA
where, DA is the increase in surface area of the liquid.
Sample Problem 17 A rectangular plate of dimensions (i) Work done in blowing a liquid drop
6 cm ´ 4 cm and thickness 2 mm is placed with its largest face If a liquid drop is blown up from a radius r1 to r2, then
flat on the surface of water. What will be the downward force work done for that is
on the plate due to surface tension? (Surface tension of water is W = S ( A2 - A1 ) = S × 4 p (r22 - r12 )
7.0 ´ 10 –2 Nm -1)
(ii) Work done in blowing a soap bubble
(a) 1.8 ´ 10 –2 N (b) 1.4 ´ 10 –2 N
As a soap bubble has two free surfaces, hence, work
(c) 2 ´ 10 -2 N (d) 2.5 ´ 10 –2 N
done in blowing a soap bubble so as to increase its
Interpret (b) Here, l = 6 ´ 10 -2 m; radius from r1 to r2 is given by
b = 4 ´ 10 -2 m; W = S × 8 p (r22 - r12 )

d = 2 ´ 10 -3 m and (iii) Work done in splitting a bigger drop into n smaller


S = 7.0 ´ 10 –2
Nm–1 droplets
If a liquid drop of radius R is split up into n smaller
Force on the plate due to surface tension is
droplets, all of same size, then radius of each droplet
F = 2 ( l + b) S
r = R × (n )-1/ 3
= 2 (6 ´ 10 -2 + 4 ´ 10 -2) ´ 7.0 ´ 10 –2
and work done W = S × 4 p (nr 2 - R2 ) = S × 4 pR2 (n1/ 3 - 1)
= 1.4 ´ 10 –2 N
(iv) Coalescence of drops
If n small liquid drops of radius r each combine
14.10 Surface Energy
1 3/
together so as to form a single bigger drop of radius
= × R, nthen
r in the process energy is released.
The molecules on the liquid surface experience a net Release of energy is given by
downward force. So, to bring a molecule from the interior DU = S × 4 p (nr 2 - R2 ) = S × 4 pr 2 n (1 - n -1/ 3 )
of the liquid to the free surface some work is required to be
done against the intermolecular forces of attraction. This Sample Problem 18 A rectangular film of liquid is
work done is stored in the surface film of the liquid as its extended from 5 cm ´ 3 cm to 6 cm ´ 5 cm. If the work done is
potential energy. 3.0 ´ 10 –4 J. The surface tension of liquid is
The potential energy per unit area of the surface film is called (a) 0.5 Nm–1 (b) 0.1 Nm–1
the surface energy. (c) 0.2 Nm–1 (d) 2 Nm–1
It may also be defined as the amount of work done in Interpret (b) Increase in area,
increasing the surface area of the film by unity. Thus, DA = 2 (6 ´ 5 – 5 ´ 3) = 2 ´ 15 cm2 Film has 2 free surfaces
surface energy = 30 ´ 10 -4 m
work done in increasing the surface area
= As, work done, W = surface tension ´ increase in area
increase in surface area 3.0 ´ 10 –4 = S ´ 2 ´ 30 ´ 10 -4
or S = 0.1Nm–1
Telegram @unacademyplusdiscounts

Properties of Liquids 447

Sample Problem 19 Surface tension of a detergent When two soap bubbles of same material having different
–2
solution is 2.8×10 What is the work done in blowing a radii r and R ( > r ) are combined to form a double bubble,
bubble of 2 cm diameter? then
(a) 4 ´ 10 -6 J (b) 70.3 ´ 10 –6 J p0
p0
(c) 50.8 ´ 10 –6 J (d) 60.8 ´ 10 –6 J
r R
Interpret (b) Given that, pr
pr p
R
–2 –1 pR
S = 2.8 ´ 10 Nm
2
and R = = 1 cm = 0.01m
2 4S
As soap bubble has two free surfaces, pr - p0 =
r
\ Work done, W = S ×8 pR 2 4S
= 2.8 ´ 10 –2 ´ 8 ´ 3.14 ´ (0.01) 2 and pR - p0 =
R
–6
= 70.3 ´ 10 J æ 1 1ö
\ pr - pR = 4 S ç - ÷
è r Rø
Applications of Surface Tension Radius of the common surface is given,
(i) Oil spreads over the water surface, because the surface rR
tension of oil is smaller than the water. R0 = .
R-r
(ii) The surface tension of points and all lubricating oils is
If two plates are placed in contact with a thin film of liquid
low.
in between them to pull them apart, a large force is
(iii) The stromy waves at the sea are calmed by pouring oil needed.
on the sea water.
2S
(iv) The surface tension of antise ptics like dettol is low Excess pressure in this case is , where d is the
d
because they spread faster.
separation between the plates.
(v) The surface tension of soap solution is low, therefore, it
can spread over large area. Force required to separate two plates, each of area A, is
2 A´S
given by F = .
Surface Tension of Drops and Bubbles d

Due to the property of surface tension, a drop or bubble Sample Problem 20 There is an air bubble of radius
tends to contract and so compresses the matter enclosed. 1.0 mm in a liquid of surface tension 0.075 Nm–1 and density
This in turn increases the internal pressure which 103 kgm–3. The bubble is at a depth of 10.0 cm below the free
prevents further contraction and equilibrium is achieved. surface. By what amount is the pressure inside the bubble
So, in equilibrium, the pressure inside a bubble or drop is greater than the atmospheric pressure?
greater than outside and difference of pressure between (a) 1030 Nm–1
two sides of the liquid surface is called excess pressure. (b) 1230 Nm–1
Excess pressure in different cases is given below : (c) 1130 Nm–1
p0 (d) None of the above

p0 p0 Interpret (c) Here, r = 1.0 mm = 10 -3 m ;


S = 0.075 Nm–1,
p p p p1i
r = 10 3 kgm–3 ,
h = 10 cm = 0.10 m
Drop Air bubble Soap bubble The pressure inside the bubble, which is greater than the
atmospheric pressure is
2S
Excess pressure inside a drop, Dp = 2S
r = + hr g
r
2S
Excess pressure inside an air bubble in air, Dp = 2 ´ 0.075
r = + 0.10 ´ 10 3 ´ 9.8
10 -3
4S
Excess pressure inside a soap bubble, Dp = = 150 + 980 = 1130 Nm–2
r
Telegram @unacademyplusdiscounts

448 JEE Main Physics

Angle of Contact 12.11 Applications of Capillary


The angle of contact between a liquid and a solid is
defined as the angle enclosed between the tangents to the
Action or Capillarity
liquid surface and the solid surface inside the liquid, both (i) It is the phenomenon of rise or fall of liquid in a
the tangents being drawn at the point of contact of the capillary tube.
liquid with the solid. (ii) The root cause of capillarity is the difference of
pressure on the two sides of liquid meniscus in the
capillary tube.
θ
?
(iii) The height h through which a liquid will rise in a
θ?
capillary tube of radius r, which wets the sides of the
tube, will be given by
2 S cos q 2 S
h= =
rrg Rrg
where, S is the surface tension of liquid, q is the angle of
(i) The angle of contact depends upon contact, r is the density of liquid and g is the
(a) the nature of solid and the liquid in contact, acceleration due to gravity. R is the radius of curvature
(b) the given pair of the solid and the liquid, of liquid meniscus.
(c) the impurities. (iv) If q < 90°, cos q is positive, so h is positive i. e., liquid rises
in a capillary tube.
(ii) The angle of contact does not depend upon the
inclination of the solid in the liquid. (v) If q > 90°, cos q is negative, so h is negative i. e., liquid
falls in a capillary tube.
(iii) The value of angle of contact (q) lies between 0° and
180°. For ordinary water and glass, q = 8°. For silver and (vi) If a capillary tube is of insufficient length as compared
pure water, q = 90°. For alcohol and clean glass, q = 0°. to height to which liquid can rise in the capillary tube,
then the liquid rises upto the full length of capillary
(iv) The increase in temperature increases the angle of
tube but there is no overflowing of the liquid in the
contact.
form of fountain. It is so because the liquid meniscus
(v) The angle of contact decreases with the addition of adjusts its radius of curvature so, that hR = a constant
impurities in the liquid. i. e., hR = h ¢R¢.
(vi) The materials used for water proofing increases the (vii) The height of the liquid column in a capillary tube on
angle of contact as well as surface tension. the surface of moon is six times than that on the earth.
(vii) If a liquid wets the sides of containing vessel, then the (viii) Rise of liquid in a capillary tube does not violate law of
value of angle of contact is acute i. e., less than 90°. conservation of energy.
(viii) If a liquid does not wet the sides of containing vessel, (ix) When a capillary tube dipped vertically in a liquid is
then the value of angle of contact is obtuse i.e., greater tilted, length of the liquid (l) in capillary tube increases
than 90°. but vertical height of liquid (h) in the tube above the
surface of liquid in trough remains the same.
Shape of Menicus
(i) If a liquid wets the sides of the vessel containing liquid,
the shape of liquid meniscus is concave upwards. In h
this case, force of cohesion between liquid molecules α l
is less than force of adhesion between liquid and
vessel molecules.
(ii) If a liquid does not wet the sides of the vessel
containing liquid, the shape of liquid meniscus is
convex upwards. In this case, force of cohesion
between liquid molecules is greater than the force of
adhesion between liquid and vessel molecules. h
l= or h = l cos a
(iii) The shape of liquid meniscus depends upon the cos a
molecular forces and is independent of the gravity (x) If m is the mass of the liquid which rises in a capillary
pull. tube of radius r, then
2 prS
mg = 2 prS or m =
g
Telegram @unacademyplusdiscounts

Properties of Liquids 449

Hot Spot Capillary Rise


The water meniscus in the tube is along a circle of circumference 2 pr Interpret (b) The excess pressure in a bubble of gas in a
which is in confact with the glass. Due to the surface tension of water, 2S
liquid is given by , where S is the surface tension of the liquid
a force equal to S per unit length acts at all points of the circle. If the r
angle of contact is q, then this force is directed invward at an angle q gas interface. The radius of the bubble is r.
from the wall of the tube. In accordance with Newton’s third law, the The pressure outside the bubble p0 equals atmospheric pressure
tube exerts an equal and opposite force S per unit length on the plus the pressure due to 8 cm of water column. That is
circumference of the water meniscus. This force which is directed
p0 = (1.01 ´ 10 5 Pa +0.08 m ´ 1000 kg m–3 ´ 9.80 ms–2)
outward, can be resolved into two components. S cos q per unit length
acting vertically upward and S sin q per unit length acting horizontally p0 = 1.01784 ´ 10 5 Pa
outward. The pressure inside the bubble is
Considering the entire circumference 2 pr, for each horizontal 2S
pi = p0 +
component T sin q there is an equal and opposite component and the r
two neutralise each other. The vertical components being in the same (2 ´ 7.3 ´ 10 –2 Pa.m)
direction are added up to give a total upward force (2 pr ) ( S cos q). It is = 1.01784 ´ 10 5 Pa +
10 -3 m
this force which supports the weight of the water column so raised.
= (1.01784+0.00146) ´ 10 5 Pa
Thus,
= 1.02 ´ 10 5 Pa
S cos θ S cos θ

S cos θ (2πr) Note This is a 100% increase in pressure from surface level. At a
θ θ depth of 1 km the increase in pressure is 100 atm. Sub marines are
sin θ S sin θ designed to withstand such enormous pressures.
θ θ
Sample Problem 22 The lower end of a capillary tube
is dipped into water and it is seen that water rises through
7.5 cm in the capillary. Given surface tension of water is
7.5 ´ 10 –2 Nm -1 and angle of contact between water and
( S cos q) (2 pr ) = Weight of the liquid column = ( pr2rgh) glass capillary tube is zero. What will be the diameter of the
2S cos q
capillary tube? (Given, g = 10 ms-2.)
\ h=
rrg (a) 0.2 mm (b) 0.3 mm
(c) 0.4 mm (d) 0.5 mm
Note The result has following notable features, Interpret (c) Given, h = 7.5 cm
(i) If the contact angle q is greater than 90°, the term cos q is
negative and hence, h is negative. The expression, then gives the = 7.5 ´ 10 -2 m, S = 7.5 ´ 10 –2 Nm–1,
depression of the liquid in the tube q = 0 °, 2 r = ?
(ii) Suppose a capillary tube is held vertically in a liquid which has a 2 S cos q
concave meniscus, then capillary rise is given by As, h=
rrg
2S cos q 2S æ as R = r ö
h= = ç ÷ 4 S cos q
rrg Rrg è cos q ø 2r =
hr g
2S
or hR = = constant 4 ´ 7.5 ´ 10 –2 ´ cos 0°
rg \ =
7.5 ´ 10 –2 ´ 10 3 ´ 10
Sample Problem 21 The lower end of a capillary tube of Þ = 4 ´ 10 –4 m
diameter 2 mm is dipped 8 cm below the surface of water in a Þ = 0.4 mm
beaker. The surface tension of water at temperature of the
experiment is 7.3 ´ 10 –2 Nm –1, 1 atmospheric pressure
= 1.01 ´ 10 5 Pa, desnity of water = 1000 kg/m3, g = 9.8 ms–2, Sample Problem 23 Assuming that the density of
then the pressure inside the bubble is atmosphere does not change with altitude. How high would
(a) 2.13 ´ 10 3 Pa (b) 1.02 ´ 10 5 Pa the atmosphere extend? (Given, density of the atmosphere
(c) 5 ´ 10 -5 Pa (d) 7.3 ´ 10 –3 Pa at sea level is 1.29 kg /m3).
(a) 2 km (b) 4 km (c) 8 km (d) 16 km
Telegram @unacademyplusdiscounts

450 JEE Main Physics

Interpret (c) Pressure = rgh = density ´ gravity ´ height layer acts in a direction opposite to the relative velocity of
3 –2
Given, r = 1.29 kg/m , g = 9.8 ms , p = 1.01 ´ 10 Pa 5 flow of fluid.

\ 1.01 ´ 10 5 = 1.29 ´ 9.8 ´ h Its unit is poise or dyne cm -2 s in CGS system and
1.01 ´ 10 5 poiseuille or deca poiseuille or Newton-s-m -2 in SI system.
Þ h= » 7989 m It is a scalar quantity.
1.29 ´ 9.8
Þ h »8m 1 poiseuelle = 1 deca poiseucle = 10 poise

Note In reality the density of air decreases with height. So does the Note In case of a steady flow of a liquid of viscosity hin a capillary tube
value of g. The atmospheric cover extends with decreasing pressure over of length L and radius r under a pressure difference p across it, the
100 km. We should also note that the sea level atmospheric pressure is velocity of flow at a distance x from the axis is given by
not always 760 mm of Hg. A drop in the Hg level by 10 mm or more is a p
v= (r 2 - x 2 )
sign of an approaching storm. 4 hL

Sample Problem 28 A Film


Check Point 3 metal block of area 0.10 m 2 is
1. In summer, cotton dress is preferable. Why? connected to a 0.010 kg mass
via a string that passes over an
2. Oil is poured on calm sea waves. Why?
ideal pully (considered
3. Why smearing of glycerine over the glass window prevents massless and frictionless). A
rain drops from sticking to it?
liquid with a film thickness of
4. Why does a small piece of camphor dance about on the water 0.01 kg
0.30 mm is placed between the
surface? block and the table. When
5. What is the value of surface tension of a liquid at critical released the block moves to the right with a constant speed of
temperature? 0.085 ms -1. The coefficient of viscosity of the liquid is
6. Name the material in whose capillary, water will descend (a) 45.2 ´ 10 2 Pa-s (b) 13.4 ´ 10 –4 Pa -s
instead of rising? –3
(c) 3.45 ´ 10 Pa-s (d) 1.42 ´ 10 –2 Pa -s

Interpret (c) The metal block moves to the right because of the
tension in the string. The tension T is equal in magnitude to the
12.12 Viscosity weight of the suspended mass m. Thus, the shear force
F = T = mg = 0.01 kg ´ 9.8 ms–2 = 9.8 ´ 10 -2N
The property of a fluid due to which it opposes the relative
F 9.8 ´ 10 –2
motion between its different layers is called viscosity (or Shear stress on the fluid = =
A 0.10
fluid friction or internal friction) and the force between the
V 0.085
layers opposing the relative motion is called viscous force. Strain rate = =
i 0.030
According to Newton, the frictional force F (or viscous Stress
force) between two layers depends upon the following h=
Strain rate
factors
(9.8 ´ 10 –2 N) (0.30 ´ 10 –3 m)
(i) Force F is directly proportional to the area ( A) of the h=
(0.085 ms–1) (0.10 m2)
layers in contact, i. e.,
h = 3.45 ´ 10 -3 Pa-s
FµA
(ii) Force F is directly proportional to the velocity gradient
æ dv ö
ç ÷ between the layers. Combining these two, we
è dy ø 12.13 Poiseuille’s Formula
have
In case of steady flow of a liquid of viscosity h in a capillary
dv dv
FµA or F = -hA tube of length L and radius R under a pressure difference
dy dy
P across it, the volume of liquid flowing per second is
where, h is a constant called coefficient of viscosity or given by
simply viscosity of fluid of liquid is equal to the tangential dQ ppR4
force required to maintain a unit velocity gradient =
dt 8 hL
between two parallel layers of liquid each of area unity.
The negative sign shows that viscous force on a liquid This is called Poiseuille’s formula.
Telegram @unacademyplusdiscounts

Properties of Liquids 451

Poiseuille’s equation can also be written as, body will fall with a constant velocity, called terminal
p - p2 Dp velocity vT
Q= 1 =
æ 8 hL ö X
ç 4÷ Upthrust = πρ3σg
è pR ø 3
8 hL
Here, X= F = 6 πηrv
pR4
This equation can be compared with the current equation
DV
through a resistance, i. e. , i =
R
Here, DV = potential difference
4 πr3ρg
and R = electrical resistance W=
3
Fig (a)
For current flow through a resistance, potential difference .

is a requirement, similarly for flow of liquid through a 4


pipe, pressure difference is must. 6 phrvT = pr3 ( r - s ) g v
3
Problems of series and parallel combination of pipes can be 2 r 2 (r - s ) g vT
or vT =
solved in the similar manner as is done in case of an 9 h
electrical circuit. The only difference is, t
O
The velocity v of the body as a function
(i) Potential difference (DV ) is replaced by the pressure
of time is shown in Fig. (b). Fig (b)
difference (Dp)
L
(ii) The electrical resistance, R æç = r ö÷ is replaced by Sample Problem 25 Water is flowing through a
è Aø horizontal tube 8 cm in diameter and 4 km in length at the rate
8 hL
X æç = ö and
÷ of 20 litre/s. Assuming only viscous resistance. The pressure
è pR4 ø required to maintain the flow in terms of mercury column.
(iii) The electrical current i is replaced by volume flow rate (Coefficient of viscosity of water is 0.001 Pa-s) is
dV (a) 69.68 cm
Q or .
dt (b) 59.68 cm
(c) 49.68 cm
(d) 39.68 cm
12.14 Stokes’ Law
Interpret (b) Here, 2 r = 8 cm = 0.08 m
When a spherical body moves through a fluid, the fluid in
contact with the body is dragged with it. The fluid exerts a or r = 0.04 m; l = 4 km = 4000 m;
viscous force on the body to oppose its motion. V = 20 litre/s = 20 ´ 10 -3 m3 s–1,
The formula for the viscous force on a sphere moving h = 0.001Pa -s, p = ?
through a fluid was first derived by the English physicist p pr 4
G. Stokes in 1843. According to him, a spherical body of As, V=
8 hl
radius r moving with velocity v experiences a viscous
8 Vhl
force given by or p=
pr 4
F = 6 phrv (h = coefficient of viscosity)
8 ´ (20 ´ 10 -3) ´ 0.001 ´ 4000
This law is called Stokes’ law. =
æ 22 ö 4
ç ÷ ´ (0.04)
è7ø
Terminal Velocity
= 7.954 ´ 10 4 Pa
Let a body (density r) of radius r is falling freely in a
medium (liquid or gas) of density s. Initially, v = 0 and r > s , \Height of mercury column for pressure difference p will be,
so the body will be accelerated downwards. Because of p
h=
the acceleration, the velocity will increase and hence, rg
viscous force will increase. At a certain instant, when
7.954 ´ 10 4
viscous force F will balance the net downward force =
(weight – upthrust), acceleration will become zero and the (13.6 ´ 10 3) ´ 9.8
= 0.5968 m = 59.68 cm
Telegram @unacademyplusdiscounts

452 JEE Main Physics

Sample Problem 26 The terminal velocity of a copper (a) 0.1 ms–1


ball of radius 2 mm falling through a tank of oil at 20°C is (b) 0.2 ms–1
6.5 cms -1. The viscosity of the oil at 20°C is [Given density of (c) 0.3 ms–1
oil is 1.5 ´ 103 kgm -3, density of copper is 8.9 ´ 103 kgm -3]. (d) 0.4 ms–1
(a) 3.3 ´ 10 –1 kgm-1s-1 Interpret (d) Let r be the radius of each of the small rain drop
(b) 6.3 ´ 10 –2 kgm-1s-1 and R be the radius of big rain drop formed.
–3 -1 -1
(c) 9.2 ´ 10 kgm s As, volume of big drop = 8 ´ volume of each small drop
(d) 9.9 ´ 10 –1 kgm-1s-1 4 3 4
\ p R = 8 ´ p r3
3 3
Interpret (d) Given, v t = 6.5 ´ 10 –2 ms–1, a = 2 ´ 10 -3 m,
R = 2r
g = 9.8 ms–2,r = 8.9 ´ 10 3 kgm–3
Let terminal velocity of small drop be v1 and of big drop be v 2.
s = 1.5 ´ 10 3 kgm–3
As terminal velocity,
2 a2 (r - s ) g
\ h= 2 r 2 (r - s ) g
9 vt v=
9h
2 (2 ´ 10 –3) ´ 9.8
h= ´ ´ 7.4 ´ 10 3 kgm–3 or v µ r2
9 6.5 ´ 10 –2
h = 9.9 ´ 10 –1 kgm–1s–1 v 2 R2
\ =
v1 r 2
2
Sample Problem 27 Eight spherical rain drops of equal R2 æ2r ö
or v 2 = v1 = 0.2 ç ÷
size are falling vertically through air with a terminal velocity of r2 è r ø
0.10 ms–1. What should be the velocity, if these drops were to = 0.1 ´ 4 = 0.4 ms–1
combine to form one large spherical drop?
Telegram @unacademyplusdiscounts

WORKED OUT
Examples
Example 1 A 50 kg girl wearing high heel shoes balances Solution Here, density of ice,
on a single heel. The heel is circular with a diameter 1 cm. What r = 917 kg m-3;
is the pressure exerted by the heel on the horizontal floor?
Density of fresh water, r = 1000 kg m-3
(a) 5.2 ´ 10 6 Nm-2 (b) 4 ´ 10 6 Nm-2
(c) 6.24 ´ 10 6 Nm-2 (d) 5.24 ´ 10 6 Nm-2 Let V be the total volume of the ice and v be the volume of ice
above the water. Then volume of the water displaced by the
Solution Here, m = 50 kg; immersed part of ice = (V - v)
1 According to law of floatation,
r = D / 2 = 1 / 2 cm = m
200 Weight of ice = Weight of the water displaced
Force mg V ´ 917 ´ g = (V - v) ´ 1000 ´ g
Pressure = =
Area pr 2 or 1000 v = 1000 V - 917 V = 83 V
50 ´ 9.8 v 83
= or = = 0.083
(22 / 7) ´ (1 / 200) 2 V 1000
= 6.24 ´ 10 6 Nm-2
Example 4 A solid floats with1/ 4 th of its volume above the
Example 2 A hydraulic automobile lift is designed to lift surface of water, the density of the solid is
cars with a maximum mass of 3000 kg. The area of (a) 750 kg m-3 (b) 650 kg m-3
cross-section of the piston carrying the board is 425 cm 2. What (c) 560 kg m-3 (d) 450 kg m-3
maximum pressure would smaller piston have to bear?
(a) 5.2 ´ 10 4 Pa (b) 4.2 ´ 10 5 Pa
Solution Let V and r be volume and density of solid
respectively and r¢ be the density of water i.e., p¢ = 10 3 kg m-3
(c) 6.92 ´ 10 4 Pa (d) 5.92 ´ 10 5 Pa
Weight of body = Vrg
Solution The maximum force, which the bigger piston can Volume of solid body outside water = V / 4
bear, \Volume of solid body inside water
F = 30000 kgf = 3000 ´ 9.8 N = V - V / 4 = 3V / 4
Area of piston ,
Weight of water displaced by solid
A = 425 cm2 = 425 ´ 10 -4 m2 3V
= ´ 10 3 ´ g
\Maximum pressure on the bigger piston, 4
F 300 ´ 9.8 As solid body is floating, then
p= =
A 425 ´ 10 -4 Weight of body = Weight of water displaced by it
3V
= 6.92 ´ 10 4 Pa Vrg = ´ 10 3 g
4
Since, the liquid transmits pressare equally,therefore the 3
maximum pressure the shaller piston can bear is 6.92 ´ 10 4 Pa. r = ´ 1000 = 750 kg m-3
4

Example 3 The density of ice is 917 kg m -3. What fraction Example 5 A wire ring of 30.0 mm radius resting flat on the
of the volume of a piece of ice will be above water, when surface of the liquid is raised. The pull required is 3.03 gf force,
floating in fresh water? before the film breaks. The surface tension of the liguid is
(a) 0.083 (b) 0.053 (a) 71.76 dyne cm-1 (b) 78.76 dyne cm-1
(c) 0.045 (d) 0.043 (c) 75.58 dyne cm -1
(d) 70 dyne cm-1
Telegram @unacademyplusdiscounts

454 JEE Main Physics

Solution Here, r = 30.0 mm = 3 cm; F = 3.00 gf = 3.03 ´ 980 Incase of a soap bubble,
4S
dyne. Since, the liquid is touching the ring, both inside as well as p=
outside therefore, force acting on the ring due to surface tension is r
given by pr 2 ´ 0.8 ´ 9.8 ´ 10 -2
or S= =
F" = 2 ( S ´ circumference of ring) 4 200
= 2( S ´ 2pr) = 4Spr = 3.92 ´ 10 -2 Nm-1
22
=4´S´ ´ 3 dyne
7 Example 8 A square plate of 10 cm side moves parallel to
As, F¢ = F another plate with a velocity of 10 cm s-1; both plates immersed
22 in water. If the viscous force is 200 dyne and viscosity of water
=4´S´ ´ 3 = 3.03 ´ 980 is 0.01 poise, what is their separation distance?
7
3.03 ´ 980 ´ 7 (a) 0.05 cm (b) 1 cm
S=
4 ´ 22 ´ 3 (c) 0.07 cm (d) 7 cm
= 78.76 dyne cm-1 Solution Here, side of the square plate,
I = 10 cm
Example 6 The work done in blowing a soap bubble of
surface tension 0.06 Nm -1 from 2 cm radius to 5 cm radius is Area of the plate, A = L2 = 10 2 = 100 cm2
(a) 0.004168 J (b) 0.003168 J dv = 10 cms-1: F = 200 dyne:
(c) 0.003158 J (d) 0.004568 J h = 0.01poise, dx = ?
Solution Here, S = 0.06 Nm ; -1 dv
As F = hA
dx
r1 = 2 cm = 0.02 m; r2 = 5 cm = 0.05 m
hAdv 0.01 ´ 100 ´ 10
Since, bubble has two surfaces, initial surface area of the bubble dx = = = 0.05 cm
F 200
= 2 ´ 4pr22 = 2 ´ 4p (0.02) 2
= 32 p ´ 10 -4m2 Example 9 A rain drop of radius 0.3 mm has a terminal
velocity in air 1 ms-1. The viscosity of air is 18 ´ 10 -5 poise.
Final surface area of the bubble
Find the viscous force on the rain drops.
= 2 ´ 4pr22 = 2 ´ 4p (0.05) 2
(a) 2.05 ´ 10 -7 N
-4 2
= 200 p ´ 10 m (b) 1.018 ´ 10 -7 N
Increase in surface area (c) 1.05 ´ 10 -7 N
-4 -4
= 200 p ´ 10 - 32 p ´ 10 (d) 2.058 ´ 10 -7 N
-4 2
= 168 p ´ 10 m
Solution Here, r = 0.3 mm = 0.3 ´ 10 -3 m; v = 1ms-1
\Work done = S × Increase in surface area
= 0.06 ´ 168p ´ 10 -4 = 0.003168 J h = 18 ´ 10 -5 poise = 18 ´ 10 -6 decapoise
viscous force, F = 6 p h rv
Example 7 If excess of pressure inside a soap bubble of 22
radius 1 cm is balanced by that due to a column of oil =6 ´ ´ (18 ´ 10 -6) ´ (0.3 ´ 10 -3) ´ 1
7
(specific gravity 0.8) 2 mm high, the surface tension of soap
bubble is = 1.018 ´ 10 -7 N
(a) 2.92 ´ 10 -2 Nm-1 (b) 4.92 ´ 10 -2 Nm-1
Example 10 What is the largest average velocity of blood
(c) 5.92 ´ 10 -2 Nm-1 (d) 3.92 ´ 10 -2 Nm-1
flow in ar rtery of radius 2 ´ 10 -3, if the flow must remain
Solution Here. r = 1 cm = 10 -2 m; laminar? What is the corresponding flow rate? [Take viscosity
of blood to be 2.084 ´ 10 -3 pa-s: Density of blood is
density of oil,
1.06 ´ 103 kgm -3 ]
r = 0.8 ´ 10 3 kg m-3
(a) 9.8 ms-1, 2.5 ´ 10 -5m3 s-1
h = 2mm = 2 ´ 10 -3m
(b) 9.8 ms-1, 3.5 ´ 10 -5m3 s-1
Pressure due to 2 mm column of oil,
(c) 9.8 ms-1, 1.23 ´ 10 -4m3 s-1
p = hrg = (2 ´ 10 -3) (0.8 ´ 10 3) ´ 9.8
(d) 0.98 ms-1, 1.23 ´ 10 -5m3 s-1
= 2 ´ 0.8 ´ 9.8 Pa
Telegram @unacademyplusdiscounts

Properties of Liquids 455

Solution Here, Area of cross-section,


-3 -3 -3 22
r = 2 ´ 10 m, D = 2r = 2 ´ 2 ´ 10 = 4 ´ 10 m; r = pr 2 = ´ (0.02) 2 m2
7
h = 2.084 ´ 10 -3 Pa -s; Let v be the velocity of the flow of water at the given point.
r = 1.06 ´ 10 3 kg m-3 Clearly, V = Av
1 22
For flow to be laminar, NR = 2000 or ´ 10 -3 = ´ (0.02) 2 ´ v
3 7
NRh 2.000 ´ (2.084 ´ 10 -3) 7 ´ 10 -3
Now n= = = 0.98 ms-1 or v = 0.2639 ms-1
rD (1.06 ´ 10 3) ´ (4 ´ 10 -3) 3 ´ 22 ´ (0.02) 2
Volume flowing second = pr 2v c
22 Example 12 At what speed will the velocity of a stream of
= ´ (2 ´ 10 -3) 2 ´ 0.98
7 water be equal to 20 cm of mercury column?
= 1.23 ´ 10 -5 m3 s-1 Taking g = 10 ms-2
(a) 6.4 ms-1 (b) 7.3756 ms-1
Example 11 Water flows through a horizontal pipe of (c) 6.4756 ms-1 (d) None of these
variable cross-section at the rate of 20 I, per min. What will be
the velocity of water at a point where diameter is 4 cm? Solution Here, velocity head = 20 cm of Hg
(a) 0.2639 ms-1 (b) 0.5639 ms-1 = 20 ´ 13.6 cm of water.
(c) 0.4639 ms-1 (d) 0.3639 ms-1 v2
As velocity head =
Solution Volume of the water flowing per second, 2g
20 ´ 1000 3 -1 v2
V = 20 L min -1 = ms \ 20 ´ 13.6 =
60 ´ (100)3 2 ´ 1000
1
= ´ 10 -3m3 s-1 v = 20 ´ 13.6 ´ 2 ´ 1000
3
Radius of the pipe, = 737.56 cms-1
4 = 7.3756 ms-1
r= = 2 cm = 0.02 m
2
Telegram @unacademyplusdiscounts

Start Practice for


JEE Main
Round I (Topically Divided Problems)

Thrust and Pressure 6. A U-tube contains water and methylated spirit


separated by mercury. If the 15.0 cm of water and
1. Density of ice is r and that of water is s. What will be
spirit each are further poured into the respective
the decrease in volume when a mass M of ice melts?
arms of the tube, what is the difference in the levels
M s -r
(a) (b) of mercury in the two arms? (Specific gravity of
s -r M mercury = 13.6)
æ1 1ö 1 æ1 1ö (a) 0.221 cm (b) 2.22 cm
(c) M ç - ÷ (d) ç - ÷
èr s ø M èr s ø (c) 0.02 cm (d) None of these
2. A 50 kg girl wearing high heel shoes balances on a 7. A cylindrical vessel is filled with equal amounts of
single heel. If the heel is circular with a diameter weight of mercury on water. The overall height of the
1.0 cm. What is the pressure exerted on the two layers is 29.2 cm, specific gravity of mercury is
horizontal floor? 13.6. Then the pressure of the liquid at the bottom of
(a) 6.9 ´ 106 Pa (b) 6.2 ´ 106 Pa the vessel is
(a) 29.2 cm of water (b) 29.2/13.6 cm of mercury
(c) 9.6 ´ 106 Pa (d) 9.0 ´ 106 Pa
(c) 4 cm of mercury (d) 15.6 cm of mercury
3. The surface area of air bubble increases four times 8. The density r of water of bulk modulus B at a depth y
when it rises from bottom to top of a water tank in the ocean is related to the density at surface r 0 by
where the temperature is uniform. If the atmospheric the realation
pressure is 10 m of water, the depth of the water in æ r 0 gy ö æ r gy ö
the tank is (a) r = r 0 ç1 - ÷ (b) r = çr 01 + 0 ÷
è B ø è B ø
(a) 30 m (b) 40 m (c) 70 m (d) 80 m
æ B ö æ B ö
(c) r = r 0 ç1 + ÷ (d) r = r 0 ç1 - ÷
4. A U-tube contains water and methylated spirit è r 0 hgy ø è r 0 gy ø
separated by mercury. The mercury columns in the
two arms are in level with 10.0 cm of water in one 9. An aquarium tank is in the shape of a cube with one
arm and 12.5 cm of spirit in the other. The specific side a 4m tall glass wall. When the tank is half filled
gravity of spirit would be. and the water is 2 m deep, the water exerts a force F
on the wall. What force does the water exerts on the
(a) 0.70 (b) 0.80 (c) 0.90 (d) 0.60
wall when the tank is full and the water is 4 m drop?
5. A uniform tapering vessel shown in figure is filled with (a) 1/2 F (b) F (c) 2 F (d) 4 F
liquid of density 900 kgm–3. The force that acts on the
10. Figure shows the
base of the vessel due to liquid is (Take, g = 10 ms -2 ) vertical cross-section of
–3 2
P
Area = 10 m a vessel filled with a
liquid of density r. The θ H
normal thrust per unit O Q h
0.4 m
area on the walls of the
vessel at point P, as
–3 2
Area = 2 × 10 m shown will be
(a) hrg (b) Hrg
(a) 3.6 N (b) 7.2 N (c) 9.0 N (d) 12.0 N (c) ( H - h) rg (d) ( H - h) rg cos q
Telegram @unacademyplusdiscounts

Properties of Liquids 457

Relative Density of Substance, 18. The spring balance A reads 2 kg with a block of mass
m suspended from it. A balance B reads 5 kg when a
Archimedes’ Principle and Laws of beaker with liquid is put on the pan of the balance.
Floatation The two balances are now so arranged that the
11. A beaker containing water is balanced on the pan of a hanging mass is inside the liquid in a beaker as
common balance. A solid of specific gravity 1 and mass shown in figure.
5 g is tied to the arm of the balance and immersed in
water contained in the beaker. The scale pan with the
beaker A
(a) goes down (b) goes up
(c) remains unchanged (d) None of these
12. Torricelli’s barometer used mercury. Pascal M
duplicated it using French wine of density 984 kg/m3.
Determine the height of the wine column for normal
atmospheric pressure.
(a) 9.5 cm (b) 5.5 cm (c) 10.5 cm (d) 11.5 cm B

13. An ice block floats in a liquid whose density is less (a) The balance A will read more than 2 kg
than water. A part of block is outside the liquid. (b) The balance B will read less than 5 kg
When whole of ice has melted, the liquid level will (c) The balance A will read less than 2 kg and B will read
(a) rise (b) go down more than 5 kg
(c) remain same (d) first rise then go down (d) The balance A will read more than 2 kg and B will read
14. A tank 5m high is half filled with water and then is less than 5 kg
filled to the top with oil of density 0.85 gcm–3. The 19. A cylinder of mass m and density r hanging from a
pressure at the bottom of the tank, due to these string is lowered into a vessel of cross-sectional area A
liquids is containing a liquid of density s (< r) until it is fully
(a) 1.85 g dyne cm -3 (b) 89.25 g dyne cm -3 immersed. The increase in pressure at the bottom of
(c) 462.5 g dyne cm -3 (d) 500 dyne cm -3 the vessel is
mg mgr msg
15. A balloon of volume 1500 m3 and weighing 1650 kg (a) Zero (b) (c) (d)
with all its equipment is filled with He (density A sA rA
0.2 kg m–3). If the density of air be 1.3 kgm–3, the pull 20. A cubical block of wooden edge l and a density r floats
on the rope tied to the balloon will be in water of density 2 r. The lower surface of cube just
(a) 300 kg (b) 1950 kg (c) 1650 kg (d) zero touches the free end of a massless spring of force
16. A cubic block is floating in a liquid with half of its constant k fixed at the bottom of the vessel. The
volume immersed in the liquid. When the whole weight w put over the block so that it is completely
system accelerates upwards with acceleration of g/3 immersed in water without wetting the weight is
2
r+ (a)
( ) the r + ((b)
) ofavolume
a l g kfraction lgk immersed in the liquid will be
æ lr g ö æ kö
(c) a ç + 2k ÷ (d) l ç l2rg + ÷
g/3 è 2 ø è 2ø
21. A rectangular plate 2m × 3m is immersed in water in
such a way that its greatest and least depth are 6 m
and 4 m respectively, from the water surface. The
1 3 2 3
(a) (b) (c) (d) total thrust on the plate is
2 8 3 4
(a) 294 ×103 N (b) 294 N
17. Two cubes each weighing 22 g exactly are taken. One (c) 100 ×103 N (d) 400 ×103 N
is of iron ( d = 8 ´ 103 kgm -3) and the other is of 22. A block of aluminium of mass 1 kg and volume
marble ( D = 3 ´ 103 kgm -3). They are immersed in 3.6 ´ 10-4 m 3 is suspended from a string and then
alcohol and then weighed again completely immersed in a container of water. The
(a) iron cube weighs less (b) iron cube weighs more decrese in tension in a container of water. The
(c) both have equal weight (d) nothing can be said decrease in tension in the string after immersion is
(a) 9.8 N (b) 6.2 N (c) 3.6 N (d) 1.0 N
Telegram @unacademyplusdiscounts

458 JEE Main Physics

23. A vessel with water is placed on a weighing pan and 29. A hollow cylinder of mass m made heavy at its bottom
it reads 0.8 gcc–1 is sunk into the water with a pin of is floating vertically in water. It is tilled from its
negligible volume as shown in figure keeping it sunk. vertical position through an angle q and is left. The
The weighing pan will show a reading restoring force acting on it is
Pin (a) mg cos q (b) mg sin q
é 1 ù é 1 ù
(c) mg ê - 1ú (d) mg ê +1
ë cos q û ë cos q úû
30. A hemispherical bowl just floats without sinking in a
liquid of density 1.2 ´ 103 kgm –3. If outer diameter
and the density of the bowl are 1 m and 2 ´ 104 kgm –3
respectively, then the inner diameter of the bowl will
Weighing pan be
(a) 0.94 m (b) 0.96 m
(a) 600 g (b) 632 g (c) 0.98 m (d) 0.99 m
(c) 642 g (d) 640 g
24. A body of density r is dropped from rest at a height h Surface Tension and Surface Energy
into a lake of density s, where s > r. Neglecting all
dissipative forces, calculate the maximum depth to 31. A thin metal disc of radius r float on water surface
which the body sinks before returning to float on the and bends the surface downwards along the
surface. perimeter making an angle q with vertical edge of the
h hr disc. If the disc displaces a weight of water w and
(a) (b) surface tension of water is T, then the weight of
s -r s
hr hs metal disc is
(c) (d) (a) 2 prT + w (b) 2 prT cos q - w
s -r s -r
(c) 2 prT cos q + w (d) w - 2 prT cos q
25. Two cylinders of same cross-section and length L but
made of two material of densities r1 and r2 (in CGS
32. A ring is cut from a platinum tube 8.5 cm internal
diameter and 8.7 cm external diameter. It is
units) are cemented together to form a cylinder of
supported horizontally from a pan of a balance so,
length 2 L. If the combination floats in water with a
that it comes in contact with the water is in glass
length L/2 above the surface of water and r1 < r2 , then
vessel. If an extra 3.47 g-wt is required to pull it away
(a) r1 > 1 (b) r1 < 3 / 4
from water, surface tension of water is
(c) r1 > 1 / 2 (d) r1 > 3 / 4
(a) 72.07 dyne cm–1 (b) 70.80 dyne cm–1
26. The density of ice is 0.9 gcc–1 and that of sea water is (c) 65.35 dyne cm–1 (d) 60.00 dyne cm–1
1.1 gcc–1. An ice berg of volume V is floating in sea
33. What is the pressure inside the drop of mercury of
water. The fraction of ice berg above water level is
radius 3.00 mm at room temperature? Surface
(a) 1/11 (b) 2/11
tension of mercury at that temperature (20°C) is
(c) 3/11 (d) 4/11
4.65 ´ 10-1N/m. The atmospheric pressure is
27. A solid of density D is floating in a liquid of density d. . ´ 105 Pa. Also give the excess pressure inside the
101
If v is the volume of solid submerged in the liquid and drop.
V is the total volume of the solid, then v / V is equal to (a) 1.01 ´ 105 Pa, 320 Pa (b) 1.01 ´ 105 Pa, 310 Pa
d D 5
(a) (b) (c) 310 Pa, 1.01 ´ 10 Pa (d) 320 Pa, 1.01 ´ 105 Pa
P d
D D+d 34. What is the radius of the biggest aluminium coin of
(c) (d)
(D + d) D thickness, t and density r, which will still be able to
float on the water surface of surface tension S?
28. The total weight of a piece of wood is 6 kg. In the 4S 3S 2S S
floating state in water its 1 part remains inside the (a)
3rgt
(b)
4rgt
(c)
rgt
(d)
rgt
3
water. On this floating solid, what maximum weight is 35. 8000 identical water drops are combined to form a big
to be put such that the whole of the piece of wood is to be drop then the ratio to the final surface energy to the
drowned in the water? initial surface energy, if all the drops together is
(a) 12 kg (b) 10 kg (a) 1 : 10 (b) 1 : 15
(c) 14 kg (d) 15 kg (c) 1 : 20 (d) 1 : 25
Telegram @unacademyplusdiscounts

Properties of Liquids 459

36. A frame made of a metallic wire enclosing a surface 44. What change in surface energy will be noticed when a
area A is covered with a soap film. If the area of the drop of radius R splits up into 1000 droplets of radius
frame of metallic wire is reduced by 50%, the energy r, surface tension T ?
of the soap film will be changed by (a) 4 pR2T (b) 7 pR2T
(a) 100 % (b) 75 % (c) 16 pR T 2
(d) 36 pR2T
(c) 50 % (d) 25 %
45. Let, W be the work done, when a bubble of volume V
37. A mercury drop of radius 1 cm is broken into 106 is formed from a given solution. How much work is
droplets of equal size. The work done is
required to be done to form a bubble of volume 2 V ?
( S = 35 ´ 10-2 Nm –1)
(a) W (b) 2W
(a) 4.35 ´ 10–2 J (b) 4.35 ´ 10–3 J
(c) 21/3 W (d) 41/3 W
(c) 4.35 ´ 10–6 J (d) 4.35 ´ 10–8 J
46. What is the ratio of surface energy of 1 small drop and
38. Surface tension of a soap solution is able of 2.0 cm 1 large drop if 1000 drops combined to form 1 large
diameter will be
drop?
(a) 7.6 ´ 10–6 pJ (b) 15.2 ´10 -6 pJ
(a) 100 : 1 (b) 1000 : 1
(c) 1.9 ´ 10–6 pJ (d) 1 ´ 10–4 pJ
(c) 10 : 1 (d) 1 : 100
39. A drop of water breaks into two droplets of equal size. 47. A bigger drop of radius R is converted into n smaller
In this process, which of the following statements is
drops of radius r, the required energy is
correct?
(a) ( 4 pr2 n - 4 pR2 ) T
(a) The sum of the temperatures of the two droplets together
is equal to temperature of the original drop æ4 4 ö
(b) ç pr3n - pR3 ÷ T
è3 3 ø
(b) The sum of the masses of the two droplets is equal to
mass of drop (c) ( 4 pR2 - 4 pr2 ) nT
(c) The sum of the radii of the two droplets is equal to the (d) ( n 4 pr2 - 4 pR2 ) T
radius of the drop
(d) The sum of the surface areas of the two droplets is equal
to the surface area of the original drop Excess of Pressure, Shape of Meniscus
40. Work done in splotting a drop of water of 1mm radius and Capillarity
into 106 droplets is (surface tension of water 48. The angle of contact at the interface of water-glass is
72 ´ 10-3 J / m 2 ) 0° Ethylalcohol-glass is 0°, Mercury-glass is 140° and
(a) 9.8 ´ 10 -5 J . ´ 10 -5 J
(b) 895 Methyliodide-glass is 30°. A glass capillary is put in a
(c) 5.89 ´ 10 -5 J (d) 5.98 ´ 10 -6 J trough containing one of these four liquids. It is
observed that the meniscus is convex. The liquid in
41. A drop of liquid of diameter 2.8 mm breaks up into
the trough is [NCERT Exemplar]
125 identical drops. The change in energy is nearly
(a) water (b) ethylalcohol
(S = 75 dyne cm–1)
(c) mercury (d) methyliodide
(a) zero (b) 19 erg
(c) 46 erg (d) 74 erg 49. The diagram shows three soap bubbles A, B and C
prepared by blowing the capillary tube fitted with
42. The surface energy of a liquid drop is u. It is sprayed
stop cocks S, S1, S2 and S3. With stop cock S closed
into 1000 equal droplets. Then its surface energy
and stop cocks S1, S2 and S3 opened
becomes
(a) u (b) 10 u
C
(c) 100 u (d) 1000 u
43. A water film is made between two straight parallel
S3 S S
wires of length 10 cm separated by 5 mm from each S1 2

other. If the distance between the wires is increased A B


by 2 mm. How much work will be done? Surface
tension for water is 72 dyne cm–1. (a) B will start collapsing with volumes of A and C increasing
(a) 288 erg (b) C will start collapsing with volume of A and B increasing
(b) 72 erg (c) volume of A, B and C will become equal in equilibrium
(c) 144 erg (d) C and A will both start collapsing with volume of B
(d) 216 erg increasing
Telegram @unacademyplusdiscounts

460 JEE Main Physics

50. The amount of work done in blowing a soap


bubble such that its diameter increases from d to D (a) (b)
is
(S = surface tension of solution)
(a) p( D2 - d2 ) S (b) 2 p ( D2 - d2 ) S
2 2
(c) 4 p ( D - d ) S (d) 8 p ( D2 - d2 ) S (c) (d)
2
51. If pressure at half the depth of a lake is equal to
3
pressure at the bottom of the lake then what is the 58. By inserting a capillary tube upto a depth l in water,
depth of the lake? the water rises to a height h. If the lower end of the
(a) 10 m (b) 20 m capillary tube is closed inside water and the capillary
(c) 60 m (d) 30 m is taken out and closed end opened, to what height
52. In a test experiment on a model aeroplane in a wind the water will remain in the tube, when l > h?
tunnel, the flow speeds on the upper and lower (a) zero (b) l + h (c) 2 h (d) h
surfaces of the wing are 70 m/s and 63 m/s 59. Two capillary tubes of radii 0.2 cm and 0.4 cm are
respectively. What is the lift on the wing, if its area is dipped in the same liquid. The ratio of heights
2.5 m2 ? Take the density of air to be 1.3 kg/m 3. through which liquid will rise in the tubes is
(a) 5.1 ´ 102 N (b) 6.1 ´ 102 N (a) 1 : 2 (b) 2 : 1
3 3 (c) 1 : 4 (d) 4 : 1
(c) 1.6 ´ 10 N (d) 1.5 ´ 10 N
60. Water rises in a capillary tube to a height h. Choose
53. With the increase in temperature, the angle of the false statement regarding rise from the following.
contact (a) On the surface of Jupitor, height will be less than h.
(a) decreases (b) In a lift, moving up with constant acceleration, height is
(b) increases less than h.
(c) remains constant (c) On the surface of the moon, the height is more than h.
(d) sometimes increases and sometimes decreases (d) In a lift moving down with constant acceleration, height
54. Water rises to a height of 10 cm in a capillary tube is less than h.
and mercury falls to a depth of 3.42 cm in the same
capillary tube. If the density of mercury and water Dependence of Surface Tension
are 135° and 0° respectively, the ratio of surface tension 61. The surface tension of a liquid at its boiling point
of water and mercury is (a) becomes zero
(a) 1 : 0.15 (b) 1 : 3 (b) becomes infinity
(c) 1 : 6.5 (d) 1.5 : 1 (c) is equal to the value at room temperature
55. Water rises to a height of 16.3 cm in a capillary of (d) is half to the value at the room temperature
height 18 cm above the water level. If the tube is cut 62. When a pinch of salt or any other salt which is soluble
at a height of 12 cm in the capillary tube, in water is added to water, its surface tension
(a) water will come as a fountain from the capillary tube (a) increases
(b) water will stay at a height of 12 cm in the capillary tube (b) decreases
(c) the height of water in the capillary tube will be 10.3 cm (c) may increase or decrease depending upon salt
(d) None of the above
(d) water height flow down the sides of the capillary tube
56. Water rises in a capillary tube to a height h. It will 63. At which of the following temperatures, the value of
rise to a height more than h surface tension of water is minimum?
(a) 4°C (b) 25°C (c) 50°C (d) 75°C
(a) on the surface of sun
(b) in a lift moving down with an acceleration 64. Two spherical soap bubbles of radii a and b in
(c) at the poles vacuum coalesce under isothermal conditions. The
(d) in a lift moving up with an acceleration resulting bubble has a radius given by
( a + b) ab
57. If a liquid is placed in a vertical cylindrical vessel and (a) (b)
2 a+b
the vessel is rotated about its axis, the liquid will
(c) a2 + b2 (d) a + b
take the shape of figure.
Telegram @unacademyplusdiscounts

Properties of Liquids 461

65. When two soap bubbles of radius r1 and r2 ( r2 > r1) 71. Water in a vessel of uniform cross-section escapes
coalesce, the radius of curvature of common surface is through a narrow tube at the base of the vessel.
(a) ( r2 - r1 ) (b) ( r2 + r1 ) Which graph given below represents the variation of
r -r r r the height h of the liquid with time t?
(c) 2 1 (d) 2 1
r1 r2 r2 - r1 h h
66. Which graph represent the variation of surface
tension with temperature over small temperature (a) (b)
ranges for water?
t t
Surface tension

Surface tension
h h

(a) (b) (c) (d)

t t
Temperature Temperature
72. 16 cm3 of water flows per sec through a capillary tube
Surface tension

of radius a cm and of length l cm when connected to a


Surface tension

pressure head of h cm of water. If a tube of the same


(c) (d) length and radius a/2 cm is connected to the same
pressure head, the quantity of water flowing through
the tube per second will be
Temperature Temperature
(a) 16 cm3 (b) 1 cm3
67. A capillary tube of radius R and length L is connected (c) 4 cm3 (d) 8 cm3
in series with another tube of radius R/2 and length 73. Under a pressure head, the rate of orderly volume
L/4. If the pressure difference across the two tubes flow of liquid through a capillary tube is Q. If the
taken together is p, then the ratio of pressure length of capillary tube were doubled and the
difference across the first tube to that across the diameter of the bore is halved, the rate of flow would
second tube is become
(a) 1 : 4 (b) 1 : 1 (a)
Q
(b) 16 Q
(c) 4 : 1 (d) 2 : 1 4
Q Q
68. The relative velocity of two parallel layers of water is (c) (d)
8 32
8 cms–1. If the perpendicular distance between the
layers is 0.1 cm, then velocity gradient will be
(a) 40 s–1 (b) 50 s–1 Stokes’ Law, Terminal Velocity
(c) 60 s–1 (d) 80 s–1 and Variation of Viscosity
69. Two water pipes P and Q having diameter 2 ´ 10-2 m 74. The terminal velocity v of a spherical ball of lead of
and 4 ´ 10-2 m respectively are joined in series with radius R falling through a viscous liquid varies with
the main supply line of water. The velocity of water R such that
flowing in pipe P is v
(a) = constant (b) vR = constant
(a) 4 times that of Q R
(b) 2 times that of Q v
(c) v = constant (d) 2 = constant
(c) 1/2 times that of Q R
(d) 1/4 times that of Q 75. The rate of steady volume flow of water through a
70. The rate of flow of liquid through a capillary tube of capillary tube of length l and radius r under a
radius r is V, when the pressure difference across pressure difference of p, is V. This tube is connected
the two ends of the capillary is p. If pressure is with another tube of the same length but half the
increased by 3 p and radius is reduced to r/2, then radius in series. Then the rate of steady volume flow
the rate of flow becomes through them is (The pressure difference across the
(a) V/9 (b) 3V/8 combination is p)
(c) V/4 (d) V/3 V V 16 V 17 V
(a) (b) (c) (d)
16 17 17 16
Telegram @unacademyplusdiscounts

462 JEE Main Physics

76. A small spherical ball of steel falls through a viscous 82. A marble of mass x and diameter 2 r is gently
medium with terminal velocity v. If a ball of twice the released a tall cylinder containing honey. If the
radius of the first one but of the same mass is marble displaces mass y ( < x) of the liquid, then the
dropped through the same method, it will fall with a terminal velocity is proportional to
terminal velocity (neglect buoyancy) (a) (x + y) (b) (x –y)
v v x+ y (x - y )
(a) (b) (c) (d)
2 2 r r
(c) v (d) 2 v 83. A small iron sphere is dropped from a great height. It
77. A tall cylinder is filled with viscous oil. A round attains its terminal velocity after having fallen 32 m.
pebble is dropped from the top with zero initial Then, it covers the rest of the path with terminal
velocity. From the plot shown in figure, indicate the velocity only. The work done by air friction during
one that represents the velocity (v) of the pebble as a the first 32 m of fall is W1. The work done by air
function of time (t). [NCERT Exemplar] friction during the subsequent 32 m fall is W2 . Then
(a) W1 > W2 (b) W1 < W2
v v (c) W1 = W2 (d) W2 = 32 W1
(a) (b) 84. A spherical ball is dropped in a long column of viscous
liquid. Which of the following graphs represent the
variation of
t t
(i) gravitational force with time
v v
(ii) viscous force with time
(c) (d) (iii) net force acting on the ball with time?
F
t t P
Q
78. A rain drop of radius 0.3 mm has a terminal velocity
in air = 1 ms–1. The viscous force on it is
(a) 101.73 ´ 10–4 dyne (b) 101.73 ´ 10–5 dyne R
(c) 16.95 ´ 10–4 dyne (d) 16.95 ´ 10–5 dyne t

79. A metallic sphere of mass M falls through glycerine (a) Q, R, P (b) R, Q, P


with a terminal velcity v. If we drop a ball of mass 8 M (c) P, Q, R (d) R, P, Q
of same metal into a column of glycerine, the
85. A large tank is filled with water to a height H. A
terminal velocity of the ball will be
small hole is made at the base of the tank if takes T1
(a) 2 v (b) 4 v H
(c) 8 v (d) 16 v
time to decrease the height of water to (h > 1) and if
l
80. A rain drop of radius 1.5 mm, experiences a drag takes T2 time to take out the rest of water if T1 = T2
force F = (2 ´ 10–5 v) N, while falling through air from then the value of h is
a height 2 km, with a velocity v. The terminal velocity (a) 2 (b) 3
of the rain drop will be nearly (use g = 10 ms–2) (c) 4 (d) 2 2
(a) 200 ms–1 (b) 80 ms–1
(c) 7 ms–1 (d) 3 ms–1 Liquid Flow
81. Which of the following diagrams (figure) does not 86. An incompressible liquid flows through a horizontal
represent a streamline flow? [NCERT Exemplar]
tube as shown in the figure. Then, the velocity v of the
fluid is

(a) (b) A v2 = 1.5 m/s

v1 = 3 m/s A
1.5
A
v
(c) (d)
(a) 3 m/s (b) 1.5 m/s
(c) 1.0 m/s (d) 2.25 m/s
Telegram @unacademyplusdiscounts

Properties of Liquids 463

87. Water flowing out of the mouth of a tap and falling 92. Three tubes A, B and C are connected to a horizontal
vertically in streamline flow forms a tapering pipe in which liquid is flowing. The radii of pipe at
column, i.e., the area of cross-section of the liquid the joints of A, B and C are 2 cm, 1 cm and 2 cm
column decreases as it moves down. Which of the respectively. The height of liquid
following is the most accurate explanation for this?
A C
B

(a) in A is maximum (b) in A and B is equal


(a) Falling water tries to reach a terminal velocity and hence,
(c) is same in all three (d) in A and C is same
reduces the area of cross-section to balance upward and
downward forces 93. Figs. (i) and (ii) refer to the steady flow of a
(b) As the water moves down, its speed increases and hence, (non-viscous) liquid. Which of the two figures is/are
its pressure decreases. It is then compressed by incorrect?
atmosphere
(c) The surface tension causes the exposed surface area of
the liquid to decrease continuously
(d) The mass of water flowing out per second through any
cross-section must remain constant. As the water is
almost incompressible, so the volume of water flowing Fig. (i) Fig. (ii)
out per second must remain constant. As this is equal to
velocity ´ area, the area decreases as velocity increases (a) Fig. (i) (b) Fig. (ii)
(c) both (i) and (ii) (d) None of these
88. If two ping pong balls are suspended near each other
and a fast stream of air is producce within the space 94. A tank is filled with water upto a height H. Water is
of the balls, the balls allowed to come out of a hole P in one of the walls at a
(a) come nearer to each other. depth h below the surface of water (see figure).
(b) move away from each other. Express the horizontal distance X in terms of H
(c) remain in their original positions. and h.
(d) move far away.
89. Along a streamline [NCERT Exemplar]
h
(a) the velocity of a fluid particle remains constant p
(b) the velocity of all fluid particles crossing a given H
position is constant
(c) the velocity of all fluid particles at a given instant is
constant
(d) the speed of a fluid particle remains constant X

90. An ideal fluid flows through a pipe of circular h


cross-section made of two sections with diameters 2.5 (a) X = h ( H - h) (b) X = ( H - h)
2
cm and 3.75 cm. the ratio of the velocities in the two (c) X = 2 h ( H - h) (d) X = 4 ( H - h)
pipes is [NCERT Exemplar]
(a) 9 : 4 (b) 3 : 2 95. Water stands at level A in the arrangement shown in
(c) 3 : 2 (d) 2 : 3
the figure. What will happen if a jet of air is gently
blown into the horizontal tube in the direction shown
91. Air is streaming past a horizontal air plane wing in the figure?
such that its speed is 120 ms–1 over the upper surface Jet of air
and 90 ms–1 at the lower surface. If the density of air A
is 1.3 kgm–3, what will be the gross lift on the wing? If
the wing is 10 m long and has an average width of
2 m,
(a) 81.9 N (b) 8.19 kN
(c) 81.9 kN (d) 819 kN
Telegram @unacademyplusdiscounts

464 JEE Main Physics

(a) Water will rise above A in the capillary tube 97. The level of water in a tank is 5 m high. A hole of area
(b) Water will fall below A in the capillary tube 10 cm2 is made in the bottom of the tank. The rate of
(c) There will be no effect on the level of water in the leakage of water from the hole is
capillary tube (a) 10 -2 m3s -1 (b) 102 m3s -1
(d) Air will emerge from end B in the form of bubbles 3 -1
(c) 10 m s (d) 10 -1 m3s -1
96. A cylindrical drum, open at the top, contains 15 L of
water. It drains out through a small opening at the
98. A fluid flows through a horizontal pipe having two
different cross-sections of area A and 2 A. If the
bottom. 5 L of water comes out in time t1, the next 5 L
pressure at the thin cross-section is p and fluid
in further time t2 and the last 5 L in further time t3.
velocity is v, the velocity and pressure at the thicker
Then
cross-section is (take the density of fluid as r)
(a) t1 < t2 < t3 v 1 2 v 3
(b) t1 > t2 > t3 (a) ,p + rv (b) , p + rv2
2 2 4 8
(c) t1 = t2 = t3 v 3 2 3 2
(d) t2 > t1 = t3 (c) , p + rv (d) v , p + rv
2 8 4

Round II (Mixed Bag)

Only One Correct Option 3. A streamline body with relative density r1 falls into
1. There are two identical small holes on the opposite air from a height h1 on the surface of a liquid of
sides of a tank containing a liquid. The tank is open relative density r2 , where r2 > r1. The time of
at the top. The difference in height between the two immersion of the body into the liquid will be
holes is h. As the liquid comes out of the two holes, 2h1 2h r1
the tank will experience a net horizontal force (a) (b) ´
g g r2
proportional to
2h1 r1 2h1 r1
(c) ´ (d) ´
g r2 g (r2 - r1 )
4. A liquid of density r is filled in a U-tube is accelerated
h with an acceleration a so that the height of liquid in its
two vertical arms are h1 and h2 as shown in the figure.
If l is the length of horizontal arm of the tube, the
acceleration a is
(a) h1 /2 (b) h3 /2 (c) h (d) h2
2. Water is filled up to a height h in beaker of radius R
as shown in the figure. The density of water r the
surface tension of water is T and the atmosphere h1
pressure is p0 . Consider a vertical section ABCD of h2
the water column through n diameter of the beaker.
The force on water on one side of this section by water l
on the other side of this section has magnitude.
g ( h1 - h2 ) g ( h1 - h2 )
2R
(a) towards right (b) towards left
2l 2l
g ( h1 - h2 ) g( h1 - h2 )
(c) towards right (d) towards left
B l l
A 5. A soap film is made by dipping
a circular frame of radius b in
h
C soap solution. A bubble is R
formed by blowing air with
D speed v in the form of cylinder. b v
The radius of the bubble
(a) |2p 0 Rh + pr2rgh - 2RT | (b) |2p 0 Rh + Rrgh2 - 2RT |
formed R >> b so that the air is
(c) |p 0 pR2 + Rrgh2 - 2RT | (d) |p 0 pR2 + Rrgh2 + 2RT | incident normally on the
Telegram @unacademyplusdiscounts

Properties of Liquids 465

surface of bubble. Air stops after striking surface of Which of the following graph represents the
soap bubble. Density of air is r. The radius R of the variation of pressure p along the axis of tube?
bubble when the soap bubble separates from the ring p p
is (surface tension of liquid is S).
S 4S Sb 4 Sb
(a) (b) (c) (d)
rv2 rv2 rv rv2 (a) (b)
6. A metal ball immersed in alcohol weighs W1 at 0°C
and W2 at 59°C. The coefficient of cubical expansion x x
of the metal is less than that of alcohol. Assuming
that the density of the metal is large compared to that p
p
of alcohol, it can be shown that
(a) W1 > W2 (b) W1 < W2
(c) (d)
(c) W1 = W2 (d) W1 = 2 W2
7. A uniform rod of density r is placed in a wide tank x x
containing a liquid s (s > r ). The depth of liquid in the
tank is half the length of the rod. The rod is in
11. In this figure, an ideal liquid flows through the tube
equilibrium, with its lower end resting on the bottom of
having uniform area of cross-section and is held in
the tank. In this position, the rod makes an angle q with
vertical plane. Find the ratio of speed of liquid at A
the horizontal. Then, sin q is equal to
and B and also find the pressure difference between
1 s 1s
(a) (b) these points.
2 r 2r
A
r r
(c) (d)
s s
8. The U-tube has a uniform
cross-section as shown in h
figure. A liquid is filled in the
two arms upto heights h1 and
h1
h2 and then the liquid is B
allowed to move. Neglect h2
viscosity and surface tension. (a) 2lgh (b) lgh
When the level equalize in the 3
(c) lgh (d) zero
two arms, the liquid will 2
(a) be at rest 12. There is a hole of area A at the bottom of a cylindrical
æ h -h ö vessel. Water is filled upto a height h and water flows
(b) be moving with an acceleration of g ç 1 2 ÷
è h1 + h2 + 2 ø out in t sec. If water is filled to a height 4h, then it will
g flow out in time
(c) be moving with a velocity of 1 2 (a) 2 t (b) 4 t (c) 16 t (d) 7/4 t
2( h1 + h2 + h)
(d) exert a net force to the right on the cube 13. A fire hydrant delivers water of density r at a volume
rate L. The water travels vertically upwards through
9. A stone of relative density k is released from rest on the hydrant and then does 90° turn to emerge
the surface of a lake. If viscous effects are ignored, the
horizontally at speed v. The pipe and nozzle have
stone sinks in water with an acceleration of
uniform cross-section throughout. The force exerted
(a) g (1 - k ) (b) g (1 + k ) by water on the corner of the hydrant is
æ 1ö æ 1ö
(c) g ç1 - ÷ (d) g ç1 + ÷ v
è kø è kø
v
10. A non viscous liquid is flowing through a frictionless
duct, with cross-section varying as shown in figure.
(a) zero
o x (b) rvL
(c) 2rvL
(d) 2 rvL
Telegram @unacademyplusdiscounts

466 JEE Main Physics

14. A block is submerged in vessel filled with water by a 20. An alloy of Zn and Cu (i.e., brass) weights 16.8 g in air
spring attached to the bottom of the vessel. In and 14.7 g in water. If relative density of Cu and Zn
equilibrium, the spring is compressed. The vessel are 8.9 and 7.1 respectively then determine the
now moves downwards with an acceleration a ( < g). amount of Zn and Cu in the alloy.
The spring length (a) 2g, 4g (b) 4g, 2g
(c) 9.345g, 7.455 g (d) 0, 3g
21. Two soap bubbles A and B are kept in closed chamber
where the air is maintained at pressure 8 N / m 2 .The
radius of bubbles A and B are 2 cm and 4 cm
respectively surface tension of the soap water used to
make bubbles is 0.04 N/m. Find the ratio nB / n A ,
where n A and nB are the number of moles of air in
(a) will become zero bubbles A and B respectively
(b) will decrease but not zero [Neglect the effect of gravity]
(c) will increase (a) 2 (b) 9
(d) may increase or decrease or remain constant (c) 8 (d) 6

15. Calculate the force of attraction between two parallel 22. A jar shown in figure is filled with a liquid of density
plates separated by a distance 0.2 mm after a water r. The jar is placed in vacuum. Cross-section of the jar is
drop of mass 80 mg is introduced between them. The circular and base is having a radius R. The force
wetting is assumed to be complete. (surface tension of exerted by the liquid column on the base of the jar is
water is 0.07 Nm–1)
a
(a) 0.14 N (b) 0.28 N
(c) 0.42 N (d) 0.56 N b
30°
16. A wooden ball of density r is immersed in water of 60°
density r 0 to depth h and then released. The height H
F
above the surface of water upto which the ball jump c
out of water is
(a) zero (b) h
r h ær ö R
(c) 0 (d) ç 0 - 1÷ h
r èr ø (a) rg ( a + b + c ) pR 2

17. The bottom of a cylindrical vessel has a circular hole (b) less than rg ( a + b + c ) pR2
of radius r and at depth h below the water level. If the (c) greater than rg ( a + b + c ) pR2
diameter of the vessel is D, the find then speed with (d) 2rg ( a + b + c ) pR2
which the water level in the vessel drops.
4 r2 4 D2 23. From a steel wire of density r is suspended a brass
(a) 2gh (b) block of density r . The extension of steel wire comes to
D2 B 2
l. If the brass block is now fully immersed in a liquid of
4 D2
(c) 2 2gh (d) None of these density r L , the extension becomes l ¢. The ratio, l / l ¢ will
r
be
18. A canister has a small hole at its bottom. Water rB - r rL
(a) (b)
penetrates into the canister when its base is at a rL - r rB - rL
depth of 40 cm from the surface of water. If surface rB - rL rB
tension of water is 73.5 dyne/cm, find the radius of (c) (d)
rB rB - rL
the hole.
(a) 375 mm (b) 3.75 mm 24. A glass tube 80 cm long and open at both ends is half
(c) 0.0375 mm (d) zero immersed in mercury. Then the top of the tube is
closed and it is taken out of the mercury. A column of
19. A piece of gold weights 50 g in air and 45 g in water. If mercury 20 cm long then remains in the tube. The
there is a cavity inside the piece of gold, then find its
atmospheric pressure (in cm of Hg) is
volume [Density of gold = 19.3 g/cc].
(a) 90 (b) 75
(a) 2.4 cm3 (b) 2.4 m3
(c) 60 (d) 45
(c) 4 .2 m3 (d) 4.2 mm3
Telegram @unacademyplusdiscounts

Properties of Liquids 467

25. Equal volumes of two immiscible liquids of densities 31. A vessel whose bottom has round holes with diameter
r and 2r are filled in a vessel as shown in figure. Two of 1 mm is filled with water. Assuming that surface
small holes are made at depth h/2 and 3 h/2 from the tension acts only at holes, then the maximum height
surface of lighter liquid. If v1 and v2 are the velocities to which the water can be filled in vessel without
of efflux at these two holes, then v1/ v2 is leakage is (Surface tension of water is 75 ´ 10-3 Nm –1
and g = 10 ms–2)
(a) 3 cm (b) 0.3 cm
h (c) 3 mm (d) 3 m
v1
32. Glycerine flows steadily through a horizontal tube of
h v2 length 1.5 m and radius 1.0 cm. If the amount of
glycerine flowing per second at one end is
1 1 4.0 ´ 10-3kg/s. What is the pressure difference
(a) (b) between the two ends of the tube? (Density of
2 4
1 1 . ´ 103 kg/m 3 and viscosity of glycerine
glycerine = 13
(c) d) . Pa-s).
= 083
2 2 2
(a) 9.75 ´ 102 Pa (b) 6.75 ´ 102 Pa
26. Two capillaries of radii r1 and r2 , lengths l1 and l2 (c) 5.75 ´ 102 Pa (d) 6.95 ´ 103 kPa
respectively are in series. A liquid of viscosity h is
flowing through the combination under a pressure 33. Two soap bubbles of radii r1 and r2 equal to 4 cm and 5
difference p. What is the rate of volume flow of liquid? cm respectively are touching each other over a
-1 common surface AB (shown in figure). Its radius will
pp æ l4 l ö 8pp æ l1 l ö
(a) ç 4 + 44 ÷ (b) ç 4 + 24 ÷ be
8h è r1 r2 ø h è r1 r2 ø
-1 -1 A
pp æ r14 r24 ö pp æ l1 l ö
(d) ç + ÷ (d) ç 4 + 24 ÷ 4cm 5cm
8h è l1 l2 ø 8h è r1 r2 ø
B
27. Two soap bubbles A and B are formed at the two open
ends of a tube. The bubble A is smaller than bubble B.
(a) 4 cm (b) 4.5 cm
Valve and air can flow freely between the bubbles,
(c) 5 cm (d) 20 cm
then
(a) there is no change in the size of the bubbles 34. What is the excess pressure inside a bubble of soap
(b) the two bubbles will become of equal size solution of radius 5.00 mm, given that the surface
(c) A will become smaller and B will become larger tension of soap solution at the temperature (20° C) is
(d) B will become smaller and A will become larger . ´ 10-2 N/m? If an air bubble of the same
250
dimension were formed at a depth of 40.0 cm inside a
28. A trough contains mercury to a depth of 3.6 cm. If
container containing the soap solution (of relative
some amount of mercury is poured in it then height of
density 1.20), what would be the pressure inside the
mercury in the trough will be
. ´ 105 Pa.)
bubble? (1 atmospheric pressure is 101
(a) 3.6 cm
(a) 7.06 ´ 105 Pa (b) 2.06 ´ 105 Pa
(b) 7.2 cm
(c) 6 cm (c) 1.06 ´ 105 Pa (d) 1.86 ´ 105 Pa
(d) None of the above 35. Speed of 2 cm radius ball in a viscous liquid is
29. Two pieces of glass plate one upon the other with a 20 cms–1. Then the speed of 1 cm radius ball in the
little water in between them cannot be separated same liquid is
easily because of (a) 7.06 ´ 105 Pa (b) 2.06 ´ 105 Pa
(a) inertia (b) pressure (c) 1.06 ´ 105 Pa (d) 1.86 ´ 105 Pa
(c) surface tension (d) viscosity 36. The work done in increasing the size of a rectangular
30. An aeroplane of mass 3 ´ 104 kg and total wing area soap film with dimensions 8 cm ´ 3.75 cm to
of 120 m2 is in a level flight at some height. The 10 cm ´ 6 cm is 2 ´ 10-4 J. The surface tension of the
difference in pressure between the upper and lower film in Nm–1 is
surfaces of its wings in kilo pascal is ( g = 10 ms -2 ) (a) 1.65 × 10–2 (b) 3.3 × 10–2
(a) 2.5 (b) 5.0 (c) 6.6 × 10–2 (d) 8.25 ×10–2
(c) 10.0 (d) 12.5
Telegram @unacademyplusdiscounts

468 JEE Main Physics

37. The glycerine of density 1.25 ´ 103 kmg –3 is flowing 44. A wooden block with a coin placed on its top.floats in
through a conical tube with end radii 0.1 m and water as shown in figure
0.04 m respectively. The pressure difference across Coin
the ends is 10 Nm–2. The rate of flow of glycerine
h
through the tube is
(a) 6.4 ´ 10 –2 m3s –1 (b) 6.4 ´ 10 –4 m3s –1 h
–2 3 –1 3 3 –1
(c) 12.8 ´ 10 ms (d) 12.8 ´ 10 m s

38. A film of water is found between two straight parallel


The distance I and h are shown in the figure. After
wires of length 10 cm each separated by 0.2 cm. If
some time the coin falls into the water. Then
their separation is increased by 1 mm, while still
[NCERT Exemplar]
maintaining their parallelism, how much work will
(a) I decreases (b) h decreases
have to be done? (surface tension of water is
(c) I increases (d) h increases
7.2 ´ 10–2 Nm –1)
(a) 7.22 ´ 10 –6 J (b) 1.44 ´ 10 –5 J 45. A spring balance reads w1 when a ball of mass m is
suspended from it. A weighing machine reads w2
(c) 2.88 ´ 10 –8 J (d) 5.76 ´ 10 –5 J
when a beaker of liquid is kept on the pan of balance.
39. Water flows through a vertical tube of variable When the ball is immersed in liquid, the spring
cross-section. The area of cross-section at A and B are balance reads w3 and the weighing machine reads w4 .
6 and 3 mm2 respectively. If 12 cc of water enters per The two balances are now so arranged that the
second through A, find the pressure difference suspended mass is inside the liquid in a beaker. Then
p A - p B (g = 10 ms -2 ) The separation between (a) w3 > w1 (b) w 4 > w2
cross-section at A and B is 100 cm. (c) w3 < w1 and w 4 > w2 (d) w3 > w1 and w 4 < w2
(a) 1.6 ´ 105 dyne cm–2 (b) 2.29 ´ 105 dyne cm–2
4
(c) 5.9 ´ 10 dyne cm –2
(d) 3.9 ´ 105 dyne cm–2 Comprehension Based Questions
40. A body of uniform cross-sectional area floats in a Passage I
liquid of density thrice its value. The portion of
Water of density r at a depth h behind the vertical
exposed height will be
face of dam whose cross-sectional length is l and
(a) 2/3 (b) 5/6 (c) 1/6 (d) 9/10
cross-sectional area A. It exerts a horizontal
resultant force on the dam tending to slide it along its
More Than One Correct Option foundation and a torque tending to overturn the dam
41. Streamline flow is more likely for liquids with about the point O.
[NCERT Exemplar ]
(a) high density (b) high viscosity h
(c) low density (d) low viscosity
42. Two solid spheres A and B of equal O
volumes but of different densities
A
dA and dB are connected by a string. 46. The pressure energy per unit volume of the water
They are fully immersed in a fluid of dam is
density dF . They get arranged into 1 1
B (a) Ahrg (b) hrg (c) Ahrg (d) rgh2
an equillibrium state of as shown in 2 2
the figure with a tension in the string. The 47. The height at which the resultant force would have to
arrangement is possible only if act to the same torque is
h h h 2h
(a) dA < dF (b) dB > dF (a) (b) (c) (d)
(c) dA > dF (d) dA + dB = 2dF 6 3 2 3

43. When an air bubble moves up from the bottom of a 48. Horizontal force on the vertical face of the dam is
1 1
lake (a) rghl (b) rgh2 l (c) rglh (d) rglh2
2 2
(a) its acceleration decreases and becomes zero
(b) its acceleration increases and becomes constant 49. Pressure on the vertical face of the dam is
(c) its velocity increases and becomes constant 1 1
(a) rgh (b) rgh (c) rgh2 (d) rgh2
(d) its velocity decreases and becomes zero 2 2
Telegram @unacademyplusdiscounts

Properties of Liquids 469

50. Torque about point O is select the correct choices from the codes (a), (b), (c) and (d) given
1 1 1 below
(a) rglh3 (b) rglh3 (c) rglh3 (d) rglh3
2 3 6 (a) If both Assertion and Reason are true and the Reason
is correct explanation of the Assertion
Passage II
(b) If both Assertion and Reason are true but Reason is
A plane is in level flight at a constant speed and each not correct explanation of the Assertion
wing has an area of 25 m2. During flight the speed of (c) If Assertion is true but Reason is false
the air is 216 kmh–1 over the lower wing surface and (d) If Assertion is false but the Reason is true
252 kmh–1 over the upper wing surface of each
wing of aeroplane. Take density of air = 1 kgm -3 and 56. Assertion When height of a tube is less than calculated
g = 10 ms–2. height of liquid in the tube, the liquid does not
overflow.
51. The mass of the plane is
(a) 25 kg (b) 250 kg (c) 1750 kg (d) 3250 kg Reason The meniscus of liquid at the top of the tube
becomes flat.
52. If a plane is in level flight with a speed of 360 kmh–1
then the fractional increase in the speed of the air on 57. Assertion The velocity of flow of a liquid is smaller
the upper surface of the wing relative to the lower where pressure is larger and vice-versa.
surface is Reason This is in accordance with Bernoulli’s
(a) 13 % (b) 9 % (c) 6.5 % (d) 4.5 % theorem.
53. Pressure difference on each wing of aeroplane is 58. Assertion A hydrogen filled balloon stops rising after
(a) 5 Nm–2 (b) 50 Nm–2 (c) 350 Nm–2 (d) 650 Nm–2 it has attained a certain height in the sky.
54. Percentage of velocity difference of the upper and Reason The atmospheric pressure decreases with
lower surface of the wings of aeroplane is height and becomes zero when maximum height is
(a) 14.3 % (b) 15.4 % (c) 16.7 % (d) 17.4 % attained.
55. The total upward force on the plane is 59. Assertion For the flow to be streamline, value of
(a) 250 N (b) 2500 N (c) 17500 N (d) 32500 N critical velocity should be as low as possible.
Reason Once the actual velocity of flow of a liquid
Assertion and Reason becomes greater than the critical velocity, the flow
becomes turbulent.
Direction Question No. 56 to 60 are Assertion-Reason type.
Each of these contains two Statements: Statement I (Assertion), 60. Assertion A bubble comes from the bottom of a lake to
Statement II (Reason). Each of these questions also has four the top.
alternative choice, only one of which is correct. You have to Reason Its radius increases.

Previous Years’ Questions


61. A body floats in a liquid contained in a beaker. If the 9 4 8 3
(a) g/cc (b) g/cc (c) g/cc (d) g/cc
whole system falls under gravity, then the upthrust 4 9 3 8
on the body due to liquid is [UP SEE 2009] 64. Bernoulli’s theorem is a consequence of the law of
(a) equal to the weight of the body in air conservation of [UP SEE 2008]
(b) equal to the weight of the body in liquid (a) momentum (b) mass
(c) zero
(c) energy (d) angular momentum
(d) equal to the weight of the immersed part of the body
65. Two rain drops reach the earth with different
62. A cube made of material having a density of
3 –3 terminal velocities having ratio 9 : 4. Then, the ratio
0.9 ´ 10 kgm floats between water and a liquid of
of their volume is [EAMCET 2008]
density 0.7 ´ 103 kgm –3, which is immiscible with
water. What part of the cube is immersed in water? (a) 3 : 2 (b) 4 : 9
[BVP Engg. 2008] (c) 9 : 4 (d) 27 : 8

(a)
1
(b)
2
(c)
3
(d)
3 66. The surface tension of soap solution is 0.03 Nm–1.
3 3 4 7 The work done in blowing to form a soap bubble of
63. A body floats with one-third of its volume consider surface area 40 cm2, in joule is [EAMCET 2008]
water and 3/4 of its volume outside another liquid. (a) 1.2 ´ 10 –4 (b) 2.4 ´ 10 –4
The density of other liquid is [BVP Engg. 2008] (c) 12 ´ 10 –4 (d) 24 ´ 10 –4
Telegram @unacademyplusdiscounts

470 JEE Main Physics

67. A body weigh 50 g in air and 40 g in water. How much (a)


Vg (r1 - r2 )
(b)
Vg(r1 - r2 )
would it weight in a liquid of specific gravity 1.5? k k
[Karnataka CET 2008] Vgr1 Vgr1
(c) (d)
(a) 65 g (b) 45 g (c) 30 g (d) 35 g k k
68. When the temperature of water rises, the apparent 73. A soap bubble is charged to a potential of 16 V. Its
weight of the wood will [WB JEE 2008] radius is, then doubled. The potential of the bubble now
(a) increase (b) decrease will be [BVP Engg. 2007]
(c) may increase or decrease (d) remain same (a) 16 V (b) 8 V (c) 4 V (d) 2 V
69. The area of cross-section of one limb of an U-tube is 74. A frame made of metallic wire enclosing a surface
twice that of the other. Both the limbs contains mercury area A is covered with a soap film. If the area of the
at the same level. Water is poured in the wider tube so frame of metallic wire is reduced by 50%, the energy
that mercury level in it goes down by 1 cm. The height of of the soap film will be changed by [UP SEE 2007]
water column is (Density of water = 103 kgm–3, density (a) 100 % (b) 75 % (c) 50 % (d) 25 %
of mercury = 13.6 ´ 103 kgm –3) [Kerala CET 2008] 75. The cylindrical tube of a spray pump has a
(a) 13.6 cm (b) 40.8 cm cross-section of 8 cm2, one end of which has 40 fine
(c) 6.8 cm (d) 54.4 cm holes each of area 10–8m2. If the liquid flows inside
70. A capillary tube (A) is dipped in water. Another the tube with a speed of 0.15 m min–1, the speed with
identical tube (B) is dipped in a soap-water solution. which the liquid is ejected through the holes is
Which of the following shows the relative nature of [Karnataka CET 2007]
the liquid columns in the two tubes? [AIEEE 2008] (a) 50 ms–1 (b) 5 ms–1 (c) 0.05 ms–1 (d) 0.5 ms–1
76. A boat at anchor is rocked by waves whose crests are
A B
B 100 m apart and velocity is 25 ms–1. The boat bounces
A up once in every [UP SEE 2006]
(a) (b) (a) 2500 s (b) 75 s (c) 4 s (d) 0.25 s
77. A tank is filled with water of density 1 g per cm3 and
oil of density 0.9 g cm–3. The height of water layer is
B 100 cm and of the oil layer is 400 cm. If g = 980 cms–2,
A
A then the velocity of efflux from an opening in the
B
(c) (d) bottom of the tank is [UP SEE 2006]
(a) 900 ´ 980 cms–1 (b) 1000 ´ 980 cms–1
(c) 92 ´ 980 cms–1 (d) 920 ´ 980 cms–1
78. A body of density D1 and mass M is moving downward
71. A jar is filled with two in glycerine of density D2 . What is the viscous force
non-mixing liquids 1 and 2 acting on it? [Orissa JEE 2006]
having densities r1 and r2 Liquid 1
ρ1 æ D ö æ D ö
respectively. A solid ball, (a) Mg ç1 - 2 ÷ (b) Mg ç1 - 1 ÷
è D1 ø è D2 ø
made of a material of density ρ3
(c) MgD1 (d) MgD2
r 3 is dropped in the jar. It
comes to equilibrium in the ρ2 79. A body of mass 120 kg and density 600 kgm–3 floats in
Liquid 2
position shown in the figure. water. What additional mass could be added to the
Which of the following is true body so that the body will just sink? [Orissa JEE 2006]
for r1, r2 and r 3 ? [AIEEE 2008] (a) 20 kg (b) 80 kg (c) 100 kg (d) 120 kg
(a) r1 < r3 < r2 (b) r3 < r1 < r2 80. A solid sphere of volume V and density r floats at the
(c) r1 > r3 > r2 (d) r1 < r2 < r3 interface of two immiscible liquids of densities r1 and
72. A spherical solid ball of volume V is made of a r2 respectively. If r1 < r < r2 , then the ratio of volume
material of density r1. It is falling through a liquid of of the parts of the sphere in upper and lower liquids
density r 2(r 2 > r1 ). Assume that the liquid applied a is [Kerala CET 2006]
r - r1 r2 - r r + r1 r + r2
viscous force on the ball that is proportional to the (a) (b) (c) (d)
square of its speeds v, i.e., Fviscous = - kv2 ( k > 0). The r2 - r r - r1 r + r2 r + r1
terminal speed of the ball is [AIEEE 2008] r1 r2
(e)
r
Telegram @unacademyplusdiscounts

Properties of Liquids 471

81. An incompressible fluid flows steadily through a water then the correct statement is that the shell is
cylindrical pipe which has radius 2R at a point A and [IIT JEE 2012]
radius R at a point B. Further along the flow of (a) more than half filled of r C is less than 0.5
direction if the velocity at point A is v, its velocity at (b) more than half filled of r C is less than 1.0
point B will be [BVP Engg. 2006] (c) half filled of r C is less than 0.5
(a) v/4 (b) 2v (c) 4v (d) -
v (d) less than half filled if r C is less than 0.5
2
89. Work done in increasing the size of a soap bubble
82. If we dip capillary tubes of different radii r in water from a radius of 3 cm to 5cm is nearly (surface tension
and the water rises to different height h in them, of soap solution = 0.03 Nm -1) [IIT JEE 2012]
then we shall have constant [BVP Engg. 2006] (a) 4p mJ (b) 0.2p mJ (c) 2p mJ (d) 0.4p mJ
(a) h/r2 (b) h/r (c) hr2 (d) hr 90. The relation between surface tension T.Surface area
83. A bubble rises from bottom of a lake 90 m deep. On A and surface energy E is given by [Orissa JEE 2011]
reaching the surface, its volume becomes (take (a) T =
E
(b) T = EA (c) E =
T
(d) T =
A
atmospheric pressure correspond upto 10 m of water) A A E
[BVP Engg. 2006] 91. The lower end of a glass capillary tube is dipped in
(a) 18 times (b) 4 times (c) 8 times (d) 10 times water, water rises to height of 8 cm. The tube is then
84. Water rises to a height of 10 cm in a capillary tube broken at a height of 6 cm. The height of water
and mercury falls to a depth of 3.42 cm in the column and angle of contact will be [Orissa JEE 2010]
same capillary tube. If the density of mercury is 3 3
(a) 6 cm, sin -1 (b) 6 cm, cos -1
13.6 kgm–3 and angle of contact is 135°. The ratio of 4 4
surface tensions for water and mercury is (angle of -1 1 -1 1
(c) 4 cm, cos (d) 4 cm, cos
contact for water and glass is 8°) [BVP Engg. 2006] 2 2
(a) 1 : 0.5 (b) 1 : 65 92. Water is flowing contineously from a tap having an
(c) 1.5 : 1 (d) 1 : 3 internal diameter 8 ´ 10-3 m. The water velocity as it
85. Water is flowing through a pipe of constant leaves the tap is 0.4 ms -1. The diameter of the water
cross-section. At some point the pipe becomes narrow stream at a distance 2 ´ 10-1 m below the tap is
and the cross-section is halved. The speed of water is close to [AIEEE 2011]
[UP SEE 2005] (a) 5 ´ 10 -3 (b) 7.5 ´ 10 -3
(a) reduced to zero (c) 9.6 ´ 10 -3 (d) 3.6 ´ 10 -3
(b) decreased by factor of 2 93. A uniform long tube is bent into a circle of radius R
(c) increased by a factor of 2 and it lies in a vertical plane. Two liquids of same
(d) unchanged volume but densities r and d. Fill half tube. The
86. The force of cohesion is [BVP Engg. 2005] angle, q is [WB JEE 2010]
(a) maximum in solids
(b) maximum in liquids
(c) maximum in gases R
R
(d) same in solid, liquid and gas δ
B
87. A thin liquid film formed between a U ρ
shaped wire and a light slider supports a
Film ær - dö
weight of 1.5 ´ 10-2 N. The length of the ærö
(a) tan -1 ç ÷ (b) tan -1 ç ÷
slider is 30 cm and its weight è r + dø è dø
negligible.The surface tension of the æ dö æ r + dö
(c) tan -1 ç ÷ (d) tan -1 ç ÷
liquid film is [AIEEE 2012]
w
èrø èr - dø

94. If the terminal speed of a sphere of gold (density


(a) 0.0125 Nm-1 (b) 0.1 Nm-1
= 9.5 kg/ m 3 ) is 0.2 m/s in a viscous liquid (density
(c) 0.05 Nm-1 (d) 0.025 Nm-1
= 1.5 kg/ m 3 ). Find the terminal speed of sphere of
88. A thin uniform cylindrical shell closed at both ends is silver (density = 10.5 kg/m 3 ) of the same size in the
partially filled with water . It is floating vertically in same liquid [WB JEE 2010]
water in half-submerged slate. If r c is the relative (a) 0.133 m/s (b) 0.1 m/s
density of the material of the shell with respect to (c) 0.2 m/s (d) 0.4 m/s
Telegram @unacademyplusdiscounts

Answers
Round I
1. (c) 2. (c) 3. (c) 4. (b) 5. (b) 6. (a) 7. (c) 8. (b) 9. (d) 10. (c)
11. (c) 12. (c) 13. (b) 14. (c) 15. (d) 16. (a) 17. (b) 18. (c) 19. (d) 20. (d)
21. (a) 22. (d) 23. (d) 24. (c) 25. (b) 26. (b) 27. (b) 28. (a) 29. (c) 30. (c)
31. (c) 32. (a) 33. (b) 34. (c) 35. (c) 36. (c) 37. (a) 38. (b) 39. (b) 40. (b)
41. (d) 42. (b) 43. (a) 44. (d) 45. (d) 46. (d) 47. (d) 48. (c) 49. (b) 50. (b)
51. (b) 52. (d) 53. (a) 54. (c) 55. (b) 56. (b) 57. (c) 58. (c) 59. (b) 60. (d)
61. (a) 62. (a) 63. (d) 64. (c) 65. (d) 66. (b) 67. (a) 68. (d) 69. (a) 70. (c)
71. (a) 72. (b) 73. (d) 74. (d) 75. (b) 76. (a) 77. (c) 78. (a) 79. (b) 80. (c)
81. (d) 82. (d) 83. (b) 84. (c) 85. (c) 86. (c) 87. (d) 88. (a) 89. (b) 90. (a)
91. (c) 92. (a) 93. (a) 94. (c) 95. (a) 96. (a) 97. (a) 98. (c)

Round II
1. (c) 2. (b) 3. (d) 4. (c) 5. (b) 6. (b) 7. (a) 8. (c) 9. (c) 10. (b)
11. (b) 12. (a) 13. (d) 14. (c) 15. (b) 16. (d) 17. (a) 18. (c) 19. (a) 20. (c)
21. (d) 22. (c) 23. (b) 24. (c) 25. (a) 26. (d) 27. (c) 28. (b) 29. (c) 30. (a)
31. (a) 32. (a) 33. (d) 34. (c) 35. (a) 36. (b) 37. (b) 38. (b) 39. (a) 40. (a)
41. (b,c) 42. (a,b,d) 43. (a,c) 44. (a,b) 45. (b) 46. (b) 47. (b) 48. (b) 49. (b) 50. (c)
51. (d) 52. (c) 53. (d) 54. (b) 55. (d) 56. (a) 57. (a) 58. (b) 59. (d) 60. (a)
61. (a) 62. (b) 63. (c) 64. (c) 65. (d) 66. (b) 67. (d) 68. (d) 69. (b) 70. (c)
71. (a) 72. (b) 73. (b) 74. (c) 75. (b) 76. (c) 77. (d) 78. (a) 79. (b) 80. (b)
81. (c) 82. (d) 83. (d) 84. (b) 85. (c) 86. (a) 87. (d) 88. (d) 89. (d) 90. (a)
91. (b) 92. (d) 93. (a) 94. (b)

the Guidance
Round I
M M 3. Surface area, A = 4 pr 2
1. Volume of ice = , volume of water =
r s or r = (A / 4p )
1/ 2

M M æ1 1ö 4 4
Change in volume = - =Mç - ÷ Volume V = pr3 = p ( A / 4p )3 / 2 = kA3 / 2
r s èr s ø 3 3
4p 1
2. Given, mass of girl (m) = 50 kg where, ´ = k = constant
3 ( 4 p )3 / 2
Diameter of circular heel (2r) = 10
. cm
Using Boyle’s law, we have
\ Radius (r) = 0.5 cm = 5 ´ 10 -3 m
p1V1 = p2V2
Area of circular heel ( A) = pr 2 p V (10 + h) kA13./2
or p2 = 1 1 =
= 3.14 ´ (5 ´ 10 -3) 2 m 2 V2 kA32/ 2
3/ 2
= 78.50 ´ 10 -6 m 2 æA ö
or p2 = (10 + h) ç 1 ÷
\ Pressure exerted on the horizontal floor è A2 ø
F mg As p2 = 10 of water, so
p= =
A A 10 + h
10 =
50 ´ 9.8 8
= = 6.24 ´ 10 6 Pa
78.50 ´ 10 -6 or 80 = 10 + h or h = 70 m
Telegram @unacademyplusdiscounts

Properties of Liquids 473

4. Ax ´ 13.6 = (29.2 - x ) ´ 1
or x = 2 cm
Spirit \ Height of water column = (29.2 - 2) = 27.2 cm
12.5
Water 10 cm cm \Pressure of the liquids at the bottom
= 27.2 cm of water column + 2 cm of Hg column
27.2
= of Hg column + 2 cm of Hg column
Mercury 13.6
= 4 cm of Hg column
Height of water column h1 = 10.0 cm
Dp
8. As, Bulk modulus, B = -V0
Density of water (r1) = 1 g/cm3 Dv
Height of spirit column (h 2) = 12.5 cm Dp
Þ DV = -V0
B
Density of spirit (r 2) = ?
æ Dp ö
The mercury column in both arms of the U-tube are at same Þ V = V0 ç1 - ÷
è B ø
level, therefore pressure in both arms will be same.
-1
\Pressure exerted by water column = Pressure exerted by æ Dp ö æ Dp ö
\ Density, r = r 0 ç1 - ÷ = p0 ç1 + ÷
sprit column è B ø è B ø
\ p1 = p2 where, Dp = p - p0 = hr 0 g
h1 r1 g = h 2 r 2 g Pressure difference between depth and surface of ocean.
h1 r1 10 ´ 1 æ r gy ö
or r2 = = = 0.80 g/cm3 \ r = r 0 ç1 + 0 ÷ ( As,h = y)
h2 12.5 è B ø
Density of spirit 9. Let, b be width of the glass wall. When the tank is half filled
Specific gravity of spirit =
Density of water then the average force on the glass wall is
0.80 F = average pressure ´ area
= = 0.80
1 éæ 4ö ù æ4 ö
= ê ç ÷ rw g ú ´ ç ´ b÷
5. Force on the base of the vessel ëè 2 ø û è2 ø
= pressure ´ area of the base When tank is filled up to height 4 m, then
-3
= hrg ´ A = 0.4 ´ 900 ´ 10 ´ 2 ´ 10 F' = ( 4 rw g ) ( 4 ´ b)
= 7.2 N F¢ 4 ´ 4
= = 4 or F ¢ = 4 F
F 2 ´2
6. When 15.0 cm of water is poured in each arm then,
10. Depth of point P below the free surface of water in the vessel
height of water column (h1) = 10 + 15 = 25 cm
= (1 + h). Since, the liquid exerts equal pressure in all direction
Height of spirit column (h 2) = 12.5 + 15 = 27.5 cm at one level, hence the pressure at P = (H - h) rg .
Density of water (rw ) = 1g/cm3
11. Effective weight of solid of specific gravity 1 when immersed
Density of spirit (r s ) = 0.80 g/cm3 in water will be zero.
3
Density of mercury (r m) = 13.6 g/cm 12. Atmospheric pressure ( p) = 1013
. ´ 10 5 Pa
Let in equilibrium, the difference in the level of mercury in Density of French wire (r) = 984 kg/m3
both arms be h cm.
Let h be the height of the wine column for normal
\ hr m g = h1rw g - h2r s g atmospheric pressure.
h1 rw - h2 r s For normal atmospheric pressure ( p) = hrg
or h=
rm p 1.013 ´ 10 5
25 ´ 1 - 27.5 ´ 0.80 \ h= = = 10.5 m
= rg 984 ´ 9.8
13.6
= 0.221 cm
13. Ice is lighter than water. When ice melts, the volume
occupied by water is less than that of ice. Due to which the
Therefore, mercury will rise in the arm containing spirit by level of water goes, down.
0.221 cm.
14. Pressure at the bottom p = (h1d1 + h2 d 2) g
7. Let, A be the area of cross-section of the cylindrical vessel and
x cm be the height of mercury in vessel. The height of water in = [250 ´ 1 + 250 ´ 0.85] g
the vessel = (29.2 ´ x ) cm. = 250 [1.85] g
As per question = 462.5 g dyne/cm
Telegram @unacademyplusdiscounts

474 JEE Main Physics

15. Pull on the rope = effective weight 22. Here, mass of block = m = 1kg
= [1650 + (1500 ´ 0.2) - 1500 ´ 1.3] kgf Volume of the block, V = 3.6 ´ 10 -4m3
= 1650 +300 –1950 = 0 Tension in the string, T = mg = mg - Vr water g
ærö \ Decrease in the tension of string
16. Fraction of volume immersed in the liquid, Vin = ç ÷ V i. e. , It
ès ø
T - T ¢ = mg - [mg - vr water g ] = Vr water g
depends upon the densities of the block and liquid so, there
= 3.6 ´ 10 -4 m3 ´ 10 3 kgm-3 ´ 10 ms-2 = 3.6 N
will be no change in it if system moves upward of downward
with constant velocity i. e. , uniform acceleration. 23. The upward thrust (i. e., buoyancy force) acts on the body and
17. Since, density of iron is more than that of marble, the volume an equal and opposite force acts on the water so the weight
of iron is less than that of marble for the given mass. The will be the sum of the two = 600 + 40 = 640 g
upthrust of water on iron will be less than that on marble. Due 24. The speed of the body just before entering the liquid is
to which iron cube will weight more. v = 2gh. The buoyant force B of the lake (i. e. , upward thrust
18. The effective weight of the block in liquid will become less of liquid on the body) is greater than the weight of the body w,
than 2 kg due to buoyancy of liquid. As a result of which A since s > r. If V is the volume of the body and a is the
will read less than 2 kg. acceleration of the body inside the liquid, then
As, the body immersed in liquid has some effective weight B - w = ma
acting downwards so the reading of B will be more than 5 kg. or sVg - rVg = rVa
m
19. Volume of cylinder = or (s - r) g = ra
r
(s - r) g
æ mö or a=
Upthrust on cylinder = ç ÷ sg r
èrø
From Newton’s third law, the downward force exerted by Using the relation, v 2 = u 2 + 2as, we have
æ mö (s - r)
cylinder on the liquid is = ç ÷ sg 0 = ( 2gh) 2 - 2g s
èrø r
msg hr
\ Increase in pressure = or s=
rA s -r
20. Initially the position of wooden block is as shown in figure. 25. Mass of the cylinders = AL (r1 + r2). As cylinders float with
Since, the density of block is half than that of water, hence half
of its volume is immersed in water. length L /2 outside the water, therefore length of cylinder
w
inside the water = 3 L /2. When cylinders are floating, then,
l 2 weight of cylinder = weight of water displaced by cylinder.
So, AL (r1 + r 2) g = A(3L / 2) ´ 1 ´ g
l 2
l or r1 + r 2 = 3 / 2
As r1 < r 2, so, r1 < 3 / 4
26. Let, v be the volume of ice-berg outside the sea water while
floating. Therefore, volume of ice-berg inside the sea water
(a) (b)
= (V - v). As ice-berg is floating, so weight of ice-berg = weight
When weight, w is put on the block, the remaining half of the of sea water displaced by ice-berg.
volume of block is immersed in water, figure (b). Therefore, i. e. , V ´ 0.9 ´ g = (V - v) ´ 1.1 ´ g
w = additional upthrust + spring force
or 1.1v = 1.1V - 0 / 9 V
l ælö æ kö
= l ´ l ´ ´ 2r ´ g + k ç ÷ = l ç l 2rg + ÷ or v / V = 0.2/1.1 = 2 /11
2 è2ø è 2ø
27. As, solid is floating in liquid, so, weight of solid body = weight
21. Given, size of the plate = 2m× 5m and of liquid displaced by immersed part of the body i. e. ,
Greatest and least depths of the plate are 6m and 4m. VDg = v dg
We know that area of the plate A = 2 ´ 3 = 6 m2 or v /V = D/d
and depth of centre of the plate V
28. Given, 6 g = ´ 10 3 ´ g ...(i)
6+ 4 3
x= = 5m
2 and (6 + m) g = V ´ 10 3 ´ g …(ii)
\Total thrust on the plate
r = rw g A x Dividing Eq. (ii) by Eq. (i), we get
= 10 3 ´ 9.8 ´ 6 ´ 5 = 294 ´ 10 3 N or m = 18 - 6 = 12 kg
Telegram @unacademyplusdiscounts

Properties of Liquids 475

29. Let, l be the length of the cylinder in water when it is in the 2 ´ 4.65 ´ 10 -1
= 1.01 ´ 10 5 +
vertical position and A be the cross-sectional area of the 3 ´ 10 -3
cylinder. As cylinder is floating = 1.01 ´ 10 5 + 3.10 ´ 10 2
So, weight of cylinder = upward thrust
= 1.01 ´ 10 5 + 0.00310 ´ 10 5
or mg = Alrg
or m = Alr = 1.01310 ´ 10 5 Pa
When the cylinder is tilted through an angle q, then length 2S 2 ´ 4.65 ´ 10 -1
Excess pressure inside the drop ( Dp) = =
of cylinder in water =
l R 3 ´ 10 -3
cos q . ´ 10 2
= 310
l
Weight of water displaced = Arg = 310 Pa
cos q
lArg 34. Let R be the radius of the biggest aluminium coin which will
\ Restoring force = - lArg
cos q be supported on the surface of water due to surface tension.
é 1 ù é 1 ù Then, mg = S ´ 2 pR
= lArg ê - 1ú = mg ê - 1ú
ë cos q û ë cos q û or 2
pR t rg = S ´ 2 pR
30. Let D1 be the inner diameter of the hemispherical bowl and D2 or R = 2 S /rgt
be the outer diameter of the bowl. As, bowl is just floating so 35. As volume remains constant i. e. ,R3 = 8000 r3 or R = 20 r
3
4 æ 1ö Surface energy of one big drop
p ç ÷ ´ 1.2 ´ 10 3 Now,
3 è2ø Surface energy of 8000 small drops
3 3
4 é æ 1ö æ D ö ù 4 pR 2T R2 (20 r) 2 1
= p ê ç ÷ - ç 1 ÷ ú ´ (2 ´ 10 4) = = = =
3 êë è 2 ø è 2 ø úû 2 2
8000 ´ 4pr T 8000 r 8000 r 2 20
1.2 ´ 10 3
or = 1 - D13 36. As, surface energy = surface tension ´ surface area
2 ´ 10 4
1/ 3 1/ 3 i. e. , E = S ´2 A
æ 1.2 ö æ18.8 ö
Þ D1 = ç1 - ÷ =ç ÷ New surface energy, E1 = S ´ 2 ( A /2) = S ´ A
è 20 ø è 20 ø
E - E1
% decrease in surface energy = ´ 100
On solving, D1 = 0.98 m E
31. As, weight of metal disc = total upward force 2 SA - SA
= ´ 100 = 50%
2 SA
T
θ θ T
r 37. If r is the radius of smaller droplet and R is the radius of bigger
drop, then according to question,
4 3 4
pR = 10 6 ´ pr3
3 3
= upthrust force + force due of surface tension R
= weight of displaced water + T cos q (2 pr) or r= = 0.01R
100
= w + 2 prT cos q = 0.01 ´ 10 –2 m = 10 –4 m
32. Force on the ring due to surface tension of water \Work done = surface tension ´ increase in area
= ( pD1 + pD2) S = mg = 35 ´ 10 –2 ´ [(10 6 ´ 4 p ´ (10 -4) 2 - 4p ´ (10 -3) 2]
mg 3.47 ´ 980
So, S= = = 4.35 ´ 10 –2 J
p (D1 + D2) (22/7) ´ (8.5+8.7)
= 72.07 dyne cm–1
38. As, work done = surface tension ´ surface area
= 1.9 ´ 10 –2 ´ ( 4 pR 2) ´ 2
33. Given, radius of drop (R) = 3.00 mm = 1.9 ´ 10 –2 ´ 4 ´ p (1 ´ 10 -2) 2 ´ 2
-3
= 3 ´ 10 m = 15.2 ´ 10 –6 pJ
-1
Surface tension of mercury (S) = 4.65 ´ 10 N/m
39. When two drops are splitted, the law of conservation of mass
5
Atmospheric pressure (p0 ) = 1.01 ´ 10 Pa is obeyed.
Pressure inside the drop 40. Work done in splotting a water drop of radius R into n drops of
= Atmospheric pressure + Excess inside the liquid drop equal size = 4pR 2T (n1/3 - 1)
2S = 4 p ´ (10 -3) 2 ´ 72 ´ 10 -3 ´ 10 6 /3 - 1
= p0 +
R = 4 p ´ 10 -6 ´ 72 ´ 10 -3 ´ 99 = 8.95 ´ 10 –5 J
Telegram @unacademyplusdiscounts

476 JEE Main Physics

41. Here, R = 2.8 /2 = 1.4 mm = 0.14 cm 50. Change in surface area = 2 ´ 4 p [(D /2) 2 - (d /2) 2]
4 3 4 = 2 p (D 2 - d 2)
Now, pR = 125 ´ pr3 (equality of volume)
3 3 \Work done = surface tension ´ change in area
R 0.14
or r= = = 0.028 cm = 2pS (D 2 - d 2)
5 5
h
\Change in energy = surface tension ´ increase in area 51. Pressure at half the depth = p0 + dg
2
= 75 ´ (125 ´ 4 ´ 4 pr 2 - 4 pR 2) = 74 erg
Pressure at the bottom = p0 + hdg
42. Given ; u = S ´ 4 pR 2; when droplet is splitted into According to given condition,
1000 droplets each of radius r, then h 2
4 3 4 p0 + dg = ( p0 + hdg )
pR = 1000 ´ pr3 or r = R /10 2 3
3 3 3h
Þ 3 p0 + dg = 2p0 + 2 hdg
\ Surface energy of all droplets 2
= S ´ 1000 ´ 4 pr 2 = S ´ 1000 ´ 4 p (R /10) 2 2 r0 2 ´ 10 5
Þ h= = 3 = 20 m
= 10 ( S 4pR 2) = 10 u dg 10 ´ 10
43. As, work done = surface tension × increase in area 52. Let the lower and upper surface of the wings of the aeroplane
= 72 ´ [10 ´ 0.7 - 10 ´ 0.5] ´ 2 = 288 erg be at the same height h and speeds of air on the upper and
lower surfaces of the wings be v1 and v 2.
44. Increase in surface energy = surface tension ´ increase in
surface area Speed of air on the upper surface of the wing v1 = 70 m/s
æ 4 4 Rö Speed of air on the lower surface of the wings v 2 = 63 m/s
= S (1000 ´ 4 pr 2 - 4 pR 2) ç100 ´ pr3 = R 2 or r = ÷
è 3 3 10 ø Density of the air r = 1.3 kg/m3
æ R2 ö
Area A = 2.5 m 2
= S ´ 4 p ç1000 ´ - R 2÷ = 36 pR 2S
è 100 ø
According to Bernoulli’s theorem,
45. Let R and R ¢ be the radius of bubble of volume V and 2 V 1 1
p1 + rv12 + rgh = p2 + rv 2 + rgh
respectively. Then 2 2
4 3 4 1 2
pR = V and pR ¢3 = 2 V or p2 - P1 = r(v1 - v 22)
3 3 2
R ¢3 \Lifting force acting on the wings,
So, = 2 or R ¢ = (2)1/3R
R3 1
F = ( p2 - p1) ´ A = r(v12 - v 22) ´ A
As W = S ´ ( 4 pR 2) 2 2
é Force ù
and W ¢ = S ´ ( 4 pR ¢2) 2 êëQ Pressure = Area úû
W ¢ R ¢2
= 2 = 2 2/3 = ( 4)1/3 1
W R = ´ 1.3 ´ [( 70) 2 - (63) 2] ´ 2.5
2
or W ¢ = ( 4)1/3 W
1
4 3 4 = ´ 1.3 [4900 - 3969] ´ 2.5
46. As, pR = 1000 ´ pr3 2
3 3 1
= ´ 1.3 ´ 931 ´ 2.5 = 1.51 ´ 10 3 N
Þ R = 10 r 2
Surface energy of small drop E1 = S ´ 4 pr 2
53. With the increase in temperature, the surface tension of liquid
Surface energy of large drop E 2 = S ´ 4 p (10 r) 2 decreases and angle of contact also decreases.
\ E1 /E 2 = 1/100 2S cos q
54. As, h= (height raised = h)
47. As, work done = surface tension ´ increase in surface area r rg
= T (n 4pr 2 - 4 pR 2) hr rg hr
or S= or S µ
2 cos q cos q
48. The meniscus of liquid in a capillary tube will be convex
Sw h cos q2 r1
upwards if angle of contact is obtuse. It is so when one end of \ = 1´ ´
glass capillary tube is immersed in a trough of mercury. S Hg h2 cos q1 r 2

49. As excess pressure, p µ1/ r, therefore, pressure inside C is 10 cos 135° 1


= ´ ´
highest and pressure inside B is lowest. The pressure inside A ( -3.42) cos 0° 13.6
is in between. Therefore, C starts collapsing with volume of A 10 0.707 1
= ´ =
and B increasing. 3.42 13.6 6.5
Telegram @unacademyplusdiscounts

Properties of Liquids 477

55. There will be no over flowing of liquid in a tube of insufficient ppr 4 p (3p + p) (r /2) 4
70. As, V = and V ¢ =
height but there will be adjustment of the radius of curvature 8 hl 8 hl
of meniscus so that hR = a finite constant. V¢ 1
\ = 4 ´ (1 / 2) 4 =
56. When lift is accelerated downwards, the observed weight of V 4
body in a lift decreases. Hence, to counter balance the V
or V¢ =
upward pull due to surface tension on the liquid meniscus, 4
the height through which the liquid rises must increase.
71. Let at a time t dV be the decrease in volume of water in vessel
57. For the given angular velocity of rotation, the centrifugal force in time dt. Therefore rate of decrease of water in vessel = rate
F µ r; Therefore, more liquid will be accumulated near the of water flowing out of narrow tube
wall of tube and the liquid meniscus will become concave dV p ( p1 - p2) r 4
upwards. So, =
dt 8 hl
58. Due to surface tension, water rises in the capillary tube upto a But, p1 = p2 = hrg
height, h with concave meniscus on both the sides. Therefore,
the total height of water column in the capillary tube dV p (hrg ) r 4 ( prgr 4)
\ - = = ´ (h ´ A)
= h + h = 2 h. dt 8 hl 8 hl ´ A

59. Height, h µ1/R where h ´ A = volume of water in vessel at a time t = V

So, h1 /h2 = R2 /R1 = 0.4 /0.2 = 2 æ p rgr 4 ö


\ dV = - ç ÷ ´ V dt = - lV dt
è 8 hlA ø
62. When a highly soluble salt (like sodium chloride) is dissolved
in water, the surface tension of water increases. dV
or = -l dt
63. Surface tension of water decreases with rise in temperature. V
prgr 4
64. Since, the bubbles coalesce in vacuum and there is no change where, = l = constant
in temperature, hence its surface energy does not change. 8 hlA
This means that the surface area remains unchanged. Hence, Integrating it within the limits as time changes 0 to t, volume
4 p a2 + 4 p b 2 = 4 p R 2 changes from V0 to V.
V
or R = a2 + b 2 or log e = -lt
V0
65. The excess of pressure inside the first bubble of radius r1 is or V = V0 e-lt
p1 = 4 S /r1; and in the second bubble of radius r2 is, p2 = 4 S /r2.
where, V0 = initial volume of water in vessel = Ah0
4S 4S 4S
\Excess pressure p = = - Therefore, h ´ A = h0 Ae- lt
r r1 r2 r2
1 r2 - r1 or h = h0 e- lt
Þ =
r r1 r2
Thus, the variation of h and t will be represented by
r1 r2 r1
Þ r= exponential curve as given by (a).
r2 - r1
ppr 4
72. As, V = ,i. e. ,V µ r 4
8 hl
66. As, Tc = T0(1- at), i. e. ,surface tension decreases with increase
in temperature. V ¢ ( a /2) 4 1
Þ = =
67. Volume of liquid flowing per second through each of the two V a4 16
tubes in series will be the same. So, V 16
or V¢ = = = 1 cm3
prR4
pp (R /2) p 1 4 16 16
1
V= = 2 or 1 =
8 hL 8h (L /2) p2 4 ppr 4
73. As, Q=
8 hl
DV 8 pr (r /2) 4 Q
68. The velocity gradient, = = 80 s–1 and Q1 = =
Dr 0.1 8 h (2l) 32
69. Using theorem of continuity, we have 2R 2 (r - r 0) g
74. Terminal velocity, v =
pDp2v p = pDQ2 vQ 9h

æD ö
2
æ 4 ´ 10 -2 ö v 2 (r - r 0) g
v p = ç Q ÷ vQ = ç ÷ ´ vQ = 4 vQ or = = constant
è DP ø è 2 ´ 10 -2 ø R2 9h
Telegram @unacademyplusdiscounts

478 JEE Main Physics

p 83. Work done against air friction is the average gain in kinetic
75. Rate of flow of liquid V =
R energy before attaining the terminal velocity
8 hl
where liquid resistance, R = 1 2
pr 2 0 + mv ter
2 1 2
For another tube liquid resistance W1 = = mv ter
2 4
8 hl 8 hl Work done against air friction after attaining terminal velocity
R¢ = 4
= 4 ×16 = 16 R
r
æ ö pr is
pç ÷
è2ø 1 2
W2 = mv max
For series combination 2
p p p V \ W2 > W1
Vnew = = = =
R + R ¢ R + 16 R 17 R 17
84. Gravitational force remains constant on the falling spherical
2 r 2rg ball. It is represented by straight line P. The viscous force
76. Given, v= …(i) (F = 6 phrv) increases as the velocity increases with time.
9h
Hence, it is represented by curve Q. Net force = gravitational
4 3 4
Mass = pr r = p (2 r)3r1 force – viscous force. As viscous force increases, net force
3 3 decreases and finally becomes zero. Then the body falls with
or r1 = r /8 a constant terminal velocity. It is thus represented by curve R.
Terminal velocity of second ball is
A 2
2 (2 r) 2 (r /8) g v 85. t = [ H1 - H2 ]
v1 = = a g
8h 2
A 2 é Hù
Now, T1 = ê H- ú
77. When a round pebble is dropped from the top of a tall a g ë hû
cylinder, filled with viscous oil the pebble acquires terminal
velocity (i.e., constant velocity) after some time. A T é H ù
and T2 = ê - 0ú
a g ë h û
78. From Stokes’ law, F = 6 phrv
According to problem, T1 = T2
= 6 ´ 3.14 ´ (18 ´ 10 –5) ´ 0.03 ´ 100
H H
= 101.73 ´ 10 –4 dyne \ H- -0 Þ H =2 Þn = 4
N n
4 4
79. As, M = pr3r and 8 M = pR3r, 86. If the liquid is incompressible, then mass of liquid entering
3 3
3 3 through left end should be equal to mass of liquid coming out
So, R = 8r
from the right end.
Þ R = 2r
M = M1 + M2
Now v µ r 2 so,
2 Þ AV1 = AV2 + 1.5 A. v
v1 æ 2 r ö
=ç ÷ =4 A ´ 3 = A ´ 1.5 A. v
v è r ø
v = 1m/s
or v1 = 4 v
80. When terminal velocity v is reaching, then 87. According to equation of continuity, av = constant. As v
increases, a decreases.
4 3
F = 2 ´ 10 -5 v = pr rg
3 88. When air stream is produced in between two suspended
4 22 balls, the pressure there becomes less than the pressure on the
Þ 2 ´ 10 -5v = ´ ´ (1.5 ´ 10 –3)3 ´ 103 ´ 10 opposite faces of the balls. Due to which the balls are pushed
3 7
towards each other.
On solving, v = 7.07 ms–1 » 7 ms–1
89. Along a streamline, the velocity of every fluid particle while
81. In a streamline flow the two streams cannot cross each other. crossing a given position is the same.
82. If v is the terminal velocity, then equation of force, 90. According to equation of continuity
xg - yg = 6 p hrv v1 a2 pd 22 / 4 æd ö
2
æ 3.75 ö 9
( x - y) g a1v1 = a2v 2 or = = = ç 2÷ = ç ÷= .
or v= 2
v 2 a1 pd1 / 4 è d1 ø è 2.50 ø 4
r 6 ph
1 1
( x - y) 91. As, p1 + rv12 = p2 + rv 22 (from Bernoulli’s equation)
or vµ 2 2
r
1
or p1 - p2 = r (v 22 - v12)
2
Telegram @unacademyplusdiscounts

Properties of Liquids 479

1
= ´ 1.3 ´ (120 2 - 90 2) 95. When, air is blown in the horizontal tube, the pressure of air
2 decreases in the tube. Due to which the water will rise above
= 4.095 ´ 10 3 Nm–2 the tube A.
Gross lift on the wing = ( p1 - p2) ´ area 96. If h is the initial height of liquid in drum above the small
= 4.095 ´ 10 3 ´ 10 ´ 2 opening, then velocity of efflux, v = 2 gh. As the water
= 81.9 ´ 10 3 N drains out, h decreases, hence v decreases. This reduces the
rate of drainage of water. Due to which, as the drainage
92. Since, the tubes A and C are connected to a tube of same area continues, a longer time is required to drain out the same
of cross-section, and the liquid flowing there will have same
volume of water. So, clearly t1 < t 2 < 3.
velocity, hence, the height of liquid in A and C will be same.
Since, tube B is connected to a tube of smaller area of 97. As, velocity of efflux, v = 2 gh;
cross-section, therefore the liquid is flowing faster in this tube
Volume of liquid flowing out per sec
and pressure there is less according to Bernoulli’s theorem.
= v ´ A = 2 gh ´ A
93. Fig. (a) is incorrect. From equation of continuity, the speed of
liquid is larger at smaller area. According to Bernoulli’s = 2 ´ 10 ´ 5 ´ (10 ´ 10 -4) = 10 -2 m3 s–1
theorem due to larger speed the pressure will be lower at
smaller area and therefore, height of liquid column will also 98. As, Av = 2 Av ¢ or v ¢ = v /2
be at lesser height, while in Fig. (a) height of liquid column at For a horizontal pipe, according to Bernoulli’s theorem
narrow area in higher. 2
1 2 1 æv ö
p+ rv = p ¢ + r ç ÷
94. Vertical distance covered by water before striking ground 2 2 è2ø
= (H - h). Time taken is, t = 2 (H - g ) × g ; Horizontal velocity
1 2 æ 1ö
of water coming out of hole at P , u = 2 gh or p¢ = p + rv ç1 - ÷
2 è 4ø
\ Horizontal range = ut = 2 gh ´ 2 (H - g )/ g 3 2
Þ p¢ = p + rv
= 2 h (H - h) 8

Round II
1. Here, v1 = 2g (h + x); v 2 = 2gx 2. As, net force = Averge pressure ´ Area - T ´ 2 R
æ hö
= ç p0 + rg ÷ (2 Rh) - T 2R
x è 2ø
v2 = |2p0Rh + Rrgh 2 - 2 RT|
h
3. If V is the volume of the body, its weight = V r1 g . Velocity
v1 gained by body when it falls from a height h1 = 2gh1. The
weight of liquid displaced by the body as body starts
immersing into the liquid = V r 2 g . The net retarding force on
Let, a = area of cross-section of each hole the body when it starts going in the liquid, F = V (r 2 - r1) g
r = density of the liquid F é V (r 2 - r1) g ù
\Retardation, a = =ê ú
The momentum of the liquid flowing out per second through V r1 ë V r1 û
lower hole = mass ´ velocity
The time of immersion of the body is that time in which the
= av1 r ´ v1 = a r v12 velocity of the body becomes zero. Using the relation
The force exerted on the lower hole towards left v = u + at , we have v = 0 , u = 2gh1 ,
= a r v12 v(r 2 - r1) g ær -r ö
a= = -ç 2 1 ÷ g
Similarly, the force exerted on the upper hole towards right V r1 è r1 ø

= a r v 22 ær -r ö
we have 0 = 2gh1 = ç 2 1 ÷ g
è r1 ø
Net force on the tank, F = a r(v12 - v 22)
2h1 æ r1 ö
= a r[2g (h + x) - 2gx] = 2argh or t= ´ç ÷
g è r 2 - r1 ø
Þ F µh
Telegram @unacademyplusdiscounts

480 JEE Main Physics

4. Pressure on left end of horizontal tube, Upthrust of liquid on rod = A l s g acting upwards through the
mid-point of AD.
p1 = p0 + h1 rg
Pressure on right end of horizontal tube, For rotational equilibrium of rod, net torque about point A
should be zero. So,
p2 = p0 + h2 rg
L l
As p1 > p2, so acceleration should be towards right hand side. (LA r g ) cos q = ( lA sg ) cos q
2 2
If A is the area of cross-section of the tube in the horizontal
portion of U-tube, then l2 r
or =
p1 A - p2 A = ( lAr) a L2 s
or (h1 - h2)r g A = lAra 1 s
or sin q =
g (h1 - h2) 2 r
or a= towards right
l 8. When, there is equal level of liquid in two arms of U-tube,
5. In figure, total force on the ring due to surface tension of soap h1 + h1
then height of liquid in each arm of U-tube = . We may
film = (2pb) ´ 2S sin q 2
(h1 + h2) h1 - h2
Mass of air entering per second the bubble consider that a height, h1 - = of the liquid has
2 2
= volume ´ density = ( Av)r = pb 2 ´ vr
been transferred from left arm to right arm of U-tube
Momentum of air entering per sec, æh -h ö
= ç 1 2 ÷ Ar
pb 2v r ´ v = p 2b 2v 2r è 2 ø

The soap bubble will separate from the ring, when force of where, A = area of cross-section of tube andr = density of liquid.
surface tension of ring equal to the force æn -n ö
The decrease in height of this liquid = ç 1 2 ÷
b è 2 ø
or 2 pb ´ 2S ´ = pb 2v 2r
R 2
æh -h ö
4S Loss in potential energy of this liquid = ç 1 2 ÷ Arg
or R= è 2 ø
r v2
The mass of the entire liquid in U-tube
6. Let V0 , Vt = volume of the metal ball at 0° C and t° C = (h1 + h2 + h)rA
respectively, r 0 r 2 = density of alcohol at 0° C and t ° C If this liquid moves with velocity v, then its
respectively. Then 1
KE = (h1 + h2 + h)r Av 2
W1 = W0 - V0 r 0 g 2
W2 = Wt - Vt r t g Using law of conservation of energy, we have
r0 1 æh -h ö
2
where, Vt = V0(1 + g mt ) and r t = (h1 + h2 + h)r Av 2 = ç 1 2 ÷ A r g
(1 + g a l) 2 è 2 ø
Upthrust at t ° C = Vtr t g g
r0 or v = (h1 - h2)
= V0(1 + g mt ) ´ g 2(h1 + h2 + h)
(1 + g at )
(1 + g l) 9. If m is the mass of the stone and V its volume, the weight of the
r = V0g 0 m mg
(1 + g a l) water displaced by it = r ¢Vg = r ¢ ´ ´g =
r h
As g m < g a , hence upthrust at t° C is less than at 0° C. It means
upthrust has been decreased with increase in temperature. where r ¢ is the density of water.
Due to which W2 > W1. Therefore, k = r /r ¢. Thus, the buoyant force acting upwards is
L mg /k whereas the weight mg of the stone acts vertically
7. As, AB = L , AC = ; AD = l (say)
2 downwards. Therefore, the net force in the downward
Let A = area of cross-section of the rod. direction = mg - mg /k
æ 1ö
Weight of the rod = ALr g acting vertically downwards at C. = mg ç1 - ÷
B
è kø
If a is the acceleration of the sinking stone, then
D
æ 1ö
ma = mg ç1 - ÷
è kø
L C
h= σ
2 s æ 1ö
or a = g ç1 - ÷
A θ? è kø
Telegram @unacademyplusdiscounts

Properties of Liquids 481

kx F kx' F'
10. As we know according to equation of continuity, when
cross-section of duct decreases, the velocity of flow of liquid
increases and in accordance with Bernoulli’s theorem, in a
horizontal pipe, the place where speed of liquid is maximum, a
the value of pressure is minimum. Hence the 2 nd graph
correctly represents the variation of pressure.
11. As area of cross-section is uniform therefore according to w w
equation of continuity speed of liquid is same at all points
When the vessel moves downwards with accleration a ( < g )
i. e. , v A = vB
the effective downward acceleration = g - a. Now upthrust is
But during motion of liquid from A to B the potential energy reduced say it becomes F'
decreases. F
where F ¢ = ( g - a)
\According to Bernoulli’s theorem, g
1 2 p In figure, then
v + gh + = constant
2 r w - kx¢ - F ¢ = ma
p p æ g - aö wa
Þ gh + A = B or w - kx¢ - ç ÷F =
r r è g ø g
Þ pB - pA = rgh a wa
or (w - F) - kx¢ + F =
g g
12. Volume of water in the vessel of base area A' and height h is a wa
V = A' h. Averege velocity of out flowing water when height of or kx - kx¢ + F =
g g
water changes from h to 0 is
a
2gh + 0 2gh or x¢ = x + (F - w)
v= = gk
2 2
Hence, the spring length will increase.
\ V = Av t
When vessel is filled to height 4 h, then volume in vessel 15. Let A be the circular area over which the liquid wets the plate
and d be the distance between two plates. Mass of liquid
2gh
= 4V = 4Avt = 4A ´t drop, m = Adr. If S is the force of surface tension of water,
2 then excess of pressure inside the liquid film in excess of
If t is the time taken for the out flowing liquid and v1 is the atmospheric pressure is given by
averege velocity of out flowing liquid, then
4 V = Av1 t1 d
4V 4 A 2gh ´ t ´ 2
or t1 = = = 2t
Av1 2 ´ A ´ 2g ´ 4h S S 2S
p= = =
r d /2 d
13. In time Dt , momentum of water entering the hydrant
Force of attraction between the plates,
p1 = (rLDt )v$j 2S é Fù
F= A êë\ p = A úû
Momentum of water while leaving the hydrant in time Dt is d
p = (rLDt )v( - $i) 2S 2 Sm
2 F= ´ Ard =
rd 2 rd 2
Change in momentum in time Dt is –6
2 ´ 0.07 ´ (80 ´ 10 )
D p = p 2 - p1 = r L t v( - $i - $j) Þ = 0.28 N
10 3 ´ ( 4 ´ 10 -8)
| D p| = r L D t v ( -1) 2 + ( -1) 2
16. Let V be the volume of wooden ball. The mass of ball is
= 2 r LD t v m = V r.
| Dp | Upward acceleration,
Force exerted by water, F = = 2rLv upward thrust - weight of ball
Dt a=
mass of ball
14. Let k be the spring constant of spring and its gets compressed V r 0 g - Vrg (r 0 - r) g
= =
by length x in equilibrium position. Let m be the mass of the Vr r
block and F be the upward thrust of water on block. When the If v ¢ is the velocity of ball on reaching the surface after being
block is at rest, released at depth h is
w = kx + F 1/ 2
é æ p -rö ù
or w - F = kx …(i) v = 2as = ê2ç 0 ÷ ghú
ë è r ø û
Telegram @unacademyplusdiscounts

482 JEE Main Physics

If h' is the vertical distance reached by ball above the surface Also, the total mass of alloy
of water, then m1 + m2 = 16.8 g
v 2 2(r 0 - r) 1
h' = = gh ´ Solving Eqs. (i) and (ii), we get
2g r 2g
m1 = 9.345 g and m2 = 7.455 g
ær -rö æ r0 ö 4S
=ç 0 ÷h = ç - 1÷h 21. Excess pressure inside the soap bubble = . So, the pressure
è r ø èr ø
r
17. The velocity of efflux = 2gh 4S
inside the soap bubble = patm+
r
The rate of flow of liquid out of hole = Av
From ideal gas equation, pV = nRT
= pr 2 2gh
pAVA nA
By using equation of continuity =
pBVB nB
( Av) container = ( Av) hole æ 4Sö 4
ç8 + ÷ 3
p
D2
v = pr 2 2gh è rA ø 3 prA nA
4 Þ = …(i)
æ 4 S ö 4 pr 2 nB
4r 2 ç8 + ÷ 3 B
v= 2gh è rB ø
D2
Substituting, we get, S = 0.04 N/m, rA = 2 cm, rB = 4 cm in
4r 2 Eq. (i)
\Speed with which water level falls = 2gh
D2 nA 1
=
18. As the water tries to enter the hole, it forms a liquid surface nB 6
through the hole with its concave surface downward. Due to nB
\ =6
which it can withstand the pressure of the liquid upto which nA
the canister is lowered
2T 2s
22. When jar is placed in vacuum, the liquid level rises up to the
\In equilibrium, = hr g Þ h = top of jar. The force exerted by liquid on the base of jar = force
r rrg due to vertical column of liquid of height ( a + b + c) + vertical
Putting the given values, we get downward.
2 ´ 73.5
r= = 0.00375 cm a
40 ´ 1 ´ 980
b
= 0.0375 mm 30°
60°
19. Let Vc is the volume of cavity and V is the actual volume of
gold piece [excluding volume of cavity] F c
50
\ V= = 2.6 cm3
19.3
Now, loss in wt. of gold in water = Thrust due to water R
Þ 50 g - 45 g = [V + Vc ] rw g Component of thrust F acting on the portion BC of jar
Þ 5 = (2.6 + Vc ) ´ 1 = ( a + b + c) rg ´ pR 2 + F sin 60°
Þ Vc = 2.4 cm3 = greater than ( a + b + c) r g ´ pR 2
20. Let m1g and m2g be the mass of Cu and Zn respectively in 23. Let V be the volume of the brass block weight of brass block
alloy, = V rBg - V rLg . If A is the area of cross-section of steel wire,
m then
\ Volume of Cu = 1 cc
8.9 V rBg L (V rBg - V rLg ) L
Y= ´ = ´
m A l l¢
and Volume of Zn = 2 cc
7.1 l rB
or =
ém m ù l¢ rB - rL
\Total volume of alloy = ê 1 + 2 ú cc
ë 8.9 7.1û 24. Let p0 = atmospheric pressure. Then,
Now, loss of wt. in water = thrust due to water
p1V1 = p2V2
ém m ù V
Þ (16.8 - 14.7) g = ê 1 + 2 ú ´ 1g or p2 = p1 1
ë 8.9 7.1û V2
m m æ 40 ö 2
Þ 2.1 = 1 + 2 ...(i) or p2 = p0 ç ÷ = p0
8.9 71 . è 60 ø 3
Telegram @unacademyplusdiscounts

Properties of Liquids 483

Now, p2 + (20 cm of Hg) = p0 32. Given, length of the tube ( l) = 1.5 m


2
or p0 + (20 cm of Hg) = p0 . cm = 1 ´ 10 -2 m
Radius of the tube (r) = 10
3
p0 Mass of glycerine flowing per second = 4 ´ 10 -3 kg/s
or = 20 cm of Hg
3 Density of glycerine (r) = 1.3 ´ 10 3 kg/m3
p0 = 60 cm of Hg
Viscosity of glycerine (h) = 0.83 Pa-s
m
Volume of glycerine flowing per second (V ) =
40 cm r
p1 60 cm
é Mass ù
p2 êëQ Density = Volume úû

4 ´ 10 -3 3 4
20 cm = 3
m /s = ´ 10 -6 m3 /s
1.3 ´ 10 1.3
(a) (b)
According to Poiseuille’s formula, the rate of flow of liquid
through a tube
26. The rate of flow of liquid (V ) through capillary tube is p pr 4
2 V=
p pr æ pr 4 ö p pressure difference 8 hr
V= = pç ÷= =
8hhl è 8hl ø R resistance where, p is the pressure difference between the two ends of
8hhl the tube.
where, T= 8hrV
pr 4 or p=
When two tubes are in series, pr 4
. ´ 4 ´ 10 -6
8 ´ 0.83 ´ 15
total resistance, R = R1 + R2 = = 975.37 Pa
p . ´ (1 ´ 10 -2) 2 ´ 1.3
314
Rate of flow of liquid, V ' =
R1 + R2 = 9.75 ´ 10 2 Pa
-1 4S 4S 4S
p p p é l1 l2 ù 33. Excess pressure - =
= = ê + ú r1 r2 r
é
8 h l1 l2 ù 8 h ë r14 r24 û
ê + ú 1 1 1 1 1 1
p ë r 4 r24 û or = - = - = or r = 20 cm
r r1 r2 4 5 20
27. Excess of pressure inside the bubble, p = 4S / r. So, smaller is
the radius r, the larger is the excess of pressure p. It means, the
34. Given, surface tension of soap solution ( S) = 2.5 ´ 10 -2 N/m
pressure of air is more in bubble A to bubble B. . ´ 10 3 kg/m3
Density of soap solution (r) = 12
28. Let A be the area of cross-section of through and r be the Radius of soap bubble (r) = 5.00 mm
density of mercury. = 5.0 ´ 10 -3 m
Initial mass of mercury in trough = A ´ 3.6 ´ r . ´ 10 5 Pa
Atmospheric pressure ( p0) = 101
Final mass of mercury in trough = Ah' r = ( A ´ 3.6 ´ r) ´ 2 4S
Excess pressure inside the soap bubble =
or h ¢ = 7.2 cm r
4 ´ 2.5 ´ 10 -2
29. The force of surface tension pulls the plates towards each = = 20 Pa
5.0 ´ 10 -3
other.
2S 2 ´ 2.5 ´ 10 -2
30. In level flight of aeroplane, mg = pA Excess pressure inside the air bubble = =
R 5.0 ´ 10 -3
mg 3 ´ 10 4 ´ 10 = 10 Pa
or = = = 2.5 kPa
A 120 Pa \ Pressure inside the air bubble = Atmospheric pressure +
2S Pressure due to 40 cm of soap solution column +
31. As, hr g = Excess pressure inside the bubble
r
2S . ´ 10 5) + (0.40 ´ 12
= (101 . ´ 10 3 ´ 9.8) + 10
or h=
rrg . ´ 10 5) + 4.704 ´ 10 3 + 10
= (101
2 ´ 75 ´ 10 -3 . ´ 10 5 + 0.04704 ´ 105 + 0.00010 ´ 105
= 101
= = 0.03 m = 3 cm
æ1 -3 ö 3
= 105714
. ´ 10 5 Pa
ç ´ 10 ÷ ´ 10 ´ 10
è2 ø . ´ 10 5 Pa
= 106
Telegram @unacademyplusdiscounts

484 JEE Main Physics

2r 2(r - r 0) g 42. The angle of contact at the free liquid surface inside the
35. Terminal velocity, v =
9h capillary tube will change in such a way so that vertical
2. component of the surface tension forces just balance the
i. e. , v µr
weight of liquid column.
v1 r12
Þ = 43. An air bubble moves up from the bottom of take due to
v r2
2
upward thrust of water acting on bubble which is greater than
æ 1ö the weight of air bubble. So, upward acceleration is
= 20 ç ÷ = 5 cms-1
è2ø maximum. As bubbles moves a viscous force act on it which
increase with the increases in speed and finally a stage comes
36. Change in surface energy = 2 ´10 -4 J when upward thrust becomes equal to weight of bubble and
As, DA = 10 ´ 6 - 8 ´ 3.75 = 30 cm2 viscous force. Then the bubble moves with constant velocity
and zero acceleration.
= 30 ´ 10 -4 m2
44. When a coin placed on the top of a wooden box floating in
\ Work done, W = T ´ 2 ´ (change in area)
water falls in water, upthrust on the block decreases. Due to it,
Now, change in surface energy = Work done
l decreases as well as h decreases.
Þ 2 ´ 10 -4 = T ´ 2 ´ 30 ´ 10 -4
or T = 3.3 ´ 10 -2 Nm-1 45. The effective weight of ball in liquid w3 becomes less then w1
due to buoyancy of liquid. As, the ball immersed in liquid has
2( p1 - p2) some effective weight acting vertically downwards, so,
37. v = a1a2
r( a12 - a22) w4 > w2.
2( p1 - p2) 46. Pressure energy per unit volume of the dam is equal to
= pr12 ´ pr22
r[( pr12) 2 - ( pr22) 2] hydrostatic pressure (i. e. , pressure due to a column of
liquid p) = hrg
2( p1 - p2)
= pr12r22 47. Let, H be the height above O at which the total force F would
r(r14 - r24)
have to act to produce the given torque. Then
22 2 ´ 10 H ´F = t
= ´ (0.1) 2 ´ (0.04) 2
7 . ´ 10 3)[(0.1) 4 - (0.04) 4 ]
(125 t
or H=
= 6.4 ´ 10 -4m3 s-1 F
rglh3 / 6 h
38. As, work done = surface tension ´ increase in area H= 2
=
(r glh / 2) 3
Þ W = surface tension ´ [0.10 ´ 0.006 - 0.10 ´ 0.005] ´ 2
= 7.2 ´ 10 -2 ´ 0.10 ´ 0.001 ´ 2 48. Consider a strip of dam of thickness, dy at a depth, y as shown
in figure. Pressure at depth, y is
= 1.44 ´ 10 -5 J
p = r gy
V 12 ´ 10 -6
39. As, v1 = = = 2 ms-1 = 200 cms-1 The force against the shaded strip in the figure is
A1 6 ´ 10 -6
dF = p ´ l dy = r gyl dy
V 12 ´ 10 -6 h
and v2 = = = 4 ms-1 = 400 cms-1 h æy2ö 1
A2 3´ 10 -6 Total force, F = ò r gy l dy = rgl ç ÷ = r glh 2
0 è 2 ø0 2
r 2 2
Now, pA - pB = rg (h2 - h1) + (v 2 - v1 )
2 49. Pressure due ot water at the bottom end of vertical face = hrg
1 Pressure due to water at the top end of vertical face = 0
= 1 ´ 1000(100) + (16 ´ 10 4 - 4 ´ 10 4)
2 \ Average pressure on the vertical face of dam
= 10 5 + 6 ´ 10 4 = 1.6 ´ 10 5 dyne cm-2 hr g + 0 1
= = hr g
2 2
40. Let x be the portion of exposed height of the body of length l,
area of cross-section A. As, the body is floating, so 50. Torque of the force dF about an axis through O is
A l r g = A( l - x)3 rg dt = dF ´ (h - y) = (rgyldy) ´ (h - y)
or l = 3l - 3x = rgyl (h - y) dy
or x = 2l/3 Total torque about the point O is
h
x 2 t = ò rgly (h - y) dy
or = 0
l 3 h
é hy 2 y3 ù rglh3
41. Streamline flow more likely for liquids with high viscosity and = r gl ê - ú =
low density. ë 2 3 û0 6
Telegram @unacademyplusdiscounts

Properties of Liquids 485

51. If, m is the mass of the aeroplane, then, mg = 32500 62. Let l = side of the tube
32500 32500 x = side of cube immersed in water,
or m= = = 3250 kg
g 10 l - x = side of cube immersed in liquid.
52. If v1 and v 2 are the speeds of air on the lower and upper surface According to law of floatation,
of the wings of aeroplane and p1, p2 are the pressures there, l3 ´ 0.9 ´ 10 3 ´ g = ( l 2 ´ x) ´ 1000 g + l 2 ( l - x) ´ 0.7 ´ 10 3 g
then l ´ 0.9 = x + ( l - x) ´ 0.7
Assume difference, x 2
or 0.3 = 0.2 l or =
1 l 3
p1 - p2 = r (v 22 - v12)
2
63. Let V be volume of body and r its density, then by law of
æv + v ö floatation in water
or Dp = r ç 2 1 ÷ (v 2 - v1) = rv av (v 2 - v1)
è 2 ø 2
-1 -1
Vrg = V ´ rw g …(i)
Here, v av = 360 kmh = 100 ms 3
2
v 2 - v1 Dp mg / A 3250 ´ 10 / 50 (\ V is immersed in water of density r w )
Þ = 2 = 2
= 3
v av rv av rv av 1 ´ (100) 2
Similarly, in a liquid.
= 0.65 = 6.5% 1
Vrg = Vr l g …(ii)
53. Here, A = 25 ´ 2 = 50 m2 4
From Eqs.(i) and (ii), we have
v1 = 216 kmh -1 = 60 ms-1
2 1
Þ Vrw g = Vr l g
v 2 = 252 kmh -1 = 70 ms-1 3 4
Pressure difference on each wing of aeroplane rl æ 2 ö æ 4 ö
Þ = ç ÷ ´ç ÷
1 rw è 3 ø è 1 ø
Dp = p1 - p2 = r (v 22 - v12)
2 rl 8
=
1 rw 3
= ´ 1(70 - 60 2) = 650 Nm-2
2
2 8 8
\ r l = rw = g /cc (Q rw = 1g / cc)
54. Percentage of velocity difference on the upper and lower 3 3
surface of the wings of aeroplane is 64. According to the Bernoulli’s theorem, the total energy
v -v 70 - 60 10 (pressure energy, potential energy and kinetic energy) of an
= 2 1= = = 0.154 = 15.4%
v av (70 + 60) / 2 15 incompressible and non-viscous fluid in steady flow through a
pipe remains constant throughout the flow
55. Upward force on the aeroplane = Dp ´ A 1
i. e. , r + rgh + rv 2 = constant
= 650 ´ 50 = 32500 N 2
56. It can be shown that R ´ h = constant, where R is radius of So, it is clear that Bernoulli's theorem is a consequence of
curvature of the meniscus of liquid in the tube. When height the law of conservation of energy.
of tube is less, the meniscus becomes flat i. e. , R = ¥ . That is 65. As terminal velocity,
why liquid does not overflow. 2 (r - s ) 2
v= r g
59. The moment when actual velocity of flow of liquid exceeds 9 h
critical velocity, the flow becomes turbulent. Hence for the i. e. , v µ r2
flow to be streamline, the limiting value of critical velocity 2
v1 æ r1 ö
should be as large as possible. Þ =ç ÷
v 2 è r2 ø
60. The shape of liquid drop is spherical due to surface tension of 2
liquid. 9 æ r1 ö
or =ç ÷
4 è r2 ø
61. Upthrust is independent of all factors of the body such as its r1 3
mass, size, density etc, except the volume of the body inside or =
r2 2
the fluid. Fraction of volume immersed in the liquid
ærö 4 3
Vin = ç ÷ V i.e., it depends upon the densities of the block pr 3
ès ø (Volume)1 3 1 æ r1 ö
= =ç ÷
(Volume) 2 4 pr3 è r2 ø
and liquid. So, there will be no change in it if system moves 2
upward or downward with constant velocity or some 3
3
acceleration. Therefore, the upthrust on the body due to æ3ö 27
ç ÷ =
liquid is equal to the weight of the body in air. è2ø 8
Telegram @unacademyplusdiscounts

486 JEE Main Physics

66. As, work done = surface tension ´ change in area 72. When falling drop attains the terminal velocity v, then the
= 0.03 ´ 2 ´ ( 40 ´ 10 -4) J acceleration of the drop is zero. In this situation
FV FB
= 2.4 ´ 10 -4 J mg = FB + FV
or Vr1 g = Vr 2g + kv 2
67. Density of water, rw = 1gcc-1
v
Density of liquid r1 = specific gravity of liquid ´ density of or kv 2 = V g (r1 - r 2)
water = 1.5 ´ 1 = 1.5 gcc-1
Vg (r1 - r 2)
or v=
If V is the volume of the body, then weight of body in water k
= weight of body air- upthrust of liquid mg
1 q
73. Potential on bubble, V=
= 50 g - Vr1 g = (50 - 10 ´ 1.5) g = 35g 4pe 0 r
68. When a piece of wood is floating in water, then the weight of 1 V r
Þ Vµ \ 1 = 2
wood is balanced by the upthrust of water, i.e., weight of r V2 r1
wood is equal to weight of the water displaced by the 16 2
immersed part of the body. Þ = or V2 = 8 V
V2 1
When temperature of water is raised, the wood will remain
float in water. Due to it, the apparent weight of wood will 74. As, surface energy = surface tension ´ surface area
remain same. Þ E = S ´ 2A
69. A and C be the initial levels of Hg in limb I and limb II of æ Aö
\New surface energy, E1 = S ´ 2 ç ÷ = S ´ A
U-tube when mercury is in equilibrium state. Let, h be the è2ø
height of water column added in limb I of U-tube. Due to it, Now % decrease in surface energy,
the mercury level gets depressed to level B in limb I and gets E - E1 2SA - SA
raised to level D in limb II, ´ 100 = ´ 100% = 50%
E 2SA
I II
75. Using equation of continuity,
D a1v1 = a2v 2
2 cm æ 0.15 ö
h Þ ( 40 ´ 10 -8) v1 = (8 ´ 10 -4) ´ ç ÷
è 60 ø
A C
1 cm On solving, v1 = 5 ms–1
B E 76. The distance between the two consecutive crests in transverse
wave motion is called wavelength.
The boat bounces up, i.e., it travels from crest to the
consecutive crest along wave motion.
When, AB = 1cm and CD = x (say) As, wavelength = distance between two consecutive crests
Then, 2a ´1 = a ´ x So, l = 100 m
or x = 2 cm Velocity of wave, v = 25 ms–1
Hence, time in one bounce of boat
If point E of limb II, is in level with B of limb I, then
l 100
hr 0 g = (2 + 1) r HG g t= = =4s
v 25
3 ´ 13.6
or h = 3r HG / rw = = 40.8 cm 77. Pressure at the bottom of tank must equal pressure due to
1
water of height, h.
70. Surface tension of soap solution is less than that of water. Let, dw and d o be the densities of water and oil, then the
2S cos q pressure at the bottom of the tank = hwdw g + hod o g
As, h= ,
rrg Let this pressure be equivalent to pressure due to water of
So, h µS height h . Then
Hence, the height of liquid raised in the capillary tube is less hdw g = hw g + hod o g
for soap solution and more for pure water, i. e., option (c) is hd 400 ´ 0.9
h = hw + o o = 100 +
correct. dw 1
71. Since, liquid 1 is over the liquid 2, so r1 < r2. If r3 is greater = 100 + 360 = 460
than r 2 or r3 < r 2, the ball would not have been partially According to Torricelli’s theorem,
inside liquid 2 but would have sunk totally. Therefore, r 2 < r3 v = 2gh = 2 ´ 980 ´ 460
and r1 < r3 Hence, r1 < r3 < r 2. = 920 ´ 980 cms–1
Telegram @unacademyplusdiscounts

Properties of Liquids 487

78. Volume of the body, V = m / D1. 84. From the formula, rise of liquid in capillary tube is
Mass of the liquid displaced by body, 2S cos q
h=
mD2 rgr
m¢ = VD2 =
D1 S cos q S cos q
Þ hµ or h µ
\ Viscous force = effective weight of the body r r

mD2 æ D ö Angle of contact for water and glass


= mg - m¢g = mg - g = mg ç1 - 2 ÷ q1 = 80°
D1 è D1 ø
Angle of contact for mercury and glass
120
79. Volume of the body, V = = 0.2 m3 q2 = 135° (given)
600 h1 S1 cos q1 ´ r 2
Weight of water displaced = 0.2 ´ 10 3 ´ g = 200 kg -wt Hence, =
h2 r1 ´ S 2 cos q2
Mass of water displaced = 20 kg S1 h1 r1 cos q2
or =
Therefore, additional mass which can be added to body, so S 2 h2 r 2 cos q2
that body can just sink 0.1 1 cos135° 1
= ´ ´ =
= 200 –120 = 80 kg 0.324 13.6 cos 8° 65
80. Let V be the total volume of a solid sphere, V1 is the volume of 85. From the equation of continuity, the amount of mass that
the part of the sphere immersed in a liquid of density r1 and V2 flows past any cross-section of a pipe has to be the same as the
is the volume of the part of the sphere immersed in a liquid of amount of mass that flows past any other cross-section,
density r 2 . Then i. e. , m1 = m2
V = V1 + V2 r1A1v1 = r 2A2v 2
As sphere is floating
A1
So, Vrg = V1 r1g + V2 r 2g
(V1 + V2) rg = V1 r1g + V2 r 2g A2
or V1(r - r1) g = V2(r 2 - r) g v1 v2
V1 r 2 - r
or =
V2 r - r1
Given, r1 = r 2
81. Since, the volume of liquid flowing per second at A is equal to A
the volume of liquid flowing per second at B and A2 = 1
2
Þ v ´ p (2R) 2 = v ¢ ´ pR 2
Þ v 2 = 2v1
v¢ = 4 v
86. We know that the force of cohesion (attraction between
82. If the height h is the rise of liquid in capillary tube of radius r, molecules) is maximum in solids.
then expression of height is given by
87. Here, 2TL = mg
2S cos q
h= …(i) mg . ´ 10 -2
15 1.5
rgr Þ T= = = = 0.025 N / m
-2
2L ´ ´2 30 10 600
Since, we dip capillary tube of different radii in water and
water rises to different height in them, then equation becomes V0
88. As, net force, mg + rc ´ Vl ´ g = ´ rc ´ g
2S cos q 2
hr = = constant V m
rg Þ Vl = 0 -
2 rc rc
83. Pressure on the bottom of the lake is given by v
V
So, Vl < 0
p1 = 90 + 10 = 100 m of water 2
Let its volume be V1 89. Here, W = T ´ DA
Let its surface, the pressure will be, p2 = 10 m of water
(Q work = surface tension ´ area)
Now, on reaching the surface its volume be V2
= 0.03 [2 ´ 4p ´ (5 2 - 3 2) 10 -4 ]
Now according to Boyle's law
= 24p (16) ´ 10 -6
p1V1 = p2V2
= 0.384 p ´ 10 -3 J = 0.4 p mJ
\ 100 ´ V = 10 ´ V2
100 ´ V Surface energy E
V2 = = 10 V 90. As,surface tension = or T =
10 Area A
Telegram @unacademyplusdiscounts

488 JEE Main Physics

91. When a capillary tube is broken at height of 6 cm, the height


of water column will be 6 cm
2S cos q h A θ
As h= = = constant
rrg cos q D
R
8 6 θ
\ = (from question) h1 θ B
cos 0° cos q
6 cos 0° 3 E
h3
Þ cos q = = h2 C
8 4 P
3
q = cos-1 Þ dgh1 + rgh2 = rgh3
4
or dgR (sin q + cos q) + rgR (1 - cos q) = rg (1 - sin q) R
92. Given, diameter = 8 ´10 -3 m or d (sin q + cos q) + r (1 - cos q) = r (1 - sin q)
and v1 = 0.4 m/ s
or (r + d) sin q = (r - d) cos q
\ v 2 = v12 + 2gh ær - dö
or tan q = ç ÷
2
= (0.4) + 2 ´ 10 ´ 0.2 = 2 m/ s è r + dø

Now, A1v1 = A2v 2 ær - dö


or q = tan -1ç ÷
-3 ö 2 è r + dø
æ 8 ´ 10 æd2ö
Þ pç ÷ ´ 0.4 = p ´ ç ÷ ´ 2
è 2 ø è 4ø 2 r 2g
94. As, terminal speed, v = (r - s )
9 h
Þ d = 3.6 ´ 10 -3 m
where, r = density of the substance
93. Vertical height of the liquid in portion AC,
s = density of the liquid
h1 = DO + OE
If h and r are constants then, v µ (r - s )
= R sin q + R cos q
v 2 r Ag - r Li
= R (sin q + cos q) Þ =
v1 r Au - r Li
Vertical height of the liquid in portion CP,
v 2 10.5 - 1.5
h2 = R - R cos q = R (1 - cos q) or =
v1 19.5 - 1.5
Vertical height of the liquid in portion PB,
9 1
h3 = R - R sin q = R (1 - sin q) = =
18 2
In equilibrium, the pressure due to liquid on the both sides v 0.2
must be equal at the lowest point P or v2 = 1 = = 0.1m/s
2 2
Telegram @unacademyplusdiscounts

13 Heat and
Kinetic Theory of Gases
JEE Main MILESTONE
< Heat < Ideal Gas or Perfect Gas
< Thermometry < Equation of State of a Perfect Gas
< Temperature Scales < Ideal Gas Law with Constraints
< Thermal Expansion < Kinetic Theory of Gases
< Calorimetry < Degree of Freedom
< Specific Heat < Internal Energy of an Ideal Gas
< Phase Changes and Latent Heat < Law of Equipartition of Energy
< Heat Transfer < Mean Free Path
< Heat Transfer Through Radiation < Avogadro’s Number

13.1 Heat
Heat is a form of energy which produces in as the sensation of warmth. The SI unit
of heat joule (J). It is popularly measured in calorie.

13.2 Thermometry Heat is a form of energy and can


The branch dealing with measurement of temperature is called thermometry and perform a work. After getting the
the devices used to measure temperature are called thermometers. heat, a closed container filled by
some gas gets energised, i . e ., its
To establish the measurement of temperature that property of a substance is used
enternal energy is increased and
which changes linearly with temperature. For example, at changing temperature,
then it can perform some work.
change in pressure of a gas at constant volume, change in electric resistance of a
The theory devlop a model of
metallic wire etc. Such property of a substance is called thermometric property. Let
molecular behaviour that should
thermometric properties at temperatures 0°C (ice point), 100°C (steam point) and
result in the observed behaviour of
t°C (unknown temperature) are X 0, X100, and X t respectively. Then
an ideal gas is studied under
X t - X 0 X100 - X 0 kinetic theory of gases.
=
t 100
Xt - X0 t
or =
X100 - X 0 100
æ X - X0 ö
Thus, t=ç t ÷ ´ 100° C
è X100 - X 0 ø
Telegram @unacademyplusdiscounts

490 JEE Main Physics

Triple Point Table 13.1 Ranges of Different Thermometers


Thermometer Lower Limit Upper Limit
Triple point is a state in which ice, water and water vapour
can stay together in equilibrium. It refers to temperature at Mercury thermometer – 30°C 300°C
the equilibrium. Gas thermometer – 268°C 1500°C
\The temperature scale by the equation Platinum resistance thermometer – 200°C 1200°C
p Thermo-couple thermometer – 200°C 1600°C
T = lim ´ 273.16 K
ptr ® 0 ptr Radiation thermometer 800°C – 6000ºC

where, p = pressure
Saturated and Unsaturated Vapour Pressure
ptr = pressure at equilibrium and K stands for kelvin
scale of temperature. When a space actually, contains the maximum possible
amount of vapour, the vapour is called saturated. If the
Now-a-days in modern technology instead of two fixed amount is less than the maximum possible, the vapour is
points only one reference point is chosen, which is triple called unsaturated.
point of water (temperature at which ice, water and water
vapour co-exist) and has been assigned arbitrarily a value Dew Point
273.16 K. So, if values of thermometric properties at 0 K,
The temperature at which the saturation vapour is equal
273.16 K and T K are 0, X tr and X respectively, then
to the present vapour pressure is called dew point.
T X
= If the temperature is decreased below the dew point,
Ttr X tr
source of the vapour condenses.
X é X ù
or T = Ttr = ê ´ 273.16 ú K
X tr X
ë tr û Humidity and Relative Humidity
The amount of water vapour present in a unit volume of
Different Thermometers air is called the absolute humidity of air. It is denoted by
gm-3. The ratio of the amount of water vapour required to
Constant-Volume Gas Thermometer saturated the volume at same temperature is called
If p0, p100, ptr and pt are the pressures of gas at relative humidity. Relative humidity is generally
temperatures 0°C, 100°C, triple point of water and expressed as a percentage.
unknown temperature (t°C) respectively keeping the
volume constant, then
æ p - p0 ö æ pö 13.3 Temperature Scales
t=ç ´ 100÷° C or T = ç273.16 ÷K
è p100 - p0 ø è ptr ø The centigrade (°C), Fahrenheit (°F), Kelvin (K), Reaumer (R)
and Rankine (Ra) are commonly used temperature scales.
Platinum Resistance Thermometer
If R0, R100, Rtr and Rt are the resistances of a platinum wire
Table 13.2 Various Temperature Scales
with LFP and UFP
at temperatures 0°C, 100°C, triple point of water and
unknown temperature (t°C) respectively, then Temperature
°C °F K °R °Ra
Another
æ R - T0 ö Scale Scale
t=ç t ´ 100÷°C
è R100 - R0 ø LFP 0 32 273 0 492 TL
æR ö æR ö UFP 100 212 373 80 672 TU
or T = ç T ´ Ttr ÷ K = ç t ´ 273.16 ÷ K
è Rtr ø è Rtr ø Number of 100 180 100 80 180 TU - TL
divisions (N)
Mercury Thermometer All these temperatures are related to each other by the
In this thermometer, the length of a mercury column from following relationship
some fixed point is taken as thermometric property. Thus, C -0 F - 32
=
æ l -l ö 100 - 0 212 - 32
t = ç t 0 ÷ ´ 100°C K - 273 R-0
è l100 - l0 ø = =
373 - 273 80 - 0
æl ö
or T = ç t ´ 273.16 ÷ K Ra - 492 T - TL
è ltr ø = =
672 - 492 TU - T L
Telegram @unacademyplusdiscounts

Heat and Kinetic Theory of Gases 491

C - 0 F - 32 and the upper fixed points. If a temperature C on Celsius scale


or =
100 180 corresponds to temperature q on the scale of faulty thermometer,
K - 273 R - 0 then
= = C - 0 q - q0
100 80 = …(i)
Ra - 492 T - T L 100 n
= =
180 TU - T L Here, q0 = 5°; n = 95 - 5 = 90 and q = 59°
Therefore, the Eq. (i) becomes
In general, whenever we are to go from any known scale to
C - 0 59 - 5
any unknown scale, then we follow the equation =
100 95 - 5
æ Temperature onö æ LFP for ö 54
ç ÷ -ç ÷ or C= ´ 100
è known scale ø èknown scaleø 90
(UFP – LFP)known scale or C = 60° C
æ Temperature onö æ LFP for ö
ç ÷ -ç ÷
è unknown scale ø èunknown scaleø
= Check Point 1
(UFP - LFP )unknown scale
1. Tea gets cooled, when sugar is added to it. Why?
Sample Problem 1 The resistance of a platinum-resistance 2. In cold countries, juice bottles are placed under water, so as to
thermometer is found to be 11.00 W ,when dipped in a triple avoid freezing. Why?
point cell. When it is dipped in a bath the resistance is found to 3. Two thermometers are constructed in the same way, except
be 28.887 W ? Find the temperature of bath in °C on platinum that one has a spherical bulb and the other a cylindrical bulb.
scale. Which will respond quickly to temperature changes?
(a) 444.17 °C (b) 333.17 °C 4. The coolant in a chemical or a nuclear plant should have high
(c) 450.17 °C (d) 350.17 °C specific heat. Why?

Interpret (a) In terms of triple point of water


é Rù
TK = ê273.16 ú K
R
ë tr û
13.4 Thermal Expansion
28.887
So, TK = 273.16 ´ = 717.32 K
1100
. It has been a general observation that heating a body is
Now TC = TK - 273.15 accompanied by an increase in size of the body. This
phenomenon is called thermal expansion.
TC = 717.32 - 273.15 = 444.17° C
There are three types of thermal expansion 1. Expansion
Sample Problem 2 At what temperature, if any, do the of solids 2. expansion of liquids 3. expansion of gases.
following pairs of readings give the same reading on Fahrenheit
and Kelvin scales? Expansion of Solids
(a) 574.6° (b) – 574.6°
On heating a body, if its length increases we call it linear
(c) 474.6° (d) – 474.6°
expansion, if its surface area increases, we call it
Interpret (a) If the temperature is q at which the reading of two superficial expansion and if its volume increases, we call
q - 32 q - 273.15 it volume expansion or cubical expansion.
scales is same, then = i. e. , q = 574.6°
180 100
Linear expansion (expansion in length of a solid) Consider a
i. e. , reading on Fahrenheit and Kelvin scales coincides at 574.6°. rod of length l1 at a temperature q1. Let it be heated to a
temperature q2 and the increased length of the rod be l2,
Sample Problem 3 A faulty thermometer has its fixed then
points marked as 5° and 95°. The temperature of a body as
measured by the faulty thermometer is 59°. Find the correct l2 = l1 (1 + aq)
temperature of the body on Celsius scale. a = coefficient of linear expansion and q = q2 - q1
(a) 30°C (b) 45°C Superficial expansion (expansion in surface area) If A1 is the
(c) 60°C (d) 75°C
area of solid at q1° C and A2 is the area at q2° C, then
Interpret (c) Let q0 be the lower fixed point of the faulty A2 = A1 (1 + bq)
thermometer and n be the number of divisions between its lower b = coefficient of superficial (areal) expansion and q = q2 - q1
Telegram @unacademyplusdiscounts

492 JEE Main Physics

Volume expansion (expansion in volume) If V1 is the volume All gases have coefficient of volume expansion gV with
of solid at q1° C and V2 is the volume at q2°C, then volume variation given by
V2 = V1 (1 + g q) V = V0 (1 + gV Dq)
g = coefficient of cubical (volume) expansion, and pressure variation given by p = p0 (1 + g pDq)
and q = q2 - q1

Note For isotropic solids : b = 2 a, g = 3 a, i .e ., a : b : g = 1: 2 : 3


Applications of Thermal Expansion
As the temperature increases, density of solid decreases. If d 1 is the 1. If a solid object has a hole in it, what happens to the
density atT1° C,d 2 is the density atT2 °C, then size of the hole, when the temperature of the object
d1 increases ? A common misconception is that, if the
d2 =
(1 + g q) object expands, the hole will shrink because material
expands into the hole. But the truth is that, if the object
where, q = ( q2 - q1)
expands, the hole will expand too, because every
linear dimension of an object changes in the same way
Expansion of Liquids when the temperature changes.
Thermal expansion in liquids is identical to that of volume
expansion in solids and is governed by the relationship a
Ti
which is V = V0 (1 + g Dq), g for liquids is generally higher
than that of solids. b
Since, liquids are always heated in a vessel, so initially on
heating the system (liquid + vessel), the level of liquid in a + ∆a
vessel falls (as vessel expands more since it absorbs heat Ti + ∆T
and liquid expands less) but later on, it starts rising due to
faster expansion of the liquid. Thus, liquids have two b + ∆b
coefficients of volume expansion.
2. Expansion of a bimetallic strip Each substance has its
(i) Coefficient of real expansion g r , which is due to the own characteristic average coefficient of expansion.
actual increase in volume of liquid due to heating. For example, when the temperatures of a brass rod
and a steel rod of equal length are raised by the same
(ii) Coefficient of apparent expansion g a, which is apparent
amount from some common initial value, the brass
increase in the volume of liquid if expansion of vessel rod expands more than the steel rod because brass
containing the liquid is not taken into account. has a greater average coefficient of expansion than
\ g r = g a + gV steel.
Here, g V = expansion of vessel Steel
Generally, with increasing temperature the volume
expansion coefficient of liquids is about ten times greater
than that of solids. Water is an exception to this rule. From
0°C to 4°C water contracts and beyond 4°C it expands.
Brass
Thus, density of water reaches a maximum value of
1000 kgm–3 at 4°C. Room temperature Higher temperature
Such type of bimetallic strip is found in practical
devices such as thermostats to break or make electrical
contact.
Volume

Density

4°C 4°C
Temperature Temperature
(a) (b) Bimetallic strip

Expansion of Gases
On heating gases expand more than solids or liquids and
On 25°C On 30°C
equal volumes of different gases expand equally, when
heated by the same amount.
Telegram @unacademyplusdiscounts

Heat and Kinetic Theory of Gases 493

3. Variation of density with temperature Most l


T = 2p
substances expand when they are heated, i.e., volume g
of a given mass of a substance increases on heating, so or T µ l
1
the density should decrease æç as r µ ö÷. Let us see how
è Vø As the temperature is increased length of the
the density (r ) varies with increase in temperature. pendulum and hence, time period gets increased or a
m pendulum clock becomes slow and it loses the time.
r=
V T¢ l¢ l + Dl
= =
1 T l l
or rµ (for a given mass)
V Here, we put Dl = l aDq in place of l aDT , so as to avoid
r¢ V V the confusion with change in time period. Thus,
\ = =
r V ¢ V + DV T¢ l + laDq
= = (1 + aDq)1/ 2
V 1 T l
= =
V + gVDT 1 + gDT 1
or T ¢ » T æç1 + aDqö÷
r è 2 ø
\ r¢ =
1 + gDT 1
or DT = T ¢ – T = TaDq
2
This expression can also be written as,
r ¢ = r (1 + gDT )–1 Time lost in time t (by a pendulum clock whose actual
time period is T and the changed time period at some
As g is small, (1 + gDT )–1 » 1 – gDT higher temperature is T ¢ ) is
DT ö
\ r ¢ » r (1 – gDT ) Dt = æç ÷t
è T¢ ø
4. Effect of temperature on upthrust When a solid body
is completely immersed in a liquid its apparent weight Similarly, if the temperature is decreased the length
gets decreased due to an upthrust acting on it by the and hence, the time period gets decreased. A
liquid. The apparent weight is given by, pendulum clock in this case runs fast and it gains the
wapp = w – F time.
Here, F = upthrust = VS r L g T¢ l¢ l – laDq
= =
where VS = volume of solid T l l
1
and r L = density of liquid » 1 - aDq
2
Now, as the temperature is increased VS increases
1
while r L decreases. So, F may increase or decrease (or or æ
T ¢ = T ç1 – aDqö÷
è 2 ø
may remain constant also) depending upon the
condition that which factor dominates on the other. 1
DT = T – T ¢ = T a Dq
We can write 2
F µ VS r L and time gained in time t is the same, i.e.,
DT ö
or
F ¢ VS¢ r L¢
= × Dt = æç ÷t
F VS r L è T¢ ø

(VS + DVS ) æ 1 ö 6. At some higher temperature a scale will expand and


= ×ç ÷ scale reading will be lesser than true values, so that
VS è 1 + g L DT ø
true value = scale reading (1 + aDT )
æ V + g S VS DT ö æ 1 ö
=ç S ÷ ç ÷ Here, DT is the temperature difference.
è VS ø è 1 + g L D T ø
However, at lower temperature scale reading will be
æ 1 + g S DT ö
or F¢ = F ç ÷ more or true value will be less.
è 1 + g L DT ø
7. When a rod whose ends are rigidly fixed such as to
Now, if g S > g L, F¢ > F or ¢ < wapp
wapp and prevent from expansion or contraction undergoes a
vice-versa. change in temperature, thermal stresses are
And if g S = g L, F ¢ = F or ¢ = wapp
wapp developed in the rod. This is because, if the
temperature is increased, the rod has a tendency to
5. Effect of temperature on the time period of a expand but since, it is fixed at two ends, the rod exerts
pendulum The time period of a simple pendulum is a force on supports.
given by
Telegram @unacademyplusdiscounts

494 JEE Main Physics

l, α Let q be the temperature at which the clock is correct.


Time lost per day = 1/2 a(rise in temperature ) ´ 86400
Þ 12 = 1/2 a ( 40 - q) ´ 86400 …(i)
Dl
Thermal strain = = a × DT Time gained per day
l
= 1/2 a (drop in temperature) ´ 86400
So, thermal stress = (g ) (thermal strain)
1
= YaDT 4 = a ( q - 20) ´ 86400 …(ii)
2
or force on supports F = A (stress) = YA aDT
Here, Y = Young’s modulus of elasticity of the rod.
On adding Eqs. (i) and (ii), we get
F = YAaDT
32 = 86400 a ( 40 - 20)
Sample Problem 4 A surveyor’s 30 m steel tape is correct Þ a = 1.85 ´ 10 –5° C–1
at a temperature of 20°C. The distance between two points, as On dividing Eqs. (i) and (ii), we get
measured by this tape on a day when the temperature is 35°C, 12 ( q - 20) = 4 ( 40 - q)
is 26 m. What is the true distance between the points? Þ q = 25° C
(a steel = 1.2 ´ 10 –5 / ° C ) Clock shows correct time at 25°C
(a) 26.00476 m (b) 27.00468 m
(c) 25.6658 m (d) None of these
Interpret (a) Let temperature above the correct temperature be
13.5 Calorimetry
Þ q = 35 - 20 = 15° C (Using the relation) Calorimetry means measurement of heat. When a body at
Correct length = measured length (1+ aq) higher temperature is brought in contact with another
True distance between the points = 26 (1 + 1.2 ´ 10–5 ´ 1.5) body at lower temperature, the heat lost by the hot body is
Þ True distance = 26.00476 m equal to the heat gained by the colder body and provided
no heat is allowed to escape to the surrounding. A device
Sample Problem 5 A steel ring of 3.000 inch inside in which heat measurement can be made is called a
diameter at 20°C is to be heated and slipped over a brass shaft calorimeter.
measuring 3.002 inch in diameter at 20°C. To what 1 calorie is the quantity of heat required to raise the
temperature should the ring be heated? ( asteel = 1.1 ´ 10–5 °C-1) temperature of 1 g of water by 1°C.
(a) 70.6°C (b) 75.6°C
(c) 80.6°C (d) 78.6°C Water Equivalent
Interpret (b) Let qbe the temperature to which the ring must be It is the quantity of water whose thermal capacity is same
heated. as the heat capacity of the body. It is denoted by W.
Final diameter of ring should be 3.002 inch.
W = ms = Heat capacity of the body
Þ 3.002 = 3 [1 + a ( q - 20)]
3.002 - 3
Þ q= + 20 Principle of Calorimetry
3a
When two bodies at different temperatures are placed in
Þ q = 75.6° C
contact with each other or mixed with each other
(liquid-in-liquid, solid-in-liquid), the heat will pass from
Sample Problem 6 A pendulum clock loses 12 s a day, if
the body at higher temperature to the body at lower
the temperature is 40°C and goes fast by 4 s a day if the
temperature until both bodies reach a common
temperature is 20°C. Find the temperature at which the clock
will show correct time and the coefficient of linear expansion of temperature. This state is called as thermal equilibrium.
the metal of the pendulum clock. At this state,
(a) 120°C, 1.85 ´ 10–5 ºC–1 (b) 28°C, 1.85 ´ 10–6 ºC–1 Heat lost by one body = Heat gained by the other body
(c) 25 °C, 1.85 ´ 10–5 ºC–1 (d) 27°C, 1.85 ´ 10–6 ºC–1
Let two bodies of masses m1 and m2, specific heats s1 and
Interpret (c) A pendulum clock keeps proper time at s 2 and at temperatures q1 and q2 are brought in contact
temperature q1 and if temperature is increased to q2 > ( q1), then due with each other. Assuming q1 > q2, heat will flow from
to linear expansion length of pendulum and hence its time period
body 1 to body 2. If q is the common temperature of two
will increase Fractional change in time period
bodies at the state of thermal equilibrium, then (assuming
DT 1
= a Dq no heat is gained or lost from or to the surroundings)
T 2
Telegram @unacademyplusdiscounts

Heat and Kinetic Theory of Gases 495

Heat lost by body 1 = Heat gained by body 2 measured in terms of calg–1 or kcalkg–1. It is given by
m1s1 (q1 - q) = m2s 2 (q - q2 ) (q2 < q < q1 ) Q = mL, where L is the latent heat.

Latent Heat of Fusion


13.6 Specific Heat It is the quantity of heat required to change the unit mass
of a solid substance to the liquid state at its melting point.
The specific heat (s) of a substance is the quantity of heat in
For ice, latent heat of fusion is 80 calg–1.
calorie required to raise the temperature of 1 g of that
substance by 1°C. Its unit is cal g–1C–1.
Latent Heat of Vaporisation
The heat lost by a body or gained from a body depends
It is the quantity of heat required to convert unit mass of a
upon the difference in the temperature.
liquid to gaseous state at the boiling point of the liquid. For
The heat lost or gained by a body Q = msDq water, latent heat of vaporisation is 540 calg–1.
m = mass of the body, s = specific heat,
Dq = rise or fall in the temperature of body. Sample Problem 7 Calculate the heat of fusion of ice from
–1 –1
the following data for ice at 0ºC added to water. Mass of
Specific heat for ice s ice = 0.5 cal g C calorimeter = 60 g, mass of calorimeter + water = 460 g, mass
for water s water = 1 cal g–1C–1 of calorimeter + water + ice = 618 g, initial temperature of
for steam s steam = 0.47 cal g–1C–1 water = 38°C, temperature of the mixture = 5°C. The specific
heat of calorimeter = 0.10 ca lg -1 °C -1.
Heat Capacity (a) 73.85 calg -1
(b) 78.35 calg -1
The heat capacity of a body is the quantity of heat required
(c) 88.7 calg -1
by the body to raise its temperature by 1°C. It is also
known as thermal capacity. (d) 84.3 calg -1
Heat capacity = ms (mass ´ specific heat) Interpret (b) Mass of water = 460 - 60 = 400 g
Mass of ice = 618 - 460 = 158 g
13.7 Phase Changes and Latent Heat lost by water = Heat gained by ice to melt +
Heat gained by (water + calorimeter) to reach 5ºC
Heat Þ 400 ´ 1 ´ (38 - 5) = 158 ´ L + 158 ´ 1 ´ 5 + 60 ´ 0.1 ´ 5
Normally, matter exists in three states : solid, liquid and (where L is the latent heat of fusion of ice)
gas. The conversion of one of these states of matter to Þ L = 78.35 calg –1
another is called the change of state.
Sample Problem 8 What will be the temperature, when
There are two common changes of states
150 g of ice at 0°C is mixed with 300 g of water at 50°C?
(i) The change of state from solid to liquid is called Specific heat of water = 1ca lg -1° C -1. Latent heat of fusion of ice
melting and from liquid to solid is called fusion. Both = 80 ca lg -1.
the solid and liquid states of the substance coexist in
thermal equilibrium during the change states from (a) 6.0°C (b) 5.6°C
solid to liquid. (c) 6.7°C (d) 17.6°C
(ii) The change of state from liquid to gas (or vapour) is Interpret (c) Let us assume that T > 0° C
called vaporisation. The temperature at which the Heat lost by water = Heat gained by ice to melt
liquid and the vapour states of the substance coexist is
+ heat gained by water formed from ice
called its boiling point. The change of state from solid
state to vapour state without passing through the 300 ´ 1 ´ (50 - T) = 150 ´ 80 + 150 ´ 1 ´ (T - 0)
liquid state is called sublimation. Þ T = 6.7° C
Hence, our assumption that T > 0°C is correct.
Latent Heat
For example water at 1 atm latent heat of fusion is
The latent heat is the amount of heat that has to be 80.0 cal/g. This simply means 80.0 cal of heat are required
supplied to (or taken from) the body during the change of to melt 1.0 g of water or 80.0 cal heat is liberated when 1.0 g
state while temperature remaining constant. It is of water freezes at 0°C. Similarly latent heat of vaporization
for water at 1 atm is 539 cal/g.
Telegram @unacademyplusdiscounts

496 JEE Main Physics

Figure shows how the temperature varies when we add Sample Problem 10 10 g of water at 70°C is mixed with
heat continuously to a specimen of ice with an initial 5 g of water at 30°C. Find the temperature of the mixture in
temperature below 0°C. Suppose we have taken 1 g of ice equilibrium.
at –20° C specific heat of ice is 0.53 cal/g-°C.
Interpret Let t° C be the temperature of the mixture. From
T(°C) energy conservation,
Heat given by 10 g of water
d = Heat taken by 5 g of water
100 e
or m1cwater | Dt1| = m2cwater | Dt 2|
\ (10) (70 – t ) = 5 (t – 30)
0
b \ t = 36.67° C
c

Q (cal) Sample Problem 11 In a container of negligible mass


–20 a Q1 Q2 Q3 Q4
30 g of steam at 100°C is added to 200 g of water that has a
temperature of 40°C. If no heat is lost to the surroundings, what
In the figure is the final temperature of the system? Also, find masses of
a to b Temperature of ice increases until it reaches its water and steam in equilibrium. (Take Lv = 539 cal / g and
melting point 0°C. c water = 1cal / g -° C )
Q1 = mcice [0 – (–20)] Interpret Let Q be the heat required to convert 200 g of water
w= (1) (0.53) (20) = 10.6 cal at 40°C into 100°C, then
b to c Temperature remains constant until all the ice has Q = mcDT
melted. = (200) (1.0) (100 – 40)
Q2 = mLf = (1) (80) = 80 cal = 12,000 cal
c to d Temperature of water again rises until it reaches Now, suppose m0 mass of steam converts into water to liberate this
its boiling point 100°C. much amount of heat, then
Q3 = mcwater [100 – 0] = (1) (1.0) (100) = 100 cal Q 12000
m0 = = = 22.26 g
d to e Temperature is again constant until all the water is L 539
transformed into the vapour phase. Since, it is less than 30 g, the temperature of the mixture is 100°C
Q4 = mLv = (1) (539) = 539 cal Mass of steam in the mixture = 30 – 22.26 = 7.74 g
Thus, the net heat required to convert 1 g of ice at – 20° C and mass of water in the mixture
into steam at 100°C is = 200 + 22.26 = 222.26 g
Q = Q1 + Q2 + Q3 + Q4 = 729.6 cal

Sample Problem 9 How much heat is required to convert 13.8 Heat Transfer
8.0 g of ice at –15° C to steam at 100°C?
There are three different ways in which heat can be
(Given c ice = 0.53 cal/g-°C,
f L cal g
transferred; conduction, convection and radiation.
and c water = 1cal/g-°C)
ice ice water water steam Conduction
–15°C 0°C 0°C 100°C 100°C
Q1 Q2 Q3 Q4 It is a process by which the heat is transferred in solid. In
conduction, molecules vibrate about a fixed location and
transfer the heat by collision.
Interpret
Q1 = mcice (Tf – Ti ) = (8.0) (0.53) [0 – (–15)] = 63.6 cal When a metallic rod is put in a flame, the other end of rod
Q 2 = mLf = (8) (80) = 640 cal will soon be so hot that you cannot hold it by your hands.
Q3 = mcwater (Tf – Ti ) = (8.0) (1.0) [100 – 0] = 800 cal It means heat transfer take place by conduction from hot
Q 4 = mLv = (8.0) (539) = 4312 cal end of rod through its different parts of the other ends.
\ Net heat required
Q = Q1 + Q 2 + Q3 + Q 4 Thermal Conductivity
= 5815.6 cal In solids, heat is transferred through conduction. We will
study conduction of heat through a solid bar.
Telegram @unacademyplusdiscounts

Heat and Kinetic Theory of Gases 497

Regarding conduction following points are worth noting If different rods are connected in parallel, then
temperature difference is same, i. e.,
(i) The amount of heat flowing in a rod of surface area A in
time t is i. e., DT1 = DT2 = DT3 =¼
DQ Dq 1 1 1 1
= - KA \ = = + +¼
Dt Dx Rp R1 R2 R3
Here, K = coefficient of heat conduction (vi) Heat current
Dq
= temperature gradient between faces of a rod dQ DT æ l ö
Dx H= = ç where R = ÷
dt R è KAø
DQ
In the above relation, negative sign is used to make Current flow through a resistance
Dt
a positive quantity since, is negative. dq DV æ l ö
i= = ç where R = ÷
(ii) The ratio of thermal and electrical conductivities is the dt R è sAø
same for the metals at a particular temperature and is We find the following similarities in heat flow through
proportional to the absolute temperature of the metal. a rod and current flow through a resistance.
If T is the absolute temperature, then
K Heat flow through a Current flow through a
µT conducting rod resistance
s
K dQ dq
or = constant Heat current H = Electric current i =
sT dt dt
= rate of heat flow = rate of charge flow
(iii) Let two rods of thermal conductivities K1, K2 lengths
l1, l2 and cross-sectional area A are connected in series. DT TD DV PD
H= = i = =
In steady state the temperatures of ends of rod are T1 R R R R
and T2 and the temperature of junction is T. Then l l
R= R=
H1 H2 KA sA
K = thermal conductivity s = electrical conductivity
K1 K2
From the above table, it is evident that flow of heat
T1 T T2 through rods in series and parallel is analogous to the
l1 l2 flow of current through resistances in series and
parallel. This analogy is of great importance in solving
K1T1l2 + K2T2l1
T = complicated problems of heat conduction.
K1l2 + K2l1
(vii) In series combination of rods of different materials,
(iv) The thermoelectric conductivity or diffusivity is defined equivalent conductivity
as the ratio of the coefficients of thermal conductivity.
K1 K2 K3
So,
m
Thermal capacity per unit volume = æç ö÷ c = rc, where r
èV ø
l1 l2 l3
is the density of substance. L
K
Diffusivity D = l1 + l2 + l3 l l l
rc = 1 + 2 + 3
Ks K1 K2 K3
(v) The hindrance offered by a body to the flow of heat is
If lengths of rods are equal, then
called its thermal resistance.
1 1 1
Temperature difference (DT ) + +
R= 1 K1 K2 K3
Heat current ( H ) =
Ks 3
DT l
= =
H KA (viii) In parallel combination of slabs of different materials,
equivalent conductivity
where l is length of rod, A the area and DT the
K 1 A 1 + K 2 A 2 + K 3 A3
temperature difference across its ends. Kp =
If different rods are connected in series, then heat A1 + A 2 + A3
flowing per second is same. If areas of slabs are equal, then
i. e., H1 = H2 = H3 =¼
\ Rs = R1 + R2 + R3 + ¼
Telegram @unacademyplusdiscounts

498 JEE Main Physics

l Which collection of the answer is correct ?


(i) (ii) (iii) (iv)
A1 (a) 3 kW -1 6W 5 Cm-1 8°C
K1 (b) 5.9 kW -1 60 W -50 Cm-1 87.5°C
A2
(c) 15.9 kW -1 6.3 W -50 Cm-1 67.5°C
K2
A3 (d) 15.9 kW -1 6.3 W -50 Cm-1 87.5°C
K3
Interpret (d) (i) Thermal resistance
K1 + K2 + K3 l l
Kp = R= =
3 KA K ( pr 2)
(ix) Ingen Hauz’s experiment If a number of identical rods (2)
or R=
state of different metals are coated with wax and one of ( 401) ( p ) (10 -2) 2
their ends is put in boiling water, then in steady state
= 15.9 kW –1
the square of length of the bar over which wax melts is
directly proportional to the thermal conductivity of the DT Dq 100
(ii) Thermal current, H = = =
metal, i. e., R R 15.9
K or H = 6.3 W
= constant
l2 0 - 100
(iii) Temperature gradient = = -50 km–1
(x) When the atmospheric pressure falls below 0°C 2
(say-T°C), the cold air above water extracts heat from = -50° Cm–1
the water. As a result, the water begins to freeze into
(iv) Let q°C, be the temperature at 25 cm from the hot end, then
the ice layers. Let at any time the thickness of ice is x
and further layer of ice of thickness dx is formed in 100°C °C 0°C
time dt. If r is density of ice and L be the latent heat of
fusion, then 0.25 m
2.0 m
at – T°C
( q - 100) = ( temperature gradient) ´ (distance)
Air
or q - 100 = ( -50) (0.25)
q = 87.5° C
Ice
x Sample Problem 13 Two metal cubes with 3 cm edges of
at 0°C copper and aluminium are arranged as shown in figure. Find
dx Water at 4°C

time taken by ice to grow to a thickness x is Al


100°C 20°C
rL x rL 2 Cu
Kq ò 0
t= x dx = x
2 Kq
If the thickness is from x1 to x2, then time taken (i) the total thermal current from one reservoir to the other.
rL (ii) the ratio of the thermal currents carried by the copper cube
t= ( x22 - x12 )
2 KT to that carried by the aluminium cube.
Here, K = coefficient of thermal conductivity of ice. Thermal conductivity of copper is 401 Wm-1K -1 and that of
aluminium is 237 Wm-1K -1.
Sample Problem 12 A copper rod 2m long has a circular (a) 0.08 kW -1, 1.75 (b) 0.01 kW -1, 1.05
cross-section of radius 1 cm. One end is kept at 100°C and the (c) 0.25 kW -1, 1.32 (d) 0.02 kW -1, 1.02
other at 0°C and the surface is insulated so that negligible heat l
is lost through the surface. Find Interpret (a) Thermal resistance of aluminium cube, R1 = ,
KA
(i) the thermal resistance of the bar (3.0 ´ 10 –2)
(ii) the thermal current H or R1 = = 0.14 kW –1
(237) (3.0 ´ 10 –2) 2
dT
(iii) the temperature gradient and l
dx and thermal resistance of copper cube R2 = ,
(iv) the temperature 25 cm from the hot end. KA
Thermal conductivity of copper is 401 Wm-1K -1
Telegram @unacademyplusdiscounts

Heat and Kinetic Theory of Gases 499

(3.0 ´ 10 –2) Interpret (a) Let q be the temperature of inner surface of box.
or R2 = = 0.08 kW –1
( 401) (3.0 ´ 10 –2) 2 Heat transfer per second through A + Heat produced by source per
second = Heat transfer per second through B
As these two resistances are in parallel, their equivalent
æ dQ ö æ dQ ö
resistance will be Þ ç ÷ + 36 = ç ÷
è dt ø A è dt ø B
RR (0.14) (0.08)
R= 1 2 = = 0.05 kW –1 KA (100 - q ) KA ( q - 4)
R1 + R2 (0.14) (0.08) Þ + 36 =
Temperature difference d d
\ Thermal current, H = Þ KA ( q - 4 - 100 + q ) = 36 ´ d
Thermal resistance
(100 - 20)
= = 1.6 ´ 10 3 W
0.05
In parallel, thermal current distributes in inverse ratio of A B
resistances 100°C 4°C
HCu RAl R1 0.14 Source
Hence, = = = = 1.75
HAl RCu R2 0.08

Sample Problem 14 Water is boiled in flat bottom kettle Now, d = 8 cm, A = 12 cm2,K = 0.5 cals–1° C–1cm–1
2
placed on a stove. The area of the bottom is 3000 cm and the 36 ´ 8
Þ 2 q - 104 =
thickness is 2 mm. If the amount of steam produced is 1 gmin -1, 12 ´ 0.5
calculate the difference of temperature between the inner Þ q = 76° C
and outer surfaces of the bottom. K for the material of kettle is
0.5 cal°C -1s -1cm -1 Convection
(a) 2.1 ´ 10 –3 °C (b) 3.1 ´ 10 –3 °C
It is a process by which heat is transferred in fluids (liquids
(c) 1.2 ´ 10 –3 °C (d) 2.5 ´ 10 –3 °C
and gases). In convection, transfer of heat takes place by
Interpret (c) Mass of steam produced = dm = 1 gs–1 transport of matter (in form of motion of particles).
dt 60
When a liquid in a container is heated, the molecules at
dQ dm
Heat transferred per second = =L the lower layers are heated up and their densities
dt dt
decrease. As a result the molecules rise up and heavier
dQ 1
Þ = 540 ´ cal° C–1s–1cm–1 ones come down and hence a continuous movement of
dt 60 molecules takes place giving rise to convection currents.
q = temperature difference In this manner the whole of liquid gets heated.
d = thickness = 2 m = 0.2 cm
dQ K Aq Radiation
Þ =
dt d
In radiation, heat is transferred from one body to other or
dm K Aq
Þ L = to the surroundings even in the absence of any medium in
dt d
the intervening space. Heat energy of the sun is
0.5 ´ 3000 ´ q
Þ 9= transmitted to earth through radiations.
0.2
Þ q = 1.2 ´ 10 –3° C

Sample Problem 15 A closed cubical box made of


13.9 Heat Transfer through
perfectly insulating material has walls of thickness 8 cm and the Radiation
only way for the heat to enter or leave the box is through the
solid cylindrical metallic plugs each of cross-sectional area Radiation is only a mode of transfer of energy by
12 cm 2 and length 8 cm fixed in the opposite walls of the box as transverse electromagnetic waves. While studying heat
radiations (Radiant energy) we are concerned with
shown in figure. The outer surface A is kept at 100°C while the
outer surface B of other plug is kept at 4°C. K if the material of thermal radiations which form the infrared region of
electromagnetic waves. All bodies emit heat to the
the plugs is 0.5 cals-1 ° C -1cm -1. A source of energy generating
surroundings at all temperatures and at all times. When
36 cals-1 is enclosed inside the box. Find the equilibrium of the
the temperature of a body remains constant, it emits as
inner surface of the box, assuming that it is same at all points on much heat to the surroundings as it gains from them. The
the inner surface. body is then in a state of dynamic (thermal) equilibrium.
(a) 76°C (b) 86°C (c) 66°C (d) 56°C
Telegram @unacademyplusdiscounts

500 JEE Main Physics

Absorption, Reflection and Transmission where, Q represents the energy of thermal radiation.
Qabsorbed
When radiations are incident on a surface, then three Absorptivity or absorptive power, a =
Qincident
things happen¾a part of the radiation is absorbed, some
is reflected back, and remaining is transmitted. Qreflected
Reflectivity, r=
Qincident
Incident
Q
Transmissivity, t = transmitted
Qincident
Qs Qv Qt
Absorption Reflected + + = a+ r+t =1
Q Q Q

For a perfect black body, a = 1, r = t = 0


Transmitted For a perfect reflector, a = t = 0, r = 1

Qincident = Qabsorbed + Qreflected + Qtransmitted For a perfect transmitter, a = r = 0, t = 1

Some Common Terms and Points


The thermal radiation emitted by a body comprises of all Emissive power (e) For a given surface it is defined as the
the wavelengths; intensities of radiation corresponding to radiant energy emitted per second per unit area of the
different wavelengths are different. surface.
Absorptive power (a) It is defined as the ratio of the radiant It is the total amount of energy radiated by a body per
energy absorbed by it in a given time to the total radiant second per unit area of surface.
energy incident on it in the same interval of time. 1 DQ
e=
Energy absorbed A Dt
a=
Energy incident Spectral emissive power (el ) It is emissive power for a
As a perfectly black body absorbs all radiations incident particular wavelength l. Thus,
on it, the absorptive power of a perfectly black body is ¥
e = ò el dl
maximum and unity. 0

Spectral Absorptive Power (al ) The spectral absorptive


Emissivity ( e )
power is the ratio of radiant energy absorbed by a surface
to the radiant energy incident on it for a particular Emissivity of a body at a given temperature is defined as
wavelength l. It may have different values for different the ratio of the total emissive power of the body (e) to the
wavelengths for a given surface. total emissive power of a perfect black body (E) at that
temperature,
The spectral absorptive power al is related to absorptive
e
power a through the relation i. e. , e=
E
¥
a = ò al dl
0

Perfectly Black Body


A body which can absorb all radiations of each
wavelength at any temperature, which are incident on its
P
and emits the full radiation spectrum on being heated is Q
known as a black body.
Such a body neither reflects nor transmits any part of the
incident heat radiation and hence, appears black
irrespective of the colour of the incident radiation. This Cavity approximating an ideal black-body. Radiation entering the
implies that a perfectly black body has unit absorptance. cavity has little chance of leaving before it is
Also, a perfectly black body when heated emits radiation completely absorbed.
of all possible wavelengths at that temperature.
Telegram @unacademyplusdiscounts

Heat and Kinetic Theory of Gases 501

Materials like black velvet or lamp black come close to If Q is the total energy radiated by the ordinary body, then
being ideal black bodies but the best practical realization Q
e= = esT 4
of an ideal black body is a small hole leading into a cavity, A ´t
as this absorbs 98% of the radiation incident on them. Þ Q = A esT 4t
Ferry’s black body Ferry suggested and designed a
perfectly black body on the principle that any space which Net Heat Loss from the Surface of a Body
is almost wholly closed having a small hole is capable of The rate at which a body radiates energy is determined by
emitting and absorbing full radiation spectrum. the temperature of the body and its surroundings. When a
Ferry’s black body consists of a double walled hollow body is hotter than its surroundings, the rate of emission
sphere having a small opening O on one side and a exceeds the rate of absorption; there is net loss of energy,
conical projection P just opposite to it. and the body cools down, unless it is heated by some
other means.
The inner wall of enclosure is painted with lamp black.
The heat radiations entering the hollow sphere through O When a body is cooler than its surroundings, the rate of
get completely absorbed due to multiple reflections. The absorption is greater than the rate of emission, and its
chance of heat radiations getting and through O is temperature rises. At thermal equilibrium the two rates
reduced by conical projection and the lamp black coating. are equal.
Walls
Therefore, all the radiations are absorbed completely and
hence, the absorptance of the enclosure is 100%
approximately. T1

Kirchhoff’s Law
At any temperature and for particular wavelength, the T2
ratio of the emissive power to the absorptive power of all
the bodies is same and is equal to the emissive power of a Hence, for a body at a temperature of T1, surrounded by
perfectly black body. walls at a temperature T2 (as in figure), the net rate of loss
e (or gain) of energy per second by radiation is
i. e. , =E (constant)
a Hnet = AesT14 - AesT24
Now, E =1 (for perfectly black body)
Hnet = Aes (T14 - T24 )
So, e=a
Kirchhoff’s law signifies that good absorbers are always Newton’s Law of Cooling
good emitters.
According to this law, if the temperature T of the body is
not very different from that of the surroundings T0, then
Stefan’s Law dT
rate of cooling - is proportional to the temperature
The energy emitted per second per unit area of a black dt
body (emissive power = 1) is proportional to the fourth difference between them. To prove it let us assume that
power of the absolute temperature. T = T0 + DT
4 4
i. e. , E = sT æ DT ö
So that T 4 = (T0 + DT )4 = T04 ç1 + ÷
Here, s = Stefan’s constant è T0 ø
= 5.67 ´ 10–8 Jm–2s–1K –4 æ 4 DT ö
» T04 ç1 + ÷ (from binomial expansion)
è T0 ø
For any other body,
e = e sT 4 \ (T 4 - T04 ) = 4 T03 (DT )

Here, e = emissivity of body (e = 1for a black body) or (T 4 - T04 ) µ DT …(i) (T0 = constant)
Telegram @unacademyplusdiscounts

502 JEE Main Physics

Rate at which heat is emitted, ● The figure above shows the experimental curves for
dQ1 radiation emitted by a black body versus wavelength
= seAT 4
dt for different temperatures.
Rate at which radiation is absorbed, ● The most significant feature of the curves obtained is
dQ2 that they are universal i. e. , black-body radiation
= seAT04 curves obtained depend only on the temperature and
dt
not on the shape, size or material of the black body.
Net rate of heat loss, H = seA [ T 4 - T04] [Q ee = a]
● As the temperature of the body increases, the
dT seA 4
Rate of cooling, =- [ T - T04] wavelength at which the spectral intensity (El ) is
dt ms
maximum shifts towards left.
where, m is mass of the body and s its specific heat
Note
capacity.
1. Diathermanous A surface or a medium which transmits most
Negative sign is there because temperature is falling with of the radiation (t = 1) is called diathermanous, also the
time. substances, which allow heat radiation to pass through them are
dT called diathermanous e.g., dry air, rock salt etc.
Now, = - K DT
dt 2. Adiathermanous A surface or a medium which does not
seA ´ 4 T03 transmit radiation at all (t = 0) is known as a opaque or
where, K =
ms adiathermanous medium. Moreover, the substances which
Solving above equation, we get absorb heat radiation and get themselves heated are called
adiathermanous, e . g ., water, wood and solid..
T ( t ) = T0 + (T1 - T0 ) e- kt
3. Solar constant The amount of heat received from the sun by
where T1 is the temperature of body at t = 0. one square centimetre area of a surface placed normally to the
sun rays at mean distance of the earth from the sun is known as
Note Approximate solution for Newton’s law of cooling is solar constant. It is denoted by S.
T1 - T2 T +T
= K éê 1 2 - T0 ùú , where t is the time in which temperature of 3
S = æç ö÷ sT 4
r
t ë 2 û èRø
body changes fromT1 toT2 .
Here, r is the radius of the sun and R the mean distance of the
earth from the centre of the sun. Value of solar constant is
Wien’s Displacement Law 1.937 calcm–2 min–1.
According to this law, the wavelength (l m ) of maximum
intensity of emission of black body radiation is inversely Sample Problem 16 The emissivity of tungsten is
proportional to absolute temperature of the black body, approximately 0.35. A tungsten sphere 1 cm in radius is
i. e. , suspended within a large evacuated enclosure whose walls are
1 at 300 K. What power input is required to maintain the sphere
lm µ
T at a temperature of 3000 K, if heat conduction along the
or l mT = b constant supports is neglected? s = 5.67 ´ 10 –8 SI units.
where b is Wien’s constant and has value 2.89 ´ 10–3 m- K (a) 2119.8 W
(b) 2019.8 W
and l m is the wavelength corresponding to maximum
(c) 2219.8 W
intensity (energy constant) of radiation emitted by body at
(d) 1919.8 W
temperature T.
● The thermal radiation emitted by a body at any
Interpret (b) Net heat lost by sphere per second
temperature consists of all wavelengths from small to Hnet = es A (T 4 - T04)
large values. The intensities of all wavelengths are where, T = temperature of sphere = 3000 K
different.
T0 = temperature of surrounding = 300 K

T1 > T2 > T3 Area, A = 4 pr 2 = 4 p (0.01) 2

T1 To maintain constant temperature,

T2
Power input required = net heat loss from the surface
Pinput = es A (T 4 - T04)
T3
= 0.35 ´ 5.67 ´ 10 –8 ´ 4 p (0.01) 2 ´ (3000 4 - 300 4)
λ
λm1 λm2 λm3
Pinput = 2019.8 W
Telegram @unacademyplusdiscounts

Heat and Kinetic Theory of Gases 503

Sample Problem 17 The rate at which the radiant energy Sample Problem 19 A body cools down from 60°C to
reaches the surface of the earth from the sun is about 55°C in 30 s. Using Newton’s law of cooling, calculate the
1.4 kWm -2. The distance from the earth to the sun is about approximate time taken by same body to cool down from 55°C
1.5 ´ 1011 m, and the radius of the sun is about 0.7 ´ 109 m. to 50°C. Assume that the temperature of surroundings is 45°C.
What is the rate of radiation of energy per unit area from the (a) 41.28 s (b) 55.28 s
sun’s surface? (c) 51.28 s (d) 60.28 s
(a) 6.43 ´ 10 7 Wm-2 (b) 6.43 ´ 10 6 Wm-2 Interpret (c) According to Newton’s law of cooling
-2
7
(c) 5.43 ´ 10 Wm (d) 6.43 ´ 10 -7 Wm-2 q1 - q2 é q1 + q2 ù
=Kêë 2 - q0 úû
Interpret (a) Let D = distance from the sun to the earth t
= 1.5 ´ 10 11 m 60 - 55 é 60 + 55 ù
=K ê - 45ú …(i)
Let R = radius of the sun = 0.7 ´ 10 9 m 30 ë 2 û
Let power of the sun, P = energy radiated from the surface of the sun Similarly, for 2nd case,
per second. 55 - 50 é 55 + 50 ù
Hence, in every one second, P joule of energy are radiated from the =K êë 2 - 45ú …(ii)
t û
surface of the sun and this energy passes through a big sphere of
radius D centred at the sun. Dividing Eq. (i) by Eq. (ii), we get, t = 51.28 s
Hence, at the circumference of this big sphere (i. e. , near the surface
of the earth), the energy crossing through a unit area per second
Sample Problem 20 A black body at 227°C radiates heat
P p at a rate of 7 cal/cm 2s. At a temperature of 727°C, the rate of
= = heat radiated in the same units will be
area of big sphere 4 pD2
(a) 112 (b) 105
P
= 1.4 ´ 10 3 Wm–2 (c) 101 (d) 89
4 pD 2
sphere o
Interpret (a) According to Stefan’s law E = sT 4
big f \ 7 = s (227 + 273) 4 = s ´ (500) 4
ra
diu

and x = s (727 + 273) 4 = s ´ (1000) 4


sD

Sun x (1000) 4
Hence, = = 16
D 7 (500) 4
Þ x = 16 ´ 7 = 112 cal/cm 2s

Þ P = 4 p (1.5 ´ 10 11) 2 ´ 1.4 ´ 103 W Sample Problem 21 Which of the following is vm -T graph
Þ P = 3.96 ´ 10 26
W for perfectly black body? n m is the frequency of radiations with
maximum intensity and T is the absolute temperature.
Rate of radiation of energy per second per unit area of the sun’s νm(Hz)
surface is given by B

P P D
= 2 C
area of big sphere 4 p
= 6.43 ´ 10 7 Wm–2
A
Sample Problem 18 In the above problem, if the sun O T(K)
radiates as an ideal black body, what is the temperature of its (a) A (b) B (c) C (d) D
surface?
(a) 6803 K (b) 5603 K Interpret (c) According to Wein’s displacement law
(c) 5803 K (d) 5503 K l mT = b = Wein’s constant
Interpret (c) If the sun is an ideal black body, e = 1 If n m is the frequency corresponding to wavelength l m then
Þ E = sT 4 æ C ö
ç ÷T = b
1/ 4 è nm ø
æEö
Þ T=ç ÷ C
ès ø or nm = T
1/ 4 b
æ 6.43 ´ 10 7 ö
=ç ÷ = 5803 K i. e. , nm µ T
è 5.67 ´ 10 –8 ø \ n m - T graph is straight line shown by the curve C.
Telegram @unacademyplusdiscounts

504 JEE Main Physics

V/T V/T
13.10 Ideal Gas or Perfect Gas m = constant m = constant
p = constant p = constant
An ideal gas or perfect gas is that gas which strictly obeys
the gas laws such as Boyle’s law, Charles, law,
Gay-Lussac’s law etc.
1/V 1/T
Boyle’s Law (d) (e)
According to it for a given mass of ideal gas at constant
temperature (called isothermal process), the volume of a Gay-Lussac’s Law or Pressure Law
gas is inversely proportional to its pressure i. e. ,
1 According to it for a given mass of an ideal gas at constant
V µ (if m and T = constant)
p volume (called isochoric process), pressure of a gas is
directly proportional to its absolute temperature i. e. ,
Graphical forms of such law are shown in figure
p µT (if m and V = constant)
p p
m = constant m = constant This is shown graphically
T = constant T = constant p p/T
m = constant
V = constant m = constant
V = constant

V 1/V
(a) (b)

pV 1/p
(a) T(in K) (b) p or T
m = constant m = constant
T = constant T = constant
Avogadro’s Law
According to it at same temperature and pressure equal
1/V volumes of all the gases contain equal number of
(c) p or V (d)
molecules, i. e. ,
N1 = N 2
pV
m = constant
T = constant
13.11 Equation of State of a
Perfect Gas
p
(e) In practice, the gases do not obey the gas laws at all values
of temperature and pressure. It is because of the
intermolecular forces between the gas molecules.
Charles’ Law
An ideal gas is one whose molecules are free from
According to it for a given mass of an ideal gas at constant intermolecular attraction and obeys gas laws at all values
pressure (called isobaric process), volume of a gas is of temperature and pressure.
directly proportional to its absolute temperature i. e. ,
Ideal gas equation is a form of combined effect of above
V µT (if m and p = consant)
first four laws. Thus, the equation is given by
Graphical forms of such law are shown in figure m
pV = nRT = RT
V V/T 1/T M
m = constant
p = constant m = constant m = constant m
p = constant p = constant Here, n = number of moles of the gas =
M
m = total mass of the gas
M = molecular mass

(a)
R = universal gas constant
T(in K) (b) V or T (c) V
= 8.31 Jmol–1 K –1 = 2.0 cal mol–1K –1
Telegram @unacademyplusdiscounts

Heat and Kinetic Theory of Gases 505

The above first four laws can be obtained by this ideal gas T
Þ pV n × = constant …(iv)
equation. For example, for a given mass of a gas pV
pV = constant (at constant temperature) Þ TV n-1 = constant …(v)
(Boyle’s law) Eq. (ii) can now be rewritten as
V
= constant (at constant pressure) T
T V = constant × …(vi)
p
(Charles’ law)
p Putting this value is Eq. (i), we get
= constant (at constant volume)
T
pT n
(Gay-Lussac’s law) = constant …(vii)
pn
and if p, V and T are constants then n = constant for all
T
gases. Since, equal number of moles contain equal Þ n-1
= constant …(viii)
number of molecules, thus at constants p, V and T all p n
gases will contain equal number of molecules which is
nothing but Avogadro’s law. Work done by a gas which is compressing from state 1 to
state 2 is given by
2
W = -ò p dV …(ix)
13.12 Ideal Gas Law with 1

From Eq. (i), we have


Constraints C
p= …(x)
For the purpose of calculations, it is convenient to place Vn
the ideal gas in the form Putting this value in equation (ix), we get
pf V f 2
pV
i i W = - ò V -n dV …(xi)
= 1
Ti Tf
On integration it leads to
where the subscripts i and f refer to the initial and final 2
æ V - n+1 ö
states of some process. W =C ç ÷ …(xii)
è - n + 1ø 1
If the temperature is constrained to be constant, this
becomes Using equation pV n = C , we have
2
i i = pf V f
pV æ V 1- n ö
W = - ç pV n ÷ …(xiii)
which is referred to as Boyle’s law. è 1- n ø
1
If the pressure is constant, then the ideal gas law takes the n
form æ pV ö
W = -ç ÷ …(xiv)
è 1- n ø
Vi V f Tf 1
= or V f = Vi
Ti T f Ti ( p2V2 - p1V1 )
Þ W =- …(xv)
1- n
which is referred to as Charles’ law.
Ideal gas law also follows the equation

Work Done on Compressing a Gas p1V1 = mRT1 …(xvi)


p2V2 = mRT2 …(xvii)
The expansion and compression of ideal gases follow the
expression. Work done now becomes
n
pV = constant …(i) mR (T2 - T1 )
W =- …(xviii)
1- n
where n is number of moles of the gas. Ideal gases also
follow the combined gas law Note Similar expressions are obtained by similar methods for work
pV done during expansion of gas but starting from
= constant …(ii) 2
T W= ò1 pdV
pV
Dividing Eq. (i) by , we get p 2V 2 - p1V1
T We obtain, W=
n -1
pV
pV n ¸ = constant …(iii) mR (T2 - T1)
T W=
n -1
Telegram @unacademyplusdiscounts

506 JEE Main Physics

Sample Problem 22 Two moles of an ideal gas is Sample Problem 23 A closed container of volume 0.02 m3
contained in a cylinder fitted with a frictionless movable piston, contains a mixture of neon and argon gases at a temperature of
exposed to the atmosphere, at an initial temperature T0. The gas 27°C and pressure of1 ´ 10 5 Nm 2. The total mass of the mixture
is slowly heated so that its volume becomes four times the
is 28 g. If the gram molar weights of neon and argon are 20 and
initial value. The work done by the gas is
40 respectively, find the masses of the individual gases in the
(a) zero (b) 2 RT0
container, assuming them to be ideal. (R = 8.31 Jmol -1K -1).
(c) 4 RT0 (d) 6 RT0
(a) 24 g (b) 25 g
Interpret (d) Given that gas is slowly heated, which means it (c) 26 g (d) 27 g
remains in equilibrium (more or less) with the atmosphere, i. e. , the
process takes place at constant pressure. Interpret (a) Let in the given container mass of neon be m and
mass of argon be (28 - m) g, so that
m
nNe =
Piston 20
28 - m
Gas and nA =
40
m (28 - m) 28 + m
n = nNe + nA = + = …(i)
From the equation of ideal gas law 20 40 40
pV = nRT and using ideal gas equation for the mixture, we have
For infinitesimal change pV 1 ´ 10 5 ´ 0.02
n= = = 0.8 ...(ii)
pdV = nR dT RT 8.314 ´ 300
or pDV = nR DT Comparing Eqs.(i) and (ii), we get
28 + m
Also, pDV = Work done by the gas = DW = 0.8
40
\ DW = nRDT
Þ m= 4g
Also DV µ DT
\ mNe = 4 g
\ DT µ DV µ V2 - V1
and mA = 28 - 4 = 24 g
Given, V2 = 4 V1
\ DT µ 4 V1 - V1 µ 3 V1 µ 3 T0 Sample Problem 24 During an experiment, an ideal gas is
Also given m = 2 moles found to obey an additional law Vp 2 = constant. The gas is
The expression for work done becomes initially at temperature T and volume V. What will be the
DW = nRDT temperature of the gas when it expands to a volume 2V?
DW = 2 R 3T0 = 6 RT0 (a) 3 T (b) 1/2 T
(c) 2 T (d) 3 T
Note
1. STP or NTP refers to standard (normal) temperature of 273K Interpret (c) Here it is given that Vp2 = constant K (say). Hence,
and 1 atm pressure of 1.01 × 105 Pa. we may write the gas equation as,
2. Whatever be the process, in equilibrium state, an ideal gas pV = nRT
satisfies the equation pV = nRT.
K
3. In terms of density, the ideal gas equation may be expressed as or × V = nRT
p V
= constant
rT nR
or V = T
4. In terms of number of molecules (n) per unit volume of a gas, K
the ideal gas equation may be expressed as p = nkT V1 T1
Þ =
5. If n1 mole of a non-reactive gas in thermodynamical state V2 T2
( p1, V1, T1) be mixed with n2 mole of another non-reactive gas at
( p2 , V2 , T2 ) and the resultant gas mixture is at a state ( p, V , T ) V2
\ T2 = T1
then V1
p1V1 p2V2 pV 2V
+ = =T = 2T
T1 T2 T V
Telegram @unacademyplusdiscounts

Heat and Kinetic Theory of Gases 507

1/ 2
é v2 + v22 + ¼+ vN2 ù
13.13 Kinetic Theory of Gases vrms =v =ê 1 ú
ë N û
The kinetic theory of gases correlates the macroscopic y
properties of gases e. g. , pressure, temperature etc., to the
microscopic properties of gas molecules e. g. , speed,
momentum, kinetic energy of molecules etc. The kinetic
theory of gases is based on the following assumptions v
(i) A gas consists of a large number of tiny, identical,
spherical and electrically neutral, stable elastic d m
particles called molecules. vx
(ii) The space occupied by the molecules of a gas is
extremely small as compared to the volume of the gas. z x
d d
(iii) The molecules of a gas are in a state of continuous,
random motion with all possible speeds ranging from A cubical box with sides of length d containing
an ideal gas. The molecule shown moves with velocity v.
zero to infinity in different possible directions. The
speed distribution is in accordance with Maxwell’s 1 mN 2 1 2
distribution law of molecular speeds and has been Thus, pressure exerted by a gas p = v = rv ,
3 V 3
shown in figure.
mN
where r = = density of given gas.
V
T1
1 æNö 2 2 N æ1 2ö
of molecules (n)

Now p=
mç ÷v = ç mv ÷
3 èV ø 3 V è2 ø
Number

T2 > T1
2
or pV = N KE
3
1 2 2 æ 1 2ö
Also p = rv = ç rv ÷
3 3 è2 ø

(vmp) Molecular speed (v) 1 2


Now, rv = average kinetic energy of the gas per unit
2
(iv) Each molecule behaves as an independent entity. volume.
There is no force of attraction among the molecules. 2
\ p = ´ average kinetic energy per unit volume
Thus, gas molecules have no potential energy but 3
possess only kinetic energy which is directly 2
proportional to temperature of the gas. p= E
3
(v) The pressure of a gas is due to elastic collision of gas
molecules with the walls of the container. Kinetic Energy and Temperature
(vi) The dynamics of the particles is governed
1 by Newton’s
2
laws of motion. According to kinetic theory of gases, pV = mNv
3
(vii) The time of contact of a moving molecule with the
but according to equation of state for an ideal gas
container walls at the time of collision is negligible as
compared to the time between two successive pV = nRT
collisions with the same wall of the container. 1 1 3 RT 3
mN v 2 = nRT to mv 2 = = kT ,
3 3 2 n 2
Concept of Pressure where k is the Boltzmann’s constant. Its value is
On the basis of these assumptions we can do 1.38 ´ 10–23 J mol–1 K –1.
mathematical calculations to find expression for pressure
\ Mean translational kinetic energy of a gas molecule
1
exerted by a gas. Accordingly, we find that pV = mN (v ) 2 3
3 = kT i. e. , the mean translational kinetic energy of a gas
2
where, m = mass of 1 gas molecule and N = total number
molecule depends only on its temperature and is
of gas molecules, v is as root mean square velocity
independent of its nature or mass etc.
Telegram @unacademyplusdiscounts

508 JEE Main Physics

On this basis, we can define absolute zero temperature as Sample Problem 25 At what temperature does the
the temperature at which translational kinetic energy of a average translation kinetic energy of a molecule in a gas
gas molecule becomes zero i. e. , at which the molecular becomes equal to the kinetic energy of a electron accelerated
motion ceases altogether. from rest through a potential difference of one volt?
(k = 1.38 ´ 10 –23 JK -1)
Critical temperature, pressure and volume Gases cannot be
liquified above a temperature called critical temperature (a) 7330 K (b) 7730 K
(Tc ) however large the pressure may be. The pressure (c) 7530 K (d) 7430 K
required to liquify the gas at critical temperature is called Interpret (b) Kinetic energy gained by an electron when
critical pressure ( pc ) and the volume of the gas at critical accelerated by a potential difference of 1 V is1 eV = 1.6 ´ 10 –19 J.
temperature and pressure is called critical volume (Vc ). 3
According to kinetic theory of gases, kinetic energy = kT
Value of critical constants in terms of van der Waals’ 2
constants a and b are as under 3
As kT = 1 eV = 1.6 ´ 10 19 J
a 2
Vc = 3 b, pc =
27 b2 2 ´ 1.6 ´ 10 –19
Þ T=
8a 3k
and Tc =
27 Rb 2 ´ 1.6 ´ 10 –19
= = 7730 K
RTc 8 3 ´ 1.38 ´ 10 –28
Further, = is called critical coefficient and is same
pcVc 3
for all gases.
Van der Waals’ Gas Equation
The gases actually found in nature are called real gases.
RMS Speed of Gas Molecules They do not obey gas laws. A real gas behaves as ideal
Root mean square (rms) speed. It is defined as the square gas most closely at low pressure and high temperature.
root of the mean of squares of the speeds of different Equation of state for real gases is given by van der Waals’
molecules i. e. , equation with two corrections in ideal gas (i) volume
vrms = v = (v12 + v22 + ¼+ vN2 ) /N correction (ii) pressure correction. van der Waals’ gas
equation for 1 mole of gas is given by
According to kinetic theory of gases it is observed that
æ a ö
3p 3 pV ç p + 2 ÷ (V - b) = RT
vrms = v = = è V ø
r M
æ an2 ö
3 RT 3 kT For n moles ç p + 2 ÷ (V - nb) = nRT
= = è V ø
M m
Here, a and b are constants called van der Waals’
where, M is the molar mass of gas while m is the mass of a constants.
single gas molecule.

Average speed It is the arithmetic mean of the speeds of


molecules in a gas. Thus,
13.14 Degree of Freedom (f)
v + v2 + ¼+ vN The term degree of freedom refers to the number of
vav = v = 1
N possible independent ways in which a system can have
On the basis of kinetic theory it is observed that energy.
8 RT 8p 8 kT y
vav = = =
pM pr pm
f=2
f=1
Most probable speed It is the speed possessed by
maximum number of gas molecules in a given gas. On x
kinetic theory basis it is found that (a) (b)
2 RT 2 kT 2p f=3
vmp = = =
M m r
Thus, we find that for a given gas vmp < vav < v and
8
vmp : vav : v = 2 : : 3. (c)
p
Telegram @unacademyplusdiscounts

Heat and Kinetic Theory of Gases 509

For example In Fig. (a) block has one degree of freedom, 3 translational, 2 rotational and 2 vibrational.
freedom, because it is confined to move in a straight line Thus,
f =5
and has only one translational degree of freedom.
(3 translational + 2 rotational) at room temperatures
In Fig. (b), the projectile has two degrees of freedom
and f =7
because it is confined to move in a plane and so it has two
translational degrees of freedom. (3 translational + 2 rotational + 2 vibrational)
at high temperatures
In Fig. (c), the sphere has two degrees of freedom one
rotational and another translational.
Degree of Freedom of Non-linear
Similarly a particle free to move in space will have three
translational degrees of freedom. Polyatomic Gas
A non-linear polyatomic molecule (such as NH3) can
Vibrational Energy rotate about any of three coordinate axes. Hence, it has
6 degrees of freedom 3 translational and 3 rotational. At
The forces between different atoms of a gas molecule may
room temperatures a polyatomic gas molecule has
be visualized by imagining every atom as being connected
vibrational energy greater than that of a diatomic gas. But
to its neighbours by springs. Each atom can vibrate along
at high enough temperatures it is also significant. So, it has
the line joining the atoms. Energy associated with this is
8 degrees of freedom 3 rotational, 3 translational and
called vibrational energy.
2 vibrational. Thus,

Degree of Freedom of Monoatomic Gas z

A monoatomic gas molecule (like He) consists of a single y


atom. It can have translational motion in any direction in
space. Thus, it has 3 translational degrees of freedom.
x
f =3 (all translational)
It can also rotate but due to its small moment of inertia,
rotational kinetic energy is neglected.
(3 translational + 3 rotation)
Degree of Freedom of a Diatomic and (3 translational + 3 rotational + 2 vibrational)
Linear Polyatomic Gas at high temperatures

The molecules of a diatomic and linear polyatomic gas


Degree of Freedom of a Solid
(like O 2, CO 2 and H2) cannot only move bodily but also
rotate about any one of the three coordinate axes as An atom in a solid has no degrees of freedom for
translational and rotational motion. At high temperatures
shown in figure. However, its moment of inertia about the
due to vibration along 3 axes it has 3 ´ 2 = 6 degrees of
axis joining the two atoms (x-axis) is negligible. Hence, it
freedom.
can have only two rotational degrees of freedom. Thus, a
f = 6 (all vibrational) at high temperatures
diatomic molecule has 5 degrees of freedom

z
Note
y (i) Degrees of freedom of a diatomic and polyatomic gas depends
on temperature and since there is no clear cut demarcation line
above which vibrational energy become significant. Moreover,
x this temperature varies from gas to gas. On the other hand, for a
monoatomic gas there is no such confusion.
Degree of freedom here is 3 at all temperatures. Unless and until
stated in the question you can take f = 3 for monoatomic gas,
3 translational and 2 rotational. At sufficiently high f = 5 for a diatomic gas andf = 6 for a non-linear polyatomic gas.
temperatures it has vibrational energy as well providing it (ii) When a diatomic or polyatomic gas dissociates into atoms it
two more degrees of freedom (one vibrational kinetic behaves as a monoatomic gas. Whose degrees of freedom are
energy and another vibrational potential energy). Thus, at changed accordingly.
high temperatures a diatomic molecule has 7 degrees of
Telegram @unacademyplusdiscounts

510 JEE Main Physics

13.15 Internal Energy of an 13.16 Law of Equipartition of


Ideal Gas Energy
Suppose a gas is contained in a closed vessel as shown in An ideal gas is just like an ideal father. As an ideal father
figure. If the container as a whole is moving with some distributes whole of its assets equally among his children.
speed, then this motion is called the ordered motion of the Same is the case with an ideal gas. It distributes its internal
gas. Source of this motion is some external force. The energy equally in all degrees of freedom. In each degree of
1
zig-zag motion of gas molecules within the vessel is freedom energy of one mole of an ideal gas is RT , where
2
known as the disordered motion. This motion is directly
T is the absolute temperature of the gas. Thus, if f be the
related to the temperature of the gas. As the temperature is
number of degrees of freedom, the internal energy of
increased, the disordered motion of the gas molecules
f
gets fast. The internal energy (U) of the gas is concerned 1 mole of the gas will be RT or internal energy of n moles
2
only with its disordered motion. It is in no way concerned n
with its ordered motion. When the temperature of the gas of the gas will be fRT . Thus,
2
is increased, its disordered motion and hence its internal
n
energy is increased. U = fRT …(i)
2
Disordered
motion For a monoatomic gas, f = 3.
Ordered 3
Therefore, U = RT
motion 2
(for 1 mole of a monoatomic gas)
Intermolecular forces in an ideal gas is zero. Thus, PE due For a dia and linear polyatomic gas at low temperatures,
to intermolecular forces of an ideal gas is zero. A f = 5, so,
monoatomic gas is having a single atom. Hence, its 5
U = RT (for 1 mole)
vibrational energy is zero. For dia and polyatomic gases 2
vibrational energy is significant only at high temperatures.
and for non-linear polyatomic gas at low temperatures,
So, they also have only translational and rotational KE. We
f = 6, so
may thus, conclude that at room temperature the internal
6
energy of an ideal gas (whether it is mono, dia or poly) U = RT = 3RT (for 1 mole)
2
consists of only translational and rotational KE. Thus,
U ( of an ideal gas ) = K T + K R at room temperatures. Note From Eq. (i) we can see that internal energy of an ideal gas
Internal Energy depends only on its temperature and which is directly proportional to its
(U) absolute temperature T. In an isothermal processT = constant. Therefore,
the internal energy of the gas does not change or dU = 0.

Check Point 2
Potential Energy

1. Although the rms speed of gas molecules is of the order of the


Due to Due to Translational Rotational Vibrational speed of sound in that gas, yet on opening a bottle of
intermolecular interatomic KE KE KE ammonia in one corner of a room, its smell takes time in
forces Forces reaching the other corner. Explain why?
(vibrational)
2. The ratio of vapour densities on two gases at the same
temperature is 8 : 9. Compare the rms velocities of their
Later in the next article we will see that K T (translational molecules.
KE) and K R (rotational KE) depends on T only. They are 3. Can the temperature of a gas be increased keeping its pressure
directly proportional to the absolute temperature of the and volume constant?
gas. Thus, internal energy of an ideal gas depends only on its 4. On driving the scooter for a long time, the air pressure in the
absolute temperature (T ) and is directly proportional to T. tyres slightly increases. Why?
or U µT
Telegram @unacademyplusdiscounts

Heat and Kinetic Theory of Gases 511

Molar Specific Heat of the Gases Sample Problem 26 A flask contains argon and chlorine
in the ratio of 2 : 1 by mass. the temperature of the mixture is
Consider a container containing m gram of gas of
27° C. Root mean square speed vrms of the molecules of the two
molecular mass M. If n is the number of moles of gas in
gases is [Given, atomic mass of argon = 39.9 u, molecular mass
container, DQ is the heat supplied and rise in temperature of chlorine = 70.9 u] [NCERT]
is DT , then
1 DQ (a) 0.22 (b) 2.2
c= (c) 1.33 (d) 3.3
m DT
m Interpret (c) The average kinetic energy (per molecule) of any
Further, n= or m = nM
M 3
(ideal) gas is always equal to k BT . It depends only on temperature
1 DQ 2
so c=
nM DT and is independent of the nature of the gas. Since argon and
Thus, molar specific heat chlorine both have the same temperature in the flask, the ratio of
1 æ DQ ö average kinetic energy (per molecule) of the two gases is 1 : 1.
C = Mc = ç ÷ 1
2 è DT ø Now, 2
mv rms = average kinetic energy per molecule
2
We can write this relation as 3
[Molar specific heat (C) of the mass] = kBT
2
= [Molar mass (M) of the gas] 2
(v rms) Ar (m) Cl (M) Cl
´ [gram specific heat (c) of the gas] 2
= =
(v rms) Cl (M) Ar (M) Al
Molar specific heat has two kinds
70.9
(i) Specific heat at constant volume (CV ) When heat is = = 1.77
39.9
supplied to gas at constant volume the entire heat
where M denotes the molecular mass of the gas. Taking square root
supplied just increases the internal energy of gas and
(v rms) Ar
does nothing else. = 1.33
1 DQ ö (v rms) Cl
CV = æç ÷
n è DT øV
1 æ DU ö Sample Problem 27 A vessel contains two non-reactive
Thus, CV = ç ÷
n è DT ø gases neon (monoatomic) and oxygen (diatomic). The ratio of
their partial pressure is 3 : 2. The ratio of number of molecules
(ii) Specific heat at constant pressure (C p ) When heat is is [Given, atomic mass of Ne = 20.2 u, molecular mass of
supplied to the gas at constant pressure a part of it O 2 = 32 u] [NCERT]
increases the internal energy of the gas and remaining 2 3 4 3
(a) (b) (c) (d)
does an external work. 3 2 3 4
So, specific heat at constant pressure
1 DQ ö
Interpret (b) Each gas (assumed ideal) obeys gas laws. Since V
C p = æç ÷ and T are common to the two gases, we have
n è DT ø p
p1V = m1 RT
At constant pressure to increase in the internal energy of and p2V = m 2 RT
the gas by the same amount (as in case of heat supplied at p1 m1
constant volume), more amount of heat has to be =
p2 m 2
supplied. Hence, we conclude that
Here 1 and 2 refer to neon and oxygen respectivley.
C p > CV
p1 3
=
Note p2 2
● The relation betweenC p andCV is given by Mayer’s relation which is
m1 3
C p - CV = R Given =
m2 2
Here R is gas constant.
N
● C p and CV in terms of degrees of freedom f can be written as By definition m1 = 1
f NA
CV = R
2 N
and m2 = 2
C p = CV + R = R + R = æç + 1ö÷ R NA
f f
and
2 è2 ø where N1 and N2 are the number of molecules of 1 and 2, and NA
● Ratio of specific heats C p and CV is is Avogadro’s number.
C 2 N1 m1 3
g = p = 1+ = =
CV f N2 m 2 2
Telegram @unacademyplusdiscounts

512 JEE Main Physics

Hot Spot Specific Heat Capacity


of Monoatomic, Diatomic and Polyatomic Gases

The molecule of a monoatomic gas has only three translational Note Specific heat of lighter elements is higher than heaveir elements
degrees of freedom. Thus, the average energy of a molecule at and vice versa. Specific heat of the same substance in different states
3 (solid, liquid and vapour) is different. For example, specific heat of water is
temperature T is kBT . The total internal energy of a mole of such
2 1 cal g–1° C–1 and that of ice is 0.5 cal g–1° C–1.
gas is
3 3
U = kBT ´ N A = RT Table Values of f, U, C V , C p and g for Different Gases
2 2
Monoatomic Gases f dU f Cp
Nature of gas f U= RT CV = = R Cp = CV + R Y =
The molar specific heat at constant volume CV is 2 dT 2 CV
dU 3
C V (monoatomic gas) = = RT Monoatomic 3 3 3 5 1.67
dT 2 RT R R
2 2 2
From Mayer’s formula C p - CV = R 5 5 7
Di and linear 5 RT R R 1.4
where, C p is molar specific heat at constant pressure. Thus polyatomic 2 2 2
5
Cp = R Non-linear 6 3RT 3R 4R 1.33
2
polyatomic
Cp 5
Ratio of specific heat g = =
CV 3
Sample Problem 28 A cylinder of fixed capacity 44.8
Diatomic Gases litres contains helium gas at standard temperature and pressure.
The amount of heat needed to raise the temperrature of the gas
A diatomic molecule has 5 degree of freedom, 3 translational and 2
rotational. Using the law of equipartition of energy the total internal in the cylinder by 15°C is [Given, R = 8.31 J mol -1K -1]
energy of a mole of such a gas is (a) 45 J
5 5 (b) 374 J
U = kBT ´ N A = RT
2 2 (c) 273 J
The molar specific heats are then given by (d) 432 J
5 7 Interpret (b) From ideal gas law
C V (rigid diatomic) = R,C p = R
2 2
pV = mRT
7
g (rigid diatomic) =
5 1 mol of any (ideal) gas at standard temperature (273 K) and
pressure (1 atm = 1.01 ´ 10 5 Pa) occupies a volume of 22.4 litres. This
If the diatomic molecule is not rigid but has in addition a vibrational
mode universal volume is called molar volume. Thus, the cylinder in this
example contains 2 mol of helium. Further, since helium is
U = æç kBT + kBT ö÷ N A = RT
5 7
è2 ø 2 monoatomic, its predicted (and observed) molar specific heat at
3
7 9
C V = R ,C p = R , g = R
9 constant volume CV = R, and molar specific heat at constant
2 2 7 2
pressure.
Polyatomic Gases 3 5
Cp = R + R = R
A polyatomic molecule has 3 translational, 3 rotational degrees of 2 2
freedom and a certain number (f ) of vibrational modes. Since, volume of the cylinder is fixed, the heat required is
determined by CV .
From law of equipartition of energy, one mole of such a gas has
æ3 ö \ Heat required = Number of moles ´ Molar specific heat
3
U = çç kBT + kBT + f kBT ÷÷ N A ´ rise in temperature
è 2 2 ø
= 2 ´ 1.5 R ´ 15 = 45 R
C V = (3 + f )R ,
= 45 ´ 8.31
C p = (4 + f )R
(4 + f ) = 374 J
g=
(3 + f )
Telegram @unacademyplusdiscounts

Heat and Kinetic Theory of Gases 513

Sample Problem 29 A sphere of aluminium of 0.047 kg The number of molecules per unit volume can be
placed for sufficient time in a vessel containing boiling water, determined from Avogadro’s number and the ideal gas
so that the sphere is at 100°C. It is then immediately transferred law leading to
to 0.14 kg copper calorimeter containing 0.25 kg of water at nN A nN A N A p
nV = = =
20°C. The temperature of water rises and attains a steady state V nRT RT
at 23°C. The specific heat capacity of aluminium is [NCERT] p
(a) 0.911 kJ kg -1K -1 (b) 211 kJ kg -1K -1 RT
\ l=
(c) 423 kJ kg -1K -1 (d) 143 kJ kg -1K -1 2 pd 2N A p
Interpret (a) At a steady state, heat given by an aluminium
sphere will be equal to the heat absorbed by the water and
calorimeter. 13.18 Avogadro’s Number
Mass of aluminium sphere (mi ) = 0.047 kg
A mole (abbreviated mol) of a pure substance is a mass of
Initial temperatue of aluminium sphere = 100° C
the material in grams that is numerically equal to the
Final temperatuee = 23° C
molecular mass in atomic mass unit (amu). A mole of any
Change in temperature ( DT) = 100° C - 23° C = 77° C
material will contain Avogadro’s number of molecules.
Let the specific heat capacity of aluminium be S Al,
For example, carbon has an atomic mass of exactly 12.0
The amount of heat lost by the aluminium sphere
atomic mass units a mole of carbon is therefore 12 grams.
= m1S AlDT = 0.047 ´ S Al ´ 77
Mass of water (M2) = 0.25 kg One mole of an ideal gas will occupy a volume of
Mass of calorimeter (m3) = 0.14 kg 22.4 litres at STP.
Initial temperature of water and calorimeter = 20° C Avogadro’s number N A = 6.0221367 ´ 1023 mol
Final temperature of the mixture = 23° C
Change in temperature ( DT2) = 23° C - 20° C = 3° C Standard Temperature and Pressure
Specific heat capacity of water ( Sw ) = 4.18 ´ 10 3 Jkg –1 K –1
(STP)
Specific heat capacity of copper calorimeter
STP is used widely as a standard reference point for
= 0.386 ´ 10 3 Jkg –1 K –1
expression of the properties and processes of ideal gases.
The amount of heat gained by water and calorimeter The standard temperature is the freezing point of water
= m2Sw DT2 + m3 S Cu DT2 and the standard pressure is one standard atmosphere.
= 0.25 ´ 4.18 ´ 10 3 + 0.14 ´ 386 ´ 10 3 ´ (23 - 20) These can be quantified as follows :
In the steady state heat lost by the aluminium sphere Standard temperature 0° C = 273.15 K
= heat gained by water + calorimeter Standard pressure = 1atmosphere = 760 mm of Hg
So, 0.047 kg ´ S Al ´ 77° C
= 101.3 kPa
= 0.25 ´ 4.18 ´ 10 3 + 0.14 ´ 0.386 ´ 3
S Al = 0.911kJ kg –1 K –1 Standard volume of 1 mole of an ideal gas at STP = 22.4
litres.

Sample Problem 30 A vessel contains a mixture of 7 g of


13.17 Mean Free Path nitrogen and11g of carbon dioxide at temperature T = 300 K. If
Every gas consists of a very large number of molecules. the pressure of the mixture is 1 atm (1 ´ 10 5 N / m 2), its density is
These molecules are in a state of continuous rapid and (gas constant R = 2513 J / mol K)
random motion. They undergo perfectly elastic collision (a) 0.72 kg /m3 (b) 1.44 kg /m3
against one another. Therefore, path of a single gas (c) 2.88 kg /m3
(d) 5.16 kg /m3
molecule consists of a series of short zig-zig paths of
different lengths. The mean free path of a gas molecule is Interpret (b) The expression for density of a mixture of gases is
the average distance between two successive collisions. given by,
Mathematically it is expressed as pMmix
r mix =
1 RT
l= where, Mmix is the mass of mixture of non-reactive gases, p is
2 pd 2nV
pressure, R is gas constant and T is temperature.
where, d = diameter of molecules, n1M1 + n2 M2
nV = number of molecules per unit volume. Mmix =
n1 + n2
Telegram @unacademyplusdiscounts

514 JEE Main Physics

Given, Interpret (b) We have mass of ice m = 3 kg


Mass of nitrogen gas, mN = 7 g = 7 ´ 10 -3 kg Specific heat capacity of ice, S ice = 2100 Jkg –1 K –1
Mass of carbon dioxide, mCO2 = 11 g = 11 ´ 10 –3 kg
Specific heat capacity of water, S water = 4186 Jkg –1 K –1
Molecular weight of nitrogen gas, mN = 28 ´ 10 -3 kg
Latent heat of fusion of ice, Lice = 3.35 ´ 10 5 Jkg –1
Molecular weight of carbon dioxide, MCO2 = 44 ´ 10 -3 kg
Latent heat of steam, Lsteam = 2.256 ´ 10 6 Jkg –1
mN mCO2
MN + MCO2 Q = heat required to convert 3 kg of ice at –12°C to steam at
nNMN MN MCO2
Mmix = = 100°C
nN + nCO2 mN mCO2
+ Q1 = heat required to convert ice at –12°C to ice at 0°C
MN MCO2
= m S iceDT1 = (3 kg) (2100 Jkg –1 K –1) [0 - ( -12)] C
mN + mCO2 (7 + 11) ´ 10 -3
Mmix = = = 75600 J
mN mCO2 æ 7 11 ö
+ ç + ÷ Q 2 = heat required to melt ice at 0°C to water at 0°C
MN M CO2 è 28 44 ø
= mLfice = (3) (3.35 ´ 10 5 Jkg –1)
-3
18 ´ 10
Mmix = = 36 ´ 10 -3 kg = 1005000 J
1 1
+ Q3 = heat required to convert water at 0°C to water at 100°C
4 4
= mSw DT2 = (3 kg) (4186 Jkg –1 K –1) (100° C)
(1 ´ 10 5) (36 ´ 10 -3)
\ r= = 1.44 kg /m3
25 = 1255800 J
´ 300
3 Q 4 = heat required to convert water at 100°C to steam at 100°C
= mLsteam = (3 kg) (2.256 ´ 10 6)
Sample Problem 31 When 0.15 kg of ice at 0° C is mixed
with 0.30 kg of water at 50° C in a container the resulting = 6768000 J
temperature is 6.7°C. The heat of fusion of ice is So, Q = Q1 + Q 2 + Q3 + Q 4
(Swater = 4186 J kg -1K -1) [NCERT] = 75600 J+1005000 J+1255800 J + 6768000 J
3
(a) 1.45 ´ 10 J kg -1 5
(b) 3.34 ´ 10 J kg -1 = 9.1 ´ 10 6 J
(c) 5.23 ´ 10 6 J kg -1 (d) 6.23 ´ 10 7 J kg -1
Sample Problem 33 The temperature of the steel-copper
Interpret (b) Heat lost by water = msw (Q f - Qi )w junction in the steady state of the system as shown in the figure
= (0.30) (4186) (50 - 6.7) is [Given length of the steel rod = 15 cm, length of the copper
rod = 10 cm, temperature of furnace = 300° C, temperature of
= 54376.14 J
other end = 0° C, the area of cross-section of the steel rod is
Heat required to melt ice = m2 Lf = (0.15) Lf twice that of the copper rod, thermal conductivity of steel
Heat required to raise temperature 0 ice water to final = 50.2 Js-1m -1 K -1 and of copper = 385 Js-1 m -1 K -1] [NCERT]
temperature = m1 sw (Q f - Qi )i
= (0.15 kg) (4186 Jkg –1 K –1) (6.7 - 0) Furnace
Steel
Ice box
300°C 0°C
= 4206.93 J
Insulating
Heat lost = heat gained material copper
54376.14 J = (0.15 kg) Lf + 4206.93 J
(a) 4.4°C (b) 40°C
Lf = 3.34 ´ 10 5 Jkg –1
(c) 44.4°C (d) 52°C

Sample Problem 32 Heat required to convert 3 kg of ice Interpret (c) The insulating material around the rod reduces
at -12° C kept in a calorimeter to steam at 100° C at heat loss from the sides of the rods. Therefore, heat flows only along
atmospheric pressure. [Given specific heat capacity of ice the length of the rods. Consider any cross-section of the rod in the
= 2100 J kg -1 K -1, specific heat capacity of water steady state, heat flowing into the element must equal the heat
flowing out of it, otherwise there would be a net gain or loss of heat
= 4186 J kg -1 K -1, latent heat of fusion of ice by the element and its temperature would not be steady. Thus, in
= 3.35 ´ 10 5 J kg -1, and latent heat of steam the steady state, rate of heat flowing across a cross-seciton of the rod
= 2.256 ´ 106 J kg -1] is the same at every point along the length of the combined
[NCERT]
steel-copper rod. Let T be the temperature of the steel copper
(a) 1250 J (b) 9.1 ´ 10 6 J
junction in the stedy state. Then
(c) 9.1 ´ 10 –6 J (d) 1.250 ´ 10 3 J k1A1 (300 - T) k2A2 (T - 0)
=
L1 L2
Telegram @unacademyplusdiscounts

Heat and Kinetic Theory of Gases 515

where 1 and 2 refer to the steel and copper rod. For A1 = 2 A2, Interpret (a) Equilibrium of piston gives pa S
L1 = 15 cm, L2 = 10 cm, k1 = 50.2 Js–1 m–1 K –1, k2 = 385 Js–1 m–1 K –1, pS = pa S + mg + kx0
we have mg kx0
50.2 ´ 2 ´ (300 - T) 385 T p = pa + +
= S S
15 10 pS
(p = final pressure of gas)
which gives T = 44.4° C mg kx0
Work done by the gas is equal to work done against atmospheric
Sample Problem 34 A pan filled with hot food cools from pressure + elastic potential energy stored in the spring + increase
94° C to 86° C in 2 min, when the room temperature is at 20° C, in gravitational potential energy of the piston.
how long will it take to cool from 71° C to 69° C? 1 1
= pa DV + kx02 + mgx0 = pa Sx0 + kx02 + mgx0
[NCERT Exemplar] 2 2
This is also the decrease in internal energy of the gas, because the
(a) 14 s (b) 3 s (c) 42 s (d) 13 s gas is thermally insulated and this work is done at the expense of
Interpret (c) The average temperature of 94°C and 86°C is internal energy of the gas.
90°C, which is 70°C above the room temperature, under these
conditions the pan cools 8°C in 2 minutes, we have Sample Problem 37 Carbon
b
Change in temperature monoxide is carried around a closed p2
= kD T cycle abc, in which bc is an isothermal
Time
8° C process, as shown.
= K (70° C) …(i) The gas absorbs 7000 J of heat as its p1 c
2 min a
temperature is increased from 300 K to
The average of 69°C and 71 °C is 70°C, which is 50°C above 1000 K is going from a to b. The O V1 V2
V
room temperature K is the same for this situation is for the original quantity of heat ejected by the gas.
2° C process ca is
= K (50° C) …(ii)
Time (a) 4200 J (b) 5000 J
Dividing Eqs. (i) and (ii), we get (c) 9800 J (d) 3500 J
8° C/2 min K (70° C)
= ,T = 0.7 min = 42 s Interpret (c) Given that gas absorbs 7000 J of energy, hence
2° C/time K (50° C)
( DQ) ab = mCV DT
Sample Problem 35 A blacksmith fixes iron ring on the \ + 7000 = mCV (1000 - 300)
rim of the wooden wheel of a bullock cart. The diameter of the For the process ca
rim and the iron ring are 5.243 m and 5.231m respectively at Ta = 300 K
27° C. The temperature to which the ring be heated so as to fit Tc = Tb = 1000 K
the rim of the wheel is [NCERT]
( DQ) ca = mC p DT = mC p (300 - 1000)
(a) 100°C (b) 50°C
= - mC p ´ 700
(c) 218°C (d) 420°C
Also C p - CV = R
Interpret (c) Given, T = 27° C, LT1 = 5.231m,LT2 = 5.243 m \ C p = R + CV
So, ca VLT 2T = L 1 = [11+2 1a (T - T )] ( DQ) = -m (C + R) 700
5.243 = 5.231[1+1.20 ´ 10 –5 (T2 - 27° C)] For carbon monoxide,
Þ T2 = 218° C 2 2 7
g = 1+ = 1+ =
n 5 5
Sample Problem 36 In the R R 5R
CV = = =
arrangement shown in the figure gas is k g -1 7 -1 2
thermally insulated. An ideal gas is filled in 5
the cylinder having pressure p 0 (> m, S
Hence, we have
atmospheric pressure p a ). The spring of force
( DQ) ab = mCV 700
constant k is initially unstretched. The piston p0
5R
of mass m and area S is frictionless. In ( DQ) ab = m ´ 700 = 7000
equilibrium, the piston rises up a distance x0, 2
then the decrease in internal energy of the gas is given by 20
or mR = =4
1 4 5
(a) pa Sx0 + kx02 + mgx0 (b) pa Sx0 + x02 + 2 mgx0
2 5 \ ( DQ) ca = - (7000 + 4 ´ 700) = -9800 J
4 2 Negative sign shows that heat is ejected.
(c) 2 pa Sx0 + x02 + mgx0 (d) pa Sx0 + 2 kx02 + 2 mgx0
5 3
Telegram @unacademyplusdiscounts

WORKED OUT
Examples
Example 1 The coefficient of volume expansion of Fall in temperature of metal
glycerine is 49 ´ 10 -5° C -1 . What is the fractional change in DT = 150 - 40 = 110°C
density for a 30° C rise in temperature? If c is specific heat of the metal, then heat lost by the metal,
(a) 0.0155 (b) 0.0145 DQ = mcDT = 200 s ´ 110 …(i)
(c) 0.0255 (d) 0.0355 Volume of water150 cc
Mass of water, m¢ =150 g
Solution Here, g = 49 ´ 19 -5 ° C -1 Water equivalent of calorimeter
DT = 30° C w = 0.025 kg = 25g
V ¢ = V + DV = V (1 + g DT) Rise in temperature of water in calorimeter
\ V ¢ = V (1 + 49 ´ 10 -5 ´ 30) = 1.0147 V DT ¢ = 40 - 27 = 13°C
m r - 0.9855 r Heat gained by water and calorimeter
As r= , r¢ = = 0.0145
V r DQ ¢ = (m¢ + w) DT ¢
= (150 + 25) ´ 13
Example 2 How much should the temperature of a brass DQ¢ = 175 ´13 …(ii)
rod be increased so as to increase its length by 1%? Given a for As DQ = DQ ¢
brass is 0.00002°C -1. \ From Eqs. (i) and (ii)
(a) 300°C (b) 400°C 200 ´ s ´ 100 = 175 ´ 13
(c) 500° C (d) 550° C 175 ´ 13
s= » 0.1
DL 1 200 ´ 110
Solution Here, DT = ?, =
L 100
If some heat is lost to the surroundings, value of s so, obtained will
a = 0.00002°C-1 be less than the actual value of s.
As DL = aLDT
\ a DT =
DL Example 4 A geyser heats water glowing at the rate of
L 3.0 Lmin-1 from 27°C to 77°C. If the geyser operates on a gas
DL 1 burner, what is the rate of combustion of fuel, if its heat of
or DT =
4-1
=
La 100 ´ 0.00002 combustion is 4.0 ´ 10 Jg ?
10 5 (a) 25.75 gmin -1 (b) 10.75 gmin -1
DT = = 500°C
2 ´ 10 2 (c) 15.75 gmin -1 (d) 35.75 gmin -1

Solution Here, volume of water heated = 3.0 Lmin -1


Example 3 In an experiment on the specific heat of a metal,
Mass of water heated, m = 3000 gmin -1
a 0.20 kg block of the metal at 150°C is dropped in a copper
calorimeter (of water equivalent 0.025 kg) containing 150 cc of Rise of temperature, DT = 77 - 27 = 50° C
water at 27°C. The final temperature is 40ºC. Calculate the Specific heat of water, s = 4.2 Jg -1 ° C- 1
specific heat of the metal. If heat losses to the surroundings are Amount of heat used, DQ = msDT = 3000 ´ 4.2 ´ 50
not negligible, is our answer greater or smaller than the actual = 63 ´ 10 4 Jmin -1
value of specific heat of the metal?
Heat of combustion = 4 ´ 10 4 Jg -1
(a) 0.02 (b) 0.2
(c) 0.01 (d) 0.1 63 ´ 10 4
Rate of combustion of fuel =
4 ´ 10 4
Solution Here mass of metal,
= 15.75 gmin -1
m = 0.20 kg = 200 g
Telegram @unacademyplusdiscounts

Heat and Kinetic Theory of Gases 517

Y ´ a ´ Dl 0.0024
Example 5 A 19 kW drilling machine is used to drill a bore or F = = 0.91 ´ 10 11 ´ 3.142 ´ 10 -6 ´
in a small aluminium block of mass 8.0 kg. How much is the l2 1.8
rise in temperature of the block in 2.5 min assuming 50% of = 3.81 ´ 10 2 N
power is used up in heating the machine itself or lost to the
surrounding, specific heat of aluminium is 0.91 Jg -1 ° C -1 ? Example 7 From the following data, find the magnitude of
(a) 105°C (b) 103°C Joule's mechanical equivalent of heat : C p for hydrogen
(c) 106°C (d) 108°C = 3.409 cal g -1 C -1; Cv for hydrogen = 2.409 cal g -1° C -1 and
molecular weight of hydrogen = 2.
Solution Here, P = 10kW = 10 4 W
(a) J = 2.11 J cal -1 (b) J = 1.11 J cal -1
3
Mass, m = 8.0 kg = 8 ´ 10 g (c) J = 3.11 J cal -1 (d) J = 4.11 J cal -1
Rise in temperature DT = ?
Time, t = 2.5 min = 2.5 ´ 60 = 150 s Solution Here, J = ?
Specific heat, s = 0.91 Jg -1
°C -1 C p = 3.409 calg -1° C-1
Total energy P ´ t = 10 4 ´ 150 CV = 2.409 calg -1° C-1, M = 2
= 15 ´ 10 5 J R = 8.31 J mol-1K -1
As 50% of energy is lost. r R
As C p - CV = =
1 J MJ
\ Energy available, DQ = ´ 15 ´ 10 5
2 8.31
\ 3.409 - 2.409 =
= 7.5 ´ 10 5 J 2J
As DQ = msDT 4155
.
1=
DQ 7.5 ´ 10 5 J
\ DT = = = 103°C
ms 8 ´ 10 3 ´ 0.91 . J cal-1
J = 411

Example 6 A brass wire 1.8 m long at 27ºC is held taut with Example 8 An air bubble of volume 1.0 cm3 rises from the
little tension between two rigid supports. If the wire is cooled to bottom of a lake 40 m deep at a temperature of 12°C. To what
a temperature of - 39° C, what is the tension developed in the valume does if grow when it reaches the surface, which is at
wire, if the diameter is 2.0 mm? Coefficient of linear expansion temperature of 3°C?
of brass = 2.0 ´ 10 -5 °C -1 and Young's modulus of brass Given, 1 atm = 1.01 ´ 10 5Pa
= 0.91 ´ 1011 pascal (Nm -2). (a) 5.275 ´ 10 -6m3 (b) 6.275 ´ 10 -6m3
(a) 3.81 ´ 10 1 N (b) 3.81 ´ 10 2 N (c) 3.275 ´ 10 -6m3 (d) 4.275 ´ 10 -6m3
(c) 3.81 ´ 10 3 N (d) 3.81 ´ 10 4 N
Solution When the air bubble is at 40 m depth, then
Solution Here, l1 = 1.8 m; DT = ( -39) - 27 = - 66° C V1 = 1 cm3 = 1.0 ´ 10 - 6 m3

a = 2.0 ´ 10 -5 ° C-1 T1 = 12 ° C = 12 + 273 = 285 K


P1 = 1atm+ h1 r g
If l2 is length of the wire at - 39° C, then
= 1.01 ´ 10 5 + 40 ´ 10 3 ´ 9.8
l2 = l1 (1+ aDT) = 1.8 (1+ 2.0 ´ 10 -5 ´ ( -66)
= 493000 Pa
= 1.8 (1 - 1.32 ´ 10 3 ) = 1.7976 m
When the air bubble reaches at the surface of lake, then
Therefore, decrease in length, V2 = ?, T2 = 35° C = 35 + 273 = 308 K
Dl = l1 - l2 = 1.8 - 1.7976 = 0.0024 m P2 = 1atm = 1.01 ´ 10 5 Pa
Also, Y = 0.91 ´ 10 11 Pa (Nm-1)
p1V1 PV
Now, = 2 2
Diameter of wire, d = 2.0 min = 2.0 ´ 10 -3 m T1 T2
Therefore, area of cross-section of wire, p1V1T2
or V2 =
p d2 p T2 p2
a= = ´ (2.0 ´ 10 -3) 2 = 3.142 ´ 10 -6 m2
4 4 (493000) ´ 1.0 ´ 10 -6 ´ 308
\ V2 =
Now, Young's modulus of the material of the wire is given by 285 ´ 1.01 ´ 10 5
F/a F/l
Y= = = 5.275 ´ 10 - 6 m3
Dl / l2 a ´ Dl
Telegram @unacademyplusdiscounts

518 JEE Main Physics

Example 9. Estimate the total number of air molecules (i) Average speed,
(inclusive of oxygen, nitrogen, water vapour and other c1 + c2 + c3 + c4
cav =
constituents) in a room of capacity 25.0 m3 at a temperature of 4
27°C and 1 atm pressure. 2+ 4+ 6+ 8
= = 5 kms-1
(Boltzmann constant = 1.38 ´ 10 -23 JK -1) 4
(a) 5.10 ´ 10 26 (b) 4.10 ´ 10 26 (ii) Root mean square speed,
(c) 6.10 ´ 10 26 (d) 2.10 ´ 10 26 c12 + c22 + c32 + c42
c=
4
Solution Here, V = 25.0 m3
22 + 42 + 62 + 82
T = 27 + 273 = 300 K = = 5.48 kms-1
4
k = 1.38 ´ 10 -23 JK -1
Now, pV = nRT = n (Nk)T Example 11 What is the mean kinetic energy of one gram
= (nN)kT = N ¢ kT molecule of hydrogen at STP. Given density of hydrogen at STP
is 0.09 kgm -3
where nN = N ¢ = total number of air molcules in the given gas
(a) 3403.4 J (b) 4403.4 J
pV
N¢ = (c) 3203.4 J (d) 2403.4 J
kT
(1.01 ´ 10 5) ´25 Solution Here, r = 0.09 kgm-3
=
(1.38 ´ 10 -23) ´ 300 At STP, pressure p = 1.01 ´ 10 5 Pa
26
= 6.10 ´ 10 According to kinetic theory of gases,
1
p = rc 2
Example 10 Four molecules of a gas have speed 3
2, 4, 6, 8 kms-1 respectively. Calculate (i) average speed and
3p 3 ´ 1.01 ´ 10 5
(ii) root mean square speed. or c= = = 1837.5 ms-1
r 0.09
(a) 5 kms -1, 5.48 kms -1
Volume occupied by one mole of hydrogen at STP
(b) 4 kms -1, 3.48 kms -1
(c) 5 kms -1 , 8.48 kms -1 = 22.4 L = 22.4 ´ 10 -3 m3
(d) 4 kms -1, 2.48 kms -1 \ Mass of hydrogen,
-1 -1 M = Volume ´ Density
Solution Here, c1 = 2 kms ; c2 = 4 kms
= 22.4 ´ 10 -3 ´ 0.09
c3 = 6 kms-1 and c4 = 8 kms-1
= 2.016 ´ 10 -3 kg
Telegram @unacademyplusdiscounts

Start Practice for


JEE Main
Round I (Topically Divided Problems)

Thermometry and Calorimetry 7. If the ratio of densities of two substances is 5 : 6 and


1. Two absolute scales A and B have triple points of that of the specific heats is 3 : 5. Then, the ratio
water defined to be 200 A and 350 B. What is the between heat capacities per unit volume is
relation between TA and TB ? [NCERT] (a) 1 : 1 (b) 2 : 1
TA 4 TA 3 TA 7 TA 7 (c) 1 : 2 (d) 1 : 3
(a) = (b) = (c) = (d) =
TB 7 TB 7 TB 3 TB 4 8. Heat capacity of a substance is infinite. It means
(a) heat is given out
2. A faulty thermometer has its fixed points marked 5
(b) heat is taken in
and 95. When this thermometer reads 68, the correct
(c) no change in temperature whether heat is taken in or
temperature in celsius is
given out
(a) 68°C (b) 70°C (c) 66°C (d) 72°C (d) All of the above
3. The Fahrenheit and Kelvin scales of temperature 9. A cylinder containing an ideal gas is in vertical
will give the same reading at position and has a piston of mass M that is able to
(a) – 40 (b) 313 (c) 574.25 (d) 732.75 move up or down without friction. If the temperature
4. An amount of water of mass 20 g at 0°C is mixed with 40 is increased. [NCERT Exemplar]
g of water at 10°C, final temperature of the mixture is
(a) 5°C (b) 0°C
(c) 20°C (d) 6.66°C

5. The graph between two 180


Temperature (°A)

temperature scales A
and B is shown in figure. ∆tA =150°
Between upper fixed
point and lower point (a) both p and V of the gas will change
there are 150 equal ∆tB =100° (b) only p will increases according to Charles' law
division on scale A and 0 Temperature (°B)100 (c) V will change but not p
100 on scale B. The (d) p will change but not V
relationship for conversion between the two scales is 10. Water falls from a height of 500 m. What is the rise in
given by [NCERT Exemplar] temperature of water at the bottom if whole energy is
t - 180 t B t - 30 t B used up in heating water ?
(a) A = (b) A =
100 150 150 100 (a) 0.96°C (b) 1.02°C
t B - 180 t A t B - 40 t A (c) 1.16°C (d) 0.23°C
(c) = (d) =
150 100 100 180 11. 540 g of ice at 0°C is mixed with 540 g of water at
6. One gram of ice is mixed with one gram of steam. At 80°C. The final temperature of the mixture is
thermal equilibrium the temperature of mixture is (a) 0°C (b) 53°C
(a) 0°C (b) 100°C (c) 55°C (d) 80°C (c) 80°C (d) less than 0°C
Telegram @unacademyplusdiscounts

520 JEE Main Physics

12. Which one of the following would raise the 18. As the temperature is increased, the time period of a
temperature of 20 g of water at 30°C most when pendulum [NCERT Exemplar]
mixed with it? (a) increases as its effective length increases even though its
(a) 20 g of water at 40°C (b) 40 g of water at 35°C centre of mass still remains at the centre of the bob
(c) 10 g of water at 50°C (d) 4 g of water at 80°C (b) decreases as its effective length increases even though its
centre of mass still remains at the centre of the bob
13. A metal sphere of radius r and specific heat c is
(c) increases as its effective length increases due to shifting of
rotated about an axis passing through its centre at a
centre of mass below the centre of the bob
speed of n rotations per second. It is suddenly
(d) decreases as its effective length increases remains same
stopped and 50% of its energy is used in increasing
but the centre of mass shifts above the centre of the bob
its temperature. Then the rise in temperature of the
sphere is
2 p2 n2 r2 1 p2 n2 Thermal Expansion of Solids and Liquids
(a) (b)
5 c 10 r2 c 19. What should be the lengths of a steel and copper rod
7 2 2 é prn ù
2 at 0°C so that the length of the steel rod is 5 cm longer
(c) pr n c (d) 5 ê ú than the copper rod at any temperature ?
8 ë14 c û
a (Steel) = 1.1 ´ 10–5 ° C–1
14. Volume versus temperature graphs for a mass of an a (Copper) = 1.7 ´ 10–5 ° C
ideal gas are shown in figure at two different values
(a) 14.17 cm; 9.17 cm (b) 9.17 cm, 14.17 cm
of constant pressure. What can be inferred about
(c) 28.34 cm; 18.34 cm (d) 14.17 cm, 18.34 cm
relation between P1 and P2 ? [NCERT Exemplar]
20. When a liquid in a glass vessel is heated, its apparent
V (L ) expansion is 10.30 ´ 10–4 C–1. When the same liquid
p2 is heated in a metal vessel, its apparent expansion is
40
10.06 ´ 10–4 °C–1. If the coefficient of linear expansion
30 p1 of glass = 9 ´ 106 °C–1, what is the coefficient of linear
20 expansion of metal?
10 (a) 51 ´ 10 -6 ° C–1 (b) 17 ´ 10 -6 ° C–1
T (K) -6 –1
100 200 300 400 500 (c) 25 ´ 10 °C (d) 43 ´ 10 –6 ° C–1
(a) p1 > p2 (b) p1 = p2 21. A steel wire of uniform area 2 mm2 is heated upto
(c) p1 < p2 (d) data is insufficient 50°C and is stretched by tying its ends rigidly. The
15. A sphere, a cube and a thin circular plate, all of same change in tension when the temperature falls from
material and same mass are initialy heated to same 50°C to 30°C is
high temperature. [NCERT Exemplar] (Take Y = 2 ´ 1011 Nm -2 , a = 1.1 ´ 10–5 ° C–1)
(a) Plate will cool fasted and cube the slowest (a) 1.5 ´ 1010 N (b) 5 N
(b) Sphere will cool fasted and cube the slowest (c) 88 N (d) 2.5 ´ 1010 N
(c) Plate will cool fasted and sphere the slowest
(d) Cube will cool fastest and plate the slowest
22. Density of substance at 0°C is 10 g/cc and at 100°C,
its density is 9.7 g/cc. The coefficient of linear
16. When the room temperature becomes equal to the expansion of the substance is
dew point the relative humidity of the room is (a) 1.03 × 10–4°C -1 (b) 3 ´ 10 -4 °C -1
(a) 100% (b) 0% (c) 19.7 ´ 10 –3°C -1 (d) 10 -3 °C -1
(c) 70% (d) 85%
23. A rectangular block is heated from 0°C to 100°C. The
17. An aluminium sphere is dipped into water. Which of percentage increase in its length is 0.2%. What is the
the following is true? [NCERT Exemplar] percentage increase in its volume?
(a) Buoyancy will be less in water at 0° C than that is water at (a) 0.6% (b) 0.10% (c) 0.2% (d) 0.4%
4° C
(b) Buoyancy will be more in water at 0° C than that is water 24. A cubic vessel (with faces horizontal + vertical)
at 4° C contains an ideal gas at NTP. The vessel is being
(c) Buoyancy in water at 0° C will be same as that in water carried by a rocket which is moving at a speed of
at 4° C 500 ms -1 in vertical direction. The pressure of the
(d) Buoyancy may be more or less in water at 4°C depending gas inside the vessel as observed by us on the ground
on the radius of the sphere [NCERT Exemplar]
Telegram @unacademyplusdiscounts

Heat and Kinetic Theory of Gases 521

(a) remains the same because 500 ms -1 is very much smaller 31. A vertical column 50 cm long at 50°C balances
than Vrms of the gas another column of same liquid 60 cm along at 100°C.
(b) remains the same because motion of the vessel as a whole The coefficient of absolute expansion of the liquid is
does not affect the relative motion of the gas molecules (a) 0.005/°C (b) 0.0005/°C
and the walls (c) 0.002/°C (d) 0.0002/°C
(c) will increase by a factor equal to ( v2rms + (500 )2 / v2rms
32. A bar of iron is 10 cm at 20°C. At 19°C it will be (a of
where v rms was the original mean square velocity of the
gas
iron = 11 ´ 10-6 /°C)
(d) will be different on the top wall and bottom wall of the (a) 11 ´ 10 -6 cm longer (b) 11 ´ 10 -6 cm shorter
vessel (c) 11 ´ 10 -5 cm shorter (d) 11 ´ 10 -5 cm longer

25. A metal rod having linear expansion coefficient 33. The radius of a metal sphere at room temperature T
-5 is R, and the coefficient of liner expansion of the
2 ´ 10 °C–1 has a length of 1 m at 20°C. The
metal is a . The sphere is heated a little by a
temperature at which it is shortened by 1 mm is temperature DT so that its new temperature is
(a) –20°C (b) –15°C T + DT. The increase in the volume of the sphere is
(c) –30°C (d) –25°C approximately [NCERT Exemplar]
26. A bimetallic strip is made of aluminium and steel (a) 2p Ra DT (b) p R2 a DT
(a Al > a steel ). On heating, the strip will (c) 4 p R3a DT /3 (d) 4 pR3aDT
[NCERT Exemplar]
(a) remain straight 34. The volume of a metal sphere increases by 0.24%
(b) get twisted when its temperature is raised by 40°C. The
(c) will bend with aluminium on concave side coefficient of linear expansion of the metal is …°C.
(d) will bend with steel on concave side (a) 2 ´ 10 –5 per°C (b) 6 ´ 10 -5 per°C
(c) 2.1 ´ 10 –5 per°C (d) 1.2 ´ 10 –5 per°C
27. A bimetallic is made of two strips A and B having
coefficients of linear expansion a A and a B . If
a A < a B , then on heating, the strip will Thermal Conduction and Convection
(a) bend with A on outer side 35. A wall has two layers A and B, made of two different
(b) bend with B on outer side materials. The thermal conductivity of material A is
(c) not bend at all twice that of B. If the two layers have same thickness
(d) None of the above and under thermal equilibrium, the temperature
28. A clock with an iron pendulum keeps correct time at difference across the wall is 48°C, the temperature
15°C. What will be the error in second per day, if the difference across layer B is
room temperature is 20°C? (a) 40°C (b) 32°C (c) 16°C (d) 24°C
(The coefficient of linear expansion of iron is 36. Two plates of same
0.000012°C–1.) thickness, of coefficients of
K1
A1
(a) 2.6 s (b) 6.2 s thermal conductivity K 1 Q
(c) 1.3 s (d) 3.1 s 1 K2 Q2
and K 2 and areas of cross
A2
29. A uniform metallic rod rotates about its section A1 and A2 are
perpendicular bisector with constant angular speed. connected as shown in
If it is heated uniformly to raise its temperature figure. The common coefficient of thermal
slightly conductivity K will be
(a) its speed of rotation increases K1 A1
(a) K1 A1 + K2 A2 (b)
(b) its speed of rotation decreases K2 A2
(c) its speed of rotation remains same K1 A1 + K2 A2 K A + K2 A1
(c) (d) 1 2
(d) its speed increases because its moment of inertia increases A1 + A2 K1 + K2
30. A uniform metal rod is used as a bar pendulum. If the 37. Ice starts forming in a lake with water at 0°C, when
room temperature rises by 10°C and coefficient of the atmospheric temperature is –10°C. If time taken
linear expansion of the metal of the rod is 2 ´ 106 °C–1 for 1 cm of ice to be formed is 7 h, the time taken for
the period of pendulum will increase by the thickness of ice to increase from 1 cm to 2 cm is
(a) 1 ´ 10 -3 % (b) -1 ´ 10 -3 % (a) 7 h
(b) less than 7 h
(c) 2 ´ 10 -3 % (d) -2 ´ 10 -3 %
(c) more than 7 h but less than 14 h
(d) more than 14 h
Telegram @unacademyplusdiscounts

522 JEE Main Physics

38. When a bimetallic strip is heated, it 44. Three rods of material X and three rods of material Y
(a) does not bend at all are connected as shown in figure. All are identical in
(b) gets twisted in the form of an helix length and cross-sectional area. If end A is
(c) bend in the form of an arc with the more expandable maintained at 60°C, end E at 10°C, thermal
metal outside conductivity of X is 0.92 cals–1 cm–1°C–1 and that of Y
(d) bends in the form of an arc with the more expandable is 0.46 cals–1 cm–1°C–1, then find the temperatures of
metal inside junctions B, C, D.
C
39. Four rods of different radii r and length l are used to
connect two reservoirs of heat at different x x
temperatures. Which one will conduct heat fastest? x
A y E
(a) r = 2 cm, l = 0.5 m (b) r =1 cm, l = 0.5 m B
(c) r = 2 cm, l = 2 m (d) r = 1 cm, l = 1 m y y
40. Two rods of equal length and area of cross-section are D
kept parallel and lagged between temperatures 20°C (a) 20°C, 30°C, 20°C (b) 30°C, 20°C, 20°C
and 80°C. The ratio of the effective thermal (c) 20°C, 20°C, 30°C (d) 20°C, 20°C, 20°C
conductivity to that of the first rod is
é æ K1 ö 3 ù 45. A cylindrical rod with one end in
ê the ratio ç ÷= ú a steam chamber and the other θ1
ë è K2 ø 4 û R2
end in ice results in melting of
(a) 7 : 4 (b) 7 : 6 θ
0.1 g of ice per second. If the rod
(c) 4 : 7 (d) 7 : 8 R1
is replaced by another with half
θ2
41. Two rods of same length and material transfer a the length and double the
given amount of heat in 12 s, when they are joined radius of the first and if the
end to end (i.e., in series). But when they are joined in thermal conductivity of the material of the second rod
parallel, they will transfer same heat under same is 1/4 that of the first, the rate at which ice melts in
conditions in gs–1 will be
(a) 24 s (b) 3 s (a) 3.2 (b) 1.6 (c) 0.2 (d) 0.1
(c) 48 s (d) 1.5 s 46. Consider two insulating sheets with thermal
42. The coefficient of thermal conductivity of copper is resistances R1 and R2 as shown in figure. The
nine times that of steel. In the composite cylindrical temperature q is
bar shown in figure, what will be the temperature at q1R2 + q2 R1
(a)
the junction of copper and steel? R1 + R2 θ1
( q1 + q2 ) R1R2 R2
(b)
100°C Copper Steel 0°C R21 + R22 θ
q R + q2 R2 R1
18 cm 6 cm (c) 1 1 θ2
R1 + R2
(a) 75°C (b) 67°C q1q2 R1R2
(d)
(c) 33°C (d) 25°C ( q1 + q2 ) ( R1 + R2 )
43. Five rods of same dimensions C
47. Two rods P and Q have equal lengths. Their thermal
are arranged as shown in K1 conductivities are K 1 and K 2 and cross-sectional
K2
figure. They have thermal areas are A1 and A2 . When the temperature at ends
conductivities K 1, K 2 , K 3, K 4 A K5 B of each rod are T1 and T2 respectively, the rate of flow
and K 5 . When points A and B of heat through P and Q will be equal, if
are maintained at different K3 K4 A1 K2 A1 K2 T2
temperatures, no heat would (a) = (b) = ´
D
A2 K1 A2 K1 T1
flow through central rod, if 2
(a) K1K4 = K2 K3 A1 K1 A1 æ K2 ö
(c) = (d) =ç ÷
(b) K1 = K4 and K2 = K3 A2 K2 A2 è K1 ø
K K
(c) 1 = 2 48. If l is length, A is the area of cross-section and K is
K4 K3
thermal conductivity, then the thermal resistance of
(d) K1 K2 = K3 K4 the block is given by
(a) K l A (b) 1/ KlA (c) l + KA (d) l / KA
Telegram @unacademyplusdiscounts

Heat and Kinetic Theory of Gases 523

49. The amount of heat conducted out per second 56. The rate of radiation of a black body at 0°C is E watt.
through a window, when inside temperature is 10°C The rate of radiation of this body at 273°C will be
and outside temperature is –10°C, is 1000 J. Same (a) 16 E (b) 8 E (c) 4 E (d) E
heat will be conducted in through the window, 57. Two circular discs A and B with equal radii are
when outside temperature is –23°C and inside blackened. They are heated to same temperature and
temperature is are cooled under identical conditions. What inference
(a) 23°C (b) 230 K do you draw from their cooling curves, shown below?
(c) 270 K (d) 296 K A
50. Two identical square rods of metal are welded end to
end as shown in figure (i) 20 cal of heat flows through R
B
it in 4 minutes. If the rods are welded as shown in
figure (ii) the same amount of heat will flow through
the rods in [NCERT Exemplar]
(θ–θ0)
0°C
(a) A and B have same specific heats
0°C 100°C 100°C (b) specific heat of A is less
(c) specific heat of B is less
(a) 1 min (b) 2 min (c) 4 min (d) 16 min
(d) nothing can be said
51. The ratio of thermal conductivity of two rods is 5 : 4.
The ratio of their cross-sectional areas is 1 : 1 and
58. The temperature of a black body is increased by 50%,
then the percentage of increase of radiation is
they have the same thermal resistances. The ratio of
approximately
their lengths, must will be
(a) 100% (b) 25% (c) 400% (d) 500%
(a) 4 : 5 (b) 9 : 1 (c) 1 : 9 (d) 5 : 4
52. In heat transfer which method is based on 59. A body cools from 80°C to 50°C in 5 min. Calculate
the time it takes to cool from 60°C to 30°C. The
gravitation
temperature of the surroundings is 20°C. [NCERT]
(a) Natural convection (b) Conduction
(c) Radiation (d) Stirrling of liquid (a) 9 min (b) 7 min (c) 8 min (d) 10 min

53. If a liquid is heated in weightlessness the heat is 60. The frequency ( nm ) y

transmitted through corresponding to which D


A
(a) conduction
energy emitted by a black vm C
(b) convection
body is maximum may vary
B
(c) radiation
with temperature T of the
(d) neither because the liquid cannot be heated in
body as shown in figure.
weightlessness Which of the curves T
x
represents correct variation?
(a) A (b) B (c) C (d) D
Thermal Radiation ; Stefan’s Law, Wien’s
61. If temperature of a black body increases from 7°C to
Law and Newton’s Law of Cooling 287°C, then the rate of energy radiation increases by
54. The wavelength of maximum intensity of radiation (a) (287 /7) 4 (b) 16 (c) 4 (d) 2
emitted by a star is 289.8 nm and the radiation
intensity for the star is 62. A black body is at a temperature of 2880 K. The
(Stefan’s constant = 5.67 ´ 10–8 Wm –2K –2 , Wien’s energy of radiation emitted by this object with
constant b = 2878 mK). wavelength between 499 nm and 500 nm is U1 and
between 999 nm and 1000 nm is U2 . The Wien
(a) 5.67 ´ 10 8 Wm–2 (b) 5.67 ´ 10 –12 Wm–2
constant = 2.88 ´ 106 nm K. Then
(c) 10.67 ´ 107 Wm–2 (d) 10.67 ´ 1014 Wm–2
(a) U1 = 0 (b) U3 = 0
55. A polished metal plate with a rough black spot on it is (c) U1 > U2 (d) U2 > U1
heated to about 1400 K and quickly taken to a dark 63. If wavelength of maximum intensity of radiation
room. The spot will appear emitted by sun and moon are 0.5 ´ 10–6 m and 10–4 m
(a) darker than plate (b) brighter than plate respectively, the ratio of their temperatures is
(c) equally bright (d) equally dark (a) 1 : 100 (b) 1 : 200 (c) 200 : 1 (d) 400 : 1
Telegram @unacademyplusdiscounts

524 JEE Main Physics

64. The maximum energy in the thermal radiation from 71. A metallic sphere cools from 50°C to 40°C in 300 s. If
a hot source occurs at l = 11 ´ 10–5 cm. If the room temperature is 20°C, then its temperature
temperature of another source is n times, for which in next 5 min will be
wavelength of maximum energy is 5.5 ´ 10–5 cm, (a) 38°C (b) 33.3°C (c) 30°C (d) 36°C
then n is 72. A black body radiates heat energy at the rate of
1
(a) 2 (b) 4 (c) (d) 1 2 ´ 105 Js–1m–2 at a temperature 127°C. The
2
temperature of black body, at which the rate of heat
65. A black body radiates at two temperatures T1 and T2 , radiation is 32 ´ 105 Js–1m–2 is
such that T1 < T2 . The frequency corresponding to (a) 273°C (b) 527°C (c) 873°C (d) 927°C
maximum intensity is
73. A liquid is filled in a container which is kept in a room
(a) less at T1 (b) more at T1
whose temperature is 20°C. When temperature of
(c) equal in the two cases (d) cannot say
liquid is 80°C, it emits heat at the rate of 45 cals–1.
66. An object is cooled from 75°C to 65°C in 2 min in a When temperature of liquid falls to 40°C, its rate of
room at 30°C. The time taken to cool another heat loss will be
identical object from 55°C to 45°C in the same room, (a) 15 cals–1 (b) 30 cals–1
in minutes is (c) 45 cals–1 (d) 60 cals–1
(a) 4 (b) 5 (c) 6 (d) 7
74. The maximum wavelength of radiation emitted at
67. A black body at 1373°C emits maximum energy 2000 K is 4 mm. What will be the maximum
corresponding to a wavelength of 1.78 micron. The wavelength emitted at 2400 K?
temperature of moon for which l m = 14 micron would (a) 3.3 mm (b) 0.66 mm
be (c) 1 m (d) 1 mm
(a) 62.6°C (b) –58.9°C (c) 63.7°C (d) 64.2°C
75. Two bodies A and B are placed in an evacuated vessel
68. A planet is at an average distance d from the sun and maintained at a temperature of 27°C. The
its average surface temperature is T. Assume that temperature of A is 327°C and that of B is 227°C. The
the planet receives energy only from the sun, and ratio of heat loss from A and B is about
loses energy only through radiation from its surface. (a) 2 : 1 (b) 4 : 1
Neglect atmospheric effects. If T µ d- n ,the value of n (c) 1 : 2 (d) 1 : 4
is
76. The reflectance and emittance of a perfectly black
(a) 2 (b) 1 (c) 1/2 (d) 1/4
body are respectively
69. The rectangular surface of area 8 cm ´ 4 cm of a black (a) 0, 1 (b) 1, 0
body at a temperature of 127°C emits energy at the (c) 0.5, 0.5 (d) 0, 0
rate of E per second. If the length and breadth of the
77. The rate of emission of radiation of a black body at
surface are each reduced to half of its initial value,
temperature 27°C is E1. If its temperature is
and the temperature is raised to 327°C, the rate of
increased to 327°C, the rate of emission of radiation
emission of energy will become
is E2 . The relation between E1 and E2 is
3 81
(a) E (a) E2 = 24 E1 (b) E2 = 16 E1
8 16
(c) E2 = 8 E1 (d) E2 = 4 E1
9 81
(c) E (d) E
16 64 78. The rates of heat radiation from two patches of skin
each of area A, on a patient’s chest differ by 2%. If the
70. The plots for intensity versus wavelength for three
patch of the lower temperature is at 300 K and
black bodies at temperatures T1, T2 , T3 respectively
emissivity of both the patches is assumed to be unity,
are shown in figure. Their temperatures are such
the temperature of other patch would be
that
(a) 306 K (b) 312 K
T3
T2 (c) 308.5 K (d) 301.5 K
I T1
79. The rays of sun are focussed on a piece of ice through
a lens of diameter 5 cm, as a result of which 10 g ice
melts in 10 min. The amount of heat received from
λ sun, per unit area per min is
(a) T1 > T2 > T3 (b) T1 > T3 > T2 (a) 4 cal cm–2 min–1 (b) 40 cal cm–2 min–1
(c) T2 > T3 > T1 (d) T3 > T2 > T1 (c) 4 Jm–2 min (d) 400 cal cm–2 min–1
Telegram @unacademyplusdiscounts

Heat and Kinetic Theory of Gases 525

80. Solar radiation emitted by sun resembles that 87. If a given mass of gas occupies a volume of 100 cc at
emitted by a black body at a temperature of 6000 K. 1 atm pressure and temperature of 100°C (373.15 K).
Maximum intensity is emitted at a wavelength of What will be its volume at 4 atm pressure; the
about 4800Å. If the sun were cooled down from temperature being the same?
6000 K to 3000 K, then the peak intensity would (a) 100 cc (b) 400 cc
occur at a wavelength of (c) 25 cc (d) 104 cc
(a) 4800 Å (b) 9600 Å 88. 1 mole of H2 gas is contained in a box of volume
(c) 2400 Å (d) 19200 Å . m 3 at T = 300K. The gas is heated to a
V = 100
81. When the temperature of a black body increases, it is temperature of T = 3000K and the gas gets converted
observed that the wavelength corresponding of to a gas of hydrogen atoms The final pressure would
maximum energy changes from 0.26 mm to 0.13 mm to be (considering all gases to be ideal) [NCERT Exemplar]
a body at the respective temperature. Then ratio of (a) same as the pressure initially
E (b) 2 times the pressure initially
the emissivities 2 is
E1 (c) 10 times the pressure initially
(a) 16/1 (b) 4/1 (c) 1/4 (d) 1/16 (d) 20 times the pressure initially
82. The energy emitted per second by a black body at 89. Two gases A and B having the same temperature T,
27°C is 10 J. If the temperature of black body is same pressure p and same volume V are mixed. If the
increased to 327°C, the energy emitted per second mixture is at same temperature T and occupies a
will be volume V, the pressure of the mixture is
(a) 20 J (b) 40 J (c) 80 J (d) 160 J (a) 2 p (b) p
83. A black body at a temperature of 327°C radiates (c) p/2 (d) 4 p
4 cal cm–2s–1. At a temperature of 927°C, the rate of 90. When a gas filled in a closed vessel is heated through
heat radiated per unit area in cal cm–2s–1 will be 1°C, its pressure increases by 0.4%. The initial
(a) 16 (b) 32 (c) 64 (d) 128 temperature of the gas was
84. The temperature of coffee in a cup with time is most (a) 250 K (b) 2500K
likely given by the curve in figure. (c) 250°C (d) 25°C
91. A vessel of volume V contains a mixture of 1 mole of
θ hydrogen and 1 mole of oxygen (both considered as
(a) θ (b) ideal). Let f1 (v) dv, denote the fraction of molecules
with speed between v and (v + dv) with f2 (v) dv,
t(time) t(time) similarly for oxygen. Then [NCERT Exemplar]
(a) f1 ( v ) + f2 ( v ) = f ( v ) obeys the Maxwell's distribution law
(b) f1 ( v ), f2 ( v ) will obey the Maxwell's distribution law
θ θ separately
(c) (d) (c) Neither f1 ( v ), nor f2 ( v ) will obey the Maxwell's
distribution law
t(time) t(time) (d) f2 ( v ) and f1 ( v ) will be the same

85. A solid cube and a solid sphere have equal surface 92. An inflated rubber balloon contains one mole of an
areas. Both are at the same temperature of 120°C. ideal gas, has a pressure p, volume V and
Then temperature T. If the temperature rises to 1.1 T, and
(a) both of them will cool down at the same rate the volume is increased to 1.05 V. the final pressure
(b) the cube will cool down faster than the sphere will be [NCERT Exemplar]
(c) the sphere will cool down faster than the cube (a) 1.1 p (b) p
(d) whichever of the two is heavier will cool down faster (c) less than p (d) between p and 1.1 p
86. A surface at temperature T0 K receives power P by 93. The air density at Mount Everest is less than that at
radiation from a small sphere at temperature T > T0 the sea level. It is found by mountainers that for one
and at a distance d. If both T and d are doubled, the trip lasting few hours, the extra oxygen needed by
power received by the surface will become them corresponds to 30000 cc at sea level (pressure
(a) P (b) 2 P 1 atm, temperature 27°C). Assuming that the
(c) 4 P (d) 16 P temperature around Mount Everest is –73°C and
Telegram @unacademyplusdiscounts

526 JEE Main Physics

that the pressure cylinder has capacity of 5.2 L, the 100. At room temperature, the rms speed of the molecules
pressure at which oxygen be filled (at site) in the of a certain diatomic gas is found to be 1930 ms–1.
cylinder is The gas is
(a) 3.86 atm (b) 5.00 atm (a) H2 (b) F2
(c) 5.77 atm (d) 1 atm (c) O2 (d) Cl2
94. A fixed amount of nitrogen gas (1 mole) is taken and 101. Calculate the rms speed of smoke particles each of
is subjected to pressure and temperature variation. mass 5 ´ 10-17 kg in their Brownian motion in air at
The experiment is performed at high pressures as NTP ( k = 1.38 ´ 10–23 JK –1 )
well as high temperature. The result obtained are (a) 1.5 mm s–1 (b) 1.5 ms–1
pV (c) 1.5 cms–1 (d) 1.5 kms–1
shown in the figure. The correct variation of with
RT
102. At a certain temperature, the ratio of the rms velocity
p will be exhibited by
of H2 molecules to O2 molecule is
4 3 (a) 1 : 1 (b) 1 : 4
2.5
2 (c) 4 : 1 (d) 16 : 1
2.0
pV 103. An oxygen cylinder of volume 30 L has an initial
RT 1.5 gauge pressure of 15 atm and a temperature of 27°C.
1.0 1
After some oxygen is withdrawn from the cylinder
0.5 the gauge pressure drops to 11 atm and its
0.0
temperature drops to 17°C. The mass of oxygen
100 200 300 400 500 600 taken out of the cylinder ( R = 8.31Jmol –1 K -1) .
p (in atm)
molecular mas of O2 = 32 u) is [NCERT]
(a) curve (4) (b) curve (3) (c) curve (2) (d) curve (1) (a) 0.14 g (b) 0.02 g
95. How much should the pressure be increased in order (c) 0.14 kg (d) 0.014 kg
to decrease the volume of a gas by 5% at a constant 104. RMS velocity of a particle is c at pressure p. If
temperature? pressure is increased two times, then rms velocity
(a) 5% (b) 5.26% (c) 10% (d) 4.26% becomes
96. An ideal gas is found to obey an additional law (a) 0.5 c (b) c (c) 2 c (d) 3 c
2
pV = constant. The gas is initially at temperature 105. If the molecular weight of two gases are M1 and M2 ,
T and volume V. Then it expands to a volume 2 V, then at a temperature the ratio of rms velocity c1 and
the temperature becomes c2 will be
(a) T / 2 (b) 2 T (c) 2 T / 2 (d) 4 T æM ö
1 /2
æM ö
1 /2
(a) ç 1 ÷ (b) ç 2 ÷
97. The rms velocity of gas molecules is 300 ms–1. The è M2 ø è M1 ø
rms velocity of molecules of gas with twice the æ M - M2 ö
1 /2
æ M + M2 ö
1 /2

molecular weight and half the absolute temperature (c) ç 1 ÷ (d) ç 1 ÷


è M1 + M2 ø è M1 - M2 ø
is
(a) 300 ms–1 (b) 600 ms 106. The root mean square velocity of the molecules in a
(c) 75 ms–1 (d) 150 ms–1 sample of helium is 5/7th that of the molecules in a
98. If cs is the velocity of sound in air and c is the rms sample of hydrogen. If the temperature of the
velocity, then hydrogen as is 0°C, that of helium sample is about
(a) c s < c (b) c s = c (a) 0°C (b) 4 K (c) 273°C (d) 100°C
1 /2
ægö 107. The average translatory energy and rms speed of
(c) c s = c ç ÷ (d) None of these
è 3ø
molecules in a sample of oxygen gas at 300 K are
99. N molecules, each of mass m, of gas A and 2 N 6.21 ´ 10–21 J and 484 ms–1 respectively. The
molecules, each of mass 2 m, of gas B are contained in corresponding values at 600 K are nearly (assuming
the same vessel which is maintained at a ideal gas behaviour)
temperature T. The mean square velocity of (a) 12.42 ´ 10 –21 J, 968 ms –1
molecules of B type is denoted by V2 and the mean
V (b) 7.78 ´ 10 –21 J, 684 ms –1
square velocity of A type is denoted by V1, then 1 is
V2 (c) 6.21 ´ 10 –21 J, 968 ms –1
(a) 2 (b) 1 (c) 1/3 (d) 2/3 (d) 12.42 ´ 10 –21 J, 684 ms –1
Telegram @unacademyplusdiscounts

Heat and Kinetic Theory of Gases 527

108. The average energy and the rms speed of molecules 114. The value of g for gas X is 1.33, the X is
in a sample of oxygen gas at 400 K are (a) Ne (b) O3 (c) N2 (d) NH3
7.21 ´ 10-21 J and 524 ms–1 respectively. The 115. There is a rough black spot on a polished metallic
corresponding values at 800 K are nearly plate. It is heated upto 1400 K. Approximately and
(a) 14.42 ´ 10 –21 J, 1048 ms –1 then at once taken in a dark room which of the
(b) 10.18 ´ 10 –21 J, 741 ms –1 following statements is true?
(c) 7.21 ´ 10 –21 J, 1048 ms –1 (a) In comparision with the plate the spot will shine more
(d) 14.42 ´ 10 –21
J, 741 ms –1 (b) In comparision with the plate the spot will appear were
black
109. The average kinetic energy of a gas molecule at 27°C (c) The spot and the plate will be equally bright
is 6.21 ´ 10–21 J. Its average kinetic energy at 127°C (d) The plate and the black spot can not be seen in the dark
will be room
(a) 12.2 ´ 10 –21 J (b) 8.28 ´ 10 –21 J 116. The thermal radiation from a hot body travels with a
(c) 10.35 ´ 10 –21 J (d) 11.35 ´ 10 –21 J velocity of
(a) 330 ms–1 (b) 2 ´ 10 8 ms –1
110. The value of molar specific heat at constant volume
for 1 mole of polyatomic gas having n number of (c) 3 ´ 10 8 ms -1 (d) 230 ´ 10 8 ms –1
degrees of freedom at temperature T K is 117. Assuming the sun to have a spherical outer surface of
(R = universal gas constant) radius r radiating like a black body at temperature
nR nR nRT t° C, the power received by a unit surface (normal to
(a) (b) (c) (d) 2 nRT
2T 2 2 the incident rays) at a distance R from the centre of
111. For a gas, if the ratio of specific heats at constant the sun is (s is stefan’s constant)
pressure and constant volume is g, then the value of r2s (t + 273) 4
(a) 4 pr2st 4 (b)
degrees of freedom is 4 pR 2
g +1 g -1 ( g - 1) 2
(a) (b) (c) (d) 16 p2 r2st 4 r2s (1 + 273) 4
g -1 g +1 2 g -1 (c) (d)
R2 R2
112. The value of molar specific heat at constant pressure
118. The temperature of sun is 5500 K and it emits
for one mole of triatomic gas (triangular
maximum intensity radiation in the yellow region
arrangement) at temperature T K is (R = universal
(5.5 ´ 10–7 m). The maximum radiation from a
gas constant)
furnace occurs at wavelength 11 ´ 10-7 m. The
2 5
(a) 3 R (b) R (c) R (d) 4 R temperature of furnace is
7 2
(a) 2550 K (b) 2750 K (c) 2650 K (d) 2850 K
113. The diameter of a gas molecule is 2.4 ´ 10–10 m. The
mean free path of gas molecule at NTP is 119. The temperature of a liquid drops from 365 K
(k = 1.38 ´ 10–23 JK–1) to 361 K in 2 minutes. Find the time during which
(a) 1.46 ´ 10 –7 m (b) 2.46 ´ 10 –6
m temperature of the liquid drops from 344 K to 342 K.
–6 –7 Temperature of room is 292 K
(c) 1.46 ´ 10 m (d) 2.46 ´ 10 m
(a) 84 s (b) 72 s (c) 66 s (d) 60 s

Round II (Mixed Bag)

Only One Correct Option 2. A steel ball of mass 0.1 kg falls freely from a height of
of 10 m and bounces to a height of 5.4 m from the
1. A thin copper wire of length l increase in length by
ground. If the dissipated energy in this process is
1%, when heated from 0°C to 100°C. If a thin copper
absorbed by the ball, the rise in its temperature is
plate of area 2 l ´ l is heated from 0°C to 100°C, the
(a) 0.01°C (b) 0.1°C
percentage increase in its area would be
(c) 1.1°C (d) 1°C
(a) 1% (b) 4%
(c) 3% (d) 2%
Telegram @unacademyplusdiscounts

528 JEE Main Physics

3. The triple point of neon and carbon dioxide are 12. The ends of 2 different materials with their thermal
24.57 K and 216.55 K respectively. These conductivities, radii of cross-section and length all in
temperatures on the celsius and fahrenheit scales the ratio of 1 : 2 maintained at temperature
are respectively [NCERT] difference. If the rate of the flow of heat in the longer
(a) - 415.44° F and - 69.88° F rod is 4 cals–1, that in the shorter rod in cals–1 will be
(b) 415.44° F and 69.88° F (a) 1 (b) 2 (c) 8 (d) 6
(c) - 315.44° F and 69.88° F 13. A hole is drilled in a copper sheet. The diameter of the
(d) - 69.88° F and 415.44° F hole is 4.24 cm at 27.0°C. What is the change in the
4. The density of a substance at 0°C is 10 g/cc and at diameter of the hole when the sheet is heated to
100°C, its density is 9.7 g/cc. The coefficient of linear 227°C? Coefficient of linear expansion of copper is
expansion of the substance is . ´ 10-5 / ° C.
170 [NCERT]
(a) 10–4 °C–1 (b) 10–2 °C–1 (a) 1.44 ´ 10 -2 cm (b) 2.44 ´ 10 -3 cm
(c) 10–3 °C–1 (d) 102 °C–1
(c) 1.44 ´ 10 -2 mm (d) 2.44 ´ 10 -3 mm
5. The rate of cooling at 600 K, if surrounding
temperature is 300 K is R. The rate of cooling at 900 K 14. A body takes 10 min to cool from 60°C to 50°C. If the
is temperature of surroundings is 25°C and 527°C
16 2 respectively. The ratio of energy radiated by P and Q
(a) R (b) 2 R (c) 3 R (d) R
3 3 is
(a) 48°C (b) 46°C (c) 49°C (d) 42.85°C
6. The temperature of a piece of metal is increased from
27°C to 84°C. The rate at which energy is radiated is 15. Three rods made of same material and having same
increased to cross-section have been joined as shown in figure.
(a) four times (b) two times Each rod is of same length. The left and right ends
(c) six times (d) eight times are kept at 0°C and 90°C respectively. The
temperature of the junction of the three rods will be
7. Two identical square rods of metal are welded end to 90°C
end as shown in figure, Q cal of heat flows through 2
this combination in 4 min. If the rods were welded as 1
shown in figure, the same amount of heat will flow 0°C
through the combination in 3
90°C
(a) 45°C (b) 60°C (c) 30°C (d) 20°C
(a)
16. A bimetallic strip consists of brass and iron when it is
(b) heated it bends into an arc with brass on the convex
and iron on the concave side of the arc. This happens
(a) 16 min (b) 12 min (c) 1 min (d) 4 min because
8. 22 g of CO2 at 27°C is mixed with 16 g of oxygen at (a) brass has a higher specific heat capacity than iron
37°C. The temperature of the mixture is (b) density of brass is more than that of iron
(a) 32°C (b) 27°C (c) 37°C (d) 30°C (c) it is easier to bend an iron strip than a brass strip of the
same size
9. Two cylindrical conductors A and B of same metallic (d) brass has a higher coefficient of linear expansion than iron
material have their diameters in the ratio 1 : 2 and
lengths in the ratio 2 : 1. If the temperature 17. The efficiency of a Carnot engine is 50% and
difference between their ends is same, the ratio of temperature of sink is 500 K. If temperature of
heat conducted respectively by A and B per second is source is kept constant and its efficiency raised to
(a) 1 : 2 (b) 1 : 4 (c) 1 : 16 (d) 1 : 8 60%, then the required temperature of sink will be
(a) 100 K (b) 600 K (c) 400 K (d) 500 K
10. The amount of heat required to convert 10 g of ice at
–10°C into steam at 100°C is (in calories) 18. 70 cal of heat are required to raise the temperature of
(a) 6400 (b) 5400 (c) 7200 (d) 7250 2 mole of an ideal gas at constant pressure from 30°C
11. A lead bullet of 10 g travelling at 300 ms–1
strikes to 35°C. The amount of heat required to raise the
against a block of wood comes to rest. Assuming 50% temperature of the same sample of the gas through
of heat is absorbed by the bullet, the increase in its the same range at constant volume is nearly
temperature is (Specific heat of lead = 150 JkgK –1) (Gas constant = 1.99 cal K–1-mol–1)
(a) 100°C (b) 125°C (c) 150°C (d) 200°C (a) 30 cal (b) 50 cal (c) 70 cal (d) 90 cal
Telegram @unacademyplusdiscounts

Heat and Kinetic Theory of Gases 529

19. A flask of volume 103 cc is completely filled with 26. A metal ball immersed in water weighs w1 at 0°C and
mercury at 0°C. The coefficient of cubical expansion w2 at 50°C. The coefficient of cubical expansion of
of mercury is 1.80 ´ 10–6 ° C–1 and that of glass is metal is less than that of water. Then
1.4 ´ 10–6 ° C–1. If the flask is now placed in boiling (a) w1 < w2 (b) w1 > w2
water at 100°C, how much mercury will overflow? (c) w1 = w2 (d) data is not sufficient
(a) 7 cc (b) 1.4 cc 27. A steel tape measures the length of a copper rod as
(c) 21 cc (d) 28 cc 90.0 cm, when both are at 10°C, the calibration
20. The coefficiency of apparent expansion of a liquid temperature, for the tape. What would be tape read
when determined using two different vessels A and B for the length of the rod when both are at 30°C?
are l1 and l2 , respectively. If the coefficient of linear Given, a for steel 1.2 ´ 10–5 ° C–1 and a for copper is
expansion of the vessel A is a, the coefficient of linear 1.7 ´ 10–5 ° C–1
expansion of vessel B is (a) 90.01 cm (b) 89.90 cm
a g1g2 g -g (c) 90.22 cm (d) 89.80 cm
(a) (b) 1 2
g1 + g2 2a 28. A cylinder of radius r and thermal conductivity K 1 is
g -g +a g -g surrounded by a cylindrical shell of linear radius r
(c) 1 2 (d) 1 2 + a
3a 3 and outer radius 2 r, whose thermal conductivity is
21. Which of the following cylindrical rods will conduct K 2 . There is no loss of heat across cylindrical
maximum heat, when their ends are maintained at a surfaces, when the ends of the combined system are
constant temperature difference ? maintained at temperatures T1 and T2 . The effective
thermal conductivity of the system, in the steady
(a) l = 1 m, r = 0.2 m (b) l = 1m, r = 0.1 m
state is
(c) l = 10 m, r = 0.1 m (d) l = 0.1m, r = 0.3 m
K1K2
(a) (b) K1 + K2
22. Two spheres made of same substance have diameters K1 + K2
in the ratio 1 : 2. Their thermal capacities are in the K + 3 K2 3 K1 + K2
ratio of (c) 1 (d)
4 4
(a) 1 : 2 (b) 1 : 8 (c) 1 : 4 (d) 2 : 1
29. The power radiated by a black body is P, and it
23. A child running at a temperature of 101°F is given an radiates maximum energy around the wavelength
antipyrin (i.e., medicine that lowers fever) which l 0 . If the temperature of black body is now changed
causes an increase in the rate of evaporation of sweat so that it radiates maximum energy around a
from his body. If the fever is brought down to 98°F in wavelength l 0 / 4, the power radiated by it will
20 min, what is the average rate of extra evaporation increase by a factor of
caused by the drug. Assume the evaporation 4 16
(a) (b)
mechanism to be the only way by which heat is lost. 3 9
The mass of child is 30 kg. The specific heat of the 64 256
(c) (d)
human body is approximately the same as that of 27 81
water and latent heat of evaporation of water at that
30. A solid whose volume does not change with
[NCERT] temperature
temperature is in
floats about 580For
liquid. cal/g.
two different
(a) 4.31 g/min (b) 4.31 g/s temperatures t1 and t2 , the fractions f1 and f2 of
(c) 2.31 g/min (d) 2.31 g/s volume of solid remain submerged. What is the
24. A wheel is 80.3 cm in circumference. An iron tyre coefficient of volume expansion of liquid?
measures 80.0 cm around its inner face. If the coefficient f1 - f2 f1 - f2
(a) (b)
of linear expansion for iron is 1.2 ´ 0-5 ° C-1, the f2t1 - f1t2 f1t1 - f2t2
temperature of the tyre must be raised by f +f f +f
(c) 1 2 (d) 1 2
(a) 105°C (b) 417°C f2t1 - f1t2 f1t1 - f2t2
(c) 312°C (d) 223°C
31. A vessel of volume 4 L contains a mixture of 8 g of
25. The temperature gradient in the earth’s crust is oxygen, 14 g of nitrogen and 22 g of carbon dioxide at
32°C km–1 and the mean conductivity of earth is 27°C. The pressure exerted by the mixture is
0.008 cals–1cm–1°C -1. Considering earth to be a (a) 5.79 ´ 105 Nm–2
sphere of radius 6000 km loss of heat by earth
(b) 6.79 ´ 105 Nm–2
everyday is about
(a) 1030 cal (b) 1040 cal (c) 7.79 ´ 103 Nm–2
(c) 1020 cal (d) 1018 cal (d) 7.79 ´ 105 Nm–2
Telegram @unacademyplusdiscounts

530 JEE Main Physics

32. 22 g of carbon dioxide at 27°C is mixed in a closed 39. The coefficient of apparent expansion of mercury in a
container with 16 g of oxygen at 37°C. If both gases glass vessel is 153 ´ 10-6 /°C and in a steel vessel is
are considered as ideal gases, then the temperature 144 ´ 106 /°C. If a for steel is 12 ´ 10-6 /°C, then that of
of the mixture is glass is
(a) 24.2°C (b) 28.5°C (a) 9 ´ 10 -6 /°C (b) 6 ´ 10 -6 /°C
(c) 31.5°C (d) 33.5°C (c) 36 ´ 10 -6 /°C (d) 27 ´ 10 -6 /°C
33. Two chambers containing m1 and m2 gram of a gas at 40. Solids expand on heating because
pressures p1 and p2 respectively are put in (a) kinetic energy of the atoms increases
communication with each other, temperature (b) potential energy of the atoms increases
remaining constant. The common pressure reached (c) total energy of the atoms increases
will be (d) the potential energy curve is asymmetric about the
p1p2 ( m1 + m2 ) p1p2 m1 equilibrium distance between neighbouring atoms
(a) (b)
p2 m1 + p1m2 p2 m1 + p1m2
41. An iron tyre is to be fitted on a wooden wheel 1 m in
m m ( p + p2 ) m1m2 p2 diameter. The diameter of tyre is 6 mm smaller than
(c) 1 2 1 (d)
p2 m1 + p1m2 p2 m1 + p1m2 that of wheel. The tyre should be heated so that its
34. At room temperature (27°C) the rms speed of the temperature increases by a minimum of (the
molecules of a certain diatomic gas is found to be coefficient of cubical expansion of iron is
1920 ms–1.The gas is 3.6 ´ 10–5 / ° C
(a) Cl2 (b) O2 (a) 167°C (b) 334°C (c) 500°C (d) 1000°C
(c) N2 (d) H2 42. A glass flask of volume one litre at 0°C is filled, level
35. 8 g of O2 , 14 g of N2 and 22 g of CO2 is mixed in a full of mercury at this temperature. The flask and
container of 10 L capacity at 27°C. The pressure mercury are now heated to 100°C. How much
exerted by the mixture in terms of atmospheric mercury will spill out, if coefficient of volume
pressure is (R = 0.082 L atm K–1 mol–1) expansion of mercury is 1.82 ´ 10–4 / ° C and linear
(a) 1.4 atm (b) 2.5 atm expansion of glass is 0.1 ´ 10–4 / ° C respectively?
(c) 3.7 atm (d) 8.7 atm (a) 21.2 cc (b) 15.2 cc
(c) 1.52 cc (d) 2.12 cc
36. Inside a cylinder closed at both ends is a movable
piston. On one side of the piston is a mass m of a gas, 43. A steel scale measures the length of a copper wire as
and on the other side a mass 2 m of the same gas. 80.0 cm, when both are at 20°C (the calibration
What fraction of the volume of the cylinder will be temperature for scale). What would be the scale read
occupied by the larger mass of the gas when the for the length of the wire when both are at 40°C? (Given
piston is in equilibrium? The temperature is the a steel = 11 ´ 10–6 per° C and a copper = 17 ´ 10–6 per°C)
same throughout. (a) 80.0096 cm (b) 80.0272 cm
2 1 1 1 (c) 1 cm (d) 25.2 cm
(a) (b) (c) (d)
3 3 2 4 44. When the temperature of a rod increases from t to
37. Two containers of equal volume contain the same gas t + Dt, its moment of inertia increases from I to
at the pressures p1 and p2 and absolute temperatures I + DI . If a be the coefficient of linear expansion of
DI
T1 and T2 respectively. On joining the vessels, the gas the rod, the then the value of is
reaches a common pressure p and a common I
temperature T. The ratio p/T is equal to (a) 2 a Dt (b) a Dt
p T + p2T2 p T + p2T1 a Dt Dt
(a) 1 2 (b) 1 2 (c) (d)
T1 ´ T2 T1 + T2 2 a
Dt
1 é p1T2 + p2T1 ù p1T2 - p2T1 (e)
(c) ê ú (d) 2a
2ë T1T2 û T1 ´ T2
45. Two metal strips that constitute a thermostat must
38. Two moles of monoatomic gas is mixed with three necessarily differ in their
moles of a diatomic gas. The molar specific heat of the (a) mass
mixture at constant volume is (b) length
(a) 1.55 R (b) 2.10 R (c) resistivity
(c) 1.63 R (d) 2.20 R (d) coefficient of linear expansion
Telegram @unacademyplusdiscounts

Heat and Kinetic Theory of Gases 531

46. A metal ball immersed in alcohol weighs w1 at 0°C 53. Water of volume 2L in a container is heated with a
and w2 at 59°C. The coefficient of cubical expansion of coil of 1 kW at 27°C. The lid of the container is open
the metal is less than that of alcohol. Assuming that and energy dissipates at rate of 160 J/s. In how much
the density of metal is large compared to that of time temperature will rise from 27°C to 77°C ?
alcohol, it can be shown that [Given specific heat of water is 4.2 kJ/kg]
(a) w1 > w2 (b) w1 = w2 (a) 8 min 20 s (b) 6 min 2 s
(c) w1 < w2 (d) w2 = ( w1/ 2) (c) 7 min (d) 14 min
47. The coefficient of volume expansion of glycerine is 54. A lead bullet at 27°C just melts when stopped by an
49 ´ 10-5 /K. What is the fractional change in its obstacle. Assuming that 25% of heat is absorbed by
density for a 30°C rise in temperature? [NCERT] the obstacle, then the velocity of the bullet at the time
(a) 1.45 ´ 10 -3 (b) 2.45 ´ 10 -3 of striking (MP of lead = 327°C, specific heat of lead =
(c) 2.45 ´ 10 -2 (d) 1.45 ´ 10 -2
0.03 cal/g°C, latent heat of fusion of lead = 6 cal/g and
J = 4.2 joule/cal)
48. A piece of metal weighs 46 g in air. When it is (a) 410 m/s (b) 1230 m/s
immersed in the liquid of specific gravity 1.24 at 27°C (c) 307.5 m/s (d) None of these
it weighs 30 g. When the temperature of liquid is
55. We have seen that a gamma-ray dose of 3 Gy is lethal
raised to 42°C the metal piece weighs 30.5 g, specific
to half the people exposed to it. If the equivalent
gravity of the liquid at 42°C is 1.20, then the linear
energy were absorbed as heat, what rise in body
expansion of the metal will be
temperature would result?
(a) 3.316 ´ 10 –5 /° C (b) 2.316 ´ 10 –5 /° C
(a) 300 mK (b) 700 mK
(c) 4.316 ´ 10 –5 /° C (d) None of these (c) 455 mK (d) 390 mK
49. It is known that wax contracts on solidification. If 56. The temperature of equal masses of three different
molten wax is taken in a large vessel and it is allowed liquids A, B and C are 12°C, 19°C and 28°C
to cool slowly, then respectively. The temperature when A and B are
(a) it will start solidifying from the top to downward mixed is 16°C and when B and C are mixed is 23°C.
(b) it will starts solidifying from the bottom to upward The temperature when A and C are mixed, is
(c) it will start solidifying from the middle, upward and (a) 18.2°C (b) 22°C
downward at equal rates (c) 20.2°C (d) 25.2°C
(d) the whole mass will solidify simultaneously
57. In an industrial process 10 kg of water per hour is to
50. A substance of mass m kg requires a power input of P be heated from 20°C to 80°C. To do this steam at
watts to remain in the molten state at its melting 150°C is passed from a boiler into a copper coil
point. When the power is turned off, the sample immersed in water. The steam condenses in the coil
completely solidifies in time t sec. What is the latent and is returned to the boiler as water at 90°C. How
heat of fusion of the substance? many kg of steam is required per hour? (Specific heat
Pm Pt of steam = 1 calorie per g°C, Latent heat of
(a) (b)
t m vaporisation = 540 cal/g)
m t
(c) (d) (a) 1 g (b) 1 kg (c) 10 g (d) 10 kg
Pt Pm
58. In a vertical U-tube containing a liquid, the two arms
51. Steam at 100°C is passed into 1.1 kg of water are maintained at different temperatures t1 and t2 .
contained in a calorimeter of water equivalent to The liquid columns in the two arms have heights l1
0.02 kg at 15°C till the temperature of the and l2 respectively. The coefficient of volume
calorimeter and its contents rises to 80°C. The mass expansion of the liquid is equal to
of the steam condensed in kg is
(a) 0.130 (b) 0.065 (c) 0.260 (d) 0.135
t1
52. 2 kg of ice at –20°C is mixed with 5 kg of water at t2
20°C in an insulating vessel having a negligible heat l1
capacity. Calculate the final mass of water remaining l2
in the container. It is given that the specific heats of
water and ice are 1 kcal/kg per°C and 0.5 kcal/kg/°C
while the latent heat of fusion of ice is 80 kcal/kg. l1 - l2 l1 - l2 l1 + l2 l1 + l2
(a) (b) (c) (d)
(a) 7 kg (b) 6 kg (c) 4 kg (d) 2 kg l2t1 + l1t2 l1t1 - l2t2 l2t1 + l1t2 l1t1 + l2t2
Telegram @unacademyplusdiscounts

532 JEE Main Physics

59. The coefficient of linear expansion of crystal in one 66. A closed compartment containing gas is moving with
direction is a1 and that in every direction some acceleration in horizontal direction. Neglect
perpendicular to it is a2 . The coefficient of cubical effect of gravity. Then, the pressure in the
expansion is compartment is
(a) a1 + a2 (b) 2 a1 + a2 (a) same everywhere (b) lower in front side
(c) a2 + 2 a2 (d) None of these (c) lower in rear side (d) lower in upper side
60. Three rods of equal length l are R 67. A room is maintained at 20°C by a heater of
joined to form an equilateral resistance 20 W connected to 200 V mains. The
triangle PQR. O is the mid point temperature is uniform throughout the room and
of PQ. Distance OR remains heat is transmitted through a glass window of area
same for small change in 1 m 2 and thickness 0.2 cm. What will be the
temperature. Coefficient of P Q temperature outside? Given that thermal
O
linear expansion for PR and RQ conductivity K for glass is 0.2 cal/m/°C sec and
is same, i. e., a 2 but that for PQ is a 1. Then J = 4.2 J /cal.
(a) 15.24°C (b) 15.00°C
(a) a2 = 3 a1 (b) a2 = 4 a1
(c) 24.15°C (d) None of these
(c) a1 = 3 a2 (d) a1 = 4 a2
68. There is formation of layer of snow x cm thick on
61. An electric kettle takes 4 A current at 220 V. How
water, when the temperature of air is - q °C (less than
much time will it take to boil 1 kg of water from
freezing point). The thickness of layer increases from
temperature 20°C? The temperature of boiling water
x to y in the time t, then the value of t is given by
is 100°C
( x + y ) ( x - y ) rL ( x - y ) rL
(a) 12.6 min (b) 4.2 min (a) (b)
2 kq 2 kq
(c) 6.3 min (d) 8.4 min
( x + y ) ( x - y ) rL ( x - y ) rLk
62. 10 g of ice at –20°C is droped into a calorimeter (c) (d)
kq 2q
containing 10 g of water at 10°C; the specific heat of
water is twice that of ice. When equilibrium is 69. A composite metal bar of uniform section is made up
reached, the calorimeter will contain of length 25 cm of copper, 10 cm of nickel and 15 cm of
(a) 20 g of water aluminium. Each part being in perfect thermal
(b) 20 g of ice contact with the adjoining part. The copper end of the
(c) 10 g ice and 10 g water composite rod is maintained at 100°C and the
(d) 5 g ice and 15 g water aluminium end at 0°C. The whole rod is covered with
belt so that no heat loss occurs at the side. If
63. A copper block of mass 2.5 kg is heated in furnace to a K Cu = 2 K Al and K Al = 3 K Ni , then what will be the
temperature of 500°C and then placed on a large ice
temperatures of Cu-Ni and Ni-Al junctions
block. What is the maximum amount of ice that can
repectively
melt? (Specific heat of copper = 039
. J/g-K; heat of
fusion of water = 335 J/g) [NCERT] Cu Ni Al
(a) 25 kg (b) 15 kg 100°C 0°C
(c) 9 kg (d) 13 kg (a) 23.33°C and 78.8°C (b) 83.33°C and 20°C
64. Steam is passed into 22 g of water at 20°C. The mass (c) 50°C and 30°C (d) 30°C and 50°C
of water that will be present when the water acquires 70. Three rods of identical area of cross-section and made
a temperature of 90°C (Latent heat of steam is from the same metal form the sides of an isosceles
540 cal/g) is triangle ABC right angled at B. The points A and B
(a) 24.8 g (b) 24 g are maintaned temperatures T and 2T
(c) 36.6 g (d) 30 g respectively. In the steady state the temperature of
65. Ice starts forming in lake with water at 0°C and when the point C is TC . Assuming that only heat conduction
the atmospheric temperature is –10°C. If the time T
takes place, C is equal to
taken for 1 cm of ice be 7 h, then the time taken for T
the thickness of ice to change from 1 cm to 2 cm is 1 3
(a) (b)
(a) 7 h (b) 14 h ( 2 + 1) ( 2 + 1)
(c) Less than 7 h (d) More than 7 h 1 1
(c) (d)
2 ( 2 - 1) 3 (2 - 1)
Telegram @unacademyplusdiscounts

Heat and Kinetic Theory of Gases 533

71. The only possibility of heat flow in a 77. Two metallic spheres S1 and S2 are made of the same
themros flask is through its cork which is material and have identical surface finish. The mass
75 cm2 in area and 5 cm thick its thermal of S1 is three times that of S2 . Both the spheres are
conductivity is 0.075 cal/cm sec°C. The heated to the same high temperature and placed in
outside temperatue is 40°C and latent the same room having lower temperature but are
heat of ice is 80 cal g -1. Time taken by thermally insulated from each other. The ratio of the
500 g of ice at 0°C in the flask to melt into initial rate of cooling of S1 to that of S2 is
water at 0°C is (a) 1/3 (b) (1 / 3)1 /3
(a) 2.47 h (b) 4.27 h (c) 1 / 3 (d) 3 /1
(c) 7.42 h (d) 4.82 h
78. Three discs A, B and C having radii 2 m, 4 m and 6 m
72. Two identical conducting rods are first connected respectively are coated with carbon black on their
independently to two vessels, one containing water at
other surfaces. The wavelengths corresponding to
100°C and the other containing ice at 0°C. In the
maximum intensity are 300 nm, 400 nm and 500 nm
second case, the rods are joined end to end and
respectively. The power radiated by them are Qa , Qb
connected to the same vessels. Let q1 and q2 g/s be the
and Qc respectively
rate of melting of ice in two cases respectively. The
(a) Qa is maximum (b) Qb is maximum
ratio of q1/ q2 is
(c) Qc is maximum (d) Qa = Qb = Qc
(a) 1/2 (b) 2/1 (c) 4/1 (d) 1/4
79. The total energy radiated from a black body source is
73. A solid cube and a solid sphere of the same material collected for one minute and is used to heat a
have equal surface area. Both are at the same
quantity of water. The temperature of water is found
temperature 120°C, then
to increase from 20°C to 20.5°C. If the absolute
(a) both the cube and the sphere cool down at the same rate
temperature of the black body is doubled and the
(b) the cube cools down faster than the sphere
experiment is repeated with the same quantity of
(c) the sphere cools down faster than the cube
water at 20°C, the temperature of water will be
(d) whichever is having more mass will cool down faster
(a) 21°C (b) 22°C
74. A black body is at a temperature of 2880 K. The energy (c) 24°C (d) 28°C
of radiation emitted by this object with wavelength
80. A solid sphere and a hollow sphere of the same
between 499 nm and 500 nm is U1, between 999 and
material and size are heated to the same
1000 nm is U2 and between 1499 nm and 1500 nm is
temperature and allowed to cool in the same
U3. The Wien’s constant b = 2.88 ´ 106 nmK. Then
surroundings. If the temperature difference between
(a) U1 = 0 (b) U3 = 0 (c) U1 > U2 (d) U2 > U1
each sphere and its surroundings is T, then
75. A black metal foil is warmed by radiation from a (a) the hollow sphere will cool at a faster rate for all values of T
small sphere at temperature T and at a distance d. It (b) the solid sphere will cool at a faster rate for all values of T
is found that the power received by the foil is P. If (c) both spheres will cool at the same rate for all values of T
both the temperature and distance are doubled, the (d) both spheres will cool at the same rate only for small
power received by the foil will be values of T
(a) 16 P (b) 4 P (c) 2 P (d) P 81. A solid copper cube of edges 1 cm is suspended in an
76. Three rods of same dimensions R evacuated enclosure. Its temperature is found to fall
are arranged as shown in figure. from 100°C to 99°C in 100 s. Another solid copper
They have thermal cube of edges 2 cm, with similar surface nature, is
K1 K2
conductivities K 1, K 2 and K 3. suspended in a similar manner. The time required
The points P and Q are for this cube to cool from 100°C to 99°C will be
maintained at different P Q approximately
K3
temperatures for the heat to (a) 25 s (b) 50 s
flow at the same rate along PRQ and PQ, then which (c) 200 s (d) 400 s
of the following options is correct? 82. A body initially at 80°C cools to 64°C in 5 min and to
1 52°C in 10 min. The temperature of the body after
(a) K3 = ( K1 + K2 ) (b) K3 = K1 + K2
2 15 min will be
K1K2 (a) 42.7°C (b) 35°C
(c) K3 = (d) K3 = 2 ( K1 + K2 )
K1 + K2 (c) 47°C (d) 40°C
Telegram @unacademyplusdiscounts

534 JEE Main Physics

83. A 5 cm thick ice block is there on the surface of water 89. A hot metallic sphere of radius r radiates heat. Its
in a lake. The temperature of air is –10°C; how much cooling is
time it will take to double the thickness of the block ? (a) independent of r (b) proportional to
(L = 80 cal/g, K ice = 0.004 erg/s-K, dice = 0.92 g cm –3) (c) proportional to r2 (d) proportional to 1 / r
(a) 1 h (b) 191 h (c) 19.1 h (d) 1.91 h
90. A solid copper sphere (density r and specific heat
84. Four identical rods of same material are joined end to capacity c) of radius r at an initial temperature 200 K
end to form a square. If the temperature difference is supended inside a chamber whose walls are at
between the ends of a diagonal is 100°C then the almost 0 K. The time required (in ms) for the
temperature difference between the ends of other temperature of the sphere to 100 K is
diagonal will be 72 rrc 7 rrc 27 rrc 7 rrc
(a) (b) (c) (d)
(a) 0°C 7 s 72 s 7 s 27 s
100
(b) °C; where l is the length of each rod 91. A metal rod AB of length 10 x has its one end A in ice
l
100
at 0°C and the other end B in water at 100°C. If a
(c) °C point P on the rod is maintained at 400°C, then it is
2l
found that equal amounts of water and ice evaporate
(d) 100°C and melt per unit time. The latent heat of
85. A cylindrical rod with one end in a steam chamber and evaporationof water is 540 cal/g and latent heat of
the other end in ice results in melting of 0.1 g of ice per melting of ice is 80 cal/g. If the point P is at a distance
second. If the rod is replaced by another with half the of lx from the ice end A, find the value of l
length and double the radius of the first and if the (Neglect any heat loss to the surroundings).
thermal conductivity of material of second rod is 1/4 (a) 9 (b) 2 (c) 6 (d) 1
that of first, the rate at which ice melts in g/s will be
92. A sphere and a cube of same material and same
(a) 3.2 (b) 1.6 (c) 0.2 (d) 0.1 volume. One heated upto same temperature and
86. One end of a copper rod of length 1.0 m and area of allowed to cool in the same surroundings. The ratio of
cross-section 10-3 m 2 is immersed in boiling water the amounts of radiation emitted will be
and the other end in ice. If the coefficient of thermal 4p
(a) 1: 1 (b) :1
conductivity of copper is 92 cal/m-s-°C and the latent 3
heat of ice is 8 ´ 104 cal/kg, then the amount of ice æpö
1 /3
1 æ4pö
2 /3
(c) ç ÷ :1 (d) ç ÷ :1
which will melt in one min is è6ø 2è 3 ø
-3 -3
(a) 9.2 ´ 10 kg (b) 8 ´ 10 kg
-3
93. Four rods of identical cross-sectional area and made
(c) 6.9 ´ 10 kg (d) 5.4 ´ 10 -3 kg from the same metal form the sides of square. The
87. An ice box used for keeping eatable cold has a total temperature of two diagonally opposite points are T
wall area of 1 m2 and a wall thickness of 5.0 cm. The and 2 T respectively in the steady state. Assuming
thermal conductivity of the ice box is K = 0.01 that only heat conduction takes place, what will be the
joule/metre-°C. It is filled with ice at 0°C along with temperature difference between other two points?
eatables on a day when the temperature is 30°C. The 2 +1 2
(a) T (b) T
latent heat of fusion of ice is 334 ´ 103 joule/kg. The 2 2 +1
amount of ice melted in one day is (1 day = 86.400 s) (c) 0 (d) None of these
(a) 776 g (b) 7760 g (c) 11520 g (d) 1552 g 94. The graph AB shown in figure is a plot of
88. Five rods of same dimensions C temperature body in degree celsius and degree
are aranged as shown in the K1 K2 fahrenheit. Then,
figure. They have thermal
conductivities K 1, K 2 , K 3, K 4 A K5 B 100°C B
and K 5 . When points A and B
Centigrade

K3
are maintained at different K4
temperatures, no heat flows D
through the central rod if 32°F 212°F Fahrenheit
(a) K1 = K4 and K2 = K3 (b) K1K4 = K2 K3 A
K K2
(c) K1K2 = K3 K4 (d) 1 = (a) slope of line AB is 9/5 (b) slope of line AB is 5/9
K4 K3
(c) slope of line AB is 1/9 (d) slope of line AB is 3/9
Telegram @unacademyplusdiscounts

Heat and Kinetic Theory of Gases 535

95. The graph shows the variation of temperature ( T) of Y


one kilogram of a material with the heat ( H ) supplied T°C
A
to it. At O, the substance in the solid state. From the 90
graph, we can conclude that B D
80
T
2 min 4 min
C(H3, T2) γ X
Time (min.)
D(H4, T2)
A(H1, T1) β (a) 40 (b) 80 (c) 100 (d) 20
B(H2, T1) 99. A solid substance is at 30°C. To this substance heat
α
H energy is supplied at a constant rate. Then
O
temperature versus time graph is as shown in the
(a) T2 is the melting point of the solid figure. The substance is in liquid state for the portion
(b) BC represents the change of state from solid to liquid (of the graph)
(c) ( H2 - H1 ) represents the latent heat of fusion of the
substance

Temperature (T°C)
240 F
(d) ( H3 - H1 ) represents the latent heat of vaporization of the D
210
liquid E
B
96. A block of ice at –110°C is slowly heated and 60 C
converted to steam at 100°C. Which of the following 30
curves represents the phenomenon qualitatively A Time
(a) BC (b) CD (c) ED (d) EF
100. The variation of density of water with temperature is
Temperature

Temperature

represented by the
(a) (b)
Density

Density
Heat supplied Heat supplied (a) (b)
Temperature

Temperature

Temperature Temperature
(c) (d)
Density

Density

Heat supplied Heat supplied


(c) (d)
97. The portion AB of the indicator diagram representing
the state of mattter denotes Temperature Temperature
p A
101. If a graph is plotted taking the temperature in
C Fahrenheit along Y -axis and the corresponding
B temperature in celsius along the X-axis, it will be
D straight line
V (a) having a +ve intercept on Y-axis
(b) having a +ve intercept on X-axis
(a) the liquid state of matter
(c) passing through the origin
(b) gaseous state of matter
(d) having a –ve intercepts on both the axis
(c) change from liquid to gaseous state
(d) change from gaseous state of liquid state 102. Which of the curves in figure °C

98. The figure given below shows the cooling curve of represents, the relation
2 3
pure wax material after heating. It cools from A to B between Celsius and
and solidifies along BD. If L and C are respective Fahrenheit temperatures? °F
4 1
values of latent heat and the specific heat of the (a) 1 (b) 2
liquid wax, the ratio L / C is (c) 3 (d) 4
Telegram @unacademyplusdiscounts

536 JEE Main Physics

103. Heat is supplied to a certain homogeneous sample of 107. Two substances Aand B of equal mass m are heated
matter, at a uniform rate. Its temperature is plotted at uniform rate of 6 cal s -1 under similar conditions.
against time, as shown. Which of the following A graph between temperature and time is shown in
conclusions can be drawn? figure. Ratio of heat absorbed H A / H B by them for
complete fusion is
Temperature

Temperature (°C)
100
80 A
60
40 B
Time
20
(a) Its specific heat capacity is greater in the solid state than t
1 2 3 4 5 6 7
in the liquid state
(b) Its specific heat capacity is greater in the liquid state than (a) 9/4 (b) 4/9 (c) 8/5 (d) 5/8
in the solid state 108. Which one of the figures gives the temperature
(c) Its latent heat of vaporization is greater than its latent dependence of density of water correctly?
heat of fusion
(d) Its latent heat of vaporization is smaller than its latent of

Density (D)

Density (D)
fusion
104. At what temperature is the root mean square speed (a) (b)
50°C 50°C
of an atom in an argon gas cylinder equal to the rms
speed of a helium gas atom at –20°C? (Atomic mass of Temperature (T) Temperature (T)
Ar = 39.9 u, of H = 4.0 u). [NCERT] Density (D)

Density (D)
(a) 3.52 ´ 103 K (b) 2.52 ´ 103 K
(c) (d)
(c) 2.52 ´ 10 4 K (d) 3.52 ´ 103 K 50°C 50°C
105. The graph signifies Temperature (T) Temperature (T)
Temperature

109. Which curve shows the rise of temperature with the


amount of heat supplied, for a piece of ice?
B A
500 C
Temperature in K

Time D
400
(a) adiabatic expansion of a gas
(b) isothermal expansion of a gas 300
(c) change of state from liquid to solid
(d) cooling of a heated soil 200
Amount of heat supplied
106. Which of the substances A, B or C has the highest
specific heat? The temperature versus time graph is (a) A (b) B (c) C (d)D
shown 110. A solid material is supplied
Temperature

A with heat at constant rate and E


Temperature (T)

the temperature of the C


B D
material changes as shown. A
C B CD = 2AB
From the graph, the false
conclusion drawn is O Heat input

Time (t) (a) AB and CD of the graph represent phase changes


(b) AB represents the change of state from solid to liquid
(a) A (c) Latent heat of fusion is twice the latent heat of
(b) B vaporization
(c) C (d) CD represents change of state from liquid to vapour
(d) All have equal specific heat (e) Latent heat of vaporization is twice the latent heat of
fusion
Telegram @unacademyplusdiscounts

Heat and Kinetic Theory of Gases 537

111. Liquid oxygen at 50 K is heated to 300 K at constant 115. Following graph shows the correct variation in
pressure of 1 atm. The rate of heating is constant. intensity of heat radiations by black body and
Which one of the following graphs represents the frequency at a fixed temperature
variation of temperature with time?

Ultraviolet
Visible Infrared

3500 K
Temperature

Temperature
(a) 2500 K
(a) (b) 1500 K

Time Time ν

Ultraviolet
Visible Infrared

Temperature

Temperature

1500 K
(c) (d) (b) 2500 K
3500 K

Time Time
ν
112. The graph shown in the adjacent diagram,
represents the variation of temperature ( T) of two Infrared Visible Ultraviolet

bodies, x and y having same surface area, with time 3500 K
( t) due to the emission of radiation. Find the correct (c)
relation between the emissivity ( e) and absorptivity 2500 K
(a) of the two bodies 1500 K
T
ν
y
Infrared Visible Ultraviolet

x
1500 K
t (d)
(a) ex > e y and ax < a y (b) ex < e y and ax > a y 2500 K
(c) ex > e y and ax > a y (d) ex < e y and ax < a y 3500 K

113. The plots of intensity versus T3 ν


wavelength for three black T2 116. A brass boiler has a base area 0.15 m2 and thickness
bodies at temperatures T1, T2 I T1 1.0 cm. It boils water at the rate of 6.0 kg/min when
and T3 respectively are as placed on a gas stove. The temperature of the part of
shown. Their temperatures are the flame in contact with the boiler will be. (Thermal
λ
such that conductivity of brass = 109 J/s-m-K, Heat of
(a) T1 > T2 > T3 (b) T1 > T3 > T2 vapourization of water = 2256 ´ 103 J/kg) [NCERT]
(c) T2 > T3 > T1 (d) T3 > T2 > T1 (a) 158°C (b) 208°C (c) 238°C (d) 264° C
114. The adjoining diagram shows the spectral energy 117. A body cools in a surrounding which is at a constant
density distribution El of a black body at two temperature of q 0 . Assume that it obeys Newton’s
different temperatures. If the areas under the curves law of cooling. Its temperature q is plotted against
are in the ratio 16 : 1, the value of temperature T is time t. Tangents are drawn to the curve at the points
TK
P (q = q 2 ) and Q(q = q 1). These tangents meet the
time axis at angles of f2 and f1, as shown

2000 K
θ

θ2 P
λ
(a) 32000 K (b) 16000 K θ1 Q
(c) 8000 K (d) 4000 K
θ0 φ2 φ1
t
Telegram @unacademyplusdiscounts

538 JEE Main Physics

tan f2 q - q0 tan f2 q - q0 121. The energy distribution E with E


(a) = 1 (b) = 2
tan f1 q2 - q0 tan f1 q1 - q0 the wavelength ( l) for the black
tan f1 q1 tan f1 q2 body radiation at temperature
(c) = (d) =
tan f2 q2 tan f2 q1 T kelvin is shown in the figure.
As the temperature is
118. Shown below are the black body radiation curves at O λ
increased the maxima will
temperatures T1 and T2 ( T2 > T1). Which of the
(a) shift towards left and become higher
following plots is correct?
(b) rise high but will not shift
T2 T2 (c) shift towards right and become higher
I I
T1 (d) shift towards left and the curve will become broader
T1
(a) (b) 122. Heat is flowing through a conductor of length l from
x = 0 to x = l. If its thermal resistance per unit length
λ λ
is uniform which of the following graph is correct?
T T
I T2 I T1
(c) T1 (a) (b)
(d) T2

O x O x
λ λ
T T
119. The spectrum of a black body at two temperatures
27°C and 327°C is shown in the figure. Let A1 and A2
be the areas under the two curves respectively. The (c) (d)
A
value of 2 is
A1 O O
x x

123. Radius of a conductor increases uniformly from left


Intensity

2 and to right end as shown in figure.


327°C
1
27°C T1 T2
Wavelength
x
(a) 1 : 16 (b) 4 : 1
(c) 2 : 1 (d) 16 : 1 Material of the conductor is isotropic and its curved
surface is thermally insulated from surrounding. Its
120. A block of metal is heated to a temperature much ends are maintained at temperatures T1 and
T T higher
T state,than
> ( ). If 2in1 2steady heat the
flowroom
rate istemperature
equal and allowed to
cool in a room free from air currents. Which of the to H, then which of the following graphs is correct?
following curves correctly represents the rate of H H
cooling?

(a) (b)
Temperature

Temperature

(a) (b)
O O
x x

Time Time H H
Temperature

Temperature

(c) (d)
(c) (d)
O O
x x
Time Time
Telegram @unacademyplusdiscounts

Heat and Kinetic Theory of Gases 539

124. Which of the following graphs correctly represents


R A
the relation between ln E and ln T, where E is the
amount of radiation emitted per unit time from unit B
area of a body and T is the absolute temperature?
ln E ln E

(a) (b) (θ – θ 0 )

(a) A and B have same specific heats


ln T ln T (b) Specific heat of A is less
(c) Specific heat of B is less
ln E ln E
(d) Nothing can be said
128. Two spheres made of same material have radii in the
(c) (d)
ratio 1 : 2. Both are at same temperature. Ratio of
ln T heat radiation energy emitted per second by them is
ln T
O (a) 1 : 2 (b) 1 : 4 (c) 1 : 8 (d) 1 : 16
125. A hollow copper sphere S and a hollow copper cube C,
both of negligible thin walls of same area, are filled More Than One Correct Option
with water at 90°C and allowed to cool in the same 129. If a and l are coefficiehnts of linear, superficial and
environment. The graph that correctly represents volume expansion respectively, then
their coolings is b 1 b 2
(a) = (b) =
T T a 2 g 3
g 3 b g
(c) = (d) =
a 1 a b
(a) (b) S
C C 130. Mark the correct options [NCERT Exemplar]
S
t t (a) A system X is in thermal equilibrium with Y but not with
Z. System Y and Z may be in thermal equilibrium with
T T each other.
S
(b) A system X is in thermal equilibrium with Y but not with
Z. System Y and Z are not in thermal equilibrium with
(c) C, S (d) each other.
C
(c) A system X is neither in thermal equilibrium with Y nor
t t with Z. The systems Y and Z must be in thermal
equilibrium with each other.
126. Which of the following is the nm - T graph for a (d) A system X is neither in thermal equilibrium with Y nor
perfectly black body (n m = maximum frequency of with Z. The system Y and Z may be in thermal equilibrium
radiation) ? with each other.

νm
131. ‘Gulab Jamuns’ (assumed to be spherical) are to be
D
B heated in an oven. They are available in two sizes,
C one twice bigger (n radius) than the other. Pizzas
(assumed to be discs) are also to be heated in oven.
A They are also in two sizes, one twice big (in radius)
T than the other. All four are put together to be heated
(a) A (b) B to oven temperature. Choose the correct option from
(c) C (d) D the following [NCERT Exemplar]
(a) Both size gulab jamuns will get heated in the same time
127. Two circular discs A and B with equal radii are (b) Smaller gulab jamuns are heated before bigger ones
blackened. They are heated to same temperature and
(c) Smaller pizzas are heated before bigger ones
are cooled under identical conditions. What
(d) Bigger pizzas are heated before smaller ones
interference do you draw from their coolings curves?
Telegram @unacademyplusdiscounts

540 JEE Main Physics

132. A spherical body of radius r radiates power P and its 137. Two bodies A and B have thermal emissivities of 0.01
rate of cooling is R. and 0.81 respectively. The outer surface areas of the
1 two bodies are the same. The two bodies emit total
(a) P µ r (b) P µ r2 (c) R µ r2 (d) R µ
r radiant power at the same rate. The wavelength l B
133. Refer to the plot of temperature versus time showing corresponding to maximum spectral radiancy in the
the changes in the state of ice on heating (not to scale) radiation from B is shifted from the wavelength
Which of the following is correct? [NCERT Exemplar] corresponding to maximum spectral radiancy in the
radiation from A, by 1.00 mm. If the temperature of A
Temperature (°C)

E is 5802 K
(a) The temperature of B is 1934 K
100 (b) l B = 1.5 mm
C D
(c) The temperature of B is 11604 K
A B (d) The temperature of B is 2901 K
0 tm time (min) 138. ABCDEFGH is hollow cube made of an insulator.
(a) The region AB represents ice and water in thermal Face ABCD has positive charge on it. Inside the cube,
equilibrium we have ionized hydrogen.
(b) At B water starts boiling B C
(c) At C all the water gets converted into steam +P
(d) C to D represents water and steam in equilibrium at A D
boiling point
134. Under which of the following conditions, the law
G
pV = RT is not obeyed by a real gas? F
(a) High pressure and high temperature E H
(b) Low pressure and low temperature
(c) Low pressure and high temperature The usual kinetic theory expression for pressure
(d) High pressure and low temperature [NCERT Exemplar]
(a) will be valid
135. Which of the following graphs do/does not represent
(b) will not be valid since the ions would experience forces
the behaviour of an ideal gas? [NCERT Exemplar]
orger than due to collisions with the walls
pV pV (c) will not be valid since collisions with walls would not be
elastic
(a) (b) (d) will not be valid because isotropy is lost
139. In a diatomic molecule, the rotational energy at a
V V given temperature [NCERT Exemplar]
pV pV (a) obeys Maxwell's distribution
(b) have the same value for all molecules
(c) (d) (c) equals the translational kinetic energy for each molecule
(d) is (2/3)rd the translational kinetic energy for each
molecule
V V

136. A glass full of hot milk is poured on the table. It Comprehension Based Questions
begins to cool gradually Which of the follwong is
correct? Passage I
(a) The rate of cooling is constant till milk attains the
When two substances at different temperatures are
temperature of the surrounding
mixed together, exchange of heat occurs between
(b) The temperature of milk falls off exponentially with time
them till they acquire a common temperature. In
(c) While cooling, there is a flow of heat from milk to the
surrounding as well as from surrounding to the milk but thermal equilibrium, heat gained by one substance is
the net flow of heat is from milk to the surrounding and equal to heat lost by the other substance. This is
that is why it cools called principle of calorimetry.
(d) All three phenomenon, conduction, convection and 140. 10 g of ice at 0°C and water at 100°C are mixed
radiation are responsible for the loss of heat milk to the together. The resultant temperature would be
surroundings.
(a) 10°C (b) 5°C (c) 50°C (d) 40°C
Telegram @unacademyplusdiscounts

Heat and Kinetic Theory of Gases 541

141. Equal masses of ice at 0°C is put in 10 g of water at 150. Time taken for collision of nitrogen molecules is
80°C. The final temperature would be (a) 4 ´ 10 -13 s (b) 5 ´ 10 –13 s
(a) 10°C (b) 0°C (c) 6 ´ 10 -13
s (d) 7 ´ 10 -13 s
(c) 40°C (d) 80°C
142. An ice block of mass m at 0°C is put in water of mass
2 m at 60°C. The final temperature would be
Assertion and Reason
(a) 60°C (b) 0°C Directions Question No. 151 to 161 are Assertion-Reason type.
(c) 30°C (d) 13.3°C Each of these contains two Statements: Statement I (Assertion),
Statement II (Reason). Each of these questions also has four
Passage II alternative choice, only one of which is correct. You have to
The latent heat of fusion of ice is 80 calg–1 and latent select the correct choices from the codes (a), (b), (c) and (d) given
heat of steam is 540 calg–1. Change of state occurs below
only at melting point or boiling point of the substance. (a) If both Assertion and Reason are true and the Reason
There is no change in temperature during the entire is correct explanation of the Assertion
change of state. For rise in temperature ( DT) heat (b) If both Assertion and Reason are true but Reason is
not correct explanation of the Assertion
required DQ = mcDT, where c is specific heat of the
substance. (c) If Assertion is true but Reason is false
(d) If Assertion is false but the Reason is true
143. Heat required to convert 1 g of ice at – 5°C to water at
0°C is (specific heat of ice = 0.5 calg–1°C–1) 151. Assertion The SI unit of Stefan’s constant is Wm–2 K–4.
(a) 80 cal (b) 82.5 cal Reason This follows from Stefan’s Law,
(c) 77.5 cal (d) 802.5 cal E = aT4
144. Heat released when 10 g of steam at 100°C cools to E Wm -2
\ a= 4 =
water at 100°C is T K4
(a) 540 cal (b) 54 cal 152. Assertion The rate of loss of heat of a body at 300 K is
(c) 5400 cal (d) 54000 cal R. At 900 K, the rate of loss becomes 81 R.
145. Heat required to melt 10 g of ice at 0°C to water at Reason This is as per Newton’s law of cooling.
0°C is 153. Assertion When temperature difference across the
(a) 800 cal (b) 80 cal two sides of a wall is increased, its thermal
(c) 8 cal (d) None of these conductivity increases.
146. SI unit of latent heat is Reason Thermal conductivity depends on nature of
(a) J kg–1 (b) J kg–1 K–1 material of the wall.
(c) kg J–1 (d) kg J–1 K–1
154. Assertion Cooking in a pressure cooker is faster.
Passage III Reason Because steam does not leak out.
A cylinder is containing nitrogen at 2 atm and 155. Assertion For higher temperatures, the peak
temperature 17°C. The radius of a nitrogen molecule emission wavelength of a black body shifts to lower
to be roughly 1.0A. Molecular mass of nitrogen wavelengths.
= 28.0 u, Boltzmann constant, k = 1.38 ´ 10–23 JK –1. Reason Peak emission wavelength of a black body is
147. The mean free path of nitrogen molecule is proportional to the fourth power of temperatures.
(a) 1.1 ´ 10 –7 m (b) 2.1 ´ 10 –7 m 156. Assertion Two bodies at different temperature, if
(c) 3.1 ´ 10 –7 m (d) 0.8 ´ 10 –7 m brought in thermal contact do not necessary settle to
the mean temperature.
148. Root mean square velocity of nitrogen molecules
Reason The two bodies may have different thermal
(a) 4 ´ 102 ms –1 (b) 5 ´ 102 ms –1
2 –1
capacities.
(c) 6 ´ 10 ms (d) 7 ´ 102 ms –1
157. Assertion When small temperature difference
149. Collision frequency of the nitrogen molecules is between a liquid and its surrounding is doubled, the
(a) 4 ´ 10 9 s –1 (b) 5 ´ 10 9 s –1 rate of loss of heat of the liquid becomes twice.
(c) 6 ´ 10 9 s –1 (d) 8 ´ 10 9 s –1 Reason This is as per Newton’s law of cooling.
Telegram @unacademyplusdiscounts

542 JEE Main Physics

158. Assertion When temperature of a black body is If the temperature is doubled and oxygen gas
halved, wavelength corresponding to which energy dissociates into atomic oxygen, the rms speed would
radiated is maximum becomes twice. be 2 c.
Reason This is as per Wien’s law. Reason c µ
T
M
159. Assertion When speed of sound in a gas is c, then
3 161. Assertion The number of molecules in 1 cc of water is
crms = ´c 1
g nearly equal to ´ 1022 .
3
g p Reason The number of molecules per gram mole of
Reason c=
r water is equal to Avogadro’s number
160. Assertion The root mean speed (rms) of oxygen ( = 6.023 ´ 1023 g –1 mol –1).
molecules at a certain absolute temperature T is c.

Previous Years’ Questions


162. Statement I The temperature dependence of pressure p1 and temperature T1. The other chamber
resistance is usually given as R = R0 (1 + Dt). The has volume V2 and contains ideal gas at pressure p2
resistance of a wire changes from 100 W to 150 W and temperature T2 . If the partition is removed
when its temperature is increased from 27°C to without doing any work on the gas, the final
227°C. This implies that a = 2.5 ´ 10–3° C–1. equilibrium temperature of the gas in the container
Statement II R = R0 (1 + a Dt) is valid only when the will be [AIEEE 2008]

change in the temperature is small and T1T2 ( p1V1 + p2V2 ) 1 2 + p2VT


T1VT 2 2
(a) (b)
DR = ( R - R0 ) << R0 [AIEEE 2009] p1VT1 2 + p2VT2 1 p1V1 + p2V2
(a) Statement I is true, Statement II is true 1 2 + p2VT
T1VT 2 1 T1T2 ( p1 + p2V2 )
(c) (d)
Statement II is the correct explanation of Statement I p1V1 + p2V2 p1VT1 1 + p2VT2 2
(b) Statement I is true, Statement II is true
167. By increasing the temperature of a liquid its
Statement II is not the correct explanation of
[UP SEE 2008]
Statement I
(a) volume and density decrease
(c) Statement I is false, Statement II is true
(b) volume and density increase
(d) Statement I is true, Statement II is false
(c) volume increases and density decreases
5
163. If one mole of a monoatomic gas æç g = ö÷ is mixed with (d) volume decreases and density increases
è 3ø
æ 7ö 168. What is an ideal gas? [UP SEE 2008]
one mole of a diatomic gas ç g = ÷, the value of g for (a) One that consists of molecules
è 5ø
(b) A gas satisfying the assumptions of kinetic theory
the mixture is [UP SEE 2009]
(c) A gas having Maxwellian distribution of speed
(a) 1.40 (b) 1.50
(d) A(c)gas consisting of massless particles
1.53 (d) 3.07 169. A body cools from 50°C to 49°C in 5 s. How long will it
164. 1 kg of diatomic gas is at a pressure of 8 ´ 10 4
Nm–2. take to cool from 40°C to 39°C? Assume temperature
The density of the gas is 4 kg m–3. What is the energy of surroundings to be 30°C and Newton’s law of
of the gas due to its thermal motion? [AIEEE 2009] cooling is valid [BVP Engg. 2008]

(a) 5 ´ 10 4 J (b) 6 ´ 10 4 J (a) 2.5 s (b) 10 s


(c) 20 s (d) 5 s
(c) 7 ´ 10 4 J (d) 3 ´ 10 4 J
170. A slab consists of two portions of different materials
165. Six molecules have speeds 2 unit, 5 unit, 3 unit, of same thickness and having the conductivities K 1
6 unit, 3 unit and 5 unit. The rms speed is [WB JEE 2008] and K 2 . The equivalent thermal conductivity of the
(a) 4 unit (b) 1.7 unit slab is [Karnataka CET 2008]
(c) 4.2 unit (d) 5 unit K1K2
(a) K1 + K2 (b)
166. An insulated container of gas has two chambers K1 + K2
separated by an insulating partition. One of the 2 K1K2
(c) (d) K1 + K2
chambers has volume V1 and contains ideal gas at K1 + K2
Telegram @unacademyplusdiscounts

Heat and Kinetic Theory of Gases 543

171. The plots of intensity of radiation versus wavelength 176. On the basis of kinetic theory of gases, the mean
of three black bodies at temperatures T1, T2 and T3 kinetic energy of 1 mol per degree of freedom is
are shown in figure. Then, [Kerala CET 2008] [BVP Engg. 2006]
I 1 1 3 3
(a) kT (b) RT (c) kT (d) RT
T1 T2 2 2 2 2
T3
177. Thermal radiations are electromagnetic waves
belonging to [BVP Engg. 2006]

λ (a) ultraviolet region (b) visible region


(c) gamma region (d) infrared region
(a) T3 > T2 > T1 (b) T2 > T3 > T1
(c) T1 > T2 > T3 (d) T1 > T3 > T2 178. Pressure of an ideal gas is increased by keeping
temperature constant. What is the effect on kinetic
172. Two rigid boxes containing different ideal gases are energy of molecules? [UP SEE 2006]
placed on table. Box A contains one mole of nitrogen
(a) Increase (b) Decrease
at temperature T0 , while box B contains 1 mole of
(c) No change (d) Can’t be determined
helium at temperature (7/3) T0 . The boxes are then
put into thermal contact with each other, and heat 179. Two balloons are filled, one with pure helium gas and
flows between them until the gases reach a common the other by air, respectively. If the pressure and
final temperature (Ignore the heat capacity of boxes) temperature of these balloons are same then the
then, the final temperature of gases, Tf , in terms of number of molecules per unit volume is [UP SEE 2006]
T0 is [Kerala CET 2008] (a) more in the helium filled balloon
3 7 (b) same in both balloons
(a) Tf = T0 (b) Tf = T0
7 3 (c) more in air filled balloon
3 5 (d) in the ratio of 1 : 4
(c) Tf = T0 (d) Tf = T0
2 3 180. When you make ice cubes, the entropy of water
pV [UP SEE 2006]
173. The value of for one mole of an ideal gas is nearly
T (a) does not change
equal to [BVP Engg. 2007] (b) increase
(a) 2 J mol–1K–1 (b) 8.3 J mol–1K–1 (c) decreases
(c) 4.2 J mol–1K–1 (d) 2 cal mol–1K–1 (d) may either increase or decrease depending on the process
174. The value of a metal sphere increase by 0.24% when used
its temperature is raised by 40°C. The coefficient of 181. The average energy for molecules in one degree of
linear expansion of the metal is...°C–1. freedom is [BVP Engg. 2006]
3 kT 3
[BVP Engg. 2007] (a) kT (b) (c) kT (d) kT
2 2 4
(a) 2 ´ 10 -5 (b) 6 ´ 10 -5
(c) 18 ´ 10 -5 (d) 1.2 ´ 10 –5 182. The thermoelectric power for a thermocouple at the
neutral temperature is [BVP Engg. 2005]
175. The temperature of the two outer surface of a
composite slab, consisting of two materials having (a) zero (b) maximum
coefficients of thermal conductivity K and 2K and (c) negative (d) minimum but not zero
thickness x and 4x, respectively are T2 and 183. The temperature of equal masses of three different
T1 ( T2 > T1). The rate of heat transfer through the liquids A, B and C are 12°C and 28°C respectively.
æ A ( T2 - T1) K ö The temperature when A and B are mixed is 16°C
slab, in a steady state is ç ÷ f , with f
è x ø and when B and C are mixed is 23°C. The
equal to [UP SEE 2007] temperature when A and C are mixed is
x 4x [Kerala CET 2005]
(a) 18.2°C (b) 22°C (c) 20.2°C (d) 24.2°C

T2 K 2K T1 184. A black body has maximum wavelength l m at


2000 K. Its corresponding wavelength at 3000 K will
be [Kerala CET 2005]
3 2 16 81
1 2 1 (a) lm (b) lm (c) lm (d) lm
(a) 1 (b) (c) (d) 2 3 81 16
2 3 3
Telegram @unacademyplusdiscounts

544 JEE Main Physics

185. A body with area A at maintained temperature T and maintained at temperatures 2 T and 3 T respectively.
emissivity e = 0.6 is kept inside a spherical black The temperature of the middle ( i. e., second) plate
body. What will be the maximum energy radiated per under steady state condition is [IIT JEE 2012]
second? [IIT JEE 2005] æ 65 ö
1/4
æ 97 ö
1/4
4 4 (a) ç ÷ T (b) ç ÷ T
(a) 0.60 s AT (b) 0.80 s AT è 2ø è4ø
(c) 1.00 s AT 4 (d) 0.40 s AT 4 æ 97 ö
1/4
(c) ç ÷ T (d) (97)1/4 T
186. Calorie is defined as the amount of heat required to è2ø
raise temperature of 1 g of water by 1°C and it is 191. A long metallic bar is carrying heat from one of its
defined under which of the following conditions ? ends to the other end under steady state. The
[IIT JEE 2005] variation of temperature q along the length x of the
(a) From 14.5 °C to 15.5 °C at 760 mm of Hg bar from its hot end is best described by which of the
(b) From 98.5 °C to 99.5 °C at 760 mm of Hg following figures? [AIEEE 2009]
(c) From 13.5 °C to 14.5 °C at 76 mm of Hg
(d) From 3.5 °C to 4.5 °C at 76 mm of Hg
187. In which of the following process, convection does not (a) θ (b)
take place primarily ? [IIT JEE 2005]
x x
(a) Sea and land breeze
(b) Boiling of water
(c) Warming of glass of bulb due to filament θ
(d) Heating air around a furnace (c) (d)

188. Figure shows a system of two x x


concentric spheres of radii r1 and r2
kept at temperatures T1 and T2 , r1
192. Assuming the sun to be a spherical body of radius R
respectively. The radial rate of flow
T1 at a temperature of T K evaluate the total radiant
of heat in a substance between the T2 power incident on the earth at a distance r from the
r2
two concentric spheres is sun. [AIEEE 2006]

proportional to [AIEEE 2005] (a) pr20 R2sT 4 / r2 (b) r20 R2sT 4 / 4 pr2
r12r ær ö r2 - r1 (c) R2sT 4 / r2 (d) 4 pr20 R2sT 4 / r2
(a) ( r2 - r1 ) (b) (c) ln ç 2 ÷ (d)
( r2 - r1 ) è r1 ø r12r where r0 is the radius of the earth and s is stefan’s constant.

189. A liquid in a beaker has temperature q ( t) at time t 193. Two rigid boxes containing different ideal gases are
and q 0 is temperature of surroundings, then placed on a table box A contains one mole of nitrogen
according to Newton’s law of cooling the correct at temperature T0 while box B contains one mole of
7
graph between log e (q - q 0 ) and t is [AIEEE 2012[ helium at temperature æç ö÷ T0 . The boxes are then
è3 ø
put into thermal contact with 0each other and heat
loge (θ – θ0)

loge (θ – θ0)

flows between them untill the gases reach a common


(a) (b) final temperature (ignore the heat capacity of boxes).
Then, the final temperature of the gases in terms of
T0 is [AIEEE 2006]
O t O t 7 3 5 3
(a) Tf = T0 (b) Tf = T0 (c) Tf = T0 (d) Tf = T0
3 2 2 7
loge (θ – θ0)

loge (θ – θ0)

194. Three perfect gases at absolute temperature T1, T2


(c) (d) and T3 are mixed. The masses of molecules are m1, m2
and m3 and the number of molecules are n1, n2 and n3
respectively. Assuming no loss of energy the final
O t O t temperature of the mixture is [AIEEE 2011]
190. Three very large plates of some area are kept parallel (T1 + T2 + T3 ) n1T1 + n2T2 + nT33
(a) (b)
and close to each other. They are considered as ideal 3 ( n1 + n2 + n3 )
black surfaces and have very high thermal n1T12 + n2T22 + nT
33
2
n21T12 + n22T22 + n32T32
(c) (d)
conductivity. The first and third plates are n1T1 + n2T2 + nT33 ( n1T1 + n2T2 + nT 3 3)
Telegram @unacademyplusdiscounts

Heat and Kinetic Theory of Gases 545

195. One kg of a diatomic gas is at a pressure of 197. If a piece of metal is heated to temperature q and then
8 ´ 106 N / m 2 . The density of the gas is 4 kg/m 3. allowed to cool in a room which is at temperature q 0 ,
What is the energy of the gas due to its thermal the graph between the temperature T of the metal
motion? [AIEEE 2009] and time t will be closed to [JEE Main 2013]
(a) 3 ´ 10 4 J (b) 5 ´ 10 4 J
(c) 6 ´ 10 4 J (d) 7 ´ 10 4 J T T
196. Two thermally insulated vessels 1 and 2 are filled (a) (b) θ0
with air at temperatures ( T1, T2 ) volumes ( V1, V2 ) and
pressures ( p1, p2 ) respectively of the value joining the O t O t
two vessels is opened the temperature inside the
vessel at equilibrium will be [AIEEE 2008, 04] T T
(a) T1 + T2 (b) (T1 + T2 ) / 2 (c) θ0 (d) θ0
T T ( p V + p2V2 ) T T ( p V + p2V2 )
(c) 1 2 1 1 (d) 1 2 1 1
1 2 + p2VT
p1VT 2 1 1 1 + p2VT
p1VT 2 2 O t O t

Answers
Round I
1. (a) 2. (b) 3. (c) 4. (d) 5. (b) 6. (b) 7. (c) 8. (c) 9. (c) 10. (c)
11. (b) 12. (c) 13. (a) 14. (a) 15. (c) 16. (a) 17. (a) 18. (a) 19. (a) 20. (b)
21. (c) 22. (a) 23. (a) 24. (b) 25. (c) 26. (d) 27. (b) 28. (a) 29. (b) 30. (a)
31. (a) 32. (c) 33. (d) 34. (a) 35. (b) 36. (c) 37. (d) 38. (c) 39. (a) 40. (b)
41. (b) 42. (a) 43. (a) 44. (b) 45. (c) 46. (a) 47. (a) 48. (d) 49. (c) 50. (a)
51. (d) 52. (a) 53. (a) 54. (a) 55. (b) 56. (a) 57. (b) 58. (c) 59. (b) 60. (a)
61. (b) 62. (d) 63. (c) 64. (a) 65. (a) 66. (a) 67. (c) 68. (c) 69. (d) 70. (a)
71. (b) 72. (b) 73. (a) 74. (a) 75. (a) 76. (a) 77. (b) 78. (d) 79. (a) 80. (b)
81. (d) 82. (d) 83. (c) 84. (c) 85. (a) 86. (c) 87. (c) 88. (d) 89. (a) 90. (a)
91. (b) 92. (d) 93. (a) 94. (d) 95. (b) 96. (a) 97. (d) 98. (c) 99. (b) 100. (a)
101. (c) 102. (c) 103. (b) 104. (b) 105. (b) 106. (c) 107. (d) 108. (d) 109. (b) 110. (b)
111. (d) 112. (d) 113. (a) 114. (b) 115. (a) 116. (c) 117. (d) 118. (b) 119. (a)

Round II
1. (d) 2. (b) 3. (a) 4. (a) 5. (a) 6. (b) 7. (c) 8. (a) 9. (d) 10. (d)
11. (c) 12. (a) 13. (a) 14. (d) 15. (b) 16. (d) 17. (c) 18. (b) 19. (b) 20. (d)
21. (d) 22. (b) 23. (a) 24. (c) 25. (d) 26. (a) 27. (a) 28. (c) 29. (d) 30. (a)
31. (d) 32. (c) 33. (a) 34. (d) 35. (c) 36. (a) 37. (c) 38. (b) 39. (a) 40. (d)
45. (d) 41. (c)
46. (c) (d) (b)
47. 42. (b)
48. 43. 49. (b) 50. (b)
51. (a) 52. (b) 53. (a) 54. (a) 55. (b) 56. (c) 57. (b) 58. (a) 59. (a) 60. (c,d)
61. (c) 62. (c) 63. (b) 64. (a) 65. (b) 66. (b) 67. (a) 68. (a) 69. (b) 70. (b)
71. (a) 72. (c) 73. (b) 74. (d) 75. (b) 76. (c) 77. (b) 78. (b) 79. (d) 80. (a)
81. (c) 82. (a) 83. (c) 84. (a) 85. (c) 86. (c) 87. (d) 88. (b) 89. (d) 90. (b)
91. (a) 92. (c) 93. (c) 94. (d) 95. (c) 96. (a) 97. (a) 98. (d) 99. (b) 100. (a)
101. (a) 102. (a) 103. (b,c) 104. (b) 105. (c) 106. (c) 107. (c) 108. (a) 109. (c) 110. (c)
111. (a) 112. (c) 113. (a) 114. (d) 115. (c) 116. (c) 117. (b) 118. (a) 119. (d) 120. (b)
121. (a) 122. (c) 123. (b) 124. (d) 125. (c) 126. (b) 127. (b) 128. (b) 129. (b,c) 130. (c)
131. (b,d) 132. (b,d) 133. (a,d) 134. (a,b,d) 135. (a,b,c) 136. (a,b,d) 137. (a,b) 138. (b,d) 139. (a,d) 140. (a)
141. (b) 142. (d) 143. (b) 144. (c) 145. (a) 146. (a) 147. (a) 148. (b) 149. (b) 150. (a)
151. (a) 152. (c) 153. (d) 154. (c) 155. (c) 156. (a) 157. (a) 158. (a) 159. (b) 160. (a)
161. (a) 162. (c) 163. (b) 164. (a) 165. (c) 166. (a) 167. (c) 168. (b) 169. (b) 170. (c)
171. (c) 172. (d) 173. (d) 174. (a) 175. (d) 176. (b) 177. (d) 178. (c) 179. (b) 180. (c)
181. (b) 182. (b) 183. (c) 184. (b) 185. (a) 186. (a) 187. (c) 188. (d) 189. (a) 190. (c)
191. (c) 192. (a) 193. (b) 194. (b) 195. (b) 196. (c) 197. (c)
Telegram @unacademyplusdiscounts

the Guidance
Round I
1. Given, triple point of water on scale A = 200 A m
7. Heat capacity/volume = c ´ = c ´r
V
Triple point of water on scale B = 350 B c1 r1 3 5
We know that triple point of water on absolute scale Desired ratio = = ´ =1 :2
c2 r 2 5 6
= 27316
. K
\ 200 A = 350B = 27316. K 8. Infinite thermal capacity implies that there would be
27316
. 27316
. practically no change in temprature whether heat is taken in
\ 1A = K and 1B = K
200 350 or given out.
If TA and TB are the triple point of water on two scales A and 9. As piston is of fixed mass M, and it is able to move up or down
B, then without friction, therefore, when temperature is increased,
27316. 27316
. piston moves out, increasing the volume V. The pressure p on
TA = TB
200 350 the gas remains the same, because of fixed mass.
TA 200 4 4 mgh
\ = = or TA = TB 10. Heat absorbed by water = Heat produced mc DT =
TB 350 7 7 J
gh 980 ´ 500 ´ 100 900
C x - lower fixed point 68 - 5 63 7 DT = = = = 1.16° C
2. = = = = Jc 4.2 ´ 10 7 ´ 1 420
100 upper fixed point – lower point 95 - 5 90 10
700 11. From the principle of calorimetry,
ÞC= = 70°
10 m1s1Dq1 = m2s2Dq2
sw
3. Let, F = K = X Þ 540 ´ sw ´ (80 - x ) = 540 ´ ´ ( x - 0)
2
F - 32 K - 273 x - 32 x - 273 160°
As = \ = Þ x= C
9 5 9 5 3
9x - 2457 = 5x - 160
12. Let m gram of water, whose temperature is q ( > 30° C) be
4x - 2457 + 160 = 0
2297 added to 20 g of water at 30°C. Then
x= = 574.25°
4 m ´ 1 ( q - q0) = 20 ´ 1 ( q0 - 30)
(m + 20) q0 = 60 + mq
4. Let, q be the temperature of the mixture.
600 + mq
Heat gained by water at 0°C = Heat lost by water at 10°C q0 =
20 + m
c1 m1 ( q - 0) = cm2 (10 - q)
400 For q0 to be maximum m should be small and q should be
q= = 6.66° C large.
60
50
5. From graph, we note that for scale A, the lowest fixed point is 13. Here, (KE of rotation) = cm q
100
higher than 0° A and the higest point is 180°A. For scale B, 1 æ 1 2ö 1 æ 2 2ö 2
the lowest point is 0°B and the highest point 100°B ç Iw ÷ = cm q ç Ir ÷ (2 pn) = cmq
2 2è ø 4 è5 ø
t - 30 t -0 t
Therefore, the relation A = B = B is correct. 2 p 2n 2r 2
150 100 100 Þ q=
5 c
6. Heat required to melt 1 g of ice at 0°C to water at 0°C
= 1 ´ 80 cal
14. When pressure of an ideal gas is constant, Charles’ law is
obeyed i.e.,
Heat required to raise temperature of 1 g of water from 0°C
V 1
to 100°C = 1 ´ 1 ´ 100 = 100 cal V µ T or = constant =
T p
Total heat required for maximum temperature of100° C
From the slope of curve shown in figure 3; p1 > p2.
= 80 + 100 = 180 cal
Dq Aes (T 4 - T04) Dq
As one gram of steam gives 540 cal of heat when it is 15. Rate of cooling = Þ µA
converted to water at 100°C therefore, temperature of the t mc t
mixture would be 100°C. Since area of plate is largest so it will cool fastest and sphere
the slowest.
Telegram @unacademyplusdiscounts

Heat and Kinetic Theory of Gases 547

f From, r 0 = r100 (1 + g ´ 100)


16. Relative humidity (RH) = ´100%
F r -r 10 - 9.7
g = 0 100 = = 3.09 ´ 10 –4
where, f and F are vapour pressures at closed point and air r100 ´ 100 9.7 ´ 100
temperature.
g 3.09 ´ 10 –4
temperature at dew point a= = = 1.03 ´ 10 –4° C–1
Þ RH = ´100 3 3
Air temperature
x 23. As, V = L3
Þ RH = t ´ 100 = 100% DV ´ 100 DL ´ 100
x \ =3 = 3 ´ 0.2% = 0.6%
V L
where, dew point = air temperature
17. Let V be the volume of the sphere and r be the density of 24. The pressure of the gas inside the vessel, as observed by us,
on the ground remains the same. This is because motion of
water. Buoyancy (F) on sphere due to water is the vessel as a whole does not effect the relative motion of gas
F = V r g or F µ r molecules and the walls of the vessel.
Since, r 0°C > r 4°C, so, F0°C > F4°C,
25. From, l2 = l1 [1 + a (t 2 - t1)]
18. With increases in temperature, the effective length (l) of l2 - l1
simple pendulum increases even though its centre of mass Þ t 2 = t1 +
l1a
still remains at the centre of the bob. As time period,
T = 2 p l/g or T µ l. So, T increases as temperature -10 3
= 20 + = -30°C
increases. 1.0 ´ 2 ´ 10 –5

19. Here, a (steel) = 1.1´10 –5°C–1 26. Since, a Al > asteel, so in bimetallic strip on heating,
aluminium strip will expand more than that of steel strip. Due
a (copper) = 1.7 ´ 10 –5°C–1
to it, aluminium strip will bend more on convex side and steel
l0( s) a( c) 1.7 ´ 10 –5 strip on concave side.
As, = = = 1.545
l0( c) a( s) 1.1 ´ 10 –5 Al
\ l0( s) = 1.545 l0( c) Steel

Also, l0( s) - l0( c) = 5


0.545 l0( c) = 5 27. As aB > aA , therefore strip B will appear on outer side.
5 28. Here, DT = 20 - 15 = 5°C
Þ l0( c) = = 9.17 cm
0.545
a = 0.00012°C–1 = 12 ´ 10 -6°C–1
and l0( s) = 1.545 ´ 9.17 cm = 14.17 cm
1
20. Here, g ag = 10.30 ´ 10 –4°C–1 Time lost per day = a ( DT) ´ 86400 s
2
g am = 10.06 ´ 10 –4°C–1 1
= ´ 12 ´ 10 -6 ´ 5 ´ 86400 s = 2.590 s
2
a a = 9 ´ 10 -60 °C-1, a m = ? ~
- 2.6 s
Now, g r = g ag + g glass = g am + g m 29. When a metallic rod is heated it expands. Its moment of
\ 10.30 ´ 10 –4 + 3 ´ 9 ´ 10 -6 = 10.06 ´ 10 –4 + g m inertia (I) about a perpendicular bisector increases. According
(Q g g = 3 ´ a a ) to law of conservation of angular momentum, its angular
–4 speed ( w) decreases, since w µ1/ l (according to law of
\ g m = (10.30 +0.27 - 10.06)10 = 0.51´ 10 –4
conservation of angular momentum)
1 0.50 ´ 10 -4
am = gm = 30. From, T = 2p
l
, we get
3 3 g
= 0.17 ´ 10 –4 = 17 ´ 10 -6°C–1 DT 1 Dl a DT
= =
= 1.7 ´ 10 –5°C–1 T 2 l 2
F L 1
21. As, DL = aL ( DT) = = ´ 2 ´ 10 -6 ´ 10 = 10 -3%
AY 2
\ F = a ( DT) AY h1 r 2 (1 + g q1) é p0 ù
31. As, = = êQ r = 1 ú
= 1.1 ´ 10 –5 ´ (50 - 30) ´ 2 ´ 10 -6 ´ 2 ´ 1011 h2 r1 (1 + g q2) ë + g qû
= 88 N 50 1 + g ´ 50
Þ =
22. Here, r 0 = 10 g / cc 60 1 + g ´ 100
and r100 = 9.7 g /cc, a = ? Þ g = 0.005/° C
Telegram @unacademyplusdiscounts

548 JEE Main Physics

32. As, L = L0 (1 + a Dq) 38. A bimetallic strip on being heated bends in form of an arc with
-6 more expandable metal ( A) outside (as shown)
L1 1 + a ( Dq1) 10 1 + 11 ´ 10 ´ 20
= = =
L2 1 + a ( Dq2) L2 1 + 11 ´ 10 -6 ´ 19
Þ L2 = 9.99989
Length is shorter by A B B
–5
» 10 - 9.99989 = 0.00011 = 11 ´ 10 cm A αB

33. Let V be the volume of sphere of radius R at temperature. then αA αB αA


4
V= pR3 .
3 DQ æ DT ö 2 DT
39. = KA ç ÷ = K ( pr )
Increase in volume of sphere with rise in temperature D T is Dt è Dx ø ( l)
4 æ DQ ö r
2
D V = g V D T = 3a ´ p R3 D T = 4 p R3 a D T \ç ÷ µ , which is maximum in case (a).
3 è Dt ø l
DV 0.24
34. Here, g= = = 6 ´ 10 -5 /°C 40. For parallel combination of two rods of equal length and
V ´ DT 100 ´ 40
equal area of cross-section.
g
a = = 2 ´ 10 -5 /°C 4 K1
3 K1 +
K + K2 3 = 7 K1
K= 1 =
35. Here, KA = 2 KB (dx) A = d (dx)B, 2 2 6
If q is temperature of junction, (dT) A = qA - q, K 7
Hence, =
(dT)B = ( q - qB) K1 6
æ dQ ö æ dQ ö DQ ( Dx )
As, ç ÷ =ç ÷ 41. As, Dt =
è dt ø A è dt ø KA ( DT)
(dT) A A (dT)B
\ KA A = KB when two rods of same length are joined in parallel, A ® 2
(dx) A (dx)B 1
times and ( Dx) = times.
2 KB( qA - q) = KB ( q - qB) 2
2 qA - 2 q = q - qB 1 1
\ D t becomes times, i. e. , ´ 12 s = 3 s
2 qA + qB = 3 q 4 4
As qA - qB = 48° …(i) 42. Let the temperature of junction be q.
qA = 48 + qB æ DQ ö æ DQ ö
ç ÷ =ç ÷
Put in Eq. (i) è Dt ø copper è Dt ø steel
2 ( 48 + qB) + qB = 3 q (100 - q) K2 A ( q - 0)
Þ K1A =
96 + 3 qB = 3 q 18 6
96 = 3 ( q - qB) (100 - q)
Þ 9 K2 = K2q
\ q - qB = 96 /3 = 32° C 3
or 3 q = 900 - 9 q
36. As is clear from figure.
or 12 q = 900
dQ dQ1 dQ 2
= + or q = 75°C
dt dt dt
K ( A1 + A2) dT dT dT 43. The equivalent electrical circuit, figure in this case is of
= K1A1 + K2A2
dx dx dx Wheatstone bridge. No current would flow through central
K1A1 + K2A2 rod CD when the bridge is balanced. The condition for
K=
A1 + A2 P R
balanced Wheatstone bridge is = (in terms of resistances)
37. Growth of ice in a pond is conduction process governed by Q S
rL y 2 1 / K1 1 / K3
the relation, t = =
Kq 2 1 / K2 1 / K4
The ratio of times for thickness of ice from 0 to y is 1 : 3 K2 K4
or =
\ Time taken to increase the thickness from 1 cm to 2 cm is K1 K3
equal to 3 ´ 7 = 21 h.
or K1K4 = K2 K3
Telegram @unacademyplusdiscounts

Heat and Kinetic Theory of Gases 549

44. Let L be the length of each rod. æ DQ ö æ DQ ö


47. As, ç ÷ =ç ÷
Temperature of A = 60° C temperature of E = 10° C è D t øP è D t øQ

Let q1, q2, q3 be respective temperture of B, C , D. (T1 - T2) (T - T )


Þ K1A1 = K2A2 1 2
If Q1, Q 2, Q3 , Q 4 , Q5 , Q 6 are the amounts of heat l l
following/sec respectively from A to B; B to C ; B to D; C to D; D or K1A1 = K2A2
to E and C to E, then using figure. A1 K2
or =
C (θ2) A2 K1
Q2 DT Dx l
Q6
x x 48. Rth = = (Q Dx = l )
Q1 ( DQ / Dt ) KA KA
x Q4 E (10°)
A(60°) y B
(θ1)
y y 49. For the same heat to be conducted, temperature difference
Q3 Q5 must be same.
D (θ ) Initial temperature difference = 10 - ( -10) = 20° C = 20 K
3
Outside temperature = -23° C = -23 + 273 = 250 K
0.46 A (60 - q1) 0.92 A ( q1 - q2) Inside temperature = 250 + 20 = 270 K
Q1 = , Q2 =
L L
Q kADq Dq Dq
0.46 A ( q1 - q3) 0.92 A ( q2 - q3) 50. As, = = = (R = Thermal resistance)
Q3 = , Q4 = t l ( l / kA) R
L L
0.46 A ( q3 - 10) 0.92 A ( q2 - 10) Þ t µR
Q5 = , Q6 = t p Rp R /2 1
L L Þ = = =
Now Q1 = Q 2 + Q3 t s Rs 2 R 4
0.46 A (60 - q1) 0.92 A ( q1 - q2) 0.46 A ( q1 - q3) ts 4
= + Þ tp = = = 1min
L L L 4 4
60 - q1 = 2 ( q1 - q2) + q1 - q3 Series resistance, Rs = R1 + R2 and parallel resistance,
or 4 q1 = 2 q2 - q3 = 60° …(i) RR
Rp = 1 2
Again, Q 2 = Q 4 + Q 6 gives R1 + R2
q1 - 3 q2 - q3 = 10° …(ii) K1 5
51. Given A1 = A2 and =
Again, Q5 = Q3 + Q 4 given K2 4
q1 + 2 q2 - 4 q3 = 10° …(iii) Q R1 = R2
Solving Eqs. (i), (ii) and (iii), we get l1 l
Þ = 2
q1 = 30° C, q2 = 20° C, q3 = 20° C k1A k2A
dQ dT l1 k1 5
45. As = KA , therefore, when Þ = =
dt dx l2 k2 4
1 1
dt ® , A ® (2) 2 = 4, K ® 52. Natural convection arises due to difference of density at two
2 4 places and is a consequence of gravity.
dQ
becomes twice; m would become twice. 53. Convection is not possible in weigtlessness so the liquid will
dt
be heated through conduction.
Mass of ice melted/s = 2 ´ 0.1 g = 0.2 g s -1
54. Here, lm = 289.8 nm = 289.8 ´10 –9 m
46. For the two sheets, shown in figure, rate of heat transfer is
s = 5.67 ´ 10 –8 Wm–2 K –4
same, i. e. ,
dQ1 dQ 2 b = 2889 m mK = 2889 ´ 10 -6 mK
=
dt dt If T is temperature of star, then
dT1 dT2
\ = according to Wien’s law l mT = b
R1 R2
b 2889 ´ 10 –6
q1 - q q - q2 T= = = 10 4
= l m 289.8 ´ 10 –9
R1 R
From Stefan’s law, E = sT 4
Þ q1R2 - qR2 = qR1 - q2R1
q R + q2R1 = 5.67 ´ 10 –8 (10 4) 4
or q= 1 2
R1 + R2 = 5.67 ´ 10 8 Wm–2
Telegram @unacademyplusdiscounts

550 JEE Main Physics

55. On heating, black spot absorbs maximum radiation. 60. According to Wien’s displacement law,
Therefore, when plate is taken to a dark room, the spot will 1 c 1
lm µ or µ
emit more radiations than the rest of the plate. Hence, it will T vm T
appear brighter than the plate. i. e. , vm µ T
4
E æT ö Hence, curve A represents the correct variation.
56. As, 2 = ç 2 ÷
E1 è T1 ø 61. Given, T1 = 7° C = 7 + 273 = 280 K
4
E 2 æ 727 + 273 ö T2 = 287° C = 287 + 273 = 560 K
\ =ç ÷ = 16 or E 2 = 16 E
7 è 0 + 273 ø 4 4
E 2 æ T2 ö æ 560 ö 4
\ =ç ÷ =ç ÷ = 2 = 16
57. Cooling rate (R) is rate of fall of temperature, according to E1 è T1 ø è 280 ø
Newton’s law. It varies inversely as specific heat of the liquid.
For A, rate of cooling is larger. Therefore, specific heat of A is
62. From Wien’s law lmT = b
smaller. b 2.88 ´ 10 6
lm =
= = 10 3 nm
58. When temperature of a black body is increased by 50% T 2880
150 2 From the shape of the E versus graph U2 > U1
T2 = T1 = T1
100 3 T1 l m2 10 -4
63. As, = = = 200
According to Stefan’s law T2 l m1 0.5 ´ 10 -6
4 4
E 2 æ T2 ö æ3ö 81 T1 lm 11 ´ 10 -5
=ç ÷ =ç ÷ = 64. As, =n = 1 = =2
E1 è T1 ø è2ø 16 T2 l m2 5.5 ´ 10 –5
Percentage increase in radiation
(E 2 - E1) (81 - 16)
65. According to Wein’s law, as T increases, lm decreases and v m
´ 100 = ´ 100 » 400% increase.
E1 16
\ T1 < T2
59. Given, initial temperature T1 = 80°C Therefore, v m1 < v m2

Final temperature T2 = 50 °C 66. According to Newton’s law of cooling,


Temperature of the surroundings T0 = 20 °C dq
- µ ( q - q0)
t1 = 5 min dt
dq
According to Newton’s law of cooling. - = K ( q - q0)
dt
dT é T + T2 ù
Rate of cooling, =kê 1 - T0 ú where, K is a constant of proportionality.
dt ë 2 û
75 - 65 æ 75 + 65 ö 1
(80 - 50) é 80 + 50 ù Now, =K ç - 30 ÷ = K ´ 40 Þ K =
=kê - 20 ú 2 è 2 ø 8
5 ë 2 û
30 In case of second identical object
= k(65 - 20) 55 - 45 æ 55 + 45 ö
5 =K ç - 30 ÷
t è 2 ø
6 = k ´ 45
6 2 10 1
or k= = …(i) = ´ 20
45 15 t 8
80
In second condition, Þ t= = 4 min
20
Initial temperature T1¢ = 60°C
T2 l m1 1.78
Final temperature T2¢ = 30°C 67. As, = =
T1 l m2 14
t¢ = ?
(60 - 30) 2 æ 60 + 30 ö 1.78 1.78
Now, = ç - 20 ÷ T2 = ´ T1 = (1373 + 273)
t¢ 15 è 2 ø 14 14
30 2 = 209.3 K = 309.3 - 273 = 63.7° C
= ( 45 - 20)
t ¢ 15 68. Let R = radius of planet
30 ´ 15 P = power radiated by the sun
or t¢ =
2 ´ 25 P
Energy received by planet = ´ 4 pR 2
= 9 min 4 pd 2
Energy radiated by planet = ( 4 pR) 2sT 4
Telegram @unacademyplusdiscounts

Heat and Kinetic Theory of Gases 551

For thermal equilibrium, E1 T14 - T04 (327 + 273) 4 - (27 + 273) 4


75. As, = =
P E 2 T24 - T04 (227 + 273) 4 - (27 + 273) 4
4 pR 2(sT 4) = ´ 4 pR 2
4 pd 2
(600) 4 - (300) 4 10 8 (1296 - 81)
1 = =
T µ 2 4 (500) 4 - (300) 4 10 8 (625 - 81)
d
1215
1 = = 2.23
or T µ d -1/ 2 \ n = 544
2
76. For a perfectly black body, a = e = 1and r = 0
69. As energy emitted µ AT 4
4 77. Here, T1 = 27° C = (27 + 273) = 300 K
E 2 E ¢ 1 (327 + 273) 4 1 æ 600 ö 81
\ = = = ´ç ÷ =
E1 E 4 (127 + 273) 4
4 è 400 ø 64 and T2 = 327° C = (327+273) K = 600 K
81 According to Stefan’s law,
or E¢ = E 4 4
64 E 2 æ T2 ö æ 600 ö
=ç ÷ =ç ÷ = 16
70. According to Wein’s displacement law E1 è T1 ø è 300 ø
1 1 Þ E 2 = 16 E1
lm µ or T µ
T lm
78. According to Stefan’s law,
From figure, ( l m)1 < ( l m) 2 < ( l m)3 \ T1 > T2 > T3 E1 = sT 4 and E 2 = s (T + DT) 4
71. According to Newton’s law of cooling E 2 - E1 s [(T + DT) 4 - T 4 ] DT
q1 - q2 \ = =4
æq -q ö E1 sT 4 T
= K ç 1 2 - q0 ÷
t è t ø
E 2 - E1 2 DT
50 - 40 æ 50 + 40 ö Now, = =4
Þ =K ç - 20 ÷ E1 100 300
300 è 2 ø
300 ´ 2
1 \ DT = = 1.5 K
or K= 4 ´ 100
25 ´ 30
\Temperature of other patch = T + DT
40 - q æ 40 + q ö Kq q
Now, =K ç - 20 ÷ = = = 300 + 1.5 = 301.5 K
300 è 2 ø 2 1500
10
or 300 q = 60000 - 1500 q 79. Mass of ice melted/min = = 1g
10
60000
q= = 33.3° C Quantity of heat used = 1 ´ 80 cal
1800
4 Area of lens = pr 2 = 3.14 (2.5) 2 = 19.625 cm2
æ T2 ö E
72. As, ç ÷ = 2 \Amount of heat received/min/cm 2 from sun
è T1 ø E1
80
1/ 4 1/ 4 = » 4 cal cm–2 min –1
T2 æ E 2 ö æ 32 ´ 10 5 ö 19.625
or =ç ÷ =ç ÷ =2
T1 è E1 ø è 2 ´ 10 5 ø 80. Here, T1 = 6000 K, l1 = 4800 Å
\ T2 = 2 T1 = 2 ´ (127 + 273) = 800 K T2 = 3000 K, l 2 = ?
= 800 - 273 = 527° C According to Wein’s law
73. According to Newton’s law of cooling, l 2 T1
\ =
Rate of heat loss = Temperature difference l1 T2
R2 40 - 20 1 T1 6000
\ = = Þ l2 = ´ l1 = ´ 4800 = 9600 Å
R1 80 - 20 3 T2 3000
R1 45 81. According to Wein’s displacement law
R2 = = = 15 cals–1
3 3 ( l m)1 T2 0.26
= = =2
l m2 T ( l y ) 2 T1 0.13
74. As, = 1 (Wein’s displacement law)
l m1 T2 4 4
E1 æ T1 ö æ 1ö 1
l m2 2000 \ =ç ÷ =ç ÷ =
Þ = E 2 è T2 ø è2ø 16
l m4 2400
4 4
æT ö 327 + 273 ö
Þ l m2 = 4 ´
20
= 3.3 mm 82. As, E 2 = E1 = ç 2 ÷ = 10 æç ÷ = 160 J
24 è T1 ø è ø
27 + 273
Telegram @unacademyplusdiscounts

552 JEE Main Physics

83. Here, T1 = 327° C = (327 + 273) K = 600 K 92. Here, p1 = p , V1 = V , T1 = T


and T2 = 927° C = (927+273) K = 1200 K T2 = 1.1 T , V2 = 1.05 V , p2 = ?
According to Stefan’s law p2V2 p1 V1
From =
E µT4 T2 T1
4 4
E 2 æ T2 ö æ1200 ö V1 T2
=ç ÷ =ç ÷ = 16 p2 = p1 ×
E1 è T1 ø è 600 ø V2 T1
E 2 = 16 E1 = 16 ´ 4 = 64 cal cm–2 s–1 =p´
V T
´ 1.1 = 1.1
p
1.05V T 1.05
84. Fall of temperature with time follows an exponential curve as
shown in option (c) figure. This is as per Newton’s law of p2 = 1.05 p
cooling. 93. Here, p1 = 1 atm; T1 = 300 K;
85. When both have same area and same temperature, they will V1 = 30000 cc = 30 L;
cool at the same rate.
T2 = 200 K; V2 = 5.2 L
1
86. As P µ T 4 and P µ 2, therefore, p V T 1 ´ 30 ´ 200
d \ p2 = 1 1 2 = = 3.86 atm
4 4 T1V2 300 ´ 5.2
T 2
Pµ µ = 2 2 = 4 times, i. e. , 4 P pV pV
d 2 22 94. As, pV = n RT, so = constant. Thus, the variation of
RT RT
pV 1 ´ 100
87. As, V1 = = = 25 cc and p will be horizontal straight line (1).
p1 4
5 95 V
95. New volume, V1 = V - V=
88. Applying standard gas equation, 100 100
p2V2 p1V1 pV pV 100
= New pressure, p1 = = = p
T2 T1 V1 (95 /100) V 95
p2 V1 T2 \% increase in pressure
= ×
p1 V2 T1 æ p - pö æ p1 ö
=ç 1 ÷ ´ 100 = ç - 1÷ ´ 100
T2 3000 è p ø èp ø
As = = 10 and every molecule of H2 splits into
T1 300 æ100 ö
=ç - 1÷ ´ 100 = 5.26
hydrogen atoms, doubling the number, therefore volume è 95 ø
available to given number of entities becomes half i.e.,
1 p 96. As pV 2 = a constant, therefore, when V becomes 2 V , p2
V2 = V1 therefore, 2 = 2 ´ 10 = 20 .
2 p1 1 1
becomes half i. e. , p 2 = or p = times.
2 2
89. Here, initially p1 = p,V1 = V + V = 2 V p
As = a constant, so T µ p.
Finally, p1 = p V2 = V T
As p1V1 = p2V2 Thus, T becomes T / 2.
pV p ´2V
or p2 = 1 1 = =2p 3 pV 3 RT
V V 97. As, c = =
M M
90. Here, p1 = p and T1 = T and the new rms speed,
0.4
p2 = p + p = 1.004 p 3 R (T /2) 1 3 RT
100 c1 = =
(2 M) 2 M
T2 = (T + 1)
c 300
p2 T2 = = = 150 ms–1
As = 2 2
p1 T1
gp 3 pV 3p
1.004 p T + 1 1 98. As, cs = and c = =
\ = = 1+ r M r
p T T
1 cs g
or T= = 250 K =
0.004 c 3
g
91. As the vessel contains 1 mole of hydrogen and 1 mole of or cs = c
exygen, therefore, as per Maxwell's law of speed distribution, 3
f1 (v) and f2 (v) will obey the law separately.
Telegram @unacademyplusdiscounts

Heat and Kinetic Theory of Gases 553

1M 2 3 pV 3 RT 1 3 3 RT ö
1/ 2
99. As, p =
3V
c or c2 =
M
=
M 105. As, Mc2 = RT or c = æç ÷
2 2 è M ø
2 3 ´ RT 3 RT
For gas A, V1 = = 1
M M Þ cµ
M
2 3 RT
For gas B, V2 = ; c1 æ M2 ö
1/ 2
M So, =ç ÷
V12 c2 è M1 ø
So, =1
V22 æ cHe ö rH 1 1
106. As, ç ÷= = =
V1 è cH ø r He 4 2
Þ =1
V2 T
( cHe) t = ( cHe) 0
3 kT 3 RT T0
100. As, m = 2 = 2
c Nc ( cHe) t ( cHe) 0 T 5
3 ´ 8.31 ´ 300 \ = =
= = 3.3 ´ 10 –27 kg ( cH) 0 ( cH) 0 T0 7
6.023 ´ 10 23 ´ (1930) 2
or T » 2 T0 = 2 ´ 273 = 546° K = 273° C
Mass of H2 molecule = 1.66 ´ 10 –27 ´ 2
c2 T
= 3.32 ´ 10 –27 kg 107. As, = 2
c1 T1
Thus, the gas is hydrogen.
T2 2T
3 kT 3 ´ 1.38 ´ 10 –23 ´ 273 or c1 =c = c 2 = 484 2 = 684 ms–1
101. As, c = = T1 T
m 5 ´ 10 -17
E 2 c22 ( c 2) 2
= 15 ´ 10 -3 ms–1 = 1.5 cms–1 \ = = =2
E1 c12 c2
3 RT c MO 16
102. As c = ; so H = = =4 or E 2 = 2 E1 = 2 ´ 6.21 ´ 10 –21 J
M cO MH 1
= 12.42 ´ 10 –21 J
103. Absolute pressure p1 = (15 + 1) atm
108. Average energy E µ T
[ Q Absolute pressure = Gauge pressure + 1 atm] E 800 800
= 16 ´ 1013
. ´ 10 5 Pa \ = =2
E 400 400
V1 = 30 L = 30 ´ 10 -3 m3 or E 800 = 2 ´ E 400 = 2 ´ 7.21 ´ 10 –21
T1 = 27315
. + 27 = 300.15 K = 14.42 ´ 10 –21 J
Using ideal gas equation
RMS velocity, cµ T
pV = nRT
800
pV \ c800 = c400 = 524 ´ 2 » 741 ms–1
or n= 400
RT
p1V1 16 ´ 1013
. ´ 10 5 ´ 30 ´ 10 -3 109. As, KE µ T
= =
RT1 8.314 ´ 300.15 So, E127 = E 27 ´ (27 + 273) / (27 + 273)
= 19.48 = 6.21 ´ 10 –21 ´ 400 / 300
Final p2 = (11 + 1) = 12 atm = 12 ´ 1013
. ´ 10 5 Pa = 8.28 ´ 10 –21 J

V2 = 30 L = 30 ´ 10 -3m3 110. According to law of equipartition of energy, average kinetic


energy per molecule per degree of freedom at temperature T
T2 = 27315
. + 17 = 290.15 K 1
is kT. The average kinetic energy per molecule of
p2V2 12 ´ 1013
. ´ 10 5 ´ 30 ´ 10 -3 2
Number of moles = =
RT2 8.314 ´ 300.15 n
polyatomic gas molecule = kT (n = number of mole)
= 1512
. 2
. = 4.36
Hence, moles removed = 19.48 - 1512 The average kinetic energy per mol of polyatomic gas
n n
Mass removed = 4.36 ´ 32 g = 0.1396 kg E = kT ´ N = RT
2 2
104. RMS velocity does not change with pressure, till temperature d æn ö n
remains constant. CV = ç RT ÷ = R
dT è 2 ø 2
Telegram @unacademyplusdiscounts

554 JEE Main Physics

2 2 2 116. The thermal radiation from a hot body travels with a velocity
111. As, g = 1+ or = g - 1 or n =
n n g -1 of light in vaccum, i. e. ,3 ´ 10 8 ms–1.
112. Here, n = 6, 117. Power radiated by sun at t° C = s (t + 273) 4 4pr 2
æ nö æ 6ö
C p = ç1 + ÷ R = ç1 + ÷ R = 4 R s (t + 273) 4 4 pr 2
è 2ø è 2ø Power received by a unit surface =
4 pR 2
113. At NTP, T = 273 K, p = 1.01´105 Nm–2
r 2s (t + 273) 4
Here, d = 2.4 ´ 10 –10 m =
R2
kT
l= 118. lm1T1 = lm2T2
2 pd 2p
(1.38 ´ 10 –23) ´ 273 Þ 5.5 ´ 10 –7 ´ 5500 = 11 ´ 10 -7 T
=
1.414 ´ 3.14 ´ (2.4 ´ 10 –10) 2 ´ 1.01 ´ 105 T = 550 ´ 5 K = 2750 K
= 1.46 ´ 10 –7
m 365 - 361 é 365 + 361 ù
119. As, =Kê - 293ú
2 ë 2 û
2 2 4 2
114. g = 1+ or 1.33+ or = 1+ (from Newton’s law of cooling)
n n 3 n
1
On solving n = 6. It is triatomic gas molecule having triangular Þ K=
35
structure, i. e. , O3 .
344 - 342 1 é 344 + 342 ù 10
115. The black spot on heating absorbs radiation and emits it then Again, = êë - 293ú =
t 35 2 û 7
in the dark room while the polished shining part reflects
14 14
radiation and absorbs nothing and so does not emit radiations t= min = ´ 60 = 84 s
and becomes invisible in the dark. 10 10

Round II
1 ö
1. As, l = l0 æç1+ ÷
On Fahrenheit scale,
è 100 ø K - 27315. F - 32
=
æ 1 ö
2 5 9
\ 2 l 2 = 2 l02 ç1 + ÷ 9
è 100 ø or F = (K1 - 27315
. )´ + 32
5
2
or 2 l 2 - 2 l02 = 2 l02 ´ 9
100 Triple point of neon, F1 = (K1 - 27315
. ) ´ + 32
5
2
Þ DS = S ´ 9
100 = (24.57 - 27315
. ) ´ + 32
5
DS 2 9
or = = 2% = - 248.58 ´ + 32 = - 415.44°F
S 100 5
2. Here, m = 0.1kg,h1 = 10 m,h2 = 5.4 m 9
Triple point of CO2, F2 = (K2 - 27315. ) ´ + 32
5
c = 460 J kg -1°C–1, g = 10 ms–2, q = ?
9
= (216.55 - 27315
. ) ´ + 32
Energy dissipated, Q = mg (h1 - h2) 5
= 0.1 ´ 10 (10 – 5.4) = 4.6 J 9
= - 56.6 ´ + 32 = - 69.88°F
From Q = cmq 5
Q 4.6 4. As r = r0 (1- g DT)
q= = = 0.1°C
cm 460 ´ 0.1
\ 9.7 = 10 (1 - g ´ 100)
3. Triple point of neon, (T1) = 24.57 K 9.7
= 1 - g ´ 100
Triple point of CO2, (T2) = 216.55 K 10
°C = K - 27315
. 9.7 0.3
On celsius scale, g ´ 100 = 1 - = = 3 ´ 10 -2
Triple point of neon, 10 10
t1° C = 24.57 - 27315
. = - 248.58°C g = 3 ´ 10 -4
Triple point of CO2, t 2° C = 216.55 - 27315
. 1
\ a = g = 10 -4° C–1
= - 56.60°C 3
Telegram @unacademyplusdiscounts

Heat and Kinetic Theory of Gases 555

5. Rate of cooling is proportional to (T 4 - T04), as per Stefan’s law. 11. Here, m = 10g = 10 –2 kg
R ¢ (900) 4 - (300) 4 v = 300 ms–1, q = ?, C = 150 J - kg –1 K –1
\ =
R (600) 4 - (300) 4 50 æ 1 2ö 1 -2 2
Q= ç mv ÷ = ´ 10 (300) = 225 J
4
9 -3 4 4 4
3 (3 - 1) 80 16 100 è 2 ø 4
= = = =
6 4 - 3 4 3 4 (2 4 - 1) 15 3 From Q = cm q
16 Q 225
R¢ = R q= = = 150° C
3 cm 150 ´ 10 -2
4 4 4 K 1 r 1
E 2 æ T2 ö æ 273 + 84 ö æ 357 ö 12. Here, 1 = , 1 =
6. As, =ç ÷ =ç ÷ =ç ÷ = 2.0 K2 2 r2 2
E1 è T1 ø è 273 + 27 ø è 300 ø
A1 1
DQ æ DT ö \ =
7. From, = KA ç ÷ A2 4
Dt è Dx ø
dx1 1 dQ 2 dQ1
DQ Dx = , = 4 cals–1, =?
Dt = dx2 2 dt dt
KA ( DT)
dQ 2 / dt K2 A2 dT / dx2 K2 A2 dx1
In arrangement (b), A is doubled and Dx is halved. = =
dQ1 / dt K1 A1 dT / dx1 K1 A1 dx2
1/ 2 1
\ Dt ® ® time 1
2 4 =2´4´ =4
1 2
i. e. , ´ 4 min = 1min dQ1 dQ 2 / dt 4
4 = = = 1 cals–1
dt 4 4
8. 22 g of CO2 is half mole of CO2, i. e. ,n1 = 0.5
13. Given, diameter of the hole (d1) = 4.24 cm
16 g of O2 is half mole of O2, i. e. ,n2 = 0.5
nT +n T Initial temperature T1 = 27 + 273 = 300 K
\ T= 11 22
n1 + n2 Final temperature T2 = 227 + 273 = 500 K
0.5 ´ (27+273)+0.5 (37+273) . ´ 10 -5 /°C
Coefficient of linear expansion ( a) = 170
=
0.5+0.5 Coefficient of superficial expansion (b) = 2a
= 305 K = 3.40 ´ 10 -5 /°C
= 305 - 273 = 32°C pd12
Initial area of hole at 27°C ( A1) = pr 2 =
D1 1 4
9. Here, = p
D2 2 = ( 4.24) 2 = 4.494p cm 2
4
A1 D12 1
\ = = Area of hole at 227°C ( A2) = A1(1 + b × Dt )
A2 D22 4
dx1 2 = 4.494 p [1 + 3.40 ´ 10 -5 ´ (227 - 27)]
Þ =
dx2 1
dQ1 æ dT ö dQ 2 1 = 4.494p [1 + 3.40 ´ 10 -5 ´ 200 ]
ç = KA1 ÷: ç = KA2 ÷=
dt è dx1 ø dt è dx2 ø dQ 2 /dt = 4.495p ´ 10068
.
A dx 1 1 1 = 4.525p cm 2
= 1× 2= ´ =
dx1 A2 4 2 8
pd 22
If diameter of hole becomes d 2 at 227°C, then A2 =
10. 10 g of ice at –10°C to ice at 0°C 4
Q1 = cm Dq = 0.5 ´ 100 = 50 cal pd 22
4.525p =
10 g of ice 0°C to water at 0°C 4
Q 2 = mL = 10 ´ 80 = 800 cal or d 22 = 4.525 ´ 4
10 g of water at 0°C to water at 100°C or d 2 = 4.2544 cm
Q3 = cm Dq = 1 ´ 10 ´ 100 = 1000 cal \Change in diameter ( Dd) = d 2 - d1
10 g water at 100°C to steam at 100°C = 4.2544 - 4.24
Q 4 = mL = 10 ´ 540 = 5400 cal = 0.0144 cm
Total heat required, Q + Q1 + Q 2 + Q3 + Q 4 . ´ 10 -2 cm
= 144
= 50+800+1000+5400 = 7250 cal
Telegram @unacademyplusdiscounts

556 JEE Main Physics

14. According to Newton’s law g a = g r - g = (180° - 4.0) 10 -6


q1 - q2 æ q + q2 ö Vt = V0 (1 + 1.40 ´ 10 –6 ´ 10 2)
=K ç 1 - q0 ÷
t è 2 ø
= (10 3 + 1.4) cc
60 - 50 æ 60 + 50 ö
\ =K ç - 25÷ …(i) \Volume of mercury that will overflow
10 è 2 ø
= Vt - V0 = 14 cc
Let q be the temperature after another 10 min
50 - q æ q + 50 ö
20. Using g r = g a + g , we get
\ =K ç - 25÷ …(ii)
10 è 2 ø g r = g1 + 3 a = g 2 + 3 b
g -g
Dividing Eq. (i) by Eq. (ii), we get \ b= 1 2+a
10 30 ´ 2 3
=
50 - q q DQ æ DT ö 2 æ DT ö r2
21. As, = KA ç ÷ = K pr ç ÷µ
\ q = 42.85° C Dt è Dx ø è l ø l

15. Let the temperature of junction be Q. In equilibrium, rate of r2


As is maximum for (d), it is correct choice.
flow of heat through rod 1 = sum of rate of flow of heat l
through rods 2 and 3. 22. Thermal capacity = Mass ´ specific heat
æ dQ ö æ dQ ö æ dQ ö Due to same material both spheres will have same specific
ç ÷ =ç ÷+ç ÷
è dt ø 1 è dt ø è dt ø 3
heat.
( q - 0) KA (90° - q) KA (90° - q) Also mass = Volume (V ) ´ density (r)
KA = +
l l l \Ratio of thermal capacity
Þ q = 2 (90° - q) 4 3
pr 3 3
or 3 q = 180° m1 V1 r 3 1 æ r1 ö æ 1ö 1
= = = =ç ÷ =ç ÷ =
180° m2 V2 r 3 pr3 è r2 ø è2ø 8
Þ q= = 60° 2
3 4

16. Two strips of equal lengths but of different materials (different 23. Given, mass of the child (m) = 30 kg
coefficient of linear expansion) when joined together, is Time taken (t ) = 20 min
called bimetallic strip, and can be used in thermostat to break
Fall in temperature = (101 - 98)°F
or make electrical contact. This strip has the characteristic
5 5
property of bending on heating due to unequal linear DT = 3°F = 3 ´ ° C = °C
expansion of the two metals. 9 3
The strip will bend with metal of greater a on outer side i.e., Specific heat of human body ( s) = 4.2 ´ 10 3 J/kg-°C
convex side. Latent heat of evaporation (L) = 580 cal/g = 580 ´ 10 3 cal/kg
T = (580 ´ 10 3 ´ 4.2) J/kg
17. As, h = 1- 2
T1
Heat given by body during fall in temperature
50 500
\ =1= or T1 = 1000 K Q1 = msDT
100 T1
Let m¢ be the mass of sweat evaporates from the human
60 T body.
Again, = 1- 2
100 1000 Heat taken in evaporation
or T2 = 400 K Q 2 = m¢ L
18. As, dQ = C pm DT But Q1 = Q 2
\ 70 = C p ´ 2 (35 - 30) \ msDT = m¢ L
CV = C p - R msDT 30 ´ 4.2 ´ 10 3 ´ 5 / 3
or m¢ = =
= 7 - 1.99 = 5.01 cal mol–1°C–1 L 580 ´ 4.2 ´ 10 3
\ dQ ¢ = CVm DT 10
== 0.0862 kg
= 5.01 ´ 2 ´ (35 – 30) = 50.1 cal 116
0.0862
19. Here, V0 = 103 cc \ Rate of evaporation of sweat =
20
g r = 180 ´ 10 -6°C–1 = 0.00431 kg/min
g = 40 ´ 10 -6°C–1,t = 100°C = 4.31 g/min
Telegram @unacademyplusdiscounts

Heat and Kinetic Theory of Gases 557

24. Here, Dl = 80.3 – 80.0 = 0.3 cm 4


or T= T0
3
l = 80 cm, a = 1.2 ´ 10 –6 °C–1 4
æ 4ö
Dl Power radiated, P = CT 4 = CT04 ç ÷
Rise in temperature DT = è3ø
la
256
0.3 P = P0 ´
DT = = 312.5°C 81
80 ´ 1.2 ´ 10 -5 P 256
DT ö or =
25. As, DQ = KA æç ÷ Dt , where, A = 4 pr
2 P0 81
è Dx ø
22 æ 32 ö
30. Let V be the volume of solid; d be its density and m be its mass;
= 0.008 ´ 4 ´ (6 ´ 10 8) 2 ´ ç 5 ÷ ´ 86400 if g coefficient of volume expansion of liquid, then
7 è10 ø d0
Density at temperature t1 is d1 =
= 10 18 cal 1 + g t1
26. Apprent weight (wa ) = Actual weight (w) - upthrust (F) d0
Density at temperature t 2 is, d 2 =
1+ g t 2
where, upthrust = weight of water displaced = V rw g
F50 V50 r50 g 1 + g m ´ 50 According to Archimedes’s principle,
Now, = = f1Vd1 = m = f2Vd 2
F0 V0 r 0 g 1 + gw ´ 50
d1 f2 d 0 (1 + g t 2)
As g m < gw , therefore, F50 < F0 . or = =
d 2 f1 1 + g t1) d 0
Hence, w2 > w1
or f1 + f1g t 2 = f2 + f2g t1
27. As the steel tape is calibrated at 10°C, therefore, adjacent f1 - f2 = g ( f2 t1 - f1 t 2)
centimetre marks on the steel tape will be separated by a (f - f )
distance of g= 1 2
f2 t1 - f1 t 2
lt = l10 (1 + a s DT) = (1 + a s 20) cm
Length of copper rod at 30°C 31. As, p = p1 + p2 + p3
= 90 (1 + a c 20) cm æ n RT ö æ n RT ö æ n RT ö
=ç ÷ +ç ÷ +ç ÷
è V øO è V ø N è V ø CO
Therefore, number of centimetres read on the tape will be 2 2 2

90 (1 + a c 20)) RT
= = (nO2 + nN2 + nCO2 )
1(1 + a s 20) V
(0.25+0.5+0.5) (8.31) ´ 300
90 (1 + 1.7 ´ 10 –5 ´ 20) =
= 4 ´ 10 -3
1 (1+1.2 ´ 10 –5 ´ 20)
= 7.79 ´ 10 5 Nm–2
90 ´ 1.00034
= = 90.01 cm 22 1
1.00024 32. For carbon dioxide, number of mole (n1) = = ;
44 2
DQ ö æ DQ ö æ DQ ö
28. As, æç ÷ +ç ÷ =ç ÷ molar specific heat of CO2 at constant volume CV1 = 3 R
è Dt ø inner è Dt ø outer è Dt ø total
16 1
K1pr 2(T2 - T1) K2p [(2 r) 2 - r 2] (T2 - T1) For oxygen, number of moles (n2) = = ;
+ 32 2
l l 5R
molar specific heat of O2 at constant volume CV2 = .
K p (2 r) 2 (T2 - T1) 2
=
l Let T K be the temperature of mixture.
pr 2 (T2 - T1) K p 4 r 2(T2 - T1) Heat lost by O2 = Heat gained by CO2.
or (K1 + 3 K2) =
l l n2CV2 DT2 = n1CV1 DT1
K1 + 3 K2 1 æ5 ö 1
or K= ç R ÷ (310 - T) = ´ (3 R) (T - 300)
4 2 2è ø 2
29. Let, T0 be the initial temperature of the black body or 1550 - 5 T = 6 T - 1800
\ l 0T0 = b (Wien’s law) or T = 304.54 K = 31.5° C
Power radiated, P0 = CT04 , where C is constant. 33. According to Boyle’s law, pV = k (a constant)
If T is new temperature of black body, then m pm
or p = k or r =
3 l0 r k
T = b = l 0T0
4 p æ k ö
or r= ç where, = k = constant ÷
k è m ø
Telegram @unacademyplusdiscounts

558 JEE Main Physics

p1 38. Molar specific heat of the mixture at constant volume is


So, r1 =
k æ3 ö æ5 ö
p1 m1 m km 2 ç R÷ + 3 ç R÷
and V1 = = 1 = 1 n1CV1 + n2CV2 è2 ø è2 ø
k r1 p1/ k p1 CV = = = 2.1 R
(n1 + n2) 2+3
km2
Similarly, V2 = 39. g real = g app. + g vessel
p2
So ( g app. + g versel) glass = ( g app. + g versel) steel
æm m ö
Total volume = V1 + V2 = k ç 1 + 2 ÷ Þ 153 ´ 10 -6 + ( g versel) glass = (144 ´ 10 -6 + g vessel) steel
è p1 p1 ø
Let p be the common pressure and r be the common density Further, ( g vessel) steel = 3a = 3 ´ (12 ´ 10 -6)
of mixture. Then = 36 ´ 10 -6 /°C
m + m2 m1 + m2
r= 1 = Þ 153 ´ 10 -6 + ( g vessel) glass = 144 ´ 10 - -6 + 36 ´ 10 -6
V1 + V2 æ m1 m2 ö
kç + ÷
è p1 p2 ø Þ ( g vessel) glass = 3a = 27 ´ 10 -6 /°C
m1 + m2 p1p2 (m1 + m2) Þ a = 9 ´ 10 -6 /°C
\ p = kr = =
m1 m2 (m1p2 + m2p1)
+ 40. The expansion of solids can be well understood by potential
p1 p2
energy curve for two adjacent atoms in crystalline solid as a
3 RT function of their intermolecular separation (r).
34. As, crms =
M
U
3 RT 3 ´ 8.31 ´ 300
or M= 2
=
crms (1920) 2
= 2 ´ 10 -3 kg = 2 g r
P3
Since, M = 2 for the hydrogen molecule. Hence, the gas is E P2 FT
3
hydrogen. C P1 D T2
n RT + n2RT + n3RT RT A BT
35. As, p = 1 = (n1 + n2 + n3) r0 1
V V r1
r2
æ 8 14 22 ö 0.082 ´ 300
=ç + + ÷´ = 3.69 atm
è16 28 44 ø 10 T3 > T2 > T1
36. When the piston is in equilibrium, the pressure is same on At ordinary temperature Each molecule of the solid
both the sides of the piston. It is given that temperature and vibrates about its equilibrium position P1 between A and B (r0
weight of gas on the two sides of piston not change. From is the equilibrium distance of it from some other molecules)
ideal gas equation, pV = n RT , we have V µ mass of the gas. At high temperature Amplitude of vibration increases
V1 m1 (C « D and E « F). Due to asymmetry in the curve, the
So, =
V2 m2 equilibrium positions (P2 and P3) of the molecule is displaced.
V1 m Hence its distance from other molecules increases
or +1= 1 +1 (r2 > r1 > r0). Thus, on raising the temperature, the average
V2 m2
equilibrium distance between the molecules increase and
V1 + V2 m1 + m2
or = the solid as a whole expands.
V2 m2
V2 m2 2m 2 41. Initial diameter of tyre = (1000 - 6) mm = 994 mm
or = = = 994
V1 + V2 m1 + m2 m + 2 m 3 Initial radius of tyre R = = 497 mm
2
37. For a closed system, the total number of moles remains and change in diameter DD = 6 mm
constant. So, 6
p1V = n1RT1 and p2V = n2RT2 DR = = 3 mm
2
\ p (2 V ) = (n1 + n2) RT
After increasing temperature by Dq tyre will fit on wheel
p (n1 + n2) 1 æp p ö
\ = R = ç 1 + 2÷ Increment in the length (circumference) of the iron tyre
T 2 2 è T1 T2 ø
g g
DL = L ´ a ´ Dq = L ´ ´ Dq [As a = ]
1 æ p1T2 + p2T1 ö 3 3
= ç ÷
2 è T1T2 ø ægö
2 pDR = 2 pR ç ÷ Dq
è3ø
Telegram @unacademyplusdiscounts

Heat and Kinetic Theory of Gases 559

3 DR 3 ´3 As p59 < p0 ,
Þ Dq = =
g R 3.6 ´ 10 -5 ´ 497 Þ w2 > w1
Þ Dq @ 500°C or w1 < w2
42. Due to volume expansion of both liquid and vessel change in 47. Given, coefficient of volume expansion ( g) = 49 ´ 10 -5/K
volume of liquid relative to container is given by Rise in temperature ( Dt ) = 30°C
DV = V0[ gL - gg ]Dq.
Let initial volume of glycerine be V0 .
Given V0 = 1000 cc, ag = 0.1 ´ 10 -4 /°C
\ Volume of glycerine when temperature is increased by
\ gg = 3ag = 3 ´ 0.1 ´ 10 -4 /°C = 0.3 ´ 10 -4 /°C 30°C
\ DV = 1000[1.82 ´ 10 -4 - 0.3 ´ 10 -4 ] ´ 100 = 152 V = V0[1 + gDt ]
= V0[1 + 49 ´ 10 -5 ´ 30 ]
43. With temperature rise (same 20°C for both), steel scale and = V0[1 + 0.01470 ] = 10147
. V0
copper wire both expand. Hence length of copper wire w.r.t. V0 1
steel scale or apparent length of copper after rise in \ = …(i)
V 10147
.
temperature
If mass of glycerine is m, then
Lapp = L'cu - L'steel
m
= [L0(1 + a cuDq) - L0(1 + a s Dq)] initial density of glycerine (r 0) =
V0
Þ Lapp = L0( a cu - a s ) Dq
m
= 80(17 ´ 10 -6 - 11 ´ 10 -6) ´ 20 and final density of glycerine (r) =
V
= 0.0096 cm r m / V V0 1
\ = = = …(ii)
\Length of the wire read = 80.0096 cm r 0 m / V0 V 10147
.
44. Moment of inertia of a rod, Dr r - r 0 r
Fractional change in density = = = -1
1 r0 r0 r0
l= ML2 ...(i)
12 Substituting value from Eq. (ii), we get
where M is the mass of the rod and L is the length of the rod 1
Fractional change in density = -1
1 10147
.
\ Dl = 2MLDL (\M is a constant) ...(ii)
12 . ´ 10 -2
= - 0.0145 = - 145
Divide Eq. (ii) by (i), we get
Negative sign shows that the density of glycerine decreases
Dl DL with rise in temperature.
=2 ...(iii)
l L
48. Loss of weight at 27°C is
As DL = LaDt
DL = 46 - 30 = 16 = V1 ´ 1. 24 r1 ´ g ...(i)
or = aDt Loss of weight at 42°C is
L
DL = 46 - 30.5 = 15.5 = V2 ´ 12. r1 ´ g ...(ii)
Substituting the value of in Eq. (iii), we get 16 V 1.24
L Now dividing Eq. (i) by (ii), we get = 1 ´
Dl 15.5 V2 1.2
= 2aDt
l V2
But = 1 + 3a(t 2 - t1)
45. Thermostat is used in electric apparatus like refrigerator, V1
iron, etc., for automatic cut-off. Therefore, for metallic strips 15.5 ´ 1.24
to bend on heating their coefficient of linear expansion = = 1.001042
16 ´1.2
should be different.
Þ 3a( 42° - 27° ) = 0.001042
46. As the coefficient of cubical expansion of metal is less as
compared to the coefficient of cubical expansion of liquid, we Þ a = 2.316 ´ 10 -5 /°C
may neglect the expansion of metal ball. So when the ball is
49. Substances are classified into two categories
immersed in alchohol at 0°C, it displaces some volume V of
(i) Water like substances which expand on solidification
alchohol at 0°C and has weight w1.
\ w1 = w0 - v p 0 g (ii) CO2 like (Wax, Ghee etc. ) substances which contract on
solidification.
where, w0 = weight of ball in air
Their behaviour regarding solidification is opposite.
Similarly, w2 = w0 - v p0 g Melting point of with rise of pressure but that of wax etc
where, p 0 = density of alchohol at 0°C increases with increase in pressure. Similarly ice starts
and p59 = density of alchohol at 59°C forming from top to downwards whereas wax starts its
formation from bottom to upwards.
Telegram @unacademyplusdiscounts

560 JEE Main Physics

mL Now the bullet will melt if Q 2 ³ Q1


50. Heat lost in t sec = mL or heat lost per sec =. This must be
t 3
i. e. , mv 2 ´ 10 -3 ³ 15 m ´ 4.2
the heat supplied for keeping the substance in molten state 8
per sec.
Þ v min = 410 m/s
mL Pt
\ = P or L =
t m 55. We can relate an absorbed energy Q and the resulting
temperature increase DT with relation Q = cmDT. In that
51. Heat is lost by steam in two stages
equation, m is the mass of the material absorbing the energy
(i) for change of state from steam at 100°C to water at 100°C and c is the specific heat of that material. An absorbed dose of
is m ´ 540 3 Gy corresponds to an absorbed energy per unit mass of
(ii) to change water at 100°C to water at 80°C is 3 J/kg. Let us assume that c the specific heat of human body, is
m ´ 1 ´ (100 - 80), where m is the mass of steam the same as that of water, 4180 J/kg K. Then we find that
condensed. Q/m 3
Total heat lost by steam is m ´ 540 + m ´ 20 = 560 m(cals.) DT = = = 7.2 ´ 10 -4K » 700 mK
c 4180
Heat gained by calorimeter and its contents is
Obviously the damage done by ionizing radiation has nothing
= (1.1 + 0.02) ´ (80 - 15)
to do with thermal heating. The harmful effects arise because
= 1.12 ´ 65 cal the radiation damages DNA and thus interferes with the
Using principle of calorimetery, Heat gained = heat lost normal functioning of tissues in which it is absorbed.
\ 560m = 112. ´ 65
56. Heat gain = Heat lost
m = 0.130 g
C A (16 - 12) = CB(19 - 16)
52. Initially ice will absorb heat to raise its temperature to 0°C CA 3
then its melting takes place Þ =
CB 4
If mi = Initial mass of ice,
and CB(23 - 19) = CC (28 - 23)
mi ¢ = Mass of ice that melts
CB 5
and mw = Initial mass of water Þ =
CC 4
By law of mixture,
C A 15
Heat gained by ice = Heat lost by water Þ = …(i)
CC 16
Þ mi ´ c ´ (20) + mi ¢ ´ L = mw cw (20)
Þ 2 ´ 0.5(20) + m1¢ ´ 80 = 5 ´ 1 ´ 20 If q is the temperature when A and C are mixed, then
Þ m1¢ = 1 kg C A ( q - 12) = CC (28 - q)
So final mass of water C A 28 - q
Þ = …(ii)
= Initial mass of water + Mass of ice that melts CC q - 12
= 5 + 1 = 6 kg On solving Eqs. (i) and (ii), q = 20.2°C
53. Heat gained by the water = (Heat supplied by the coil) 57. Suppose m kg steam is required per hour
- (Heat dissipated to environment) Heat is released by steam in following three steps
Þ mcDq = PCoil t - PLosst (i) When 150°C steam ¾® 100°C steam
Þ 2 ´ 4.2 ´ 10 3 ´ (77 - 27) = 1000t - 160t Q1

5 Q1 = mcsteam Dq
4.2 ´ 10
Þ t= = 500 s = 8 min 20 s = m ´ 1150
( - 100) = 50 m cal
840
(ii) When 100°C steam ¾® 100°C water
Q2
54. If mass of the bullet is m g
then total heat required for bullet to just melt down Q 2 = mLv = m ´ 540 = 540 m cal
Q1 = mcDq + mL = m ´ 0.03(327 - 27) + m ´ 6 (iii) When 100°C water ¾® 90°C water
Q2
= 15 m cal = (15m ´ 4.2)J Q3 = mcw Dq = m ´ 1 ´ (100 - 90) = 10 m cal
Now when bullet is stopped by the obstacle, the loss in its Hence total heat given by the steam
1 Q = Q1 + Q 2 + Q3 = 600 mcal
mechanical energy = (m ´ 10 -3)v 2J
2 Heat taken by 10 kg water
(As mg = m ´ 10 -3 kg) Q ¢ = mcw Dq = 10 ´ 10 3 ´ 1 ´ (80 - 20) = 600 ´ 10 3 cal
As 25% of this energy is absorbed by the obstacle, Hence Q = Q'
The energy absorbed by the bullet Þ 600m = 600 ´ 10 3
75 1 3 Þ m = 10 3 g = 1 kg
Q2 = ´ mv 2 ´ 10 -3 = mv 2 ´ 10 -3 J
100 2 8
Telegram @unacademyplusdiscounts

Heat and Kinetic Theory of Gases 561

58. Suppose, height of liquid in each arm before rising the 63. Given, mass of copper block (m) = 2.5 kg
temperature is l.
Change in temperature ( Dt ) = 500°C
t1 Specific heat ( s) = 0.39 J/g-K
= 390 J/kg-K
t2
l l l1 Latent heat of fusion of water (L) = 335 J/g
l2
= 335 ´ 10 3 J/kg
Heat energy absorbed by copper block
With temperature rise height of liquid in each arm increases
Q1 = msDt
i. e., l1 > l and l2 > l
l1 l2 = 2.5 ´ 390 ´ 500 J
Also, l= = Let m¢ kg of ice be melted.
1 + gt1 1 + gt 2
Þ l1 + gl1t 2 = l2 + gl2t1 Heat energy required to melt ice, Q 2 = m¢ L
l -l But heat energy absorbed by copper block
Þ g= 1 2 .
l2t1 - l1t 2 = Heat energy utilized by ice in melting
59. V = V0(1 + gDq) Q1 = Q 2
3 msDt = m¢ L
L = L0(1 + a1Dq)L20(1 + a 2Dq) 2
msDt
= L30(1 + a1Dq)(1 + a 2Dq) 2 or m¢ =
L
Since, L30 = V0 and L3 = V 2.5 ´ 390 ´ 500
=
Hence, 1 + gDq = (1 + a1Dq)(1 + a 2Dq) 2 335 ´ 10 3
@ (1 + a1Dq)(1 + a 2Dq) = 1455
. kg
@ (1 + a1Dq + 2a 2Dq) » 15
. kg
Þ g = a1 + 2a 2 64. Let m g of steam get condensed into water (By heat
2 loss). This happens in following two steps
60. (OR) 2 = (PR) 2 - (PO) 2 = l 2 - æç ö÷
l
è2ø
2 100ºC 100ºC
él ù Steam (H1 = m × 540) Water
= [ l (1 + a 2t )]2 - ê (1 + a1t ú
ë2 û
l2 l2 [(H2 = m × 1 × (100 – 90)]
l - = l (1 + a 2 t + 2a 2 t ) - (1 + a12t 2 + 2a1t )
2 2 2 2
4 4
Neglecting a 22 t 2 and a12t 2 90ºC
2 Water
l
0 = l 2(2a 2t ) - (2a 2t )
4
2a1 Heat gained by water (20°C) to raise its temperature upto
Þ 2a 2 = Þ a1 = 4 a 2
4 90° = 22 ´ 1 ´ (90 - 20)
Hence, in equilibrium;
´ =PDq
61. t mc heat lost = heat gain
mcDq 4200mDq 4200 ´ m ´ Dq Þ m ´ 540 + m ´ 1 ´ (100 - 90) = 22 ´ 1 ´ (90 - 20)
Þ t= = =
P P VI Þ m = 2.8 g
{QC water = 4200 J / kg ´°C } The net mass of the water present in the mixture
4200 ´ 1 ´ (100 - 20) = 22 + 2.8 = 24.8 g
Þ t= = 381 s » 6.3 min
220 ´ 4 Pl 2
65. t = ( x2 - x12)
62. Heat given by water Q1 = 10 ´ 10 = 100 cal 2Kq
Þ t µ ( x22 - x12)
Heat taken by ice to melt
t ( x2 - x2)
Q 2 = 10 ´ 0.5 ´ [0 - ( -20)] + 10 ´ 80 = 900 cal Þ = 22 12
t ' ( x'2 - x'1 )
As Q1 < Q 2, so ice will not completely melt and final
temperature = 0°C 7 (12 - 0 2)
Þ =
t' (2 2 - 12)
As heat given by water in cooling up to 0°C is only just
sufficient to increase the temperature of ice from -20° C to Þ t ¢ = 21h
0°C, hence mixture in equilibrium will consist of 10 g ice and
10 g water at 0°C.
Telegram @unacademyplusdiscounts

562 JEE Main Physics

66. The pressure on the rear side would be more due to fictitious Rate of flow of heat in path BCA will be same
force (acting in the opposite direction of acceleration) on the æQ ö æQ ö
i. e. ç ÷ =ç ÷
rear face. Consequently, the pressure in the front side would è t ø BC è t ø CA
be lowered.
K( 2T - Tc ) A K(Tc - T) A
V 2 t (200) 2 ´ t Þ =
67. Heat developed by the heater H = . = a 2a
R J 20 ´ 4.2 Tc 3
0.2 ´ 1 ´ (20 - q)t Þ =
Heat conducted by the glass H = T 1+ 2
0.002
(200) 2 ´ t 0.2 ´ (20 - q) t KADqt
Hence, = 71. mL =
20 ´ 4.2 0.002 Dx
0.0075 ´ 75 ´ ( 40 - 0)t
Þ q = 15.24°C Þ 500 ´ 80 =
5
rL 2
68. Since, t = ( x2 - x12) Þ t = 8.9 ´ 10 3 s = 2.47 h
2kq
rL 2 rL( x + y)( x - y) 72. Initially the rods are placed in vessels as shown below
\ t= ( x - y 2) =
2kq 2kq
RP = R/ 2
69. If suppose KNi = K R

Þ KAl = 3K and KCu = 6K 100ºC R 0ºC


Since all metal bars are connected in series. l
æQ ö æQ ö æQ ö æQ ö 100ºC 0ºC
So, ç ÷ =ç ÷ =ç ÷ =ç ÷
è t ø Combination è t ø Cu è t ø Al è t ø Ni
Q ( q1 - q2)
3 1 1 1 =
and = + + t R
Keq KCu KAl KNi Q
æ ö mL (100 - 0)
Þ ç ÷ = = q1L = ...(i)
1 1 1 9 è t ø1 t R /2
= + + =
6K 3K K 6K Finally when rods are joined end to end as shown
Req=2R
Þ Keq = 2 K
æQ ö mL R R
25 cm 10 cm 15 cm Þ ç ÷ = = q2 L
è t ø2 t
Q Cu Ni Al Q 100ºC 0ºC
l l
100ºC θ1 θ2 0ºC (100 - 0)
= …(ii)
2R
æQ ö æQ ö q1 4
Hence, if ç ÷ =ç ÷ From Eqs. (i) and (ii), =
è t ø Combination è t ø Cu
q2 1
Keq A (100 - 0) KCu A(100 - q1)
Þ = 73. Rate of cooling of a body
lCombination lCu
2KA(100 - 0) 6KA(100 - q1) Dq Aes(T 4 - T04) A Area
Þ = R= = Þ Rµ µ
(25 + 10 + 15) 25 t mc m Volume
1
Þ q1 = 83.33°C For the same surface area R µ
Volume
æQ ö æQ ö
Similar if ç ÷ =ç ÷ Q Volume of cube < Volume of sphere
è t ø Combination è t ø Al
2KA(100 - 0) 3 KA( q2 - 0) Þ RCube > RSphere i. e., cube, cools down with faster rate.
Þ =
50 15 74. Wien's displacement law is lmT = b
Þ q2 = 20 °C b 2.88 ´ 10 6
Þ lm = = = 1000 nm
70. QTB > TA Þ Heat will flow B to A via two paths (i) B to A (ii) and t 2880
along BCA as shown. Energy distribution with wavelength will be as follows

(T )A

a√2 U2
a U1
U3
499
500
999
1000
1499
1500

λ (nm)
√2T B a C(TC)
From the graph it is clear that U2 > U1.
Telegram @unacademyplusdiscounts

Heat and Kinetic Theory of Gases 563

75. Energy received per second i. e., power P µ (T 4 - T04) Þ Q 2 = 16 Q1

Þ P µT4 (Q T0 << T) if m be mass of water taken and S be its specific heat capacity,
then Q1 = ms(20.5 - 20) and Q 2 = ms( q - 20)
1
Also energy received per sec (P) µ (inverse square law) q°C = Final temperature of water
d2
Q 2 q - 20
T4 Þ =
Þ Pµ Q1 0.5
d2
16 q - 20
4 2 Þ =
P1 æT ö æd ö 1 0.5
Þ = çç 1 ÷÷ ´ ç 2 ÷
P2 è T2 ø è d1 ø Þ q = 28°C

P æT ö
4 2 2 Dq Aes (T 4 - T04)
æ 2d ö æ 2d ö 1 80. Rate of cooling =
Þ =ç ÷ ´ç ÷ ´ç ÷ = t mc
P 2 è 2T ø èd ø èd ø 4
As surface area, material and temperature difference are
Þ P2 = 4 P same, so rate of loss of heat is same in both spheres. Now in
76. The given arrangement of rods can be redrawn as follows this case rate of cooling depends on mass.
2K1K2 Dq 1
K= Þ Rate of cooling µ
K1+K2 T m
K1 K2
Qmsolid > mhollow . Hence hollow sphere will cool fast.
Dq Aes (T 4 - T04)
H1 H1 81. Rate of cooling =
l l t mc
m
H H2 H Þ tµ (Q Dq, t , s , (T 4 - T04) are constants)
K3 A
It is given that H1 = H2 m Volume a3
Þ tµ µ µ 2
KA( q1 - q2) K3 A( q1 - q2) A Area a
Þ =
2l l Þ t µa
K KK t1 a1
Þ K3 = = 1 2 Þ =
2 K1 + K2 t 2 a2
100 1
Dq Aes (T 4 - T04) Þ =
77. Rate of cooling (R) = = t2 2
t mc
Þ t 2 = 200 s
A area r2 1
Þ Rµ µ µ 3 µ 82. According to Newton's law of cooling
m volume r r
1 1 4 q1 - q2 é q + q2 ù
Þ
é ù
Rate (R) µ µ 1/3 ê\ m = r ´ pr3 Þ r µ m1/3 ú = Kê 1 - q0 ú
r m 3 t ë 2 û
ë û
1/3 1/3 1 5 min
R1 æ m2 ö æ 1ö 80ºC 64ºC
Þ =ç ÷ =ç ÷ 2 10 min
R2 è m1 ø è3ø 52ºC
3 15 min
θ=?
78. Radiated power P = AesT 4 Þ P µ AT 4
1 For first process,
From Wien's law, l mT = constant Þ T µ (80 - 64)
lm é 80 + 64 ù
= Kê - q0 ú ...(i)
2 5 ë 2 û
A r
\ Pµ µ For second process,
( l m) 4 ( l m ) 4
(80 - 52) é 80 + 52 ù
22 42 62 = Kê - q0 ú ...(ii)
Þ Q A : QB : QC = : : 10 ë 2 û
(300) ( 400) (500) 4
4 4
For third process,
\ QB will be maximum. (80 - q) é 80 + q ù
= Kê - q0 ú ...(iii)
79. The total energy radiated from a black body per minute. 15 ë 2 û
Q µT4 1
On solving Eqs. (i) and (ii), we get K = and q0 = 24° C
Q 2 æ 2T ö
4 15
Þ = ç ÷ = 16 Putting these values in Eq. (iii), we get
Q1 è T ø
q0 = 42.7°C
Telegram @unacademyplusdiscounts

564 JEE Main Physics

Ql mLl 88. For no current flow between C and D


83. t = =
KA( q1 - q2) KA( q1 - q2) æQ ö æQ ö
VrLl ç ÷ =ç ÷
= è t ø AC è t ø CB
KA( q1 - q2)
K1A( qA - qC ) K2A( qC - qB)
5 + 10 Þ =
5 ´ A ´ 0.92 ´ 80 ´ l l
= 2 = 191
.h qA - qC K2
0.004 ´ A ´ 10 ´ 3600 Þ = ...(i)
qC - qB K1
84. Suppose temperature difference between A and B is 100°C
æQ ö æQ ö
and qA > qB Also, ç ÷ =ç ÷
è t ø AD è t ø DB
C
K3 A( qA - qD ) K4 A( qD - qB)
H/2 H/2 Þ =
l l
A B
qA - qD K4
H H Þ = ...(ii)
qD - qB K3
H/2 H/2
It is given that qC = qD , hence from Eqs. (i) and (ii),
D K2 K4
we get, =
Heat current will flow from A to B via path ACB and ADB. K1 K3
Since, all the rods are identical, so ( Dq) AC = ( Dq) AD Þ K1K4 = K2 K3
Dq
[because heat current H = , here R = same for all] dq Aes(T 4 - T04)
R 89. Rate of cooling Rc = =
dt mc
Þ qA - qC = qA - qD dq A r 2 dq 1
Þ qC = qD Þ µ µ 3 Þ µ
dt V r dt r
i. e., temperature difference between C and D will be zero. dT sA 4
Q KADq 90. = (T - T04) [ In the given problem fall in temperature
85. = dt mcJ
t l of body dT = (200 - 100) = 100K , temperature of
mL K( pr 2) Dq surrounding T0 = 0K, Initial temperature of body T = 200 K]
Þ =
t l 100 s 4 pr 2
æ m ö Kr
2 = (200 4 - 0 4)
Rate of melting of ice ç ÷ µ dt 4 3
èt ø pr rcJ
l 3
1 rrcJ rrc 4.2
Since for second rod K becomes th r becomes double and Þ dt = ´ 10 -6 s = . ´ 10 -6
4 48s s 48
length becomes half, so rate of melting will be twice i. e
7 rrc 7 rrc
æ mö æ mö = ms » ms [As J = 4.2]
ç ÷ = 2 ç ÷ = 2 ´ 0.1 = 0.2 g/s 80 s 72 s
è t ø2 è t ø1
91. P(400°C)
86. Heat transferred in one minute is utilised in melting the
Ice A QA QB B Water
ice so,
KA( q1 - q2)t 0°C λx (10 – λ)x 100°C
=m´L
l
10 -3 ´ 92 ´ (100 - 0) ´ 60 Heat received by end A, for melting of ice
m= KA( 400 - 0)t
1 ´ 8 ´ 10 4 QA = = mLice ...(i)
lx
-3
= 6.9 ´ 10 kg Heat received by end B, for vaporisation of water
dQ KA 0.01 ´ 1 KA( 400 - 100)t
87. = dq = ´ 30 = 6 J/s QB = = mLvap ...(ii)
dt l 0.05 (10 - l) x
Heat transferred in one day (86400 s) 400
lx L
Q = 6 ´ 86400 = 518400 J Dividing both equations, = ice
300 Lvap
Now, Q = mL
(10 - l) x
Q 518400
Þ m= = 4 (10 - l) 80
L 334 ´ 10 3 Þ =
3 l 540
= 1552
. kg = 1552 g Þ l =9
Telegram @unacademyplusdiscounts

Heat and Kinetic Theory of Gases 565

92. Q = sAt(T 4 - T04) 98. Let the quantity of heat supplied per minute be Q. Then,
If T ,T0 , s and t are same for both bodies quantity of heat supplied in 2 min = mC(90 - 80) In 4 min,
heat supplied = 2mC(90 - 80)
Q sphere Asphere 4pr 2
= = L
Q cube Acube 6 a2 \ 2mC(90 - 80) = mL Þ = 20
C
But according to problem,
volume of sphere = volume of cube
99. In the given graph CD represents liquid state.
4 3 100. Density of water is maximum at 4°C and is less on either side
Þ pr = a3
3 of this temperature.
1/3
æ4 ö C F - 32 9
Þ a = ç p÷ r 101. We know that, = or F = C + 32
è3 ø 100 180 5
Substituting the value of a in Eq. (i), we get Y
Q sphere 4pr 2 4 pr 2
= = F
Q cube 6 a2 ìï æ 4 ö 1/3 üï
2

6í ç p ÷ rý
ïî è 3 ø ïþ X
O C
1/3
4 pr 2 æpö
= =ç ÷ :1 Equation of straight line is, y = mx + c
æ4 ö
2/3 è6ø
6ç p ÷ r2 Hence, m = (9 / 5), positive and c = 32 positive. The graph is
è3 ø
shown in figure.
93. Temperature difference between C and D is zero. C F - 32 æ5ö 20
102. = Þ C =ç ÷F -
C 5 9 è9ø 3
R R Hence, graph between °C and ° F will be a straight line with
positive slope and negative intercept.
A B T
√2T 103. The horizontal parts of the curve, where the system absorbs
R R heat at constant temperature must depict changes of state.
D
Here, the latent heats are proportional to lengths of the
horizontal parts. In the sloping parts, specific heat capacity is
94. Relation between Celsius and Fahrenheit scale of temperature inversely proportional to the slopes.
C F - 32 5 160 104. Root mean square speed of organ atom
is = ÞC = F -
5 9 9 9
3RTAr
Equating above equation with standard equation of line (v rms) Ar = …(i)
MAr
5
y = mx + c, we get slope of the line AB is m = Root mean square speed of helium atom
9
3RTHe
95. Since in the region AB temperature is constant, therefore at (v rms) He = …(ii)
MHe
this temperature phase of the material changes from solid to
liquid and (H2 - H1) heat will be absorb by the material. This Dividing Eq. (i) by Eq. (ii)
heat is known as the heat of melting of the solid. (v rms) Ar 3RTAr MHe
= ´
Similarly in the region CD temperature is constant, therefore at (v rms) He MAr 3RTHe
this temperature phase of the material changes from liquid to
gas and (H4 - H3) heat will be absorbed by the material. This Given (v rms) Ar = (v rms) He
heat as known as the heat is vaporisation of the liquid. æ T ö æM ö
Þ 1 = ç Ar ÷ × ç He ÷
96. Initially, on heating temperature rises from -10°C to 0°C. è THe ø è MAr ø
Then ice melts and temperature does not rise. After the whole MHe THe
ice has melted, temperature begins to rise until it reaches Þ =
MAr TAr
100°C. Then it becomes constant, as at the boiling point will
not rise. æM ö
\ TAr = THe ç Ar ÷
è MHe ø
97. The volume of matter in portion AB of the curve is almost
constant and pressure is decreasing. These are the æ 39.9 ö
= 253.15ç ÷
characteristics of liquid state. è 4 ø
= 2523.675 K= 2.52 ´ 10 3 K
Telegram @unacademyplusdiscounts

566 JEE Main Physics

106. Substances having more specific heat take longer time to get 1
115. According to Wien's law, l m µ Þ v m µ T. As the
heated to a higher temperature and longer time to get cooled. T
temperature of body increases, frequency corresponding to
T
A maximum energy in radiation (v m) increases this is shown in
graph ( c).
B
C 116. Given, base area of boiler ( A) = 0.15 m 2
. cm = 1 ´ 10 -2 m
Thickness (d) = 10
6.0
t Rate of water boils = 6.0 kg/min = kg /s = 0.1 kg /s
tA tB tC 60
Thermal conductivity of brass (K) = 109 J/s-m-K
If we draw a line parallel to the time axis, then it cuts the
Latent heat of vapourization of water (L) = 2256 ´ 10 3 J/kg
given graphs at three different points. Corresponding points
on the times axis shows that Let q1 be the temperature of the part of the boiler in contact
tC > tB > t A with the stove.
Þ CC > CB > C A Rate of heat energy supplied
107. From given curve, = Rate of heat energy utilized in vaporization
KADq
Melting point for A = 60°C = mL
d
and melting point for B = 20°C
KA( q1 - q2)
Time taken by A for fusion = (6 - 2) = 4 min = mL
d
Time taken by B for fusion = (6.5 - 4) = 2.5 min 109 ´ 0.15( q1 - 100)
HA 6 ´ 4 ´ 60 8 = 0.1 ´ 2256 ´ 10 3
Then, = = 1 ´ 10 -2
HB 6 ´ 2.5 ´ 60 5
1635 ( q1 - 100) = 2256 ´ 10 2
108. Anomalous density of water is given by (a). It has maximum 225600
density at 4°C. When ice is formed it floats. q1 = + 100
1635
111. Initially liquid oxygen will gain the temperature up to its = 137.98 + 100
boiling temperature, then it change its state to gas. After this = 237.98°C » 238°C
again its temperature will increase.
dq
dT ö 117. For q - t plot, rate of cooling = = slope of the curve.
112. Rate of cooling æç - ÷ µ emissivity ( e) dt
è dt ø dq
At P, = tan f2 = k ( q2 - q0),
æ dT ö æ dT ö dt
From graph, ç - ÷ > ç- ÷ Þ ex > ey
è dt ø x è dt ø y where, k = constant.
dq
Further emissivity ( e) µ Absorptive power ( a) Þ ax > ay At Q, = tan f1 = k ( q1 - q0)
dt
(Q good absorbers are good emitters).
tan f 2 q2 - q0
1 Þ =
113. According to Wien's law, l µ and from the figure tan f1 q1 - q0

( l m)1 < ( l m)3 < ( l m) 2, therefore T1 > T3 > T2 118.According to Wien's displacement law,
Ar 16 1
114. = [Given] lm µ
A2000 1 T
Þ l m2 < l m1 (QT1 < T2)
Area under el - l curve reperesents the emissive power of
body and emissive power µ T 4 Therefore I - l graph for T2 has lesser wavelength ( l m) and so
4
curve for T2 will shift towards left side.
(Hence, area under el - l curve) µ T
4 119. Area under given curve represents emissive power and
AT æ T ö emissive power µ T 4
Þ =ç ÷
A2000 è 2000 ø
Þ A µT4
4
16 æ T ö 4
Þ =ç ÷ A2 T24 (273 + 327) 4 æ 600 ö 16
1 è 2000 ø = = =ç ÷ =
A1 T14 (273 + 27) 4 è 300 ø 1
Þ T = 4000 K
Telegram @unacademyplusdiscounts

Heat and Kinetic Theory of Gases 567

120. According to Newton's law of cooling, Negative sign shows that temperature decreases i. e. , the body
θ cools, c is the specific heat of the material and q0 is the
θi surrounding temperature.
dq 1
Þ µ
dt c
θ0
æ dq ö
i. e., rate of cooling çR = ÷ is inversely proportional to
t è dt ø
Rateof cooling µTemperature differernce the specific heat of material. For A, rate of cooling is large.
dq Therefore, specific heat of A is smaller.
Þ - µ ( q - q0) 4
dt E1 A1 æ T1 ö 4 pr12 æ 1ö
2
1
dq 128. As, = ×ç ÷ = ´1 = ç ÷ =
Þ - = a( q - q0) (a = constant) E 2 A2 è T2 ø 4 pr22 è2ø 4
dt
q dq t 129. As, b = 2 a and g = 3 a
Þ òqi ( q - q0) = - aò0 dt b 2a 2 g 3a 3
\ = = and = =
Þ q = q0 + ( qi - q0) e- at g 3a 3 a a 1
This relation tells us that, temperature of the body varies 130. If Tx = Ty and Tx ¹ Tz, then Ty ¹ Tz
exponentially with time from qi to q0 .
1 If Tx ¹ Ty and Tx ¹ Tz, then Ty ¹ Tz but Ty may be equal to Tz.
121. According to Wien's displacement law, lm µ . Hence,
T
131. With the given amount of heat, the heating effect is more on a
if temperature increases l m decreases i. e. , peak of the E - l body of smaller surface area than that of larger surface area.
curve shift towards left.
dQ dq
132. According to Stefan’s law, E = aT 4
122. = - KA æ dT ö
dt dx Power radiated, P = ( 4 pr 2) sT 4 = ms ç - ÷
è dt ø
dQ
Q , K and A are constants for all points 3 3 dT
dt = pr rs
Þ dq µ - dx , i. e., temperature will decrease linearly with x. 4 dt
dT
123. Since the curved surface of the conductor is thermally where, = R = rate of cooling
dt
insulated, therefore in steady state, the rate of flow of heat at
\ P µ r2
every section will be the same. Hence, the curve between H
and x will be straight line parallel to x-axis. 4
Also, from 4 rr 2T 4 = pr3r s (R)
3
124. According to Stefan's law, E = sT 4
3 sT 4
Þ In E = In s + 4 In T R=
rrs
Þ ln E = 4 In T + In s 1

On comparing this equation with y = mx + c, r
we find that graph between In E and In T will be a straight line, 133. In the given graph, the region AB represents no change in
having positive slope (m = 4) and intercept on In E axis equal temperature with time. It means ice and water are in thermal
to In s. equilibrium.
dq eAs 3 The region BC shows the change in temperature with time.
125. = 4q0 Dq
dt mc The region CD represents a constant temperature (100°C)
eAs 3 with time. It means, water and steam are in thermal
For given sphere and cube 4q0 Dq is constant, so for both
mc equilibrium at boiling point.
dq
rate of fall temperature = constant. 134. A real gas can behave as an ideal gas under low pressure and
dt
high temperature, then all the gas laws are obeyed.
1
126. Wien's law lm µ or n m µ T 135. For an ideal gas, pV = a constant, when temperature is
T
constant. Thus, the variation between pV and V is a straight
n m increases with temperature. So, the graph will be straight line parallel to V-axis. Hence, graphs (a), (b) and (c) are wrong.
line.
136. The heat from hot milk spread on the table is transferred to the
127. When a body cools by radiation, the rate of cooling is given surrounding by conduction, convection and radiation, and
by the temperature of milk falls off exponentially with time
dq eAs 4
=- ( q - q40) according to Newton's law of cooling.
dt mc
Telegram @unacademyplusdiscounts

568 JEE Main Physics

137. According to Stefan's law, 143. As, DQ = cm DT + mL = 0.5 ´1(5)+1´ 80 = 82.5 cal
E = eAsT 4
144. As, DQ = mL = 10 ´ 540 = 5400 cal
Þ E A = eA AsTA4
and EB = eB AsTB4 145. As, DQ = mL = 10 ´ 80 = 800 cal
Q E A = EB 146. SI unit of latent heat is Jkg –1
\ eATA4 = eBTB4
147. Here, p = 2 atm = 2 ´1.013 ´105 Nm–2
1 1
æ eA 4 ö 4 æ1 4ö 4 T = 17 + 273 = 290 K, s = 2 r = 2 ´ 1 Å = 2 ´ 10 –10 m
Þ TB = ç TA ÷ = ç ´ (5802) ÷
è eB ø è 81 ø
kT (1.38 ´ 10 –23) ´ 290
l= =
Þ TB = 1934 2 ms 2p 1.414 ´ 3.14 ´ (2 ´ 10 –10) 2 ´ 2.026 ´ 10 5
And, from Wien's law l A ´ TA = lB ´ TB
= 1.11 ´ 10 –7 m
l A TB
Þ = 148. For nitrogen molecule,
lB TA
M = 28 g = 28 ´ 10 -3 kg
lB - l A TA - TB
Þ =
lB TA 3 RT 3 ´ 8.31 ´ 290
v rms = =
1 5802 - 1934 3868
M 28 ´ 10 –3
Þ = =
lB 5802 5802 = 5.1 ´ 10 2 ms–1

Þ lB = 1.5 mm 149. As, collision frequency = number of collisions per second


138. As face ABCD has positive charge on it and the gas consists of v rms 5.1 ´ 10 2
= =
ionized hydrogen, therefore, isotropy is lost, The usual l 1.1 ´ 10 –7
expression for pressure on the basis of kinetic theory will not = 4.58 ´ 10 9 s–1 » 5 ´ 10 9 s–1
be valid as ions would also experience forces, other than the
forces due to collisions with the walls of the container. 150. Time taken for collision of nitrogen molecules
s 2 ´ 10 -10
139. In a diatomic molecule, the rotational energy at a given t1 = = = 4 ´ 10 -13 s
temperature obeys Maxwell's energy distribution law. As v rms 5.1 ´ 10 2
each such atom has three translational degrees of freedom
152. Stefan’s law applies here and not the Newton’s law of
and two rotational degrees of freedom, therefore, at a given
2 cooling.
temperature, rotational energy is rd the translational KE of According to Stefan’s law,
3
4 4
each molecule. Energy associated with each molecule per E 2 æ T2 ö æ 900 ö
æ 1 ö =ç ÷ =ç ÷ = 81
degree of freedom is fixed ç = kB T ÷ . E1 è T1 ø è 300 ø
è 2 ø
E2
mL = 81 \ E 2 = 81R
mwQw - l i 10 ´ 100 - 10 ´ 80 R
Cw 1
140. As, Q mix = = 153.Thermal conductivity of the wall depends only on nature of
mi + mw 10 + 10
material of the wall; and not on temperature difference across
1000 - 800
= its two sides.
20
200 154. On increasing pressure, boiling point of water increases.
= = 10°C Therefore, cooking is faster.
20
1
141. 10 g water heat taken by ice to melt at 0°C is 155. lm µ as per Wien’s displacement law.
T
Q1 = mL = 10 ´ 80 = 800 cal
156.When two bodies at temperatures T1 and T2 are brought in
Heat given by water or cool upto 0°C is
thermal contact, they do settle to the mean temperature
Q 2 = ms Dq = 10 ´ 1 (80 - 0) = 800 cal æ T1 + T2 ö
ç ÷ . They will do so, in case the two bodies were of
Hence, heat given by water is just sufficient to melt the è 2 ø
whole ice and final temperature of mixture is 0ºC. same mass and material i. e. , same thermal capacities. In other
2 m ´ 60 ´ (1 - m) ´ 80 words, the two bodies may have different thermal capacities
142. Temperature of mixture =
(2 m + m) that is why they do not settle to the mean temperature, when
= 13.3° C brought together.
Telegram @unacademyplusdiscounts

Heat and Kinetic Theory of Gases 569

157.According to Newton’s law of cooling p1V1 + p2V2


\ p= …(ii)
dQ V1 + V2
µ ( q - q0)
dt For mixture of two gases
1 (m1 + m 2) RT = p (V1 + V2)
158.According to Wien’s law, lm µ when T is halved, lm
T Using Eqs. (i) and (ii), we get
becomes twice. æ p1V1 p2V2 ö ( p V + p2V2) (V1 + V2)
3p gp c 3 ç + ÷ RT = 1 1
159.We know, crms = and c = Þ rms = è RT1 RT2 ø (V1 + V2)
r r c g
æ p1V1 p2V2 ö
or ç + ÷ T = ( p1V1 + p2V2)
T è T1 T2 ø
160. As, crms µ ; When T is doubled and M has become half,
M ( p1V1 + p2V2) T1T2
the, crms will become two times. On solving, we get T=
( p1V1T2 + p2V2T1)
161.Density of water = 1g/cc
167. When we increase the temperature of a liquid, the liquid will
\ Mass of 1 cc of water = volume×density expand. So, the volume of the liquid will increase and hence,
= 1 ´ 1 = 1g of water the density of the liquid will decrease.
In 1 g mole (or 18 g) of water, the total number of molecules
168. An ideal gas is a gas which satisfying the assumptions of the
= 6.023 ´ 10 23 kinetic energy.
\Number of molecules of water in 1 g
169. From Newton’s law of cooling
6.023 ´ 10 23 1 q1 - q2 æ q1 + q2
= = ´ 10 23 µç
ö
- q0 ÷
18 3 t è 2 ø
5 7
1´ 1´ where, q0 = temperature of surroundings
3 + 5
m1 g1 m 2g 2 æ 5 ö æ 7 ö 59 - 49 æ 50 + 49 ö
+ - 1 - 1÷ \ µç - 30 ÷ …(i)
ç ÷ ç t1 è 2 ø
g -1 g 2 -1 è 3 ø è 5 ø 3
163. g mix = 1 = = = 1.5
m1 m æ ö æ ö 2 40 - 39 æ 40 + 39 ö
+ 2 and µç - 30 ÷ …(ii)
g1 - 1 g 2 - 1 ç 1 ÷ + ç 1 ÷ t2 è 2 ø
ç ÷ ç ÷
ç 5 - 1÷ ç 7 - 1÷ Dividing Eq. (ii) by Eq. (i), we get
è3 ø è5 ø
t 2 39
=
164. For diatomic gas, number of degrees of freedom per t1 19
molecules is, n = 5 39
Þ t2 = ´ 5 = 10 s
Average kinetic energy of diatomic gas due to thermal motion, is 19
5 5 5 m
E = (nR) T = pV = ´ p ´ 170. As, the two portions of the slab are connected in series,
2 2 2 J Sxi x+ x 2 K1K2
therefore, K = = =
5 ´ (8 ´ 10 4) ´ 1 Sxi / Ki x x K1 + K2
= = 5 ´ 10 4 J +
2´4 K1 K2
c12 + c22 + ¼+ c62 171. According to Wien’s displacement law,
165. As, crms =
6 b
lm =
2 2 2
2 +5 +3 +6 +3 +5 2 2 2 T
= 1
6 \ Tµ
lm
= 3 2 = 4.242 unit
As l m increases, hence T decreases Þ T1 > T2 > T3 .
166. According to standard gas equation
nT +n T
p1V1 172. As T = 1 1 2 2
= m1R n1 + n2
T1
æ7 ö
p2V2 1(T0) + 1 ç T0 ÷
and = m 2R …(i) è3 ø 10 T0 5 T0
T2 \ Tf = = =
1+ 1 3 ´2 3
As no work is done in removing the partition, total energy pV
remains conserved. Therefore, 173. The value of for one mole of an ideal gas
T
3 3
( p1V1 + p2V2) = p (V1 + V2) = gas constant = 2 cal mol–1 K –1
2 2
Telegram @unacademyplusdiscounts

570 JEE Main Physics

174. As, DV = V g Dt In this equation,


Þ 0.24 = 100 ´ g ´ 40 N = number of moles of the gas
0.24 p = pressure of the gas
g= = 0.00006 = 6 ´ 10 –5
100 ´ 40 V = volume of the gas
g R = universal gas constant
a=
3 and T = temperature of the gas from Eq. (i),
Þ a = 2 ´ 10 -5° C–1 N p
we have = = constant all gases will contain,
175. Let the temperature of common interface be T°C. V RT
Rate of heat flow So, at constant pressure and temperature equal, numbers of
Q KA DT molecule per unit volume.
H= = Note This result is nothing but Avogadro’s laws.
t l
æQ ö 2 KA (T - T1) 180. The entropy function gives us a numerical measure of the
\ H1 = ç ÷ =
è t ø1 4x irreversibility of a given process, i. e. , it is a measure of
KA (T2 - T) disorder of a system. During formation of ice cubes
æQ ö
and H2 = ç ÷ = orderedness increases i.e., disorderness decreases, hence
è t ø1 x
entropy decreases.
In steady state the rate of heat flow should be same in whole 1
system i.e., 181.Average energy per molecule per degree of freedom = kT
2
H1 = H2
2 KA (T - T1) KA (T2 - T) 182. We know that the thermo electromotive force is maximum at
Þ = neutral temperature. Hence, the thermoelectric power will
4x x
also maximum at neutral temperature.
T - T1
Þ = T2 - T 183. Let m1 = m2 = m3 = m
2
Þ T - T1 = 2 T2 - 2 T Let s1, s2, s3 be the respective specific heats of the three
2T + T liquids,
Þ T= 2 1 …(i) When A and B are mixed, temperature of mixture = 16°C
3
As heat gained by A = heat lost by B
Hence, heat flow from composite slab is
é KA (T2 - T) ù KA æ 2 T2 + T1 ö KA \ ms1 (16 - 12) = ms2 (19 - 16)
H=ê úû = x çèT2 - ÷= (T2 - T1) …(ii) 4 s1 = 3 s2 …(i)
ë x 3 ø 3x
When B and C are mixed, temperature of mixture = 23°C.
[From Eq. (i)]
As heat gained by B = heat lost by C,
é A (T2 - T1) K ù
Accordingly, H=ê úû f ms2 (23 - 19) = ms3 (28 - 23)
ë x
\ 4 s2 = 5 s3 …(ii)
By comparing Eqs. (ii) and (iii), we get
3 15
1 From Eqs.(i) and (ii), s1 = s2 = s3
Þ f= 4 16
3
When A and C are mixed, suppose temperature of
176. According to kinetic theory of gas, the mean kinetic energy of mixture = t
1 heat gained by A = heat lost by C
molecules per degree of freedom is given by kT
2 ms1 (t - 12) = ms3 (28 - t )
and for a gram mole, the kinetic energy
15
NkT RT æ Rö s3 (t - 12) = s3 (28 - t )
= = ç since, k = ÷ 16
2 2 è Nø
15 t - 180 = 448 - 16 t
177. We know that thermal radiations consists of larger 628
t= = 20.2° C
wavelength as compared to gamma rays and wavelength in 31
visible regions and so, thermal radiations belong to infrared
region. 184. Here, lm1 = lm ,T1 = 2000 K
178. Kinetic energy of ideal gas depends only on its temperature. l m2 = ?, T2 = 3000 K
Hence, it remains constant whether its pressure is increased According to Wein’s displacement law,
or decreased. l m2T2 = l m1T1
179. Ideal gas equation can be written as T 2000 2
l m2 = l m1 1 = l m ´ = lm
pV = n RT ...(i) T2 3000 3
Telegram @unacademyplusdiscounts

Heat and Kinetic Theory of Gases 571

185. Energy radiated per second by a body at temperature T K as sA (3T) 4 - sA (T ¢) 4 = sT (T ¢) 4 - sA (2 T) 4


per Stefan’s law is (3 T) 4 - (T ¢) 4 = (T ¢) 4 - (2 T) 4
4 4
E = esAT = 0.6 sAT
(2 T ¢) 4 = (16 + 81) T 4
186. One calorie is defined as the amount of heat required to raise æ 97 ö
1/ 4

the temperature of 1 g of water from 14.5°C to 15.5°C at T¢ = ç ÷ T


è2ø
atmospheric pressure.
187. In convection process, the heat is transferred by the bodily 191. In steady state the temperature decreases exponentially from
motion of the heated particles. It is not so in case of warming hot end to cold end. Also the temperature decreases more
of glass bulb due to filament heating. In fact, warming of glass rapidly near hot end and goes down to slow towards cold
bulb is due to radiation. end.
R 2sT 4
188. To measure the radial rate of heat 192. Solar constant =
flow, we have to go for integration r2
technique as here the area of the R 2sT 4
dx \Radiation power incident to earth = pr02 ´
surface through which heat will r2
r1
flow is not constant.
193. When two gases are mixed together, then
Let us consider an element x Heat lost by the Helium gas = Heat gained by the Nitrogen gas
(spherical shell) of thickness dx r2
æ7 ö
and radius x as shown in figure. Þ mB ´ (CV ) He ´ ç T0 - Tf ÷ = m A ´ (CV ) N2 ´ (Tf - T0)
è3 ø
Let us first find the equivalent
thermal resistance between inner Box A Box B
and outer sphere.
dx 1 mole N 2 1 mole He
Resistance of shell = dR = Temperature = T0 7
K ´ 4 px2 Temperature = T0
3
é 1 ù
êFrom R = KA ú 3 æ7 ö 5
ê ú Þ 1 ´ R ´ ç T0 - Tf ÷ = 1 ´ R ´ (Tf - T0)
2 è3 ø 2
ë where, K ® thermal conductivity û
3
As, ò dR = R By solving, we get Tf = T0
2
r2 dx 1 é 1 1ù r2 - r1 f f f f
Þ R=ò = ê - ú= 194. As, n1k1T1 + n2kT2 + n3kT3 = (n1 + n2 + n3) kT
r1 4 pKx2 4 pK r r
ë 1 2û 4 pK (rr1 2) 2 2 2 2
Rate of heat flow = H n1T1 + n2T2 + n3T3
Þ T=
T -T T -T n1 + n2 + n3
= 1 2 = 1 2 ´ 4pK (rr
1 2)
R r2 - r1 f
195. For 1 kg gas energy E = rT
rr
12
2
µ
r2 - r1 As p = rrT
dq Therefore, rT = p / r
189. As, = -K ( q - q0)
dt 5 8 ´ 10 4
loge (θ – θ0)

\ E= ´ [f = 5, for diatomic gas]


(Newton’s law of cooling) 2 4
q dq t or E = 5 ´ 10 4 J
Þ òq 0 q - q0
= -k ò dt
0 196. The number of moles of the system temains same
or log ( q - q0) = - kt + C
O t p1V1 p2V2 p (V1 + V2)
\ + =
So, graph is straight line. RT1 RT2 RT
p (V1 + V2) T1T2
190. In steady state energy absorbed by middle plate is equal to Þ T=
p1V1T2 + p2V2T1
energy released by middle plate
According to Boyle’s law
p1V1 + p2V2 = p (V1 + V2)
( p V + p2V2) T1T2
\ T= 11
( p1V1T2 + p2V2T1)
3T T 2T 197. According to Newton’s cooling law, options (c) is correct
answer.
Telegram @unacademyplusdiscounts

14 Thermodynamics
JEE Main MILESTONE
< Thermal Equilibrium < Heat Engine
< Zeroth Law of Thermodynamics and < Different Thermodynamic Processes
Concept of Temperature < Efficiency of a Cycle
< Work < Carnot Cycle
< First Law of Thermodynamics < Carnot’s Engine
< Second Law of Thermodynamics

14.1 Thermal Equilibrium


It is observed that a higher temperature object which is in contact with a lower
temperature object transfers heat to the lower temperature object. The objects will
approach the same temperature and in the absence of loss to other objects, they
will then maintain a constant temperature. They are then said to be in thermal
equilibrium.

14.2 Zeroth Law of Thermodynamics and


Concept of Temperature Thermodynamics is a branch of
science which deals with
According to this law, if two systems A and B are each in thermal equilibrium with transformation of heat energy
a third system C, then A and B will be in thermal equilibrium with each other. into other forms of energy and
vice-versa.
A
C Insulated wall

Diathermic wall

Therefore, there must be a certain scalar physical quantity which is identical for all
systems in thermal equilibrium. This quantity (scalar) is the temperature.
Suppose for systems A, B and C are in thermal equilibrium. Then,
T A = TB = TC
So for a body, temperature is that physical quantity which decides the degree of
hotness or coldness of a body and is responsible for heat flow.
Telegram @unacademyplusdiscounts

Thermodynamics 573

A process can be represented by a curve on the p-V


14.3 Work diagram. If the gas goes from initial state A( p1, V1 ) to the
It is defined as the product of force and its displacement in final state B( p2, V2 ), the work done is given by W = area
the direction of force. Its unit is joule or N-m, i. e. , under the curve AB above the X-axis (shaded portion).
W = F dx
Internal Energy
Work Done by a Gas During Expansion Internal energy of a system is the energy possessed by the
system due to molecular motion and molecular
Let us consider an ideal gas
configuration. The energy due to molecular motion is
enclosed in a perfectly insulated
called internal kinetic energy U K and that due to
cylindner fitted with a
dx molecular configuration internal potential energy U P
non-conducting and frictionless
piston. Let p be the pressure exerted i. e. , U = U K + UP
by the gas and V be the volume of Regarding internal energy it is worth noting that
the gas at any particular instant and Gas (i) Change in internal energy is path independent and
A be the area of cross-section of the depends only on the initial and final states of the
piston. system,
The force exerted by the gas on the piston = pA i. e., DU = U F - U I

If the piston moves through an infinitesimal distance dx, (ii) Change in internal energy in a cyclic process is always
zero as for cyclic process U F = U I , so that
this force can be assumed constant and the work done
(dW ) is given as DU = U F - U I = 0

dW = ( pA) dx = pdV (iii) In case of ideal gas as there is no molecular attraction


U P = 0, i. e., internal energy of an ideal gas is totally
(Q A dx = dV = infinitesimal change in volume)
kinetic and is given by
During expansion of the gas work is done by the gas and is 3
U = U K = nRT
taken as positive while if work is done on the gas, it is 2
taken as negative. 3
with DU = nR DT
2
Work Done from p-V Diagram where, n = number of moles

An ideal gas contained in a cylinder fitted with a massless and R = gas constant.
and frictionless piston can be considered as a (iv) In case of gases, whatever be the process
thermodynamical system. Its state can be represented by nR DT
DU = nCV DT =
variables ( p, V , T ). If p and V are known, T can be (g - 1)
calculated as pV = nRT . The change in internal energy (DU ) of a system in case of
p
gain is taken as positive while it is taken as negative in

p1
case of loss of energy. A

B
p2
14.4 The First Law of
Thermodynamics
V
O V1 V2 When a system changes for a given initial state to a given
p-V diagram
final state, both the work W and heat Q depend on the
nature of the process.
The state of system at any instant of time can be specified
by two variables ( p, V ). The relation between pressure p Experimentally, however, we find a surprising thing, the
and volume V can be studied on a pressure-volume (p -V ) quantities Qand W is the same for all processes. It depends
graph known as indicator diagram. only on initial and final states and does not depend at all
on how the system gets one state from the another. All
On such a graph, each equilibrium state of a
other combinations of Qand W including Qalone, W alone,
thermodynamic system can be represented by a point
Q`+ W and Q - 2W are path dependent, only the quantity
whose x-coordinate represents volume (V ) and
Q - W is not.
y-coordinate represents pressure ( p).
Telegram @unacademyplusdiscounts

574 JEE Main Physics

The quantity Q - W must represent a change in same Sample Problem 4 A certain p


intrinsic property of the system. This property is the amount of an ideal gas passes from state
1
internal energy U and we write A to B first by means of process 1, then
DU = U f - U i = Q - W …(i) by means of process 2. In which of the A B
process is the amount of heat observed
This equation is the first law of thermodynamics. by the gas greater? 2
If the thermodynamic system undergoes only a V
(a) Process 1
differential change, we can write Eq. (i) as
(b) Process 2
dU = dQ - dW …(ii) (c) Equal in both process
We can not write Eq. (ii) as (d) None of these
dQ = dU + dW …(iii) Interpret (a) For process 1, DQ1 = DW1 + DU1
Thus, we can say that, heat supplied to the system is the and For process 2, DQ 2 = DW2 + DU2
sum of external work done (dW ) by the system and U is a state function. Hence, DU depends only on the initial and
increase in its internal energy (dU ). final positions. Therefore,
DU1 = DU2
Note First law of thermodynamics is a direct consequence of law of But W1 > W2
conservation of energy.
as the area under 1 is greater than area under 2. Hence, Q1 > Q 2
Sample Problem 1 When water is boiled under a pressure p p
6 -1
of 2 atm, the heat of vaporization is 2.20 ´ 10 Jkg and the
boiling point is 120°C. At this pressure, 1 kg of water has a A B A B
volume of10 -3 m3 and 1 kg of steam has a volume of 0. 824 m3. W1
What is the work done when 1 kg of steam is formed at this W2
temperature?
V V
(a) 166.74 kJ (b) 266.74 kJ
(c) 366.74 kJ (d) 466.74 kJ
Interpret (a) Work done = p ( DV ) (at constant pressure) 14.5 The Second Law of
3
\ W = 2 atm ´ (0.824 - 0.001) m Thermodynamics
Þ W = 2 ´ 1.013 ´ 10 5 Nm-2 ´ 0.823 m3
The first law of thermodynamics tells us that in a
= 166.74 kJ
thermodynamic process taking place energy will be
Sample Problem 2 When a system goes from state A to conserved. However, it does not tell us whether a given
state B, is supplied with 400 J of heat and it does 100 J of work. process in which energy is conserved will actually take
For this translation what will be the change in internal energy of place or not. So, there must be a law of nature other than
the system? 1st law, which decides whether a given process, allowed
(a) 250 J (b) 300 J by 1st law, will actually take place or not. This law is the
(c) 350 J (d) 150 J second law of thermodynamics. The following two forms are
Interpret (b) From the first law of thermodynamics, worth mentioning about this law.
DUAB = Q AB - WAB = ( 400 - 100) J = 300 J
Kelvin’s Statement
Sample Problem 3 In the above example, if the system It is impossible for an engine operating in a cyclic process
moves from B to A, what is the change in internal energy? to extract heat from a reservoir and convert it completely
(a) 300 J into work. In other words, whole of heat can never be
(b) - 300 J converted into work. Heat engine works on this principle.
(c) 400 J
(d) - 400 J Clausius Statement
Interpret (b) Consider a closed path that passes through the It is impossible for a self-acting machine unaided by any
state A and B. Internal energy is a state function so, DU is zero for a external agency to transfer heat from a colder to a hotter
closed path. reservoir. In other words, heat by itself can not pass from a
Thus, DU = DUAB + DUBA = 0 colder to a hotter body without an external agency.
or DUBA = - DUAB = - 300 J Refrigerator is based on this statement.
Telegram @unacademyplusdiscounts

Thermodynamics 575

14.6 Heat Engine Sample Problem 5 An automobile engine absorbs 1600 J


of heat from a hot reservoir and expels 1000 J to a cold reservoir
A heat engine is a evice which converts thermal energy in each cycle. What maximum work is done in each cycle?
into another useful forms of energy and work. (a) 400 J (b) 500 J
(c) 600 J (d) 450 J
The efficiency of heat engine
Work done by working substance Interpret (c) We know that,
h=
Heat given to working substance W = QH - QL
= 1600 - 1000 = 600 J
Source Working Sink
T1 Q1 substance Q2 T2
Sample Problem 6 In the above example what is the
efficiency of the cycle?
(a) 37.5% (b) 47.5%
W
W Q1 - Q2 (c) 40% (d) 30%
h= =
Q1 Q1 W
Interpret (a) As, h =
W Q - Q2 QH
%h = ´ 100 = 1 ´ 100
Q1 Q1 600
= = 0.375 = 37.5%
æ Q ö æ T ö 1600
= ç1 - 2 ÷ ´ 100 = ç1 - 2 ÷ ´ 100
è Q1 ø è T1 ø
Sample Problem 7 In the above example what is the
Note If Q 2 = 0 orT2 = 0 K ,then h = 100%, which is impossible. power output of the engine if it operates at 200 cycles per
minute?
(a) 3 HP (b) 4 HP
Types of Heat Engines (c) 2.7 HP (d) 3.7 HP
In practice, heat engine are of two types
Interpret (c) As the engine is operating at 200 cycles min -1
t = (60 / 200) = 0.3 s.
(a) External Combustion Engine
So, power output
In which heat is produced by burning the fuel in a W 600
chamber outside the main body (working substance) of = = 2 kW ( = 2.7 HP)
t 0.3
the engine. Steam engine is an external combustion
engine. The thermal efficiency of a steam engine varies
from 10 to 20%. Sample Problem 8 Calculate the least amount of work
that must be done to freeze one gram of water at 0°C by means
(b) Internal Combustion Engine of a refrigerator. Temperature of surrounding is 27°C. How
much heat is passed on the surrounding in this process ? Latent
In which heat is produced by burning the fuel inside the heat of fusion L = 80 calg -1.
main body of the engine. Petrol engine and diesel engines
(a) 87.91 cal
are internal combustion engine.
(b) 97.91 cal
(c) 88.95 cal
Refrigerator (d) 89.95 cal
A refrigerator or heat pump is basically a heat engine
Interpret (a) As, Q 2 = mL = 1 ´ 80 = 80 cal,
running in reverse
direction. It takes heat Source Working Sink T2 = 0° C = 273 K
from colder body (sink)
T1 Q1 substance Q2 T2
and T1 = 27°C = 300 K
and after doing some Q2 T2
=
work gives the rest heat W T1 - T2
W
to the hotter body Q 2(T1 - T2)
(source). Þ W= cal
T2
The coefficient of performance of refrigerator is 80(300 - 273)
= = 7.91
Heat extracted Q2 273
b= =
Work done W Þ Q1 = Q 2 + W
Q2 T2 1- h = (80 + 7.91) = 87.91 cal
= = =
Q1 - Q2 T1 - T2 h
Telegram @unacademyplusdiscounts

576 JEE Main Physics

14.7 Different Thermodynamics


Isothermal
Process Pressure
Vf
W = nRT ln ——
Reversible and Irreversible Processes Vi
Volume
A reversible process means, if a process takes up the path
AB (as shown in figure) then on reversing the conditions it Since for an ideal gas the internal energy is proportional to
comes back by BA. temperature, it follows that there is no change in the
p internal energy of the gas during an isothermal process.
B The first law of thermodynamics then becomes
DU = 0 = Q - W
\ Q=W
A
V All the heat added to the system is used to do work.

Note A thermal process however cannot be reversible. It could be Adiabatic Process


reversible, if the change is extremely small (infinitesimally small). In an adiabatic process, heat is neither allowed to enter
In irreversible process, one will not reach back to A if the nor allowed to escape the system. Specific heat in an
process AB has occured. adiabatic process is zero.
p p p
B
x –ve work +ve work
y
A Compression Expansion
V V V

p p
Compression
Isothermal Process Isothermal Ad
iab
atic
In isothermal process the temperature remains constant. Expansion
Melting and boiling points are examples. Specific heat in Adiabatic Isothermal
isothermal process is infinity. V V
For a constant temperature process involving an ideal gas,
Since, dQ = 0
pressure can be expressed in terms of the volume as
follows \ dU = - p dV
nRT In an adiabatic process,
p=
V (a) pV g = constant
The result of a heat engine process leading to expansion (b) p1 - g T g = constant
gives the work done
(c) TV g - 1 = constant
V 2 dV
W = ò pdV = nRT ò
V1 V Work done in an adiabatic process
V p1V1 - p2V2 nR(T1 - T2 )
= nRT loge 2 W= = ,
V1 g -1 g -1
V
= 2. 303 nRT log 2 Cp
V1 where g=
CV
p1
= 2. 303 nRT log
p2 Adiabatic elasticity (bulk modulus) = gp .
Isothermal elasticity = p (bulk modulus)
Note Slope of adiabatic curve is more in magnitude in comparison to the
The pressure versus volume curve for isothermal process slope of isothermal curve.
is as follows Slope of adiabatic curve = g (slope of isothermal curve)
Telegram @unacademyplusdiscounts

Thermodynamics 577

p2V2 3p0
Cyclic Process \ V3 =
p3
=
p0
(3V0) = 9V0
Figure shows a cyclic process ABCA, the work done
W31 = p0(V1 - V3) = p0(V0 - 9V0) = - 8p0V0
during the cycle can be calculated as
p W31 = - 8RT0
Applying gas law in process1 ® 2
p3 A p0V0 = RT
p2 C or aV02 = RT
p1 The net work,
B
\ Wnet = W12 + W23 + W31
V1 V2 V3
V = 4RT0 + 9.81RT0 - 8RT0 = 5.81RT0

Work done during the process AB = WAB. Sample Problem 10 A thermodynamic system is taken
Work done during the process BC = WBC . through the cycle abcda shown in figure. Find the work done by
Work done during the process CA = WCA. the gas during the parts ab, bc, cd and da.
The net work done during the cycle is
200 kPa d c
W = WAB + WBC + WCA
p
100 kPa a b
Sample Problem 9 One mole of an ideal monoatomic gas
undergoes thermodynamic cycle 1 ® 2 ® 3 ® 1 as shown in
the figure. Initial temperature of the gas is T0 = 300 K. 100 cm3 300 cm3
2 V
3p0
(a) 20 J (b) 0 J
p (c) - 40 J (d) zero
p0 1
3
Interpret (a, b, c, d) The work done during the part ab,
b b
= ò p dV = (100 kPa) ò dV
a a
V0 3V0
V = (100 kPa) (300 cm3 - 100 cm3) = 20 J
Process 1 ® 2 : p = aV
The work during bc is zero as volume does not change.
Process 2 ® 3 : pV = constant
The work done during cd,
Process 3 ® 1 : p = constant d d
[Take ln|3| = 109
. ] = ò p dV = (200 kPa) òc dV
c
The net work done by the cycle is = (200 kPa) (100 cm3 – 300 cm3) = - 40 J
(a) 3.27 RT0 (b) 6.83 RT0
(c) 4.53 RT0 (d) 5.81 RT0 The work done during da is zero as the volume does not change.

Interpret (d) For process 1 ® 2,


2 3V
Isochoric Process
W12 = ò aVdV = a ò VdV = (9V02 - V02) = 4 aV02
1 V0 2 An isochoric process, also called a constant volume
pV pV process, an isovolumetric process or an isometric process
Using gas law, 1 1 = 2 2
T1 T2 is one in which volume of the closed system undergoing
p2V2 V2 such a process remains constant.
Þ T2 = T1 = 22 T1
p1V1 V1 ● An isochoric process is exemplified by the heating or the
2 cooling of the contents of a sealed, in elastic container.
æ 3V ö
= ç 0 ÷ T0 = 9T0 ● An isochroric thermodynamic process is characterized
è V0 ø by constant volume DV = 0.
For process 2 ® 3, ● If the process during which the pressure of the system
p 3p remain constant.
W23 = RT2 log 2 = R(9T0) log 0
p3 p0 The process does no pressure-volume work since such
= 9RT0 log|3| = 9.81RT0 work is defined by
For isothermal process, DW = pDV
p2V2 = p3V3 The sign convention is such that positive work is
performed by the system on the environment.
Telegram @unacademyplusdiscounts

578 JEE Main Physics

we can say adiabatic curve is more steep than an isotherm


Isobaric Process for expansion and just reverse for compression.
An isobaric process is one in which volume and
temperature of system may change but pressure remain It is clear from the figure that for expansion that occurs
constant, i. e. , Dp = 0 within same limits.
1. For this process Charles’ law is obeyed. Wisobaric > Wisothermal > Wadiabatic > Wisochoric
V V
Hence, V µT Þ 1 = 2
T1 T2 Important Points
2. Specific heat of a gas during an isobaric process 1. For a reversible process, the first law of thermodynamics gives the
æf ö Q change in the internal energy of the system.
C p = ç + 1÷ R =
è2 ø nDT dU = dQ - dW
Replacing work with a change in volume gives
3. Work done in a isobaric process
dU = dQ - pdV
W = p(Vf - Vi ) = nR (T f - Ti ) = nRDT Since the process is isochoric, dV = 0 the above equation becomes
4. From first law of thermodynamics dU = dQ
DQ = DU + DW 2. Also for specific heat capacity at constant volume
Here, DW = nRDT CV =
dU
dQ = mC V dT
DU = nCV DT dT
\ DQ = nCV DT + nRDT Integrating both sides yields
b
= nC p DT Q = m ò C V dT
a
5. Bulk modulus of an isobaric process is zero where C V is the specific heat capacity at constant volume, a is initial
Dp temperature and b is final temperature, hence
Kisobaric = =0 (As Dp = 0)
DV Q = mC V DT
-
V
On pressure-volume diagram an isochoric process appears as a
6. p-V curve is a straight line parallel to volume axis, straight vertical line.
dp p p
slope of p-V curve for an isobaric process = 0.
dV
–ve work +ve work
(i) Graph 1 incidates isobaric p
expansion. The heat is 1 Isobaric compression Isobaric expansion
given to the gas. The V V
2
volume and temperature
of the gas both will rise. 3. Work done on gas in some process
The gas expands during p p
positive work. V 1 1

(ii) Graph 2 indicates isobaric compression. In this


process, heat is taken out of the gas. The
temperature falls and the gas contracts causing 2 2
negative work. V V
DV = 0 but dV ¹ 0 DW can be zero
Indication Diagram of p-V Curves Work done = + ve but dW ¹ 0
p
We can determine the work done from area under p-V
curve.
Isothermal

(compression)
p
Adiabatic
V
Isobaric For clockwise DW = + ve
O
For anti-clockwise DW = - ve
Isothermal
Isochoric Adiabatic (expansion) 4. Work done is least for monoatomic gas expansion
V p
Dp p
For isothermal process, = - × For adiabatic process,
DV V Isothermal
Dp gp Polyatomic adiabatic process
=- i. e. , it means that at a particular point, slope Monoatomic
DV V
(value) of adiabatic curve is more than that for isotherm or V
V1 V2
Telegram @unacademyplusdiscounts

Thermodynamics 579

p
14.8 Efficiency of a Cycle T1
(p1V1)Q1 Isothermal
expansion
A
In a cyclic process, (p2V2)
Adiabatic B
DU = 0 compression Heat in
and Qnet = Wnet
V4
T2 p4 p3 V
(from first law of thermodynamics) 3 T2
D C
Isothermal Heat out
First we see what is the meaning of efficiency of a cycle. compression Q2

Suppose 100 J of heat is supplied to a system (in our case it a c V


b d
is an ideal gas) and the system does 60 J of work. Then,
TH - TC
efficiency of the cycle is 60%. Thus, efficiency (h) of a cycle Carnot efficiency, h = ´ 100%
can be defined as TH

æ Work done by the working substance ö The temperature in the Carnot efficiency expression must
ç ÷ be expressed in kelvins.
(an ideal gas in our case) during a cycle ÷
h=ç ´ 100
ç Heat supplied to the gas during the cycle ÷ The efficiency of a heat engine cycle is given by
ç ÷
è ø W QH - QC
h= =
Wtotal | Q | – | Q–ve | QH QH
= ´ 100 = +ve ´ 100
| Q+ve | | Q+ve | For the ideal case of the Carnot cycle, this efficiency can
be written as
ì Q ü
= í1 – –ve ý ´ 100 T - TC
Q h= H
î +ve þ TH
Wtotal
Thus, h= ´ 100 Using these two expressions together
| Q+ve |
Q T
ì 1- C = 1- C
Q ü QH TH
= í1 – –ve ý ´ 100
î Q+ve þ QC QH
=
TC TH
Note
QH QC
(i) There cannot be a cycle whose efficiency is 100%. Hence, h is or - =0
always less than100%. TH TC
Thus, WTotal ¹ Q + ve If we take Q to represent heat added to the system, then
(ii) It is just like a shopkeeper. He takes some money from you. heat taken from the system will have a negative value. For
(Suppose he takes ` 100/- from you). In lieu of this he provides the Carnot cycle
services to you (suppose he provides services of worth ` 80/-).
Q
Then, the efficiency of the shopkeeper is 80%. There cannot be a S i =0
i Ti
shopkeeper whose efficiency is 100%. Otherwise what will he
save?
Which can be generalized as an integral around a
reversible cycle
dQ
14.9 Carnot Cycle ò T =0 (Clausius theorem)

The most efficient heat engine cycle is the Carnot cycle For any part of the heat engine cycle, this can be used to
consisting of two isothermal processes and two adiabatic define a change in entropy S for the system
process. In order to approach the Carnot efficiency, the 2 dQ
processes involved in the heat engine cycle must be S2 - S1 = ò
1 T
reversible and involve no change in entropy. This means
Or in differential form at any point in the cycle
that the carnot cycle is an idealization, since no real
engine processes are reversible and all real physical dQ
dS =
processes involve some increase in entropy. T
Telegram @unacademyplusdiscounts

580 JEE Main Physics

For any irreversible process, the efficiency is less than that Sample Problem 12 A gasoline engine takes in 5 moles
of the Carnot cycle. of air at 20°C and 1 atm, and compresses it adiabatically to
This can be associated with less heat flow to the system 1/10 th of the original volume. Find the final temperature and
and/or more heat flow out of the system. The inevitable pressure. Assume air to be diatomic. The work done and
result is change in internal energy is
dQ (a) 46 kJ , - 46 kJ
ò T £ 0 (Clausius inequality) (b) 36 kJ , - 36 kJ
(c) -46 kJ , 46 kJ
(d) 36 kJ , - 46 kJ
14.10 Carnot’s Engine Interpret (c) Let p1 = 1 atm, n = 5 moles, T1 = 293 K
A heat engine is a device which transforms heat into V1
V2 =
mechanical work continuously. Carnot designed a 10
theoretical engine which is free from all defects of Using, T1V1g -1 = T2V2g -1
practical engine. This engine cannot be realised in g -1
practice. æV ö
Þ T2 = T1ç 1 ÷ = 293(10) 0.4 = 736 K
è V2 ø
It consists of a cylinder having perfectly insulating walls
and perfectly conducting base. Heat can enter or leave the nR(T1 - T2) 5 ´ 8.3 ´ (293 - 736)
Work done = = = - 46 kJ
cylinder through its conducting base. It is fitted with a g -1 0.4
piston in the cylinder. The working substance draws heat DU = DQ - W = 0 - W = 46 kJ
from the heat source but the temperature of the source
remains constant (T1 ). The engine also consists a heat sink Sample Problem 13 How much work is done by ideal
of infinite thermal capacity, the heat may be rejected by gas in expanding isothermally from an initial volume of 3 L at
the working substance to the sink whose temperature also 20 atm to a final volume of 24 L?
remains constant (T2 ). The engine has an insulating stand (a) 1.36 ´ 10 5 J (b) 1.26 ´ 10 4 J
which thermally isolates the working substance from the (c) 1.36 ´ 10 4 J (d) 2.36 ´ 10 5 J
surrounding. An ideal gas is used as the working
substance. Interpret (b) In isothermal process at temperature T
V2
W = 2.303 nRT log10
Sample Problem 11 Carnot’s engine takes in a thousand V1
kilo calories of heat from a reservoir at 827°C and exhausts it to V
W = 2.303( p1V1) log10 2 (Using p1V1 = nRT)
a sink at 27°C. How much work does it perform? What is the V1
efficiency of the engine? 24
= 2.303(20 ´ 2) log10 L-atm
(a) 2.70 ´ 10 5 cal , 70.70% (b) 2.70 ´ 10 5 cal, 72.72% 3
(c) 2.70 ´ 10 5 cal , 80.70% (d) 3.70 ´ 10 5 cal , 70.70% = [2.303 ´ 60 ´ log10 8] (101) J = 1.26 ´ 10 4 J

Interpret (b) Given, Q = 10 6 cal


T1 = (827 + 273) = 1100 K Check Point
and T2 = (27 + 273) = 300 K
1. A piece of metal is hammered. Does its internal energy
Q1 Q 2 increase?
As, =
T1 T2
2. A cylinder filled with gas is placed in heat-proof jacket. How
T æ 300 ö 6 will the temperature of the gas change, if the volume of the
\ Q 2 = 2 Q1 = ç ÷ (10 )
T1 è1100 ø cylinder is gradually increased?
= 2.720 ´ 10 5 cal 3. If hot air rises, why is it cooler at the top of a mountain than
near sea level?
Efficiency of the engine
4. On removing the valve, the air escaping from a cycle tube
æ T ö becomes cool. Why?
h = ç1 - 2 ÷ ´ 100
è T1 ø 5. Why is it impossible for a ship to use the internal energy of sea
æ 300 ö water to operate its engine?
or h = ç1 - ÷ ´ 100 = 72.72%
è 1100 ø
Telegram @unacademyplusdiscounts

Thermodynamics 581

Table 16.1 Different Thermodynamic Processes at a Glance

Change or Name of
S. No. Isobaric Isochoric Isothermal Adiabatic
Process
1. Definition p = constant V = constant T = constant (a) Q = constant
(b) Entropy, S = constant
2. dQ (i) For solids dQ (i) For solids dQ = dW Zero
= mC p dT dQ = mC VdT
(ii) For gases (ii) For gases,
dQ = mC p dT dQ = nC VdT
= nC p dT
(iii) For change in state
dQ = mL
3. dU (i) dQ - pdV dQ Zero - dW
(ii) dQ - nRdT
4. dW (i) pdV Zero
(ii) nRdT V2 R(T2 - T1 )
(i) 2. 303 nRT log10 (i)
V1 (1 - g )
V2 p2 V2 - p1V1
(ii) 2.303 p1V1 log10 (ii)
V1 (1 - g )
p1
(iii) 2.303 p1V1 log10
p2

5. Equation of state V
= constant
p
= constant pV = constant (i) pV g = constant
T T or p1V1 = p2 V2 (ii) TV g - 1 = constant
p1 p2
or
V1 V2
= or = (iii) p1 - g T g = constant
T1 T2 T1 T2

6. p-V graph p p Isotherm p


pV = constant
pi Adiabatic
p
p Hyperbola
pf
V V V
Vi Vf Isobaric expansion V Asiabatic expansion
Isobaric expansion Vi Vf
Isothermal expansion

7. Slope of p-V curve Zero ¥ p gp


- -
V V
8. Law Charle’s law Gay-Lussac’s law Boyle’s law Poisson’s law
9. Form of first law dQ = dU + dW dQ = dU = nC VdT dQ = dW = pdV (i) dW = - dU
= nC p dT + pd V (ii) dU = - dW
10. Bulk modulus Zero Infinity - p -gp
11. Result of maximum Maximum Zero Less from isobaric Minimum but not zero
work process but greater
from adiabatic process
Telegram @unacademyplusdiscounts

WORKED OUT
Examples
Example 1 A cylinder containing an ideal gas and closed by Example 2 A fixed mass of gas is taken through a process
a movable piston is submerged in an ice-water mixture. The A ® B ® C ® A. Here A ® B is isobaric B ® C is adiabatic and
piston is quickly pushed down from position (1) to position (2) C ® A is isothermal
(process AB).
A B
105
1
p(N/m2)
C
2
V
1 4
The pressure at C is given by (g = 1.5)
10 5 10 5
(a) N / m2 (b) N / m2
The piston is held at position (2) until the gas is again at 0°C 64 32
(process BC). Then the piston is slowly raised back to position (c) zero (d) 10 5 N /m2
(1) (process CA). Which one of the following p-V diagrams
correctly represent the processes. AB, BC and CA and the cycle Solution (a) For adiabatic process BC (pressure constant)
ABCA pB = pC …(i)
For isothermal process CA,
p p
pAVA = pC VC …(ii)
A A C From Eqs. (i) and (ii), we get
(a) B C ( b) 1 1
B
é V g ù g -1 é 4g ù g -1
VC = ê B ú =ê ú
V2 V1
V
V1 V2
V
ë VA û ë 1û
p p For g = 1.5, VC = 43 = 64 m3
A B B pAVA 10 5
\ pC = = N / m2
(c) C ( d) C
A VC 64

V2 V1
V
V1 V2
V Example 3 An ideal gas is taken through a cyclic
thermodynamic process through four steps. The amount of heat
involved in these steps are Q1 = 5960 J , Q 2 = - 5585 J ,
Solution In an adiabatic process, heat is neither allowed to Q3 = - 2980 J and Q 4 = 3645 J respectively. The efficiency of
enter nor allowed to escape the system, the process AB is adiabatic the cycle is
compression because piston is pushed very quickly from position 1 (a) 100% (b) 10.82%
to position 2. (c) 28% (d) 15%
The process BC is isochoric because in this case volume remains
constant, whereas process CA is an isothermal expansion because
Solution From the given problem,
temperature remains constant. These are shown on the p-V DQ = Q1 + Q 2 + Q3 + Q 4
diagram correctly in option (d). = 5960 - 5585 - 2980 + 3645
Note Slope of adiabatic curve is more in magnitude in comparison to be DQ = 9605 - 8565 = 1040 J
slope of the isothermal curve.
Telegram @unacademyplusdiscounts

Thermodynamics 583

Efficiency of a cycle is defined as Example 6 A Carnot's engine, with its cold body at 17°C has
50% efficiency. If the temperature of its hot body is now increased
Network DW DQ
h= = = by 145° C, the efficiency becomes
Input heat Q1 + Q 4 Q1 + Q 4
(a) 55% (b) 60%
Putting DQ = 1040 J (c) 40% (d) 45%
and Q 2 + Q 4 = 5960 + 3645 = 9605 J
T2
1040 Solution 1- = 0.5 or T1 = 2T2 = 2 (17 + 273) = 580 K
\ h= = 0.1082 = 10.82% T1
9605
Temperature of hot body is increased by 145°C or 145 K,
Example 4 The efficiency of an ideal gas with adiabatic \ T1¢ = (580 + 145) = 725 K
exponent g for the shown cyclic process would be and T 2 = (17 + 273) = 290 K
V
æ 290 ö
2V0 \ h = ç1 - ÷ ´ 100 = 60%
C è 725 ø

Example 7 A steam engine delivers 5.4 ´ 10 8 J of work per


V0
B
A minute and takes 36 . ´ 109 J of heat per minute from the boiler.
T What is the efficiency of the engine? How much heat is wasted
T0 2T0
per minute?
(2 ln 2 - 1) (1 - 1 ln 2) (2 ln 2 + 1) (2 ln 2 - 1) . ´ 10 9 J/min
(a) 31 (b) 2 ´ 10 9 J/min
(a) (b) (c) (d)
g / ( g - 1) g / ( g - 1) g /( g - 1) g /( g + 1) (c) 4 ´ 10 9 J/min (d) 6 ´ 10 9 J/min
Solution As, WBC = pDV = nRDT = - nRT0 Solution Here, Q1 = Heat absorbed per minute
and WCA = + 2nRT0 ln 2
nRg T0 Q 2 = Heat rejected per minute
Also, DQBC = nC p DT = W
g -1 We know that h % = ´ 100
Q1
Work (2 ln 2 - 1)
\ Efficiency = =
Input heat g /( g - 1) 5.4 ´ 10 8 J
\ h% = ´ 100
3.6 ´ 10 9 J
Example 5 During an adiabatic expansion, the increase in 3
= ´ 100 = 15%
volume is associated with which of the following possiblities 20
w.r.t. pressure and temperature? Also using the relation, Q 1 = W + Q 2, we get
Pressure Temperature Q 2 = Q1 - W
(a) increase increase = 36 ´ 10 8 - 5.4 ´ 10 8
(b) decrease decrease = 30.6 ´ 10 8 J / min
(c) increase decrease
= 3.06 ´ 10 9 J / min
(d) decrease increase
. ´ 10 9 J / min
= 31
Solution According to first law of thermodynamics
DQ = DU + DW Example 8 The volume of system produced by 1g of water at
For an adiabatic process, 100°C is 1650 cm3. What is the change in internal energy
DQ = 0 during the change of state? Given, J = 4.2 ´ 107 erg cal -1,
\ DU = - W g = 981 cms-2. Latent heat of steam = 50 calg -1.
In adiabatic process, (a) 2.0 ´ 10 9 erg (b) 2.0 ´ 10 11 erg
1 (c) 2.0 ´ 10 10 erg (d) 2.0 ´ 10 12 erg
pµ g
V
1 Solution Here, mass of water = 1 g
and Tµ g -1
V \ Initial volume of water, V1 = 1 cm3
g > 1, because volume increases. Volume of steam, V2 = 1650 cm3
then, p and T will decrease.
Change of internal energy, dU = ?
Telegram @unacademyplusdiscounts

584 JEE Main Physics

As the state of water is changing, Solution Let the original volume V1 = V


\ dQ = mL = 1 ´ 540 cal \Final volume V2 = V / 2
= 540 ´ 4.2 ´ 10 7 erg Initial pressure p1 = 0.76 m of Hg column.
= 22.68 ´ 10 9 erg Let p2 be the final pressure after compressions
Taking p = 1atm, As the change in adiabatic,
= 76 ´ 13.6 ´ 981 dyne cm -2 \ p1V1g = p2V2g
g 1.4
dW = pdV = p (V2 - V1) æV ö æ V ö
or p2 = p1 ç 1 ÷ = p1 ç ÷
= 76 ´ 13.6 ´ 981 (1650 - 1) è V2 ø èV / 2ø
= 76 ´ 13.6 ´ 981 ´ 1649 erg p2 = 0.76 ´ (2)1.4
= 1.67 ´ 10 9 erg
As dQ = dU + dW 1
Example 11 A gas is suddenly compressed to th of its
\ dU = dQ - dW = 22.68 ´ 10 9 - 1.67 ´ 10 9 4
original volume. What is the rise in temperature, the original
dU = 21.01 ´ 10 9 erg temperature being 27°C and g = 1.5?
(a) 300°C (b) 350°C
Example 9 Find the work required to compress adiabatically (c) 400°C (d) 450°C
1g of air initially at NTP to half its volume. Density of air at NTP
= 1.29 ´ 10 -3 gcm -3 and g = 1.4. Solution Let the initial volume, V1 = V
(a) 62.75 J (b) - 62.75 J Final volume, V2 = V/ 4
(c) 82.75 J (d) - 82.75 J Here, initial temperature,
T1 = 27° C = 273 + 27 = 300 K and g = 1.5
Solution Here, T1 = 0 + 273 = 273 K; Let T2 K be the final temperature after compression.
p1 = 1.013 ´ 10 6 dyne cm -2; g = 1.4 Since the change is adiabatic,
and density of air at NTP, r = 1.29 ´ 10 -3 gcm-3 \ T1V1g - 1 = T2V2g - 1
g -1 1.5 - 1
Mass 1 æV ö é V ù
\ V1 = = = 775.2cm3 or T2 = T1 ç 1 ÷ = 300 ê
Density 1.29 ´ 10 -3 è V2 ø ë V/ 4 úû
V 775.2
and V2 = 1 = = 387.6 cm3 = 300 ( 4)1/ 2 = 300 ´ 2 = 600 K
2 2
g T2 = 600 - 273 = 327° C
æV ö
Now, p1V1g = p2V2g or p2 = p1 ´ ç 1 ÷ \Rise in temperature = 317° C - 27° C = 300° C or 300 K
è V2 ø
1× 4
æ V ö Example 12 A gram molecule of a gas at 127°C expands
or p2 = 1.013 ´ 10 6 ´ ç 1 ÷ = 1.013 ´ 10 6 ´ (2)1.4 isothermally until its volume is doubled. How much will be the
è V1 / 2 ø
amount of heat energy absorbed?
= 1.013 ´ 10 6 ´ 2.639 = 2.673 ´ 10 6 dyne cm -2 (a) 400 cal (b) 548 cal
Work done during adiabatic change, (c) 500 cal (d) 580 cal
p V - p2V2
W= 11 Solution Here, temperature of the gas,
g -1
6 6 T = 273 + 127 = 400 K
1.013 ´ 10 ´ 775.2 - 2.673 ´ 10 ´ 387.6
= Let initial volume of the gas, V1 = V
1.4 - 1
\ Final volume of the gas, V2 = 2V
7.853 ´ 10 8 - 10.36 ´ 10 8
= = - 6.27 ´ 10 8 erg In an isothermal expansion,
0.4 V
= - 62.7 J Work done (W ) = 2.3026 RT log10 2
V1
2V
Example 10 A certain gas at atmospheric pressure is = 2.3026 ´ 8.2 ´ 400 ´ log10
V
compressed adiabatically so that its volume becomes half of its
= 2.3026 ´ 8.3 ´ 400 ´ 0.3010
original volume. What will be the resulting pressure in Nm -2?
or W = 2.30 ´ 10 3 J
Take, g = 1.4, for air.
(a) 76 ´ (2)1.4
(b) 0.76 ´ (2) 1.4 If H is the amount of heat absorbed
0.76 76 W 2.30 ´ 10 3
(c) (d) H= = = 548 cal
(2)1.4 (2)1.4 J 4.2
Telegram @unacademyplusdiscounts

Thermodynamics 585

Example 13 A cylinder containing one gram molecule of Example 15 The efficiency of a Carnot’s engine working
the gas was compressed adiabatically until its temperature rose between steam point and ice point is
from 27°C to 97°C. The heat produced in the gas (g = 1.5) is (a) 25% (b) 35%
(a) 250.6 cal (b) 276.7 cal (c) 298.5 cal (d) 320.6 cal (c) 40% (d) 50%

Solution Here, initial temperature, Solution Here, steam point,


T1 = 27° C = 273 + 27 = 300 K T1 = 100° C = 100 + 273 = 373 K
Final temperature, T2 = 97° C = 273 + 97 = 370 K Ice point, T2 = 0° C = 0 + 273 = 273 K
When a gas is compressed adiabatically, work done on the 273 100
As h =1- =
gas is given by 373 373
R 273 100
W= (T2 - T1) \ h =1- -
(1 - g) 373 373
8.3 ´ (370 - 300) 100
= = ´ 100% = 26.81%
1 - 1.5 2373
or W = - 11.62 ´ 10 2J
Example 16 A Carnot engine intakes steam at 200°C and
\Heat produced, after doing work, exhausts it to a sink at 100°C. The percentage
W 11.62 ´ 10 2 of heat which is utilised for doing work is
H= = = 276.7 cal
J 4.2 (a) 21% (b) 25% (c) 27% (d) 30%

Solution Here, T1 = 200°C = 200 + 273 = 473 K


Example 14 5 moles of an ideal gas is carried by a
quasi-static isothermal process at 500 K to twice its volume as T2 = 100°C = 100 + 273 = 373 K
shown in figure. T 273 100
h =1- 2 =1- =
T1 473 473
pA A
100
h= ´ 100% = 21.14%
pB 473
B
This is the percentage of heat which is utilized for doing work.

O VA VB Example 17 A Carnot engine absorbs 6 ´ 10 5 cal at 227°C.


V
The work done per cycle by the engine if its sink is maintained
at 127°C is
(i) How much work was done by the gas along the path AB?
(a) 15 ´ 10 8 J (b) 15 ´ 10 4 J
p
(ii) Calculate the pressure ratio B (c) 5 ´ 10 5 J (d) 2 ´ 10 4 J
pA
Given, R = 8.31J mol -1 K -1 Solution Here, Q1 = 6 ´ 10 5 cal
(a) 14,401.3 J, 2 (b) 34,28.9 J, 2 T1 = 227° C = 227 + 273 = 500 K
1 1
(c) 14,401.3 J, (d) 3428.9 J, T2 = 127° C = 127 + 273 = 400 K
2 2
Work done/cycle, W = ?
Solution (i) Here, n = 5; T = 500K; VB = 2 VA ; Q 2 T2
As =
R = 8.31 mol-1 K -1 Q1 T1
VB T2 400
Now, Wiso = n ´ 2.303 ´ RT log \ Q2 = ´ Q1 = ´ 6 ´ 10 5
VA T1 500
2VA = 4.8 ´ 10 5 cal
= 5 ´ 2.303 ´ 8.31 ´ 500 log
VA
As W = Q1 - Q 2 = 6 ´ 10 5 - 4.8 ´ 10 5
= 5 ´ 2.303 ´ 8.31 ´ 500 log2
= 5 ´ 2.303 ´ 8.31 ´ 500 ´ 0.3010 W = 1.2 ´ 10 5 cal
= 14,401.3 J = 1.2 ´ 10 5 ´ 4.2 J
(ii) For an isothermal change, pA VB = pBVB W = 5.04 ´ 10 5 J
pB VA V 1
\ = = A =
PA VB 2VA 2
Telegram @unacademyplusdiscounts

Start Practice for


JEE Main
Round I (Topically Divided Problems)

Thermodynamic Processes 6. If an average person jogs, hse produces


14.5 ´ 103 cal/min. This is removed by the
1. The ratio of the slopes of p-V graphs of adiabatic and
evaporation of sweat. The amount of sweat
isothermal is
g -1 evaporated per minute (assuming 1 kg requires
(a) (b) g - 1 580 ´ 103 cal for evaporation) is [NCERT Exemplar]
g
(a) 0.25 kg
(c) g /1 (d) g
(b) 2.25 kg
2. An ideal gas at a pressure 1 atm and temperature of (c) 0.05 kg
27°C is compressed adiabatically until its pressure (d) 0.20 kg
becomes 8 times, the initial pressure. Then, the final
æ 3ö 7. An ideal gas is taken from state A to state B following
temperature is ç g = ÷ three different paths as shown in p-V diagram.
è 2ø
(a) 627°C (b) 527°C (c) 427°C (d) 327°C Which one of the following is true?
y
3. A cylinder with a movable piston contains 3 moles of A C
hydrogen at standard temperature and pressure. p
The walls of the cylinder are made of heat insulator,
and the piston is insulated by having a pile of sand on
it/ By what factor does the pressure of the gas D B
increase, if the gas is compressed to half its original O
V
x
volume? [NCERT Exemplar)
(a) 1.40 (b) 1.60 (c) 2.64 (d) 1.94 (a) Work done is maximum along AB
(b) Work done is minimum along AB
4. p-V diagram of an ideal gas is as shown in figure. (c) Work done along ACB = work done along ADB
s (d)Work
Work done by the
done along gasis in
ADB the process
minimum.

C D 8. Figure shows a thermodynamic process on one mole


2p0
of a gas. How does the work done in the process
p changes with time?
p0 A
B y
B
p
V0 2V0 3V0
V
(a) 4 pV0 (b) 2p 0V0 (c) 3p 0V0 (d) p 0V0 A
O x
5. If for hydrogen C p - CV = m and for the nitrogen V
C p - CV = n, where C p , CV refer to specific heats per
(a) decreases continuously
unit mass respectively at constant pressure and
(b) increases continuously
constant volume, the relation between m and n is
(a) m = 14 n (b) n = 7 n (c) remains constant
(c) m = 7 n (d) n = 14 n (d) first increases and then decreases
Telegram @unacademyplusdiscounts

Thermodynamics 587

9. During an adiabatic process, the pressure p of a fixed 14. In the indicator diagram, net amount of work done
mass of an ideal gas changes by Dp and its volume V will be
changes byDV. If g = C p / CV , then DV / V is given by
Dp Dp
(a) - (b) -g p
p p 1 2
Dp Dp
(c) - (d)
gp g2 p
V
10. Figure shows four p-V diagrams. Which of these
(a) positive (b) zero (c) infinity (d) negative
curves represent isothermal and adiabatic process?
y 15. A cyclic process is shown in figure. Work done during
isobaric expansion is
p
A D
2 × 102 A B
B C
p

(Nm–2)
O x 102 D C
V
(a) D and C (b) A and C (c) A and B (d) B and D
O
1 2 3
11. A thermodynamic system is taken from state A to V (Vm–3)
state B along ACB and is brought back to A along
BDA as shown in figure. Net work done during one (a) 1600 J (b) 100 J (c) 400 J (d) 600 J
complete cycle is given by area 16. In a p-V diagram for an ideal gas (where p is along
p2 y-axis and V is along x-axis), the value of the ratio
C
B “slope of adiabatic curve/slope of the isothermal
p curve” at any point will be (where symbols have their
p1
D usual meanings).
A
(a) 1 (b) 2
O
(c) C p / CV (d) CV / C p
X
V1 V2
V 17. An ideal gas undergoes cyclic process ABCDA as
(a) ACBDA (b) ACB p2 p1 A shown in given p-V diagram. The amount of work
(c) AVV (D) BD Ap1p2 B done by the gas is [NCERT Exemplar]
1 2 BDA
p
12. A gas at pressure p is adiabatically compressed so
that its density becomes twice that of initial value. D
2p0 C
Given that g = C p /CV = 7/5, what will be the final
pressure of the gas? p0 B
7 A
(a) 2p (b) p
5
V
(c) 2.63 p (d) p V0 3V0

13. Two isothermal curves are shown in figure at (a) 6 p 0V0 (b) - 2p 0V0
temperature T1 and T2 . Which of the following (c) + 2 p 0V0 (d) + 4 p 0V0
relations is correct?
18. In the following p-V diagram figure two adiabates cut
two isothermals at T1 and T2 . The value of Vb/ Vc is
p
A B
p T1
T1
T2 D C T2
V Va Vd Vb Vc
(a) T1 > T2 (b) T1 < T2 V
1
(c) T1 = T2 (d) T1 = T2 (a) = Va / Vd (b) < Va / Vd
2 (c) > Va / Vd (d) Cannot say
Telegram @unacademyplusdiscounts

588 JEE Main Physics

19. In figure a certain mass of gas traces three paths 1, 2, 23. An ideal monoatomic gas is taken around the cycle
3 from state A to state B. If work done by the gas ABCD as shown in p versus V diagram. Work done
along three paths are W1, W2 , W3 respectively, then during the cycle is
y
B (V, 2p) (2V, 2p)
D C
p 1
p
2
3

A B
A (2V, p)
(V, p)
x
V V
(a) W1 < W2 < W3 (b) W1 = W2 = W3 (a) pV (b) 0.5 pV
(c) W1 > W2 > W3 (d) Cannot say (c) 2 pV (d) 3 pV

20. Work done by the system in closed path ABCA, is 24. By what percentage should the pressure of the given
mass of gas be increased so to decrease its volume by
y
10% at a constant temperature?
p2 A (a) 5% (b) 7.2%
p (c) 12.5% (d) 11.1%

p1 25. One mole of an ideal gas expands adiabatically from


C an initial temperature T1 to a final temperature T2 .
B
O x The work done by the gas would be
V1 V2 (a) (C p - CV )(T1 - T2 ) (b) C p (T1 - T2 )
V
(c) CV (T1 - T2 ) (d) (C p - CV )(T1 + T2 )
(a) zero (b) (V1 - V2 )( p1 - p2 )
( p - p1 )(V2 - V1 ) ( p + p1 )(V2 - V1 )
26. A gas at pressure 6 × 105 Nm–2 and volume 1 m3 and
(c) 2 (d) 2 . its pressure falls to 4 × 105 Nm–2. When its volume is
2 2 3m3.Given that the indicator diagram is a straight
21. Figure shows four thermodynamic process to which a line, work done by the system is
gas sample may be subjected. The isobaric and (a) 6 × 105 J (b) 3 × 105 J (c) 4 × 105 J (d) 10 × 105 J
isothermal curves are 27. A thermodynamic system goes from state (i) (p, V) to
y (2 p, V) and (ii) (p,V) to (p, 2V). Work done in the two
cases is
IV
(a) zero, zero (b) zero, pV (c) pV, zero (d) pV, pV.
p
III
28. Consider p-V diagram for an p
ideal gas shown in figure 1
II Constant
I
p=
V
V 2
Out of the following diagrams
(a) IV
(figure), and III
which represents the (b) II and IV V
(c) I and III (d) II and III T-p diagram? [NCERT Exemplar]
22. In the indicator diagram, Ta , Tb, Tc , Td represents T T
temperatures of gas at A, B, C, D respectively. Which 2
2
of the following is correct relation?
(a) (b)
1 1
A
p B
p p
D T T
C

V (c) 2 1 (d) 1 2
(a) Ta = Tb = Tc = Td (b) Ta ¹ Tb ¹ Tc ¹ Td
(c) Ta = Tb and Tc = Td (d) None of these
p p
Telegram @unacademyplusdiscounts

Thermodynamics 589

29. The pressure inside a tyre is 4 atm at 27°C. If the tyre 37. In a thermodynamic process pressure of a fixed mass
bursts suddenly, new temperature will be (g = 7/5) of a gas is changed in such a manner that the gas
(a) 300 (4)7/2 (b) 300 (4)2/7 molecules gives out 20 J of heat and 10 J of work is
(c) 300 (2)7/2 (d) 300 (4)–2/7 done on the gas. If the internal energy of gas 40 J
30. A monoatomic ideal gas, initially at temperature T1 is then the final internal energy will be
(a) 30 J (b) 20 J (c) 60 J (d) 40 J
enclosed in a cylinder fitted with a frictionless piston.
The gas is allowed to expand adiabatically to a 38. In changing the state of a gas adiabatically from an
temperature T2 by releasing the piston suddenly. If equilibrium state A to another equilibrium state B,
L1, L2 are the lengths of the gas column before and an amount of work equal to 22.3 J is done on the
after expansion respectively, then T1/ T2 is given by system. If the gas is taken from state A to B via a
(a) ( L1 / L2 ) 2 /3 (b) ( L1 / L2 ) process in which the net heat absorbed by the system
(c) L1 / L2 (d) ( L2 / L1 ) 2 /3 is 9.35 cal, how much is the net work done by the
system in the latter case?
31. For adiabatic expansion of a perfect monoatomic gas, (a) 15.6 J (b) 11.2 J (c) 14.9 J (d) 16.9 J
when volume increases by 24%, what is the
percentage decrease in pressure?
39. During an isothermal expansion, a confined ideal gas
(a) 24% (b) 30% does –150 J of work against its surrounding. This
(c) 48% (d) 71% implies that
(a) 150 J of heat has been added of the gas
32. Starting with the same initial conditions, an ideal (b) 150 J heat has been removed from the gas
gas expands from volume V1 to V2 in three different (c) 300 J of heat has been added to the gas
ways. The work done by the gas is W1 if the process is (d) No heat is transferred because the process is isothermal
purely isothermal, W2 if purely isobaric and W3 if
purely adiabatic. Then, Laws of Thermodynamics and Internal
(a) W2 > W1 > W3
(b) W2 > W3 > W1 Energy
(c) W1 > W2 > W3 40. In a certain process, 400 cal of heat are supplied to a
(d) W1 > W3 > W2 system and at the same time 105 J of mechanical
33. A litre of dry air at STP is allowed to expand to a work was done on the system. The increase in its
volume of 3 L under adiabatic conditions. If g = 1.40, internal energy is
the work done is (31.4 = 4.6555) (a) 20 cal (b) 303 cal
(a) 48 J (b) 60.7 J (c) 404 cal (d) 425 cal
(c) 90.5 J (d) 100.8 J 41. If the heat of 110 J is added to a gaseous system and
34. As shown in figure three p-V it acquires internal energy of 40 J, then the amount
I
diagrams. In which case, work of internal work done is
done is minimum II (a) 40 J (b) 70 J (c) 150 J (d) 110 J
p
(a) I (b) II III 42. A gas is expanded from volume V0 to 2 V0 under three
(c) III (d) Cannot say different processes shown in figure.
V Process 1 is isobaric process, process 2 is isothermal
35. An ideal gas is heated at constant pressure and and process 3 is adiabatic. Let DU1, DU2 and DU3 be
absorbs amount of heat Q. If the adiabatic exponent the change in internal energy of the gas in these
is g then the fraction of heat absorbed in raising the three processes. Then,
internal energy and performing the work, is
1 1
(a) 1 - (b) 1 + p0
1
g g
2 2 p 2
(c) 1 - (d) 1 +
g g 3

36. A thermodynamical system is changed from state


( p1V1) to ( p2 V2 ) by two different process. The quantity V0 V 2V2
which will remain same is (a) DU1 > DU2 > DU3 (b) DU1 < DU2 < DU3
(a) DQ (b) DW
(c) DU2 < DU1 > DU3 (d) DU2 < DU3 < DU1
(c) DQ + W (d) DQ - DW
Telegram @unacademyplusdiscounts

590 JEE Main Physics

43. In a thermodynamic process, pressure of a fixed mass 48. If amount of heat given to a system be 50 J and work
of gas is changed in such a manner that the gas done on the system be 15 J, then change in internal
releases 20 J of heat and 8 J of work is done on the energy of the system is
gas. If internal energy of the gas was 30 J, then the (a) 35 J (b) 50 J (c) 65 J (d) 15 J
final internal energy will be 49. In an isothermal change of an ideal gas, DU = 0. The
(a) 42 J (b) 18 J (c) 12 J (d) 60 J
change in heat energy DQ is equal to
44. What is the nature of change in internal energy in (a) 0.5 W (b) W (c) 1.5 W (d) 2 W
the following three thermodynamic processes shown 50. 5 mole of an ideal gas with (g = 7/5) initially at STP
in figure? are compressed adiabatically so that its temperature
becomes 400°C. The increase in the internal energy
p p of gas in kJ is
(a) 21.55 (b) 41.55
(c) 65.55 (d) 50.55

V V
51. Which one of the following statements is true in
(I) (ii) respect of usual quantities represented by DQ, DU
and DW?
(a) DU and DW are path dependent
p (b) DQ and DU are path dependent
(c) DU does not depend on path
(d) DQ does not depend upon path

(iii)
V 52. When an ideal monoatomic gas is heated at constant
pressure, fraction of heat energy supplied which
(a) DU is positive in all the three cases increases the internal energy of gas is
(b) DU is negative in all the three cases (a) 2/5 (b) 3/5
(c) DU is positive for (i), negative for (ii), zero for (iii) (c) 3/7 (d) 3/4
(d) DU = 0, in all the cases
53. Consider two containers A and B containing identical
45. Figure shows two processes a and b for a given gases at the same pressure, volume and temperature.
sample of a gas, if DQ1, DQ2 are the amounts of heat The gas in container A is compressed to half of its
absorbed by the system in the two cases and DU1, DU2 original volume isothermally while the gas in
are changes in internal energies respectively, then container B is compressed to half of its original value
y adiabatically. The ratio of final pressure of gas in B to
that of gas in A is [NCERT Exemplar]
p a g -1
æ 1ö
(a) 2g - 1 (b) ç ÷
è 2ø
b
2 2
æ 1 ö æ 1 ö
(c) ç ÷ (d) ç ÷
O x è1 - g ø è g - 1ø
V
(a) DQ1 = DQ2 ; DU1 = DU2 (b) DQ1 > DQ2 ; DU1 > DU2 54. Three copper blocks of masses M1, M2 and M 3 kg
(c) DQ1 < DQ2 ; DU1 < DU2 (d) DQ1 > DQ2 ; DU1 = DU2 . respectively are brought into thermal contact till
they reach equilibrium. Before contact, they were at
46. 1 cm3 of water at its boiling point absorbs T1, T2 , T3 ( T1 > T2 > T3). Assuming there is no heat
540 cal of heat to become steam with a volume = loss to the surroundings, the equilibrium temprature
1.013 ´ 105 Nm–2 and the mechanical equivalent of T is (s is specific heat of copper) [NCERT Exemplar]
heat = 4.19 Jcal–1. The energy spend in this process T1 + T2 + T3
in overcoming intermolecular forces is (a) T =
3
(a) 540 cal (b) 40 cal (c) 500 cal (d) zero M1T1 + M2T2 + M3T3
(b) T =
47. During adiabatic expansion of 10 moles of a gas, the M1 + M2 + M3
internal energy decreases by 50 J. Work done during M1T1 + M2T2 + M3T3
(c) T =
the process is 3( M1 + M2 + M3 )
(a) + 50 J (b) – 50 J M1T1s + M2T2 s + M3T3s
(d) T =
(c) zero (d) Cannot say M1 + M2 + M3
Telegram @unacademyplusdiscounts

Thermodynamics 591

55. In which of processess does the internal energy of the 64. In a refrigerator, the low temperature coil of
system remains constant evaporator is at – 23°C and the compressed gas in the
(a) isobaric (b) isothermal condenser has a temperature of 77°C. How much
(c) adiabatic (d) isochoric electrical energy is spent in freezing 1 kg of water
56. A thermodynamic system is taken through the cycle already at 0°C ?
(a) 134400 J (b) 1344 J
PQRSP process. The net work done by the system is
(c) 80000 J (d) 3200 J
p
S R 65. A refrigerator absorbs 2000 cal of heat from ice trays.
200 kPa
If the coefficient of performance is 4, then work done
by the motor is
100 kPa Q
(a) 2100 J (b) 4200 J
P (c) 8400 J (d) 500 J
V
100 cc 300 cc 66. A Carnot engine has same efficiency between (i) 100 K
(a) 20 J (b) –20 J (c) 400 J (d) –374 J and 500 K , (ii) T K and 900 K. The value of T is
(a) 180 K (b) 90 K (c) 270 K (d) 360 K
Carnot Engine and Refrigerator 67. A refrigerator works between temperature of melting
ice and room temperatures (17°C). The amount of
57. The efficiency of a Carnot engine working between
energy in kWh that must be supplied to freeze 1 kg of
800 K and 500 K is
water at 0°C is
(a) 0.4 (b) 0.625 (c) 0.375 (d) 0.5
(a) 1.4 (b) 1.8
58. A Carnot engine whose sink is at 300 K has an (c) 0.058 (d) 2.5
efficiency of 40%. By how much should the
68. A Carnot engine has an efficiency of 1/6. When
temperature of source be increased so as to increase
temperature of sink is reduced by 62°C, its efficiency
its efficiency by 50% of original efficiency ?
is doubled. Temperature of source and sink are,
(a) 280 K (b) 275 K (c) 325 K (d) 250 K
(a) 99°C, 37°C (b) 124°C, 62°C
59. An ideal Carnot engine whose efficiency is 40% (c) 37°C, 99°C (d) 62°C, 124°C
receives heat at 500 K. If its efficiency were 50%, then
69. A Carnot engine whose low temperature reservoir
in take temperature for same exhaust temperature
is at 27°C has an efficiency 37.5%. The high
would be
temperature reservoir is at
(a) 700 K (b) 900 K
(a) 480°C (b) 327°C
(c) 800 K (d) 600 K
(c) 307°C (d) 207°C
60. A Carnot’s engine works between a source at a
70. The coefficient of performance of a refrigerator
temperature of 27°C and a sink at – 123°C. Its
working between 10°C and 20°C is
efficiency is
(a) 28.3 (b) 29.3
(a) 0.5 (b) 0.25
(c) 2 (d) Cannot be calculated
(c) 0.75 (d) 0.4
1
61. Four engines are working between the given 71. A reversible heat engine converts th of heat it absorbs
6
temperatures ranges given below. For which from source into work. When temperature of source is
temperature range the efficiency is maximum? 600 K, temperature at which heat exhausts is
(a) 100 K, 80 K (b) 40 K, 20 K (a) 500 K (b) 100 K (c) 0 K (d) 600 K
(c) 60 K, 40 K (d) 120 K, 100 K
72. A Carnot engine used first ideal monoatomic gas and
62. An engine has an efficiency of 1/3. The amount of then an ideal diatomic gas, if the source and sink
work this engine can perform per kilocalorie of heat temperatures are 411°C and 69°C, respectively and
input is the engine extracts 1000 J of heat from the source in
(a) 1400 cal (b) 700 cal
each cycle, then
(c) 700 J (d) 1400 J (a) area enclosed by the p-V diagram is 10 J
63. A Carnot engine works between 600 K and 300 K. In (b) heat energy rejected by engine is 1st case is 600 J while
each cycle of operation, the engine draws 1000 J of that in 2nd case in 113 J
heat energy from the source. The efficiency of the (c) area enclosed by the p-V diagram is 500 J
engine is (d) efficiencies of the engine in both the cases are in the ratio
(a) 50% (b) 70% (c) 20% (d) 80% 21 : 25
Telegram @unacademyplusdiscounts

592 JEE Main Physics

73. A Carnot engine whose source is at 400 K takes Specific Heat of Gases and Degrees of
200 cal of heat and rejects 150 cal to the sink. What is
the temperature of the sink?
Freedom
(a) 800 K (b) 400 K 83. For a gas the difference between the two specific heats
(c) 300 K (d) Cannot say is 4150 J/kg-K. What is the specific heats at constant
74. An ideal gas heat engine operates in Carnot cycle volume of gas it the ratio of the specific heat is 1.4?
(a) 8475 J/kg-K (b) 5186 J/kg-K
between 227°C and 127°C. It absorbs 6 × 104 cal of
(c) 1660 J/kg-K (d) 10375 J/kg-K
heat at higher temperature. Amount of heat
converted into work is 84. For a gas if the ratio of specific heats at constant
(a) 1.2 × 104 cal (b) 2.4 × 104 cal pressure and volume is g, then value of degree of
(c) 6 × 104 cal (d) 4.8 × 104 cal freedom is
3g -1 2
75. Two heat engines A and B have their sources at (a) (b)
2g -1 g -1
1000 K and 1100 K and their sinks are at 500 K and
9 25
400 K respectively. What is true about their (c) ( g - 1) (d) ( g - 1)
efficiencies? 2 2
(a) h A = h B (b) h A > h B 85. If 70 cal of heat is required to raise the temperature
(c) h A < h B (d) Cannot say of 2 moles of an ideal gas at constant pressure from
76. An engine takes compressed steam at 127°C and 30°C to 35°C, then the amount of heat required to
rejects it at 47°C. Efficiency of the engine is raise the temperature of same gas through same
(a) 60% (b) 35% range at constant volume is
(c) 20% (d) 40% (a) 50 cal (b) 70 cal (c) 60 cal (d) 65 cal

77. A Carnot engine has the same efficiency between 86. One mole of an ideal gas requires 207 J heat to raise
800 K to 500 K and x K to 600 K. The value of x is the temperature by 1 K, when heated at constant
(a) 100 K (b) 960 K pressure. If the same gas is heated at constant
(c) 846 K (d) 754 K volume to raise the temperature by the same range,
the heat required will be (Take R = 8.3 Jmol–1 K–1)
78. What is the temperature of source in Carnot cycle of (a) 215.3 J (b) 198.7 J
10% efficiency when heat exhausts at 270 K? (c) 207 J (d) None of these
(a) 400 K (b) 500 K
(c) 300 K (d) 600 K 87. One mole of a gas enclosed in a vessel is heated at
constant pressure 1 K. Work done by the gas is
79. Even Carnot engine cannot give 100% efficiency 1
(a) 1 J (b) J
because we cannot R
(a) prevent radiation (c) R J (d) None of these
(b) final ideal sources
(c) reach absolute zero temperature 88. One mole of a monoatomic gas is heated at a constant
(d) eliminate friction pressure of 1 atm from 0 K to 100 K. If the gas
constant R = 8.32 Jmol–1 K–1, the change in internal
80. A Carnot engine take energy of the gas is approximately
reservoir at 627°C and gives it to a sink at 27°C. The (a) 2.3 J (b) 46 J
work done by the engine is (c) 8.67 × 103 J (d) 1.25 × 103 J
(a) 4.2 ´ 106 J (b) 8.4 ´ 106 J
6 89. Which one of the following gases possesses the
(c) 16.8 ´ 10 J (d) zero
largest internal energy ?
81. A Carnot engine has the same efficiency between (a) 2 moles of helium occupying 1 m3 at 300 K
800 K to 500 K and x K to 600 K. The value of x is (b) 56 g of nitrogen at 107 N m –2 at 300 K
(a) 100 K (b) 960 K (c) 8 g of oxygen at 8 atm at 300 K
(c) 846 K (d) 754 K (d) 6 × 1026 molecules of argon occupying 40 m3 at 900 K

82. What is the value of sink temperature when 90. For the same rise in temperature of one mole of gas at
efficiency of engine is 100%? constant volume, heat required for a non linear
(a) 0 K (b) 300 K triatomic gas is K times that required for monoatomic
(c) 273 K (d) 400 K gas. The value of K is
(a) 1 (b) 0.5 (c) 2 (d) 2.5
Telegram @unacademyplusdiscounts

Thermodynamics 593

91. Value of two principal specific heats of a gas in cal (a) 5186 Jkg–1K–1 (b) 10375 Jkg–1K–1
(mol K)–1 determined by different students are given. (c) 1660 Jkg–1K–1 (d) 8475 Jkg–1K–1
Which is most reliable? 97. If the degrees of freedom of a gas molecule be f, then
(a) 5, 2 (b) 6, 5 (c) 7, 5 (d) 7, 4 the ratio of two specific heat C p / CV is given by
92. In the above question, if g = 1.5, the gas may 2 2 1 1
(a) +1 (b) 1 - (c) 1 + (d) 1 -
(a) monoatomic f f f f
(b) diatomic 98. During an adiabatic process the pressure of a gas is
(c) a mixture of monoatomic and diatomic gases found to be proportional to the cube of its absolute
(d) a mixture of diatomic and triatomic gases temperature. The ratio C p / CV for the gas is
3 4 5
93. Calculate change in internal energy when 5 mole of (a) (b) (c) 2 (d)
hydrogen is heated to 20°C from 10°C, specific heat of 2 3 3
hydrogen at constant pressure is 8 cal (mol°C)–1. 99. The adiabatic elasticity of hydrogen gas (g = 1.4) at
(a) 200 cal (b) 350 cal (c) 300 cal (d) 475 cal NTP is
94. A gas expands with temperature according to the (a) 1 ´ 105 N /m2 (b) 1 ´ 10 -8 N /m2
2/ 3 2
relation V = kT , Calculate work done when the (c) 1.4 N /m (d) 1.4 ´ 105 N /m2
temperature changes by 60 K.
(a) 10 R (b) 30 R (c) 40 R (d) 20 R 100. A system is taken through a cyclic process
represented by a circle as shown. The heat absorbed
95. A gaseous mixture contains equal number of by the system is
hydrogen and nitrogen molecules. Specific heat V (in) cc
measurements on this mixture at temperature below
150 K would indicate the value of g = C p/CV for the 40
mixture as 30
(a) 3/2 (b) 4/3 (c) 5/3 (d) 7/5
20
96. For a gas, the difference between the two principal
specific heats is 4150 Jkg–1K–1. What is the specific 50 100 150 200 p (in kPa)
heat of the gas at constant volume if, the ratio of p
specific heat is 1.4? (a) p ´ 103 J (b) J (c) 4 p ´ 102 J (d) p J
2

Round II (Mixed Bag)


Only One Correct Option 5. An ideal gas heat engine is operating between 227°C
and 127°C. It absorbs 104 J of heat at the higher
1. A given system undergoes a change in which the
temperature. The amount of heat converted into
work done by the system equals the decrease in its
work is
internal energy. The system must have undergone an
(a) 2000 J (b) 4000 J
(a) isothermal change (b) adiabatic change
(c) 8000 J (d) 5600 J
(c) isobaric change (d) isochoric change
2. The change in internal energy, when a gas is cooled 6. In figure two indicator diagrams are shown. If the
amounts of work done in the two cases are W1 and W2
from 927°C to 27°C
(a) 300% (b) 400% (c) 200% (d) 100% respectively, then

3. When a small amount of heat DQ is added to an A A


enclosed gas, then increase in internal energy and p p
external work done are related as
(a) mCV DT = DQ + pDV (b) DQ = mCV DT + pDV
(c) mCV = DQ + pDV (d) DQ = mC p DT + pDV B B
V V
4. Value of adiabatic bulk modulus of elasticity of
helium at NTP is (a) W1 = W2 (b) W1 > W2
(a) 1.01 × 105 Nm–2 (b) 1.01 × 10–5 Nm–2
(c) W1 < W2 (d) Cannot say
(c) 1.69 × 105 Nm–2 (d) 1.69 × 10–5 Nm–2
Telegram @unacademyplusdiscounts

594 JEE Main Physics

7. At constant temperature, the volume of a gas is to be 1


14. A mass of dry air at NTP is compressed to
th of its
decreased by 4%. The pressure must be increased by 20
(a) 4% (b) 4.16% original volume suddenly. If g = 1.4, the final
(c) 8% (d) 3.86% pressure would be
(a) 20 atm (b) 66.28 atm
8. One mole of an ideal monoatomic gas is heated at a (c) 30 atm (d) 150 atm
constant pressure of 1 atm from 0°C to 100°C. Work
done by the gas is 15. A gas under constant pressure of 4.5 ´ 105 Pa when
(a) 8.31 × 103
J (b) 8.31 × 10–3
J subjected to 800 kJ of heat changes the volume from
(c) 8.31 × 10–2 J (d) 8.31 × 102 J 0.5 m3 to 2.0 m3. The change in the internal energy of
the gas is
9. 500 J of heat energy is removed from 4 moles of a (a) 6.75 × 105 J (b) 5.25 × 105 J
monoatomic ideal gas at constant volume. The (c) 3.25 × 105 J (d) 1.25 × 105 J
temperature drops by
(a) 40°C (b) 30°C 16. A perfect gas goes from state A to state B by
(c) 10°C (d) 0°C absorbing 8 ´ 105 J of heat and doing 6.5 ´ 105 J of
external work. It is now transferred between the
10. The ratio of specific heat of a gas at constant pressure same two states in another process in which it
to that at constant volume is g. The change in
absorbs 105 J of heat. In the second process,
internal energy of one mole of gas when volume (a) work done on gas is 105 J
change from V to 2 V at constant pressure p is (b) work done on gas is –0.5 × 10 5 J
(a) R/( g - 1) (b) pV
(c) work done by gas is 105 J
g pV
(c) pV /( g - 1) (d) (d) work done by gas is 0.5 × 105 J
g -1
17. A Carnot engine is made to work between 200°C and
11. In an adiabatic change, the pressure and 0°C first and then between 0°C to –200°C. The ratio
temperature of a monoatomic gas are related as of efficiencies of the engine in the two cases is
p µ T - c where c equals (a) 1 : 2 (b) 1 : 1
2 5 3 5
(a) (b) (c) (d) (c) 1.73 : 1 (d) 1 : 1.73
5 2 5 3
18. For an engine operating between t1°C and t2 °C, the
12. If AB is an isothermal, BC is an isochoric and AC is efficiency will be
an adiabatic curve; which of the graph correctly t1 t2
represents them in figure (a) (b) 1 -
t2 t1
p p t1 - t2 t1 - t2
A
(c) (d)
t2 t1 + 273
B
(a) (b) B 19. p-V plots for two gases during
C C adiabatic processes are shown in
figure. Plots 1 and 2 should p
V V correspond respectively to 1
p p
(a) He and O2 2
A A B (b) O2 and He
B V
(c) He and Ar
(c) (d)
(d) O2 and N2
C C
20. An ideal gas is taken through 2 C B
V V the cycle A ® B ® C ® A, as
shown in figure. If the net V A
13. Ten moles of an ideal gas at constant temperature heat supplied to the gas in 1
(m3)

600 K is compressed from 100 L to 10 L. The work


cycle is 5J, work done by the
done in the process is
gas in the process C ® A
(a) 4.11 × 104 J
0 5 10
(b) – 4.11 × 104 J (a) –5 J p(Pa)
(c) 11.4 × 104 J (b) –10 J
(d) – 11.4 × 104 J (c) 15 J
(d) –20 J
Telegram @unacademyplusdiscounts

Thermodynamics 595

21. 200 cal of heat is given to a heat engine so that it 30. Certain amount of an ideal gas is contained in a
reject 150 cal of heat. If source temperature is 400 K, closed vessel. The vessel is moving with a constant
then the sink temperature is velocity v. The rise in temperature of the gas when
(a) 300 K (b) 200 K the vessel is suddenly stopped is (M is molecular
(c) 100 K (d) 50 K mass, g = C p / CV )
Mv2 ( g - 1) Mv2 ( g + 1)
22. Calculate change in internal energy of a system (a) (b)
which has absorbed 2 kcal of heat and done 500 J of 2R 2R
work Mv2 Mv2
(c) (d)
(a) 7900 J (b) 8900 J 2R y 2 R ( g + 1)
(c) 6400 J (d) 5400 J
31. The specific heat of hydrogen gas at constant
23. During the adiabatic expansion of 2 moles of a gas, pressure is C p = 3.4 ´ 103 cal / kg° C and at constant
change in internal energy was found to be equal to volume is CV = 2.4 ´ 103 cal / kg° C. If one kilogram
100 J. Work done in the process will be equal to hydrogen gas is heated from 10°C to 20°C at constant
(a) 100 J (b) 50 J
pressure, the exterted work done on the gas to
(c) 200 J (d) 400 J
maintain it at constant pressure is
24. A gas undergoes a process in which its pressure p and (a) 105 cal (b) 10 4 cal
volume V are related as Vp = constant. The bulk
n
(c) 103 cal (d) 5 ´ 103 cal
modulus for the gas in this process is
32. An ideal gas is made to go through a cyclic thermo-
(a) np (b) p1/n
dynamical process in four steps. The amount of heat
p
(c) (d) p n involved are Q1 = 600 J, Q2 = -400 J, QB = -300 J and
n Q4 = 200 J respectively. The corresponding work
25. For a monoatomic gas, work done at constant involved are W1 = 300 J,W2 = -200 J,W3 = -150 J and
pressure is W. The heat supplied at constant volume W4 . What is the value of W4 ?
for the same rise in temperature of the gas is (a) –50 J (b) 100 J (c) 150 J (d) 50 J
(a) W /2 (b) 3 W / 2 33. One mole of an ideal monoatomic gas is heated at a
(c) 5 W / 2 (d) W constant pressure of one atmosphere from 0°C to
26. An ideal gas expands isothermally from a volume V1 100°C. Then the change in the internal energy is
to V2 and then compressed to original volume V1 (a) 6.56 J (b) 8.32 ´ 102 J
adiabatically. Initial pressure is p1 and final (c) 12.48 ´ 102 J (d) 20.80 J
pressure is p3. Total work done is W. Then, 34. If the ratio of specific heat of gas at constant pressure
(a) p3 > p1; W > 0 (b) p3 < p1; W < 0
to that at constant volume is g. The change in
(c) p3 > p1; W < 0 (d) p3 = p1; W = 0
internal energy of a mass of gas when the volume
27. Temperature of an ideal gas is 300 K. The change in changes from V to 2 V at constant pressure p is
temperature of the gas when its volume changes from (a)
R
(b) pV
V to 2 V in the process p = aV (here a is a positive ( g - 1)
constant)
pV is g pV
(c) (d)
(a) 900 K (b) 1200 K ( g - 1) ( g - 1)
(c) 600 K (d) 300 K
35. When the ideal monoatomic gas is heated at constant
28. The specific heats of an ideal gas at constant pressure fraction of heat energy supplied which
pressure and constant volume are 525 J (kg°C)–1 and
increases the internal energy of gas is
315 J (kg°C )–1 respectively. Its density at NTP is 2 3 3 3
(a) 0.64 kgm–3 (b) 1.20 kgm–3 (a) (b) (c) (d)
5 5 7 4
(c) 1.75 kgm–3 (d) 2.62 kgm–3
36. A steam engine delivers 5.4 ´ 108 J of work per
29. Pressure p, volume V and temperature T of a certain
minute and services 3.6 ´ 109 J of heat per minute
material are related by p = aT2 / V , where a is
from the boiler. What is the efficiency of the engine?
constant. Work done by the material when
How much heat is wasted per minute?
temperature changes from T0 to 2 T0 and pressure
[NCERT Exemplar]
remains constant is 8 8
(a) 1.1 ´ 10 J/min (b) 2.1 ´ 10 J/min
(a) 3 aT20 (b) 5 aT20
3 (c) 1.9 ´ 10 9 J/min (d) 3.1 ´ 10 9 J/min
(c) aT20 (d) 7 aT20
2
Telegram @unacademyplusdiscounts

596 JEE Main Physics

37. In an isothermal reversible expansion if the volume 45. Two spheres of the same material have radii 1 m and
of 96 g of oxygen at 27°C in increased from 70 L of 4 m and temperatures 4000 K and 2000 K
140 L, then the work done by the gas will be respectively. The ratio of the energy radiated per
(a) 300 R log10 2 second by the first sphere to that by the second is
(b) 81 R log e 2 (a) 16:1 (b) 4:1
(c) 900 R log10 2 (c) 1:1 (d) 1:9
(d) 2.3 ´ 900 R log10 2
46. Ideal gas undergoes an adiabatic change in its state
38. One mole of O2 gas having a volume equal to 22.4 L at from ( p1V1T1) to ( p2 V2 T2 ). The work done (W ) in the
0°C and 1 atmospheric pressure is compressed process is (m = number of moles C p and CV are molar
isothermally so that its volume reduces to 11.2 L. The specific heats of gas)
work done in this process is (a) W = m (T1 - T2 ) C p (b) W = m (T1 - T2 ) CV
(a) 1672.5 J (b) 1728 J (c) W = m (T1 + T2 ) C p (d) W = m (T1 + T2 ) CV
(c) –1728 J (d) –1572.5 J
47. Compressed air in the tube of a wheel of a cycle at
39. A refrigerator is to remove heat from the eatable kept normal temperature suddenly starts coming out from
inside at 90°C. Calculate the coefficient of a puncture. The air inside [NCERT]
performance, if the room temperature is 36°C. [NCERT] (a) starts becoming hotter
(a) 10.4 (b) 11.5 (b) remains at the same temperature
(c) 9.8 (d) None of these (c) starts becoming cooler
(d) may become hotter or cooler depending upon the
40. 540 cal of heat converts 1 cubic centimeter of water at amount of water vapour present
100°C into 1671 cubic centimeter of steam at 100°C
at a pressure of one atmosphere. Then, the work done 48. How many times a diatomic gas should be expanded
against the atmospheric pressure is nearly adiabatically so as to reduce the root mean square
(a) 540 cal (b) 40 cal velocity to half
(c) 200 cal (d) 500 cal (a) 64 (b) 32
(c) 16 (d) 8
41. The volume of an ideal gas is 1 litre and its pressure
is equal to 72 cm of mercury column. The volume of 49. At NTP one mole of diatomic gas is compressed
gas is made 900 cm 3 by compressing it isothermally. adiabatically to half of its volume, g = 1.41. The work
The stress of the gas will be done on gas will be
(a) 8 cm (mercury) (b) 7 cm (mercury) (a) 1280 J (b) 1610 J
(c) 6 cm (mercury) (d) 4 m (mercury) (c) 1815 J (d) 2025 J

42. 1 mm 3 of a gas is compressed at 1 atmospheric 50. Two moles of an ideal monoatomic gas at 27°C
pressure and from temperature 27°C to 627°C. What occupies a volume of V. If the gas is expanded
is the final pressure under adiabatic condition (g for adiabatically to the volume 2 V, then the work done
æ 5 ö
the gas = 1.5)? by the gas will be ç g = , R = 8.31 J/ mol K ÷
(a) 27 ´ 105 N /m2 (b) 80 ´ 105 N /m2
è 3 ø
(a) + 2767.23 J (b) 2627.23 J
(c) 36 ´ 105 N /m2 (d) 56 ´ 105 N /m2
(c) 2500 J (d) –2500 J
43. The pressure and density of a diatomic gas ( g = 7 / 5) 51. 5.6 L of helium gas at STP is adiabatically compressed

change adiabatically from ( p, d) to ( p¢, d¢) if = 32, to 0.7 L. Taking the initial temperature to be T1, the
d
p¢ work done in the process is
then should be 9 3 15 9
p (a) RT1 (b) RT1 (c) RT1 (d) RT1
8 2 8 2
(a) 1/128 (b) 32
(c) 128 (d) None of these 52. In changing the state of a gas adiabatically from an
equillibrium state A to another equillibrium state B,
44. An ideal gas at 27°C is compressed adiabatically to an amount of work equal to 22.3 J is done on the
8 5
of its original volume. If g = , then the rise of system. If the gas is taken from state A to B via a
27 3
temperature is process in which the net heat absorbed by the system
(a) 450 K (b) 375 K is 9.35 cal the net work done by the system in latter
(c) 225 K (d) 405 K case will be
(a) 5.9 J (b) 16.9 J (c) 9.3 J (d) 4.6 J
Telegram @unacademyplusdiscounts

Thermodynamics 597

53. If the temperature of 10 mole of ideal gas is changed 3 to 1 is adiabatic


from 0°C to 100°C at constant pressure of 2 atm, then Such a process does not exist because
the work done in the process is (R = 8.3 J/ mol-K)
p
(a) 8.3 ´ 10 –3 J (b) 8.3 ´ 10 –2 J
1
(c) 8.3 ´ 10 4 J (d) 8.3 ´ 103 J 2

54. 2 kg of water is converted into steam by boiling at


atmospheric pressure. The volume changes from 3
2 ´ 10-3 m 3 to 3.34 m 3. The work done by the system V
is about
(a) heat is completely converted to mechanical energy in
(a) –340 kJ (b) –170 kJ
such a process, which is not possible.
(c) 170 kJ (d) 340 kJ
(b) mechanical energy is completely converted to heat in this
process, which is not possible.
More Than One Correct Option (c) curves representing two adiabatic processes don’t
intersect.
55. Figure shows the p-V diagram of an ideal gas
(d) curves representing an adiabatic process and an
undergoing a change of state from A to B. Four
isothermal process don’t intersect.
different parts I, II, III and IV as shown in the
figure may lead to the same change of state. 58. An ideal gas is taken from the state A (p, V) to the
state B (p/2, 2 V) along a straight line path as shown
[NCERT Exemplar]
in figure. Select the correct statement from the
following.
A
p p
p
A
B
C
p/2
B
V
(a) DQA ® B = negative (b) DUC ® A = negative
V V 2V V0

(c) DWCAB = negative (d) DUB ® C = negative (a) Work done by the gas in going from A to B exceeds the
work done in going from A to B under isothermal
56. Figure shows the p-V diagram of an ideal gas conditions
undergoing a change of state from A to B. Four (b) In the T-V diagram, part AB would become a parabola,
different paths I, II, III and IV as shown in the figure (c) In the p-T diagram, path AB would be part of hyperbola
may lead to the same changes of state. (d) In going from A to B, the temperature T of gas first
[NCERT Exemplar]
increases to a maximum value 1 and then decreases
p
I 59. Consider a heat engine as shown in figure. Q1 and Q2
IV are heat added to heat bath T1 and heat T2 taken from
A II in one cycle of engine. W is the mechanical work done
on the engine. If W > 0, then possibilities are
B
III [NCERT Exemplar]
V T1
(a) change in internal energy is same in IV and III cases, but
not in I and II Q1
W
(b) Change in internal energy is same in all the four cases
(c) Work done is maximum in case I Q2
(d) Work done is minimum in case II T2
57. Consider a cycle followed by an engine (figure) (a) Q1 > Q2 > 0
[NCERT Exemplar] (b) Q2 > Q1 > 0
(c) Q2 < Q1 < 0
1 to 2 is isothermal
(d) Q1 < O, Q2 > 0
2 to 3 is adiabatic
Telegram @unacademyplusdiscounts

598 JEE Main Physics

Comprehension Based Questions (a) If both Assertion and Reason are true and the Reason
is correct explanation of the Assertion
Passage I (b) If both Assertion and Reason are true but Reason is
not correct explanation of the Assertion
The efficiency of a Carnot engine working between (c) If Assertion is true but Reason is false
source temperature T1 and sink temperature T2 is
T (d) If Assertion is false but the Reason is true
h = 1 - 2 . The efficiency cannot be 100% as we
T1 65. Assertion Work done by a gas in isothermal expansion
cannot maintain T2 = 0. Coefficient of performance of is more than the work done by the gas in the same
a refrigerator working between the same two expansion, adiabatically.
temperature is Reason Temperature remains constant in isothermal
T2 1 -h expansion, and not in adiabatic expansion.
=
T1 - T2 h 66. Assertion A reversible engine working between 127°C
60. The efficiency of a Carnot engine working between and 227°C cannot have efficiency more than 20%.
T
27°C and – 73°C is Reason Under ideal conditions h = 1 - 2
T1
(a) 100% (b) 60% (c) 33% (d) zero
61. The efficiency of a Carnot engine is 40%. If 67. Assertion Reversible systems are difficult to find in
temperature of sink is 27°C, what is the source real world.
temperature? ReasonMost processes are dissipative in nature.
(a) 300 K (b) 400 K (c) 600 K (d) 500 K 68. Assertion A Carnot engine working between 100 K
and 400 K has an efficiency of 75%.
62. The efficiency of a Carnot engine is 60%. If T
temperature of source is 127°C. The sink must be Reason It follows from h = 1 - 2
T1
maintained at
(a) 113 K (b) – 113°C (c) 113°C (d) –113 K 69. Assertion An adiabatic process is an isotropic process.
DQ
Reason DS = = 0 \ DQ = 0
Passage II T
which represents an adiabatic process.
The changes in pressure and volume of a gas when
heat content of the gas remains constant are called 70. Assertion It is not possible for a system unaided by an
adiabatic changes. The equation of such changes is external agency to transfer heat from a body at a
pV g = constant. The changes must be sudden and the lower temperature to another at a higher
container must be perfectly insulting to disallow any temperature.
exchange of heat with the surroundings. In such Reason It is not possible to violate the second law of
changes, dQ = 0, then as per first law of thermodynamics.
thermodynamics, dQ = dU + W = 0. dU = - dW . 71. Assertion Internal energy of an ideal gas depends only
63. A gas in a container is compressed suddenly. Its on temperature and not on volume.
temperature would Reason Temperature is more important than volume.
(a) increase 72. Assertion Change of state is an example of isothermal
(b) decrease process.
(c) stay constant Reason Change of state from solid to liquid occurs
(d) change depending upon surrounding temperature. only at melting point of solid and change of state from
64. The tyre of a bicycle bursts suddenly. The changes in liquid to gas occurs only at boiling point of liquid.
pressure and volume of air are Thus, there is no change of temperature during
(a) isothermal (b) adiabatic change of state.
(c) isobaric (d) isochoric 73. Assertion Efficiency of a Carnot engine decreases with
decrease in temperature difference between the
Assertion and Reason source and the sink.
T2 T1 - T2
Reason h = 1 - =
Direction Question No. 65 to 74 are Assertion-Reason type. T1 T1
Each of these contains two Statements: Statement I (Assertion),
Statement II (Reason). Each of these questions also has four 74. Assertion First law of thermodynamics is a
alternative choice, only one of which is correct. You have to re-statement of the principle of conservation of
select the correct choices from the codes (a), (b), (c) and (d) given energy.
below Reason Energy is something fundamental.
Telegram @unacademyplusdiscounts

Thermodynamics 599

Previous Years’ Questions


Passage
B (3 p, 4V)
Two moles of helium gas are taken over the cycle
p
ABCDA, as shown in p-T diagram.
y
A
2 × 105 B B (p, V) B (p, 4V)
p Pa
V

1× 105
D
C (a) 2 pV (b) 3 pV
x (c) 6 pV (d) zero
300 500
T(K) 81. The p-V diagram of a gas undergoing a cyclic process
(ABCDA) is shown in the figure, where p is in units of
75. Assuming the gas to be ideal, the work done on the Nm–2 and V in cm3. Identify the incorrect statement.
gas in taking it from A to B is [AIEEE 2009] [Kerala CET 2008]
(a) 300 R (b) 400 R y
(c) 500 R (d) 200 R 2 × 105 A B
p(Nm–2)
76. The work done on the gas in taking it from D to A is
[AIEEE 2009]
(a) 415.9 R (b) – 690 R 1 × 105 C
D
(c) + 690 R (d) – 414 R 2.0 4.0
V(cm3)
77. Which of the following p-V diagrams best represents
an isothermal process? [UP SEE 2009] (a) 0.4 J of work is done by the gas from A to B
(b) 0.2 J of work is done on the gas from C to D
(c) Now work is done by the gas from B to C
p p (d) Work is done by the gas in going from B to C and on the
gas from D to A
(a) (b)
82. The work of 146 kJ is performed in order to compress
one kilomole of a gas adiabatically and in this process
V V the temperature of the gas increases by 7°C. The gas
is (R = 8.3 J mol–1 K–1) [UP SEE 2007]
(a) diatomic
p p
(b) triatomic
(c) (d) (c) a mixture of monoatomic and diatomic
(d) monoatomic

V V 83. A gas is taken through a number of thermodynamic


states. What happen to its specific heat?
78. An ideal heat engine exhausting heat at 27°C is to [BVP Engg. 2006]
have 25% efficiency. It must take heat at [UP SEE 2008] (a) It is always constant
(a) 127ºC (b) 227ºC (b) It increases
(c) 327ºC (d) None of these (c) It decreases
79. A heat engine has an efficiency h. Temperatures of (d) It can have any value depending upon process of heat
absorbed or evolved
source and sink are each decreased by 100 K. The
efficiency of the engine [Karnataka CET 2008] 84. Six moles of O2 gas is heated from 20°C to 35°C at
(a) increases (b) decreases constant volume. If specific heat capacity at constant
(c) remains constant (d) becomes 1 pressure is 8 cal (mol-K)–1 and R = 8.31 J(mol-K)–1,
what is change in internal energy of gas? [UP SEE 2006]
80. An ideal monoatomic gas is taken through a cyclic
(a) 180 cal (b) 300 cal
process as shown in p-V diagram, Work done per
(c) 360 cal (d) 540 cal
cycle is [Karnataka CET 2008]
Telegram @unacademyplusdiscounts

600 JEE Main Physics

85. In Carnot engine efficiency is 40% at hot reservoir T


temperature T. For efficiency 50%, what will be 2T0
temperature of hot reservoir? [UP SEE 2006]
T 2T
(a) (b)
5 5
6T T0
(c) 6 T (d)
5 S0 2S0
S

86. During an adiabatic process, the pressure of a gas is 2 1


(a) (b)
found to be proportional to cube of its absolute 3 3
Cp 1 1
temperature. Then ratio of g = for the gas is (c) (d)
CV 4 2
[BVP Engg. 2006]
92. A system goes from A to B via two process I and II as
(a) 4/3 (b) 5/3 shown in figure. If DU1 and DU2 are the changes in
(c) 3/2 (d) 2 internal energies in the processes I and II
87. In adiabatic expansion of a gas [BVP Engg. 2005] respectively, then [AIEEE 2005]
(a) its pressure increases p
(b) its temperature falls
II
(c) its density increases
(d) its thermal energy increases A B

88. A gas at 27°C has a volume V and pressure p. On I


heating, its pressure is doubled and volume becomes V
three times. The resulting temperature of the gas
(a) DU2 < DU1
will be [BVP Engg. 2005]
(b) DU2 > DU1
(a) 1800ºC (b) 162ºC
(c) relation between DU1 and DU2 cannot be determined
(c) 1527ºC (d) 600ºC
(d) DU1 = DU2
pV
89. The value of for one mole of an ideal gas is nearly 93. The above p-V diagram represents the
T
thermodynamic cycle of an engine, operating with an
equal to [Kerala CET 2005]
ideal monoatomic gas. The amount of heat, extracted
(a) 2 J mol–1 K–1 (b) 8.3 cal mol–1 K–1
from the source in a single cycle is [JEE Main 2013]
(c) 4.2 J mol–1 K–1 (d) 2 cal mol–1 K–1
90. A rigid container with thermally insulated walls
2T0 4T0
contains a coil of resistance 100 W carrying current 2p0
1 A. Change in internal energy after 5 min will be
p0
[IIT JEE 2005]
2T0
(a) 0 kJ (b) 10 kJ
(c) 20 kJ (d) 30 kJ V0 2V0
91. The temperature-energy diagram of a reversible æ13 ö
(a) p 0V0 (b) ç ÷ p 0V0
engine cycle is given in figure. Its efficiency is è2ø
[AIEEE 2005] æ11ö
(c) ç ÷ p 0V0 (d) 4 p 0V0
è2ø
Telegram @unacademyplusdiscounts

Answers
Round I
1. (d) 2. (d) 3. (c) 4. (c) 5. (a) 6. (a) 7. (d) 8. (b) 9. (c) 10. (a)
11. (a) 12. (c) 13. (a) 14. (a) 15. (c) 16. (c) 17. (b) 18. (a) 19. (c) 20. (c)
21. (a) 22. (c) 23. (a) 24. (d) 25. (c) 26. (d) 27. (b) 28. (c) 29. (d) 30. (d)
31. (b) 32. (a) 33. (c) 34. (c) 35. (a) 36. (d) 37. (a) 38. (d) 39. (b) 40. (d)
41. (b) 42. (a) 43. (b) 44. (d) 45. (d) 46. (c) 47. (a) 48. (c) 49. (b) 50. (b)
51. (c) 52. (b) 53. (a) 54. (b) 55. (b) 56. (b) 57. (c) 58. (d) 59. (d) 60. (a)
61. (b) 62. (d) 63. (a) 64. (a) 65. (a) 66. (a) 67. (c) 68. (a) 69. (d) 70. (a)
71. (a) 72. (c) 73. (c) 74. (a) 75. (c) 76. (c) 77. (b) 78. (c) 79. (c) 80. (b)
81. (b) 82. (a) 83. (d) 84. (b) 85. (a) 86. (b) 87. (c) 88. (d) 89. (d) 90. (c)
91. (c) 92. (c) 93. (c) 94. (c) 95. (a) 96. (b) 97. (a) 98. (a) 99. (d) 100. (b)

Round II
1. (b) 2. (a) 3. (b) 4. (c) 5. (a) 6. (b) 7. (b) 8. (d) 9. (c) 10. (c)
11. (a) 12. (b) 13. (d) 14. (b) 15. (d) 16. (b) 17. (d) 18. (d) 19. (b) 20. (a)
21. (a) 22. (a) 23. (a) 24. (c) 25. (b) 26. (c) 27. (b) 28. (c) 29. (a) 30. (a)
31. (b) 32. (c) 33. (c) 34. (c) 35. (b) 36. (d) 37. (d) 38. (d) 39. (a) 40. (b)
41. (a) 42. (a) 43. (c) 44. (b) 45. (c) 46. (b) 47. (c) 48. (b) 49. (c) 50. (a)
51. (a) 52. (b) 53. (c) 54. (d) 55. (a,c,d) 56. (b,c) 57. (a,c) 58. (a,b) 59. (a,c) 60. (c)
61. (d) 62. (b) 63. (a) 64. (b) 65. (b) 66. (a) 67. (a) 68. (a) 69. (a) 70. (a)
71. (c) 72. (a) 73. (a) 74. (c) 75. (b) 76. (a) 77. (c) 78. (a) 79. (a) 80. (b)
81. (d) 82. (a) 83. (d) 84. (d) 85. (d) 86. (c) 87. (b) 88. (c) 89. (d) 90. (d)
91. (b) 92. (d) 93. (b)

the Guidance
Round I
1. Slope of p-V graph of adiabatics = g p / V p2
=?
p1
Slope of p-V graph of isothermal = p / V
As hydrogen is a diatomic gas.
Required ratio = g
7
2. Here, p1 = 1 atm, T1 = 27° C \ g = = 14
.
5
= 27 + 273 = 300 K For an adiabatic change,
p2 = 8 atm, T2 = ?, g = 3 / 2 p1V1g = p2V2g
As change are adiabatic, g
p2 æ V1 ö
\ As changes are adiabatic, \ =ç ÷
p1 è V2 ø
\ P1g -1T1-g = p2g -1T2-g 1. 4
æ V ö
æ T2 ö
-g
æp ö
g -1 =ç ÷ = (2)1. 4
= ç 1÷ èV / 2ø
ç ÷
è T1 ø è p2 ø
= 2.64
( g -1 )/ g
æp ö 4. As, WAB = - p0V0 ,
T2 = T1ç 2 ÷ = 300 ´ (8)(1.5-1)/1.5 = 300 ´ (8)1/3
è p1 ø
WBC = 0
Þ T2 = 600 K = (600 - 273)° C = 327° C and WCD = 4p0V0
3. Let initial volume of the gas in the cylinder be V. \ WABCD = WAB + WBC + WCD
\ V1 = V = - p0V0 + 0 + 4p0V0 = 3p0V0
and V2 = V / 2
Telegram @unacademyplusdiscounts

602 JEE Main Physics

R 17. Work done by the gas is equal to area of rectangle ABCDA, .


5. Here, for hydrogen, C p - CV = m =
2
= AB ´ BC = (2V0) p0 = 2 p0V0
or R = 2m
As the trace is anticlockwise, this work is done on the gas.
R
and for nitrogen, C p - CV = n = or R = 28n \Work done by the gas = - 2p0V0
28
Þ 2m = 28n 18. From symmetry considerations and also from theory,
m = 14n Va Vb
=
Vd Vc
6. Amount of sweat evaporated/minute
Sweat produced / Minute 19. As work done by the gas = are under the p-V curve, therefore
=
Number of cals required for evaporation / kg W1 > W2 > W3 .
4
14.5 ´ 10 145 20. Work done = Area of DABC
= = = 0.25 kg
580 ´ 10 3 580 AB ´ BC ( p2 - p1) (V2 - V1)
= =
2 2
7. As, W = ò p dV = area under the p -V curve
21. Curve IV is parallel to volume axis. It represents isobaric
= minimum along ADB curve. Out of curve II and III, slope of curve III is smaller.
8. As work done in a process = area under the curve, which Therefore, curve III represents an isothermal curve.
increases continuously. 22. AB and CD are isothermal curves therefore Ta = Tb and Tc = Td
Dp but all the four temperatures are not equal.
9. As, Ka = gp = -
DV / V
23. Work done during the complete cycle
DV Dp = Area ABCDA = AD ´ AB = p ´ V = pV
\ =-
V gp
24. When T is constant, pV = constant. When volume is
10. In curves A and B, pressure and volume both increase. 90
decreased by 10% i. e. , volume become , the pressure
Therefore, temperature must rise and heat must be 100
supplied/work is done. Therefore, A and B cannot be required must become100 / 90 .
curves. Out of C and D, slope of D smaller. Therefore, D is (100 - 90) ´ 100
isothermal curve and C is adiabatic curve. \ % increase in pressure = = 11.1%
90
11. In a cyclic process, work done is equal to area of the loop 25. As, dW = dU = mCV DT = - CV (T2 - T1) = CV (T1 - T2) (Q m = 1)
ACBDA, representing the cycle of changes.
12. As, p2V2g = p1V1g 26. According to question, the figure can be drawn as below
g g 7 /5
æV ö ær ö æ2ö
\ p2 = p1ç 1 ÷ = p1ç 2 ÷ = p ç ÷ = 2.63p 6 × 105 A
è 2ø
V r
è 1ø è 1ø
4 × 105 C B
13. As isothermal curve at T1 is farther from the origin than the
isothermal at T2, therefore, T1 > T2. D E
1m3 1 is anti-clockwise,
3m3 O
14. Figure shows that loop therefore W1 is V
negative, loop 2 is clockwise, therefore W2 is positive. Now, work done by the system
Also, loop 2 is bigger. = Area under p-V diagram
W2 > W1 = Area of rectangle BCDE + area of DABC
Hence, net work done W = - W1 + W2 = W2 - W1 = ( + ) ve 2 ´ 10 5 ´ 2
= 4 ´ 10 5 ´ 2 +
15. Isobaric expansion is represented by curve AB. 2
Work done = Area under AB Þ W = 10 ´ 10 5 J
2 2
= 2 ´ 10 ´ (3 - 1) = 4 ´ 10 = 400 J
27. As dW = pdV
16. As, it is known, \ (i) dW = p ´ 0 = 0 (Q change in volume = 0)
Slope of adiabatic curve Cp and (ii) dW = p(2V - V ) = pV
=g=
Slope of isothermal curve CV
Telegram @unacademyplusdiscounts

Thermodynamics 603

28. In figure, T is constant and p1 > p2. This situation is 33. Here, V1 = 1L = 10 -3m3 , V2 = 3, L = 3 ´ 10 -3m3
represented by curve (iii) in figure, in which p1 > p2 and p1 = 1 atm = 1.013 ´ 10 5 Nm-2, g = 1.40, W = ?
straight line graph is parallel to pressure axis indicating
constant temperature. As changes are adiabatic.
\ p1V1g = p2V2g
29. In adiabatic operation ( e. g . , bursting of tyre)
g
p(21- g )T2g = p1(1- g )T1g p1 æ V2 ö
= ç ÷ = (3)1.4 = 4.6555
(1- g )/ g p2 è V1 ø
æp ö
or T2 = T1ç 1 ÷ p1 1.013 ´ 10 5
è p2 ø \ p2 = =
4.6555 4.6555
æ 1- 7 /5 ö
æ 4öè
ç
7 /5 ø
÷
= 0.217 ´ 10 5 Nm-2
= 300 ç ÷ = 300( 4) -2/ 7
è 1ø p1V1 - p2V2
Now, work done =
(Q atmospheric pressure = 1atm) g -1

30. During adiabatic expansion. 1.013 ´ 10 5 ´ 10 -3 - 0.217 ´ 10 5 ´ 3 ´ 10 -3


=
TV g -1 = constant Þ T2V2g -1 = T1V1g -1 1.4 - 1
g -1 = 90.5 J
T1 æ V2 ö
or =ç ÷ 34. Area under curve III is minimum. Therefore, work done is
T2 è V1 ø
minimum.
For monoatomic gas, g = 5 / 3
5 /3 -1 2/3 35. Heat absorbed by the system at constant pressure, Q = nC pDT
T1 æ AL2 ö æL ö
=ç ÷ = ç 2÷ Change in internal energy DU = nCV Dt
T2 è AL1 ø è L1 ø
g As, W = Q - DU
æV ö
31. From, p2V2g = p1V1g Þ p2 = p1ç 1 ÷ (1st law of thermodynamics)
è V2 ø W Q - DU DU
5 /3
\ = =1-
æ100 ö Q Q Q
p2 = ç ÷ p1
è 124 ø RC g DT C
=1- =1- V
p2 = 0.6985p1 RC g DT Cp
\% decrease in pressure æ 1ö
= ç1 - ÷
p - p2 è gø
= 1 ´ 100%
p1
36. Change in internal energy does not depend upon path so
p1 - 0.6985p1
= ´ 100 DU = DQ - DW remains constant.
p1
0.3015p1
37. DQ = - 20 J ; DW = -10 J. Now, DQ = (Uf - Ui ) + DW
= ´ 100%
p1 Þ -20 = (Uf - 40) - 10
= 30.15% » 30% Þ Uf = - 10 + 40 = 30 J

32. The p -V graphs three given processes are shown in figure. 38. Given, work done (W ) = - 22 .3 J
Work done is taken negative as work is done on the system.
2 In an adiabatic change DQ = 0
p0 Isobaric
Using first law of thermodynamics,
p 1
Isothermal DU = DQ - W
= 0 - ( -22.3)
3
Adiabatic = 22. 3 J
V1 V V2 For another process between state A and state B,
Heat absorbed ( DQ) = + 9.35 cal
As work done by the gas = area under the p -V graph (between
the curve and V axis) = + (9.35 ´ 419
. )J
Þ (Area) 2 > ( Area)1 > ( Area)3 = + 3918
. J
\ W2 > W1 > W3 Change in internal energy between two states via different
paths are equal.
Telegram @unacademyplusdiscounts

604 JEE Main Physics

\ DU = 22.3 J 48. Here dQ = 50 J, dW = -15 J


\ From first law of thermodynamics
\ dU = dQ - dW = 50 - ( -15) = 65 J
DU = DQ - W
49. According first law of thermodynamics,
or W = DQ - DU
DQ = DU + DW
= 3918
. - 22.3
Þ DQ = 0 + W = W
= 16.88 J
7
» 16.9 J 50. Here, n = 5, g = , T1 = 0° C, T2 = 400° C
5
39. DQ = DU + DW nRdT
\ dU =
DQ = 0 - 150 J = -150 J 7
-1
So, heat has been given by the system. 5
5 ´ 8.31 ´ ( 400 - 0)
40. Here, dQ = 400 cal, dW = -105 J dU = = 41550 J
7
-1
= - 105 / 4.2 cal = - 25 cal; dU = ? 5
Now, dU = dQ - dW dU = 41.55 kJ
dU = 400 - ( -25) = 425 cal 51. Internal energy ( DU) does not depend upon path. It depends
only on initial and final states.
Note dW is negative because work is done on the system.
dU CVdT CV (3 / 2)R 3
52. As, = = = =
41. Here, dQ = 110 J, dU = 40 J, dW = ? dQ C pdT C p (5 / 2)R 5
From dQ = dU + dW (Here, number of moles of gas is constant)
dW = dQ - dU = 110 - 40 = 70 J
53. When the compression is isothermal for gas in A,
42. Process 1 is isobaric (p = constant) expansion. p2V2 = p1V1
Hence, temperature of gas will increase. pV V
p2 = 1 1 = p1 1 = 2p1
\ DU1 = negative V2 V1/ 2
Process 2 is an isothermal process For gas in B, when compression is adiabatic,
\ DU2 = 0 p2¢ V2g ¢ = p1V1g
Process 3 is an adiabatic expansion. g g
Hence, temperature of gas will fall. æV ö æ V ö
p2¢ = p1 ç 1 ÷ = p1 ç 1 ÷ = 2 g p1
\ DU3 = constant, è V2¢ ø è V1/ 2 ø
\ DU1 > DU2 > DU3 p2¢ 2 g p1
\ = = 2g - 1
43. As, DU = DQ - DW p2 2p1
\ DU = ( -20) - ( -8) = - 12 J 54. If the equilibrium temperature T > T1 and T2 but less than T3,
DU = Uf - Ui = -12 then as there is no heat loss to the surroundings therefore heat
\ Uf = - 12 + Ui = - 12 + 30 = 18 J lost by M1 and M2 = heat gained by M3
44. As indicator diagrams in all the three cases are closed curves, M1s (T1 - T2) + M2s(T2 - T) = M3 s(T - T3)
representing cyclic changes, therefore, U = constant and M1T1 + M2T2 + M2T3 = (M1 + M2 + M3) T
DU = 0 in all the cases. M T + M2T2 + M3T3
Þ T= 11
45. As initial and final states in the two processes are same. M1 + M2 + M3
Therefore, DU1 = DU2. As area under curve a > area under
curve b, i.e., DW1 > DW2
55. The internal energy of a system remains constant when the
temperature does not change i. e. , when the system is
As DQ = DU + DW isothermal.
\ DQ1 > DQ 2
56. Work done by the system = Area of shaded portion on the p-V
46. As, dU = dQ - dW = mL - p(dV ) diagram
1.013 ´ 10 5(1671 - 1)10 -6 = (300 - 100)10 6 ´ (100 - 200)10 3
= 1 ´ 540 -
4.2 = - 20 J
= 540 - 40 = 500 cal T2 500 3
57. As, h = 1 - =1- = = 0.375
47. In adiabatic expansion, dQ = 0 , T1 800 8
\ dW = - dU = - ( -50 J ) = 50 J
Telegram @unacademyplusdiscounts

Thermodynamics 605

T2 40 3 T2 100 T
58. As, =1- h =1- = 66. As, h = 1 - Þ1- =1-
T1 100 5 T1 500 900
5 5 T 1
\ T1 =T2 = ´ 300 = 500 K \ =
3 3 900 5
Increase in efficiency = 50% of 40% = 20% or T = 180 K
\ New efficiency, h¢ = 40 + 20 = 60% 67. Given, T2 = 0°C = 273 K, T1 = 17°C = 17 + 273 = 290 K
T2 60 2
\ = 1 - h¢ = 1 - = Q2 T2
T1¢ 100 5 COP = =
W T1 - T2
5
T1¢ =
´ 300 = 750 K 80 ´ 1000 ´ 4.2 273 273
2 Þ = =
W 290 - 273 17
Increase in temperature of source = T1¢ - T1
80 ´ 1000 ´ 4.2 ´ 17
= 750 - 500 = 250 K W= J
273
T2
59. From h = 1 - , 33.6 ´ 17 ´ 10 4
T1 W= kWh
273 ´ 3.6 ´ 10 5
T2 40 3
Þ =1- h =1- = = 0.058 kWh
T1 100 5
T
3 3 68. As, h = 1 - 2
\ T2 = T1 = ´ 500 = 300 K T1
5 5
T2 T2 1 5
Again =1- h Þ =1-h =1- = …(i)
T1¢ T1 6 6
300 50 1 T2 - 62 2 2
or =1- = In second case = 1 - h¢ = 1 - = …(ii)
T1¢ 100 2 T1 6 3

or T1¢ = 600 K 1 2
Þ h¢ = 2 ´ =
6 6
60. Given, T1 = 27° C = (27 + 273) K = 300 K ,
From Eqs. (i) and (ii)
and T2 = - 123 + 273 = 150 K
2 2 6
T 150 Now, T2 - 62 = T1 = ´ T2
\ h =1- 2 =1- = 0.5 3 3 5
T1 300
Þ T2 = 310 K = 310 - 273 = 37°C
T2 T1 - T2
61. As, h = 1 - = 6 6
T1 T1 T1 = T2 = ´ 310 = 372 K
5 5
In all the four cases, T1 - T2 = 20 K. Therefore, h is highest,
when T1 is lowest. = 372 - 273 = 99°C
W 69. Given, T2 = 27 + 273 = 300 K and h = 37.5%
62. We have, h =
Q1 T2
As, h =1-
1 1000 T1
Þ W = hQ1 = ´ 1000 cal = ´ 4.2 = 1400 J
3 3 37.5 300
T 300 1 \ =1-
63. As, h = 1 - 2 = 1 - = = 50% 100 T1
T 600 2
300 62.5 5
T2 273 - 23 250 or = =
64. COP = = = = 2.5 T1 100 8
T1 - T2 (273 + 77) - (273 - 23) 100
2400
Q T1 = = 480 K
As, COP = 2 5
W
= 480 - 273 = 207°C
1000 ´ 80 ´ 4.2
\ 2.5 = 70. Givne, T1 = 273 + 20 = 293 K, T2 = 273 + 10 = 283 K
W
1000 ´ 80 ´ 4.2 \Coefficient of performance
or W= = 134400 J
2.5 T2 283
= =
Q T1 - T2 293 - 283
65. Here, Q 2 = 2000 cal. As, COP = 2
W 283
= = 28.3
\ 4 = 2000 / W 10
W = 500 cal = 500 ´ 4.2 = 2100 J
Telegram @unacademyplusdiscounts

606 JEE Main Physics

T2 T2 W
71. As, h = 1 - 80. As, h = 1 - =
T1 T1 Q
T2 1 5 æ T ö ì (273 + 27) ü
or =1- h =1- = Þ W = ç1 - 2 ÷Q = í1 - ý ´Q
T1 6 6 è T1 ø î (273 + 627) þ
5 5
T2 = T1 = ´ 600 = 500 K æ 300 ö 6 6 6
6 6 Þ W = ç1 - ÷ ´ 3 ´ 10 = 2 ´ 10 ´ 4.2 J = 8.4 ´ 10 J
è 900 ø
72. Here, T1 = 411°C = ( 411 + 273) K = 684 K 500 3
81. In first case(h1) = 1 - =
T2 = 69°C = (69 + 273) K = 342 K 800 8
and Q1 = 1000 J 600
and in second case (h 2) = 1 -
W T 342 1 x
Q h= =1- 2 =1- = 3 600
Q1 T1 684 2 Since h1 = h 2 therefore = 1 -
1000 8 x
Þ W = hQ1 = = 500 J 600 3
2 =1-
x 8
Q 2 T2
73. As, = 5 600 ´ 8
Q1 T1 or x = = 960 K
8 5
Q2 150 T
\ T2 = ´ T1 = ´ 400 = 300 K 82. Efficiency of an engine is h = 1 - 2
Q1 200 T1
Q 2 T2
74. As, = where, T1 is the source temperature (higher) and T2 is the sink
Q1 T1
temperature (lower)
Q2 127 + 273 400
\ 4
= = For h =1 (i. e. , 100%)
6 ´ 10 227 + 273 500
either T1 = ¥
4
Q2 = ´ 6 ´ 10 4 = 4.8 ´ 10 4 cal or T2 = 0 K
5
\ W = Q1 - Q 2 = 6 ´ 10 4 - 4.8 ´ 10 4 = 1.2 ´ 10 4 cal
83. As, C p - CV = R = 4150 J/kg-K
Cp
75. The efficiency of two engines are and = g = 1.4
CV
T 500 1
hA = 1 - 2 = 1 - = R 4150
T1 1000 2 CV = = = 10375 J / kg -K
g - 1 (1.4 - 1)
T2 400 7
and hB = 1 - =1- = 2
T1 1100 11 84. As, g = 1 + (where f = degree of freedom)
f
Clearly, h A < hB 2
Þ g -1=
T ( 47 + 273) 320 1 f
76. As, h = 1 - 2 = 1 - =1- = = 20%
T1 (127 + 273) 400 5 f 1
Þ =
T2 500 600 2 g -1
77. As, h = 1 - =1- =1-
2
T1 800 x Þ f=
3 600 g -1
\ =1-
8 x 85. From, DQ = m C p( DT)
600 3 5
=1- = ( m = number of moles)
x 8 8
70 = 2 ´ C p ´ (35 - 30),
5x = 4800
4800 \ C p = 70 / 10 = 7 cal (mol°C) -1
x= = 960 K
5 CV = C p - R = 7 - 2 = 5 cal/mol°C
T T 10 90 Now, DQ ¢ = m CV ( DT)
78. As, h = 1 - 2 \ 2 = 1 - h = 1 - =
T1 T1 100 100 = 2 ´ 5 ´ 5 = 50 cal
100T2
or T1 = 86. As, CV = C p - R = 207 - 8.3 = 198.7 J
90
100 87. Work done in expansion = C p - CV = R joule
= ´ 270 = 300 K
90
Telegram @unacademyplusdiscounts

Thermodynamics 607

3 3 dV T2 2 dT
T2
88. As, dU = CV dT = æç Rö÷ dT = ´ 8.32 ´ 100 = 1.25 ´ 103 J W = ò RT
V òT1
Work done, RT
è2 ø 2 T1 3 T
2 2
89. Internal energy U = number of moles ´ number of degrees of W = R(T2 - T1) = R ´ 60 = 40R
3 3
1
freedom ´ RT
2 95. Below 150 K, hydrogen behaves as monoatomic gas
Out of four cases, product of number of moles (1000) degrees 1 1 æ5 7ö 3
\ For the mixture,g = [ g mono + g di ] = ç + ÷ =
of freedom (3) and T( = 900 K) is maximum for argon gas. 2 2 è3 5ø 2
90. For a non-linear triatomic gas, CV = 3R 96. Given, C p - CV = 4150
3
and for monoatomic gas, CV ¢ = R Cp
2 and = 1.4, C p = 1.4 CV
Q CV 3R CV
\ = =K = =2
Q ¢ CV ¢ 3 \ 1.4 CV - CV = 4150
R
2 CV = 4150 / 0.4 = 10375 J - kg K -1
91. As, C p - CV = R = 2 cal (mol K) -1. Difference in the two values Cp 2 2
97. As, h = =1+ =1+
must be 2. CV n f
5
92. For monoatomic gas, g = = 1.67 98. Given, p µ T3 but we know that, for an adiabatic process the
3
pressure, p µ T g / g -1
7
and for diatomic gas g = = 1.40 g
5 So, =3
g -1
As actual g = 1.5. Therefore, gas must be a mixture of
monoatomic and diatomic gases. 3
Þ g=
-1
2
93. Given, C p = 8 cal (mol°C) , Cp 3
Þ =
CV = C p - R = 8 - 2 = 6 cal (mol °C ) -1 CV 2
\ dU = mCV (T2 - T1) = 5 ´ 6(20 - 10) = 300 cal 99. We know that, E f = gp = 1.4 ´ (1 ´ 105) = 1.4 ´ 105 N/m 2
94. As V = KT 2/3
100. In cyclic process, DQ = Work done = Area inside the closed
2 p
\ dV = K T -1/3dT (after differentiating) curve treat the circle as an ellipse of area =
( p2 - p1) (V2 - V1)
3 4
2 -1/3 p p
KT dT Þ DQ = {(180 - 50) ´ 10 3 ´ ( 40 - 20) ´ 10 -6} = J
dV 3 2 dT
\ = 2/3
= 4 2
V KT 3 T

Round II
According T2 (127 + 273) 1
1.
1 1h
5. As, = - = - to first
= law of thermodynamics,
dQ = dU + dW T1 (227 + 273) 5
1
As dW = - dU W = hQ1 = ´ 10 4 J = 2000 J
5
Þ dQ = dU - dU = 0
6. As, work done = area under the p-V diagram
The change must be adiabatic.
\ W1 > W2
2. Here, T1 = 927° C = (927 + 273) K = 1200 K 7. At constant temperature,
and T2 = 27° C = (27 + 273) K = 300 K p1V1 = p2V2
As U µT p1 V2
or =
DU U1 - U2 1200 - 300 p2 V1
\ = = ´ 100 = 300%
U2 U2 300 Fractional change in volume
3. As, DQ = DU + DW = mCV ( DT) + p( DV ) V1 - V2 4 1
= =
V1 100 25
5
4. We know that, ka = gp = æç ö÷ ´ 1.01 ´ 105 Nm-2 V 1
è3ø 1- 2 =
V1 25
= 1.69 ´ 10 5 Nm-2
Telegram @unacademyplusdiscounts

608 JEE Main Physics

V2 24 æV ö
= 13. As, W = 2.3026 n RT log10 ç 2 ÷
V1 25 è V1 ø
p1 V2 24 p 25
\ = = or 2 = æ 10 ö
p2 V1 25 p1 24 = 2.3026 ´ 10 ´ 9.3 ´ 600 log10 ç ÷
è100 ø
p2 - p1 25 1
= -1= = - 11.4 ´ 10 4 J
p1 24 24
100 14. From p2V2g = p1V1g
% increase in pressure = = 4.16
24 g 1.4
æV ö æ V1 ö
Þ p2 = p1ç 1 ÷ = 1 ç ÷
8. As, dW = dQ - dU è V2 ø è1 / 20V1 ø
= C p(T2 - T1) - CV (T2 - T1) = 66.28 atm
= R[T2 - T1] (Q C p - CV = R)
15. Here, p = 4.5 ´ 10 5 Pa,
= 8.31 ´ 100 = 8.31 ´ 10 2 J
dV = (2.0 - 0.5) m3 = 1.5 m3
9. For monoatomic gas,
and dQ = 800 kJ = 8 ´ 10 5 J, dU = ?
3 3
CV = R = ´ 8.31 Jmol-1°C-1 dW = pdV = 4.5 ´ 10 5 ´ 1.5 = 6.75 ´ 10 5 J
2 2
Given, Q = 500 J, n = 4q = ? dU = dQ - dW = 8 ´ 10 5 - 6.75 ´ 10 5
Q 500 = 1.25 ´ 10 5 J
\ q= = = 10°C
nCV 4 ´ 3 ´ 8.31
2 16. As, dU = dQ - dW = 8 ´ 105 - 6.5 ´ 105 = 1.5 ´ 105 J
10. As, C p / CV = g In the 2nd process, dU remains the same
C p - CV \ dW = dQ - dU = 10 5 - 1.5 ´ 10 5
\ = g -1
CV = - 0.5 ´ 10 5 J .
C p - CV R 17. Given, T1 = 200° C = 200 + 273 = 473 K
or CV = =
g -1
g -1
T2 = 0° C = 0 + 273 = 273 K
RdT npdV T 273 200
\ DU = nCVdT = n = \ h1 = 1 - 2 = 1 - =
( g - 1) g -1 T1 473 473
np(2V - V ) npV
= = Again, T1¢ = 0° C = (0 + 273) K = 273 K
g -1 g -1
and T2¢ = - 200° C = ( -200 + 273) K = 73 K
As n = 1,
T 73 200
pV \ h2 = 1 - 2 = 1 - =
\ DU = T1¢ 273 273
(g - 1)
h1 200 273 273 1
Now, = ´ = =
11. In an adiabatic change, h 2 473 200 473 1.732
p1- g T g = constant T1 - T2 (t1 + 273) - (t 2 + 273) t -t
18. As, h = = = 1 2
or pT/1-g g = constant T1 t1 + 273 t1 + 273
or p µ T -(1- g )/ g 19. As is clear from figure,
1- g
Thus, c=- Slope of curve 2 > Slope of curve 1
g
( gp) 2 > ( gp)1
5
For a monoatomic gas, g = g 2 > g1
3
1- 5 /3 2 As g He > go 2
\ -c = =- \ Adiabatic curve 2 corresponds to helium and adiabatic
5 /3 3
2 curve 1 corresponds to oxygen.
Þ c=
5 20. As, DWAB = pDV = 10(2 - 1) = 10 J
12. As slope of adiabatic AC is more than the slope of isothermal and DWBC = 0 , because V is constant
AB, and BC is isochoric (i. e. , at constant volume), therefore, From first law of thermodynamics,
Fig. (b) represents the curves correctly.
DQ = DW + DU
As ABCA is a cyclic process, therefore,
Telegram @unacademyplusdiscounts

Thermodynamics 609

DU = 0
\ DQ = DWAB + DWBC + DWCA p3 C
p
= DWAB + DWCA
p1
or DWCA = DQ - DWAB = 5 - 10 = - 5 J A

21. Here Q1 = 200 cal, Q 2 = 150 cal, T1 = 400 K B


Q1 T1
As, = V1 V2
Q 2 T2
V
Q2 150 In going from A to B, volume is increasing
T2 = ´ T1 = ´ 400 = 300 K
Q1 200
\ WAB = positive
22. Given, dQ = 2 kcal = 200 cal = 2000 ´ 4.2 J = 8400 J In going from B to C volume is decreasing
and dW = 500 J \ WBC = negative
\ dU = dQ - dW As work done is area under p-V graph, therefore,
= 8400 - 500 = 7900 J |WBc| > |WAB|
\ W = WAB + WBC = Negative i. e. ,W < 0.
23. Here, dU = -100 J, in adiabatic expansion
From the graph, it is clear that p3 > p1.
\ dW = - dU = 100 J
n
27. The given relation is p = aV
24. Given that, Vp = constant
Therefore, p µV
\ Vp n = (V + DV ) ( p + Dp) n When V changes from V to 2V , pressure p is also doubled.
æ DV ö æ Dp ö pV
= Vp n ç1 + ÷ ç1 + n ÷ For an ideal gas, = constant
è V øè p ø T
DV Dp DV Dp \ T µ pV
1=1+ +n +n
V p V p Hence, T becomes 2 ´ 2 = 4 times
DV Dp i. e. , 4 ´ 300 K = 1200 K
or = -n , (neglecting the product)
V p 28. If M is molecular mass of the gas, then from
- Dp p M(C p - CV ) = R
Þ Bulk modulus, k = =
DV / V n 8.31
Þ M= = 0.0392
210
25. For the process at constant pressure
If r is density of the gas at NTP, then mass of 1 m 2 of gas at NTP
dQ = C pdT + dW
= r kg
dQ - dW
\ dT = \ Mass of 22.4 L ( = 22.4 ´ 10 -3 m3) of gas at NTP
Cp
= r ´ 22.4 ´ 10 -3 kg,
For the process at constant volume, Which is the molecular mass of the gas
dQ = CVdT (\ dW = 0) \ r ´ 22.4 ´ 10 -3 = 0.0392
æ dQ - dW ö 0.0392
= CV ç ÷ r= = 1.75 kgm-3
è Cp ø 22.4 ´ 10 -3
dQ - dW aT 2
= 29. The given relation is p =
C p / CV V
dQ - dW aT 2
= \ V=
g p
or ( g - 1dQ
) = dW As pressure is kept constant,
æ5 ö æ 2aT ö
ç - 1÷dQ = W \ dV = ç ÷dT (after differentiating)
è3 ø è p ø
3 2T 0 æ 2at ö
\ dQ = W Now, W = ò p dV = ò pç ÷ dT
2 T0 è p ø
26. As, slope of adiabatic process at a given state is more than the 2T 0
éT2ù
slope of isothermal process, therefore in figure, AB is = - 2a ê ú = 3aT02
2
ë ûT0
isothermal and BC is an adiabatic.
Telegram @unacademyplusdiscounts

610 JEE Main Physics

1 We know that,
30. KE of the vessel = Mv 2
2 DQ = mC p DT
when the vessel is suddenly stopped, the ordered motion of and DU = mCV DT
the gas molecules is converted into disordered motion of the
DU CV 3
molecules increasing thereby the internal energy of the gas. Þ = =
Thus, DQ C p 5
1 1 i. e. , Fraction of heat energy to increases the internal energy
DU = nCV DT = mv 2 = (nM)v 2
2 2 3
be ×
where n is number of moles of the gas in the vessel and M is 5
molecular weight of the gas. 36. Given, work done (W ) = 5.4 ´ 10 8 J/min
Mv 2
\ DT = Total heat energy taken from the boiler,
2CV
Q = 3.6 ´ 10 9 J/min
R Mv 2( g - 1)
As CV = \ DT = W
g -1 2R Efficiency of heat engine (h) = ´ 100
Q
31. From first law of thermodynamics, DQ = DU + W 5.4 ´ 10 8
= ´ 100
Work done at constant pressure (DW ) p = ( DQ) p - DU 3.6 ´ 10 9
= ( DQ) p - ( DQ)V (As, we know DQV = DU) 3
= ´ 100%
Also ( DQ) p = MC p DT 20
and ( DQ)V = MCV DT = 15%
Þ ( DWp) = M(C p - CV ) DT Heat wasted per minute = Q - W
= 1 ´ (3.4 ´ 10 3 - 2.4 ´ 10 3) ´ 10 = 10 4 cal = 3.6 ´ 10 9 - 5.4 ´ 10 8

32. From the first law of thermodynamics, = (36 - 5.4) ´ 10 8 J/min

DQ = DU + DW ..(i) = 30.6 ´ 10 8 J/min


For a cyclic process, DU = 0 . ´ 10 9 J/min
» 31
\ DQ = DW
Now, DQ = Q1 + Q 2 + Q3 + Q 4
Note DW is negative because work is done on the system.
= 600 J - 400 J - 300 J + 200 J = 100 J 37. As work done in an isothermal process is
and DW = W1 + W2 + W3 + W4 V2
W = mRT log e
Þ DW = 300 J - 200 J - 150 J + W4 V1
= - 50 J + W4 æmö V m V
= ç ÷RT log e 2 = 2.3 ´ RT log10 2
Substitute the value of DQ and DW in Eq. (i), we get èMø V1 M V1
100 J = - 50 J + W4 96 140
= 2.3 ´ R(273 + 27) log10 = 2.3 ´ 900R log10 2
W4 = 150 J 32 70
V æ 22.4 ö
33. Change in internal
e energy is always equal to the
e ç heat ÷
supplied 38. As, W = - mRT log 2 = - 1 ´ 8.31 ´ (273 + 0) log
V1 è 11.2 ø
at constant volume.
i. e. , DU = ( DQ)V = mCV DT (m = number of moles) (–ve sign shows compression)
3 = - 8.31 ´ 273 ´ log e 2
For monoatomic gas, CV = R
2 = -1572.5 J [Q log e 2 = 0.693]
æ3 ö 3
39. Given, temperature of source (T1) = (36 + 273) K
DU = r ç R ÷ DT = 1 ´ ´ 8.31 ´ (100 - 0)
è2 ø 2
= 309 K
= 12.48 ´ 10 2 J
Temperature of sink (T1) = (9 + 273) K = 282 K
æ R ö Coefficient of performance of a refrigerator
34. As, DU = mCV DT = m ç ÷ DT
è g - 1ø T2 282
b= =
pD V p (2V - v) pV T1 - T2 309 - 282
\ DU = = =
( g - 1) (g - 1) ( g - 1) 282
=
Cp 5 27
35. For monoatomic gas, g = =
= 10.4
CV 3
Telegram @unacademyplusdiscounts

Thermodynamics 611

40. Amount of heat given = 540 cal 3 RT (v rms)1 T


48. As, v rms = = = 1
M (v rms) 2 T2
Change in volume DV = 1670 cc
atmospheric pressure p = 1.01 ´ 10 6 dyne/cm 2 Now, T1Vgg -1 = T2V2g -1
g -1
\Work done against atmospheric pressure, T1 æ V2 ö
=ç ÷
1.01 ´ 10 6 ´ 1670 T2 è V1 ø
W = pD V = = 40 cal
4.2 ´ 10 7 g -1
(Vrms)1 æ V2 ö 2
41. For isothermal process, p1V1 = p2V2 Thus, =ç ÷
(Vrms) 2 è V2 ø
p1V1 72 ´ 1000
Þ p2 = = = 80 cm of mercury 7
-1
V2 900 5
V æV ö 2
\Stress, Dp = p2 - p12 = 80 - 72 = 8 cm of mercury = = ç 2÷ =2
V / 2 è V1 ø
42. Here, P = 105 N/m 2, T1 = 27 + 273 = 300 K 2 1 1
´
T2 = 627 + 273 = 900 K and g = 1.5 æV ö5 2 æV ö5 æV ö
= ç 2÷ = ç 1 ÷ = ç 2 ÷ = 25 = 32
Tg è V1 ø è V1 ø è V1 ø
For adiabatic change, g -1 = constant g -1
p æV ö
1/ 2 3/ 2 49. As, T2 = T1ç 1 ÷ = 273(2) 0.41 = 273 ´ 1.328 = 363 K
æ p2 ö æT ö è V2 ø
ç ÷ = ç 2÷
è p1 ø è T1 ø Now, for one mole of the gas, the work done
1/ 2 3/ 2 R(T1 - T2) 8.31(273 - 363)
æ p2 ö æ 900 ö |W | = = = - 1815
Þ ç ÷ =ç ÷ g -1 1.41 - 1
è105 ø è 300 ø
Þ p2 = 27 ´ 10 5 N / m2 Þ |W| = 1815 J
M mR(T1 - T2) mRT1 é T2 ù
43. As, volume of the gas, V = 50. As, W = = ê1 - T ú
d ( g - 1) ( g - 1) ë 1û

Now, using pV g = constant, we get g -1 é 5


-1ù
mRT1 é æ V1 ö ù 2 ´ 8.31 ´ 300 ê1 - æç 1 ö÷ 3 ú
g = ê1 - ç ÷ ú =
p¢ æ V ö ( g - 1) ê è V2 ø ú æ5 ö ê è2ø ú
= ç ÷ = (32) 7 /5 = 128 ë û ç - 1÷ ë û
p è V ¢ø è3 ø
g -1
T2 æ V1 ö = + 2767.23 J
44. As, =ç ÷
T1 è V2 ø 1
51. Number of moles of He =
5 2 4
-1 2
æ 27 ö 3 æ 27 ö 3 æ3ö Now, T1(5.6) g -1 = T2(0.7) g -1
Þ T2 = 300 ç ÷ = 300 ç ÷ = 300 ç ÷ = 675 K
è8ø è8ø è2ø 2/3
æ 1ö
Þ DT = 576 - 300 = 375 K T1 = T2ç ÷
è8ø
dQ
45. = er sAT 4 = E 1 2 4T = T
dt
1
E µ r 2T 4 - R[3T1]
-nR[T2 - T1] 4 -9
4 \Work done W = = = RT1
2
æT ö g -1 2 8
E ær ö 1
\ 1 = ç 1÷ ç 1÷ = 3
E 2 è r2 ø ç T2 ÷ 1
è ø
9
Þ |W| = RT1
mR(T1 - T2) 8
46. Work done during an adiabatic change is, W =
( g - 1)
52. According to 1st law of thermodynamics,
Cp
Now, g= DQ = DU + DW , in adiabatic process DQ = 0
CV 0 = DU - DW (work done on the system negative
C p - CV R DU = + DW = + 22.3
\ g -1= =
CV CV [Q work done on the system\internal energy increases]
mR(T1 - T2)CV in 2nd process DQ = DU + DW
\ W= = m(T1 - T2)CV
R 9.35 ´ 4.18 = 22.3 + DW
47. Pressure is reduced so the temperature falls. Work done by system, DW = 16.95 J
Telegram @unacademyplusdiscounts

612 JEE Main Physics

53. From 1st law of thermodynamics, If Q 2 is negative, Q1 is also negative (but less negative as
W > 0).
DQ = DU + DW
\ Q 2 < Q1 < 0
Þ DW = DQ - DU
Choices (a) and (c) are correct.
Þ DW = nC p DT - nCV DT
= 10 (C p - CV ) DT = 10 RDT 60. Here, T1 = 27° C = (27 + 273) K = 300 K
= 10 ´ 8.300 J T2 = - 73° C = ( -73 + 273) K = 200 K
= 8.3 ´ 10 4 J T 200 1
\ h =1- 2 =1- = = 33%
5 -3
T1 300 3
54. As, W = pDV = 1.01 ´ 10 (3.34 - 2 ´ 10 )
T2
= 337 ´ 10 3 J = 340 kJ 61. From h = 1 -
T1
55. As shown in figure, during the process A to B, p and V both 40 (27 + 273)
=1-
decrease. As T µ pV , therefore, T must also be decreasing. So 100 T1
internal energy must be decreasing. 300 40 3
=1- =
\ DUA ® B is negative. As volume is decreasing, therefore, T1 100 5
DWA ® B is also negative. Thus, DQ A ® B = negative. 300 ´ 5
During the process B to C, volume is increasing at constant T1 = = 500 K
3
pressure. Therefore, T( µ V ) must increase and so does the T 60 T2
internal energy, DUC ® A = positive. During the process CAB, 62. From h = 1 - 2 , =1-
T1 100 (127 + 273)
volume is decreasing. Therefore, DWCAB = negative.
T2 60 2
56. From the given initial state A to final state B, change in internal =1- =
400 100 5
energy is same in all the four cases, as it is independent of the
path from A to B. 800
\ T2 = = 160 K = (160 - 273)°C = - 113°C
As work done = area under p - V curve, therefore, work done 5
is maximum in case I. Choices (b) and (c) are correct. 63. As compression is sudden, changes are adiabatic, dQ = 0.
57. In the given one complete cycle, 1 ® 2 ® 3 ® 1, the system Therefore, work done on the gas increases the temperature.
returns to its initial state. 64. The bursting of tyre is sudden. Therefore, the changes are
\ dU = 0 and dQ = dW , i. e. , heat is completely converted adiabatic.
into mechanical energy, which is not possible in such a
65. Adiabatic curve is steeper than isothermal curve. Therefore,
process. Further, the two adiabatic curves (2 3) and (3 1)
area under adiabatic curve is smaller than the area under
cannot intersect each other. Choices (a) and (c) are correct.
isothermal curve i. e. , work done by the gas in adiabatic
58. Isothermal curve from A to B will be parabolic with lesser area expansion is smaller than the work done by the gas in
under the curve than the area under straight line AB. isothermal expansion.
Therefore, work done by the gas in going straight from A to Bis
66. Here, T1 = 227 + 273 = 500 K
more. Therefore is correct.
T2 = 127 + 273 = 400 K
If p0 , V0 be the intercepts of curve on p and V axes, then its
equation T2 400 1
11 h = is
= - =20-% = obtained from y mx c
p T1 500 5
i. e. , p = 0 V + p0
V0 This is the maximum value of efficiency.
RT p0V T 100 3
or = + p0 68. As, h = 1 - 2 = 1 - = = 75%
V V0 T1 400 4
p0 2 p0V DQ
or T= V + , 69. Change in entropy, DS = × In an adiabatic change, DQ = 0
V0R R T
\ DS = 0
Which is the equation of a parabola. Hence T-V curve is
parabolic. Therefore (b) is correct. \ S = constant
Also ( p / 2) ´ (2V ) = pV = constant i. e. , process is isothermal. i. e. , entropy remains constant, or it is an isotropic process.

59. Figure represents the working of a refrigerator, wherein 71. In an ideal gas, we assume that intermolecular force are zero.
No work is done in changing the distance between the
Q1 = Q 2 + W
molecules. Therefore, internal energy is only kinetic and not
If W > 0 , Q1 > Q 2 > 0 . potential. Therefore, internal energy of an ideal gas depends
Both Q1 and Q 2 are positive. only on temperature and not on volume.
Telegram @unacademyplusdiscounts

Thermodynamics 613

T2 T1 - T2 81. In going from B to C and in going from D to A, V = constant.


73. As h = 1 - = , therefore, h will decrease if (T1 - T2)
T1 T1 Therefore, dV = 0 , dW = pdV = 0 .
decreases.
82. For adiabatic process,
74. First law of thermodynamics is a restatement of the principle dQ = 0
of conservation of energy as applied to heat energy.
So, dU = - DW
75. From A to B in figure the process is isobaric. Þ nCVdT = + 146 ´ 10 3 J
\ W = nRDT nfR
= 2 ´ R ´ (500 - 300) Þ ´ 7 = 146 ´ 10 3
2
= 400R (f ® Degree of freedom)
76. From D to A, in figure, the process is isothermal 10 3 ´ f ´ 8.3 ´ 7
Þ = 146 ´ 10 3
æp ö 2
\ W = nRT log e ç 1 ÷
è p2 ø f = 5.02 » 5
æ 1 ´ 10 5 ö Therefore gas is diatomic.
= 2 ´ R ´ 300 log e ç ÷
è 2 ´ 10 5 ø 83. The specific heat of a gas depends upon the process and can
have any value as specific heat at constant volume is CV and at
= 600R ´ 2.303(0 - 0.3010) constant pressure is C p .
= - 415.9 R
84. Consider n moles of a gas which undergo isochoric process,
\ Work done on the gas = 415.9 R i. e. ,V = constant. From first law of thermodynamics,
77. In this process, p and V changes but T = constant i. e. , change DQ = DW + DU …(i)
in temperature DT = 0 Here, DW = 0 as V = constant
Boyle’s law is obeyed i. e. , pV = constant Substituting in Eq. (i), we get
Þ p1V1 = p2V2 DU = nCV DT …(ii)
According to equation, pV = constant, graph between p and V Mayor’s relation can be written as
is a part of rectangular hyperbola. C p - CV = R
1 Þ CV = C p - R …(iii)
78. Here, T2 = 27° C = (27 + 273) K = 300 K , h = 25% =
4 From Eqs. (ii) and (iii), we have
We know that, DU = n(C p - R) DT
T
h =1- 2 Given, n = 6, C p = 8 cal (mol-K) -1
T1
1 300 R = 8.31 Jmol-1 K -1
Þ =1-
4 T1 Hence, DU = 6(8 - 2) (35 - 20) = 6 ´ 6 ´ 15 = 540 cal
300 1 85. The coefficiency of a heat engine is defined as the ratio of
or =1-
T1 4 work done to the heat supplied, i. e. ,
work done W
Þ h = = 300 = 3
T1 4 heat input Q
300 ´ 4 T
or T1 = or h =1- 2
3 T1
T1 = 400 K where, T2 = temperature of sink,
or T1 = ( 4500 - 273)° C = 127°C T1 = temperature of hot reservoir.
T2 T1 - T2 40 T
79. h = 1 - = \ =1- 2
T1 T1 100 T1

When T1 and T2 are decreased by 100 K each, (T1 - T2) stays T2


Þ = 0.6
constant and T1 decreases, hence h increases. T1
Þ T2 = 0.6T1
50 T
80. Work done = area of DABC Again, =1- 2
100 T1¢
AB ´ BC ( 4V - V ) (3p - p)
= = = 3pV T2
2 2 Þ = 0.5
T1¢
Telegram @unacademyplusdiscounts

614 JEE Main Physics

0.6T1 DW Area of DABC


Þ = 0.5 91. As, h = =
T1¢ QBC Area under curve BC
0.6 6 S 0T0 / 2 1
T1¢ = T1 = T ( as T1 = T) Þ h= =
0.5 5 3S 0T0 / 2 3
Tg
86. From the relation, = constant T
p g -1 B
2T0
g / g -1
p µT …(i)
3
But p µ T (given) …(ii)
From Eqs. (i) and (ii), we get T0
A
C
3 g /( g -1)
T =T S
S0 2S0
g
or =3
g -1 92. As change in internal energy does not depend upon the path
3 followed between the two given points on p-V diagram,
or g= therefore, DU1 = DU2.
2
87. In an adiabatic expansion of gas energy is consumed in the 93.
gas. On account of the consumption of energy the 2T0 4T0
temperature of system falls. 2p0
p1V1 p2V2
88. = p0
T1 T2 2T0
(Here, p1 = p, V1 = V , p2 = 2p, V2 = 3V ,T1 = 27°C = 300 K)
V0 2V0
pV 2p ´ 3V
= Heat supplied
T T2
H = nCV + Dt + nC p DT
Hence, T2 = 300 ´ 2 ´ 3 = 1800 K = nCV (2T0 - T0) + nC p + ( 4T0 - T0)
= 1800 - 273 = 1527°C 3R 5R
For monoatomic gas CV = and C p =
89. For one mole of an ideal gas, 2 2
pV 3 RT0 æ5Rö
= R = gas constant for one mole \ H =n +nç ÷ 2T
T 2 è 2 ø 0
= 2 cal mol -1 K -1 3R
= nT0 + 5nRT0
2
90. Change in internal energy of gas is equal to the heat produced 13 13
due to current flowing = nRT0 = p0V0
2 2
i. e. , dU = I 2Rt = 12 ´ 100 ´ (5 ´ 60) = 30 , 000 J = 30 kJ
Telegram @unacademyplusdiscounts

15 Oscillations
JEE Main MILESTONE
< Periodic Motion < Oscillations of Spring Combination
< Periodic Functions < Restoring Force and Force Constant
< Simple Harmonic Motion < Free, Forced, Damped and Resonant Vibrations
< Energy in simple Harmonic Motion

15.1 Periodic Motion


A motion that repeats itself over and over again after a regular interval of time is
called a periodic motion. The regular interval of time after which the periodic motion
is repeated is called its time period. Revolution of the earth around the sun, rotation
of the earth about its axis are common examples of periodic motion.

Oscillatory Motion In our daily life, we come across


A special type of periodic motion in which a particle moves to and fro (back and
the various types of motion such
forth or up and down) about a fixed point after regular interval of time is termed as as rectilinear and periodic motion.
oscillatory or vibratory motion. The fixed point about which the body oscillates is Such motions do not repeat with
called mean position or equilibrium position. Thus, a periodic or bounded motion of a time and are called non-repetitive
body about a fixed point is called an oscillatory or vibratory motion. Examples of motions.
oscillatory motion are simple pendulum, spring pendulum, etc.
A uniform circular motion and
A body that undergoes oscillatory motion is always having a stable equilibrium orbital motion of a planet around
position (where net force on the body is zero). As the body is displaced from its the sun. Such motions repeat after
mean/equilibrium position, a force (torque) comes into existence, which tends to
a definite interval of time are
bring the body back to the equilibrium position, this force or torque is termed as
called periodic motions.
the restoring force (torque).
As the object comes back from the displaced position to initial mean equilibrium
position under the action of restoring force (torque), it acquires some kinetic energy
and hence, overshoots to the other side, as soon as it crosses the mean position, the
direction of restoring force (torque) reverses and the object slows down and stops
after travelling some distance.
As restoring force (torque) is still acting, the object will approach mean position
and move to the other side and this motion continues. The maximum
displacement of the particle from the mean position, where the object stops
momentarily, is termed as the amplitude of oscillation.

Note There is not significant difference between oscillations and vibrations. When to end fro motion of
the body about a fixed position has small frequency, we call it oscillation such as the oscillation of a
simple pendulum. When to end fro motion of the body about a fixed position has high frequency. We call
it oscillation in vibrations such as oscillation of a musical instrument.
Telegram @unacademyplusdiscounts

616 JEE Main Physics

The period of the first term is a multiple of the periods of the last two
15.2 Periodic Functions terms. Therefore, the smallest interval of time after which the sum of
the three terms repeats is T0 and thus the sum is a periodic function
Periodic functions are those functions which are used to 2p
represent periodic motion. with a period .
w
A function f ( t ) is said to be periodic, if
Sample Problem 2 The period of the following function of
f ( t) = f (t + T ) = f (t + 2 T ) …(i) time is sin2 wt.
Q sine and cosine functions are example of periodic w 2w
(a) (b)
functions. p p
p p
\ the particle performing a periodic motion must return to (c) (d)
w 2w
its initial position after one period of motion.
When T is the period of this periodic motion, then for Interpret (a) Given, sin2 wt
periodic motion, Using the trignometric identity
y = A sin wt = A sin w ( t + T ) …(ii) cos 2q = 1 - 2 sin 2 q
1 1
and x = A cos wt = A cos w ( t + T ) …(iii) We have, sin 2 wt = - cos 2 wt
2 2
But the value of sine or cosine functions repeat after a p
The function is periodic having a period T = . It also represents a
period of 2 p radian. w
1
\ w ( t + T ) = wt + 2 p harmonic motion with the point of equilibrium occuring at
2
or wT = 2 p …(iv) instead of zero.
or w = 2 pn …(v)
Consider a linear combination of sine and cosine Sample Problem 3 The following figure depicts circular
functions are given as motion. The radius of the circle, the period of revolution, the
initial position and sense of revolution are indicated in the
x = f ( t ) = A sin wt + B cos wt
figure.
Taking, A = R cos f and B = R sin f y
Then, x = R cos f sin wt + R sin f cos wt = R sin (wt + f ) …(vi) P
P (t = 0)
It represents a period function of time period T and
amplitude R, T = 4s 45° x
2 2
O
where R= A +B
and tan f = B / A.
The combination of any number of periodic functions will The simple harmonic motion of the x-projection of the radius
also be periodic one, whose time period will be minimum vector of the rotating particle P is as follows
of the periodic functions used in the combinations. æ 2p pö æp 2p ö
(a) x (t ) = A cos ç t+ ÷ (b) x (t ) = A cos ç t - ÷
è 4 4ø è4 4ø
Sample Problem 1 A function of time is represented as æ 2p pö æp pö
follows sin wt + cos 2 wt + sin 4 wt (c) x (t ) = A sin ç t+ ÷ (d) x (t ) = A sin ç t - ÷
è 4 2ø è4 2ø
The motion represented by it is
(a) non-periodic p
Interpret (a) At, t = 0, OP makes an angle of 45° = rad with
(b) periodic 4
2p
(c) both non-periodic and periodic the (positive direction) of x-axis. After time t, it covers an angle t
(d) data insufficient T
2p p
in the anti-clockwise sense, and makes an angle of t + with
Interpret (b) This is an example of a periodic motion. It can be T 4
noted that each term represents a periodic function with different the x-axis.
angular frequency. Since period is the least interval of time after The projection of OP on the x-axis at time t is given by
2p æ 2p pö
which a function repeats its value sin wt has a period T0 = x (t ) = A cos ç t+ ÷
w èT 4ø
p T0 æ 2p pö
cos 2wt has a period = For, T = 4 s x (t ) = A cos ç t+ ÷
w 2 è 4 4ø
2p T0
and sin 4wt has a period = p
4w 4 which is a SHM of amplitude A, period 4 s and an initial phase = .
4
Telegram @unacademyplusdiscounts

Oscillations 617

2. A simple harmonic motion may also be considered as


15.3 Simple Harmonic Motion the projection of a uniform circular motion on any
diameter of the circle. As shown in figure, let us
Simple harmonic motion (SHM) is that type of oscillatory
consider uniform circular motion in a plane with
motion in which the particle moves to and fro about a
constant angular velocity w and let at an instant t the
fixed point under a restoring force (or torque) whose particle executing circular motion be at point P
magnitude is directly proportional to its displacement (or making an angle q = wt from X-axis, then the
angular displacement). projection of this circular motion along a diameter YY ¢
Y′ ω is given by
P
N
ωt y = A sin wt
y A and projection along diameter XX¢ is given by
ωt
X′ x M X x = A coswt
O
Such a motion is simple harmonic motion. The radius
A of the reference circle is the maximum value of
displacement to and fro about the mean position O
Y and is known as the amplitude of SHM.
1. A simple harmonic motion may be mathematically
expressed by a single sinusoidal (sine or cosine)
function of time.

Terms Related to SHM


1/ 2
1. Displacement Displacement of a particle in the case of æ x2 ö
SHM is always measured from mean position. = Aw ç 1 - 2 ÷
è A ø
(a) If the particle is at the mean position at t = 0, then
displacement x = A sin wt. = w A2 - x 2
(b) If the particle is at the extreme position at t = 0, then (a) At mean position ( x = 0) velocity is maximum, i. e.,
x = A cos wt . vmax = Aw .
(c) In general x = A sin (wt + f), where f is the initial phase (b) At extreme positions ( x = ± A), velocity is zero, i. e.,
or epoch. v = Aw.

2. Time period The time taken by the particle to complete 5. Acceleration Acceleration is
one oscillation is called time period of oscillation. It is dv
a= = - Aw2 sin (wt + f )
denoted by T. dt
We know that cosine or sine function repeats itself every = - w2x
time the angle wt increases by 2p. Thus, the displacement
(a) Acceleration is zero at the mean position, i. e., a = 0, at
2p
of the particle repeats itself after a time interval, T = . x = 0.
w
(b) Acceleration is maximum at extreme position, i. e.,
3. Frequency The number of oscillations made by the amax = - w22 A at x = A
particle in one second is called frequency of oscillations. It
1 w The above expressions also indicate that in SHM, the
is denoted by n or n. Thus, n = = . acceleration is proportional and opposite to the
T 2p
displacement.
w being the angular frequency of the oscillating particle.
6. Phase relationship between displacement, velocity and
4. Velocity acceleration of SHM
As, x = A sin (wt + f )
dx As we have seen that
v= = Aw cos (wt + f ) x = A sin (wt + f )
dt
= Aw [1 - sin2 (wt + f )]1/ 2 v = Aw cos (wt + f )
Telegram @unacademyplusdiscounts

618 JEE Main Physics

æ pö v
= Aw sin ç wt + f + ÷
è 2ø
+Aω
and a = - Aw2 sin (wt + f )
(ii)
= Aw2 sin (wt + f + p ) t

Thus, we conclude that in SHM, particle velocity is ahead –Aω


dx
in phase by p/2 as compared to the displacement and v = dt = –Aω cos ωt
acceleration is further ahead in phase by p/2. a
In figure, x, v and a as functions of time are illustrated.
x + ω2A
(iii)
+A t
(i)
t – ω2A
dv
a = dt = – 2 At sin t
–A
x = A sin ωt

Sample Problem 4 The periodic time of a body executing \ p = - Ap sin ( p ´ 0 + f) = - Ap sin f …(i)
SHM is 2 s. After how much interval from t = 0, will its or 1 = -A sin f
displacement be half of its amplitude? or A sin f = -1
(a) 1/5 s (b) 1/6 s
Dividing Eq. (ii) by (i), we get
(c) 1/10 s (d) 1/8 s
p æ pö 3p
A tan f = -1 = - tan = tan ç p - ÷ = tan
Interpret (b) Given, T = 2 s,t = ? ; x = 4 è 4ø 4
2
2p 3p
Now, x = A sin wt = A sin t or f=
T 4
A 2p
\ = A sin t = A sin pt Identifying Linear SHM
2 2
1 p In linear SHM, the acceleration a and displacement x of
Þ sin pt = sin 30° = sin
2 6 the system are related by an equation of the form
p 1 a = - ( a positive constant ) x
or pt = or t = s
6 6
which says that the acceleration is proportional to the
displacement from the equilibrium position, but is in an
Sample Problem 5 A particle in SHM is described by the
opposite direction.
displacement function.
x = A cos ( wt + f), w = 2p /T Also in linear SHM, the force and displacement are related
by
If the initial (t = 0) position of the particle is 1 cm and its initial
velocity is p cm s-1 , what is the initial phase angle? (The F = - ( a positive constant ) x
-1 which says that the force is proportional to the
angular frequency of the particle is p s .)
(a) 3p / 4 (b) 2p / 4 displacement but is in an opposite direction.
(c) 5p / 4 (d) 7p / 4
Differential Equations of SHM
Interpret (a) Here, at t = 0 , x = 1 cm and v = p cms-1; f = ?;
For linear SHM,
w = p s -1
d 2x
Given, x = A cos ( wt + f) + w2x = 0
dt 2
1 = A cos ( p ´ 0 + f)
For angular SHM,
1 = A cos f …(i)
d 2q
dx + w2q = 0
Velocity, v = = - Aw sin ( wt + f) dt 2
dt
Telegram @unacademyplusdiscounts

Oscillations 619

In SHM, F = - kx or a = - w2x, i. e. , F-x graph or a-x graph is a 2. Potential energy This is an account of the
straight line passing through the origin with negative displacement of the particle from its mean position.
1 1
slope. The corresponding graphs are shown below U = mw2 x2 = mA2w2 sin 2 (wt + f)
F a 2 2
Thus, potential energy has its minimum at the centre
( x = 0) and increases as the particle approaches either
F = – kx a = – ω2x extreme of oscillation ( x = ± A).
x x Thus, total energy = kinetic energy + potential energy
1
or E= mw2 A2
2
Obviously, the total energy is constant and is
(a) (b)
proportional to the square of amplitude (A) of motion.
Slope = – k Slope = – ω2
Figures show the variations of total energy (E),
potential energy (U) and kinetic energy (K) with
Dynamics of SHM displacement (x).
We have seen that in SHM, a = - w2x and we know that Total energy = U (t) + K (t)
E
F = ma

Energy
\ F = -mw2x (a)
U(t)
K(t)
or F = - kx
2
where, k = mw = force constant. O
T/ 2 T
t

Thus, in SHM the restoring force is proportional and


1 U, K
opposite to the displacement. E = kA2 1
U = kx2
2 2
k 2p
Moreover, w= = (From k = mw2)
m T
(b) 1
m K= k(A2 – x2)
Þ T = 2p 2
k
1 k
or frequency, v= – O +A
2p m
Displacement

15.4 Energy in Simple Harmonic Sample Problem 6 A particle executes SHM of amplitude
A. At what distance from the mean position is its KE equal to its
Motion PE?
(a) 0.71 A (b) 0.61 A
A particle executing SHM possesses two types of energy
(c) 0.65 A (d) 0.8 A
If a particle executes SHM, its kinetic energy changes into
1
potential energy and vice-versa keeping total energy Interpret (a) As, KE = mw2 ( A2 - x2)
2
constant (if friction of air is neglected).
1
1. Kinetic energy This is an account of the velocity of the and PE = mw2x2
2
particle.
1 As, KE = PE
K = mv2 1 1
2 \ mw ( A - x2) = mw2x2
2 2

1 1 2 2
= mA2w2 cos2 (wt + f ) = mw2 ( A2 - x 2 )
2 2 or A2 - x2 = x2
From this expression, we conclude that kinetic or x2 = A2/ 2
energy is maximum at the centre (x = 0) and zero at or x = A / 2 = 0.71 A
the extremes of oscillation (x = ± A).
Telegram @unacademyplusdiscounts

620 JEE Main Physics

Angular Simple Harmonic Motion Identifying Angular SHM


If the angular displacement of the body at an instant is q, When a system undergoes simple harmonic motion, its
the resultant torque acting on the body in angular SHM angular acceleration a and angular displacement q are
should be related by
t = -kq a = - (a positive constant) q
If the moment of inertia is I, the angular acceleration is which says that the angular acceleration a is proportional
t k to the angular displacement q from the equilibrium
a= =- q position but tends to rotate the system in the direction
I I
opposite to the displacement.
d 2q
or = - w2q …(i) Also in angular SHM, the torque t in terms of the angular
dt 2
displacement q is given by
k
where, w= t = - (a positive constant) q
I
It says that the torque t is proportional to the angular
The Eq. (i) may be integrated in the similar manner and we
displacement q from the equilibrium position but tends to
shall get
rotate the system in an opposite direction.
q = q0 sin (wt + d) ...(ii)
where, q0 is the maximum angular displacement on either Sample Problem 7 If two SHMs are represented by
side.
equations
æ pö
The angular velocity at time t is y1 = 10 sin ç3pt + ÷ and y2 = 5 [sin 3pt + 3 cos 3pt],
è 4ø
dq then the ratio of their amplitudes is
w= = q0w cos (wt + d) ...(iii)
dt (a) 1 : 2 (b) 2 : 1 (c) 1 : 1 (d) 1 : 3

Interpret (c) Here, y1 = 10 sin æç3 pt + p ö÷


The time period of oscillation is è 4ø
2p I But y 2 = 5 (sin 3p + 3 cos 3pt ) does not appear to be a single
T = = 2p ...(iv)
w k compact function of time t. We therefore, assume that
and the frequency of oscillation is 5 = A cos f
1 1 k and 5 3 = A sin f
n= = ...(v)
T 2p I Thus, A = (5) 2 + (5 3) 2 = 10
p
k and tan f = 3 or f =
The quantity w = is the angular frequency. 3
I
then, y 2 = A cos f sin 3pt + A sin f cos 3pt
Energy in Angular SHM = A sin (3pt + f)
æ pö
1 2 = 10 sin ç3pt + ÷
The potential energy is U = kq è 3ø
2
Now, we find that A1 = 10 and A2 = 10
1
= Iw2q2 or A1 : A2 = 10 : 10 = 1 : 1
2
1 2 Sample Problem 8 On an average a human heart is found
The kinetic energy is K = Iw
2 to beat 75 times in a minute. It’s time period is [NCERT]
Total energy, E =U + K (a) 0.2 s (b) 0.4 s
1 1 (c) 0.8 s (d) 1 s
= Iw2q2 + Iw2
2 2 75
Interpret (c) The beat frequency of human heart =
From q = q0 sin (wt + d) we have, 1min
1 1 75
E = Iw2q20 sin2 (wt + d) + Iq20w2 cos2 (wt + d) = = 1.25 s–1 = 1.25 Hz
2 2 60
1 1
= Iw2q20 Time period T = = 0.8 s
2 1.25
Telegram @unacademyplusdiscounts

Oscillations 621

m (k1 + k2 )
Spring Block System T = 2p = 2p
ks k1k2
Let a mass m be attached to the free end of a massless
spring of spring constant (also known as force constant or
spring factor or stiffness) k, with its other end fixed to a k1 k1
rigid support. If the mass be displaced through a distance k1 k2 m
x and then released, a linear restoring force F = - kx acts
k2 k2
on the mass due to elastic nature of the spring. Under the
action of this restoring force, the system oscillates in SHM
whose time period is given by m
m
(a) (b) (c)

S F (ii) If in a spring pendulum two springs are joined in


m parallel arrangement as shown in Fig. (b) and (c), then
m
x kp = k1 + k2, and hence, T = 2p
k1 + k2

m These rules are applicable for horizontal motion of


T = 2p spring mass systems too.
k
(iii) The force constant of a spring is inversely proportional
1 k 1
Frequency, n= to its length, i. e., k µ . Thus, if we change the length of a
2p m l
spring, the time of oscillation of spring pendulum will
Spring Pendulum change. As an example, if a spring is cut into two parts
of equal length and a pendulum is prepared by using
A point mass suspended from a massless one part of spring and some mass m, then the new
(or light) spring constitutes a spring pendulum. time period will be
If the mass is once pulled downwards so as to m T
stretch the spring and then released, the system k T ¢ = 2p =
2k 2
oscillates up and down about its mean position
simple harmonically. Time period and (iv) If two masses m1 and m2 are connected by a spring as
shown in figure and the arrangement is made to
frequency of oscillations are given by
m oscillate on a horizontal surface, then time period is
m given by
T = 2p
k k
m1 m2
1 k
or n=
2p m
m
T ¢ = 2p
If the spring is not light but has a definite mass m s , then it k
can be easily shown that period of oscillation will be m1m2
where, m= = reduced mass
m m1 + m2
m+ s
T = 2p 3
k Sample Problem 9 A block with a mass of 2 kg hangs
without vibrating at the end of a spring of spring constant
800 Nm–1, which is attached to the ceiling of an elevator. The
elevator is going upwards with an acceleration g/3. At a certain
15.5 Oscillations of Spring instant, the acceleration suddenly ceases and elevator starts
Combination moving with constant velocity. What is the angular frequency
of oscillation of block when the acceleration ceases?
(i) If a spring pendulum is constructed by using two (a) 12 rad s-1 (b) 20 rad s-1
springs in series and a mass m as shown in Fig. (a), the
(c) 21 rad s-1 (d) 19 rad s-1
resultant spring constant of the combination is given
by k 800
Interpret (b) Angular frequency, w = = = 20 rads–1
1 1 1 k + k2 m 2
= + = 1 , and hence
ks k1 k2 k1k2
Telegram @unacademyplusdiscounts

622 JEE Main Physics

Sample Problem 10 In the above example by what Sample Problem 12 A body oscillates with SHM
amount is the spring stretched during the time when the according to the equation (in SI units)
elevator is accelerating? æ pö
x = 5 cos ç2 pt + ÷
(a) 2.3 cm (b) 3.3 cm (c) 5.3 cm (d) 4.4 cm è 4ø
Interpret (b) When the elevator is accelerating upwards with at t = 1.5 s, the displacement is
an acceleration a = g /3, the equation of motion of the block is (a) –1.53 m (b) –2.1 m
æ gö (c) –3.535 m (d) – 4.23 m
çky - mg = ma = m ÷
è 3ø
Interpret (c) The angular frequency w of the body = 2p s-1 and
its time period T = 1s. At t = 1.5 s,
æ pö
displacement = (5 m) cos ç2p s–1 ´ 1.5 s+ ÷
k = 800 Nm–1 è 4ø
æ pö
= (5 m) cos ç3 p + ÷
è 4ø
2 kg = -5 ´ 0.707
= -3.535 m
mg 4
Þ ky = mg + = mg
3 3 Sample Problem 13 Two identical springs of spring
4 mg 4 ´ 2 ´ 10 constant k are attached to a block of mass m and to fixed
Þ y= = = 0.033 m or 3.3 cm
3k 3 ´ 800 supports as shown when the mass is displaced from its
equilibrium position on either side, it executes SHM, the period
Sample Problem 11 In the above example, what is the of oscillation is
amplitude of oscillation? (Take g = 10 ms-2) m
(a) 0.1 cm (b) 0.7 cm k k
(c) 0.8 cm (d) 0.5 cm

Interpret (c) In equilibrium, when the elevator has no


acceleration, the equation of motion is m 2m
(a) 2p (b) 2p
ky 0 = mg k k
mg 2 ´ 10 m m
Þ y0 = = = 0.025 m or 2.5 cm (c) 2p (d) 2p
k 800 3k 2k
\Amplitude of oscillation, Interpret (d) Let the mass be displaced by a small distance x to
A = y - y 0 = 3.3 - 2.5 = 0.8 cm the right side of the equilibrium position. Under this situation, the
spring on the left side elongated by a length equal to x and that on
the right side gets compressed by the same length. The forces acting
15.6 Restoring Force and Force on the mass are then [NCERT]

Constant F1 F2

From Newton’s second law of motion and the expression O


for acceleration of a particle undergoing SHM, the force
acting on a particle of mass m in SHM is x
2
F ( t ) = ma = -mw x ( t ) F1 = - kx
i. e. , F ( t ) = - kx ( t ) (force exerted by the spring on the left side, trying to pull the mass
where , k = mw2 towards the mean position)
k Fl = - kx
or w=
m (force exerted by the spring on the right side, trying to push the mass
Force is always directed towards mean position and is towards the mean position)
also called restoring force and k is called the spring F2 = - kx
constant, its value is governed by the elastic properties of The net force, F acting on the mass is then
the spring. A stiff spring has large k and a soft spring has F = -2 kx
small k.
Telegram @unacademyplusdiscounts

Oscillations 623

Hence, the force acting on the mass is proportional to the Note At maximum displacement kinetic energy is zero and hence the
displacement and is directed towards the mean position, therefore total energy of the system is equal to potential energy. The result is also in
the motion executed by the mass is simple harmonics. The time conformity with the principle of conservation of energy.
period of oscillation is
m Sample Problem 15 A 5 kg collar is attached to a spring
T = 2p
2k of spring constant 500 Nm -1. It slides without friction over a
horizontal rod. The collar is displaced from its equilibrium
Sample Problem 14 A block whose mass is 1 kg is position by 10 cm and released. The maximum speed is
fastenedto a spring. The spring has a (a) 1 ms-1
m
spring constant of 50 Nm -1. The block is (b) 5 ms-1
k
pulled to a distance x = 10 cm from its (c) 10 ms-1
equilibrium position at x = 0 on a (d) 20 ms-1
frictionless surface from rest at t = 0. The
total energy of the block when it is 5 cm Interpret (a) The velocity of the collar executing SHM is given
away from the mean position is by
v (t ) = - Aw sin ( wt + f)
(a) 0.19 J (b) 0.0625 J
The maximum speed is given by
(c) 0.09 J (d) 0.25 J
v m = Aw
Interpret (d) The block executes SHM to angular frequency is Given, A = 10 cm = 0.1 m,
k 50
w= = = 7.07 rad s–1 k
m 1 w=
m
Its displacement at any time t is given by
500
x (t ) = 0.1 cos (7.07 t ) v m = 0.1 ´
5
Therefore, when the particle is 5 cm away from the mean position,
v m = 1 ms–1
we have
0.05 = 0.1 cos (7.07 t ) it occurs at x=0
or cos (7.07 t ) = 0.5
3
sin(7.07 t ) = = 0.866 Check Point 1
2
Then, the velocity of the block at x = 5 cm is 1. Why a point on a rotating wheel cannot be considered as
executing SHM?
= 0.1 ´ 7.07 ´ 0.866 ms–1
2. What is the (a) distance moved (b) displacement of a body
= 0.61ms–1
executing SHM in a time equal to its period if its amplitude is A?
Hence, the kinetic energy of the block 3. A man is standing on a platform moving up and down as a
1 SHM. Will there be any change in his weight as recorded by a
KE = mv 2
2 weighing machine on the platform?
1 4. Will a pendulum clock gain or loose time, when taken to the
= [1kg ´ (0.6123) 2] = 0.19 J
2 top of a mountain?
and the potential energy, 5. Two simple pendulums of unequal lengths meet each other at
1 1 the mean position while oscillating. What is their phase
PE = kx2 = (50 ´ 0.05 ´ 0.05) difference?
2 2
= 0.0625 J 6. When will the motion of a simple pendulum be simple
harmonic?
\ Total energy = KE + PE = 0.25 J
Telegram @unacademyplusdiscounts

624 JEE Main Physics

Hot Spot Time Period


of Simple Pendulum
A small bob of massmtied to an in extensible massless string of length L. The other end of the string is fixed to a support
in the ceiling. The bob oscillates in a plane about the vertical line through the support.
Rigid
support

The various force acting on the system are as shown in the diagram.
Let q be the angle made by the string with the vertical. When the bob is If q is small, sin q can be approximated by q. Eq. (iv) reduces to
at the mean position q = 0. There are only two forces acting on the mgL
bob, the tension T along the string and the vertical force due to gravity a=- q ...(vi)
I
( = mg ). Resolving force ( = mg ) into the component mg cos q along the
Also, a = - w2 q …(vii)
string and mg sin q perpendicular to it.
Comparing Eqs. (vi) and (vii), we get
mgL
θ L w=
I
T
2p I
m and time period T is T = = 2p …(viii)
w mgL
mg sin θ mg cos θ
mg Since, the string of the simple pendulum is massless, the moment
of inertia I = mL2 , putting this value in equation above, we get
Since the motion of the bob is along a circle of length L and centre at
the support point, the bob has a radial acceleration ( w2 L) and also a mL2 L
T = 2p T = 2p
tangential acceleration, the later arises since motion along the arc of mgL g
the circle is not uniform. The radial acceleration is provided by the net
radial force T - mg cos q, while the tangential acceleration is provided This expression gives the time period of a simple pendulum.
by mg sin q.
Sample Problem 16 A ball is suspended by a thread of
Torque t about the support is entirely provided by the tangential length l at the point O on the wall PQ which is inclined to
component of force the vertical by a. The thread with the ball is displaced by a
t = -L ( mg sin q) …(i) small angle b away from the vertical and also away from the
This is a restoring torque that tends to reduce angular displacement. wall. If the ball is released, assuming the collision to be
From Newton’s law of rotational motion perfectly elastic the period of oscillation of the pendulum
t = Ia …(ii) for b > a is
where I is the moment of inertia of the system about the support and a L é bù
is the angular acceleration. Thus, (a) ê p + 2 cos-1 ú
g ë aû
Ia = - mg sin q × L …(iii)
P O Q
mgL L é aù
or a=- sin q …(iv) (b) p + 2 sin -1 ú
I g êë bû α β
Since, displacement q is small, sin q is expressed as g é aù
(c) ê p + 2 tan -1 ú
q3 q5 L ë bû
sin q = q - + +¼ …(v)
3! 5 !
g é -1 b ù C
êë2p + 2 tan a úû
where, q is in radians (d ) A
L
B
Telegram @unacademyplusdiscounts

Oscillations 625

Interpret (b) When b > a times taken by pendulum from B to C Using, q = q0 sin wt
and C to B is a = b sin wt
T 1 L L 1 æ aö
t1 = = ´ 2p =p or t = sin -1 ç ÷
2 2 g g w èbø

2 æ aö L é aù
sin -1 ç ÷ Time period of motion, T = t1 + t 2 = p + 2 sin -1 ú
and t2 = 2 t =
w èbø g êë bû

Sample Problem 17 Two physical pendulums perform Interpret (a) The time period of a simple pendulum is given by
small oscillations about the same horizontal axis with L gT 2
frequencies w1 and w2. Their moments of inertia relative to the T = 2p Þ L=
g 4 p2
given axes are I1 and I2 respectively. In the equilibrium
positions, they are joined rigidly. The frequency of small The time period of a simple pendulum which ticks seconds is 2 s.
oscillations of the combined pendulum is Therefore, for g = 9.8 ms–2 and T = 2 s, L is
I1w12 + I2w22 I1 + I2 9.8 ´ 4
(a) (b) L= » 1m
I1 + I2 I12w1 + I22w2 4 p2

I1w12 - I2w22 I1 - I2
(c) (d) Sample Problem 19 The acceleration due to gravity on
I1 - I2 I12w1 - I22w2 the surface of the moon is 1.7 ms–2. What is the time period of a
simple pendulum on the moon, if its time period on the earth is
Interpret (a) When the pendulums are rigidly joined and set 3.5 s? (g on the earth 9.8 ms–2).
to oscillate, each exerts a torque on the other. These torques are (a) 8.4 s (b) 8.2 s
equal and opposite, thus
(c) 7.4 s (d) 6.4 s
I1a = - w12I1q + G …(i)
Interpret (a) Given, g m = 1.7 ms–2,
I2a = - w 22 I2q - G …(ii)
g e = 9.8 ms–2,Tm = ? ; T = 3.5 s
Adding Eqs. (i) and (ii), we get
l
(I1w12 + I2w22) q As, Te = 2p
a=- ge
I1 + I2
Comparing with a = - w2q, we get and Tm = 2p
l
gm
I1w12 + I2w22
Frequency, w= Tm ge
I1 + I2 \ =
Te gm
Sample Problem 18 Length of a simple pendulum which ge
or Tm = Te
ticks seconds is gm
(a) 1 m (b) 2 m
(c) 3 m (d) 4 m 9.8
= 3.5 = 8.4 s
1.7
Telegram @unacademyplusdiscounts

626 JEE Main Physics

Some Important Points


1. The time period of a simple pendulum is (vi) Finally, the graph between T and l/g is also a straight line.
T = 2p l/g Y
1
Þ T µ l or T µ T2
g
l
Þ T µ
g
O X
Using these relations. We may conclude l/g

(i) The graph between T 2 and l is a straight line. 2. In the case of water oscillating in a U-tube
Y

T2

h
O X
l

(ii) The graph between T and l is a parabola.


æ hö
T = 2p ç ÷
Y è gø

T where, h is the height of liquid column in each limb.


3. When a ball of mass m is made to oscillate in the neck of an air
chamber having volume V and neck area A, then

T = 2 p æç 2 ö÷
mV
O l
X è pA ø

4. When a pendulum is kept in a car which is sliding down, then


(iii) The graphs l-T and l-T 2 intersect at T = 1s.
æ l ö
T = 1s T = 2p ç ÷
Y è g cos q ø
l-T
where, q is the angle of inclination.
5. If a simple pendulum oscillates in a non-viscous liquid of density
r, then its time period is
é l ù
l-T 2 T = 2p ê æ ú

O X ê ç1 - ÷ g ú
êë è rø úû
(iv) The graph between T 2 and 1/g is a straight line.
r = density of suspended mass.
Y
6. If the mass m attached to a spring oscillates in a non-viscous liquid of
T2 density s, then its time period is
1/2
ém æ s ö ù
T = 2 p ê ç1 - ÷ ú
ë k è r øû
where, k = force constant.
O X
1/g 7. If a small ball is rolling down in hemispherical bowl. Time period,
R -r
2
(v) The graph between T and g is a rectangular hyperbola. T = 2p
g
Y
where, R = radius of the bowl and
T2 r = radius of the ball
8. For a body executing SHM in a tunnel dug along any chord of earth.
Re
Time period, T = 2p = 84.6 min
O X g
g
where Re is the radius of earth.
Telegram @unacademyplusdiscounts

Oscillations 627

9. If the time period of simple pendulum is 2s, then it is called as second’s In a freely falling lift, geff = 0 and T = ¥, i .e ., the pendulum will not
pendulum. oscillate.
10. If the simple pendulum is placed in some non-inertial frame of 14. If in addition to gravity one additional force F (e .g ., electrostatic force
reference like an accelerated lift, g is replaced by geff whose value can Fe ) is also acting on the bob, then in that case
be computed by considering the inertial force. In these cases, the F
geff = g +
equilibrium position may also change. m
Here, m is the mass of the bob.
11. If the length of simple pendulum is very large, then g can’t be
taken along vertical direction. Torsional pendulum In a torsional pendulum, an object is
suspended from a wire. If such a wire is twisted due to elasticity, it
1
In this case, T = 2 p exerts a restoring torque t = C q
g æç + ö÷
1 1
èl Rø
where, R = Radius of length of the pendulum. l

12. If temperature of system changes, then time period of simple


pendulum changes due to change in length of the simple pendulum.
13. If a simple pendulum is in a carriage which is accelerating with an θ
acceleration a, then
In this case, time period is given by
g eff = g - a
l
e .g ., if the acceleration a is upwards, then T = 2p
C
l
| geff | = g + a and T = 2p where, I = moment of inertia of the object
g+a phr n
C = torsional constant of wire =
If the acceleration a is downwards, then (g > a ) 2l
| geff | = g - a h = modulus of elasticity of wire
l r = radius of wire
and T = 2p l = length of wire
g -a

If the acceleration a is in the horizontal direction, then


|g eff | = a 2 + g 2

Physical Pendulum Þ
d 2q mgdq
=
dt 2
P
When a rigid body of any shape is I
capable of oscillating about an axis θ d 2q
[As q and are oppositely directed]
(may or may not be passing through it), d dt 2
it constitutes a physical pendulum. C
d 2q
Comparing with the equation = - w2q, we get
Consider an arbitrary shaped body C mg dt 2
whose centre of gravity is at C, being
mgd
pivoted about a point P at distance d w=
I
from C. When P and C are in the same vertical line, the
body is in its equilibrium position. When the body is l
T = 2p
displaced (rotated) slightly by an angle q about point P, mgd
then gravity force will provide the necessary restoring ● The simple pendulum whose time period is same as that
torque to execute oscillations. of a physical pendulum is termed as an equivalent
t = mg ´ d sin q simple pendulum.
For small, q, t = mgd ´ q I
T = 2p
mgd
i. e. , body will perform SHM.
Þ Ia = mgdq l
= 2p
g
where I is moment of inertia of the body about a
horizontal axis passing through P.
● The length of an equivalent simple pendulum is given by
I
l=
md
Telegram @unacademyplusdiscounts

628 JEE Main Physics

In these oscillations, the amplitude of oscillation


15.7 Free, Forced, Damped and decreases exponentially and hence, energy also decreases
exponentially.
Resonant Vibrations
If the velocity of an oscillator is v, the damping force
Free Vibrations Fd = - bv
If a given body is once set into vibrations and then let free where, b = damping constant.
to vibrate with its own natural frequency, the vibrations Resultant force on a damped oscillator is given by
are said to be free vibrations. The natural frequency of free
F = FR + FD = - kx - bv
vibrations depends on the nature and structure of the
md 2x bdx
body and in ideal situation, the amplitude, frequency and or + + kx = 0
dt 2 dt
the energy of the vibrating body remain constant.
Displacement of a damped oscillator is given by
Forced Vibrations x = xme- bt / 2m sin (w¢t + f )
The vibrations in which a body oscillates under the effect
w¢ = w20 - (b /2 m ) 2
of an external periodic force, whose frequency is different
from the natural frequency of oscillating body are called where, w¢ = angular frequency of the damped oscillator
forced vibrations. In forced vibrations, the oscillating body
For a damped oscillator, if the damping is small then the
vibrates with the frequency of external force and
mechanical energy decreases exponentially with time
amplitude of oscillations is generally small.
1 2 - bt / m
If an external driving force is represented by as E= kxme
2
F ( t ) = F0 cos wd t
The motion of particle is under combined action of Resonant Vibrations
(i) restoring force (-kx)
It is a special case of forced vibrations in which frequency
(ii) damping force (-bv), and
of external force is exactly same as the natural frequency
(iii) driving force F (t )
of oscillator. As a result the oscillating body begins to
Now, ma = - kx - bv + F0 cos wd t vibrate with a large amplitude leading to the resonance
d 2x kx b dx F0 cos wd t phenomenon to occur. Resonant vibrations play a very
or 2
=- - +
dt m m dt m important role in music and tuning of station/channel in
The solution of this equation gives x = x0 sin (wd t + f ) with a radio/TV.
F0/ m
amplitude x0 =
2
æ bw ö Check Point 2
(w20 - w2d ) + ç ÷
èmø
1. How would the period of spring mass system change, when it
w2 - w2d is made to oscillate horizontally and then vertically?
tan q = 0
bwd / m 2. Glass windows may break due to an explosion far away
k Explain, why?
and w0 = = natural frequency
m 3. There are two springs, one delicate and another stout one. For
which spring the frequency of oscillation is more?
Damped Vibrations 4. Water in a U-tube executes SHM. Will the time period for
mercury filled upto the same height in U-tube be lesser or
When a body is set in free vibrations, generally there is a greater than that in case of water?
dissipation of energy due to dissipative causes like 5. Why are soldiers asked to break their steps while marching
viscous drag of a fluid, frictional force, hysteresis, over a bridge?
electromagnetic damping force, etc., and as a result
amplitude of vibration regularly decreases with time.
Such vibrations of continuously falling amplitudes are
called damped vibrations.
Telegram @unacademyplusdiscounts

WORKED OUT
Examples
1
Example 1 Two SHMs are represented by y1 = A sin( wt - f) (a) proportional to (b) independent of a
a
and y2 = B cos( wt - f). The phase difference between the two, is
p p (c) proportional to a (d) proportional to a3 / 2
(a) (b)
2 4 Solution As potential energy U( x ) = k| x|3 , hence maximum
p
(c) p (d) value of potential energy
3
1
Umax = U( a) = mw2a2 = ka2
Solution Here, y1 = A sin ( wt - f) and 2
y 2 = B cos ( wt - f) Þ wµ a
æ p ö 2p 1
= B sin ç wt + - f ÷ As, T= , hence, T µ
è 2 ø w a
A simple comparison of arguments of sine terms shows that phase
æ p ö p Example 4 Two springs of force constants k1 and k2, have
difference is ç wt + - f ÷ - ( wt - f) = .
è 2 ø 2 equal highest velocities when executing SHM. Then the ratio of
their amplitudes (given their masses are equal) will be
Example 2 A mass of Hg is executing SHM which is given k1 k1
(a) (b)
æ pö k2 k2
by x = 6.0 cos ç100t + ÷ cm. What is the maximum kinetic
è 4ø k2 k2
(c) (d)
energy? k1 k1
(a) 3 J (b) 6 J
(c) 9 J (d) 18 J Solution At highest velocities A1w1 = A2w2
A1 w2 k /m k
Solution Here, m =1kg \ = = 2 2= 2 (since, m1 = m2)
A2 w1 k1 /m1 k1
The given equation of SHM is
æ pö
x = 6.0 cos ç100t + ÷ Example 5 Two point masses of 3.0 kg and 1.0 kg are
è 4ø
attached to opposite ends of a horizontal spring whose spring
Comparing it with equation of SHM constant is 300 Nm -1 as shown in adjacent figure. The natural
x = A cos ( wt + f), we have frequency of vibration of the system is
6 k = 300 Nm–1
A = 6.0 cm = m 3 kg 1 kg
100
and w = 100 rad s-1
(a) 4 Hz (b) 3 Hz
1 1 1
Maximum kinetic energy = m (v max ) 2 (c) Hz (d) Hz
2 3 4
2
1 1 é 6 ù m1m2 3 ´1
= m ( Aw) 2 = ´ 1 ´ ê ´ 100 ú =18 J Solution Here reduced mass of the system, m = =
2 2 ë 100 û m1 + m2 3 + 1
= 0.75kg
Example 3 A particle of mass m is executing oscillations
\Vibrational frequency,
about the origin on the x-axis. Its potential energy is
U( x) = k| x|3 , where k is a positive constant. If the amplitude of 1 k 1 300 20 10
n= = = = @ 3 Hz
oscillation is a, then its time period T is 2p m 2p 0.75 2p p
Telegram @unacademyplusdiscounts

630 JEE Main Physics

Example 6 A particle is executing SHM of amplitude 25 cm Example 8 The displacement of a particle executing
and time period 3 s. What is the minimum time required for the periodic motion is given by
particle to move between two points12.5 cm on either side of æt ö
y = 4 cos2 ç ÷ sin(1000t)
the mean position? è2 ø
(a) 0.5 s (b) 1s Find independent constituents of SHMs
(c) 1.5 s (d) 2 s (a) y1 = 2 sin 1000 t , y 2 = sin 1001t , y3 = sin 999 t
Solution Here, A = 25 cm; T = 3 s; (b) y1 = 3 sin 1000 t , y 2 = sin 1000 t , y3 = sin 899 t
(c) y1 = sin 1001t , y 2 = sin 999 t , y3 = 2 sin 1000 t
Let the particle be at the locating -12.5 cm at time t1 and (d) None of the above
+ 12.5 cm at time t 2.

Using the relation,


æ 2pt
x = A cos ç
ö
+ f÷
Solution y = 4 cos2 æç t ö÷ sin(1000t)
è2ø
è T ø
= 2 (1 + cos t ) sin(1000t )
æ 2 p t1 ö
First condition, -12.5 = 25 cos ç + f÷ = 2 sin(1000t ) + 2 cos t sin(1000t )
è 3 ø
= 2 sin(1000t ) + sin (1001t ) + sin(999t )
æ 2 pt 2 ö Thus, the given periodic motion is a combination of three
Second condition, 12.5 = 25 cos ç + f÷
è 3 ø independent SHMs, which are given by
æ 2 p t1 ö -12.5 1 2p y1 = 2 sin 1000t , y 2 = sin 10001t and y3 = 999t
From Eq. (i), cos ç + f÷ = = - = cos
è 3 ø 25 2 3
2 p t1 2p Example 9 As shown in figure, system consisting of
\ +f= massless pulley, a light spring of force constant k and a block of
3 3
mass m. If the block is slightly displaced vertically downwards
or 2 p t1 + 3 f = 2 p from its equilibrium position and released, find the expressing
æ 2 pt 2 ö for frequency of vertical oscillations.
From Eq. (ii), cos ç + f÷
è 2 ø
12.5 1 p
= = = cos
25 2 3
2 pt 2 p
\ + f= P
3 3
or 2 p t 2 + 3f = p
Subtracting Eq. (iv) from Eq. (iii), we get k
2 p (t1 - t 2) = p m
or (t1 - t 2) = p / 2p =1/2 = 0.5 s

Example 7 A man stands on a weighing machine placed on


1 k 1 m
a horizontal platform. The machine reads 50 kg . By means of a (a) n = (b) n =
suitable mechanism, the platform is made to execute harmonic 3p m 3p k
vibrations up and down with a frequency of 2 vibrations per 1 k 2 m
(c) n = (d) n =
second. What will be the maximum reading of the weighing 2p m 3p k
machine? The amplitude of vibration of platform is 5 cm. Solution In equilibrium, due to weight mg spring is stretched
[Take g = 10 ms-2] by y 0 as shown in figure. When further depressed by a small
(a) 64.5 kgf (b) 74.5 kgf distance y, the restoring force will be
(c) 89.5 kgf (d) 95.5 kgf

Solution Here, m = 50 kg, n = 2s-1,


A = 5 cm = 0.05 m
Maximum, acceleration, amax = w2 A m
= (2p n) 2 A = 4p 2n 2A y0
2
æ 22 ö m
= 4 ´ ç ÷ ´ (2) 2 ´ 0.05 = 7.9 ms-2
è7ø
y
Maximum force on the man = m ( g + amax ) = 50 (10 + 7.9) m
= 89500 N = 89.5 kgf
Telegram @unacademyplusdiscounts

Oscillations 631

F = - [k (y + y 0) - mg ] \Time period of oscillation


= [k (y + y 0) - ky 0 ] = - ky 1 2
ml 0
F k I 3
\ Acceleration, a = = y T = 2p = 2p
m m mgl æl ö
mg ç 0 ÷
As acceleration is proportional to the displacement y and opposite è 2ø
to y, the motion will be SHM. Frequency of oscillations will be 2l0
Þ T = 2p
1 k 3g
n=
2p m
Example 11 The bob of a simple pendulum executes SHM
Example 10 A uniform rod of mass m and length l0 is in water with a period t, while the period of oscillation of the
pivoted at one end and is hanging in the vertical direction. The bob is t 0 in air. Neglecting frictional force of water and given
period of small angular oscillations of the rod is æ 4ö
that the density of the bob is ç ÷ ´ 1000 kgm -3. What
è3ø
l0 relationship between t and t 0 is true?
2 l0 O CM
(a) t = t 0 (b) t = 4 t 0
t
(c) t = 2t 0 (d) t = 0
2
4
2l0 l Solution Here density of bob, r = ´ 1000 kgm-3 and density of
(a) T = 3p (b) T = 4p 0 3
3g 3g water, s = 1000 kgm-3
2l0 2l0
(c) T = 4p (d) T = 2p L
3g 3g \In air t 0 = 2p and in water
g
Solution Here the rod is oscillating about an end point O. L L
t = 2p = 2p
Hence, moment of inertia of rod about the point of oscillating is æ sö æ 3ö
g ç1 - ÷ g ç1 - ÷
1 è rø è 4ø
I = ml02
3
L
Moreover, length l of the pendulum = distance from the = 2 ´ 2p = 2t 0
g
oscillation axis to centre of mass of rod = l0 /2
Telegram @unacademyplusdiscounts

Start Practice for


JEE Main
Round I (Topically Divided Problems)

Oscillatory Motion and Simple 6. Starting from y = A sin wt and y = A cos wt


Harmonic Motion with its (a) acceleration lags the displacement by a phase p/4
(b) acceleration lags the displacement by a phase p/2
Characteristics (c) acceleration leads the displacement by a phase p/2
1. The displacement of two particles executing SHM are (d) acceleration leads the displacement by a phase p
represented by equations
7. The displacement of a particle is represented by the
y1 = 2 sin (10 t + q ), y2 = 3 cos 10 t. equation y = sin 3 wt. The motion is [NCERT Exemplar]
The phase difference between the velocity of these (a) non-periodic
particles is (b) periodic but not simple harmonic
(a) q (b) - q (c) simple harmonic with period 2p /w
(c) q + p /2 (d) q - p /2 (d) simple harmonic with period p /w
2. Two pendulums of length 121 cm and 100 cm start 8. A simple harmonic oscillator has and amplitude a
vibrating. At same instant the two are in the mean and time period T. The time required by it to travel
position in the same phase. After how many a
vibrations of the shorter pendulum, the two will be in from x = a to x = is
2
phase at the mean position?
T T
(a) 10 (b) 11 (c) 20 (d) 21 (a) (b)
6 4
3. The displacement of the particle varies with time T T
according to the relation. [NCERT Exemplar] (c) (d)
3 2
y = a sin wt + b cos wt, then
(a) The motion is oscillating but not SHM 9. The piston in the cylinder head of a locomotive has a
(b) The motion is SHM with amplitude a + b stroke (twice the amplitude) of 1.0 m. If the piston
(c) The motion is SHM with amplitude a + b moves with simple harmonic motion with an angular
frequency of 200 rad/min, what is its maximum
(d) The motion is SHM with amplitude a2 + b2
speed? [NCERT]
4. Two pendulums have time period T and 5T/4. They
(a) 100 m/min (b) 200 m/min
start SHM at the same time from the mean position.
What will be the phase difference between them after (c) 300 m/min (d) 50 m/min
the bigger pendulum completed one oscillation? 10. Two points are located at a distance of 10 m and 15 m
(a) 45º (b) 90º (c) 60º (d) 30º from the source of oscillation. The period of
oscillation is 0.05s and the velocity of the wave is
5. The displacement of a particle is represented by
p 300 m/s. What is the phase difference between the
the equation y = 3 cos æç - 2wt ö÷. The motion of the oscillations of two points?
è4 ø
particle is [NCERT Exemplar] p
(a) p (b)
(a) simple harmonic with period 2p/w 6
(b) simple harmonic with period p /w p 2p
(c) (d)
(c) periodic but not simple harmonic 3 3
(d) non-periodic
Telegram @unacademyplusdiscounts

Oscillations 633

11. A horizontal plank has a rectangular block placed on 18. The acceleration d2 x / dt2 of a particle varies with
it. The plank starts oscillating vertically and simple d 2x
displacement x as = - kx
harmonically with an amplitude of 40 cm. The block dt 2
just loses contact with the plank when the later is where k is a constant of the motion. The time period T
momentarily at rest. Then of the motion is equal to
(a) the period of oscillation is 2p/5 s (a) 2pk (b) 2p k
(b) the block weighs double its weight when the plank is at (c) 2 p / k (d) 2 p / k
one of the positions of momentary at rest
(c) the block weighs 1.5 times its weight on the plank half 19. Two linear SHMs of equal amplitude A and angular
way down frequencies w and 2w are impressed on a particle
(d) the block weighs its true weight on the plank, when the along the axes x and y respectively. If the initial
latter moves fastest phase difference between them is p/2, the resultant
12. A body has a time period T1 under the action of one path followed by the particle is
force and T2 under the action of another force, the (a) y2 = x2 (1 - x2 / A2 ) (b) y2 = 2x2 (1 - x2 / A2 )
square of the time period when both the forces are (c) y2 = 4 x2 (1 - x2 / A2 ) (d) y2 = 8x2 (1 - x2 / A2 )
acting in the same direction is
20. A coin is placed on a horizontal platform, which
(a) T12T22 (b) T12T22
undergoes horizontal SHM about a mean position O.
(c) T12 + T22 (d) T12T22 /(T12 + T22 ) The coin placed on platform does not slip, coefficient
13. Motion of an oscillating liquid column in a U-tube is of friction between the coin and the platform is m. The
[NCERT Exemplar]
amplitude of oscillation is gradually increased. The
coil will begin to slip on the platform for the first time
(a) periodic but not simple harmonic
(a) at the mean position
(b) non-periodic
(b) at the extreme position of oscillations
(c) simple harmonic and time period is independent of the
density of the liquid (c) for an amplitude of mg/ w2
(d) simple harmonic and time-period is directly proportional (d) for an amplitude of g/mw2
to the density of the liquid
21. A block is resting on a piston which is moving
14. A particle is acted simultaneously by mutually vertically with SHM of period 1.0 s. At what
perpendicular simple hormonic motions x = a cos wt amplitude of motion will the block and piston
and y = a sin wt. The trajectory of motion of the separate?
particle will be [NCERT Exemplar] (a) 0.2 m (b) 0.25 m
(a) an ellipse (b) a parabola (c) 0.3 m (d) 0.35 m
(c) a circle (d) a straight line pt
22. A particle moves according to the law, x = r cos
.
15. This time period of a particle undergoing SHM is 2
16 s. It starts motion from the mean position. After The distance covered by it in the time interval
2 s, its velocity is 0.4 ms–1. The amplitude is between t = 0 to t = 3 s is
(a) 1.44 m (b) 0.72 m (a) r (b) 2r (c) 3r (d) 4r
(c) 2.88 m (d) 0.36 m 23. A particle is executing SHM of period 24s and of
16. A particle is performing simple harmonic motion amplitude 41 cm with O as equilibrium position. The
along x-axis with amplitude 4 cm and time period minimum time in seconds taken by the particle to go
1.2 s. The minimum time taken by the particle to from P to Q, where OP = - 9 cm and OQ = 40 cm is
move from x = + 2 to x = 4 cm and back again is given (a) 5 (b) 6
by (c) 7 (d) 9
(a) 0.4 s (b) 0.3 s 24. Figure shows the circular motion of a particle. The
(c) 0.2 s (d) 0.6 s radius of the circle, the period, y
17. The acceleration of a particle performing SHM is sense of revolution and the initial P (t = 0)
12 cms–2 at a distance of 3 cm from the mean position. position are indicated on the T = 30 s
Its time period is figure. The simple harmonic B x
motion of the x-projection of the O
(a) 2.0 s (b) 3.14 s
(c) 0.5 s (d) 1.0 s radius vector of the rotating
particle P is
[NCERT Exemplar]
Telegram @unacademyplusdiscounts

634 JEE Main Physics

æ 2pt ö æ pt ö 32. Out the following functions representing motion of a


(a) x (t ) = B sin ç ÷ (b) x (t ) = B cos ç ÷
è 30 ø è 15 ø particle which represents SHM
æ pt p ö æ pt p ö (1) y = sin wt - cos wt
(c) x (t ) = B sin ç + ÷ (d) x (t ) = B ç + ÷
è 15 2 ø è 15 2 ø (2) y = sin 3 wt
æ3p ö
25. A 1.00 ´ 10-20 kg particle is vibrating with simple (3) y = 5 cos ç - 3 wt ÷
è 4 ø
harmonic motion with a period of 1.00 ´ 10-5 s and a
(4) y = 1 + wt + w2t 2
maximum speed of 1.00 ´ 103 m/s. The maximum
(a) Only (1) and (2)
displacement of the particle is
(b) Only (1)
(a) 1.59 mm (b) 1.00 cm
(c) Only (4) does not represent SHM
(c) 10 m (d) None of these
(d) Only (1) and (2)
26. Which one of the following equations does not 33. A large horizontal surface moves up and down in
represent SHM, x = displacement and t = time.
SHM with an amplitude of 1 cm. If a mass of 10 kg
Parameters a, b and c are the constants of motion?
(which is placed on the surface) is to remain
(a) x = a sin bt
continuously is in contact with it, the maximum
(b) x = a cos bt + c
frequency of SHM will be
(c) x = a sin bt + c cos bt
(a) 5 Hz (b) 0.5 Hz
(d) x = a sec bt + c cosec bt
(c) 1.5 Hz (d) 10 Hz
27. A particle executing SHM has a maximum speed of 34. The composition of two simple harmonic motions of
30 cm/s and a maximum acceleration of 60 cm/s2 . The
equal periods at right angle to each other and with a
period of oscillation is [NCERT Exemplar]
phase difference of p results in the displacement of
p
(a) p s (b) s the particle along
2
(a) circle (b) figure of eight
p
(c) 2p s (d) s (c) straight line (d) ellipse
t
35. A horizontal platform vibrates with simple harmonic
28. The bob of a simple pendulum of length L is released
motion in the horizontal direction with a period 2 s. A
at time t = 0 from a position of small angular
body of mass 0.5 kg is placed on the platform. The
displacement. Its linear displacement at time t is
coefficient of static friction between the body and
given by
platform is 0.3. What is the maximum frictional force
L g
(a) X = a sin 2p ´t (b) X = a cos 2p ´t on the body when the platform is oscillating with an
g L amplitude 0.2 m? Assume p2 = 10 = g.
g g (a) 0.5 N (b) 1 N
(c) X = a sin ´t (d) X = a cos ´t
L L (c) 1.5 N (d) 2 N
29. Displacement-time equation of a particle executing
SHM is , x = 4 sin w t + 3 sin (wt + p/ 3 ). Here x is in Energy in SHM
centimetre and t in second. The amplitude of 36. The angular velocity and the amplitude of a simple
oscillation of the particle is approximately pendulum is w and a respectively. At a displacement
(a) 5 cm (b) 6 cm x from the mean position if its kinetic energy is T and
(c) 7 cm (d) 9 cm potential energy is V, then the ratio of T to V is
30. A particle in SHM is described by the displacement (a) ( a2 - x2 w2 ) / x2 w2 (b) x2 w2 /( a2 - x2 w2 )
function x( t) = A cos( wt + f), w = 2p/ T. If the initial (c) ( a2 - x2 ) / x (d) x2 / ( a2 - x2 )
( t = 0) position of the particle is 1 cm, its initial
velocity is p cm s–1 and its angular frequency is ps–1, 37. A particle of mass m is executing oscillations about
then the amplitude of its motion is the origin on the x-axis with amplitude A. Its
(a) p cm (b) 2 cm (c) 2 cm (d) 1 cm potential energy is given as U( x) = a x4 , where a is
positive constant. The x-coordinate of mass where
31. A particle moves in xy-plane according to the rule potential energy is one-third of the kinetic energy of
x = a sin wt and y = a cos wt. The particle follows particle, is
(a) an elliptical path A A
(b) a circular path (a) ± (b) ±
3 2
(c) a parabolic path A A
(d) a straight line path inclined equally to x and y-axis (c) ± (d) ±
3 2
Telegram @unacademyplusdiscounts

Oscillations 635

38. A particle starts SHM from the mean position. Its Springs and their Oscillations
amplitude is a and total energy E. At one instant its
kinetic energy is 3E/4 its displacement at this 46. A body of mass 500 g is attached to a horizontal
instant is spring of spring constant 8 p2 Nm -1. If the body is
a pulled to a distance of 10 cm from its mean position
(a) y = a / 2 (b) y =
2 then its frequency of oscillation is
a
(c) y = (d) y = a (a) 2 Hz (b) 4 Hz (c) 8 Hz (d) 0.5 Hz
3 /2
(e) 4 p Hz
39. A point particle of mass 0.1 kg is executing SHM of
amplitude 0.1 m. When the particle passes through 47. A simple spring has length l and force constant k. It is
the mean position, its kinetic energy is 8 ´ 10 -3 J. The cut into two springs of length l1 and l2 such that
equation of motion of this particle, if its initial phase l1 = nl2 (n = an integer). The force constant of the
of oscillation is 45°, is spring of length l2 is
ær pö æt pö ( n + 1) k
(a) y = 0.1sin ç + ÷ (b) y = 0.1sin ç + ÷ (a) k (1 + n ) (b)
è4 4ø è2 4 ø n
æ pö æ pö (c) k (d) k / ( n + 1)
(c) y = 0.1sin ç 4 t - ÷ (d) y = 0.1sin ç 4t + ÷
è 4ø è 4ø 48. Two springs of force constants k and 2 k are connected
40. A particle is vibrating in a simple harmonic motion to a mass as shown below. The frequency of
with and amplitude of 4cm. At what displacement oscillation of the mass is
from the equilibrium position is its energy half
potential and half kinetic?
(a) 1cm (b) 2 cm (c) 3 cm (d) 2 2 cm k 2k
m
41. When the potential energy of a particle executing
simple harmonic motion is one-fourth of its maximum
value during the oscillation, the displacement of the
particle from the equilibrium position in terms of its 1 1
(a) k/ m (b) 2k/ m
amplitude a is 2p 2p
a a a 2a 1 3k 1 m
(a) (b) (c) (d) (c) (d)
4 3 2 3 2p m 2p k
42. The potential energy of a particle (U X ) executing 49. A weightless spring which has a force constant k
SHM is given by
oscillates with frequency n when a mass m is
k
(a) Ux = ( x - a )2 (b) Ux = k1x + k2 x2 + k3x3 suspended from it. The spring is cut into two equal
2
(c) Ux = Ae - bx (d) Ux = constant halves and a mass 2 m is suspended from one part of
spring. The frequency of oscillation will now become
43. If a simple pendulum of length l has maximum n
angular displacement q, then the maximum kinetic (a) n (b) 2n (c) (d) n (2)1 /2
2
energy of bob of mass m is
1 ælö 50. An object suspended from a spring exhibits
(a) ´ç ÷ (b) ´ oscillations of period T. Now, the spring is cut in two
2 è gø 2 l
1 halves and the same object is suspended with two
(c) mgl ´ (1 - cos q) (d) ´ mgl sin q halves as shown in figure. The new time period of
2
oscillation will become
44. When the displacement is half of the amplitude, then
what fraction of the total energy of a simple harmonic
oscillator is kinetic?
(a) 2/7th (b) 3/4th (c) 2/9th (d) 5/7th
45. For a particle executing SHM, the kinetic energy K is
given by K = K 0 cos2 wt. The equation of its
displacement can be
1 /2 1 /2
æ K ö æ 2K ö m m
(a) ç 02 ÷ sin wt (b) ç 02 ÷ sin wt
è mw ø è mw ø
1 /2 1 /2 T T T
æ 2w2 ö æ 2K ö (a) (b) (c) (d) 2T
(c) ç ÷ sin wt (d) ç 0 ÷ sin wt 2 2 2 2
è mK0 ø è mw ø
Telegram @unacademyplusdiscounts

636 JEE Main Physics

51. On a smooth inclined plane, a body of mass M is 57. A uniform spring of force constant k is cut into two
attached between two springs. The other ends of the pieces, the lengths of which are in the ratio 1 : 2. The
springs are fixed to firm support. If each spring has ratio of the force constants of the shorter and longer
force constant k, the period of oscillation of the body piece is
(assuming the springs as massless) is (a) 1 : 2 (b) 2 : 1
(c) 1 : 3 (d) 2 : 3
58. What will be the force constant of the spring system
M
shown in figure?

θ k1 k1

(a) 2p[ M/ 2k ]1 /2 (b) 2p[2M/ k ]1 /2


(c) 2p [ Mg sin q/ 2k ]1 /2 (d) 2p[2Mg/ k ]1 /2
k2
52. A mass M, attached to a spring, oscillates with a
period of 2 s. If the mass is increased by 4 kg, the time
period increases by 1 s. Assuming that Hooke’s law is -1
obeyed, the initial mass M was k1 é 1 1ù
(a) + k2 (b) ê + ú
(a) 3.2 kg (b) 1 kg 2 ë 2k1 k2 û
(c) 2 kg (d) 8 kg -1
1 1 é2 1ù
(c) + (d) ê + ú
53. A mass M is suspended from a light spring. An 2k1 k2 k
ë 1 k2û
additional mass m added displaces the spring further
by a distance X. Now the combined mass will oscillate 59. One end of a spring of force constant k is fixed to a
on the spring with period vertical wall and the other to a block of mass m
mg (M + m)X resting on a smooth horizontal surface. There is
(a) T = 2p (b) T = 2p another wall at a distance x0 from the block. The
X (M + m) mg
spring is then compressed by 2 x0 and then released.
mg (M + m) The time taken to strike the wall is
(c) T = p / 2 (d) T = 2p
X( M + m ) mg

54. Time period of mass m suspended by a spring is T. If M


the spring is cut to one-half and made to oscillate by
A B C
suspending double mass, the time period of the mass
2x0 x0
will be
(a) 8T (b) 4T 1 k k 2p m p k
T (a) p (b) (c) (d)
(c) (d) T 6 m m 3 k 4 m
2
55. Two blocks with masses m1 = 1 kg and m2 = 2 kg are Simple Pendulum and other Systems
connected by a spring of spring constant k = 24 Nm–1
and placed on a frictionless horizontal surface. The 60. A simple pendulum of length l and mass (bob) m is
block m1 is imparted an initial velocity v0 = 12 cms–1 suspended vertically. The string makes an angle q
to the right, the amplitude of oscillation is with the vertical. The restoring force acting on the
(a) 1 cm (b) 2 cm pendulum is
(c) 3 cm (d) 4 cm (a) mg tan q (b) - mg sin q
(c) mg sin q (d) - mg cos q
56. A mass 1 kg suspended from a spring whose force
constant is 400 Nm–1, executes simple harmonic 61. A simple pendulum has a length l. The inertial and
oscillation. When the total energy of the oscillator is gravitational masses of the bob are m1 and m g
2 J, the maximum acceleration experienced by the respectively. Then the time period T is given by
mass will be mg l mil
(a) T = 2p (b) T = 2p
(a) 2 ms–2 (b) 4 ms–2 mig mg g
(c) 40 ms–2 (d) 400 ms–2 mi ´ mg ´ l l
(c) T = 2p (d) T = 2p
g mi ´ mg ´ g
Telegram @unacademyplusdiscounts

Oscillations 637

62. A man measures the period of a simple pendulum 68. Two simple pendulums of length 0.5 m and 20 m
inside a stationary lift and finds it to be T second. If respectively are given small linear displacement in
the lift accelerates upwards with an acceleration g/4, one direction at the same time. They will again be in
then the period of pendulum will be the phase when the pendulum of shorter length has
(a) 2T 5 (b) T completed ....oscillations.
2T T (a) 5 (b) 1 (c) 2 (d) 3
(c) (d)
5 4 69. In damped oscillation the amplitude of oscillations is
63. Four pendulums A, B, C and D are suspended from reduced to one-third of its initial value a0 at the end of
the same elastic support as shown in figure. A and C 100 oscillation. When the oscillation completes 200
are of the same length, while B is smaller than A and oscillations, its amplitude must be
D is larger than A. If A is a0 a0
[NCERT Exemplar] (a) (b)
2 4
a0 a0
(c) (d)
6 9

B
70. A and B are fixed points and A B
A the mass M is tied by strings at
C
D A and B. If the mass M is
displaced slightly out of this
(a) D will vibrate with maximum amplitude
plane and released, it will
(b) C will vibrate with maximum amplitude
execute oscillations with M
(c) B will vibrate with maximum amplitude
period.
(d) All the four will oscillate with equal amplitude
(Given, AM = BM = L, AB = 2 d)
64. If the length of second’s pendulum is increased by
2%. How many seconds it will lose per day? L ( L2 - d 2 )1 /2
(a) 2p (b) 2p
(a) 3927 s (b) 3427 s g g
(c) 3737 s (d) 864 s ( L2 + d 2 )1 /2 (2d 2 ) 3 /2
(c) 2p (d) 2p
65. A pendulum bob of mass m is hanging from a fixed g g
point by a light thread of length l. A horizontal speed
71. A simple pendulum of length l has been set up inside
v0 is imparted to the bob so that it takes up horizontal
a railway wagon sliding down a frictionless inclined
position. If g is the acceleration due to gravity, then
plane having an angle of inclination q = 30° with the
v0 is
horizontal. What will be its period of oscillation as
(a) mgl (b) 2gl (c) gl (d) gl
recorded by an observer inside the wagon?
66. A tunnel is made across the earth of radius R, 2l
(a) 2p (b) 2p 2l / g
passing through its centre. A ball is dropped from a 3g
height h in the tunnel. The motion will be periodic 3l
with time period. (c) 2p l / g (d) 2p
2g
R h
(a) 2p +4 72. If a simple pendulum is taken to a place where g
g g
decreases by 2%, then the time period
R 2h
(b) 2p +4 (a) increases by 0.5% (b) increases by 1%
g g
(c) increases by 2.0% (d) decreases by 0.5%
R h
(c) 2p + 73. A heavy sphere of mass m is suspended by string of
g g length l. The sphere is made to revolve about a
R 2h vertical line passing through the point of suspension
(d) 2p +
g g in a horizontal circle such that the string always
remains inclined to the vertical at an angle q . What
67. The bob of a pendulum of length l is pulled a side is its period of revolution?
from its equilibrium position through an angle q and l l cos q
then released. The bob will then pass through its (a) T = 2p (b) T = 2p
g g
equilibrium position with a speed v, where v equals
l sin q l tan q
(a) 2gl(1 - cos q) (b) 2gl(1 + sin q) (c) T = 2p (d) T = 2p
g g
(c) 2gl(1 - sin q) (d) 2gl(1 + cos q)
Telegram @unacademyplusdiscounts

638 JEE Main Physics

74. A piece of wood has dimensions a, b and c. Its relative


density is d. It is floating in water such that the side c

Displacement
is vertical. It is now pushed down gently and
released. The time period is Time
æ abc ö æ ba ö
(a) T = 2p ç ÷ (b) T = 2p ç ÷
è g ø è dg ø (2)

æ gö æ dc ö
(c) T = 2p ç ÷ (d) T = 2p ç ÷
è dc ø è gø

Displacement
75. A pendulum clock is placed on the moon, where object Time
weighs only one-sixth as much as on the earth, how
many seconds the clock tick out in an actual time of
(3)
1 min the clock keeps good time on the earth?
(a) 12.25 (b) 24.5
(c) 2.45 (d) 0.245

Displacement
76. A uniform cylinder of length L and mass M having Time
cross-sectional area A is suspended with its vertical
length, from a fixed point by a massless spring, such
that it is half submerged in a liquid of density d at (4)
equilibrium position. When released, it starts
oscillating vertically with a small amplitude. If the (a) Fig. 1 alone (b) Fig. 2 alone
force constant of the spring is k, the frequency of (c) Fig. 4 alone (d) Fig. 3 and 4
oscillation of the cylinder is 79. A particle of mass m is released from rest and follows
1 /2 1 /2 a parabolic path as shown. Assuming that the
1 æ k - Adg ö 1 æ k + Adg ö
(a) ç ÷ (b) ç ÷ displacement of the mass from the origin is small.
2p è M ø 2p è M ø Which graph correctly depicts the position of the
1 /2
1 æ k - dgL ö
1 /2
1 æ k + AgL ö particle as a function of time
(c) ç ÷ (d) ç ÷
2p è M ø 2p è Adg ø v(x)

77. A particle, with restoring force proportional to


displacement and resisting force proportional to
velocity is subjected to a force, F = F0 sin wt O (x)
If, the amplitude of the particle is maximum for w= w1
and the energy of the particle is maximum for w = w2 , x(t) x(t)
then
(a) w1 = w0 and w2 ¹ w0 (a) (b)
O t O t
(b) w1 = w0 and w2 = w0
(c) w1 ¹ w0 and w2 = w0
(d) w1 ¹ w0 and w2 ¹ w0
x(t) x(t)
78. Which of the following figure represent(s) damped
simple harmonic motions?
(c) (d)
O t O t
Displacement

Time 80. The amplitude of damped oscillator becomes in 2 s.


Its amplitude after 6 s is 1/n times the original. Then
(1) n is equal to
(a) 23 (b) 32 (c) 31/3 (d) 33
Telegram @unacademyplusdiscounts

Oscillations 639

Round II (Mixed Bag)

Only One Correct Option


1. A simple pendulum of length l has a bob of mass m,
with a charge q on it. A vertical sheet of charge, with L
surface charge density s passes through the point of
suspension. At equilibrium, the spring makes an
angle q with the vertical. Its time period of
oscillations is T in this position. Then m
sq sq
(a) tan q = (b) tan q =
2e 0 mg e 0 mg
(a) 9 (b) 18
1 1
(c) T > 2p (d) T = 2p (c) 27 (d) 36
g g
8. The period of particle in SHM is 8 s. At t = 0, it is at
2. An instantaneous displacement of a simple harmonic the mean position. The ratio of the distances
oscillator is x = A cos ( wt + p / 4). Its speed will be travelled by it in Ist second and 2nd second is
maximum at time
(a) 1.6 : 1 (b) 2.4 : 1
(a) p / 4 w (b) p /2w
(c) p / w (d) 2p / w (c) 3.2 : 1 (d) 4.2 : 1

3. The period of oscillation of a mass m suspended from 9. A simple pendulum of length l and having a bob of
a spring is 2 s. If along with it another mass 2 kg is mass M is suspended in a car. The car is moving on a
also suspended, the period of oscillation increases by circular track of radius R with a uniform speed v. If
1 s. The mass m will be the pendulum makes small oscillations in a radial
(a) 2 kg (b) 1 kg direction about its equilibrium position, what will be
(c) 1.6 kg (d) 2.6 kg its time period?
4. Two pendulums begin to swing simultaneously. The 3l æ v4 ö
(a) 2p (b) 2p l ç g2 + 2 ÷
first pendulum makes 9 full oscillations when the v2 è r ø
g+
other makes 7. The ratio of lengths of the two r
pendulums is 2l 2l
(a) 9/7 (b) 7/9 (c) 2p (d) 2p
æ 2 v2 ö ( g 2 + v 2 /r )
(c) 49/81 (d) 81/49 çg + 2 ÷
è r ø
5. A bottle weighing 220 g and area of cross-section
50 cm2 and height 4 cm oscillates on the surface of 10. A spring balance has a scale that reads from 0 to
water in vertical position. Its frequency of oscillation is 50 kg. The length of the scale is 20 cm. A body
(a) 1.5 Hz (b) 2.5 Hz suspended from this balance, when displaced and
(c) 3.5 Hz (d) 4.5 Hz released, oscillates with a period of 0.6 s. What is the
weight of the body? [NCERT]
6. A body of mass 4.9 kg hangs from a spring and
oscillates with a period 0.5 s on the removal of the (a) 222.13 N (b) 200.13 N
body, the spring is shortened by (c) 193.13 N (d) 219.13 N
(Take g =10 ms -2 , p 2 = 10) 11. One end of a U-tube containing mercury is connected
(a) 6.3 m (b) 0.63 m to a suction pump and the other end to atmosphere. A
(c) 6.25 cm (d) 63.5 cm small pressure difference is maintained between the
(e) 0.625 cm two columns. The suction pump is removed, the
7. A ball of mass (m) 0.5 kg is attached to the end of a column of mercury in the U-tube will show
string having length (L) 0.5m. The ball is rotated on a [NCERT]
horizontal circular path about vertical axis. The (a) periodic motion
maximum tension that string can bear is 324 N.The (b) oscillation
maximum possible value of angular velocity of ball (c) simple harmonic motion
(in rad/s) is (d) None of the above
Telegram @unacademyplusdiscounts

640 JEE Main Physics

12. The time period of a particle in simple harmonic 18. A highly rigid cubical block A of small mass M and
motion is 8 s. At t = 0, it is at the mean position. The side L is fixed rigidly on the another cubical block of
ratio of the distances travelled by it in the first and same dimensions and low modulus of rigidity h such
second, seconds is that the lower face of A completely covers the upper
(a) 1/2 (b) 1 / 2 (c) 1 / ( 2 - 1) (d) 1 / 3 face of B. The lower face of B is rigidly held on a
horizontal surface. A small force F is applied
13. The equation of SHM is given by perpendicular to one of the side faces of Z. After the
x = 3 sin 20pt + 4 cos 20t force is withdrawn, block A executes small
where x is in cm and t is in second. The amplitude is oscillations, the time period of which is given by
(a) 7 cm (b) 4 cm (c) 5 cm (d) 3 cm (a) 2p MLh (b) 2p Mh /L

14. When a body of mass 1.0 kg is suspended from a (c) 2p ML / h (d) 2p M / h L


certain light spring hanging vertically, its length 19. The bob of a simple pendulum is of mass 10 g. It is
increases by 5 cm. By suspending 2.0 kg block to the suspended with a thread of 1 m. If we hold the bob so
spring and if the block is pulled through 10 cm and as to stretch the string horizontally and release it,
released, the maximum velocity of it, in ms–1 is what will be the tension at the lowest position?
(g = 10 ms–2) (g = 10 ms–2)
(a) 0.5 (b) 1 (c) 2 (d) 4 (a) Zero (b) 0.1 N
15. A pendulum is made to hang from a ceilling of an (c) 0.3 N (d) 1.0 N
elevator.It has period of Tsec . (for small angles). The 20. A block of mass M is suspended from a light spring of
elevator is made to accelerate upwards with force constant k. Another mass m moving upwards
10 m/s2.The period of the pendulum now will be with velocity v hits the mass M and gets embedded in
(assume g =10 m/s2) it. What will be the amplitude of the combined mass ?
(a) T 2 (b) infinite (c) T/ 2 (d) zero mv mv
(a) (b)
16. A mass M is attached to a horizontal spring of force k (M - m) (M - m)k
constant k fixed on one side to a rigid support as mv mv
(c) (d)
shown in figure. The mass oscillates on a frictionless k (M + m) (M + m)k
surface with time period T and amplitude A. When
21. Two pendulums of length 1 m and 16 m start
the mass is in equilibrium position, another mass m
vibrating one behind the other from the same stand.
is gently placed on it. What will be the new amplitude
of oscillations? At some instant, the two are in the mean position in
the same phase. The time period of shorter pendulum
is T. The minimum time after which the two threads
k
m of the pendulum will be one behind the other is
M (a) T/4 (b) T/3
(c) 4 T/3 (d) 4 T
æ M ö æ M - mö
(a) A ç ÷
è M - mø
(b) A ç
è M ø
÷ 22. Two pendulums of lengths 1m and 1.21m
respectively start swinging together with same
æ M ö æ M + mö amplitude. The number of vibrations that will be
(c) A ç ÷ (d) A ç ÷
è M + mø è M ø executed by the longer pendulum before the two will
17. Lissajous figure shown in figure corresponds to swing together again are
which one of the following? (a) 9 (b) 10
(c) 11 (d) 12
23. An elastic string has a length l when tension in it is
5 N. Its length is h when tension is of 4 N. On
subjecting the string to a tension of 9 N, its length
will be
(a) Phase difference p/2 and period 1 : 2 (a) l + h
(b) Phase difference 3p/4 and period 1 : 2 (b) l - h
(c) Phase difference p/4 and period 2 : 1 (c) (5 l - 4 h)
(d) Phase difference 2p/3 and period 2 : 1 (d) (l + h)/(h – l)
Telegram @unacademyplusdiscounts

Oscillations 641

24. A point mass is subjected to two simultaneous 30. A particle in SHM is described by the displacement
sinusoidal displacement in X-direction function x( t) = A cos ( wt + q ). If the initial ( t = 0)
2p position of the particle is 1 cm and its initial velocity
X1( t) = A sin wt and X2 ( t) = A sin æç wt + ö÷ . Adding
è 3 ø is p cms–1, what is its amplitude? The angular
a third sinusoidal displacement X 3( t) = B sin( wt + f) frequency of the particle is p s–1.
brings the mass to a complete rest. The value of B (a) 1 cm (b) 2 cm
and f (c) 2 cm (d) 2.5 cm
3p 4p
(a) 2 A, (b) A, 31. The acceleration due to gravity on the surface of
4 3 moon is 1.7 m/s2 . What is the time period of a simple
5p p
(c) 3 A, (d) A, pendulum on the surface of moon, if its time period on
6 3
the surface of earth is 3.5 s?
25. Let T1 and T2 be the time period of spring A and B (g on the surface of earth is 9.8 m/s2 .) [NCERT]
when mass M is suspended from one end of each (a) 6.4 s (b) 7.4 s
spring. If both springs are taken in series and the (c) 9.4 s (d) 8.4 s
same mass M is suspended from the series
32. The total energy of a particle executing SHM is 80 J.
combination, the time period is T, then
What is the potential energy when the particle is at a
1 1 1
(a) T = T1 + T2 (b) = + distance of 3/4 of amplitude from the mean position?
T T1 T2
(a) 60 J (b) 10 J
1 1 1
(c) T 2 = T12 + T22 (d) 2 = 2 + 2 (c) 40 J (d) 45 J
T T1 T2
33. Two simple harmonic motions act on a particle.
26. A particle is having kinetic energy 1/3 of the These harmonic motions are
maximum value at a distance of 4 cm from the mean
x = A cos( wt + a ); y = A cos( wt + a ), when
position. Find the amplitude of motion.
(a) an ellipse and the actual motion is counter clockwise
(a) 2 6 cm (b) 2 / 6 cm
(b) an ellipse and the actual motion is clockwise
(c) 2 cm (d) 6 / 2 (c) a circle and the actual motion is counter clockwise
27. If a spring extends by x on loading, then the energy (d) a circle and the actual motion is clockwise
stored in the spring is (if T is the tension and k is the 34. The time period of a mass suspended from a spring is
force constant of the spring) 5 s. The spring is cut into four equal parts and the
T 2
T 2 same mass is now suspended from one of its parts.
(a) (b)
2x 2k The period is now
2k 2T 2 (a) 5 s (b) 2.5 s
(c) 2 (d) 1
T k (c) 1.25 s (d) s
16
28. Average value of kinetic energy and potential energy
over entire time period is
35. A block whose mass is 650 g is fastened to a spring
1 1 whose spring constantly is 65 Nm–1. The block is
(a) 0, wm2 2A pulled a distance x = 11 cm from its equilibrium
2 2
1 1 1 1 position at x = 0. On a frictionless surface and
(c) mw2 A2 , mw2 A2 (d) mw2 A2 , mw2 A2 released from rest at t = 0. The maximum velocity of
2 2 4 4
the vibrating block is
29. A mass m is suspended separately (a) 1.1 ms–1 (b) 0.65 ms–1
by two different springs in (c) 1.30 ms–1 (d) 2.6 ms–1
successive order, then time
periods is t1 and t2 respectively. If k1 k2

m is connected by both springs as


More Than One Correct Option
shown in figure, then time period 36. Motion of a ball bearing inside a smooth curved bowl,
is t0 , the correct relation is m when released from a point slightly above the lower
(a) t 02 = t12 + t22 point is [NCERT Exemplar]

(b) t 0-2 = t1-2 + t2-2 (a) simple harmonic motion


(b) non-periodic motion
(c) t 0-1 = t1-1 + t2-1
(c) periodic motion
(d) t 0 = t1 + t2 (d) periodic but not SHM
Telegram @unacademyplusdiscounts

642 JEE Main Physics

37. Displacement vs. time curve for a particle executing 40. A metal rod length L and mass m is pivoted at one
SHM is shown in figure. Choose the correct end. A thin disc of mass M and radius R ( < L) is
statements. [NCERT Exemplar] attached at its centre to the free end of the rod.
Consider two ways the disc is attached ( case A). The
Displacement

disc is not free to rotate about its centre. The rod-disc


system perform SHM in vertical plane after being
released from the same displaced position which of
0
1 2 3 4 5 6 7 Time (s) the following statements is (are) true?

(a) Phase of the oscillator is same at t = 0 s and t = 2 s


(b) Phase of the oscillator is same at t = 2 s and t = 6 s
(c) Phase of the oscillator is same at t =1 s and t =7 s
(d) Phase of the oscillator is same at t =1 s and t = 5 s
38. The displacement time graph of a particle executing (a) Restoring torque in case A = Restoring torque in case B
SHM is shown in figure. Which of the following
(b) Restoring torque in case A < Restoring torque in case B
statement is/are true ? [NCERT Exemplar]
(c) Angular frequency for case A > Angular frequency for
case B
Displacement

(d) Angular frequency for case A < angular frequency for


case B
2T/4
0
3T/4 T 5T/4
T/4 Time (s) Comprehension Based Questions
Passage I
A uniform cylindrical metal rod A of length L and
3T
(a) The force is zero at t = radius R is suspended at its mid-point from a rigid
4 support through a strong metal wire of length l. The
4T
(b) The acceleration is maximum at t = rod is given a small angular twist and released so that
4 it oscillates to and fro about its mean position with a
T time period T1. Another body B of an irregular shape
(c) The velocity is maximum at t =
4 is suspended from the same rigid support using the
T same length of given suspension wire and its time
(d) The PE is equal to KE of oscillation at t =
2 period is found to be T2 .
39. A particle is in linear simple harmonic motion
between two points A and B, 10 cm apart (Fig). Take
the direction from A to B as the +ve direction and
choose the correct statements. [NCERT Exemplar] A B

B O C A

41. The rotational inertia of metal rod about the wire as


AO = OB = 5 cm
an axis is
BC = 8 cm ML2 MR2
(a) The sign of velocity, acceleration and force on the (a) (b)
12 2
particle when it is 3 cm away from A going towards B are
é L2 R2 ù é L2 R2 ù
positive (c) M ê + (d) M ê +
(b) The sign of velocity of the particle at C going towards O is ë 12 2 úû ë 12 4 úû
negative 42. The motion of rod is
(c) The sign of velocity, acceleration and force on the
(a) periodic but non-oscillatory
particle when it is 4 cm away from B going towards A are
(b) oscillatory but non-simple harmonic
negative
(c) linear harmonic motion
(d) The sign of acceleration and force on the particle when it
is at point B is negative (d) angular harmonic oscillation
Telegram @unacademyplusdiscounts

Oscillations 643

43. Time period of oscillations of rod is given by (b) If both Assertion and Reason are true but Reason not
m k correct explanation of the Assertion
(a) T = 2p (b) T = 2p (c) If Assertion is true but Reason is false
k m
(d) If Assertion is false but the Reason is true
I k
(c) T = 2p (d) T = 2p
k I 46. Assertion In SHM, the motion is to end fro and
periodic.
Passage II Reason Velocity of the particle (v) = w K 2 - x2 ,where
A particle performs harmonic oscillation along the x is the displacement and k is amplitude.
x-axis about the equilibrium position x = 0. The
oscillation frequency is w= 4.00 s–1. At a certain moment 47. Assertion Soldiers are asked to break steps while
of time the particle has a coordinate x0 = 25.0 cm and crossing the bridge.
its velocity is equal to n x = 100 cms–2. Reason The frequency of marching may be equal to
0
44. Find the amplitude of oscillation. the natural frequency of bridge and may lead to
(a) 13 3 cm (b) 25 2 cm resonance which can break the bridge.
(c) 27 5 cm (d) 2 3 cm 48. Assertion The percentage change in time period is
1.5%, if the length of simple pendulum increases
45. Find the equation of motion of the particle.
by 3%.
æ pö æ pö
(a) y = 13 3 sin ç 4t + ÷ (b) y = 25 2 sin ç 4t + ÷ Reason Time period is directly proportional to length
è 4ø è 4ø
of pendulum.
æ pö æ pö
(c) y = 27 2 sin ç 4t + ÷ (d) y = 27 5 sin çt + ÷ 49. Assertion If the length of a spring is made n times, the
è 4ø è 2ø
spring factor of the spring becomes 1/nth of its
original value.
Assertion and Reason Reason Time of oscillation of a spring pendulum is
m
Direction Question No. 46 and 50 are Assertion-Reason type. T = 2p .
Each of these contains two Statements : Statement I (Assertion),
k
Statement II (Reason). Each of these questions also has four 50. Assertion The amplitude of a particle executing SHM
alternative choice, only one of which is correct. You have to with a frequency of 60 Hz is 0.01 m. The maximum
select the correct choices from the code (a), (b), (c) and (d) given value of acceleration of the particle is ±144 p2 ms -2 .
below Reason Acceleration amplitude = w2 A,where A is
(a) If both Assertion and Reason are true and the Reason displacement amplitude.
is correct explanation of the Assertion

Previous Years’ Questions


51. If a simple pendulum has significant amplitude (up them is ( X 0 + A) the phase difference during their
motion 1
tois
a factor of of original) only in the period
[AIEEE between
2011]
e p p
t = 0 s to t = ts. Then t may be called the average life (a) (b)
2 3
of the pendulum. When the spherical bob of the p p
pendulum suffers a retardation (due to viscous drag) (c) (d)
4 6
proportionality the average life time of the pendulum
is (assuming damping is small ) in seconds
53. A mass M attached to a horizontal spring executes
SHM with a amplitude A when the mass M passes
[AIEEE 2012]
through its mean position then a smaller mass m is
0.693
(a) (b) b placed over it and both of them move together with
b æA ö
1 amplitude A2 .The ratio of ç 1 ÷ is
(c) (d) 2/b è A2 ø
b [AIEEE 2011]
M M+m
52. Two particles are executing simple harmonic motion (a) (b)
M+m M
of the same amplitude A and frequency w along the 1 /2 1 /2
x-axis. Their mean position is separated by distance æ M ö æ M + mö
(c) ç ÷ (d) ç ÷
X 0 ( X 0 > A). If the maximum separation between è M + mø è M ø
Telegram @unacademyplusdiscounts

644 JEE Main Physics

54. If x, v and a denotes the displacement, the velocity 62. The particle execute simple harmonic motion with a
and the acceleration of a particle executing simple time period of 16 s. At time t = 2 s, the particle crosses
harmonic motion of time period T. Then which of the the mean position while at t = 4 s, its velocity is
following does not change with time? [AIEEE 2009] 4 ms–1. The amplitude of motion in metre is
(a) a2T2 + 4 p2 v2 (b) aT /x [Kerala CET 2007]

(c) aT + 2pv (d) aT / v (a) 2 p (b) 16 2p


(c) 32 2 /p (d) 4 / p
55. If the maximum velocity and acceleration of a
particle executing SHM are equal in magnitude, then (e) 24 2p
the time period will be [BVP Engg. 2008] 63. A mass oscillates along the x-axis according to the law,
(a) 1.57 s (b) 3.14 s x = x0 cos ( wt - p/ 4). If the acceleration of the particle
(c) 6.28 s (d) 12.56 s is written as a = A cos( wt + d), then [AIEEE 2007]
56. A particle is executing simple harmonic motion with (a) A = x0 w2 , d = 3p / 4 (b) A = x0 , d = - p / 4
an amplitude A and time period T. The displacement 2
(c) A = x0 w , d = p / 4 (d) A = x0 w2 , d = - p / 4
of the particle after 2 T period from its initial position
64. A particle of mass m executes simple harmonic
is [EAMCET 2008]
motion with amplitude a and frequency n. The
(a) A (b) 4 A (c) 8 A (d) zero average kinetic energy during its motion from the
57. Two springs are joined and attached to a mass of position of equilibrium to the end is [AIEEE 2007]
16 kg. The system is then suspended vertically from a (a) 2 p2 ma2 v2 (b) p2 ma2 v2
rigid support. The spring constant of the two springs 1
are k1 and k2 respectively. The period of vertical (c) p2 ma2 v2 (d) 4 p2 ma2 v2
4
oscillations of the system will be [WB JEE 2008]
65. A particle executes simple harmonic oscillation with
1 k1 + k2
(a) k1 + k2 (b) 8p an amplitude a. The period of oscillation is T. The
8p k1k2
minimum time taken by the particle to travel half of
p p k1 the amplitude from the equilibrium position is
(c) k1 - k2 (d)
2 2 k2 [UP SEE 2007]
T T
58. A simple pendulum is suspended from the ceiling of a (a) (b)
4 8
lift. When the lift is at rest its time period is T. With
T T
what acceleration should the lift be accelerated (c) (d)
12 2
upwards in order to reduce its period to T/2?
(g is the acceleration due to gravity) 66. A mass of 2.0 kg is put on a flat pan m
[Karnataka CET 2008] attached to a vertical spring fixed on the
(a) 4 g (b) g (c) 2 g (d) 3 g ground as shown in the figure. The mass
of the spring and the pan is negligible.
59. The total energy of a simple harmonic oscillator is When pressed slightly and released the
proportional to [Kerala CET 2008]
mass executes a simple harmonic motion.
(a) square root of displacement The spring constant is 200 Nm–1. What
(b) velocity should be the minimum amplitude of the
(c) frequency motion, so that the mass gets detached
(d) amplitude from the pan? (Take g = 10 ms–2) [UP SEE 2007]
(e) square of the amplitude (a) 8.0 cm
60. The amplitude of SHM (b) 10.0 cm
y = 2 (sin 5 pt + 3 cos 5 pt) is [Kerala CET 2008] (c) Any value less than 12.0 cm
(a) 2 (b) 2 2 (d) 4.0 cm
(c) 4 (d) 2 3 67. The kinetic energy and potential energy of a particle
61. The period of a simple pendulum inside a stationary executing SHM of amplitude a will be equal when
lift is T. The lift accelerates upwards with an displacement is [BVP Engg. 2007]

acceleration of g/3. The time period of pendulum will a


(a) (b) a 2
be [Kerala CET 2008] 2
a
T 3 T (c) 2a (d)
(a) 2 T (b) (c) T (d) 2
2 2 3
Telegram @unacademyplusdiscounts

Oscillations 645

68. One end of a long metallic wire of length L is tied to 73. A particle starts SHM from the mean position. Its
the ceiling. The other end is tied to massless spring of amplitude is a and total energy E. At one instant its
spring constant k. A mass m hangs freely from the kinetic energy is 3E/4. Its displacement at that
free end of the spring. The area of cross-section and instant is [Kerala CET 2005]
Young’s modulus of the wire are A and Y respectively. (a) a / 2 (b) a/2
If the mass is slightly pulled down and released, it (c) a / ( 3 / 2) (d) a / 3
will oscillate with a time period T equal to [DCE 2006]
m(YA + KL) 74. If the length of the pendulum is made 9 times and
(a) 2p m / k (b) 2p mass of the bob is made 4 times, then the value of
YAK
time period becomes [BHU 2005]
mYA ML
(c) 2p (d) 2p (a) 3 T (b) 3 T/2 (c) 4 T (d) 2 T
KL YA
75. A simple pendulum has time period T1. The point of
69. Two identical springs, each of spring constant k are
suspension is now moved upward according to the
connected first in series and then in parallel. A mass
relation) y = kt2 . ( k = 1 ms -2 ) where y is the vertical
M is suspended from them. The ratio of their
displacement. The time period now becomes T2 . The
frequencies of vertical oscillations will be T2
[Kerala CET 2006] ratio of 12 is (g = 10 ms–2)
T2
(a) 2 : 1 (b) 1 : 1
(a) 6/5 (b) 5/6 (c) 1 (d) 4/5
(c) 1 : 4 (d) 4 : 1
(e) 1 : 2 76. The bob of a simple pendulum is a spherical hollow
ball filled with water. A plugged hole near the bottom
70. A coin is placed on a horizontal platform which
of the oscillation bob gets suddenly unplugged.
undergoes vertical simple harmonic motion of
During observation, till water is coming out, the time
angular frequency w . period of oscillation would [AIEEE 2005]
The amplitude of oscillation is gradually increased.
(a) increase towards a saturation value
The coin will leave contact with the platform for the
(b) remain unchanged
first time [AIEEE 2006]
(c) first decrease and then increase to the original value
(a) at the highest position of the platform (d) first increase and then decrease to the original value
(b) at the mean position of the platform
(c) for an amplitude of g/w2
77. If a simple harmonic motion is represented by
d2 x
(d) for an amplitude of g2/w2 + ax = 0, its time period is
dt2 [AIEEE 2005]
71. Starting from the origin, a body oscillates simple 2p 2p
(a) 2p a (b) 2pa (c) (d)
harmonically with a period of 2 s. After what time a a
will its kinetic energy be 75% of the total energy?
78. The function sin2 (wt) represents [AIEEE 2005]
[AIEEE 2006]
(a) a simple harmonic motion with a period p / w
1 1
(a) s (b) s (b) a simple harmonic motion with a period 2p / w
12 6 p
1 1 (c) a periodic with a period , but not simple harmonic
(c) s (d) s 3
4 3 motion
72. A particle executes SHM with an amplitude of 2 cm. 2p
(d) a periodic with a period , but not simple harmonic
When the particle is at 1 cm from the mean position 3
motion
the magnitude of its velocity is equal to that of its
acceleration. Then its time period in second is 79. Two simple harmonic motions are represented by the
p
[Kerala CET 2005] equations y1 = 0.1sin æç100 pt + ö÷ and y2 = 0.1cosp t.
è 3ø
1
(a) (b) 2p 3 The phase difference of the velocity of particle 1 with
2p 3
respect to the velocity of particle 2 is [AIEEE 2005]
2p 3
(c) (d) p -p
3 2p (a) (b)
6 3
2 p -p
(e) (c) (d)
p 3 6
Telegram @unacademyplusdiscounts

646 JEE Main Physics

80. A particle is executing simple harmonic motion with that the system is completely isolated from its
amplitude of 0.1 m. At a certain instant when its surrounding the piston executes a simple haromonic
displacement is 0.02, its acceleration is 0.5 ms–2. The motion with frequency. [JEE Main 2013]
maximum velocity of the particle is (in ms–1) 1 Ag p 0 1 V0 Mp 0
(a) (b)
[BVP Engg. 2005] 2p V0 M 2p A2 g
(a) 0.01 (b) 0.05
1 A2 g p 0 1 MV0
(c) 0.5 (d) 0.25 (c) (d)
2p MV0 2p Ag p 0
81. An ideal gas enclosed in a vertical cylindrical
container supports a freely moving piston of mass M. 82. The amplitude of a damped oscillator decreases to
The piston and the cylinder have equal 0.9 times its original magnitude is 5 s. In another
cross-sectional area A. When the piston is in 10 s it will decrease to a time its original magnitude,
equilibrium, the volume of the gas is V0 and its where a equals [JEE Main 2013]
pressure is p0 . The piston is slightly displaced from (a) 0.7 (b) 0.81
the equilibrium position and released. Assuming (c) 0.729 (d) 0.6

Answers
Round I
1. (d) 2. (b) 3. (d) 4. (b) 5. (b) 6. (d) 7. (b) 8. (a) 9. (a) 10. (d)
11. (b) 12. (d) 13. (c) 14. (c) 15. (a) 16. (a) 17. (b) 18. (c) 19. (c) 20. (c)
21. (b) 22. (c) 23. (b) 24. (a) 25. (a) 26. (d) 27. (a) 28. (d) 29. (b) 30. (c)
31. (b) 32. (d) 33. (a) 34. (c) 35. (b) 36. (c) 37. (b) 38. (b) 39. (d) 40. (d)
41. (c) 42. (a) 43. (c) 44. (b) 45. (b) 46. (a) 47. (b) 48. (c) 49. (a) 50. (b)
51. (a) 52. (a) 53. (d) 54. (d) 55. (b) 56. (c) 57. (b) 58. (b) 59. (c) 60. (b)
61. (b) 62. (c) 63. (b) 64. (d) 65. (b) 66. (b) 67. (a) 68. (a) 69. (d) 70. (b)
71. (a) 72. (b) 73. (b) 74. (d) 75. (b) 76. (b) 77. (c) 78. (a) 79. (a) 80. (d)

Round II
1. (a) 2. (a) 3. (c) 4. (c) 5. (b) 6. (c) 7. (d) 8. (b) 9. (b) 10. (d)
11. (c) 12. (c) 13. (c) 14. (b) 15. (c) 16. (c) 17. (a) 18. (d) 19. (c) 20. (d)
21. (c) 22. (b) 23. (c) 24. (b) 25. (c) 26. (a) 27. (d) 28. (d) 29. (b) 30. (b)
31. (d) 32. (d) 33. (c) 34. (b) 35. (a) 36. (a,c) 37. (b,d) 38. (a,b,c) 39. (a,c,d) 40. (a,d)
41. (d) 42. (d) 43. (c) 44. (b) 45. (b) 46. (b) 47. (a) 48. (c) 49. (a) 50. (b)
51. (d) 52. (b) 53. (d) 54. (b) 55. (c) 56. (d) 57. (b) 58. (d) 59. (e) 60. (c)
61. (c) 62. (c) 63. (a) 64. (b) 65. (c) 66. (b) 67. (d) 68. (b) 69. (e) 70. (c)
75. (a) 76. (d)71. (b)77. (c) 72. 78. (c) 73.
(c) 79. (d) 80. (c)
81. (c) 82. (c)
Telegram @unacademyplusdiscounts

the Guidance
Round I
dy1 6. Given, y = A sin wt
1. We can find the velocities, v1 = = 2 ´ 10 cos (10 t + q)
dt
dy
and v 2 = -3 ´ 10 sin 10t = 30 cos(10t + p / 2) \ v= = Aw cos wt
dt
\ Phase difference = (10t + q) - (10t + p / 2) = q - p / 2
= A w sin ( wt + p / 2)
121 100 dv
2. The time periods, T1 = 2p and T2 = 2p Acceleration a= = - w2A sin wt
g g dt
So, T1 > T2. , Let the shorter pendulum makes n vibrations, then = w2A sin( wt + p )
the longer pendulum will make less than n vibrations to come 1
in phase again. 7. Given, y = sin3 wt = [3 sin wt - sin 3 wt ]
4
So, nT2 = (n - 1)T1
As this motion is not represented by single harmonic function,
100 121 hence it is not SHM. As this motion involves sine and cosine
or n ´ 2p = (n - 1) ´ 2p
g g functions, hence it is periodic motion.
or 10n = (n - 111
) 8. It is required to calculate the time for extreme position.
or n = 11 Hence in this case, equation of displacement of particle can
æ pö
3. Given, y = a sin wt + b cos wt be written as x = a sin ç wt + ÷ = a cos wt
è 2ø
Let a = A cos q and b = A sin q …(i) a
Þ = a cos wt (as per question)
then y = A cos q sin wt + A sin q cos wt 2
y = A sin ( wt + q) p
Þ wt =
which is in the form of SHM 3
From Eq. (i) 2p p T
Þ ×t = Þ t=
a2 + b 2 = A2 cos2 q + A2 sin2 q T 3 6
Þ A = a2 + b 2 9. Given, angular frequency of the piston, w = 200 rad/min
4. When bigger pendulum of time period (5T / 4) completes one Stroke length = 1m
vibrations, the smaller pendulum will complete (5 / 4) Stroke length 1
\Amplitude of SHM, A = =
vibrations. It means the smaller pendulum will be leading the 2 2
p = 0.5 m
bigger pendulum by phase T /4 sec = rad = 90°
2
Now, v max = wA
æp ö
5. Given, y = 3 cos ç - 2wt ÷ ... (i) = 200 ´ 0.5 = 100 m/min
è4 ø
10. As, Wavelength = Velocity of wave ´ time period
dy æp ö
Velocity, v= = 3 ´ 2 w sin ç - 2 wt ÷ i. e. , l = 300 ´ 0.05
dt è4 ø
or l = 15 m
dv æp ö
Acceleration, A= = - 4 w2 ´ 3 cos ç - 2wt ÷ = - 4 w2y According to the problem path difference between two points
dt è4 ø
= 15 - 10 = 5 m
As A µ y and - ve sign shows that it is directed towards
equilibrium (or mean position), hence particle will execute 2p
\ Phase difference = ´ path difference
SHM Comparing Eq. (i) with equation l
y = r cos ( f - w¢ t ) 2p
Df = ´ Dx
l
2p
we have, w¢ = 2 w or =2 w 2p 2p
T¢ = ´5 =
15 3
p
or T¢ =
w
Telegram @unacademyplusdiscounts

648 JEE Main Physics

11. At one of the extreme position, weight of block = restoring - 4p 2


17. Acceleration, a = - w2y = y
force. At the other extreme position, weight of block and T2
restoring force both act downward direction. So the force on 1/ 2
æ 4p 2y ö y
block at is doubled than its weight. or T=ç ÷ = 2p
è a ø a
m 4 p 2a m 4p 2a
12. As, F1 = 2
and F2 =
22 3
p T22 =2´ ´ = 3.14 s
7 12
4p 2ma 4p 2ma
Net force, F = F1 + F2 = +
T12 T22 18. As, d 2x / dt 2 = - kx
æ 1 1ö displacemant
= 4p 2maç 2 + 2 ÷ and T = 2p
è T1 T2 ø acceleration
x 1
4p 2ma æ 1 1ö So, T = 2p = 2p
or 2
= 4p 2ma ç 2 + 2 ÷ kx k
T è T1 T2 ø
1 1 1
19. As, x = Asin( w+ p / 2) = Acoswt
or = +
T 2 T12 T22 \ cos wt = x / A

1 T12 + T22 and sin wt = 1 - ( x2 / A2)


or =
T2 T12T22 y = A sin 2wt = 1 - ( x2 / DA2)
T12T22 y = A sin 2 wt = 2 A sin wt cos wt
or T2 =
T12 + T22 or y 2 = 4 A2 sin 2w t cos2 wt

13. Motion of an oscillating liquid column in a U tube is SHM x2 æ A2 - x2 ö


= 4 A2 ´ ´ç ÷
l A2 è A2 ø
with period, T = 2p , where l is the height of liquid column
g æ x2 ö
in one arm of U tube in equilibrium position of liquid. = 4x2ç1 - 2 ÷
è A ø
Therefore, T is independent of density of liquid.
14. x = a cos wt and y = a sin wt 20. Let, O be the position and x be the distance of coin from O.
The coin will slip if centrifugal force on coin just becomes
\ x2 + y 2 = a2(cos2 wt + sin2 wt ) = a2 equal to force of friction i. e., mxw2 = mmg
It is an equation of a circle. Thus trajectory of motion of the The coin will slip if, x = maximum = amplitude A
particle will be a circle.
m Aw2 = m mg
15. Velocity, v = rw cos wt
or A = mg / w2
2p 2p 2p 1
Þ 0.4 = r ´ cos ´2 = r ´ ´ 21. Weight kept on the system will separate from the piston when
16 16 16 2
the maximum force just exceeds the weight of the body.
0.4 ´ 16 ´ 2 3.2 2 Hence,
or r= = = 1.44 m
2 2p p
mw y = mg
16. When particle is at x = 2, the displacement is y = 4 - 2 = 2 cm. or y = g / w2 = 9.8 /(2p ) 2 = 0.25 m
If t is the time taken by the particle to go from x = 4 cm to
2 pt 2 pt p ´0
x = 2 cm, then y = a cos wt = a cos = a cos 22. When t = 0 , x = r cos =r
T 1.2 2
2pt y 2 p ´3
or cos = = when t = 3s, x = r cos =0
1.2 a 4 2
1 p p
= = cos Here w=
2 3 2
2t 1 2p p
or = or =
1.2 3 T 2
1.2 or T = 4s
or t= = 0.26
6 \ In 3 s, the particle goes from one extreme to other extreme
and then back to mean positing. So, the distance travelled
Time taken to move from x = +2 cm to x = +4 cm and back
= 2r + r = 3r
again = 2t = 2 ´ 0.2 s = 0.4 s
Telegram @unacademyplusdiscounts

Oscillations 649

23. For displacemant OQ = 40 cm; let t1 be the time taken, then 29. Given, x = 3 sin wt + 4 sin ( wt + p / 3)
2p Comparing it with the equation
40 = 41sin t1 ,
24 x = r1 sin wt + r2 sin( wt + f)
On solving, t1 = 516
. s We have, r1 = 3 cm, r2 = 4 cm and f = p /3
For displacement OQ = -9 cm, let t 2 be the time taken, then The amplitude of combination is
2p
9 = 41 sin t1 r = r12 + r22 + 2rr
1 2 cos f
12
On solving t 2 = 0.84s = 3 2 + 4 2 + 2 ´ 3 ´ 4 ´ cos p / 3
Total time = 5.16 + 0.84 = 6.00 s = 37 » 6 cm
24. Given, T = 30 s, OQ = B. The projection of the radius vector 30. As, x (t ) = A cos ( wt + f) …(i)
on the diameter of the circle when a particle is moving with \ 1 = A cos( p ´ 0 + f) = A cos f …(ii)
uniform angular velocity ( w) on a circle of reference is SHM. d [ x (t )]
Let the particle go from P to Q in time t. velocity = = - Aw sin ( wt + f)
dt
Then ÐPOQ = wt = ÐOQP. The projection of radius OQ on
x-axis will be OR = x (t ) say. p = - A ´ p sin (0 + f) = - pA sin f - 1
= A sin f …(ii)
Squaring and adding Eqs. (ii) and (iii), we have
P (t = 0)
1 + 1 = A2(cos2 w + sin 2 w) = A2
Q
or A = 2 cm
ωt ωt
x y
x 31. Given, = sin wt and = cos wt
O x(t) R a a
y 2 x2
Now + =1
a2 a2
Þ y 2 + x2 = a2
x (t )
In DOQR , sin w t = This is equation of a circle having radius a.
B
2p 2p d 2y
or x (t ) = B sin wt = B sin t = B sin t 32. For SHM, µ -y
T 30 dx2
2p 33. Here, a = 1cm = 0.01m . The mass will remain in contact with
25. v max = 1 ´ 10 3 = a ´
T surface, if
v max ´ T
Þ a= mg = mw2a
2p
1 ´ 10 3 ´ 1 ´ 10 -5 or w= g /a
a= = 1.59 mm
2p or 2pn = g / a
1 g 7 980
26. As, sec bt is not define for bt = p / 2 or n= =
a sin bt + c cos bt 2p a 2 ´ 22 1
and x = a sec bt + c cosec bt = = 4.9 Hz » 5 Hz
sin bt cos bt
This equation cannot be modified in the form of simple 34. As, x = a sin wt
equation of SHM and y = b sin( wt + p ) = - b sin wt
i. e., x = a sin( wt + f) x y b
or =- Þy=- x
So, it cannot represent SHM a b a
27. Here, v max = wr = 30 cm/s It is an equation of a straight line.
2 2
Amax = w r = 60 cm/s 35. Maximum force on body while in SHM
w2r 60 2p = mw2a = 0.5 (2p /2) 2 ´ 0.2 = 1 N
\ = = 2 or w = 2 or = 2 or T = ps.
wr 30 T Maximum force of friction = m mg = 0.3 ´ 0.5 ´ 10 = 1.5 N
L 2p g Since the maximum force on the body due to SHM of the
28. Time period, T = 2p and w = =
g T L platform is less than the maximum possible frictional force, so
g the maximum force of friction will be equal to the maximum
\Displacement, x = a cos wt = a cos t
L force acting on body due to SHM of platform i. e., 1 N.
Telegram @unacademyplusdiscounts

650 JEE Main Physics

1 43. From the figure, AC = l cos q


36. Potential energy, V = mw2x2
2 O
1
and kinetic energy E, T = mw2( a2 - x2)
2 θ
T a2 - x2 T
\ =
V x2 l
37. Energy of oscillation,E = aA4 (at maximum displacement) C
Kinetic energy of mass at x = x is
K = E - U = a ( A2 - x4) A mg mg cos θ
mg sin θ
As K = 3U
a ( A4 - x4) = 3 ax4 \ OC = OA - AC
A = l - l cos q = l (1 - cos q)
or x=±
2 Maximum KE of bob at O = Maximum PE of bob at B
1 = ma ´ OC = mgl (1 - cos q)
38. Total energy, E= mw2a2
2 1
44. Kinetic energy, E k = m ( a2 - y 2)
3E 1 2
KE = = mw2( a2 - y 2)
4 2 1 æ 2 a2 ö 1 3
2 2 = m ç a - ÷ = mw2a2
3 a -y 2 è 4ø 2 4
So, =
4 a2 1
Total energy, E= mw2a2
or y 2 = a2 / 4 2
or y = a/2 So, Ek / E = 3 / 4
1
39. Kinetic energy at mean position = mw2a2 = 8 ´ 10 -3 45. If m is the mass and r is the amplitude of oscillation, then
2 maximum kinetic energy,
1/ 2 1/ 2
æ 2 ´ 8 ´ 10 -3 ö é 2 ´ 8 ´ 10 -3 ù 1
or w=ç ÷ =ê =4 K0 = mw2r 2
2ú 2
è ma2 ø ë 0.1 ´ (0.1) û
1/ 2
Equation of SHM is, æ 2K ö
or r = ç 02 ÷
è mw ø
æ pö
y = a sin( wt + q) = 0.1sin ç 4 t + ÷
è 4ø The displacement equation can be
1/ 2
æ 2K ö
40. Let x be point, where KE = PE y = r sin wt = ç 0 ÷ sin w t
è mw ø
1 1
Hence, mw2( a2 - x2) = mw2x2
2 2 46. Here, mass of body, m = 500 g = 500 ´10 -3 kg
2 2
Þ 2x = a Spring constant k = 8 p 2 Nm–1
a y
Þ x= = = 2 2 cm The frequency of oscillation is
2 2
1 k 1 8 p 2 Nm–1
1 v= = = 2 Hz
mw2y 2 2p m 2p 500 ´ 10 -3 kg
U 2 1
41. As, = =
Umax 1
mw2a2 4 47. Let k be the force constant of spring of length l2. Since, l2 = nl2,
2
where n is an integer, so the spring is made of (n + 1) equal parts
y2 1
= in length, each of length l2.
a2 4
1 (n + 1)
a \ =
Þ y= k k
2
or k = (n + 1)k
42. Potential energy of body in SHM at an instant, The spring of length l2( = n l2) will be equivalent to n spring
1 2 k (n + 1) k
U= ky connected in series where spring constant k¢ = =
2 n n
if the displacement, y = ( a - x), then 1 keffective 1 k + 2k 1 3k
1 1 48. As, n = = =
U = k ( a - x) 2 = k ( x - a) 2 2p m 2p m 2p m
2 2
Telegram @unacademyplusdiscounts

Oscillations 651

1 k Using law of conservation of energy, we have


49. We have, n= ;
2p m 1 1 1
m1v 02 = kx2 + (m1 + m2)v 2
1 k' 1 2k 2 2 2
n¢ = = =n (Qk' = 2k )
2p 2m 2p 2m Putting the value of v = 4 cms–1 and solving, we get x = 2 cm.

50. Let, k be the spring constant of each half part of the spring. For 56. As, energy stored = work done
a complete spring, the spring constant k¢ = k / 2 (spring in 1 2
series). When two splitted parts of a spring are connected to Þ E= kr (where, r = displacement)
2
the body, then spring are in parallel. Their effective spring
2E 2 ´2 1
constant, k¢ = k + k = 2 k. or r= = = m
k 400 10
m
As T =2p 2
k æ kö 1
Now, a = w2r = ç ÷ ´
1 è m ø 10
or Tµ (for a fixed value of m)
k æ 400 ö 1
=ç ÷´ = 40 ms-2
T¢ k/2 1 è 1 ø 10
\ = =
T 2k 2
57. Let k be the force constant of the shorter part of the spring of
T length l / 3. In a complete spring, three springs are in series
or T¢ =
2 each of force constant k.
51. It is a system of two springs in parallel. The restoring force on 3k
k1 =
the body is due to springs and not due to gravity. 2
Therefore slope is irrelevent. k 3k
\ = =2
Here, the effective spring constant = k + k = 2 k k1 3 k / 2
Thus time period, T = 2p M / 2k or k : k1 = 2 : 1
M 58. Two spring each of spring constant k1 in parallel, given
52. As, T = 2 = 2p
k equivalent spring constant of 2k1 and this is in series with
M+ 4 spring of constant k2, so equivalent spring constant
and 2 + 1 = 2p (from questions) -1
k æ1 1 ö
k=ç + ÷
k+ 4 è k2 2k1 ø
or 3 = 2p
k 59. The total time to go from A to C
4 M
So, = t AC = t AB + t AC = (T /4) + t AC
9 M+ 4
where, T = time period of oscillation of spring mass system
or 4 M + 16 = 9 M
Now, t AB can be obtained from, BC = AB sin(2p /T) tBC
16
or M= = 3.2 kg BC 1
5 Putting =
AB 2
53. A total restoring force, F = kX = mg T
We obtain tBC =
or k = mg / X 12
T T 2p m
(M + m) (M + m) Þ t AC = + =
\ T = 2p = 2p 4 12 3 k
mg / X mg
Period of oscillation,
m m 3m / 4 3m
54. As, T = 2p T = 2p = 2p =p
k k k k
2m m
and T ¢ = 2p = 2p =T 60. S
2k k
55. The amplitude of oscillations will be the maximum when θ
compression in the spring is maximum. At the time of l T
maximum compression, velocities of both the blocks are
equal say v, then using law of conservation of momentum, P
C
m1v 0 = (m1 + m2) v θ
or 1 ´ 12 = (1 + 2) v O B mg mg cos θ
or v = 4 cms-1 mg sin θ
Telegram @unacademyplusdiscounts

652 JEE Main Physics

When the bob is displaced to position P, through a small 66. When the ball m falls from a height h, it reaches the surface of
angle q from the vertical, the various forces acting on the bob earth in time, t = 2h/g . Its velocity is v = 2gh. It then moves
at P are
into the tunnel and reaches on the other side of earth and goes
(i) the weight mg of the bob acting vertically downwards again upto a height h from that side of earth. The ball again
(ii) the tension T in the string acting along PS returns back and thus executes periodic motion. Outside the
Resolving mg into two rectangular components, we get earth ball crosses distance h four times. When the ball is in the
(a) mg cos q acts along PA, opposite to tensions, we get tunnel at distance x from the centre of the earth, then
(b) mg sin q acts along PB, tangent to the arc OP and directed gravitational force acting on ball is
towards O. Gm æ 4 ö æ4 ö
F = 2 ´ ç px2r ÷ = G ´ ç pr ÷mx
If the string neither slackens nor breaks but remains taut, then x è3 ø è3 ø
T = mg cos q 4
Mass of the earth, M = pR 2r
The force mg sin q tends to bring the bob back to its mean 3
position O. 4 M
or pr = 3
\Restoring force acting on the bob is 3 R
F = - mg sin q GMmx
\ F=
61. Torque acting on the bob = Ia = - (mg ) l sin q R3
i. e. , F µx
or (mi l 2) a = - (mg g ) l q
As, this force, F is directed towards the centre of earth i. e. , the
æ mg g ö mean position. So, the ball will execute periodic motion
or a = -ç ÷ q = - w2q
è mi l ø about the centre of earth.
Here, inertia factor = mass of ball = m
mg g 2p mi l
where, w2 = \T = = 2p GMm gm
mi l w mg g Spring factor = =
R3 R
l \Time period of oscillation of ball in the tunnel is
62. As, T = 2p . When lift is accelerated upwards with
g inertia factor
T ¢ = 2p
acceleration a ( = g /4), then effective acceleration due to spring factor
gravity inside the lift m R
g 5g = 2p =p
g1 = g + a = g + = gm / R g
4 4
l l 2 2T Time spent by ball outside the tunnel on both the sides will
\ T1 = 2p = 2p ´ =
5g /4 g 5 5 be = 4 2h/g

63. Since length of pendulums A and C is same and T = 2p l/ g , Therefore, total time period of oscillation of ball is
R 2h
hence their time period is same and they will have same = 2p +4
frequency of vibration. Due to it, a resonance will take place g g
and the pendulum C will vibrate with maximum amplitude.
67. When the bob of pendulum is brought is brought to a position
64. According to question, making and angle q with the equilibrium position, then height
l + 2 l / 100 of bob of pendulum will be, h = l - l cos q = l (1 - cos q). Taking
T =T free fall of the
l
2 ö
1/ 2
1 ö u = 0 , a = g , g = h = l (1 - cos q), v = ?
æ æ
= T ç1 + ÷ = 2ç1 + ÷ Now, v 2 = u 2 + 2 gh = 0 + 2 gl (1 - cos q) f
è 100 ø è 100 ø
2 1 or v = 2 gl (1 - cos q)
\ T¢ -T = = s
100 50
Therefore, loss in seconds per day
68. Let T1 and T2 be the time period of shorter length and longer
1 / 50 length pendulums reapectively. As per question,
= ´ 24 ´ 60 ´ 60 = 864 s nT1 = (n - 1)T2
2
0.5 20
65. According to the law of conservation of So n 2p = (n - 1) 2p
g g
mechanical energy, we get
1 l or n = (n - 1) 40 » (n - 16
)
mv 02 = mgl
2 m Hence, 5n = 6
v0
Þ v 0 = 2 gl Hence, after 5 oscillations they will be in same phase.
Telegram @unacademyplusdiscounts

Oscillations 653

69. In damped oscillation amplitude goes on decaying or g tan q = rw2 = r ´ 4p 2/ T 2


exponentially r
or T = 2p
a = a0 e- bt g tan q
where, b = damping coefficient initially l sin q
a0 = 2p
= a0 e- b ´ 100T , g tan q
3
l cos q
T = time of one oscillation = 2p
g
1
or = e-100 bT …(i)
3 74. Let the distance of vertical disc c of block be pushed in liquid,
Finally a = a0 e - b ´ 200T when black is floating, then buoyancy force
= abxdw g = abxg (Q dw = 1)
or a = a0[ e-100bt ]2
2
The mass of piece of wood = abcd
æ 1ö æ g ö
or a = a0 ´ ç ÷ [ \from Eq. (i) ] So, acceleration = - abxg / abcd = - ç ÷ x
è3ø è cd ø
a0
a= dc
9 Hence, time period, T = 2p
g
70. The motion of M is SHM, with length cm = L2 - d 2
75. We have, T ¢ = 2p l/( g /6) = 6T
(L2 - d 2)1/ 2
T = 2p Hence, the clock will tick in one minute.
g
= 60 / 6 =24.5 times
A C B
2d 76. When the cylinder is given a small downward displacement,
say y, the additional restoring force is due to (i) additional
L L extension y, which is, F1 = ky (ii) Additional buoyancy which
is F2 = AYd g
Total restoring force,
M - F = F1 + F2 = (k + Adg ) y
71. On the inclined plane, the effective acceleration due to = new force constant
gravity 1 k¢
\ n=
g ¢ = g cos 30° = g ´ 3 /2 2p k
l 2l 1 k + Adg
T = 2p = 2p =
g¢ 3g 2p M

72. As, T = 2p l /g 77. Amplitude resonance takes place at a frequency of external


force which is less than the frequency of undamped maximum
1 1 vibration, velocity-resonance takes place ( i. e. , maximum
Þ log T = log 2 + log p + log l - log g
2 2 energy) when frequency of external periodic force is equal to
Differentiating it, we get natural frequency of undamped vibrations.
dT 1 dl 1 dg 1 dg 78. Fig. (1) alone represents damped SHM.
= - =- ( \l is constant )
T 2 l 2 g 2 g
79. Motion given here is SHM starting from rest .
% change in time period
dT 1 dg 80. For damped motion a = a0e-bt
= ´ 100 = ´ 100
T 2 g For first case,
1 æ -2 ö a0
=- ç ÷ ´ 100 = 1% (increase) = a0 e-b ´ 2
2 è100 ø 3
1
73. Resolving tension T in string into two rectangular or = e-2b
3
components, we get
For second case,
T cos q = mg a0
= a0 eb ´ 6
and T sin q = mr w2 n
8
T sin q rw2 1 æ 1ö
So, = tan q = or = e-6b = ( e-2b)3 = ç ÷ n = 33
T cos q g n è3ø
Telegram @unacademyplusdiscounts

654 JEE Main Physics

Round II
1. Electric intensity at B due to sheet of charge, Substituting the given values, we get
1 sq 4 ´ 10 ´ 4.9
E= O K= Nm–1 …(i)
2 e0 (0.5) 2
Force on the bob due to sheet of charge θ On the removal of the body the spring is shortened by x
1 sq T \ mg = kx
F = qE =
2 e0 mg 4.9 ´ 10 ´ (0.5) 2
B Þ x= = [ from Eq. (i) ]
As, the bob is in equilibrium, so C k 4 ´ 10 ´ 4.9
F 0.25
mg F T = = 0.0625 m = 6.25 cm
= =
OC CB BO
mg 4
Sheet of charge
1 7. From the figure, T sin q = mL sin qw2
sq /e 0
CB F sq
\ tan q = = =2 = 324 = 0.5 ´ 0.5 ´ w2
OC mg mg 2e 0mg
324
w2 =
dx 0.5 ´ 0.5
2. Velocity v = = - Aw sin ( wt + p / 4)
dt 324 18
Þ w= = = 36 rad/s
Velocity will be maximum, when 0.5 ´ 0.5 0.5
wt + p / 4 = p / 2
8. As, x1 = a sin( w ´1) = a sin w
or wt = p / 2 - p / 4 = p / 4
and x2 = a sin( w ´ 2) - a sin w
or t = p /4w
x2 sin(2 w) - sin w
m Now =
3. Here, 2 = 2p x1 sin w
k
= sin 2 ´ (2p / 8) - sin 2p / 8 (from question)
m+2
and 3 = 2p 1 - (1 / 2) 2 -1
k = =
(1 / 2) 1
3 m+2
So, = x1 1 2 +1
2 m or = =
x2 2 - 1 ( 2 - 1)( 2 + 1)
or 9m = 4m + 8
or m = 1.6 kg 2 +1
= = 2.414 = 2.4
2 -1
t0 l1 t l2
4. = 2p and 0 = 2p
9 g 7 g 9. The bob is subjected to two simultaneo us, accelerations
2 perpendicular to each other viz acceleration due to gravity g
l1 æ 7 ö æ 49 ö v2
\ =ç ÷ =ç ÷ and radial acceleration aR = towards the centre of the
l2 è 9 ø è 81 ø
R
5. Let h be the depth of in water, then circular path.
Ah r g = mg
m 200
or h= = = 4 cm
Ar 50 ´ 1
2
h ar = v
T = 2p R
g g
1 1 g
Now, n= = 2
T 2p h æv2ö2
\Effective acceleration aeff = g +ç ÷
7 980 èRø
= = 2.5 Hz
2 ´ 22 4 l
\Time period of the simple pendulum T = 2p
m aeff
6. Time period of oscillation, T = 2p
k l l
= 2p = 2p
where m is the mass of body suspended from a spring and K is 2 2 v4
æv ö g2 +
4 p 2m g2 + ç ÷
spring constant of the spring and K = èRø R2
T2
Telegram @unacademyplusdiscounts

Oscillations 655

10. As the length of the scale is 20 cm and it can read upto 50 kg. where, m = mass of the mercury column of length l
The maximum extension of 20 cm will correspond to If r Hg is density of mercury,
maximum weight of 50 kg ´ 9.8 m/s 2. then m = Alr
Alr l
\ T = 2p = 2p
2Arg 2g

12. When t = 1s, y1 = r sin w ´1 = r sin w


when t = 2 s, y 2 = r sin w ´ 2 = r sin 2w
y1 r sin w
\ =
y 2 r sin 2w
Using, F = - kx
1 1 1
|F| = F = kx = = =
2 cos w 2 cos 2p / T 2 cos 2p / 8
Here, x = 20 ´ 10 -2 m 1 1 1
50 ´ 9.8 = = =
k= 2 cos p / 4 2(1 / 2) 2
20 ´ 10 -2
\ y 2 = 2y1
= 2450 N/m
Distance covered in 2nd second = y 2 - y1 = ( 2 - 1)y1
We have for loaded oscillation
\ Ratio = 1 : ( 2 - 1)
m
T = 2p
k 13. As, x = 3 sin 20 p t + 4 cos 20 p t
2 2m é3 4 ù
or T = 4p =5 sin 20 pt + cos 20 pt
k êë 5 5 úû
T 2k (0.60) 2 ´ 2450 = 5 (cos q sin 20 pt + sin q cos 20 pt )
m= =
4p 2 4 ´ (3.14) 2 = 5 sin(20 pt + q)
= 22.36 kg It is a SHM of amplitude 5 cm
\ Weight = mg = 22.36 ´ 9.8
14. As, F = mg = kx
= 219.13 N
For first case,
11. Density of mercury column = r mg 1 ´ 10 N
k= = = 200 Nm-1
Acceleration due to gravity = g x 0.05
For second case,
Area of cross-section = A k 200
w= = = 100 = 10Hz
m 2.0
2h m¢g 2 ´ 10
r= = = 0.1 m
k 200
\ v max = rw = 0.1 ´ 10 = 1 ms-1
l
and T ¢ = 2p
g net

Restoring force, g net = g + a = 10 + 10


F = - Weight of mercury column in excess of g net = 20 m /s2
one arm = - (Volume ´ density ´ g) T
T¢ =
= - ( A ´ 2h ´ r ´ g ) 2
= - 2Argh = - k ´ Displacement in one arm (h)
16. When a mass m is placed on mass M, the new system is of
Clearly, 2Arg = constant = k (say)
mass = (M + m), attached to the spring. New time period of
(As F = - kx) oscillation,
F µ-h M+m
T ¢ = 2p
Hence, motion in SHM, k = 2Arg k
m m M
\Time period, T = 2p = 2p T = 2p
k 2Arg k
Telegram @unacademyplusdiscounts

656 JEE Main Physics

Let v = velocity of mass M while passing through the mean mv 2 0.01 ´ 20


Centrifugal force = = = 0.20 N
position. r 1
v ¢ = velocity of mass (M + m), while passing through the mean Net tension = weight + centrifugal force
position. = (0.01 ´ 10 + 0.20) = 0.30 N
According to law of conservation of linear momentum
20. If v and v' are the velocities of the block of mass M and (M + m)
Mv = (M + m)v' while passing from the mean position when executing SHM.
At mean position, v = A w = and v ¢ = A¢w¢
Using law of conservation of linear momentum, we have
\ MA w = (M + m) A' w'
mv = (M + m) v ¢
æ M ö w M T¢
or A¢ = ç ÷ A= ´ ´A or v ¢ = mv / (M + m)
è M + m ø w' M+m T
Also, maximum PE = maximum KE
æ M ö M+m
=ç ÷´ ´A 1 1
è M + mø M \ k A¢2 = (M + m) v ¢2
2 2
M 1/ 2
=A æ M + mö mv
M+m or A¢ = ç ÷ ´
è k ø (M + m)
17. The Lissajous figure will be parabola if period ratio is 1 : 2 and mv
phase difference is p / 2. =
(M + m)k
Let x = a sin(2wt + p / 2) and y = b sin wt
T1 l1 16
\ sin wt = y / b 21. As, = = =4
T l 1
x
Now, = sin(2wt + p / 2) = cos 2w / t or T1 = 4 T
a
2y2 Let after time t , the pendulum be in the same phase. It will be
= 1 - 2 sin 2 wt = 1 - so, then
b2
t t t -T
2y 2 x æ x - aö = -1 =
or = 1- = -ç ÷ T1 T T
b2 a è a ø
t t -T
b2 or =
or y2 = - ( x - a) 4T T
2a
or t = 4t - 4T
It is an equation of a parabola as given in figure.
or 3 t = 4T
18. When the force F is applied to one side of block A, let the or t = 4T / 3
upper face of A be displaced through distance DL. Then
∆L
22. Let T1 and T2 be the time periods of the pendulum with lengths
1.0 m and 1.21 m respectively
L T2 l2 1.21
θ
A
θ = = = 1.1 … (i)
T1 l1 1

1 2 Let v and v be the vibrations made by two pendulum to


B swing together.
\ v1T1 = v 2T2 …(ii)
For the two pendulum to swing together, required
F / L2 condition is
h= or F = hLDL
DL / L
v1 - v 2 = 1
So, F µ DL and this force is restoring one. So, if the force is
or v1 = v 2 + 1
removed, the block will execute SHM.
\ (v 2 + 1)T1 = v 2T2
From Eq. (i) spring factor = hL
Here, inertia factor = M or (v 2 + 1)v 2 = T2 / T1 = 1.1
M 1
\Time period, T = 2p or 1+ = 1.1
hL v2
1 1
19. When the bob falls through a vertical height of h, the velocity or = 1.1 - 0.1 =
v2 10
acquired at the lowest point,
v = 2gh = 2 ´ 10 ´ 1 = 20 ms-1 or v 2 = 10
Telegram @unacademyplusdiscounts

Oscillations 657

23. If L is the original length of spring, and k is a spring constant of 1


28. Maximum KE = mw2A2; minimum KE = 0
the spring, then 2
1
L + (5 / k) = l ...(i) 0+ mw2A2
2 1
and L + ( 4 / k) = h ...(ii) Averege KE = = mw2A2
2 4
\ l - h = 1/ k æ 1 2 2ö
or k = 1/ ( l - h) ç 0 + mw A ÷ 1
Similarly average PE = ç 2 2 2
÷ / 2 = mw A
and L = (5 h - l) ç 2 ÷ 4
è ø
\Length of spring when subjected to tension 9 N is
= L + 9 / k = (5h - 4l ) + 9 ( l - h) m
29. As, t1 = 2p
= (5l - 4h) k1

24. 4.9 2p 2m
or t12 =
k1

120° 4p 2m
240° or k1 =
A t12
B=A
4p 2m
Similarly, k2 =
t 22
A
4p 2m
and (k1 + k2) =
It is clear from figure, that t 02
2p 4p 2m 4p 2m 4p 2m
B = A, f = 240 = \ = 2 + 2
3 t 02 t1 t2
M 4p 2M 1 1 1
25. As, T1 = 2p or k1 = or = +
k1 T12 t 02 t12 t 22
4p 2M
and k2 = 30. Given, (t) = A cos ( wt + q)
T22
dx
In series combination, Velocity, v= = - Aw sin( wt + q)
dt
k1k2 4p 2M
keff = = 2 = - Aw 1 - cos2( wt + q)
k1 + k2 T1 + T22
M = - A w 1 - x2/A2 = - w A2 - x2
T = 2p = T12 + T22
keff Here, v = p cms-1, x = 1 cm, w = p s-1
1 1 1 So, p = - p A2 - 12
26. As, mw2(r 2 - y 2) = ´ mw2r 2
2 3 2
or ( -1) 2 = A2 - 1
1
or r - y = r2
2 2
or A2 = 2
3
or 3r 2 - 3y 2 = r 2 or A = 2 cm

or 2r 2 - 3y 2 = 0 . m/s 2
31. Given, acceleration due to gravity on moon ( g m) = 17
3 3 Acceleration due to gravity on earth ( g e) = 9.8 m/s 2
or r= ´y = ´4
2 2
Time period on earth Te = 3.5 s
= 2 6 cm Time period on moon Tm = ?
27. In equilibrium, T = mg On the surface of the earth, time period = Te
1 2 l
Work done = mg = mgx = kx \ Te = 2p …(i)
2 ge
2mg 2T On the surface of the moon, time period = Tm
or x= =
k k l
\ Tm = 2p …(ii)
Energy stored = mg x = T x gm
2T 2T 2 g e , g m are acceleration due to gravity on the earth and
=T ´ =
k k moon surface respectively.
Telegram @unacademyplusdiscounts

658 JEE Main Physics

On dividing Eq. (i) by Eq. (ii), p 3p


Force is zero, when cos wt = 0 or wt = or ,
Te 2p l gm 2 2
= ´ 2p p 3p
Tm 2p l ge i. e. , t = or
T 2 2
Te gm 2p p T
Þ = If t = , then, t =
Tm ge T 2 4
æ ge ö 2p 3p 3T
Þ Tm = ç ÷ Te If t= , then t = s (given)
è gm ø T 2 4
2p
Putting the values, we get Acceleration is maximum if cos wt =1or 2 p or t = 2p or
T
9.8 4T
Tm = ´ 3.5 t =T = s (given)
1.7 4
= 8.4 s Velocity is maximum if sin ( wt + p ) =1or wt + p = p /2
1 p 2p p T
32. Given, mw2r 2 = 80 J; or wt = - p = - p / 2 or t = - or t = - s
2 2 T 2 4
2 1 2 2 1 2 2 2
1 1 æ3 ö PE = m w y = mw a cos wt
\ PE = mw2y 2 = mw2 ´ ç r ÷ 2 2
2 2 è4 ø
1
9 æ1 KE = m w2a2 sin 2 wt
ö 9 2
= ç mw2r 2÷ = ´ 80 = 45 J
16 è 2 ø 16
If PE = KE, then cos2 wt = sin 2 wt or cos wt = sin wt or
33. Given, x = A cos ( wt + a) tan wt =1
p 2p p T
= - A sin (w + a) ...(i) or wt = or t = or t = s
4 T 4 8
y = A cos( wt + a) ...(ii)
39. As per question, the direction from A to B, i. e. , from A towards
Squaring and adding Eqs. (i) and (ii), we get mean position O is positive, therefore if a particle starting from
x2 + y 2 = A2[sin 2( wt + a) + cos2( wt + a)] = A2 A reaches at C, where AC = 3 cm, then its direction of motion
is towards the mean position O. Hence, its velocity is positive,
It is an equation of a circle. The given motion is anti-clockwise. acceleration is positive and force is positive.
m 3 cm
34. As, T = 2p
k
B D O C
m T 5
and T' = 2p = = s = 2.5 s
4k 2 2 4 cm 5 cm

35. Maximum KE = Maximum PE When a particle from B reaches point D, where BD = 4 cm,
1 1 1 then its direction if motion is towards BA i. e. , along BO, then
mv 2 = kx2 = ´ 65 ´ (0.11) 2
2 2 2 velocity, acceleration and force are negative.
65 ´ (0.11) 2 When particle reaches at B, its velocity becomes zero but its
or v2 = . ms-1
or v = 11
650 ´ 10 -3 acceleration and force are towards BA, i. e. , negative.

36. A ball bearing when released a little above the lower limit 40. Torque is same for both the cases
inside a smooth curved ball, will execute SHM with a definite I
T = 2p
period. mgd L
37. Phase is the state of a particle as regards its position and IA > IB
direction is motion w.r.t. mean position. In the given curve \ WA < WB mg
phase is same when t = 1s and t = 5 s. Also phase is same when
t = 2 s and t = 6 s.
38. For the given SHM, the displacement is given by y = a cos wt
dy
Velocity, v= = - w sin wt = aw sin( wt + p ) 41. The moment of inertia of a cylindrical rod about axis of wire
dt
(i. e. , an axis passing through the centre of rod and
dV perpendicular to its length) is
Acceleration, A = = aw2 cos wt
dt
é L2 R 2 ù
Force = mass ´ acceleration = - m a w2 cos wt I =Mê + ú
ë12 4 û
Telegram @unacademyplusdiscounts

Oscillations 659

42. As in torsional vibrations of this type restoring torque is As during SHM the direction of deflection is opposite of
directly proportional to angular displacement (e.g., t = - kq, displacement. It may be +ve or -ve.
where, k = restoring torque per unit twist = spring factor), Hence, maximum acceleration = ±144p 2 ms-2
hence the vibrational are angular harmonic oscillations.
51. As, retardation= bv
43. As torsional vibrations are angular SHMs, hence m will be \ Retardation force = mbv
replaced by I, the moment of inertia of oscillating body.
\ Net restoring torque when angular displacement q, is given
l by
\ T = 2p
k = - mgl sin q + mbvl
44. We can write, v 2x0 2
= w (A 2
- x02) bn
\ Ia = - mgl sin q +
l
v 2x
or 2
A = 20 + x20 (\w = 4) where, I = ml 2
w
d 2q g bn
or A = 25 2 cm \ 2
= a = - sin q +
dt l l
45. Let x (t) = A sin( wt + f 0) For small damping the solution of the above differential
and v(t ) = Aw cos( wt + f 0) equation will be
bt
We have, -
\ q = q0 e 2 sin ( wt + f)
x (0) = A sin f 0 = 25
bt
25 1 -
\ sin f 0 = = (\ A = 25 2 ) \ Angular amplitude will be q0 × e 2
A 2
According to question, in t time (average life time)
v(0) = Aw cos f 0
1
100 1 p angular amplitude drops to value of its original value ( q)
\ cos f 0 = = \ f0 = e
Aw 2 4 bt
q -
The equation of motion \ = q0 e 2
e
æ pö
x (t ) = 25 2 sin ç 4 t + ÷ bt
è 4ø Þ =1
2
æ pö 2
or y = 25 2 sin ç 4 t + ÷ \ t=
è 4ø b
47. If the soldiers while crossing a suspended bridge march in 52. X1 = A sin ( wt + f1) = X 2 = A sin ( wt + f 2)
steps the frequency of marching steps of soldiers may match
with the natural frequency of oscillations of the suspended é æ f + f2 ö æ f - f2 ö ù
\ X1 - X 2 = A ê2 sin ç wt + 1 ÷ sin ç 1 ÷
bridge. In that situated resonance will take place then the ë è 2 ø è 2 ø úû
amplitude of oscillation of the suspended bridge will increase æ f - f2 ö
enormously which may cause the collapsing of the bridge. To Þ A = 2 A sin ç 1 ÷
è 2 ø
avoid such situation the soldiers are advised to break steps on
f1 - f 2 p
suspended bridge. =
2 6
48. Time period of simple pendulum of length l is p
Þ f1 - f 2 =
l 3
T = 2p
g
53. Law of conservation of momentum gives,
Tµ l
Mv max = (m + M) v new and v max = A1w1
DT 1 Dl Mv max
= \ v new =
T 2 l (m + M)
DT 1
\ = ´ 3 = 15
. % Now, v new = A2 × w2
T 2
MA1 k k
49. Correct explanation of the assertion is that spring constant of a = A2
(m + M) M (m + M)
given material and thickness so, inversely proportional to its
1 M
length, i. e., k µ A2 = A1
l (m + M)
1/ 2
50. Maximum acceleration A1 æ m + M ö
Þ =ç ÷
= w2A = 4p 2v 2A = 4p 2 ´ (60) 2 ´ 0.01 = 144p 2 ms-2 A2 è M ø
Telegram @unacademyplusdiscounts

660 JEE Main Physics

54. Acceleration a = w2x 61. When lift acceleration upwards with acceleration g /3, then
2
aT w2xT æ2 p ö 4 p2 effective acceleration due to gravity is g ¢ = g + g /3 = 4g /3.
\ = = w2T = ç ÷ T=
x x è T ø T l
Now, T = 2p
It is a constant term for SHM i. e. ,it does not change with time. g

55. According to question, l 3 l 3


and T ¢ = 2p = 2p = T
( 4 g /3) 2 g 2
|v max| = | amax|
i. e. , wa = w2a 2p
62. As, y = a sin wt = a sin t
2p T
Þ w =1=
T when t = 2 s, let y = y1. Then
\ T = 2p é 2p ù æpö a
y1 = a sin ´ 2 = a sin ç ÷ = …(i)
Þ T = 2 ´ 3.14 ëê 16 ûú è 4ø 2
Þ T = 6.28 s After 4 second from mean position,
56. The particle completes one oscillation in time T. Therefore, in a
y1 =
time 2T, it will complete two oscillations and will reach to its 2
starting point, i. e. , initial position. Therefore, the
and velocity = 4 ms-1
displecement is zero.
57. Here the two springs are in series. Their effective spring Since, velocity = w a2 - y12
k1 k2
constant, k = æ 2p ö a2 p a
k1 + k2 \ 4 = ç ÷ a2 - = ´
è 16 ø 2 8 2
\The period of oscillation, 32 2
or a= m
M p
T = 2p
k
63. Given, x = x0 cos( wt - p / 4)
16 (k + k )
= 2p = 8p 1 2 dx æ pö
k1k2 / (k1 + k2) k1 k2 velocity, v= = - x0 w sin ç w t - ÷
dt è 4ø
l dv pö
58. Here, T = 2p …(i) æ
g Acceleration, a = = - x0 w2 cos ç wt - ÷
dt è 4ø
When lift moves upward with acceleration a, let time period
= x0 w2 cos [ p + ( w t - p / 4)]
becomes T / 2. Then
T l = x0 w2 cos [ w t + 3p / 4)]
= 2p …(ii)
2 g+a Comparing it with acceleration,
Dividing Eq. (i) by Eq. (ii), we have a = A cos( w t + d)
1/ 2 we have, A = x0 w2, d = (3p / 4)
g+a æ aö
2= = ç1 + ÷
g è gø 64. Kinetic energy of the particle executing SHM at an instant
a when its displacement from the mean position is y, is
\ 4 = 1+
g 1
K = mw2( a2 - y 2)
2
or a=3g
1
59. As, total energy, E = 2p 2m2a2, i. e. , E µ a2 At the mean position, y = 0; K1 = mw2a2
2
60. Equation, is y = 2 (sin 5pt + 3 cos 5pt) At the extreme position, y = a;
1
é1 3 ù K2 = mw2( a2 - a2) = 0
= 2 ´ 2ê sin 5pt + cos 5pt ú 2
ë2 2 û K + K2 æ 1 ö1
\ Average kinetic energy = 1 = ç mw2a2 + 0 ÷
é p p ù 2 è2 ø2
= 4 cos sin 5pt + sin cos 5pt
ëê 3 3 ûú 1
= mw2a2
æ pö 4
= 4 sin ç5 p t + ÷
è 3ø 1
= m( 4p 2v 2) a2 = mp 2v 2a2
It represents a SHM. Its amplitude is 4. 4
Telegram @unacademyplusdiscounts

Oscillations 661

65. Let displacemant equation of particle executing SHM is k ´k k


69. In series, ks = =
y = a sin wt k+k 2
As particle travels half of the amplitude from the equilibrium In parallel, kp = k + k = 2k
position, so In series, frequency of vertical oscillation,
a 1 ks
y= vs =
2 2p M
a In parallel, frequency of vertical oscillation,
Therefore, = sin wt
2
1 kp
1 p vp =
or sin wt = sin 2p M
2 6
p p vs k k/2 1
or wt = or t = \ = s = =
6 6w vp kp 2k 2
p æ 2p ö
or t= ç as w = ÷ 70. The coin will be leaving its constant at the lowest point when
2
æ öp è T ø
6ç ÷ restoring force is equal to or greater than the weight of the
èT ø
coin, i. e. ,
or t=
T mw2 ³ mg or a ³ g / w2
12
71. As per question,
Hence, the particle travels half of the amplitude from the
T 75 75 1
equilibrium in seconds. KE = E= ´ m a2w2
12 100 100 2
1 75 1
66. Let the minimum amplitude of SHM is a. Restoring force on \ m a2w2 cos2 w t = ´ m a2w2
2 100 2
spring 3
F = ka or cos2 wt =
4
Restoring force is balanced by weight mg of block. For mass 3 p p
to execute simple harmonic motion of amplitude a. or cos w t = = cos or w t =
2 6 6
\ ka = mg p p 1
mg or t= = = s
or a= 6p 6(2p / 2) 6
k
Here, m = 2 kg , k = 200 Nm-1, g = 10 ms-2 72. Given, w a2 - u2 = w2y
2p
2 ´ 10 10 10 or a2 - y 2 = w y = y
a= = m= cm = 10 cm T
200 100 100
2py 2p ´1 2p
Hence, minimum amplitude of the motion should be 10 cm, or T= = = s
2
a -y 2 2
2 -1 2 3
so the mass gets detached from the pan.
67. Kinetic energy = Potential energy 73. Kinetic energy of a particle while displacement is y, will be
1 1 1 3 3 1
Þ mw2( a2 - y 2) = mw2y 2 = mw2( a2 - y 2) = E = ´ mw2a2
2 2 2 4 4 2
a 2 2 3 a2
Þ y= or a -y =
2 4
2
a a
68. Let x be the extension in the wire, when mass m hangs from or y2 = or y =
4 2
the end of spring. Then
F F F YA l
y = ´ or ´ 74. As, T = 2p i. e. , T µ l
A x x L g
The wire and the spring are in series, so total spring constant, T¢ 9l
or = =3 or T' = 3T
YA T l

kk¢ L = kY A dy
K= = 75. Given, y = kt 2, = 2kt
k + k¢ k + YA / L kL + Y A dt
m d 2y
As time period, T = 2p and = 2k = 2 ´ 1 = 2 ms-2
k dt 2
m m(kL + YA) So point of suspension of pendulum is moving upwards with
\ T = 2p = 2p acceleration a = 2 ms-2.
kYA / (kL + YA) kYA
Telegram @unacademyplusdiscounts

662 JEE Main Physics

Then, effective acceleration due to gravity on pendulum, 80. Accleration, a = 0.5 ms-2
g ¢ = ( g + a) = 10 + 2 = 12 ms-2
amplitude, a = 0.1 a
l displacement, y = 0.1 m
T1 = 2p
g
Using the formula of maximum acceleration a = w2y
l 0.5
and T2 = 2p or 0.5 = w2 ´ 0.02 or w2 = = 25
g' 0.02
T12 g ' 12 6 So, w=5
Þ = = =
T22 g 10 5 Now, maximum velocity is
l v = aw = 0.1x5 = 0.5 ms-1
76. T = 2p ; so T µ l
g 81. Pressure applied by piston
When plugged hole near the bottom of the oscilliating bob Mg
= p0
gets suddenly unplugged, water flows out, the value of l A
increase because when level of the position of centre of Mg = p0 A …(i)
gravity of bob and water falls down. It is so water falls upto the
centre of the bob. After that as water comes out, the position
of the centre gravity of bob and water rises and finally it x
reaches at the centre of bob when whole water leakes out of
bob. Therefore, T first increases and then decreases to the x0
original value.
77. The equation of SHM,
d 2x d 2x Here, the system is completely isolated, so the process will
2
+ ax = 0 or = - ax be adiabatic.
dt dt 2
Comparing it with the equation of SHM p0V0g = pV g
d 2x p0 Ax0g = pA ( x0 - x ) g
= - w2x = ax
dt 2 p0 x0g
p=
w2 = a or w = a ( x0 - x) g
2p 2p
or T= = Let piston is displaced by x
w a
æ p xg ö
1 - cos 2wt 1 1 Mg - ç 0 0 g ÷ A = frestoring
78. Given, y = sin2 wt = = - cos 2 w t è ( x0 - x) ø
2 2 2
dy 1 æ x0g ö
v= = ´ 2w sin 2wt = w sin wt p0 A ç1- ÷ = Frestoring ( x0 - x » x0)
dt 2 è ( x0 - x) g ø
dv g p Ax
Acceleration = = 2w2 cos 2w t F=- 0
dt x0
As accleration is directly proportional to displacement and 1 g p0 A 1 g p0 A2
directed towards the mean position, hence motion is not in \ f= =
2p x0M 2p MV0
SHM but a periodic motion. Its period,
bt
2p p -
T' = = 82. Amplitude of damped oscillator A = A0e 2m
2w w b (5 )
-
79. As, y1 = 0.1sin(100 pt + p / 3) After 5 s, 0.9 A0 = A0 e 2m

b (5 )
dy1 -
\Velocity, v1 = = 0.1 ´ 100 p cos (100 pt + p /3) Þ 0.9 = e 2m …(i)
dt
After 10 more second
and y 2 = 0.1cos pt = 0.1sin ( pt + p /2) (15 )
dy -b
\Velocity, v 2 = 2 = 0.1 ´ p cos ( pt + p /2) A = A0 e 2m
dt 3
æ - 5b ö
\ Phase difference of the velocity of particle 1 w.r.t. particle 2. A = A0 çç e 2m ÷÷ …(ii)
= (100 pt + p / 3) - ( pt + p / 2) è ø
= 99pt + p / 3 = p / 2 From Eqs. (i) and (ii)
p p p A = 0.729 A0
At t = 0 , Phase difference, = - = -
3 2 6 Hence, a = 0.729
Telegram @unacademyplusdiscounts

16 Waves
JEE Main MILESTONE
< Wave Motion < Principle of Superposition of Waves
< Speed of Waves < Standing or Stationary Waves
< Progressive Wave < Fundamental Mode and Harmonics
< Sound Waves < Musical Sound and Noise
< Reflection and Transmission of Waves < Beats

16.1 Wave Motion


In simple terms, we can say that wave motion involves transfer of disturbance
(energy) from one point to the other with particles of medium oscillating about
their mean positions i. e. , the particles of the medium do not travel themselves
along with the wave. Instead, they oscillate back and forth about the same
equilibrium position as the wave passes by. Only the disturbance is propagated.
There are two types of wave motion
Waves occur, when a system is
(i) Transverse Wave disturbed from its equilibrium
In transverse wave, the particles of the medium oscillate perpendicular to the position and this disturbance
direction in which the wave travels. Travelling waves on a tight rope are transverse propagates from one region to
waves. If one end of the rope is rigidly fixed and the other end is given periodic up other.
and down jerks, the disturbance propagates along the length of the rope but the
particles of the rope oscillate up and down. Disturbance travels along the rope in
the form of crests (upward peaks) and troughs (downward peaks).

Wave motion

Oscillating particles

C Crest
T — Trough
C

Vertical arrows show the oscillation of particles and the horizontal line arrows
show the direction of the motion.
Telegram @unacademyplusdiscounts

664 JEE Main Physics


Compression
The stretched string of sitar, violin, sonometer, etc.,
execute transverse vibrations. Further, all the λ
electromagnetic waves are also transverse in nature.
Various terms related to wave motion are as follows
1. Wavelength The distance travelled by the disturbance
in the time, the particle of the medium completes one
vibration, is called wavelength.
It is found that in case of a transverse wave,
wavelength is equal to the distance between two
consecutive crests or troughs. It is denoted by l. λ
Rarefaction
2. Frequency It is the number of waves produced per
second, in the medium. It is found that in case of a longitudinal wave, wavelength
is equal to the distance between two consecutive
It is denoted by n. If T is period of vibration of the
compressions or rarefactions. The velocity of longitudinal
particle of the medium, then
waves is also given by v = nl.
1
n= Longitudinal wave motion is possible in a medium
T
possessing elasticity of volume i. e. , in solid, liquid and
3. Velocity The velocity of transverse wave motion is gases.
given by
distance travelled by the wave
v=
time taken 16.2 Speed of Waves
Since, the distance equal to wavelength (l ) is travelled
by the wave in a time equal to the period of vibration
(i) Speed of Transverse Waves
(T) of the particle of the medium, we have The expression for speed of transverse waves in a solid can
l 1 be obtained theoretically and verified experimentally.
v = = æç ö÷ l
T èT ø (a) If medium is solid, then
h
or v = nl v=
r

(ii) Longitudinal Wave where, h is modulus of rigidity and r is density of solid


materials.
In this type of wave motion, the particles of the medium
(b) In a stretched string,
oscillate about their mean or equilibrium position along T
the direction of propagation of the wave motion itself. v=
m
Longitudinal wave travels in the form of compressions and where, T is tension and m is linear mass density of the
rarefactions. string.
A compression is a region of the medium in which particles
(ii) Speed
come to of Longitudinal
distances less than theWaves (sound)
normal distance between
them. Following are the expressions for the speed of
A rarefaction is a region of the medium in which particles longitudinal waves in the different types of media
of the medium go apart to distances greater than the (a) If the medium is solid,
normal distance between them. 4
B+ h
When a tuning fork is set into vibrations, its prong v= 3
r
compresses the air medium just in front of it. As a wave of
compression progresses in the air along horizontal, the where B, h and r are values of bulk modulus, modulus
particles of the air medium also execute periodic motion of rigidity and density of the solid respectively.
along horizontal. The longitudinal wave can also be set in If the solid is in the form of a long rod, then
a clamped rod (by pulling it along its length) or in an organ Y
v=
pipe. r
where, Y is the Young’s modulus of the solid material.
Telegram @unacademyplusdiscounts

Waves 665

(b) In a liquid, m = 6.9 ´ 10 –3 kgm–1


B
v= Tension, T = 60 N
r
The speed of the wave on the wire is given by
where B is the bulk modulus of the liquid. T 60 N
v= = = 93 ms–1
(c) According to Newton’s formula, speed of sound in a m 6.9 ´ 10 –3 kgm–1
gas is obtained by B replaced by initial pressure p of
the gas i.e., B = p.
p
Sample Problem 3 Mass of 1 mole of air is 29 ´ 10 -3 kg.
v= The speed of sound in air at standard temperature and pressure is
r
(a) 331 ms -1 (b) 3 ´ 10 8 ms -1
Laplace’s correction According to Laplace, the formula (c) 280 ms -1 (d) 4 ´ 10 6 ms -1
for speed of sound in the gases should be Interpret (c) 1 mole of any gas occupies 22.4 L at STP.
gp gRT Therefore, density of air at STP is
v= =
r M mass of one mole of air
r0 =
Cp volume of one mole of air at STP
where, g= 29 ´ 10 -3 kg
CV r0 = = 1.29 kgm–3
22.4 ´ 10 –3 m3
After substituting the appropriate values in the relation,
From Newton’s formula,
we get the theoretical value of speed of sound in air at NTP
p pressure
which is found to be 332.5 ms–1, which is in good v= =
r0 density
agreement with the experimental value.
At STP, p = 1.01 ´ 10 5 Nm–2
Note The propagation of longitudinal waves through a medium
requires that medium should possess bulk modulus of elasticity, while 1.01 ´ 10 5 Nm–2
v= = 280 ms–1
transverse waves can propagate only in the medium, which possesses 1.29 kgm–3
shear modulus of elasticity.
Sample Problem 4 Determine the speed of sound
Sample Problem 1 A steel wire has a length of 12.0 m and waves in water, and find the wavelength of a wave having a
a mass of 2.10 kg. What should be the tension in the wire so that frequency of 242 Hz. (Take, Bwater = 2 ´ 10 9 Pa.)
the speed of a transverse wave on the wire equals the speed of
(a) 5.5 m (b) 5.84 m
sound in dry air at 20°C = 343 ms–1)?
(c) 6.84 m (d) 6.5 m
(a) 2.06 × 103 N (b) 2.06 × 104 N
(c) 3.8 × 105 N (d) 3.8 × 106 N Interpret (b) Speed of sound wave,
Interpret (b) Here, speed of sound in air, v = 343 ms–1; B (2 ´ 10 9)
v= = = 1414 ms–1
Length of the wire, l = 12.0 m; r 10 3
Total mass of the wire, M = 2.10 kg v
Wavelength, l = = 5.84 m
Therefore, mass per unit length of the wire, f
M 2.10
m= = = 0.175 kgm–1 –3
Sample Problem 5 The density of air at NTP is 1.29 kgm .
l 12.0
Assume air to be diatomic with g = 1.4. The velocity of sound at
T
Now, v= 127ºC is
m
(a) 382.8 ms–1 (b) 350 ms–1
or T = v 2m = (343) 2 ´ 0.175 (c) 350.6 ms–1 (d) 348.6 ms–1
= 20588.6 N = 2.06 ´ 10 4 N Interpret (a) Velocity of sound in air at NTP
gp 1.4 ´ 1.013 ´ 105 Nm–2
Sample Problem 2 A steel wire 0.72 long has a mass of . = = = 331.6 ms–1
r 1.29 kgm–3
5 ´ 10 -3 kg. If the wire is under a tension of 60 N, speed of the
transverse wave on the wire is [NCERT] The velocity of sound is proportional to the square root of absolute
(a) 40 ms -1 (b) 93 ms -1 temperature
(c) 32 ms -1 (d) 15 ms -1 v2 T
Þ = 2
Interpret (b) Mass per unit length of the wire v1 T1
5 ´ 10 -3 kg T2 273+127
m= Þ v 2 = v1 = 331.6 = 382.8 ms–1
0.72 m T1 273 + 27
Telegram @unacademyplusdiscounts

666 JEE Main Physics


y
16.3 Progressive Waves λ
A wave which travels continuously in the same direction A
without any change in its amplitude is called a x
O
progressive wave or a travelling wave. A progressive wave
may be transverse or longitudinal.

(ii) The equation of progressive or harmonic wave moving


General form of a Wave Function along positive x-axis is
Let us consider a wave travelling along positive direction y = A sin (wt - kx)
of x-axis with velocity v. The displacement of the wave is
In general, y = A sin (wt - kx + f)
given by
y (x, t ) = f (x - vt ) (iii) The equation of progressive or harmonic wave moving
along negative axis is
x being the distance of wave pulse from origin.
y = A sin (wt + kx)
The following points may be noted about the wave In general, y = A sin (wt + kx + f)
function representing the travelling wave pulse.
(iv) Different forms of progressive or harmonic wave
(i) For a wave pulse travelling from right to left, i.e., along
x x
negative direction of x-axis, the wave function will be y = A sin æçt ± ö÷ , y = A cosw æçt ± ö÷
è vø è vø
y (x, t ) = f (x + vt )
y = A sin (wt ± kx), y = A cos (wt ± kx)
(ii) For a wave pulse travelling with velocity v, the wave t x t x
y = A sin2p æç ± ö÷ , y = A cos2p æç ± ö÷
function depends on x and t only through the èT l ø èT l ø
combination x - vt (for the wave pulse travelling from
y = A sin k (vt ± x), y = A cos k (vt + x)
left to right) or x + vt (for the wave pulse travelling from
right to left.) (v) The coefficient of sin or cos function i.e., A gives the
amplitude of the wave while its argument (wt ± kx)
(iii) If phase wt - kx = phase = constant, then the shape of
denotes phase.
wave remains constant.
¶y (vi) In progressive or harmonic wave, we have two
(iv) Particle velocity, vparticle = . periodicities, one in time given by the period T, and
¶t
one in space given by the wavelength l, with the two
(v) For a wave, vparticle = - C (slope).
related by
(vi) For a given t, y-x graph gives the shape of pulse on l = vT
string. l wl w
Þ v = ln = = =
T 2p k
Displacement Relation for Progressive or
(vii) Phase difference and path difference At any instant t, if
Harmonic Wave
f1 and f2 are the phases of two particles whose
(i) If a travelling wave is a sine or cosine function of ( x - vt ) distances from the origin are x1 and x2 respectively,
or ( x + vt ), the wave is said to be progressive or then
harmonic wave. f1 = (wt - kx1 )
Therefore, y ( x, t ) = A sin k ( x - vt ) and f2 = (wt - kx2 )
y ( x, t ) = A cos k ( x - vt ) Þ f1 - f2 = k ( x2 - x1 )
Here, 2p
Phase difference, Df = (Path difference Dx)
2p l
k= = number of wavelengths in the distance 2p.
l (viii) Phase difference and time difference If the phases of a
= wave number or propagation constant particle distance x from the origin is f1 at time t1 and f2
2p at time t2, then f1 = (wt1 - kx) and f2 = (wt2 - kx)
Thus, y ( x, t ) = A sin ( x - vt ) = A sin (kx - wt )
l Þ f1 - f2 = w (t1 - t2 )
2 pn 2p
where, w = kv = Phase difference (Df) = time difference (Dt )
l T
= angular frequency
Telegram @unacademyplusdiscounts

Waves 667

Energy in Wave Motion Sample Problem 6 A wave travelling along a string is


described by
The energy is associated with every wave motion. y ( x, t ) = 0.005 sin (80 x - 3 t )
Regarding the energy in wave motion, we come across
in which numerical constants are in SI units (0.005 m,
three terms namely, energy density (u), power (P) and
80 rad m –1, 3 rad s -1). The wavelength l is then given by
intensity (I). Let us consider them one by one.
(a) 2.85 cm (b) 7.85 cm
Energy Density (u) The energy density is defined as the (c) 10.83 cm (d) 18.2 cm
total mechanical energy (kinetic + potential) per unit
volume of the medium through which the wave is passing. Interpret (b) Comparing the given displacement equation,
with standard equation
So, kinetic energy per unit volume y ( x, t ) = a sin (kx - wt )
1
DK = rw2 A2 cos2 (kx - wt ) We have amplitude of the wave is 0.005 m = 5 mm angular wave
2 number k and angular frequency w are k = 80 m–1 and w = 3 s–1.
Potential energy per unit volume Then, from equation
1 2p 2p
DU = rw2 A2 cos2 (kx - wt ) l= = = 7.85 cm
2 k 80 m–1
Total energy per unit volume
DE = DK + DU = rw2 A2 cos2 (kx - wt ) Sample Problem 7 A stretched string is forced to transmit
Thus, energy density transverse waves by means of an oscillator coupled to one end.
The string has a diameter of 4 mm. The amplitude of the
u = < DE > = < (DK + DU ) >
oscillation is10 -4 m and the frequency is 10 Hz. Tension in the
= rw2 A2 < cos2 (kx - wt ) >
. ´ 103 kgm -3. Find
string is 100 N and mass density of wire is 42
1
= rw2 A2 (a) the equation of the waves along the string
2
(b) the energy per unit volume of the wave
Power (P) If we consider a transverse wave on a string,
(c) the average energy flow per unit time across any section
then the instantaneous rate at which energy is transferred
of the string
along the string is called power. Thus,
P = Energy density ´ Volume Interpret (a) Speed of transverse wave on the string is,
1 T
= rw2 A2 ´ Dv v= (as m = rS)
2 rS
1 Substituting the values, we have
= rw2 A2Dv
2 100
v=
Intensity (I) Flow of energy per unit area of cross-section of æ pö
( 4.2 ´ 10 3) ç ÷ ( 4.0 ´ 10 –3) 2
the string in unit time is known as intensity of the wave. è 4ø
Thus, = 43.53 ms–1
Power P
I = = w = 2 pf = 20 p rad/s
Area of cross - section S
= 62.83 rad /s
1
or I = rw2 A2v w
2 k = = 1.44 m–1
v
Note The intensity of sound waves is given by \ Equation of the waves along the string
2
Pmax y ( x, t ) = A sin (kx - wt )
I=
2rv = (10 -4 m) sin [(1.44 m–1) x - (62.83 rad s–1) t ]
where, Pmax is the maximum change of pressure in the medium. (b) Energy per unit volume of the string,
The intensity of waves emitting in all directions due to a point source 1
varies inversely as the square of the distance (r). u = energy density = rw2A2
2
1
i.e., Iµ 2 Substituting the values, we have
r
æ 1ö
The intensity of waves from a linear source varies inversely as the u = ç ÷ ( 4.2 ´ 10 3) (62.83) 2 (10 -4) 2
1 è2ø
distance (r) perpendicular to the source, i.e., I µ
r = 8.29 ´ 10 –2 Jm–3
Telegram @unacademyplusdiscounts

668 JEE Main Physics

(c) Average energy flow per unit time


P = power
16.4 Sound Waves
æ1 ö Sound wave is defined roughly as any longitudinal wave.
= ç rw2A2÷ ( Sv) = (u) ( Sv)
è2 ø These waves travel with a speed of 332 ms–1 (approx).
Substituting the values, we have They are classified into following categories
æpö (i) Infrasonics The longitudinal waves having
P = (8.29 ´ 10 –2) ç ÷ ( 4.0 ´ 10 –3) 2 ( 43.53)
è 4ø frequencies below 20 Hz are called infrasonics. These
= 4.53 ´ 10 –5 Js–1 waves cannot be heard. These waves can be heard by
snakes.

Sample Problem 8 Equation of a transverse wave (ii) Audible waves The longitudinal waves having the
frequency between 20 Hz and 20000 Hz are called
travelling in a rope is given by y = 5 sin (4.0 t –0.02 x), where y
audible waves. Human can hear these waves.
and x are expressed in cm and time in seconds. Calculate the
amplitude, frequency and velocity of the wave. (iii) Ultrasonics The longitudinal waves having the
(a) 8 cm, 0.8673 cycle s–1,
200 cms–1 frequencies above 20000 Hz are called ultrasonics.
These waves are also called supersonic waves or
(b) 5 cm, 0.673 cycle s–1, 200 cms–1
supersonics.
(c) 5.8 cm, 0.673 cycle s–1, 250 cms–1
(d) None of the above
Interpret (b) Given, y = 5 sin ( 4.0 t - 0.02 x ). Comparing this
Check Point 1
2p ö
with the standard equation of wave motion, y = A sin æç2 pft - x÷ 1. What type of mechanical waves do you expect to exist in
è l ø (a) vacuum, (b) air, (c) inside the water, (d) rock and (e) on the
where, A, f and l are amplitude, frequency and wavelength surface of water?
respectively. Thus, amplitude A = 5 cm, 2 pf = 4 2. Is it possible to have longitudinal waves on a string?
4 3. What is the phase difference between the waves
Frequency, f= = 0.673 cycle s–1
2p y = A cos ( wt + kx ) and y = A sin ( wt + kx ) ?
2p 4. Why is sound heard in CO 2 more intense in comparison to
Again, = 0.02
l sound heard in air?
2p 5. What will be the speed of sound in a perfectly rigid rod?
or wavelength, l = = 100 p cm
0.02 6. What do mechanical waves transfer energy, matter, both or
4 2p neither?
Velocity of the wave v = f l = ´
2 p 0.02 7. Which characteristic of the medium determines the velocity of
= 200 cms–1 sound waves?
8. The speed of sound waves depends on temperature but speed
Sample Problem 9 In above example, find the maximum of light waves does not. Why?
transverse speed and acceleration of a particle in the rope.
(a) 40 cms–2
(b) 50 cms–2
(c) 80 cms–2
16.5 Reflection and Transmission
(d) 75 cms–2 of Waves
Interpret (c) Transverse velocity of the particle, When sound waves are incident on a boundary
¶y separating two media, a part of it is reflected back into the
u= = 5 ´ 4 cos ( 4.0 t - 0.02 x )
¶t initial medium while the remaining is partly absorbed
= 20 cos ( 4.0 t - 0.02 x ) and partly transmitted into the second medium.
Maximum velocity of the particle = 20 cms–1
¶y
Characteristics
Particle acceleration, a = = 20 ´ 4 cos ( 4.0 t - 0.02 x)
¶t (i) In case of reflection and transmission of sound, the
Maximum particle acceleration = 80 cms–2 frequency of the wave remains unchanged, i.e.,
wi = wr = wt = w.
Telegram @unacademyplusdiscounts

Waves 669

(ii) The incident ray, the reflected ray, normal and the
refracted ray are always in the same plane. 16.6 Principle of Superposition of
(iii) In case of reflection of sound, Waves
angle of incidence = angle of reflection
(iv) In case of refraction of sound, Two or more waves can traverse the same space
sin i vi independently of one another. Thus, the resultant
= displacement of each particle of the medium at any
sin r ¢ vt
instant is equal to the vector sum of displacements
(v) In case of reflection from a denser medium or rigid produced by the two waves separately. This principle is
support or fixed end there is inversion of the reflected called principle of superposition of waves.
displacement wave, i.e., if the incident wave is
y = At sin (wt - kx) the reflected wave will be
y = - Ar sin (wt + kx)
Interference
= Ar sin (wt + kx + p ) When two waves of the same frequency, superimpose
each other, there occurs redistribution of energy in the
i.e., in case of reflection from a denser medium,
displacement wave changes in phase by p while in medium which causes either a minimum intensity or
case of reflection from a rarer medium, no inversion of maximum intensity. This phenomenon is called
wave or phase change occurs. The transmitted wave is interference of waves.
never inverted. Let at a given point two waves arrive with a phase
(vi) On reflection, the amplitude and intensity of wave may difference f and the equation of these waves is given by
decrease. y1 = A1 sin wt,
(vii) When a transverse wave is reflected from a denser y2 = A2 sin (wt + f )
medium, the trough is reflected as crest and
vice-versa.
Then by principle of superposition
(viii) When a transverse wave is reflected from a rarer y = y1 + y2
medium, crest and trough do not invert after reflection. Þ y1 = A1 sin wt + A2 sin (wt + f )
(ix) When a longitudinal wave is reflected from a denser = A sin (wt + f )
medium, the compression and rarefaction do not
A2
invert after reflection. A
(x) When a longitudinal wave is reflected from a rarer
medium, compression is reflected as rarefaction and
vice-versa.
φ
(xi) Waves on reflection from a θ
A1
fixed end undergo a phase
Incident wave
change of 180°.
where, A= A12 + A22 + 2 A1 A2 cos f
Reflected wave
A2 sin f
tan q =
A1 + A2 cos f
(xii) While a wave reflected from If I1 and I 2 are intensities of the interfering waves and f is
a free end is reflected the phase difference, then the resultant intensity is given
Incident wave
without a change in phase. by
I = I1 + I 2 + 2 I1I 2 cos f
Reflected wave Now, Imax = ( I1 + I 2 ) 2

(xiii) In case of pressure wave, there is no phase change for f = 2 pn


when reflected from a denser medium or fixed end. Imin = ( I1 - I 2 ) 2
Note The concept of rarer or denser medium for a wave is through for f = (2 n + 1) p
speed (and not density of medium). For example, water is rarer for sound
Moreover, Intensity (I ) µ Amplitude ( A) 2
and denser for light than air, as for soundvw > v a , while for lightvw < v a .
Telegram @unacademyplusdiscounts

670 JEE Main Physics

Sample Problem 10 Two coherent sound sources are at


distances x1 = 0.2 m and x 2 = 0.48 m from a point. The
Important Points
intensity of the resultant wave at that point if the frequency 1. In this equation, it is seen that a particle at any particular point x
of each wave is f = 400 Hz and velocity of wave in the executes simple harmonic motion and all particles vibrate with the
medium is v = 448 ms -1 is (The intensity of each wave is same frequency.
I 0 = 60 Wm -2 .) 2. The amplitude is not the same for different particles but varies with
(a) 120 Wm–2 (b) 125 Wm–2 the location x of the particle.
(c) 130 Wm–2 (d) 135 Wm–2 3. The points having maximum amplitudes are those for which 2a sinkx
has a maximum value of 2A, these are at the positions,
Interpret (a) Path difference, Dx = x2 - x1 = 0.48 – 0.28 m
kx = p /2 , 3p /2 , 5 p /2 , ¼
Phase difference,
or x = l /4 , 3l /4 , 5 l /4 , ¼
2p æ 2 pf ö
f= Dx = ç ÷ Dx These points are called antinodes.
l è v ø
2 p ( 400) (0.28) p 4. The amplitude has a minimum value zero at positions where,
= = kx = p , 2 p , 3 p , ¼
448 2
I = I1 + I2 + 2 I1I2 cos f or x = l / 2 , l , 3l / 2 , 2 l , ¼
These points are called nodes.
or I = I0 + I0 + 2 I0 cos ( p /2)
= 2 I0 = 2 (60) = 120 Wm–2 5. Energy is not transported along the string to the right or to the left,
because energy cannot flow past the nodes points in the string which
are permanently at rest.

16.7 Standing or Stationary Waves 6. Standing wave is an example of interference. Node means
destructive interference and antinode means constructive
A standing wave is formed when two identical waves interference.
travelling in the opposite directions along the same line, 7. Due to persistance of vision these waves appear in the form of loops.
interfere. All the particles in a loop are in the same phase. But the particles in
adjacent loops differ in phase by p.
On the path of the stationary wave, there are points where
the amplitude is zero, these points are known as nodes. 8. Stationary waves may be transverse or longitudinal.
On the other hand, there are points where the amplitude 9. As in stationary waves nodes are permanently at rest, so energy
is maximum, these points are known as antinodes. cannot be transmitted across them.
● The distance between two consecutive nodes or two 10. Two identical waves moving in opposite directions along the string
consecutive antinodes is l /2. will still produce standing waves even if their amplitudes are unequal
(as shown in figure).
● The distance between a node and the next antinode is l /4.
Consider two waves of the same frequency, speed and
amplitude, which are travelling in opposite direction
Antinode Node
along a string. Two such waves may be represented by the
equations
y1 = A sin (kx - wt )
and y2 = A sin (kx + wt ) The standing wave ratio (SWR) is defined as
A
Hence, the resultant may be written as = max
Amin
A1 + Ay
=
A1 - Ay
y = y1 + y2 = A sin (kx - wt ) + A sin (kx + wt )
y = 2 A sin kx cos wt For 100% reflection SWR = ¥ and for no reflection SWR = 1

This is the equation of a standing wave.


Telegram @unacademyplusdiscounts

Waves 671

i. e., f2 = 2 f1
16.8 Fundamental Mode and It is called the first overtone or second harmonic. It is of
Harmonics twice the frequency of fundamental and is called an
octave higher than the fundamental frequency.
Stationary Waves in Strings (iii) The same string under the same conditions may also
A string of length L is stretched between two points. When vibrate in three segments.
the string is set into vibrations, a transverse progressive N A
N
A
N
A N
wave begins to travel along the string. It is reflected at the
other fixed end. The incident and the reflected waves L
interfere to produce a stationary transverse wave in which (c)
the ends are always nodes. Various modes of vibrations of
l3 2
a stretched string are shown below \ L=3 \ l3 = L
2 3
(i) In the simplest form, when the N A N If f3 is the frequency in this mode of vibration, then
string is plucked in the middle,
2
it vibrates in one loop in which L v = l3 f3 \ v = Lf3
3
the ends are the nodes and the (a)
3v
centre is the antinode. This or f3 = …(iii)
mode of vibration is known as the fundamental node 2L
and the frequency of vibration is known as the The frequency f3 is known as the third harmonic or
fundamental frequency of first harmonic. Since, the second overtone. Thus, a stretched string in addition to
distance between consecutive nodes is l/2. the fundamental mode, also vibrates with frequencies
l1 which are integral multiples of the fundamental
L= \ l1 = 2 L
2 frequencies. These frequencies are known as
The velocity of transverse waves is given as, harmonics.
v = l1 f1 (iv) In general, when the string vibrates in p loops,
\ Frequency of vibration wavelength of the pth mode of vibration is given by
v 2L
f1 = lp =
2L p
Þ v = 2 Lf1 …(i) and frequency is given by
T v
As we know, v= fp = p = pf1
m 2L
1 T It is called the (p – 1)th overtone or the pth harmonic.
f1 =
2L m
Note
This (first) normal made of vibration is called
1. Harmonics on a string When you need to obtain information
fundamental mode. The frequency of vibration ( f1 ) of on a stretched string of given length l, we draw harmonics. If you
string is minimum and is called fundamental are asked about, say, the Vth harmonic, you need to draw five
loops between the
frequency fixedsound
. The support points.
or note That
sowould mean thatis called
produced
fundamental note or first hormonic. five loops, each of length l / 2, occupy the length l of the string.
Thus, 5 ( l / 2) = l and l = 2l / 5. You can then use n = v / l to
(ii) The same string under the A
N
A find the frequency of the harmonic. Keep in mind that the
N N
same conditions may also wavelength of a harmonic is set only by the length l of the string,
vibrate in two loops, such that L but the frequency depends also on the wave speed v, which is set
the centre is also the node. (b) T
by the tension and the linear density of the string via, v = .
l2 m
\ L =2 \ l2 = L
2 2. If a string is vibrating in nth mode of vibration, then
If f2 is the frequency of vibrations, then the velocity of (a) the number of harmonics = n
transverse waves is given as, (b) fn = nf , where f is frequency of first or fundamental mode of
vibration.
v = l 2 f2
(c) the number of loops = n
\ v = Lf2
(d) the number of antinodes = n
or f2 = v/ L …(ii)
(e) the number of nodes = n + 1
1 T
f2 = (f) the number of overtones = n - 1
L m
But at n = 1, overtone is fundamental.
Telegram @unacademyplusdiscounts

672 JEE Main Physics

(c) The particle velocity is equal to


Laws of Vibration of Stretched String æ ¶y ö = 4 sin æ px ö (96 p) ( - sin96 pt )
ç ÷ ç ÷
(i) Law of length For a given wire under a given tension, è ¶t ø è15 ø
the frequency of wire varies inversely as its vibrating px
= -384 sin æç ö÷ sin (96 pt )
length i.e., è15 ø
1
v µ or v1l1 = v2l2 at, x = 7.5 and t = 0.25, we get
l
æ ¶y ö = -384 p sin æ px ö sin (96 pt )
where, T and m are constants. ç ÷ ç ÷
è ¶t ø è15 ø
(ii) Law of tension For a uniform wire of given length and p
= -384 p sin æç ö÷ sin (24 p) = 0
material, the frequency of the wire varies directly as è2 ø
the square root of tension
(d) The equations of the component waves are
vµ T px
y1 = 2 sin æç + 96 pt ö÷
v1 æ T1 ö è15 ø
or = ç ÷
v2 è T2 ø px
and y 2 = 2 sin æç - 96 pt ö÷
è15 ø
where l and m are constants.
(iii) Law of mass When l and T are constants, the frequency
of vibration of the wire varies inversely as the square Stationary Waves in Air Column
root of mass per unit length of the wire i.e., Open pipe
1
vµ If both ends of a pipe are open and a system of air is
m
directed against an edge, standing longitudinal waves can
where l and T are constant.
be set up in the tube. The open end has a displacement
1 1
So, vµ µ antinode
l m First Second Third
Hence, a graph between l and m is a straight line. harmonic harmonic harmonic
A A λ3 A
λ —
—2 4 N
Sample Problem 11 The vibrations of a string of length 4
60 cm fixed at both ends are represented by the equation λ
N —3 A
æ px ö 2
y = 4 sin ç ÷ cos (96 pt), where x and y are in cm and t in sec. λ λ N
è15 ø L N —1 —2 A
2 2
(a) What is the maximum displacement of a point at λ
—3 A
x = 5 cm? 2
λ N N
(b) Where are the nodes located along the string? —2 λ
4 —3
(c) What is the velocity of the particle at x = 7.5 cm and at A A 4 A
t = 0.25 s ? (a) (b) (c)
(d) Write down the equations of the component waves
whose superposition gives the above wave. (i) For fundamental mode or first harmonic,
l1
Interpret Given, y = 4 sin ( px /15) cos (96 pt ). It can be broken L= \ l1 = 2 L
2
up into velocity, v = l1 f1 \ v = 2 Lf1
é px px ù
y = 2 ê sin æç + 96pt ö÷ + sin æç - 96 pt ö÷ ú v
ë è15 ø è15 øû Þ f1 = ...(i)
2L
Thus, the waves are of the same amplitude and frequency but (ii) For the second harmonic or first overtone,
travelling in opposite directions which thus, superimpose to give a
L = l2
standing wave,
velocity, v = l 2 f2 \ v = Lf2
(a) At x = 5 cm the standing wave equation gives v
Þ f2 = …(ii)
y = 4 sin ( 5p /15) cos (96 pt ) = 4 ´ 3 / 2 cos (96 pt ) 2L
\ Maximum displacement = 2 3 cm (iii) For the third harmonic or second overtone,
(b) The nodes are points permanently at rest. Thus, they are l3 2
those points for which L = 3´ \ l3 = L
2 3
sin ( px /15) = 0 2
velocity, v = l3 f3 \ v = Lf3
i. e., px /15 = np, 3
n = 0, 1, 2, 3, 4, … 3v
Þ f3 = …(iii)
x = 15n, i. e., at x = 0, 15, 30, 45 and 60 cm. 2L
Telegram @unacademyplusdiscounts

Waves 673

From Eqs. (i), (ii) and (iii), we get 4L


\ l2 =
f1 : f2 : f3 : ¼ = 1 : 2 : 3 : ¼ 3
i.e., for a cylindrical tube, open at both ends, the velocity, v = l 2 f2
harmonics excitable in the tube are all integral v
\ f2 =
multiples of its fundamental. l2
2L 3v
In the general case, l = , where n = 1, 2, ¼ Þ f2 =
n 4L
v nv
Frequency, f = = , where n = 1, 2, ¼ (iii) It is the fifth harmonic or second overtone,
l 2L
l
L = 5´ 3
In open organ pipe, all (even and odd) harmonics are 4
present. Ratio of harmonics is f1 : f 2 : f3 ¼ = 1 : 2 : 3 ¼ and 4
\ l3 = L ...(ii)
ratio of overtones = 2 : 3 : 4 : 5 5
l 3l 5l Velocity, v = l3 f3
Position of nodes from one end x = , , ,¼
4 4 4 4
\ v = Lf3
l 3l 5
Position of antinodes from one end x = 0, , l, ,¼ 5v
2 2 Þ f3 = ...(iii)
4L
Close pipe From Eqs. (i), (ii) and (iii), we get
f1 : f2 : f3 ¼ = 1 : 3 : 5 ¼
If one end of a pipe is closed, the reflected wave is 180° out
4l
of phase with the incoming wave. The displacement of the In the general case, l = , where n = 0, 1, 2, ¼
(2n - 1)
small volume elements at the closed end must always be
zero. Hence, the closed end must be a displacement node. (2n - 1)v
Frequency = , where n = 0, 1, 2, ¼
First 4L
Trird Fifth
harmonic harmonic harmonic l 3l
A A A
Position of nodes from closed end x = 0, , l, ,¼
λ 2 2
λ —3
—2 4 N Position of antinodes from closed end
4
N l 3l 5l
λ x= , , ,¼
—3 A 4 4 4
λ 2
L —1 A
4 λ N
—2
Sample Problem 12 A pipe, 30 cm long is open at both
2
λ
ends, harmonic mode of the pipe which resonates at 1.1 kHz
—3 A source is [Given, speed of sound in air is 330 ms -1] [NCERT]
2
N N (a) first (b) second
N (c) third (d) four
(a) (b) (c)
Interpret (b) The first harmonic frequency is
(i) It represents the fundamental mode or first harmonic. v n
f1 = = (open pipe)
l li 2 L
L= 1
4
whereL is the length of the pipe. The frequency of itsnth harmonic is
\ l1 = 4 L nv
fn = for n = 1, 2, 3, ¼ (open pipe)
If f1 is the fundamental frequency, then the velocity of 2L
waves is given as, Given, L = 30 cm, v = 330 ms–1
v = l1 f1
n (330 ms–1)
\ v = 4 Lf1 fn = = 550 ms–1
0.6 m
v
\ f1 = …(i)
4L The frequencies of 2nd harmonic, 3rd hormonic, 4th hormonic, …
(ii) It is the third harmonic or first overtone,
are 2 ´ 550 = 1100 Hz, 3 ´ 550 = 1650 Hz, 4 ´ 550 = 2200 Hz.
3 l2 Clearly, a source of frequency 1.1 kHz will resonate at f2, i. e. , the
L=
4 second harmonic.
Telegram @unacademyplusdiscounts

674 JEE Main Physics

End Correction
It was found that the antinode is not formed exactly at the
open end of the organ pipe but actually due to finite
momentum of the particles the reflection takes place a
little above the open end; that is why the antinode is A A
formed a little above the open end. For this, a correction is
applied being known as end correction. This is denoted by l1 = λ/4 N
e. If length of organ pipe is l and end correction is e, then l2 = 3 λ/4 N
length of air-column in closed pipe will be (l + e) and in
open pipe, ( l + 2 e). Thus, for a closed organ pipe. A

e e
N

λ λ (a) (b)
— —
4 2
l l l1 + e = l/ 4
and l2 + e = 3l/ 4
l2 + e
=3
l1 + e

e Þ l2 = 3 l1 + 2 e
(a) (b) l - 3 l1
Hence, end correction, e= 2
2
v
f1 =
4 ( l + e)
and for an open organ pipe,
16.9 Musical Sound and Noise
v A musical sound consists of a quick succession of regular
f2 =
2 ( l + 2 e) and periodic compressions and rarefactions without any
sudden change in amplitude.
Note The value of end correction e is 0.6r for closed organ pipe and While a noise consists of slow succession of irregular and
1.2r for an open organ pipe, where r is the radius of the pipe.
periodic rarefactions and compressions accompanied by a
sudden change in amplitude.
Resonance Tube
It is a closed organ pipe in which length of air-column can Characteristics of Musical Sound
be increased or decreased. When a vibrating tuning fork is
Musical sounds differ from each other due to the
brought at its mouth as shown in figure, then forced
following three characteristics
vibrations are set up in its air-column. If we adjust the
length of air-column as such its any natural frequency
Pitch
equals to the frequency of tuning fork, then the amplitude
of forced vibrations of air-column increases very much. Pitch is the characteristic of sound that depends on
This is the state of resonance. frequency. It determines the shrillness or graveness of
sound.
When length of air-column is l = l/4, then the first
resonance occurs. As shown in Fig. (a), antinode is formed Smaller the frequency smaller is the pitch, greater the
at an open end and a node is formed at the water surface. frequency greater is the pitch. Frequency of ladies voice is
usually higher than that of gents’. Therefore, ladies voice
Now, when length of air-column is l2 = 3l/ 4, then second has higher pitch (sharper) than gents’.
resonance occurs. In this condition, two antinodes and
two nodes are formed as shown in Fig. (b). Loudness
End correction In resonance tube, antinode is not formed The loudness being a sensation, depends upon the
exactly at open end but it is formed a little above the open sensitivity of the listener’s ears. Therefore, loudness of a
end known as end correction (e). So, in first and second sound of given intensity may be different for different
state of air-column, the lengths are l1 + e and l2 + e. listeners. Hence, it depends on intensity of sound.
Telegram @unacademyplusdiscounts

Waves 675

æ I ö note and the highest (which is double of first) is called an


L = 10 log10 ç ÷ decibel (dB)
è I0 ø octave. Harmonium, piano, etc., are based on this scale.

Here, I 0 is the intensity of minimum audible sound which Tempered scale It contains 13 notes. The ratio of
is 10-12 Wm-2. frequencies of successive notes is 21/12. This scale is used
in harmonium. The frequencies of successive tones of an
Humming of mosquito has a high pitch (high frequency) equal tone temperature scale, form a geometric series.
but low intensity (low loudness) while the roar of a lion
has high intensity (loudness) but low pitch. Sample Problem 13 An open pipe is suddenly closed at
one end with the result that the frequency of third harmonic of
Quality the closed pipe is found to be higher by 100 Hz, than the
Quality is that characteristic of sound by which we can fundamental frequency of the open pipe. The fundamental
differentiate between the sounds coming from different frequency of the open pipe is
sources. Quality of sound depends on the number of (a) 100 Hz (b) 150 Hz
overtones and their relative intensities. (c) 200 Hz (d) 250 Hz

If same note is played on different instruments say sitar Interpret (c) For fundamental mode in open pipe,
and veena, at same loudness and same frequency, they L = l /2
produce different sensation on our ears due to their Þ l = 2L
different quality. v v
and fF = = …(i)
l 2L
Musical Interval For third harmonic in closed pipe
3l
The ratio between the frequencies of two notes is called L=
4
the musical interval.
4L
Þ l=
The combined effect of two tones is musical, if the interval 3
can be expressed as a ratio of two small numbers v v 3v
fH 2 = = = …(ii)
æ 2 3 ö l 4 L 4L
ç as , , ¼÷ and the combined effect of two tones is a
è 1 2 ø 3
noise, if the interval is given by the ratio of two large From Eqs. (i) and (ii), we have
æ 21 22 ö fH 3
numbers ç as , , ¼÷ . =
è 20 20 ø fF 2
3
Following are the names of some musical intervals Þ fH = fF ´ …(iii)
2
n
(a) Unison 2 = 1 but fH - fF = 100
n1 3
n fF - fF = 100
(b) Octave 2 = 2 2
n1 Þ fF = 200 Hz
n 9
(c) Major tone 2 =
n1 8 Sample Problem 14 A window whose area is 2 m2 opens
(d) Minor tone 2 =
n 10 on a street where the street noise results in an intensity level at
n1 9 the window of 60 dB. Now, if a sound absorber is fitted at the
n2 16 window, how much energy from the street will it collect in a
(e) Semi tone =
n1 15 day?
n 3 (a) 0.73 J (b) 0.173 J
(f) Fifth tone 2 =
n1 2 (c) 2.73 J (d) 1.73 J
I
Musical Scale Interpret (b) By definition sound level = 10 log = 60
I0
The arrangement of notes having a definite ratio with I
or = 10 6
respect to fundamental frequencies is called a musical scale. I0

Musical scales are of two types I = 10 -12 ´ 10 6 = 1 m Wm–2 [I0 = 10 -12 Wm–2]
Power entering the room = 1 ´ 10 -6 ´ 2 = 2 mW
Diatonic scale It is known as ‘Sargam’ in Indian system. It
contains eight notes with definite ratios in their Energy collected in a day = 2 ´ 10 -6 ´ 86400 = 0.173 J
frequencies. The note of lowest frequency is called key
Telegram @unacademyplusdiscounts

676 JEE Main Physics

For the first overtone of an open pipe, the length of the pipe l equals
16.10 Beats the wavelength of the vibration.
v
Beats is an interesting phenomenon arising from Hence, 345 =
l
interference of waves. When two sound waves of nearly
34000
same frequency are produced simultaneously, then the or l= = 101.5 cm
intensity of resultant sound wave increases and decreases 335
with time. This change in the intensity of sound is called as Other possible length l ¢ is given by
the phenomenon of ‘beats.’ v
335 =

The time interval between two successive beats is called
34000
beat period and the number of beats per second is called l¢ = = 101.5 cm
335
the beat frequency.
Hence, possible lengths of the open pipe are 98.5 and 101.5 cm.
If f1 and f 2 are the frequencies ( f1 > f 2 ) of the two waves,
then the beat frequency Sample Problem 16 Two tuning forks A and B sounded
b = f1 - f 2 together give 8 beat s-1. With an air resonance tube closed at
one end, the two forks give resonance when the two air
Important Points columns are 32 cm and 33 cm respectively. The frequencies of
forks are
1. At frequency difference greater than about 6 or 7 Hz, we no longer (a) 260 Hz, 250 Hz
hear individual beats. For example, if you listen to a whistle that (b) 264 Hz, 256 Hz
produces sounds at 2000 Hz and 2100 Hz, you will hear not only
(c) 274 Hz, 256 Hz
these tones but also a much lower 100 Hz tone.
(d) 2709 Hz, 250 Hz
2. If the frequency of a tuning fork is f and it produces Df beats per
second with a standard fork of frequency f 0 , then Interpret (b) Let the frequency of the first fork be f1 and that of
second be f2. Then, we have
f = f 0 ± Df
v v
If on filing the arms of an unknown fork, the beat frequency f1 = and f2 =
4 ´ 32 4 ´ 33
decreases, then
We also see that f1 > f2
f = f 0 - Df
\ f1 - f2 = 8 …(i)
This is because filing an arm of a tuning fork increases its frequency.
f1 33
Similarly, if on loading/waxing of the unknown fork, the beat and = …(ii)
f2 32
frequency decreases, then the frequency of the unknown fork is
f = f 0 + Df . This is because loading/waxing decreases the frequency Solving Eqs. (i) and (ii), we get
of tuning fork. f1 = 264 Hz
Similarly, f = f 0 + Df , if on filing beat frequency decreases and and f2 = 256 Hz
f = f 0 - Df if on loading/waxing beat frequency increases.
Sample Problem 17 Two sitar strings A and B playing the
Sample Problem 15 The first overtone of an open pipe note ‘Dha’ are slightly out of tune and produce beats of
and the fundamental note of a pipe closed at one end, gives frequency 5 Hz. The tension of the string B is slightly increased
-1
5 beats s , when sounded together. If the length of the pipe, and the beat frequency is found to decrease to 3 Hz. The
closed at one end is 25 cm, what are the possible lengths of the original frequency of B, if the frequency of A is 427 Hz is
open pipe? (Neglect end corrections and take the velocity of [NCERT]
sound in air to be 340 ms-1). (a) 427 Hz (b) 422 Hz
(a) 90.5 and 120.5 cm (b) 98.5 and 101.5 cm (c) 5 Hz (d) 10 Hz
(c) 95.5 and 102.5 cm (d) 95.5 and 200 cm Interpret (b) Increase in the tension of a string increases its
frequency. If the original frequency of B ( nB) were greater than that
Interpret (b) Let the fundamental frequency of the closed end
of A ( n A ), further increase in nB should have resulted in an increase
pipe of length 25 cm be f0 . Then
in the beat frequency. But, the beat frequency is found to decrease.
v 340 ´ 100
f0 = = = 340 Hz This shows that nB < n A .
4l 4 ´ 25
Since, n A - nB = 5 Hz and n A = 427 Hz
Possible frequencies of first overtone of the required open pipe are We get, nB = 427 - 5 = 422 Hz
340 ± 5, i.e., 345 or 335 Hz.
Telegram @unacademyplusdiscounts

Waves 677

Hot Spot Doppler’s Effect In SOUND


When there is a relative motion between the source and the observer the apparent frequency changes. This change in
apparent frequency because of relative motion is called Doppler’s effect. Doppler’s effect is a wave phenomena, it holds
not only for sound waves but also for electromagnetic waves.
Change in frequency can be analyzed under three different situations.
T = T 0 æç1 + S ö÷
v
...(i)
1. Observer is stationary, but the source is moving. è v ø
2. Observer is moving but the source is stationary. 1
3. Both the observer and source are moving. The above equation is terms of actual frequency f = and apparent
T
In our focus, we take the first case in detail frequency (fo ) that would be measured, if the source and observer were
stationary and the frequency f observed, when the source is moving as
1. Observer is stationary but source is moving Taking the -1
f = fo æç1 + S ö÷
v
direction from the observer to the source as the positive direction of
è v ø
velocity. Consider a source S moving with velocity vS and an observer
who is stationary in a frame in which the medium is also at rest. If vS is small compared with the wave speed v, taking Binomial
vS v
expansion to terms in first order is S and neglecting higher powers
O S1 S2 v
the above equation becomes
f = f 0 æç1 - S ö÷
v
è v ø
For a source approaching the observer, then
L + vST0
f = fo æç1 + S ö÷
v
è v ø
vST0

Let the speed of a wave of angular frequency w and period T0 both Note The observer thus measures a lower frequency, when the source
recedes from him, then he does when it is at rest. He measures a higher
measured by an observer at rest w.r.t. the medium be v. At time t = 0,
frequency when the source approaches him.
the source is at point S1, located at a distance L from the observer and
emits a crest. This reaches the observer at time
Sample Problem 19 A rocket is moving at a speed of
L
t1 = 200 ms -1 towards a stationary target. While moving it emits a
v
wave of frequency 1000 Hz. Some of the sound reaching the
At time, t = T0 , the source has moved a distance vST0 and is at point S2 , target gets reflected back to the rocket as an echo. The
located at distance ( L + vST0 ) from the observer. At S2 , the source frequency of sound as detected by the target is
emits a second crest. This reaches the observer at (a) 2540 Hz (b) 1500 Hz
(L + v ST 0 )
= +2 0 t T (c) 4240 Hz (d) 6200 Hz
v
Interpret (a) The observer is at rest and the source is moving
At time, t = nT0 , the source emits its ( n + 1)th crest and this reaches
with a speed of 200 ms -1. Since, this is comparable with the
the observer at time
velocity of sound 330 ms -1, using equation
(L + nv ST 0 )
tn + 1 = nT 0 + -1
æ v ö
v f = f0 ç1 + S ÷
è vø
Hence, in time interval,
énT + (L + nv ST 0 ) - L ù Since, the source is approaching a stationary target v 0 = 0 and v S
êë 0 v v úû must be replaced by -v S.
We have
The observer’s detector counts n crests and the observer records the -1
æ v ö
period of the wave as T given f = f0 ç1 - S ÷
è vø
(L + nv ST 0 ) L ù
T = éênT 0 + - ú n é 200 ù
ë v vû f = 1000 ê1 - Hz
v ST 0 ë 330 úû
T = T0 + f = 2540 Hz
v
Telegram @unacademyplusdiscounts

678 JEE Main Physics

Sample Problem 20 A siren emitting a sound of Interpret (b) From Doppler’s effect frequency fapp
frequency 1000 Hz moves away from you towards a cliff with a 800 m
v S
speed of 10 ms-1. What is the frequency of the sound you hear
coming directly from the siren? vS cos θ
(a) 33/34 ×1000 Hz 1000 600 m
(b) 34/33 × 1000 Hz
(c) 35/34 × 1000 Hz
P
(d) 34/35 × 1000 Hz
v
fapp = f
Interpret (a) Sound heard directly, v - v S cos q
æ v ö 340
f1 = f0 ç ÷ = ´ 800 = 839.5 Hz
èv + v s ø 340 - 16
v s = 10 ms–1
Sample Problem 23 A source S of acoustic wave of the
æ 330 ö
\ f1 = ç ÷ ´ 1000 frequency, n 0 = 1700 Hz and a receiver R are located at the
è 330 + 10 ø
same point. At the instant, t = 0, the source starts from rest to
33 move away from the receiver with a constant acceleration a.
= ´ 1000 Hz
34 The velocity of sound in air is 340 m/s. If a = 10 m / s2, the
apparent frequency that will be recorded by the stationary
Sample Problem 21 A bullet passes past a person at a receiver at t = 10 s will be
speed 220 ms -1. The fractional change in the frequency of the
(a) 1700 Hz (b) 1.35 Hz
whistling sound heard by the person as the bullet crosses the
(c) 2.89 Hz (d) 1300 Hz
person is (Speed of sound = 330 ms-1)
(a) 0.67 Interpret (b) Source frequency, n 0 = 1700 Hz. Source
(b) 0.8 (coinciding with observer at t = 0) moves away with uniform
(c) 1.2 acceleration a. Consider the wave which is received by the observer
at instant t = t. It will have left the source at an earlier instant of
(d) 3.0
time, say t ( < t), when the distance of source was r (say), if u be
Interpret (b) Limiting cases when it is just at the verge of æ 1ö
velocity of source at instant t, then r = ç ÷ at 2 and u = at . The
crossing and when it has just crossed are taken. From Doppler’s è2ø
effect, we have relation between t and t, is
æ v ö r at 2
f1 = ç ÷ f = 0.6 f t =t + =t +
è v + vS ø v 2v
v This is a quadratic equation in t, giving the solution
f2 = f =3f
v - vS
- 2v + 4 v 2 + 8 vat
f +f at =
fnet = 1 2 2
2
æ 2 at ö
3.6 f u = at = v ç 1 + - 1÷
= = 1.8 f è v ø
2
Df = 0.8 f æ 2 ´ 10 ´ 10 ö
u = 340 ç 1 + - 1÷
Df è 340 ø
or = 0.8
f
æ 27 ö
= 340 ç - 1÷
Sample Problem 22 A person P is 600 m away from the è 17 ø
station, when train is approaching station with 72 km/hr it flows Then apparent frequency is given by
a whistle of frequency 800 Hz when 800 m away from the æ v ö
station. Frequency heard by the person is na = ç ÷n
èv + uø 0
[Given, speed of sound = 340 ms–1]
Putting the values v = 340 m/s, t = 10 s, a = 10 m/s2, we have
(a) 800 Hz
æ 340 ö
(b) 839.5 Hz na = ç ÷ 1700
è 340 + u ø
(c) 829.5 Hz
(d) 834.5 Hz 17
n a = 1700 = 1.35 Hz
27
Telegram @unacademyplusdiscounts

Waves 679

After Cases of Doppler’s Effects (b) If both are moving in the same direction and source is ahead of
2. Observer is moving and source at rest the observer, then apparent frequency is
æ v + vo ö
(a) When observer is coming towards the source, then apparent f0 = f ç ÷
frequency is è v + vs ø
v + v0 ö
f0 = f æç ÷ (c) If both are moving towards each other, then apparent frequency is
è v ø æ v + vo ö
f0 = f ç ÷
(b) When observer is going away from the source, then apparent è v - vs ø
frequency is
v - v0 ö (d) If both are moving away from each other, then
f0 = f æç ÷ apparent frequency is
è v ø
æ v - vo ö
f0 = f ç ÷
3. Both observer and source are moving è v + vs ø
(a) If observer and source are moving in same direction and observer
is ahead of the source, then apparent frequency is Note Frequency is abbreviated as either n or f.
æ v - vo ö
f0 = f ç ÷
è v - vs ø

Transverse Doppler’s Effect are much lesser than that of sound, the change in frequency
becomes independent of the fact whether the source is moving
(i) The Doppler’s effect in sound does not take place in
the transverse direction. or the observer. This can be shown as under.
Suppose a source is moving towards a stationary observer, with
(ii) From the figure, the position of a source is S and of
speed u and the speed of sound is v, then
observer is O. The component of velocity of source
æ ö
towards the observer is v cosq. For this situation, the ç 1 ÷ -1
f ¢ = æç
v ö æ uö
approach frequency is ÷ f = ç u ÷ f = ç1 - ÷ f
èv - u ø ç1 - ÷ è vø
θ è vø
v P Using the Binomial expansion, we have
v

S v
θ θ S -1
æ1 - u ö u u2
ç ÷ » 1+ +
è vø v v2
v
co

θ T
s

æ u u2 ö
f ¢ = çç1 + + 2 ÷÷ f
O è v v ø
v
f¢ = ´f
f ¢ » æç1 + ö÷ f , ifu << v
u
v - v s cosq è vø
f ¢ which will now be a function of q so, it will no more On the other hand, if an observer moves towards a stationary
be constant. source with same speed u, then
Similarly, if the source is moving away from the v + uö
f ¢ = æç æ uö
÷ f = ç1 + ÷ f
observer as shown above, with velocity component è v ø è vø
v s cos q, then
v Which is same as above.
f¢ = ´f
v + v s cosq
(iii) If q = 90°, the v s cos q = 0 and there is no shift in the Check Point 2
frequency. Thus, at point P, Doppler’s effect does not
occur. 1. When a source moves with a speed greater than that of sound, will
Doppler formula hold? What will happen?
Note 2. If oil of density higher than density of water is used in a
1. If wind blows at a speed vw from the source to the observer, take resonance tube, how will the frequency change?
v ® v + vw (both in numerator and denominator) and if in 3. What is the nature of thermal changes in air, when a sound
opposite direction (i.e., from observer to source), take v ® v - vw . wave propagates through it?
Thus, the modified formula is 4. Why the pitch of an organ pipe on a hot summer day higher?
æ v ± vw ± v o ö 5. The beats are not heard, if the difference in frequencies of two
f¢ = ç ÷f
è v ± vw ± v s ø sounding notes is more than 10. Why?
2. Change in frequency depends on the fact that whether the 6. Where will a man hear a louder sound at node or at the
source is moved towards the observer or the observer is moved antinode, in a case of stationary wave?
towards the source. But when the speed of source and observer
Telegram @unacademyplusdiscounts

WORKED OUT
Examples
Example 1 A wave is represented by the equation Example 4 The frequency of a vibrating wire is f. If tension is
y = 0.1sin (100pt + kx). doubled, density is halved and diameter is doubled, then the
If wave velocity is 100 ms -1, its wave number is equal to new frequency will be
(a) f (b) f 2
(a) 1m -1 (b) 2m -1
(c) pm -1 (d) 2 pm -1 (c) 2f (d) 4f

Solution Given, y = 0.1 sin (100 pt + kx)


Solution If l be the length of the wire then frequency
1 T
Compare it with standard equation of wave form f= .
lD pr
é 2p 2p ù
y = A sin ê t+ x When tension T is doubled,
ëT l úû
density r is halved,
2p 2p 1
= 100 p Þ T = = s diameter D is doubled
T 100 p 50
v = 100 ms-1 f would remain the same.
1
Wavelength, l = v ´ T = 100 ´ =2m Example 5 A standing wave having 3 nodes and 2 antinodes
50
2p 2p
is formed between two atoms having a distance of 1.21Å
Wave number, k = = = pm-1 between them. The wavelength of the standing wave is
l 2
(a) 1.21Å (b) 2.42Å
(c) 3.63Å (d) 6.05Å
Example 2 In a longitudinal wave, pressure variation and
displacement variation are Solution As is clear from figure
(a) in phase (b) 90° out of phase l l
+ = 1.21
(c) 45° out of phase (d) 180° out of phase 2 2
Solution In a longitudinal wave, pressure is maximum where l = 1.21Å
displacement is minimum. Therefore, pressure and displacement λ/2 λ/2
variations are 180° out of phase.
X X
N N
Example 3 In an experiment with sonometer, a tuning fork A A
of frequency 256 Hz resonates with a length of 25 cm and 1.21Å
another tuning fork resonates with a length of 16 cm. Tension of
the string remaining constant, the frequency of the second Example 6 A glass tube 1.5 m long and open at both ends is
turning fork is immersed vertically in a water tank completely. A tuning fork of
(a) 204.8 Hz (b) 163.84 Hz 660 Hz is vibrated and kept at the upper end of tube and the
(c) 400 Hz (d) 320 Hz tube is gradually raised out of water. The total number of
1 resonances heard before when the tube comes out of water,
Solution As, fµ
(taking v = 330 ms-1) is
l
f2 l1 (a) 6 (b) 12
\ =
f1 l2 (c) 8 (d) 4
f2 25
= Solution Here, n = 660Hz, v = 330 ms- 1
256 16
256 ´ 25 v 330
f2 = = 400 Hz Wavelength, l= = = 0.5 m
16 f 660
Telegram @unacademyplusdiscounts

Waves 681

Resonance lengths are Solution When the listener moves, wavelength ( l) remains
l 3l 5l 7l 9l 11l 13l v 340
, , , , , , and so on unaffected l = = =2m
4 4 4 4 4 4 4 n 170
13l 13 ´ 0.5l
= = 1.6 m Distance between adjacent position of minimum intensity (nodes)
4 4 l 2
= = = 1m .
Which is greater than 1.5 m, therefore, total number of resonances 2 2
heard is 6.
Example 11 If the splash is heard 4.23 s after a stone is
Example 7 A cylinder tube, open at both ends, has a dropped into a well 78.4 m deep, find the velocity of sound in
fundamental frequency f in air. The tube is dipped vertically in air.
water so that half of it is in water. The fundamental frequency of (a) 340 ms-1 (b) 320 ms-1
the air column is (c) 300 ms-1 (d) 280 ms-1
(a) 2f (b) 3f/4
(c) f/2 (d) f Solution Here, depth of the well, s = 78.4 m
v Total time after which splash is heard = 4.23 s,
Solution When the tube is open at both ends, f =
2l If t1 = time taken by stone to hit the water surface in the well,
When the tube is dipped vertically in water and half of it is in water, t 2 = time taken by splash of sound to reach the top of the well.
v Then, t1 + t 2 = 4.23s
f¢ = =f
4( l / 2) Now, for downwards journey of stone,
u = 0 , a = 9.8ms-2, s = 78.4 m
Example 8 An organ pipe filled with a gas at 27°C resonates
t = t1 = ?
at 40 Hz in its fundamental mode. If it is filled with the same gas
1 2
at 90°C, the resonance frequency will be As s = ut + at
2
(a) 420 Hz (b) 440 Hz
1
(c) 484 Hz (d) 512 Hz \ 78.4 = 0 + ´ 9.8 t12 = 4.9 t12
2
Solution Assuming that length of pipe remains unchanged, 78.4
or t12 = = 16 or t1 = 16 = 4 s
f ¢ v 90 273 + 90 4.9
= = = 1.21 = 1.1
f v 27 272 + 27 From t1 + t 2 = 4.23
f ¢ = 1.1 f = 1.1 ´ 400 = 440 Hz t 2 = 4.23 - t1 = 4.23 - 4 = 0.23 s
If v is the velocity of sound in air, then
Example 9 A tuning fork of frequency 480 Hz produces distance ( s) 78.4
v= = = 340.87ms-1
10 beats s-1 when sounded with a vibrating sonometer string. time (t 2) 0.23
What must have been the frequency of the string if a slight
increase in tension produces fewer beats s-1 than before? Example 12 The mass of 1 mol of air is 29 ´ 10 -3 kg, then
(a) 480 Hz (b) 490 Hz the speed of sound in air at standard temperature and pressure
(c) 460 Hz (d) 470 Hz is
(a) 332.5 ms-1 (b) 300 ms-1
Solution Frequency of tuning fork, f1 = 480 Hz
(c) 280 ms-1 (d) 250 ms-1
Number of beats s-1, n = 10
Frequency of string f2 = ( 480 ± 10) Hz
Solution Here, p = 1atmospheric pressure
= 1.01 ´ 10 5 N / m2
A slight increase in tension increases f2
mass of one mole 29.0 ´ 10 -3 (kg)
\ f2 = 480 - 10 = 470 Hz Density of air, r = =
volume of one mole 22.4 ´ 10 -3 (m3)
Example 10 A sound of frequency 170 Hz is placed near a r = 1.29 kgm-3
wall. A man walking from the source normally towards the wall For air, g = 1.41
finds that there is a periodic rise and fall of sound intensity. If According to corrected Newton’s formula,
speed of sound in air is 340 ms-1, the distance in metres gp
v=
separating the two adjacent positions of minimum intensity is r
(a) 1/2 (b) 1
. ´ 10 5
1.41 ´ 101
(c) 3/2 (d) 2 v= = 332.5 ms-1
1.29
Telegram @unacademyplusdiscounts

Start Practice for


JEE Main
Round I (Topically Divided Problems)

Wave Motion, Speed of Sound 7. v1 and v2 are the velocities of sound at the same
temperature in two monoatomic gases of densities r1
1. Velocity of sound wave in air is 330 m/s for a r1 1
particular sound in air ; a path difference of 40 cm and r2 respectively. If = , then the ratio of
r2 4
is equivalent to a phase difference of 1.6 p. The
velocities v1 and v2 will be
frequency of this wave is
(a) 1: 2 (b) 4 :1
(a) 165 Hz (b) 150 Hz
(c) 2 : 1 (d) 1: 4
(c) 660 Hz (d) 330 Hz
2. The equation of wave is represented by 8. A 1000 Hz sound wave in air strikes the surface of a
lake and penetrates into water. If speed of sound in
Y = 10-4 sin é100t -

m, then the velocity of wave water is 1500 ms–1, the frequency and wavelength of
êë 10 úû
waves in water are
will be (a) 1500 Hz, 1m (b) 1000 Hz, 1.5 m
(a) 100 ms–1 (b) 4 ms–1 (c) 1000 Hz, 1m (d) 1500 Hz, 1.5 m
(c) 1000 ms–1 (d) zero
9. Sound waves of wavelength l travelling in a medium
3. A steel rod 100 cm long is clamped at its middle. The with a speed of v m/s enter into another medium
fundamental frequency of longitudinal vibrations of where its speed is 2v m/s. Wavelength of sound waves
the rod are given to be 2.53 kHz. What is the speed of in the second medium is [NCERT Exemplar]
sound in steel? [NCERT] l
(a) 5 km/s (b) 6 km/s (a) l (b)
2
(c) 7 km/s (d) 4 km/s (c) 2l (d) 4l
4. From a point source, if amplitude of waves at a 10. The displacement x (in metre) of a particle
distance r is A, its amplitude at a distance 2r will be performing simple harmonic motion is related to time
p
t (in second) as x = 0.05 cos æç 4 pt + ö÷. The frequency
(a) A (b) 2 A
(c) A/2 (d) A/4 è 4ø
5. A simple harmonic progressive wave is represented of the motion will be
by the equation y = 8 sin 2p (01
. x - 2 t) (a) 0.5 Hz (b) 1.0 Hz
where x and y are in cm and t is in seconds. At any (c) 1.5 Hz (d) 2.0 Hz
instant, the phase difference between two particles 11. The temperature at which the speed of sound in air
separated by 2.0 cm in the x-direction is becomes double of its value at 0°C is
(a) 18° (b) 54° (a) 273 K (b) 546 K
(c) 36° (d) 72° (c) 1092 K (d) 0 K
6. In a sinusoidal wave, the time required for a 12. A wave of frequency 500 Hz has a velocity 360 ms–1.
particular point to move from maximum dis- The phase difference between two displacements at a
placement to zero displacement is 0.170 s. The certain point at time 10-3 s apart will be [NCERT]
frequency of the wave is (a) p rad (b) p/2 rad
(a) 1.47 Hz (b) 0.36 Hz (c) p/4 rad (d) 2 p rad
(c) 0.73 Hz (d) 2.94 Hz
Telegram @unacademyplusdiscounts

Waves 683

13. The phase difference between two points separated 22. A transverse wave is described by the equation
y = y0 sin 2p é ft - ù. The maximum particle velocity
by 0.8 m in a wave of frequency 120 Hz is 0.5 p. The x
wave velocity is êë l úû
(a) 144 ms–1 (b) 384 ms–1 is equal to four times the wave velocity, if
(c) 256 ms–1 (d) 720 ms–1 (a) l = p y 0 / 4 (b) l = 2 py 0
14. Two sitar strings A and B playing the note ‘Ga’ are (c) l = p / y 0 (d) l = p y 0 / 2
slightly out of tune and produce beats of frequency 23. The wavelength of infrasonics in air is of the order of
6 Hz. The tension in the string A is slightly reduced (a) 100 m (b) 101 m
and the beat frequency is found to reduce to 3 Hz. If (c) 10–1 m (d) 10–2 m
the original frequency of A is 324 Hz, what is the
frequency of B ? [NCERT] 24. Which of the following statements are true for wave
motion? [NCERT Exemplar]
(a) 218 Hz (b) 518 Hz
(c) 418 Hz (d) 318 Hz (a) Mechanical transverse waves can propagate through all
mediums.
15. A stone dropped from the top of a tower of height 300 (b) Longitudinal waves can propagate through solids only.
m high splashes into the water of a pond near the (c) Mechanical transverse waves can propagate through solids
base of the tower. When is the splash heard at the top only.
given that the speed of sound in air is 340 m/s? (d) Longitudinal waves can propagate through vacuum.
( g = 9.8 m/s2 ) [NCERT]
25. The speed of sound in a mixture of 1 mole of helium
(a) 8.7 s (b) 9.7 s
and 2 moles of oxygen at 27°C is
(c) 6.7 s (d) 10 s
(a) 800 ms–1 (b) 400.8 ms–1
16. Speed of sound in mercury at a certain temperature (c) 600 ms–1 (d) 1200 ms–1
is 1450 m/s. Given the density of mercury as
26. A wave is represented by the equation
13.6 ´ 103 kg/m 3, the bulk modulus for mercury is
10 3 10 3
y = 0.5 sin (10 t + x) m
(a) 2.86 ´ 10 N/m (b) 3.86 ´ 10 N / m It is a travelling wave propagating along + X
(c) 4.86 ´ 1010 N / m3 (d) 5.86 ´ 1010 N / m3 direction with velocity
17. If the temperature is raised by 1 K from 300 K, the (a) 40 ms–1 (b) 20 ms–1
percentage change in the speed of sound in the (c) 5 ms–1 (d) None of these
gaseous mixture is (R = 8.31 J/mol-K) 27. Under identical conditions of pressure and density,
(a) 0.167% (b) 0.334% the speed of sound is highest in a
(c) 1% (d) 2% (a) monoatomic gas (b) diatomic gas
18. The path difference between two waves (c) triatomic gas (d) polyatomic gas
æ 2 px ö æ 2px
y1 = a1 sin ç wt - ÷ and y2 = a2 cos ç wt - + f ö÷ 28. A sound absorber attenuates the sound level by
è l ø è l ø 20 dB. The intensity decreases by a factor of
is (a) 100 (b) 1000
l l æ pö (c) 10000 (d) 10
(a) (f) (b) çf + ÷
2p 2p è 2ø 29. The velocity of sound in air is 330 ms–1 and the
2p æ pö 2p velocity of light in air is 3 ´ 108 ms -1. What
(c) çf - ÷ (d) (f)
l è 2ø l frequency, in Hz does a BBC station which transmits
19. The amplitude of a wave is given by A =
c at 1500 m broadcast?
( a + b - c) (a) 2 × 105 Hz (b) 595 × 103 Hz
Resonance will occur when (c) 0.22 Hz (d) 5 × 10–6 Hz
c a
(a) b = - (b) b = - 30. Walls of auditorium should be
2 2
(a) good absorber (b) reflector
(c) b = 0, a = c (d) None of these
(c) amplifier (d) modifier
20. Which of the following equations represents a wave ?
31. The velocity of sound in air is 330 ms–1. The rms
(a) y = A sin wt (b) y = A cos kx
velocity of air molecules (g = 1.4) is approximately
(c) y = A sin ( at - bx + c ) (d) y = A ( wt - kx)
equal to
21. Ultrasonic waves are produced by (a) 400 ms–1 (b) 471.4 ms–1
(a) Piezoelectric effect (b) Pettiro’s effect (c) 231 ms–1 (d) 462 ms–1
(c) Doppler’s effect (d) Coulomb’s law
Telegram @unacademyplusdiscounts

684 JEE Main Physics

32. The ratio of intensities between two coherent sound tension, the ratio between the fundamental
sources is 4 : 1. The difference of loudness in decibels frequencies is
(dB) between maximum and minimum intensities, (a) 1 : 2 (b) 2 : 1
on their interference in space is (c) 1 : 4 (d) 4 : 1
(a) 20 log 2 (b) 10 log 2 40. n waves are produced on a string in one second.
(c) 20 log 3 (d) 10 log 3 When the radius of the string is doubled and the
33. A sine wave has an amplitude A and a wavelength l. tension is maintained the same, the number of waves
Let v be the wave velocity, and V be maximum produced in one second for the same harmonic will be
velocity of a particle in the medium. n n n
(a) (b) (c) 2n (d)
(a) V cannot be equal to v 2 3 2
(b) V = v , if A = l/2p 41. A sound wave is passing through air column in the
(c) V = v , if A = 2pl form of compression and rarefaction. In consecutive
(d) V = v , if l = A / p compressions and rarefactions, [NCERT Exemplar]

34. Two identical sounds A and B reach a point in the (a) density remains constant
same phase. The resultant sound is C. The loudness (b) Boyle’s law is obeyed
of C is ndB higher than the loudness of A. (c) bulk modulus of air oscillates
(a) 2 (b) 3 (d) there is no transfer of heat
(c) 4 (d) 6 42. A string in a musical instrument is 50 cm long and its
35. The velocity of sound hydrogen is 1224 Its ms–1. fundamental frequency is 800 Hz. If a frequency of
velocity in mixture of hydrogen and oxygen 1000 Hz is to be produced, then required length of
containing 4 parts by volume of hydrogen and 1 part string is
oxygen is (a) 62.5 cm (b) 50 cm
(a) 1224 ms–1 (b) 612 ms–1 (c) 40 cm (d) 37.5 cm
(c) 2448 ms–1 (d) 306 ms–1 43. The tension in a wire is decreased by 19%. The
percentage decrease in frequency will be
Stationary Waves in Strings and in Organ (a) 19% (b) 10%
(c) 0.19% (d) None of these
Pipes
44. An echo repeats two syllables. If the velocity of sound
36. A wave represented by the given equation is 330 ms–1, then the distance of the reflecting
y = a cos( kx - wt) is superposed with another wave to surface is
form a stationary wave such that the point x = 0 is a (a) 66.0 m (b) 33.0 m
node. The equation for the other wave is (c) 99.0 m (d) 16.5 m
(a) y = a sin ( kx + wt ) (b) y = - a cos ( kx + wt )
(c) y = - a cos ( kx - wt ) (d) y = - a sin ( kx - wt ) 45. A tuning fork of frequency 512 Hz is used to produce
vibrations in a sonometer wire of natural frequency
37. A string of mass 2.5 kg is under a tension of 200 N. 256 Hz. The wire will vibrate in
The length of the stretched string is 20.0 m. If the (a) one segment (b) two segments
transverse jerk is struck at one end of the string, the (c) four segments (d) three segments
disturbance will reach the other end in
[NCERT Exemplar] 46. The following equations represent progressive
transverse waves
(a) 1 s (b) 0.5 s
Z1 = A cos ( wt - kX ), Z2 = A cos ( wt + kX )
(c) 2 s (d) data is insufficient
Z 3 = A cos ( wt - kY ), Z4 = A cos (2 wt - 2kY )
38. A wave of wavelength 2 m is reflected from a surface A stationary wave will be formed by superposing
if a node is formed at 3 m from the surface, then at (a) Z1 and Z2 (b) Z1 and Z 4
what distance from the surface another node will be (c) Z2 and Z3 (d) Z3 and Z 4
formed?
(a) 3 m (b) 2 m 47. The equation of a stationary wave along a stretched
2px
(c) 3 m (d) 4 m string is given by y = 4 sin cos 40pt
2
39. Two wires made up of same material are of equal where x and y are in cms and t is in sec. The
lengths but their radii are in the ratio 1 : 2. On
separation between two adjacent nodes is
stretching each of these two strings by the same
(a) 3 cm (b) 1.5 cm (c) 6 cm (d) 4 cm
Telegram @unacademyplusdiscounts

Waves 685

48. The length of a sonometer wire AB is 110 cm. Where 55. A wave of frequency 100 Hz is sent along a string
should the two bridges be placed from A to divide the towards a fixed end. When this wave travels back,
wire in three segments whose fundamental after reflection, a node is formed at a distance of
frequencies are in the ratio of 1 : 2 : 3. 10 cm from the fixed end of the string. The speeds of
(a) 30 cm, 90 cm (b) 60 cm, 90 cm incident (and reflected) waves are
(c) 40 cm, 70 cm (d) None of these (a) 5 ms–1 (b) 10 ms–1 (c) 20 ms–1 (d) 40 ms–1
49. If n1, n2 and n3 are the fundamental frequencies of 56. An open pipe is suddenly closed at one end with the
three segments into which a string is divided, then result that the frequency of third harmonic of the
the original fundamental frequency n of the string is closed pipe is found to be higher at 100 Hz. The
given by fundamental frequency of the open pipe is
1 1 1 1 (a) 200 Hz (b) 480 Hz (c) 240 Hz (d) 300 Hz
(a) = + +
n n1 n2 n3 57. The vibrating of four air columns are represented in
1 1 1 1 the figure. The ratio of frequencies n p : nq : nr : ns is
(b) = + +
n n1 n2 n3
(c) n = n1 + n2 + n3
(d) n = n1 + n2 + n3 p q

50. An organ pipe open at one end is vibrating in first


overtone and is in resonance with another pipe open
at both ends and vibrating in third harmonic. The r s
ratio of length of two pipes is (a) 12 : 6 : 3 : 5 (b) 1 : 2 : 4 : 3
(a) 3 : 8 (b) 8 : 3 (c) 1 : 2 (d) 4 : 1 (c) 4 : 2 : 3 : 1 (d) 6 : 2 : 3 : 4
51. In Melde’s experiment, three loops are formed by 58. In a resonance column first and second resonance are
putting a weight of 8 g in a massless pan. The weight obtained at depths 22.7 cm and 70.2 cm. The third
required to form two loops is resonance will be obtained at a depth of
(a) 18 g (b) 8 g (c) 36 g (d) 24 g (a) 117.7 cm (b) 92.9 cm
52. A stretched string of length l fixed at both ends can (c) 115.5 cm (d) 113.5 cm
sustain stationary waves of wavelength l given by 59. A cylindrical tube open at both ends, has a
(a) l = 2ln (b) l = 2l / n fundamental frequency f0 in air. The tube is dipped
(c) l = l2 / 2n (d) l = n 2 / 2l vertically into water such that half of its length
53. In Melde’s experiment, the string vibrates in 4 loops inside water. The fundamental frequency of the air
when a 50 kg-wt weight is placed in the pan of weight column now is
15 kg-wt. To made the string vibrate in 6 loops, the (a) 3 f0 / 4 (b) f0 (c) f0 / 2 (d) 2 f0
weight that has to be removed from the pan is 60. A metre-long tube open at one end, with a movable
approximately piston at the other end, shows resonance with a fixed
(a) 7 kg-wt (b) 36 kg-wt frequency source (a tunning fork of frequency 340
(c) 21 kg-wt (d) 29 kg-wt Hz) when the tube length is 25.5 cm or 79.3 cm.
Estimate the speed of sound in air at the temperature
54. Equation of a plane progressive wave is given by of the experiment. The edge effect may be neglected.
y = 0.6 sin 2p æç t - ö÷. On reflection from a denser
x
[NCERT]
è 2ø
(a) 336 m/s (b) 331 m/s (c) 356 m/s (d) 366 m/s
medium its amplitude becomes 2/3 of the amplitude 61. If l1, l2 , l 3 are the wavelengths of the waves giving
of the incident wave. The equation of the reflected resonance with the fundamental, first and second
wave is [NCERT Exemplar] overtones respectively of a closed organ pipe, then
æ xö the ratio of l1, l2 , l 3 is
(a) y = 0.6 sin 2p çt + ÷
è 2ø 1 1
æ xö (a) 1 : 3 : 5 (b) 1 : 2 : 3 (c) 5 : 3 : 1 (d) 1 : :
(b) y = - 0.4 sin 2p çt + ÷ 3 5
è 2ø 62. In a resonance tube, the first resonance with a tuning
æ x ö fork occurs at 16 cm and second at 49 cm. If the
(c) y = 0.4 sin 2p çt + ÷
è 2ø velocity of sound is 330 m/s, the frequency of tunning
æ xö fork is
(d) y = - 0.4 sin 2p çt - ÷
è 2ø (a) 500 (b) 300 (c) 330 (d) 165
Telegram @unacademyplusdiscounts

686 JEE Main Physics

63. An open organ pipe has fundamental frequency Beats


100 Hz. What frequency will be produced if its one
end is closed? 71. In two similar wires of tensions 16 N and T, 3 beats
(a) 100, 200, 300 .... (b) 50, 150, 250.... are heard, then T =
(c) 50, 100, 200, 300... (d) 50, 100, 150, 200... (a) 49 N (b) 25 N
(c) 64 N (d) None of these
64. An open pipe is suddenly closed at one end with the
result that the frequency of third harmonic of the 72. When 2 tuning forks (fork 1 and fork 2) are sounded
closed pipe is found to be higher by 100 Hz. Then the simultaneously, 4 beats s -1 are heard. Now, some
fundamental frequency of open pipe is tape is attached on the prong of fork 2. When the
(a) 480 Hz (b) 300 Hz tuning forks are sounded again, 6 beats s -1 are heard
(c) 240 Hz (d) 200 Hz if the frequency of fork 1 is 200 Hz, then what was the
original frequency of fork 2 ?
65. A column of air of length 50 cm resonates with a (a) 196 Hz (b) 200 Hz
stretched string of length 40 cm. The length of the (c) 202 Hz (d) 204 Hz
same air column which will resonate with 60 cm of
the same string at the same tension is 73. A wire stretched between two rigid supports vibrates
(a) 100 cm (b) 75 cm in its fundamental mode with a frequency of 45 Hz.
(c) 50 cm (d) 25 cm The mass of the wire is 3.5 ´ 10-2 kg and its linear
mass density is 4.0 ´ 10-2 kg/m. What is (i) the speed
66. In a resonance tube, using a tuning fork of frequency of a transverse wave on the string and (ii) the tension
325 Hz, two successive resonance length are in the string? [NCERT]
observed as 25.4 cm and 77.4 cm respectively. The
(a) (i) 80 m/s (ii) 250 N (b) (i) 88 m/s (ii) 208 N
velocity of sound in air is
(c) (i) 90 m/s (ii) 249 N (d) (i) 78.75 m/s (ii) 248 N
(a) 338 ms–1 (b) 328 ms–1
(c) 330 ms–1 (d) 320 ms–1 74. Ten tuning forks are arranged in increasing order of
frequency in such a way that any two nearest tuning
67. An organ pipe P1 closed at one end vibrating in its forks produce 4 beats s–1. The highest frequency is
first harmonic and another pipe P2 open at both ends twice that of the lowest. Possible highest and lowest
vibrating in its third harmonic are in resonance with frequencies are
a given tuning fork. The ratio of the length of P1 to
(a) 80 and 40 (b) 100 and 50
that of P2 is
(c) 44 and 32 (d) 72 and 36
(a) 1/3 (b) 1/6
(c) 3/8 (d) 8/3 75. A source of frequency n given 5 beats s–1, when
sounded with a source of frequency 200 s–1. The
68. The tones that are separated by three octaves have a second harmonic (2n) gives 10 beats s–1, when
frequency ratio of sounded with a source of frequency 420 s–1. n is equal
(a) 3 (b) 4 to
(c) 8 (d) 16
(a) 200 s–1 (b) 205 s–1
69. Air is blown at the mouth of a tube of length 25 cm (c) 195 s–1 (d) 210 s–1
and diameter equal to 2 cm open at both ends. If 76. Two forks A and B when sounded together produce
velocity of sound in air is 330 ms–1, the sound emitted four beats s–1. The fork A is in unison with 30 cm
will have all the frequencies in the group length of a sonometer wire and B is in unison with
(a) 330, 990, 1690 Hz 25 cm length of the same wire at the same tension.
(b) 302, 664, 1320 Hz The frequencies of the forks are
(c) 660, 1320, 1980 Hz (a) 24 Hz, 28 Hz (b) 20 Hz, 24 Hz
(d) 660, 100, 3300 Hz (c) 16 Hz, 20 Hz (d) 26 Hz, 30 Hz
70. A pipe closed at one end and open at the other end, 77. Beats are produced by two travelling waves each of
resonates with sound waves of frequency 135 Hz and loudness I and nearly equal frequencies n1 and n2 .
also 165 Hz, but not with any wave of frequency The beat frequency will be ... and maximum loudness
intermediate between these two. Then the frequency hard will be
of the fundamental note is (a) ( n1 - n2 ), 2 I (b) ( n1 - n2 ), 4 I
(a) 30 Hz (b) 15 Hz (c) ( n1 - n2 ), 3 I (d) ( n1 - n2 ), I
(c) 60 Hz (d) 7.5 Hz
Telegram @unacademyplusdiscounts

Waves 687

78. A tuning fork of frequency 200 Hz is in unison with a Doppler’s Effect


sonometer wire. The number of beats heard per
second when the tension is increased by 1% is 86. Two sound sources emitting sound each of
(a)1 (2) 2 (c) 4 (d) 1/2 wavelength l are fixed at a given distance apart. A
listener moves with a velocity u along the line joining
79. Two organ pipes both closed at on end have length l the two sources. The number of beats heard by him
and ( l + Dl). Neglect end correction. If velocity of per second is
sound in air is v, the number of beats s–1 is (a) 2 u / l (b) u / l
(a) v /4 l (b) v /2 l u 2l
v v (c) (d)
(c) 2 ( Dl ) (d) 2 ( Dl ) 3l u
4l 2l
87. A bus is moving with a velocity of 5 ms–1 towards a
80. A fork of unknown frequency gives four beats s–1 huge wall. The driver sounds a horn of frequency
when sounded with another of frequency 256. The 165 Hz. If the speed of sound in air is 335 ms–1, the
fork is now loaded with a piece of wax and again four number of beats heard per second by the passengers
beats s–1 are heard. Then the frequency of the in the bus will be
unknown fork is (a) 3 (b) 4 (c) 5 (d) 6
(a) 256 Hz (b) 252 Hz (c) 264 Hz (d) 260 Hz
88. A train whistling at constant frequency is moving
81. When two waves of almost equal frequencies n1 and towards a station at a constant speed v. The train
n2 are produced simultaneously, then the time goes past a stationary observer on the station. The
interval between successive maxima is frequency n¢ of the sound as heard by the observer is
1 1 1
(a) (b) - plotted as a function of time t (figure.). Identify the
n1 - n2 n1 n2
expected curve. [NCERT Exemplar]
1 1 1
(c) + (d) n' n'
n1 n2 n1 + n2

82. A tuning fork of frequency 480 Hz produces 10 (a) (b)


beats –1 when sounded with a vibrating sonometer
string. What must have been the frequency of string
if slight increase in tension produces fewer beats s–1 t t
than before?
n'
(a) 490 Hz (b) 470 Hz
(c) 460 Hz (d) 480 Hz n
83. Two organ pipes, each closed at one end, give (c) (d)
5 beats s -1 when emitting their fundamental notes.
If their lengths are in the ratio 50 : 51, their t
t
fundamental frequencies are
(a) 250, 255 (b) 255, 260
(c) 260, 265 (d) 265, 270 89. A rocket is receding away from earth with velocity
= 0.2 c. The rocket emits signal or frequency
84. A source of sound gives 5 beats s–1 when sounded 4 ´ 107 Hz. The apparent frequency of the signal
with another source of frequency 100 Hz. The second produced by the rocket observed by the observer on
harmonic of the source together with a source of earth will be
frequency 205 Hz gives 5 beats s–1. What is the (a) 3 × 106 Hz (b) 4 × 106 Hz
frequency of the source? (c) 2.4 × 107 Hz (d) 5 × 107 Hz
(a) 105 Hz (b) 205 Hz
(c) 95 Hz (d) 100 Hz 90. A source of sound of frequency 256 Hz is moving
towards a wall with a velocity of 5 ms–1. Velocity of
85. Two uniform wires are vibrating simultaneously in sound is 330 ms–1. The number of beats s–1 heard by
their fundamental notes. The tension, lengths an observer standing between the source and the
diameters and the densities of the two wires are in wall is nearly
the ratio 8 : 1, 36 : 35, 4 : 1 and 1 : 2 respectively. If the 256 ´ 330 256 ´ 330 256 ´ 330
note of the higher pitch has a frequency 360 Hz, the (a) - (b) 256 -
325 325 325
number of beats produced per second is 256 ´ 330 256 ´ 330 256 ´ 330
(c) - (d) - 256
(a) 5 (b) 15 (c) 10 (d) 20 325 325 325
Telegram @unacademyplusdiscounts

688 JEE Main Physics

91. A bat flies at a steady speed of 4 m/s emitting a sound 95. A whistle giving out 450 Hz approaches a stationary
of f = 90 ´ 103 Hz. It is flying horizontally towards a observer at a speed of 33 ms–1. The frequency
vertical wall. The frequency of the reflected sound as heard by the observer in Hz is [velocity of sound in
detected by the bat will be (Take velocity of sound in air = 333 ms–1]
air as 330 m/s) (a) 409 (b) 429 (c) 517 (d) 500
3 3
(a) 88.1 ´ 10 Hz (b) 87.1 ´ 10 Hz 96. A train moves towards a stationary observer with
(c) 92.1 ´ 103 Hz (d) 89.1 ´ 103 Hz speed 34 ms–1. The train sounds a whistle and its
92. A source of sound emitting a tone of frequency 200 Hz frequency registered by the observer is f1. If the
moves towards an observer with a velocity v equal to train’s speed is reduced to 17 ms–1, the frequency
the velocity of sound. If the observer also moves away registered is f2 . If the speed of sound is 340 ms–1,
from the source with the same velocity v, the then the ratio f1/ f2 is
apparent frequency heard by the observer is (a) 18/19 (b) 1/2 (c) 2 (d) 19/18
(a) 50 Hz (b) 100 Hz (c) 150 Hz (d) 200 Hz 97. A racing car moving towards a cliff sounds its horn.
93. A source and an observer move away from each other The driver observes that the sound reflected from the
with a velocity of 10 m/s with respect to ground. If the cliff has a pitch one octave higher than the actual
observer finds the frequency of sound coming from sound of the horn. If v = the velocity of sound, the
velocity of the car is
the source as 1950 Hz, then actual frequency of the
source is (velocity of sound in air = 340 m/s) (a) v / 2 (b) v/2 (c) v/3 (d) v/4
(a) 1950 Hz (b) 2068 Hz 98. A source of sound S is moving with a velocity of
(c) 2132 Hz (d) 2486 Hz 50 ms–1 towards a stationary observer. The observer
94. A sound wave of frequency n travels horizontally to measures the frequency of the source as 1000 Hz.
the right. It is reflected from a large vertical plane What will be the apparent frequency of the source
surface moving to the left with speed v. The speed of when it is moving away from the observer after
the sound in the medium is c. Then, crossing him? The velocity of the sound in medium is
éc + vù 350 ms–1.
(a) the frequency of the reflected wave is ê
ë c - v úû (a) 750 Hz (b) 857 Hz
écù éc + vù (c) 1143 Hz (d) 1333 Hz
(b) the wavelength of the reflected wave is ê ú ê
ë n û ë c - v úû 99. A source of sound of frequency 500 Hz is moving
(c) the number of waves striking the surface per second is towards an observer with velocity 30 ms–1. The speed
éc + vù of sound is 330 ms–1. The frequency heard by the
êë c úû observer will be
(d) the number of beats heard by a stationary listener to the (a) 545 Hz (b) 580 Hz
nv (c) 558.3 Hz (d) 550 Hz
left to the reflecting surface is
c-v

Round II (Mixed Bag)

Only One Correct Option 2. Two uniform strings A and B made of steel are made
1. The ends of a stretched wire of length L are fixed at to vibrate under the same tension. If the first
X = 0 and X = L. In one experiment the displacement overtone of A is equal to the second overtone of B and
px if the radius of A is twice that of B, the ratio of the
of the wire is Y1 = A sin æç ö÷ sin wt and energy is E1 lengths of the strings is
èLø
(a) 2 : 1
and in another experiment its displacement is
(b) 3 : 4
y2 = A sin (2 px / L) sin 2 wt and energy is E2 . Then
(c) 3 : 2
(a) E2 = E1 (b) E2 = 2 E1
(d) 1 : 3
(c) E2 = 4 E1 (d) E2 = 16 E1
Telegram @unacademyplusdiscounts

Waves 689

3. In a sine wave, position of different particles at time 8. In an experiment, it was found that string vibrates in
t = 0 is shown in figure. The equation for this wave n loops when a mass M is placed on the pan. What
travelling along positive x-axis can be mass should be placed on the pan to make it vibrate
y in 2n loops, with same frequency. Neglect the mass of
the pan.
x (a) M / 4 (b) 4 M (c) 2 M (d) M/2
9. A heavy uniform rope changes vertically from the
ceiling, with its lower end free. A disturbance on the
(a) y = A sin( wt - kx) (b) y = A cos ( kx - wt ) rope travelling upwards from the lower end has a
(c) y = A cos ( wt - kx) (d) y = A sin( kx - wt ) velocity v at a distance x from the lower end such that
(a) v µ x (b) v µ x
4. A closed organ pipe and an open organ pipe of same
1 1
length produce 2 beats s–1
when they are set into (c) v µ (d) v µ
x x
vibrations together in fundamental mode. The length
of open pipe is now halved and that of closed pipe is 10. A string is under tension so that its length is
doubled. The number of beats produced will be increased by 1/n times its original length. The ratio of
(a) 7 (b) 4 (c) 8 (d) 2 fundamental frequency of longitudinal vibrations
5. Source and observer, both start moving and transverse vibrations will be
simultaneously from origin, one along X-axis and the (a) 1 : n (b) n2 : 1
other along Y-axis with speed of source equal to twice (c) n : 1 (d) n : 1
the speed of observer. The graph between the
11. A uniform rope having mass m hangs vertically from
apparent frequency ( n¢) observed by observer and
a rigid support. A transverse wave pulse is produced
time t in figure would be
at the lower end. The speed (v) of wave pulse varies
n' n' with height h from the lower end as shown in figure.
n' n
(a) n (b) n'
v v
(a) (b)
t t

n' n'
h h
n' n
(c) n (d) n'
v v
(c) (d)
t t

6. In question, the shape of the wave at time t = 3 s, if O


h h
is a fixed end (not free)in is.
12. The frequency of a sonometer wire is 100 Hz. When
(a) (b) 1 cm
O the weights producing the tension are completely
1 cm immersed in water, the frequency becomes 80 Hz and
1 cm on immersing the weights in a certain liquid, the
2 cm

(c) (d) frequency becomes 60 Hz. The specific gravity of the


1 cm
liquid is
2 cm
(a) 1.42 (b) 1.77
7. A string of mass 0.2 kg/m has length l = 0.6 m. It is (c) 1.21 (d) 1.82
fixed at both ends and stretched such that it has a 13. Oxygen is 16 times heavier than hydrogen. Equal
tension of 80 N. The string vibrates in three volumes of hydrogen and oxygen are mixed. The ratio
segments with amplitude = 0.5 cm. The amplitude of of speed of sound in the mixture to that in hydrogen is
transverse velocity is 1 32 2
(a) (b) (c) 8 (d)
(a) 9.42 ms–1 (b) 3.14 ms–1 8 17 17
(c) 1.57 ms–1 (d) 6.28 ms–1
Telegram @unacademyplusdiscounts

690 JEE Main Physics

14. The equation of a simple harmonic wave is given by (a) 0.5 ms–1 (b) 2.0 ms–1
p (c) 1.0 ms–1 (d) 4.0 ms–1
y = 5 sin (100 t - x) where x and y are in metre and
2
22. The difference between the apparent frequency of a
time is in second. The period of the wave in second
source of sound as perceived by the observer during
will be
its approach and recession is 2% of the natural
(a) 0.04 (b) 0.01 (c) 1 (d) 5
frequency of the source. If the velocity of sound in air
15. A table is revolving on its axis at 5 revolutions per is 300 ms–1, the velocity of source is
second. A sound source of frequency 1000 Hz is fixed (a) 12 ms–1 (b) 1.5 ms–1
on the table at 70 cm from the axis. The minimum (c) 3 ms–1 (d) 6 ms–1
frequency heard by a listener standing at a distance
23. A band playing music at a frequency f is moving
from the table will be (speed of sound 352 ms–1)
towards a wall at a speed vb. A motorist is following
(a) 1000 Hz (b) 1066 Hz
the band with a speed vm . If v is speed of sound, the
(c) 941 Hz (d) 352 Hz
expression for the beat frequency heard by the
16. The frequency of a tuning fork A is 2% more than the motorist is
frequency of a standard tuning fork. The frequency of ( v + v m )f ( v + v m )f
(a) (b)
the same standard tuning fork. If 6 beats s–1 are v + vb v - vb
heard when the two tuning forks A and B are excited, 2v b ( v + v m )f 2v m ( v + v b )f
(c) (d)
the frequency of A is v2 - v2b v2 - v2b
(a) 120 Hz (b) 122.4 Hz
(c) 116.4 Hz (d) 130 Hz 24. A massless rod is suspended by two identical strings
AB and CD of equal length.
17. The line of sight of a jet plane makes an angle of 60° A block of mass m is A C
with the vertical, and the sound appears to be coming suspended from point O
from over the head of the observer. The speed of jet such that BO is equal to (x)
plane is (taking speed of sound waves to be v) further it is observed that
(a) v (b) v / 3 the frequency of 1st B
O
D
(c) v 3 (d) 2 v harmonic (fundamental L
x
frequency) in AB is equal to
18. In brass, the velocity of longitudinal wave is M
2nd harmonic frequency in
100 times the velocity of the transverse wave. If
CD. Then length of BO is
y = 1 ´ 1011 Nm–2, then stress in the wire is
L 4L
(a) 1 ´ 1013Nm-2 (b) 1 ´ 10 9 Nm-2 (a) (b)
5 5
(c) 1 ´ 1011Nm-2 (d) 1 ´ 107 Nm-2 3L L
(c) (d)
19. A wire of density 9 ´ 103 kg/m 3 is stretched between 4 4
two clamps 1m apart and is subjected to an extension
25. Two string with masses per unit length of 25 g cm–1
of 4.9 ´ 10-4 cm. The lowest frequency of transverse
and 9 gcm–1 are joined together in series. The
vibration in the wire is ( Y = 9 ´ 1010 N/m 2 )
reflection coefficient for the vibration waves is
(a) 40 Hz (b) 35 Hz (c) 30 Hz (d) 25 Hz 9 3
(a) (b)
20. The displacement of a particle executing periodic 25 5
motion is given by y = 4 cos2 ( t / 2) sin (1000 t). This (c)
1
(d)
9
expression may be considered to be a result of 16 16
superposition of 26. Two radio stations broadcast their programmes at the
(a) two waves (b) three waves same amplitude A and at slightly different frequenies
(c) four waves (d) five waves w1 and w2 respectively where w2 - w1 = 103 Hz. A
21. The amplitude of wave disturbance propagating in detector is receiving signals from the two stations
1 simultaneously. It can only detect signals of intensity
positive direction of X-axis is given by y = at
1 + x2 > 2 A2 .The time interval between successive maxima of
1 the intensity of the signal received by the detector is
t = 0 and by y = at t = 2 s
1 + ( x - 1)2 (a) 103 s
where x and y are in metres. The shape of the wave (b) 10–3 s
disturbance does not change during propagation. The (c) 10–4 s
velocity of the wave is (d) 104 s
Telegram @unacademyplusdiscounts

Waves 691

27. Four simple harmonic vibrations, 32. Equations of a stationary wave and a travelling wave
y1 = 8 cos wt are y1 = a sin kx cos wt and y2 = a sin( wt - kx). The
p p
y2 = 4 cos æç wt + ö÷ phase difference between two points x1 =
3k
and
è 2ø
3p
y3 = 2 cos( wt + p) x2 = are f1 and f2 respectively for the two waves.
2k
3p ö
y4 = cos æç wt + ÷ The ratio f1/f2 is
è 2 ø
(a) 1 (b) 5/6
are superimposed on one another. The resulting (c) 3/4 (d) 6/7
amplitude and phase are respectively
æ 1ö æ 1ö
33. The displacement-time graphs for two sound waves A
(a) 45 and tan -1 ç ÷ (b) 45 and tan -1 ç ÷ and B are shown in figure, then the ratio of their
è 2ø è 3ø
intensities I A / I B is equal to
æ 1ö
(c) 75 and tan -1(2) (d) 75 and tan -1 ç ÷
è 3ø 2 A
28. When a train approaches a stationary observer, the 1
B
apparent frequency of the whistle is n¢ and when the 0
same train recedes away from the observer, the –1
C
apparent frequency is n¢¢. Then the apparent –2
frequency n when the observer moves with the train is
n¢ + n (a) 1 : 4 (b) 1 : 16 (c) 1 : 2 (d) 1 : 1
(a) n = (b) n = n ¢n ¢¢
2 34. Sound of wavelength l passes through a Quincke’s
2n ¢ n ¢¢ 2n ¢ n ¢¢
(c) n = (d) n = tube, which is adjusted to give a maximum intensity
n ¢ + n ¢¢ n ¢ - n ¢¢ I 0 . Through what distance should the sliding tube be
29. A stone is hung in air from a wire, which is stretched moved to give an intensity I 0 / 2 ?
over a sonometer. The bridges of the sonometer are (a) l /2 (b) l /3 (c) l / 4 (d) l /8
40 cm apart when the wire is in unison with a tuning
35. An observer starts moving with uniform acceleration
fork of frequency 256. When the stone is completely
a, towards a stationary sound source of frequency f0 .
immersed in water, the length between the bridges is
As the observer approaches the source, the apparent
22 cm for re-establishing unison. The specific gravity
frequency (f ) heard by the observer varies with time
of material of stone is
(t) is
( 40 2 ) ( 40 2 )
(a) (b)
( 40 ) + (22) 2
2
( 40 ) - (22) 2
2

22 40 r r
(c) 256 ´ (d) 256 ´
40 22 (a) (b)
30. An aeroplane be is above the head of an observer and
the sound appears to be coming at an angle of 60°
with the vertical. If velocity of sound is v, then the t t
speed of aeroplane is
3
(a) v (b) v r r
2
v (c) (d)
(c) (d) 2
2
31. Standing waves are produced by the superposition of t t
two waves
l
y1 = 0.05sin (3 pt + 2 x) 36. A string l is stretched by and transverse waves in
30
y2 = 0.05 sin (3 pt + 2x) the string are found to travel at a speed v0. Speed of
l
where x and y are in metres and t is in second. What transverse waves when the string is stretched by
15
is the amplitude of the particle at x = 0.5 m?. Given,
will be
cos 57.3° = 0.54.
v0
(a) 2.7 cm (b) 5.4 cm (a) (b) 2v 0
2
(c) 8.1 cm (d) 10.8 cm
(c) 2 2v 0 (d) 2v 0
Telegram @unacademyplusdiscounts

692 JEE Main Physics

37. Which of the following statements is correct? 43. A transverse harmonic wave on a string is described
(a) Both sound and light waves in air are longitudinal by y ( x, t) = 3.0 sin (36t + 0.018 x + p/4)
(b) Both sound and light waves in air are transverse where x and y are in cm and t is in s. The positive
(c) Sound waves in air are longitudinal while light transverse direction of x is from left to right. [NCERT Exemplar]
(d) Sound waves in air are transverse while light longitudinal (a) The wave is travelling from right to left
38. A light pointer fixed to one prong of a tuning fork (b) The speed of the wave is 20m/s
touches a vertical plate. The fork is set vibrating and (c) Frequency of the wave is 5.7 Hz
the plate is allowed to fall freely. Eight complete (d) The least distance between two successive crests in the
oscillations are counted when the plate falls through wave is 2.5 cm
10 cm. What is the frequency of the tuning fork? 44. A string of length L is stretched along the x-axis and
8 7
(a) 112 Hz (b) 56 Hz (c) Hz (d) Hz is rigidly clamped at its two ends. It undergoes
7 8 transverse vibrations. If n is an integer, which of the
following relations may represent the shape of the
More Than One Option Correct string at any time t?
39. In the principle of superposition, the characteristic æ n p xö æ n p xö
(a) y = A sin ç ÷ cos wt (b) y = A sin ç ÷ sin wt
that gets added vectorially is è L ø è L ø
(a) displacement (b) velocity æ n p xö æ n p xö
(c) y = A cos ç ÷ cos wt (d) y = A cos ç ÷ sin wt
(c) amplitude (d) frequency è L ø è L ø
40. The equation of a progressive wave is 45. Which one of the following represents a travelling
y = a sin (200 t - x) wave?
(a) y = A ( x - vt ) (b) y = A cos ( ax - bt )
where x is in metre and t is in second. The velocity of
wave is (c) y = A log ( x - vt ) (d) y = f ( x2 - vt 2 )
(a) 200 ms–1 (b) 100 ms–1 46. The displacement of a string is given by
(c) 50 ms–1 (d) None of these y( x, t) = 0.06 sin (2px/3) cos (120 pt)
41. A wave is represented by the equation where x and y are in m and t in s. The length of the
y = A sin [10 p x - 15 p t + ( p / 3)] string is 1.5m and its mass is 3.0 ´ 10-2 kg.
[NCERT Exemplar]
where x is in metre and t is in second. The expression
(a) It represents a progressive wave of frequency 60 Hz
represents
(b) It represents a stationary wave of frequency 60 Hz
(a) a wave travelling in positive x-direction with velocity
(c) It is the result of superposition of two waves of
1.5 ms -1
wavelength 3 m, frequency 60 Hz each travelling with a
(b) a wave travelling in negative x-direction with velocity speed of 180 m/s in opposite direction
1.5 ms -1 (d) Amplitude of this wave is constant
(c) a wave travelling in negative x-direction with wavelength
47. Speed of sound waves in a fluid depends upon
2.0 m
[NCERT Exemplar]
(d) a wave travelling in positive x-direction with wavelength
0.2 m (a) directly on density of the medium
(b) square of Bulk modulus of the medium
42. A particle is executing SHM with amplitude A. At
A (c) inversly on the square root of density
displacement x2 = - , force acting on the particle is (d) directly on the square root of bulk modulus of the
4
medium
F, potential energy of the particle is U, velocity of
48. For the y = 20 sin p æç + ö÷ the correct statement is
particle is v and kinetic energy is K. Assuming x t
potential energy to be zero at mean position. At 2ø è4
A (where x is in metre and time is in second)
displacement x =
2 (a) amplitude is 20 m and frequency is 0.25
(a) force acting on the particle will be 2F (b) wavelength is 20 m and frequency is 1
(b) potential energy of particle will be 4U 1
(c) frequency is and wavelength is 20 cm
4 2
(c) velocity of particle must be v
5 p
(d) w = 2p and k =
(d) kinetic energy of particle will be 0.8 K 2
Telegram @unacademyplusdiscounts

Waves 693

49. An air column in a pipe, which is closed at one end, is 55. What is the velocity of sound wave?
in resonance with a vibrating tuning fork of (a) 200 ms–1 (b) 180 ms–1
frequency 264 Hz. If v = 330 ms–1, the length of the (c) 100 ms–1 (d) 194 ms–1
column in cm is 56. At x = 0, how many times does the net amplitude be
(a) 31.25 (b) 62.50 zero between 0 and 1 s?
(c) 93.75 (d) 125 (a) 46 (b) 42 (c) 50 (d) 100
50. A thin plane membrane separates hydrogen at 27°C
from hydrogen at 127°C, both being at the same Passage II
pressure. A plane sound wave enters from the cooler According to Doppler’s effect in sound, the apparent
to the hotter side. If angle of incidence on the frequency ( n¢) of sound heard by a listener when there
membrane is 30°C, then the angle of refraction is relative motion between the listener and the source is
æ v - vL ö
(a) sin -1(1 / 3 ) (b) sin -1(2 / 8) given by n¢ = ç ÷ n,
-1
(c) sin (3 / 8) (d) sin -1(2 / 3) è v - vS ø
where n is actual frequency of sound emitted by the
51. The transverse displacement of a string (clamped at source, v is velocity of sound in air, vL is velocity of
its both ends) is given by listener and vS is velocity of source. The velocities
y ( x, t) =0.06 sin (2px/3) cos (120pt) along SL are taken as positive and velocities along LS
are taken as negative.
All the points on the string between two consecutive
nodes vibrate with [NCERT Exemplar] 57. A tuning fork producing sound of frequency 256 is
(a) same frequency (b) same phase moving towards a listener with a velocity of 20 ms–1.
(c) same energy (d) different amplitude If velocity of sound in air is 340 ms–1, the apparent
frequency of sound heard is
52. In case of interference of two waves each of intensity (a) 272 (b) 256 (c) 300 (d) 340
I 0 , the intensity at a point of constructive
interference will be 58. In the above case, if the fork moves at right angles to
(a) 4 I0 for coherent source the line SL, the apparent frequency of sound heard
(b) 2 I0 for coherent source would be
(c) 4 I0 for incoherent source (a) zero (b) 256 (c) 272 (d) 240
(d) 2 I0 for incoherent source
53. A train, standing in a station yard, blows a whistle of Assertion and Reason
frequency 400 Hz in still air. The wind starts blowing
in the direction from the yard to the station with a
Direction Question No. 59 to 68 are Assertion-Reason type.
Each of these contains two Statements: Statement I (Assertion),
speed of 10 m/s. Given that the speed of sound in still Statement II (Reason). Each of these questions also has four
air is 340m/s. [NCERT Exemplar] alternative choice, only one of which is correct. You have to
(a) the frequency of sound as heard by an observer standing select the correct choices from the codes (a), (b), (c) and (d) given
on the platform is 400 Hz below
(b) the speed of sound for the observer standing on the (a) If both Assertion and Reason are true and the Reason
platform is 350 m/s is correct explanation of the Assertion
(c) the frequency of sound as heard by the observer standing (b) If both Assertion and Reason are true but Reason is
on the platform will increase not correct explanation of the Assertion
(d) the frequency of sound as heard by the observer standing (c) If Assertion is true but Reason is false
on the platform will decrease (d) If Assertion is false but the Reason is true
59. Assertion Sound, light cannot propagate in the
Comprehension Based Questions vaccum.
Passage I Reason Sound is a square wave. It propagates in a
Two waves y1 = A cos (0.5 px - 100 pt) and medium by a virtue of damping oscillation.
y2 = A cos (0.46 px - 92 pt) are travelling in a pipe 60. Assertion Under given conditions of pressure and
along x-axis. temperature, sound travels faster in a monoatomic
54. How many times in a second does a stationary any gas than in the diatomic gas.
observer hear loud sound (maximum intensity)? Reason Opposition to travel is more in diatomic gas
(a) 4 (b) 8 than in monoatomic gas.
(c) 10 (d) 12
Telegram @unacademyplusdiscounts

694 JEE Main Physics

61. Assertion A wave of frequency 500 Hz is propagating The distance between two consecutive anti-nodes will be
with a velocity of 350 ms–1. Distance between two 4 m.
particles with 60° phase difference is 12 cm. Reason The data is insufficient.
l
Reason x = f. 66. Assertion Displacements produced by two waves at a
2p p
point are y1 = a sin wt, y2 = a sin æç wt + ö÷. The
62. Assertion Violet shift indicates that a star is è 2ø
approaching the earth. resultant amplitude is a a 2.
Reason Violet shift indicates decrease in apparent Reason R = a z + b z + 2 ab cos p / 2
wavelength of light.
67. Assertion When two waves each of amplitude a
63. Assertion Quality of sound depends on number and produce a resultant wave of amplitude a, the phase
frequency of overtones produced by the instrument.
difference between them must be 120°.
Reason Pitch of sound depends on frequency of the
Reason If follows from
source.
R = a 2 + b2 + 2 ab cos f .
64. Assertion A tuning fork produces 4 beats s–1 with
49 cm lengths of a stretched sonometer wire. The 68. Assertion If speed of sound in a gas is 336.6 ms–1,
frequency of fork is 396 Hz. number of beats s–1 by 2 waves of length 1 m and 1.01
m is 3.
Reason n = 4( 49 + 50) = 396 Hz.
Reason using the relation v = nl.
65. Assertion The equation of a stationary wave is
px
y = 20 sin cos wt.
4

Previous Years’ Questions


69. Fundamental frequency of a sonometer wire is n. If motor cycle, there is a stationary electric siren. How
the length and diameter of the wire are doubled far has the motor cycle gone when the driver hears
keeping the tension same, then the new fundamental the frequency of the siren at 94% of its value when
frequency is [UP SEE 2009] the motor cycle was at rest? (speed of sound = 330
2n n n ms–1) [AIEEE 2009]
(a) (b) (c) 2n (d)
2 2 2 4 (a) 98 m (b) 147 m
70. A car sounding its horn at 480 Hz moves towards a (c) 196 m (d) 49 m
high wall at a speed of 20 ms–1. If the speed of sound 74. Three sound waves of equal amplitudes have
is 340 ms–1, the frequency of the reflected sound frequencies ( n - 1), n, ( n + 1). They superpose to give
heard by the girl sitting in the car will be closest to beats. The number of beats produced per second will
[UP SEE 2009] be [AIEEE 2009]
(a) 540 Hz (b) 524 Hz (c) 568 Hz (d) 480 Hz (a) 3 (b) 2 (c) 1 (d) 4
71. A cylindrical tube open at both ends, has a 75. The speed of sound in oxygen (O2 ) at a certain
fundamental frequency f in air. The tube is dipped temperature is 460 ms–1. The speed of sound in
vertically in water so that half of it is in water. The helium (He) at the same temperature will be (assume
fundamental frequency of air column is now both gases to be ideal) [AIEEE 2008]
[UP SEE 2009] (a) 330 ms–1 (b) 1420 ms–1
(a) f /2 (b) f (c) 3 f / 4 (d) 2f (c) 500 ms–1 (d) 650 ms–1
72. The driver of a car travelling with speed 30 ms–1 76. While measuring the speed of sound by performing a
towards a hill sounds a horn of frequency 600 Hz. If resonance column experiment, a student gets the
the velocity of sound in air is 330 ms–1, the frequency first resonance condition at a column length of 18 cm
of reflected sound as heard by driver is [AIEEE 2009] during winter. Repeating the same experiment
(a) 500 Hz (b) 550 Hz during summer, he measures the column length to be
(c) 555.5 Hz (d) 720 Hz x cm for the second resonance. Then, [AIEEE 2008]

73. A motor cycle starts from rest and accelerates along a (a) 36 > x > 18 (b) 18 > x
straight path at 2 ms–2. At the starting point of the (c) x > 54 (d) 54 > x > 36
Telegram @unacademyplusdiscounts

Waves 695

77. A wave travelling along the x-axis is described by the 85. The wavelength of special line for a given element in
equation y (x, t) = 0.005 cos (ax - b t). If the light received from a distant star is increased by
wavelength and the time period of wave are 0.08 m 0.32%. Velocity of the star is [Karnataka CET 2008]
and 2.0 s respectively, then a and b in appropriate (a) 4.8 × 105 ms–1, away from the earth
units are [AIEEE 2008] (b) 9.6 × 105 ms–1, towards earth
p (c) 4.8 × 104 ms–1, receding from the earth
(a) a = 12.50 p , b = (b) a = 25.00 p , b = p
2.0 (d) 9.6 × 105 ms–1, away from the earth
0.08 2.0 0.04 1.0
(c) a = ,b= (d) a = ,b= 86. A tuning fork produces 8 beats s–1 with both, 80 and
p p p p
70 cm of stretched wire of sonometer. Frequency of
78. A wave travelling along a string is described by the the fork is [Karnataka CET 2008]
equation y = A sin ( wt - kx) [UP SEE 2008] (a) 120 Hz (b) 128 Hz
The maximum particle velocity is (c) 112 Hz (d) 240 Hz
(a) Aw (b) w/k (c) dx /dk (d) x /1 87. When two tuning forks (fork 1 and fork 2) are
79. Two waves represented by y = a sin ( wt - kx) and sounded simultaneously, 4 beats s–1 are heard. Now,
y = a cos ( wt - kx) are superposed. The resultant some tape is attached on the prong of the fork 2.
wave will have an amplitude [UP SEE 2008] When the tuning forks are sounded again, 6 beats s–1
(a) a (b) 2a are heard. If the frequency of fork 1 is 200 Hz, then
(c) 2a (d) zero what was the original frequency of fork 2?
[UP SEE 2007, AIEEE 2005]
80. The temperature at which speed of sound in air
(a) 200 Hz (b) 202 Hz
becomes double of its value at 27°C is [UP SEE 2008]
(c) 196 Hz (d) 204 Hz
(a) 54ºC (b) 327ºC
(c) 927ºC (d) None of these 88. A sound absorber attenuates the sound level by
81. A tuning fork of frequency 250 Hz produces a beat 20 dB. The intensity decreases by a factor of
[UP SEE 2007]
frequency of 10 Hz when sounded with a sonometer
vibrating as its fundamental frequency. When the (a) 1000 (b) 10000
tuning fork is filed, the beat frequency decreases. If (c) 10 (d) 100
the length of sonometer wire is 0.5 m, the speed of 89. A pipe open at both the ends produces a note of
transverse wave is [Kerala CET 2008] fundamental frequency f1. When the pipe is kept
(a) 260 ms–1 (b) 250 ms–1 3
with th of its length in water, it produces a note of
(c) 240 ms–1 (d) 500 ms–1 4
f
82. A glass tube of length 1.0 m is completely filled with fundamental frequency f2 . The ratio of 1 is
f2
water. A vibrating tuning fork of frequency 500 Hz is
[BVP Engg. 2007]
kept over the mouth of the tube and the water is
4 3
drained out slowly at the bottom of the tube. If the (a) (b)
3 4
velocity of sound in air is 330 ms–1, then the total 1
number of resonances that occur will be (c) 2 (d)
2
[Kerala CET 2008]
(a) 2 (b) 3 (c) 1 (d) 5 90. An observer standing near the sea shore min–1 . If the
83. A bus is moving with a velocity of 5 ms–1 towards a wavelength of the water wave is 10 m, then the
huge wall. The driver sounds a horn of frequency velocity of water wave is [BVP Engg. 2007]
165 Hz. If the speed of sound in air is 335 ms–1, the (a) 540 ms–1 (b) 5.4 ms–1
number of beats heard per second by a passenger (c) 0.184 ms–1 (d) 9 ms–1
inside the bus will be [Kerala CET 2008] 91. Two organ pipes of length 50 cm and 50.5 cm produce
(a) 3 (b) 4 three beats. Then, the velocity of sound is
(c) 5 (d) 6 [BVP Engg. 2006]
84. Stationary waves of frequency 200 Hz are formed in (a) 30 ms–1 (b) 300 ms–1
air. If the velocity of the wave is 360 ms–1, the (c) 303 ms–1 (d) 606 ms–1
shortest distance between two anti-nodes is 92. Musical interval between two notes of frequencies
[Karnataka CET 2008] 320 and 240 is [BVP Engg. 2006]
(a) 1.8 m (b) 3.6 m
(a) 1.33 (b) 8
(c) 0.9 m (d) 0.45 m
(c) 7 (d) 1.78
Telegram @unacademyplusdiscounts

696 JEE Main Physics

93. Two balls of different mass have the same kinetic 101. A source of sound of frequency 500 Hz is moving
energy. The ball have the greater momentum, will be towards a stationary observer with velocity 30 ms–1.
[BVP Engg. 2006] The speed of sound is 330 ms–1. The frequency heard
(a) both having equal masses by the observer will be [UP SEE 2005]
(b) lighter one
(a) 545 Hz (b) 580 Hz (c) 458.3 Hz (d) 550 Hz
(c) heavier one
(d) None of the above 102. A stone is dropped into a lake from a tower 500 m
high. The sound of the splash will be heard at the top
94. If the velocity of sound in air is 350 ms–1, then
of the tower approximately after a time of (take
fundamental frequency of an open organ pipe of
velocity of sound in air = 330 ms–1) [Kerala CET 2005]
length 50 cm is [BVP Engg. 2006]
(a) 11.5 s (b) 1.5 s (c) 10 s (d) 14 s
(a) 350 Hz (b) 520 Hz (c) 300 Hz (d) 20 Hz
95. A string is stretched between fixed points separated 103. A tuning fork of 512 Hz is used to produce resonance
by 75.0 cm. It is observed to have resonant in a resonance tube experiment. The level of water at
frequencies of 420 Hz and 315 Hz. There are other first resonance is 30.7 cm and at second resonance is
resonant frequencies between these two. Then, the 63.2 cm. The error in calculating velocity of sound is
lowest resonant frequency for this string is [AIEEE 2006] [IIT JEE 2005]
(a) 1.05 Hz (b) 1050 Hz (a) 204.8 cms–1 (b) 102.4 cms–1
(c) 10.5 Hz (d) 105 Hz (c) 51.2 cms–1 (d) 161.3 cms–1
96. A whistle producing sound waves of frequencies 104. An open pipe is in resonance in 2nd harmonic with
9500 Hz and above is approaching a stationary frequency f1. Now one end of the tube is closed and
person with speed v ms–1. The person can hear frequency is increased to f2 such that the resonance
frequencies upto a maximum of 1000 Hz, the again occurs in nth harmonic. Choose the correct
maximum value of v upto which he can hear the option. [IIT JEE 2005]
whistle is [AIEEE 2006]
3 5
(a) 15 / 2 ms -1 (b) 15 ms -1 (a) n = 3, f2 = f1 (b) n = 3, f2 = f1
4 4
(c) 30 ms -1 (d) 15 2 ms -1 5 3
(c) n = 5, f2 = f1 (d) n = 5, f2 = f1
4 4
97. Two running forks P and Q when set vibrating, given
4 beats s–1. If a prong of the fork P is filed, the beats 105. An observer moves towards a stationary source of
are reduced to 2/s. What is frequency of P, if that of Q sound, with a velocity one-fifth of the velocity of
is 250 Hz? [UP SEE 2006] sound. What is the percentage increase in the
(a) 246 Hz (b) 250 Hz (c) 254 Hz (d) 252 Hz apparent frequency? [AIEEE 2005]
(a) 20% (b) 5% (c) 0.5% (d) Zero
98. A source and an observer are moving towards each
other with a speed equal to v/2, where v is the speed 106. The function sin2 (wt) represents [AIEEE 2005]
of sound. The source is emitting sound of frequency n. (a) a simple harmonic motion with a period p / w
The frequency heard by one observer will be (b) a periodic, simple harmonic motion with a period 2p / w
[BVP Engg. 2005]
(c) a periodic, but not simple harmonic motion with a period
n
p/ w (a) zero (b) n
3
(d) a periodic, but not simple harmonic motion with a period
99. An open pipe is suddenly closed at one end with the 2p / w
result that the frequency of third harmonic of the 107. Two simple harmonic motions are represented by the
closed pipe is termed to be higher by 100 Hz, then the p
equations y1 = 0.1sin æç100 pt + ö÷ and y2 = 0.1 cos pt.
fundamental frequency of the open pipe is è 3ø
[UP SEE 2005] The phase difference of the velocity of particle 1 with
(a) 200 Hz (b) 150 Hz (c) 100 Hz (d) 250 Hz respect to the velocity of particle 2 is [AIEEE 2005]
100. A string vibrates according to the equation p -p p -p
(a) (b) (c) (d)
æ2 p x ö 6 3 3 6
Y = 5 sin ç ÷ cos 20 pt, where x and y are in cm
è 3 ø 108. A cylindrical tube open both ends has a fundamental
and t in second. The distance between two adjacent frequency f in air. The tube is dipped vertically in
nodes is [UP SEE 2005] water so that half of it is in water. The fundamental
(a) 3 cm (b) 4.5 cm frequency of the air-column is now [AIEEE 2012]
(c) 6 cm (d) 1.5 cm (a) f (b) f /2 (c) 3 f / 4 (d) 2f
Telegram @unacademyplusdiscounts

Waves 697

109. The transverse displacement y ( x, t) of a wave on a beats. The number of beats produced per second will
- (ax 2 + bx 2 +2 ab ´ t)
string is given by y ( x, t) = e . This be [AIEEE 2009]
represents a [AIEEE 2011] (a) 4 (b) 3 (c) 2 (d) 1
a 112. A student is performing the experiment of resonance
(a) wave moving in + x direction with speed
b column. The diameter of the column tube is 4 cm. The
b frequency of the tuning fork is 512 Hz. The air
(b) wave moving in -x direction with speed temperature is 30°C in which the speed of sound is
a
(c) standing wave of frequency b 336 m/s. The zero of the meter scale coincides with
1 the top end of the resonance column tube. When the
(d) standing wave of frequency first resonance occurs the reading of the water level
b
in the column is [IIT JEE 2012]
110. The equation of the wave on a string of linear (a) 14.0 cm (b) 1.2 cm (c) 16.4 cm (d) 17.6 cm
mass density 0.04 kgm -1 is given by
113. A police car with a siren of frequency 8 Hz is moving
é æ t x öù
y = 0.02 (m ) sin ê2p ç - ÷ú with uniform velocity 36 km/h towards a tall building
ë è 0.04 (s ) 0.50 (m ) øû which reflects the sound waves. The speed of sound
The tension in the string is in air is 320 m/s. The frequency of the siren heard by
(a) 6.25 N (b) 4.0 N (c) 12.5 N (d) 0.5 N the car driver is [IIT JEE 2011]
(a) 0.50 Hz (b) 8.50 Hz
111. Three sound waves of equal amplitudes have
(c) 9.75 Hz (d) 7.50 Hz
frequencies ( n - 1), n, ( n + 1). They superpose to give

Answers
Round I
1. (c) 2. (a) 3. (a) 4. (c) 5. (d) 6. (a) 7. (c) 8. (b) 9. (c) 10. (d)
11. (c) 12. (a) 13. (b) 14. (d) 15. (a) 16. (a) 17. (a) 18. (b) 19. (c) 20. (c)
21. (a) 22. (d) 23. (b) 24. (c) 25. (b) 26. (d) 27. (a) 28. (a) 29. (a) 30. (a)
31. (b) 32. (c) 33. (b) 34. (d) 35. (b) 36. (b) 37. (b) 38. (d) 39. (b) 40. (a)
41. (d) 42. (c) 43. (b) 44. (a) 45. (b) 46. (a) 47. (b) 48. (b) 49. (a) 50. (c)
51. (a) 52. (b) 53. (b) 54. (b) 55. (c) 56. (a) 57. (b) 58. (a) 59. (b) 60. (d)
61. (d) 62. (a) 63. (b) 64. (d) 65. (b) 66. (a) 67. (b) 68. (c) 69. (c) 70. (b)
71. (a) 72. (a) 73. (d) 74. (b) 75. (b) 76. (b) 77. (b) 78. (a) 79. (c) 80. (d)
81. (a) 82. (b) 83. (a) 84. (a) 85. (c) 86. (a) 87. (c) 88. (c) 89. (c) 90. (a)
91. (c) 92. (d) 93. (b) 94. (c) 95. (d) 96. (d) 97. (c) 98. (a) 99. (d)

Round II
1. (c) 2. (d) 3. (d) 4. (a) 5. (b) 6. (a) 7. (c) 8. (a) 9. (b) 10. (c)
11. (a) 12. (b) 13. (d) 14. (a) 15. (c) 16. (a) 17. (b) 18. (d) 19. (b) 20. (b)
21. (a) 22. (c) 23. (c) 24. (a) 25. (c) 26. (b) 27. (a) 28. (c) 29. (b) 30. (b)
31. (b) 32. (d) 33. (d) 34. (d) 35. (d) 36. (d) 37. (c) 38. (b) 39. (a,c) 40. (a)
41. (b,c) 42. (b,d) 43. (a,b,c) 44. (a,b) 45. (a,b,c) 46. (b,c) 47. (c,d) 48. (a) 49. (a,c) 50. (a)
51. (a,b,d) 52. (a,d) 53. (a,b) 54. (a) 55. (a) 56. (b) 57. (a) 58. (a) 59. (b) 60. (c)
61. (a) 62. (b) 63. (d) 64. (a) 65. (c) 66. (a) 67. (a) 68. (a) 69. (d) 70. (a)
71. (b) 72. (d) 73. (a) 74. (c) 75. (b) 76. (c) 77. (b) 78. (a) 79. (b) 80. (c)
81. (a) 82. (b) 83. (c) 84. (c) 85. (d) 86. (a) 87. (c) 88. (d) 89. (d) 90. (d)
91. (c) 92. (a) 93. (c) 94. (a) 95. (d) 96. (b) 97. (a) 98. (d) 99. (a) 100. (a)
101. (d) 102. (a) 103. (c) 104. (c) 105. (a) 106. (a) 107. (b) 108. (a) 109. (b) 110. (a)
111. (c) 112. (b) 113. (b)
Telegram @unacademyplusdiscounts

the Guidance
Round I
2p But Intensity = (Amplitude)2
1. As, phase difference = ´ path difference
l 1
\ (Amplitude)2 µ 2
2p r
Þ 1.6 p = ´ 40
l 1
or Amplitude µ
Þ l = 50 cm = 0.5 m r
Now as v = lf Clearly, at distance 2r, amplitude becomes A/2.
v 300
Þ f= = = 660 Hz 5. Given, y = 8 sin 2p ( 0.1 x - 2 t)
l 0.5
Compare it with the equation of wave motion

2. Given, y = 10 - 4 sin éê100t - æx tö
ë 10 úû y = r sin 2p ç - ÷
èl Tø
Comparing it with the standard equation of wave motion
1
é 2p 2p ù = 0.1= 10 cm
y = r sin ê t- x , we get l
ëT l úû
2p 2p
2p 2p p From, f= Dx = ´ 2 = 0.4 ´ 180° = 72°
= 100 or T = = s l 10
T 100 50
2p 1 6. Time required for a point to move from maximum
= T 1
l 10 displacement to zero displacement is t = =
4 4n
or l = 20 p
1 1 æ 1ö
l 20 p Þ n= = = 1.47 Hz ç as T = ÷
and velocity, v= = = 100 ms -1 4 t 4 ´ 0.170 è nø
T p / 50
l l 7. At given temperature and pressure
3. In fundamental mode, l = 2æç ö÷ = 1
è 4ø 2 vµ
r
v1 r2 4
Þ = = = 2 :1
N v2 r1 1

A A 8. Frequency remains the same i. e. , 1000 Hz and wavelength


changes to
λ/4 λ/4 vw 1500
w l = = = 1.5 m
v 1000
l
v
Þ l = 2l 9. In the first medium, frequency, n =
l
Given l = 100 cm, n = 2.53 kHz It remains the same in second medium, i. e. ,v ¢ = v
Using v = nl v ¢ 2v v
\ = =
Þ v = 2.53 ´ 10 3 ´ 2 ´ 100 ´ 10 -2 l¢ l¢ l
= 5.06 ´ 10 3 m/s \ l¢ = 2l
= 5.06 km/s 10. Comparing the given equation with the standard form
4. As, intensity = energy/s/area = power/area é 2 pt 2 px ù
y = r cosê - , we get,
From a point source, energy spreads over the surface of a ë T l úû
sphere of radius r. 2p
Coefficient of t = = 2 pn = 4 p ,
P P 1 T
\ Intensity = = µ
A 4 pr 2 r 2 n = 2 Hz
Telegram @unacademyplusdiscounts

Waves 699

11. As, v µ T g RT
17. From, v =
T2 v 2 M
Þ =
T1 v1 Dv 1 æ DT ö
= ç ÷
2 v 2è T ø
æv ö
Þ T2 = T1ç 2 ÷ Dv 1 æ 1ö
è v1 ø Þ ´ 100 = ç ÷ ´ 100
v 2 èT ø
Þ T2 = 273 ´ 4 = 1092 K 1 1
1 1 = ´ ´ 100
12. Here, n = 500 Hz, T = = = 2 ´ 10 -3 s 2 300
n 500 = 0.167%
Phase differences corresponding to 2 ´ 10 -3 s = 2p rad 2 px
æ ö
18. As, y 2 = a2 cosç wt - + f÷
Phase differences corresponding to è l ø
2p ´ 1 ´ 10 -3 ép æ 2 px öù
1 ´ 10 -3 s = = p rad = a2 sinê + ç wt - + f÷ ú
2 ´ 10 -3 ë2 è l øû
13. Here, n = 120 Hz, æ 2 px ö
Compare it with, y1 = a1 sinç wt - ÷
è l ø
x = 0.8m, f = 0.5 p
2p æp ö
From f= x, Phase difference = ç + f ÷
l è2 ø
2 px 2p ´ 0.8 l æp ö
l= = = 3.2m \ Path difference = ç + f÷
f 0.5 p 2p è 2 ø

Þ v = nl = 120 ´ 3.2 = 384 ms-1 19. Resonance occurs when amplitude is maximum i.e., when
the denominator of this equation is minimum.
14. Given, frequency of A, fA = 324 Hz
Now, frequency of B, fB = fA ± beat frequency
20. y = A sin( at - bx + c) represents a wave, when a may
correspond to w and b may correspond to k.
= 324 ± 6
or fB = 330 or 318 Hz 21. Ultrasonic waves are produced by piezoelectric effect.
Now, if tension in the string is slightly reduced its frequency dy é xù
22. As, = y 0 cos 2p ê ft - ú ´ 2 pf
will also reduce from 324 Hz. dt ë lû
Now, if fB = 330 and fA reduces, then beat frequency should æ dy ö
Maximum particle velocity = ç ÷ = 2 pfy 0 ´ 1
increase which is not the case but if fB = 318 Hz and fA è dt ø max
decreases the beat frequency should decrease, which is the Wave velocity = f l
case and hence fB = 318 Hz.
As 2 pfy 0 = 4 fl ,
15. Height of the tower (h) = 300 m 2 py 0 py 0
\ l= =
Initial velocity u = 0 4 2
Acceleration due to gravity g = 9.8 m/s 2 23. For infrasonics, frequency n < 20 cms–1
Speed of sound in air = 340 1 u 330
l= > » 10 m
Time taken by stone to reach the pond = t1 n 20
Using equation of motion, 24. Mechanical transverse waves can propagate through solid
1 1 only as solids have elasticity of shape.
h = ut + gt12 Þ 300 = 0 + ´ 9.8 t12
2 2 25. Molecular weight of mixture
300 ´ 2 n1M1 + n2 M2
Þ t1 = = 7.82 s Mmix =
9.8 n1 + n2
Time taken by the sound to reach the top of the tower 1 ´ 4 + 2 ´ 32 68
= =
h 300 1´ 2 3
t2 = = = 0.88 s
v 340 68
= ´ 10 -3kg mol-1
\Total time t = t1 + t 2 = 7.82 + 0.88 = 8.7 s 3
k 3
16. As, v = For helium, CV1 = R
r 2
5
\ k = v 2r = 2.86 ´ 1010 N/m3 For oxygen, CV2 = R
2
Telegram @unacademyplusdiscounts

700 JEE Main Physics

3R 5R 33. Let the equation of wave be y = A sin ( wt - kx )


n1CV1 + n2CV2 1´ +2´
(CV ) mix = = 2 2 = 13R
n1 + n2 1+ 2 6 where, w = 2 pn
2p
Now, (C p) mix = (CV ) mix + R and k=
13R 19R l
= +R = w 2p w
6 6 Wave velocity, v =nl = ´ =
(C p) mix 19 2p k k
Þ g mix = = Maximum particle velocity v = Aw
(CV ) mix 13
w
g mixRT 19 8.31 ´ 300 For V = v; = Aw
Sound, v= = ´ k
Mmix 13 68 ´ 10 -3
1 l
3 or A= =
k 2p
= 400.8 ms–1
34. If a amplitude of sound from A and B each,then IA = IB ka2,
26. Comparing the given equation with the standard form where k is constant and is amplitude
é 2 pt 2 px ù
y = r sin ê + , we get Loudness due to C (i. e. , A + B )
ë T l úû
IC = k (2a) 2 4 IA
2p 2p
= 10 , =1 æI ö
T l \ n = 10 log10 ç C ÷ = 10 log10 4
l 10 è IA ø
and v= = = 10 ms–1
T 1 = 10 ´ 0.6 = 6
gp 4 ´ 1 + 1 ´ 16
27. As, v = , The speed (v) will be highest for the gas for 35. If rH = 1, then rmix = =4
r ( 4 + 1)
which g is highest, which is a monoatomic gas. v mix rH 1 1
Þ = = =
28. It is known that when loudness decreases by 10 dB, intensity vH r mix 4 2
of sound decreases by a factor 10. Therefore, when loudness vH 1224
decreases by 20 dB, i. e. , 2 ´ 10 dB, the intensity of sound \ v mix = = = 612 ms-1
2 2
would decrease by a factor of10 2, i. e. , 100.
36. Since the point x = 0 is a node and reflection is taking place
c 3 ´ 10 8 from point X = 0.This means that reflection must be taking
29. As, v = = = 2 ´ 10 5 Hz
l 1500 place from the fixed end hence the reflected ray must suffer
l
30. Walls of auditorium should be good absorbers to provide an additional phase change of p or a path change of
0.16 V 2
optimum value of reverberation time, T = , where V is
S as So, if y incident = a cos (kx - wt )
volume of hall and S as is total absorption of the hall. Þ y reflected = a cos ( -kx - wt + p ) = - a cos ( wt + kx )
3 3 2.5
31. As, C rms = v = 330 ´ » 471.4 ms-1 37. Here, m = kg/m, T = 200 N
g 1.4 20
T 200
I1 4 a2 v= =
32. As, = = (where I1 and l2 are the intensities) m 2.5/ 20
I2 1 b 2
a 2 200 ´ 200
\ = v=
b 1 25
Imax ( a + b) 2 200
\ = = = 40 m/s
Imin ( a - b) 2 5
l 20
(2 + 1) 2 Time taken, t= = = 0.5s
= =9 v 40
(2 - 1) 2
l
I I 38. Distance between two consecutive node is
Now, L1 - L2 = 10 log max - 10 log min 2
I0 I0 l 2
\ = m = 1m
I 2 2
= 10 log max = 10 log 9
Imin So, the distance of another node from the surface will be
\ L1 - L2 = 10 log 3 2 l
3 + = 3 +1= 4m
2
= 20 log3
Telegram @unacademyplusdiscounts

Waves 701

r1 1 1 1
39. Here, r1 = r2 ; = , T1 = T2 49. As n µ or l µ
r2 2 l n
1 T1 and l = l1 + l2 + l3
Þ n1 = 1 1 1 1
2lr1 p r1 Þ = + +
n n1 n2 n3
1 T2 n r
and n2 = \ 1 = 1 =2
2lr2 p r2 n2 r2 50. For an organ pipe open at one end,
3v
l T frequency of 1st overtone, n1 =
40. From, n = 4 l1
lD pr
For the organ pipe open at both ends,
When radius of string is doubled, Diameter D becomes twice. 3v
As Tand r are same , n becomes 1 / 2, i. e., n / 2. frequency of 3rd harmonic, n2 =
2 l2
41. There is no transfer of heat from compression to rarefaction as As n1 = n2
air is a bad conductor of heat. And time of compression
3v 3v
rarefaction is too small. \ =
4 l1 2 l2
42. According to the law of length, l1 2 1
n1l1 = n2 l2 or = =
l2 4 2
n l 800 ´ 50
\ l2 = 1 1 = = 40 cm
n2 1000 51. Here, p1 = 3, T1 = 8, p2 = 2, T2 = ?
n2 T 81 9 T2 p12
43. As, = 2 = = As =
n1 T1 100 10 T1 p22
(n1 - n2) p12 9
\ ´ 100 = 10 % \ T2 = 2
´ T1 = ´ 8 = 18 g
n1 p2 4
2 52. Both the ends will behave as nodes. In the nth mode of
44. Time taken for two syllables t = s
5 vibration,
2 æ lö 2l
\ x + x = v ´ t = 330 ´ \ nç ÷ = l \ l =
5 è2ø n
Þ x = 66 m
53. As, p T = constant
45. As the wire is forced to have a frequency= 512 = 2 ´ 256 = 2n,
therefore, it must vibrate in two segments. T2 p12 4 2
\ = =
T1 p22 6 2
46. Z1 and Z2 are displacements of two waves of same frequency 16
travelling in opposite directions.They will form a stationary T2 = T1
36
wave.
16
= ´ 65 = 29
47. Compare the given equation with the standard form of 36
stationary wave equation \ Weight to be removed = 65 - 29 = 36 kg-wt
2 px 2pvt
y = 2r sin cos 54. On reflection from a denser medium, there is a phase reversal
l l
2px 2pvt of 180°.
we get, = \ l = 3 cm 2
l 3 New amplitude = ´ 0.6 = 0.4
l 3
\Separation between two adjacent nodes = = 1.5 cm
2 \Equation of reflected wave is
é x ù
48. As, n1 : n2 : n3 = 1: 2 : 3 y = 0.4 sin 2p êt + + 180° ú
ë 2 û
1 1 1
\ l1 : l2 : l3 = : : = 6 : 3 : 2 = - 0.4 sin 2p (t + x/ 2)
1 2 3
sum of the ratios = 6 + 3 + 2 = 11 55. As fixed end is a node, therefore, distance between two
l
110 consecutive nodes = = 10 cm
\ l1 = ´ 6 = 60 cm, 2
11
l = 20 cm = 0.2 m
110
l2 = ´ 3 = 30 cm Now, v =nl
11
Thus, wedges should be placed from A at 60 cm and 90 cm. \ v = 100 ´ 0.2 = 20 ms-1
Telegram @unacademyplusdiscounts

702 JEE Main Physics

56. Frequency of third harmonic of closed pipe 60. Length at which first resonance occurs
3v l1 = 25.5 cm = 25.5 ´ 10 -2 cm
n1 =
4l
Length at which second resonance occurs l2 = 79.3 m
Fundemental frequency of open pipe \ wavelength l = 2( l2 - l1)
v = 2(79.3 - 25.5) = 2 ´ 53.8
n2 =
2l
= 107.6 cm = 1076
. m
As n1 - n2 = 100 Using v = nl, n = 340 Hz
v
Þ = 100 \ Speed of sound in air v = 340(107.6) ´ 10 -2
4l
= 365.84 m/s
Fundamental frequency,
v 61. As is clear from figure,
= 200 Hz
2l

57. As is clear from figure of question, 2nd overtone


l = λ1/4
lp v v
l= , l p = 4 l, np = =
4 lp 4 l 1st overtone
l = 5λ3/4
lq v v
l= , l q = 2 l, nq = =
2 lq 2 l
l = 3λ3/4
v v l1
l = lr, l r = 1,nr = = l= , l1 = 4 l
lr l 4
3ls 4l v 3v 3l 4l
l= , l s = , ns = = l = 2, l2 =
4 3 ls 4 l 4 3
3 ls 4l 5l3 4l
l= , ls = and l= ,l3 =
4 3 4 5
1 1
v 3v \ l1 : l 2 : l3 = 1: :
hs = = 3 5
ls 4 l
v 3v
v v v 3v 62. For closed pipe l1 = , l2 =
\ np : nq : nr : ns = : : : = 1 : 2 : 4 :3 4n 4n
4l 2l l 4l
v 330
l Þ n= = = 500 Hz
58. As, l1 + x = = 22.7 2 ( l2 - l1) 2 ´ (0.49 - 0.16)
4
3l 100
l2 + x = = 70.2 63. When one end is closed, n1 = = 50 Hz
4 2
5l n2 = 3 n1 = 150 Hz
and l3 + x =
4 and n3 = 5 n = 250 Hz
l2 - 3 l1
\ x= v
64. Fundamental frequency of open2 organ pipe =
70.2 - 68.1 2.1 2l
= = = 1.05 cm 3v
2 2 Frequency of third harmonic of closed pipe =
4l
l3 + x
Now, =5 3v v
l1 + x \From question, = 100 +
4l 2l
Þ l3 = 5 l1 + 4x = 5 ´ 22.7+ 4 ´ 1.05 = 117.7 cm 3v 2v v
Þ - = = 100
v 4l 4l 4l
59. As, nopen =
2lopen v
= 200 Hz
v v v 2l
and nclosed = = =
4lclosed 4lopen /2 2lopen l2 l2'
65. As =
æ lopen ö 2 l l1'
As ç lclosed = ÷ 60 l'
è 2 ø \ = 2
40 50
i. e. , frequency remain unchanged. 50 ´ 60
l2 = = 75 cm
40
Telegram @unacademyplusdiscounts

Waves 703

66. v = 2 n ( l2 - l1) = 2 ´ 325 (77.4 - 25.4) cms-1 Now, from Eq. (i),
650 ´ 52 (i) v = 45 ´ 2 ´ 0.875 = 78.75 m/s [Q l = 2l]
= ms-1 = 338 ms-1
100 T
(ii) As v =
v 3v l 2 1 m
67. As = \ 1 = =
4l1 2l2 l2 12 6 Þ T = v 2(m) = (78.75) 2 ´ 4.0 ´ 10 -2

68. Octave stands for an interval 2 :1. Three octaves will have a = 248.06 N
3
frequency ratio = 2 = 8 74. In n is frequency of first fork, then frequency of the last (10th
fork) = n + 4 (10 - 1) = 2 n
69. Fundamental frequency
v 330 \ n = 36 and 2 n = 72
n= = = 660 Hz
2 l 2 ´ 0.25 75. Here, n = 200 ± 5 and 2 n = 420 ± 10. This is possible only
Frequency of overtones are 2n,3n, 4n,…= 1320,1980 2640 Hz. when n = 200 + 5 = 205
n1 l2 25 5
70. In a closed pipe, resonance frequency, n = (2r -1) v / 4l 76. = = =
n2 l1 30 6
= 135 and 165.
and n2 - n1 = 4
The lowest frequency must be highest common factor of
On solving, we get n2 = 24 Hz
125 and 165, which is 15 Hz.
n1 = 20 Hz
71. Here, T1 = 16 N, T2 = T = ?
77. Beat frequency = number of beats per second = n1 - n2
As per the choice given, T2 > T1
and maximum loudness = ( 2a + 2a) 2 = 4a2 = 4 I
\ n2 > n1 and given that (n2 - n1) = 3 …(i)
1/ 2
Now nµ T n2 101 æ 1 ö 1
78. = = ç1 + ÷ = 1+
n2 T T n1 100 è 100 ø 200
\ = =
n1 16 4 (From Binomial Expansion)
If n1 corresponds to 4; then n2 corresponds to 3 + 4 = 7, n
n2 = n1 + 1
which is T . Therefore, T = 49 N 200
n 200
Number of beats s-1 = n2 - n1 = 1 = =1
72. Here, n1 = 200 Hz 200 200
Number of beats s-1; m = 4 79. Number of beats s-1 = n1 - n2 =
v
-
v
\ n2 = 200 ± 4 = 204 or 196 Hz 3 l 4( l + Dl)
On loading fork 2, its frequency decreases. And number of v é l + Dl = l ù v Dl
= =
beats per second increases to 6. Therefore, m is negative. 1 êë l( l + Dl) úû 4 l 2
n2 = 200 - 4 = 196 Hz
80. Frequency of unknown fork = 256 ± 4 = 260 or 252. As
73. A
frequency decreases on loading, therefore, original frequency
of unknown fork = 260 Hz.
81. Number of beats per second = n1 - n2
N N
\Time interval between two successive beats/successive
1
λ/2 maxima =
n1 - n2
e
In fundamental mode of vibration, 82. Here, n1 = 480 ,m = 10
l \ n2 = n1 ± m = 480 ± 10 = 490 or 470.
= l or l = 2l
2 when tension is increased, n2 will increase (\ n2 µ T ).
v = nl …(i) As number of beats s -1
decrease, n2 = 470 Hz
n = 45 Hz
n1 l2 51
m 83. As, = =
As m = = 4 ´ 10 -2 kg/m n2 l1 50
l
m = 3.5 ´ 10 -2 kg and n1 - n2 = 5
On solving, we get
m 3.5 ´ 10 -2
\ l= = = 0 .875 m n2 = 250 ,n1 = 255
m 4 ´ 10 -2
Telegram @unacademyplusdiscounts

704 JEE Main Physics

84. Two possible frequencies of source are = 100 ± 5 = 105 or 91. Here bat is a source of sound and the wall is observer at rest
95 Hz vf
\ frequency of sound reaching the wall is f ¢ =
Frequencies of 2nd harmonic = 210 or 190 Hz v - vS
5 beats with source of frequency 205 Hz are possible only where v is the velocity of sound in the air and VS is the
when 2nd harmonic has frequency = 210 Hz velocity of source.
\Frequency of source = 105 Hz On reflection the wall is the source of sound of frequency f ¢ at
T 8 l 36 D 4 rest and bat is an observer approaching the wall
85. Here, 1 = , 1 = , 1 =
T2 1 l2 35 D2 1 \ frequency heard by the bat is
r1 1 f ¢(v + v 0) (v + v 0)
= f ¢¢ = =f [using Eq. (i)]
r2 2 v (v - v s )
and n1 = 360 Hz, n2 = ? æ 330 + 4 ö
= 90 ´ 10 3 ç ÷
è 330 - 4 ø
n2 l1D1 T2 r1
Now, =
n1 l2D2 r 2T1 90 ´ 10 3 ´ 334
=
326
n2 36 4 1 1 36
= ´ ´ = = 92.1 ´ 10 3 Hz
n1 35 1 8 2 35
Clearly n2 > n1.
92. As source and observer both are moving in the same direction
with the same velocity, their relative velocity is zero.
When n2 = 360 Hz; n1 = 350 Hz Therefore, n ¢ = n = 200 Hz.
Number of beats per second = n2 - n1 = 360 - 350 = 10
93. From Doppler’s effect,
u 2u
86. Number of extra waves received per second= - ( -u / l) = æ v - v0 ö æ 340 - 10 ö
l l n¢ = nç ÷ = nç ÷ = 1950 (from question)
è v + v sø
è 340 + 10 ø
87. n = 165 Hz, and
n = 2068 Hz
335 + 5 335
n¢ = ´ ´ 165 = 170 Hz
335 330 94. Large vertical plane acts as listener moving per second.
,
\Number of beats per second = n - n = 170 - 165 = 5 ( c + v)n
\ n¢ =
c
88. Whistling train is the source of sound, v s = V . Before crossing This is the number of waves striking the surface per second.
a stationary observer on station, frequency heard is
vn vn 95. As source is moving towards observer,
n¢ = = = constant and n¢ > n.
(v - v s ) v - V uv 333 ´ 450
\ v¢ = = = 400.5 » 500
u - vs 333 - 30
Here, v is velocity of sound in air and n is actual frequency of
whistle. 96. Here, v s1 = 34 ms -1,
After crossing the stationary observer, frequency heard is v = 340 ms -1
vn vn
n¢ = = = constant and n¢ < n v ´n 340 ´ n 340
(v + v s ) v + V f1 = = = n
v - v s1 340 - 34 306
Therefore, the expected curve is (c). v ´n 340 ´ n 340n
f2 = = =
1 Dn v - v s 2 (340 - 17) 323
89. As n= c
2 n f1 323 19
1 Dn = =
\ 0.2c = c f2 306 18
2 ( 4 ´ 10 7)
Dn = 1.6 ´ 10 7 Hz
97. Let n be the actual frequency of sound of horn.
As the rocket is receding away If v s is velocity of car, then frequency of sound striking the
7 7 cliff (source moving towards listener)
\ n ¢ = n - Dn = 4 ´ 10 - 1.6 ´ 10
(v + v s )n ¢ (v + v s ) v ´ n
= 2.4 ´ 10 7 Hz n¢ = = ´
v v (v - v s )
90. No beat is heard, because frequency received by listener n ¢¢ v + v s
or = =2
directly from the source and that received on reflection from n v - vs
256 ´ 330
the wall is same i. e. , = Hz v + vs = 2 v - 2 vs
330 - 5 v
3v s = v ,v s =
3
Telegram @unacademyplusdiscounts

Waves 705

98. Here, us = 50 ms-1,vL = 0 ,v = 350 ms-1 When source is moving away from observer, the apparent
frequency
When source is moving towards observer, u ´v 350 6000
frequency n¢ = 1000 v¢ = = ´
u + v s (350 + 50) 7
u ´v
n¢ = = 750 Hz
u - us
u ´v
(u - us )v ¢ 99.From Doppler’s effect, v ¢ =
\ n¢ = u - us
u
(350 - 50)1000 6000 330 ´ 500
= = Hz = = 550 H
350 7 330 - 30

Round II
1. As, energy E µ (amplitude) 2(frequency) 2 v v
4. nc = and n0 =
4l 2l
Amplitude is same in both the cases; but frequency 2w in the
Now, n0 - nc = 2
second case is two times the frequency ( w) in the first case.
v v
Hence E 2 = 4E1 \ - =2
2l 4l
2. Frequency of 1st overtone of A v
or =8
2 T 2 T l
n1 = = v v
2l1 m l1 D1 pr Also n0¢ = =
2l / 2 l
Frequency of 2nd overtone of B; v v
and nc¢ = =
3 T 2 T 4 (2l) 8 l
n2 = =
2 l2 m l2 D2 pr Number of beats per second = n0¢ - nc¢
As n1 = n2 (resonance condition) v v 7v
= - =
2 T 2 T l 8l 8l
\ = =
l1D1 m l1D2 pr 7
= ´8 = 7
l1D1 2 8
=
l1D2 3 5. Let speed of observer be vL = v along Y-axis and speed of
l1 2D2 2 l source the v s = 2vL = 2v along X-axis
= = or 1 = 1: 3
l2 D2 3 l2 Y
vL = v
3. As, it is clear from figure, at t = 0, x = 0 , displacement y = 0.
Therefore, options (a) or (d) may be correct. In case of (d); P
β
y = A sin (kx - wt )
dy O θ S
= A cos(kx - wt ) [ - w] X
vs = 2 v L
dt
(after differentiating w.r.t. t ) \ PS = 2 (OL)
2
dy cos a =
and = A cos(kx - wt ) [k] 5
dx
2
(after differentiating w.r.t. x ) and cos b =
5
dy
dt = - wA cos (kx - wt ) = - w = -v Now, apparent frequency n ¢ is given by
\ (v - vL cos b)n
dy / dx kA cos (kx - wt ) k n¢ =
dy æ dy ö (v + vL cos a)
\ = -v ç ÷
dt è dx ø where v is velocity of sound.
i. e. , particle velocity = - (wave speed) ´ slope (v - v 5)n
n¢ =
And slope at x = 0 and t = 0 is positive, in figure. Therefore, (v + 4v 5)
particle velocity is in negative y- direction. Clearly, n ¢ is constant but n ¢ < n. This is shown in curve (b).
Telegram @unacademyplusdiscounts

706 JEE Main Physics

6. When O is a fixed end, the formation of reflected pulse is mass mass


m= = ´ area = ra
equivalent to overlapping of two inverted pulses travelling in length volume
opposite direction as shown in figure. T
\ v2 =
3 cm ra
t=0
v1 Y ra Ya
3 cm = . =
v2 r T T
F T
t = 3s As Y= =
aDl / l a( Dl / l)
-1/ 2
Hence, at t = 3s, net displacement of all particles of the string v1 T a æ Dl ö
Þ = =ç ÷
will be zero i. e. , the string will as shown in figure. v2 æ Dl ö T è l ø
aç ÷
è l ø
7. As the string is vibrating in three segments, therefore,
3l Dl 1
l= We are given, =
2 l n
-1/ 2
2l 2(0.6) v1 æ 1 ö
or l= = = 0.4 m \ =ç ÷ = n
3 3 v2 è n ø
T If f1, f2 are the corresponding fundamental frequencies of
Now, v=
m longitudinal and transverse vibrations, then
80 v1 = f1 l
\ v= = 20 ms-1
2.0 and v 2 = f2 l
v 20 v1 f1
n= = = 50 Hz \ = = n
l 0.4 v 2 f2
Amplitude of particle velocity
11. Let m be the total mass of the rope of length l. Tension in the
æ dy ö rope at a height h from lower end = weight of rope of length h
=ç ÷ = ( amax ) w = amax (2p n)
è dt ø max mg
i. e. , T= (h)
= (0.5 ´ 10 -2) ´ 2p ´ 50 = 1.57 ms-1 l
T
8. As the string vibrates in n loops therefore, As v=
(m / l)
nl
l=
2 mg (h)
\ v= = gh
1 l(m / l)
therefore, v would become time.
2 v 2 = gh
As vµ T
which is a parabola. Therefore, h versus v graph is a parabola
1 option (a) is correct.
Therefore, to make v half -time , T must be made time i. e. ,
4
M / 4. 12. As is known, frequency of vibration of a stretched string
9. Let m = mass per unit length of rope n µ T µ mg µ g

T = tension in the rope at a distance xfrom the lower end 80


As nw = na = 0.8 na
\ T = (mg ) x = weight of x metre of rope 100
2
T mgx g ¢ æ nw ö
As v = \ v= = gx \ = ç ÷ = (0.8) 2 = 0.64
m m g è na ø
i. e. , vµ x
If, r w = relative density of water (= 1)
10. Velocity of longitudinal waves, r m = relative density of mass
Y r t = relative density of liquid, then
v1 =
r
g ¢ æ rw ö
and velocity of transverse waves = ç1 - ÷ = 0.64
g çè rm ÷ø
T
v2 =
m rw
= 1 - 0.64 = 0.36 …(i)
If a is area of cross-section of string, then rm
Telegram @unacademyplusdiscounts

Waves 707

Similarly, in the liquid 17. Proceeding as in above question,


2
g ¢ æ nL ö vp
= ç ÷ = (0.6) 2 = 0.36 tan q = tan 60° =
g è na ø v
g ¢ æ rL ö \ v p = v ´ tan 60 º = v 3
= ç1 - ÷ = 0.36
g è rm ø Y T T
18. As, vL = and vT = =
rL r m pr 2r
= 1 - 0.36 = 0.64 …(ii)
rm
vL Y pr 2r Y Y
Dividing Eq. (i) by Eq. (ii), we get = ´ = =
vT r T T / pr 2 stress
rL 0.64
= = 1.77 Y 1 ´ 10 11
r w 0.34 \ Stress = = = 10 7Nm-2
(vL / vT ) 2 (100) 2
Hence, specific gravity of liquid = 1.77
VO r O + VH2r H2 19. For wire, if
13. Density of mixture = r mix = 2 2 M = Mass, r = density, A = Area of cross-section
VO2 + VH2
V = volume, l = length, Dl = change in length
V (r O2 + r H2 ) r O2 + r H2
= = [ since VO2 = VH2 = V ] M Alr
2v 2 The mass per unit length m = = = Ar
l l
r H2 + 16 r H2 )
= 8.5 r H2 T/A
2 And Young's modulus of elasticity = Y =
Dl / l
1
Þ As Vµ YDlA
r T=
l
Vmix r H2 r H2 2 Hence, lowest frequency of vibration
= = =
VH2 r mix 8.5 r H2 17 æ Dl ö
gç ÷A
p 1 T 1 è l ø
14. As, y = 5 sin (100t - x ) n= =
2 2l M 2l Ar
æ100 p p ö 1 gDl
y = 5 sin ç t- x÷ =
è 2 2 ø 2l lp
æ p ö 1 9 ´ 10 10 ´ 4.9 ´ 10 -4
y = 5 sin ç50 pt - x÷ n= = 35 Hz
è 2 ø 2 ´1 1 ´ 9 ´ 10 3
The general equation y = a sin ( wt - kx)
20. As, y = 4 cos2(t / 2) sin (1000t )
On comparing w = 50 p
2p 2p 1 = 2 [2 cos2(t / 2) sin (1000t )]
T= = =
w 50 p 25 = 2 [2(1 cos t ) sin (1000t )]
or T = 0.04 s = 2 sin 1000t + 2 sin 1000t cos t
70 22 y = 2 sin 1000t + sin (1001)t + sin (999t )
15. As, v s = rw = r ´ 2pn = ´2 ´ ´ 5 = 22 ms-1
100 7 \ The given wave equation represents the super position of
Frequency is minimum when source is moving away from three waves.
listenery.
21. In a wave equation, x and t must be related in the form ( x - vt).
Therefore from Doppler’s effect, Therefore, we rewrite the given equation as
u ´ v 352 ´ 1000 1
n' = = = 941Hz y=
u + us 352 + 22 1 + ( x - vt ) 2
16. Let the frequency of standard fork = x 1
For t = 0, it becomes Y =
102 97 1 + x2
\ nA = x and nB = x
100 100 and for t = 2, it becomes
Number of beats per second = nA - nB = 6 1 1
y= =
102 97 [1 + ( x - 2v) ] 1 + ( x - 1) 2
2
x- x=6
100 100 \ 2v = 1
6 ´ 100 or v = 0.5 ms-1
x= = 120 Hz
5
Telegram @unacademyplusdiscounts

708 JEE Main Physics

22. When source is moving towards observer, then apparent For rotational equilibrium, net torque should be equal to
frequency zero
vn Þ T1 x = T2 (L - x )
n¢ =
v + vs L
x=
5
When source is moving away from observer, then apparent
frequency 25. SupposeIi andIr are intensities of incident and reflected waves.
vn 2
n ¢¢ = Ir æ m - 1ö
v - vs Reflection coefficient = =ç ÷
Ii è m + 1ø
Now, change in frequency,
é v + v s - v + v s ù (2v s ) vm v1 T / m1 m2
where , m= = =
n ¢ - n ¢¢ = vn ê ú= 2 2 v2 T / m2 m1
ë v 2 - v s2 û v - vs
2 v sn 25 5
When v S,n ¢ - n ¢¢ = = =
v 9 3
2
n ¢ - n ¢¢ 2 2v 2 vs æ 5 / 3 - 1ö 1
Now, = = s = \ Reflection coefficient = ç ÷ =
n 100 v 300 è 5 / 3 + 1ø 16
\ v s = 3 ms-1
26. Here,A1 = A2; n1 = w, n2 = w2
23. The motorist receives two sound waves, one direct from the \ y1 = A sin 2 pw1t ,
band and second reflected from the wall which is shown is
figure. For direct sound waves, apparent frequency and y 2 = A sin 2 pw2 t
(v + v m) f Now, y = y1 + y 2 (in case of superposition)
f¢ = cos 2p ( w2 - w1) t 2p ( w2 + w1) t
v + vb = 2A sin
2 2
= A¢ sin p ( w2 + w1) t
Motorist Band Master
(Listener) (Source) Wall where, A¢ = 2A cos p ( w2 - w1)t
vm vb
Sound heard will be of maximum intensity ( > 2A2)
when cos p ( w2 - w1) t = max = ±1
For reflected sound waves, p ( w2 - w1)t = 0 , p, 2p
Frequency of sound wave, reflected from the wall 1 2
t = 0, ; ;…
v´f w2 - w1 w2 - w1
f ¢¢ =
v - vb Time interval between two successive maxima
Frequency of reflected waves as received by the moving 1 2
= = = 10 -3 s
motorist, w2 - w1 10 3
(v + v m) f ¢¢ (v + v m) f 27. Resultant displacement along X-axis is x = y1 - y3 = 8 - 2 = 6
f ¢¢ = =
v v - vb
Resultant displacement along Y-axis is y = y 2 - y 4 = 4 - 1 = 3
\ Beat frequency = f ¢¢¢ - f ¢
Net displacement,
(v + v m) f (v + v m) f 2 v b(v + v m) f
= - =
v - vb v + vb v 2 - v b2 r = x2 + y 2 = 6 2 + 3 2 = 45
y 3 1
24. According to the question Also, tan q = = =
x 6 2
1 T1 1 T2 q = tan -1(1 / 2)
=
2l m l m vp vn
28. As, n¢ = and n ¢¢ =
T2 = T1 / 4 v - vs v + vs
A D n v
\ = 1- s ,
n¢ v
T1 T2 n vs
= 1+
O
n ¢¢ v
B C Adding the two, we get
x
L n n
+ =2
M n ¢ n ¢¢
2n ¢n ¢¢
\ n=
n ¢ + n ¢¢
Telegram @unacademyplusdiscounts

Waves 709

29. When the stone is suspended in air 32. Equation of stationary wave is
1 Wa y1 = a sin kx cos wt ,
n=
2L m and equation of progressive wave is
When the stone is suspended in water, y 2 = a sin ( wt - kx)
1 Ww = a (sin wt cos kx - cos wt sin kx)
n=
2L ¢ m p 3p
At x1 = and x2 =
Wa Ww 3k 2k
\ = sin kx1 or sin kx2 is zero.
L L¢
Wa L 2 so, neither x1 nor x2 is node.
or = 3p p 7p
Ww L ¢2 Dx = x2 - x1 = = =
2k 3k 6k
Specific gravity of stone
7p
Wa 1 1 As Dx = ,
= = = 6x
Wa - Ww 1 - Ww L ¢2
1- 2 2p p
Wa L Therefore, > Dx >
k k
L2 ( 40) 2
= 2 2= 2p
L - L ¢ ( 40) 2 - (22) 2 But =l
k
30. When aeroplane is at P2 vertically above the observer O, l
So, l > Dx >
sound comes along P1 O at 60° with the vertical. 2
(vs) t In case of a stationary wave, phase difference between any
P1 P2 two points is either zero or p.
\ f1 = p
7p 7
and f 2 = kD x = k= p
vt
6k 6
f1 p 6
60°
\ = =
f2 7 p 7
6
O 33. As intensity µ a2w2
\ P1O = v ´ t , aA 2 w 1
here, = and A =
P1 P2 = v pt aB 7 wB 2
2 2
P1 P2 v pt vp iA æ 2 ö æ 1 ö 1
sin 60° = = = Þ = ç ÷ ´ç ÷ =
P1O v ´ t v iB è 1 ø è 2 ø 1
\ v p = v sin 60° = v 3 /2 34. If a is amplitude of each wave,
31. Here, y1 = 0.05sin(3pt - 2x ) I0 = k( 2a + 2a) 2 = 4ka2

y 2 = 0.05sin(3pt + 2x ) Let f be the phase difference to obtain intensity I0 / 2


I0
According to superposition principle, the resultant \ = kar2 = k( a2 + a2 + 2aa cos f)
2
displacements is
f
y = y1 + y 2 = k 2a2(1 + cos f) = k4a2 cos2
2
= 0.05[sin (3pt - 2x)+ sin (3pt +2x)]
= I0 cos2 f/2
y = 0.05 ´ 2 sin 3pt cos 2x
f 1
y = (0.1cos 2x) sin 3pt = R sin 3pt \ cos = = cos 45°
2 2
where, R = 0.1 cos 2x = amplitude of the resultant standing
wave. \ f = 90°
At x = 0.5 m If Dx is path difference between the two waves, then
R = 0.1cos 2x = 0.1 cos 2 ´ 0.5 l l æpö l
Dx = f= ç ÷=
180° 2p 2p è 2 ø 4
= 0.1cos 1(radian) = 0.1 cos 1
p Therefore, displacement of sliding tube ( Dx) = l / 8
= 0.1 cos 57.3° 2
or R = 0.1 ´ 0.54 m = 0.054 m = 5.4 cm
Telegram @unacademyplusdiscounts

710 JEE Main Physics

35. Let, v be the speed of sound in air, vL velocity of observer at 4


i. e. , A2 - x2 has become times
time t. As, the observer approaches the source, therefore, 5
apparent frequency A 4
Therefore, velocity at x = may be ± v or kinetic energy
(v + vL) é v + (0 + at ) ù æ f0 at ö 2 5
f= f0 = ê úû f0 = f0 + çè v ÷ø
v ë v 4
will become times or 0.8 times.
5
This is the equation of a straight line with a positive intercept
p
æ f aö
( f0) and positive slope ç 0 ÷ . Therefore, option (d) is correct. 43. The given equation is y ( x, t) = 3.0 sin éê36 t + 0.018x + ùú
è v ø ë 4û

36. vT µ strain (a) As positive direction is from left to right and x is positive,
therefore, wave is travelling from right to left.
v0 1 ´ 15
= (b) Compare the given equation with the standard from
vT 30 ´ 1
é 2 pt 2 px ù
vT = 2 v 0 y = r sin ê + + fú
ë T l û
37. Sound waves in air are longitudinal while light transverse. 2p
= 36,
T
38. The time taken by the plate falling through a distance y is 2p
given by = 0.018
l
æ 2 ´ 10 ö æ 1 ö l 36
t = (2yg ) = ç ÷ =ç ÷s
è 980 ø è 7 ø Speed of wave, v = = = 2000 cm/s = 20 m/s
T 0.018
1 2p p
The number of oscillations completed s is 8. (c) Again, T = =
7 36 18
\Frequency = number of oscillations completed in one sec 1 18
Frequency v = = Hz = 5.7Hz
8 T p
= = 56 Hz
1/ 7
44. As the string is rigidly clamped at its two ends, therefore, y = 0
39. Displacement and amplitude both, are added vectorially in at x = 0. This can be satisfied only by the term
superposition principle. np x
sin = ,
40. Compare the given equation with the standard equation L
y = a sin( wt - x ), we get where m is an integer.
-1
v = 200 ms Therefore, options (a) and (b) correct.

41. In the given equation as x is positive, therefore, the wave is 45. A travelling wave is of the form F ( ax ± bt). Therefore, choice
travelling along negative direction of x-axis in which, (a), (b), (c) are correct.
2p 2p 2 px ö
= 10 p , l = = 0.2m 46. (a) The given equation is y ( x, t) = 0.06 sin æç ÷ cos (120 pt )
l 10 p è 3 ø
2p 2p 2
and = 15p , T = = s (b) As terms involving x and t are independent of each other,
T 15p 15 the given equation represents a stationary wave.
l 0.2
v= = = 1.5 ms-1 (c) Compare the given equation with the standard form of
T 2 /15
equation of stationary wave
42. Force increases linearly therefore, force acting on the particle y ( x, t ) = 2r sin kx cos wt
A
at x = will be –2F. Potential energy U µ x2 2p 2p
2 k= =
l 3
A
i. e. , potential energy at x = will become 4U. \ l = 3m
2
Speed of particle is given by w = 120 p
2 2 w 120 p
v = w A -x \ v= = = 60 Hz
2 2
2p 2p
i. e. , v µ A -x
and v = vl = 60 ´ 3 = 180 m/s
A 15
at x=- , A2 - x2 = A Hence the given stationary wave is the result of superposition
4 16 of two waves of wavelength 3 m and frequency 60 Hz each,
A 3 travelling with a velocity of 180 m/s in opposite directions.
and at x= , A2 - x2 = A
2 4
Telegram @unacademyplusdiscounts

Waves 711

47. According to Newton’s formula for velocity of sound in a 53. Here, n = 400Hz, v = 340 m/s, w = 10 m/s
fluid,
(a) The frequency of sound as heard by an observer standing
Ba on platform = 400 Hz only, as it is not affected by motion
v=
r of wind alone.
v µ Ba (b) Speed of sound for observer standing on platform,
1 v ¢ + w = 340 + 10 = 350 m/s.
and vµ
r 54. Here, w1 = 100 p and w2 = 92p ,
Choices (c) and (d) are correct. 100 p
Hence n1 =
2p
48. Compare the given equation 92p
æp pt ö = 50 Hz and n 2 == 46 Hz
y = 20 sinç x + ÷ 2p
è4 2ø
\ Number of beats per second = n1 - n 2 = 50 - 46 = 4
with the standard form w 100 p
55. Wave velocity n = 1 = = 200 ms-1
æ 2p 2 pt ö k2 0.5p
y = a sinç x+ ÷ , we get
è l T ø
56. At x = 0 , y = y1 + y 2
\ a = 20
2p p = A cos (0.5 px - 100 pt ) + A cos (0.46 px - 2 pt )
= ,l =8 = A cos100 pt + A cos 92 pt = 2 A cos 96 pt cos 4 pt
l 4
2p p y = 0 when either cos 96 pt = 0 or cos 4 pt = 0
= ,T = 4 p p
T 2 Þ 96pt = (2n + 1) and 4pt = (2m + 1) ,
1 1 2 2
n = = = 0.25
T 4 where n and m are integers.

49. In first normal mode of vibration For 0 < t < 1, n can have 96 integer values and m can have 4
v v 330 ´ 100 integer values. Hence net amplitude becomes zero
n= , l= = = 31.25 cm 96 + 4 = 100 times
4l 4n 4 ´ 264
In second normal mode of vibration, 58. When the source moves at 90°to the line joining the source
3v 3v and the listener, apparent frequency remains unaffected.
n= , l= = 3 ´ 31.25 = 93.75 cm
4l 4n gp
60. The correct formula for velocity of sound in a gas is v =
sin i v cooler T 273 + 27 3 r
50. m = = = 1 = =
sin r v hotter T2 273 + 127 4 For monoatomic gas, g = 1.67
4 4 4 1 1 For diatomic gas, g = 1.40
\ sin r = ´ sin i = sin 30° = ´ =
3 3 3 2 3 \ v is larger in case of monoatomic gas compared to its value
-1 in diatomic gas.
\ r = sin (1 / 3)
v 350
2 px ö 61. As, l = = = 0.7m
51. The given equation is y ( x, t) = 0.06 sin æç ÷ cos (120 pt ) n 500
è 3 ø p p
f = 60° = 60 ´ = rad
It represents a stationary wave. Therefore, all the points 180 3
between two consecutive nodes. l
As, x= f
(a) vibrate with same frequency 2p
0.7 é 60 p ù
(b) in same phase, but \ x= ´ = 0.12 m = 12 cm
2p êë 180 úû
(d) different amplitudes. The amplitude is zero at nodes and
maximum at antinodes (between the nodes). 62. As lv < lr
\ Violet shift means apparent wavelength of light from a star
52. When the sources are coherent,
2 2 2
decreases. Obviously, apparent frequency increases. This
R = a + b + 2ab cos f would happen when the star is approaching the earth.
For constructive interference, f = 0 64. Let n be the frequency of fork.
\ I = I0 + I0 + 2 I0I0 cos 0° = 4I0 n1 - n = 4
When the sources are incoherent, intensities just add and n - n2 = 4 ...(i)
I = I0 + I0 = 2I0 \ n1 - n2 = 8 ...(ii)
Telegram @unacademyplusdiscounts

712 JEE Main Physics

n1 l2 50 70. The frequency of reflected sound heard by the girl,


Also, = =
n2 l1 49
é v - ( -v 0) ù é v + v0 ù
50 n¢ = nê ú = nê ú
\ n1` = n2 ë v - v 0 û ë v - v0 û
49
50 1 é 340 + 20 ù
From Eq. (ii), n2 - n2 = 8, n2 = 8 = 480 ê = 540 Hz
49 49 ë 340 - 20 úû
n2 = 49 ´ 8 = 392. v
71. fclosed =
From Eq. (i), n = 4 + n2 = 4 + 392 = 396 Hz 2lopen
65. The equation of stationary waves is v v é lopen ù
px fclosed = = ê As lclosed = ú
y = 20 sin cos wt 4lclosed 4lopen / 2 ë 2 û
4
v
Compare with y = 2a sin kx cos wt = = fopen
p 2fopen
k=
4 72. Frequency of sound reaching the hill
2p
As l= v ´ n 330 ´ 600
k n¢ = = = 660 Hz
2p v - vs 330 - 30
\ l= =8m
p/4 On reflection frequency of sound heard by driver,
Distance between two consecutive antinodes (v + vL)n ¢ (330 + 30)600
n ¢¢ = = = 720 Hz
l 8 v 330
= = =4m
2 2
73. As the listener on motor cycle is moving away from the
66. Equations show that phase difference between two waves n ¢ v - vL 94
source (siren), therefore = =
f = p /2 n v 100
vL 94
\ Resultant amplitudes R = a2 + b 2 + 2ab cos p / 2 Þ 1- =
v 100
= a2 + a2 + 2a2 cos 90° vL 94 6
Þ = 1- =
= 2 a2 = a 2 v 100 100
6 ´ v 6 ´ 330
67. When b = a, then from Þ vL = = = 19.8 ms-1
2 2
100 100
R = a + b + 2ab cos f, we get
vL2 19.8 ´ 19.8
a2 = a2 + a2 + 2a a cos f = 2a2(1 + cos f) Distance covered = =
2a 2 ´2
1 = 98 m
Þ 1 + cos f =
2
1 1 74. If we assume that all the three waves are in same phase at t = 0,
Þ cos f - 1 = - , f = 120° we shall hear only one beat per second.
2 2
v He g HeMoxygen
68. Number of beats per second n = n1 - n2 75. As, =
oxygen oxygen He v g M
v v 1 1
= - = vê - ú
l1 l 2 v He (5 / 3) ´ 32
ë 1 l2 û
l
=
é1 1 ù 460 (7 / 5) ´ 4
= 336.6 ê - ú =3
ë 1 1.01û 200
=
21
69. If the lenght of the wire between the two bridges is l, then the
frequency of vibration is 200
v He = 460
1 T 1 T 21
n= =
2l m 2 l pr 2d = 1420 ms–1

æ radius ö 76. Here, l1 = 18 cm


In the length and diameter ç = ÷ of the wire are doubled
è 2 ø
During summer, v increases, g increases.
keep in the tension same, then new fundamental frequency
n \ l1 increases.
will be .
4 As l2 > 3 l1
\ l2 > 54 cm
Telegram @unacademyplusdiscounts

Waves 713

77. Given, y ( x, t) = 0.005 cos ( ax - bt) 1 300 1 300


or = or =
2 T2 4 T2
Compare it with standard equation
é 2p 2p ù Þ T2 = 1200 K
y ( x, t ) = r cos ê x- t , we get
ë l T úû or T2 = (1200 - 273)° C = 927° C
2p 2p 81. Frequency of fork = 250 Hz.
a= = = 25.00 p
l 0.08 Possible frequencies of sonometer wire = (250 ± 10) Hz
2p 2p On filling the fork, number of beats per second decreases
b= = =p
T 2.0 \ Frequency of sonometer wire,n = 260 Hz
78. Given that, the displacement of a particle is v = nl = 260 (2 l) = 260 (2 ´ 0.5)
y = A sin ( wt = kx ) …(i) = 260 ms-1
The particle velocity v 330
82. As, l = = = 0.66 m = 66 cm
dy n 500
vp = …(ii)
dt The successive resonance lengths are at
Now, on differentiating Eq. (i) with respect to t, l 3l 5l 7l
, , , and so on.
dy 4 4 4 4
= A cos( wt - kx ) ´ w
dt Within one metre, length of the tube, total number to
dy 7l
Þ = Aw cos ( wt - kx ) resonances is 3(as is more than1.0 m).
dt 4
From Eq. (ii) 83. The apparent frequency of sound striking the wall,
Þ v p = Aw cos( wt - kx ) v ´n 335 ´ 165 335
n¢ = = = Hz
For maximum particle velocity, (v - v s ) (335 - 5) 2
cos ( wt - kx ) = 1 For reflected sound, wall is the source and passenger in the
So, v p = Aw ´ 1Þ v p = Aw bus is as listener.
(v + nL)n ¢
79. Given that, two waves \ n ¢¢ =
v
y = a sin ( wt - kx ) (335 + 5) 335
and y = a cos( wt - kx ) = ´ = 170 Hz
335 2
p \ Number of beats s-1 = n ¢¢ - n = 170 - 165 = 5
Here, the phase difference between the two waves is .
2
So, the resultant amplitude 84. Here, n = 200 Hz, v = 360 ms-1
v 360
A = a12 + a22 + 2a1a2 cos f \ l= = = 1.8 m
n 200
é pù
êëHere, a1 = a, a2 = a and f = 2 úû Distance between two consecutive antinodes
l 1.8
p = = = 0.9 m
\ A = a2 + a2 + 2as cos 2 2
2 v dl
85. From, = ; (where, dl = change in wavelength)
or A = a2 + a2 + 0 Þ A = 2 a c l
80. The speed of sound in air is defined as, dl
Þ v= ´c
gRT l
v= 0.32
M = ´ 2 ´ 10 8 ms-1
100
Þ vµ T
= 9.6 ´ 10 5 ms-1
v1 T
\ = 1 …(i)
v2 T2 As wavelength increases, apparent frequency decreases.
But according to question, The star must be moving away from rarth.
v1 = v ,v 2 = 2v , 86. Here, l1 = 80 cm, l2 = 70 cm
T1 = 27° C = (27 + 273) K = 300 K, n2 l1 80 8
\ = = =
On pulling these value in Eq. (i), we have n1 l2 70 7
v 300 If n is frequency of tuning fork, then
=
2v T2 n2 - n = 8
Telegram @unacademyplusdiscounts

714 JEE Main Physics

and n - n1 = 8 92. Musical interval produced between two notes of frequencies


\ n2 - n1 = 16 320
is given as = 1.33
8n1 240
- n1 = 16,n1 = 112
7 93. Suppose, m1, v1 and m2, v 2 are the masses and velocities of the
\ n = 120 Hz balls. Since, kinetic energy is same
87. The frequency of fork 2 = 200 ± 4 = 196 or 204 Hz. Since, on i. e. , KE1 = KE2
attaching the tape on the prong of fork 2, its frequency 1 1
So, m1v12 = m2v 22
decreases, but now the number of beats per second is 6 i.e., 2 2
the frequency difference now increases. It is possible only
(m1v12) (m2v 2) 2
when before attaching the tape, the frequency of fork 2 is less or =
than the frequency of tunning fork 1. Hence the frequency of m1 m2
fork 2 is 196 Hz. p12 p22
or =
æIö m1 m2
88. As, B1 = 10 log ç ÷
è I0 ø i. e. , pµ m
æ I¢ ö Therefore, the heavier ball will have greater momentum.
and B2 = 10 log ç ÷
è I0 ø 94. The fundamental frequency of an open prgan pipe is given by
Given, B2 - B1 = 20 v
n=
æ I¢ ö 2l
20 = 10 log ç ÷
è I0 ø 350
= = 350 Hz
Þ I ¢ = 100I 2 ´ 0.5

89. According to problem 95. Between two fixed points, resonance is obtained, when one
loop, two loops, three loops etc., are formed. The resonant
v
f1 = ...(i) frequencies are in the ratio 1 :2 :3 :4.
2l
As the two resonance frequencies are 315 Hz and 420 Hz,
v v
and f2 = = ...(ii) with highest commom factor = 105.Therefore, the lowest
4l / 4 l resonant frequency for this string = 105 Hz.
f1 1
We, get = 96. Here, n = 9500 Hz, v S = ?
f2 2
v = 300 ms–1,
54
90. Frequency, n=
60 n = 1000 Hz
9 As source is moving towards the listener,
Þ n=
10 v´n
\ n¢ = (from Doppler effect)
9 v - vS
\ Velocity, v = nl = ´ 10 = 9 ms-1
10 300 ´ 9500
Þ 1000 =
v 300 - v S
91. The frequency of an open organ pipe is given by, n =
2l or 300 - v S = 285
v or v S = 300 - 285 = 15 ms–1
n1 = …(i)
2 ´ 0.5
v 97. There are four beats between P and Q, therefore the possible
and n2 = …(ii)
2 ´ 0.505 frequencies of P are 246 or 254 (that is 250 ±4) Hz.
(Given l2 = 50 cm = 0.5 m, l2 = 50.5 cm = 0.505 cm) When the prong of P is filed, its frequency becomes greater
than the original frequency of P is 254, then on filing its
From Eqs. (i) and (ii), we get frequency will be greater than 254. The beats between P and
n1 - n2 = 3 Q will be more than 4. But it is given than the beats are
æ v v ö 0.01v reduced to 2, therefore, 254 is not possible.
\ 3=ç - ÷=
è1.0 1.01ø 1.01 Therefore, the required frequency must be 246 Hz.
0.01v = 3.03 (This is true, because on filling the frequency may increase
3.03 to 248, giving 2 beats with Q of frequency 250 Hz).
or v= = 303 ms-1
0.01
Telegram @unacademyplusdiscounts

Waves 715

98. When the source and observer are approaching towards each 102. Time taken by stone to drop into lake is obtained from
other, then apparent frequency heard by the observer is 1 2
s= gt
æ v + v0 ö 2
n¢ = ç ÷n
è v - v0 ø 2s 2 ´ 500
t= = = 10 s
æ vö g 10
çv + ÷
è 2ø Sound so produced travels to the top of the tower.
= n
æ v ö s 500
çv - ÷ Time taken t¢ = = = 1.5 s
è 2ø v 330
æ3 2ö Total time = t + t ¢ = 10 + 1.5 = 11.5 s
or n ¢ = ç ´ ÷ n = 3n
è 2 1ø
103. For first resonance,
99. An open pipe forms antinode at both ends. If length of pipe is l l
and v the velocity, then the fundamental frequency is given by 30.7 = +x …(i)
4
v
n1 = For second resonance,
2l 3l
3v 63.2 = +x …(ii)
Frequency of third harmonic of closed pipe is n ¢ = 4
4l
Solving Eqs., (i) and (ii), l = 65.0 cm
Given, n ¢ - n = 100
3v v The effort in measuring the length using metric scale would
\ - = 100 be 0.1 cm, which is the least count of metric scale.
3l 2l
v Therefore , l = (65.0 ± 0.1) cm.
Þ = 100
4l As, v = nl = 512 (65.2 ± 0.1)
1æ v ö Therefore, error in velocity = 51.2 cms-1
Þ ç ÷ = 100
2 è2 lø 104. In case of open pipe, the frequency of second harmonic is,
\ n1 = 200 Hz f1 = 2v / 2L = v / L
100.The node and antinodes are formed in a standing wave In case of closed pipe, the frequency of nth harmonic is
pattern as a result of the interference of two waves. Distance f2 = nv / 4L = nf1 / 4
between two nodes is half of wavelength (l). Standard where, n = 1, 3, 5, ¼ , i. e. ,n is odd and f1 > f2
equation of standing wave is It will be so if n = 5
2 px 2pvt 5
y = 2a sin cos …(i) \ f2 = f1
l l 4
where a is amplitude, l the wavelength, v the velocity and t 1
the time. v+ v
v ¢ (v + vL) 5 5
105. As, = = =
Given equation is v v v 6
2 px
y = 5 sin cos 20 pt …(ii) Percentage increase in frequency
3
(v ¢ - v) (6 - 5)
Comparing Eqs. (ii) with (i), we have = ´ 100 = = 20%
2 px 2 px v 100
= 1 - cos 2wt 1 1
l 3 106. As, y = sin2 wt = = = wt
Þ l = 3 cm 2 2 2
dy 1
101. From Doppler’s effect, the perceived frequency ( f ¢ ) is given by velocity, = ´ 2w sin 2wt
dt 2
v
f ¢= f d 2y æ1 ö
v - vs Acceleration, = 2w2 cos 2wt = 4 w2 ç - y ÷
dt 2 è2 ø
where v S is velocity of source, v is the speed of sound and f
the original frequency. As acceleration µ displacement and negative sign shows that
Given, f = 500 Hz, v s = 30 ms-1, v = 330 ms-1 it is direction towards mean position.
330 \ Motion is simple harmonic and its period = p / w
f ¢ = 500 ´ ,
330 - 30 107. Given, y1 = 0.1sin(100 pt+ p / 3)
330
f ¢ = 500 ´ = 550 Hz dy1
300 \velocity, v1 = = 0.1 ´ 100 p cos (100 pt - p / 3)
dt
y 2 = 0.1cos pt = 0.1sin( pt + p / 2)
Telegram @unacademyplusdiscounts

716 JEE Main Physics

dy 2 111. Maximum number of beats = ( n + 1) - ( n -1) = 2


Velocity, v2 = = 0.1 ´ p cos ( pt+ p / 2)
dt
v
\ Phase difference of the velocity of particle 1 w.r.t. 112. As, =f
particle 2 = ( pt + p / 3) - ( pt + p / 2) = - p / 6 4 ( l + e)
v v
108. As, f = Þ ( l + e) =
2l 4f
v
Now, it will act like one end opened and other closed or l= =e
4f
v v v
So, f0 = 0 = = =f Here, e = (0.6) lr = (0.6) (2) = 1.2 cm
4 l ¢ 4l2 2l
ax + bt )2
113. Here, fincident = freflected
109. Given, y = ( xt) = e-(
320
It is transverse type y( x, t ) = e-( ax+ bt )
2 Þ = ´ 8 Hz
320 - 10
b
Speed, v =
a
The wave is moving along x-direction. 36 km/h
= 10 m/s
é t x öù
110. Given, y = 0.02 sin ê2p æç - ÷
ë è 0.04 0.50 ø úû
320 + 10
T w T 1 / 0.04 Now, fobserved = freflected
v= = = = 320
M k 0.04 1 / 0.50 330
2 =8´ = 8.51kHz = 8.5 kHz
æ 0.05 ö 2 310
T=ç ÷ ´ 0.04 = (12.5) ´ 0.04 = 6.25 N
è 0.04 ø
Telegram @unacademyplusdiscounts

Part - II
th
Chapters from Class 12 Syllabus
Telegram @unacademyplusdiscounts
Telegram @unacademyplusdiscounts

17 Electrostatics
JEE Main MILESTONE
< Electric Charges < Equipotential Surfaces
< Coulomb’s Law : Force between two Point < Van de Graaff Generator
Charges < Electric Field Lines
< Superposition Principle < Electric Dipole
< Electric Field < Electric Flux
< Electric Field due to a Point Charge < Gauss’s Law
< Motion of a Charged Particle in an Electric < Conductors and Insulators
Field < Polar and Non-Polar Substances
< Electric Potential < Polariszation
< Electric Potential due to Continuous < Capacitor
Charge Distribution < Energy Stored in a Conductor
< Electric Potential Energy

17.1 Electric Charges


All substances are made up of atoms and an atom is made up of electrons, protons
and neutrons which are known as fundamental particles.
The electricity developed on
In an atom, there is a nucleus in which protons and neutrons (collectively called
objects, when they are rubbed
nucleons) are found and around the nucleus, electrons revolve in
circular/elliptical orbits with high speed (106 ms -1). Due to presence of protons in
with each other, is called frictional
electricity. One electric charges so
nucleus, nucleus is always positively charged. Charge on nucleus = + Ze, where
Z = atomic number. developed cannot move from one
part to other part of the object.
The negative charge on electron cloud formed by electrons in a neutral atom = - Ze.
So, the frictional electricity is
Excess of electrons on a body is known as electric charge. Electric charge are of two known as electrostatics or static
types-positive and negative. electricity.
On a neutral body, the net charge is equal to zero. The charges are produced by
transfer of electrons from one body to another. When electrons are lost by an atom,
molecule or a body, it becomes positively charged and when electrons are gained
by them, they become negatively charged. The amount of charge developed
q = ± ne, where n = number of electrons lost or gained, (e = 1.6 ´ 10–19 C).

Unit of charge SI unit coulomb


CGS unit stat coulomb or esu of charge
Telegram @unacademyplusdiscounts

720 JEE Main Physics

Properties of Charges
1. Like charges repel while opposite charges attract each other. 7. A charge at rest produces only electric field around it while a charge
2. As electron is a fundamental particle which cannot be sub-divided, having unaccelerated motion produces electric field as well as
hence smallest number of electron lost or gained as n = 1and hence magnetic field around itself.
charge on a body, q = ne = 1 ´ e. Hence, smallest charge in nature is 8. A charge in accelerated motion emits electromagnetic radiation in
electronic charge. Charge smaller than e does not exist and any addition to produce electric field and magnetic field around itself.
amount of charge is an integral multiple of e,q = ne, where n = an 9. According to the classical electrodynamics theory, an accelerated
integer. Thus, charge is quantized. charge radiates energy in the form of electromagnetic wave, i .e .,
3. Quantization If a physical quantity exists in discrete values photons.
called quanta and any amount is integral multiple of quanta, then 10. When charge is given to a non-conducting body, then the charge
the physical quantity is said to be quantized. remains localised. But when charge is given to an isolated
4. Charge is invariant i .e ., charge does not change with change in conducting body, the charge gets uniformally distributed over the
velocity. outer surface of the conducting body.
According to theory of relativity, the mass, time and length change 11. Linear charge density, l = charge per unit length. Linear
with a charge in velocity but charge does not change. charge q
charge density, l = = cm–1
length L
5. A charged body attracts a lighter neutral body.
6. Electronic charge is additive i .e ., the total charge on a body is the 12. Surface charge density, s = charge per unit area
Charge q
algebraic sum of all the charges present in different parts of the Surface charge density, s = = cm-2
body. For example, if a body has different charges as + 2q, + 4q, -3q, Total surface area A
-q, then the total charge on the body is + 2q.

Sample Problem 1 A polythene piece rubbed with wool is as the amount of charge developed may be less than or
found to have a negative charge of 3 ´ 10 -7 C. Estimate the equal to or greater than the charge left on the charged
number of electrons transferred from wool to polythene. body but the total charge remains equal to the original
(a)1.8 ´ 1015 (b)1.8 ´ 1012 (c)1.2 ´ 1011 (d)1.2 ´ 1010 charge on charged body.

Interpret (b) Here, total charge transferred, q = - 3 ´ 10 –7 C Charging by Induction


Charge on an electron, e = - 1.6 ´ 10 –19 C Charging a neutral body with the help of a charged body
From quantization of charge, q = n e without making actual contact between the two bodies is
Therefore, number of electrons transferred, called charging by induction. The charge developed on
q - 3 ´ 10 –7 the neutral body is called induced charge and the charge
n= = on the charging body is called induced charge. The
e - 1.6 ´ 10 –19
induced charge is less than or equal to induction charge
= 1.8 ´ 10 12 and their nature is opposite. The induced charge is equal
to inducing charge only when inducing charge is very
Ways
close of Charging
to the surface a Body
or completely enclosed by the surface
on which induced charge develops.
There are three methods of charging a neutral body.
If q be the inducing charge, then the induced charge on a
é 1ù
Charging by Friction body having dielectric constant K is q¢ = - q 1 -
êë
. In case
K úû
When two neutral bodies are rubbed together, then some
of conductor, K = ¥, hence induced charge q¢ = - q.
electrons get transferred from on body to the another as a
result of which they get charged. Amount of charge
developed on the two bodies is equal in magnitude, but
Sample Problem 2 Two conductors identical in shape
the nature of charge is opposite. and size, but one of silver and the other of aluminium (which is
less conducting) are both placed in an identical electric field. In
Charging by Conduction which metal will more charge be induced?
(a) The induced charge will be equal
When a charged body is brought in contact with a neutral (b) The induced charge will not be equal
body, then some charge gets transferred from the charged (c) May be equal or may not be equal
body to the neutral body due to which the neutral body (d) None of the above
gets charged. The nature of charge developed same where
Telegram @unacademyplusdiscounts

Electrostatics 721

Interpret (a) In case of induction, maximum induced charge is Coulomb’s Law in Vector Form
æ 1ö
given by q ¢ = - q ç1 - ÷
è Kø Let q1 and q2 be the two point charges separated by
distance r in vacuum. Let F21 be the force on q2 due to q1
Q In case of conductor K = ¥
and F12 the force on q1 due to q2.
æ 1ö
Hence induced charge, q ¢ = - q ç1 - ÷ = - q qq
è ¥ø Then, F21 = k 1 2 2 r$12 …(i)
r
i. e. , in both the metals induced charge will be equal. However, in
where, r$12 is a unit vector pointing from q1 to q2.
case of conduction, the current in silver will be more than in
qq
aluminium as it is more conducting than aluminium. Also, F12 = k 1 2 2 r$21 …(ii)
r
Conservation of Charges where, $r21 is a unit vector pointing from q2 to q1.
Charge is always conserved i. e. , constant. Charge can
Importance of Coulomb’s law in vector form
neither be created nor be destroyed. In any physical
phenomenon. It follows from Eqs. (i) and (ii), thus F21 = - F12.
Total initial charge = Total final charge. The force exerted by the two charges on each other is
equal and opposite. (Newton’s third law is obeyed)
e. g. , charge annihilation reaction
e- (electron) + e+ (positron) = hn (photon) Effect of Medium
Force between two charges in vacuum
1 q1q2
17.2 Coulomb’s Law : Force F0 =
4pe 0 r 2
newton …(i)

between Two Point Charges Force between the same two charges in case of medium
1 q1q2
It states that, the electrostatic force of interaction between Fm = newton …(ii)
4pe 0e m r 2
two point charges at rest is directly proportional to the
product of the charge and inversely proportional to the where, e m is absolute permittivity of the medium.
square of the distance between them. Dividing Eq. (i) by Eq. (ii), we get
q1 q2 1 q1q2
r
F0 4pe 0 r 2 e
F µ q1q2 = = m = er
Fm 1 q1q2 e0
1 4pe 0 r 2
i. e. , F µ 2
r F0
qq = er
F µ 12 2 Fm
r F
Fm = 0
qq er
or F = k 12 2 …(i)
r
Fir air, er = 1 (minimum)
where, k is constant of proportionality. Its value depends \ Fair = F0
upon the system of units and on the nature of medium The force between two charges is always maximum in
between the charges. When the two charges are located in case of air. Dielectric constant (K ) may be defined as the
vacuum or air ratio of force in vacuum and force in a given medium
1 between two charges separated by same distance.
k= (In SI unit)
4 pe 0 F
i. e. , K = 0
Fm
where, e 0 is called absolute permittivity of free space and its
æe ö F
value is 8.85 ´ 10–12 C2N–1m–2. (i) K = er = ç m ÷ = 0 is also called relative permittivity of
è e0 ø Fm
\In SI unit, Eq. (i) in presence of vacuum can be written as medium.
1 q1q2 (ii) K is a dimensionless constant.
F =
4 pe 0 r 2 (iii) K = ¥ for conducting medium.
qq (iv) Higher is the dielectric constant of a medium, lesser is
or F = 9 ´ 109 ´ 1 2 2
r the number of electric lines of force passing through
that medium.
Telegram @unacademyplusdiscounts

722 JEE Main Physics

Force between Multiple Charges \ Q - 2q = 0


\ Q = 2q
The mutual electric force between two charges is given by
Q
Coulomb’s law. When we have to calculate the force on a and q=
2
charge due to several stationary charges, we use
superposition principle in addition to Coulomb’s law. Q æ Qö Q Q
Hence, the two charges are and çQ - ÷ or and , when a
2 è 2ø 2 2
charge is divided into two equal parts, then force between them is
17.3 Superposition Principle always maximum.

It states that, the net force on any one charge is

You might also like